Download as pdf or txt
Download as pdf or txt
You are on page 1of 1088

TOPNOTCH MEDICAL BOARD PREP BIOCHEMISTRY SUPEREXAM

For inquiries visit www.topnotchboardprep.com.ph or email us at topnotchmedicalboardprep@gmail.com


DEAR TOPNOTCH FRIENDS:

PLEASE FOLLOW THESE INSTRUCTIONS:

1. These questions are previous diagnostic, midterm, and finals exams of Topnotch, almost all of them made by Topnotch Board Exam Topnotchers.
2. Answer this Topnotch Superexam seriously 100-items at a time. Cover the “Explanations” Column. Do not immediately look at the answers from the
answer key. That’s not the correct way of answering sample exams. You need to treat these MCQs as exercises and not as handouts.
3. Time yourself. 1.5 hours per 100-item block.
4. After answering each 100-item block, refer to the Topnotch Answer Key for the correct answers. Please be careful of “frameshift mutations” when
checking your answers – check every 10 items. (the format of the answer key was designed for you to practice against “frameshift mutations”)
5. The Topnotch Superexams are EXERCISES for the actual med boards. They will not appear verbatim in your future exams. More than knowing what’s
the correct answer, it’s more important for you to:
a. Know why the other choices are wrong
b. Know why the other choices were included in the first place
c. Know the explanation to the correct answer
6. Sharpen your mind by answering the Topnotch Superexams. Most of these questions based on past feedback are more difficult than the actual questions
in the med boards. In these exams made by Board Exam Topnotchers, if you’re getting a score of 60/100 , that’s already a good score. More than 80/100
is outstanding.


Item QUESTION EXPLANATION AUTHOR TOPNOTCH
# EXAM
1 The pH at which majority of amino acid exists in a form A zwitterion is a molecule with a positive and a KRISTEL DIAGNOSTIC
where it has an equal number of positive and negative negative charge in different locations yielding to a net TANHUI EXAM -
charges and thus is electrically neutral is called the: charge of zero for the whole molecule. The pI, or the (TOP 3 - MARCH 2016
A. pK1 isoelectric pH is the pH at which this AUG 2015
B. pK2 isoelectric/zwitterion form of the amino acid MED
C. pK1 + pK2 predominates. Source: Harper 27th ed p.17 BOARDS;
D. pIE. pH TOPNOTCH
MD FROM
LA SALLE)
2 Glutathione is an important molecule for the clearance of Remember the 3 amino acids which make up KRISTEL DIAGNOSTIC
free radicals in the body. It consists of only 3 amino Glutathione. (Similar principle/concept to previous TANHUI EXAM -
acids. Which of the following polypeptides is the primary exam questions) The most important amino acid is (TOP 3 - MARCH 2016
structure of glutathione? cysteine which contains the active -SH group. Source: AUG 2015
A. Glutamyl – cysteinyl – glycine Harper 27th ed p. 19 MED
B. Glutamine – cysteinyl – glycine BOARDS;
C. Glutamine – threonyl – methionine TOPNOTCH
D. Glutamine – cysteinyl – methionine MD FROM
E. None of the above LA SALLE)

3 Which of the following amino acid is incorrectly paired Tyrosine does not contain a thiol group. Tyrosine KRISTEL DIAGNOSTIC
with its side chain? contains a phenol group. Source: Harper 27th ed p. 18 TANHUI EXAM -
A. Tyrosine – Thiol group containing (TOP 3 - MARCH 2016
B. Tryptophan – indole group containing AUG 2015
C. Proline – imino group containing MED
D. Arginine – guandino group containing BOARDS;
E. Histidine – imidazole group containing TOPNOTCH
MD FROM
LA SALLE)
4 A neonate, born at home, with no newborn care This is a classic case of hemorrhagic disease of the KRISTEL DIAGNOSTIC
administered is rushed to the ER for heavy bleeding newborn. They can present with seizures for TANHUI EXAM -
from the umbilical stump and recent onset generalized intracranial hemorrhage. Source: Harper 27th ed p (TOP 3 - MARCH 2016
seizures. What vitamin deficiency is responsible for this? 495 AUG 2015
A. Vitamin A MED
B. Vitamin D BOARDS;
C. Vitamin E TOPNOTCH
D. Vitamin K MD FROM
E. Vitamin C LA SALLE)

5 What is the rate limiting enzyme of ketogenesis? HMG CoA reductase is the rate limiting enzyme for KRISTEL DIAGNOSTIC
A. HMG-CoA synthase cholesterol synthesis. Source: Harper 27th ed p 191 TANHUI EXAM -
B. HMG-CoA reductase (TOP 3 - MARCH 2016
C. HMG-CoA lyase AUG 2015
D. HMG-CoA transferase MED
E. All of the above BOARDS;
TOPNOTCH
MD FROM
LA SALLE)
6 An infant presents with hepatosplenomegaly and failure Glycogen storage diseases: enzyme deficienct: signs KRISTEL DIAGNOSTIC
to thrive. A liver biopsy was done which revealed and symptoms/buzzwords TANHUI EXAM -
accumulation of polysaccharide with few branch points (TOP 3 - MARCH 2016
because of a deficiency for an important enzyme in - Von Gierke’s disease: Glucose 6 phosphatase AUG 2015
carbohydrate metabolism. What is the diagnosis for this deficiency: severe hypoglycemia (because glucose MED
patient? generated from all pathways (gluconeogenesis and BOARDS;
A. Von Gierke Disease glycogenolysis) is trapped in the hepatocyte and TOPNOTCH
B. Andersen’s Disease cannot be released into the blood) MD FROM
C. Tarui’s Disease - Pompe’s disease: acid maltase: heart failure LA SALLE)
D. Fabry’s Disease - Cori’s disease: debranching enzyme: accumulation of
E. Her’s Disease branched polysaccharide
- Anderson’s disease: branching enzyme:
accumulation of polysaccharide with few branch
points
- McArdle syndrome: muscle phosphorylase: poor
exercise tolerance, accumulation of muscle glycogen
TOPNOTCH MEDICAL BOARD PREP BIOCHEMISTRY SUPEREXAM Page 1 of 92
For inquiries visit www.topnotchboardprep.com.ph or email us at topnotchmedicalboardprep@gmail.com
TOPNOTCH MEDICAL BOARD PREP BIOCHEMISTRY SUPEREXAM
For inquiries visit www.topnotchboardprep.com.ph or email us at topnotchmedicalboardprep@gmail.com
Item QUESTION EXPLANATION AUTHOR TOPNOTCH
# EXAM
- Her’s disease: liver phosphorylase: accumulation of
liver glycogen, mild hypoglycemia (because only
glycogenolysis is impaired, gluconeogenesis can
replenish glucose)
Source: Harper 27th ed p. 166

7 A patient who presented with mild jaundice had work up Try to master direct and indirect hyperbilirubinemia. KRISTEL DIAGNOSTIC
done, which revealed increased direct bilirubin, absent It’s a favorite theme and there are many question TANHUI EXAM -
urine urobilinogen, (+) urine bilirubin. Liver biopsy variations that can be done across many subjects. (TOP 3 - MARCH 2016
showed liver hyperpigmentation. Which of the following AUG 2015
is the most likely diagnosis? This is a case of conjugated hyperbilirubinemia (note MED
A. Crigler- Najjar syndrome sometimes, the examinar can be unforgiving and may BOARDS;
B. Gilbert syndrome not put increased direct bilirubin so be familiar with TOPNOTCH
C. Dubin Johnson syndrome other lab findings too). Rotor syndrome and dubin MD FROM
D. Rotor syndrome Johnson syndrome present similarly except dubin LA SALLE)
E. Hepatitis Johnson has liver hyperpigmentation (dubin – dark
liver)

Conjugated hyperbil – increased direct serum
bilirubin, absent urine urobilinogen, (+) urine
bilirubin
Ex. Obstructive jaundice, dubin Johnson, Rotor
syndrome

Unconjugated – increased indirect serum bilirubin, (+)
urine urobilinogen, (-) urine bilirubin
Ex. Crigler-Najjar, Gilbert

Source: Harper 27th ed p292
8 If 20% of DNA consists Guanosine nucleotides, how According to Chargaff’s rule, the concentration of A KRISTEL DIAGNOSTIC
many percent are thymidine nucleotides? nucleotides equals T nucleotides while C nucleotides TANHUI EXAM -
A. 20% equals G nucleotides. (TOP 3 - MARCH 2016
B. 30% AUG 2015
C. 40% 100% - 20% (G) – 20%(C) = 60% (A+T) MED
D. 50% 60% / 2 = 30% BOARDS;
E. Cannot be identified from given information TOPNOTCH
MD FROM
LA SALLE)

9 Covalent modification is one of the most rapid ways of Acetylation and methylation are covalent KRISTEL DIAGNOSTIC
regulating enzyme activity in the cell. During fasting modifications on histones to activate or inactivate TANHUI EXAM -
which of the following is expected to occur? transcription. Metabolic enzymes are regulated via (TOP 3 - MARCH 2016
A. Phosphorylation of Glucose-6-phosphatase phosphorylation or dephosphorylation.Glucagon AUG 2015
B. Dephosphorylation of Glycogen synthase phosphorylates, insulin dephosphorylates. (My MED
C. Acetylation mnemonic is DIG-P) This results to either activation or BOARDS;
D. Methylation inactivation of particular enzymes in certain TOPNOTCH
E. None of the above pathways. MD FROM
Make sure you master this concept and which LA SALLE)
enzyme/pathways are activated or inactivated by
insulin and glucagon. They can easily ask 5-10
different varieties of these sort of questions.
10 After a meal, insulin levels in the blood become elevated Glucagon phosphorylates, insulin dephosphorylates. KRISTEL DIAGNOSTIC
to effect which of the following changes in enzyme (My mnemonic is DIG-P) This results to either TANHUI EXAM -
activity? activation or inactivation of particular enzymes in (TOP 3 - MARCH 2016
A. Glucokinase is dephosphorylated and inactivated certain pathways. AUG 2015
B. Glycogen synthase is dephosphorylated and activated Make sure you master this concept and which MED
C. HMG CoA synthase is phosphorylated and activated enzyme/pathways are activated or inactivated by BOARDS;
D. Acetyl CoA carboxylase is phosphorylated and insulin and glucagon. They can easily ask 5-10 TOPNOTCH
inactivated different varieties of these sort of questions. MD FROM
E. A and B LA SALLE)

11 Which is the main source of NADPH for Lipogenesis? NADPH is utilized in many pathways in the cell such as KRISTEL DIAGNOSTIC
A. Pentose Phosphate Pathway lipogenesis, nucleic acid metabolism, and even in the TANHUI EXAM -
B. Krebs Cycle pathways utilized by the innate immunity. There are 2 (TOP 3 - MARCH 2016
C. Urea Cycle sources of NADPH – the Pentose phosphate pathway AUG 2015
D. Gluconeogenesis and the malic enzyme. The PPP is most significant. MED
E. A and C Source: Harper 27th ed p199 BOARDS;
TOPNOTCH
MD FROM
LA SALLE)
12 Cyanide inhibits which of the following complex of the Inhibitors: KRISTEL DIAGNOSTIC
electron transport chain? Complex I – Barbiturates TANHUI EXAM -
A. NADH Q oxidoreductase Complex II – Malonate (TOP 3 - MARCH 2016
B. Q cytochrome c oxidoreductase Complex III – Antimycin A and dimercaprol AUG 2015
C. Cytochrome C oxidase Complex IV – H2S, Cyanide, CO MED
D. Succinate Q reductase BOARDS;
E. None of the above Source: Harper 27th ed p107 TOPNOTCH
MD FROM
LA SALLE)

TOPNOTCH MEDICAL BOARD PREP BIOCHEMISTRY SUPEREXAM Page 2 of 92


For inquiries visit www.topnotchboardprep.com.ph or email us at topnotchmedicalboardprep@gmail.com
TOPNOTCH MEDICAL BOARD PREP BIOCHEMISTRY SUPEREXAM
For inquiries visit www.topnotchboardprep.com.ph or email us at topnotchmedicalboardprep@gmail.com
Item QUESTION EXPLANATION AUTHOR TOPNOTCH
# EXAM
13 Which of the following is not an essential fatty acid? Both are essential FA. KRISTEL DIAGNOSTIC
A. Linoleate TANHUI EXAM -
B. Alpha-linolenate Source: Harper 27th ed p204 (TOP 3 - MARCH 2016
C. Both AUG 2015
D. Neither MED
E. Fatty acids are not essential in diet and can be BOARDS;
synthesized from carbohydrates and proteins TOPNOTCH
MD FROM
LA SALLE)

14 Starch is a polymer of which of the following? Source: Harper 27th ed 116 KRISTEL DIAGNOSTIC
A. Glucose TANHUI EXAM -
B. Galactose (TOP 3 - MARCH 2016
C. Fructose AUG 2015
D. Tryptophan MED
E. Cellulose BOARDS;
TOPNOTCH
MD FROM
LA SALLE)
15 If the enthalpy change is negative and the entropy Thermodynamics is still chemistry! Don’t sacrifice this KRISTEL DIAGNOSTIC
change is positive in a particular reaction, will this topic, they can still ask a few questions on this. Know TANHUI EXAM -
proceed spontaneously? when a reaction is spontaneous, the definitions of (TOP 3 - MARCH 2016
A. Yes, always entropy and enthalpy and the basic laws of AUG 2015
B. No, always thermodynamics. Source: Topnotch handout in MED
C. Maybe, but only at high temp biochemistry BOARDS;
D. Maybe, but only at low temp TOPNOTCH
E. Cannot be determined with the given information MD FROM
LA SALLE)

16 A 6 month old infant presented with hyperacusis Be familiar with the clinical presentation as well as KRISTEL DIAGNOSTIC
(exaggerated startle response), cherry red spot on the enzyme deficiency and substance which accumulates TANHUI EXAM -
macula and froglike position. What substance is expected for lysosomal storage diseases. (favorite cases are (TOP 3 - MARCH 2016
to accumulate in the brain? taysach’s, neimann pick, gaucher’s) They can ask AUG 2015
A. GM2 Ganglioside similar 2 or 3 step analysis types of question in MED
B. Hexoseaminidase A biochemistry or pediatrics/IM or even patho. BOARDS;
C. Sphingomyelin TOPNOTCH
D. Glycosphingolipids This is a case of Tay Sach’s. The enzyme deficient is MD FROM
E. No accumulation Hexoseaminidase A and the substance GM2 LA SALLE)
ganglioside accumulates in the brain. Note the
buzzwords - exaggerated startle response and cherry
red spot in the macula. They do not present with
hepatomegaly.

Source: Harper 27th ed p.215
Clinical case adapted from first aid for the pediatrics
clerkship 2nd ed.
17 Patient presents with arthritis and connective tissue This is a case of alkaptonuria. KRISTEL DIAGNOSTIC
pigmentation. On letting his urine stand, his urine is TANHUI EXAM -
noted to darken. Which enzyme is deficient in this Source: Harper 27th ed p 259 (TOP 3 - MARCH 2016
patient? AUG 2015
A. Tyrosine aminotransferase MED
B. Phenylalanine hydroxylase BOARDS;
C. Homogentisate oxidase TOPNOTCH
D. Branched chain ketoacid dehydrogenase MD FROM
E. A and C LA SALLE)

18 The following is ATP, Biotin and CO2 requiring: These 3 enzymes need 3 things to function (ATP, KRISTEL DIAGNOSTIC
A. Pyruvate dehydrogenase Biotin, CO2): Acetyl CoA carboxylase, propionyl TANHUI EXAM -
B. Acetyl CoA Carboxylase carboxylase, pyruvate carboxylase. (TOP 3 - MARCH 2016
C. HMG CoA reductase My mnemonic is the ABC enzymes. AUG 2015
D. SGPT MED
E. All of the above Source: Topnotch lecture BOARDS;
TOPNOTCH
MD FROM
LA SALLE)
19 Patient presents with urine smelling of maple syrup or The patient is deficient of branched chain ketoacid KRISTEL DIAGNOSTIC
burnt sugar and is diagnosed with maple syrup urine dehydrogenase. TANHUI EXAM -
disease. Which of the following is not necessary for the (TOP 3 - MARCH 2016
function of the enzyme deficient in this disease? BCKD, like pyruvate dehydrogenase and succinate AUG 2015
A. Thiamine dehydrogenase require the following for function: MED
B. Lipoic acid Thiamine, Lipoic acid, CoA, FAD, NAD BOARDS;
C. FAD TOPNOTCH
D. Biotin Familiarize yourself with these 3 enzymes and the 5 MD FROM
E. None of the above components required for their function. It’s also a LA SALLE)
popular question.
Source: Topnotch lecture

TOPNOTCH MEDICAL BOARD PREP BIOCHEMISTRY SUPEREXAM Page 3 of 92


For inquiries visit www.topnotchboardprep.com.ph or email us at topnotchmedicalboardprep@gmail.com
TOPNOTCH MEDICAL BOARD PREP BIOCHEMISTRY SUPEREXAM
For inquiries visit www.topnotchboardprep.com.ph or email us at topnotchmedicalboardprep@gmail.com
Item QUESTION EXPLANATION AUTHOR TOPNOTCH
# EXAM
20 Patient has an autosomal recessive genetic disease This is a case of xeroderma pigmentosum. KRISTEL DIAGNOSTIC
which makes him markedly susceptible to sunlight with TANHUI EXAM -
subsequent formation of multiple skin cancers and Source: Harper 27th ed p345 (TOP 3 - MARCH 2016
premature death. The inherited defect involves repair of AUG 2015
damaged DNA, particularly thymine dimers. Cells MED
cultured from the patient has low activity of which of the BOARDS;
following DNA repair mechanisms? TOPNOTCH
A. Mismatch repair MD FROM
B. Base excision repair LA SALLE)
C. Nucleotide excision repair
D. Double strand break repair
E. All of the above

21 Chemical added to natural gas that confers its distinctive SIMILAR TO BOARD EXAM CONCEPT. Natural gas is LESTER MIDTERM 1
odor? odorless. To detect leaks, mercaptan is added to the BRYAN CO EXAM -
A. Sulfur gas to make it smell like rotten eggs. (TOP 10 - MARCH 2016
B. Chlorine AUG 2015
C. Methane MED
D. Mercaptan BOARDS;
E. Phenol TOPNOTCH
MD FROM
UST)
22 Marize did not take any food for eight hours in SIMILAR TO BOARD EXAM CONCEPT. Fasting LESTER MIDTERM 1
preparation for a lipid profile test. Which of the increases the glucagon:insulin ratio. Glucagon acts via BRYAN CO EXAM -
following is likely to be occuring inside her cells right a G-protein coupled receptor whose alpha subunit (TOP 10 - MARCH 2016
now? adenylate cyclase, increasing intracellular levels of AUG 2015
A. Glycogen phosphorylase b is unphosphorylated. cAMP. cAMP activates protein kinase A. This enzyme, MED
B. cAMP levels are increased. in turn, activates phosphorylase kinase, which then BOARDS;
C. Pyruvate is converted to lactate. phosphorylates glycogen phosphorylase b, converting TOPNOTCH
D. PFK-2 is unphosphorylated. it into the active form called phosphorylase a. Protein MD FROM
E. Fructose 2-6 bisphosphatase is unphosphorylated. kinase A will also phosphorylate the enzymes UST)
fructose-2,6-bisphosphatase and
phosphofructokinase-2. This covalent
phosphorylation initiated by glucagon activates the
former and inhibits the latter. Conversion of pyruvate
in lactate happens in anaerobic metabolism, which
does not necessarily occur only in fasting. There were
about 10 questions involving insulin, glucagon,
feeding and fasting in Biochemistry during our
Biochemistry exam
23 Ryan experiences bloating and abdominal cramps SIMILAR TO BOARD EXAM CONCEPT. Lactase LESTER MIDTERM 1
whenever he eats dairy-containing products. Which of intolerence is caused by a deficiency of lactase (a BRYAN CO EXAM -
the following is true about his condition? disaccharidase) in the intestinal brush border. Lactose (TOP 10 - MARCH 2016
A. Galactose accumulate in his small intestine. is unable to be broken down into glucose and AUG 2015
B. Increased levels of hydrogen may be detected in his galactose, and thus accumulates in the gut. It is MED
breath. eventually fermented by bacteria to form hydrogen, BOARDS;
C. Stool pH is increased. methane and carbon dioxide, leading to various TOPNOTCH
D. There is a deficiency of a specific disaccharide from abdominal symptoms. 3 questions pertaining to MD FROM
the brush border. lactase deficiency were asked during our UST)
E. Administration of lactulose will lead to improvement Biochemistry exam
of his symptoms.
24 Which of the following amino acids contain a phenol SIMILAR TO BOARD EXAM CONCEPT. Aromatic amino LESTER MIDTERM 1
ring? acids: Phenylalanine - benzene ring; Histidine - BRYAN CO EXAM -
A. Tyrosine Imidazole ring; Tryptophan - Indole ring; Proline (TOP 10 - MARCH 2016
B. Phenylalanine contains a ring structure, but is not classified as an AUG 2015
C. Histidine aromatic amino acid MED
D. Tryptophan BOARDS;
E. Proline TOPNOTCH
MD FROM
UST)
25 The term forensic pertains to: SIMILAR TO BOARD EXAM CONCEPT. This seems to LESTER MIDTERM 1
A. Crime be the only none-Biochemistry related question asked BRYAN CO EXAM -
B. Investigatory during our Board Examination in Biochemistry. (TOP 10 - MARCH 2016
C. Autopsy AUG 2015
D. Law MED
E. Criminal BOARDS;
TOPNOTCH
MD FROM
UST)
26 The following electrophoresis technique is used in the For the different blotting techniques, remember the LESTER MIDTERM 1
study of DNA-binding proteins: mnemonic SNoW DRoP: Southern - DNA, Northern - BRYAN CO EXAM -
A. Northern blot RNA, Western - Protein (TOP 10 - MARCH 2016
B. Western blot AUG 2015
C. Southern blot MED
D. Western blot BOARDS;
E. Southwestern blot TOPNOTCH
MD FROM
UST)

TOPNOTCH MEDICAL BOARD PREP BIOCHEMISTRY SUPEREXAM Page 4 of 92


For inquiries visit www.topnotchboardprep.com.ph or email us at topnotchmedicalboardprep@gmail.com
TOPNOTCH MEDICAL BOARD PREP BIOCHEMISTRY SUPEREXAM
For inquiries visit www.topnotchboardprep.com.ph or email us at topnotchmedicalboardprep@gmail.com
Item QUESTION EXPLANATION AUTHOR TOPNOTCH
# EXAM
27 Characteristic of the genetic code that can account for Non-ambiguous/specific - a certain codon codes LESTER MIDTERM 1
silent mutations? always codes for the same amino acid; Universal - BRYAN CO EXAM -
A. Non-ambiguous codons always code for same amino acid from (TOP 10 - MARCH 2016
B. Universal generation to generation and between species; AUG 2015
C. Non-overlapping Redundant/degenerate - A given amino acid may have MED
D. Degenerate more than one triplet codon coding for it; Non- BOARDS;
E. Commaless overlapping/commaless - DNA is read from a starting TOPNOTCH
point as a continuous sequence of bases, taken three MD FROM
at a time without any gaps UST)
28 Which of the following is a nucleoside? SIMILAR TO BOARD EXAM CONCEPT. LESTER MIDTERM 1
A. Hypoxanthine Bases/Nucleobases: Nitrogenous bases = guanine, BRYAN CO EXAM -
B. Xanthine adenine, thymine, uracil, cytosine, xanthine, (TOP 10 - MARCH 2016
C. Inosine hypoxanthine; Nucleosides: base+sugar = guanosine, AUG 2015
D. cAMP adenosine, thymidine, cytidine, uridine, inosine; MED
E. Adenine Nucleotides: base+sugar+phosphate = ATP, GMP, UDO, BOARDS;
cAMP. 3 similar questions were asked during our TOPNOTCH
Biochemistry exam MD FROM
UST)
29 What is the rate-limiting step of ketogenesis? Rate limiting steps: Urea cycle - Carbamoyl phosphate LESTER MIDTERM 1
A. Carbamoyl phosphate synthetase I synthetase I; De novo pyrimidine synthesis - BRYAN CO EXAM -
B. Carbamoyl phosphate synthetase II Carbamoyl phosphate synthetase II; Fatty acid (TOP 10 - MARCH 2016
C. Acetyl CoA carboxylase synthesis - Acetyl-CoA carboxylase. Only 1 rate- AUG 2015
D. Acetyl CoA decarboxylase limiting enzyme type question appeared during our MED
E. HMG CoA synthase Biochemistry exam BOARDS;
TOPNOTCH
MD FROM
UST)
30 Alex is a thin 24-year old male who began avidly taking 2 B3 - niacin; B5 - pantothenic acid; B6 - pyridoxine; B7 LESTER MIDTERM 1
eggs every morning in order to build up his muscle bulk. - biotin; B9 - folic acid BRYAN CO EXAM -
This might put him at risk of a deficiency of which of the (TOP 10 - MARCH 2016
following vitamin? AUG 2015
A. B3 MED
B. B5 BOARDS;
C. B6 TOPNOTCH
D. B7 MD FROM
E. B9 UST)

31 Serum levels of which ketogenic amino acid are I - Isoleucine; L- Leucine; K - Lysine; V - Valine; Y - LESTER MIDTERM 1
increased in Maple Syrup Urine Disease? Tryptophan BRYAN CO EXAM -
A. I (TOP 10 - MARCH 2016
B. L AUG 2015
C. K MED
D. V BOARDS;
E. Y TOPNOTCH
MD FROM
UST)
32 True of alpha-thalassemia: In alpha thalassemia, one or more genes involved in LESTER MIDTERM 1
A. Beta chain accumulates and precipitates. alpha-globin production are defective, leading to BRYAN CO EXAM -
B. Gamma chain production is decreased. relative excess of beta globin and gamma globin (TOP 10 - MARCH 2016
C. It is transmitted in an autosomal dominant fashion. chains. Despite being a structural disease, it is AUG 2015
D. Hemoglobin C disease results if 3 alleles are affected. transmitted in a autosomal recessive manner. MED
E. Definitive diagnosis may be done through Hemoglobin H (beta globin tetramers) may result BOARDS;
karyotyping. from 3 deleted alleles. Hemoglobinopathies are TOPNOTCH
usually diagnosed by hemoglobin electrophoresis. MD FROM
UST)
33 Lipase whose activity is increased by glucagon? Pancreatic lipase - activated by trypsin; Lipoprotein LESTER MIDTERM 1
A. Pancreatic lipase lipase - activated by insulin; Hepatic lipase acts on the BRYAN CO EXAM -
B. Lipoprotein lipase remaining lipids carried on lipoproteins in the blood (TOP 10 - MARCH 2016
C. Hormone-sensitive lipase to regenerate LDL. AUG 2015
D. Hepatic lipase MED
E. None of the above BOARDS;
TOPNOTCH
MD FROM
UST)
34 Which of the following is a omega 3 fatty acid? The other choices are omega-6 fatty acids. LESTER MIDTERM 1
A. Linoleic acid BRYAN CO EXAM -
B. Linolenic acid (TOP 10 - MARCH 2016
C. Arachidonic acid AUG 2015
D. Oleic acid MED
E. All of the above BOARDS;
TOPNOTCH
MD FROM
UST)
35 A man manifests with short stature. His wife has normal The disease described fits the profile of Vitamin-D LESTER MIDTERM 1
stature, but all his daughters are short and bow-legged. resistant rickets, which is transmitted in a x-linked BRYAN CO EXAM -
All his sons are normal. The probable mode of dominant fashion. 1 genetic question in our (TOP 10 - MARCH 2016
inheritance of this disease is? Biochemistry exam. AUG 2015
A. Autosomal dominant MED
B. Autosomal recessive BOARDS;
C. Mitochondrial TOPNOTCH
D. X-linked dominant MD FROM
E. X-linked recessive UST)

TOPNOTCH MEDICAL BOARD PREP BIOCHEMISTRY SUPEREXAM Page 5 of 92


For inquiries visit www.topnotchboardprep.com.ph or email us at topnotchmedicalboardprep@gmail.com
TOPNOTCH MEDICAL BOARD PREP BIOCHEMISTRY SUPEREXAM
For inquiries visit www.topnotchboardprep.com.ph or email us at topnotchmedicalboardprep@gmail.com
Item QUESTION EXPLANATION AUTHOR TOPNOTCH
# EXAM
36 The Enzyme Commision and IUBMB classifies enzymes LESTER MIDTERM 1
based on their: BRYAN CO EXAM -
A. Functional groups (TOP 10 - MARCH 2016
B. Reactions catalyzed AUG 2015
C. Tertiary and Quaternary structure MED
D. Metabolic pathways they belong to BOARDS;
E. Locations in the body TOPNOTCH
MD FROM
UST)
37 Which of the following transporter systems utilize GLUT transporters facilitate diffusion, while CFTR use LESTER MIDTERM 1
secondary active transport? the primary active transport mechanism. BRYAN CO EXAM -
A. SGLT-1 transporter (TOP 10 - MARCH 2016
B. GLUT-2 transporter AUG 2015
C. GLUT-4 transporter MED
D. Cystic Fibrosis Transporter Receptor (CFTR) BOARDS;
E. GLUT-5 TOPNOTCH
MD FROM
UST)
38 Alcohol metabolism produces large amounts of NADH NADH shifts oxaloacetate to malate. LESTER MIDTERM 1
which inhibit gluconeogenesis by? BRYAN CO EXAM -
A. shifting the pyruvate-lactate equilibrium towards (TOP 10 - MARCH 2016
lactate AUG 2015
B. Favoring the producion of oxaloacetate from malate MED
C. Inhibiting the electron transport chain BOARDS;
D. All of the above TOPNOTCH
E. None of the above MD FROM
UST)
39 A ganglioside may contain all of the following except:
Glycosphinglipids do not contain phosphate. LESTER MIDTERM 1
A. a ceramide structure
Gangliosides, by definition, contain sialic acid BRYAN CO EXAM -
B. glucose or galactose (TOP 10 - MARCH 2016
C. phosphate AUG 2015
D. Sialic acid MED
E. Sphingosine BOARDS;
TOPNOTCH
MD FROM
UST)
40 Arsenic does not inhibit an enzyme in this metabolic Arsenic inhibits the pyruvate dehydrogenase enzyme LESTER MIDTERM 1
pathway? in the pyruvate decarboxylation pathway (Swanson BRYAN CO EXAM -
A. Pyruvate decarboxylation conversion. It also inhibits alpha-ketoglutarate (TOP 10 - MARCH 2016
B. Citric acid cycle dehydrogenase in the TCA cycle. AUG 2015
C. Oxidative phosphorylation MED
D. Pyruvate dehydrogenase pathway BOARDS;
E. None of the above TOPNOTCH
MD FROM
UST)
41 Which of the following is true? Brain cells prefer glucose as fuel source. Cardiac GEORGE MIDTERM 2
A. Brain cells prefer fatty acid as fuel source muscle prefer fatty acid as fuel source. Adipose cells MICHAEL EXAM -
B. Cardiac muscle prefer glucose as fuel source prefer fatty acid as fuel source. 3/4 to 4/5 of glycogen SOSUAN MARCH 2016
C. Adipose cells prefer ketone bodies as fuel source are stored in skeletal muscles. (TOP 5 -
D. One-half of the glycogen stores is in the muscles AUG 2015
E. NOTA MED
BOARDS;
TOPNOTCH
MD FROM
UST)
42 True 5-Fluorouracil GEORGE MIDTERM 2
A. Causes thymineless death MICHAEL EXAM -
B. Used as a chemotherapeutic agent for colonic SOSUAN MARCH 2016
carcinoma (TOP 5 -
C. Inhibitis thymidylate synthase AUG 2015
D. Both B and C MED
E. AOTA BOARDS;
TOPNOTCH
MD FROM
UST)
43 Tangier's disease is characterized by: Tangier's Disease is a rare genetic condition due to a GEORGE MIDTERM 2
A. Hypo-alphaprotenemia deficiency of HDL. This disease is also known as MICHAEL EXAM -
B. Deficiency of LDL cholesterol familial alpha lipoprotein deficiency. People affected SOSUAN MARCH 2016
C. Affected people have xathomas often have mild hypertrigylceridemia, neuropathy and (TOP 5 -
D. Common genetic condition premature atherosclerosis AUG 2015
E. NOTA MED
BOARDS;
TOPNOTCH
MD FROM
UST)
44 Most potent counter-regulatory hormone that stimulates GEORGE MIDTERM 2
gluconeogenesis MICHAEL EXAM -
A. Insulin SOSUAN MARCH 2016
B. Cortisol (TOP 5 -
C. Growth hormone AUG 2015
D. Epinephrine MED
E. Thyroxine BOARDS;
TOPNOTCH
MD FROM
UST)

TOPNOTCH MEDICAL BOARD PREP BIOCHEMISTRY SUPEREXAM Page 6 of 92


For inquiries visit www.topnotchboardprep.com.ph or email us at topnotchmedicalboardprep@gmail.com
TOPNOTCH MEDICAL BOARD PREP BIOCHEMISTRY SUPEREXAM
For inquiries visit www.topnotchboardprep.com.ph or email us at topnotchmedicalboardprep@gmail.com
Item QUESTION EXPLANATION AUTHOR TOPNOTCH
# EXAM
45 The normal glycogen stores last for how long? GEORGE MIDTERM 2
A. 6 hours MICHAEL EXAM -
B. 18 hours SOSUAN MARCH 2016
C. 26 hours (TOP 5 -
D. 36 hours AUG 2015
E. 48 hours MED
BOARDS;
TOPNOTCH
MD FROM
UST)
46 Gamma-glutamyl-cysteinyl-glycine is Gamma-glutamyl-cysteinyl-glycine is glutathionine. It GEORGE MIDTERM 2
A. A potent anti-oxidant is potent anti-oxidant and needs NADPH from PPP to MICHAEL EXAM -
B. Needs pentose-phosphate pathway to be be regenerated. SOSUAN MARCH 2016
regenerated (TOP 5 -
C. Similar to Vitamin E AUG 2015
D. Both A and B MED
E. AOTA BOARDS;
TOPNOTCH
MD FROM
UST)
47 Final common pathway by which electrons are passed to GEORGE MIDTERM 2
the mitochondria to generate ATP MICHAEL EXAM -
A. Glycolyis SOSUAN MARCH 2016
B. TCA (TOP 5 -
C. ETC AUG 2015
D. B-oxidation MED
E. Uronic pathway BOARDS;
TOPNOTCH
MD FROM
UST)
48 Which of the following is a complex 4 inhibitor Amobarbital - Complex 1 inhibitor, CO - complex 4 GEORGE MIDTERM 2
A. Amobarbital inhibitor, Antimycin A and Dimercaprol - complex 3 MICHAEL EXAM -
B. CO inhibitor. 2,4 dintrophenol - uncoupler SOSUAN MARCH 2016
C. Antimycin A (TOP 5 -
D. Dimercaprol AUG 2015
E. 2,4 dintrophenol MED
BOARDS;
TOPNOTCH
MD FROM
UST)
49 True of Central Dogma of Molecular Biology GEORGE MIDTERM 2
A. Translation --> Transcription --> replication MICHAEL EXAM -
B. Replication --> transcription --> translation SOSUAN MARCH 2016
C. Transcription --> translation --> replication (TOP 5 -
D. Translation --> replication --> transcription AUG 2015
E. NOTA MED
BOARDS;
TOPNOTCH
MD FROM
UST)
50 Most of dietary fat are composed of 90% of dietary fats are composed of triacylglycerol GEORGE MIDTERM 2
A. TAG MICHAEL EXAM -
B. Free fatty acid SOSUAN MARCH 2016
C. Cholesterol (TOP 5 -
D. Phospholipid AUG 2015
E. Glycerol MED
BOARDS;
TOPNOTCH
MD FROM
UST)
51 Lactic acid is considered to be a weak acid because: Weak acids like lactic acid never completely GEORGE MIDTERM 2
A. It is insoluble in water at standard temperature and dissociate. The indicated pKa of 5.2 is consistent with MICHAEL EXAM -
pressure. the idea that the lactate anion retains a strong affinity SOSUAN MARCH 2016
B. It fails to obey the Henderson-Hasselbalch equation. for protons, a hallmark of a weak acid. The lactate (TOP 5 -
C. The equilibrium between the acid and its conjugate anion is highly water-soluble. All weak acids obey the AUG 2015
base has a pKa of 5.2.. Henderson- Hasselbalch equation. MED
D. The lactate anion has minimal tendency to attract a BOARDS;
proton. TOPNOTCH
E. NOTA MD FROM
UST)
52 A noncompetitive enzyme inhibitor A noncompetitive inhibitor binds to the enzyme at a GEORGE MIDTERM 2
A. Decreases Vmax and increases Km. site other than MICHAEL EXAM -
B. Decreases Vmax and has no effect on Km. the substrate binding site, so it has little measurable SOSUAN MARCH 2016
C. Has no effect on Vmax or Km. effect on the enzyme’s affinity for substrate, as (TOP 5 -
D. Has no effect on Vmax and increases Km. represented by the Km. However, the inhibitor has the AUG 2015
E. NOTA effect of decreasing MED
the availability of active enzyme capable of catalyzing BOARDS;
the reaction, which manifests itself as a decrease in TOPNOTCH
Vmax. MD FROM
UST)

TOPNOTCH MEDICAL BOARD PREP BIOCHEMISTRY SUPEREXAM Page 7 of 92


For inquiries visit www.topnotchboardprep.com.ph or email us at topnotchmedicalboardprep@gmail.com
TOPNOTCH MEDICAL BOARD PREP BIOCHEMISTRY SUPEREXAM
For inquiries visit www.topnotchboardprep.com.ph or email us at topnotchmedicalboardprep@gmail.com
Item QUESTION EXPLANATION AUTHOR TOPNOTCH
# EXAM
53 Aspartame is composed of what amino acid? Aspartame: Phenylalanine + Methanol + Aspartic Acid GEORGE MIDTERM 2
A. Phenylalanine MICHAEL EXAM -
B. Asparagine SOSUAN MARCH 2016
C. Glutamic acid (TOP 5 -
D. Alanine AUG 2015
E. Tyrosine MED
BOARDS;
TOPNOTCH
MD FROM
UST)
54 Humans can't synthesize Vitamin C due to the absence of The inability of humans to synthesize L-ascorbic acid GEORGE MIDTERM 2
this enzyme is known to be due to a lack of L-gulono-gamma- MICHAEL EXAM -
A. Gulonolactone reductase lactone oxidase, an enzyme that is required for the SOSUAN MARCH 2016
B. Gulonolactone oxidase biosynthesis of this vitamin. (TOP 5 -
C. Gulonolactone ligase AUG 2015
D. Gulonolactone synthase MED
E. NOTA BOARDS;
TOPNOTCH
MD FROM
UST)
55 Certain drugs are thought to increase membrane fluidity Anesthetics are highly lipid-soluble and experiments GEORGE MIDTERM 2
directly, resulting in impaired neurotransmission that with isolated MICHAEL EXAM -
may be the basis for their therapeutic effects. Which membranes indicate that these molecules can dissolve SOSUAN MARCH 2016
class of drugs acts by this direct mechanism? in the hydrophobic center of the membrane bilayer. (TOP 5 -
A. Hallucinogens This causes a measurable increase in the membrane AUG 2015
B. Stimulants fluidity by disrupting MED
C. Sedatives the packed structure of phospholipids tails; thus, It BOARDS;
D. Opiates inhibits neurotransmission (pain sensations) in TOPNOTCH
E. Anesthetics neurons. Hallucinogens and opiates may also affect MD FROM
membrane fluidity, but their effects occur by indirect UST)
mechanisms, resulting from changes in the protein or
lipid composition of the membranes.
56 Rate-limiting step in urea cycle CPS 1 synthase - urea cycle GEORGE MIDTERM 2
A. CPS 1 synthase CPS 2 synthase - pyrimidine synthesis MICHAEL EXAM -
B. CPS 2 synthase Isocitarate dehydrogenase - TCA SOSUAN MARCH 2016
C. Isocitarate dehydrogenase (TOP 5 -
D. Both A and B AUG 2015
E. AOTA MED
BOARDS;
TOPNOTCH
MD FROM
UST)
57 Which of the following organs or tissues does NOT need Only the liver and kidneys can synthesize glucose by GEORGE MIDTERM 2
to be supplied with glucose gluconeogenesis. All the other organs listed are MICHAEL EXAM -
for energy production during a prolonged fast? dependent on provision of glucose from blood, either SOSUAN MARCH 2016
A. Lens supplied by the diet or by gluconeogenesis in liver and (TOP 5 -
B. Brain the kidneys. AUG 2015
C. RBCs MED
D. Liver BOARDS;
E. Cornea TOPNOTCH
MD FROM
UST)
58 The major pathway of amino groups disposal in human GEORGE MIDTERM 2
is by: MICHAEL EXAM -
A. Urea SOSUAN MARCH 2016
B. Ammonia (TOP 5 -
C. Creatinine AUG 2015
D. Uric acid MED
E. AOTA BOARDS;
TOPNOTCH
MD FROM
UST)
59 Bloatedness in lactose intolerance is due to GEORGE MIDTERM 2
A. Increased production of gas by the gut MICHAEL EXAM -
B. Lactose reduced to lactitol and converted to lactone SOSUAN MARCH 2016
C. Bacterial fermentation of undigested lactose in the (TOP 5 -
gut AUG 2015
D. Osmotically driven gfluid shift in the gut MED
E. AOTA BOARDS;
TOPNOTCH
MD FROM
UST)
60 Severe combined immunodeficiency are sensitive to dATP-induced allosteric inhibition of ribonucleotide GEORGE MIDTERM 2
allosteric inhibition of which of the following enzymes of reductase, which catalyzes reduction of the 2′- MICHAEL EXAM -
purine nucleotide metabolism? hydroxyl groups on ADP and GDP to form dADP and SOSUAN MARCH 2016
A. Xanthine oxidase dGDP (TOP 5 -
B. Dihydrofolate reductase AUG 2015
C. Adenosine deaminase MED
D. Ribonucleotide reductase BOARDS;
E. Hypoxanthine-guanine phosphoribosyltransferase TOPNOTCH
MD FROM
UST)

TOPNOTCH MEDICAL BOARD PREP BIOCHEMISTRY SUPEREXAM Page 8 of 92


For inquiries visit www.topnotchboardprep.com.ph or email us at topnotchmedicalboardprep@gmail.com
TOPNOTCH MEDICAL BOARD PREP BIOCHEMISTRY SUPEREXAM
For inquiries visit www.topnotchboardprep.com.ph or email us at topnotchmedicalboardprep@gmail.com
Item QUESTION EXPLANATION AUTHOR TOPNOTCH
# EXAM
61 Vitamin B1 or Thiamine pyrophosphate is a cofactor in Remember the mnemonics ATP. Vit B1 is a co factor of JAN MIDTERM 3
several of these enzymes/steps EXCEPT: Alpha-ketoglutarate DH of the TCA cycle, CHRISTIAN EXAM -
A. Pyruvate to Acetyl-Coenzyme A Transketolase (Phase 2 of the HMP shunt) and FELICIANO MARCH 2016
B. Pyruvate to Oxaloacetate Pyruvate DH complex (pyruvate to Acetyl-CoA). (TOP 2 -
C. Alpha-Keto Glutarate to Succinyl CoA Additonally it is also a co-factor of the the Branched AUG 2015
D. Hexose Monophosphate Shunt Phase 2 chain ketoacid DH step. Pyruvate to oxacocetate is MED
E. Branched chain ketoacid dehydrogenase faciltitated by pyruvate carboxylase which requires BOARDS;
biotin instead. TOPNOTCH
MD FROM
UST)
62 This amino acid is weakly basic but uncharged at SIMILAR TO PREVIOUS BOARD EXAM JAN MIDTERM 3
physiologic pH hence its important role in the CONCEPT/PRINCIPLE. The 3 basic amino acids are CHRISTIAN EXAM -
functioning of hemoglobin. lysine, arginine and histidine. Histidine is a weak base FELICIANO MARCH 2016
A. Proline but largely uncharged at physiologic pH. This is an (TOP 2 -
B. Arginine important property in the functioning of hemoglobin. AUG 2015
C. Lysine MED
D. Phenylalanine BOARDS;
E. Histidine TOPNOTCH
MD FROM
UST)
63 Which statement is true regarding the Krebs Henseleit The urea cycle occurs exclusively in the liver because JAN MIDTERM 3
cycle? it has arginase to cleave arginine into urea. The first 2 CHRISTIAN EXAM -
A. 90% occurs n the liver and 10% in the kidney steps occurs in the mitochondria whereas the FELICIANO MARCH 2016
B. The rate limiting step has an absolute requirement for remaining enzymes are located in the cytosol. The (TOP 2 -
N-acetylglutamate (NAG) as a positive allosteric second nitrogen of urea is from aspartate NOT AUG 2015
activator glutamate. 2 molecules of ATP are req'd to synthesize MED
C. The first 2 steps occurs in the cytoplasm and the rest carbamoly phosphate. Remember: An important BOARDS;
occurs in the mitochondria concept in the urea cycle is that NAG is an essential TOPNOTCH
D. The first nitrogen of urea is from free ammonia while activator of CPS I, the rate limiting step. MD FROM
the second nitrogen is donated by gluatamate UST)
E. Formation of carbamoyl phosphate is driven by
cleavage of 1 mol of ATP
64 The ff is true of proline EXCEPT? Cysteine and not proline contains a sulfhydryl group. JAN MIDTERM 3
A. It is considered a secondary amino acid Proline has a unique 5 membered ring structure CHRISTIAN EXAM -
B. It is an imino acid making it a secondary and an imino acid. Its unique FELICIANO MARCH 2016
C. It contains a sulfhydryl group that is an active part of and sturdy geometry interrupts the alpha helices of (TOP 2 -
many enzymes collagen. AUG 2015
D. It has an aliphatic side and a non polar side chain MED
E. Its unique geometry contributes to formation of BOARDS;
collagen TOPNOTCH
MD FROM
UST)
65 This complex of the electron transport chain is the site Complex IV or cytochrome oxidase is the site where JAN MIDTERM 3
where one-half of O2 is reduced to water. the the final electron acceptor which is 1/2 of oxygen CHRISTIAN EXAM -
A. Cytochrome oxidase is reduced to water FELICIANO MARCH 2016
B. ATP synthase (TOP 2 -
C. NADH dehydrogenase AUG 2015
D. Succinate dehydrogenase MED
E. Ubiquinol ferricytochrome oxidoreductase BOARDS;
TOPNOTCH
MD FROM
UST)
66 A patient is suspected of salicylate toxicitiy. Which of the SIMILAR TO PREVIOUS BOARD EXAM JAN MIDTERM 3
ff statement is true regarding its effects on the ETC? CONCEPT/PRINCIPLE.. Salicylates/aspirin are CHRISTIAN EXAM -
A. There is reduced permeability of the inner uncouplers that dissociates oxidation in the FELICIANO MARCH 2016
mitochondrial membrane to protons hence impaired respiratory chain from phosphorylation. There is (TOP 2 -
ATP synthesis increased permeability of the mitochondiral AUG 2015
B. Respiration dissociates from phosphorylation membrane to protons therefore ATP sysnthesis stops MED
C. The electron transport chain terminates along with but the respiratory chain continues prducing BOARDS;
ATP synthesis excessive heat hence hyperpyrexia. Oligomycin and TOPNOTCH
D. There is direct inhibition of Complex V hence ATP not aspirin is a Complex V inhibitor. MD FROM
synthesis terminates UST)
E. All of the above
67 Which of the ff statements regarding the carbohydrate Glucose, galactose, fructose and mannose are all JAN MIDTERM 3
structure is true? isomers having the chemical formula C6H12O6. CHRISTIAN EXAM -
A. Epimers are compounds that have the same chemical Epimers are compounds that differ around only 1 FELICIANO MARCH 2016
formula different only in structure specific carbon atom hence galactose and mannose (TOP 2 -
B. The L enantiomer is the major sugar of the human are NOT epimers bec they differ on 2 carbons atoms. AUG 2015
body The D enantiomer and not the L is utilzed by the body. MED
C. Galactose and mannose are epimers Anomers can mutarotate without need for enzymes. BOARDS;
D. Glucose and Fructose are isomers TOPNOTCH
E. Anomers are convertible between a linear form and MD FROM
ring form through specific enzymatic reactions UST)
68 The ff statement is true regarding glycosis EXCEPT Fructose 2-6 BP is actualy the most potent allosteric JAN MIDTERM 3
A. It is present in the cytoplasm of all cells activator of PFK-1, the rate limiting step of glycolysis. CHRISTIAN EXAM -
B. Phosphorylation of glucose is an irreversible step All the rest of the statements are true. FELICIANO MARCH 2016
C. 2 ATP molecules are initially utilized (TOP 2 -
D. Fructose 2,6 Bisphosphate is a potent inhibitor of the AUG 2015
rate limiting step MED
E. Only 2 molecules of ATP per glucose is produced BOARDS;
under anaerobic conditions TOPNOTCH
MD FROM
UST)

TOPNOTCH MEDICAL BOARD PREP BIOCHEMISTRY SUPEREXAM Page 9 of 92


For inquiries visit www.topnotchboardprep.com.ph or email us at topnotchmedicalboardprep@gmail.com
TOPNOTCH MEDICAL BOARD PREP BIOCHEMISTRY SUPEREXAM
For inquiries visit www.topnotchboardprep.com.ph or email us at topnotchmedicalboardprep@gmail.com
Item QUESTION EXPLANATION AUTHOR TOPNOTCH
# EXAM
69 Characteristics of glucokinase are the ff: SIMILAR TO PREVIOUS BOARD EXAM JAN MIDTERM 3
A. It is used as basal absorption of glucose by most CONCEPT/PRINCIPLE. Glucokinase despite its name CHRISTIAN EXAM -
tissues can also phosphorylate other hexoses. It has high Km FELICIANO MARCH 2016
B. It can also phosphorylate hexoses other than glucose (low affinity for glucose) and high Vmax and is (TOP 2 -
C. It is saturated under normal conditions present only in liver and pancreas. It is most active AUG 2015
D. It has high affinity for glucose compared to post-prandially. It is the first step but not the rate MED
hexokinase limiting step of glycolysis. Review the table BOARDS;
E. It is the rate limiting step of glycolysis highlighting the differences between hexokinase and TOPNOTCH
glucokinase. MD FROM
UST)
70 In glycogenolysis, the reason why the muscle tissue In the muscle, the enzyme, glucose-6-phosphatase is JAN MIDTERM 3
cannot produce free glucose for release into the blood ABSENT hence glucose 6 phosphate from CHRISTIAN EXAM -
stream is: glycogenolysis enters glycolysis instead. Remember FELICIANO MARCH 2016
A. The cell membrane is selectively impermeable to free that muscle can perform glycogenolysis but can only (TOP 2 -
glucose utilze the glucose for its own use. The other AUG 2015
B. Increased calcium levels in muscle tissue prevents statements are erroneous explanations. MED
exocytosis of free glucose BOARDS;
C. Absence of glucose-6-phosphatase TOPNOTCH
D. Glycogen phosphorylase is allosterically inactivated MD FROM
by increased glucose 6 phosphate levels UST)
E. Debranching enzyme is inacitve in metabolically
active msucles hence glucose is prefentially utilized
intracellularly
71 What step of the Krebs cycle utilizes substrate level The Krebs cycle has only 1 step that uses substrate JAN MIDTERM 3
phosphorylation? elvel phosphorylation to produce 1 mol of GTP namely CHRISTIAN EXAM -
A. Succinate to Fumarate Succinyl-Coa to Succinate by succinate thiokinase. FELICIANO MARCH 2016
B. Succinate to Oxaloacetate (TOP 2 -
C. Alpha keto glutarate to Succinyl CoA AUG 2015
D. Fumarate to Malate MED
E. None of the above BOARDS;
TOPNOTCH
MD FROM
UST)
72 In the fasting state which of the ff occurs? Expect PLENTY of questions regarding the JAN MIDTERM 3
A . Glycogen synthase is phosphorylated feeding/fasting cycle. Effects of insulin/glucagon? CHRISTIAN EXAM -
B. Decreased cAMP What enzymes are active when phosphorylated/ FELICIANO MARCH 2016
C. Increased phosphofructokinase-2 (PFK-2) dephosphorylated? Please master it by heart. In the (TOP 2 -
D. Increased fructose 2-6 Bisphosphate (F2,6 BP) fasting state, glucagon predominates to raise blood AUG 2015
E. Decreased activity of hormone senstive lipase glucose levels. Glucagon increases cAMP and protein MED
kinase A. It acts generally by PHOSPHORYLATING BOARDS;
enzymes. Take note: Glycogen synthase when TOPNOTCH
phosphorylated is INACTIVE hence glycogenesis is MD FROM
inhibited. There is also decreased activity of PFK2 and UST)
its product F2,6-BP which is the most important
alloesteric activator of glycolysis and inhibitor of
gluconeogenesis. Hormone senstive lipase is activated
by glucagon to release FFA for oxidation
73 A person ate a heavy meal 30 mins ago, which of the ff In the fed state, insulin predominates. In contrast to JAN MIDTERM 3
occurs? glucagon, insulin generally activates enzymes by CHRISTIAN EXAM -
A. Increased activity of carnitine acyltransferase DEPHOSPHORYLATION. Glycogen phosphorylase, in FELICIANO MARCH 2016
B. Decreased fructose 2-6 Bisphosphate (F2,6 BP) contrast to glycogen synthase, is active when (TOP 2 -
C. Most rare limiting enzymes are in the phosphorylated. Insulin upregulates F2,6BP to AUG 2015
dephosphorylated state stimulate glycolysis increasing NADH. Carnitine MED
D. Glycogen phosphorylase Is phosphorylated acyltranferase is the rate limiting step of fatty acid BOARDS;
E. Increased NAD/NADH ratio oxidation and is deactivated TOPNOTCH
MD FROM
UST)
74 All of the the ff are effects of glucagon EXCEPT? Again, glucagon (fasting state) generally acts by JAN MIDTERM 3
A. Activation of hormone sensitive lipase phosphorylating enzymes. Hormone senstive lipase is CHRISTIAN EXAM -
B. Increased activity of both ketogenesis and activated to release FFA for oxidation. Both FELICIANO MARCH 2016
ketogenolysis ketogenensis and ketogenolysis is active in the fasting (TOP 2 -
C. Allosteric activation of pyruvate carboxylase by state.Pyruvate carboxylase, an irreversible step in AUG 2015
acetyl-CoA gluconeogenesis is activated to increase free glucose MED
D. Glycogen phosphorylase Is phosphorylated levels. Glycogen phosphorlyase is active when BOARDS;
E. Activation of protein phosphatase phosphorylated. Protein phosphatase is the key TOPNOTCH
effector enzyme of insulin and acts to MD FROM
DEPHOSPHORYLATE other enzymes. UST)
75 What lipoprotein is responsible for reverse cholesterol HDL or the good cholesterol mediates reverse JAN MIDTERM 3
transport? cholesterol transport from periphery to liver. CHRISTIAN EXAM -
A. Chylomicron FELICIANO MARCH 2016
B. VLDL (TOP 2 -
C. LDL AUG 2015
D. HDL MED
E. IDL BOARDS;
TOPNOTCH
MD FROM
UST)
76 NAPDH produced from the HMP shunt is needed in the ff NADPH from HMP shunt is used for reductive JAN MIDTERM 3
pathways EXCEPT? reactions for FA and steroid synthesis, reduction of CHRISTIAN EXAM -
A. Fatty acid synthesis glutathione, NO synthesis, respiratory burst and FELICIANO MARCH 2016
B. Nitric oxide syntehsis CYP450 reactions. (TOP 2 -
C. Cholesterol synthesis AUG 2015
D. Heme synthesis MED
E. Regeneration of Glutathione BOARDS;
TOPNOTCH
MD FROM
TOPNOTCH MEDICAL BOARD PREP BIOCHEMISTRY SUPEREXAM Page 10 of 92
For inquiries visit www.topnotchboardprep.com.ph or email us at topnotchmedicalboardprep@gmail.com
TOPNOTCH MEDICAL BOARD PREP BIOCHEMISTRY SUPEREXAM
For inquiries visit www.topnotchboardprep.com.ph or email us at topnotchmedicalboardprep@gmail.com
Item QUESTION EXPLANATION AUTHOR TOPNOTCH
# EXAM
UST)

77 A patient has mental retardation and hypotonia. Further SIMILAR TO PREVIOUS BOARD EXAM JAN MIDTERM 3
examination revealed a cherry red spot on the macula. CONCEPT/PRINCIPLE. For some reason, Tay sachs CHRISTIAN EXAM -
Traube's space is obliterated. You are suspecting a type disease with characteristic cherry red spot is a FELICIANO MARCH 2016
of sphingolipidosis. What enzyme is probably deficient? favorite topic. Differentiate it from Niemann pick (TOP 2 -
A. Hexosaminadase A diease which also has cherry red spot but with AUG 2015
B. Hexosaminadase B hepatosplenomely as well. Enzyme dieficiencies are: MED
C. Glucocerebrosidase Tay sach's- Hexosaminidase A. Gaucher's- BOARDS;
D. Galactocerebrosidase Glucocerebrosidase. Krabbe's- Galactocerebrosidase. TOPNOTCH
E. Sphingomyelinase Nieman Pick- Sphingomyelinase MD FROM
UST)
78 This intermediate is common to both ketogenesis and SIMILAR TO PREVIOUS BOARD EXAM JAN MIDTERM 3
cholesterol synthesis? CONCEPT/PRINCIPLE. 3-hydroxy 3 methylglutaryl- CHRISTIAN EXAM -
A. 3-hydroxy 3 methylglutaryl-CoA CoA is termed simply as HMG-CoA and and its FELICIANO MARCH 2016
B. 3 hydroxybutyrate production serve as the first step of both cholesterol (TOP 2 -
C. Mevalonate 5-phosphate synthesis and ketogenesis. The other intermediates AUG 2015
D. Farnesyl pyrophosphate are part of ketogenesis or choelsterol synthesis but MED
E. Lanosterol not both. BOARDS;
TOPNOTCH
MD FROM
UST)
79 This is an example of a nucleoside. SIMILAR TO PREVIOUS BOARD EXAM JAN MIDTERM 3
A. Adenine CONCEPT/PRINCIPLE. Memorize the names of CHRISTIAN EXAM -
B. Cytidine nitrogenous bases with their corresponding FELICIANO MARCH 2016
C. Thymine nucleoside and nucleotide. Bases are adenine, (TOP 2 -
D. Adenosine monophosphate guanine, thymine, uracil, cytosine. Nucleoside (base AUG 2015
E. Uracil plus sugar) are adenosine, guanosine, thymidine, MED
uridine and cytidine. Nucelotide (nucleoside plus BOARDS;
phopshate group) are AMP, UMP, TMP etc. TOPNOTCH
MD FROM
UST)
80 It is a characteristic of eukaryotic mRNA. mRNA in eukaryotes are extensively modified: Exons JAN MIDTERM 3
A. Introns are spliced together are spliced together and introns are removed, a long CHRISTIAN EXAM -
B. Long sequence of adenine moelcules at the 5' end sequence of poly A tail at the 3' end and a 7 methyl- FELICIANO MARCH 2016
C. 3'-CCA sequence guanosyl group at 5' end is added. It also makes up (TOP 2 -
D. 7 methyl-guanosyl group at 3' end only 5% of the proportion of the RNA genome. 3'-CCA AUG 2015
E. Makes up only 5% of the proportion of the RNA sequence is a unique property of tRNA. MED
genome BOARDS;
TOPNOTCH
MD FROM
UST)
81 Which of the following is NOT true on how Calcium glycogen phosphorylase is activated by muscle ANDREW FINAL EXAM -
synchronizes the activation of glycogen phosphorylase? phosphorylase kinase TIU (TOP 1 MARCH 2016
a. cytosolic calcium is responsible for both contraction Harper’s Biochemistry 28th edition p. 160 - AUG 2015
and glycogenolysis MED
b. muscle phosphorylase kinase activates glycogen BOARDS;
phosphatase TOPNOTCH
c. phosphorylase kinase is made up of 4 subunits MD FROM
d. binding of calcium activates catalytic site of gamma CIM)
subunit
e. phosphorylated form is only fully activated in the
presence of calcium
82 2. Which of the following amino acids are not found in ANDREW FINAL EXAM -
glutathione? TIU (TOP 1 MARCH 2016
a. cysteine - AUG 2015
b. glutamic acid MED
c. glycine BOARDS;
d. aspartate TOPNOTCH
e. all of the above MD FROM
CIM)
83 3. Which of the following is added to LPG tanks to detect ANDREW FINAL EXAM -
leaks? TIU (TOP 1 MARCH 2016
a. propane - AUG 2015
b. ethyl mercaptan MED
c. methane BOARDS;
d. butane TOPNOTCH
e. acetone MD FROM
CIM)
84 Which of the following amino acids does not contain a Harper’s Biochemistry 28th edition p. 16 ANDREW FINAL EXAM -
basic group? TIU (TOP 1 MARCH 2016
a. arginine - AUG 2015
b. proline MED
c. lysine BOARDS;
d. histidine TOPNOTCH
e. none of the above MD FROM
CIM)

TOPNOTCH MEDICAL BOARD PREP BIOCHEMISTRY SUPEREXAM Page 11 of 92


For inquiries visit www.topnotchboardprep.com.ph or email us at topnotchmedicalboardprep@gmail.com
TOPNOTCH MEDICAL BOARD PREP BIOCHEMISTRY SUPEREXAM
For inquiries visit www.topnotchboardprep.com.ph or email us at topnotchmedicalboardprep@gmail.com
Item QUESTION EXPLANATION AUTHOR TOPNOTCH
# EXAM
85 5. According to the International Union of Biochemists ligases - joining together of 2 molecules coupled with ANDREW FINAL EXAM -
(IUB) which of the following refers to an enzyme that hydrolysis of ATP TIU (TOP 1 MARCH 2016
catalyzes cleavage of C-O bond through atom hydrolases - hydrolytic cleavage - AUG 2015
elimination? transferases - transfer of glycosyl, methyl, or MED
a. oxidoreductases phosphoryl groups, etc BOARDS;
b. transferases Harper’s Biochemistry 28th edition p. 52 TOPNOTCH
c. ligases MD FROM
d. lyases CIM)
e. hydrolases
86 6. An 8 year old child ingests Full cream milk. After 2 refers to lactase deficiency ANDREW FINAL EXAM -
hours, patient was noted to have bloating, flatulence, and Harper’s Biochemistry 28th edition p. 113 TIU (TOP 1 MARCH 2016
soft watery stools. Reduced sugars were found in stools. - AUG 2015
Which of the following is most likely true? MED
a. disaccharidase enzyme deficiency BOARDS;
b. inability to absorb lactose TOPNOTCH
c. inability to break down maltose MD FROM
d. autoimmune destruction of villi CIM)
e. none of the above
87 7. Which of the following is TRUE on regulation of Harper’s Biochemistry 28th edition p. 168 ANDREW FINAL EXAM -
glycolysis and gluconeogenesis in the liver? TIU (TOP 1 MARCH 2016
a. most potent positive allosteric activator of PFK 1 and - AUG 2015
inhibitor of fructose 1,6 bisphosphate is fructose 2,6 MED
bisphosphate BOARDS;
b. fructose 2,6 bisphosphate is formed by PFK -2 TOPNOTCH
c. PFK 2 and fructose 2,6 bisphosphatase are different MD FROM
enzymes CIM)
d. both A and B
e. both B and C
88 8. Which of the following is NOT induced by insulin in Harper’s Biochemistry 28th edition p. 168 ANDREW FINAL EXAM -
the regulatory and adaptive enzymes associated with TIU (TOP 1 MARCH 2016
carbohydrate metabolism? - AUG 2015
a. PFK 1 MED
b. pyruvate carboxylase BOARDS;
c. pyruvate kinase TOPNOTCH
d. glucokinase MD FROM
e. pyruvate dehydrogenase CIM)
89 9. Which of the following processes is NOT increased by Harper’s Biochemistry 28th edition p. 172 ANDREW FINAL EXAM -
glucagon? TIU (TOP 1 MARCH 2016
a. protein synthesis - AUG 2015
b. glycogenolysis MED
c. gluconeogenesis BOARDS;
d. ketogenesis TOPNOTCH
e. lipolysis MD FROM
CIM)
90 10. Which of the following is NOT TRUE of regulation of Free fatty acids are the precursors of ketone bodies ANDREW FINAL EXAM -
ketogenesis? Harper’s Biochemistry 28th edition p. 190 TIU (TOP 1 MARCH 2016
a. the factors regulating mobilization of free fatty acids - AUG 2015
are important in controlling ketogenesis MED
b. triacylglycerols are the precursors of ketone bodies in BOARDS;
the liver TOPNOTCH
c. regulation of entry of fatty acids into the oxidative MD FROM
pathway is through CPT-1 CIM)
d. impaired oxidation of fatty acids leads to diseases
associated with hypoglycemia
e. decreased insulin/glucagon ration promotes B -
oxidation of fatty acids
91 11. Which of the following is the regulatory step of bile Harper’s Biochemistry 28th edition p. 229 ANDREW FINAL EXAM -
acid synthesis? TIU (TOP 1 MARCH 2016
a. HMG Coa synthase - AUG 2015
b. HMG CoA reductase MED
c. 7 alpha - hydroxylase BOARDS;
d. sterol 27 hydroxylase TOPNOTCH
e. none of the above MD FROM
CIM)
92 12. A 6 year old African American presented with Here, glutamic acid, the normal amino acid in position ANDREW FINAL EXAM -
dyspnea, painful digits, splenomegaly, anemia and 6 of the B chain has been replaced by valine. Clearly, TIU (TOP 1 MARCH 2016
jaundice. Which of the following mutations is most likely this hinders normal function. - AUG 2015
present in this patient? Harper’s Biochemistry 28th edition p. 357 MED
a. nonsense BOARDS;
b. missense TOPNOTCH
c. frameshift MD FROM
d. silent CIM)
e. none of the above
93 13. WHich of the following is not a vitamin B12 Harper’s Biochemistry 28th edition p. 476 ANDREW FINAL EXAM -
dependent enzyme? TIU (TOP 1 MARCH 2016
a. methionine synthase - AUG 2015
b. methylmalonyl CoA mutase MED
c. leucine aminomutase BOARDS;
d. alpha ketoglutarate dehydrogenase TOPNOTCH
e. none of the above MD FROM
CIM)

TOPNOTCH MEDICAL BOARD PREP BIOCHEMISTRY SUPEREXAM Page 12 of 92


For inquiries visit www.topnotchboardprep.com.ph or email us at topnotchmedicalboardprep@gmail.com
TOPNOTCH MEDICAL BOARD PREP BIOCHEMISTRY SUPEREXAM
For inquiries visit www.topnotchboardprep.com.ph or email us at topnotchmedicalboardprep@gmail.com
Item QUESTION EXPLANATION AUTHOR TOPNOTCH
# EXAM
94 14. A neonate was found to have poor feeding with Harper’s Biochemistry 28th edition p. 493 ANDREW FINAL EXAM -
seizures. Biochemical tests revealed accumulation of TIU (TOP 1 MARCH 2016
very long chain fatty acids and marked reduction of - AUG 2015
plasmalogen. Mutations in genes encoding peroxisome MED
biogenesis has also been found. Which of the following is BOARDS;
the most likely diagnosis? TOPNOTCH
a. adrenoleukodystrophy MD FROM
b. Refsum’s disease CIM)
c. Zellweger syndrome
d. rhizomelic chondrodysplasia punctata
e. none of the above
95 15. Which of the following is NOT a feature of N - transfer can occur cotranslationally in the ANDREW FINAL EXAM -
glycosylation? endoplasmic reticulum TIU (TOP 1 MARCH 2016
a. the transfer of oligosaccharide occurs in Asn residues Harper’s Biochemistry 28th edition p. 516 - AUG 2015
b. protein bound oligosaccharide partially processed by MED
glucosidases and mannosidases BOARDS;
c. transfer of oligosaccharide occurs post translationally TOPNOTCH
in the golgi MD FROM
d. the enzyme which catalyzes the transfer of CIM)
oligosaccharide is inhibited by tunicamycin
e. none of the above
96 16. Which of the following is NOT true in the metabolism Harper’s Biochemistry 28th edition p. 562 ANDREW FINAL EXAM -
of skeletal muscle? TIU (TOP 1 MARCH 2016
a. skeletal muscle has receptors for glucagon - AUG 2015
b. glucose is resynthesized from lactate through Cori MED
cycle BOARDS;
c. free fatty acids are the major source of energy during TOPNOTCH
marathons MD FROM
d. alanine from muscle is destined for gluconeogenesis CIM)
e. skeletal muscle does not contain glucose 6
phosphatase
97 17. A 32 year old female presented with hepatic Harper’s Biochemistry 28th edition p. 573 ANDREW FINAL EXAM -
encephalopathy and Kayser Fleischer rings. Which of the TIU (TOP 1 MARCH 2016
following enzymes contain copper? - AUG 2015
a. amine oxidase MED
b. superoxide dismutase BOARDS;
c. cytochrome oxidase TOPNOTCH
d. tyrosinase MD FROM
e. all of the above CIM)
98 18. A neonate was seen in the NICU for delayed NADPH oxidase is used for oxygen dependent killing ANDREW FINAL EXAM -
separation of umbilical cord. Which of the following is of microbes TIU (TOP 1 MARCH 2016
not a biochemical feature of neutrophils? Harper’s Biochemistry 28th edition p. 604 - AUG 2015
a. active pentose phosphate pathway MED
b. contains CD11/CD18 integrins in plasma membrane BOARDS;
c. uses NADPH oxidase for oxygen independent killing of TOPNOTCH
microbes MD FROM
d. active glycolysis CIM)
e. moderately active oxidative phosphorylation
99 19. Which of the following is NOT true in the metabolism glycogenolysis is not the reverse of glycogenesis, but ANDREW FINAL EXAM -
of glycogen? is a separate pathway TIU (TOP 1 MARCH 2016
a. CAMP promotes simultaneous activation of Harper’s Biochemistry 28th edition p. 157 - AUG 2015
phosphorylase and inhibition of glycogen synthase MED
b. insulin inhibits glycogenolysis and stimulating BOARDS;
glycogenesis TOPNOTCH
c. glycogenesis and glycogenolysis are the same pathway MD FROM
d. glycogen biosynthesis involves UDP glucose CIM)
e. branching of glycogen involves detachment of existing
glycogen chains
100 20. Which of the following vitamins is given to a child to Harper’s Biochemistry 28th edition p. 469 ANDREW FINAL EXAM -
prevent metaplasia of surface epithelium? TIU (TOP 1 MARCH 2016
a. A - AUG 2015
b. B MED
c. C BOARDS;
d. D TOPNOTCH
e. E MD FROM
CIM)
101 Smith-Lemli-Opitz syndrome is a common autosomal SIMILAR TO PREVIOUS BOARD EXAM ANGELA DIAGNOSTIC
recessive multiple malformation syndrome. In this CONCEPT/PRINCIPLE. Pathway on page 24 of PAULINE P. EXAM - AUG
syndrome there is a deficiency in what enzyme?
Topnotch Handout. Smith-Lemli-Opitz syndrome CALIMAG- 2015
A. HMG-CoA reductase (SLOS) is a well-known malformation syndrome with LOYOLA
B. Acetyl-CoA acetyltransferase principal characteristics of psychomotor and growth (TOP 8 -
C. 3-hydroxy-3-methylglutaryl-CoA reductase retardation, cleft palate, hypospadias, postaxial FEB 2015
D. 7-dehydrocholesterol reductase polydactyly, and a distinctive craniofacial appearance MED
E. 3-hydroxy-3-methylglutaryl-CoA synthase consisting of microcephaly, ptosis, inner epicanthal BOARDS;
folds, anteverted nares, and micrognathia. Mutations TOPNOTCH
in the DHCR7 gene cause Smith-Lemli-Opitz MD FROM
syndrome.The DHCR7 gene provides instructions for UST)
making an enzyme called 7-dehydrocholesterol
reductase. This enzyme is responsible for the final
step in the production of cholesterol.

TOPNOTCH MEDICAL BOARD PREP BIOCHEMISTRY SUPEREXAM Page 13 of 92


For inquiries visit www.topnotchboardprep.com.ph or email us at topnotchmedicalboardprep@gmail.com
TOPNOTCH MEDICAL BOARD PREP BIOCHEMISTRY SUPEREXAM
For inquiries visit www.topnotchboardprep.com.ph or email us at topnotchmedicalboardprep@gmail.com
Item QUESTION EXPLANATION AUTHOR TOPNOTCH
# EXAM
102 Which of the following is the rate limiting step in SIMILAR TO PREVIOUS BOARD EXAM ANGELA DIAGNOSTIC
catecholamine synthesis?
CONCEPT/PRINCIPLE. Pathway on page 31 of PAULINE P. EXAM - AUG
A. DOPA decarboxylase Topnotch Handout. The rate limiting step is Tyrosine CALIMAG- 2015
B. Dopamine hydroxylase hydroxylase. LOYOLA
C. Tyrptophan hydroxylase (TOP 8 -
D. Phenethanolamine N-methyltransferase FEB 2015
E. Tyrosine hydroxylase MED
BOARDS;
TOPNOTCH
MD FROM
UST)
103 Sanfilippo syndrome is one of the autosomal recessive Page 44 of Topnotch Handout. Iduronate sulfatase is a ANGELA DIAGNOSTIC
mucopolysaccharidoses. It is characterized by mental deficiency in Hunters syndrome not in Sanfilippo. PAULINE P. EXAM - AUG
retardation, hyperactivity and eventual loss of motor CALIMAG- 2015
function. Each of the 4 types has a specific deficiency in LOYOLA
the following enzymes, EXCEPT:
(TOP 8 -
A. Iduronate sulfatase FEB 2015
B. heparan sulfamidase MED
C. N-acetylglucosaminidase BOARDS;
D. N-acetylglucosamine sulfatase TOPNOTCH
E. N-acetylglucosamine MD FROM
UST)
104 Which of the following is part of the Electron transport SIMILAR TO PREVIOUS BOARD EXAM ANGELA DIAGNOSTIC
chain and is also found in the Tricarboxylic acid CONCEPT/PRINCIPLE. Page 6 of Topnotch Handout. PAULINE P. EXAM - AUG
pathway: A. NADH dehydrogenase Succinate dehydrogenase is the enzyme in converting CALIMAG- 2015
B. Succinate dehydrogenase succinate to fumarate in the Kreb's cycle and its LOYOLA
C. Ubiquinol:ferricytochrome oxidoreductase product is FADH2. It is also known as Complex II of (TOP 8 -
D. Cytochrome oxidase the ETC. FEB 2015
E. Cytochrome a/a3 MED
BOARDS;
TOPNOTCH
MD FROM
UST)
105 A 27 y/o female patient went in for an executive check Page 27 of Topnotch Handout. LDL binds to specific ANGELA DIAGNOSTIC
up. Laboratory results showed elevated LDL. Which receptors on extrahepatic tissues and on the liver PAULINE P. EXAM - AUG
statement is true regarding LDL.:
where they are endocytosed. A,B and D are functions CALIMAG- 2015
A. Transports cholesterol from the peripheral tissues to of HDL. E is a function of Apo-E LOYOLA
the liver. (TOP 8 -
B. Delivers cholesterol to steroidogenic tissues via SR- FEB 2015
B1 MED
C. Taken up by tissues through receptor mediated BOARDS;
endocytosis TOPNOTCH
D. Shuttles apo-CII and apo-E in blood MD FROM
E. Mediates uptake of chylomicron remnant UST)
106 Further studies showed that the patient in the above Page 28 of Topnotch Handout. In Type II a Familial ANGELA DIAGNOSTIC
case suffers from Familial Hypercholesterolemia Type hypercholesterolemia there is a deficiency or defect in PAULINE P. EXAM - AUG
IIa this defect is brought about by which mechanism:
the LDL-receptors causing increased LDL. A and E are CALIMAG- 2015
A. LCAT deficiency defects in Familial a-Lipoprotein deficiency/ Tangiers LOYOLA
B. Defect in LDL-receptor disease. C is seen in Type I Familial lipoprotein lipase (TOP 8 -
C. Lipoprotein lipase deficiency deficiency. D is seen in Abetalipoproteinemia. FEB 2015
D. Decreased production of apo-B48 and 100 MED
E. Inefficient reverse cholesterol transport BOARDS;
TOPNOTCH
MD FROM
UST)
107 A 70 y/o male was brought to the ER presenting with Page 35 of Topnotch Handout. The patient is in ANGELA DIAGNOSTIC
severe congestive cardiac failure. He has been unwell for Respiratory alkalosis in response to the dyspnea PAULINE P. EXAM - AUG
about a week and has been vomiting for the previous 5 hence there will be a shift to the left of the oxygen CALIMAG- 2015
days. He was on no medication. He was hyperventilating binding curve. pH: pH is greater than 7.44 so an LOYOLA
and was very distressed. His vital signs are: BP 80/60, alkalaemia is present. The cause is an alkalosis (TOP 8 -
HR 85, RR 30. ABG showed pH 7.58, pCO2 21 mmHg, Pattern: pCO2 & bicarbonate are both low suggesting FEB 2015
pO2 154 mmHg, HCO3 14 mmol/l How will his current either a metabolic acidosis or a respiratory alkalosis. MED
condition affect the Oxygen saturation curve of As we already know an alkalosis is present then the BOARDS;
hemoglobin :
primary disorder is a respiratory alkalosis. Asessing TOPNOTCH
A. The oxygen binding curve will assume the shape of a the compensation for a respiratory alkalosis (using MD FROM
hyperbola similar to myoglobin rule 4 - "the 5 for 10" rule): The expected HCO3 is (24 UST)
B. There will be a rightward shift of the oxygen binding - 10) = 14.
curve Formulation: respiratory alkalosis
C. The position of the oxygen binding curve will not be
altered in any way
D. There will be a leftward shift of the oxygen binding
curve
E. Both A and C
108 Respiratory fatigue will eventually set in and bring Page 35 of Topnotch Handout. P50 is the pO2 at which ANGELA DIAGNOSTIC
about acidosis. This will affect the P50 value in the Hemoglobin is 50% saturated. PAULINE P. EXAM - AUG
oxyhemoglobin dissociation curve. The P50 value CALIMAG- 2015
represents the oxygen pressure at which hemoglobin: LOYOLA
A. is almost fully saturated with oxygen (TOP 8 -
B. releases oxygen at the capillary level FEB 2015
C. binds oxygen to 50% saturation of its binding sites MED
D. binds oxygen released by tissues BOARDS;
E. Both A and C TOPNOTCH
MD FROM
UST)

TOPNOTCH MEDICAL BOARD PREP BIOCHEMISTRY SUPEREXAM Page 14 of 92


For inquiries visit www.topnotchboardprep.com.ph or email us at topnotchmedicalboardprep@gmail.com
TOPNOTCH MEDICAL BOARD PREP BIOCHEMISTRY SUPEREXAM
For inquiries visit www.topnotchboardprep.com.ph or email us at topnotchmedicalboardprep@gmail.com
Item QUESTION EXPLANATION AUTHOR TOPNOTCH
# EXAM
109 Alpha-tocopherol has the following functions:
SIMILAR TO PREVIOUS BOARD EXAM ANGELA DIAGNOSTIC
A. Protects membrane lipids from peroxidation CONCEPT/PRINCIPLE. Page 47 of Topnotch Handout. PAULINE P. EXAM - AUG
B. Promotes epithelial cell proliferation and Alpha tocopherol is also known as Vitamin E. It CALIMAG- 2015
differentiation functions as an antioxidant in the lipid phase, protects LOYOLA
C. Increases calcium reabsorption in kidneys and bone membranes from peroxidation and prevents oxidation (TOP 8 -
resorption of LDL. B is a function of Vitamin A. C is a function of FEB 2015
D. Used as co-factors in redox reactions Vitamin D. D is a function of Vitamin B2/Riboflavin. E MED
E. Used as cofactor for acyl transfers is a function of Vitamin B5/Panthotenate. BOARDS;
TOPNOTCH
MD FROM
UST)
110 The importance of the Glucose Alanine cycle in the Page 39 of Topnotch Handout. The glucose alanine ANGELA DIAGNOSTIC
muscle in relation to amino acid metabolism is that:
cycle is used for the removal of excess nitrogen from PAULINE P. EXAM - AUG
A. It is the mechanism by which excess nitrogen from peripheral tissues through alanine. The excess CALIMAG- 2015
muscles is transfered to the liver by transamination of nitrogen will reach the liver through transamination LOYOLA
pyruvte of pyruvate to produce alanine (occurs in the muscle) (TOP 8 -
B. It is used for the synthesis of glucose in the muscle to then in the liver, alanine is converted back to pyruvate FEB 2015
replenish energy supply of the liver which may undergo gluconeogenesis and is transorted MED
C. The carbon skeletons of alanine will be back to the muscles. BOARDS;
transaminated to pyruvate for glycolysis TOPNOTCH
D. Pyruvate is reduced to lactate which is eventually MD FROM
transported to the liver UST)
E. All of the above

111 Which of the following is a true statement regarding the Page 39 of Topnotch Handout. Krebs-Henseleit cycle is ANGELA DIAGNOSTIC
Krebs-Henseleit cycle :
also known as the Ornithine or Urea cycle, which PAULINE P. EXAM - AUG
A. It is the final common pathway for the aerobic functions in the removal of nitrogenous waste CALIMAG- 2015
oxidation of ALL nutrients products in the body. A and B are both true for Kreb's LOYOLA
B. It provides majority of ATP for energy cycle. C is true for the ETC. (TOP 8 -
C. It occurs in the inner mitochondrial membrane FEB 2015
D. It is a pathway for removal of nitrogenous waste in MED
the body BOARDS;
E. Both A and B are correct TOPNOTCH
MD FROM
UST)
112 All of the following non-essential amino acids are Page 40 of Topnotch Handout. Alanine, aspartate, and ANGELA DIAGNOSTIC
synthesized from the transamination of alpha-ketoacids glutamate are synthesized from the transamination of PAULINE P. EXAM - AUG
except: A. Glutamate alpha-ketoacids. CALIMAG- 2015
B. Glycine LOYOLA
C. Alanine (TOP 8 -
D. Aspartate FEB 2015
E. All of the above MED
BOARDS;
TOPNOTCH
MD FROM
UST)
113 A 25 y/o male was diagnosed to have staghorn calculi. Page 41 of Topnotch Handout. Cystinuria is a common ANGELA DIAGNOSTIC
This condition may be due to a common inherited defect inherited defect of renal tubular amino acid PAULINE P. EXAM - AUG
of renal tubular amino acid transporter for the following transporter for cystine, ornithine, lysine and arginine CALIMAG- 2015
amino acids in the PCT of the kidneys, except:
in the PCT of the kidneys. Excess of cystine in the LOYOLA
A. Arginine urine may lead to precipitation of cystine kidney (TOP 8 -
B. Lysine stones and cause staghorn calculi. FEB 2015
C. Ornithine MED
D. Cystine BOARDS;
E. Cysteine TOPNOTCH
MD FROM
UST)
114 Fats are the best storage form of energy because they SIMILAR TO PREVIOUS BOARD EXAM ANGELA DIAGNOSTIC
have ___________ energy content :
CONCEPT/PRINCIPLE. Page 46 of Topnotch Handout. PAULINE P. EXAM - AUG
A. 6 kcal/gram Fats have 9 kcal/gram energy content. Carbohydrates, CALIMAG- 2015
B. 7 kcal/gram proteins and ketones contain 4 kcal/gram, while LOYOLA
C. 8 kcal/gram ethanol contains 7 kcal/gram. (TOP 8 -
D. 9 kcal/gram FEB 2015
E. 10 kcal/gram MED
BOARDS;
TOPNOTCH
MD FROM
UST)
115 A 17 y/o Japanese male who is fond of eating sushi was Page 48 of Topnotch Handout. D. Latum infestation ANGELA DIAGNOSTIC
noted to have diarrhea, abdominal pain, vomiting, which is usually obtained from eating raw fish can PAULINE P. EXAM - AUG
weight loss, and fatigue. He was diagnosed to have D. cause a deficiency in Vitamin B12/cobalamin. CALIMAG- 2015
Latum infestation, causing a deficiency in which of the LOYOLA
following: A. Iron (TOP 8 -
B. Folate FEB 2015
C. Vitamin A MED
D. Vitamin C BOARDS;
E. Cobalamin TOPNOTCH
MD FROM
UST)

TOPNOTCH MEDICAL BOARD PREP BIOCHEMISTRY SUPEREXAM Page 15 of 92


For inquiries visit www.topnotchboardprep.com.ph or email us at topnotchmedicalboardprep@gmail.com
TOPNOTCH MEDICAL BOARD PREP BIOCHEMISTRY SUPEREXAM
For inquiries visit www.topnotchboardprep.com.ph or email us at topnotchmedicalboardprep@gmail.com
Item QUESTION EXPLANATION AUTHOR TOPNOTCH
# EXAM
116 A 50 y/o male patient with colorectal cancer was given Page 51 of Topnotch Handout. 5-FU acts in several ANGELA DIAGNOSTIC
Fluorouracil. This is a pyrimidine analog drug which acts ways, but principally as a thymidylate synthase (TS) PAULINE P. EXAM - AUG
on rapidly dividing cancerous cells and they undergo cell inhibitor. Interrupting the action of this enzyme CALIMAG- 2015
death via thymineless death. Hence this drug is known to blocks synthesis of the pyrimidine thymidine, which is LOYOLA
act on the enzyme in the De Novo Pyrimidine synthesis a nucleoside required for DNA replication. (TOP 8 -
necessary for the:
Thymidylate synthase methylates deoxyuridine FEB 2015
A. Synthesis of carbamoyl phosphate monophosphate (dUMP) to form thymidine MED
B. Methylation of deoxyuridine monophosphate to monophosphate (dTMP). Administration of 5-FU BOARDS;
form thymidine monophosphate causes a scarcity in dTMP, so rapidly dividing TOPNOTCH
C. Formation of orotidine monophosphate cancerous cells undergo cell death via thymineless MD FROM
D. Synthesis of UTP and CTP death. UST)
E. All of the above

117 A 7 month old infant presents with presented with fever Page 52 of Topnotch Handout. Adenosine deaminase ANGELA DIAGNOSTIC
and paralysis of his left arm 3 months after receiving his deficiency leads to severe combined PAULINE P. EXAM - AUG
third oral poliovirus vaccine. Past history included immunodeficiency due to increased levels of dATP CALIMAG- 2015
chronic thrush presenting in the absence of antibiotic inhibiting ribonucleotide reductase and inhibiting LOYOLA
therapy or breastfeeding at 3 months, chronic diarrhea formation of deoxyribonucleotides. B is for Lesch- (TOP 8 -
from 4 months, and recurrent otitis media. He was at the Nyhan syndrome. C is seen in Orotic aciduria. D is for FEB 2015
90th percentile for height and weight, similar to his Menkes syndrome. E is in Tay-sachs. MED
parents. He was noted to have poor feeding and poor BOARDS;
weight gain. This is characteristic of SCID: TOPNOTCH
A. Adenosine deaminase deficiency MD FROM
B. HGPRT deficiency UST)
C. Low activity of orotidine phosphate decarboxylase
D. x-linked recessive disorder caused by deficiency of
ATP-dependent membrane transporter for copper
E. Hexosaminidase deficiency

118 DNA replication is a semi-conservative process where in Page 54 of Topnotch Handout. This occurs during the ANGELA DIAGNOSTIC
each strand serves as a template for the complementary S phase of the cell cycle. PAULINE P. EXAM - AUG
daughter strand and each strand becomes part of the CALIMAG- 2015
daugther strand. This process usually occurs during LOYOLA
which part of the cell cycle?
(TOP 8 -
A. G0 phase FEB 2015
B. G1 phase MED
C. G2 phase BOARDS;
D. S phase TOPNOTCH
E. M phase MD FROM
UST)
119 Which of the following antibiotics used in TB inhibits SIMILAR TO PREVIOUS BOARD EXAM ANGELA DIAGNOSTIC
RNA synthesis:
CONCEPT/PRINCIPLE. Page 59 of Topnotch Handout. PAULINE P. EXAM - AUG
A. Streptomycin Rifampicin binds to the Beta subunit of bacterial DNA- CALIMAG- 2015
B. Rifampicin dependent RNA polymerase and thereby inhibits RNA LOYOLA
C. Isoniazid synthesis. Streptomycin binds to the 30s subunit and (TOP 8 -
D. Pyrazinamide distorts its structure thereby interfering with the FEB 2015
E. Ethambutol initiation of protein synthesis. MED
BOARDS;
TOPNOTCH
MD FROM
UST)
120 This heat stable DNA polymerase used in PCR replicates Page 60 of Topnotch Handout. Taq polymerase is a ANGELA DIAGNOSTIC
the DNA sequence following each primer:
thermostable DNA polymerase named after the PAULINE P. EXAM - AUG
A. Swivelase thermophilic bacterium Thermus aquaticus. As an CALIMAG- 2015
B. Gyrase enzyme able to withstand the protein-denaturing LOYOLA
C. Taq polymerase conditions (high temperature) required during PCR. (TOP 8 -
D. Ligase Taq's optimum temperature for activity is 75–80°C, FEB 2015
E. Helicase with a half-life of greater than 2 hours at 92.5°C, 40 MED
minutes at 95°C and 9 minutes at 97.5°C, and can BOARDS;
replicate a 1000 base pair strand of DNA in less than TOPNOTCH
10 seconds at 72°C. MD FROM
UST)
121 The following amino acids are important in creatine Creatinine is formed in muscle from creatine LYNN MIDTERM 1
biosynthesis except: phosphate by irreversible, non-enzymatic dehydratio, DARYL EXAM - AUG
A. Methionine and loss of phosphate. Glycine, arginine, and FELICIANO 2015
B. Arginine methionine all participate in creatine biosynthesis. VILLAMAT
C. Glycine (Harpers) SIMILAR TO PREVIOUS BOARD EXAM ER, MD
D. Lysine CONCEPT (TOP 5 -
E. None of the above FEB 2015
MED
BOARDS;
TOPNOTCH
MD FROM
EAC)

TOPNOTCH MEDICAL BOARD PREP BIOCHEMISTRY SUPEREXAM Page 16 of 92


For inquiries visit www.topnotchboardprep.com.ph or email us at topnotchmedicalboardprep@gmail.com
TOPNOTCH MEDICAL BOARD PREP BIOCHEMISTRY SUPEREXAM
For inquiries visit www.topnotchboardprep.com.ph or email us at topnotchmedicalboardprep@gmail.com
Item QUESTION EXPLANATION AUTHOR TOPNOTCH
# EXAM
122 Which of the following is NOT a function of Vitamin C? Ascorbic acid role/functions are the following : LYNN MIDTERM 1
A. Antioxidant coenzyme in hydroxylation of proline and lysine in DARYL EXAM - AUG
B. Regulation of cell cycle collagen synthesis, antioxidant, enhances absorption FELICIANO 2015
C. It is necessary for conversion of dopamine to of iron. Regulation of cell cycle is a function of Biotin. VILLAMAT
norepinephrine (Harpers). "Ascorbic acid is the coenzyme for copper ER, MD
D. Coenzyme in hydroxylation of proline and lysine in containing hydroxylases and a ketoglutarate linked (TOP 5 -
collagen synthesis iron containing hydroxylases. Dopamine B- FEB 2015
E. None of the above hydroxylase is a copper-containing enzyme involved MED
in the synthesis of the catecholamines, BOARDS;
norepinephrine, and epinephrine, from tyrosine in the TOPNOTCH
adrenal medulla and central nervous system. During MD FROM
hydroxylation the Cu+ is oxidized to Cu2+; reduction EAC)
back to Cu+ specifically requires ascorbate, which is
oxidized to monodehydroascorbate. Harpers p. 495.
Regulation of cell cycle is a function of biotin. Harpers"
123 Which of the following is INCORRECT regarding Deficiency of lipoprotein lipase will cause severe LYNN MIDTERM 1
hormone-sensitive lipase? chylomicronemia (Harpers, Topnotch handouts) DARYL EXAM - AUG
A. It catalyzes hydrolysis of triacylglycerol stores in FELICIANO 2015
adipose tissue. VILLAMAT
B. Its activity is mainly inhibited by insulin by ER, MD
stimulating phosphodiesterase and lipase phosphatase. (TOP 5 -
C. Deficiency of this enzyme will cause severe FEB 2015
chylomicronemia. MED
D. It is activated by cAMP-dependent protein kinase BOARDS;
and removes fatty acid from carbon 1 and carbon 3 of TOPNOTCH
triacylglycerol. MD FROM
E. Its action yields free fatty acids and glycerol. EAC)
124 What is the rate-limiting step in catecholamine Tyrosine hydroxylase is the rate-enzyme if LYNN MIDTERM 1
synthesis? catecholamine biosynthesis; it uses DARYL EXAM - AUG
A. Phenylalanine hydroxylase tetrahydrobiopterin and molecular oxygen to convert FELICIANO 2015
B. DOPA decarboxylase tyrosine to DOPA. (Pubmed: Archives of Biochemistry VILLAMAT
C. Phenylethanolamine N-methyl transferase and Biophysics, April 2011). SIMILAR TO PREVIOUS ER, MD
D. Tyrosine hydroxylase BOARD EXAM CONCEPT (TOP 5 -
E. Dopamine beta-oxidase FEB 2015
MED
BOARDS;
TOPNOTCH
MD FROM
EAC)
125 Which of the following enzyymes is common to both All the enzymes listed are specific to either glycolysis LYNN MIDTERM 1
glycolysis and gluconeogenesis? or gluconeogenesis. Glucokinase/Hexokinase, DARYL EXAM - AUG
A. Phosphoglycerate kinase Phosphofructokinase, and pyruvate kinase catalyzed FELICIANO 2015
B. Fructose-1,6-biphosphatase irreversible reactions unique to glycolysis. Fructose- VILLAMAT
C. Glucokinase 1,6-biphosphatase and pyruvate carboxylase are ER, MD
D. Pyruvate carboxylase irreversible enzymes in gluconeogenesis. (TOP 5 -
E. Pyruvate kinase FEB 2015
MED
BOARDS;
TOPNOTCH
MD FROM
EAC)
126 The greatest quantitative source of high energy Oxidative phosphorylation occurs only in aerobic LYNN MIDTERM 1
phosphate in aerobic organism is? organism DARYL EXAM - AUG
A. Oxidative phosphorylation FELICIANO 2015
B. Substrate level phosphorylation VILLAMAT
C. Pentose Phosphate pathway ER, MD
D. Citric acid cycle (TOP 5 -
E. Beta-oxidation FEB 2015
MED
BOARDS;
TOPNOTCH
MD FROM
EAC)
127 Which of the following is true regarding myoglobin? Myoglobin follows a hyperbolic curve. Its main LYNN MIDTERM 1
A. It follows a sigmoidal curve. function is for oxygen storage. It is mainly found in DARYL EXAM - AUG
B. Its main function is for oxygen transport. heart and muscle. Its structure is composed of 1 FELICIANO 2015
C. It is mainly affected by allosteric receptors. polypeptide only. In contrast, hemoglobin follows a VILLAMAT
D. It is found in the the muscles and red blood cells. sigmoidal curve and shows cooperativity. It is mainly ER, MD
E. It shows saturation in the oxygen dissociation curve. found in the red blood cells and is affected by (TOP 5 -
allosteric effectors whose interaction with one site of FEB 2015
hemoglobin affects the binding of oxygen to heme MED
groups at other location. It has 4 polypeptides. BOARDS;
TOPNOTCH
MD FROM
EAC)
128 Which of the following is true regarding glycemic index? Carbohydrates that are absorbed less rapidly are food LYNN MIDTERM 1
A. It is described as the increase in blood glucose after that have low glycemic index and are thus more DARYL EXAM - AUG
a test dose of a carbohydrate compared with that after beneficial since it causes less fluctuation in insulin FELICIANO 2015
an equivalent amount of glucose. secretion. Fructose and sugar alcohols are examples VILLAMAT
B. Foods that high glycemic index are considered to be of carbohydrates with low glycemic index. ER, MD
more beneficial since they cause less fluctuation in (TOP 5 -
insulin secretion. FEB 2015
C. Carbohydrates that are absorbed less rapidly have a MED
higher glycemic index. BOARDS;
D. Fructose and sugar alcohols have high glycemic TOPNOTCH
index. MD FROM
TOPNOTCH MEDICAL BOARD PREP BIOCHEMISTRY SUPEREXAM Page 17 of 92
For inquiries visit www.topnotchboardprep.com.ph or email us at topnotchmedicalboardprep@gmail.com
TOPNOTCH MEDICAL BOARD PREP BIOCHEMISTRY SUPEREXAM
For inquiries visit www.topnotchboardprep.com.ph or email us at topnotchmedicalboardprep@gmail.com
Item QUESTION EXPLANATION AUTHOR TOPNOTCH
# EXAM
E. None of the above. EAC)

129 Which statement is true regarding competitive Only option D is correct. All others are description of LYNN MIDTERM 1
inhibition? non-competitive inhibitor. DARYL EXAM - AUG
A. Km is constant. FELICIANO 2015
B. Inhibitor binds to allosteric site of the enzyme and VILLAMAT
halts the enzyme catalysis. ER, MD
C. Vmax is lowered. (TOP 5 -
D. Reversal of the inhibition is through increasing the FEB 2015
substrate. MED
E. Inhibitor changes the shape of the enzyme so it BOARDS;
cannot bind to the substrate. TOPNOTCH
MD FROM
EAC)
130 The most important step in cholesterol excretion: The provision of glycerol 3-phosphate regulates LYNN MIDTERM 1
A. Bile acid secretion esterification. Hormone-sensitive lipase catalyzes DARYL EXAM - AUG
B. Provision of glycerol-3-phosphate hydrolysis of triacylglycerol to form fatty acids and FELICIANO 2015
C. Action of hormone-sensitive lipase glycerol. Biosynthesis of mevalonate (enzyme is HMG- VILLAMAT
D. Biosynthesis of mevalonate CoA reductase is part of cholesterol synthesis. ER, MD
E. Action of lipoproteins Lipoprotein are important in transport About 1 g of (TOP 5 -
cholesterol is eliminated from the body FEB 2015
per day. Approximately half is excreted in the feces MED
after BOARDS;
conversion to bile acids. The remainder is excreted as TOPNOTCH
cholesterol. MD FROM
EAC)
131 What is/are the effects of epinephrine in the body as a Epinephrine, together with glucagon, cortisol, and LYNN MIDTERM 1
result of stressful stimuli? growth hormone counteract the actions of insulin. It DARYL EXAM - AUG
A. Stimulation of phosphorylase via generation of cAMP increases gluconeogenesis and glycogenolysis. Cyclic FELICIANO 2015
B. Increase in glycogenolysis AMP is formed from ATP by adenylyl cyclase and acts VILLAMAT
C. Inactivation of pyruvate kinase as an intracellular 2nd messenger in response to ER, MD
D. All of these. hormones such as epinephrine, NE, and glucagon. (TOP 5 -
E. A & B only. Phosphorylase in muscle is activated in response to FEB 2015
epinephrine acting via cAMP. Pyruvate kinase is an MED
irreversible reaction in glycolysis and is inhibited by BOARDS;
high levels of epinephrine, glucagon and ATP. TOPNOTCH
MD FROM
EAC)
132 The main difference of glucokinase from hexokinase Only option A is correct. The rest are characteristics of LYNN MIDTERM 1
A. It promotes increased hepatic utilization of glucose hexokinase. Glucokinase is present only in liver and DARYL EXAM - AUG
following a meal. islet cells of pancreas, inhibited by fructose 6 FELICIANO 2015
B. It has a low Km and thus, a high affinity for glucose. phosphate, has high Km/low affinity, high Vmax, and VILLAMAT
C. It acts at a constant rate under all normal condition. removes glucose from blood following a meal. ER, MD
D. It has a lower Vmax compared to hexokinase. (TOP 5 -
E. It is saturated under normal conditions and acts to FEB 2015
provide glucose-6-phosphate to meet the cell's need. MED
BOARDS;
TOPNOTCH
MD FROM
EAC)
133 How many ATPs are produced by NADH in the liver? Liver, kidney and heart utilizes the Malate-aspartate LYNN MIDTERM 1
A. 1 shuttle producing 3 ATPs=1 NADH. Skeletal muscle DARYL EXAM - AUG
B. 2 and brain uses glycerol-phosphate shuttle 1 NADH=2 FELICIANO 2015
C. 3 ATP. (SIMILAR TO PREVIOUS BOARD EXAM VILLAMAT
D. 4 CONCEPT) ER, MD
E. 5 (TOP 5 -
FEB 2015
MED
BOARDS;
TOPNOTCH
MD FROM
EAC)
134 The first proponent of the double helix DNA model SIMILAR TO PREVIOUS BOARD EXAM CONCEPT LYNN MIDTERM 1
A. Watson-Crick DARYL EXAM - AUG
B. Wilson-Crick FELICIANO 2015
C. Wilkins-Chargaff VILLAMAT
D. Wilkins-Crick ER, MD
E. Watson-Chargaff (TOP 5 -
FEB 2015
MED
BOARDS;
TOPNOTCH
MD FROM
EAC)

TOPNOTCH MEDICAL BOARD PREP BIOCHEMISTRY SUPEREXAM Page 18 of 92


For inquiries visit www.topnotchboardprep.com.ph or email us at topnotchmedicalboardprep@gmail.com
TOPNOTCH MEDICAL BOARD PREP BIOCHEMISTRY SUPEREXAM
For inquiries visit www.topnotchboardprep.com.ph or email us at topnotchmedicalboardprep@gmail.com
Item QUESTION EXPLANATION AUTHOR TOPNOTCH
# EXAM
135 The process by which a particular segment of DNA is SIMILAR TO PREVIOUS BOARD EXAM CONCEPT LYNN MIDTERM 1
copied by the RNA polymerase? DARYL EXAM - AUG
A. Replication FELICIANO 2015
B. Translation VILLAMAT
C. Transcription ER, MD
D. Initiation (TOP 5 -
E. Elongation FEB 2015
MED
BOARDS;
TOPNOTCH
MD FROM
EAC)
136 The following are effects of insulin except: Effects of insulin: WELL FED STATE. Increased in LYNN MIDTERM 1
A. Stimulation of glycogen synthase. glycogenesis (glycogen synthase), decreased DARYL EXAM - AUG
B. Inhibition of phosphofructokinase-2 glycogenolysis (inhibition of glycogen FELICIANO 2015
C. Inhibition of glycogen phosphorylase phosphorylase), decreased gluconeogenesis VILLAMAT
D. Decreased activity of fructose 1,6 biphosphatase (decreased activity of Fructose-1,6-biphosphatase), ER, MD
E. Activation of carboxylation of acetyl-CoA to malonyl- increase fatty acid synthesis (activation of acetyl CoA (TOP 5 -
CoA carboxylase involved in Acetyl coa --> Malonyl CoA). FEB 2015
Option B is the answer. Insulin activates PFK-2 which MED
activates PFK-1, which catalyzes the rate-limiting step BOARDS;
in glycolysis TOPNOTCH
MD FROM
EAC)
137 A 20-month old child born full terrm via vaginal delivery, β-galactosidase -- Krabbes; Ceramidase --Farber'ss LYNN MIDTERM 1
was brought to the hospital due to inability to sit up. She disease; Sphingomyelinase --Niemann-Pick; α- DARYL EXAM - AUG
was diagnosed to have Tay-Sach's disease. Which of the galactosidase -- Fabry's SIMILAR TO PREVIOUS FELICIANO 2015
following enzymes is deficient in this condition? BOARD EXAM CONCEPT VILLAMAT
A. Hexosaminidase A ER, MD
B. β-galactosidase (TOP 5 -
C. Ceramidase FEB 2015
D. Sphingomyelinase MED
E. α-galactosidase BOARDS;
TOPNOTCH
MD FROM
EAC)
138 A 10-month old infant was admitted due to vomiting and Fructose intolerance is an autosomal recessive LYNN MIDTERM 1
decreased sensorium after her mother gave him an disorder resulting from deficiency in Aldolase B. DARYL EXAM - AUG
orange juice. On history, he was born full term after an Patient present wirh hypoglycemia, jaundice, cirrhosis FELICIANO 2015
uncomplicated pregnancy, solely breastfed for the first 6 and vvomiting due to accumulation of fructose 1-P VILLAMAT
months. Urine was noted to contain non-reducing phosphate. (Decrease glycogenolysis, and decrease ER, MD
sugars. What is the patient's most likely condition? gluconeogenesis). In Essential fructosuria, there is a (TOP 5 -
A. Essential fructosuria defect in fructokinase. However, it is more benign and FEB 2015
B. Classic Galactosemia asymptomatic. Galactokinase deficiency presents with MED
C. Aldolase B deficiency galactosuria and cataracts in early childhood. Classic BOARDS;
D. Galactokinase deficiency galactosemia may present with vomiting, diarrhea TOPNOTCH
E. Aldose reductase deficiency after milk ingestion (not after ingestion of fruit juice). MD FROM
EAC)

139 Which of the following vitamins has a protective effect SIMILAR TO PREVIOUS BOARD EXAM CONCEPT. LYNN MIDTERM 1
by combating lipid peroxidation? DARYL EXAM - AUG
A. Vitamin A FELICIANO 2015
B. Pyridoxine VILLAMAT
C. Ascorbic Acid ER, MD
D. Vitamin D (TOP 5 -
E. Vitamin E FEB 2015
MED
BOARDS;
TOPNOTCH
MD FROM
EAC)
140 Which of the following antibiotics prevents binding of Chloramphenicol - inhibits peptidyltransferase; LYNN MIDTERM 1
aminoacyl-tRNA to the A site? Streptomycin - interferes with initiation of protein DARYL EXAM - AUG
A. Chloramphenicol synthesis, Rifampicin - binds to bacterial RNA FELICIANO 2015
B. Streptomycin polymerase, inhibiting RNA synthesis; Clindamycin - VILLAMAT
C. Tetracycline inhibit translocation ER, MD
D. Rifampicin (TOP 5 -
E. Clindamycin FEB 2015
MED
BOARDS;
TOPNOTCH
MD FROM
EAC)
141 Which of the following is not a participant in creatinine SIMILAR TO PREVIOUS BOARD EXAM EDWARD MIDTERM 2
formation? CONCEPT/PRINCIPLE. The AA lysine is not involved in HARRY EXAM - AUG
A. Lysine creatinine formation VALLAJER 2015
B. Arginine A, MD (TOP
C. Methionine 8 - FEB
D. Glycine 2015 MED
E. None of the above BOARDS;
TOPNOTCH
MD FROM
PERPETUA
L BINAN)

TOPNOTCH MEDICAL BOARD PREP BIOCHEMISTRY SUPEREXAM Page 19 of 92


For inquiries visit www.topnotchboardprep.com.ph or email us at topnotchmedicalboardprep@gmail.com
TOPNOTCH MEDICAL BOARD PREP BIOCHEMISTRY SUPEREXAM
For inquiries visit www.topnotchboardprep.com.ph or email us at topnotchmedicalboardprep@gmail.com
Item QUESTION EXPLANATION AUTHOR TOPNOTCH
# EXAM
142 Which of the following enzymes is elevated in Gout? The activity of xanthine oxidase is elevated in gout EDWARD MIDTERM 2
A. HGPRT because elevated activity of this enzyme causes HARRY EXAM - AUG
B. Adenosine deaminase increased formation of uric acid thus precipitating VALLAJER 2015
C. Xanthine Oxidase attacks of gout. The rest are enzymes which shunt A, MD (TOP
D. A & B purine derivatives back into the available nucleic acid 8 - FEB
E. None of the above pool and thus deplete xanthine oxidase of its 2015 MED
substrate. BOARDS;
TOPNOTCH
MD FROM
PERPETUA
L BINAN)
143 The sequencing of a genetic make up of an individual is Genetics is the study of genes, heredity, and genetic EDWARD MIDTERM 2
known as? variation in living organisms. Genomics is the study of HARRY EXAM - AUG
A. Genetics the genetic make-up of an individual VALLAJER 2015
B. Genomics A, MD (TOP
C. Gene Therapy 8 - FEB
D. Genetic Engineering 2015 MED
E. None of the above BOARDS;
TOPNOTCH
MD FROM
PERPETUA
L BINAN)
144 What is the principal cholesterol excreted in human Cholic acid and chenodeoxycholic acid are primary EDWARD MIDTERM 2
feces? bile acids secreted in bile, lithocholic acid is a HARRY EXAM - AUG
A. Cholic Acid secondary bile acid. Coprostanol is the primary VALLAJER 2015
B. Chenodeoxycholic acid cholesterol in the stool synthesized from cholesterol A, MD (TOP
C. Coprostanol from the GIT by the gut bacteria. 8 - FEB
D. Lithocholic acid 2015 MED
E. None of the above BOARDS;
TOPNOTCH
MD FROM
PERPETUA
L BINAN)
145 How many base pairs are there in the human genome? Reference: Harper's biochem EDWARD MIDTERM 2
A. 3 x 1010 HARRY EXAM - AUG
B. 3 x 1011 VALLAJER 2015
C. 3 x 1012 A, MD (TOP
D. 3 x 109 8 - FEB
E. 3 x 108 2015 MED
BOARDS;
TOPNOTCH
MD FROM
PERPETUA
L BINAN)
146 What is the enzyme is defective in Smith-Lemli-Opitz SIMILAR TO PREVIOUS BOARD EXAM EDWARD MIDTERM 2
syndrome? CONCEPT/PRINCIPLE. The enzyme 7- HARRY EXAM - AUG
A. HMG-CoA reductase dehydrocholesterol reductase is deficient in patients VALLAJER 2015
B. 25-Hydroxyvitamin D3 1-alpha-hydroxylase with Smith-Lemli-Opitz syndrome A, MD (TOP
C. HMG CoA synthase 8 - FEB
D. Acetyl CoA carboxylase 2015 MED
E. 7-dehydrocholesterol reductase BOARDS;
TOPNOTCH
MD FROM
PERPETUA
L BINAN)
147 Which of the following cells do not synthesize heme RBCs when they reach maturity do not synthesize EDWARD MIDTERM 2
despite being mature? heme anymore as they rely on the preformed heme HARRY EXAM - AUG
A. Erythroid Bone Marrow inside the cytoplasm, the other options are the VALLAJER 2015
B. RBC progenitor sources of the RBCs and therefore A, MD (TOP
C. Yolk Sac synthesize heme. 8 - FEB
D. Liver 2015 MED
E. Spleen BOARDS;
TOPNOTCH
MD FROM
PERPETUA
L BINAN)
148 RM, a 30 year old political activist was imprisoned, he At this point, ketogenesis predominates which EDWARD MIDTERM 2
was brought to the infirmary due to weakness, upon produces ketone bodies to serve as the body's main HARRY EXAM - AUG
questioning, you learned that he was deliberately energy source. VALLAJER 2015
withheld food for 2 weeks, thus when the brain is A, MD (TOP
deprived of glucose at this point in time, which of the 8 - FEB
following is expected to happen? 2015 MED
A. Glycolysis BOARDS;
B. Fatty acid synthesis TOPNOTCH
C. Cholesterol synthesis MD FROM
D. Ketogenesis PERPETUA
E. A & B L BINAN)

TOPNOTCH MEDICAL BOARD PREP BIOCHEMISTRY SUPEREXAM Page 20 of 92


For inquiries visit www.topnotchboardprep.com.ph or email us at topnotchmedicalboardprep@gmail.com
TOPNOTCH MEDICAL BOARD PREP BIOCHEMISTRY SUPEREXAM
For inquiries visit www.topnotchboardprep.com.ph or email us at topnotchmedicalboardprep@gmail.com
Item QUESTION EXPLANATION AUTHOR TOPNOTCH
# EXAM
149 RVD, a 60 year old man with hypertension and diabetes Anaerobic glycolysis yields only 2 ATP per molecule of EDWARD MIDTERM 2
mellitus came in due to chest pain, you know that the glucose HARRY EXAM - AUG
cause of his chest pain is lactic acid accumulation in the VALLAJER 2015
heart muscle. How many ATPs are generated by the A, MD (TOP
cardiac muscle in anaerobic respiration? 8 - FEB
A. 4 2015 MED
B. 2 BOARDS;
C. 16 TOPNOTCH
D. 8 MD FROM
E. 10 PERPETUA
L BINAN)
150 Pyruvate is oxidized to which substrate to enter the Acetyl CoA is formed when Pyruvate is oxidized by EDWARD MIDTERM 2
Kreb's cycle? pyruvate dehydrogenase in aerobic respiration, in the HARRY EXAM - AUG
A. Acetyl CoA absence of aerobic respiration as in hypoxia or lack of VALLAJER 2015
B. Lactic Acid mitochondria (e.g. RBC) the pyruvate is not oxidized A, MD (TOP
C. Phosphoenolpyruvate but is reduced by LDH to lactate. 8 - FEB
D. Glyceraldhyde 3-Phosphate 2015 MED
E. Oxaloacetate BOARDS;
TOPNOTCH
MD FROM
PERPETUA
L BINAN)
151 What is the complex amino acid alcohol found in Sphingosine is the complex amino acid alcohol found EDWARD MIDTERM 2
sphingomyelin? in sphingomyelin, ceramide is a combination of HARRY EXAM - AUG
A. Sphingosine sphingosine and a fatty acid. VALLAJER 2015
B. Ceramide A, MD (TOP
C. Phosphatidylserine 8 - FEB
D. Phosphatidylinositol 2015 MED
E. Lysophosphatidylcholine BOARDS;
TOPNOTCH
MD FROM
PERPETUA
L BINAN)
152 Sickle cell is an example of point mutation in the gene Valine is the amino acid that substituted glutamic acid EDWARD MIDTERM 2
coding for the globin molecule. What is the amino acid in the normal Hgb molecule. HARRY EXAM - AUG
implicated in the polymerization of hemoglobin in sickle VALLAJER 2015
cell disease A, MD (TOP
A. Aspartic acid 8 - FEB
B. Glutamic acid 2015 MED
C. Isoleucine BOARDS;
D. Valine TOPNOTCH
E. Phenylalanine MD FROM
PERPETUA
L BINAN)
153 An infant was brought to your clinic because of yellowish Crigler-Najjar and Gilbert's syndrome are defects in EDWARD MIDTERM 2
discoloration of the skin, upon PE, you noted jaundice. the conjugation of bilirubin while Rotor syndrome HARRY EXAM - AUG
Labs indicated elevated levels of direct bilirubin and also causes elevated direct bilirubin but the liver VALLAJER 2015
liver biopsy showed darkly stained hepatocytes, what is biopsy shows that the hepatocytes are not darkly A, MD (TOP
the most likely cause? stained unlike in Dubin-Johnson syndrome. 8 - FEB
A. Crigler-Najjar syndrome 2015 MED
B. Dubin-Johnson syndrome BOARDS;
C. Rotor syndrome TOPNOTCH
D. Gilbert's syndrome MD FROM
E. None of the above PERPETUA
L BINAN)
154 A patient came in to the ER suspected of suffering from a Clopidogrel inhibits the ADP receptor on platelet cell EDWARD MIDTERM 2
MI, you know that the mechanism of action of which of membranes HARRY EXAM - AUG
the following involves the inhibition of ADP binding to VALLAJER 2015
its receptor in the platelet cell membrane? A, MD (TOP
A. Aspirin 8 - FEB
B. Tirofiban 2015 MED
C. Abciximab BOARDS;
D. Clopidogrel TOPNOTCH
E. Dipyridamole MD FROM
PERPETUA
L BINAN)
155 These are enzymes whose main function is to catalyze EDWARD MIDTERM 2
the union of 2 molecules with hydrolysis of ATP HARRY EXAM - AUG
A. Ligases VALLAJER 2015
B. Lyases A, MD (TOP
C. Hydrolases 8 - FEB
D. Oxidoreductases 2015 MED
E. None of the above BOARDS;
TOPNOTCH
MD FROM
PERPETUA
L BINAN)
156 All of the following require the co-factor biotin except: It is catalyzed by pyruvate dehydrogenase and does EDWARD MIDTERM 2
A. pyruvate ---> oxaloacetate not require biotin as co-factor. HARRY EXAM - AUG
B. pyruvate ---> acetyl CoA VALLAJER 2015
C. acetyl CoA ---> malonyl CoA A, MD (TOP
D. propionyl CoA ---> succinyl CoA 8 - FEB
E. None of the above 2015 MED
BOARDS;
TOPNOTCH
TOPNOTCH MEDICAL BOARD PREP BIOCHEMISTRY SUPEREXAM Page 21 of 92
For inquiries visit www.topnotchboardprep.com.ph or email us at topnotchmedicalboardprep@gmail.com
TOPNOTCH MEDICAL BOARD PREP BIOCHEMISTRY SUPEREXAM
For inquiries visit www.topnotchboardprep.com.ph or email us at topnotchmedicalboardprep@gmail.com
Item QUESTION EXPLANATION AUTHOR TOPNOTCH
# EXAM
MD FROM
PERPETUA
L BINAN)

157 Our understanding today of DNA was made possible James Watson and Francis Crick were the first to EDWARD MIDTERM 2
through the contribution of? describe the double helical structure of DNA. HARRY EXAM - AUG
A. Oliver and Wilbur Wright VALLAJER 2015
B. Jane Goodall and Marie Leakey A, MD (TOP
C. Gregor Mendel and James Watson 8 - FEB
D. James Watson and Francis Crick 2015 MED
E. None of the above BOARDS;
TOPNOTCH
MD FROM
PERPETUA
L BINAN)
158 2,3-Bisphosphoglycerate formation is catalyzed by Bisphosphoglycerate mutase catalyzes the conversion EDWARD MIDTERM 2
which enzyme? of 1,3-BPG to 2,3-BPG HARRY EXAM - AUG
A. Phosphoglycerate kinase VALLAJER 2015
B. Phosphoglycerate mutase A, MD (TOP
C. Phosphotriose isomerase 8 - FEB
D. Bisphosphoglycerate mutase 2015 MED
E. Glyceraldehyde 3-phosphate dehydrogenase BOARDS;
TOPNOTCH
MD FROM
PERPETUA
L BINAN)
159 RDL, a 17 year old female was brought to you due to CO2 and H2O are the substrates of carbonic anhydrase EDWARD MIDTERM 2
stunted growth and recurrent fractures, a diagnosis of to form carbonic acid which in an aqueous solution HARRY EXAM - AUG
osteopetrosis was made, biopsy showed osteoclasts dissociates into H ions and bicarbonate ions VALLAJER 2015
lacking carbonic anhydrase activity, what are the A, MD (TOP
substrates of this enzyme? 8 - FEB
A. H+ and sodium 2015 MED
B. Potassium and HCO3- BOARDS;
C. CO2 and HCl TOPNOTCH
D. CO2 and H2O MD FROM
E. None of the above PERPETUA
L BINAN)

160 Acetyl Co-A combines with which substrate to enter the The first step in Kreb's cycle is when Acetyl Co-A EDWARD MIDTERM 2
Kreb's cycle? combines with oxaloacetate to form citrate catalyzed HARRY EXAM - AUG
A. Oxaloacetate by citrate synthase VALLAJER 2015
B. Malonate A, MD (TOP
C. Citrate 8 - FEB
D. Succinate 2015 MED
E. Alpha keto-glutarate BOARDS;
TOPNOTCH
MD FROM
PERPETUA
L BINAN)
161 Oxidative phosphorylation which involves several oligomycin inhibits complex 5 or atp synthase. HAROLD MIDTERM 3
compounds and complexes that transfer electron Barbiturates inhibit complex 1. cyanide and hydrogen JAY S. EXAM - AUG
between mitochondrial membranes is an efficient sulfate inhibits complex 4. malonate inhibits complex BAYTEC, 2015
process of ATP production in aerobic species. Therefore 3. MD (TOP
when one of its components is blocked, ATP production 10 - FEB
may cause pathologic disturbance. Which of the 2015 MED
following compounds inhibits ATP synthase? BOARDS;
A.Barbiturates TOPNOTCH
B.Oligomycin MD FROM
C.Cyanide FEU)
D.Malonate
E.Hydrogen Sulfate

162 How many net ATPs will be produced in a single glucose 4 will be produced 2 ATPs will be used (SIMILAR TO HAROLD MIDTERM 3
molecule when a cell undergoes anaerobic glycolysis? PREVIOUS BOARD EXAM CONCEPT/PRINCIPLE) JAY S. EXAM - AUG
A. 2 BAYTEC, 2015
B. 3 MD (TOP
C. 4 10 - FEB
D. 5 2015 MED
E. 0 BOARDS;
TOPNOTCH
MD FROM
FEU)
163 3. NADH is an important compound for ATP production 2 NADH will be produced but they will be used in HAROLD MIDTERM 3
in both aerobic and anaerobic glycolysis. What is the net converting pyruvate to lactate (SIMILAR TO JAY S. EXAM - AUG
NADH produced in a cell undergoing anaerobic PREVIOUS BOARD EXAM CONCEPT/PRINCIPLE) BAYTEC, 2015
glycolysis? MD (TOP
A. 0 10 - FEB
B. 1 2015 MED
C. 2 BOARDS;
D. 3 TOPNOTCH
E. 4 MD FROM
FEU)

TOPNOTCH MEDICAL BOARD PREP BIOCHEMISTRY SUPEREXAM Page 22 of 92


For inquiries visit www.topnotchboardprep.com.ph or email us at topnotchmedicalboardprep@gmail.com
TOPNOTCH MEDICAL BOARD PREP BIOCHEMISTRY SUPEREXAM
For inquiries visit www.topnotchboardprep.com.ph or email us at topnotchmedicalboardprep@gmail.com
Item QUESTION EXPLANATION AUTHOR TOPNOTCH
# EXAM
164 4. Which of the following enzyme is both involved in phosphoglycerate kinase can convert 1,3- HAROLD MIDTERM 3
gluconeogenesis and glycolysis? bisphosphoglycerate to 3-phosphoglycerate in JAY S. EXAM - AUG
A. PFK 1 glycolysis and vice versa for gluconeogenesis BAYTEC, 2015
B. Hexokinase (SIMILAR TO PREVIOUS BOARD EXAM MD (TOP
C. Pyruvate dehydrogenase CONCEPT/PRINCIPLE) 10 - FEB
D. Phosphoglycerate kinase 2015 MED
E. None of the above BOARDS;
TOPNOTCH
MD FROM
FEU)
165 5. Smith-Lemil-Opitz syndrome (SLOS) is an autosomal Previous board exam question concept. This is an HAROLD MIDTERM 3
recessive inborn error of cholesterol synthesis which autosomal recessive disorder characterized by JAY S. EXAM - AUG
causes broad spectrum of effects ranging from mild multiple malformation syndrome caused by mutation BAYTEC, 2015
intellectual disability and behavioral problems to lethal of 7-dehydrocholesterol reductase MD (TOP
malformations. Which of the following enzymes is 10 - FEB
defective in SLOS? 2015 MED
A. HMG Co A reductase BOARDS;
B. HMG Co A synthase TOPNOTCH
C. 7-dehydrocholesterol reductase MD FROM
D. 11-dehydrocholesterol synthase FEU)
E. carnitine transport
166 6. What is the Body Mass Index of a 34 year old male BMI= wt in kg/ height in m2.. Factor for pounds to kg HAROLD MIDTERM 3
patient who has a weight of190 lbs and a height of 5 foot is 2.2 while inches to meter is 0.0254 JAY S. EXAM - AUG
10 inches? BAYTEC, 2015
A. 27.3 MD (TOP
B. 32.2 10 - FEB
C. 26.6 2015 MED
D. 31.4 BOARDS;
E. 25.5 TOPNOTCH
MD FROM
FEU)
167 7. Which of the following sugar molecules is an epimer of mannose and glucose only differ structurally at carbon HAROLD MIDTERM 3
D-glucose at carbon number 2? number 2 JAY S. EXAM - AUG
A. fructose BAYTEC, 2015
B. galactose MD (TOP
C. mannose 10 - FEB
D. ribose 2015 MED
E. maltose BOARDS;
TOPNOTCH
MD FROM
FEU)
168 8. Which among the following amino acids is not Essential amino acids are Phenylalanine, valine, HAROLD MIDTERM 3
essential for human survival for as long as all the tryptophan, threonine, isoleucine, methionine, JAY S. EXAM - AUG
essential amino acids are present in a normal person’s histidine, arginine, leucine, lysine ( PVT TIM HALL BAYTEC, 2015
diet? always Argues never Tyres) MD (TOP
A. threonine 10 - FEB
B. tryptophan 2015 MED
C. tyrosine BOARDS;
D. lysine TOPNOTCH
E. Leucine MD FROM
FEU)
169 9. Which of the following vitamins is/are needed by pyruvate dehydrogenase needs Thiamine (B1), FAD HAROLD MIDTERM 3
pyruvate dehydrogenase to convert Pyruvate to Acetyl (B2), NAD (B3), Coenzyme A (B5), and lipoic acid JAY S. EXAM - AUG
CoA before it enters the Kreb's cycle? BAYTEC, 2015
A. B1 MD (TOP
B. B2 10 - FEB
C. B3 2015 MED
D. B5 BOARDS;
E. All of the above TOPNOTCH
MD FROM
FEU)
170 10. Which among the lipoproteins has the greatest the higher the density, the more its protein HAROLD MIDTERM 3
concentration of proteins? component JAY S. EXAM - AUG
A. chylomicrons BAYTEC, 2015
B. HDL MD (TOP
C. LDL 10 - FEB
D. VLDL 2015 MED
E. IDL BOARDS;
TOPNOTCH
MD FROM
FEU)
171 Sphingomyelin is an important constituent of myelin SIMILAR TO PREVIOUS BOARD EXAM HAROLD MIDTERM 3
sheath of nerves. What is the backbone structure of CONCEPT/PRINCIPLE JAY S. EXAM - AUG
sphingomyelin? BAYTEC, 2015
A. Sphingosine MD (TOP
B. Fatty Acid 10 - FEB
C. Palmitic acid 2015 MED
D. oligosaccharide BOARDS;
E. N-acetyl neuraminic acid TOPNOTCH
MD FROM
FEU)

TOPNOTCH MEDICAL BOARD PREP BIOCHEMISTRY SUPEREXAM Page 23 of 92


For inquiries visit www.topnotchboardprep.com.ph or email us at topnotchmedicalboardprep@gmail.com
TOPNOTCH MEDICAL BOARD PREP BIOCHEMISTRY SUPEREXAM
For inquiries visit www.topnotchboardprep.com.ph or email us at topnotchmedicalboardprep@gmail.com
Item QUESTION EXPLANATION AUTHOR TOPNOTCH
# EXAM
172 Tay-Sach's disease is a lipid storage disease which can (alpha galactosidase: Fabry's disease); (Ceraminidase: HAROLD MIDTERM 3
manifest as mental retardation, hypotonia and cherry Farbers); (beta-glucosidase: gauchers); (beta- JAY S. EXAM - AUG
red macula. Which of th following enzymes is deficient in galactosidase:Krabbe's disease) BAYTEC, 2015
the disease? MD (TOP
A. Alpha-galactosidase 10 - FEB
B. ceramidase 2015 MED
C. Beta-glucosidase BOARDS;
D. Hexosaminidase A TOPNOTCH
E. Beta-galactosidase MD FROM
FEU)
173 LT, a 24 year old male, went to a blood donation center (type A: N-acetyl galactosamine) (Type B: galactose) HAROLD MIDTERM 3
to donate blood. Upon examination, he found out that he (Type AB: both) (Type O:none) JAY S. EXAM - AUG
is blood type B. Which of the following BAYTEC, 2015
glycosphingolipids is found is his RBC? MD (TOP
A.N-acetyl-galactosamine 10 - FEB
B. galactose 2015 MED
C. N-acetyl-glucosamine BOARDS;
D. B and C TOPNOTCH
E. None MD FROM
FEU)
174 Which of the following techniques involves the use of Southern blot is for DNA; Northern blot is for RNA; HAROLD MIDTERM 3
radioactive DNA probe binding to sample RNA and this is Western blot is for proteins JAY S. EXAM - AUG
used to measure sizes and amounts of specific mRNA BAYTEC, 2015
molecules? MD (TOP
A. Southern Blot 10 - FEB
B. Western Blot 2015 MED
C. Northern Blot BOARDS;
D. Eastern Blot TOPNOTCH
E. ELISA MD FROM
FEU)
175 During transciption, a mutation happened which UGG was changed to UGA which is one of the stop HAROLD MIDTERM 3
changed the codon UGG-GGU-UGG-UGG-UGA to UGG- codons JAY S. EXAM - AUG
GGU-UGG-UGA-UGA. What kind of mutation happened? BAYTEC, 2015
A. Silent mutation MD (TOP
B. Missense mutation 10 - FEB
C. Frameshift mutation 2015 MED
D. Nonsense mutation BOARDS;
E. None of the above TOPNOTCH
MD FROM
FEU)
176 Lead poisoning causes several manifestations like Lead inhibits FALAD (ferrocheletase and ALA HAROLD MIDTERM 3
basophilic stipling of RBC, headache, memory loss, dehydratase) JAY S. EXAM - AUG
nausea, abdominal pain, neuropathy, and lead deposits BAYTEC, 2015
in some organ systems. Which of the following enzymes MD (TOP
is/are inactivated by lead? 10 - FEB
A. ALA dehydratase 2015 MED
B. Ferrochelatase BOARDS;
C. ALA synthase TOPNOTCH
D. Uroporphyrinogen MD FROM
E. A and B FEU)

177 Which of the following nucleotides are pyrimidines pyrimidines are cytosine, uracil, and thymine. Uracil HAROLD MIDTERM 3
found in an RNA molecule? and cytosine are for RNA while cytosine and thymine JAY S. EXAM - AUG
A. Uracil and Cytosine are for DNA BAYTEC, 2015
B. Thymine and Cytosine MD (TOP
C. Uracil ang Guanine 10 - FEB
D. Adenine and Guanin 2015 MED
E. Thymine and Guanine BOARDS;
TOPNOTCH
MD FROM
FEU)
178 Which of the following nucleotides is a purine found in purines are guanine and adenine and both are found HAROLD MIDTERM 3
DNA but not in RNA? in DNA and RNA. SIMILAR TO PREVIOUS BOARD JAY S. EXAM - AUG
A. thymine EXAM CONCEPT/PRINCIPLE BAYTEC, 2015
B. Uracil MD (TOP
C. Adenine 10 - FEB
D. Guanine 2015 MED
E. None of the above BOARDS;
TOPNOTCH
MD FROM
FEU)
179 Which of the following amino acids is purely ketogenic? purely ketogeneic amino acids are the two Ls (lysine HAROLD MIDTERM 3
A. Tyrosine and leucine), purely glucogenic are WIFY(tryptophan, JAY S. EXAM - AUG
B. Arginine isoleucine, phenyalanine and tyrosine) SIMILAR TO BAYTEC, 2015
C. Histidine PREVIOUS BOARD EXAM CONCEPT/PRINCIPLE MD (TOP
D. Methionie 10 - FEB
E. Lysine 2015 MED
BOARDS;
TOPNOTCH
MD FROM
FEU)

TOPNOTCH MEDICAL BOARD PREP BIOCHEMISTRY SUPEREXAM Page 24 of 92


For inquiries visit www.topnotchboardprep.com.ph or email us at topnotchmedicalboardprep@gmail.com
TOPNOTCH MEDICAL BOARD PREP BIOCHEMISTRY SUPEREXAM
For inquiries visit www.topnotchboardprep.com.ph or email us at topnotchmedicalboardprep@gmail.com
Item QUESTION EXPLANATION AUTHOR TOPNOTCH
# EXAM
180 Sickle cell anemia involves a mutation substituting glutamate is a polar amino acid while valine is HAROLD MIDTERM 3
thymine for adenine which causes the coding of Valine nonpolar. Valine will tend to fold the hemoglobin so JAY S. EXAM - AUG
instead of glutamate at position 6 of the hemoglobin beta that it will position itself to the internal parts of the BAYTEC, 2015
chain. Which of the following best describes the amino hgb MD (TOP
acid change in the disease? 10 - FEB
A. from non-polar amino acid to polar amino acid 2015 MED
B. from polar amino acid to non-polar amino acid BOARDS;
C. from acidic amino acid to basic amino acid TOPNOTCH
D. from basic amino acid to acidic amino acid MD FROM
E. from essential amino acid to non-essential amino FEU)
acid
181 Creatine is derived from the following amino acids (SIMILAR TO PREVIOUS BOARD EXAM JEAN FINAL EXAM -
except? CONCEPT/PRINCIPLE.). Creatine can be derived from PAOLO M. AUG 2015
A. methionine methionine, arginine and glysine. Choline is derived DELFINO,
B. lysine from methionine. Carnitine is from methionine and MD (TOP
C. arginine lysine. Carnosine and anserine are from alanine. 10 - FEB
D. glycine Spermine and spermidine are from methionine. 2015 MED
E. None of the above BOARDS;
TOPNOTCH
MD FROM
FATIMA)
182 Which of the following is found in the basement SIMILAR TO PREVIOUS BOARD EXAM JEAN FINAL EXAM -
membrane? CONCEPT/PRINCIPLE.. Heparan sulfate is found in the PAOLO M. AUG 2015
A. Chondroitin sulfate basement membrane of the kidney where it plays a DELFINO,
B. Hyaluronic acid major role in determining the charge selectiveness of MD (TOP
C. Dermatan sulfate glomerular filtration. Memorize this! Hyaluronic acid- 10 - FEB
D. Heparan sulfate found in synovial fluid, vitreous humor, loose 2015 MED
E. Heparin connective tissue. Chondroitin sulfate- cartilage, bone, BOARDS;
cornea. Keratan sulfate- cornea. Heparan sulfate- skin TOPNOTCH
fibroblast, aortic wall. Dermatan sulfate- wide MD FROM
distribution. FATIMA)
183 True statement about fructose intolerance. SIMILAR TO PREVIOUS BOARD EXAM JEAN FINAL EXAM -
A. It is autosomal recessive CONCEPT/PRINCIPLE.. topnotch biochem handout PAOLO M. AUG 2015
B. It is caused by deficiency of aldolase B page 17. DELFINO,
C. Symptoms include hypoglycemia, jaundice, vomiting MD (TOP
D. There is accumulation of fructose-1-phosphate 10 - FEB
E. All of the above 2015 MED
BOARDS;
TOPNOTCH
MD FROM
FATIMA)
184 The last step in the synthesis of cortisol is catalyzed by The order of enzymes used in cortisol synthesis: 17 -> JEAN FINAL EXAM -
what enzyme? 21 -> 11. Please refer to steroid hormone synthesis PAOLO M. AUG 2015
A. 17-α hydroxylase pathway diagram for better comprehension. You can DELFINO,
B. 21 hydroxylase easily eliminate 1-hydroxylase and 25-hydroxylase MD (TOP
C. 11-β hydroxylase because they are involved in Vit.D activation, not in 10 - FEB
D. 1- hydroxylase cortisol synthesis. 2015 MED
E. 25- hydroxylase BOARDS;
TOPNOTCH
MD FROM
FATIMA)
185 What is the only amino acid that undergoes oxidative Glutamate undergoes oxidative deamination to yield JEAN FINAL EXAM -
deamination in order to yield free ammonia? free ammonia which will be used in the urea cycle. PAOLO M. AUG 2015
A. arginine DELFINO,
B. glutamine MD (TOP
C. asparagine 10 - FEB
D. aspartate 2015 MED
E. glutamate BOARDS;
TOPNOTCH
MD FROM
FATIMA)
186 In the liver, how many ATPs can be produced from 1 mol 2 from substrate level phosphorylation and 6 from 2 JEAN FINAL EXAM -
of glucose via aerobic glycolysis? NADH that undergo oxidative phosphorylation. Take PAOLO M. AUG 2015
A. 6 note the transport shuttle used here is malate- DELFINO,
B. 10 aspartate shuttle (because it is in the liver), so 1 NADH MD (TOP
C. 2 yields 3 ATP. Whereas if glycerol phosphate shuttle is 10 - FEB
D. 8 used, 1 NADH yields only 2 ATP. 2015 MED
E. 38 BOARDS;
TOPNOTCH
MD FROM
FATIMA)
187 Which of the following statements about DNA is false? In DNA, adenine pairs with thymine while guanine JEAN FINAL EXAM -
A. DNA serves as storage of genetic information. pairs with cytosine. PAOLO M. AUG 2015
B. The most common form is the B-DNA DELFINO,
C. The 2 strands of the double helix are held in register MD (TOP
by H-bond 10 - FEB
D. Watson, Crick and Wilkins proposed the double- 2015 MED
stranded DNA molecule model in the 1950s BOARDS;
E. Adenine pairs with guanine while cytosine pairs with TOPNOTCH
thymine MD FROM
FATIMA)

TOPNOTCH MEDICAL BOARD PREP BIOCHEMISTRY SUPEREXAM Page 25 of 92


For inquiries visit www.topnotchboardprep.com.ph or email us at topnotchmedicalboardprep@gmail.com
TOPNOTCH MEDICAL BOARD PREP BIOCHEMISTRY SUPEREXAM
For inquiries visit www.topnotchboardprep.com.ph or email us at topnotchmedicalboardprep@gmail.com
Item QUESTION EXPLANATION AUTHOR TOPNOTCH
# EXAM
188 Which of these is true during fasting state? In fasting state, glycogen synthase is inactive and JEAN FINAL EXAM -
A. Glycogen synthase is dephosphorylated phosphorylated while glycogen phosphorylase is PAOLO M. AUG 2015
B. Glycogen synthase is active active and dephosphorylated. DELFINO,
C. Glycogen phosphorylase is phosphorylated MD (TOP
D. Glycogen phosphorylase is inactive 10 - FEB
E. None of the above 2015 MED
BOARDS;
TOPNOTCH
MD FROM
FATIMA)
189 What enzyme is defective in Richner-Hanhart syndrome? Tyrosinemia has 2 types: Type 1 is caused by a defect JEAN FINAL EXAM -
A. Tyrosine aminotransferase on the enzyme fumarylacetoacetate and affects PAOLO M. AUG 2015
B. Fumarylacetoacetate hydrolase hepatorenal. Type 2 (aka Richner-Hanhart Syndrome) DELFINO,
C. Phenylalanine hydroxylase is caused by defect in tyrosine aminotransferase and MD (TOP
D. Homogentisate oxidase has oculocutaneous effects. Note: expect at least 1 10 - FEB
E. Arginosuccinate synthase question about never-before-heard eponyms. 2015 MED
BOARDS;
TOPNOTCH
MD FROM
FATIMA)
190 Which of the following functions as the storage of genetic giveaway question last boards JEAN FINAL EXAM -
material? PAOLO M. AUG 2015
A. protein DELFINO,
B. lipid MD (TOP
C. nucleic acid 10 - FEB
D. carbohydrate 2015 MED
E. Both A and C BOARDS;
TOPNOTCH
MD FROM
FATIMA)
191 Which of the following is the main glucose transporter in Glut 1 and Glut 3 are both found primarily in the brain. JEAN FINAL EXAM -
the brain? Glut 1 is the most abundant and is expressed in high PAOLO M. AUG 2015
A. Glut 1 levels in erythrocytes and endothelial cells of the DELFINO,
B. Glut 2 blood brain barrier, whereas Glut3 is expressed MD (TOP
C. Glut 3 mostly in neurons. So yes, I think Glut 3 is the better 10 - FEB
D. Glut 4 answer between the two since it is particularly 2015 MED
E. Glut 5 present in the parenchyma of the brain. BOARDS;
TOPNOTCH
MD FROM
FATIMA)
192 Which of the following relies on anaerobic glycolysis as Glycolysis in erythrocytes always terminates in JEAN FINAL EXAM -
source of energy? lactate, because the subsequent reactions of pyruvate PAOLO M. AUG 2015
A. RBC oxidation are mitochondrial, and erythrocytes lack DELFINO,
B. Renal medulla mitochondria. Other tissues that normally derive MD (TOP
C. Retina much of their energy from glycolysis and produce 10 - FEB
D. All of the above lactate include brain, gastrointestinal tract, renal 2015 MED
E. A and C medulla, retina, and skin. BOARDS;
TOPNOTCH
MD FROM
FATIMA)
193 Which of these enzymes is used in both glycolysis and phosphoglycerate kinase catalyzes a reversible JEAN FINAL EXAM -
gluconeogenesis? reaction, thus utilized in both glycolysis and PAOLO M. AUG 2015
A. Hexokinase gluconeogenesis. DELFINO,
B. Phosphoglycerate kinase MD (TOP
C. Phosphofructokinase I 10 - FEB
D. Pyruvate kinase 2015 MED
E. Glucokinase BOARDS;
TOPNOTCH
MD FROM
FATIMA)
194 True statement about Kreb's Cycle except? 1 mol of GTP (not ATP) is produced via substrate level JEAN FINAL EXAM -
A. 1 mol of ATP is produced via substrate level phosphorylation in the Kreb's cycle. PAOLO M. AUG 2015
phosphorylation DELFINO,
B. 3 NADH and 1 FADH2 are produced per turn MD (TOP
C. The rate limiting step is catalyzed by isocitrate 10 - FEB
dehydrogenase 2015 MED
D. The enzyme that catalyzes substrate level BOARDS;
phosphorylation is succinate thiokinase TOPNOTCH
E. α-ketoglutarate dehydrogenase is an enzyme MD FROM
complex that requires vitamins B1, B2, B3, B5 and lipoic FATIMA)
acid
195 Amyloid seen in Alzheimers is an example of what level SIMILAR TO PREVIOUS BOARD EXAM JEAN FINAL EXAM -
of protein structure? CONCEPT/PRINCIPLE. PAOLO M. AUG 2015
A. Primary DELFINO,
B. Secondary MD (TOP
C. Tertiary 10 - FEB
D. Quaternary 2015 MED
E. All of the above BOARDS;
TOPNOTCH
MD FROM
FATIMA)

TOPNOTCH MEDICAL BOARD PREP BIOCHEMISTRY SUPEREXAM Page 26 of 92


For inquiries visit www.topnotchboardprep.com.ph or email us at topnotchmedicalboardprep@gmail.com
TOPNOTCH MEDICAL BOARD PREP BIOCHEMISTRY SUPEREXAM
For inquiries visit www.topnotchboardprep.com.ph or email us at topnotchmedicalboardprep@gmail.com
Item QUESTION EXPLANATION AUTHOR TOPNOTCH
# EXAM
196 A patient was diagnosed with a hereditary condition SIMILAR TO PREVIOUS BOARD EXAM JEAN FINAL EXAM -
affecting the mitochondria. What would be the most CONCEPT/PRINCIPLE. PAOLO M. AUG 2015
likely manifestation? DELFINO,
A. myopathy MD (TOP
B. mental retardation 10 - FEB
C. stroke 2015 MED
D. seizure BOARDS;
E. MI TOPNOTCH
MD FROM
FATIMA)
197 Which among these amino acids is polar and uncharged? serine, threonine and tyrosine contain hydroxyl JEAN FINAL EXAM -
A. serine groups making it polar but uncharged. PAOLO M. AUG 2015
B. threonine DELFINO,
C. tyrosine MD (TOP
D. A and C 10 - FEB
E. All of the above 2015 MED
BOARDS;
TOPNOTCH
MD FROM
FATIMA)
198 Hormones that bind to cytoplasmic receptor Among all the hormones that bind to intracellular JEAN FINAL EXAM -
A. cortisol receptors, only aldosterone and cortisol have PAOLO M. AUG 2015
B. aldosterone receptors located in the cytoplasm. Others are in the DELFINO,
C. testosterone nucleus. MD (TOP
D. A and B 10 - FEB
E. None of the above 2015 MED
BOARDS;
TOPNOTCH
MD FROM
FATIMA)
199 What is the mechanism of action of tetracycline? A- rifampicin. B- chloramphenicol. D- Aminoglycoside. JEAN FINAL EXAM -
A. Inhibits DNA-dependent RNA polymerase E. Clindamycin and macrolides PAOLO M. AUG 2015
B. Inhibits prokaryotic peptidyltransferase and peptide DELFINO,
bond formation MD (TOP
C. prevents binding of aminoacyl-tRNAs to the A site 10 - FEB
D. Binds to 30s subunit and blocks the formation of 2015 MED
initiation complex BOARDS;
E. binds to 50s subunit and inhibit translocation TOPNOTCH
MD FROM
FATIMA)
200 What do adipose tissues use for the synthesis of TAG? Glycerokinase is only present in liver. DHAP is an JEAN FINAL EXAM -
A. diacylglycerol intermediate of glycolysis which undergoes reduction PAOLO M. AUG 2015
B. Phosphatidic acid to become glycerol phosphate. DELFINO,
C. Through a reaction catalyzed by glycerokinase MD (TOP
D. DHAP 10 - FEB
E. Acetyl coa 2015 MED
BOARDS;
TOPNOTCH
MD FROM
FATIMA)
201 Zero-order kinetics of enzyme reactions occur when: In zero-order kinetics, the rate of reaction depends on GRACE DIAGNOSTIC
A. Substrate concentration is above Km the availability of enzyme and not in the substrate ARVIOLA, EXAM - FEB
B. Velocity of enzyme reaction is below Vmax concentration. This occurs when all enzymes are MD (TOP 3 2015
C. Allosteric effectors are present active; that is, when the substrate concentration is - AUG 2014
D. The Lineweaver-Burke plot assumes a sigmoidal above Km. MED
curve BOARDS;
E. Enzyme inhibitors are not present TOPNOTCH
MD)
202 In oxidative phosphorylation, energy is generated when The electron transport chain occurs in the GRACE DIAGNOSTIC
hydrogen accumulates in the ________. mitochondrial matrix. However, the proton gradient ARVIOLA, EXAM - FEB
A. Cytosol created by the accumulation of hydrogen atoms MD (TOP 3 2015
B. Mitochondrial matrix occurs in the intermembranous space. - AUG 2014
C. Intermembranous space MED
D. Inner mitochondrial membrane BOARDS;
E. Outer mitochondrial membrane TOPNOTCH
MD)

203 Which of the following is NOT true of anabolic reactions? Anabolic reactions are divergent. GRACE DIAGNOSTIC
A. Endergonic ARVIOLA, EXAM - FEB
B. Convergent MD (TOP 3 2015
C. ΔG > 0 - AUG 2014
D. Non-spontaneous MED
E. Synthesis of compounds occurs. BOARDS;
TOPNOTCH
MD)
204 Which among the following is amphibolic? Amphibolic means a link between anabolic and GRACE DIAGNOSTIC
A. Glycolysis catabolic systems. The TCA is an amphibolic cycle. ARVIOLA, EXAM - FEB
B. TCA MD (TOP 3 2015
C. ETC - AUG 2014
D. PPP MED
E. Urea cycle BOARDS;
TOPNOTCH
MD)

TOPNOTCH MEDICAL BOARD PREP BIOCHEMISTRY SUPEREXAM Page 27 of 92


For inquiries visit www.topnotchboardprep.com.ph or email us at topnotchmedicalboardprep@gmail.com
TOPNOTCH MEDICAL BOARD PREP BIOCHEMISTRY SUPEREXAM
For inquiries visit www.topnotchboardprep.com.ph or email us at topnotchmedicalboardprep@gmail.com
Item QUESTION EXPLANATION AUTHOR TOPNOTCH
# EXAM
205 Which enzyme catalyzes a reversible reaction in Choices B, C, and D catalyze irreversible reactions in GRACE DIAGNOSTIC
glycolysis? glycolysis. ARVIOLA, EXAM - FEB
A. phosphoglycerate kinase MD (TOP 3 2015
B. phosphofructokinase-1 - AUG 2014
C. hexokinase MED
D. pyruvate kinase BOARDS;
E. glycogen synthase TOPNOTCH
MD)
206 What is an important role of phosphofructokinase-2 in In the well-fed state, fructose-2,6-bisphosphate is the GRACE DIAGNOSTIC
the well-fed state? most potent activator of PFK-1, the rate-limiting ARVIOLA, EXAM - FEB
A. It promotes glycogenesis enzyme in glycolysis, so that the glycolytic cycle will MD (TOP 3 2015
B. It inactivates insulin continue. - AUG 2014
C. It forms fructose-2,6-bosphosphate MED
D. It inhibits glycolysis BOARDS;
E. It metabolizes sucrose into fructose and glucose. TOPNOTCH
MD)
207 Which form of arsenic inhibits dehydrogenases by its Arsenite (the trivalent form of arsenic) forms a stable GRACE DIAGNOSTIC
affinity to lipoic acid? complex with the thiol (-SH) groups of lipoic acid, ARVIOLA, EXAM - FEB
A. Univalent make this compound unavailable to serve as a MD (TOP 3 2015
B. Divalent coenzyme. - AUG 2014
C. Trivalent MED
D. Tetravalent BOARDS;
E. Pentavalent TOPNOTCH
MD)
208 Which TCA enzyme is located in the inner mitochondrial Succinate dehydrogenase is the only enzyme of the GRACE DIAGNOSTIC
membrane? TCA that is not found in the mitochondrial matrix. ARVIOLA, EXAM - FEB
A. Aconitase Instead, it functions in the inner mitochondrial MD (TOP 3 2015
B. Alpha-ketoglutarate dehydrogenase membrane. - AUG 2014
C. Succinate thiokinase MED
D. Succinate dehydrogenase BOARDS;
E. Fumarase TOPNOTCH
MD)
209 In glycogenesis, bonds between glucose residues are Elongation of a glycogen chain involves the transfer of GRACE DIAGNOSTIC
formed at the non-reducing end of the growing chain. glucose from UDP-glucose to the non-reducing end of ARVIOLA, EXAM - FEB
This is carbon ____. the growing chain, forming a new glycosidic bond MD (TOP 3 2015
A. 1 between the anomeric hydroxyl of carbon 1 of the - AUG 2014
B. 2 activated glucose and carbon 4 of the accepting MED
C. 3 glucosyl residue. Hence, the alpha(1→4) linkages. BOARDS;
D. 4 TOPNOTCH
E. 5 MD)

210 This is the most important product of the pentose Ribose-5-phosphate is a precursor of the nucleotides GRACE DIAGNOSTIC
phosphate pathway. DNA and RNA. ARVIOLA, EXAM - FEB
A. ATP MD (TOP 3 2015
B. NADPH - AUG 2014
C. Ribose-5-phosphate MED
D. GTP BOARDS;
E. NADH TOPNOTCH
MD)
211 What form of hemoglobin is produced when 2,3-BPG When 2,3-BPG combines with hemoglobin, it forms GRACE DIAGNOSTIC
forms salt bridges with hemoglobin thus promoting salt bridges leading to the formation of the T ('taut") ARVIOLA, EXAM - FEB
oxygen release? form of hemoglobin. This form has lesser affinity to MD (TOP 3 2015
A. T form oxygen. Oxygen is, thus, released for use by the - AUG 2014
B. R form tissues. MED
C. S form BOARDS;
D. C form TOPNOTCH
E. A form MD)

212 What functional group is absent in DNA rendering it In contrast to RNA, DNA has no 2-hydroxyl group. GRACE DIAGNOSTIC
more stable in alkali solutions? Hence, its name 2-deoxyribonucleic acid. ARVIOLA, EXAM - FEB
A. 1-carbonyl group MD (TOP 3 2015
B. 2-hydroxyl group - AUG 2014
C. 1-phosphate group MED
D. 2-sulfate group BOARDS;
E. 1-alkyl group TOPNOTCH
MD)

213 What amino acid plays a crucial part in hemoglobin's Histidine located at F8 and E7 positions of hemoglobin GRACE DIAGNOSTIC
affinity to oxygen? participate in oxygen binding. ARVIOLA, EXAM - FEB
A. Glycine MD (TOP 3 2015
B. Tyrosine - AUG 2014
C. Histidine MED
D. Leucine BOARDS;
E. Proline TOPNOTCH
MD)
214 Sucrose is a non-reducing sugar because __________. The reducing property of sugars is determined by its GRACE DIAGNOSTIC
A. It is not a pentose free anomeric carbon. ARVIOLA, EXAM - FEB
B. It has no hydroxyl group MD (TOP 3 2015
C. It is a disaccharide - AUG 2014
D. It's anomeric carbon is not free MED
E. Its enantiomers do not exist BOARDS;
TOPNOTCH
MD)

TOPNOTCH MEDICAL BOARD PREP BIOCHEMISTRY SUPEREXAM Page 28 of 92


For inquiries visit www.topnotchboardprep.com.ph or email us at topnotchmedicalboardprep@gmail.com
TOPNOTCH MEDICAL BOARD PREP BIOCHEMISTRY SUPEREXAM
For inquiries visit www.topnotchboardprep.com.ph or email us at topnotchmedicalboardprep@gmail.com
Item QUESTION EXPLANATION AUTHOR TOPNOTCH
# EXAM
215 What is the important role of pyruvate carboxylase in Pyruvate carboxylase catalyzes the conversion of GRACE DIAGNOSTIC
the Krebs Cycle? pyruvate to oxaloacetate so that enough oxaloacetate ARVIOLA, EXAM - FEB
A. It depletes pyruvate thereby allowing glycolysis to is present to combine with pyruvate in the Krebs MD (TOP 3 2015
continue cycle. - AUG 2014
B. It prevents flooding of intermediates in the TCA MED
C. It conserves carbon atoms by generating two BOARDS;
molecules of carbon dioxide TOPNOTCH
D. It inhibits lactate dehydrogenase MD)
E. It ensures that enough oxaloacetate is present
216 This glycosaminoglycan is implicated in atherosclerosis Dermatan sulfate appears to be the major GAG GRACE DIAGNOSTIC
as it is known to bind LDL in arteries. synthesized by arterial smooth muscle cells, cells ARVIOLA, EXAM - FEB
A. Heparan sulfate which are known to proliferate in atherosclerotic MD (TOP 3 2015
B. Keratan sulfate lesions. - AUG 2014
C. Dermatan sulfate MED
D. Chondroitin sulfate BOARDS;
E. Hyaluronic acid TOPNOTCH
MD)

217 The Van Den Bergh reaction measures: More specifically, it measures the amount of GRACE DIAGNOSTIC
A. Glycosylated hemoglobin conjugated bilirubin in the blood. ARVIOLA, EXAM - FEB
B. Bilirubin MD (TOP 3 2015
C. Creatinine - AUG 2014
D. Amylase MED
E. Ck-MB BOARDS;
TOPNOTCH
MD)
218 Which B vitamin is part of fatty acid synthesis? Pantothenate or vitamin B5 is a component of the GRACE DIAGNOSTIC
A. B1 fatty acyl coA synthase complex of fatty acid synthesis. ARVIOLA, EXAM - FEB
B. B2 MD (TOP 3 2015
C. B3 - AUG 2014
D. B5 MED
E. B6 BOARDS;
TOPNOTCH
MD)
219 Which of the following is TRUE regarding uncouplers of Uncouplers increase the permeability of the inner GRACE DIAGNOSTIC
the ETC? mitochondrial membrane to hydrogen ions. As a ARVIOLA, EXAM - FEB
A. A proton gradient is formed; however, no ATP is result, no protein gradient is formed. Complex V is MD (TOP 3 2015
generated. affected. Aspirin is an uncoupler. - AUG 2014
B. Uncouplers decrease the permeability of the MED
intermembranous space to hydrogen atoms. BOARDS;
C. There is increased oxygen consumption. TOPNOTCH
D. Uncouplers affect complexes I to IV of the ETC. MD)
E. Acetylsalicylic acid is not an uncoupler.
220 In DNA transcription, the template strand is also known The template strand is the one that is GRACE DIAGNOSTIC
as the ________. read/copied/transcribed during DNA transcription to ARVIOLA, EXAM - FEB
A. Coding strand form the mRNA transcript. It is also known as the non- MD (TOP 3 2015
B. Non-coding strand coding strand and is complementary to the mRNA - AUG 2014
C. Intron transcript. The other strand is called the coding strand MED
D. Exon because this strand bears the exact same code as the BOARDS;
E. None of the above mRNA transcript (with the exception of T for U TOPNOTCH
changes). MD)
221 Which of the following is true about myoglobin except? SIMILAR TO PREVIOUS BOARD EXAM LEAN MIDTERM
A.it contains a tetrapyrrole ring connected to its CONCEPT/PRINCIPLE. Bohr effect is dependent ANGELO EXAM 1 - FEB
individual components through methylene bridges cooperative interaction between hemes of more than SILVERIO, 2015
B. it it similar to the secondary-tertiary structure of the one domain.this is only represented in hemoglobin MD (TOP 4
hemoglobin which has a tetrameric structure. Letter A is correct - AUG 2014
C. it exhibits bohr effect during extreme oxygen since myoglobin contains a heme protein. B- is also MED
deprivation correct, since the only major difference between BOARDS;
D. oxidation of Fe2 to Fe3 completely destroys its hemoglobin and myoglobin is that the latter has only TOPNOTCH
biologic activity domain while the former exhibits 4 domains. their MD), MD
E. it has only one domain therefore it is monomeric. alpha helix and tertiary configuration is very similar.
both myoglobin and hemoglobin needs a reduced
state of iron (ferrous form) to allow binding of oxygen
for storage or for transport.
222 which of the following is true about a competitive Competitive inhibition is best extrapolated using line LEAN MIDTERM
enzyme inhibitor? weaver burk plot since it can easily demonstrate the ANGELO EXAM 1 - FEB
A. It can be best extrapolated using Michaelis Menten values of Km and Vmax. It is also called as substrate SILVERIO, 2015
equation analog they bind to the catalytic site decreasing the MD (TOP 4
B. It increases the value of the Km number of free enzyme molecules availabe for - AUG 2014
C. It affects the Vmax by one half during high levels of substrate binding. kinetics will show that affinity will MED
inhibitor compared to original substrate be decreased as manifested by increase in Km ( note BOARDS;
D. it acts by decreasing the number of free substrate that Km and affinity has inverse relationship) and TOPNOTCH
molecules availabole to bind to the enzyme with no effect to the Vmax regardless of the amount of MD), MD
E. all of the above are correct the inhibitor
223 which of the following sites of respiratory chain does barbiturates exert its inhibitory effect by binding to LEAN MIDTERM
Barbiturates act? complex I. Malonate is an inhibitor of complex II, ANGELO EXAM 1 - FEB
A. Complex I Antimycin A and Dimercaprol is for Complex III, SILVERIO, 2015
B. Complex II Hydrogen sulfide, carbon monoxide and cyanide MD (TOP 4
C. Complex III inhibits complex IV and lastly oligomycin inhibits ATP - AUG 2014
D. Complex IV synthase. MED
E. ATP synthase BOARDS;
TOPNOTCH
MD), MD

TOPNOTCH MEDICAL BOARD PREP BIOCHEMISTRY SUPEREXAM Page 29 of 92


For inquiries visit www.topnotchboardprep.com.ph or email us at topnotchmedicalboardprep@gmail.com
TOPNOTCH MEDICAL BOARD PREP BIOCHEMISTRY SUPEREXAM
For inquiries visit www.topnotchboardprep.com.ph or email us at topnotchmedicalboardprep@gmail.com
Item QUESTION EXPLANATION AUTHOR TOPNOTCH
# EXAM
224 which of the following correctly describes Glucokinase? Glucokinase is predominantly found in the liver and LEAN MIDTERM
A. Found in hepatocytes beta cells of the pancreas. It has a higher Km ANGELO EXAM 1 - FEB
B. High affinity to glucose compared to Hexokinase, therefore it only SILVERIO, 2015
C. Low Vmax phosphorylate glucose during high glucose load ( after MD (TOP 4
D. Can be inhibited by glucose 6 phosphate meal consumption). It has a high Vmax such that it - AUG 2014
E. None of the above effectively remove glucose from portal blood. MED
furthermore, it is not allosterically inhibited by its BOARDS;
product Glucose 6 phosphate. all the other choices TOPNOTCH
describes Hexokinase which is most tissues of the MD), MD
body.
225 What is the most common glycolytic enzyme deficiency? Pyruvate kinase deficiency is responsible for about LEAN MIDTERM
A. Glucokinase 96% of patient suffering from glycolytic enzyme ANGELO EXAM 1 - FEB
B. Pyruvate kinase defect. SILVERIO, 2015
C. Glucose phosphate isomerase MD (TOP 4
D. Phosphoglycerate kinase - AUG 2014
E. Phosphoglycerate mutase MED
BOARDS;
TOPNOTCH
MD), MD
226 which of the following is an effect of high glucagon high glucagon/insulin ratio will promote LEAN MIDTERM
states? gluconeogenesis through a cascade of events: increase ANGELO EXAM 1 - FEB
A. Phosphorylation of PFK2 leading to decrease activity cAMP leads to increase Protein Kinase A which will SILVERIO, 2015
B. Phosphorylation of Fructose 1,6 biphosphatase leads inactivate PFK2 and activate FBP both via MD (TOP 4
to decrease activity phosphorylation. - AUG 2014
C. Decrease levels of Protein kinase A MED
D. all of the above BOARDS;
E. none of the above TOPNOTCH
MD), MD
227 which of the following enzyme catalyze the formation of Succinyl CoA synthetase or succinate thiokinase LEAN MIDTERM
GTP via substrate level phosphorylation in Kreb cycle? converts succinyl CoA to succinate in mammalian ANGELO EXAM 1 - FEB
A. Succinate dehydrogenase tissues resulting to phosphorylation of GDP to GTP. SILVERIO, 2015
B. Succinyl CoA synthetase MD (TOP 4
C. Aconitase - AUG 2014
D. Isocitrate dehydrogenase MED
E. Fumarase BOARDS;
TOPNOTCH
MD), MD
228 In Kreb cycle, which of the following dehydrogenase in kreb cycle, it is necessary to reduce citrate ( a C6 LEAN MIDTERM
catalyze the last step for CO2 production? atom) to oxaloacetate ( C4 atom). There are only two ANGELO EXAM 1 - FEB
A. Isocitrate dehydrogenase sites for CO2 production. The first step is catalyzed by SILVERIO, 2015
B. Malate dehydrogenase isocitrate dehydrogenase and the last is by the alpha MD (TOP 4
C. Succinate dehydrogenase ketoglutarate dehydrogenase. All the other proceeding - AUG 2014
D. Alpha ketoglutarate dehydrogenase dehydrogenase will only reduce FAD or NAD without MED
E. None of the above forming CO2. BOARDS;
TOPNOTCH
MD), MD
229 A 1 month old male presents with severe hypotonia and Glycogen storage disease type II aka Pompe disease is LEAN MIDTERM
cardiomegaly, which enzyme deficiency most likely a fatal condition most commonly manifested during ANGELO EXAM 1 - FEB
represent this type of glycogen storage disease. the first month of life. It primarily affects the heart SILVERIO, 2015
A. Glucose 6 phosphatase muscle causing cardiomyopathy. This is secondary to MD (TOP 4
B. Acid maltase deficiency of acid maltase of alpha 1-4 glucosidase. - AUG 2014
C. Glycogen debranching enzyme MED
D. muscle phosphorylase BOARDS;
E. none of the above TOPNOTCH
MD), MD
230 which of the following is a correctly paired enzyme Hunter syndrome is the only X linked type of LEAN MIDTERM
deficiency among mucopolysaccharidoses except? mucopolysaccharidosis characterized by deficiency in ANGELO EXAM 1 - FEB
A. Hurler syndrome : a-L iduronidase iduronate sulfatase. It presents with mental SILVERIO, 2015
B. Hunter syndrome: N - acetyltransferase retardation, variable physical deformities however, MD (TOP 4
C. Sly syndrome: B- Glucuronidase minimal to no corneal clouding. It is secondary to - AUG 2014
D. Sanfilippo syndrome: Heparan sulfamidase accumulation of dermatan sulfate and heparan sulfate. MED
E. none of the above BOARDS;
TOPNOTCH
MD), MD
231 which of the following is a false statement regarding acetyl CoA carboxylase is activated by citrate and LEAN MIDTERM
acetyl CoA carboxylase? inhibited by its product palmitate. ANGELO EXAM 1 - FEB
A. It is activated by palmitate SILVERIO, 2015
B. It is inhibited by phosphorylation during high MD (TOP 4
glucagon/insulin ratio - AUG 2014
C. It requires biotin MED
D. It is the rate limiting step in fatty acid synthesis BOARDS;
E. only A and C are incorrect statement TOPNOTCH
MD), MD
232 this lack of enzyme is responsible why the liver cannot acetoacetate: succinylCoa transferase or thiophorase LEAN MIDTERM
utilize its own ketone bodies? is responsible for conversion of acetoacetate to its CoA ANGELO EXAM 1 - FEB
A. Acetoacetate: succinyl CoA transferase derivative. This is present in almost all tissues but SILVERIO, 2015
B. B- ketothiolase absent in the liver. MD (TOP 4
C. HMG CoA lyase - AUG 2014
D. B hydroxybutyrate dehydrogenase MED
E. None of the above BOARDS;
TOPNOTCH
MD), MD

TOPNOTCH MEDICAL BOARD PREP BIOCHEMISTRY SUPEREXAM Page 30 of 92


For inquiries visit www.topnotchboardprep.com.ph or email us at topnotchmedicalboardprep@gmail.com
TOPNOTCH MEDICAL BOARD PREP BIOCHEMISTRY SUPEREXAM
For inquiries visit www.topnotchboardprep.com.ph or email us at topnotchmedicalboardprep@gmail.com
Item QUESTION EXPLANATION AUTHOR TOPNOTCH
# EXAM
233 A 27 y/o female presents with hypertriglyceridemia with A- is a pathologic basis of familial LEAN MIDTERM
a normal or low cholesterol count. Which of the familial hyperbetalipoproteinemia. In this disease, the ANGELO EXAM 1 - FEB
syndrome will explain the abnormal lipid profile of the cholesterol rises (inc LDL) while the clearance of SILVERIO, 2015
patient? chylomicrons and VLDL is normal ( Normal TAG). B- MD (TOP 4
A. LDL receptor defect impaired clearance of VLDL and chylomicrons by an - AUG 2014
B. Lipoprotein lipase deficiency absent or abnormal lipoprotein lipase will lead to MED
C. overproduction of VLDL increase TAG. since less is transformed into LDL, BOARDS;
D. apolipoprotein E deficiency cholesterol will be normal or low. overproduction of TOPNOTCH
E. none of the above VLDL alone will lead to increase in both Cholesterol MD), MD
and TAG. Apolipoprotein E deficiency decreasing the
clearance of VLDL remnant, will result to increase
VLDL,LDL and TAG.
234 what type of cell is affected in terms of glucose transport Among the GLUT transporters, it is only GLUT 4 that is LEAN MIDTERM
if there is an insulin deficiency states? insulin dependent. It is primarily distributed in two ANGELO EXAM 1 - FEB
A. Neurons major cell population: the adipocytes and myocytes. SILVERIO, 2015
B. Pancreatic acinar cells Pancreatic acinar cells particularly beta cells and MD (TOP 4
C. Adipocytes hepatocytes uses Glut 2 receptors, Neuorns and RBC - AUG 2014
D. Erythrocytes uses GLut 1 receptors. MED
E. Hepatocytes BOARDS;
TOPNOTCH
MD), MD
235 Which of the following intermediate products of heme Lead intoxication affects two enzyme in the heme LEAN MIDTERM
synthesis will accumulate if the patient suffers from lead synthesis. It is ALA dehydratase and ferrochelatase. ANGELO EXAM 1 - FEB
intoxication? The former combines two molecules of d-ALA to form SILVERIO, 2015
A. Succinyl CoA Porphobilinogen, while ferrochelatase catalyzes the MD (TOP 4
B. Uroporpholinogen incorporation of ferrous iron to the Protoporphyrin IX - AUG 2014
C. Aminolevulinic acid to form heme. therefore both B and D will be MED
D. porphobilinogen deficient.succinyl CoA along with glycine will not BOARDS;
E. All of the above accumulate since its conversion to d-ALA is TOPNOTCH
irreversible by the enzyme ALA synthase ( rate MD), MD
limiting step)
236 which of the following is true regarding Phenylketonuria Hyperphenylalaninemia (PKU) can arise from either LEAN MIDTERM
? absence of phenylalanine hydroxylase(PH) which ANGELO EXAM 1 - FEB
A. It can be secondary to dihydrobiopterin reductase convert phenylalanine to tyrosine or absence of its SILVERIO, 2015
deficiency enzyme cofactor tetrahydrobiopterin. Blood MD (TOP 4
B. Blood phenylalanine levels rises significantly at Day 2 phenylalanine levels accumulates only after Day 3 or - AUG 2014
postnatal life. Day 4 of life. absence of PH will make tyrosine MED
C. There would be normal levels of catecholamines in essential AA, without complete supplementation, BOARDS;
the blood there would be a depletion of catecholamine levels TOPNOTCH
D. tyrosine need not be supplemented since tyrosine is deficient. MD), MD
E. none of the above
237 which of the following is an essential amino acid? Essential amino acid: phenylalanine, valine, LEAN MIDTERM
A. asparagine tryptophan, threonine, isoleucine, methionine, ANGELO EXAM 1 - FEB
B. aspartate histidine, arginine, leucine, lysine. SILVERIO, 2015
C. arginine MD (TOP 4
D. Cysteine - AUG 2014
E. Alalnine MED
BOARDS;
TOPNOTCH
MD), MD
238 what bacterialDNA polymerase is responsible for gap there are only three types of bacterial DNA LEAN MIDTERM
filling and synthesis of lagging strand during replication? polymerase. Pol I is responsible ofr gap filling and ANGELO EXAM 1 - FEB
A. DNA pol I lagging strand synthesis. Pol II is for DNA SILVERIO, 2015
B. DNA pol II proofreading and repair. Pol III is for leading strand MD (TOP 4
C. DNA pol III synthesis - AUG 2014
D. DNA pol IV MED
E. DNA pol V BOARDS;
TOPNOTCH
MD), MD
239 which of the following is false about the Genetic code? one amino acid can be encoded by more than one LEAN MIDTERM
A. One amino acid is encoded only by one codon codon.this is the degenerate feature of a genetic code. ANGELO EXAM 1 - FEB
B. There are no punctation marks in the reading of the SILVERIO, 2015
code MD (TOP 4
C. A specific codon indicates only one specific amino - AUG 2014
acid MED
D. It is universal BOARDS;
E. none of the above TOPNOTCH
MD), MD
240 which of the following is true about type 1 muscle fiber? type 1 are slow twitch muscle fiber widely distributed LEAN MIDTERM
A. Fast contraction rate along the antigravity muscles. It is important in ANGELO EXAM 1 - FEB
B. Predominant in soleus and erector spinae muscle maintaining posture. All the other choices describes SILVERIO, 2015
C. High energy utilization Type II fast twitch or glycolytic fibers. MD (TOP 4
D. High myosin ATPase activity - AUG 2014
E. Short duration of action MED
BOARDS;
TOPNOTCH
MD), MD
241 Sphingomyelin is a substance participating in nerve SIMILAR TO PREVIOUS BOARD EXAM KEVIN MIDTERM 2
conduction and insulation, what is its predominant fatty CONCEPT/PRINCIPLE, very difficult to answer BRYAN LO, EXAM - FEB
acid composition? because there are a lot to consider especially type of MD (TOP 7 2015
A. palmitic acid tissue involved, but assuming human neural tissue, - AUG 2014
B. stearic acid based on journals from pubmed the most MED
C. lignoceric acid predominant fatty acid is stearic acid C18 with BOARDS;
D. nervonic acid lignocericC24 and nervonic C24:1 coming next TOPNOTCH
E. sphanganine MD)

TOPNOTCH MEDICAL BOARD PREP BIOCHEMISTRY SUPEREXAM Page 31 of 92


For inquiries visit www.topnotchboardprep.com.ph or email us at topnotchmedicalboardprep@gmail.com
TOPNOTCH MEDICAL BOARD PREP BIOCHEMISTRY SUPEREXAM
For inquiries visit www.topnotchboardprep.com.ph or email us at topnotchmedicalboardprep@gmail.com
Item QUESTION EXPLANATION AUTHOR TOPNOTCH
# EXAM
242 Which of the following precursors is the one converted SIMILAR TO PREVIOUS BOARD EXAM KEVIN MIDTERM 2
into vitamin D3 in the skin by activation with the help of CONCEPT/PRINCIPLE, The presence of this compound BRYAN LO, EXAM - FEB
sunlight? in human skin enables humans to manufacture MD (TOP 7 2015
A. 7-dehydrocholesterol vitamin D3 from ultraviolet rays in the sun light, via - AUG 2014
B. 3-hydroxycholesterol an intermediate isomer pre-vitamin D3. MED
C. 25-dehydrocholesterol BOARDS;
D. 1,25 - dihydroxycholecalciferol TOPNOTCH
E. pregnenolone MD)

243 Which of the following is a common enzyme in both SIMILAR TO PREVIOUS BOARD EXAM KEVIN MIDTERM 2
gluconeogenesis and glycolysis CONCEPT/PRINCIPLE. Of all the enzymes given only BRYAN LO, EXAM - FEB
A. pyruvate carboxylase phosphoglycerate kinase is common to both MD (TOP 7 2015
B. pyruvate kinase pathways, pyruvate kinase is glycolytic, the others are - AUG 2014
C. phosphoenolpyruvate carboxykinase involved in gluconeogenesis MED
D. phosphoglycerate kinase BOARDS;
E. fructose 1,6 bisphosphatase TOPNOTCH
MD)

244 The committed step in the glycolytic pathway is SIMILAR TO PREVIOUS BOARD EXAM KEVIN MIDTERM 2
catalyzed by the enzyme phosphofructokinase 1, the CONCEPT/PRINCIPLE. All are positive regulators BRYAN LO, EXAM - FEB
following substrates and or substances positively except citrate and ATP which represent high energy MD (TOP 7 2015
regulates or allosterically activates this enzyme EXCEPT states therefore exerting negative feedback regulation - AUG 2014
A. citrate on the glycolytic pathway. Fructose 2,6 bisphosphate MED
B. 5'AMP is the strongest positive allosteric activator BOARDS;
C. fructose 2,6 bisphosphate TOPNOTCH
D. fructose 6 phosphate MD)
E. none of the above (all are positive regulators)

245 Which of the following substances is known to inhibit SIMILAR TO PREVIOUS BOARD EXAM KEVIN MIDTERM 2
complex 2 succinate -q-reductase of the electron CONCEPT/PRINCIPLE. At that time we were asked BRYAN LO, EXAM - FEB
transport chain? about complex 3 and 4 inhibitors Antimycin and MD (TOP 7 2015
A. amobarbital cyanide respectively. Dimercaprol also inhibits - AUG 2014
B. malonate complex 3. Amobarbital inhibits complex 1 MED
C. antimycin A BOARDS;
D. dimercaprol TOPNOTCH
E. cyanide MD)

246 which of the following storage forms of energy in SIMILAR TO PREVIOUS BOARD EXAM KEVIN MIDTERM 2
skeletal muscle provides energy for the first few seconds CONCEPT/PRINCIPLE. The most immediate source of BRYAN LO, EXAM - FEB
of muscle activity or exertion? energy in the first few seconds of muscular activity is MD (TOP 7 2015
A. muscle glycogen phosphocreatine, muscle glycogen is then used next - AUG 2014
B. free glucose and over long periods of activity, fatty acids start to be MED
C. phosphocreatine used as the main source. BOARDS;
D. fatty acids TOPNOTCH
E. ketone bodies MD)

247 The disease characterized as having a deficiency of SIMILAR TO PREVIOUS BOARD EXAM KEVIN MIDTERM 2
homogentisic acid oxidase manifesting with dark CONCEPT/PRINCIPLE. Homocystinemia and BRYAN LO, EXAM - FEB
discoloration of the urine when allowed to stand and can homocystinuria are manifestations of cystathionine MD (TOP 7 2015
cause damage to cartilage (ochronosis), heart valves and synthase deficiency and other enzymes involved in - AUG 2014
kidney stones. methionine metabolism, may also be seen with B9 B12 MED
A. homocystinemia deficiencies. Alkaptonuria is a basement membrane BOARDS;
B. homocystinuria disease, maple syrup urine disease is deficiency of TOPNOTCH
C. maple syrup urine disease branched chain amino acid dehydrogenase MD)
D. alkaptonuria
E. alport's disease
248 Which of the following genes act as a tumor suppressor SIMILAR TO PREVIOUS BOARD EXAM KEVIN MIDTERM 2
gene which regulates cell apoptosis and the dysfunction CONCEPT/PRINCIPLE. P53 also known as the BRYAN LO, EXAM - FEB
of which is implicated in the pathogenesis of a lot of guardian of the genome regulates promotes cell MD (TOP 7 2015
human cancers? apoptosis when there is irreversible DNA damage. All - AUG 2014
A. BCR-ABL the other genes are oncogenes which function in MED
B. RAS promoting growth when mutated promotes BOARDS;
C. RET uncontrolled cellular growth. TOPNOTCH
D. C-kit MD)
E. p-53

249 Juan was trapped inside a collapsed building after an SIMILAR TO PREVIOUS BOARD EXAM KEVIN MIDTERM 2
earthquake. Having been trapped with no access to food CONCEPT/PRINCIPLE. A lot of questions on starvation BRYAN LO, EXAM - FEB
and water, his main source of energy for the first 24 and metabolic integration. First 24 hours glycogen is MD (TOP 7 2015
hours following the event would be derived from which the main source. After that gluconeogenesis to - AUG 2014
of the following sources? maintain blood glucose, then fats and ketones and MED
A. muscle glycogen then breakdown of proteins after prolonged BOARDS;
B. muslce protein starvation. Muscle glycogen cannot be utilized as free TOPNOTCH
C. liver glycogen glucose by other organs because of lack of glucose 6 MD)
D. ketones phosphatase.
E. gluconeogenesis

TOPNOTCH MEDICAL BOARD PREP BIOCHEMISTRY SUPEREXAM Page 32 of 92


For inquiries visit www.topnotchboardprep.com.ph or email us at topnotchmedicalboardprep@gmail.com
TOPNOTCH MEDICAL BOARD PREP BIOCHEMISTRY SUPEREXAM
For inquiries visit www.topnotchboardprep.com.ph or email us at topnotchmedicalboardprep@gmail.com
Item QUESTION EXPLANATION AUTHOR TOPNOTCH
# EXAM
250 Juan was trapped inside a collapsed building after an SIMILAR TO PREVIOUS BOARD EXAM KEVIN MIDTERM 2
earthquake. Having been trapped with no access to food CONCEPT/PRINCIPLE. The brain can obtain up to BRYAN LO, EXAM - FEB
and water, and experienced prolonged starvation, other 70% of its energy from ketone bodies with prolonged MD (TOP 7 2015
than glucose what predominant source of energy will his starvation. Fatty acids cannot be utilized by the brain. - AUG 2014
brain be utilizing? MED
A. protein BOARDS;
B. glycosphingolipids TOPNOTCH
C. free fatty acids MD)
D. ketone bodies
E. glycogen

251 The following substances are known to be antioxidants SIMILAR TO PREVIOUS BOARD EXAM KEVIN MIDTERM 2
preventing lipid peroxidation and buffers against free CONCEPT/PRINCIPLE. Vitamin B1 is the least BRYAN LO, EXAM - FEB
radical production EXCEPT implicated vitamin as an antixoidant among the MD (TOP 7 2015
A. glutathione choices - AUG 2014
B. vitamin D MED
C. vitamin C BOARDS;
D. vitamin E TOPNOTCH
E. vitamin B1 MD)

252 The following hormones act via intracellular nuclear SIMILAR TO PREVIOUS BOARD EXAM KEVIN MIDTERM 2
receptors binding to DNA response elements EXCEPT CONCEPT/PRINCIPLE. All steroid based hormones BRYAN LO, EXAM - FEB
A. Estrogen have intracellular nuclear receptors. Insulin binds to a MD (TOP 7 2015
B. insulin cell membrane receptor associated with tyrosine - AUG 2014
C. thyroid hormone kinase MED
D. vitamin D BOARDS;
E. cortisol TOPNOTCH
MD)

253 Which of the following cell profiles are usually SIMILAR TO PREVIOUS BOARD EXAM KEVIN MIDTERM 2
associated with chronic autoimmune diseases and CONCEPT/PRINCIPLE, difficult one, from research I BRYAN LO, EXAM - FEB
cancers like hodgkin's lymphoma? found the answer to be double positive, meaning cells MD (TOP 7 2015
A. CD4(+) CD8(-) have both CD4 CD8 characteristics evading thymic - AUG 2014
B. CD4(-) CD8(-) double negative immune regulation causing diseases like autoimmune MED
C. CD4(-) CD8(+) and malignancies. BOARDS;
D. CD4(+) CD8(+) double positive TOPNOTCH
E. all of the above MD)

254 What is the most active potent form of androgen SIMILAR TO PREVIOUS BOARD EXAM KEVIN MIDTERM 2
involved in the development of male external genitalia CONCEPT/PRINCIPLE, dihydrotestosterone or DHT is BRYAN LO, EXAM - FEB
and also found predominant in the prostate and hair the active form of testosterone and the most potent. MD (TOP 7 2015
follicles? Androstenedione is a precursor with androgen - AUG 2014
A. testosterone properties, Dehydroepiandrosterone or DHEA is MED
B. dihydrotestosterone secreted in the adrenal and gonads where it also acts BOARDS;
C. dehydroepiandrosterone as an intermediate for estrogen and androgen TOPNOTCH
D. androstenedione synthesis MD)
E. none of the above

255 From which metabolic intermediate does the synthesis SIMILAR TO PREVIOUS BOARD EXAM KEVIN MIDTERM 2
of cholesterol begin with? CONCEPT/PRINCIPLE. Cholesterol synthesis starts BRYAN LO, EXAM - FEB
A. acetylCoA with joining of 2 molecules of Acetyl CoA to form MD (TOP 7 2015
B. HMG-CoA AcetoacetylCoA with the enzyme acetoacetylCoA - AUG 2014
C. mevalonate thiolase eventually forming HMG-CoA and so on. All MED
D. squalene the others are intermediates further down the BOARDS;
E. lanosterol pathway. TOPNOTCH
MD)

256 Increasing insulin levels has which of the following SIMILAR TO PREVIOUS BOARD EXAM KEVIN MIDTERM 2
effects on the different enzyme systems? CONCEPT/PRINCIPLE, know the functions of the BRYAN LO, EXAM - FEB
A. glycogen phosphorylase will be dephosphorylated hormones by heart, a lot of metabolic integration. MD (TOP 7 2015
B. AcetylCoA carboxylase will be inhibited Insulin predominantly dephosphorylates enzymes to - AUG 2014
C. pyruvate dehydrogenase is phosphorylated activate them or inhibit them. Glucagon Epinephrine MED
D. glycogen synthase is phosphorylated predominantly phosphorylates enzymes to activate BOARDS;
E. pyruvate carboxylase is activated them via cAMP Protein Kinase A pathway. Increase TOPNOTCH
Insulin like in the well fed state will cause storage of MD)
energy as glycogen therefore glycogen phosphorylase
the one responsible for breakdown is inhibited and
dephosphorylated form, opposite is true with
glycogen synthase where it is activated and
dephosphorylated, acetylcoa carboxylase is activated
because insulin promotes energy storage by
lipogenesis, pyruvate dehydrogenase is
dephosphorylated and activated because insulin
promotes glucose entry and utilization, insulin
promotes glycolysis and inhibits gluconeogenesis
therefore pyruvate carboxylase should be inhibited no
new glucose needs to be made in a well fed state.

TOPNOTCH MEDICAL BOARD PREP BIOCHEMISTRY SUPEREXAM Page 33 of 92


For inquiries visit www.topnotchboardprep.com.ph or email us at topnotchmedicalboardprep@gmail.com
TOPNOTCH MEDICAL BOARD PREP BIOCHEMISTRY SUPEREXAM
For inquiries visit www.topnotchboardprep.com.ph or email us at topnotchmedicalboardprep@gmail.com
Item QUESTION EXPLANATION AUTHOR TOPNOTCH
# EXAM
257 Which of the following statements are true regarding SIMILAR TO PREVIOUS BOARD EXAM KEVIN MIDTERM 2
skeletal muscle? CONCEPT/PRINCIPLE, suprisingly there was a lot of BRYAN LO, EXAM - FEB
A. skeletal muscle is unable to contract in the absence of physiology question in the biochemistry exam, muscle MD (TOP 7 2015
extracellular calcium influx stimulating the release of physiology had 3-5 questions alone. Skeletal muscle - AUG 2014
calcium from the sarcoplasmic reticulum does not need extracellular calcium, dihydropyridine MED
B. action potential travels down the T-tubules closely receptors sense the action potential causing the BOARDS;
related to the 2 terminal cisternae of the sarcoplasmic Ryanodine receptors in the SR to open and release TOPNOTCH
reticulum releasing calcium into the cytoplasm calcium, calcium binds to troponin C MD)
C. the dihydropyridine receptor opens and releases the
calcium from the sarcoplasmic reticulum into the
cytoplasm to stimulate excitation contraction coupling
D. calcium released from the sarcoplasmic reticulum
binds to troponin T allowing actin myosin interaction to
occur
E. all of the above statments are correct
258 Which of the following biochemical processes SIMILAR TO PREVIOUS BOARD EXAM KEVIN MIDTERM 2
exclusively occur in the mitochondria of the cell? CONCEPT/PRINCIPLE, glycolysis and fatty acid BRYAN LO, EXAM - FEB
A. glycolysis synthesis occurs in the cytoplasm, urea cycle has MD (TOP 7 2015
B. pentose phosphate pathway steps in both the cytoplasm and mitochondria, - AUG 2014
C. tricarboxylic acid cycle pentose phosphate pathway in the cytoplasm MED
D. urea cycle BOARDS;
E. fatty acid synthesis TOPNOTCH
MD)

259 What is the predominant pathway utilized by red blood SIMILAR TO PREVIOUS BOARD EXAM KEVIN MIDTERM 2
cells for energy? CONCEPT/PRINCIPLE, specifically answer should be BRYAN LO, EXAM - FEB
A. beta oxidation anaerobic glycolysis since RBCs don’t have MD (TOP 7 2015
B. embden meyerhoff pathway mitochondria. Lueberin rapoport pathway - pathway - AUG 2014
C. luebering rapoport pathway involved in the formation of 2,3 BPG. MED
D. hexose monophosphate shunt BOARDS;
E. Citric acid cycle TOPNOTCH
MD)
260 This important coenzyme A involved in various SIMILAR TO PREVIOUS BOARD EXAM KEVIN MIDTERM 2
biochemical reactions is derived from which of the CONCEPT/PRINCIPLE, what was asked was FAD B2 BRYAN LO, EXAM - FEB
following substances? and NAD B3. memorize the coenzymes and vitamin MD (TOP 7 2015
A. vitamin B1 derivatives, this is after all both biochemistry and - AUG 2014
B. vitamin B2 nutrition MED
C. vitamin B3 BOARDS;
D. vitamin B5 TOPNOTCH
E. vitamin B6 MD)

261 If an amino acid with a nonpolar side chain has a pk1 of At this pH, the COOH group of the amino acid will have RAYMUND MIDTERM 3
2.3 and a pk2 of 9.1, what will be its expected net charge already given up its H+ making the net charge 0. MARTIN LI, EXAM - FEB
in an alkalinized urine with a pH of 8.5? MD (TOP 1 2015
A. +2 - AUG 2014
B. 0 MED
C. -1 BOARDS;
D. +1 TOPNOTCH
E. MD)
262 A patient with long-standing Diabetes mellitus type 2 is Sorbitol dehydrogenase converts sorbitol to fructose. RAYMUND MIDTERM 3
diagnosed with peripheral neuropathy. The absence of Absence of this enzyme in the presence of MARTIN LI, EXAM - FEB
this enzyme in the presence of hyperglycemia is said to hyperglycemia will result in increased sorbitol in the MD (TOP 1 2015
be responsible: tissue. - AUG 2014
A. sorbitol dehydrogenase MED
B. aldose reductase BOARDS;
C. fructokinase TOPNOTCH
D. aldolase B MD)

263 Which of the following characterizes the lagging strand A B and D are required by both strands RAYMUND MIDTERM 3
but not the leading strand in DNA replication? MARTIN LI, EXAM - FEB
A. Requires an RNA primer MD (TOP 1 2015
B. Synthesized in a 5’ to 3’ direction - AUG 2014
C. Requires DNA ligase activity MED
D. Bound by single-stranded DNA binding proteins BOARDS;
TOPNOTCH
MD)
264 A diet rich in which of the following dietary fats is most Monounsaturated fatty acids and omega 6 PUFA RAYMUND MIDTERM 3
responsible for a decrease in LDL levels: lowers LDL levels. MARTIN LI, EXAM - FEB
A. polyunsaturated omega 3 fatty acids MD (TOP 1 2015
B. monounsaturated fatty acids - AUG 2014
C. saturated fat MED
D. transfat BOARDS;
TOPNOTCH
MD)
265 Which of the following is true about enzyme kinetics? Review enzyme kinetics. Michaelis-menten shows a RAYMUND MIDTERM 3
A. Allosteric enzymes show a sigmoidal curve hyperbolic curve. Irreversible inhibitor lowers Vmax MARTIN LI, EXAM - FEB
B. A numerically small Km indicates low affinity for MD (TOP 1 2015
substrate - AUG 2014
C. Enzymes that follow Michaelis-Menten kinetics show a MED
sigmoidal curve BOARDS;
D. An irreversible competitive antagonist leads to an TOPNOTCH
increase in Km MD)
E. None of the above

TOPNOTCH MEDICAL BOARD PREP BIOCHEMISTRY SUPEREXAM Page 34 of 92


For inquiries visit www.topnotchboardprep.com.ph or email us at topnotchmedicalboardprep@gmail.com
TOPNOTCH MEDICAL BOARD PREP BIOCHEMISTRY SUPEREXAM
For inquiries visit www.topnotchboardprep.com.ph or email us at topnotchmedicalboardprep@gmail.com
Item QUESTION EXPLANATION AUTHOR TOPNOTCH
# EXAM
266 Which of the following functions as a reservoir of HDL is a reservoir of apoCII and apo E RAYMUND MIDTERM 3
apolipoproteins? MARTIN LI, EXAM - FEB
A. VLDL MD (TOP 1 2015
B. HDL - AUG 2014
C. LDL MED
D. Chylomicrons BOARDS;
E. IDL TOPNOTCH
MD)
267 A patient is diagnosed with megaloblastic anemia and All the others may also be found in folic acid RAYMUND MIDTERM 3
undergoes several laboratory tests. Which of the deficiency MARTIN LI, EXAM - FEB
following differentiates cobalamin from folic acid MD (TOP 1 2015
deficiency? - AUG 2014
A. Homocystinuria MED
B. Increased MCV BOARDS;
C. Decreased Hgb TOPNOTCH
D. Increased methylmalonic acid levels MD)
E. Decreased activity of methionine synthase

268 Interconversion of sugars between fructose 6-P, xylulose HMP shunt or Pentose phosphate pathway RAYMUND MIDTERM 3
5-P, Ribose 5-P, and Glyceraldehyde 3-P is made possible MARTIN LI, EXAM - FEB
by which pathway? MD (TOP 1 2015
A. Hexose monophosphate shunt - AUG 2014
B. Embden Meyerhoff pathway MED
C. Pentose Phosphate pathway BOARDS;
D. A and B TOPNOTCH
E. A and C MD)

269 The reaction catalyzed by the enzyme phosphoglucose A + standard free energy mostly proceed from right to RAYMUND MIDTERM 3
isomerase (glucose 6-P →fructose 6-P) in the glycolytic left and vice versa for a (-) standard free energy MARTIN LI, EXAM - FEB
pathway has a standard free energy +400 cal/mol. If MD (TOP 1 2015
isolated from the other enzymatic reactions from the - AUG 2014
pathway under standard conditions, which of the MED
following conclusions may be derived: BOARDS;
A. The reaction is in equilibrium TOPNOTCH
B. The reaction mostly proceeds from right to left MD)
C. More fructose 6-P is produced
D. A predominantly forward reaction takes place
E. No reaction takes place
270 Arsenic poisoning inhibits the conversion of pyruvate to Arsenic inhibits lipoic acid an essential component of RAYMUND MIDTERM 3
acetyl-coA. It will also inhibit the following: the enzymes pyruvate dehydrogenase, alpha- MARTIN LI, EXAM - FEB
A. reaction catalyzed by isocitrate dehydrogenase ketoglutarate dehydrogenase, and branched chain MD (TOP 1 2015
B. conversion of phosphoenolpuruvate to pyruvate dehydrogenase - AUG 2014
C. synthesis of ribose 5-phosphate MED
D. catabolism of branched-chain amino acids BOARDS;
E. None of the above TOPNOTCH
MD)

271 A patient presenting with fasting hypoglycemia and Cori disease or deficiency in debranching enzyme RAYMUND MIDTERM 3
accumulation of intracellular glycogen with abnormal MARTIN LI, EXAM - FEB
structure at branch points has which disorder: MD (TOP 1 2015
A. Glycogen phoshorylase deficiency - AUG 2014
B. Pompe disease MED
C. Glucose 6-Phosphatase deficiency BOARDS;
D. Cori disease TOPNOTCH
E. McArdle disease MD)

272 Myoglobin is a globular, flexible molecule found All the others bonds that stabilize tertiary and RAYMUND MIDTERM 3
predominantly in slow-twitch muscle fibers. It is quaternary structures MARTIN LI, EXAM - FEB
composed of alpha-helical secondary structure that is MD (TOP 1 2015
stabilized by: - AUG 2014
A. Disulfide bonds MED
B. Hydrophobic interactions BOARDS;
C. Ionic interactions TOPNOTCH
D. Hydrogen bonds MD)
E. All of the above

273 An infant presenting with increased orotic acid levels, The presence of hyperammonemia differentiates RAYMUND MIDTERM 3
hyperammonemia, and neurologic manifestations will ornithine transcarbamylase deficiency from deficiency MARTIN LI, EXAM - FEB
most likely have which deficiency? of pyrimidine synthesis enzyme deficiency that cause MD (TOP 1 2015
A. Ornithine transcarbamylase orotic aciduria - AUG 2014
B. N-acetylglutamate MED
C. Carbamoyl phosphate synthase I BOARDS;
D. Carbamoyl phosphate synthase II TOPNOTCH
E. Bifunctional pyrimidine synthesis enzyme MD)

274 What is the rate-limiting enzyme in the TCA cycle? RAYMUND MIDTERM 3
A. Alpha-ketoglutarate dehydrogenase MARTIN LI, EXAM - FEB
B. Malate dehydrogenase MD (TOP 1 2015
C. Succinate dehydrogenase - AUG 2014
D. Isocitrate dehydrogenase MED
E. None of these BOARDS;
TOPNOTCH
MD)

TOPNOTCH MEDICAL BOARD PREP BIOCHEMISTRY SUPEREXAM Page 35 of 92


For inquiries visit www.topnotchboardprep.com.ph or email us at topnotchmedicalboardprep@gmail.com
TOPNOTCH MEDICAL BOARD PREP BIOCHEMISTRY SUPEREXAM
For inquiries visit www.topnotchboardprep.com.ph or email us at topnotchmedicalboardprep@gmail.com
Item QUESTION EXPLANATION AUTHOR TOPNOTCH
# EXAM
275 A person is in his second day of religious fasting, which Glycogen stores good for 12-18 hours only. After that, RAYMUND MIDTERM 3
of the following metabolic pathway is primarily gluconeogenesis is responsible for maintaining MARTIN LI, EXAM - FEB
responsible for maintaining glucose concentration in the glucose levels MD (TOP 1 2015
circulation: - AUG 2014
A. breakdown of glycogen stores MED
B. ketones synthesis BOARDS;
C. uptake and phosphorylation of glycerol by the liver TOPNOTCH
D. Embden Meyerhoff pathway MD)
E. Pentose phosphate pathway

276 Collagen undergoes extensive posttranslational All others occur inside the cell RAYMUND MIDTERM 3
modification. Which of the following occurs outside the MARTIN LI, EXAM - FEB
cell? MD (TOP 1 2015
A. Triple helix formation - AUG 2014
B. Cleavage of C-terminal sequence MED
C. Hydroxylation of lysine and proline BOARDS;
D. Removal of signal sequence TOPNOTCH
E. Glycosylation of lysine MD)

277 Glycogenesis is enhanced by: All others enhance gluconeogenesis RAYMUND MIDTERM 3
A. enzyme phosphorylation MARTIN LI, EXAM - FEB
B. activation of protein phosphatase MD (TOP 1 2015
C. increased cAMP concentration - AUG 2014
D. activation of protein kinase A MED
E. All of the above BOARDS;
TOPNOTCH
MD)
278 Which of the following amino acids is a direct donor of Aspartate directly donates NH3 in urea synthesis RAYMUND MIDTERM 3
NH3 in urea synthesis? MARTIN LI, EXAM - FEB
A. Aspartate MD (TOP 1 2015
B. Glutamate - AUG 2014
C. Glutamine MED
D. Arginine BOARDS;
E. Alanine TOPNOTCH
MD)
279 In gluconeogenesis, reversal of the glycolytic enzyme Succinate thiokinase is responsible for substrate level RAYMUND MIDTERM 3
pyruvate kinase involves two reactions, the second of phosphorylation that produces GTP, which is MARTIN LI, EXAM - FEB
which is catalyzed by phosphoenolpyruvate preferentially used by phosphoenolpyruvate MD (TOP 1 2015
carboxykinase which utilizes a source of energy from a carboxykinase - AUG 2014
substrate level phosphorylation such as the reaction MED
catalyzed by: BOARDS;
A. alpha-ketoglutarate dehydrogenase TOPNOTCH
B. malate dehydrogenase MD)
C. phosphofructokinase 1
D. succinate thiokinase
E. A and B
280 Hypogonadotrophic dwarfism and acrodermatitis RAYMUND MIDTERM 3
enteropathica is caused by: MARTIN LI, EXAM - FEB
A. Vitamin E deficiency MD (TOP 1 2015
B. Vitamin A excess - AUG 2014
C. Zinc deficiency MED
D. Ascorbic acid deficiency BOARDS;
E. Vitamin A deficiency TOPNOTCH
MD)
281 Which of the following characterizes the enzyme Characteristics of Glucokinase: 1. Present only in liver ERIC ROYD FINAL EXAM -
glucokinase? parenchymal cells and islet cells of pancreas, 2. TALAVERA, FEB 2015
A. Low Vmax inhibited by fructose-6-phosphate, 3. High km (low MD (TOP 1
B. High Km affinity), 4. high vmax, . - AUG 2014
C. Inhibited by glucose-6-phosphate MED
D. Present in most tissues BOARDS;
E. All of the above TOPNOTCH
MD)
282 Which of the following amino acids does not undergo All amino acids with the exception of LYSINE and ERIC ROYD FINAL EXAM -
transamination at some point in its catabolism? THREONINE participate in transamination TALAVERA, FEB 2015
A. Histidine MD (TOP 1
B. Valine - AUG 2014
C. Tryptophan MED
D. Aspartate BOARDS;
E. Lysine TOPNOTCH
MD)

283 Which of the following is true regarding Vitamin K? Vitamin K, a fat soluble vitamin, is an essential ERIC ROYD FINAL EXAM -
A. It is a water soluble vitamin cofactor for the synthesis of factors II, VII, IX and X. It TALAVERA, FEB 2015
B. It is not synthesized in the human body DECREASES/SHORTENS the coagulation time MD (TOP 1
C. Plays an essential role in coagulation (particularly the PT). It is synthesized by the normal - AUG 2014
D. Present at high concentration in cow's or breast milk gut flora and is present at low concentration in MED
E. Important in management of hemorrhagic disorders cow's/breast milk. BOARDS;
by prolonging the coagulation time. TOPNOTCH
MD)

TOPNOTCH MEDICAL BOARD PREP BIOCHEMISTRY SUPEREXAM Page 36 of 92


For inquiries visit www.topnotchboardprep.com.ph or email us at topnotchmedicalboardprep@gmail.com
TOPNOTCH MEDICAL BOARD PREP BIOCHEMISTRY SUPEREXAM
For inquiries visit www.topnotchboardprep.com.ph or email us at topnotchmedicalboardprep@gmail.com
Item QUESTION EXPLANATION AUTHOR TOPNOTCH
# EXAM
284 Which of the following pathways produces carbon Heme catabolism is first carried out by the enzyme ERIC ROYD FINAL EXAM -
monoxide as a by-product? heme oxygenase which produces an equimolar TALAVERA, FEB 2015
A. Heme Catabolism amount of biliverdin, with ferric iron and carbon MD (TOP 1
B. Glycogenolysis monoxide being released - AUG 2014
C. Fatty Acid Synthesis MED
D. Urea Cycle BOARDS;
E. Hexose Monophosphate Shunt TOPNOTCH
MD)
285 How does azithromycin inhibit bacterial protein A - aminoglycoside, B - Rifampicin, D - ERIC ROYD FINAL EXAM -
synthesis in order to extert its antibacterial effect? Chloramphenicol, E- Tetracycline TALAVERA, FEB 2015
A. Interferes with the initiation of protein synthesis MD (TOP 1
B. Inhibits the DNA depended RNA polymerase - AUG 2014
C. Inhibits translocation by irreversibly binding to a MED
site on the 50s ribosome BOARDS;
D. Inhibits the enzyme peptidyltransferase TOPNOTCH
E. Blocks the binding of aminoacyl TRNA to the MD)
acceptor site
286 Chronic Alcoholism often leads to fatty liver by As part of alcohol metabolism, ethanol is oxidized to ERIC ROYD FINAL EXAM -
consuming what coenzyme which is needed for beta acetaldehyde by the enzyme alcohol dehydrogenase to TALAVERA, FEB 2015
oxidation of fatty acids? produce acetaldehyde, this is coupled with NAD being MD (TOP 1
A. NAD+ reduced to NADH2. NAD is also essential in one of the - AUG 2014
B. NADH oxidative steps in the beta oxidation of fatty acids. If MED
C. NADPH NAD is consumed by means of chronic alcohol BOARDS;
D. FAD consumption, the beta oxidation will not proceed TOPNOTCH
E. FADH2 hence accumulation of fat within the hepatocytes MD)
(steatosis)
287 What is the biochemical mechanism behind the Hepatic encephalopathy results in the shunting of ERIC ROYD FINAL EXAM -
development of seizures in individuals with hepatic nitrogenous material INSIDE the neurons. The TALAVERA, FEB 2015
encephalopathy? increased ammonia levels shift the transamination MD (TOP 1
A. Elevated levels of alpha ketoglutrate hence reaction to the synthesis of glutamate rather than - AUG 2014
producing increased ATP production alpha ketoglutarate, hence the TCA will not proceed MED
B. An increase in the level of the neurotransmitter leading to DEPLETION of ATP. Glutamate is an BOARDS;
glycine inside the brain EXCITATORY neurotransmitter and its elevated levels TOPNOTCH
C. Nitrogenous substances shunted out of the brain can stimulate the cortical neurons producing seizure MD)
resulting in depletion of ammonia levels episodes. GABA is also increased in individuals with
D. Increased synaptic release of glutamate hepatic encephalopathy as it is produced through
E. None of the above decarboxylation of glutamate. However it is an
inhibitory neurotransmitter and is primarily
responsibe for COMA in advanced stages. Glycine is an
inhibitory neurotransmitter in the spinal cord
288 Cataracts are common in poorly controlled diabetic Poorly controlled diabtetics have elevated levels of ERIC ROYD FINAL EXAM -
individuals due to what biochemical mechanism? glucose in the blood. The glucose accumulates inside TALAVERA, FEB 2015
A. Decreased sorbitol levels which produce cellular the lens (the lens does not require insulin for glucose MD (TOP 1
degradation of the proteins in the lens uptake), which is eventually metabolized to sorbitol - AUG 2014
B. Insulin deficiency inhibits synthesis of proteins in through the enzyme ALDOSE REDUCTASE. The lens MED
the lens lack the enzyme SORBITOL DEHYDROGENASE which BOARDS;
C. Osmotic shift due to the increased levels of sorbitol metabolizes sorbitol further to fructose TOPNOTCH
inside the lens MD)
D. Absence of the enzyme aldose reductase which is
responsible for sorbitol metabolism
E. Both C and D are correct
289 Increased formation of ketone bodies during starvation Free fatty acids are increased due to the increased ERIC ROYD FINAL EXAM -
is due to? activity of hormone sensitive lipase (stimulated by TALAVERA, FEB 2015
A. Decreased levels of circulating glucagon Glucagon, in the fasting state). Acetyl-CoA is the end MD (TOP 1
B. Increased levels of free fatty acids in the blood product of fatty acid oxidation which is a substrate - AUG 2014
C. Decreased levels of acetyl-CoA in the blood needed for ketone body synthesis MED
D. Decreased activity of hormone sensitive lipase BOARDS;
E. Inhibition of beta oxidation of fatty acid TOPNOTCH
MD)
290 Which of the following chemotherapeutic drugs acts by 5FU inhibits thymidylate synthase leading to a ERIC ROYD FINAL EXAM -
inhibiting the enzyme thymidylate synthase? "thymineless" cell death, TALAVERA, FEB 2015
A. Gemcitabine MD (TOP 1
B. Paclitaxel - AUG 2014
C. Bleomycin MED
D. Vincristine BOARDS;
E. 5-FU TOPNOTCH
MD)

291 Which of the following is true about the enzyme acetyl- ACC requires BIOTIN as a coenzyme. It is found in the ERIC ROYD FINAL EXAM -
CoA carboxylase? CYTOPLASM. It utilizes ACETYL-COA as a substrate. It TALAVERA, FEB 2015
A. It requires thiamine for the carboxylation of acetyl- is the rate limiting step of FATTY ACID SYNTHESIS MD (TOP 1
CoA - AUG 2014
B. It is located mainly in the matrix of the hepatic MED
mitochondria BOARDS;
C. It utilizes citrate as a substrate TOPNOTCH
D. It produces malonyl-CoA MD)
E. It catalyzes the rate limitting step in the beta
oxidation of fatty acids

TOPNOTCH MEDICAL BOARD PREP BIOCHEMISTRY SUPEREXAM Page 37 of 92


For inquiries visit www.topnotchboardprep.com.ph or email us at topnotchmedicalboardprep@gmail.com
TOPNOTCH MEDICAL BOARD PREP BIOCHEMISTRY SUPEREXAM
For inquiries visit www.topnotchboardprep.com.ph or email us at topnotchmedicalboardprep@gmail.com
Item QUESTION EXPLANATION AUTHOR TOPNOTCH
# EXAM
292 An infant diagnosed with Von Gierke's disease has the Von Gierke's disease: deficiency in glucose-6- ERIC ROYD FINAL EXAM -
following characteristics EXCEPT? phosphatase which is an enzyme utilized in TALAVERA, FEB 2015
A. Cannot utilize hepatic gluconeogenesis to maintain gluconeogenesis and glycogenolysis. Affected infants MD (TOP 1
blood glucose levels develop hypoglycemia and if severe enough could - AUG 2014
B. Characterized by a deficiency of the enzyme glucose- progress to lactic acidosis. Muscle glycogen cannot be MED
6-phosphatase used (even in normal individuals) to produce free BOARDS;
C. Can utilize the muscle glycogen to maintain blood blood glucose since muscle lacks glucose-6- TOPNOTCH
glucose levels phosphatase MD)
D. Develops lactic acidosis due to the hypoglycemia
E. None of the above
293 Which of the following substances inhibit Complex III of SIMILAR TO PREVIOUS BOARD EXAM ERIC ROYD FINAL EXAM -
the Electron Transport Chain? CONCEPT/PRINCIPLE: Complex I inhibitors TALAVERA, FEB 2015
A. Barbiturates (Barbiturate, Piericidin A, Amytal, Rotenone), MD (TOP 1
B. Dimercaprol Complex II inhibitors (Malonate, Carboxin, TTFA), - AUG 2014
C. Cyanide Complex III inhibitors (Dimercaprol, Antimycin A) MED
D. Hydrogen Sulfide Complex IV inhibitors (Cyanide, Carbon monoxide, BOARDS;
E. Malonate Sodium azide, Hydrogen sulfide) TOPNOTCH
MD)

294 An individual with lipoprotein lipase deficiency would If the activity of lipoprotein lipase is decreased, both ERIC ROYD FINAL EXAM -
have which of the following changes in blood lipid plasma chylomicrons and VLDL would become TALAVERA, FEB 2015
levels? elevated MD (TOP 1
A. Elevation of plasma HDL level - AUG 2014
B. Elevation of chylomicron level MED
C. Decreased triglyceride levels BOARDS;
D. Elevated LDL levels TOPNOTCH
E. Decreased VLDL levels MD)

295 An infant who was initially normal at birth began to Arginosuccinate synthase catalyzes the condensation ERIC ROYD FINAL EXAM -
develop lethargy, hypothermia and apnea. He was later of citrulline with L-aspartate to form arginine. Any TALAVERA, FEB 2015
found out to have a deficiency of the enzyme intermediates before this step would accumulate MD (TOP 1
arginosuccinate synthase. Which of the following would while all those that occur afterwards would have - AUG 2014
be expected? decreased levels MED
A. High blood levels of arginine BOARDS;
B. Low blood levels of ammonia TOPNOTCH
C. Low blood levels of aspartate MD)
D. High blood levels of ornithine
E. High blood levels of citrulline
296 Patients with xeroderma pigmentosum suffer DNA UV light causes formation of pyrimidine dimers in ERIC ROYD FINAL EXAM -
damage upon exposure to UV light through formation of DNA TALAVERA, FEB 2015
which of the following? MD (TOP 1
A. Pyrimidine dimers - AUG 2014
B. Purine dimers MED
C. Deoxyribose dimers BOARDS;
D. Anhydrous bonds in the DNA TOPNOTCH
E. All of the above MD)

297 The carbon skeleton of triacylglycerol can be used for Glycerol is released during TAG hydroysis, Glycerol is ERIC ROYD FINAL EXAM -
gluconeogenesis by entering the pathway as what phosphorylated to glycerol phosphate which is then TALAVERA, FEB 2015
substrate? oxidized to form dihydroxyacetone phosphate. MD (TOP 1
A. 3-phosphoglycerate - AUG 2014
B. Glucose-6-phosphate MED
C. Fructose 1,6-bisphosphate BOARDS;
D. Dihydroxyacetone phosphate TOPNOTCH
E. Phosphoenolpyruvate MD)

298 The neurologic manifestations seen in Niemann Pick Niemann Pick disease - deficiency of ERIC ROYD FINAL EXAM -
disease are associated with the accumulation of which of sphingomyelinase enzyme TALAVERA, FEB 2015
the following? MD (TOP 1
A. Gangliosides - AUG 2014
B. Cerebroside MED
C. Prostaglandin BOARDS;
D. Leukotriene TOPNOTCH
E. Sphingomyelin MD)

299 A 24 year old male medical student complains of Prolactinoma - most common functioning pituitary ERIC ROYD FINAL EXAM -
bilateral white discharge from his breast. Which of the tumor, located in the anterior pituitary gland. Most TALAVERA, FEB 2015
following is true about his probable condition ? are microadenoma and is the only pituitary tumor that MD (TOP 1
A. There is excessive production of ADH by his can be treated medically with dopamine analogs (such - AUG 2014
posterior pituitary as Bromocriptine) MED
B. His testosterone levels are markedly decreased BOARDS;
C. His disease condition would respond by TOPNOTCH
administering Bromocriptine MD)
D. Oxytocin levels are markedly elevated
E. There is most likely a tumor present that involves
the posterior pituitary
300 Which of the following minerals is essential for the In the form of selenocysteine, selenium is essential for ERIC ROYD FINAL EXAM -
activity of the enzyme glutathione peroxidase? the activity of the anti-oxidanrt glutathione TALAVERA, FEB 2015
A. Molybdenum peroxidase MD (TOP 1
B. Manganese - AUG 2014
C. Iron MED
D. Selenium BOARDS;
E. Copper TOPNOTCH
MD)

TOPNOTCH MEDICAL BOARD PREP BIOCHEMISTRY SUPEREXAM Page 38 of 92


For inquiries visit www.topnotchboardprep.com.ph or email us at topnotchmedicalboardprep@gmail.com
TOPNOTCH MEDICAL BOARD PREP BIOCHEMISTRY SUPEREXAM
For inquiries visit www.topnotchboardprep.com.ph or email us at topnotchmedicalboardprep@gmail.com
Item QUESTION EXPLANATION AUTHOR TOPNOTCH
# EXAM
301 Which of the following is true about mammalian pyruvate dehydrogenase complex is one of the most LEAN BACK-UP
pyruvate dehydrogenase complex under physiologic highly regulated enzyme in terms of carbohydrate ANGELO MIDTERM
condition ? metabolism. It is a mitochondrial enzyme complex SILVERIO, EXAM - FEB
A. It is composed of 3 types of catalytic enzyme subunits composed of 3 catalytic subunits namely: pyruvate MD (TOP 4 2015
B. Phosphorylation of this enzyme will increase the dehydrogenase, dihydrolipoyl transacetylase, and - AUG 2014
formation of acetyl CoA dihydrolipoyl dehydrogenase. cofactors essential for MED
C. the presence of epinephrine will activate the PDH in the complex include the ff: FAD, thiamine BOARDS;
adipose tissue pyrophosphate, lipoamide,Coenzyme A (pantothenic TOPNOTCH
D. high NADH/NAD ratio will promote active form of acid), and NAD. the process involves the conversion of MD), MD
the complex pyruvate to acetyl CoA along with release of CO2 and
E. pyridoxal phosphate is an esential cofactor of the NADH as byproduct. accumulation of its product (
complex acetylCoA,NADH) will inactivate the complex via
competitive feedback. furthermore, phosphorylation
of the enzyme by protein kinase will also inactivate
the complex. the reverse is true about
dephosphorylation. catecholamines particularly
epinephrine will stimulate PDH in the cardiac tissue
while insulin will stimulate this adipose tissue PDH.
Devlin biochemistry pp 540-542 6th ed.
302 Which of the following intermediate substances in the the administration of fluoroacetate will inhibit the LEAN BACK-UP
kreb cycle will accumulate when fluoroacetate enters the enzyme aconitase found in the mitochondrial matrix. ANGELO MIDTERM
mitochondrial matrix? This enzyme is part of the kreb cycel responsible for SILVERIO, EXAM - FEB
A. Alpha ketoglutarate converting citrate into isocitrate. Therefore, citrate MD (TOP 4 2015
B. citrate will accumulate as a result. On the otherhand, arsenic - AUG 2014
C. Isocitrate inhibit alpha ketoglutarate dehydrogenase complex MED
D. Succinyl CoA resulting to accumulation of alpha ketoglutarate and BOARDS;
E. None of the abvoe deficiency in succinyl CoA. harpers 27th ed pp 145- TOPNOTCH
146. MD), MD
303 Which of the following is an effect of salicylate overdose supradoses of aspirin (salicylates) will uncouple the LEAN BACK-UP
in the oxidative phosphorylation process? electron transport chain by promoting permeabilty of ANGELO MIDTERM
A. It binds to the ATP synthetase preventing the H the inner mitochondrial membrane to hydrogen. This SILVERIO, EXAM - FEB
reentry into the mitochondrial matrix will result to loss of hydrogen gradient despite MD (TOP 4 2015
B. It inhibits the electron transport via complex I by continous process of ETC. in effect, much of the energy - AUG 2014
blocking the transfer from Fe-S to Q produced in the electron transfer will dissipate as heat MED
C. competitively inhibits the succinate Q reductase( and consequently leads to fever on the part of the BOARDS;
Complex II) patient. the same mechanism is involved in ingestion TOPNOTCH
D. it promotes increase permeability of the inner of 2,4 dinitrophenol and during brown fat MD), MD
mitochondrial membrane thermogenesis via thermogenin protein in
E. inhibits the cytochrome c oxidase (complex IV) neonates.barbiturates, ptericidin A, rotenone are
resulting to inability to transfer electrons to oxygen inhibitors of complex I, carboxin, TTFA, malonate are
atom to Complex II; antimycin A and dimercaprol (BAL) are
to Complex III while hydrogen sulfide, CO, and
Cyanide are to complex IV. oligomycin on the
otherhand inhibits the H reentry in the ATP
syntethase. Harpers 27th ed pp 107-108
304 which of the following enzymes is highly active in the (SIMILAR TO PREVIOUS BOARD EXAM LEAN BACK-UP
seminal vesicles? CONCEPT/PRINCIPLE). The major energy source of ANGELO MIDTERM
A. Sorbitol dehydrogenase spermatozoa is fructose. This is produced by the cells SILVERIO, EXAM - FEB
B. Aldolase B of the seminal vesicles from glucose by two step MD (TOP 4 2015
C. Aldose reductase process. First is by NADPH dependent reduction of - AUG 2014
D. Glycerol 3 phosphate dehydrogenase glucose to sorbitol( aldose reductase) followed by MED
E. None of the above NAD dependent oxidation of sorbitol to fructose BOARDS;
(sorbitol dehydrogenase). of the two enzymes, the TOPNOTCH
activity of the former is comparable to all other cells MD), MD
while the latter enzymes is specifically active in
seminal vesicles. Devlin 6th p 615
305 Which of the following is the end product of propionyl CoA is the end product of the ff metabolism: LEAN BACK-UP
isoleucine,valine and methionine metabolism? isoleucine, valine, methionine, odd chain fatty acid ANGELO MIDTERM
A. Acetyl CoA oxidation and side chain of cholesterol degradation. SILVERIO, EXAM - FEB
B. Succinyl CoA Conversion of proprionyl CoA to methylmalonyl CoA MD (TOP 4 2015
C. Propionyl CoA is by the biotin requiring enzyme propionyl CoA - AUG 2014
D. Methylmalonyl CoA carboxylase. further conversion of methylmalonyl CoA MED
E. None of the above to succinyl CoA is by Vit B12 requiring methylmalonyl BOARDS;
CoA mutase. Devlin 6th ed p777 TOPNOTCH
MD), MD
306 how many ATP/s is/are required to produce urea in Kreb Henseleit cycle or the urea cycle requires 4 ATPs LEAN BACK-UP
Kreb Henseleit Cycle? to produce and excrete urea. Two ATPs are required ANGELO MIDTERM
A. 1 ATP in the formation of carbamoyl phosphate and 2 ATPs SILVERIO, EXAM - FEB
B. 2 ATP for the formation of arginosuccinate. Refer to your MD (TOP 4 2015
C. 3 ATP handouts for the diagram. Devlin 6th ed pp 752 - AUG 2014
D. 4 ATP MED
E. No ATP needed. BOARDS;
TOPNOTCH
MD), MD
307 which of the following amino acid is an essential phosphatidylcholine is an essential component of cell LEAN BACK-UP
component in the de novo synthesis of membranes, myelin sheath and mature pulmonary ANGELO MIDTERM
phosphatidylcholine? surfactant. It is formed by phosphorylation of choline SILVERIO, EXAM - FEB
A. arginine obtained from the diet followed by addition of 3 MD (TOP 4 2015
B. threonine methyl groups. The activated form of methionin (S- - AUG 2014
C. serine adenosyl methionine) is the only amino acid that can MED
D. Methionine donate methyl groups. BOARDS;
E. None of the above TOPNOTCH
MD), MD

TOPNOTCH MEDICAL BOARD PREP BIOCHEMISTRY SUPEREXAM Page 39 of 92


For inquiries visit www.topnotchboardprep.com.ph or email us at topnotchmedicalboardprep@gmail.com
TOPNOTCH MEDICAL BOARD PREP BIOCHEMISTRY SUPEREXAM
For inquiries visit www.topnotchboardprep.com.ph or email us at topnotchmedicalboardprep@gmail.com
Item QUESTION EXPLANATION AUTHOR TOPNOTCH
# EXAM
308 true about sphingolipidoses except? sphingolipidosis is a group of inherited disorder LEAN BACK-UP
A. There is upregulation in the synthesis of stored characterized by accumulation of complex lipids. This ANGELO MIDTERM
lipids is primarily due to defective lyososomal degradation SILVERIO, EXAM - FEB
B. Most are autosomal recessive pathway. It particularly affect the CNS because of its MD (TOP 4 2015
C. Accumulation of GM2 ganglioside is caused by reliance to complex lipid myelination processes. The - AUG 2014
hexosaminidase A deficiency synthesis of stored lipids is intact and not MED
D. Deficiency of alpha galactosidase is the cause of upregulated. Most are autosomal recessive except for BOARDS;
fabry's disease fabry disease ( deficiency in a galactosidase) which is TOPNOTCH
E. none of the above x linked recessive.Tay sach disease presenting mental MD), MD
retardation, cherry red macula and muscular
weakness, is due to deficiency in hexosaminidase A
leading accumulation of GM2 ganglioside.
309 Which of the following processes does not occur during adipocytes does not have glycerol kinase instead LEAN BACK-UP
mobilization of stored fats and fatty acid oxidation? glycerol is transported to the liver wherein it is ANGELO MIDTERM
A. cAMP mediated activation of hormone sensitive phosphorylated to produce dihydroacetone phosphate SILVERIO, EXAM - FEB
lipase which then can be converted to metabolic precursors MD (TOP 4 2015
B. Upregulation of glycerol kinase in the adipocytes for glycolysis or gluconeogenesis. Lippincott 2nd ed - AUG 2014
C. Transfer of acyl group from coenzyme A to carnitine pp 181-182 MED
for transfer to the mitochondrial matrix BOARDS;
D. shortening of the fatty acid chain via sequential TOPNOTCH
oxidation,hydration,oxidation, thiolysis reaction MD), MD
E. none of the above

310 failure in the addition of this signal sequence to I cell disease is characterized by accumulation of LEAN BACK-UP
lysosomal proteins leads to characteristic mental lysosomal proteins in the serum due to failure of the ANGELO MIDTERM
retardation, coarse facial features, and multiple joint latter to be compartmentalized in the lysosomes. This SILVERIO, EXAM - FEB
restriction of I cell disease. is due to absent addition of mannose 6 phosphate MD (TOP 4 2015
A. Glyceraldehyde 3 phosphate that is responsible for internalization of these - AUG 2014
B. apolipoproteins A2 enzymes. MED
C. Galactoacylceramide BOARDS;
D. Mannose 6 phosphate TOPNOTCH
E. none of the above MD), MD

311 During eukaryotic DNA replication, what type of important eukaryotic polymerase enzymes in DNA LEAN BACK-UP
polymerase is responsible for the synthesis of RNA replication are as follows: alpha - formation of RNA ANGELO MIDTERM
primer in both leading and lagging strand? primers for both leading and lagging strands, delta- SILVERIO, EXAM - FEB
A. Pol alpha elongation of leading strand, epsilon - elongation of MD (TOP 4 2015
B. Pol beta lagging strand, gamma - replication of mitochondrial - AUG 2014
C. Pol gamma DNA. MED
D. Pol delta BOARDS;
E. Pol epsilon TOPNOTCH
MD), MD
312 which of the following enzyme- product pair is inhibited alpha amanitin inhibits eukaryotic RNA polymerase II LEAN BACK-UP
even by low concentration amanitin (amanita phalloides enzyme that is responsible for the formation of mRNA. ANGELO MIDTERM
mushroom) poisoning ? RNA pol I forms the rRNA while RNA pol III is for SILVERIO, EXAM - FEB
A. RNA pol I/ tRNA tRNA,snRNA and 5S ribosomal RNA. Lippincott 2nd ed MD (TOP 4 2015
B. RNA pol II/mRNA p 384. - AUG 2014
C. RNA pol III/rRNA MED
D. RNA pol I/rRNA BOARDS;
E. RNA pol II/ snRNA TOPNOTCH
MD), MD
313 During bacterial RNA synthesis, what is the promoter for prokaryotic organisms, the promoter regions in LEAN BACK-UP
region for RNA polymerase to bind that is located 10 the DNA are the pribnow box ( 8-10 nucleotide ANGELO MIDTERM
bases upstream from transcription start site? upstream) and -35 sequence while for the eukaryotics SILVERIO, EXAM - FEB
A. Sigma factor it is the CAAT box (80 nucleotide upstream) and MD (TOP 4 2015
B. CAAT box TATA/hogness (25 nucleotide upstream) box. On the - AUG 2014
C. Pribnow box otherhand, sigma factor is a component of the RNA MED
D. TATA box polymerase responsible for recognizing the specific BOARDS;
E. Rho factor promoter region in the DNA strand while rho factor TOPNOTCH
promotes termination of RNA synthesis. MD), MD

314 The atoms of the purine ring are contributed by the purine ring atoms are contributed by the following: LEAN BACK-UP
following except? aspartate, glycine, glutamine, N10 ANGELO MIDTERM
A. Aspartic acid formyltetrahydrofolate and carbon dioxide. For SILVERIO, EXAM - FEB
B. glutamine pyrimidine ring it is contributed by glutamine, MD (TOP 4 2015
C. CO2 aspartic acid and CO2 lippincott 2nd ed p 345 - AUG 2014
D. Thiamine pyrophosphate MED
E. Tetrahydrofolate BOARDS;
TOPNOTCH
MD), MD
315 what is the specific mechanism on how clindamycin (SIMILAR TO PREVIOUS BOARD EXAM LEAN BACK-UP
exerts its effect in protein synthesis? CONCEPT/PRINCIPLE) A- streptomycin,B- ANGELO MIDTERM
A. Binds to 30s subunit inhibiting initiation of protein tetracycline, D- chlorampenicol. Lippincott 2nd ed p SILVERIO, EXAM - FEB
synthesis 396 MD (TOP 4 2015
B. Blocks the interaction between aminoacyl tRNA to the - AUG 2014
mRNA ribosome complex MED
C. irreversible binding to 50s subunit preventing BOARDS;
translocation TOPNOTCH
D. inhibits 50s peptidyl transferase preventing MD), MD
translocation
E. none of the above

TOPNOTCH MEDICAL BOARD PREP BIOCHEMISTRY SUPEREXAM Page 40 of 92


For inquiries visit www.topnotchboardprep.com.ph or email us at topnotchmedicalboardprep@gmail.com
TOPNOTCH MEDICAL BOARD PREP BIOCHEMISTRY SUPEREXAM
For inquiries visit www.topnotchboardprep.com.ph or email us at topnotchmedicalboardprep@gmail.com
Item QUESTION EXPLANATION AUTHOR TOPNOTCH
# EXAM
316 the compromise in the energy production seen in wet wet beriberi is a type of severe thiamine deficiency LEAN BACK-UP
beriberi is primarily due to what altered process? characterized by high output heart failure, ANGELO MIDTERM
A. Absence of cofactor in the transketolase activity neuromuscular symptoms and weakness. The lack of SILVERIO, EXAM - FEB
B. Inhibition of PDH and alpha ketoglutarate complex energy generation by the cells is due to inhibition of MD (TOP 4 2015
C. Lack of cofactor supply during oxidation and PDH and a ketogluterate dehydrogenase complex - AUG 2014
reduction reactions leading to decrease in the levels of ATP and reducing MED
D. inhibition of transamination reaction leading to equivalents. transketolase reaction is also catalyzed BOARDS;
underutilization of amino acids during gluconeogenesis by thiamine however it is not involved in energy TOPNOTCH
E. all of the above generating pathway but rather it is required in HMP. MD), MD
transamination reaction requires pyridoxine as a
cofactor. oxidation reduction reactions uses riboflavin
(FAD) and niacin (NAD and NADP) Devlin 6th ed pp
1102-1103
317 Supplementation of which trace mineral appears to chromium is the component of low molecular weight LEAN BACK-UP
improve the glycemic control of patients with type II chromodulin which potentiates the effect of insulin by ANGELO MIDTERM
diabetes mellitus? facilitating insulin binding to its receptor and receptor SILVERIO, EXAM - FEB
A. Chromium tyrosine kinase signalling. Devlin pp 1114 MD (TOP 4 2015
B. Copper - AUG 2014
C. Zinc MED
D. Selenium BOARDS;
E. Molybdenum TOPNOTCH
MD), MD
318 Formation of Nitric oxide requires which of the following Nitric oxide formerly known as EDRF is formed by the LEAN BACK-UP
substances? enzyme NO synthase. It uses arginine as a substrate ANGELO MIDTERM
A. Asparagine for oxidation to produce citrulline and NO. Harpers pp SILVERIO, EXAM - FEB
B. Acetylcholine 580 MD (TOP 4 2015
C. Citrulline - AUG 2014
D. Arginine MED
E. Glycine BOARDS;
TOPNOTCH
MD), MD
319 The following are the characteristics of smooth muscle Caldesmon is a regulatory protein ubiquitous in the LEAN BACK-UP
except? smooth muscle.at low calcium concentration, it binds ANGELO MIDTERM
A. Caldesmon promotes interaction of actin and myosin to tropomyosin and actin thereby preventing the SILVERIO, EXAM - FEB
B. Contains rudimentary sarcoplasmic reticulum interaction of actin and myosin. Harpers 27th ed p MD (TOP 4 2015
C. Extracellular fluid Ca is important for contraction 579-580 - AUG 2014
D. Exhibits latch state MED
E. none of the above BOARDS;
TOPNOTCH
MD), MD
320 A 4 weeks old neonate presents to your clinic with this is a classic case of Pompe disease caused by LEAN BACK-UP
generalized hypotonia, respiratory distress, dilated deficiency of a 1,4 glucosidase aka acid maltase. It is a ANGELO MIDTERM
jugular veins, and displaced apex beat. Laboratory shows form of enzyme defect wherein there is accumulation SILVERIO, EXAM - FEB
normal glucose levels, uric acid and blood lactate level. of glycogen granules inside the lysosomes. It doesn’t MD (TOP 4 2015
which of the following enzyme is deficient in this present with hypoglycemia and lactic acidosis since - AUG 2014
patient? enzymes for glycogen metabolism are intact. the most MED
A. glycogen debranching enzyme common presentation is cardiomyopathy with heart BOARDS;
B. muscle glycogen phosphorylase failure. no treatment is available and death usually TOPNOTCH
C. alpha 1,4 glucosidase occurs at 1 mo of age. Devlin p 621. MD), MD
D. glucose 6 phosphatase
E. none of the above
321 Which glycolytic enzyme catalyzes a reversible reaction? SIMILAR TO PREVIOUS BOARD EXAM DEBBIE BACK-UP
A. glyceraldehyde phosphate dehydrogenase CONCEPT/PRINCIPLE. Glyceraldehyde phosphate ROSE MIDTERM
B. hexokinase dehydrogenase yields 1,3-bisphosphoglycerate using TANENGSY EXAM - FEB
C. phosphofructokinase-1 glyceraldehyde-3-phosphate as its substrate. , MD (TOP 2015
D. pyruvate kinase 5 - AUG
E. none of the above 2014 MED
BOARDS;
TOPNOTCH
MD)
322 Deficiency of this fatty acid leads to decreased vision and SIMILAR TO PREVIOUS BOARD EXAM DEBBIE BACK-UP
altered learning behavior. CONCEPT/PRINCIPLE. Linolenic acid / omega 3 fatty ROSE MIDTERM
A. linoleic acid acid deficiency results in decreased vision and altered TANENGSY EXAM - FEB
B. omega 6 fatty acid learning behavior. Linoleic acid / omega 6 fatty acid is , MD (TOP 2015
C. arachidonic acid a precursor to arachidonic acid is thus essential in 5 - AUG
D. linolenic acid prostaglandin synthesis. 2014 MED
E. none of the above BOARDS;
TOPNOTCH
MD)
323 The following are causes of conjugated In Dubin Johnson syndrome, the gene encoding MRP- DEBBIE BACK-UP
hyperbilirubinemia, except: 2, involved in the secretion of already conjugated ROSE MIDTERM
A. Dubin Johnson syndrome bilirubin into bile, is mutated. Rotor syndrome is TANENGSY EXAM - FEB
B. Rotor syndrome characterized by chronic conjugated , MD (TOP 2015
C. biliary atresia hyperbilirubinemia and a normal liver histology. 5 - AUG
D. Gilbert syndrome Biliary atresia is a form of biliary tree obstruction and 2014 MED
E. no exception a cause of conjugated hyperbilirubinemia which BOARDS;
results in the regurgiation of bilirubin glucuronide TOPNOTCH
into hepatic veins and lymphatics. Gilbert syndrome is MD)
caused by mutations in the gene encoding bilirubin-
UGT. Harper Biochemistry 28th edition p.282

TOPNOTCH MEDICAL BOARD PREP BIOCHEMISTRY SUPEREXAM Page 41 of 92


For inquiries visit www.topnotchboardprep.com.ph or email us at topnotchmedicalboardprep@gmail.com
TOPNOTCH MEDICAL BOARD PREP BIOCHEMISTRY SUPEREXAM
For inquiries visit www.topnotchboardprep.com.ph or email us at topnotchmedicalboardprep@gmail.com
Item QUESTION EXPLANATION AUTHOR TOPNOTCH
# EXAM
324 True regarding the biochemical events occurring during Harper Biochemistry 28th edition p.550 DEBBIE BACK-UP
muscle contraction: ROSE MIDTERM
A. The S-1 head of myosin hydrolyzes ATP to ADP and TANENGSY EXAM - FEB
P1, which remain bound. , MD (TOP 2015
B. Formation of the actin-myosin-ADP-P1 complex 5 - AUG
promotes the release of P1, inititating the power stroke. 2014 MED
C. Myosin-ATP has a low affinity for actin, which is thus BOARDS;
released. TOPNOTCH
D. all of the above MD)
E. none of the above
325 What is the most potent positive allosteric activator of Harper Biochemistry 28th edition p.168 DEBBIE BACK-UP
phosphofructokinase-1, and an inhibitor of fructose-1,6- ROSE MIDTERM
bisphosphatase? TANENGSY EXAM - FEB
A. fructose-2,6-bisphosphate , MD (TOP 2015
B. fructose-6-phosphate 5 - AUG
C. glucose 2014 MED
D. acetyl CoA BOARDS;
E. none of the above TOPNOTCH
MD)
326 In the fed state, the following enzymes' activities are In the fasting state, glucagon stimulates the DEBBIE BACK-UP
expected to increase, except: production of cAMP, activating cAMP-dependent ROSE MIDTERM
A. phosphofructokinase-1 protein kinase, which in turn inactivates PFK-1 and TANENGSY EXAM - FEB
B. pyruvate dehydrogenase activates fructose-2,6-bisphosphatase by , MD (TOP 2015
C. glucokinase phosphorylation. Harper Biochemistry 28th edition 5 - AUG
D. glycogen synthase p.169 2014 MED
E. cAMP dependent protein kinase BOARDS;
TOPNOTCH
MD)
327 Bile acid conjugation with glycine or taurine takes place Harper Biochemistry 28th edition p.229 DEBBIE BACK-UP
in which organelle? ROSE MIDTERM
A. golgi apparatus TANENGSY EXAM - FEB
B. endoplasmic reticulum , MD (TOP 2015
C. cytosol 5 - AUG
D. peroxisomes 2014 MED
E. none of the above BOARDS;
TOPNOTCH
MD)
328 What is true regarding the pentose phosphate pathway? The enzymes involved in the pentose phosphate DEBBIE BACK-UP
A. The enzymes involved are mitochondrial. pathway are cytosolic. NADP+ is the electron acceptor, ROSE MIDTERM
B. Oxidation is achieved by dehydrogenation using NAD+ and this cycle does not yield ATP. Harper TANENGSY EXAM - FEB
as an electron acceptor. Biochemistry 28th edition p.175-1 , MD (TOP 2015
C. Like the TCA cycle, this pathway yields ATP. 5 - AUG
D. This pathway can account for the complete oxidation 2014 MED
of glucose. BOARDS;
E. Glutathione reductase contains selenocysteine at its TOPNOTCH
active site. MD)
329 Which is a true statement? Neonatal physiologic jaundice is the most common DEBBIE BACK-UP
A. Gilbert syndrome is the most common cause of cause of unconjugated hyperbilirubinemia. ROSE MIDTERM
unconjugated hyperbilirubinemia. Phenobarbital has no effect in the treatment of Crigler TANENGSY EXAM - FEB
B. Phenobarbital is effective in the treatment of Crigler Najar type I. An obstructive condition is suspected , MD (TOP 2015
Najar type I. when urinalysis reveals increased amount of 5 - AUG
C. Crigler Najar type II is the benign form. conjugated bilirubin but no urobilinogen. 2014 MED
D. A hemolytic condition is suspected when urinalysis BOARDS;
reveals increased amount of conjugated bilirubin but no TOPNOTCH
urobilinogen. MD)
E. none of the above
330 The following are glucogenic, except: Ketogenic only: Leu & Lys. Propionyl resude from an DEBBIE BACK-UP
A. leucine odd chain fatty acid is the only part of a fatty acid ROSE MIDTERM
B. propionyl CoA that's glucogenic. TANENGSY EXAM - FEB
C. lysine , MD (TOP 2015
D. A & B 5 - AUG
E. A & C 2014 MED
BOARDS;
TOPNOTCH
MD)
331 The following are true regarding non-alpha-amino acids, All statements are true. Harper Biochemistry 28th DEBBIE BACK-UP
except? edition pp.266-268 ROSE MIDTERM
A. GABA functions as an inhibitory neurotransmitter in TANENGSY EXAM - FEB
brain tissue , MD (TOP 2015
B. Catabolism of uracil yields B-alanine. 5 - AUG
C. Catabolism of thymine yields B-aminoisobutyrate. 2014 MED
D. none of the above BOARDS;
TOPNOTCH
MD)
332 Which is a true statement? In the reduction of pyruvate to lactate, NADH+ acts as DEBBIE BACK-UP
A. Pyruvate is reduced to form lactate. the electron donor; whereas oxidation of lactate uses ROSE MIDTERM
B. Lactate is oxidized to form pyruvate. NAD+ as the electron acceptor. TANENGSY EXAM - FEB
C. all of the above , MD (TOP 2015
D. none of the above 5 - AUG
2014 MED
BOARDS;
TOPNOTCH
MD)

TOPNOTCH MEDICAL BOARD PREP BIOCHEMISTRY SUPEREXAM Page 42 of 92


For inquiries visit www.topnotchboardprep.com.ph or email us at topnotchmedicalboardprep@gmail.com
TOPNOTCH MEDICAL BOARD PREP BIOCHEMISTRY SUPEREXAM
For inquiries visit www.topnotchboardprep.com.ph or email us at topnotchmedicalboardprep@gmail.com
Item QUESTION EXPLANATION AUTHOR TOPNOTCH
# EXAM
333 The following amino acids are catabolized via a The catabolism of leucine, isoleucine, and valine DEBBIE BACK-UP
multimeric enzyme complex closely resembling pyruvate involves reactions analogous to catabolism of fatty ROSE MIDTERM
dehydrogenase, except: acids. There is an involvement of the enzyme TANENGSY EXAM - FEB
A. leucine branched chain alpha keto acid dehydrogenase, a , MD (TOP 2015
B. isoleucine multimeric enzyme complex of a decarboxylase, 5 - AUG
C. lysine transacetylase, and a dihydrolipoyl dehydrogenase 2014 MED
D. valine closely resembling pyruvate dehydrogenase. Harper BOARDS;
Biochemistry 28th edition pp.257-258 TOPNOTCH
MD)
334 Of the following steps in the processing of the fibrillar Harper Biochemistry 28th edition p.529 table 48-3 DEBBIE BACK-UP
collagen precursor, which one takes place ROSE MIDTERM
extracellularly? TANENGSY EXAM - FEB
A. cleavage of the signal peptide , MD (TOP 2015
B. formation of the triple helix 5 - AUG
C. quarter staggered alignment assembly 2014 MED
D. formation of intrachain and interchain disulfide bonds BOARDS;
in extension peptides TOPNOTCH
MD)
335 What is true about cardiac muscle but not of smooth Both cardiac and smooth muscle are syncytial. Nerve DEBBIE BACK-UP
muscle? impulse initiates contraction in skeletal muscle. ROSE MIDTERM
A. syncytial Troponin system is present in skeletal and cardiac TANENGSY EXAM - FEB
B. nerve impulse initiates contraction muscle. Caldesmon is important in smooth muscle , MD (TOP 2015
C. troponin system present only. Harper Biochemistry 28th edition p.555 table 5 - AUG
D. caldesmon an important regulatory protein 49-3 2014 MED
BOARDS;
TOPNOTCH
MD)
336 True regarding skeletal muscle, except: Harper Biochemistry 28th edition p.562 table 49-12 DEBBIE BACK-UP
A. Skeletal muscle can utilize ketone bodies during ROSE MIDTERM
starvation. TANENGSY EXAM - FEB
B. Skeletal muscle can contribute to blood glucose. , MD (TOP 2015
C. Its proteolysis during starvation supplies amino acids 5 - AUG
for gluconeogenesis. 2014 MED
D. Glucagon is unable to stimulate glycogenolysis in BOARDS;
skeletal muscle. TOPNOTCH
MD)
337 The following clotting factors are properly paired with There is no factor VI. Proconvertin / cothromboplastin DEBBIE BACK-UP
their common name, except: / serum prothrombin convertion accelerator (SPCA) is ROSE MIDTERM
A. III - tissue factor factor VII. Harper Biochemistry 28th edition p. 585 TANENGSY EXAM - FEB
B. IV - calcium table 51-1 , MD (TOP 2015
C. V - proaccelerin 5 - AUG
D. VI - proconvertin 2014 MED
BOARDS;
TOPNOTCH
MD)
338 RBCs make use of this glucose transporter: Harper Biochemistry 28th edition p. 596 DEBBIE BACK-UP
A. GLUT 1 ROSE MIDTERM
B. GLUT 2 TANENGSY EXAM - FEB
C. GLUT 3 , MD (TOP 2015
D. GLUT 4 5 - AUG
2014 MED
BOARDS;
TOPNOTCH
MD)
339 Not true regarding anti-oxidants and free radicals: Beta carotene is an autocatalytic pro-oxidant at higher DEBBIE BACK-UP
A. Anti-oxidants can also be pro-oxidants. concentrations of oxygen and is only an antioxidant at ROSE MIDTERM
B. Radical damage is a known cause of atherosclerosis. low oxygen concentrations. Harper Biochemistry 28th TANENGSY EXAM - FEB
C. Oxygen radicals are the most damaging radicals in edition pp. 482-485 , MD (TOP 2015
biological systems. 5 - AUG
D. Beta carotene is an antioxidant at high concentrations 2014 MED
of oxygen. BOARDS;
TOPNOTCH
MD)
340 With regards to isomerism, glucose and galactose are: Glucose and galactose are epimers, differing only at DEBBIE BACK-UP
A. epimers, differing only at carbon 2 carbon 4. Choice A refers to glucose and mannose. ROSE MIDTERM
B. isomers TANENGSY EXAM - FEB
C. enantiomers , MD (TOP 2015
D. anomers 5 - AUG
2014 MED
BOARDS;
TOPNOTCH
MD)
341 True of the peptide bond, except: Characteristics of the peptide bond: JESSICA BACK-UP
A. Formed by a reaction of the α-COOH and α-NH2 of two - attaches the α-COOH to the α-NH2 of another MAE MIDTERM
amino acids - very stable, can only be disrupted by hydrolysis SANCHEZ, EXAM - FEB
B. Has a partial double bond character through prolonged exposure to a strong acid or base MD (TOP 4 2015
C.Generally in the cis configuration at elevated temperatures - AUG 2014
D. Rigid and planar - polar and can form hydrogen bonds MED
- partial double bond character, which makes the bond BOARDS;
rigid and planar TOPNOTCH
- generally in the trans configuration MD)

Reference: Topnotch Biochem Handouts

TOPNOTCH MEDICAL BOARD PREP BIOCHEMISTRY SUPEREXAM Page 43 of 92


For inquiries visit www.topnotchboardprep.com.ph or email us at topnotchmedicalboardprep@gmail.com
TOPNOTCH MEDICAL BOARD PREP BIOCHEMISTRY SUPEREXAM
For inquiries visit www.topnotchboardprep.com.ph or email us at topnotchmedicalboardprep@gmail.com
Item QUESTION EXPLANATION AUTHOR TOPNOTCH
# EXAM
342 Which of the following is NOT a tyrosine derived Tyrosine is a precursor of L-dopa, dopamine (also JESSICA BACK-UP
hormone? known as Prolactin Inhibiting Hormone or Prolactin MAE MIDTERM
A. Vasopressin Inhibiting Factor), norepinephrine, epinephrine, SANCHEZ, EXAM - FEB
B. Prolactin inhibiting hormone thyroxine and melanin. MD (TOP 4 2015
C. Thyroxine - AUG 2014
D. Epinephrine Vasopressin, or anti-diuretic hormone, is a peptide MED
hormone composed of 9 amino acids, and is not BOARDS;
directly derived from tyrosine. TOPNOTCH
MD)
343 Enzymes as biologic catalysts accelerate biochemical Enzymes act by lowering the free energy of activation, JESSICA BACK-UP
reactions by: but do not change the energy of the reactants and MAE MIDTERM
A. Lowering the net free energy change (ΔG0) of the products, nor the equilibrium of the reaction. SANCHEZ, EXAM - FEB
reaction MD (TOP 4 2015
B. Lowering the free energy of activation Reference: Topnotch Biochem Handouts - AUG 2014
C. Altering the thermodynamic properties of the reaction MED
D. Decreasing the Km of the enzyme for its substrate BOARDS;
TOPNOTCH
MD)
344 All of the following complexes in the Electron Transport Protons are pumped to the intermembranous space to JESSICA BACK-UP
Chain are sites of ATP synthesis, except: create a gradient (therefore, produce ATP) in 3 MAE MIDTERM
A. NADH dehydrogenase complexes: SANCHEZ, EXAM - FEB
B. Cytochrome b-c1 complex - Complex I or NADH dehydrogenase MD (TOP 4 2015
C. Succinate-CoQ dehydrogenase - Complex III or Cytochrome b/c1 complex - AUG 2014
D. Cytochrome aa3 complex - Complex IV or Cytochrome a/a3 or cytochrome MED
oxidase BOARDS;
TOPNOTCH
Complex II (Succinate dehydrogenase) does not MD)
produce ATP.
345 Which of the following compounds inhibit biological Uncouplers act by dissociating respiration from ATP JESSICA BACK-UP
oxidation by dissociating respiration from ATP synthesis. MAE MIDTERM
synthesis? Examples of uncouplers are 2,4 dinitrophenol, aspirin, SANCHEZ, EXAM - FEB
A. Antimycin A and thermogenin (brown fat). MD (TOP 4 2015
B. 2,4 dinitrophenol - AUG 2014
C. Carbon monoxide Antimycin A - inhibits Complex III MED
D. Malonate Carbon monoxide - inhibits Complex IV BOARDS;
Malonate - inhibits Complex II TOPNOTCH
MD)

346 Which of the following is true of the amino acids valine, Valine, leucine and isoleucine are branched chain JESSICA BACK-UP
leucine, and isoleucine? amino acids, with nonpolar, uncharged side chains; MAE MIDTERM
A. Characterized with a polar but uncharged side chain they are non ionizable and are considered SANCHEZ, EXAM - FEB
B. Classified under branched chain amino acids nutritionally essential. MD (TOP 4 2015
C. All are nutritionally non essential - AUG 2014
D. Possess ionizable functional groups MED
BOARDS;
TOPNOTCH
MD)
347 Hemoglobin and proteins composed of several subunits Primary, secondary, and tertiary levels of protein JESSICA BACK-UP
assume this level of protein structure: structure are composed of only one polypeptide chain. MAE MIDTERM
A. Primary SANCHEZ, EXAM - FEB
B. Secondary MD (TOP 4 2015
C. Tertiary - AUG 2014
D. Quaternary MED
BOARDS;
TOPNOTCH
MD)
348 Which of the following is true of Km? Km, or the Michaelis constant, is the substrate JESSICA BACK-UP
A. Corresponds to the substrate concentration when concentration at which the initial velocity is half the MAE MIDTERM
initial velocity is Vmax/2 maximal velocity (Vmax/2). SANCHEZ, EXAM - FEB
B. Its value increases when the affinity of the substrate to Km increases when the affinity to the substrate is low, MD (TOP 4 2015
the enzyme increases. and decreases when the affinity is high. - AUG 2014
C. Corresponds to the reciprocal value of the y-axis in the In the Lineweaver Burk plot, the Km is the reciprocal MED
Lineweaver Burk plot. value of the x-axis. BOARDS;
D. is equal to the value of Vmax when the enzyme is fully Km is always half of Vmax, and is never equal to its TOPNOTCH
saturated with the substrate value. MD)
349 This protein digestive enzyme has the capacity to Trypsin is capable of activating trypsinogen (its JESSICA BACK-UP
activate its own zymogen: zymogen) into more trypsin, which in turn activates MAE MIDTERM
A. Pepsinogen other protein digestive enzymes. SANCHEZ, EXAM - FEB
B. Elastase MD (TOP 4 2015
C. Trypsin - AUG 2014
D. Chymotrypsin MED
BOARDS;
TOPNOTCH
MD)
350 Classic galactosemia can cause cataract formation due to Galactitol, once formed from galactose by aldose JESSICA BACK-UP
the following, except: reductase, accumulates within the lens and causes MAE MIDTERM
A. UDP-Glc:galactose-1PO4 uridyltransferase deficiency early cataract formation found in classic galactosemia. SANCHEZ, EXAM - FEB
B. Accumulation of galactitol inside the lens cells MD (TOP 4 2015
C. Action of aldose reductase forming galactitol - AUG 2014
D. Ability of galactitol to diffuse out of the lens cell MED
BOARDS;
TOPNOTCH
MD)

TOPNOTCH MEDICAL BOARD PREP BIOCHEMISTRY SUPEREXAM Page 44 of 92


For inquiries visit www.topnotchboardprep.com.ph or email us at topnotchmedicalboardprep@gmail.com
TOPNOTCH MEDICAL BOARD PREP BIOCHEMISTRY SUPEREXAM
For inquiries visit www.topnotchboardprep.com.ph or email us at topnotchmedicalboardprep@gmail.com
Item QUESTION EXPLANATION AUTHOR TOPNOTCH
# EXAM
351 Carbamoyl phosphate synthase I differs from Carbamoyl All the other choices except D describe CPS II, the rate JESSICA BACK-UP
phosphate synthase II in that the former: limiting enzyme in de novo pyrimidine synthesis. MAE MIDTERM
A. Is involved in pyrimidine de novo synthesis SANCHEZ, EXAM - FEB
B. Is stimulated by PRPP MD (TOP 4 2015
C. Is found in the cytosol - AUG 2014
D. Is a mitochondrial urea cycle enzyme MED
BOARDS;
TOPNOTCH
MD)
352 Which apolipoprotein is vital in the clearance of TAGs Apo C-II activates lipoprotein lipase for the clearance JESSICA BACK-UP
from chylomicrons and VLDL by lipoprotein lipase? of triacylglycerols from chylomicrons and VLDS. MAE MIDTERM
A. Apo E Apo E - mediates uptake of the chylomicron remnant SANCHEZ, EXAM - FEB
B. Apo C-II Apo A-1 - found in HDL; activates LCT/PCAT to MD (TOP 4 2015
C. Apo A-1 produce cholesterol esters - AUG 2014
D. Apo B-48 Apo B-48 - functions in chylomicron assembly and MED
secretion BOARDS;
TOPNOTCH
Reference: Topnotch Biochem Handouts MD)
353 A 30 year old patient was seen by his physician because Familial Hypercholesterolemia, which manifests as JESSICA BACK-UP
of hypertension. Aside from the high blood pressure, lab high LDL levels in laboratory determinations, is due to MAE MIDTERM
tests revealed elevated blood cholesterol but normal a deficiency in the LDL receptor. This confers an SANCHEZ, EXAM - FEB
triglycerides. He was diagnosed as a case of Familial increased risk of atherosclerosis and coronary heart MD (TOP 4 2015
Hypercholesterolemia. The elevated cholesterol in this disease. - AUG 2014
condition is mainly due to: MED
A. Absence of feedback inhibition on the committed step BOARDS;
of cholesterol synthesis TOPNOTCH
B. Defect in LDL receptor and receptor-mediated MD)
endocytosis of LDL cholesterol
C. Decreased lipoprotein lipase activity
D. Absence of apoprotein B-48, C-II and E
354 Which condition promotes a rightward shift of the An increase in CO2, hydrogen ions (acidosis), 2,3-BPG, JESSICA BACK-UP
hemoglobin saturation curve? temperature, and exercise, promotes a shift to the MAE MIDTERM
A. Decrease in intracellular pH of 7.2 right of oxygen-hemoglobin dissociation curve. SANCHEZ, EXAM - FEB
B. Core body temperature of 32°C MD (TOP 4 2015
C. pCO2 of 25 - AUG 2014
D. Decreased 2,3 BPG concentration MED
BOARDS;
TOPNOTCH
MD)
355 The following biochemical events take place during High CO2 within the muscles during exercise should JESSICA BACK-UP
exercise, except: promote diffusion of CO2 from the tissues to the RBC. MAE MIDTERM
A. High CO2 and low O2 concentration in the muscle SANCHEZ, EXAM - FEB
inhibit diffusion of CO2 from the tissues to the RBC MD (TOP 4 2015
B. Increased [H+] from lactic acid formation will promote - AUG 2014
O2 release by oxyhemoglobin MED
C. Increase in CO2 will favor deoxyhemoglobin BOARDS;
formation, promoting O2 unloading to the tissues TOPNOTCH
D. Increased heat generated in the muscles shifts the MD)
oxyhemoglobin binding curve to the right resulting to O2
unloading at the tissue level.
356 The structural feature of myoglobin that distinguishes it All the other choices except B describe hemoglobin. JESSICA BACK-UP
from hemoglobin: MAE MIDTERM
A. Quaternary protein composed of alpha and beta SANCHEZ, EXAM - FEB
globin chains stabilized by non-covalent interactions MD (TOP 4 2015
B. A single polypeptide chains folded to form a globular - AUG 2014
protein MED
C. A central cavity occupied by 2,3 bisphosphoglycerate BOARDS;
D. Salt bridges on the 4 terminal carboxyl groups of the TOPNOTCH
tetramer MD)
357 Which type of mutation causes premature termination of JESSICA BACK-UP
protein synthesis because of the creation of a stop codon MAE MIDTERM
during the genetic change? SANCHEZ, EXAM - FEB
A. Missense MD (TOP 4 2015
B. Neutral - AUG 2014
C. Nonsense MED
D. Silent BOARDS;
TOPNOTCH
MD)

358 Muscle relaxation that takes place after actin-myosin JESSICA BACK-UP
dissociation is due to: MAE MIDTERM
A. Binding of calcium to troponin C SANCHEZ, EXAM - FEB
B. Depletion of ATP in the muscle MD (TOP 4 2015
C. Increased formation of creatine phosphate - AUG 2014
D. Binding of ATP to the S-1 head of myosin to form a MED
myosin-ATP complex BOARDS;
TOPNOTCH
MD)

TOPNOTCH MEDICAL BOARD PREP BIOCHEMISTRY SUPEREXAM Page 45 of 92


For inquiries visit www.topnotchboardprep.com.ph or email us at topnotchmedicalboardprep@gmail.com
TOPNOTCH MEDICAL BOARD PREP BIOCHEMISTRY SUPEREXAM
For inquiries visit www.topnotchboardprep.com.ph or email us at topnotchmedicalboardprep@gmail.com
Item QUESTION EXPLANATION AUTHOR TOPNOTCH
# EXAM
359 Because of the amphipathic nature of bile acids, their Bile acids are not involved in the packaging of TAGs JESSICA BACK-UP
role in lipid digestion includes the following, except: into chylomicrons, which takes place inside the MAE MIDTERM
A. Serve as detergents binding to fat globules as they are intestinal epithelial cell. SANCHEZ, EXAM - FEB
broken down by peristaltic action of the GI tract MD (TOP 4 2015
B. Suspends dietary fat in smaller particles in the - AUG 2014
aqueous environment in the small intestines MED
C. involved in the packaging of TAGs into chylomicrons BOARDS;
D. solubilize the TAGs into micelles and transport them TOPNOTCH
to the intestinal cell surface MD)
360 This component of total energy expenditure plays the 60% of energy is used to maintain Resting or Basal JESSICA BACK-UP
most significant role in the total energy requirement of metabolic rate. MAE MIDTERM
an individual: 30% for physical activity, and 10% for diet-induced SANCHEZ, EXAM - FEB
A. Physical activity thermogenesis or thermic effect of food. MD (TOP 4 2015
B. Thermic effect of food - AUG 2014
C. Diet induced thermogenesis Reference: Topnotch Biochem Handouts MED
D. Basal energy expenditure BOARDS;
TOPNOTCH
MD)
361 Reduced glutathione (GSH) is composed of which of the Reduced glutathione is a tripeptide-thiol (γ- MAIRRE BACK-UP
following amino acids EXCEPT? glutamylcysteinylglycine) present in most cells which JAMES MIDTERM
A. Glycine can chemically detoxify hydrogen peroxide Lippincott GADDI, MD EXAM - FEB
B. Cysteine 5th pg 148 (TOP 4 - 2015
C. Glutamate AUG 2013
D. Serine MED
E. None of the above BOARDS;
TOPNOTCH
MD)
362 The following are TRUE of the urea cycle EXCEPT CPS I absolutely requires N-acetyl glutamate as an MAIRRE BACK-UP
A. The rate limiting step of the cycle is catalyzed by allosteric activator. In the urea cycle, 3 molecules of JAMES MIDTERM
carbamoyl phosphate synthetase I ATP are required but ultimately 4 ATP equivalents are GADDI, MD EXAM - FEB
B. CPS I absolutely requires N-acetyl glutamate consumed, (2 ATP for CPS I and 1 ATP for (TOP 4 - 2015
C. There are two ATP requiring enzymes and one of them argininosuccinate synthetase [converted to AMP AUG 2013
is argininosuccinate synthetase hence 2 equivalents used]). The cycle can also be used MED
D. It can be used to synthesize citrulline, ornithine and to synthesize ornithine, citrulline, arginine, fumarate BOARDS;
aspartate and urea. Lippincott 5th pg 254 TOPNOTCH
E. Carbon dioxide and free ammonia provides for the MD)
atoms of urea
363 HMG-CoA is involved in the synthesis of: The first two reactions in the cholesterol synthetic MAIRRE BACK-UP
A. Cholesterol pathway are similar to those in the pathway that JAMES MIDTERM
B. Ketone bodies produces ketone bodies. They result in the production GADDI, MD EXAM - FEB
C. Palmitate of HMG CoA through HMG CoA synthase. HMG CoA (TOP 4 - 2015
D. A and B synthase is the limiting enzyme of ketone body AUG 2013
E. All of the above synthesis while HMG CoA reductase is the limiting MED
enzyme of cholesterol synthesis. Lippincott 5th pg 220 BOARDS;
TOPNOTCH
MD)
364 The following are TRUE regarding prokaryotic RNA polymerase does not need a primer and has no MAIRRE BACK-UP
transcription EXCEPT proofreading activity in contrast to DNA polymerase. JAMES MIDTERM
A. Transcription starts at the promoter which contain Lippincott 5th 419-421 GADDI, MD EXAM - FEB
consensus sequences such as the -35 sequence and the (TOP 4 - 2015
TATAAT box AUG 2013
B. Termination of transcription can be spontaneous MED
through formation of a hairpin loop BOARDS;
C. RNA polymerase needs a primer to initiate TOPNOTCH
transcription MD)
D. Transcription proceeds from a 5’ to 3’ direction and is
identical to the sense strand with U replacing T
E. All are correct
365 A 14/F recently broke up with her boyfriend. Due to this Silver jewelry cleaner usually contains cyanide and MAIRRE BACK-UP
she ingested silver jewelry cleaner causing her to lose this causes inhibition of Cytochrome c oxidase JAMES MIDTERM
consciousness after a few minutes. She was immediately (Complex IV). Other substances which inhibit complex GADDI, MD EXAM - FEB
taken to the ER. On PE, she was still unconscious and had IV include sodium azide, hydrogen sulfide and carbon (TOP 4 - 2015
cherry red cold clammy skin. The patient most likely had monoxide. NADH dehydrogenase (Complex I), AUG 2013
inhibition of: Succinate dehydrogenase (Complex II), Cytochrome MED
A. NADH dehydrogenase b/c1 (Complex III). Lippincott 5th pg 75 BOARDS;
B. Succinate dehydrogenase TOPNOTCH
C. Cytochrome b/c1 MD)
D. Cytochrome c oxidase
E. Coenzyme Q
366 True of lipid metabolism EXCEPT Short and medium chain length fatty acids do not MAIRRE BACK-UP
A. Short and medium chain fatty acids are absorbed into require the assistance of mixed micelles for JAMES MIDTERM
the enterocyte through micelles absorption by the intestinal mucosa. GADDI, MD EXAM - FEB
B. Digestion of lipids begins in the stomach through acid Lippincott 5th pgs 173-177 (TOP 4 - 2015
stable lingual lipase and gastric lipase AUG 2013
C. Short and medium chain fatty acids are released MED
directly into the portal circulation bypassing the lacteals BOARDS;
D. Pancreatic lipase preferentially cleaves the fatty acids TOPNOTCH
at carbons 1 and 3 forming 2-monoacylglycerol plus free MD)
fatty acids

TOPNOTCH MEDICAL BOARD PREP BIOCHEMISTRY SUPEREXAM Page 46 of 92


For inquiries visit www.topnotchboardprep.com.ph or email us at topnotchmedicalboardprep@gmail.com
TOPNOTCH MEDICAL BOARD PREP BIOCHEMISTRY SUPEREXAM
For inquiries visit www.topnotchboardprep.com.ph or email us at topnotchmedicalboardprep@gmail.com
Item QUESTION EXPLANATION AUTHOR TOPNOTCH
# EXAM
367 Simvastatin is a reversible competitive inhibitor of the Competitive inhibition would cause an apparent MAIRRE BACK-UP
rate-limiting step in cholesterol biosynthesis. What increase in Km causing -1/Km to move closer to zero. JAMES MIDTERM
change would you expect for Lineweaver-Burke plot? Choice D would be expected for non-competitive GADDI, MD EXAM - FEB
A. The x-intercept would be closer to the zero due to an inhibition. Lippincott 5th pgs 60-61 (TOP 4 - 2015
apparent increase in Km AUG 2013
B. The x-intercept would be farther from zero due to an MED
apparent increase in Km BOARDS;
C. The y-intercept would be closer to zero due to an TOPNOTCH
apparent decrease in Vmax MD)
D. The y-intercept would be farther from zero due to an
apparent decrease in Vmax
368 The electron transport chain is embedded in the inner Mobile electron carriers: coenzyme Q and cytochrome MAIRRE BACK-UP
mitochondrial membrane. Which of the following is/are c. Lippincott 5th pg 74 JAMES MIDTERM
the mobile electron carrier/s? GADDI, MD EXAM - FEB
A. NADH dehydrogenase (TOP 4 - 2015
B. Coenzyme Q AUG 2013
C. Cytochrome b/c1 MED
D. Cytochrome c oxidase BOARDS;
E. B and D TOPNOTCH
MD)
369 The following are TRUE regarding the HMP shunt Phase 1 reactions produce NADPH and ribulose 5-P; MAIRRE BACK-UP
EXCEPT? Phase 2 reactions produce ribose 5-P not ribulose 5-P JAMES MIDTERM
A. Phase 1 reactions are irreversible Lippincott 5th pg 146 GADDI, MD EXAM - FEB
B. The main products of phase 2 reactions are ribulose 5- (TOP 4 - 2015
P, glyceraldehyde 3-P and fructose 6-P AUG 2013
C. The only product of phase 1 reactions is NADPH MED
D. B and C BOARDS;
E. No exception TOPNOTCH
MD)
370 Which essential amino acid is both ketogenic and Leucine and lysine are ketogenic and essential; MAIRRE BACK-UP
glucogenic? Threonine glucogenic and essential; Tyrosine both JAMES MIDTERM
A. Threonine glucogenic and ketogenic but nonessential Lippincott GADDI, MD EXAM - FEB
B. Tyrosine 5th pg 262 (TOP 4 - 2015
C. Tryptophan AUG 2013
D. Leucine MED
E. Lysine BOARDS;
TOPNOTCH
MD)
371 Which of the following is/are TRUE regarding RNA? introns are removed while exons are joined together MAIRRE BACK-UP
A. Postranslational modification of mRNA includes the Lippincott 5th pg 422 JAMES MIDTERM
addition of a polyadenylate tail and a methylguanosine GADDI, MD EXAM - FEB
cap (TOP 4 - 2015
B. snRPs are responsible for the removal of exons and AUG 2013
the joining of introns MED
C. RNA polymerase II synthesizes mRNA and miRNA BOARDS;
D. A and C TOPNOTCH
E. All of the above MD)
372 What percentage of total caloric intake of fat has been AMDR for adults is 45–65% of their total calories from MAIRRE BACK-UP
associated with reduced risk of chronic disease while carbohydrates, 20–35% from fat, and 10–35% from JAMES MIDTERM
still providing adequate amounts of the nutrient? protein Lippincott 5th pg 360 GADDI, MD EXAM - FEB
A. 45-65% (TOP 4 - 2015
B. 10-20% AUG 2013
C. 10-35% MED
D. 20-35% BOARDS;
E. 5-10% TOPNOTCH
MD)
373 Cephalin is made up of: Cephalin/phosphatidylethanolamine; A - cardiolipin; MAIRRE BACK-UP
A. two phosphatidic acid residues and a glycerol C - phosphatidylglycerol; D - JAMES MIDTERM
backbone lecithin/phosphatidylcholine Lippincott 5th pg 202 GADDI, MD EXAM - FEB
B. phosphatidic acid and ethanolamine (TOP 4 - 2015
C. phosphatidic acid and glycerol AUG 2013
D. phosphatidic acid and choline MED
E. None of the above BOARDS;
TOPNOTCH
MD)
374 Which of the following is/are TRUE of Vitamin A? Rhodopsin consists of 11-cis retinal and opsin; MAIRRE BACK-UP
A. Rhodopsin consists of 11-trans retinal and opsin Pseudotumor cerebri is caused by excessive intake of JAMES MIDTERM
B. Vitamin A deficiency can produce nightblindness, vitamin A Lippincott 5th pg 384-386 GADDI, MD EXAM - FEB
xerophthalmia and pseudotumor cerebri (TOP 4 - 2015
C. Retinol and retinal can readily be interconverted and AUG 2013
can be derived from retinoic acid MED
D. A and C BOARDS;
E. All of the above TOPNOTCH
MD)

375 NADPH required for fatty acid synthesis comes from the: Hexose monophosphate pathway is the major supplier MAIRRE BACK-UP
A. Hexose monophosphate pathway of NADPH for fatty acid synthesis but cytosolic malic JAMES MIDTERM
B. Cytosolic malic enzyme enzyme or NADP+ dependent malate dehydrogenase GADDI, MD EXAM - FEB
C. Succinyl dehydrogenase also produces cytosolic NADPH Lippincott 5th pg 186 (TOP 4 - 2015
D. A and B AUG 2013
E. All of the above MED
BOARDS;
TOPNOTCH
MD)

TOPNOTCH MEDICAL BOARD PREP BIOCHEMISTRY SUPEREXAM Page 47 of 92


For inquiries visit www.topnotchboardprep.com.ph or email us at topnotchmedicalboardprep@gmail.com
TOPNOTCH MEDICAL BOARD PREP BIOCHEMISTRY SUPEREXAM
For inquiries visit www.topnotchboardprep.com.ph or email us at topnotchmedicalboardprep@gmail.com
Item QUESTION EXPLANATION AUTHOR TOPNOTCH
# EXAM
376 Triacylglycerols comprise the majority of which CM 90%; VLDL 60%; LDL 8%; HDL 5% MAIRRE BACK-UP
lipoprotein? triacylglycerols Lippincott 5th pg 232 JAMES MIDTERM
A. Chylomicrons GADDI, MD EXAM - FEB
B. VLDL (TOP 4 - 2015
C. IDL AUG 2013
D. LDL MED
E. HDL BOARDS;
TOPNOTCH
MD)
377 Which of the following is/are the needed substrate/s in Sources of the atoms in the purine ring include MAIRRE BACK-UP
de novo purine synthesis? aspartate, carbon dioxide, glutamine, glycine and N10- JAMES MIDTERM
A. Aspartate THF. Pyrimidine ring: aspartate, carbon dioxide, GADDI, MD EXAM - FEB
B. Carbon dioxide glutamine Lippincott 5th pg 293 and 302 (TOP 4 - 2015
C. Glutamate AUG 2013
D. A and B MED
E. All of the above BOARDS;
TOPNOTCH
MD)
378 Which of the following is/are INCORRECTLY matched? Southwestern blot for DNA-binding proteins MAIRRE BACK-UP
A. Northern blot: RNA JAMES MIDTERM
B. Southern blot: DNA-binding proteins GADDI, MD EXAM - FEB
C. Western blot: protein (TOP 4 - 2015
D. A and B AUG 2013
E. None of the above MED
BOARDS;
TOPNOTCH
MD)
379 Which of the following amino acids is/are non-polar? glycine, alanine, valine, leucine, isoleucine, MAIRRE BACK-UP
A. Phenylalanine phenylalanine, tryptophan, methionine, proline: non JAMES MIDTERM
B. Proline polar Lippincott 5th pg 2 GADDI, MD EXAM - FEB
C. Tryptophan (TOP 4 - 2015
D. A and B AUG 2013
E. All of the above MED
BOARDS;
TOPNOTCH
MD)
380 Which of the following is/are TRUE regarding deoxy/taut (T) form - low affinity; oxy/relaxed (R) MAIRRE BACK-UP
hemoglobin? form - high affinity for oxygen Lippincott 5th pg 28 JAMES MIDTERM
A. One gram of hemoglobin is capable of binding 1.34 mL GADDI, MD EXAM - FEB
of O2 (TOP 4 - 2015
B. Binding of an oxygen molecule to one heme group AUG 2013
increases the oxygen affinity of the other groups MED
C. The deoxy form of hemoglobin is constrained and has BOARDS;
low affinity for oxygen TOPNOTCH
D. A and B MD)
E. All of the above
381 Which of the following are non-polar amino acids? SIMILAR TO PREVIOUS BOARD EXAM SCOTT BACK-UP
A. Leucine CONCEPT/PRINCIPLE. Non-polar amino acids: Gly, RILEY ONG, MIDTERM
B. Phenylalanine Ala, Phe, Trp, Met, Pro. Uncharged, polar amino acids: MD (TOP 5 EXAM - FEB
C. Methionine Cys, Tyr, Ser, Thr, Asn, Gln. Basic amino acids: His, Arg, - AUG 2014 2015
D. All of the above Lys. Acidic amino acids: Glu, Asp. MED
E. None of the above BOARDS;
TOPNOTCH
MD)
382 Which of the following enzymes participate in both The other choices participate in non-reversible steps SCOTT BACK-UP
glycolysis and gluconeogenesis? of glycolysis only. RILEY ONG, MIDTERM
A. Phosphofructokinase-1 MD (TOP 5 EXAM - FEB
B. Hexokinase - AUG 2014 2015
C. Phosphoglycerate kinase MED
D. Pyruvate kinase BOARDS;
E. Glucokinase TOPNOTCH
MD)

383 Which of the following substances is a known inhibitor Complex I inhibitors: barbiturate, piericidin A, amytal, SCOTT BACK-UP
of complex III of the electron transport chain? rotenone. Complex II inhibitors: malonate, carboxin, RILEY ONG, MIDTERM
A. Amytal TTFA. Complex III inhibitors: antimycin A, MD (TOP 5 EXAM - FEB
B. Carboxin dimercaprol. Complex IV inhibitors: cyanide, carbon - AUG 2014 2015
C. Malonate monoxide, sodium azide, hydrogen sulfide. MED
D. Actinomycin A BOARDS;
E. Sodium azide TOPNOTCH
MD)

384 What is the rate-limiting enzyme in glycolysis? SCOTT BACK-UP


A. Hexokinase RILEY ONG, MIDTERM
B. Phosphofructokinase MD (TOP 5 EXAM - FEB
C. Phosphoglycerate kinase - AUG 2014 2015
D. Pyruvate kinase MED
E. Both A and B BOARDS;
TOPNOTCH
MD)

TOPNOTCH MEDICAL BOARD PREP BIOCHEMISTRY SUPEREXAM Page 48 of 92


For inquiries visit www.topnotchboardprep.com.ph or email us at topnotchmedicalboardprep@gmail.com
TOPNOTCH MEDICAL BOARD PREP BIOCHEMISTRY SUPEREXAM
For inquiries visit www.topnotchboardprep.com.ph or email us at topnotchmedicalboardprep@gmail.com
Item QUESTION EXPLANATION AUTHOR TOPNOTCH
# EXAM
385 Which of the following is a cofactor of xanthine oxidase? SCOTT BACK-UP
A. Molybdenum RILEY ONG, MIDTERM
B. Pyridoxine MD (TOP 5 EXAM - FEB
C. Selenium - AUG 2014 2015
D. Copper MED
E. Vitamin C BOARDS;
TOPNOTCH
MD)
386 Which of the following vitamins is required for proper SCOTT BACK-UP
hydroxylation of amino acid residues in collagen? RILEY ONG, MIDTERM
A. Vitamin A MD (TOP 5 EXAM - FEB
B. Vitamin B6 - AUG 2014 2015
C. Vitamin C MED
D. Vitamin D BOARDS;
E. Vitamin E TOPNOTCH
MD)

387 Apo-CII can be found in the following lipoproteins Apo-CII is an activator of lipoprotein lipase and is SCOTT BACK-UP
except: naturally found in chylomicrons and VLDL. HDL is RILEY ONG, MIDTERM
A. Chylomicron used to shuttle Apo-CII. MD (TOP 5 EXAM - FEB
B. VLDL - AUG 2014 2015
C. LDL MED
D. HDL BOARDS;
E. None of the above TOPNOTCH
MD)
388 In diabetes mellitus, excess glucose is converted to Sorbitol is metabolized by sorbital dehydrogenase SCOTT BACK-UP
sorbitol, causing some of its complications including into fructose. Remember that fructose is one of the RILEY ONG, MIDTERM
cataract and neuropathy. Sorbital accumulates in the secretions of the seminal vesicles for the seminal fluid. MD (TOP 5 EXAM - FEB
body because its metabolism is very slow, occurring only - AUG 2014 2015
in the liver and which other body organ? MED
A. Seminal vesicle BOARDS;
B. Brain TOPNOTCH
C. Pancreas MD)
D. Choroid body
E. Kidney

389 Which of the following enzymes can be seen in cancer SIMILAR TO PREVIOUS BOARD EXAM SCOTT BACK-UP
cells and is partly responsible for their unlimited CONCEPT/PRINCIPLE. RILEY ONG, MIDTERM
replicative potential by preventing the shortening of the MD (TOP 5 EXAM - FEB
highly repetitive DNA sequences at the ends of their - AUG 2014 2015
chromosomes? MED
A. DNA polymerase III BOARDS;
B. Telomerase TOPNOTCH
C. Primase MD)
D. DNA A protein
E. Replicase

390 Which of the following fatty acids is essential and is Linolenic acid = omega-3 fatty acid SCOTT BACK-UP
needed for proper development of vision and learning RILEY ONG, MIDTERM
behavior? MD (TOP 5 EXAM - FEB
A. Linolenic acid - AUG 2014 2015
B. Linoleic acid MED
C. Oleic acid BOARDS;
D. Arachidonic acid TOPNOTCH
E. Stearic acid MD)

391 Which of the following is the precursor of the active form Cholecalciferol, also known as 7-dehydrocholesterol, SCOTT BACK-UP
of vitamin D that is found naturally occurring in the skin? is the naturally occurring form of vitamin D in animals RILEY ONG, MIDTERM
A. Cholecalciferol and human skin. It serves as the precursor for MD (TOP 5 EXAM - FEB
B. Ergocalciferol calcitriol, the active form of vitamin D. Ergocalciferol - AUG 2014 2015
C. Sitocalciferol is the form of vitamin D obtained from vegetables. MED
D. Calcitriol BOARDS;
E. Dihydroergocalciferol TOPNOTCH
MD)

392 How much oxygen does 1 g of hemoglobin carry? SIMILAR TO PREVIOUS BOARD EXAM SCOTT BACK-UP
A. 1.00 mL CONCEPT/PRINCIPLE. RILEY ONG, MIDTERM
B. 1.25 mL MD (TOP 5 EXAM - FEB
C. 1.34 mL - AUG 2014 2015
D. 1.55 mL MED
E. 1.67 mL BOARDS;
TOPNOTCH
MD)
393 Which of the following amino acids is the precursor of SIMILAR TO PREVIOUS BOARD EXAM SCOTT BACK-UP
catecholamines such as dopamine, norepinephrine and CONCEPT/PRINCIPLE. RILEY ONG, MIDTERM
epinephrine? MD (TOP 5 EXAM - FEB
A. Asparagine - AUG 2014 2015
B. Glutamate MED
C. Methionine BOARDS;
D. Serine TOPNOTCH
E. Tyrosine MD)

TOPNOTCH MEDICAL BOARD PREP BIOCHEMISTRY SUPEREXAM Page 49 of 92


For inquiries visit www.topnotchboardprep.com.ph or email us at topnotchmedicalboardprep@gmail.com
TOPNOTCH MEDICAL BOARD PREP BIOCHEMISTRY SUPEREXAM
For inquiries visit www.topnotchboardprep.com.ph or email us at topnotchmedicalboardprep@gmail.com
Item QUESTION EXPLANATION AUTHOR TOPNOTCH
# EXAM
394 Carbamoyl phosphate synthetase I is the rate-limiting SCOTT BACK-UP
enzyme in the urea cycle. This enzyme is found in which RILEY ONG, MIDTERM
part of the cell? MD (TOP 5 EXAM - FEB
A. Cytosol - AUG 2014 2015
B. Mitochondria MED
C. Golgi body BOARDS;
D. Smooth ER TOPNOTCH
E. Peroxisome MD)

395 Fructose enters glycolysis as what intermediate Fructose is phosphorylated by fructosekinase into SCOTT BACK-UP
substance in this pathway? fructose-1-phosphate. Fructose-1-phosphate is then RILEY ONG, MIDTERM
A. Dihydroxyacetone phosphate cleaved by aldolase B to form dihydroxyacetone MD (TOP 5 EXAM - FEB
B. Fructose-6-phosphate phosphate (which can now enter glycolysis) and - AUG 2014 2015
C. Fructose-1,6-bisphosphate glyceraldehyde. Glyceraldehyde can be MED
D. Glucose phosphorylated by triokinase into glyceraldehyde-3- BOARDS;
E. 2-phosphoglycerate phosphate (which can also now enter glycolysis). TOPNOTCH
MD)
396 Which of the following substances would you expect to Homocysteine and N5-methyltetrahydrofolate SCOTT BACK-UP
accumulate when there is a deficiency of homocysteine accumulate when there is a deficiency of RILEY ONG, MIDTERM
methyltransferase, such as in cases of homocyteinuria? homocysteine methyltransferase. This enzyme is MD (TOP 5 EXAM - FEB
A. N5-methyltetrahydrofolate required to transfer a methyl group from N5- - AUG 2014 2015
B. Methionine methylTHF to homocysteine to form methionine and MED
C. N10-formyltetrahydrofolate tetrahydrofolate. BOARDS;
D. Cysteine TOPNOTCH
E. S-adenosylmethionine MD)

397 Which of the following amino acids is purely ketogenic? Purely ketogenic: Lys and Leu. Both ketogenic and SCOTT BACK-UP
A. Phenylalanine glucogenic: Phe, Tyr, Ile, Trp (mnemonic: F-Y-I-W). RILEY ONG, MIDTERM
B. Tryptophan MD (TOP 5 EXAM - FEB
C. Tyrosine - AUG 2014 2015
D. Isoleucine MED
E. Leucine BOARDS;
TOPNOTCH
MD)
398 Which of the following is not a common precursor in Precursors of purine: Asp, Gln, Gly, CO2, N10- SCOTT BACK-UP
both de novo purine and pyrimidine synthesis? formyltetrahydrofolate or N5,N10- RILEY ONG, MIDTERM
A. Aspartate methenyltetrahydrofolate. Precursors of pyrimidineL MD (TOP 5 EXAM - FEB
B. Glutamine Asp, Gln, CO2 - AUG 2014 2015
C. Glycine MED
D. Carbon dioxide BOARDS;
E. None of the above TOPNOTCH
MD)

399 Which of the following vitamins is most likely to be Intrinsic factor, which is required for vitamin B12 SCOTT BACK-UP
deficient in patients who underwent gastrectomy? absorption in the ileum, is produced by parietal cells RILEY ONG, MIDTERM
A. Vitamin C in the gastric fundus. MD (TOP 5 EXAM - FEB
B. Vitamin A - AUG 2014 2015
C. Vitamin B6 MED
D. Vitamin B12 BOARDS;
E. Vitamin E TOPNOTCH
MD)

400 Which of the following is a co-factor of glutathione SCOTT BACK-UP


peroxidase? RILEY ONG, MIDTERM
A. Molybdenum MD (TOP 5 EXAM - FEB
B. Pyridoxine - AUG 2014 2015
C. Selenium MED
D. Copper BOARDS;
E. Vitamin C TOPNOTCH
MD)
401 Which of the following substances inhibits the Electron Dimercaprol (and Antimycin A) inhibits the Ubiquinol: JOSE DIAGNOSTIC
Transport chain via the Ubiquinol: Ferricytochrome Ferricytochrome Oxidoreductase Complex simply CARLO EXAM - AUG
Oxidoreductase Complex? known as Complex III. Babiturate: Complex I; MASANGK 2014
A. Barbiturate Malonate: Complex II, Cyanide Complex IV; AY III, MD
B. Malonate Oligomycin: ATP synthase (Previous Board Ques) (TOP 8 -
C. Dimercaprol FEB 2014
D. Cyanide MED
E. Oligomycin BOARDS;
TOPNOTCH
MD)
402 Which of the following is true regarding Enzymes? JOSE DIAGNOSTIC
A. First-order kinetics is followed when the substrate CARLO EXAM - AUG
concentration is above Km MASANGK 2014
B. Temperature does not affect the reaction rate AY III, MD
C. The Km is directly proportional to the substrate (TOP 8 -
affinity FEB 2014
D. Vi is the substrate concentration at which Km is half MED
the maximal Velocity BOARDS;
E. Enzymes that follow the Michaelis-Menten kinetics TOPNOTCH
presents with a hyperbolic curve? MD)

TOPNOTCH MEDICAL BOARD PREP BIOCHEMISTRY SUPEREXAM Page 50 of 92


For inquiries visit www.topnotchboardprep.com.ph or email us at topnotchmedicalboardprep@gmail.com
TOPNOTCH MEDICAL BOARD PREP BIOCHEMISTRY SUPEREXAM
For inquiries visit www.topnotchboardprep.com.ph or email us at topnotchmedicalboardprep@gmail.com
Item QUESTION EXPLANATION AUTHOR TOPNOTCH
# EXAM
403 What Glucose transporter is present in Erythrocytes? GLUT1: Erythrocytes, brain, kidney, colon, placenta; JOSE DIAGNOSTIC
A. GLUT-1 GLUT2 Liver, pancreas, SI, kidney; GLUT-3 Brain, CARLO EXAM - AUG
B. GLUT-2 kidney, placenta; GLUT-4 Heart and skeletal muscle, MASANGK 2014
C. GLUT-3 adipose tissue; GLUT5: SI AY III, MD
D. GLUT-4 (TOP 8 -
E. GLUT-5 FEB 2014
MED
BOARDS;
TOPNOTCH
MD)
404 Utilization of Galactose by the body enters the Embden- Galactose is converted to Galactose-1-phosphate by JOSE DIAGNOSTIC
Meyerhof Pathway thru which Intermediate? galactokinase/hexokinase, Gal-1-Phosphate is then CARLO EXAM - AUG
A. Citrate converted to Glucose-1-phosphate by Hexose-1- MASANGK 2014
B. Glucose-6-Phosphate phosphate uridyltransferase and is then converted AY III, MD
C. Fructose-6-phosphate again to Glucose-6-phosphate by (TOP 8 -
D. Succinyl CoA Phosphoglucomutase, Embden Meyerhof pathway is FEB 2014
E. Fumarate the other name for Glycolysis, A,D and E are MED
intermediates of the Kreb's Cycle. BOARDS;
TOPNOTCH
MD)
405 A Gold olympic medalist for boxing has suffered an Energy intake=Energy expenditure. Sedentary:30; JOSE DIAGNOSTIC
injury and was eventually admitted, you were his Moderately active:35; Very Active:40. CARLO EXAM - AUG
admitting physician and upon ordering his diet MASANGK 2014
requirements you computed it according to his average AY III, MD
dietary energy intake which is: (TOP 8 -
A. 25 kcal/kg/day FEB 2014
B. 30 kcal/kg/day MED
C. 35 kcal/kg/day BOARDS;
D. 40 kcal/kg/day TOPNOTCH
E. 45 kcal/kg/day MD)

406 Fat provides how many percentage of the energy Carbohydrates: 45-65%; Protein 10-35%(SIMILAR TO JOSE DIAGNOSTIC
requirements in humans? PREVIOUS BOARD EXAM CONCEPT/PRINCIPLE) CARLO EXAM - AUG
A. 10-35% MASANGK 2014
B. 10-20% AY III, MD
C. 20-35% (TOP 8 -
D. 20-50% FEB 2014
E. 45-65% MED
BOARDS;
TOPNOTCH
MD)
407 Which of the following amino acids will be most likely Leucine, Isoleucine, Valine, Alanine, Glycine, JOSE DIAGNOSTIC
found in the interior of a protein? Phenylalanine, Tyrosine, Methionine and Proline are CARLO EXAM - AUG
A. Cysteine all Non-polar Amino acids, hence can be found in the MASANGK 2014
B. Leucine interior of a protein. AY III, MD
C. Isoleucine (TOP 8 -
D. B and C FEB 2014
E. All of the above MED
BOARDS;
TOPNOTCH
MD)
408 The following statements are true regarding the All are correct except B which is supposed to be JOSE DIAGNOSTIC
utilization of Pyridoxine by the human body, EXCEPT: "Coenzyme for ALA synthase" CARLO EXAM - AUG
A. Coenzyme for Glycogen phosphorylase MASANGK 2014
B. Coenzyme for ALA Reductase AY III, MD
C. Synthesis of Niacin from Tryptophan (TOP 8 -
D. Synthesis of GABA from Glutamate FEB 2014
E. Coenzyme of Cystathione synthase MED
BOARDS;
TOPNOTCH
MD)
409 A pediatric patient presented to you with noted This is a classic case of Lead poisoning. ALA JOSE DIAGNOSTIC
Basophilic stippling on RBC, Memory loss, Peripheral dehydratase together with ferrochelatase are both CARLO EXAM - AUG
Neuropathy, and lines in the gums, due to your diligent inhibited by lead. (similar to a SIMILAR TO PREVIOUS MASANGK 2014
history, you extracted from the patient's mother that the BOARD EXAM CONCEPT/PRINCIPLEion) AY III, MD
walls of their house had several "chapped" paints, which (TOP 8 -
the patient may have ingested. Which of the following FEB 2014
enzymes in heme synthesis is inhibited by the said MED
suspected ingested substance? BOARDS;
A. ALA Dehydratase TOPNOTCH
B. ALA reductase MD)
C. ALA synthase
D. Uroporphyrinogen decarboxylase
E. Uroporphyrinogen synthase

410 An 8 month-old patient presented to you with noted This is a classic case of Alkaptonuria, an Inborn error JOSE DIAGNOSTIC
black conjunctival pigment, upon examination you noted of metabolism. CARLO EXAM - AUG
that the patients diaper had black stains. What enzyme is MASANGK 2014
most likely deficient in this patient? AY III, MD
A. Dihydropteroate synthase (TOP 8 -
B. Cystathione synthase FEB 2014
C. Phenylalanine hydroxylase MED
D. Homogentisic Acid Oxidase BOARDS;
E. Tyrosine hydroxylase TOPNOTCH
MD)

TOPNOTCH MEDICAL BOARD PREP BIOCHEMISTRY SUPEREXAM Page 51 of 92


For inquiries visit www.topnotchboardprep.com.ph or email us at topnotchmedicalboardprep@gmail.com
TOPNOTCH MEDICAL BOARD PREP BIOCHEMISTRY SUPEREXAM
For inquiries visit www.topnotchboardprep.com.ph or email us at topnotchmedicalboardprep@gmail.com
Item QUESTION EXPLANATION AUTHOR TOPNOTCH
# EXAM
411 Which of the following Apolipoproteins is a cofactor for JOSE DIAGNOSTIC
Lipoprotein Lipase? CARLO EXAM - AUG
A. A-1 MASANGK 2014
B. B-100 AY III, MD
C. C-II (TOP 8 -
D. B-48 FEB 2014
E. E MED
BOARDS;
TOPNOTCH
MD)
412 A female african-american patient presented to you with This is a classic case of Sickle Cell Anemia, caused by a JOSE DIAGNOSTIC
anemia, tissue anoxia, and painful crises, Upon doing a point mutation in both genes coding for the B chain CARLO EXAM - AUG
PBS, the pathologist noted several misshapen RBCs, that results in the replacemnt of Glutamate by Valine MASANGK 2014
forming crescent shapes. Which of the following is true in the 6th position. AY III, MD
regarding the pathophysiology of this disease? (TOP 8 -
A. Lysine is substituted for Glutamate in the 6th FEB 2014
position MED
B. Lysine is substituted for Glutamate in the 8th BOARDS;
position TOPNOTCH
C. Valine is substituted for Glutamate in the 6th MD)
position
D. Valine is substituted for Glutamate in the 8th
position
E. There is a mutation in the development of
Ankyrin/Spectrin
413 When old RBCs are degraded by the spleen, Free Haptoglobin prevents loss of free hemoglobin thru the JOSE DIAGNOSTIC
hemoglobin is produced. One of the following binds kidneys, thus conserving the Iron present in CARLO EXAM - AUG
extracorpuscular hemoglobin, preventing free hemoglobin which would otherwise be lost in the MASANGK 2014
hemoglobin from entering the kidneys: body.(SIMILAR TO PREVIOUS BOARD EXAM AY III, MD
A. Ferritin CONCEPT/PRINCIPLEion) (TOP 8 -
B. Transferrin FEB 2014
C. Albumin MED
D. Haptoglobin BOARDS;
E. Hemopexin TOPNOTCH
MD)
414 Jaundice is a clinical sign when there is systemic B-Glucuronidase is an enzyme present in intestinal JOSE DIAGNOSTIC
yellowish discoloration. This is prevented by its bacteria, which converts conjugated bilirubin to a CARLO EXAM - AUG
catabolic pathway, which of the following enzymes colorless substance called Urobilinogen and is then MASANGK 2014
convert bilirubin into a colorless compound? converted to Stercobilin (Feces) or Urobilin (Urine) AY III, MD
A. Urobilinogen reductase (SIMILAR TO PREVIOUS BOARD EXAM (TOP 8 -
B. B-Glucuronidase CONCEPT/PRINCIPLEion) FEB 2014
C. UDP-Glucose Dehydrogenase MED
D. Biliverdin reductase BOARDS;
E. Cytochrome P450 TOPNOTCH
MD)
415 This is a branch of medical science wherein it involves SIMILAR TO PREVIOUS BOARD EXAM JOSE DIAGNOSTIC
the design of computer algorithms and construction of CONCEPT/PRINCIPLE CARLO EXAM - AUG
databases that enable biomedical scientists to access and MASANGK 2014
analyze the growing avalanche of biomedical data AY III, MD
A. Genomics (TOP 8 -
B. Genetics FEB 2014
C. Bioinformatics MED
D. Computational Biology BOARDS;
E. Human Genome Project TOPNOTCH
MD)
416 Eicosanoids are important groups of physiologically and Linoleic Acid is an essential fatty acid which is the JOSE DIAGNOSTIC
pharmacologically active compounds, including precursor of a membrane phospholipid, Arachidonic CARLO EXAM - AUG
Prostaglandins, thromboxanes, leukotrienes and Acid which is the main component for the synthesis of MASANGK 2014
lipoxins. Which of the following Fatty acids can Eicosanoids. AY III, MD
Eicosanoids be derived from? (TOP 8 -
A. Linoleic Acid FEB 2014
B. Palmitic Acid MED
C. Linolenic Acid BOARDS;
D. Oleic Acid TOPNOTCH
E. Stearic Acid MD)
417 Sickle Cell Anemia is a mutation were an amino acid in a Silent-New codon, same amino acid; Nonsense-Stop JOSE DIAGNOSTIC
sequence has been replaced by a different amino acid, codon; Transition-Purine-Purine change; CARLO EXAM - AUG
What type of mutation is indicated in this case? Transversion-Purine-Pyrimidine Change(PSIMILAR MASANGK 2014
A. Silent TO PREVIOUS BOARD EXAM CONCEPT/PRINCIPLE) AY III, MD
B. Missense (TOP 8 -
C. Nonsense FEB 2014
D. Transition MED
E. Transversion BOARDS;
TOPNOTCH
MD)
418 This rule indicates that the total amount of purines In any sample of dsDNA, the amount of Adenine JOSE DIAGNOSTIC
equals the total amount of pyrimidines in a sequence: equals to the amount of Thymine, the amount of CARLO EXAM - AUG
A. Shine-Dalgarno's Rule Guanine equals to the amount of Cytosine. MASANGK 2014
B. Chargaff's Rule AY III, MD
C. Pribnow's Rule (TOP 8 -
D. Hogness' Rule FEB 2014
E. Sanger's Rule MED
BOARDS;
TOPNOTCH
MD)
TOPNOTCH MEDICAL BOARD PREP BIOCHEMISTRY SUPEREXAM Page 52 of 92
For inquiries visit www.topnotchboardprep.com.ph or email us at topnotchmedicalboardprep@gmail.com
TOPNOTCH MEDICAL BOARD PREP BIOCHEMISTRY SUPEREXAM
For inquiries visit www.topnotchboardprep.com.ph or email us at topnotchmedicalboardprep@gmail.com
Item QUESTION EXPLANATION AUTHOR TOPNOTCH
# EXAM
419 Tetracycline is an antibiotic which inhibits protein JOSE DIAGNOSTIC
synthesis in prokaryotes, specifically, on what aspect of CARLO EXAM - AUG
protein synthesis does it act? MASANGK 2014
A. Inhibits prokaryotic peptidyltransferase AY III, MD
B. Binds to the 50s subunit and distorts its structure (TOP 8 -
C. Binds to the B-subunit of bacterial DNA-dependent FEB 2014
RNA polymerase MED
D. Prevents binding of aminoacyl tRNAs to the A site BOARDS;
E. Inactivates Elongation Factor-2 TOPNOTCH
MD)
420 You and your friends went to Vikings for an eat-all-you- Increased Epinephrine levels are seen in the fasting JOSE DIAGNOSTIC
can buffet, during your fed state the following state, which promotes Glycogenolysis to maintain CARLO EXAM - AUG
biochemical reactions are currently occuring in your normal blood glucose levels MASANGK 2014
body except: AY III, MD
A. Increased Protein synthesis by the muscles (TOP 8 -
B. Increased Triglyceride synthesis and storage FEB 2014
C. Replenished its glycogen stores MED
D. Decreased Glucagon:Insulin ratio BOARDS;
E. Increased Epinephrine levels TOPNOTCH
MD)
421 To maintain normal protein and energy balance in the Leucine and lysine are the only ketogenic amino acids. WEBSTER MIDTERM 1
body, the carbon skeletons of amino acids are They can be degraded directly to become acetyl CoA ALINDOG, EXAM - AUG
catabolized yielding different intermediates including by ketogenesis (acetoacetate). They cannot be MD (TOP 3 2014
ketone bodies. Which of the following amino acids is converted to glucose since the carbon atoms in the - FEB 2014
exclusively ketogenic? ketones are ultimately degraded to carbon dioxide. MED
A. Phenylalanine Phenylalanine and isoleucine, as well as tyrosine and BOARDS;
B. Leucine tryptophan are both ketogenic and glucogenic; while TOPNOTCH
C. Isoleucine methionine and the rest of the amino acids are MD)
D. Methionine glucogenic.
E. All of the above

422 Glucose-6-phosphate dehydrogenase deficiency is WEBSTER MIDTERM 1


characterized by hemolytic episodes secondary to poor ALINDOG, EXAM - AUG
defense against oxidative stress. The most common MD (TOP 3 2014
precipitating factor to hemolysis among these patients - FEB 2014
is: MED
A. Drugs BOARDS;
B. Diet (fava beans) TOPNOTCH
C. Infection MD)
D. Spontaneous, undetermined

423 A 9-month old female was observed to have excessive The first clue here is the involvement of a REDUCING WEBSTER MIDTERM 1
REDUCING sugars in the urine. She was also noted to be sugar, and fructose should be our main consideration. ALINDOG, EXAM - AUG
jaundiced, hypoglycemic, acidotic, and with elevated There are only 2 metabolic disorders involving MD (TOP 3 2014
AST. There was significant improvement upon removal fructose that we should know - essential fructosuria - FEB 2014
of the sugar from her diet. The patient is most likely (fructokinase deficiency) which is essentially benign MED
suffering from deficiency of: and asymptomatic, and hereditary fructose BOARDS;
A. Aldolase B intolerance in which aldolase B is deficient. Absence of TOPNOTCH
B. Fructokinase aldolase B leads to intracellular trapping of fructose 1- MD)
C. Glucokinase phosphate which results in vomiting, severe
D. Galactose-1-phosphate uridyltransferase A hypoglycemia, jaundice, hemorrhage and finally,
hepatic failure. Removal of dietary fructose on the
other hand serves as the main therapy.
424 Peptide bonds covalently join amino acids to form The partial double-bond feature of peptide bond WEBSTER MIDTERM 1
proteins. They are amide linkages between the alpha- makes the structure of protein rigid and planar at the ALINDOG, EXAM - AUG
carboxyl group of 1 amino acid and the alpha-amino same time, just enough to allow assumption of certain MD (TOP 3 2014
group of another. Which of the following statements configurations. Prolonged exposure to strong acid or - FEB 2014
about peptide bonds is true? base at elevated temperatures (at 110oC for 24 hours) MED
A. Rapid exposure to strong acids is required to is required to hydrolyze these bonds non- BOARDS;
hydrolyze peptide bonds non-enzymatically. enzymatically. Urea cannot break down these bonds TOPNOTCH
B. They can be broken by exposure to high whereas trypsin can cleave them only at the carbonyl MD)
concentrations of urea. side of either arginine or lysine.
C. Trypsin can cleave peptide bonds on the amino end
of proteins.
D. They possess a partial double-bond character.
E. All of these are true.

425 Deficiency of which of the following vitamins Vitamin B6 (pyridoxine phosphate) is a co-factor in WEBSTER MIDTERM 1
predisposes a patient to anemia? the formation of delta-aminolevulinic acid, the rate- ALINDOG, EXAM - AUG
A. Iron limiting step in heme synthesis; and is catalyzed by MD (TOP 3 2014
B. Vitamin B2 ALA synthase. This is also the reason why isoniazid - FEB 2014
C. Vitamin B6 therapy in which pyridoxine deficiency may be MED
D. Vitamin D common, can result in anemia. Iron, although essential BOARDS;
in heme synthesis, is not considered a vitamin but a TOPNOTCH
mineral. (Always remember the rule, CHOOSE THE MD)
BEST ANSWER). Vitamin is organic, mineral is not.

TOPNOTCH MEDICAL BOARD PREP BIOCHEMISTRY SUPEREXAM Page 53 of 92


For inquiries visit www.topnotchboardprep.com.ph or email us at topnotchmedicalboardprep@gmail.com
TOPNOTCH MEDICAL BOARD PREP BIOCHEMISTRY SUPEREXAM
For inquiries visit www.topnotchboardprep.com.ph or email us at topnotchmedicalboardprep@gmail.com
Item QUESTION EXPLANATION AUTHOR TOPNOTCH
# EXAM
426 A 24-year old asymptomatic patient was noted to have It may not be your typical biochem question but you WEBSTER MIDTERM 1
decreased hemoglobin and hematocrit. Her RBC indices should know that anything can be asked in any ALINDOG, EXAM - AUG
were MCHC 26%, MCV 74 fL - hypochromic, microcytic. subject, and biochem is no exception. Our main hint MD (TOP 3 2014
At this point, your most likely consideration for the here is the RBC indices indicating a hypochromatic, - FEB 2014
cause of anemia would be: microcytic type of anemia. You should only remember MED
A. Iron deficiency 5 anemias exhibiting this picture and the mnemonic is BOARDS;
B. Folate deficiency CLITS - anemia of Chronic disease, Lead poisoning, TOPNOTCH
C. Spherocytosis Iron deficiency anemia, Thalassemia and Sideroblastic MD)
D. Acute blood loss from menstruation anemia. For megaloblastic (macrocytic) anemias, just
remember folate and vitamin B12 deficiency, and
pernicious anemia. Acute blood loss and spherocytosis
is often times normocytic and normochromic. Keeping
this in mind, you'll get a very good chance in
answering questions not only in biochem, but more
likely in physio, patho, IM, and pedia.
427 Bilirubin is conjugated in the liver mainly for the Keep yourself from choosing "all of the above" that WEBSTER MIDTERM 1
purpose of: easily. You have to really read through and analyze. ALINDOG, EXAM - AUG
A. Increasing the levels of delta bilirubin which is the The goal of bilirubin conjugation is to make it more MD (TOP 3 2014
less toxic form. soluble in water (plasma) so that it can be easily - FEB 2014
B. Making it more water-soluble for easier excretion. excreted (direct bili/B2). Delta bilirubin is bilirubin MED
C. Inhibiting its binding with albumin to reduce risk of covalently bound to albumin, which is also non-toxic BOARDS;
jaundice. but cannot be excreted; it happens in prolonged TOPNOTCH
D. Keeping its value less than 0.1 mg/dL in the blood. periods of hyperbilirubinemia. Indirect bilirubin MD)
E. All of these. naturally binds with albumin for transport to the liver
for conjugation. Bilirubin levels must be below 2.5
mg/dl to prevent development of jaundice.
428 What is the default product of fatty acid synthesis? Be thrilled with quick and simple questions like this WEBSTER MIDTERM 1
A. Linoleic acid =). ALINDOG, EXAM - AUG
B. Arachidonic acid MD (TOP 3 2014
C. Formic acid - FEB 2014
D. Palmitic acid MED
BOARDS;
TOPNOTCH
MD)
429 In the intestine, primary bile acids are dehydroxylated to Just remember, cholic acid and chenodeoxycholic acid WEBSTER MIDTERM 1
form secondary bile acids. Which of the following is a are primary bile acids, whereas deoxycholic and ALINDOG, EXAM - AUG
secondary bile acid? lithocholic acids are secondary bile acids. When MD (TOP 3 2014
A. Cholic acid primary bile acids are conjugated with either taurine - FEB 2014
B. Cholic acid + taurine or glycine, they become bile salts. Secondary bile acids MED
C. Chenodeoxycholic acid are formed when bacteria act upon (deconjugate and BOARDS;
D. Lithocholic acid dehydroxylate) the bile salts. TOPNOTCH
MD)

430 Which secondary messenger system is utilized by the Must remember at least 2 examples for each WEBSTER MIDTERM 1
hormone glucagon? secondary messenger system: glucagon and ALINDOG, EXAM - AUG
A. Tyrosine kinase epinephrine (β and α2) for cAMP pathway; ANP and MD (TOP 3 2014
B. cGMP-guanylate cyclase NO for cGMP; insulin and growth factors for tyrosine - FEB 2014
C. None, it binds intracellularly kinase; epinephrine α1 for IP3/DAG; and MED
D. cAMP-adenylate cyclase glucocorticoids and thyroid hormone for the BOARDS;
intracellular pathway. TOPNOTCH
MD)
431 The rate-limiting step in the Krebs cycle involves the More often than not, you'll get clues from the question. WEBSTER MIDTERM 1
conversion of isocitrate to alpha-ketoglutarate. The Even without studying Krebs, the answer is already ALINDOG, EXAM - AUG
enzyme responsible for this step is: obvious from the question itself. MEMORIZE all the MD (TOP 3 2014
A. Isocitrate dehydrogenase rate-limiting enzymes and steps. - FEB 2014
B. Alpha-ketoglutarate dehydrogenase MED
C. Aconitase BOARDS;
D. Succinate thiokinase TOPNOTCH
MD)

432 The last step in the enterohepatic circulation involves: The answer lies in the main purpose of the WEBSTER MIDTERM 1
A. Removal of taurine/glycine from the bile salt enterohepatic circuit and that is to reabsorb majority ALINDOG, EXAM - AUG
compound. of bile acids and recycle them in the liver while also MD (TOP 3 2014
B. Reabsorption of about 95-99% of primary and serving as the mean for excreting body cholesterol. - FEB 2014
secondary bile acids in the ileum with excretion of some MED
unesterified cholesterol. BOARDS;
C. Dehydroxylating action of intestinal bacteria on TOPNOTCH
primary bile acids with their conversion to secondary MD)
bile acids in equal molar amounts.
D. The 7-alpha-hydroxylation of cholesterol as
catalyzed by cholesterol 7-alpha hydroxylase.
E. None of these.
433 What is the amino acid precursor of glutathione and Must know the other important amino acid WEBSTER MIDTERM 1
GABA? precursors: PHENYLALANINE --- tyrosine (another ALINDOG, EXAM - AUG
A. Glutamate amino acid), L-dopa, dopamine, norepinephrine, MD (TOP 3 2014
B. Glutamine epinephrine; TYROSINE --- thyroxine, melanin; - FEB 2014
C. Methionine TRYPTOPHAN --- niacin, serotonin, melatonin; MED
D. Glycine METHIONINE --- homocysteine. (high yield) BOARDS;
TOPNOTCH
MD)

TOPNOTCH MEDICAL BOARD PREP BIOCHEMISTRY SUPEREXAM Page 54 of 92


For inquiries visit www.topnotchboardprep.com.ph or email us at topnotchmedicalboardprep@gmail.com
TOPNOTCH MEDICAL BOARD PREP BIOCHEMISTRY SUPEREXAM
For inquiries visit www.topnotchboardprep.com.ph or email us at topnotchmedicalboardprep@gmail.com
Item QUESTION EXPLANATION AUTHOR TOPNOTCH
# EXAM
434 The enzyme responsible to replace the stretches of WEBSTER MIDTERM 1
highly repititive DNA found at the ends of linear ALINDOG, EXAM - AUG
chromosomes in cells in order to avoid aging as seen in MD (TOP 3 2014
cancer and germline cells is: - FEB 2014
A. Elongase MED
B. Transcriptase BOARDS;
C. Telomerase TOPNOTCH
D. Polymerase MD)

435 This is a disease condition resulting from formation of WEBSTER MIDTERM 1


thymine dimers secondary to exposure to UV without its ALINDOG, EXAM - AUG
normal removal by UV-specific endonuclease: MD (TOP 3 2014
A. Epidermolysis bullosa - FEB 2014
B. HNPCC MED
C. Xeroderma pigmentosum BOARDS;
D. FAP TOPNOTCH
MD)

436 What constitute the central dogma? WEBSTER MIDTERM 1


A. Replication, DNA synthesis, proofreading ALINDOG, EXAM - AUG
B. Replication, transcription, translation MD (TOP 3 2014
C. Transcription, translation, modification - FEB 2014
D. Translation, modification, labeling and secretion MED
E. All of these may constitute the central dogma. BOARDS;
TOPNOTCH
MD)
437 The type of mutation which results in a new codon Frameshift means there is deletion or addition of a WEBSTER MIDTERM 1
specifying a different amino acid which may or may not base which results in protein products that are ALINDOG, EXAM - AUG
cause variation in protein function is known as: shorter than normal and hence usually non-functional. MD (TOP 3 2014
A. Frameshift Non-sense, on the other hand, refers to formation of a - FEB 2014
B. Nonsense new stop codon making the protein shorter and also MED
C. Missense non-functional. A silent mutation involves a new BOARDS;
D. Silent codon specifying the same amino acid and therefore TOPNOTCH
has no effect in the protein function. MD)

438 Cystinuria is an inherited condition characterized by Cystinuria is the most common genetic error of amino WEBSTER MIDTERM 1
formation of kidney stones brought about by defects in acid transport. It has defective kidney tubule ALINDOG, EXAM - AUG
the renal tubular amino acid transporter for: reabsorption of the amino acids cysteine, ornithine, MD (TOP 3 2014
A. Ornithine lysine and arginine (mnemonic: COLA). There is - FEB 2014
B. Arginine precipitation of cystine to form kidney stones which MED
C. Cystine can cause urinary tract obstruction. Under the BOARDS;
D. All of these microscope, you can appreciate cystine as hexagonal- TOPNOTCH
E. None of these shaped crystals in the urine, looking much like your MD)
favorite "Piattos" chips.
439 In the regular human diet, the estimated energy This topic is must know. For carbohydrate, it is 45- WEBSTER MIDTERM 1
requirement coming from fat is set at: 65% and for protein 10-35%. The expected total ALINDOG, EXAM - AUG
A. 10-25% energy required per day for a sedentary person is set MD (TOP 3 2014
B. 20-35% at 30 kcal/kg/day; for the moderately active and in- - FEB 2014
C. 45-65% patients, 35 kcal/kg/day; and for the very active, 40 MED
D. >60% kcal/kg/day. BOARDS;
TOPNOTCH
MD)

440 Hemoglobin is different from myoglobin by: Myoglobin on the other hand, has only 1 polypetide, it WEBSTER MIDTERM 1
A. Having oxygen dissociation curve that is sigmoidal in has a hyperbolic-shaped oxygen dissociation curve ALINDOG, EXAM - AUG
shape and functions mainly as storage for oxygen both in MD (TOP 3 2014
B. Possessing 4 polypeptides in the form of globins heart and skeletal muscles. Myoglobin is the - FEB 2014
C. Exhibiting allosteric effects EARLIEST detectable cardiac element found in plasma MED
D. Being utilized for oxygen transport rather than during MI. BOARDS;
storage TOPNOTCH
E. All of these MD)
441 Electrons are passed down the transport chain to create Electron Transport Chain Inhibitors: Complex I- JULIET MIDTERM 2
proton gradient which results to ATP synthesis. Several Barbiturate, Piericidin A, Amytal, Rotenone, Complex KRISTINE EXAM - AUG
inhibitors stop electron flow from substrate to oxygen. II-Malonate, Carboxin, TTFA, Complex III-Antimycin A, EVANGELIS 2014
Inhibitor/s of Cytochrome oxidase include/s: Dimercaprol, Complex IV-Cyanide, Carbon monoxide, TA, MD
A. Cyanide sodium azide, hydrogen sulfide (TOP 9 -
B. Malonate FEB 2014
C. Hydrogen sulfide MED
D. A and B BOARDS;
E. A and C TOPNOTCH
MD)
442 A newborn was brought to a clinic by her mother due to Classic Galactosemia occurs in the absence of JULIET MIDTERM 2
vomiting every after feeding. PE revealed that the galactose 1-phosphate uridyltransferase resulting to KRISTINE EXAM - AUG
newborn had severe jaundice and enlarged liver. NB accumulation of galactitol. The manifestations are EVANGELIS 2014
screening result was positive for galactosemia and vomiting and diarrhea after milk ingestion, TA, MD
lactose-containing substances are removed from the hypoglycemia, liver disease and cirrhosis, lethargy (TOP 9 -
diet. In this case, lactose is toxic to the NB because: and hypotonia and mental retardation. FEB 2014
A. Excess galactose shuts glucose metabolism MED
B. Galactose is toxic even in little amounts BOARDS;
C. Galactose is converted to galactitol TOPNOTCH
D. Excess glucose accumulated in the blood MD)
E. Excess galactose is converted to sorbitol

TOPNOTCH MEDICAL BOARD PREP BIOCHEMISTRY SUPEREXAM Page 55 of 92


For inquiries visit www.topnotchboardprep.com.ph or email us at topnotchmedicalboardprep@gmail.com
TOPNOTCH MEDICAL BOARD PREP BIOCHEMISTRY SUPEREXAM
For inquiries visit www.topnotchboardprep.com.ph or email us at topnotchmedicalboardprep@gmail.com
Item QUESTION EXPLANATION AUTHOR TOPNOTCH
# EXAM
443 All are true of the pentose phosphate pathway, EXCEPT:
Pentose phosphate pathway occurs in the cytoplasm JULIET MIDTERM 2
A. It provides the route for metabolism of 5-carbon of RBCs, liver, adipose tissue, adrenals, thyroid, testes, KRISTINE EXAM - AUG
sugars lactating mammaries and tissues that are producing EVANGELIS 2014
B. It is the source of ribose 5-phosphate for nucleotide lipids. TA, MD
synthesis (TOP 9 -
C. It is only functional in red blood cells FEB 2014
D. It is the source of reduced NADP MED
E. It is active in tissues that produce lipids BOARDS;
TOPNOTCH
MD)
444 A non-competitive inhibitor of an enzyme:
Non-competitive inhibitor results to decreased Vmax JULIET MIDTERM 2
A. Increases Km and decreases affinity but it does not affect KM and affinity. KRISTINE EXAM - AUG
B. Decreases Km and increases affinity EVANGELIS 2014
C. Increases Vmax TA, MD
D. Decreases Vmax (TOP 9 -
E. None of the above FEB 2014
MED
BOARDS;
TOPNOTCH
MD)
445 In prokaryotic RNA synthesis, the function of the "rho" rho factor is required for termination of transcription JULIET MIDTERM 2
factor: A. of genes. KRISTINE EXAM - AUG
It initiates the binding of RNA polymerase EVANGELIS 2014
B. It participates in proper termination of transciption TA, MD
C. It binds repressor to the promoter region (TOP 9 -
D. It increases the rate of RNA synthesis FEB 2014
E. It eliminates binding of RNA polymerase to promoter MED
region BOARDS;
TOPNOTCH
MD)
446 In contrast to RNA polymerase, DNA polymerase:
DNA polymerase synthesizes DNA in both strands, the JULIET MIDTERM 2
A. Synthesizes RNA primer to initiate DNA synthesis leading and lagging strands, catlyzes chain elongation, KRISTINE EXAM - AUG
B. It fills in the gap between Okazaki fragments proofreads the newly synthesized DNA, read template EVANGELIS 2014
C. Synthesizes in a 3' to 5' direction in 3' to 5' direction and synthesize in 5' to 3' direction. TA, MD
D. Does not edit as it synthesizes Primase not DNA polymerase synthesizes short (TOP 9 -
E. Does not require a primer stretches of RNA called primers. FEB 2014
MED
BOARDS;
TOPNOTCH
MD)
447 A 1 year-old boy was seen at the ER with severe Pompe Disease or Type II Gylcogen storage disease is JULIET MIDTERM 2
difficulty of breathing. His blood sugar was normal. caused by deficiency in acid maltase or lysosomal α1- KRISTINE EXAM - AUG
Glycogen structure was normal although excessive 4 and α1:6 glucosidase. Manifestations include muscle EVANGELIS 2014
concentrations in abnormal vacuoles in the lysosomes is hypotonia, death from heart failure by age 2 TA, MD
found. He had also massive cardiomegaly and he died (TOP 9 -
from heart failure. The most probable enzyme deficient FEB 2014
in the patient leading to diagnosis is: MED
A. glucose-6-phosphatase BOARDS;
B. debranching enzyme TOPNOTCH
C. branching enzyme MD)
D. acid maltase
E. glycogen phosphorylase

448 The oxidation of pyruvate to acetyl coA by pyruvate Conversion of pyruvate to acetyl CoA is through the JULIET MIDTERM 2
dehydrogenase is dependent on this vitamin co-factor: pyruvate dehydrogenase complex which requires the KRISTINE EXAM - AUG
A. Biotin presence of 5 co-enzymes including: thiamine, FAD, EVANGELIS 2014
B. Niacin NAD, Pantothenic acid and lipoic acid. TA, MD
C. Thiamine (TOP 9 -
D. Vitamin C FEB 2014
E. Vitamin A MED
BOARDS;
TOPNOTCH
MD)
449 Which RNA viral genome encodes a precursor protein gag gene encodes a precursor protein that is cleaved JULIET MIDTERM 2
that is cleaved to form a protein core: to form a protein core, tat gene is for activation of KRISTINE EXAM - AUG
A. tat transcription of viral genes, pol encodes for enzymes EVANGELIS 2014
B. gag reverse transcriptase, protease intregrase and TA, MD
C. pol ribonuclease, env codes for envelope glycoproteins, (TOP 9 -
D. rev rev is for transport of late mRNAs to cytoplasm FEB 2014
E. env MED
BOARDS;
TOPNOTCH
MD)
450 Key enzyme in amino acid metabolism which is Through glutamate dehydrogenase, glutamate is JULIET MIDTERM 2
responsible for release of nitrogen as free ammonia, for oxidatively deaminated to liberate free ammonia. KRISTINE EXAM - AUG
subsequent detoxification in the urea cycle: EVANGELIS 2014
A. Glutamate dehydrogenase TA, MD
B. Pyruvate dehydrogenase (TOP 9 -
C. Alpha-ketoglutarate dehydrogenase FEB 2014
D. Succinate dehydrogenase MED
E. Glutaminase BOARDS;
TOPNOTCH
MD)

TOPNOTCH MEDICAL BOARD PREP BIOCHEMISTRY SUPEREXAM Page 56 of 92


For inquiries visit www.topnotchboardprep.com.ph or email us at topnotchmedicalboardprep@gmail.com
TOPNOTCH MEDICAL BOARD PREP BIOCHEMISTRY SUPEREXAM
For inquiries visit www.topnotchboardprep.com.ph or email us at topnotchmedicalboardprep@gmail.com
Item QUESTION EXPLANATION AUTHOR TOPNOTCH
# EXAM
451 All of these enzymes act on protein substrates in the polypeptides and amino acids are digested further in JULIET MIDTERM 2
small intestines the small intestines by elastase, carboxypeptidase, KRISTINE EXAM - AUG
A. Trypsin, chymotrypsin, elastase trypsin, chymotrypsin; pepsin is in the stomach, EVANGELIS 2014
B. Pepsin, trypsin, chymotrypsin amylase is for digestion of carbohydrates TA, MD
C. Amylase, pepsin, trypsin (TOP 9 -
D. Elastase, pepsin, amylase FEB 2014
E. Pepsin, chymotrypsin, elastase MED
BOARDS;
TOPNOTCH
MD)
452 A defect in peroxisomal activation of very long-chain X-linked adrenaleukodystrophy occurs with defect in JULIET MIDTERM 2
fatty acid which leads to its accumulation in the blood peroxisomal activation of VLCFA leading to its KRISTINE EXAM - AUG
and tissues: accumulation in the blood and tissues. Initial EVANGELIS 2014
A. Zellweger syndrome abnormalities are apathy and behavioral change. Late TA, MD
B. X-linked adrenaleukodystrophy findings include visual loss, spasticity, ataxia and (TOP 9 -
C. Refsum's Disease death a few years after onset of neurologic symptoms. FEB 2014
D. Jamaican vomiting sickness Zellweger syndrome is a rare inheroted absence of MED
E. Fatty liver peroxisomes in tissues. Refsum's disease is a defect BOARDS;
which results in accumulation of phytanic aid. TOPNOTCH
Jamaican vomiting sickness is caused by eating unripe MD)
fruit of akee tree which contains hypoglycin. Fatty
liver is due to alcoholism which leads to fat
accumulation in the liver.
453 It is the average daily dietary intake level that is Recommended Daily Allowance (RDA) is the average JULIET MIDTERM 2
sufficient to meet the requirements of >95% of all daily dietary intake level that is sufficient to meet the KRISTINE EXAM - AUG
individuals in a life stage and gender group: requirements of >95% of all individuals in a life stage EVANGELIS 2014
A. Dietary requirement and gender group. TA, MD
B. Total caloric requirement (TOP 9 -
C. Recommended daily allowance FEB 2014
D. Total energy requirement MED
E. Estimated average requirement BOARDS;
TOPNOTCH
MD)
454 In an adult, the energy requirement for protein is: Energy requirements in Humans: Fat=20 to 35%, JULIET MIDTERM 2
A. 10 to 35% Carbohydrate=45 to 65%, Protein=10 to 35% KRISTINE EXAM - AUG
B. 15 to 30% EVANGELIS 2014
C. 25 to 30% TA, MD
D. 20 to 35% (TOP 9 -
E. 45 to 65% FEB 2014
MED
BOARDS;
TOPNOTCH
MD)
455 In order to oxidize fatty acids through B-oxidation in the Carnithine shuttle transports fatty acids from JULIET MIDTERM 2
mitochondria, fatty acids must be ferried through the cytoplasm to mitochondria which uses 2 ATPs in Beta KRISTINE EXAM - AUG
mitochondrial membrane by: exidation. EVANGELIS 2014
A. Acetyl CoA TA, MD
B. Thiokinase (TOP 9 -
C. Thiolases FEB 2014
D. Carnithine MED
E. Citrate BOARDS;
TOPNOTCH
MD)
456 The common biochemical pathology seen in Creutzfeld- Creutzfeld-Jakob disease is a fatal JULIET MIDTERM 2
Jakob disease and Alzheimer's Disease: neurodegenerative disease caused by an altered KRISTINE EXAM - AUG
A. Improper folding of affected proteins due to absence version of normal protein to a pathologic EVANGELIS 2014
of chaperones conformation which is rich in B-sheets. Alzheimer TA, MD
B. Mutated apolipoprotein E is the primary cause disease is progressive cognitive and behavioral (TOP 9 -
C. Aggregation of infected proteins forming impairment due to accumulation of amyloid plaques FEB 2014
neurofibrillary tangles and neurofibrillary tangles. Similar to CJB, alzheimer's MED
D. Secondary helical structure of affected proteins are disease also is rich in B-sheet prone to self- BOARDS;
transformed to B sheet aggregation. Harper's, 27th Ed. p38. TOPNOTCH
E. Fatal neurodegenerative disease due to MD)
accumulation of amyloid plaques
457 This inborn error of metabolism is manifested by Homocystinuria is an inborn error of metabolism JULIET MIDTERM 2
convulsive seizures, lens subluxation, damage of the which is manifested by convulsive seizures, lens KRISTINE EXAM - AUG
arterial walls, myocardial infacrtion and skeletal subluxation, damage of the arterial walls, myocardial EVANGELIS 2014
deformities: infacrtion and skeletal deformities; I-cell Disease is TA, MD
A. I-cell Disease due to accumulation of partially degraded (TOP 9 -
B. Marfan syndrome glycoproteins in lysosomes manifested as severe FEB 2014
C. Homocystinuria mental retardation, coarse facial features, Marfan MED
D. Cystinuria syndrome is due to defect in the gene encoding for BOARDS;
E. Porphyrias fibrillin resulting to lens subluxation, aortic dilatation, TOPNOTCH
archnodactyly, Cystinuria is due to inherited defect or MD)
renal tubular amino acid transporter leading to
formation of staghorn calculi, porphyrias is due to
defect in heme synthesis resulting to photosensitivity
and overt blistering after sun exposure.
458 The rate-limiting step of fatty acid oxidation is: Transport of fatty acyl CoA to mitochondria is the JULIET MIDTERM 2
A. Activation of fatty acid rate-limiting step in B-oxidation which is KRISTINE EXAM - AUG
B. Transport of fatty acyl CoA to mitochondria catalyzed by carnitine acyltransferase. Activation EVANGELIS 2014
C. Formation of malonyl CoA of fatty acids is the only step in the complete TA, MD
D. Provision of fatty acid degradation of a fatty acid that requires energy from (TOP 9 -
E. None of the above ATP. Harper's, 27th Ed. p187. FEB 2014
MED
BOARDS;
TOPNOTCH MEDICAL BOARD PREP BIOCHEMISTRY SUPEREXAM Page 57 of 92
For inquiries visit www.topnotchboardprep.com.ph or email us at topnotchmedicalboardprep@gmail.com
TOPNOTCH MEDICAL BOARD PREP BIOCHEMISTRY SUPEREXAM
For inquiries visit www.topnotchboardprep.com.ph or email us at topnotchmedicalboardprep@gmail.com
Item QUESTION EXPLANATION AUTHOR TOPNOTCH
# EXAM
TOPNOTCH
MD)

459 Deficiency of the activities of DNA repair system leading Nucleotide excision repair is used to replace regions JULIET MIDTERM 2
to sunlight sensitivity and susceptibility in persons at of damaged DNA caused by UV light inducing the KRISTINE EXAM - AUG
risk for Xeroderma pigmentosum is in the: formation of thymine dimers. A special excision EVANGELIS 2014
A. Base excision repair system nuclease cuts the DNA upstream and downstream of TA, MD
B. Nucleotide excision repair system the defective region. The gap is then filled by a (TOP 9 -
C. Double-strand break system polymerase. Harper's p. 344-345 FEB 2014
D. Mismatch repair system MED
E. Multifunctional repair system BOARDS;
TOPNOTCH
MD)
460 A 6 year-old girl was brought to the OPD due to skin Epidermolysis bullosa is characterized by skin breaks JULIET MIDTERM 2
breaks and blisters which noted to occur during minor and blisters as a result of minor trauma with KRISTINE EXAM - AUG
scratching. This is a dystrophic form which is due to mutations affecting the structure of Type VII collagen. EVANGELIS 2014
mutations affecting the structure of collagen type: Type I collagen is affected in osteogenesis imperfecta, TA, MD
A. Type I Type III collagen in Ehlers-Danlos syndrome and Type (TOP 9 -
B. Type III IV collagen structure is affected in Alport syndrome. FEB 2014
C. Type IV MED
D. Type V BOARDS;
E. Type VII TOPNOTCH
MD)
461 Which of the following is a known inhibitor of complex I Amytal is a complex I inhibitor, while others are LUISA BACK-UP
of the electron transport chain? inhibitors of the following: malonate - complex II; SARANILL MIDTERM
A. amytal Antimycin A - complex III; cyanide - complex IV O, MD (TOP EXAM AUG
B. malonate 6 - FEB 2014 - FOR
C. Antimycin A 2014 MED INCLUSION IN
D. cyanide BOARDS; THE SAMPLEX
E. None of the choices TOPNOTCH
MD)

462 The Krebs or Citric acid cycle is the final common krebs or citric acid cycle is the final common pathway LUISA BACK-UP
pathway of the following except: of carbohydrate, protein, and lipid SARANILL MIDTERM
A. carbohydrate O, MD (TOP EXAM AUG
B. protein 6 - FEB 2014 - FOR
C. lipid 2014 MED INCLUSION IN
D. A and B BOARDS; THE SAMPLEX
E. None of the choices TOPNOTCH
MD)

463 What is the rate-limiting enzyme in cholesterol The following are the rate limiting enzymes for: HMG LUISA BACK-UP
synthesis? CoA synthase = ketogenesis; HMG CoA reductase = SARANILL MIDTERM
A. HMG CoA synthase cholesterol synthesis; acetyl CoA carboxylase = fatty O, MD (TOP EXAM AUG
B. HMG CoA reductase acid synthesis; carnitine acyltransferase = fatty acid 6 - FEB 2014 - FOR
C. Acetyl CoA carboxylase oxidation; cholesterol - 7 - α - hydoxylase = bile acid 2014 MED INCLUSION IN
D. Carnitine acyltransferase synthesis BOARDS; THE SAMPLEX
E. cholesterol - 7 - α - hydroxylase TOPNOTCH
MD)
464 Bile acids are synthesized from cholesterol. Which of the chenodeoxycholic acid and cholic acid are the primary LUISA BACK-UP
following is an example of primary bile acids? bile acids, while lithocholic and deoxycholic acid are SARANILL MIDTERM
A. Lithocholic acid the secondary bile acids O, MD (TOP EXAM AUG
B. Chenodeoxycholic acid 6 - FEB 2014 - FOR
C. Deoxycholic acid 2014 MED INCLUSION IN
D. All of the choices BOARDS; THE SAMPLEX
E. None of the choices TOPNOTCH
MD)

465 What is the "mother hormone" of all steroid hormones? cholesterol is the precursor of all steroid hormones LUISA BACK-UP
A. pregnenolone while pregnenolone is the "mother hormone" of all SARANILL MIDTERM
B. cholesterol steroid hormones. Other choices are derivatives of O, MD (TOP EXAM AUG
C. hydroxypregnenolone pregnenolone. 6 - FEB 2014 - FOR
D. progesterone 2014 MED INCLUSION IN
E. Aldosterone BOARDS; THE SAMPLEX
TOPNOTCH
MD)
466 Cardiolipin is an important phospholipid characterized Cardiolipin is a phospholipid that is antigenic which LUISA BACK-UP
as follows except: reacts with antibodies against Treponema pallidum. It SARANILL MIDTERM
A. antigenic phospholipid is found only in mitochondria which is essential for O, MD (TOP EXAM AUG
B. Found in mitochondria and cytoplasm mitochondrial function. 6 - FEB 2014 - FOR
C. Found in mitochondria only 2014 MED INCLUSION IN
D. Reacts with antibodies produced against Treponema BOARDS; THE SAMPLEX
pallidum TOPNOTCH
E. none of the choices MD)

TOPNOTCH MEDICAL BOARD PREP BIOCHEMISTRY SUPEREXAM Page 58 of 92


For inquiries visit www.topnotchboardprep.com.ph or email us at topnotchmedicalboardprep@gmail.com
TOPNOTCH MEDICAL BOARD PREP BIOCHEMISTRY SUPEREXAM
For inquiries visit www.topnotchboardprep.com.ph or email us at topnotchmedicalboardprep@gmail.com
Item QUESTION EXPLANATION AUTHOR TOPNOTCH
# EXAM
467 Which of the following is an essential fatty acids? linoleic, linolenic and arachidonic acid are the LUISA BACK-UP
A. Linoleic acid essential fatty acids. Other choices are essential amino SARANILL MIDTERM
B. isoleucine acids. O, MD (TOP EXAM AUG
C. phenylalanine 6 - FEB 2014 - FOR
D. valine 2014 MED INCLUSION IN
E. All of the above BOARDS; THE SAMPLEX
TOPNOTCH
MD)
468 A single DNA base substitution resulting to formation of frameshift mutation involves deletion or addition of a LUISA BACK-UP
a different amino acid. base resulting to a shorter nonfunctional protein. SARANILL MIDTERM
A. Frame shift mutation Nonsense mutation results to a stop codon. Missense O, MD (TOP EXAM AUG
B. Nonsense mutation mutation forms a different amino acid. Silent mutation 6 - FEB 2014 - FOR
C. Missense mutation forms same amino acid. 2014 MED INCLUSION IN
D. Silent mutation BOARDS; THE SAMPLEX
E. None of the choices TOPNOTCH
MD)
469 The following are essential amino acids that are both all of the choices are both ketogenic and glucogenic, LUISA BACK-UP
ketogenic and glucogenic except: and all are essential amino acids except tyrosine. SARANILL MIDTERM
A. phenylalanine O, MD (TOP EXAM AUG
B. tyrosine 6 - FEB 2014 - FOR
C. tryptophan 2014 MED INCLUSION IN
D. isoleucine BOARDS; THE SAMPLEX
E. None of the choices TOPNOTCH
MD)

470 In the Landsteiner ABO blood system, what determines type A= N-acetyl-galactosamine; type B = galactose; LUISA BACK-UP
specificity of blood type A? type AB = N-acetyl-galactosamine and galactose SARANILL MIDTERM
A. N-acetyl-galactosamine O, MD (TOP EXAM AUG
B. galactose 6 - FEB 2014 - FOR
C. N-acetyl-glucosamine 2014 MED INCLUSION IN
D. A and B BOARDS; THE SAMPLEX
E. All of the choices TOPNOTCH
MD)

471 A 60 year old female diabetic patient was given 20U of The counter-regulatory hormones of insulin are LUISA BACK-UP
regular insulin at bedtime by her daughter. Few hours glucagon, cortisol, epinephrine, and growth hormone. SARANILL MIDTERM
later, the blood sugar level was at 39mg/dl. In order to O, MD (TOP EXAM AUG
increased her blood sugar level, her body should 6 - FEB 2014 - FOR
produce what hormone/s? 2014 MED INCLUSION IN
A. glucagon BOARDS; THE SAMPLEX
B. epinephrine TOPNOTCH
C. cortisol MD)
D. all of the above
E. A and C only
472 Approximately, proteins comprised what percentage of 10-35% = protein; 20-35% = fat; 45-65% = LUISA BACK-UP
energy requirements in human? carbohydrates SARANILL MIDTERM
A. 10-25% O, MD (TOP EXAM AUG
B. 45-60% 6 - FEB 2014 - FOR
C. 20-35% 2014 MED INCLUSION IN
D. 40-45% BOARDS; THE SAMPLEX
E. 50-55% TOPNOTCH
MD)

473 What is the fat soluble vitamin that is considered the Vit. D is the most toxic vitamin while Vitamin E is the LUISA BACK-UP
most toxic vitamin? least toxic vitamin. SARANILL MIDTERM
A. Vitamin A O, MD (TOP EXAM AUG
B. Vitamin D 6 - FEB 2014 - FOR
C. Vitamin E 2014 MED INCLUSION IN
D. Vitamin K BOARDS; THE SAMPLEX
E. Vitamin B TOPNOTCH
MD)
474 A 6 year old child presents with pallor. As her attending This child is most likely having an iron deficiency LUISA BACK-UP
physician, you request for CBC which revealed a anemia. Iron should be given in this patient along with SARANILL MIDTERM
hemoglobin of 9g/dl. What medication/s should you give Vitamin C which increases iron absorption. O, MD (TOP EXAM AUG
to this patient. 6 - FEB 2014 - FOR
A. Iron 2014 MED INCLUSION IN
B. Vitamin C BOARDS; THE SAMPLEX
C. Zinc TOPNOTCH
D. Iron and Vitamin C MD)
E. Iron and zinc

475 Among the following pyrimidine bases, which one is Cytosine, uracil, and thymine are pyrimidine bases, LUISA BACK-UP
present only in the RNA? but uracil is present only in RNA while thymine is SARANILL MIDTERM
A. cytosine present only in DNA. Adenine and guanine are purine O, MD (TOP EXAM AUG
B. thymine bases. 6 - FEB 2014 - FOR
C. uracil 2014 MED INCLUSION IN
D. guanine BOARDS; THE SAMPLEX
E. Adenine TOPNOTCH
MD)

TOPNOTCH MEDICAL BOARD PREP BIOCHEMISTRY SUPEREXAM Page 59 of 92


For inquiries visit www.topnotchboardprep.com.ph or email us at topnotchmedicalboardprep@gmail.com
TOPNOTCH MEDICAL BOARD PREP BIOCHEMISTRY SUPEREXAM
For inquiries visit www.topnotchboardprep.com.ph or email us at topnotchmedicalboardprep@gmail.com
Item QUESTION EXPLANATION AUTHOR TOPNOTCH
# EXAM
476 This disease is caused by low activities of orotidine LUISA BACK-UP
phosphate decarboxylase and orotate SARANILL MIDTERM
phosphoribosyltransferase which causes abnormal O, MD (TOP EXAM AUG
growth and anemia. What type of anemia is present in 6 - FEB 2014 - FOR
this disease? 2014 MED INCLUSION IN
A. hypochromic anemia BOARDS; THE SAMPLEX
B. microcytic anemia TOPNOTCH
C. megaloblastic anemia MD)
D. sideroblastic anemia
E. none of the choices
477 What is the smallest type of RNA? tRNA is the smallest RNA. mRNA is the largest RNA. LUISA BACK-UP
A. tRNA rRNA is the most common type of RNA. snRNA is a SARANILL MIDTERM
B. mRNA subset of RNA for gene regulation. O, MD (TOP EXAM AUG
C. rRNA 6 - FEB 2014 - FOR
D. snRNA 2014 MED INCLUSION IN
E. None of the choices BOARDS; THE SAMPLEX
TOPNOTCH
MD)
478 In metabolic pathways, catabolic reaction is an example Catabolic reaction is an exergonic reaction. Anabolic LUISA BACK-UP
of: reaction is an endergonic reaction. Amphibolic is the SARANILL MIDTERM
A. Endergonic reaction link between anabolic and catabolic reactions. O, MD (TOP EXAM AUG
B. Exergonic reaction 6 - FEB 2014 - FOR
C. Amphibolic reaction 2014 MED INCLUSION IN
D. A and B only BOARDS; THE SAMPLEX
E. B and C only TOPNOTCH
MD)
479 NADH produced from aerobic glycolysis proceeds to the NADH needs a shuttle in order for it to enter the LUISA BACK-UP
electron transport chain. If one molecule of NADH enters mitochondrial membrane to enter the ETC. There are SARANILL MIDTERM
a neuron, how many ATP are produced? 2 shuttles used - the malate aspartate shuttle present O, MD (TOP EXAM AUG
A. 1 in liver, kidney, and heart which produces 3 ATP in 6 - FEB 2014 - FOR
B. 2 one NADH; and the glycerol phosphate shuttle present 2014 MED INCLUSION IN
C. 3 in skeletal muscle and brain which produces 2 ATP in BOARDS; THE SAMPLEX
D. 4 one NADH. TOPNOTCH
E. 5 MD)

480 A 3 day old infant presents with vomiting and diarrhea Gal-1-P uridyltransferase deficiency causes classic LUISA BACK-UP
after milk ingestion. Blood sugar level was 40 mg/dl. The galactosemia which presents with vomiting, diarrhea, SARANILL MIDTERM
infant develops a cataract few days after. What is hypoglycemia, lethargy, hypotonia, liver disease, and O, MD (TOP EXAM AUG
responsible for the disease. cataracts within a few days of birth. While in 6 - FEB 2014 - FOR
A. Galactokinase deficiency galactokinase deficiency, cataracts develop later in 2014 MED INCLUSION IN
B. galactokinase excess early childhood. In aldolase B deficiency, cataract is BOARDS; THE SAMPLEX
C. Gal-1-P uridyltransferase deficiency not characteristic. TOPNOTCH
D. hexokinase deficiency MD)
E. aldolase B deficiency

481 An excess of this vitamin causes sensory neuropathy: Ironically, intake of pyridoxine (B6) which prevents ANGELIS FINAL EXAM -
A. Vitamin A peripheral neuropathy can result in sensory ANDREA AUG 2014
B. Vitamin B neuropathy with altered sensation of touch and pain if COCOS, MD
C. Vitamin C taken in excess. (TOP 1 -
D. Vitamin D FEB 2014
MED
BOARDS;
TOPNOTCH
MD)
482 Which is FALSE regarding beta bends? Proline and glycine are frequently found in beta ANGELIS FINAL EXAM -
A. They are also known as beta turns. bends; proline because its cyclic structure is ideally ANDREA AUG 2014
B. They fall under secondary structure. suited for the beta turn, and glycine because with the COCOS, MD
C. They are usually composed of proline and alanine. smallest side chain of all the amino acids, it is the most (TOP 1 -
D. Beta bends often promote the formation of stericallly flexible. FEB 2014
antiparallel beta sheets. MED
BOARDS;
TOPNOTCH
MD)
483 The most appropriate and prompt treatment for Galactosemia is due to an enzyme deficiency ANGELIS FINAL EXAM -
galactosemia is: (galactose-1-phosphate uridyl transferase) which ANDREA AUG 2014
A. Enzyme replacement leads to accumulation of galactose-1-phosphate in the COCOS, MD
B. Hormone therapy tissues. Treatment consists of prompt elimination of (TOP 1 -
C. Special infant formula lactose-containing milk from the diet in infancy. FEB 2014
D. Vitamin therapy MED
BOARDS;
TOPNOTCH
MD)
484 A 13-year-old healthy Jewish girl, Anne Frank, is found to This is a case of Gaucher disease. The diagnosis is ANGELIS FINAL EXAM -
have mild anemia, leukopenia, and thrombocytopenia. confirmed by the absence of glucocerebrosidase ANDREA AUG 2014
Physical exam reveals an enlarged spleen. Bone marrow activity in leukocytes, in cultured skin fibroblasts and COCOS, MD
exam shows abnormal cells. The diagnosis can be liver cells. Spingomyelinase deficiency causes type A (TOP 1 -
confirmed by measurement of the activity of which of Niemann-Pick disease, hexosaminidase A deficiency FEB 2014
the following enzymes? causes Sandhoff disease, sulfatase A deficiency causes MED
A. glucocerebrosidase metachromatic leukodystrophy. SIMILAR TO BOARDS;
B. sphingomyelinase PREVIOUS BOARD EXAM CONCEPT/PRINCIPLE TOPNOTCH
C. hexosamidase A MD)
D. sulfatase A

TOPNOTCH MEDICAL BOARD PREP BIOCHEMISTRY SUPEREXAM Page 60 of 92


For inquiries visit www.topnotchboardprep.com.ph or email us at topnotchmedicalboardprep@gmail.com
TOPNOTCH MEDICAL BOARD PREP BIOCHEMISTRY SUPEREXAM
For inquiries visit www.topnotchboardprep.com.ph or email us at topnotchmedicalboardprep@gmail.com
Item QUESTION EXPLANATION AUTHOR TOPNOTCH
# EXAM
485 A 60-kg medical student rides the jeepney daily, spends Calorie intake for weight maintenance is as follows ANGELIS FINAL EXAM -
8 hours in class sitting down, goes home and studies for according to Schwartz: for sedentary lifestyle: 25 ANDREA AUG 2014
another 3 hours or so. What should be his total caloric kcal/kg, moderately active lifestyle: 30 kcal/kg, heavy COCOS, MD
intake daily in order for him to maintain his current activity lifestyle: 35 kcal/kg. Minus 5kcal each for (TOP 1 -
weight? weightloss and add 5kcal each for weight gain. There FEB 2014
A. 1,200 kcal were some computations like this during our exam. MED
B. 1,000 kcal Biochem lec values as follows: 30 kcal/kg sedentary, BOARDS;
C. 1,800 kcal 35 kcal/kg moderate, 40 kcal/kg heavy. TOPNOTCH
D. 2,400 kcal MD)
486 A 6-month old child was noted to have slow weight gain Strokes in the young should increase the suspicion of ANGELIS FINAL EXAM -
and mild delay in achieving developmental milestones. homocystinuria. It can cause thromboembolic ANDREA AUG 2014
One morning the parents noted that their child seems phenomena in the pulmonary and systemic arteries, COCOS, MD
unable to move the right side of his body. Which and particulary the cerebral vasculature. (TOP 1 -
condition could explain this child's presentation? FEB 2014
A. phenylketonuria MED
B. cystathioninuria BOARDS;
C. homocystinuria TOPNOTCH
D. maple syrup disease MD)

487 Which of the following is a ketogenic amino acid? SIMILAR TO PREVIOUS BOARD EXAM ANGELIS FINAL EXAM -
A. isoleucine CONCEPT/PRINCIPLE. Only leucine and lysine are ANDREA AUG 2014
B. valine purely ketogenic. Isoleucine gives rise to both COCOS, MD
C. lysine ketogenic and glucogenic fragments. (TOP 1 -
D. alanine FEB 2014
MED
BOARDS;
TOPNOTCH
MD)
488 Which of the following is FALSE regarding gene therapy? Several questions regarding gene therapy were asked ANGELIS FINAL EXAM -
A. It is a safe and effective therapy. during our exam. Although gene therapy is a ANDREA AUG 2014
B. It can be used to treat not only inherited disorders promising treatment option for a number of diseases, COCOS, MD
but certain types of cancer and viral infections as well. the technique remains risky and is still under study to (TOP 1 -
C. It is the use of DNA encoding for a functional, ensure that it is safe and effective. FEB 2014
therapeutic gene to replace a mutated gene. MED
D. Viruses have been used as a vector for gene therapy. BOARDS;
TOPNOTCH
MD)
489 Which of the following statements about the urea cycle is Urea is formed during the pathway wherein arginine ANGELIS FINAL EXAM -
FALSE? is converted to ornithine. ANDREA AUG 2014
A. Urinary urea is increased by a diet rich in protein. COCOS, MD
B. The two nitrogen atoms that are incorporated into (TOP 1 -
urea enter the cycle as ammonia and aspartate. FEB 2014
C. ATP is not required for the reaction in which MED
arginosuccinate is converted to arginine. BOARDS;
D. Urea is produced directly by the hydrolysis of TOPNOTCH
ornithine. MD)
490 Which apolipoprotein activates lecithin-cholesterol Apolipoprotein A-1 is found in HDL alone. ANGELIS FINAL EXAM -
acyltransferase (LCAT)? ANDREA AUG 2014
A. E COCOS, MD
B. A-1 (TOP 1 -
C. C-II FEB 2014
D. B-48 MED
BOARDS;
TOPNOTCH
MD)
491 Paulo ordered two-piece chicken and large fries for Pancreatic lipase is used for the degradation of dietary ANGELIS FINAL EXAM -
lunch. Which among the enzymes would increase in TG in the small intestine. LPL is for TG degradation ANDREA AUG 2014
activity? circulating in chylomicrons and VLDLs. HL is for the COCOS, MD
A. Pancreatic lipase degradation of TG remaining in IDL. Hormone- (TOP 1 -
B. Lipoprotein lipase sensitive lipase (sensitive to catecholamines, glucagon FEB 2014
C. Hepatic TG lipase etc.) is for the degradation of TG stored in adipocytes. MED
D. Hormone-sensitive lipase BOARDS;
TOPNOTCH
MD)
492 Urine test for ketones detect: This is the reason why serum ketones are more ANGELIS FINAL EXAM -
A. acetoacetate accurate than urine ketones. ANDREA AUG 2014
B. beta-hydroxybutyrate COCOS, MD
C. acetone (TOP 1 -
D. all of the above FEB 2014
MED
BOARDS;
TOPNOTCH
MD)
493 The liver is NOT able to utilize ketone bodies because of SIMILAR TO PREVIOUS BOARD EXAM ANGELIS FINAL EXAM -
the lack of the enzyme: CONCEPT/PRINCIPLE. Thiophorase is an enzyme that ANDREA AUG 2014
A. thiolase converts ketone bodies to acetyl CoA. COCOS, MD
B. thiosuccinase (TOP 1 -
C. thiophorase FEB 2014
D. Thionase MED
BOARDS;
TOPNOTCH
MD)

TOPNOTCH MEDICAL BOARD PREP BIOCHEMISTRY SUPEREXAM Page 61 of 92


For inquiries visit www.topnotchboardprep.com.ph or email us at topnotchmedicalboardprep@gmail.com
TOPNOTCH MEDICAL BOARD PREP BIOCHEMISTRY SUPEREXAM
For inquiries visit www.topnotchboardprep.com.ph or email us at topnotchmedicalboardprep@gmail.com
Item QUESTION EXPLANATION AUTHOR TOPNOTCH
# EXAM
494 Carbon monoxide poisoning inhibits which complex in Carbon monoxide, together with cyanide, hydrogen ANGELIS FINAL EXAM -
the mitochondrial electron transport chain? sulfide and azides, inhibits cytochrome c oxidase or ANDREA AUG 2014
A. Complex II complex IV. SIMILAR TO PREVIOUS BOARD EXAM COCOS, MD
B. Complex III CONCEPT/PRINCIPLE. Know the other complex (TOP 1 -
C. Complex I inhibitors. FEB 2014
D. None of the above MED
BOARDS;
TOPNOTCH
MD)
495 Biotin is used as a cofactor for the following enzymes, The rest of the choices use biotin which adds a 1- ANGELIS FINAL EXAM -
EXCEPT: carbon group to the substrate. ANDREA AUG 2014
A. Pyruvate carboxylase COCOS, MD
B. Acetyl-CoA carboxylase (TOP 1 -
C. Propionyl-CoA carboxylase FEB 2014
D. Succinyl-Coa carboxylase MED
BOARDS;
TOPNOTCH
MD)
496 A middle aged woman presents with depression, CGG is for Fragile X, GAA is for Friedrich's ataxia, CTG ANGELIS FINAL EXAM -
progressive dementia, and choreiform movements. You is for myotonic dystrophy. ANDREA AUG 2014
suspect that she is suffering from a trinucleotide repeat COCOS, MD
expansion disease which is multiple repeats of: (TOP 1 -
A. CGG FEB 2014
B. GAA MED
C. CAG BOARDS;
D. CTG TOPNOTCH
MD)
497 Which of the following post-transcription modifications All these steps happen to produce the mature ANGELIS FINAL EXAM -
take place to produce eukaryotic mRNA? eukaryotic mRNA. ANDREA AUG 2014
A. Addition of 7-methylguanosine cap on the 5' end COCOS, MD
B. Polyadenation on 3' end (TOP 1 -
C. Splicing out of introns FEB 2014
D. All of the above MED
BOARDS;
TOPNOTCH
MD)
498 Which is NOT a feature of the genetic code? The genetic code is unambiguous, degenerate (except ANGELIS FINAL EXAM -
A. Each codon specifies only 1 amino acid. methionine and tryptophan), ANDREA AUG 2014
B. Tryptophan and methionine is encoded by only 1 nonoverlapping/commaless, and universal. COCOS, MD
codon. (TOP 1 -
C. The genetic code is conserved throughout evolution. FEB 2014
D. It is overlapping. MED
BOARDS;
TOPNOTCH
MD)
499 A person at risk for all types of skin cancer would have a Nucleotide excision repair is the primary defect in ANGELIS FINAL EXAM -
defect in: xeroderma pigmentosum, which prevents repair of ANDREA AUG 2014
A. Base excision repair pyrimidine dimers because of ultraviolet light COCOS, MD
B. Mismatch repair exposure. (TOP 1 -
C. Nucleotide excision repair FEB 2014
D. Any of the above MED
BOARDS;
TOPNOTCH
MD)
500 A deficiency in this micronutrient results in Zinc deficiency results in delayed wound healing, ANGELIS FINAL EXAM -
hypogonadism and decrease in adult hair whether hypogonadism, decrease in axillary, facial and pubic ANDREA AUG 2014
axillary, facial, or pubic. hair, dysgeusia and anosmia. COCOS, MD
A. copper (TOP 1 -
B. zinc FEB 2014
C. manganese MED
D. selenium BOARDS;
TOPNOTCH
MD)
501 1. A sphingolipid which is present in high amounts in Harper's 27th ed., 126. SIMILAR TO PREVIOUS BOARD JAN BACK-UP
myelin sheath: EXAM CONCEPT/PRINCIPLE (board question, feb CHARMAIN MIDTERM
A. Sphingomyelin 2014) E EXAM AUG
B. Glycerol PALOMAR, 2014
C. Lysophosphatidylcholine MD (TOP 9
D. Cholesterol - FEB 2014
E. Phosphatidylinositol MED
BOARDS;
TOPNOTCH
MD)

TOPNOTCH MEDICAL BOARD PREP BIOCHEMISTRY SUPEREXAM Page 62 of 92


For inquiries visit www.topnotchboardprep.com.ph or email us at topnotchmedicalboardprep@gmail.com
TOPNOTCH MEDICAL BOARD PREP BIOCHEMISTRY SUPEREXAM
For inquiries visit www.topnotchboardprep.com.ph or email us at topnotchmedicalboardprep@gmail.com
Item QUESTION EXPLANATION AUTHOR TOPNOTCH
# EXAM
502 2. These vitamins are toxic in excess: There is only a limited capacity to metabolize vitamin JAN BACK-UP
A. Vitamin A and B A, and excessive intakes lead to accumulation beyond CHARMAIN MIDTERM
B. Vitamin B and C the capacity of binding proteins, so that unbound E EXAM AUG
C. Vitamin C and D vitamin A causes tissue damage. Symptoms of toxicity PALOMAR, 2014
D. Vitamin A and D affect the central nervous system (headache, nausea, MD (TOP 9
E. Vitamin D and E ataxia, and anorexia, all associated with increased - FEB 2014
cerebrospinal fluid pressure); the liver (hepatomegaly MED
with histologic changes and hyperlipidemia); calcium BOARDS;
homeostasis (thickening of the long bones, TOPNOTCH
hypercalcemia, and calcification of soft tissues); and MD)
the skin (excessive dryness, desquamation, and
alopecia).
Harper's 27th ed., 491-92.
Some infants are sensitive to intakes of vitamin D as
low as 50 g/day, resulting in an elevated plasma
concentration of calcium. This can lead to contraction
of blood vessels, high blood pressure, and calcinosis—
the calcification of soft tissues.
Harper's 27th ed., 93
503 3. The enzyme responsible for the yellow color of a SIMILAR TO PREVIOUS BOARD EXAM JAN BACK-UP
maturing hematoma: CONCEPT/PRINCIPLE. Heme oxygenase - enzyme CHARMAIN MIDTERM
A. Heme oxygenase system responsible for catabolism of heme from all of E EXAM AUG
B. Heme reductase the heme proteins PALOMAR, 2014
C. Biliverdin reductase Heme reductase MD (TOP 9
D. Glucoronosyl ttransferase Biliverdin reductase - reduces the methyne bridge - FEB 2014
E. a-aminolevulinate synthase b/w pyrrole III and pyrole IV of biliverdin to a MED
methylene group to produce bilirubin a yellow BOARDS;
pigment Katzung 27th ed., 287 TOPNOTCH
Glucoronosyl ttransferase - catalyzes conjugation of MD)
bilirubin
a-aminolevulinate synthase - rate-controlling enzyme
in porphyrin biosynthesis in mammalian liver
504 4. Major end product of nitrogen catabolism in humans: Harper's 27th ed., 249 JAN BACK-UP
A. glutamate CHARMAIN MIDTERM
B. urea E EXAM AUG
C. uric acid PALOMAR, 2014
D. ammonia MD (TOP 9
E. Creatinine - FEB 2014
MED
BOARDS;
TOPNOTCH
MD)
505 5. Most important intracellular buffer: Buffers are solutions of weak acids or bases and their JAN BACK-UP
A. bicarbonate conjugates that has the ability to resist a change in pH CHARMAIN MIDTERM
B. carbon dioxide following addition of strong acid or base. Bicarbonate E EXAM AUG
C. water is the most important extracellular buffer. Proteins PALOMAR, 2014
D. proteins are the most important intracellular buffer. MD (TOP 9
E. Phosphate - FEB 2014
MED
BOARDS;
TOPNOTCH
MD)
506 6. Which of the following statements regarding enzymes Enzymes lower the free energy of activation. JAN BACK-UP
is not true? CHARMAIN MIDTERM
A. Enzymes are protein catalysts that increase the E EXAM AUG
velocity of a chemical reaction but are not consumed in PALOMAR, 2014
the process. MD (TOP 9
B. Isoenzymes like hexokinase and glucokinase are - FEB 2014
physically distinct versions of a given enzyme that MED
catalyze the same reaction. BOARDS;
C. Enzymes do not change the energy of the reactants TOPNOTCH
and products and the equilibrium of the reaction. MD)
D. Enzymes are highly specific in the reactions they
catalyze.
E. Enzymes increase the free energy of activation.
507 7. Inhibitor of Complex III of the electron transport SIMILAR TO PREVIOUS BOARD EXAM JAN BACK-UP
chain: CONCEPT/PRINCIPLE. 3-4 questions of this type came CHARMAIN MIDTERM
A. Barbiturate and Amytal out in my boards Complex I inhibitors: E EXAM AUG
B. Dimercaprol and antimycin Barbiturate, Piercidin A, Amytal, Rotenone PALOMAR, 2014
C. Cyanide and hydrogen sulfide Complex II inhibitors - Malonate, Carboxin, TTFA
MD (TOP 9
D. Malonate and carboxin Complex III inhibitors - Antimycin A, Dimercaprol
- FEB 2014
E. Cyanide and carbon monoxide Complex IV inhibitors - Cyanide, Carbon monoxide, MED
sodium azide, hydrogen sulfide BOARDS;
TOPNOTCH
MD)
508 8. Which of the following substances increase the SIMILAR TO PREVIOUS BOARD EXAM JAN BACK-UP
permeability of the inner mitochondrial membrane to CONCEPT/PRINCIPLE. All of them are uncouplers. CHARMAIN MIDTERM
protons causing a decrease in proton gradient and an First Aid 2013, p. 102. E EXAM AUG
increase in oxygen consumption with resultant PALOMAR, 2014
production of heat but cessation of ATP synthesis? MD (TOP 9
A. 2,4-dinitrophenol - FEB 2014
B. brown fat MED
C. aspirin BOARDS;
D. all of the above TOPNOTCH
E. none of the above MD)

TOPNOTCH MEDICAL BOARD PREP BIOCHEMISTRY SUPEREXAM Page 63 of 92


For inquiries visit www.topnotchboardprep.com.ph or email us at topnotchmedicalboardprep@gmail.com
TOPNOTCH MEDICAL BOARD PREP BIOCHEMISTRY SUPEREXAM
For inquiries visit www.topnotchboardprep.com.ph or email us at topnotchmedicalboardprep@gmail.com
Item QUESTION EXPLANATION AUTHOR TOPNOTCH
# EXAM
509 9. Metabolic pathway that provides a source of NADPH HMP of Pentose phosphate pathway is an important JAN BACK-UP
required for reductive reactions and also yields ribose source of NADPH and ribose. Gluconeogenesis is CHARMAIN MIDTERM
for nucleotide synthesis and glycolytic intermediates. responsible for generation of glucose from non- E EXAM AUG
A. Hexose monophosphate shunt carbohydrate sources as glycerol and amino acid. PALOMAR, 2014
B. Gluconeogenesis Glycolysis is breakdown of carbohydrates into glucose MD (TOP 9
C. Aerobic Glycolysis molecules that can be absorbed by the body. - FEB 2014
D. Anaerobic Glycolysis Tricarboxylic acid cycle or Kreb's cycle produces 3 MED
E. Tricarboxylic acid cycle NADH, 1 FADH2, 2 CO2, and 1 GTP per acetyl-CoA. BOARDS;
First Aid 2013, p.103 TOPNOTCH
MD)
510 10. Deficiency of this enzyme results in intracellular First Aid 201, p. 105
JAN BACK-UP
accumulation of sorbitol, causing osmotic damage in the Aldose reductase sorbitol dehydrogenase
CHARMAIN MIDTERM
form of cataracts, retinopathy and peripheral Glucose → Sorbitol → Fructose
E EXAM AUG
neuropathy seen in uncontrolled diabetics : NADPH NAD+ PALOMAR, 2014
A. Aldose reductase MD (TOP 9
B. Sorbitol dehydrogenase - FEB 2014
C. Lactase MED
D. Galactokinase BOARDS;
E. Fructokinase TOPNOTCH
MD)
511 11. A newborn presents with dark ears and brown- SIMILAR TO PREVIOUS BOARD EXAM JAN BACK-UP
pigmented sclera. Mother also complains that his urine CONCEPT/PRINCIPLE. This is a case of alkaptonuria, CHARMAIN MIDTERM
turns black on exposure to air. Which enzyme is AKA ochronosis wherein there is a deficiency of E EXAM AUG
deficient? homogentisic oxidase. First Aid 2013, p.108 PALOMAR, 2014
A. Melanin synthase tyrosonase is deficient in albinism MD (TOP 9
B. Phenylalanine hydroxylase - FEB 2014
C. Homocysteine methyl transferase MED
D. Tyrosinase BOARDS;
E. Homogentisic oxidase TOPNOTCH
MD)
512 12. These are compounds that are mirror images of each SIMILAR TO PREVIOUS BOARD EXAM JAN BACK-UP
other. CONCEPT/PRINCIPLE.( 2 questions of this type) CHARMAIN MIDTERM
A. isomers Isomers - compounds that have the same chemical E EXAM AUG
B. anomers formula but different structures. Epimers are PALOMAR, 2014
C. epimers compounds that differ in the 3-d configuration around MD (TOP 9
D. enantiomers only one specific carbon atom with the exception of - FEB 2014
E. All of the above the carbonyl carbon. Enantiomers/optical MED
isomers/stereoisomers- are pairs of structures that BOARDS;
are mirror images of each other and are designated as TOPNOTCH
D- and L-sugars. Anomers are convertible between a MD)
linear and a ring form.
513 13. Which of the following is not true regarding HDL picks up cholesterol accumulating in blood JAN BACK-UP
lipoproteins? vessels (reverse cholesterol transport) to the liver and CHARMAIN MIDTERM
A. Chylomicrons transport triglyceride and cholesterol streroidogenic tissues via scavenger receptor (SR-B1). E EXAM AUG
from intestine to tissues following a meal. PALOMAR, 2014
B. LDL transport cholesterol into the cells. MD (TOP 9
C. VLDL transport endogenous triglyceride from liver - FEB 2014
to tissues. MED
D. HDL is responsible delivering cholesterol back to the BOARDS;
intestines for excretion. TOPNOTCH
E. None of the above MD)
514 14. This is the most common lysosomal storage disease SIMILAR TO PREVIOUS BOARD EXAM JAN BACK-UP
wherein there is a deficiency of glucocerebrosidase CONCEPT/PRINCIPLE. Niemann-Pick- deficient in CHARMAIN MIDTERM
enzyme. Patient may present with hepatosplenomegaly, sphingomyelinase enzyme; Taysach's- deficient in E EXAM AUG
aseptic necrosis of femur and macrophages look like hexosaminidase A; Hurler's- deficient in a-L- PALOMAR, 2014
crumpled tissue paper microscopically. iduronidase, a mucopolysaccharidoses; Hunter's - MD (TOP 9
A. Niemann-Pick disease Idoronate sulfatase deficiency - FEB 2014
B. Gaucher's disease MED
C. Taysachs disease BOARDS;
D. Hurler's syndrome TOPNOTCH
E. Hunter's syndrome MD)

515 15. Adipose tissue and muscles lacks this enzyme which Harper's 27th ed 161. In liver (and kidney) but not in JAN BACK-UP
explains why it cannot export glucose into the blood muscle, glucose 6 phosphatase hydrolyzes glucose 6- CHARMAIN MIDTERM
stream: phosphate, yielding glucose that is exported, leading E EXAM AUG
A. HMG CoA reductase to an increase in the blood glucose concentration. PALOMAR, 2014
B. Fatty acid synthetase MD (TOP 9
C. Glycogen phosphorylase - FEB 2014
D. Hexokinase MED
E. Glucose 6-phosphatase BOARDS;
TOPNOTCH
MD)
516 16. Which of the following regarding glycogen is not Harper's 27th ed., 161. In the liver the role of glycogen JAN BACK-UP
true? is to provide free glucose for export to maintain the CHARMAIN MIDTERM
A. In the muscle, the role of glycogen is to provide free blood concentration of glucose; in muscle, the role of E EXAM AUG
glucose for export to maintain the blood concetration of glycogen is to provide a source of glucose 6-phosphate PALOMAR, 2014
glucose. for glycolysis in response to the need for ATP for MD (TOP 9
B. Glycogen is the major storage carbohydrate in muscle contraction. - FEB 2014
animals. MED
C. Glycogenolysis is not the reverse of glycogenesis, but BOARDS;
is a separate pathway. TOPNOTCH
D. Insulin inhibits glycogenolysis and stimulates MD)
glycogenesis.
E. McArdle's syndrome is a glycogen storage disease
presenting with poor exercise tolerance and decreased
TOPNOTCH MEDICAL BOARD PREP BIOCHEMISTRY SUPEREXAM Page 64 of 92
For inquiries visit www.topnotchboardprep.com.ph or email us at topnotchmedicalboardprep@gmail.com
TOPNOTCH MEDICAL BOARD PREP BIOCHEMISTRY SUPEREXAM
For inquiries visit www.topnotchboardprep.com.ph or email us at topnotchmedicalboardprep@gmail.com
Item QUESTION EXPLANATION AUTHOR TOPNOTCH
# EXAM
blood lactate after exercise.

517 17. The following amino acids can be converted to SIMILAR TO PREVIOUS BOARD EXAM JAN BACK-UP
glucose except: CONCEPT/PRINCIPLE: Glycogenic Amino acids: CHARMAIN MIDTERM
A. Lysine and Isoleucine Alanine, Arginine, Asparagine, Cysteine, Glutamate, E EXAM AUG
B. Tryptophan and Tyrosine Histidine, Methionine, Proline, Serine, Threonine, PALOMAR, 2014
C. Isoleucine and Alanine Valine MD (TOP 9
D. Leucine and Lysine Ketogenic: Leucine and Lysine
- FEB 2014
E. Alanine and Arginine Glycogenic and Ketogenic: Phenylalanine, Tyrosine, MED
Isoleucine, Tryptophan (FYIW) BOARDS;
TOPNOTCH
MD)
518 18. Which of the following statements show degeneracy SIMILAR TO PREVIOUS BOARD EXAM JAN BACK-UP
in the genetic code? CONCEPT/PRINCIPLE. Degenerate/Redundant - Most CHARMAIN MIDTERM
A. Six different codons specify serine. amino acids are coded by multiple codos except E EXAM AUG
B. For any specific codon, only a single amino acid is methionine and tryptophan which are encoded by PALOMAR, 2014
indicated. only 1 codon (AUG and UGG, respectively). MD (TOP 9
C. The reading of the genetic code during the process of Unambiguous- Each codon specifies only 1 amino acid. - FEB 2014
protein synthesis does not involve any overlap of Commaless/non-overlapping- read from a fixed MED
codons. starting point as a continuous sequence of bases. BOARDS;
D. Once the reading of the genetic code is commenced Universal -genetic code is conserved. First aid 2013, TOPNOTCH
at a specific codon, the message is read in a continuous p.66. Choice E depicts wobble or the MD)
sequence of nucleotide triplets until translation stop ability of certain anti-codons to pair with codons that
codon is reached. differ at the 3rd base.
E. Codons differing only in the 3rd base positions may
code for the same tRNA/amino acid.
519 19. If the cytosine content of a double-helical DNA is Chargaff's rule: In DNA molecules the concentration of JAN BACK-UP
30% of the total bases, the thymidine content would be: deoxyadenine nucleotide equals that of thymidine CHARMAIN MIDTERM
A. 60% nucleotides (A=T), while the concentration of deoxy E EXAM AUG
B. 50% guanosine nucleotides equals that of deoxycytidine PALOMAR, 2014
C. 40% nucleotides (G=C). Harper's 27th ed., 311. MD (TOP 9
D. 30% - FEB 2014
E. 20% MED
BOARDS;
TOPNOTCH
MD)
520 20. This enzyme is responsible for the release of free SIMILAR TO PREVIOUS BOARD EXAM JAN BACK-UP
fatty acids from adipose in fasting states and is inhibitted CONCEPT/PRINCIPLE. Pancreatic lipase acts on CHARMAIN MIDTERM
by insulin. dietary TAG, Lipoprotein lipase acts on TAG E EXAM AUG
A. Pancreatic lipase chylomicrons and VLDL. Hormone sensitive TAG acts PALOMAR, 2014
B. Lipoprotein lipase on TAG stored in adipose and is activated by glucagon. MD (TOP 9
C. Hormone-sensitive lipase Lingual lipase originates from the back of the tongue. - - FEB 2014
D. Gastric lipase Biochem Topnotch handouts MED
E. Lingual lipase BOARDS;
TOPNOTCH
MD)
521 A 23-year-old male develops steatorrhea, weight loss, Celiac disease MIGUEL MIDTERM 1
and bloody diarrhea. He notes that his diarrhea is worse RAFAEL EXAM - FEB
when he eats breads or cereals. A gastroenterologist RAMOS, 2013
performs a biopsy during a colonoscopy, which reveals MD (TOP 3
celiac disease. This disorder is most directly due to - FEB 2012
which of the following? MED
A) excess lipids in the feces BOARDS;
B) deficiency of enterokinase TOPNOTCH
C) defective transport of amino acid cysteine MD)
D) hypersensitivity to the protein gluten

TOPNOTCH MEDICAL BOARD PREP BIOCHEMISTRY SUPEREXAM Page 65 of 92


For inquiries visit www.topnotchboardprep.com.ph or email us at topnotchmedicalboardprep@gmail.com
TOPNOTCH MEDICAL BOARD PREP BIOCHEMISTRY SUPEREXAM
For inquiries visit www.topnotchboardprep.com.ph or email us at topnotchmedicalboardprep@gmail.com
Item QUESTION EXPLANATION AUTHOR TOPNOTCH
# EXAM
522 A 32-year-old man with alcoholism is brought to the MIGUEL MIDTERM 1
emergency department by friends because he has been RAFAEL EXAM - FEB
unable to stand without support and has had "funny eye RAMOS, 2013
movements"; they report that he has been drinking MD (TOP 3
approximately 18 beers daily over the past month and - FEB 2012
has been increasingly confused over the past 5 days. He MED
is awake and confused but is noncombative. His speech BOARDS;
is slurred, and his breath smells of alcohol. His TOPNOTCH
temperature is 37.2 C (99 F), blood pressure is 180/60 MD)
mm Hg, pulse is 110/min, and respirations are 18/min.
Physical examination shows sixth cranial nerve palsy,
horizontal diplopia, strabismus, and an asymmetric
horizontal-gaze evoked nystagmus. Neurologic
examination shows no focal weakness or numbness.
When helped up and told to walk, he has a broad-based,
uncertain gait. When asked how he arrived at the
emergency department, he relates that "I drove to this
place to visit some friends." The most likely cause of
these findings is a deficiency of which of the following?
A) Folic acid
B) Magnesium
C) Vitamin B1 (thiamine)
D) Vitamin B12 (cyanocobalamin)
523 A 3-month-old boy is brought for a well-child MIGUEL MIDTERM 1
examination. He has poor head control. Examination RAFAEL EXAM - FEB
shows generalized hypotonia. The point of maximal RAMOS, 2013
impulse is at the left anterior axillary line. The liver edge MD (TOP 3
is palpated 4 cm below the right costal margin. The - FEB 2012
spleen is not palpable. Which of the following is the most MED
likely diagnosis? BOARDS;
A) Congenital muscular dystrophy TOPNOTCH
B) Glycogen storage disease, type II (Pompe's disease) MD)
C) GM1 gangliosidosis
D) Infant botulism

524 A 1-week-old newborn has had poor feeding, vomiting, MIGUEL MIDTERM 1
and progressive lethargy over the past 4 days. She was RAFAEL EXAM - FEB
born at term; pregnancy, labor, and delivery were RAMOS, 2013
uncomplicated, and she had no congenital anomalies. MD (TOP 3
She is being breast-fed. She has a healthy 2-year-old - FEB 2012
brother; a sister died at 10 days of age after a full-term MED
birth. Examination BOARDS;
shows decreased muscle tone and poor responsiveness; TOPNOTCH
reflexes are normal. Serum bicarbonate level is 8 MD)
mEq/L, pH is 7.15, and plasma ammonia level is 10 times
the upper limit of normal. Which of the following is the
most likely cause?
A) Mitochondrial disorder
B) Mucopolysaccharidoses disorder
C) Organic acid metabolism disorder
D) Renal tubular acidosis
525 A 1-year-old infant came to the physician due to fever. Trisomy 21 >> most common is nondisjunction in 95 MIGUEL MIDTERM 1
On physical examination, the patient has epicanthal % of cases. RAFAEL EXAM - FEB
folds, flat facial profile, macroglossia and presence of RAMOS, 2013
simian crease. Further history revealed that the patient MD (TOP 3
was born from a 48-year-old mother. What is the most - FEB 2012
common cause of this chromosal disorder? MED
A) Nondisjunction BOARDS;
B) Robertsonian translocation TOPNOTCH
C) Mosaicism MD)
D) Chromosomal deletion
526 What enzyme converts Norepinephrine to Epinephrine? MIGUEL MIDTERM 1
A) Dopamine B-hydroxylase RAFAEL EXAM - FEB
B) Dopa decarboxylase RAMOS, 2013
C) Norepinephrine hydroxylase MD (TOP 3
D) Phenylethanolamine -N-Methyl transferase - FEB 2012
MED
BOARDS;
TOPNOTCH
MD)
527 A molecule of palmitate will yield how many ATP? MIGUEL MIDTERM 1
A)12 RAFAEL EXAM - FEB
B) 38 RAMOS, 2013
C) 39 MD (TOP 3
D) 129 - FEB 2012
MED
BOARDS;
TOPNOTCH
MD)

TOPNOTCH MEDICAL BOARD PREP BIOCHEMISTRY SUPEREXAM Page 66 of 92


For inquiries visit www.topnotchboardprep.com.ph or email us at topnotchmedicalboardprep@gmail.com
TOPNOTCH MEDICAL BOARD PREP BIOCHEMISTRY SUPEREXAM
For inquiries visit www.topnotchboardprep.com.ph or email us at topnotchmedicalboardprep@gmail.com
Item QUESTION EXPLANATION AUTHOR TOPNOTCH
# EXAM
528 Typtophan is an amino acid with the largest side chain MIGUEL MIDTERM 1
containing an indole ring. It is a precursor of what RAFAEL EXAM - FEB
substances? RAMOS, 2013
A) Niacin MD (TOP 3
B) Melatonin - FEB 2012
C) Serotonin MED
D) All of the above BOARDS;
TOPNOTCH
MD)
529 Desmolase is the rate limiting enzyme in androgen and desmolase >> cholesterol to pregnenolone MIGUEL MIDTERM 1
steroid synthesis. It is responsible for what reaction? RAFAEL EXAM - FEB
A) Conversion of progesterone to 11 RAMOS, 2013
deoxycorticosterone MD (TOP 3
B) Conversion of pregnenolone to 17 - FEB 2012
hydroxypregnenolone MED
C) Conversion of cholesterol to pregnenolone BOARDS;
D) Conversion of DHEA to Androstenedione TOPNOTCH
MD)
530 A 45-year-old male is hospitalized for treatment of Familial hypercholesterolemia MIGUEL MIDTERM 1
myocardial infarction. His father and a paternal uncle RAFAEL EXAM - FEB
also had heart attacks at an early age. His cholesterol is RAMOS, 2013
elevated, and lipoprotein electrophoresis demonstrates MD (TOP 3
an abnormally high ratio of low- to high-density - FEB 2012
lipoproteins (LDL to HDL). Which of the following is the MED
most likely explanation for this problem? BOARDS;
A) Mutant LDL receptors are deficient in cholesterol TOPNOTCH
uptake MD)
B) Mutant LDL is not responding to high cholesterol
levels
C) Mutant caveolae proteins are not responding to high
cholesterol levels
D) Mutant HDL is not responding to high cholesterol
levels
531 A healthy 28-year-old woman comes to physician for liver will burn lipids to provide energy for MIGUEL MIDTERM 1
advice on losing weight. She is 150 cm ( 4ft 11in) tall and gluconeogenesis RAFAEL EXAM - FEB
56 kg (124 lb): BMI 25kg/m2. Physical examination RAMOS, 2013
reveals no other abnormalities. Physician recommends a MD (TOP 3
diet that will restrict her daily intake by 500 kilocalories. - FEB 2012
Which of following processes is most likely to increase in MED
this patient as a reuslt of following this diet? BOARDS;
A) Adipocyte glucose uptake TOPNOTCH
B) Cerebral keotne utilization MD)
C) Hepatic lipid oxidation
D) Muscle glucose uptake
532 A 4-year-old girl with a history of mental retardation is Marfan-like habitus >> homocystinuria >> MIGUEL MIDTERM 1
brought to the physician because of poor vision. Physical cystathionine synthase deficiency >> accumulated RAFAEL EXAM - FEB
examination shows long limbs, tall stature, and kyphosis. homocysteine is alternatively metabolized to RAMOS, 2013
Funduscopic examination shows bilateral lens methionine by homocysteine methyl transferase and MD (TOP 3
dislocations. Laboratory studies are most likely to show cofactor B12 - FEB 2012
a selective elevation of which of the following amino MED
acids in serum? BOARDS;
A) Cystathionine TOPNOTCH
B) Methionine MD)
C) Cysteine
D) Phenylalanine
533 A 25-year-old man eats 20 raw eggs a day to increase Raw eggs >> biotin deficiency >> biotin cofactor for MIGUEL MIDTERM 1
protein for his bodybuilding routine. This unusual diet carboxylation reactions RAFAEL EXAM - FEB
affects the activity of which of the following enzymes? RAMOS, 2013
A) Acetyl-CoA carboxylase MD (TOP 3
B) Acyl-CoA dehydrogenase - FEB 2012
C) Fatty acid synthase MED
D) Fatty acid thiokinase BOARDS;
TOPNOTCH
MD)
534 A 44 year old alcoholic was brought to the emergency Glucokinase >> glucose storage >> fbs is 73 so you MIGUEL MIDTERM 1
department by his friends. During their usual gathering don’t need storage RAFAEL EXAM - FEB
at the local bar, he passed out and his friends were RAMOS, 2013
unable to revive him. The physician ordered an injection MD (TOP 3
of Thiamine , followed by overnight parenteral glucose. - FEB 2012
The next morning the patient was alert and coherent, MED
serum thiamine was normal, and blood glucose was 73 BOARDS;
mg/dl (4mM). The patient was then sent home. At the TOPNOTCH
time of discharge from the hospital, which of the the MD)
following proteins would have no significant physiologic
activity in this patient?
A) Malate Dehydrogenase
B) Glucokinase
C) alpha-ketoglutarage dehydrogenase
D) GLUT 1 transporter

TOPNOTCH MEDICAL BOARD PREP BIOCHEMISTRY SUPEREXAM Page 67 of 92


For inquiries visit www.topnotchboardprep.com.ph or email us at topnotchmedicalboardprep@gmail.com
TOPNOTCH MEDICAL BOARD PREP BIOCHEMISTRY SUPEREXAM
For inquiries visit www.topnotchboardprep.com.ph or email us at topnotchmedicalboardprep@gmail.com
Item QUESTION EXPLANATION AUTHOR TOPNOTCH
# EXAM
535 Some humans are unable to generate NADPH from MIGUEL MIDTERM 1
glucose metabolism but are able to synthesize ribose RAFAEL EXAM - FEB
from fructose-6-phosphate. Which of the following RAMOS, 2013
enzymes are essential? MD (TOP 3
A) G6PD - FEB 2012
B) Glutathione reductase MED
C) Enolase BOARDS;
D) Transketolase TOPNOTCH
MD)
536 Okazaki fragments are formed on the ______ strands and MIGUEL MIDTERM 1
in the ______ direction. RAFAEL EXAM - FEB
A) Leading; 3’-5’ RAMOS, 2013
B) Lagging; 3’-5’ MD (TOP 3
C) Leading; 5’-3’ - FEB 2012
D) Lagging; 5’-3’ MED
BOARDS;
TOPNOTCH
MD)
537 Two couples present to the ER with severe nausea, a-amanitin from Amanita phylloides >> inhibits RNA MIGUEL MIDTERM 1
vomiting, and diarrhea. One of the patients admits that polymerase II RAFAEL EXAM - FEB
she served salad at the dinner party to which she had RAMOS, 2013
added a few mushrooms that she picked outside. With MD (TOP 3
such information, it is likely that their symptoms are a - FEB 2012
result of inhibition of what molecular event? MED
A) RNA polymerase II BOARDS;
B) RNA polymerase I TOPNOTCH
C) RNA splicing MD)
D) RNA polymerase III
538 Which of the following is not a characteristic of the one amino acid is translated by a codon = MIGUEL MIDTERM 1
genetic code? unambiguous; degenerate = one amino acid may be RAFAEL EXAM - FEB
A) It is a triplet code generated by more than one codon RAMOS, 2013
B) It is nonoverlapping and without punctuation MD (TOP 3
C) It is degenerate in that one codon may code for more - FEB 2012
than one amino acid MED
D) It is universal BOARDS;
TOPNOTCH
MD)
539 A 4-month-old boy is being evaluated for seizures, pyruvate carboxylase >> converts pyruvate to OAA >> MIGUEL MIDTERM 1
psychomotor retardation, and hypotonia. Work-up enters TCA or gluconeogenesis RAFAEL EXAM - FEB
reveals elevated serum levels of lactate alanine, RAMOS, 2013
pyruvate, and ketoacids. Based on the clinical MD (TOP 3
presentation, pyruvate carboxylase activity is measured - FEB 2012
using fibroblasts from a skin biopsy and is found to be MED
markedly decreased. This enzyme is normally used to BOARDS;
directly synthesize which of the following molecules? TOPNOTCH
A) Pyruvate MD)
B) Oxaloacetate
C) Malate
D) Acetyl CoA

540 In glycogen, glucose residues form a straight chain via MIGUEL MIDTERM 1
which of the following? RAFAEL EXAM - FEB
A) a-1,4 linkages RAMOS, 2013
B) a-1.6 linkages MD (TOP 3
C) a-1,4 linkages with glycogenin at the non-reducing - FEB 2012
end MED
D) a-1,6 linkages with UDP-glucose at any end BOARDS;
TOPNOTCH
MD)
541 A 65/M, heavy smoker, presented with a 5 month history In complete obstruction of the bile duct, no ABDELSIM FINAL EXAM -
of vague epigastric pain and nausea; associated with urobilinogen is found in the urine, since bilirubin has AR OMAR FEB 2014
weight loss and pruritus. Lately, patient has also started no access to the intestine, where it can be converted to II, MD (TOP
to experience back pain and appeared jaundiced. On PE, urobilinogen. In this case, the presence of bilirubin 2 - AUG
you note icteric conjunctivae and epigastric tenderness. (conjugated) in the urine without urobilinogen 2013 MED
An abdominal ultrasound reveals a pancreatic head suggests obstructive jaundice, either intrahepatic or BOARDS;
mass. Expected results of biochemical testing include the posthepatic. The commonest cause of obstructive TOPNOTCH
following except: (posthepatic) jaundice are cancer of the head of the MD - 200
A. Increased direct bilirubin pancreas and a gallstone lodged in the common bile QUESTION
B. Present urine bilirubin duct. (Harpers 26e p.284) S) AND
C. Trace fecal urobilinogen MARC
D. Increased total bilirubin DENVER
E. Increased urine urobilinogen TIONGSON,
MD (40
QUESTION
S)

TOPNOTCH MEDICAL BOARD PREP BIOCHEMISTRY SUPEREXAM Page 68 of 92


For inquiries visit www.topnotchboardprep.com.ph or email us at topnotchmedicalboardprep@gmail.com
TOPNOTCH MEDICAL BOARD PREP BIOCHEMISTRY SUPEREXAM
For inquiries visit www.topnotchboardprep.com.ph or email us at topnotchmedicalboardprep@gmail.com
Item QUESTION EXPLANATION AUTHOR TOPNOTCH
# EXAM
542 The above patient was noted to lose 10 kg in 5 months. A - C describe the three mechanisms by which cancer ABDELSIM FINAL EXAM -
On PE, you note obvious wasting. Which of the following causes cachexia. In cachexia, protein catabolism is AR OMAR FEB 2014
is NOT true regarding the biochemical and metabolic INCREASED; this differentiates it from marasmus in II, MD (TOP
changes in cancer cachexia? which protein synthesis is reduced but catabolism is 2 - AUG
A. The tumor releases lactate which is recycled into unaffected (Harpers p.479). 2013 MED
glucose consuming 6 ATPs / glucose recycles. BOARDS;
B. There is upregulation of uncoupling proteins, leading TOPNOTCH
to thermogenesis and oxidation of fuels. MD - 200
C. There is futile cycling of lipids. Hormone sensitive QUESTION
lipase is activated which breaks down triglycerides to S) AND
FFA, which ae then re-esterified at ATP cost. MARC
D. In cachexia, protein synthesis is reduced but DENVER
catabolism is unaffected. TIONGSON,
E. None of the above MD (40
QUESTION
S)
543 Which of the following immunoglobulins opsonizes Main antibody in the secondary response. Also fixes ABDELSIM FINAL EXAM -
bacteria making them easier to phagocytose? complement, which enhances bacterial killing, and AR OMAR FEB 2014
A. IgE neutralizes bacterial toxins and virus (Harper's p.594) II, MD (TOP
B. IgM 2 - AUG
C. IgG 2013 MED
D. IgA BOARDS;
E. IgD TOPNOTCH
MD - 200
QUESTION
S) AND
MARC
DENVER
TIONGSON,
MD (40
QUESTION
S)
544 A 24/M came in for pre-employment examination. On Patient has Marfan's syndrome. ABDELSIM FINAL EXAM -
PE, you note that the patient is over the 95th percentile AR OMAR FEB 2014
for height; and he has a slight scoliosis, skin striae and II, MD (TOP
pectus excavatum. Ausculation reveals a heart murmur. 2 - AUG
You then order a 2D echo which shows an enlarged 2013 MED
aortic root, aortic valve regurgitation and mitral valve BOARDS;
prolapse. You suspect a genetic condition in which there TOPNOTCH
is abnormal production of a glycoprotein, leading to MD - 200
abnormalities in the mechanical stability and elastic QUESTION
properties of connective tissue. Production of which of S) AND
the following glycoproteins is defective? MARC
A. Elastin DENVER
B. Collagen TIONGSON,
C. Fibronectin MD (40
D. Fibrillin QUESTION
E. Laminin S)
545 A neonate is admited to the intensive neonatal care and Haptoglobin was asked during the last board exam. ABDELSIM FINAL EXAM -
incubated due to jaundice. He was born to a 30-year old This protein prevents loss of free hemoglobin into the AR OMAR FEB 2014
G3P2 who is known to be Rh-negative with an Rh- kidney; conserving Fe present in Hg. Levels are II, MD (TOP
positive sexual partner. The mother has not received decreased in hemolytic anemia because of increased 2 - AUG
anti-D prophylaxis during the current pregnancy. You free Hg levels (Hg-Hp complex has a short half life of 2013 MED
are suspecting Rh incompatibility. Which of the 90 mins vs 5 days for normal halptoglobin). Increased BOARDS;
following serum proteins is expected to be decreased in levels are seen in chronic inflammatory states TOPNOTCH
the neonate? (haptoglobin is an acute phase reactant) MD - 200
A. a1-antitrypsin QUESTION
B. C reactive protein S) AND
C. Haptoglobin MARC
D. Ceruloplasmin DENVER
E. a2-macroglobulin TIONGSON,
MD (40
QUESTION
S)
546 A 78/M, car mechanic, presented with a 24 hour history MNEMONIC: CO (carbon monoxide) inhibits CO ABDELSIM FINAL EXAM -
of progressive headache, vomiting and dizziness. On (cytochrome oxidase / complex IV). AR OMAR FEB 2014
examination, he is awake, but appears confused and has II, MD (TOP
generalized weakness. His carboxyhemoglobin level is 2 - AUG
18%. You diagnose the patient as having carbon 2013 MED
monoxide poisoning. Carbon monoxide is dangerous BOARDS;
because it is a potent inhibitor of which complex in the TOPNOTCH
electron transport chain. MD - 200
A. NADH dehydrogenase QUESTION
B. Oxygen dehydrogenase S) AND
C. Succinate dehydrogenase MARC
D. Ubiquinol:ferricytochrome oxidoreductase DENVER
E. Cytochrome oxidase TIONGSON,
MD (40
QUESTION
S)

TOPNOTCH MEDICAL BOARD PREP BIOCHEMISTRY SUPEREXAM Page 69 of 92


For inquiries visit www.topnotchboardprep.com.ph or email us at topnotchmedicalboardprep@gmail.com
TOPNOTCH MEDICAL BOARD PREP BIOCHEMISTRY SUPEREXAM
For inquiries visit www.topnotchboardprep.com.ph or email us at topnotchmedicalboardprep@gmail.com
Item QUESTION EXPLANATION AUTHOR TOPNOTCH
# EXAM
547 A 24/F who came in for dysuria and frequency was Patient has G6P deficiency. G6PD is the key enzyme in ABDELSIM FINAL EXAM -
prescribed cotrimoxazole. Two days later, she develops the oxidative phase of PPP. AR OMAR FEB 2014
nausea and exhaustion and comes back to the ER after II, MD (TOP
passing "tea-colored" urine. She was also noted to be 2 - AUG
jaundiced. Work-ups reveal that the patient has anemia, 2013 MED
azotemia, deranged liver function and unconjugated BOARDS;
hyperbilirubinemia. She is diagnosed as having drug- TOPNOTCH
induced hemolytic anemia. You suspect that the patient MD - 200
has an inherited condition that makes her unable to QUESTION
tolerate oxidative stress leading to red cell hemolysis. S) AND
This condition is due to a deficiency in the key enzyme in MARC
which of the following biochemical reactions: DENVER
A. Glycolysis TIONGSON,
B. Tricarboxylic acid cycle MD (40
C. Gluconeogenesis QUESTION
D. Serial oxidative phosphorylation in the electron S)
transport chain
E. Pentose phosphate pathway
548 Sorbitol dehydrogenase is an enzyme which converts Remember that fructose is used by semen as fuel. ABDELSIM FINAL EXAM -
sorbitol to fructose. Aside from the liver, this enzyme is AR OMAR FEB 2014
also found in significant quantities in the: II, MD (TOP
A. Ovaries 2 - AUG
B. Retina 2013 MED
C. Schwann cells BOARDS;
D. Pancreas TOPNOTCH
E. Seminal vesicles MD - 200
QUESTION
S) AND
MARC
DENVER
TIONGSON,
MD (40
QUESTION
S)
549 Which of the following dyslipoproteinemia is a condition Remember that ApoA is the major lipoprotein of HDL ABDELSIM FINAL EXAM -
apparently beneficial to health and longevity? aka good cholesterol. In hyperalphalipoproteinemia, AR OMAR FEB 2014
A. Familial dysbetalipoproteinemia (ApoA = alpha) there is increased concentration of II, MD (TOP
B. Familial hypertriacylglycerolemia HDL, conferring longevity to the lucky patient. 2 - AUG
C. Familial hyperalphalipoproteinemia 2013 MED
D. Familial type III hyperlipoproteinemia BOARDS;
E. All increase risk of atherosclerosis and coronary TOPNOTCH
disease. MD - 200
QUESTION
S) AND
MARC
DENVER
TIONGSON,
MD (40
QUESTION
S)
550 Collagen is rich in which of the following amino acids: Proline facilitates kinking. ABDELSIM FINAL EXAM -
A. Lysine AR OMAR FEB 2014
B. Serine II, MD (TOP
C. Threonine 2 - AUG
D. Leucine 2013 MED
E. Proline BOARDS;
TOPNOTCH
MD - 200
QUESTION
S) AND
MARC
DENVER
TIONGSON,
MD (40
QUESTION
S)
551 A newborn presenting with vomiting and hypotonia has This is the most common biochemical cause of ABDELSIM FINAL EXAM -
been found to have severe lactic acidosis. Serum analysis congenital lactic acidosis. Because enzyme is deficient, AR OMAR FEB 2014
reveals elevated lactate and alanine. You are considering pyruvate is shunted towards lactate production. II, MD (TOP
a biochemical condition characterized by a deficiency in: Patients present with lactic acidosis and neurologic 2 - AUG
A. Muscle phosphofructokinase disturbance. Alanine increases from conversion of 2013 MED
B. Pyruvate kinase pyruvate to alanine by alanine aminotransferase. BOARDS;
C. Pyruvate carboxylase TOPNOTCH
D. Pyruvate dehydrogenase MD - 200
E. Lactate dehydrogenase QUESTION
S) AND
MARC
DENVER
TIONGSON,
MD (40
QUESTION
S)

TOPNOTCH MEDICAL BOARD PREP BIOCHEMISTRY SUPEREXAM Page 70 of 92


For inquiries visit www.topnotchboardprep.com.ph or email us at topnotchmedicalboardprep@gmail.com
TOPNOTCH MEDICAL BOARD PREP BIOCHEMISTRY SUPEREXAM
For inquiries visit www.topnotchboardprep.com.ph or email us at topnotchmedicalboardprep@gmail.com
Item QUESTION EXPLANATION AUTHOR TOPNOTCH
# EXAM
552 An 8 month old male of Ashkenazi Jewish heritage was Patient has Tay-Sach's disease (hexosaminidase A ABDELSIM FINAL EXAM -
referred to you for developmental delay. Parents noted deficiency) characterized by mental retardation, AR OMAR FEB 2014
that by 7 months, he had poor head control and blindness and muscular weakness. Buzz phrase: II, MD (TOP
remained inattentive to surroundings. Parents also cherry red spot in the macula. 2 - AUG
noted unusual eye movements and staring episodes. The 2013 MED
patient was already referred to an ophthalmologist who BOARDS;
observed a "cherry-red spot" within a pale macula. You TOPNOTCH
suspect that the patient has an inherited deificency in MD - 200
which of the following enzymes: QUESTION
A. Sphingomyelinase S) AND
B. Arylsulfatase A MARC
C. Hexosaminidase A DENVER
D. B-galactosidase TIONGSON,
E. B-glucosidase MD (40
QUESTION
S)
553 Signs and symptoms similar to pellagra are seen in Pellagra is due to niacin deficiency. Remember that ABDELSIM FINAL EXAM -
Hartnup disease due to impaired transport of this amino tryptophan is required for niacin synthesis. AR OMAR FEB 2014
acid: II, MD (TOP
A. Tyrosine 2 - AUG
B. Threonine 2013 MED
C. Tryptophan BOARDS;
D. Valine TOPNOTCH
E. Cystine MD - 200
QUESTION
S) AND
MARC
DENVER
TIONGSON,
MD (40
QUESTION
S)
554 A newborn is seen with blisters and erosions covering Patient has epidermolysis bullosa. ABDELSIM FINAL EXAM -
the entire body. It was noted that handling of the baby AR OMAR FEB 2014
easily causes tearing of the skin. You suspect an II, MD (TOP
inherited condition characterized by mutations affecting 2 - AUG
a protein which forms delicate fibrils that anchor the 2013 MED
basal lamina to the dermis. Which protein is abnormally BOARDS;
produced in this condition? TOPNOTCH
A. Type I collagen MD - 200
B. Type II collagen QUESTION
C. Type III collagen S) AND
D. Type IV collagen MARC
E. Type VII collagen DENVER
TIONGSON,
MD (40
QUESTION
S)
555 A 28/F, whose partner was recently diagnosed with HIV- ABDELSIM FINAL EXAM -
AIDS, comes in and requests to be tested for the AR OMAR FEB 2014
presence of HIV. To confirm an HIV infection, you would II, MD (TOP
need to order a test which employs: 2 - AUG
A. Enzyme linked immunoassay 2013 MED
B. Genomic outhern blot BOARDS;
C. Northern blot TOPNOTCH
D. Western blot MD - 200
E. VNTR analysis QUESTION
S) AND
MARC
DENVER
TIONGSON,
MD (40
QUESTION
S)
556 A 20/M comes in for knee pain and swelling. On PE, you Patient has alkaptonuria due to deficiency in ABDELSIM FINAL EXAM -
noted dark spots in his sclera. On probing, the patient homogentisate oxidase. Homogentisate is a product of AR OMAR FEB 2014
reports that when he leaves his urine standing, it turns tyrosine catabolism. Oxidation of homogentisate leads II, MD (TOP
black. You suspect an inherited condition characterized to darkening of the urine. Oxidation of homogentisate 2 - AUG
by a deficiency in an enzyme necessary for the to benzoquinone acetate poiymerizes and binds to 2013 MED
catabolism of this amino acid. connective tissue, leading to arthritis and BOARDS;
A. Tyrosine pigmentation (ochronosis). TOPNOTCH
B. Threonine MD - 200
C. Leucine QUESTION
D. Serine S) AND
E. Glycine MARC
DENVER
TIONGSON,
MD (40
QUESTION
S)

TOPNOTCH MEDICAL BOARD PREP BIOCHEMISTRY SUPEREXAM Page 71 of 92


For inquiries visit www.topnotchboardprep.com.ph or email us at topnotchmedicalboardprep@gmail.com
TOPNOTCH MEDICAL BOARD PREP BIOCHEMISTRY SUPEREXAM
For inquiries visit www.topnotchboardprep.com.ph or email us at topnotchmedicalboardprep@gmail.com
Item QUESTION EXPLANATION AUTHOR TOPNOTCH
# EXAM
557 A 28/M, with a history of recurrent nephrolithiasis, Cystinuria ABDELSIM FINAL EXAM -
comes in due to severe intermittent right flank pain and No transporter for COAL AR OMAR FEB 2014
hematuria. On examination, you note CVA tenderness on Cystine, Ornithine, Arginine, Lysine II, MD (TOP
the right. Urinalysis reveals hexagonal crystals upon 2 - AUG
cooling of acidified urine sediment. The patient tells you 2013 MED
that a physician who saw him for a previous bout of BOARDS;
nephrolithiasis was considering an inherited condition. TOPNOTCH
You suspect that the patient may have a condition MD - 200
characterized by impaired renal tubular absorption of QUESTION
which of the following amino acids: S) AND
A. Ornithine MARC
B. Arginine DENVER
C. Lysine TIONGSON,
D. Cystine MD (40
E. All of the above QUESTION
S)
558 Too much of a good thing can be bad for you. Excessive ABDELSIM FINAL EXAM -
ingestion of this vitamin for example can cause headache AR OMAR FEB 2014
and nasuea related to increased CSF, hepatomegaly, II, MD (TOP
thickening of long bones and hypercalcemia, and 2 - AUG
excessive dryness and desquamation of skin. 2013 MED
A. Vitamin A BOARDS;
B. Vitamin D TOPNOTCH
C. Vitamin E MD - 200
D. Vitamin K QUESTION
E. Niacin S) AND
MARC
DENVER
TIONGSON,
MD (40
QUESTION
S)
559 Activation of erythrocyte transaminases is an accepted The question could be rephrased as such: 'which of ABDELSIM FINAL EXAM -
index of nutritionals status of this vitamin: the following vitamins is a co-factor in transaminase AR OMAR FEB 2014
A. Thiamine reactions?' Answer would be pyridoxine. II, MD (TOP
B. Pyridoxine 2 - AUG
C. Riboflavin 2013 MED
D. Biotin BOARDS;
E. Folic acid TOPNOTCH
MD - 200
QUESTION
S) AND
MARC
DENVER
TIONGSON,
MD (40
QUESTION
S)
560 It has been common practice to prescribe multivitamins ABDELSIM FINAL EXAM -
for patients with peripheral neuropathy, complaining of AR OMAR FEB 2014
"pangangalay." However, the use of multivitamins for II, MD (TOP
this indication is not evidence-based. Moreover, 2 - AUG
excessive intake of this vitamin is commonly associated 2013 MED
with sensory neuropathy. BOARDS;
A. Thiamine TOPNOTCH
B. Pyridoxine MD - 200
C. Riboflavin QUESTION
D. Biotin S) AND
E. Folic acid MARC
DENVER
TIONGSON,
MD (40
QUESTION
S)
561 1. The main source of Estrogen in a 27 year old Estriol is only produced in significant amounts during BLAKE FINAL EXAM -
primigravid patient is from: pregnancy as it is made by the placenta from 16-OH WARREN FEB 2014
A. placenta DHEAS, an androgen steroid made in the fetal liver ANG, MD
B. Maternal adrenals and adrenal glands (TOP 1 -
C. fetus AUG 2013
D. Peripheral aromatization of DHEA MED
BOARDS;
TOPNOTCH
MD)
562 Hemoglobin is a functional protein belonging to what Quaternary structures are composed of at least 2 BLAKE FINAL EXAM -
structural order? polypeptide chains. Hemoglobin is composed of 2 WARREN FEB 2014
A. primary alpha and 2 beta chains and is therefore quaternary in ANG, MD
B. secondary nature. (TOP 1 -
C. tertiary AUG 2013
D. quaternary MED
BOARDS;
TOPNOTCH
MD)

TOPNOTCH MEDICAL BOARD PREP BIOCHEMISTRY SUPEREXAM Page 72 of 92


For inquiries visit www.topnotchboardprep.com.ph or email us at topnotchmedicalboardprep@gmail.com
TOPNOTCH MEDICAL BOARD PREP BIOCHEMISTRY SUPEREXAM
For inquiries visit www.topnotchboardprep.com.ph or email us at topnotchmedicalboardprep@gmail.com
Item QUESTION EXPLANATION AUTHOR TOPNOTCH
# EXAM
563 This amino acid is considered the most basic among the BLAKE FINAL EXAM -
group: WARREN FEB 2014
A. Histidine ANG, MD
B. Lysine (TOP 1 -
C. Arginine AUG 2013
D. Glycine MED
BOARDS;
TOPNOTCH
MD)
564 The major inhibitory neurotransmitter of the Brain is Glutamate, the major stimulant in the CNS is BLAKE FINAL EXAM -
derived from a decarboxylation reaction requiring consequently carboxylated by L-glutamic WARREN FEB 2014
pyridoxal phosphate. The amino acid decarboxylase into GABA, the major inhibitory ANG, MD
A. Glutamine transmitter of the CNS (TOP 1 -
B. Glycine AUG 2013
C. Glutamate MED
D. Lysine. BOARDS;
TOPNOTCH
MD)
565 In alpha thalassemia (-/-, -/-) the hemoglobin formed In Alpha thalassemia, accumulation of 4 beta chains is BLAKE FINAL EXAM -
composed of B4 (beta4) is referred to as : referred to as HbH, whereas 4 gamma chains is WARREN FEB 2014
A. HbH referred to as Hb Bart’s. Both can exist in this ANG, MD
B. HbS condition. (TOP 1 -
C. Hb Barts AUG 2013
D. Hb C MED
BOARDS;
TOPNOTCH
MD)
566 True of Noncompetitive inhibition In noncompetitive inhibition, formation of the product BLAKE FINAL EXAM -
a. Formation of product continues continues. Allosteric inhibition causes the Vmax to WARREN FEB 2014
b. The Km of the enzyme targeted decreases decrease. However, Km remains unchanged. ANG, MD
c. The Km of the enzyme targeted increases (TOP 1 -
d. The Vmax of the enzyme targeted remains unchanged AUG 2013
MED
BOARDS;
TOPNOTCH
MD)
567 Gibbs free energy predicts the spontaneity of a reaction The formula of Gibbs free energy is enthalpy minus BLAKE FINAL EXAM -
that is about to take place. The factors to be considered temperature x entropy WARREN FEB 2014
are the following except: ANG, MD
a. enthalpy (TOP 1 -
b. temperature AUG 2013
c. entropy MED
d. none of the above BOARDS;
TOPNOTCH
MD)
568 A Gibbs free energy change of 0 means that the reaction BLAKE FINAL EXAM -
is: WARREN FEB 2014
a. Spontaneously occurring Zero Gibbs free energy change means that the reaction ANG, MD
b. Non-spontaneous that requires coupling with an is in equilibrium (TOP 1 -
exothermic reaction AUG 2013
c. In equilibrium MED
d. No significance BOARDS;
TOPNOTCH
MD)
569 beta oxidation of palmitic acid into acetyl coA residues BLAKE FINAL EXAM -
occur in which site of the cell? WARREN FEB 2014
a. cytosol ANG, MD
b. peroxisome (TOP 1 -
c. mitochondrial matrix AUG 2013
d. both a and c MED
BOARDS;
TOPNOTCH
MD)

570 terminal digestion of carbohydrates into Terminal digestion occurs in the brush borders due to BLAKE FINAL EXAM -
monosaccharides occur in which site of the the presence of disaccharidases WARREN FEB 2014
gastrointestinal tract? ANG, MD
a. brush border of the intestines (TOP 1 -
b. duodenal lumen when acted upon by pancreatic AUG 2013
enzymes MED
c. enterocytes BOARDS;
d. in the mouth by the enzyme ptyalin TOPNOTCH
MD)
571 which of the following is true of phosphatidylinositol? PI is synthesized from free inositol and CDP-DAG. It is BLAKE FINAL EXAM -
a. synthesized from free inositol and CDP-diacylglycerol an unusual phospholipid in that it contains stearic WARREN FEB 2014
b. it is an unusual phospholipid in that it contains acid on C-1 and Arachidonic acid on C-2. It serves as a ANG, MD
arachidonic acid in Carbon 1 and stearic acid on carbon 2 reservoir for prostaglandin synthesis. (TOP 1 -
c. the arachidonic acid present is bound and unaccessible AUG 2013
for prostaglandin synthesis MED
d. all of the above BOARDS;
TOPNOTCH
MD)

TOPNOTCH MEDICAL BOARD PREP BIOCHEMISTRY SUPEREXAM Page 73 of 92


For inquiries visit www.topnotchboardprep.com.ph or email us at topnotchmedicalboardprep@gmail.com
TOPNOTCH MEDICAL BOARD PREP BIOCHEMISTRY SUPEREXAM
For inquiries visit www.topnotchboardprep.com.ph or email us at topnotchmedicalboardprep@gmail.com
Item QUESTION EXPLANATION AUTHOR TOPNOTCH
# EXAM
572 which component of phosphatidylinositol degradation of The products IP3 and DAG mediate the mobilization of BLAKE FINAL EXAM -
the secondary messenger complex is responsible for intracellular Ca and the activation of Protein kinase C, WARREN FEB 2014
Protein kinase C activation? respectively. ANG, MD
a. Diacylglycerol (TOP 1 -
b. IP3 AUG 2013
c. PHosphatidylinositol 4,5 diphosphate MED
d. calcium BOARDS;
TOPNOTCH
MD)
573 the irreversible inhibition of cyclooxygenase enzyme Acetyl salicylate causes acetylation of the COX enzyme BLAKE FINAL EXAM -
system in platelets by aspirin is by virtue of: rendering it ineffective permanently. WARREN FEB 2014
a. acetylation ANG, MD
b. hydroxylation (TOP 1 -
c. sulfation AUG 2013
d. conjugation MED
BOARDS;
TOPNOTCH
MD)
574 which among the following is considered a primary bile Two primary bile acids are: chenodeoxycholic and BLAKE FINAL EXAM -
acid cholic acid. WARREN FEB 2014
a. lithocolic acid ANG, MD
b. cholic acid (TOP 1 -
c. deoxycholic acid AUG 2013
d. ursodeoxycholic acid MED
BOARDS;
TOPNOTCH
MD)
575 the most common enzyme deficiency in Congenital 21-hydroxylase deficiency BLAKE FINAL EXAM -
Adrenal hyperplasia is: WARREN FEB 2014
a. 11-hydroxylase ANG, MD
b. 17- hydroxylase (TOP 1 -
c. 21-hydroxylase AUG 2013
d. Aromatase MED
BOARDS;
TOPNOTCH
MD)
576 Deficiency of this vitamin causes hemolytic anemia: Vitamin E deficiency causes hemolytic anemia; on the BLAKE FINAL EXAM -
a. Vitamin A other hand, excess Vitamin K may also cause WARREN FEB 2014
b. vitamin D hemolysis. ANG, MD
c. vitamin E (TOP 1 -
d. vitamin K AUG 2013
MED
BOARDS;
TOPNOTCH
MD)
577 Warfarin inhibits vitamin K dependent clotting factors Vitamin K Epoxide reductase inhibition is the main BLAKE FINAL EXAM -
by direct inhibition of: mechanism of Warfarin WARREN FEB 2014
a. vitamin K epoxide reductase ANG, MD
b. glutamyl carboxylase (TOP 1 -
c. Lipid absorption of vitamin K in the GI tract and the AUG 2013
liver MED
d. protein synthesis of factors 2,7,9,10 BOARDS;
TOPNOTCH
MD)
578 a disorder of purine metabolism characterized by Lysch Nyhan Syndrome is due to the deficiency of the BLAKE FINAL EXAM -
hyperuricemia, self-mutilation, and neurologic features Purine Salvage Mechanism caused by HGPRT WARREN FEB 2014
with involuntary movements is mainly due to a defect of deficiency. ANG, MD
what enzyme? (TOP 1 -
a. glucose 6-phosphatase AUG 2013
b. HGPRT MED
c. Adenine phosphoribosyl transferase BOARDS;
d. ribonucleotide reductase TOPNOTCH
MD)

579 Adenosine deaminase deficiency causes this disorder BLAKE FINAL EXAM -
among children characterized by both Tcell and Bcell WARREN FEB 2014
dysfunction: ANG, MD
a. Severe combined immunodeficiency (TOP 1 -
b. combined variable immunodeficiency AUG 2013
c. DiGeorge syndrome MED
d. X-linked agammaglobulinemia BOARDS;
TOPNOTCH
MD)
580 which among the following amino acids does not Valine does not contribute any carbon to the purine BLAKE FINAL EXAM -
contribute directly to the synthesis of a purine ring? ring. WARREN FEB 2014
a. glycine ANG, MD
b. glutamine (TOP 1 -
c. aspartate AUG 2013
d. valine MED
BOARDS;
TOPNOTCH
MD)

TOPNOTCH MEDICAL BOARD PREP BIOCHEMISTRY SUPEREXAM Page 74 of 92


For inquiries visit www.topnotchboardprep.com.ph or email us at topnotchmedicalboardprep@gmail.com
TOPNOTCH MEDICAL BOARD PREP BIOCHEMISTRY SUPEREXAM
For inquiries visit www.topnotchboardprep.com.ph or email us at topnotchmedicalboardprep@gmail.com
Item QUESTION EXPLANATION AUTHOR TOPNOTCH
# EXAM
581 Which compounds act as inhibitors for Cytochrome All compounds are inhibitors of cytochrome oxidase. TIMOTHY MIDTERM 1
Oxidase (Complex IV) of the Electron Transport Chain? TANG LEE EXAM - FEB
A. CO SAY, MD 2014
B. CN (TOP 4 -
C. H2S AUG 2013
D. All of the Above MED
E. None of the Above BOARDS;
TOPNOTCH
MD)
582 Which irreversible enzymes in glycolysis is coupled to Although all 3 enzymes catalyzed irreversible steps in TIMOTHY MIDTERM 1
the release of energy and requires different enzymes to glycolysis, only pyruvate kinase is exergonic or is TANG LEE EXAM - FEB
catalyzed the reverse reactions in the pathway of coupled to the release of energy. Hexokinase and PFK SAY, MD 2014
gluconeogenesis? are in the energy investment stage of the Glycolysis. (TOP 4 -
A. Hexokinase AUG 2013
B. Phosphofructokinase MED
C. Pyruvate Kinase BOARDS;
D. All of the above TOPNOTCH
E. None of the above MD)

583 The reaction rates of adding substance A in a solution Remember the equation y=mx+b where m is the slope TIMOTHY MIDTERM 1
with known enzyme kinetics can be exemplified using of the line and b is the y-intercept (ordinate). TANG LEE EXAM - FEB
the Lineweaver-Burk Plot (1/Vi=(Km/Vmax)(1/S) + Increasing the ordinate will result in a decrease in SAY, MD 2014
1/Vmax). When adding substance A, the ordinate of the Vmax (since we get the reciprocal of Vmax as the (TOP 4 -
Lineweaver-Burk plot increases while the abscissa ordinate). A decrease in Vmax while Km (-1/Km is the AUG 2013
remains the same. Substance A can then be classified as abscissa or x-intercept) remains the same is MED
a? characteristic of Non-competitive inhibition. BOARDS;
A. Complete Agonist TOPNOTCH
B. Partial Agonist MD)
C. Competitive Antagonist
D. Non-competitive Antagonist
E. Uncompetitive Antagonist

584 In the intestinal brush border which monosaccaharide All monosaccharides can use the GLUT-5 transporter TIMOTHY MIDTERM 1
can only used the GLUT-5 (facilitated diffusion) and not while only aldoses can use the SGLT transporter. TANG LEE EXAM - FEB
the SGLT transporter (active co-transport)? Fructose is the only ketose sugar in the group. SAY, MD 2014
A. Glucose Glucose=Dextrose. Glucose, galactose and mannose (TOP 4 -
B. Mannose are aldoses AUG 2013
C. Galactose MED
D. Dextrose BOARDS;
E. Fructose TOPNOTCH
MD)
585 Aerobic glycolysis in the heart produces a net total of Aerobic glycolysis produces either 36 or 38 ATP TIMOTHY MIDTERM 1
how many ATP molecules for each molecule of glucose? molecules for each molecule of glucose depending on TANG LEE EXAM - FEB
A. 30 the shuttle used. The liver, kidney, and the heart uses SAY, MD 2014
B. 32 the malate aspartate shuttle so each NADH produce in (TOP 4 -
C. 34 glycolysis in the cytoplasm nets 3 ATP. The skeletal AUG 2013
D. 36 muscle and the brain uses the gycerol phosphate MED
E. 38 shuttle so each NADH only produces 2 ATP. BOARDS;
TOPNOTCH
MD)
586 A patient always experienced pain and cramps after The signs and symptoms are typical of McArdle's TIMOTHY MIDTERM 1
exercise. Initial work-up shows normal ECG disease (Cramps+Myoglobinuria (brownish TANG LEE EXAM - FEB
tracing,electrolytes were within normal limits and discoloration) with no lactate acidosis) SAY, MD 2014
normal lactate levels. The patient's urine had a brownish (TOP 4 -
color. The patient likely has which glycogen storage AUG 2013
disease? MED
A. Von Gierke's BOARDS;
B. Pompe's TOPNOTCH
C. Cori's MD)
D. McArdle's
E. Andersen's
587 Which of the following statements are TRUE for lipids? The melting temperature is inversely proportional to TIMOTHY MIDTERM 1
A. All lipids are substances that are hydrophobic. the number of double bonds. Cholesterol is not used to TANG LEE EXAM - FEB
B. The melting temperature of fatty acids is directly obtain energy, only fatty acids and glycerol. Proteins, SAY, MD 2014
proportional to the number of double bonds. not lipids are the main components of enzymes. Fatty (TOP 4 -
C. Lipids such as cholesterol can be used by the body to acid synthesis occur in the cytoplasm but β-oxidation AUG 2013
obtain energy through β-oxidation. occurs in the mitochondria. MED
D. Lipids are the main component of cellular enzymes BOARDS;
needed for metabolism. TOPNOTCH
E. Fatty acid metabolism occurs in the cytoplasm. MD)

588 In the formation of oleic acid, aside from enzymes The end of fatty acid synthesis in the cytoplasm is the TIMOTHY MIDTERM 1
located in the cytoplasm which organelle is needed for formation of the 14-C palmitic acid. Further TANG LEE EXAM - FEB
its synthesis? elongation can be done in both the SER and SAY, MD 2014
A. Peroxisomes mitochondria but insertion of double bonds, such as in (TOP 4 -
B. Lysosomes Oleic acid can only be done in the ER. Peroxisomes are AUG 2013
C. Endoplasmic Reticulum needed to oxidize fatty acids with double bonds. MED
D. Golgi Bodies BOARDS;
E. Mitochondria TOPNOTCH
MD)

TOPNOTCH MEDICAL BOARD PREP BIOCHEMISTRY SUPEREXAM Page 75 of 92


For inquiries visit www.topnotchboardprep.com.ph or email us at topnotchmedicalboardprep@gmail.com
TOPNOTCH MEDICAL BOARD PREP BIOCHEMISTRY SUPEREXAM
For inquiries visit www.topnotchboardprep.com.ph or email us at topnotchmedicalboardprep@gmail.com
Item QUESTION EXPLANATION AUTHOR TOPNOTCH
# EXAM
589 Which of the following is not an essential amino acid? PVT TIM HALL, always ARGues, never TYRes. TIMOTHY MIDTERM 1
A. Histidine Asparagine can be synthesized from aspartate. TANG LEE EXAM - FEB
B. Threonine SAY, MD 2014
C. Leucine (TOP 4 -
D. Asparagine AUG 2013
E. Methionine MED
BOARDS;
TOPNOTCH
MD)
590 Denaturation is the disruption of a protein's structure Altering pH whether increasing or decreasing it TIMOTHY MIDTERM 1
(secondary structure and above). This could be disrupts the protein structure. However only heating TANG LEE EXAM - FEB
accomplished by any of these mechanisms EXCEPT? will result in denaturation, cooling would decrease the SAY, MD 2014
A. Decreasing pH velocity of the catalyzed reactions but the structure of (TOP 4 -
B. Increasing pH the protein is unchanged. AUG 2013
C. Cooling MED
D. Addition of heavy metals BOARDS;
E. Addition of detergents TOPNOTCH
MD)
591 What amino acid is the precursor of the main inhibitory The main inhibitory neurotransmitter of the spinal TIMOTHY MIDTERM 1
neurotransmitter of the spinal cord? cord is glycine. Serine is necessary for its synthesis. TANG LEE EXAM - FEB
A. Serine SAY, MD 2014
B. Glycine (TOP 4 -
C. Glutamate AUG 2013
D. Glutamine MED
E. Tryptophan BOARDS;
TOPNOTCH
MD)
592 Maple syrup disease is a disorder characterized by Lysine is not a branched chain amino acid. TIMOTHY MIDTERM 1
blocked degradation of branched chain amino acids. It is TANG LEE EXAM - FEB
therefore important to limit the following amino acids SAY, MD 2014
EXCEPT? (TOP 4 -
A. Lysine AUG 2013
B. Leucine MED
C. Valine BOARDS;
D. Isoleucine TOPNOTCH
E. None, all needs to be limited MD)

593 A 5-yr old boy has distinctive coarseness in his facial Enzyme defects: TIMOTHY MIDTERM 1
features, including a prominent forehead, a nose with a Hunter's Syndrome: Iduronate Sulfatase TANG LEE EXAM - FEB
flattened bridge, and an enlarged tongue. There are Hurler's Syndrome and Scheie's Syndrome: α- SAY, MD 2014
developmental delays and nervous system problems. iduronidase (TOP 4 -
Biochemical testing reveals there is a defect in the Morquio's Syndrome: Galactose-6-sulfatase AUG 2013
degradation of dermatan sulfate and keratan sulfate. Sly Syndrome: β-Glucuronidase MED
Genetic testing shows there is a polymorphism in the BOARDS;
gene encoding for iduronate-2-sulfatase. The patient TOPNOTCH
most likely has? MD)
A. Hunter's Syndrome
B. Hurler's Syndrome
C. Morquio's Syndrome
D. Scheie's Syndrome
E. Sly Syndrome
594 A 25 yr old obese person had just ate one cup of fried The person is in the well-fed state so we know that the TIMOTHY MIDTERM 1
rice, 2 pieces of fried chicken and a mixed berry shake. metabolic pathways active are glycolysis and TANG LEE EXAM - FEB
Which enzymes are dephosphorylated and active? glycogenesis and not gluconeogenesis and SAY, MD 2014
A. Hexokinase glycogenolysis which are active in the fasting state. (TOP 4 -
B. Phosphofructokinase-1 Hexokinase is not the rate-limiting step of glycolysis AUG 2013
C. Glucose-6-phosphatase and is not subject to phosphorylation and MED
D. Fructose 1,6-bisphophatase dephosphorylation so the answer is PFK-1, the rate BOARDS;
E. Glycogen phosphorylase limiting step of glycolysis. TOPNOTCH
MD)

595 A group of four cavers were rescued today after Glucose is the main energy fuel in the well-fed state. TIMOTHY MIDTERM 1
spending five days lost underground with no food. Glycogen stores are only sufficient for 18-24 hrs. TANG LEE EXAM - FEB
Which of the following metabolites were used for fuel by Prolonged fasting will make the body used ketones as SAY, MD 2014
their bodies just before they were rescued? its main energy fuel (TOP 4 -
A. Glucose AUG 2013
B. Glycogen MED
C. Fatty Acids BOARDS;
D. Ketone bodies TOPNOTCH
E. Glycerol MD)

TOPNOTCH MEDICAL BOARD PREP BIOCHEMISTRY SUPEREXAM Page 76 of 92


For inquiries visit www.topnotchboardprep.com.ph or email us at topnotchmedicalboardprep@gmail.com
TOPNOTCH MEDICAL BOARD PREP BIOCHEMISTRY SUPEREXAM
For inquiries visit www.topnotchboardprep.com.ph or email us at topnotchmedicalboardprep@gmail.com
Item QUESTION EXPLANATION AUTHOR TOPNOTCH
# EXAM
596 An athlete who has extensive training of 8 hours per day No matter the physical activity, the basal metabolic TIMOTHY MIDTERM 1
has a higher energy requirement than people who have rate accounts for at least 50% of energy requirements. TANG LEE EXAM - FEB
sedentary lifestyles. Most of the energy requirement is Most of the basal metabolic rate is used for the SAY, MD 2014
used for? maintenance of cellular functions such as the (TOP 4 -
A. Metabolism in skeletal muscles during physical maintenace of the cellular electrolyte composition by AUG 2013
activity for rapid and sustain effort the Na/K pump. MED
B. Metabolism in cardiac and respiratory cells to BOARDS;
maintain adequate oxygen and nutrient delivery to all TOPNOTCH
cells of the body MD)
C. Metabolism in nerve cells needed to conduct
electrical impulses and control bodily functions
D. Metabolism in the GIT to extract nutrients needed to
sustain the energy requirements
E. Metabolism in cells to maintain homeostasis such as
the maintenance of the Na/K pump

597 Which precursors of nucleotide synthesis is necessary All are needed for either purine or pyrimidine TIMOTHY MIDTERM 1
for both purine and pyrimidine synthesis? synthesis. For purine synthesis, there is the addition TANG LEE EXAM - FEB
A. Glutamine of glycine and tetrahydrofolate as necessary SAY, MD 2014
B. Aspartic acid components. (TOP 4 -
C. Carbon dioxide AUG 2013
D. All of the above MED
E. None of the above BOARDS;
TOPNOTCH
MD)
598 You are task to elucidate the identity of a heriditary Mitochondrial diseases are always maternally derived TIMOTHY MIDTERM 1
disease X. A father inflicted with the disease does not since the mitochondria of a child all comes from the TANG LEE EXAM - FEB
transmit it to any of his children. An afflicted mother mother. Remember the only contribution of the sperm SAY, MD 2014
transmit it to all her offsprings. The disease most likely is its haploid DNA. (TOP 4 -
shows this characteristic? AUG 2013
A. X-linked MED
B. Autosomal Dominant BOARDS;
C. Autosomal Recessive TOPNOTCH
D. Mitochondrial MD)
E. Variable Penetrance

599 What is known as the Central Dogma of Molecular The central dogma involves DNA replication to form TIMOTHY MIDTERM 1
Biology? copies of DNA, DNA transcription to mRNA. mRNA TANG LEE EXAM - FEB
A. The Genetic Code has been conserved from the early serves as a template for the translation of proteins. SAY, MD 2014
stages of evolutions with only slight changes in the (TOP 4 -
manner of which the code is translated. AUG 2013
B. DNA is replicated semi-conservatively with the MED
daughter DNA strands containing half of the original BOARDS;
strands. TOPNOTCH
C. The Genetic Code is specific, universal, MD)
degenerate/redundant and non-overlapping.
D. Southern blot is used to identify DNA, Northern blot
to identify RNA and Western blot to identify proteins
E. DNA is replicated in the nucleus, transcribed to
mRNA, and then translated to proteins in the ribosomes.
600 In biotechnology, restriction enzymes are enzymes that Restriction enzymes target palindromic sequences or TIMOTHY MIDTERM 1
cleave certain locations in the DNA sequence that helps those sequence that are mirror images of each other .. TANG LEE EXAM - FEB
identify the presence of genetic polymorphisms. They Ex SAY, MD 2014
preferentially target which sites in the DNA molecule? AAAAGTCGACAAAAA (TOP 4 -
A. Sites where there is a high GC content TTTTCAGCTGTTTTT AUG 2013
B. Sites where there is a palindromic sequence MED
C. Sites that have altered DNA bases such as thymidine BOARDS;
dimers TOPNOTCH
D. Sites that have trinucleotide repeats (CAGs, etc.) MD)
E. Sites that are "trash" or those that do not encode
proteins
601 Water is the universal solvent. This is because water: Having a high dielectric constant permits water to RACHELLE FINAL EXAM -
A. is an excellent nucleophile decrease force of attraction between charged and MENDOZA, FEB 2013
B. is amphoteric polar species. Water as a dipole means it has electric MD (TOP 9
C. has high dielectric constant charge distributed asymetrically about its structure. - AUG 2012
D. is a dipole Water as an excellent nucleophile means it can cleave MED
E. All of the above amide, glycoside or ester bonds. Water as an BOARDS;
amphoteric means it acts both as an acid and a base. TOPNOTCH
MD)
602 The following are electron transport chain uncouplers, Uncouplers are agents that allow dissipation of RACHELLE FINAL EXAM -
EXCEPT: hydrogen ions across the inner mitochondrial MENDOZA, FEB 2013
A. Aspirin membrane without production of ATP. These MD (TOP 9
B. Thermogenin substances include aspirin, 2,4 DNP and thermogenin - AUG 2012
C. 2, 4-dinitrophenol (brown fat) MED
D. Oligomycin BOARDS;
E. None of the above TOPNOTCH
MD)

TOPNOTCH MEDICAL BOARD PREP BIOCHEMISTRY SUPEREXAM Page 77 of 92


For inquiries visit www.topnotchboardprep.com.ph or email us at topnotchmedicalboardprep@gmail.com
TOPNOTCH MEDICAL BOARD PREP BIOCHEMISTRY SUPEREXAM
For inquiries visit www.topnotchboardprep.com.ph or email us at topnotchmedicalboardprep@gmail.com
Item QUESTION EXPLANATION AUTHOR TOPNOTCH
# EXAM
603 Enzyme 1 has Km (michaelis constant) of 4, while A greater Km value indicates lesser affinity of the RACHELLE FINAL EXAM -
Enzyme 2 has Km of 2. Assuming that Enzymes 1 and 2 enzyme for a given substrate. Km is substrate MENDOZA, FEB 2013
act on the same substrate, which of the following is concentration at which the speed of reaction is half MD (TOP 9
correct? the maximal velocity (Vmax/2) - AUG 2012
A. Enzyme 1 has greater substrate affinity than Enzyme MED
2 BOARDS;
B. Enzyme 2 has greater substrate affinity than Enzyme 1 TOPNOTCH
C. Enzyme 1 and 2 have the same substrate affinity MD)
D. Affinity cannot be determined by Km
E. None of the above
604 After a high carbohydrate meal, blood glucose level Liver, kidney and heart utilizes the malate-aspartate RACHELLE FINAL EXAM -
increases. In the brain, how many ATPs will be shuttle, thereby producing 38 ATPs over-all (NADH MENDOZA, FEB 2013
produced from 1 molecule of glucose after complete will be converted to 3 ATPs). Skeletal muscle and MD (TOP 9
oxidative glycolysis? brain utilizes the glycerol-phosphate shuttle, thereby - AUG 2012
A. 32 producing only 36 ATPs (NADH will be converted to 2 MED
B. 34 ATPs only). BOARDS;
C. 36 TOPNOTCH
D. 38 MD)
E. Cannot be determined

605 The liver cannot utilize ketones as a source of energy, Thiophorase (also known as succinyl coa - acetoacetyl RACHELLE FINAL EXAM -
due to the absence of which enzyme/s? coa transferase) is needed for tissues to utilize MENDOZA, FEB 2013
A. Thiophorase ketones as source of energy. Liver lacks this enzyme. MD (TOP 9
B. Succinyl CoA - Acetoacteyl CoA transferase - AUG 2012
C. Acetoacetyl - acyl transferase MED
D. A and B BOARDS;
E. B and C TOPNOTCH
MD)

606 Which of the following apoproteins attached to Apo C-II activates lipoprotein lipase. Apo B serves as a RACHELLE FINAL EXAM -
lipoproteins are responsible for activation of lipoprotein receptor ligand for LDL (B 100) or Chylomicrons (B MENDOZA, FEB 2013
lipase? 48). Apo E facilitates uptake of lipoproteins by the MD (TOP 9
A. Apo C-II liver. - AUG 2012
B. Apo B-100 MED
C. Apo B-48 BOARDS;
D. Apo E TOPNOTCH
E. B and C MD)

607 A preterm newborn male was observed to be dyspneic Dipalmitoylphosphatidycholine (aka RACHELLE FINAL EXAM -
after 12 hours of life during your NICU rotation. You dipalmitoylecithin) is the major lung surfactant. MENDOZA, FEB 2013
noted chest indrawing, alar flaring and cyanosis. As the Inadequate levels lead to respiratory distress MD (TOP 9
intern-in-charge, you suggested chest x-ray to be done, syndrome in the newborn, more commonly observed - AUG 2012
revealing "white out lungs." Which of the following among preterms. MED
is/are most probably decreased in this patient? BOARDS;
A. Phosphatidylcholine TOPNOTCH
B. Dipalmitoylphosphatidylcholine MD)
C. Dipalmitoyllecithin
D. A and B
E. B and C

608 Which of the following amino acids lacks an amino group Proline has an imino group in its structure. RACHELLE FINAL EXAM -
in its structure? MENDOZA, FEB 2013
A. Histidine MD (TOP 9
B. Arginine - AUG 2012
C. Tryptophan MED
D. Proline BOARDS;
E. None of the above TOPNOTCH
MD)
609 The following are synthesized from tryptophan, EXCEPT: Tryptophan is the precursor for niacin, serotonin and RACHELLE FINAL EXAM -
A. Melanin melatonin. Melanin is synthesized from tyrosine. MENDOZA, FEB 2013
B. Melatonin MD (TOP 9
C. Niacin - AUG 2012
D. Serotonin MED
E. B and C BOARDS;
TOPNOTCH
MD)
610 Your patient has muscular weakness and degeneration Leber's hereditary optic neuropathy is caused by an RACHELLE FINAL EXAM -
of the optic nerve. Your tentative diagnosis is Leber’s abnormality involving complex III of ETC. This is MENDOZA, FEB 2013
hereditary optic neuropathy. If this is the correct characterized by acute onset of unilateral visual loss MD (TOP 9
diagnosis, which of the following pathways will contain which may become bilateral. - AUG 2012
the MED
defect? BOARDS;
A. tricarboxylic acid cycle TOPNOTCH
B. electron transport chain MD)
C. glycolysis
D. gluconeogenesis
E. pyruvate dehdyrogenase multienzyme complex
611 The eukaryotic DNA has the following The eukaryotic DNA has the following properties: RACHELLE FINAL EXAM -
property/properties: 1)coding regions are often interrupted by intervening MENDOZA, FEB 2013
A. Coding regions are not interrupted by intervening sequences, 2)more than half of the DNA is unique or MD (TOP 9
sequeces nonrepetitive sequences, 3)at least 30% of the - AUG 2012
B. All of the cellular DNA is located within the nucleus genome consist of repitive sequences, 4)1% of cellular MED
C. Contains no repetitive sequences DNA is in mitochondria, 5) replication is bidirectional BOARDS;
D. Replication of double stranded DNA is unidirectional TOPNOTCH

TOPNOTCH MEDICAL BOARD PREP BIOCHEMISTRY SUPEREXAM Page 78 of 92


For inquiries visit www.topnotchboardprep.com.ph or email us at topnotchmedicalboardprep@gmail.com
TOPNOTCH MEDICAL BOARD PREP BIOCHEMISTRY SUPEREXAM
For inquiries visit www.topnotchboardprep.com.ph or email us at topnotchmedicalboardprep@gmail.com
Item QUESTION EXPLANATION AUTHOR TOPNOTCH
# EXAM
E. None of the above MD)

612 Which of the following pairs is INCORRECT? RNA polymerase 1 is used in the transcription of large RACHELLE FINAL EXAM -
A. RNA polymerase I - rRNA rRNAs in the nucleolus, II for mRNAs and III for tRNAs MENDOZA, FEB 2013
B. RNA polymerase II - ssRNA MD (TOP 9
C. RNA polymerase III - tRNA - AUG 2012
D. A and C MED
E. All are incorrect BOARDS;
TOPNOTCH
MD)
613 Which of the following pertains to a set of structure Promoter regions are part of the DNA where RNA RACHELLE FINAL EXAM -
genes coding for a group of proteins required for a polymerase binds to start transcription. The primary MENDOZA, FEB 2013
particular metabolic function along with the regulatory transcript is a linear copy of the transcriptional unit, MD (TOP 9
region that controls the expression of the structural the segment of DNA between specific initiation and - AUG 2012
genes? termination sequences. Rho factor is involved in MED
A. Promoter region termination of DNA transcription by binding to a C- BOARDS;
B. Primary transcript rich region near the 3'-end of the newly synthesized TOPNOTCH
C. Rho factor RNA and migrates along the 5' to 3' direction until MD)
D. Operon termination site is reached.
E. None of the above
614 A sample of DNA-RNA hybrid is to be examined for its Southern blot is utilized for purely DNA sample. RACHELLE FINAL EXAM -
component. The best method to utilize is: Northern blot is utlized for RNA and mixed DNA-RNA MENDOZA, FEB 2013
A. Southern blot samples, while Western blot can be utilized for MD (TOP 9
B. Northern blot protein-containing sample. - AUG 2012
C. Western blot MED
D. Eastern blot BOARDS;
E. None of the above TOPNOTCH
MD)

615 A 6-year old boy was brought to your clinic due to This patient is suffering from mucopolysaccharidoses. RACHELLE FINAL EXAM -
persistent back pain. You noted that he was short for his Morquio's syndrome is the only MENDOZA, FEB 2013
age and has some degree of skeletal dysplasia. Upon mucopolysaccharidoses which does not present with MD (TOP 9
further examination, his mental development was found mental retardation or other CNS involvement. - AUG 2012
to be at par with his age. Suspecting a type of Findings usually include skeletal dysplasia and short MED
mucopolysacchridoses, which of the following is most stature. This is due to deficiency in galactose 6- BOARDS;
likely affecting this patient? sulfatase. TOPNOTCH
A. Morquio's syndrome MD)
B. Hunter's syndrome
C. San fillippo syndrome
D. Sly syndrome
E. None of the above

616 A 65-year old diabetic female was brought to the ER due The very low blood glucose of the patient would RACHELLE FINAL EXAM -
to body weakness, followed by seizures and sudden loss trigger the release of counterregulatory hormones MENDOZA, FEB 2013
of consciousness. History revealed that she was on that are supposed to elevate glucose levels by MD (TOP 9
insulin therapy for 6 years now and has missed several increasing mobilization of glucose from liver and - AUG 2012
meals that day due to a bingo tournament she joined in inducing gluconeogenesis. These counterregulatory MED
the neighborhood. Blood glucose level was 18 mg/dl. hormones include glucagon, epinephrine and cortisol. BOARDS;
Which of the following can be expected in this patient? Since the patient is being maintained on exogenous TOPNOTCH
A. Increased glucagon insulin for sugar control, endogenous insulin secretion MD)
B. Increased epinephrine can be presumed to be decreased. C peptide is
C. Decreased C-peptide produced only in endogenous insulin. Hence, in this
D. All of the above patient, C peptide can be presumed to be decreased,as
E. A and B well.

617 Which of the following enzyme/s involved in heme Lead inhibits ALA dehydratase and ferrochelatase. RACHELLE FINAL EXAM -
synthesis is/are inhibited by lead intoxication, thereby MENDOZA, FEB 2013
causing anemia? MD (TOP 9
A. aminolevulinic acid synthase - AUG 2012
B. aminolevulinic dehydratase MED
C. ferrochelatase BOARDS;
D. A and B TOPNOTCH
E. B and C MD)

618 A 3-month old infant prsented with fair skin, eczema, The patient has phenyketonuria. This is either due to RACHELLE FINAL EXAM -
growth retardation and musty body odor. Which of the deficiency in phenylalanine hydroxylase or decreased MENDOZA, FEB 2013
following should be done to prevent further worsening tetrahydrobipterin cofactor. This sually presents with MD (TOP 9
of his condition? mental retardation, growth retardation, fair skin, - AUG 2012
A. Administration of branched chain amino acid in the eczema and musty body odor. Treatment includes MED
diet elimination of phenylalanine from the diet and BOARDS;
B. Eliminate branched chain amino acid from the diet administer tyrosine-rich diet TOPNOTCH
C. Administer phenylalanine in the diet and eliminate MD)
tyrosine
D. Eliminate phenylalanine from the diet and administer
tyrosine supplement
E. None of the above

TOPNOTCH MEDICAL BOARD PREP BIOCHEMISTRY SUPEREXAM Page 79 of 92


For inquiries visit www.topnotchboardprep.com.ph or email us at topnotchmedicalboardprep@gmail.com
TOPNOTCH MEDICAL BOARD PREP BIOCHEMISTRY SUPEREXAM
For inquiries visit www.topnotchboardprep.com.ph or email us at topnotchmedicalboardprep@gmail.com
Item QUESTION EXPLANATION AUTHOR TOPNOTCH
# EXAM
619 The following amino acids can be converted to ketones, Strictly ketogenic amino acids are: leucine and lysine. RACHELLE FINAL EXAM -
EXCEPT: Ketogenic and glucogenic amino acids are: MENDOZA, FEB 2013
A. Glycine phenylalanine, tyrosine, tryptophan and isoleucine. MD (TOP 9
B. Leucine All others are strictly glucogenic. - AUG 2012
C. Phenylalanine MED
D. Tryptophan BOARDS;
E. B and C TOPNOTCH
MD)
620 A mitochondrion was experimentally divided into its All components of ETC are fixed to the inner RACHELLE FINAL EXAM -
individual parts. On examining the inner membrane, mitochondrial membrane except for: conenzyme Q MENDOZA, FEB 2013
which of the following electron transport chain and cytochrome C. MD (TOP 9
components will NOT be present? - AUG 2012
A. Complex I MED
B. Complex III BOARDS;
C. Cytochrome oxidase TOPNOTCH
D. Cytochome c MD)
E. C and D

621 A 49 year old man with a 10 year history of poorly Aldose reductase catalyzes the breakdown of glucose VON DIAGNOSTIC
controlled diabetes mellitus presents to his physician into sorbitol. Sorbitol is then metabolized to fructose. ANDRE EXAM - AUG
complaining of changes in his vision. Physical In patients with hyperglycemia as in a case of DM, MEDINA, 2012
examination reveals opacities on the lens of the eye. sorbitol accumulation with the cells leads in the rise in MD (TOP 4
Which enzyme most likely contributed to this the intracellular osmolality, causing water movement - FEB 2012
complication? in the cell resulting to osmolar swelling and osmotic MED
A. Adenosine deaminase damage. swelling of the lens fiber can lead into BOARDS;
B. Aldose reductase rupture and cataract formation. TOPNOTCH
C. Galactose-1-phosphate uridyltransferase MD)
D. Hexokinase
E. Glucokinase
622 A woman gives birth to a full term baby. Upon delivery, This is a case of Phenylketonuria (PKU). Patients with VON DIAGNOSTIC
her pediatrician, Dra. Casimiro, noticed that the baby is this kind of disease are unable to convert ANDRE EXAM - AUG
small and has a musty odor. Upon questioning, the phenylalanine to tyrosine due to a deficiency of MEDINA, 2012
woman says that she did not smoke nor drink alcohol phenylalanine hydroxylase. MD (TOP 4
during the course of her pregnancy and she only drinks - FEB 2012
diet soda and water. Which of the following amino acid is MED
most likely to be deficient in the newborn? BOARDS;
A. Alanine TOPNOTCH
B. Tyrosine MD)
C. Phenylalanine
D. Serine
E. B and C

623 Which amino acid would most likely to be found in the Phenylalanine is a non-polar amino acid. VON DIAGNOSTIC
interior of the a protein? ANDRE EXAM - AUG
A. Tyrosine MEDINA, 2012
B. Phenylalanine MD (TOP 4
C. Histidine - FEB 2012
D. Asparagine MED
E. A and C BOARDS;
TOPNOTCH
MD)
624 Water is considered a dipolar molecule because? water is a dipole- a molecule with electrical charge VON DIAGNOSTIC
A. It can act both as a proton donor and a proton distributed asymmetrically it its structure. ANDRE EXAM - AUG
acceptor MEDINA, 2012
B. It has a partial positive and negative charges MD (TOP 4
unequally distibuted in its structure - FEB 2012
C. It is made up of two elements- oxygen and hydrogen MED
D. it can exist in a solid (ice) as well as the liquid state BOARDS;
E. all of the above TOPNOTCH
MD)
625 The higher the oxygen affinity of hemoglobin for oxygen, The higher the affinity of oxygen for hemoglobin the VON DIAGNOSTIC
A. The higher its P50 for oxygen becomes difficult it becomes to give up or to distribute the ANDRE EXAM - AUG
B. The more difficult it is to saturate with oxygen oxygen to the tissues. MEDINA, 2012
C. The more difficult it becomes for oxygen to MD (TOP 4
dissociate from the hemoglobin molecule - FEB 2012
D. A & B MED
E. none of the above BOARDS;
TOPNOTCH
MD)
626 A drug that is a competitive inhibitor of an enzyme a competitive inhibitor competes with a substrate for VON DIAGNOSTIC
A. Increases the apparent Km but does not affect the the active site of the enzyme, in effect increasing the ANDRE EXAM - AUG
Vmax apparent Km. As the substrate concentration is MEDINA, 2012
B. Decreases the apparent Km but does not affect the increased, the substrate, by competing with the MD (TOP 4
Vmax inhibitor, can overcome its inhibitory effects and - FEB 2012
C. Increases Vmax but does not affect the Km eventually the normal Vmax is reached. MED
D. Decreases the Vmax but does not affect the Km BOARDS;
E. Decreases both Vmax and Km TOPNOTCH
MD)
627 What enzyme converts Norepinephrine to Epinephrine? PNMT converts NE to E, with SAM (S adenosyl VON DIAGNOSTIC
A. Dopamine B-hydroxylase methionine) which adds CH3 and serves as a donor ANDRE EXAM - AUG
B. Dopa decarboxylase for the conversion. MEDINA, 2012
C. Norepinephrine hydroxylase MD (TOP 4
D. Epinephrine decarboxylase - FEB 2012
E. Phenylethanolamine -N-Methyl transferase MED
BOARDS;
TOPNOTCH
TOPNOTCH MEDICAL BOARD PREP BIOCHEMISTRY SUPEREXAM Page 80 of 92
For inquiries visit www.topnotchboardprep.com.ph or email us at topnotchmedicalboardprep@gmail.com
TOPNOTCH MEDICAL BOARD PREP BIOCHEMISTRY SUPEREXAM
For inquiries visit www.topnotchboardprep.com.ph or email us at topnotchmedicalboardprep@gmail.com
Item QUESTION EXPLANATION AUTHOR TOPNOTCH
# EXAM
MD)

628 What vitamin is required for the synthesis of niacin from Pyridoxine of B6 is required for the synthesis of niacin VON DIAGNOSTIC
typtophan? from tryptophan. ANDRE EXAM - AUG
A. Thiamine MEDINA, 2012
B. Riboflavin MD (TOP 4
C. Pantothenic acid - FEB 2012
D. Pyridoxine MED
E. Cobalamin BOARDS;
TOPNOTCH
MD)
629 Mr. Dorado was diagnosed to have a deficiency of muscle this is a case of McArdle's disease, muscle VON DIAGNOSTIC
phosphorylase enzyme. An individual with deficiency of phosphorylase deficiency. Muscle glycogen could not ANDRE EXAM - AUG
this enzyme would; be oxidized during exercise. Therfore, lactate level MEDINA, 2012
A. Produce a higher than normal amount of lactate would be low and the person could not tolerate MD (TOP 4
during a brief period of intense exercise like push-ups intense exercise of brief duration and would rely on - FEB 2012
B. Be incapable of performing mild exercises of long fuels from the blood for energy. the person could MED
duration (a 10-mile walk) engage in mild exercises of long duration using these BOARDS;
C. have lower than normal amounts of glycogen in blood fuels. the liver would not be affected because it TOPNOTCH
muscle tissue contains a different phosphorylase enzyme and it MD)
D. be less dependent than normal on blood glucose to could still respond to glucagon by breaking down
supply energy for exercise glycogen.
E. produce normal amounts of blood glucose in
response to increased glucagon
630 Which of the following statements about the conversion Citrate transports acetyl unit from mitochondria to VON DIAGNOSTIC
of glucose to triacylglycerol in the liver is CORRECT? cytosol. NADPH is provided by PPP and MALIC ANDRE EXAM - AUG
A. Malate serves to transport acetyl units across the enzyme. The liver has glycerol kinase, thus blood MEDINA, 2012
mitochondrial membrane glycerol can be used. 2 monoacylglycerol is produced MD (TOP 4
B. Reducing equivalents are provided by the reactions only in intestinal cells. - FEB 2012
of glycolysis MED
C. Reducing equivalents are provided by the malic BOARDS;
enzyme (a decarboxylating malate dehydrogenase) TOPNOTCH
D. The glycerol moiety can be derived from MD)
dihydroxyacetone phosphate (DHAP) but not from blood
glycerol
E. 2-Monoacylglycerol is an intermediate
631 What reaction/pathway generates the greatest amount VON DIAGNOSTIC
of ATP? ANDRE EXAM - AUG
A. Glycolysis MEDINA, 2012
B. Citric acid cycle MD (TOP 4
C. Oxidative Phosphorylation - FEB 2012
D. Adenylate kinase reaction MED
E. None of the above BOARDS;
TOPNOTCH
MD)
632 When it transfers extramitochondrial NADH to the Glycerophosphate shuttle 2 ATPs; malate 3 ATPS VON DIAGNOSTIC
respiratory chain, it generates 2 rather than 3 ATPs ANDRE EXAM - AUG
A. Malate shuttle MEDINA, 2012
B. Glycerophosphate shuttle MD (TOP 4
C. Citrate shuttle - FEB 2012
D. Both A and B MED
E. None of the above BOARDS;
TOPNOTCH
MD)
633 A neonate born 4 hours ago is having a difficulty of Dipalmitoyl phosphatidylcholine is the same as VON DIAGNOSTIC
breathing. The baby was born at 32 weeks AOG. He is lecithin. ANDRE EXAM - AUG
tachypneic and is flaring and grunting. The baby's heart MEDINA, 2012
rate is 120bpm, BP 100/60 mmHg and respiratory rate MD (TOP 4
is 55 cpm. What is lacking in this patient? - FEB 2012
A. Dipalmitoyl phosphatidylcholine MED
B. Lecithin BOARDS;
C. Elastase TOPNOTCH
D. Collagen MD)
E. A and B

634 Which of the following statements about nitrogen because of the sulfur of methionine is used for VON DIAGNOSTIC
metabolism is correct? cysteine synthesis, as cysteine increases in the diet, ANDRE EXAM - AUG
A. Cysteine "spares" methionine; that is ingestion of less methionine is required. Glutamate DH catalyzes MEDINA, 2012
cysteine reduces the need for methionine in the diet the addition of ammonia to alpha ketoglutarate to MD (TOP 4
B. The enzyme glutamate dehydrogenase catalyzes the form glutamate. For creatine synthesis, arginine, not - FEB 2012
transamination of glutamate ornithine, is required. FIGLU is produced during the MED
C. Creatine requires glycine, ornithine, and methionine degradation of histidine. BOARDS;
for the synthesis of its carbon skeleton TOPNOTCH
D. Formiminoglutamate (FIGLU) is an intermediate in MD)
glutamine degradation
E. All of the above

TOPNOTCH MEDICAL BOARD PREP BIOCHEMISTRY SUPEREXAM Page 81 of 92


For inquiries visit www.topnotchboardprep.com.ph or email us at topnotchmedicalboardprep@gmail.com
TOPNOTCH MEDICAL BOARD PREP BIOCHEMISTRY SUPEREXAM
For inquiries visit www.topnotchboardprep.com.ph or email us at topnotchmedicalboardprep@gmail.com
Item QUESTION EXPLANATION AUTHOR TOPNOTCH
# EXAM
635 Which of the following statements concerning Arachidonic acid is cleaved from membrane VON DIAGNOSTIC
metabolism of arachidonic acid is TRUE? phospholipids by phospholipase A2. ANDRE EXAM - AUG
A. It is converted to prostagalndins by a process that is MEDINA, 2012
stimulated by aspirin MD (TOP 4
B. It is coverted to prostaglandins by a process that is - FEB 2012
stimulated by glucocorticoids MED
C. It is produced from thromboxanes and leukotrienes BOARDS;
D. It is derived from palmitate TOPNOTCH
E. it is cleaved from membrane phospholipids by a MD)
phospholipase
636 Which of the following statements is TRUE of de novo In pyrimidine biosynthesis, carbamoyl phosphate VON DIAGNOSTIC
pyrimidine synthesis but not of de novo purine produced from glutamine, C02 and ATP, reacts with ANDRE EXAM - AUG
synthesis? aspartate to form a base which, after oxidation, reacts MEDINA, 2012
A. The base is synthesized while attached to ribose 5 with PRPP to form a nucleotide. MD (TOP 4
phosphate - FEB 2012
B. One carbon fragments are donated by folic acid MED
derivatives BOARDS;
C. Carbamoyl PO4 donates a carbamoyl group TOPNOTCH
D. The entire glycine molecule is incorporated into a MD)
precursor of the base
E. Glutamine donates nitrogen that becomes one of the
atoms in the ring
637 A molecule of palmitate will yield how many ATP? VON DIAGNOSTIC
A. 12 ANDRE EXAM - AUG
B. 38 MEDINA, 2012
C. 39 MD (TOP 4
D. 126 - FEB 2012
E. 129 MED
BOARDS;
TOPNOTCH
MD)
638 Which level of protein structure is involved in the B pleated sheet= secondary structure VON DIAGNOSTIC
pathology of prion diseases? ANDRE EXAM - AUG
A. Primary structure MEDINA, 2012
B. Secondary structure MD (TOP 4
C. Tertiary structure - FEB 2012
D. Quaternary structure MED
E. A and C BOARDS;
TOPNOTCH
MD)
639 Typtophan is an amino acid with the largest side chain tryptophan is a precursor of niacin, melatonin and VON DIAGNOSTIC
containing an indole ring. It is a precursor of what serotonin ANDRE EXAM - AUG
substances? MEDINA, 2012
A. niacin MD (TOP 4
B. melatonin - FEB 2012
C. serotonin MED
D. A and B BOARDS;
E. All of the above TOPNOTCH
MD)
640 Desmolase is the rate limiting enzyme in androgen and desmolase converts cholesterol to pregnenolone, VON DIAGNOSTIC
steroid synthesis. It is responsible for what reaction? which is the rate limiting step ANDRE EXAM - AUG
A. Conversion of progesterone to 11 MEDINA, 2012
deoxycorticosterone MD (TOP 4
B. Conversion of pregnenolone to 17 - FEB 2012
hydroxypregnenolone MED
C. Conversion of cholesterol to pregnenolone BOARDS;
D. Conversion of DHEA to Androstenedione TOPNOTCH
E. None of the above MD)
641 Glycosaminoglycans (GAG's) are long, unbranched All of the choices given are GAG's. However, in terms LITO JAY DIAGNOSTIC
heteropolysaccharide chains that are generally of cell migration and wound repair (specifically on MACARAIG, EXAM - AUG
composed of repeating dissacharide units. Which among synovial fluid and vitreous humor), Hyaluronic Acid is MD (TOP 8 2013
the following GAG's is mainly found on synovial fluid and the best answer. Chondroitin sulfate is present on - FEB 2013
vitreous humor; and plays an important role in cartilage and cornea. Keratan Sulfate and Dermatan MED
permitting cell migration during morphogenesis and sulfate are responsible for corneal transparency. BOARDS;
wound repair? Heparan Sulfate determines the charge-selectiveness TOPNOTCH
A. Hyaluronic Acid (GlcNAc, GLCUA) of the renal glomerulus. MD)
B. Chondroitin Sulfate (GalNAc, GlcUA)
C. Keratan Sulfate (GlcNAc, Gal)
D. Dermatan Sulfate (GalNAc, IdUA)
E. Heparan Sulfate (GalNAc, GlcUA)

642 After acquiring your Physician PRC license, you decided Morquio's Syndrome is characterized by accumulation LITO JAY DIAGNOSTIC
to go on moonlighting. At the OPD, a mother brought her of GAG's in the lysosomes due to galactose-6-sulfatase MACARAIG, EXAM - AUG
4 year old child who presented with short stature, deficiency, and is the only mucopolysaccharidoses MD (TOP 8 2013
hypermobile joints and widely-spaced teeth. Upon chest syndrome that is NOT associated with CNS - FEB 2013
examination, you heard a grade 4/6 systolic murmur. abnormality. MED
You are suspecting a mucopolysaccharidoses syndrome BOARDS;
and upon knowing that the patient has NO CNS TOPNOTCH
abnormality, you already know that the patient is MD)
suffering from?
A. Hurler's Syndrome
B. Hunter's Syndrome
C. Morquio's Syndrome
D. Sanfilippo Syndrome
E. Zellweger Syndrome

TOPNOTCH MEDICAL BOARD PREP BIOCHEMISTRY SUPEREXAM Page 82 of 92


For inquiries visit www.topnotchboardprep.com.ph or email us at topnotchmedicalboardprep@gmail.com
TOPNOTCH MEDICAL BOARD PREP BIOCHEMISTRY SUPEREXAM
For inquiries visit www.topnotchboardprep.com.ph or email us at topnotchmedicalboardprep@gmail.com
Item QUESTION EXPLANATION AUTHOR TOPNOTCH
# EXAM
643 Myoglobin is a heme protein found in heart and skeletal Histidine residues play an essential role in O2-binding LITO JAY DIAGNOSTIC
muscle. It acts as reservoir of oxygen in the said of myoglobin. Glutamine and Alanine are the main MACARAIG, EXAM - AUG
locations and releases oxygen in times of hypoxia. The carriers of Nitrogen from peripheral tissues towards MD (TOP 8 2013
residue of which amino acid is responsible for the liver. Serine and Threonine are the sites for O- - FEB 2013
myoglobin's oxygen-binding capacity? linked glycosylation in the Golgi apparatus. MED
A. Glutamine BOARDS;
B. Histidine TOPNOTCH
C. Alanine MD)
D. Serine
E. Threonine

644 Which among the following amino acids is a weak base, Histidine, Arginine, and Lysine are all basic in nature. LITO JAY DIAGNOSTIC
and therefore has no charge on neutral pH? Arginine and Lysine are positively charged at neutral MACARAIG, EXAM - AUG
A. Histidine pH. Leucine and Isoleucine are branched-chain amino MD (TOP 8 2013
B. Arginine acids. - FEB 2013
C. Lysine MED
D. Leucine BOARDS;
E. Isoleucine TOPNOTCH
MD)

645 On a research laboratory, you are asked to determine the The linear sequence of a protein is called its Primary LITO JAY DIAGNOSTIC
linear sequence of a specific protein. While doing so, you Structure. It is composed by specific amino acids MACARAIG, EXAM - AUG
recalled that the kind of bond you least likely to break is attached to each other by Peptide Bonds. All other MD (TOP 8 2013
the? choices are found on higher levels of protein - FEB 2013
A. Hydrogen Bond structure. MED
B. Ionic Bond BOARDS;
C. Covalent Bond TOPNOTCH
D. Peptide Bond MD)
E. Hydrophobic Bond

646 These are specialized group of proteins required for the SIMILAR TO PREVIOUS BOARD EXAM LITO JAY DIAGNOSTIC
folding of many species of proteins. They can also CONCEPT/PRINCIPLE. Motifs are the non-functional MACARAIG, EXAM - AUG
"rescue" proteins that have become thermodynamically combinations of secondary protein structures. MD (TOP 8 2013
trapped in a misfolded dead-end by unfolding Domains are made of Motifs and are the functional - FEB 2013
hydrophobic regions. forms. Chaperones are the folding-guides and are also MED
A. Domains called the "rescue" proteins. The rest are just nuisance BOARDS;
B. Chaperones choices. TOPNOTCH
C. Motifs MD)
D. Protein-guiding Benders
E. Hydrophilic β-benders

647 Hemoglobin A1C (HbA1C) is being used in clinics to When blood glucose enters the erythocytes, it LITO JAY DIAGNOSTIC
monitor a diabetic patient's compliance in taking his/her glycosylates the ξ-amino group of Lysine residues and MACARAIG, EXAM - AUG
medications. On what amino acid residue does the amino terminals of hemoglobin. MD (TOP 8 2013
glycosylation of erythrocytes occur? - FEB 2013
A. Glycine MED
B. Alanine BOARDS;
C. Valine TOPNOTCH
D. Lysine MD)
E. Leucine

648 α1-antitrypsin is said to be a protective factor against Elastase destroys alveolar walls, resulting to LITO JAY DIAGNOSTIC
developing emphysema. What proteolytic enzyme does emphysema. Α1-antitrypsin regulates this enzyme and MACARAIG, EXAM - AUG
this substance inhibit? serves as a protection. MD (TOP 8 2013
A. Fibrillin Kinase - FEB 2013
B. Desmosinase MED
C. Elastase BOARDS;
D. Elastinase TOPNOTCH
E. Protease MD)

649 In bioenergetics, the measure of heat content of the Joules is the unit of measurement used for describing LITO JAY DIAGNOSTIC
reactants and products is called Enthalpy. What unit is Enthalpy. Joules per Kelvin is used to measure MACARAIG, EXAM - AUG
used to describe it? Entropy. MD (TOP 8 2013
A. Kelvin - FEB 2013
B. Joules MED
C. Joules per Kilogram BOARDS;
D. Joules per Kelvin TOPNOTCH
E. Joules per Fahrenheit MD)

650 The electron transport chain (ETC) is the final common Complex II is the Succinate Dehydrogenase enzyme. LITO JAY DIAGNOSTIC
pathway by which electrons from the different fuels of Aside from accepting FADH2 in the ETC, it also MACARAIG, EXAM - AUG
the body flow and create energy. What complex of ETC is participates in the Kreb's cycle. It converts Succinate MD (TOP 8 2013
also an essential part of the Kreb's Cycle? to Fumarate and yields FADH2 as a reducing - FEB 2013
A. Complex I equivalent. MED
B. Complex II BOARDS;
C. Coenzyme Q TOPNOTCH
D. Complex III MD)
E. Complex IV

TOPNOTCH MEDICAL BOARD PREP BIOCHEMISTRY SUPEREXAM Page 83 of 92


For inquiries visit www.topnotchboardprep.com.ph or email us at topnotchmedicalboardprep@gmail.com
TOPNOTCH MEDICAL BOARD PREP BIOCHEMISTRY SUPEREXAM
For inquiries visit www.topnotchboardprep.com.ph or email us at topnotchmedicalboardprep@gmail.com
Item QUESTION EXPLANATION AUTHOR TOPNOTCH
# EXAM
651 Uncouplers are substances that increase the In the presence of uncouplers, ETC still continue to LITO JAY DIAGNOSTIC
permeability of the inner mitochonadrial membrane to produce energy BUT in the form of HEAT and NOT MACARAIG, EXAM - AUG
protons. Which among the following statements is/are ATP. Uncouplers increase oxygen consumption and MD (TOP 8 2013
true about ETC in the presence of uncouplers? oxidation of NADH. - FEB 2013
A. Continue to produce energy MED
B. Increase ATP synthesis BOARDS;
C. Decrease oxidation of NADH TOPNOTCH
D. choices A and B are both correct MD)
E. choices B and C are both correct

652 Lactose is from? It is a common mistake that medical students believe LITO JAY DIAGNOSTIC
A. Glucose + Glucose that Galactose is the dissacharide form of Lactose MACARAIG, EXAM - AUG
B. Galactose + Glucose (which is the other way around). Lactose is the MD (TOP 8 2013
C. Galactose - Glucose condensation product of Galactose plus Glucose. - FEB 2013
D. Galactose - Fructose Galactose is only a monosaccharide, just like Glucose. MED
E. Galactose + Maltose BOARDS;
TOPNOTCH
MD)
653 Glucose is the main fuel of most of the cells in the body. The brain utilizes Glut-1 and Glut-3 to transport LITO JAY DIAGNOSTIC
Before it can be utilized, some cells need specific glucose inside the cell. Glut-2 is present in the liver, MACARAIG, EXAM - AUG
transporters to mediate its use. Which among the pancreatic B-cells, kidneys, and basement membrane MD (TOP 8 2013
following glucose transporters is/are present in the of the small intestines. - FEB 2013
brain? MED
A. Glut 1 BOARDS;
B. Glut 2 TOPNOTCH
C. Glut 3 MD)
D. choices A and B are both correct
E. choices A and C are both correct
654 Phosphofructokinase 1 (PFK1) is the rate-limiting step During the well-fed state (when there is increased LITO JAY DIAGNOSTIC
for Glycolysis. Which among the following statements Insulin and decreased Glucagon levels), PFK2 converts MACARAIG, EXAM - AUG
is/are true about this enzyme? fructose-6-phosphate (F-6-P) to fructose-2,6- MD (TOP 8 2013
A. Activated by excess ATP bisphosphate (F-2,6-BP). Accordingly, F-2,6-BP - FEB 2013
B. Inhibited by excess Citrate activates PFK1 and glycolysis will proceed to produce MED
C. Active in Starved state energy from the food we eat. In contrary, when there BOARDS;
D. Choices A and B are both correct is excess energy (in the form of ATP and Citrate), PFK- TOPNOTCH
E. Choices A and C are both correct 1 is inhibited. MD)

655 Pyruvate Dehydrogense enzyme deficiency is the most Pyruvate Dehydrogenase enzyme is the link between LITO JAY DIAGNOSTIC
common biochemical cause of congenital lactic acidosis. Glycolysis and Kreb's Cycle. It needs the following co- MACARAIG, EXAM - AUG
Which among the following substances is/are needed by enzymes: 1. Thiamine pyrophosphate, 2. FAD, 3. MD (TOP 8 2013
this enzyme as co-factor/s? NAD+, 4. Coenzyme-A, 5. Lipoic Acid - FEB 2013
A. Thiamine bisphosphate MED
B. FAD BOARDS;
C. NAD+ TOPNOTCH
D. Choices A and B are both correct MD)
E. Choices B and C are both correct

656 Gluconeogenesis pathway is technically the reverse of The steps 10, 3 and 1 of Glycolysis are catalyzed by LITO JAY DIAGNOSTIC
Glycolysis. They just differ in three enzymes. Which Pyruvate Kinase, PFK-1, and Hexokinase/Glucokinase MACARAIG, EXAM - AUG
among the following groups of enzymes is PRESENT in respectively. In Gluconeogenesis, these enzymes are MD (TOP 8 2013
Gluconeogenesis and NOT in the later? subtituted by: PEP carboxykinase + Pyruvate - FEB 2013
A. PEP carboxykinase + Pyruvate Carboxylase, Carboxylase, Fructose-1,6-bisphosphatase, and Glucose- MED
Fructose-1,6-bisphosphatase, Glucose-6-phosphatase 6-phosphatase respectively. BOARDS;
B. PEP carboxykinase + Pyruvate Carboxylase, TOPNOTCH
Fructose-1,6-bisphosphatase, Hexokinase/Glucokinase MD)
C. PEP carboxykinase + Pyruvate Carboxylase, Fructose-
2,6-bisphosphatase, Glucose-6-phosphatase
D. PEP carboxykinase + Pyruvate Carboxylase,
Fructose-2,6-bisphosphatase, Hexokinase/Glucokinase
E. PEP carboxykinase + Pyruvate Carboxykinase,
Fructose-2,6-bisphosphatase, Hexokinase/Glucokinase

657 Cellulose is the primary structure of a plant cell wall. Cellulose is a polysaccharide consisiting of a linear LITO JAY DIAGNOSTIC
Being a stool-former, it is an important part of human chain of several hundred to ten thousand of β(1->4) MACARAIG, EXAM - AUG
diet and is said to be protective from colon cancer. What linkage which cannot be digested by humans. MD (TOP 8 2013
kind of glycosidic bond is present in cellulose that - FEB 2013
humans cannot digest? MED
A. α(1->4) linkage BOARDS;
B. α(1->6) linkage TOPNOTCH
C. β(1->4) linkage MD)
D. β(1->6) linkage
E. none of the choices is true

TOPNOTCH MEDICAL BOARD PREP BIOCHEMISTRY SUPEREXAM Page 84 of 92


For inquiries visit www.topnotchboardprep.com.ph or email us at topnotchmedicalboardprep@gmail.com
TOPNOTCH MEDICAL BOARD PREP BIOCHEMISTRY SUPEREXAM
For inquiries visit www.topnotchboardprep.com.ph or email us at topnotchmedicalboardprep@gmail.com
Item QUESTION EXPLANATION AUTHOR TOPNOTCH
# EXAM
658 During your rotation in the Department of Pediatrics, Glucose-6-phosphate dehydrogenase (G6PD) LITO JAY DIAGNOSTIC
you encountered a 5 year-old patient with a chief deficiency is the most common disease-producing MACARAIG, EXAM - AUG
complaint of loose watery stools accompanied by enzyme abnormality in humans. Precipitating factors MD (TOP 8 2013
undocumented fever. During PE, patient is febrile with involves infection and certain medications. - FEB 2013
macular rashes on the chest (Rose Spots). CBC showed MED
lymphocytosis and the Widal Test turned out to be BOARDS;
positive. You prescribed Chloramphenicol but the TOPNOTCH
mother told you that the patient experienced bleeding in MD)
the past when he took the said medication. What kind of
abnormality are you thinking of?
A. Glucose-6-phosphatase deficiency
B. Glucose-6-phosphate dehydroxylase deficiency
C. Glucose-6-phosphate dehydrogenase deficiency
D. Glucose-6-phosphate dihydrogenase deficiency
E. Glucose-6-phosphatase dehydroxylase deficiency
659 This is a rare neurologic disorder due to abnormal In excess, Phytanic Acid blocks beta oxidation and LITO JAY DIAGNOSTIC
accumulation of Phytanic Acid. causes neurologic symptoms due to improper MACARAIG, EXAM - AUG
A. Zellweger's Syndrome myelinization. This disorder is called Refsum's MD (TOP 8 2013
B. Carnitine palmitoyl transferase I deficiency Disease. - FEB 2013
C. Carnitine palmitoyl transferase II deficiency MED
D. Ornithine transcarbamoylase deficiency BOARDS;
E. Refsum's Disease TOPNOTCH
MD)

660 Which among the following codons is/are "stop There are only three "stop codons". UAA, UAG and LITO JAY DIAGNOSTIC
codon/s"? UGA. MACARAIG, EXAM - AUG
A. UAA MD (TOP 8 2013
B. UGG - FEB 2013
C. UAU MED
D. Choices A and B are both correct BOARDS;
E. Choices A and C are both correct TOPNOTCH
MD)
661 This type of chemical bond is formed between 2 amino Peptide bond is formed between two amino acids and HAZEL MIDTERM 2 -
acidsand destroyed by heating the protein in a strong is the bonds that form the primary structure of KAREN AUG 2013
acid solution for 24 hours. proteins. Phosphodiester bonds exist between RAZ, MD
A. Peptide bond nucleotides in a DNA strand. H-bonds exist between (TOP 6 -
B. Phosphodiester bond water and an electron rich molecule such as sulfur, FEB 2013
C. Hydrogen bond oxygen, nitrogen and fluoride. Disulfide bonds exist MED
D. Disulfide bond between cysteine residues in 3o structures. BOARDS;
E. None of the above TOPNOTCH
MD)

662 Precursor of purine amino acids: De Novo Purine synthesis uses IMP as precursor HAZEL MIDTERM 2 -
A. IMP molecule while Pyrimidine synthesis uses OMP. KAREN AUG 2013
B. OMP RAZ, MD
C. UMP (TOP 6 -
D. dUMP FEB 2013
E. None of the above MED
BOARDS;
TOPNOTCH
MD)
663 Deficiency in Vitamin B6 would lead to decreased Vitamin B6 (Pyridoxine) is a co-factor in several HAZEL MIDTERM 2 -
synthesis of the following except? amino acid biosynthetic pathway including KAREN AUG 2013
A. Niacin Tryptophan, Histidine, Glycine and Glutamate to RAZ, MD
B. GABA produce Niacin, Histamine, Porphyrin and GABA (TOP 6 -
C. Epinephrine respectively. Tetrahydrobiopterin (BH4) is needed in FEB 2013
D. Histamine the synthesis of Epinephrine, to convert Phenylalanine MED
E. None of the above into Tyrosine. BOARDS;
TOPNOTCH
MD)
664 Tyrosine becomes an essential amino acid in the In a normal person, Tyrosine is synthesized form HAZEL MIDTERM 2 -
presence of _________? Phenylalanine. However in the presence of KAREN AUG 2013
A. Phenylketonuria phenylketonuria, Phe intake is restricted, so Tyrosine RAZ, MD
B. Tryptophan deficiency must be derived from the diet for the synthesis of (TOP 6 -
C. Adenosine deaminase deficiency catecholamines. FEB 2013
D. Vit. B6 Deficiency MED
E. None of the above BOARDS;
TOPNOTCH
MD)
665 The following are the only ketogenic amino acids? The only ketogenic amino acids are Leucine and HAZEL MIDTERM 2 -
A. Lysine, Leucine Lysine. Ile, Thr, Phe, Tyr and Trp are both ketogenic KAREN AUG 2013
B. Isoleucine, Lysine and glucogenic. Others are purely glucogenic. RAZ, MD
C. Tyrosine, Methionine (TOP 6 -
D. Leucine, Arginine FEB 2013
E. None MED
BOARDS;
TOPNOTCH
MD)

TOPNOTCH MEDICAL BOARD PREP BIOCHEMISTRY SUPEREXAM Page 85 of 92


For inquiries visit www.topnotchboardprep.com.ph or email us at topnotchmedicalboardprep@gmail.com
TOPNOTCH MEDICAL BOARD PREP BIOCHEMISTRY SUPEREXAM
For inquiries visit www.topnotchboardprep.com.ph or email us at topnotchmedicalboardprep@gmail.com
Item QUESTION EXPLANATION AUTHOR TOPNOTCH
# EXAM
666 Keloid is made up of what type of collagen? CONTROVERSIAL. It's made up of both type I and III, HAZEL MIDTERM 2 -
A. I but more type I rather than III. II) Keloid tissues KAREN AUG 2013
B. II contained 31.6 +/- 2.2 percent type III collagen as RAZ, MD
C. III compared to 21.4 +/- 2.7 percent type III present in (TOP 6 -
D. IV normal human skin dermis. FEB 2013
E. V MED
Alteration of collagen composition and cross-linking BOARDS;
in keloid tissues. TOPNOTCH
Di Cesare PE, Cheung DT, Perelman N, Libaw E, Peng L, MD)
Nimni ME.
Matrix. 1990 Jul;10(3):172-8. PubMed PMID: 2215357
667 Post - transcriptional modification in eukaryotes, except? Co-transcriptional modification is a process in cell HAZEL MIDTERM 2 -
A. Capping of 5' end biology by which, in eukaryotic cells, primary KAREN AUG 2013
B. Polyadenylation of 3' end transcript RNA is converted into mature RNA. RAZ, MD
C. Splicing of introns (TOP 6 -
D. Splicing of exons FEB 2013
E. None of the above MED
BOARDS;
TOPNOTCH
MD)
668 A protein is separated by gel electrophoresis and Southern blot uses DNA as sample and DNA as probe. HAZEL MIDTERM 2 -
antibody is bound to a protein. This process describes? Northern blot uses RNA as sample, DNA as probe and KAREN AUG 2013
A. Southern blot Western blot uses Protein as probe and sample. RAZ, MD
B. Northern blot (TOP 6 -
C. Western blot FEB 2013
D. Southwestern blot MED
E. Northwestern blot BOARDS;
TOPNOTCH
MD)
669 This type of inhibition leads to decreased affinity and Non - competitive inhibition decreases efficacy but HAZEL MIDTERM 2 -
does not affect efficacy? does not affect affinity. Allosteric inhibition is a type KAREN AUG 2013
A. Non - competitive Inhibition wherein the substrate binds to a different site from RAZ, MD
B. Allosteric inhibition the inhibitor. Mixed inhibition leads to deacreased (TOP 6 -
C. Mixed inhibition affinity and decreased efficacy. FEB 2013
D. Competitive inhibition MED
E. None of the above BOARDS;
TOPNOTCH
MD)
670 This process occurs both in the cytoplasm and Processes that occur both in the cytoplasm and HAZEL MIDTERM 2 -
mitochondria: mitochondria includes Heme synthesis, Urea cycle and KAREN AUG 2013
A. Glycolysis Gluconeogenesis. RAZ, MD
B. Heme synthesis (TOP 6 -
C. B-oxidation of fatty acids FEB 2013
D. ketogenesis MED
E. None of the above BOARDS;
TOPNOTCH
MD)
671 Rate Limiting step in de Novo Pyrimidine Synthesis? Glutamine - PRPP amidotransferase - de novo purine HAZEL MIDTERM 2 -
A. Carbamoyl PO4 synthetase II synthesis KAREN AUG 2013
B. Glutamine - PRPP amidotransferase Carnitine acyltransferase I - FA oxidation RAZ, MD
C. Carnitine acyltransferase I Carbamoyl phosphate synthetase I - FA oxidation (TOP 6 -
D. Carbamoyl phosphate synthetase i G6PD - HMP shunt FEB 2013
E. G6PD MED
BOARDS;
TOPNOTCH
MD)
672 Gross ATP production per molecule of glucose during Two enzymes in glycolysis produce ATP through HAZEL MIDTERM 2 -
glycolysis through substrate level phosphorylation? substrate level phosphorylation: PG Kinase and KAREN AUG 2013
A. 2 Pyruvate kinase producing 2 ATPs each, making it a RAZ, MD
B. 4 total of 4 ATPs. (TOP 6 -
C. 6 FEB 2013
D. 8 MED
E. 10 BOARDS;
TOPNOTCH
MD)
673 Main function of Pentose Phosphate Pathway? The pentose phosphate pathway (also called HAZEL MIDTERM 2 -
A. NADPH production the phosphogluconate pathway and the hexose KAREN AUG 2013
B. ATP production monophosphate shunt) is a process that RAZ, MD
C. Removal of nitrogenous waste generates NADPH and pentoses (5-carbon sugars). (TOP 6 -
D. Transport of FA from cytosol to inner mitochondrial ATP production is through Kreb's cycle and ETC, FEB 2013
membrane nitrogenous waste is removed through Urea cycle and MED
E. None of the above FA transport is carnitine shuttle for B-oxidation of FA. BOARDS;
TOPNOTCH
MD)
674 Reactions proceed spontaneously in which of the Spontaneous reactions occur when ΔG is negative, that HAZEL MIDTERM 2 -
following? ΔG is <0. Positive ΔG needs energy for it to occur and if ΔG KAREN AUG 2013
A. = 0 = 0, rate of formation is equal to rate of degradation. RAZ, MD
B. < 0 (TOP 6 -
C. >0 FEB 2013
D. ΔH - TΔS MED
E. None of the above. BOARDS;
TOPNOTCH
MD)

TOPNOTCH MEDICAL BOARD PREP BIOCHEMISTRY SUPEREXAM Page 86 of 92


For inquiries visit www.topnotchboardprep.com.ph or email us at topnotchmedicalboardprep@gmail.com
TOPNOTCH MEDICAL BOARD PREP BIOCHEMISTRY SUPEREXAM
For inquiries visit www.topnotchboardprep.com.ph or email us at topnotchmedicalboardprep@gmail.com
Item QUESTION EXPLANATION AUTHOR TOPNOTCH
# EXAM
675 This lipoprotein transports triglyceride from liver to the LDL transports cholesterol into cells. HDL reverses HAZEL MIDTERM 2 -
periphery. cholesterol transport, transporting it from the KAREN AUG 2013
A. LDL periphery into the liver, chylomicrons transport RAZ, MD
B. HDL dietary TG and cholesterol into tissues while micelles (TOP 6 -
C. VLDL are formed through the biliary saponification of FEB 2013
D. Chylomicrons dietary fat from the intestinal lumen for absorption MED
E. Micelles into the intestinal brush border. BOARDS;
TOPNOTCH
MD)
676 Component of surfactant produced by type II Dipalmitoyl phosphatidylcholine is the major HAZEL MIDTERM 2 -
pneumocytes? component of lung surfactant. Phosphatidyl inositol is KAREN AUG 2013
A. Phosphatidyl inositol a reservoir of arachidonic acid in cellular membranes, RAZ, MD
B. Phosphatidyl choline needed for synthesis of prostaglandins and (TOP 6 -
C. Phospholipase leukotrienes. FEB 2013
D. Glycerophospholipid MED
E. Glycerol BOARDS;
TOPNOTCH
MD)
677 Major reactants for heme synthesis includes: Glutamine, glycine and aspartate are the reactants for HAZEL MIDTERM 2 -
A. Glutamine, glycine, aspartate purine synthesis, aspartate and glutamine for KAREN AUG 2013
B. Aspartate, glutamine pyrimidine synthesis, acetyl CoA for FA synthesis. RAZ, MD
C. 2 acetyl CoA (TOP 6 -
D. Glycine, succinyl CoA FEB 2013
E. None MED
BOARDS;
TOPNOTCH
MD)
678 A 6 - month old presents with recurrent sinus infection, This condition is also known as Severe Combined HAZEL MIDTERM 2 -
chronic cough and pneumonia. CBC showed Immunodeficiency (SCID) which is a genetic disorder KAREN AUG 2013
lymphopenia. This may be caused by a deficiency in characterized by the absence of fuctional T- RAZ, MD
which enzyme? lymphocytes,which results in a defective antibody (TOP 6 -
A. HGPRT response. FEB 2013
B. Adenine deaminase MED
C. G6PD BOARDS;
D. Homogentisic acid oxidase TOPNOTCH
E. HMG CoA synthetase MD)

679 This vitamin deficiency may result secondary to Tryptophan is a precursor for synthesis of Niacin (Vit HAZEL MIDTERM 2 -
decreased absorption of tryptophan, also known as B3). KAREN AUG 2013
Hartnup disease. RAZ, MD
A. Thiamine (TOP 6 -
B. Riboflavin FEB 2013
C. Niacin MED
D. Pyridoxine BOARDS;
E. Folic acid TOPNOTCH
MD)
680 Protein - energy malnutrition which occurs more often Kwashiorkor is a form of malnutrition wherein there HAZEL MIDTERM 2 -
in children >1y/o, with hair and skin changes, edema is adequate intake of calories but with low protein KAREN AUG 2013
and stunted growth. Plasma protein is very low when content, which leads to the above symptoms. RAZ, MD
measured. Marasmus on the other hand is due to lack of calories (TOP 6 -
A. Kwashiorkor but with adequate protein. FEB 2013
B. Marasmus MED
C. Cachexia BOARDS;
D. Anorexia TOPNOTCH
E. None of the above MD)

681 The model proposed by Watson, Crick and Wilkins A. Hydrogen bond (pp. 311, Harper's Biochemistry, MICHELLE MIDTERM 1 -
consists of a double-stranded helical structure of the B 27th ed.) JAY AUG 2013
form of DNA. Several bonds maintain the integrity of the Notes: FRANCISC
DNA molecule. What bond is found in between the • van der Waals and hydrophobic interactions - O, MD (TOP
purine and pyrimidine bases of its respective linear between stacked adjacent base pairs 9 - FEB
molecules holding the two strands together of the • 3'-5'-phosphodiester bridges - connects monomeric 2013 MED
double-stranded helix? units of DNA to become polymeric forms BOARDS;
A. Hydrogen bond TOPNOTCH
B. van der Waals MD)
C. Hydrophobic interactions
D. 3'-5'-phosphodiester bridges
682 In the prokaryotic gene expression, what is the genetic C. Cistron (pp 381, Harper's Biochemistry, 27th ed). MICHELLE MIDTERM 1 -
unit coding for the structure of the subunit of a protein SIMILAR TO PREVIOUS BOARD EXAM JAY AUG 2013
molecule, acting as it does as the smallest unit of genetic CONCEPT/PRINCIPLE. FRANCISC
expression? O, MD (TOP
A. Operon 9 - FEB
B. Intron 2013 MED
C. Cistron BOARDS;
D. Exon TOPNOTCH
MD)

TOPNOTCH MEDICAL BOARD PREP BIOCHEMISTRY SUPEREXAM Page 87 of 92


For inquiries visit www.topnotchboardprep.com.ph or email us at topnotchmedicalboardprep@gmail.com
TOPNOTCH MEDICAL BOARD PREP BIOCHEMISTRY SUPEREXAM
For inquiries visit www.topnotchboardprep.com.ph or email us at topnotchmedicalboardprep@gmail.com
Item QUESTION EXPLANATION AUTHOR TOPNOTCH
# EXAM
683 The occurrence of the 21st amino acid in proteins is Answer: B. Selenocysteine (pp. 243, Harper's MICHELLE MIDTERM 1 -
uncommon; however, it is present at the active site of Biochemistry, 27th ed.) JAY AUG 2013
several human enzymes that catalyze redox reactions. Notes: Keshan Disease = selenium deficiency FRANCISC
Replacement of this amino acid with another can result cardiomyopathy O, MD (TOP
in significantly decreased catalytic activity and might 9 - FEB
impair a kind of human protein which has been 2013 MED
implicated in tumorigenesis, atherosclerosis and Keshan BOARDS;
disease. What is this amino acid? TOPNOTCH
A. Isoleucine MD)
B. Selenocysteine
C. Hydroxyproline
D. Selenoleucine

684 The Kreb's-Henseleit Cycle is a pathway present only in Answer: A. Carbamoyl Phosphate Synthetase-1 MICHELLE MIDTERM 1 -
the liver useful in the removal of nitrogenous waste (Biochemistry, Topnotch Handouts). Notes: This JAY AUG 2013
products in the body. The "bottleneck" reaction and the question can be rephrased into: What is the rate- FRANCISC
metabolite flux of this pathway is reduced when the limiting (bottleneck) enzyme for urea cycle? *Kreb's- O, MD (TOP
catalytic efficiency or quantity of what catalyst is Henseleit Cycle (Urea Cycle/Ornithine Cycle) is NOT 9 - FEB
decreased? synonymous with Kreb's Cycle (TCA). 2013 MED
A. Carbamoyl Phosphate Synthetase-1 BOARDS;
B. Phosphofructokinase-1 TOPNOTCH
C. Isocitrate Dehydrogenase MD)
D. Carnitine Palmitoyl Transferase-1
685 5. The different etiologies of jaundice can be classified Answer: B. Hemolytic Anemia (pp. 292, Harper's MICHELLE MIDTERM 1 -
into prehepatic, hepatic and posthepatic causes. Biochemistry, 27th ed.) JAY AUG 2013
Differentiation of these causes can be evaluated by Notes: The presence of bilirubin in the urine is FRANCISC
determination of bilirubin levels in the serum, urine and sometimes referred to as Choluria. O, MD (TOP
feces. What is the clinical significance of acholuric • Hepatitis and obstruction of CBD = choluric jaundice 9 - FEB
jaundice? • Hemolytic anemia = acholuric jaundice 2013 MED
A. Normal • Normal = (obviously) absent jaundice and absent BOARDS;
B. Hemolytic Anemia urine bilirubin TOPNOTCH
C. Hepatitis MD)
D. Obstructive Jaundice
686 Impairment of oxidation of fatty acids gives rise to Answer: C. Phytanic Acid (pp. 194, Harper's MICHELLE MIDTERM 1 -
several diseases. An example of which is Refsum's Biochemistry, 27th ed.) JAY AUG 2013
disease, a rare neurologic disorder due to a metabolic Notes: FRANCISC
defect resulting in the accumulation of a certain • Jamaican Vomiting Sickness = Hypoglycin (from O, MD (TOP
substance found in dairy products and ruminant fat and Akee Tree) = inactivates medium- and short-chain 9 - FEB
meat. What is this substance thought to have acyl-CoA Dehydrogenase 2013 MED
pathological effects on membrane function, protein • Dicarboxylic Aciduria = w-dicarboxylic acids = lack BOARDS;
prenylation and gene expression? mitochondrial medium-chain acyl-CoA TOPNOTCH
A. Hypoglycin Dehydrogenase MD)
B. Dicarboxylic Acid • Refsum disease = Phytanic Acid
C. Phytanic Acid • Zellweger's (cerebrohepatorenal) syndrome =
D. Polyenoic Acid accum. polyenoic acid in the brain =absence of
peroxisomes in all tissues
• *This topic came out during Feb 2013 Physician
Licensure Exam.
687 There are several types of mutation which involves any Answer: B. Missense (Biochemistry, Topnotch MICHELLE MIDTERM 1 -
permanent heritable change in the DNA base sequence of Handouts) JAY AUG 2013
an organism. What type of mutation causes a possible FRANCISC
decrease in protein function because the new codon O, MD (TOP
specifies for a different amino acid? 9 - FEB
A. Silent 2013 MED
B. Missense BOARDS;
C. Nonsense TOPNOTCH
D. Frame shift MD)

688 Blotting is a method used to study macromolecules like Answer: A. Eastern Blot (pp. 20, Comment Section, MICHELLE MIDTERM 1 -
DNA, RNA and proteins which are separated by gel Rush University Medical Center, Review of Surgery, JAY AUG 2013
electrophoresis and transferred into a carrier. The 5th edition) FRANCISC
macromolecules can then be visualized by specific Notes: Southern Blot - DNA sample, DNA probe; O, MD (TOP
probes or staining methods. Which of the following Northern Blot - RNA or mRNA sample, DNA probe; 9 - FEB
methods is a modification of the Western Blot Technique Western Blot - Protein Sample, Antibody Probe; 2013 MED
by detecting posttranslational modification of proteins? Eastern Blot - Posttranslational modification of BOARDS;
A. Eastern Blot protein (sample); Southwestern Blot - DNA-binding TOPNOTCH
B. Polymerase Chain Reaction proteins (sample). MD)
C. Southwestern Blot
D. Northern Blot
689 Which of the following conditions involve severe Answer: B. Fatal Infantile Mitochondrial Myopathy MICHELLE MIDTERM 1 -
diminution or absence of most oxidoreductases of the (Biochemistry, Topnotch Handouts) JAY AUG 2013
respiratory chain as this affects all complexes of the FRANCISC
ETC? O, MD (TOP
A. MELAS 9 - FEB
B. Fatal Infantile Mitochondrial Myopathy 2013 MED
C. Leber's Hereditary Optic Neuropathy BOARDS;
D. Kearns-Sayre Syndrome TOPNOTCH
MD)

TOPNOTCH MEDICAL BOARD PREP BIOCHEMISTRY SUPEREXAM Page 88 of 92


For inquiries visit www.topnotchboardprep.com.ph or email us at topnotchmedicalboardprep@gmail.com
TOPNOTCH MEDICAL BOARD PREP BIOCHEMISTRY SUPEREXAM
For inquiries visit www.topnotchboardprep.com.ph or email us at topnotchmedicalboardprep@gmail.com
Item QUESTION EXPLANATION AUTHOR TOPNOTCH
# EXAM
690 Immunoglobulins (Ig) play a key role in the defense Answer: C. IgM and secretory IgA (pp 601, Harper’s MICHELLE MIDTERM 1 -
mechanism of the body. Each class of Ig has its own Biochemistry, 27th edition) JAY AUG 2013
distinct property for it to confer its specific function. Notes: FRANCISC
Among the 5 classes, based on its structure, which of the • serum IgA – may exist both as a monomer and a O, MD (TOP
Ig may exist as a dimer? dimer 9 - FEB
A. IgG1 and IgM • secretory IgA – exist as a dimer with a secretory 2013 MED
B. serum IgA and IgD component BOARDS;
C. IgM and secretory IgA • IgM – monomer, dimer or pentamer TOPNOTCH
D. IgD and IgG1 • IgD, IgG, IgE – monomer MD)
691 An adult man suffered from stable angina pectoris for 15 Fabry disease is an X-linked disorder that results from MICHELLE MIDTERM 1 -
yrs. During which time there was progressive heart a deficiency in alpha-galactosidase A. This leads to JAY AUG 2013
failure and repeated pulmonary thromboembolism, on deposition of neutral glycosphingolipids. Most FRANCISC
his death at age 63, autopsy disclosed enormous affected tissues are heart, kidneys and eyes. O, MD (TOP
cardiomyopathy (1100 g), cardiac storage of 9 - FEB
globotriaosylceramide (11 mg lipid/g wet weight) and 2013 MED
restricted cardiocytes. Which of the following lipid BOARDS;
storage disease would result in these clinical findings? TOPNOTCH
A. Fabry disease MD)
B. Gaucher disease
C. Krabbe disease
D. Niemann-Pick disease type 1A
E. Tay-Sachs disease

692 Lipoprotein lipase (LPL) is the endothelial cell- The presence of Apo CII on the surfaces of lipoprotein MICHELLE MIDTERM 1 -
associated enzyme necessary for release of fatty acid particles is necessary for the activation of endothelial JAY AUG 2013
from circulating lipoproteins. Which of the following cell LPL. FRANCISC
apoplipoproteins is required to activate LPL-mediated O, MD (TOP
release of fatty acids from chylomicrons? 9 - FEB
A. Apo A 2013 MED
B. Apo B100 BOARDS;
C. Apo B48 TOPNOTCH
D. Apo CII MD)
E. Apo E

693 Hemochromatosis, a disorder that is the result of excess Hemochromatosis is a disorder in iron metabolism MICHELLE MIDTERM 1 -
iron accumulation, is caused by deficiencies in which of that is characterized by excess iron absorption, JAY AUG 2013
the following proteins? saturation of iron-binding proteins and deposition of FRANCISC
A. Divalent metal transporter-1 (DMT-1) hemosiderin in the tissues. O, MD (TOP
B. Human leukocyte antigen (HLA) complex iron protein 9 - FEB
(HFE) 2013 MED
C. Ferritin BOARDS;
D. Ferroportin TOPNOTCH
E. Transferrin MD)
694 One important function of nitric oxide is the induction of NO is generated from arginine catalyzed ny NOS. MICHELLE MIDTERM 1 -
vascular smooth muscle relaxation in response to Other product is citrulline. JAY AUG 2013
acetylcholine. The production of NO requires which FRANCISC
amino acid? O, MD (TOP
A. Arginine 9 - FEB
B. Asparagines 2013 MED
C. Cysteine BOARDS;
D. Glutamine TOPNOTCH
E. Lysine MD)

695 Hepatocytes deliver ketone bodies to the circulation Succinuylo CoA Acetoacetate CoA transferase is the MICHELLE MIDTERM 1 -
because they lack which of the following enzymes? long name of thiophorase which is an enzyme not JAY AUG 2013
A. Beta-hydroxybutyrate dehdrogenase found in the liver FRANCISC
B. Hydroxymethyglutaryl-CoA-lyase O, MD (TOP
C. Hydroxymethylglutaryl-CoA-synthetase 9 - FEB
D. Succinyl-CoA-acetoacetate-CoA-tranferase 2013 MED
E. Beta-ketothiasolase to hydrolyze acetoacetyl-CoA BOARDS;
TOPNOTCH
MD)
696 The inability to rapidly synthesize DNA during the MICHELLE MIDTERM 1 -
process of erythrocyte maturation leads to abnormally JAY AUG 2013
enlarged erythrocytes. This disorder referred to FRANCISC
macrocytic anemia and is caused by a deficiency in O, MD (TOP
which of the following vitamins? 9 - FEB
A. Ascorbate 2013 MED
B. Biotin BOARDS;
C. Folate TOPNOTCH
D. Niacin MD)
E. Thiamine

697 Hypersensitive individuals have IgE to specific antigens PAF is a unique complex lipid of the plasmalogen MICHELLE MIDTERM 1 -
on the surface of their WBC. When these individuals are family and functions in hypersensitivity rxns, acute JAY AUG 2013
challenged with antigen, the antigen-IgE complexes inflammatory rxns and anaphylactic shock by inc. FRANCISC
induce synthesis and release of which of the following Vasopermeability, vasodilation and O, MD (TOP
physiologically potent lipids? bronchoconstriction. 9 - FEB
A. Arachidonic acid 2013 MED
B. Leukotriene B4 BOARDS;
C. Platelet activating factor (PAF) TOPNOTCH
D. Prostaglandin E2 MD)
E. Thromboxane A2

TOPNOTCH MEDICAL BOARD PREP BIOCHEMISTRY SUPEREXAM Page 89 of 92


For inquiries visit www.topnotchboardprep.com.ph or email us at topnotchmedicalboardprep@gmail.com
TOPNOTCH MEDICAL BOARD PREP BIOCHEMISTRY SUPEREXAM
For inquiries visit www.topnotchboardprep.com.ph or email us at topnotchmedicalboardprep@gmail.com
Item QUESTION EXPLANATION AUTHOR TOPNOTCH
# EXAM
698 Which of the following represents the enzyme deficiency Essential fructosuria is an autosomal recessive MICHELLE MIDTERM 1 -
that leads essential fructosuria? disorder manifesting benign asymtotomatology due to JAY AUG 2013
A. Fructose 1 phosphate aldolase (aldolase B) lack of fructokinase. FRANCISC
B. Fructose 1,6 bis phosphate aldolase (aldolase A) O, MD (TOP
C. Fructokinase 9 - FEB
D. Hexokinase 2013 MED
E. 6PFK-1 BOARDS;
TOPNOTCH
MD)
699 There is but a single enzyme-catalyzed reaction in the Heme is oxidized, w/ heme ring being opened by MICHELLE MIDTERM 1 -
human body known to generate carbon monoxide as one endoplasmic reticulum enzyme, heme oxygenase. JAY AUG 2013
of its products. Which of the following enzymes FRANCISC
represents the CO-producing reaction? O, MD (TOP
A. Biliverdin reductase 9 - FEB
B. Coproporphyrinogen oxidase 2013 MED
C. Heme oxygenase BOARDS;
D. Protoporphyrin oxidase TOPNOTCH
E. Uroporphysin decarboxylase MD)

700 Obesity, genetic profile and aging all contribute to the The overall level of significant influence is at the level MICHELLE MIDTERM 1 -
development of Type II DM. Which is the most important of skeletal muscle sensitivity to the action of insulin. JAY AUG 2013
additive factor for these 3 conditions in the development FRANCISC
of type II DM? O, MD (TOP
A. Inc.hepatic gluconeogenesis 9 - FEB
B. Inc. Pancreatic glucagon secretion 2013 MED
C. Impaired renal clearance of glucose BOARDS;
D. Inc. Adipose tissue activity TOPNOTCH
E. Muscle resistance to insulin MD)

TOPNOTCH MEDICAL BOARD PREP BIOCHEMISTRY SUPEREXAM Page 90 of 92


For inquiries visit www.topnotchboardprep.com.ph or email us at topnotchmedicalboardprep@gmail.com
TOPNOTCH MEDICAL BOARD PREP BIOCHEMISTRY SUPEREXAM
For inquiries visit www.topnotchboardprep.com.ph or email us at topnotchmedicalboardprep@gmail.com
Item # KEY 88 B 177 A 266 B 355 A
ANSWER 89 A 178 E 267 D 356 B
1 D 90 B 179 E 268 E 357 C
2 A 91 C 180 B 269 B 358 D
3 A 92 B 181 B 270 D 359 C
4 D 93 D 182 D 271 D 360 D
5 A 94 C 183 E 272 D 361 D
6 B 95 C 184 C 273 A 362 D
7 C 96 A 185 E 274 D 363 D
8 B 97 E 186 D 275 C 364 C
9 A 98 C 187 E 276 B 365 D
10 B 99 C 188 C 277 B 366 A
11 A 100 A 189 A 278 A 367 A
12 C 101 D 190 C 279 D 368 B
13 D 102 E 191 C 280 C 369 D
14 A 103 A 192 D 281 B 370 B
15 A 104 B 193 B 282 E 371 D
16 A 105 C 194 A 283 C 372 D
17 C 106 B 195 B 284 A 373 B
18 B 107 D 196 A 285 C 374 C
19 D 108 C 197 E 286 A 375 D
20 C 109 A 198 D 287 D 376 A
21 D 110 A 199 C 288 C 377 D
22 B 111 D 200 D 289 B 378 B
23 B 112 B 201 A 290 E 379 E
24 A 113 E 202 C 291 D 380 E
25 D 114 D 203 B 292 C 381 D
26 E 115 E 204 B 293 B 382 C
27 D 116 B 205 A 294 B 383 D
28 C 117 A 206 C 295 E 384 B
29 E 118 D 207 C 296 A 385 A
30 D 119 B 208 D 297 D 386 C
31 B 120 C 209 D 298 E 387 C
32 A 121 D 210 C 299 C 388 A
33 C 122 B 211 A 300 D 389 B
34 B 123 C 212 B 301 A 390 A
35 D 124 D 213 C 302 B 391 A
36 B 125 A 214 D 303 D 392 C
37 A 126 A 215 E 304 A 393 E
38 A 127 E 216 C 305 C 394 B
39 C 128 A 217 B 306 D 395 A
40 C 129 D 218 D 307 D 396 A
41 E 130 A 219 C 308 A 397 E
42 E 131 D 220 B 309 B 398 C
43 A 132 A 221 C 310 D 399 D
44 B 133 C 222 B 311 A 400 C
45 B 134 A 223 A 312 B 401 C
46 D 135 C 224 A 313 C 402 E
47 C 136 B 225 B 314 D 403 A
48 B 137 A 226 A 315 C 404 B
49 B 138 C 227 B 316 B 405 D
50 A 139 E 228 D 317 A 406 C
51 C 140 C 229 B 318 D 407 D
52 B 141 A 230 B 319 A 408 B
53 A 142 C 231 A 320 C 409 A
54 B 143 B 232 A 321 A 410 D
55 E 144 C 233 B 322 D 411 C
56 A 145 D 234 C 323 D 412 C
57 D 146 E 235 C 324 D 413 D
58 A 147 B 236 A 325 A 414 B
59 C 148 D 237 C 326 E 415 C
60 D 149 B 238 A 327 D 416 A
61 B 150 A 239 A 328 D 417 B
62 E 151 A 240 B 329 C 418 B
63 B 152 D 241 B 330 E 419 D
64 C 153 B 242 A 331 D 420 E
65 A 154 D 243 D 332 C 421 B
66 B 155 A 244 A 333 C 422 C
67 D 156 B 245 B 334 C 423 A
68 D 157 D 246 C 335 C 424 D
69 B 158 D 247 D 336 B 425 C
70 C 159 D 248 E 337 D 426 A
71 E 160 A 249 C 338 A 427 B
72 A 161 B 250 D 339 D 428 D
73 C 162 A 251 E 340 B 429 D
74 E 163 A 252 B 341 C 430 D
75 D 164 D 253 D 342 A 431 A
76 D 165 C 254 B 343 B 432 B
77 E 166 A 255 A 344 C 433 A
78 A 167 C 256 A 345 B 434 C
79 B 168 C 257 B 346 B 435 C
80 E 169 E 258 C 347 D 436 B
81 B 170 B 259 B 348 A 437 C
82 D 171 A 260 D 349 C 438 D
83 B 172 D 261 B 350 D 439 B
84 B 173 B 262 A 351 D 440 E
85 D 174 C 263 C 352 B 441 E
86 A 175 D 264 B 353 B 442 C
87 D 176 E 265 A 354 A 443 C
TOPNOTCH MEDICAL BOARD PREP BIOCHEMISTRY SUPEREXAM Page 91 of 92
For inquiries visit www.topnotchboardprep.com.ph or email us at topnotchmedicalboardprep@gmail.com
TOPNOTCH MEDICAL BOARD PREP BIOCHEMISTRY SUPEREXAM
For inquiries visit www.topnotchboardprep.com.ph or email us at topnotchmedicalboardprep@gmail.com
444 D 533 A 622 B
445 B 534 B 623 B
446 B 535 D 624 B
447 D 536 B 625 C
448 C 537 A 626 A
449 B 538 C 627 E
450 A 539 B 628 D
451 A 540 A 629 E
452 B 541 E 630 C
453 C 542 D 631 C
454 A 543 C 632 B
455 D 544 D 633 E
456 D 545 C 634 A
457 C 546 E 635 E
458 B 547 E 636 C
459 B 548 E 637 E
460 E 549 C 638 B
461 A 550 E 639 E
462 E 551 D 640 C
463 B 552 C 641 A
464 B 553 C 642 C
465 A 554 E 643 B
466 B 555 D 644 A
467 A 556 A 645 D
468 C 557 E 646 B
469 B 558 A 647 D
470 A 559 B 648 C
471 D 560 B 649 B
472 A 561 A 650 B
473 B 562 D 651 A
474 D 563 C 652 B
475 C 564 C 653 E
476 C 565 A 654 B
477 A 566 A 655 E
478 B 567 D 656 A
479 B 568 C 657 C
480 C 569 C 658 C
481 B 570 A 659 E
482 C 571 A 660 A
483 C 572 A 661 A
484 A 573 A 662 A
485 C 574 B 663 C
486 C 575 C 664 A
487 C 576 C 665 A
488 A 577 A 666 A
489 D 578 B 667 D
490 B 579 A 668 C
491 A 580 D 669 D
492 A 581 D 670 B
493 C 582 C 671 A
494 D 583 D 672 B
495 D 584 E 673 A
496 C 585 E 674 B
497 D 586 D 675 C
498 D 587 A 676 B
499 C 588 C 677 D
500 B 589 D 678 B
501 A 590 C 679 C
502 D 591 A 680 A
503 C 592 A 681 A
504 B 593 A 682 C
505 D 594 B 683 B
506 E 595 D 684 A
507 B 596 E 685 B
508 D 597 D 686 C
509 A 598 D 687 B
510 B 599 E 688 A
511 E 600 B 689 B
512 D 601 C 690 C
513 D 602 D 691 A
514 B 603 B 692 D
515 E 604 C 693 B
516 A 605 D 694 A
517 D 606 A 695 D
518 A 607 E 696 C
519 E 608 D 697 C
520 C 609 A 698 C
521 D 610 B 699 C
522 C 611 E 700 E
523 B 612 B
524 C 613 D
525 A 614 B
526 D 615 A
527 D 616 D
528 D 617 E
529 C 618 D
530 A 619 A
531 C 620 D
532 B 621 B
TOPNOTCH MEDICAL BOARD PREP BIOCHEMISTRY SUPEREXAM Page 92 of 92
For inquiries visit www.topnotchboardprep.com.ph or email us at topnotchmedicalboardprep@gmail.com
TOPNOTCH MEDICAL BOARD PREP ANATOMY SUPEREXAM
For inquiries visit www.topnotchboardprep.com.ph or email us at topnotchmedicalboardprep@gmail.com
DEAR TOPNOTCH FRIENDS:

PLEASE FOLLOW THESE INSTRUCTIONS:

1. These questions are previous diagnostic, midterm, and finals exams of Topnotch, almost all of them made by Topnotch Board Exam Topnotchers.
2. Answer this Topnotch Superexam seriously 100-items at a time. Cover the “Explanations” Column. Do not immediately look at the answers from the
answer key. That’s not the correct way of answering sample exams. You need to treat these MCQs as exercises and not as handouts.
3. Time yourself. 1.5 hours per 100-item block.
4. After answering each 100-item block, refer to the Topnotch Answer Key for the correct answers. Please be careful of “frameshift mutations” when
checking your answers – check every 10 items. (the format of the answer key was designed for you to practice against “frameshift mutations”)
5. The Topnotch Superexams are EXERCISES for the actual med boards. They will not appear verbatim in your future exams. More than knowing what’s
the correct answer, it’s more important for you to:
a. Know why the other choices are wrong
b. Know why the other choices were included in the first place
c. Know the explanation to the correct answer
6. Sharpen your mind by answering the Topnotch Superexams. Most of these questions based on past feedback are more difficult than the actual questions
in the med boards. In these exams made by Board Exam Topnotchers, if you’re getting a score of 60/100 , that’s already a good score. More than 80/100
is outstanding.

Item QUESTION EXPLANATION AUTHOR TOPNOTCH
# EXAM
1 Patient had improperly fitted axillary crutch and suffered The affected nerve with improperly fitted crutches KRISTEL DIAGNOSTIC
injury to a branch of the brachial plexus. Which is the most is the radial nerve. Yes, the first impulse for this TANHUI EXAM -
likely finding? question is to think axillary nerve is the answer (TOP 3 - AUG MARCH
A. Weakness of arm abduction because it’s an axillary crutch. BUT THAT’S 2015 MED 2016
B. Hand of benediction WRONG. The correct answer is radial nerve! BOARDS;
C. Ape hand TOPNOTCH
D. Cyclist palsy The axillary nerve, with the posterior humeral MD FROM LA
E. Weakness in extending the forearm and wristdrop circumflex artery actually courses much higher SALLE)
posteriorly via the quadrangular space.
The radial nerve, along with the deep brachial
artery courses posteriorly via the triangular
interval and is more susceptible to injury from the
axilla.

Weakness of arm abduction - axillary nerve
Hand of benediction – Median nerve
Ape hand – median nerve
Cyclist palsy – ulnar nerve
Weakness in extending the forearm – radial nerve

Source: Topnotch handout and pearls in anatomy

2 Which of the following is not involved in the transmission of The fibula is just a bone for anchoring muscles. In KRISTEL DIAGNOSTIC
weight while standing? fact, it’s so insignificant, you can use it for bone TANHUI EXAM -
A. Sacrum grafting! The part used for grafting is the middle (TOP 3 - AUG MARCH
B. Pelvis third because this contains the nutrient artery. 2015 MED 2016
C. Femur BOARDS;
D. Tibia Source: Moore Clinically oriented anatomy 4th ed TOPNOTCH
E. Fibula Chapter 5 (Ebook) MD FROM LA
SALLE)

3 Which is the equivalent layer of the Deep investing fascia of I can’t fit a table here but the ideal way to KRISTEL DIAGNOSTIC
the abdomen in the perineum? memorize the following is to put them in a table TANHUI EXAM -
A. Colles’ fascia side by side: (TOP 3 - AUG MARCH
B. Gallaudet’s fascia 2015 MED 2016
C. Scarpa’s fascia Abdomen (superficial to deep) BOARDS;
D. Dartos fascia Camper’s fascia (fatty superficial layer) TOPNOTCH
E. Buck’s fascia Scarpa’s fascia (membranous superficial layer) MD FROM LA
Deep investing fascia SALLE)

Perineum (superficial to deep)
Fatty superficial layer
Colles’ fascia (membranous superficial layer)
Gallaudet’s fascia/investing fascia

Penis (superficial to deep)
Dartos muscle
Dartos fascia
Buck’s fascia

Source: Moore Clinically oriented anatomy 4th ed
Chapter 2 and 3 (Ebook)

TOPNOTCH MEDICAL BOARD PREP ANATOMY SUPEREXAM Page 1 of 94


For inquiries visit www.topnotchboardprep.com.ph or email us at topnotchmedicalboardprep@gmail.com
TOPNOTCH MEDICAL BOARD PREP ANATOMY SUPEREXAM
For inquiries visit www.topnotchboardprep.com.ph or email us at topnotchmedicalboardprep@gmail.com
Item QUESTION EXPLANATION AUTHOR TOPNOTCH
# EXAM
4 Which of the following is false of the Greater pelvis? Lesser pelvis (true pelvis, pelvis minor) – below KRISTEL DIAGNOSTIC
A. Obstetric significance the arcuate line, obstetric significance TANHUI EXAM -
B. Lies above the linea terminales (TOP 3 - AUG MARCH
C. Pelvis major Source: Moore Clinically oriented anatomy 4th ed 2015 MED 2016
D. False pelvis Chapter 3 (Ebook) BOARDS;
E. None of the above TOPNOTCH
MD FROM LA
SALLE)

5 What is the shape of the left adrenal gland? Left crescent, Right triangle KRISTEL DIAGNOSTIC
A. Triangular TANHUI EXAM -
B. Kidney For those who are math geeks, my mnemonic is (TOP 3 - AUG MARCH
C. Crescent “right triangle” (a triangle with a right angle in it) 2015 MED 2016
D. Oval BOARDS;
E. Round Source: Moore Clinically oriented anatomy 4th ed TOPNOTCH
Chapter 2 (Ebook) MD FROM LA
SALLE)

6 Which of the following cartilages is pyramidal in shape? Arytenoid: 3 sided pyramid KRISTEL DIAGNOSTIC
A. Epiglottis Apex superiorly – bears corniculate cartilage and TANHUI EXAM -
B. Arytenoid attaches to aryepiglottic fold (TOP 3 - AUG MARCH
C. Corniculate Vocal process anteriorly – posterior attachement 2015 MED 2016
D. Cuneiform for vocal cord BOARDS;
E. Thyroid Muscular process laterally – attachment for TOPNOTCH
posterior and lateral cricoarytenoid muscle MD FROM LA
Base: cricoarytenoid joint SALLE)

Source: Moore Clinically oriented anatomy 4th ed
Chapter 8 (Ebook)

7 Which of the following is not true of the quadrangular The vestibular ligament forms the inferior border KRISTEL DIAGNOSTIC
membrane? of the quadrangular membrane. TANHUI EXAM -
A. Extends from the lateral aspects of the arytenoid and (TOP 3 - AUG MARCH
epiglottic cartilage Source: Moore Clinically oriented anatomy 4th ed 2015 MED 2016
B. The aryepiglottic ligament/fold forms the free superior Chapter 8 (Ebook) BOARDS;
margin TOPNOTCH
C. The vestibular ligament/fold forms the free inferior MD FROM LA
margin SALLE)
D. The vestibular fold forms the false vocal cord
E. None of the above

8 What do you call the junction where the nasal bones and SIMILAR TO PREVIOUS BOARD EXAM KRISTEL DIAGNOSTIC
frontal bones meet? CONCEPT/PRINCIPLE. TANHUI EXAM -
A. Inion (TOP 3 - AUG MARCH
B. Rhinion Source: Moore Clinically oriented anatomy 4th ed 2015 MED 2016
C. Glabella Chapter 7 (Ebook) BOARDS;
D. Nasion TOPNOTCH
E. Pterion MD FROM LA
SALLE)

9 FSH is secreted by which type of cell in the pituitary gland? Acidophils – Growth hormone, prolactin KRISTEL DIAGNOSTIC
A. Acidophils Basophils – FSH, LH, TSH, ACTH TANHUI EXAM -
B. Basophils Supraoptic nuclei – Vasopressin mainly (TOP 3 - AUG MARCH
C. Chromophobes Paraventricular nuclei – Oxytocin mainly 2015 MED 2016
D. Supraoptic nuclei BOARDS;
E. Paraventricular nuclei Source: Topnotch TOPNOTCH
MD FROM LA
SALLE)

10 A trauma patient suffers a tear at the junction of the SVC and The SA node is located in the right atrium at the KRISTEL DIAGNOSTIC
the right atrium. This tear would likely damage the: junction where it is joined by the SVC. TANHUI EXAM -
A. SA node (TOP 3 - AUG MARCH
B. AV node Source: Moore Clinically oriented anatomy 4th ed 2015 MED 2016
C. AV bundle Chapter 1 (Ebook BOARDS;
D. Right bundle branch TOPNOTCH
E. Left bundle branch MD FROM LA
SALLE)

11 The lowest level of the lung on end expiration at the Lungs – 6-8-10 KRISTEL DIAGNOSTIC
midaxillary line is? Pleura – 8-10-12 TANHUI EXAM -
A. 6th intercostal space (TOP 3 - AUG MARCH
B. 7th intercostal space Source: Topnotch handout on anatomy 2015 MED 2016
C. 8th intercostal space BOARDS;
D. 9th intercostal space TOPNOTCH
E. 10th intercostal space MD FROM LA
SALLE)

TOPNOTCH MEDICAL BOARD PREP ANATOMY SUPEREXAM Page 2 of 94


For inquiries visit www.topnotchboardprep.com.ph or email us at topnotchmedicalboardprep@gmail.com
TOPNOTCH MEDICAL BOARD PREP ANATOMY SUPEREXAM
For inquiries visit www.topnotchboardprep.com.ph or email us at topnotchmedicalboardprep@gmail.com
Item QUESTION EXPLANATION AUTHOR TOPNOTCH
# EXAM
12 While doing a spinal tap, which describes the most accurate Source: Topnotch handout on anatomy KRISTEL DIAGNOSTIC
sequence of layers from the skin to the subarachnoid space? TANHUI EXAM -
A. Skin, interspinous ligament, dura mater, subarachnoid (TOP 3 - AUG MARCH
space 2015 MED 2016
B. Skin, supraspinous ligament, interspinous ligament, BOARDS;
posterior longitudinal ligament, dura mater, subarachnoid TOPNOTCH
space MD FROM LA
C. Skin, supraspinous ligament, interspinous ligament, dura SALLE)
mater, subarachnoid space
D. Skin supraspinous ligament, intertransverse ligament,
arachnoid space, subarachnoid space
E. Skin, interspinous ligament, anterior longitudinal
ligament, dura mater, subarachnoid space

13 A 5-year-old male presented with painless testicular mass. If Tumor cells from the gonads that metastasize KRISTEL DIAGNOSTIC
the mass proved to be a malignancy, the earliest affected through the lymphatics metastasize to the lumbar TANHUI EXAM -
lymph nodes would be: nodes initially. Remember that the origin of the (TOP 3 - AUG MARCH
A. Lumbar (aortic) lymph nodes gonadal arteries is the aorta, therefore the 2015 MED 2016
B. Inferior mesenteric nodes lymphatics will also course similarly. BOARDS;
C. Deep inguinal nodes TOPNOTCH
D. Common iliac nodes Source: Moore Clinically oriented anatomy 4th ed MD FROM LA
E. Superficial inguinal nodes Chapter 2 (Ebook) SALLE)

14 A surgeon is about to make an incision through the fundus of Lesser curvature – right and left gastric artery KRISTEL DIAGNOSTIC
the stomach, which of the following requires clamping to Greater curvature – right and left gastroomental TANHUI EXAM -
prevent bleeding? artery (TOP 3 - AUG MARCH
A. Right gastric artery Fundus – short gastric artery 2015 MED 2016
B. Left gastric artery BOARDS;
C. Right gastroomental artery Source: Moore Clinically oriented anatomy 4th ed TOPNOTCH
D. Left gastroomental artery Chapter 2 (Ebook) MD FROM LA
E. Short gastric artery SALLE)

15 A surgeon is about to perform cholecystectomy. The Source: Moore Clinically oriented anatomy 4th ed KRISTEL DIAGNOSTIC
gallbladder can be localized in its fossa between which 2 Chapter 2 (Ebook) TANHUI EXAM -
anatomical lobes? (TOP 3 - AUG MARCH
A. Quadrate and left lobes 2015 MED 2016
B. Quadrate and caudate lobes BOARDS;
C. Right and quadrate lobes TOPNOTCH
D. Caudate and right lobes MD FROM LA
E. Caudate and left lobes SALLE)

16 The right pulmonary artery: There are 2 pulmonary arteries – 1 right and 1 left. KRISTEL DIAGNOSTIC
A. Are 2 in number There are 4 pulmonary veins. TANHUI EXAM -
B. Are 4 in number (TOP 3 - AUG MARCH
C. Courses posterior to the ascending aorta and the SVC The right pulmonary artery courses posterior to 2015 MED 2016
D. B and C the ascending aorta and SVC! BOARDS;
E. None of the above TOPNOTCH
Source: Moore Clinically oriented anatomy 4th ed MD FROM LA
Chapter 1 (Ebook) SALLE)

17 What happens in May-Thurner syndrome? This condition predisposes to Deep vein KRISTEL DIAGNOSTIC
A. The aorta compresses the IVC thrombosis. TANHUI EXAM -
B. The left common iliac vein is compressed by the left (TOP 3 - AUG MARCH
common iliac artery 2015 MED 2016
C. The right common iliac vein is compressed by the right BOARDS;
common iliac artery TOPNOTCH
D. The left common iliac vein is compressed by the right MD FROM LA
common iliac artery SALLE)
E. The right common iliac vein is compressed by the left
common iliac artery

18 Which of the following is a syndesmosis? Source: Topnotch handout on anatomy KRISTEL DIAGNOSTIC
A. Interosseus membrane TANHUI EXAM -
B. Gomphosis (TOP 3 - AUG MARCH
C. Distal tibiofibular joint 2015 MED 2016
D. A and B BOARDS;
E. All of the above TOPNOTCH
MD FROM LA
SALLE)

TOPNOTCH MEDICAL BOARD PREP ANATOMY SUPEREXAM Page 3 of 94


For inquiries visit www.topnotchboardprep.com.ph or email us at topnotchmedicalboardprep@gmail.com
TOPNOTCH MEDICAL BOARD PREP ANATOMY SUPEREXAM
For inquiries visit www.topnotchboardprep.com.ph or email us at topnotchmedicalboardprep@gmail.com
Item QUESTION EXPLANATION AUTHOR TOPNOTCH
# EXAM
19 Kiesselbach’s plexus is vascular anastomosis on the anterior Arterial supply KRISTEL DIAGNOSTIC
part of the nose. Juan dela Cruz damaged this plexus while Sphenopalatine artery (Maxillary artery) TANHUI EXAM -
picking his nose. Which of the following is least likely to Anterior and posterior ethmoidal artery (TOP 3 - AUG MARCH
contribute to the epistaxis? (ophthalmic artery branch) 2015 MED 2016
A. Ophthalmic artery Greater palatine artery (Maxillary artery) BOARDS;
B. Maxillary artery Superior labial artery and lateral nasal branches of TOPNOTCH
C. Mandibular artery the facial artery MD FROM LA
D. Facial artery SALLE)
E. Ethmoidal artery Source: Moore Clinically oriented anatomy 4th ed
Chapter 7 (Ebook)

20 Which of the following is has a brush border lining its lumen? The brush border differentiates the proximal from KRISTEL DIAGNOSTIC
A. Small intestine the distal convoluted tubule. TANHUI EXAM -
B. Proximal convoluted tubule (TOP 3 - AUG MARCH
C. Distal convoluted tubule Source: Topnotch handout on anatomy 2015 MED 2016
D. A and B BOARDS;
E. All of the above TOPNOTCH
MD FROM LA
SALLE)

21 The inner ear is housed within which bone? SIMILAR TO PREVIOUS BOARD EXAM LESTER MIDTERM 1
A. Temporal CONCEPT/PRINCIPLE. A few ENT questions were BRYAN CO EXAM -
B. Sphenoid present in our Anatomy exam, and were generally (TOP 10 - MARCH
C. Ethmoid tougher than the anatomy questions. AUG 2015 2016
D. Maxillary MED
E. Zygomatic BOARDS;
TOPNOTCH
MD FROM
UST)
22 Which type of pelvis presents with a larger AP diameter SIMILAR TO PREVIOUS BOARD EXAM LESTER MIDTERM 1
compared to the transverse diameter? CONCEPT/PRINCIPLE. Anthropoid=AP BRYAN CO EXAM -
A. Gynecoid diameter>transverse; Platypelloid (TOP 10 - MARCH
B. Android ("flatypelloid")=transverse diameter>AP diameter. AUG 2015 2016
C. Anthropoid Both came out in 2 seaparate questions during our MED
D. Platypelloid Anatomy exam. BOARDS;
E. Arthropod TOPNOTCH
MD FROM
UST)
23 Correct boundaries of the greater sciatic foramen except: SIMILAR TO PREVIOUS BOARD EXAM LESTER MIDTERM 1
A. anterolaterally by the greater sciatic notch CONCEPT/PRINCIPLE. The piriformis muscle BRYAN CO EXAM -
B. posteromedially by the sacrotuberous ligament divides the greater sciatic foramen into a (TOP 10 - MARCH
C. inferiorly by the sacrospinous ligament and ischial spine suprapiriform and infrapiriform foramen. There AUG 2015 2016
D. inferolaterally by the piriformis muscle were about 3 questions about the greater sciatic MED
E. superiorly by the anterior sacroilliac ligament notch in our Anatomy exam. Expect a few difficult BOARDS;
questions about pelvic anatamy TOPNOTCH
MD FROM
UST)
24 Secretes surfactant in the lung: SIMILAR TO PREVIOUS BOARD EXAM LESTER MIDTERM 1
A. Type I alveolar CONCEPT/PRINCIPLE. The few histology BRYAN CO EXAM -
B. Type II alveolar cells questions asked during our anatomy exam were (TOP 10 - MARCH
C. Kulchitsky cells generally easy. AUG 2015 2016
D. Clara cells MED
E. B, C, D are correct. BOARDS;
TOPNOTCH
MD FROM
UST)
25 A 63 year-old male smoker develops an apical sulcus tumor SIMILAR TO PREVIOUS BOARD EXAM LESTER MIDTERM 1
that compresses the brachial plexus and cervical stellate CONCEPT/PRINCIPLE. Thoracic outlet and inlet BRYAN CO EXAM -
ganglion. This may lead to: syndrome generally pertain to the same thing. (TOP 10 - MARCH
A. Pancoast syndrome Horner syndrome involves the cervical AUG 2015 2016
B. Superior vena cava syndrome sympathetic trunk. 2 questions about this topic in MED
C. Horner's syndrome our Anatomy exam. BOARDS;
D. Thoracic outlet syndrome TOPNOTCH
E. Thoracic inlet syndrome MD FROM
UST)

26 To pass a needle into the pleural space in the midaxillary LESTER MIDTERM 1
line, the following structures will have to be pierced except? BRYAN CO EXAM -
A. Internal intercostals (TOP 10 - MARCH
B. Levatores costarum AUG 2015 2016
C. External intercostals MED
D. Parietal pleura BOARDS;
E. Innermost intercostals TOPNOTCH
MD FROM
UST)
27 On percussing the anterior chest of a patient, you find the The right border of the heart is formed by the LESTER MIDTERM 1
right margin of the heart to lie 5 cm to right of the edge of the right atrium. Inferior border is mostly the right BRYAN CO EXAM -
sternum. Which chamber of the heart is likely to be ventricle with the apex of the left ventricle (TOP 10 - MARCH
enlarged? A. left ventricle B. left atrium C. right ventricle D. AUG 2015 2016
right atrium E. both ventricles MED
BOARDS;
TOPNOTCH
MD FROM
UST)

TOPNOTCH MEDICAL BOARD PREP ANATOMY SUPEREXAM Page 4 of 94


For inquiries visit www.topnotchboardprep.com.ph or email us at topnotchmedicalboardprep@gmail.com
TOPNOTCH MEDICAL BOARD PREP ANATOMY SUPEREXAM
For inquiries visit www.topnotchboardprep.com.ph or email us at topnotchmedicalboardprep@gmail.com
Item QUESTION EXPLANATION AUTHOR TOPNOTCH
# EXAM
28 The following structures form the walls of the inguinal canal LESTER MIDTERM 1
except? BRYAN CO EXAM -
A. Conjoint tendon (TOP 10 - MARCH
B. Aponeurosis of external obliique muscle AUG 2015 2016
C. Internal oblique muscle MED
D. Lacunar ligament BOARDS;
E. Fascia transversalis TOPNOTCH
MD FROM
UST)
29 To pass a needle into the cavity of the tunica vaginalis in the SIMILAR TO PREVIOUS BOARD EXAM LESTER MIDTERM 1
scrotum, the following structures have to be pierced except? CONCEPT/PRINCIPLE. The tunica albuginea is the BRYAN CO EXAM -
A. Dartos muscle fibrous covering of the testis. There were also (TOP 10 - MARCH
B. Colles' fascia other questions about Colles's and Scarpa's fascia. AUG 2015 2016
C. Tunica albuginea MED
D. Internal spermatic fascia BOARDS;
E. Cremasteric fascia TOPNOTCH
MD FROM
UST)
30 The hilum of the right kidney contains the following The right suprarenal adrenal gland does not LESTER MIDTERM 1
important structures except? extend downward to the hilum of the right kidney. BRYAN CO EXAM -
A. Renal pelvis (TOP 10 - MARCH
B. Tributaries of right renal vein AUG 2015 2016
C. Sympathetic nerve fibers MED
D. Part of the right suprarenal gland BOARDS;
E. Branches of the right renal artery TOPNOTCH
MD FROM
UST)
31 A 53-year-old man has difficulty with breathing through his The posterior ethmoidal sinus opens into the LESTER MIDTERM 1
nose. On examination, his physician finds that he has superior nasal meatus. The maxillary, frontal, and BRYAN CO EXAM -
swelling of the mucous membranes of the superior nasal anterior and middle ethmoidal sinuses drain into (TOP 10 - MARCH
meatus. Which opening of the paranasal sinuses is most the middle nasal meatus. AUG 2015 2016
likely plugged? MED
A. Middle ethmoidal sinus BOARDS;
B. Maxillary sinus TOPNOTCH
C. Posterior ethmoidal sinus MD FROM
D. Anterior ethmoidal sinus UST)
E. Frontal sinus

32 An oncologist is reviewing a CT scan of a 74-year-old man The quadrate lobe of the liver drains bile into the LESTER MIDTERM 1
with newly diagnosed hepatocellular carcinoma. He locates left hepatic duct and receives blood from the left BRYAN CO EXAM -
the affected quadrate lobe of the liver that: hepatic artery. It lies between the gallbladder (TOP 10 - MARCH
A. Lies between the IVC and ligamentum venosum fossa and the ligamentum teres hepatic, is a medial AUG 2015 2016
B. Receives blood from the right hepatic artery inferior segment, and is a part of the left lobe. MED
C. Drains bile into the left hepatic duct BOARDS;
D. Is a medial superior segment TOPNOTCH
E. Is functionally a part of the right lobe MD FROM
UST)

33 In a patient with portal hypertension, which of the following The right colic vein belongs to the portal venous LESTER MIDTERM 1
veins is most likely to be dilated? system and empties into the superior mesenteric BRYAN CO EXAM -
A. Right colic vein vein, which joins the splenic vein to form the (TOP 10 - MARCH
B. Inferior epigastric vein portal vein. The inferior epigastric, AUG 2015 2016
C. Inferior phrenic vein inferior phrenic, suprarenal, and ovarian veins MED
D. Suprarenal vein belong to the systemic (or caval) venous system BOARDS;
E. Ovarian vein and drain directly or indirectly into the IVC. TOPNOTCH
MD FROM
UST)
34 A 2-year-old boy presents with pain in his groin that has The deep inguinal ring lies in the transversalis LESTER MIDTERM 1
been increasing over the past few weeks. He is found to have fascia, just lateral to the inferior epigastric vessels. BRYAN CO EXAM -
a The superficial inguinal ring is in the aponeurosis (TOP 10 - MARCH
degenerative malformation of the transversalis fascia during of the external oblique muscle. The inguinal AUG 2015 2016
development. Which of the following structures on the ligament and the anterior wall of the inguinal MED
anterior abdominal wall is likely defective? canal are formed by the aponeurosis of the BOARDS;
A. Superficial inguinal ring external oblique muscle. The sac of a direct TOPNOTCH
B. Deep inguinal ring inguinal hernia is formed by the peritoneum. MD FROM
C. Inguinal ligament UST)
D. Sac of a direct inguinal hernia
E. Anterior wall of the inguinal canal

35 A pediatric surgeon is resecting a possible malignant mass The left umbilical vein becomes the round LESTER MIDTERM 1
from the liver of a neonate with cerebral palsy. The surgeon ligament of the liver after birth. The right BRYAN CO EXAM -
divides the round ligament of the liver during surgery. A umbilical vein does not leave a fibrous remnant (TOP 10 - MARCH
fibrous remnant of which of the following fetal vessels is because it degenerates during the early embryonic AUG 2015 2016
severed? period. The ductus venosus forms the ligamentum MED
A. Ductus venosus venosum; the ductus arteriosus forms the BOARDS;
B. Ductus arteriosus ligamentum arteriosum; the umbilical artery TOPNOTCH
C. Left umbilical vein forms the medial umbilical MD FROM
D. Right umbilical vein ligament. UST)
E. Umbilical artery

TOPNOTCH MEDICAL BOARD PREP ANATOMY SUPEREXAM Page 5 of 94


For inquiries visit www.topnotchboardprep.com.ph or email us at topnotchmedicalboardprep@gmail.com
TOPNOTCH MEDICAL BOARD PREP ANATOMY SUPEREXAM
For inquiries visit www.topnotchboardprep.com.ph or email us at topnotchmedicalboardprep@gmail.com
Item QUESTION EXPLANATION AUTHOR TOPNOTCH
# EXAM
36 A 33-year-old patient is suffering from a sudden occlusion at The first two posterior intercostal arteries are LESTER MIDTERM 1
the origin of the descending (thoracic) aorta. This condition branches of the highest (superior) intercostal BRYAN CO EXAM -
would most likely decrease blood flow in which of the artery of the costocervical trunk; the remaining (TOP 10 - MARCH
following intercostal arteries? nine branches are from the thoracic aorta. The AUG 2015 2016
A. Upper six anterior internal thoracic artery gives off the upper six MED
B. All of the posterior
anterior intercostal arteries and is divided into the BOARDS;
C. Upper two posterior superior epigastric and musculophrenic arteries, TOPNOTCH
D. Lower anterior which gives off anterior intercostal arteries in the MD FROM
E. Lower six posterior 7th, 8th, and 9th intercostal spaces and ends in the UST)
10th intercostal space where it anastomoses with
the deep circumflex iliac artery.
37 The following statements concerning the left suprarenal The medulla is innervated by preganglionic LESTER MIDTERM 1
gland are correct except? sympathetic fibers. BRYAN CO EXAM -
A. The gland extends along the medial border of the left (TOP 10 - MARCH
kidney from the upper pole to the hilus. AUG 2015 2016
B. The left suprarenal vein drains into the left renal vein. MED
C. The gland is separated from the left kidney by perirenal BOARDS;
fat. TOPNOTCH
D. The gland lies behind the lesser sac. MD FROM
E. The medulla is innervated by post-ganglionic UST)
sympathetic nerve fibers

38 A thoracic surgeon removed the right middle lobar bronchus The right middle lobar (secondary) bronchus LESTER MIDTERM 1
along with lung tissue from a 57-year-old heavy smoker with leads to the medial and lateral bronchopulmonary BRYAN CO EXAM -
lung cancer. Which of the following bronchopulmonary segments. The right superior lobar bronchus (TOP 10 - MARCH
segments must contain cancerous tissues? divides into the superior, posterior, and anterior AUG 2015 2016
A. Medial and lateral segmental (tertiary) bronchi. The right inferior MED
B. Anterior and posterior lobar bronchus has the anterior, lateral, posterior, BOARDS;
C. Anterior basal and medial basal and anterior segmental bronchi. TOPNOTCH
D. Anterior basal and posterior basal MD FROM
E. Lateral basal and posterior basal UST)

39 A 19-year-old man came to the emergency department, and The middle cardiac vein ascends in the posterior LESTER MIDTERM 1
his angiogram exhibited that he was bleeding from the vein interventricular groove, accompanied by the BRYAN CO EXAM -
that is accompanied by the posterior interventricular artery. posterior interventricular branch of the right (TOP 10 - MARCH
Which of the following veins is most likely to be ruptured? coronary artery. The great cardiac vein is AUG 2015 2016
A. Great cardiac vein accompanied by the anterior interventricular MED
B. Middle cardiac vein artery, the anterior cardiac vein drains directly BOARDS;
C. Anterior cardiac vein into the right atrium, and the small cardiac vein is TOPNOTCH
D. Small cardiac vein accompanied by the marginal artery. MD FROM
E. Oblique veins of the left atrium UST)

40 After an automobile accident, a back muscle that forms the The latissimus dorsi forms boundaries of the LESTER MIDTERM 1
boundaries of the triangle of auscultation and the lumbar auscultation and lumbar triangles and receives BRYAN CO EXAM -
triangle receives no blood. Which of the following muscles blood from the thoracodorsal artery. The levator (TOP 10 - MARCH
might be ischemic? scapulae, rhomboid minor, and splenius capitis AUG 2015 2016
A. Levator scapulae muscles do not form boundaries of these two MED
B. Rhomboid minor triangles. The trapezius muscle forms a boundary BOARDS;
C. Latissimus dorsi of the auscultation triangle but not the lumbar TOPNOTCH
D. Trapezius triangle. The levator scapulae, rhomboid minor, MD FROM
E. Splenius capitis and trapezius muscles receive blood from the UST)
transverse cervical artery. The splenius capitis
muscle receives blood from the occipital and
transverse cervical arteries.
41 The isthmus of the thyroid gland lies anterior to the GEORGE MIDTERM 2
A. Cricoid cartilage MICHAEL EXAM -
B. Thyroid cartilage SOSUAN MARCH
C. 2-3 tracheal rings (TOP 5 - AUG 2016
D. 4-5 tracheal rings 2015 MED
E. NOTA BOARDS;
TOPNOTCH
MD FROM
UST)
42 The nasal septum is comprised of the following, except: GEORGE MIDTERM 2
A. Septal cartilage MICHAEL EXAM -
B. Cribiform plate of ethmoid SOSUAN MARCH
C. Vomer (TOP 5 - AUG 2016
D. Perpendicular plate of ethmoid 2015 MED
E. NOTA BOARDS;
TOPNOTCH
MD FROM
UST)
43 The structure that drains into the inferior nasal meatus all other choices drain to the middle nasal meatus GEORGE MIDTERM 2
A. Nasolacrimal duct MICHAEL EXAM -
B. Anterior ethmoid sinus SOSUAN MARCH
C. Frontal sinus (TOP 5 - AUG 2016
D. Maxillary sinus 2015 MED
E. NOTA BOARDS;
TOPNOTCH
MD FROM
UST)
TOPNOTCH MEDICAL BOARD PREP ANATOMY SUPEREXAM Page 6 of 94
For inquiries visit www.topnotchboardprep.com.ph or email us at topnotchmedicalboardprep@gmail.com
TOPNOTCH MEDICAL BOARD PREP ANATOMY SUPEREXAM
For inquiries visit www.topnotchboardprep.com.ph or email us at topnotchmedicalboardprep@gmail.com
Item QUESTION EXPLANATION AUTHOR TOPNOTCH
# EXAM
44 Aqueous humor is produced by the The aqueous humor is produced by the ciliary GEORGE MIDTERM 2
A. Choroid process of the uveal tract and drained into the MICHAEL EXAM -
B. Ciliary process anterior chamber angle. SOSUAN MARCH
C. Cornea (TOP 5 - AUG 2016
D. Retina 2015 MED
E. Iris BOARDS;
TOPNOTCH
MD FROM
UST)
45 Origins of the arterial supply to the rectum, except Superior rectal artery from the inferior GEORGE MIDTERM 2
A. Superior mesenteric artery mesenteric; Middle rectal artery from the internal MICHAEL EXAM -
B. Inferior mesenteric artery iliac artery; inferior rectal artery from the internal SOSUAN MARCH
C. Internal iliac artery pudendal artery from the internal iliac artery (TOP 5 - AUG 2016
D. Internal pudendal artery 2015 MED
E. NOTA BOARDS;
TOPNOTCH
MD FROM
UST)
46 Blood supply ot the liver includes 75% from the portal vein and 25%from the GEORGE MIDTERM 2
A. 25% from portal vein hepatoc artery MICHAEL EXAM -
B. 75% from hepatic artery SOSUAN MARCH
C. 75% from portal vein (TOP 5 - AUG 2016
D. 20% from the celiac artery 2015 MED
E. Both A and B BOARDS;
TOPNOTCH
MD FROM
UST)
47 Most common used vein in the antecubittal fossa for GEORGE MIDTERM 2
phlebotomy MICHAEL EXAM -
A. Median cubittal vein SOSUAN MARCH
B. Brachial vein (TOP 5 - AUG 2016
C. Basilic vein 2015 MED
D. Cephalic vein BOARDS;
E. AOTA TOPNOTCH
MD FROM
UST)
48 Divides the axillary artery into three parts GEORGE MIDTERM 2
A. Teres major MICHAEL EXAM -
B. Teres minor SOSUAN MARCH
C. Scalene medius (TOP 5 - AUG 2016
D. Pectoralis major 2015 MED
E. Pectoralis minor BOARDS;
TOPNOTCH
MD FROM
UST)
49 Support of the uterus is provided by the following, except: The other choices comprise the endopelvic fascia GEORGE MIDTERM 2
A. Uterosacral ligament MICHAEL EXAM -
B. Cardinal ligament SOSUAN MARCH
C. Ileococcygeus (TOP 5 - AUG 2016
D. Puborectalis 2015 MED
E. Broad ligament BOARDS;
TOPNOTCH
MD FROM
UST)
50 The left primary bronchus GEORGE MIDTERM 2
A. Has one eparterial and one hyparterial bronchi MICHAEL EXAM -
B. Is narrower and shorter than the right SOSUAN MARCH
C. Has the aorta arching over it (TOP 5 - AUG 2016
D. Has a more vertical direction than the right 2015 MED
E. NOTA BOARDS;
TOPNOTCH
MD FROM
UST)
51 A fluid found in the membranous labyrinth Endolymph is the fluid contained in the GEORGE MIDTERM 2
A. Perilymph membranous labyrinth of the inner ear and rich in MICHAEL EXAM -
B. Corticolymph potassium. SOSUAN MARCH
C. Blood (TOP 5 - AUG 2016
D. Endolymph 2015 MED
E. NOTA BOARDS;
TOPNOTCH
MD FROM
UST)
52 A pair of opening on the lateral wall of the oral cavity The Stensen's duct or parotid duct opens opposite GEORGE MIDTERM 2
A. Ducts of Rivinius the 2nd upper molar. MICHAEL EXAM -
B. Wharton's duct SOSUAN MARCH
C. Stensen's duct (TOP 5 - AUG 2016
D. Boath A and B 2015 MED
E. AOTA BOARDS;
TOPNOTCH
MD FROM
UST)

TOPNOTCH MEDICAL BOARD PREP ANATOMY SUPEREXAM Page 7 of 94


For inquiries visit www.topnotchboardprep.com.ph or email us at topnotchmedicalboardprep@gmail.com
TOPNOTCH MEDICAL BOARD PREP ANATOMY SUPEREXAM
For inquiries visit www.topnotchboardprep.com.ph or email us at topnotchmedicalboardprep@gmail.com
Item QUESTION EXPLANATION AUTHOR TOPNOTCH
# EXAM
53 The bulk of muscles that make up the floor of the oral cavity The mylohyoid forms the floor of the oral cavity GEORGE MIDTERM 2
A. Genioglossus and the roof of the submandibular triangle. MICHAEL EXAM -
B. Stylohyoid SOSUAN MARCH
C. Mylohyoid (TOP 5 - AUG 2016
D. Digastric 2015 MED
E. NOTA BOARDS;
TOPNOTCH
MD FROM
UST)
54 The Stensen's duct pierces this structure to open into the The Stensen's duct or parotid duct crosses the GEORGE MIDTERM 2
oral cavity masseter to pierce the buccinator to open into the MICHAEL EXAM -
A. Buccinator oral cavity. SOSUAN MARCH
B. Masster (TOP 5 - AUG 2016
C. Mentalis 2015 MED
D. Zygoma BOARDS;
E. NOTA TOPNOTCH
MD FROM
UST)
55 Spongiocytes are located in this layer A spongiocyte is a cell in the zona fasciculata of the GEORGE MIDTERM 2
A. Zona glomerulosa adrenal cortex containing lipid droplets that show MICHAEL EXAM -
B. Zona fasciculata pronounced vacuolization. SOSUAN MARCH
C. Zona reticularis (TOP 5 - AUG 2016
D. Adrenal medulla 2015 MED
E. AOTA BOARDS;
TOPNOTCH
MD FROM
UST)
56 Origin of blood supply of the suprarenal glands The suprarenal glands has three origins of blood GEORGE MIDTERM 2
A. Inferior phrenic artery supply. MICHAEL EXAM -
B. Renal artery SOSUAN MARCH
C. Abdominal aorta (TOP 5 - AUG 2016
D. Both B and C 2015 MED
E. AOTA BOARDS;
TOPNOTCH
MD FROM
UST)
57 Most common direction of hip dislocation 90% of the hip dislocation occurs posteriorly. GEORGE MIDTERM 2
A. Anterior MICHAEL EXAM -
B. Posterior SOSUAN MARCH
C. Superior (TOP 5 - AUG 2016
D. Inferior 2015 MED
E. Antero-inferior BOARDS;
TOPNOTCH
MD FROM
UST)
58 Crutch paralysis injures this nerve Crutch paralysis is a form of paralysis which can GEORGE MIDTERM 2
A. Axillary nerve occur when the radial nerve, is under constant MICHAEL EXAM -
B. Radial nerve pressure, such as by the use of a crutch. SOSUAN MARCH
C. Median nerve (TOP 5 - AUG 2016
D. Musculocutaneous nerve 2015 MED
E. Ulnar nerve BOARDS;
TOPNOTCH
MD FROM
UST)
59 Forms the hood of the clitoris GEORGE MIDTERM 2
A. Frenulum MICHAEL EXAM -
B. Labia majora SOSUAN MARCH
C. Prepuce (TOP 5 - AUG 2016
D. Labia minora 2015 MED
E. NOTA BOARDS;
TOPNOTCH
MD FROM
UST)
60 The ovarian vessels are enclosed by this ligament The suspensory ligament of the ovary, also GEORGE MIDTERM 2
A. True ovarian ligament infundibulopelvic ligament, is a fold of peritoneum MICHAEL EXAM -
B. Suspensory ligament that extends out from the ovary to the wall of the SOSUAN MARCH
C. Round ligament pelvis and encloses the ovarian vessels. (TOP 5 - AUG 2016
D. Cardinal ligament 2015 MED
E. NOTA BOARDS;
TOPNOTCH
MD FROM
UST)
61 The lens of the eye is made up of what epithelial layer? Other parts of the eye: The corneal epithelium is JAN MIDTERM 3
A. Simple squamous made up of stratified squaomous. The corneal CHRISTIAN EXAM -
B. Simple columnar endothelium is made up of simple squamous. FELICIANO MARCH
C.Simple cuboidal (TOP 2 - AUG 2016
D. Stratified squamous non keratinizing 2015 MED
E. Pseudostratified columnar BOARDS;
TOPNOTCH
MD FROM
UST)

TOPNOTCH MEDICAL BOARD PREP ANATOMY SUPEREXAM Page 8 of 94


For inquiries visit www.topnotchboardprep.com.ph or email us at topnotchmedicalboardprep@gmail.com
TOPNOTCH MEDICAL BOARD PREP ANATOMY SUPEREXAM
For inquiries visit www.topnotchboardprep.com.ph or email us at topnotchmedicalboardprep@gmail.com
Item QUESTION EXPLANATION AUTHOR TOPNOTCH
# EXAM
62 Frey's syndrome commonly termed as crocodile tears is a The Auriculotemporal branch of the trigeminal JAN MIDTERM 3
condition wherein beads of perspiration appear on the skin nerve carries parasympathetic fibers to the sweat CHRISTIAN EXAM -
when the patient eats due to penetrating injury to the glands of the scalp and the parotid salivary gland. FELICIANO MARCH
parotid glands. This is due to faulty regeneration of the As a result of severance and inappropriate (TOP 2 - AUG 2016
auriculotemporal nerve which is a branch of what regeneration, the parasympathetic nerve fibers 2015 MED
nerve/ganglion? may switch course, resulting in "gustatory BOARDS;
A. Trigeminal nerve sweating" or sweating in the anticipation of eating, TOPNOTCH
B. Facial nerve instead of the normal salivatory response. MD FROM
C. Glossopharyngeal nerve UST)
D. Vagus nerve
E. Sueprior cervical ganglion

63 90% of epistaxis occurs in the Keiseelbach's plexus or Little's SIMILAR TO PREVIOUS BOARD EXAM JAN MIDTERM 3
triangle. 4 arteries contribute to this plexus. Which artery CONCEPT/PRINCIPLE. The dorsal nasal artery is CHRISTIAN EXAM -
does NOT contribute? the termminal branch of the opthalmic artery and FELICIANO MARCH
A. Anterior ethmoidal artery of ophthalmic artery supplies the dorsum and root of the nose. All the (TOP 2 - AUG 2016
B. Sphenopalatine artery of maxillary artery other 4 arteries make up the Keisselbach's plexus 2015 MED
C. Greater palatine artery of maxillary artery BOARDS;
D. Dorsal nasal artery of ophthalmic artery TOPNOTCH
E. Superior labial artery of facial artery MD FROM
UST)

64 When thyroid follicular cells are stimulated by TSH and fiiled SIMILAR TO PREVIOUS BOARD EXAM JAN MIDTERM 3
with colloid they assume what configuration? CONCEPT/PRINCIPLE. When inactive thyroid CHRISTIAN EXAM -
A. Transitional follicles are simple cuboidal but assume a tall FELICIANO MARCH
B. Tall columnar columnar configuration when stimulated by TSH. (TOP 2 - AUG 2016
C. Simple cuboidal 2015 MED
D. Stratified squamous non keratinizing BOARDS;
E. No specific configuration TOPNOTCH
MD FROM
UST)
65 All of the ff nerves arises from the posterior cord of the For branches of the posterior cord. Remember the JAN MIDTERM 3
brachial plexus EXCEPT? mnemonic STAR. Subscapular nerve, CHRISTIAN EXAM -
A. Subscapular Thoracodorsal, Axillary and Radial. The long FELICIANO MARCH
B. Long thoracic throacic nerve arises from the C5-C7 roots of the (TOP 2 - AUG 2016
C. Thoracodorsal plexus not the cord. 2015 MED
D. Axillary BOARDS;
E. Radial TOPNOTCH
MD FROM
UST)
66 The dorsalis pedis pulse can be located on what landmark? Choice C should have been: Lateral to the extensor JAN MIDTERM 3
A. Lateral to the flexor hallucis longus and medial to flexor hallucis longus and medial to extensor digitorum CHRISTIAN EXAM -
hallucis longus longus . SIMILAR TO PREVIOUS BOARD EXAM FELICIANO MARCH
B .Lateral to the extensor hallucis brevis and medial to CONCEPT/PRINCIPLE. The dorsalis pedis artery (TOP 2 - AUG 2016
extnesor hallucis brevis pulse can be palpated readily lateral to the 2015 MED
C. Lateral to the extensor hallucis longus and medial to extensor hallucis longus tendon and medially to BOARDS;
extnesor hallucis longus the extensor digitorum longus tendon on the TOPNOTCH
D. Lateral to the flexor hallucis brevis and medial to flexor dorsal surface of the foot. Choice E refers to the MD FROM
hallucis brevis femoral artery UST)
E. Behind the inguinal ligament between the ASIS and
symphysis pubis

67 What cell prodcues the hormone that activates bone SIMILAR TO PREVIOUS BOARD EXAM JAN MIDTERM 3
resorption and increases blood calcium level? CONCEPT/PRINCIPLE. The questions refes to CHRISTIAN EXAM -
A. Follicular cells Parathormone or PTH secreted by the chief cells of FELICIANO MARCH
B. Parafolllicular cells the parathyroid gland. Parafollicular cellsof the (TOP 2 - AUG 2016
C. Oxyntic cells thyroid gland secrete calcitonin. Oxyphil cells 2015 MED
D. Oxyphil cells although part of the parathyroids have unknown BOARDS;
E. Chief cells functions. Oxyntic cells is in the stomach. TOPNOTCH
MD FROM
UST)
68 All of the ff are contents of the spermatic cord EXCEPT? SIMILAR TO PREVIOUS BOARD EXAM JAN MIDTERM 3
A. Testicular artery CONCEPT/PRINCIPLE The ilioinguinal nerve psses CHRISTIAN EXAM -
B. Processus vaginalis through the inguinal canal but lies outside the FELICIANO MARCH
C. Deferential artery spermatic cord. All the other structures plus the (TOP 2 - AUG 2016
D. Ilioinguinal nerve vas deferens, cremasteric artery and genital 2015 MED
E. Pampiniform plexus branch of the genitofemoral nerve are contents. BOARDS;
TOPNOTCH
MD FROM
UST)
69 What structure is termed as the false vocal cords? SIMILAR TO PREVIOUS BOARD EXAM JAN MIDTERM 3
A. Quadrangular membrane CONCEPT/PRINCIPLE. The vestibular fold is a CHRISTIAN EXAM -
B. Vestibular folds fixed fold on each side of the larynx and is termed FELICIANO MARCH
C. Aryepiglottic folds as the false vocal cords. While the mobile vocal (TOP 2 - AUG 2016
D. Vocal folds folds immediately below it are the true vocal 2015 MED
E. Rima glottidis cords. The gap b/w the true vocal cord is termed BOARDS;
as the rima glottidis or glottis. The quadrangular TOPNOTCH
membrane extends between the epiglottis and MD FROM
arytenoid cartilage and its inferior margin forms UST)
the vestibular folds.

TOPNOTCH MEDICAL BOARD PREP ANATOMY SUPEREXAM Page 9 of 94


For inquiries visit www.topnotchboardprep.com.ph or email us at topnotchmedicalboardprep@gmail.com
TOPNOTCH MEDICAL BOARD PREP ANATOMY SUPEREXAM
For inquiries visit www.topnotchboardprep.com.ph or email us at topnotchmedicalboardprep@gmail.com
Item QUESTION EXPLANATION AUTHOR TOPNOTCH
# EXAM
70 The medial wall of the bony orbit is formed by the ff bones SIMILAR TO PREVIOUS BOARD EXAM JAN MIDTERM 3
EXCEPT? CONCEPT/PRINCIPLE. The orbital plate of the CHRISTIAN EXAM -
A. Frontal process of the maxilla maxilla forms the floor of the orbit. The 4 other FELICIANO MARCH
B. Lacrimal bone choices forms the medial wall of the orbit. The (TOP 2 - AUG 2016
C. Orbital plate of the maxilla orbital plate of the frontal bone forms the roof of 2015 MED
D. Body of sphenoid the orbit while the zygomatic and the greater wing BOARDS;
E. Orbital plate of the ethmoid of the sphenoid forms the lateral wall. TOPNOTCH
MD FROM
UST)
71 What rotator cuff muscle is a lateral rotator of the shoulder There are 2 lateral rotators of the shoulder: JAN MIDTERM 3
and supplied by a trunk of the brachial plexus? infraspinatus supplied by suprascapular nerve CHRISTIAN EXAM -
A. Supraspinatus from the trunk of the brachial plexus and the teres FELICIANO MARCH
B. Infraspinatus minor supplied by the axillary nerve from the (TOP 2 - AUG 2016
C. Teres Minor posteriro cord. 2015 MED
D.Teres Major BOARDS;
E. Subscapularis TOPNOTCH
MD FROM
UST)
72 Which spinal nerves innervates the main muscle of The question is refers to the phrenic nerve. JAN MIDTERM 3
respiration? Remember: C3 4 5 keeps the diaphragm alive. CHRISTIAN EXAM -
A. C2 C3 C4 FELICIANO MARCH
B. C3 C4 C5 (TOP 2 - AUG 2016
C. C4 C5 C6 2015 MED
D. C5 C6 C7 BOARDS;
E. C6 C7 T1 TOPNOTCH
MD FROM
UST)
73 What is the correct sequence of the CSF pathway? Knowthe CSF pathway by heart.. Lateral ventricle- JAN MIDTERM 3
A .Lateral ventricle- Foramen of Magendie- 3rd ventricle- Foramen of Monroe- 3rd ventricle- Cerebral CHRISTIAN EXAM -
Cerebral aqueduct- 4th ventricle- Foramen of Monroe - aqueduct- 4th ventricle- Foramen of Lushka and FELICIANO MARCH
Arachnoid villi Magendie -Subarachnoid space-Arachnoid villi (TOP 2 - AUG 2016
B. 3rd ventricle- Foramen of Magendie- Latreral ventricle- 2015 MED
Cerebral aqueduct- 4th ventricle- Foramen of Monroe - BOARDS;
Arachnoid villi TOPNOTCH
C. 3rd ventricle- Foramen of Monroe- Lateral ventricle- MD FROM
Cerebral aqueduct- 4th ventricle- Foramen of Magendie - UST)
Arachnoid villi
D. Lateral ventricle- Foramen of Monroe- 3rd ventricle-
Cerebral aqueduct- 4th ventricle- Foramen of Magendie -
Arachnoid villi
E. Lateral ventricle- Cerebral aqueduct- 3rd ventricle-
Foramen of Monroe- 4th ventricle- Foramen of Magendie -
Arachnoid villi
74 The outer hair cells of the organ of Corti are attached SIMILAR TO PREVIOUS BOARD EXAM JAN MIDTERM 3
inferiorly to basilar membrane and superiorly to what CONCEPT/PRINCIPLE. The Reissner's aka the CHRISTIAN EXAM -
membrane? Vestibualr membrane is the roof of the scala media FELICIANO MARCH
A. Reissner's membrane while the floor is the basilar membrane. The (TOP 2 - AUG 2016
B. Shrapnel's membrane tectorial membrane overlies the outer hair cells of 2015 MED
C. Vestibular membrane the organ of Corti and during acoustic stimulation BOARDS;
D. Tympanic membrane stimulates the inner hair cells through fluid TOPNOTCH
E. Tectorial membrane coupling. MD FROM
UST)

75 What cranial foramina and structure passing through it is Ophthalmic artery passes through the optic canal. JAN MIDTERM 3
correctly paired? The maxillary nerve passes through the formaen CHRISTIAN EXAM -
A. Superior orbital fissure- opthalmic artery rotundum while the Mandibular nerve passes FELICIANO MARCH
B. Foramen spinosum- Middle meningeal artery through the foramen ovale. GP nerve passes (TOP 2 - AUG 2016
C. Foramen rotundum- Mandibular nerve through jugular foramen while the facial nerve 2015 MED
D. Internal accoustic meatus- Glossopharyngeal nerve passes through the internal accoustic meatus BOARDS;
E. Jugular foramen- Facial nerve TOPNOTCH
MD FROM
UST)
76 Which statement regarding the lungs is correct? The right lung is divided by the oblique and JAN MIDTERM 3
A.The left lung is divided by the oblique and horizontal horizontal fissure into three lobes. The apex is 1 CHRISTIAN EXAM -
fissure into three lobes inch or 2.5 cm above the clavicle. The FELICIANO MARCH
B. Each lung has an apex which projects upward into the bornchopulmonary segment and not the lobe is (TOP 2 - AUG 2016
neck 2.5 inches above the clavicle the anatomical and functional unit of the lung. 2015 MED
C. The pulmonary lobe is the anatomical and functional unit BOARDS;
of the lung TOPNOTCH
D. All the lymph from all parts of the lungs leaves the hilum MD FROM
and drains into the tracheobronchial nodes and into the UST)
bronchomediastinal trunks
E. None of the above

77 What lobe of the prostate is primarily affected in prostate Lateral and middle lobe (periurethral zone) is JAN MIDTERM 3
adenocarcinoma? affected in BPH while posterior lobe (peripheral CHRISTIAN EXAM -
A. Anterior lobe zone) is affected in prostate adenoCA FELICIANO MARCH
B. Lateral lobe (TOP 2 - AUG 2016
C. Posterior lobe 2015 MED
D. Middle lobe BOARDS;
E. Inferior lobe TOPNOTCH
MD FROM
UST)

TOPNOTCH MEDICAL BOARD PREP ANATOMY SUPEREXAM Page 10 of 94


For inquiries visit www.topnotchboardprep.com.ph or email us at topnotchmedicalboardprep@gmail.com
TOPNOTCH MEDICAL BOARD PREP ANATOMY SUPEREXAM
For inquiries visit www.topnotchboardprep.com.ph or email us at topnotchmedicalboardprep@gmail.com
Item QUESTION EXPLANATION AUTHOR TOPNOTCH
# EXAM
78 What statement regarding the liver is true? Experiments have shown that the quadrate and JAN MIDTERM 3
A. The quadrate and caudate lobes are a functional part of caudate lobes are a functional part of the left lobe CHRISTIAN EXAM -
the left lobe of the liver of the liver. It is divided into a large right lobe and FELICIANO MARCH
B. Divided into a large right lobe and small left lobe by the small left lobe by the falciform ligament. The first (TOP 2 - AUG 2016
coronary ligament liver segment is regarded as the caudate lobe and 2015 MED
C. The first liver segment is regarded as the quadrate lobe 4th segment is the quadrate lobe. Blood supply is BOARDS;
and 4th segment is the caudate lobe 70% portal vein and 30% hepatic vein. TOPNOTCH
D. Half of blood supply is from the portal vein and the other MD FROM
half is from the hepatic artery UST)
E. None of the above

79 Which of the ff nerve does not pass through the greater SIMILAR TO PREVIOUS BOARD EXAM JAN MIDTERM 3
sciatic foramen? CONCEPT/PRINCIPLE. The ff structures passes CHRISTIAN EXAM -
A. Superior gluteal nerve thru the greater sciatic foramen: Sciatic Nerve, FELICIANO MARCH
B. Inferior gluteal nerve Superior Gluteal Nerve, Inferior Gluteal Nerve, (TOP 2 - AUG 2016
C. Sciatic nerve Pudendal Nerve, Posterior, Femoral Cutaneous 2015 MED
D. Obturator nerve Nerve, Nerve to Quadratus Femoris, Nerve to BOARDS;
E. Pudendal nerve Obturator Internus, Superior Gluteal Artery & TOPNOTCH
Vein, Inferior Gluteal Artery & vein, Internal MD FROM
Pudendal Artery & vein and the Piriformis muscle. UST)
Please review the contents of the lesser sciatic
foramen as well.
80 Which among the ff parts of the ear is part of the bony SIMILAR TO PREVIOUS BOARD EXAM JAN MIDTERM 3
labyrynth? CONCEPT/PRINCIPLE. The inner ear is divided CHRISTIAN EXAM -
A. Utricle into the bony labyrynth (external) and FELICIANO MARCH
B. Saccule membranous labyrnth (internal). Bony: Vestibule, (TOP 2 - AUG 2016
C. Semicircular canal Semicircular canal, cochlea. Membranous: Utricle, 2015 MED
D. Cohclear duct saccule, cochlear duct, semicircular duct BOARDS;
E. Auditory ossicles TOPNOTCH
MD FROM
UST)
81 1. Which of the following does NOT play a role in parasitic ANDREW FINAL
infections? TIU (TOP 1 - EXAM -
a. IL - 6 AUG 2015 MARCH
b. IL - 4 MED 2016
c. IgE BOARDS;
d. IL - 5 TOPNOTCH
e. none of the above MD FROM
CIM)
82 2. Which of the following structures does not pass through obturator nerve arises from the lumbar plexus (L2 ANDREW FINAL
both greater and lesser sciatic foramen? – L4) and emerges on the medial border of the TIU (TOP 1 - EXAM -
a. Internal pudendal artery psoas muscle within the abdomen. It runs forward AUG 2015 MARCH
b. pudendal nerve on the lateral wall of the pelvis to reach the upper MED 2016
c. obturator internus nerve part of the obturator foramen where it divides into BOARDS;
d. obturator nerve anterior and posterior divisions. TOPNOTCH
e. none of the above Snells’ Clinical anatomy by regions 9th edition p. MD FROM
465 CIM)
83 3. If the aryepiglottic fold forms the superior border, which The thickened inferior margin forms the ANDREW FINAL
of the following is the inferior border of the quadrangular vestibular ligament and the vestibular ligaments TIU (TOP 1 - EXAM -
membrane? form the interior of the vestibular folds. AUG 2015 MARCH
a. Vocal ligament Snells’ Clinical anatomy by regions 9th edition p. MED 2016
b. Vestibular ligament 647 BOARDS;
c. Median thyrohyoid ligament TOPNOTCH
d. Cricotracheal ligament MD FROM
e. Cricoid cartilage CIM)
84 4. Which of the following cartilages is found in the These two small rod shaped cartilages are found in ANDREW FINAL
quadrangular membrane? the aryepiglottic folds and serve to strengthen TIU (TOP 1 - EXAM -
a. Corniculate cartilage them. AUG 2015 MARCH
b. Cuneiform cartilage Snells’ Clinical anatomy by regions 9th edition p. MED 2016
c. Epiglottis 646 BOARDS;
d. Thyroid cartilage TOPNOTCH
e. Arytenoid cartilage MD FROM
CIM)
85 5. Which of the following is found superior to the arytenoid Snells’ Clinical anatomy by regions 9th edition p. ANDREW FINAL
cartilages? 646 TIU (TOP 1 - EXAM -
a. Corniculate cartilage AUG 2015 MARCH
b. Cuneiform cartilage MED 2016
c. Epiglottis BOARDS;
d. Thyroid cartilage TOPNOTCH
e. Arytenoid cartilage MD FROM
CIM)
86 6. Which of the following is a membranous labyrinth? The membranous labyrinth is lodged within the ANDREW FINAL
a. Vestibule bony labyrinth. It is filed with endolymph and TIU (TOP 1 - EXAM -
b. Semicircular canals surrounded by perilymph. It consists of utricle and AUG 2015 MARCH
c. Cochlea saccule which are lodged in the bony vestibule; MED 2016
d. Utricle three semicircular ducts, which lie within the bony BOARDS;
e. None of the above semicircular canals; and the duct of the cochlea, TOPNOTCH
which lies within the bony cochlea. All these MD FROM
structures freely communicate with each other. CIM)
Snells’ Clinical anatomy by regions 9th edition p.
569

TOPNOTCH MEDICAL BOARD PREP ANATOMY SUPEREXAM Page 11 of 94


For inquiries visit www.topnotchboardprep.com.ph or email us at topnotchmedicalboardprep@gmail.com
TOPNOTCH MEDICAL BOARD PREP ANATOMY SUPEREXAM
For inquiries visit www.topnotchboardprep.com.ph or email us at topnotchmedicalboardprep@gmail.com
Item QUESTION EXPLANATION AUTHOR TOPNOTCH
# EXAM
87 7. Which of the following is responsible for depression of the Contraction of the lateral pterygoids pulls forward ANDREW FINAL
mandible? the neck of the mandible and articular disc so that TIU (TOP 1 - EXAM -
a. Temporalis the latter moves onto the articular tubercle. The AUG 2015 MARCH
b. Lateral pterygoid forward movement of the disc is limited by the MED 2016
c. Medial pterygoid tension of the fibroelastic tissue, which tethers the BOARDS;
d. Masseter disc to the temporal bone posteriorly. TOPNOTCH
e. None of the above Snells’ Clinical anatomy by regions 9th edition p. MD FROM
572 CIM)
88 8. Which of the following divides the cochlear canal into scala Snells’ Clinical anatomy by regions 9th edition p. ANDREW FINAL
vestibule and scala tympani? 569 TIU (TOP 1 - EXAM -
a. Basilar membrane AUG 2015 MARCH
b. Reissner’s membrane MED 2016
c. Tectorial membrane BOARDS;
d. Vestibular membrane TOPNOTCH
e. Spiral ligament MD FROM
CIM)
89 9. Which of the following membranes vibrates to code Hair cells are attached through the phalangeal ANDREW FINAL
acoustic information into nerve impulses and is responsible cells to the basilar membrane, which vibrates TIU (TOP 1 - EXAM -
for amplitude discrimination? during sound reception. The stereocilia of these AUG 2015 MARCH
a. Basilar membrane hair cells are in turn attached to the tectorial MED 2016
b. Reissner’s membrane membrane which also vibrates. However, the BOARDS;
c. Tectorial membrane tectorial membrane and the basilar membrane are TOPNOTCH
d. Vestibular membrane hinged at different points. Thus, a shearing effect MD FROM
e. Spiral ligament occurs between the basilar membrane ( and the CIM)
cells attached to it) and the tectorial membrane
when sound vibrations impinge on the ear.
Histology 7th edition. Pawlina. P. 952
90 10. Which of the following landmarks signify the intersection Rhinion refers to the lower end of the suture ANDREW FINAL
between the frontal bone and 2 nasal bones? between the nasal bones. TIU (TOP 1 - EXAM -
a. Nasion Wikipedia (cant find it in snells) AUG 2015 MARCH
b. Rhinion MED 2016
c. Pterion BOARDS;
d. Glabella TOPNOTCH
e. Asterion MD FROM
CIM)
91 11. Which of the following lies above the pelvic inlet? SNell's CLinical anatomy for Medical students 5th ANDREW FINAL
a. true pelvis edition p. 277 TIU (TOP 1 - EXAM -
b. false pelvis AUG 2015 MARCH
c. pelvic cavity MED 2016
d. lesser pelvis BOARDS;
e. none of the above TOPNOTCH
MD FROM
CIM)
92 12. Which of the following types of pelvis is described with a A - heart shaped with triangular anterior segment ANDREW FINAL
greater anteroposterior diameter, straight walls, small C - normal female pelvis TIU (TOP 1 - EXAM -
subpubic arch, and large sacrosciatic notches? D - transversely wide with flattened AUG 2015 MARCH
a. android anteroposterior diameter MED 2016
b. anthropoid SNell's CLinical anatomy for Medical students 5th BOARDS;
c. gynecoid edition p. 299 TOPNOTCH
d. platypelloid MD FROM
e. none of the above CIM)
93 13. Which of the following is a branch from the aorta? this confusing question came out. Best answer is ANDREW FINAL
a. inferior rectal artery still superior rectal artery, a branch of the inferior TIU (TOP 1 - EXAM -
b. superior vesical artery mesenteric artery. AUG 2015 MARCH
c. superior rectal artery SNell's CLinical anatomy for Medical students 5th MED 2016
d. vaginal artery edition p. 291 BOARDS;
e. all of the above TOPNOTCH
MD FROM
CIM)
94 14. A 5 year old male comes to you for epistaxis. Which of the The following contributes: ANDREW FINAL
following does NOT contribute to the blood supply in the anterior ethmoidal artery from ophthalmic artery TIU (TOP 1 - EXAM -
Kiesselbach area? sphenopalatine artery from terminal branch of AUG 2015 MARCH
a. ophthalmic artery maxillary artery MED 2016
b. sphenopalatine artery greater palatine artery from maxillary artery BOARDS;
c. greater palatine artery septal branch of superior labial artery from facial TOPNOTCH
d. superior nasal artery artery MD FROM
e. superior labial artery CIM)
95 15. Which of the following is NOT a globulin? Junqueira's Basic Histology 12th edition p. 203 ANDREW FINAL
a. transferrin TIU (TOP 1 - EXAM -
b. fibrinogen AUG 2015 MARCH
c. Beta2 - macroglobulin MED 2016
d. fibronectin BOARDS;
e. immunoglobulin TOPNOTCH
MD FROM
CIM)
96 16. Which of the following develops from a reticular cell? Junqueira's Basic Histology 12th edition p. 209 ANDREW FINAL
a. eosinophil TIU (TOP 1 - EXAM -
b. platelet AUG 2015 MARCH
c. RBC MED 2016
d. neutrophil BOARDS;
e. basophil TOPNOTCH
MD FROM
CIM)

TOPNOTCH MEDICAL BOARD PREP ANATOMY SUPEREXAM Page 12 of 94


For inquiries visit www.topnotchboardprep.com.ph or email us at topnotchmedicalboardprep@gmail.com
TOPNOTCH MEDICAL BOARD PREP ANATOMY SUPEREXAM
For inquiries visit www.topnotchboardprep.com.ph or email us at topnotchmedicalboardprep@gmail.com
Item QUESTION EXPLANATION AUTHOR TOPNOTCH
# EXAM
97 17. Which of the following zones of the adrenal cortex has The cells are most densely filled with cytoplasmic ANDREW FINAL
long cords of large polyhedral cells separated by fenestrated lipid droplets and as a result of lipid dissolution TIU (TOP 1 - EXAM -
sinusoidal capillaries? during tissue preparation, often appear vacuolated AUG 2015 MARCH
a. zona glomerulosa or spongy. MED 2016
b. zona fasciculata Junqueira's Basic Histology 12th edition p. 356 BOARDS;
c. zona reticularis TOPNOTCH
d. zona pellucida MD FROM
e. zona medulla CIM)
98 18. The skin is not the largest organ of the body as it is Junqueira's Basic Histology 12th edition p. 320 ANDREW FINAL
considered an organ system. Part of its functions is TIU (TOP 1 - EXAM -
protection of the body by triggering an immune response. AUG 2015 MARCH
One of the cells that play a role in alerting the immune MED 2016
system is bone marrow derived and capable of binding, BOARDS;
processing, and presenting antigens to T lymphocytes. Which TOPNOTCH
of the following layers of the skin is this cell predominantly MD FROM
located? CIM)
a. stratum corneum
b. stratum lucidum
c. stratum germinativum
d. stratum granulosum
e. stratum spinosum
99 19. Which of following supporting cells of the CNS is derived oligodendrocyte - neural tube - CNS - myelin ANDREW FINAL
from neural tube and functions in the repair processes? production TIU (TOP 1 - EXAM -
a. oligodendrocyte neurolemmocyte - neural crest - PNS - myelin AUG 2015 MARCH
b. neurolemmocyte production MED 2016
c. astrocyte ependymal cell - neural tube - CNS - lines the BOARDS;
d. ependymal cell cavities TOPNOTCH
e. microglia microglia - bone marrow - CNS - immune related MD FROM
activity CIM)
Junqueira's Basic Histology 12th edition p. 151
100 20. At what level does the ureter turns forward and medially http://radiopaedia.org/articles/ureter ANDREW FINAL
to insert into the posterolateral wall of the urinary bladder? TIU (TOP 1 - EXAM -
a. pelvic brim AUG 2015 MARCH
b. coccyx MED 2016
c. ischial spine BOARDS;
d. bifurcation of common iliac vessels TOPNOTCH
e. sacroiliac joint MD FROM
CIM)
101 A 30 y/o female patient is brought to the emergency room in Page 11 of Topnotch Handout. To establish an ANGELA DIAGNOSTIC
respiratory distress. On PE she has an anterior neck mass, emergency airway you do a cricothyroidotomy. PAULINE P. EXAM - AUG
which relatives recounted that it recently suddenly increased Through the cricothyroid membrane located CALIMAG- 2015
in size. You decided to create an emergency airway to retore inferior to the thyroid cartilage and superior to the LOYOLA
respiration. At what level could you rapidly create an airway cricoid cartilage, the true vocal cords lie superior (TOP 8 - FEB
with minimum danger of hemorrhage and damage to the to it. 2015 MED
vocal cords? BOARDS;
A. Through the third tracheal ring TOPNOTCH
B. Just above the jugular notch MD FROM
C. Just above the thyroid cartilage UST)
D. Just below the thyroid cartilage
E. Between the 3rd and 4th tracheal ring

102 During thyroidectomy of the patient in the above scenario, Page 11 of Topnotch Handout. The External ANGELA DIAGNOSTIC
the surgeon blindly clamped the superior thyroid artery and laryngeal nerve may be injured during ligation of PAULINE P. EXAM - AUG
ligates it. After 24 hours post-op the patient was noted to the superior thyroid artery. The cricothyroid is CALIMAG- 2015
have a change in vocal quality. Which of the following supplied by the External laryngeal nerve. The LOYOLA
muscles may be affected due to an injury to its innervation other muscles of phonation are supplied by the (TOP 8 - FEB
during the ligation of the superior thyroid artery?
Recurrent laryngeal nerve. 2015 MED
A. Thyroarytenoid BOARDS;
B. Lateral cricoarytenoid TOPNOTCH
C. Posterior cricoarytenoid MD FROM
D. Cricothyroid UST)
E. All of the above

103 A term newborn was noted to have grunting and retractions. SIMILAR TO PREVIOUS BOARD EXAM ANGELA DIAGNOSTIC
His chest xray showed a markedly elevated right CONCEPT/PRINCIPLE. Phrenic nerve palsy will be PAULINE P. EXAM - AUG
hemidiaphragm with essentially clear lung fields. PE showed seen as elevation of diagphram on chest xray. CALIMAG- 2015
a globular abdomen. The most likely diagnosis is: LOYOLA
A. Phrenic nerve palsy (TOP 8 - FEB
B. Congenital diaphragmatic hernia 2015 MED
C. Pulmonary hypoplasia BOARDS;
D. TTN TOPNOTCH
E. PPHN MD FROM
UST)

TOPNOTCH MEDICAL BOARD PREP ANATOMY SUPEREXAM Page 13 of 94


For inquiries visit www.topnotchboardprep.com.ph or email us at topnotchmedicalboardprep@gmail.com
TOPNOTCH MEDICAL BOARD PREP ANATOMY SUPEREXAM
For inquiries visit www.topnotchboardprep.com.ph or email us at topnotchmedicalboardprep@gmail.com
Item QUESTION EXPLANATION AUTHOR TOPNOTCH
# EXAM
104 An 80 y/o male went to the OPD with complaints of low Page 1 and 3 of supplement Topnotch Handout. L5 ANGELA DIAGNOSTIC
back pain. Further tests showed a herniated disc root compression will manifest at the lateral PAULINE P. EXAM - AUG
compressing the L5 root. Where will the patient have surface of the leg and dorsum of the foot. CALIMAG- 2015
dermatomal pain? LOYOLA
A. Groin (TOP 8 - FEB
B. Posterior thigh 2015 MED
C. Lateral part of the lower leg BOARDS;
D. Medial aspect of the calf TOPNOTCH
E. Big toe MD FROM
UST)
105 A 29 y/o male presented at the ER due to abdominal pain. Page 10 and 11 of Topnotch Handout. The rule is ANGELA DIAGNOSTIC
History revealed that the pain started in the periumbilical that pain originating from the midgut is generally PAULINE P. EXAM - AUG
area. This was accompanied by anorexia. After 6 hours, the perceived in the periumbilical region. The midgut: CALIMAG- 2015
patient started vomiting and the pain became localized in the in the 5 mm embryo, it is seen beginning just LOYOLA
right lower quadrant The organ involved is most likely a caudal to the entrance of the bile duct into the (TOP 8 - FEB
derivative of the:
duodenum and terminating at the beginning of the 2015 MED
A. Foregut last third of the transverse colon (from anterior to BOARDS;
B. Midgut posterior intestinal portals). It is suspended from TOPNOTCH
C. Hindgut the dorsal abdominal wall by a short mesentery MD FROM
D. All of the above and communicates with the yolk sac via the UST)
E. None of the above vitelline duct. Its derivatives consist of the small
intestines (except the first part of the duodenum
to the common bile duct entrance); the cecum, the
appendix, the ascending colon; and the right one-
half to two-thirds or proximal part of the
transverse colon. It is supplied by the superior
mesenteric artery and vagus nerve.
106 The patient was then brought to the OR for emergency Page 1-2 of Day 3 Topnotch Handout. McBurney's ANGELA DIAGNOSTIC
appendectomy, a McBurney's incision was done. This incision, is oblique beginning laterally from above PAULINE P. EXAM - AUG
incision will pass through the following except:
and ending medially and located at about one- CALIMAG- 2015
A. External oblique third the distance along a line from the ASIS to the LOYOLA
B. Internal oblique umbilicus. The skin and subcutaneous tissue are (TOP 8 - FEB
C. Transversus abdominis incised down to the external oblique muscle, the 2015 MED
D. Rectus abdominis underlying internal oblique and transversus BOARDS;
E. None of the above abdominis are split and separated. TOPNOTCH
MD FROM
UST)
107 A 10 month old male was diagnosed with Congenital SIMILAR TO PREVIOUS BOARD EXAM ANGELA DIAGNOSTIC
megacolon. On barium studies which segment is considered CONCEPT/PRINCIPLE. Page 11 of Day 3 Topnotch PAULINE P. EXAM - AUG
normal: A. the dilated proximal colon Handout. In Hirscprungs disease/Congenital CALIMAG- 2015
B. the contracted distal colon megacolon there is absence of autonomic ganglion LOYOLA
C. the whole colon cells in the myenteric plexus distal to the dilated (TOP 8 - FEB
D. Both A and B segment of colon. 2015 MED
E. None of the above BOARDS;
TOPNOTCH
MD FROM
UST)
108 In a bilateral recurrent layngeal nerve injury the vocal cords SIMILAR TO PREVIOUS BOARD EXAM ANGELA DIAGNOSTIC
assume what position? CONCEPT/PRINCIPLE. Bilateral recurrent PAULINE P. EXAM - AUG
A. Midway between full abduction and adduction laryngeal nerve palsy: In this condition both cords CALIMAG- 2015
B. Paramedian assume a paramedian position compromising the LOYOLA
C. Full Abduction airway. This commonly occurs following total (TOP 8 - FEB
D. Full Adduction thyroidectomy or in thyroid malignancies. The 2015 MED
E. Either C or D patient will commonly manifest with stridor. The BOARDS;
voice will be near normal. TOPNOTCH
MD FROM
UST)
109 As a surgeon who will perform a laparoscopic Page 9 of Day 3 Topnotch Handout. Liver is ANGELA DIAGNOSTIC
cholecystectomy, you must know the boundaries of the superior, cystic duct is inferior, common hepatic PAULINE P. EXAM - AUG
triangle of Calot which consists of:
duct is medial. CALIMAG- 2015
A. Cystic duct, common hepatic duct, inferior border of the LOYOLA
liver (TOP 8 - FEB
B. Cystic duct, common hepatic duct, common bile duct 2015 MED
C. Common bile duct, common hepatic duct, inferior margin BOARDS;
of the liver TOPNOTCH
D. Cystic duct, common bile duct, inferior border of the MD FROM
liver UST)
E. Cystic duct, common hepatic artery, inferior border of
the liver

110 A 9 month old male was noted to have an inguinal mass. A SIMILAR TO PREVIOUS BOARD EXAM ANGELA DIAGNOSTIC
diagnosis of cryptorchid testes was made. Which of the CONCEPT/PRINCIPLE. Page 22 of day 3 Topnotch PAULINE P. EXAM - AUG
following controls the descent of testes into the scrotum:
Handout. 26-28 weeks-the testes descended CALIMAG- 2015
A. Withdrawal of maternal estrogen retroperitoneally from the posterior abdominal LOYOLA
B. Enlargement of fetal pelvis wall to the deep inguinal rings; due to (TOP 8 - FEB
C. Androgens enlargement of fetal pelvis. 2-3 days descent into 2015 MED
D. All of the above the scrotum; controlled by androgens. BOARDS;
E. Both A and B TOPNOTCH
MD FROM
UST)

TOPNOTCH MEDICAL BOARD PREP ANATOMY SUPEREXAM Page 14 of 94


For inquiries visit www.topnotchboardprep.com.ph or email us at topnotchmedicalboardprep@gmail.com
TOPNOTCH MEDICAL BOARD PREP ANATOMY SUPEREXAM
For inquiries visit www.topnotchboardprep.com.ph or email us at topnotchmedicalboardprep@gmail.com
Item QUESTION EXPLANATION AUTHOR TOPNOTCH
# EXAM
111 A 23 y/o male arrived at the ER. He a suffered severe head Page 33 of supplement Topnotch Handout. The ANGELA DIAGNOSTIC
trauma in a motorcycle accident. Radiographic studies of the mandibular branch/V3 of the trigeminal exits thru PAULINE P. EXAM - AUG
head revealed a basilar skull fracture in the region of the the foramen ovale. It will manifest as Loss of CALIMAG- 2015
foramen ovale. Which of the following functional losses general sensation in skin over mandible, LOYOLA
would most likely be related to this injury? :
mandibular teeth, tongue, paralysis of muscles of (TOP 8 - FEB
A. Inability to abduct eye mastication; jaw deviation to injured side, 2015 MED
B. Loss of general sensation in skin over maxilla Hypoacusis due to paralysis of the tensor tympani BOARDS;
C. Ageusia muscle. B is related to maxillary branch of TOPNOTCH
D. Loss of afferent limb of corneal reflex trigeminal/V2 which exits from the foramen MD FROM
E. Hypoacusis rotundum, C is alteration or loss of taste realted to UST)
facial nerve injury which exits from the internal
auditory meatus, D loss of afferent limb of corneal
reflex is due to injury to the Ophthalmic branch of
the trigeminal/V1 which exits from the superior
orbital fissure, Inability to abduct the eye is due to
injury of the abducens which also exits from the
superior orbital fissure.
112 The physician was listening for breath sounds over the right Page 20 of Day 3 Topnotch Handout. The parietal ANGELA DIAGNOSTIC
hemithorax and was concerned when no sounds were heard pleura reflection passes obliquely across the 8th PAULINE P. EXAM - AUG
on the front of the chest wall at the level of the tenth rib in rib in MCL not the 10th. CALIMAG- 2015
the MCL, the following statements are correct, except: LOYOLA
A. In a healthy individual, the lower border of the right lung (TOP 8 - FEB
in the MCL in the midrespiratory position is at the level of 2015 MED
the 6th rib B. No breath sounds are heard because the BOARDS;
stethoscope was located over the liver TOPNOTCH
C. The costodiaphragmatic recess is situated between the MD FROM
lower border of the lung and the parietal pleura UST)
D. The parietal pleura in the MCL crosses the tenth rib
E. The lower margin of the lung crosses the tenth rib on the
sides of the vertebral column

113 A nexus is an area of low electrical resistance and is involved Page 5 of Histology Topnotch Handout. Gap ANGELA DIAGNOSTIC
in metabolic and electrical coupling of cells. These are found junctions or Nexus are found in nearly all tissues PAULINE P. EXAM - AUG
in nearly all tissues except:
except skeletal muscle. CALIMAG- 2015
A. Osteocytes LOYOLA
B. Skeletal muscle (TOP 8 - FEB
C. Neurons 2015 MED
D. Smooth muscle BOARDS;
E. Cardiac muscle TOPNOTCH
MD FROM
UST)
114 Type IIa muscle fibers are also known as:
Page 13 of Histology Topnotch Handout. Type IIa ANGELA DIAGNOSTIC
A. Red, slow, oxidative fibers are Red, fast, oxidative-glycolytic fibers. Type I are PAULINE P. EXAM - AUG
B. White, fast, glycolytic fibers Red, slow, oxidative fibers, while Type IIb are fast, CALIMAG- 2015
C. Red, fast, oxidative-glycolytic fibers glycolytic fibers. LOYOLA
D. White, slow, oxidative-glycolytic fibers (TOP 8 - FEB
E. Red, slow, oxidative-glycolytic fibers 2015 MED
BOARDS;
TOPNOTCH
MD FROM
UST)
115 The liver has several functions including the storage of Page 24 of Histology Topnotch Handout. Ito ANGELA DIAGNOSTIC
vitamin A and other fat soluble vitamins. This is specifically a cells/Stellate Cells/Lipocytes stores vitamin A and PAULINE P. EXAM - AUG
function of the Ito Cells which are found in the:
other fat soluble vitamins and are found in the CALIMAG- 2015
A. Sinusoids space of disse or perisinusoidal space. LOYOLA
B. Bile canaliculi (TOP 8 - FEB
C. Principal parenchyma cell 2015 MED
D. Space of disse BOARDS;
E. Rokitansky aschoff sinus TOPNOTCH
MD FROM
UST)

TOPNOTCH MEDICAL BOARD PREP ANATOMY SUPEREXAM Page 15 of 94


For inquiries visit www.topnotchboardprep.com.ph or email us at topnotchmedicalboardprep@gmail.com
TOPNOTCH MEDICAL BOARD PREP ANATOMY SUPEREXAM
For inquiries visit www.topnotchboardprep.com.ph or email us at topnotchmedicalboardprep@gmail.com
Item QUESTION EXPLANATION AUTHOR TOPNOTCH
# EXAM
116 The normal anatomical position of the epididymis is SIMILAR TO PREVIOUS BOARD EXAM ANGELA DIAGNOSTIC
________________ in relation to the testes: CONCEPT/PRINCIPLE. The testes are located PAULINE P. EXAM - AUG
A. Postero-lateral within the scrotum, with the epididymis situated CALIMAG- 2015
B. Inferior on the posterolateral aspect of each testicle. LOYOLA
C. Anterior Commonly, the left testicle lies lower than the (TOP 8 - FEB
D. Superior right. They are suspended from the abdomen by 2015 MED
E. Lateral the spermatic cord – collection of vessels, nerves BOARDS;
and ducts that supply the testes. The epididymis is TOPNOTCH
5–10 mm thick and extends from the upper to the MD FROM
caudal pole of the testis. The epididymis can be UST)
divided in caput, corpus and cauda. A thin capsule
and the serosa of the tunica vaginalis cover the
epididymis.

8–10 efferent ducts transport the sperm from the
rete testis into the ductus epididymidis. The
epididymis consists mostly of the coiled
epididymal duct, at the cauda of the epididymis
the ductus deferens starts.
Reference:
http://www.ncbi.nlm.nih.gov/pubmed/16944482
J Clin Ultrasound. 2006 Oct;34(8):385-92.
Normal and variant appearances of the adult
epididymis and vas deferens on high-resolution
sonography.
Puttemans T1, Delvigne A, Murillo D.
117 The prostatic segment of the male urethra is lined with this Page 27 of Histology Topnotch Handout. ANGELA DIAGNOSTIC
type of epithelium: PAULINE P. EXAM - AUG
A. Stratified columnar CALIMAG- 2015
B. Urothelium LOYOLA
C. Pseudostratified columnar epithelium (TOP 8 - FEB
D. Stratified squamous epithelium 2015 MED
E. Pseudostratified squamous epithelium BOARDS;
TOPNOTCH
MD FROM
UST)
118 The anal canal above the dentate line is sensitive to:
SIMILAR TO PREVIOUS BOARD EXAM ANGELA DIAGNOSTIC
A. Pain CONCEPT/PRINCIPLE. Page 11 of Day 3 Topnotch PAULINE P. EXAM - AUG
B. Temperature Handout. The anal canal segment above the CALIMAG- 2015
C. Touch dentate line is derived from the hindgut, supplied LOYOLA
D. Pressure and drained by the superior rectal, sensitive to (TOP 8 - FEB
E. Stretch stretch. 2015 MED
BOARDS;
TOPNOTCH
MD FROM
UST)
119 Which of the following blood cells has the longest life span:
SIMILAR TO PREVIOUS BOARD EXAM ANGELA DIAGNOSTIC
A. RBC CONCEPT/PRINCIPLE. Page 10 of Histology PAULINE P. EXAM - AUG
B. Neutrophils Topnotch Handout. ADULT RBC- 120 days, Fetal CALIMAG- 2015
C. Platelets RBC-90 days, Platelets-10 days, Reticulocytes- 1-2 LOYOLA
D. Eosinophils days, Neutrophils- 1-4 days, Eosinophils-1-2 (TOP 8 - FEB
E. Reticulocytes weeks, Basophils-Several months, Lymphocytes- 2015 MED
hours to years, Monocytes- hours to years. BOARDS;
TOPNOTCH
MD FROM
UST)
120 A 56 y/o male came to the cardiac catheterization SIMILAR TO PREVIOUS BOARD EXAM ANGELA DIAGNOSTIC
laboratory for suspected acute coronary syndrome. Coronary CONCEPT/PRINCIPLE. Page 17 of Day 2 Topnotch PAULINE P. EXAM - AUG
angiography showed a 90% occlusion in the distal left Handout. In approximately 70% of the population, CALIMAG- 2015
circumflex. The patient had arrived at the hospital 24 hours the PDA originates from the right coronary artery; LOYOLA
earlier with crescendo angina. He had no prior history of it is codominant in 20%, meaning both the right (TOP 8 - FEB
coronary artery disease. His ECG showed Sinus rhythm at coronary artery and LCx feed the PDA; and 10% 2015 MED
75/min with acute myocardial injury of the posterior are left dominant, meaning the LCx alone supplies BOARDS;
myocardium manifested as down-sloping ST-segment the PDA. TOPNOTCH
depression in leads V1 to V5. The coronary artery that MD FROM
supplies the PDA determines coronary “dominance.” In UST)
approximately 70% of the population, the PDA originates
from: A. Left circumflex artery
B. Left Coronary artery
C. Right Coronary artery
D. Marginal artery
E. Both A and C

121 The following layers of abdominal wall have scrotal Superficial fascia - Dartos muscle; External oblique LYNN DARYL MIDTERM 1
derivatives except: - External spermatic fascia; Internal Oblique - FELICIANO EXAM - AUG
A. External oblique Cremaster muscle; Transversalis fascia - Internal VILLAMATE 2015
B. Peritoneum spermatic fascia; Peritoneum- Tunica vaginalis R, MD (TOP 5
C. Transversus abdominis - FEB 2015
D. Transversalis fascia MED
E. Superficial fascia BOARDS;
TOPNOTCH
MD FROM
EAC)
TOPNOTCH MEDICAL BOARD PREP ANATOMY SUPEREXAM Page 16 of 94
For inquiries visit www.topnotchboardprep.com.ph or email us at topnotchmedicalboardprep@gmail.com
TOPNOTCH MEDICAL BOARD PREP ANATOMY SUPEREXAM
For inquiries visit www.topnotchboardprep.com.ph or email us at topnotchmedicalboardprep@gmail.com
Item QUESTION EXPLANATION AUTHOR TOPNOTCH
# EXAM
122 The following are parts of the greater omentum EXCEPT? Gastrohepatic/hepatogastric (and LYNN DARYL MIDTERM 1
A. Gastrohepatic hepatoduodenal) are parts of the lesser omentum. FELICIANO EXAM - AUG
B. Gastrocolic VILLAMATE 2015
C. Gastrosplenic R, MD (TOP 5
D. Gastrophrenic - FEB 2015
E. No exception MED
BOARDS;
TOPNOTCH
MD FROM
EAC)
123 The most common site of diverticulitis is the: LYNN DARYL MIDTERM 1
A. Cecum FELICIANO EXAM - AUG
B. Ascending colon VILLAMATE 2015
C. Transverse colon R, MD (TOP 5
D. Descending colon - FEB 2015
E. Sigmoid MED
BOARDS;
TOPNOTCH
MD FROM
EAC)
124 A 28-year old male tennis player comes to the clinic and This is a case of tennis elbow or lateral LYNN DARYL MIDTERM 1
complained of pain when he opens the door or lift a glass. epicondylitis. Muscle attached to lateral FELICIANO EXAM - AUG
Which of the following muscles is least likely involved in this epicondyle include the following: Anconeus VILLAMATE 2015
condition? muscle, the supinator, extensor carpi radialis R, MD (TOP 5
A. Extensor carpi radialis brevis brevis, extensor digitorum, extensor digiti, and - FEB 2015
B. Extensor digitorum extensor carpi ulnaris. ECRL is the exception. Its MED
C. Extensor carpi ulnaris origin is the lateral supracondylar ridge of the BOARDS;
D. Supinator humerus. This is a similar question asked in the TOPNOTCH
E. Extensor carpi radialis longus boards. MD FROM
EAC)

125 The radial artery is the smaller of the terminal branches of The radial artery is the smaller of the terminal LYNN DARYL MIDTERM 1
the brachial artery. It begins at the level of: branches of brachial artery. It begins in the cubital FELICIANO EXAM - AUG
A. Lateral epicondyle of the humerus fossa at the level of the neck of the radius and VILLAMATE 2015
B. Head of the radius passes downward and laterally, beneath the R, MD (TOP 5
C. Radial neck bracioradialis muscle. In the middle third of its - FEB 2015
D. Ulnar head course, the superficial branch of the radial nerve MED
E. Styloid process of the radius lies on its lateral side. SIMILAR TO PREVIOUS BOARDS;
BOARD EXAM CONCEPT TOPNOTCH
MD FROM
EAC)
126 A 50 year old male patient was noted to have cyanosis and Dorsalis pedis artery can be palpated medially to LYNN DARYL MIDTERM 1
decreased sensation of left foot. You decided to assess the the extensor digitorum longus tendon or laterally FELICIANO EXAM - AUG
dorsalis pedis pulse. It is palpated: to the EHL tendon. Structures that pass in front of VILLAMATE 2015
A. Medially to the extensor hallucis longus tendon medial malleolus: Great saphenous vein and R, MD (TOP 5
B. Medially to the extensor digitorum longus tendon saphenous nerve. Behind the medial malleolus: - FEB 2015
C. In front of the medial malleolus Posterior tibial artery, tibial nerve, tibialis MED
D. Behind the medial malleolus posterior tendon, FDL and FHL. Behind the lateral BOARDS;
E. Behind the lateral malleolus malleolus: Sural nerve and small saphenous vein. TOPNOTCH
SIMILAR TO PREVIOUS BOARD EXAM CONCEPT MD FROM
EAC)
127 A 35 y/o patient suffered from a vehicular crash affecting his Layers of the kidney from outer to inner: LYNN DARYL MIDTERM 1
right kidney. On exploration, the first layer of the kidney to Pararenal fat, renal fascia/Gerota's fascia, FELICIANO EXAM - AUG
be encountered during surgery is the: perirenal fat, fibrous/true capsule. Renal fascia VILLAMATE 2015
A. Pararenal fat encloses both the kidney and suprarenal glands. R, MD (TOP 5
B. Perirenal fat SIMILAR TO PREVIOUS BOARD EXAM CONCEPT - FEB 2015
C. Renal fascia MED
D. Outer corte BOARDS;
E. Fibrous capsule TOPNOTCH
MD FROM
EAC)
128 The least dilatable portion of the male urethra: PROSTATIC URETHRA - widest and most dilatable LYNN DARYL MIDTERM 1
A. Prostatic urethra portion; MEMBRANOUS - shortest and least FELICIANO EXAM - AUG
B. Membranous urethra dilatable; PENILE URETHRA - longest, EXTERNAL VILLAMATE 2015
C. Penile urethra MEATUS - narrowest part; FOSSA R, MD (TOP 5
D. Sphincter urethra TERMINALIS/NAVICULAR FOSSA - lies within the - FEB 2015
E. None of the above glans penis. SIMILAR TO PREVIOUS BOARD EXAM MED
CONCEPT BOARDS;
TOPNOTCH
MD FROM
EAC)
129 The most commonly injured portion of the ureter during The ureter runs vertically downward behind and LYNN DARYL MIDTERM 1
gynecologic surgery? enters the pelvis by crossing the bifurcation of the FELICIANO EXAM - AUG
A. Abdominal portion common iliac artery in front of the sacroiliac joint. VILLAMATE 2015
B. Middle portion In vaginal hysterectomy, the ureter courses lateral R, MD (TOP 5
C. Pelvic portion to the uterine cervix. In oophorectomy, it lies - FEB 2015
D. Near the bifurcation of aorta medial to ovarian vessels. It also passes inferior to MED
E. As it crosses the pelvic brim the uterine vessels. SIMILAR TO PREVIOUS BOARDS;
BOARD EXAM CONCEPT TOPNOTCH
MD FROM
EAC)

TOPNOTCH MEDICAL BOARD PREP ANATOMY SUPEREXAM Page 17 of 94


For inquiries visit www.topnotchboardprep.com.ph or email us at topnotchmedicalboardprep@gmail.com
TOPNOTCH MEDICAL BOARD PREP ANATOMY SUPEREXAM
For inquiries visit www.topnotchboardprep.com.ph or email us at topnotchmedicalboardprep@gmail.com
Item QUESTION EXPLANATION AUTHOR TOPNOTCH
# EXAM
130 Which of the following is true regarding safe subclavian vein Supraclavicular approach - Patient in LYNN DARYL MIDTERM 1
catherization using the infraclavicular approach? Trendelenburg position with head turned to FELICIANO EXAM - AUG
A. Needle is inserted below the lower border of the clavicle opposite side, the posterior border of clavicular VILLAMATE 2015
at the junction of medial third and outer 2/3 of the clavicle origin of SCM is palpated. Needle is inserted and is R, MD (TOP 5
and is pointed downward and posteriorly toward the middle directed downward in the direction of opposite - FEB 2015
suprasternal notch nipple and enters the junction of IJV and MED
B. The needle is directed upward in the direction of the subclavian vein. Infraclavicular approach - BOARDS;
opposite nipple. needle is inserted through the skin just below the TOPNOTCH
C. The needle is directed downward in the direction of the lower border of th clavicle at the junction of the MD FROM
opposite nipple. medial third and outer 2/3. Needle is pointed and EAC)
D. Needle is inserted below the lower border of the clavicle upward and posteriorly toward the middle of the
at the junction of medial third and outer 2/3 of the clavicle suprasternal notch. SIMILAR TO PREVIOUS
and is pointed upward and posteriorly toward the middle BOARD EXAM CONCEPT.
suprasternal notch
E. None of the above.

131 Hyaline cartilage is composed of what type of collagen? Type I - Fibrocartilage and elastic cartilage; Type II LYNN DARYL MIDTERM 1
A. Type I - Hyaline cartilage. SIMILAR TO PREVIOUS FELICIANO EXAM - AUG
B. Type II BOARD EXAM CONCEPT. VILLAMATE 2015
C. Type III R, MD (TOP 5
D. Type IV - FEB 2015
E. Type V MED
BOARDS;
TOPNOTCH
MD FROM
EAC)
132 The main glycosaminoglycan content of the bone is Hyaluronic acid - synovial fluid, vitreous humor; LYNN DARYL MIDTERM 1
A. Chondroitin sulfate Chrondoitin sulfate - none, cornea, skin; FELICIANO EXAM - AUG
B. Hyaluronic acid Dermatan sulftage - skin, tendon; Heparan sulfate - VILLAMATE 2015
C. Dermatan sulfate basal laminae, Keratan sulfate - cornea, nucleus R, MD (TOP 5
D. Heparan sulfate pulposus, annulus fibosus - FEB 2015
E. Keratan sulfate MED
BOARDS;
TOPNOTCH
MD FROM
EAC)
133 Patient presented with chronic cough and weight loss and The superficial (subpleural) plexus lies beneath LYNN DARYL MIDTERM 1
was diagnosed with lung mass at the hilar area. Lymph the visceral pleura and drains over the surface of FELICIANO EXAM - AUG
metastases will first affect nodes at the: ? the lungs toward the hilum, where the lymph VILLAMATE 2015
A. Bronchomediastinal lymph trunks vessels enter the bronchopulmonary nodes. All R, MD (TOP 5
B. Bronchopulmonary nodes the lymph from the lung then leaves the hilum and - FEB 2015
C. Paratracheal drains into the tracheobronchial nodes and then MED
D. Tracheobronchial nodes into the bronchomediastinal lymph trunks. BOARDS;
E. Parasternal SIMILAR TO PREVIOUS BOARD EXAM CONCEPT. TOPNOTCH
MD FROM
EAC)
134 A 50 year old woman was noted to have 2 cm breast cancer Lymph from medial breast quadrants, drains to LYNN DARYL MIDTERM 1
located in the nipple areolar complex of the left breast. the parasternal lymph nodes or to the opposite FELICIANO EXAM - AUG
Which of the following lymph nodes is most likely to be breast while lymph from the inferior quadrants VILLAMATE 2015
affected? may pass deeply to the abdominal lymph nodes R, MD (TOP 5
A. Parasternal lymph nodes (subdiaphragmatic inferior phrenic lymph nodes). - FEB 2015
B. Inferior phrenic lymph nodes More than 75% of the lymph, especially from the MED
C. Axillary lymph nodes lateral breast quadrants drains to the axillary BOARDS;
D. Subscapular lymph nodes lymph nodes, iniitially to the anterior or pectoral TOPNOTCH
E. Supraclavicular lymph nodes nodes for the most part. MD FROM
EAC)
135 A 15-year-old male suffered from a sprained ankle due to Ankle is the most frequently injured major joint in LYNN DARYL MIDTERM 1
excessive inversion of the foot. This would most likely the body. Ankle sprains (torn fibers of ligaments) FELICIANO EXAM - AUG
demenstrated a torn of? are most common. A sprained ankle is nearly VILLAMATE 2015
A. Anterior talofibular ligament always an inversion injury. Lateral ligament is R, MD (TOP 5
B. Posterior talofibular ligament more frequently injured that medial ligament - FEB 2015
C. Tendocalcaneus because it is much weaker. The anterior MED
D. Deltoid ligament talofibular ligament, a part of lateral ligament is BOARDS;
E. Long plantar ligament most vulnerable and most commonly torn during TOPNOTCH
ankle sprains because it is much weaker. Severe MD FROM
eversion - deltoid or medial ligament. (Moore) EAC)
136 Patient underwent splenectomy after he suffered from a Splenorenal ligament connects the spleen to the LYNN DARYL MIDTERM 1
vehicular crash severe injury to the spleen. Which of the left kidney. Between the layers of ths ligament, FELICIANO EXAM - AUG
following ligaments is the most vascular? the splenic artery divides into 5 or more branches VILLAMATE 2015
A. Gastrosplenic ligament that enter the hilum. Gastrosplenic ligament is R, MD (TOP 5
B. Splenorenal ligament closely related to to short gastricvessels. - FEB 2015
C. Splenocolic MED
D. Splenophrenic BOARDS;
E. Spleno-omental TOPNOTCH
MD FROM
EAC)

TOPNOTCH MEDICAL BOARD PREP ANATOMY SUPEREXAM Page 18 of 94


For inquiries visit www.topnotchboardprep.com.ph or email us at topnotchmedicalboardprep@gmail.com
TOPNOTCH MEDICAL BOARD PREP ANATOMY SUPEREXAM
For inquiries visit www.topnotchboardprep.com.ph or email us at topnotchmedicalboardprep@gmail.com
Item QUESTION EXPLANATION AUTHOR TOPNOTCH
# EXAM
137 Which of the following blood vessel joins the splenic vein to The portal vein is about 5 cm long and is formed LYNN DARYL MIDTERM 1
form the portal vein? behind the neck of the pancreas by the union of FELICIANO EXAM - AUG
A. Inferior mesenteric vein superior mesenteric and splenic veins. It ascends VILLAMATE 2015
B. Right gastric vein to the right, behind the first part of duodenum. It R, MD (TOP 5
C. Inferior mesenteric vein then runs upward in front of the opening into the - FEB 2015
D. Superior mesenteric vein lesser sac to the porta hepatis, where it divides MED
E. Hepatic vein into right and left terminal branches. BOARDS;
TOPNOTCH
MD FROM
EAC)
138 When oculomotor nerve is affected, patient may present with loss of corneal/blink reflex - CN VII; Extorsion of LYNN DARYL MIDTERM 1
A. Inability to blink the eye and weakness of downward gaze - CN IV; FELICIANO EXAM - AUG
B. Extorsion of the eye loss of afferent pupillary reflex, blindness - CN II; VILLAMATE 2015
C. Loss of afferent pupillary reflex Medial deviation - CN VI; Ptosis, dilated and fixed R, MD (TOP 5
D. Drooping of the eyelid pupil, eyes turned down and out - CN III - FEB 2015
E. Medial deviation of the affected eye MED
BOARDS;
TOPNOTCH
MD FROM
EAC)
139 Which of the following segment of the colon is most LYNN DARYL MIDTERM 1
susceptible to ischemia? FELICIANO EXAM - AUG
A. Cecum VILLAMATE 2015
B. Ascending colon R, MD (TOP 5
C. Descending colon - FEB 2015
D. Splenic flexure MED
E. Sigmoid BOARDS;
TOPNOTCH
MD FROM
EAC)
140 Which of the following statements is true regarding renal The kidney has a dark brown outer cortex and a LYNN DARYL MIDTERM 1
cortex? light brown inner medulla. The cortex closely FELICIANO EXAM - AUG
A. It is thicker and has a lighter brown in color thnan the adheres to renal capsule, not the renal fascia. The VILLAMATE 2015
medulla. renal cortex extends into the medulla between R, MD (TOP 5
B. It contains the upper expanded end of the ureter. adjacent pyramids as the renal columns, while the - FEB 2015
C. The apices of the renal pyramids extends into the renal medulla is composed of renal pyramids, each MED
cortex. having its base oriented toward the cortex and itx BOARDS;
D. The renal fascia closely adheres to the renal cortex and is apex, the renal papilla, projecting medially. TOPNOTCH
continuously laterally with fascia transversalis. MD FROM
E. It extends into the medulla between adjacent pyramids. EAC)

141 A patient with leukemia needs to undergo bone marrow All the other options are the usual sites for bone EDWARD MIDTERM 2
biopsy. The following bones are ideal for bone marrow marrow biopsy, the Posterior iliac crest and HARRY EXAM - AUG
biopsy except: sternum are for adults, in children, the tibia VALLAJERA, 2015
A. Sternum especially the anterior portion may be used and MD (TOP 8 -
B. Femur the vertebra FEB 2015
C. Posterior iliac crest MED
D. Tibia BOARDS;
E. None of the above TOPNOTCH
MD FROM
PERPETUAL
BINAN)
142 Articular cartilage is mostly made up of this proteoglycan Chondroitin sulfate is the major component of EDWARD MIDTERM 2
A. Chondroitin sulfate articular cartilage, in combination with some HARRY EXAM - AUG
B. Keratan sulfate keratan sulfate, it forms aggrecan which is the VALLAJERA, 2015
C. Heparan sulfate major component of articular cartilage. MD (TOP 8 -
D. Dermatan sulfate FEB 2015
E. Heparin MED
BOARDS;
TOPNOTCH
MD FROM
PERPETUAL
BINAN)
143 Tennis elbow involves which among of the following muscle? The extensor muscles of the wrist is involved in EDWARD MIDTERM 2
A. Extensor carpi radialis tennis elbow HARRY EXAM - AUG
B. Biceps brachii VALLAJERA, 2015
C. Brachioradialis MD (TOP 8 -
D. Flexor carpi ulnaris FEB 2015
E. Flexor carpi radialis MED
BOARDS;
TOPNOTCH
MD FROM
PERPETUAL
BINAN)
144 The following are muscles for dorsiflexion of the ankle All of the muscles above are used for dorsiflexion EDWARD MIDTERM 2
except: of the ankle HARRY EXAM - AUG
A. Extensor hallucis longus VALLAJERA, 2015
B. Extensor digitorum longus MD (TOP 8 -
C. Peroneus tertius FEB 2015
D. Tibialis anterior MED
E. None of the above BOARDS;
TOPNOTCH
MD FROM
PERPETUAL

TOPNOTCH MEDICAL BOARD PREP ANATOMY SUPEREXAM Page 19 of 94


For inquiries visit www.topnotchboardprep.com.ph or email us at topnotchmedicalboardprep@gmail.com
TOPNOTCH MEDICAL BOARD PREP ANATOMY SUPEREXAM
For inquiries visit www.topnotchboardprep.com.ph or email us at topnotchmedicalboardprep@gmail.com
Item QUESTION EXPLANATION AUTHOR TOPNOTCH
# EXAM
BINAN)

145 During subclavian vein catheterization, how would you The 18-gauge introducer needle is inserted 1 cm EDWARD MIDTERM 2
insert the needle as you puncture the inferior border of the inferior to the junction of the middle and proximal HARRY EXAM - AUG
clavicle in the infraclavicular approach? third of the clavicle while aiming slightly cephalad VALLAJERA, 2015
A. Upward and directed posterior to the middle of the SCM toward your index finger in the suprasternal MD (TOP 8 -
B. Upward and directed to the suprasternal notch notch. FEB 2015
C. Downward MED
D. Straight BOARDS;
E. None of the above TOPNOTCH
MD FROM
PERPETUAL
BINAN)
146 Which of the following veins in the arm is used for large vein The basilic vein is most commonly used as a EDWARD MIDTERM 2
catheterization due to its increasing diameter as it goes venous access for venous catheterization because HARRY EXAM - AUG
proximally? it becomes larger as goes proximally VALLAJERA, 2015
A. Cephalic MD (TOP 8 -
B. Basilic FEB 2015
C. Median cubital vein MED
D. Innominate vein BOARDS;
E. None of the above TOPNOTCH
MD FROM
PERPETUAL
BINAN)
147 A man sustained a stab wound located at the 4th ICS directed The needle is inserted at the subxiphoid area and EDWARD MIDTERM 2
inferiorly, you were able to appreciate Beck's triad. What is is directed superiorly and to the left aiming for the HARRY EXAM - AUG
the quickest way to perform pericardiocentesis? patient's left shoulder. VALLAJERA, 2015
A. 4th ICS parasternal line left MD (TOP 8 -
B. 4th ICS parasternal line right FEB 2015
C. 5th ICS midclavicular line MED
D. Subxiphoid BOARDS;
E. 2nd ICS parasternal line left TOPNOTCH
MD FROM
PERPETUAL
BINAN)
148 At which tracheal cartilage would you perform The 2nd and 3rd tracheal rings are the best sites to EDWARD MIDTERM 2
tracheostomy? perform tracheostomy but care must be noted to HARRY EXAM - AUG
A. 1st and 2nd avoid hitting the isthmus of the thyroid gland VALLAJERA, 2015
B. 2nd and 3rd which is highly vascularized. If ever the thyroid MD (TOP 8 -
C. 3rd and 4th isthmus is encountered, it should be retracted FEB 2015
D. 4th and 5th inferiorly. MED
E. cricothyroid membrane BOARDS;
TOPNOTCH
MD FROM
PERPETUAL
BINAN)
149 The dorsalis pedis pulse is best appreciated where? It is appreciated best between the 1st and 2nd EDWARD MIDTERM 2
A. Between 1st and 2nd metatarsal space metatarsal space of the foot. HARRY EXAM - AUG
B. Between 2nd and 3rd metatarsal space VALLAJERA, 2015
C. 3rd and 4th metatarsal space MD (TOP 8 -
D. Between 4th and 5th metatarsal space FEB 2015
E. Behind medial malleolus MED
BOARDS;
TOPNOTCH
MD FROM
PERPETUAL
BINAN)
150 The dorsalis pedis pulse is related to which structure? It is closely related to the lateral border of the EDWARD MIDTERM 2
A. Lateral to the extensor hallucis longus extensor hallucis longus of the 1st toe. HARRY EXAM - AUG
B. Medial to the extensor hallucis longus VALLAJERA, 2015
C. Lateral to the flexor hallucis longus MD (TOP 8 -
D. Lateral to the medial malleolus FEB 2015
E. None of the above MED
BOARDS;
TOPNOTCH
MD FROM
PERPETUAL
BINAN)
151 The spleen is closest to which of the following? The spleen is closest to the inferior border of the EDWARD MIDTERM 2
A. Kidney diaphragm HARRY EXAM - AUG
B. Stomach VALLAJERA, 2015
C. Splenic flexure MD (TOP 8 -
D. Tail of the pancreas FEB 2015
E. Inferior border of the diaphragm MED
BOARDS;
TOPNOTCH
MD FROM
PERPETUAL
BINAN)

TOPNOTCH MEDICAL BOARD PREP ANATOMY SUPEREXAM Page 20 of 94


For inquiries visit www.topnotchboardprep.com.ph or email us at topnotchmedicalboardprep@gmail.com
TOPNOTCH MEDICAL BOARD PREP ANATOMY SUPEREXAM
For inquiries visit www.topnotchboardprep.com.ph or email us at topnotchmedicalboardprep@gmail.com
Item QUESTION EXPLANATION AUTHOR TOPNOTCH
# EXAM
152 Which of the following cells produces the fibers that form the These cells produce the fibers that form the EDWARD MIDTERM 2
structural framework of most lymphatic structure of lymphatic organs HARRY EXAM - AUG
organs? VALLAJERA, 2015
A. Mononuclear cells MD (TOP 8 -
B. Reticular cells FEB 2015
C. Macrophages MED
D. Bilobed cells BOARDS;
E. None of the above TOPNOTCH
MD FROM
PERPETUAL
BINAN)
153 The spleen is easily ruptured in blunt abdominal trauma All of the above are possible causes of the spleen EDWARD MIDTERM 2
because of which of the following factors: ruputuring in blunt abdominal trauma. HARRY EXAM - AUG
A. Highly vascular organ VALLAJERA, 2015
B. Soft parenchyma MD (TOP 8 -
C. Sudden deceleration and acceleration FEB 2015
D. Direct blow to the area of the spleen MED
E. All of the above BOARDS;
TOPNOTCH
MD FROM
PERPETUAL
BINAN)
154 A patient came in the ER due to blunt abdominal trauma The patient has probably a ruptured spleen, Kehr's EDWARD MIDTERM 2
secondary to a vehicular accident, the patient is awake and sign is the occurrence of acute pain in the tip of HARRY EXAM - AUG
conversant and just complains of a minimal vague the shoulder due to the presence of blood or other VALLAJERA, 2015
abdominal pain, vital signs are still stable yet you were able irritants in the peritoneal cavity when a person is MD (TOP 8 -
to elicit a positive Kehr's sign, what would you do next? lying down and the legs are elevated. Kehr's sign FEB 2015
A. Perform emergency exploratory laparotomy in the left shoulder is considered a classical MED
B. Observe the patient symptom of a ruptured spleen. BOARDS;
C. GIve analgesics TOPNOTCH
D. Send the patient home MD FROM
E. None of the above PERPETUAL
BINAN)

155 A 1 year old boy is brought to you due to vomiting everytime The treatment of choice for intussusception in this EDWARD MIDTERM 2
after feeding and passage of jelly like stools, you were able to age group is barium enema or air enema HARRY EXAM - AUG
appreciate a small hard mass on the right lower quadrant, accompanied by GI decompression by inserting a VALLAJERA, 2015
what would you do next? NGT. MD (TOP 8 -
A. Perform exploratory laparotomy FEB 2015
B. Perform a barium enema MED
C. Observe the patient BOARDS;
D. Give an anxiolytic TOPNOTCH
E. None of the above MD FROM
PERPETUAL
BINAN)
156 A 5 year old accidentally swallowed a 5 peso coin, on The orientation of the coin on both anterior and EDWARD MIDTERM 2
performing radiograph, the coin was located at the level of lateral views suggest that the coin is at the level of HARRY EXAM - AUG
T4, the coin on AP CXR looks like a slit while on lateral view the bifurcation of the trachea VALLAJERA, 2015
it looks round, you know that the coin is most likely in the: MD (TOP 8 -
A. Esophagus FEB 2015
B. Bifurcation of the trachea MED
C. Right main bronchus BOARDS;
D. Left main bronchus TOPNOTCH
E. Inferior right bronchopulmonary segment MD FROM
PERPETUAL
BINAN)
157 All of the following organs actively fight pathogens except: The thymus is the site of maturation and selection EDWARD MIDTERM 2
A. Cervical lymph nodes of self-tolerant T lymphocytes HARRY EXAM - AUG
B. Thymus VALLAJERA, 2015
C. Spleen MD (TOP 8 -
D. Axillary lymph nodes FEB 2015
E. Tonsils MED
BOARDS;
TOPNOTCH
MD FROM
PERPETUAL
BINAN)
158 Which of the following does not empty into the thoracic The right subclavian trunk drains into the right EDWARD MIDTERM 2
duct? lymphatic duct to enter the right subclavian vein HARRY EXAM - AUG
A. Right subclavian trunk VALLAJERA, 2015
B. Left jugular trunk MD (TOP 8 -
C. Cisterna chyli FEB 2015
D. Left subclavian trunk MED
E. None of the above BOARDS;
TOPNOTCH
MD FROM
PERPETUAL
BINAN)

TOPNOTCH MEDICAL BOARD PREP ANATOMY SUPEREXAM Page 21 of 94


For inquiries visit www.topnotchboardprep.com.ph or email us at topnotchmedicalboardprep@gmail.com
TOPNOTCH MEDICAL BOARD PREP ANATOMY SUPEREXAM
For inquiries visit www.topnotchboardprep.com.ph or email us at topnotchmedicalboardprep@gmail.com
Item QUESTION EXPLANATION AUTHOR TOPNOTCH
# EXAM
159 As lymph flows through a lymph node, which of the following It flows from the afferent vessel then to the EDWARD MIDTERM 2
would it go through last subcapsular sinus then to the cortical sinus then to HARRY EXAM - AUG
A. Subcapsular sinus the medullary sinus and finally into the efferent VALLAJERA, 2015
B. Afferent lymphatic vessel vessel MD (TOP 8 -
C. Medullary sinus FEB 2015
D. Cortical sinus MED
E. None of the above BOARDS;
TOPNOTCH
MD FROM
PERPETUAL
BINAN)
160 Most of the body's mucosa-associated lymphatic tissue can These 2 systems are constantly exposed to EDWARD MIDTERM 2
be found in which of the following pathogens and are therefore with the most HARRY EXAM - AUG
A. Respiratory and Reproductive tract number of mucosa associated lymphatic tissue. VALLAJERA, 2015
B. Gastrointestinal and urinary tract MD (TOP 8 -
C. Urinary and reproductive tract FEB 2015
D. Respiratory and gastrointestinal tract MED
E. None of the above BOARDS;
TOPNOTCH
MD FROM
PERPETUAL
BINAN)
161 The dorsalis pedis pulse can be palpated immediately on PBEQ HAROLD JAY MIDTERM 3
which part of the foot? S. BAYTEC, EXAM - AUG
A. Medially to the external hallucis longus tendon MD (TOP 10 2015
B. Lateral to the external digitorum longus tendon - FEB 2015
C. Medial to the tibialis anterior tendon MED
D. Lateral to the external hallucis longus tendon BOARDS;
E. lateral to the tibialis anterior tendon TOPNOTCH
MD FROM
FEU)
162 The dorsalis pedis artery courses its way distally and can be PBEQ HAROLD JAY MIDTERM 3
palpated between the following landmarks. S. BAYTEC, EXAM - AUG
A. 1st and 2nd metatarsals MD (TOP 10 2015
B. 2nd and 3rd metatarsals - FEB 2015
C. 3rd an 4th metatarsals MED
D. 1st and 2nd proximal phalanges BOARDS;
E. 2nd and 3rd proximal phalanges TOPNOTCH
MD FROM
FEU)
163 Which of the following part is mostly affected in tennis tennis elbow is lateral epicondylitis HAROLD JAY MIDTERM 3
elbow? S. BAYTEC, EXAM - AUG
A. Medial part of the elbow MD (TOP 10 2015
B. lateral part of the elbow - FEB 2015
C. superior part of the elbow MED
D. Inferior part of the elbow BOARDS;
E. Posterior part of the elbow TOPNOTCH
MD FROM
FEU)
164 25. SB, a 35 year old male, was diagnosed with hypertension hydrochlorothiazide inhibits sodium reabsorption HAROLD JAY MIDTERM 3
just last week. Her doctor decided to start him with in distal renal tubules resulting in increased S. BAYTEC, EXAM - AUG
hydrochlorothiazide. Where is the site of action of the drug? excretion of water and of Na, K, and hydrogen ions MD (TOP 10 2015
A. PCT - FEB 2015
B. DCT MED
C. CT BOARDS;
D. LOH TOPNOTCH
E. Glomerulus MD FROM
FEU)

165 The breast receives its blood supply from which of the HAROLD JAY MIDTERM 3
following blood vessels? S. BAYTEC, EXAM - AUG
A. Perforating branches of internal mammary artery MD (TOP 10 2015
B. Lateral branches of the posterior intercostal arteries - FEB 2015
C. Branches from axillary artery MED
D. A and B BOARDS;
E. All of the above TOPNOTCH
MD FROM
FEU)
166 By weight, which of the following is considered the largest ft if the question is largest organ, the answer HAROLD JAY MIDTERM 3
organ of the body? should be liver since the skin is not considered an S. BAYTEC, EXAM - AUG
A. Skin organ but an organ system MD (TOP 10 2015
B. Liver - FEB 2015
C. Lung MED
D. Heart BOARDS;
E. Brain TOPNOTCH
MD FROM
FEU)
167 Which among the following muscle is considered as the gluteus maximus is generally the biggest. HAROLD JAY MIDTERM 3
longest muscle of the human body? Latissimus dorsi is the widest S. BAYTEC, EXAM - AUG
A. Gluteus maximus MD (TOP 10 2015
B. Latissimus dorsi - FEB 2015
C. sartorius MED
D. Quadriceps femoris BOARDS;
E. Psoas major TOPNOTCH
MD FROM
FEU)
TOPNOTCH MEDICAL BOARD PREP ANATOMY SUPEREXAM Page 22 of 94
For inquiries visit www.topnotchboardprep.com.ph or email us at topnotchmedicalboardprep@gmail.com
TOPNOTCH MEDICAL BOARD PREP ANATOMY SUPEREXAM
For inquiries visit www.topnotchboardprep.com.ph or email us at topnotchmedicalboardprep@gmail.com
Item QUESTION EXPLANATION AUTHOR TOPNOTCH
# EXAM
168 All of the following statements regarding the vessels of the lymphatics from the middle third drain into the HAROLD JAY MIDTERM 3
vagina EXCEPT: internal iliac nodes. the table combined the S. BAYTEC, EXAM - AUG
A. Lymphatics from the middle third drain into the external lymphatic drainage of the upper and middle 3rd. MD (TOP 10 2015
iliac nodes OB Williams 23rd edition page 18. "Those from the - FEB 2015
B. The proximal portion is supplied by the cervical branch middle third drain into the internal iliac nodes, MED
of the uterine artery and those from the upper third drain into the BOARDS;
C. the middle rectal artery contributes to supply the external, internal, and common iliac nodes." TOPNOTCH
posterior vaginal wall MD FROM
D. distal walls receive contributions from the internal FEU)
pudendal artery
E. extensive venous plexus immediately sorrounds the
vagina and follows the course of the arteries

169 This structure originated from the densest portion of the A B and C are all the same. HAROLD JAY MIDTERM 3
broad ligament and is considered as provider of the major S. BAYTEC, EXAM - AUG
support for the uterus and cervix MD (TOP 10 2015
A. Cardinal ligament - FEB 2015
B. Mackenrodt ligament MED
C. Transverse cervical ligament BOARDS;
D. None of the above TOPNOTCH
E. all of the above MD FROM
FEU)

170 What is the most common type of leiomyoma of the uterus? SIMILAR TO PREVIOUS BOARD EXAM HAROLD JAY MIDTERM 3
A. subserous CONCEPT/PRINCIPLE S. BAYTEC, EXAM - AUG
B. Intramural MD (TOP 10 2015
C. submucous - FEB 2015
D. cervical MED
E. None of the above BOARDS;
TOPNOTCH
MD FROM
FEU)
171 In a kidney surgery, when the surgeon penetrated the the choices are arranged from the outer to inner HAROLD JAY MIDTERM 3
peritoneum, the next layer that he will encounter is layers. almost all the pictures that i saw online and S. BAYTEC, EXAM - AUG
A. Pararenal fat on books have a fat in between the peritoneum MD (TOP 10 2015
B. Renal fascia and the gerotas fascia, and almost all sources - FEB 2015
C. Perinephric fat or perirenal fat describe this layer as the paranephric fat.. MED
D. Renal capsule SIMILAR TO PREVIOUS BOARD EXAM BOARDS;
E. Renal cortex CONCEPT/PRINCIPLE TOPNOTCH
MD FROM
FEU)
172 Gerota's fascia is the structure that immediately encloses SIMILAR TO PREVIOUS BOARD EXAM HAROLD JAY MIDTERM 3
both the kidneys and suprarenal glands. This fascia is also CONCEPT/PRINCIPLE S. BAYTEC, EXAM - AUG
called as MD (TOP 10 2015
A. Pararenal fat - FEB 2015
B. Renal fascia MED
C. Perinephric fat or perirenal fat BOARDS;
D. Renal capsule TOPNOTCH
E. Renal cortex MD FROM
FEU)
173 Which of the following structures is NOT derived from spleen is from mesoderm HAROLD JAY MIDTERM 3
endoderm? S. BAYTEC, EXAM - AUG
A. bronchus MD (TOP 10 2015
B. trachea - FEB 2015
C. spleen MED
D. stomach BOARDS;
E. Duodenum TOPNOTCH
MD FROM
FEU)
174 Peyer's patches are seen in what part of the GIT? SIMILAR TO PREVIOUS BOARD EXAM HAROLD JAY MIDTERM 3
A. Large intestines CONCEPT/PRINCIPLE. Peyer's patches are only S. BAYTEC, EXAM - AUG
B. Ileum found in ileum MD (TOP 10 2015
C. jejunum - FEB 2015
D. duodenum MED
E. Stomach BOARDS;
TOPNOTCH
MD FROM
FEU)
175 Brunner's glands are found in what part/s of the GIT Brunner's gland are found in duodenum and HAROLD JAY MIDTERM 3
A. Stomach peyer's patches are found in ileum. No peyer's S. BAYTEC, EXAM - AUG
B. jejunum patches nor brunner's glands are found in jejunum MD (TOP 10 2015
C. ileum - FEB 2015
D. duodenum MED
E. B and D BOARDS;
TOPNOTCH
MD FROM
FEU)

TOPNOTCH MEDICAL BOARD PREP ANATOMY SUPEREXAM Page 23 of 94


For inquiries visit www.topnotchboardprep.com.ph or email us at topnotchmedicalboardprep@gmail.com
TOPNOTCH MEDICAL BOARD PREP ANATOMY SUPEREXAM
For inquiries visit www.topnotchboardprep.com.ph or email us at topnotchmedicalboardprep@gmail.com
Item QUESTION EXPLANATION AUTHOR TOPNOTCH
# EXAM
176 Which of the following muscles is LEAST likely to cause Thyrohyoid is attached to the inferior of the hyoid HAROLD JAY MIDTERM 3
elevation of the hyoid bone? bone so it depresses the latter. choice A is S. BAYTEC, EXAM - AUG
A. hypoglossus hypoglossus and not hyoglossus. i intentionally MD (TOP 10 2015
B. stylohyoid put the choice A which is hyPOglossus (CN XI) to - FEB 2015
C. Geniohyoid make a little trick to the question just like a typical MED
D. Mylohyoid board exam question. some might have read the BOARDS;
E. Thyrohyoid choice A as hyoglossus which is a muscle which TOPNOTCH
does not elevate the hyoid bone. MD FROM
FEU)
177 The sternal angle of Louis is an important landmark of the IVD of T4 and T5 HAROLD JAY MIDTERM 3
chest because it lies on the following structures EXCEPT: S. BAYTEC, EXAM - AUG
A. Second costal cartilage MD (TOP 10 2015
B. Bifurcation of the trachea - FEB 2015
C. Intervertebral disc between T2 and T3 MED
D. Junction of the superior and inferior mediastinum BOARDS;
E. junction of the ascending aorta and the aortic arch TOPNOTCH
MD FROM
FEU)
178 A patient came in at the ER due to gunshot wound on the SIMILAR TO PREVIOUS BOARD EXAM HAROLD JAY MIDTERM 3
abdomen which prompted exploratory laparotomy. Upon CONCEPT/PRINCIPLE. It is the part of the large S. BAYTEC, EXAM - AUG
opening, you saw that a portion of the abdominal aorta was intestines that is farthest from the SMA and IMA MD (TOP 10 2015
hit. Which of the following part of the large intestine is most - FEB 2015
vulnerable for ischemia? MED
A. cecum BOARDS;
B. ascending colon TOPNOTCH
C. hepatic flexure MD FROM
D. splenic flexure FEU)
E. sigmoid

179 The total refractive power of the eye is mostly contributed 2/3 of the total refractive power is contributed by HAROLD JAY MIDTERM 3
by: cornea S. BAYTEC, EXAM - AUG
A. cornea MD (TOP 10 2015
B. pupil - FEB 2015
C. lens MED
D. Vitreous humor BOARDS;
E. Acqueous humor TOPNOTCH
MD FROM
FEU)
180 Cricoid cartilage can be seen at what vertebral level? HAROLD JAY MIDTERM 3
A. C4 S. BAYTEC, EXAM - AUG
B. C5 MD (TOP 10 2015
C. C6 - FEB 2015
D. C7 MED
E. T1 BOARDS;
TOPNOTCH
MD FROM
FEU)
181 You can palpate the pulsations of dorsalis pedis artery in SIMILAR TO PREVIOUS BOARD EXAM JEAN PAOLO FINAL EXAM
what location in the foot? CONCEPT/PRINCIPLE. M. DELFINO, - AUG 2015
A. Between 1st and 2nd metatarsals MD (TOP 10
B. Between 2nd and 3rd metatarsals - FEB 2015
C. Between 3rd and 4th metatarsals MED
D. In front of the ankle joint BOARDS;
E. Medial to 1st metatarsal TOPNOTCH
MD FROM
FATIMA)
182 What is the lymph drainage of the lower 1/3 of esophagus? SIMILAR TO PREVIOUS BOARD EXAM JEAN PAOLO FINAL EXAM
A. Bronchomediastinal nodes CONCEPT/PRINCIPLE.. Lymph vessels from the M. DELFINO, - AUG 2015
B. Tracheobronchial nodes upper third of the esophagus drain into the deep MD (TOP 10
C. Paraaortic nodes cervical nodes, from the middle third into the - FEB 2015
D. Left gastric nodes superior and posterior mediastinal nodes, and MED
E. Posterior mediastinal nodes from the lower third into nodes along the left BOARDS;
gastric blood vessels and the celiac nodes TOPNOTCH
MD FROM
FATIMA)
183 During TAHBSO of a 45 year old G3P3, what part of the SIMILAR TO PREVIOUS BOARD EXAM JEAN PAOLO FINAL EXAM
ureter is most commonly injured? CONCEPT/PRINCIPLE. M. DELFINO, - AUG 2015
A. Proximal 1/3 MD (TOP 10
B. Middle 1/3 - FEB 2015
C. Distal 1/3 MED
D. B and C BOARDS;
E. None of the above TOPNOTCH
MD FROM
FATIMA)

TOPNOTCH MEDICAL BOARD PREP ANATOMY SUPEREXAM Page 24 of 94


For inquiries visit www.topnotchboardprep.com.ph or email us at topnotchmedicalboardprep@gmail.com
TOPNOTCH MEDICAL BOARD PREP ANATOMY SUPEREXAM
For inquiries visit www.topnotchboardprep.com.ph or email us at topnotchmedicalboardprep@gmail.com
Item QUESTION EXPLANATION AUTHOR TOPNOTCH
# EXAM
184 In doing subclavian vein catheterization, the needle should SIMILAR TO PREVIOUS BOARD EXAM JEAN PAOLO FINAL EXAM
be inserted through the skin just below the lower border of CONCEPT/PRINCIPLE.. The question describes an M. DELFINO, - AUG 2015
the clavicle at the junction of the medial third and outer two Infraclavicular approach: needle is inserted at MD (TOP 10
thirds. The needle is then pointed in what direction? lower border of clavicle, then it is pointed upward - FEB 2015
A. downward and medially toward the mediastinum and posteriorly toward the middle of the MED
B. downward and medially towards the opposite nipple suprasternal notch. Supraclavicular approach: The BOARDS;
C. downward and posteriorly towards the suprasternal needle is inserted through the skin at the site TOPNOTCH
notch where the posterior border of the clavicular origin MD FROM
D. upward and posteriorly toward the suprasternal notch of sternocleidomastoid is attached to the upper FATIMA)
E. upward and medially towards the opposite nipple border of the clavicle. then it is pointed downward
and medially toward the mediastinum.
185 The first branch of the internal iliac artery supplies what JEAN PAOLO FINAL EXAM
organ? M. DELFINO, - AUG 2015
A. Middle portion of ureter MD (TOP 10
B. Bladder - FEB 2015
C. Uterus MED
D. Rectum BOARDS;
E. Prostate gland TOPNOTCH
MD FROM
FATIMA)
186 Which of the following intrinsic muscles of the larynx can Posterior cricoarytenoid abducts the vocal cords JEAN PAOLO FINAL EXAM
abduct the vocal fold? by rotating arytenoid cartilage M. DELFINO, - AUG 2015
A. Cricothyroid MD (TOP 10
B. Transverse arytenoid - FEB 2015
C. Posterior cricoarytenoid MED
D. Lateral cricoarytenoid BOARDS;
E. Oblique arytenoid TOPNOTCH
MD FROM
FATIMA)
187 During splenectomy, injury to what structure can result to SIMILAR TO PREVIOUS BOARD EXAM JEAN PAOLO FINAL EXAM
damage of pancreas? CONCEPT/PRINCIPLE.. The presence of the tail of M. DELFINO, - AUG 2015
A. Gastrosplenic ligament the pancreas in the splenicorenal ligament MD (TOP 10
B. Splenorenal ligament sometimes results in its damage during - FEB 2015
C. Hepatosplenic ligament splenectomy MED
D. Splenocolic ligament BOARDS;
E. Pancreaticosplenic ligament TOPNOTCH
MD FROM
FATIMA)
188 What is the most common cause of aortic aneurysm? SIMILAR TO PREVIOUS BOARD EXAM JEAN PAOLO FINAL EXAM
A. Atherosclerosis CONCEPT/PRINCIPLE.. Atherosclerosis is the most M. DELFINO, - AUG 2015
B. Medial degeneration common cause of aortic aneurysm. MD (TOP 10
C. Congenital - FEB 2015
D. Infection MED
E. None of the above BOARDS;
TOPNOTCH
MD FROM
FATIMA)
189 What is the location of epididymis in relation to the testis? SIMILAR TO PREVIOUS BOARD EXAM JEAN PAOLO FINAL EXAM
A. anteromedial CONCEPT/PRINCIPLE. M. DELFINO, - AUG 2015
B. medial MD (TOP 10
C. posterolateral - FEB 2015
D. posterior MED
E. superolateral BOARDS;
TOPNOTCH
MD FROM
FATIMA)
190 What is the lining epithelium of seminal vesicles? seminal vesicles, prostate gland, epididymis and JEAN PAOLO FINAL EXAM
A. transitional vas deferens have pseudostratified columnar non- M. DELFINO, - AUG 2015
B. pseudostratified columnar ciliated lining epithelium. The latter 2 have MD (TOP 10
C. simple columnar stereocilia. - FEB 2015
D. simple cuboidal MED
E. stratified columnar BOARDS;
TOPNOTCH
MD FROM
FATIMA)
191 What structure divides the subclavian artery into 3 Scalene anterior divides the subclavian artery into JEAN PAOLO FINAL EXAM
segments? 3 segments. 1st segment is medial to the muscle, M. DELFINO, - AUG 2015
A. 1st rib 2nd segment is posterior, and the 3rd segment is MD (TOP 10
B. subclavius lateral to the muscle. - FEB 2015
C. pectoralis minor MED
D. teres major BOARDS;
E. scalene anterior TOPNOTCH
MD FROM
FATIMA)
192 The posterior belly of digastric is innervated by what nerve? Facial nerve innervates structures derived from JEAN PAOLO FINAL EXAM
A. CN X the 2nd pharyngeal arch: muscles of facial M. DELFINO, - AUG 2015
B. CN IX expression, stapedius, stylohyoid and posterior MD (TOP 10
C. CN VIII bellt of digastric. The anterior belly of digastric is - FEB 2015
D. CN VII innervated by trigeminal nerve. MED
E. CN V BOARDS;
TOPNOTCH
MD FROM
FATIMA)

TOPNOTCH MEDICAL BOARD PREP ANATOMY SUPEREXAM Page 25 of 94


For inquiries visit www.topnotchboardprep.com.ph or email us at topnotchmedicalboardprep@gmail.com
TOPNOTCH MEDICAL BOARD PREP ANATOMY SUPEREXAM
For inquiries visit www.topnotchboardprep.com.ph or email us at topnotchmedicalboardprep@gmail.com
Item QUESTION EXPLANATION AUTHOR TOPNOTCH
# EXAM
193 All of the following drains into the right atrium except SIMILAR TO PREVIOUS BOARD EXAM JEAN PAOLO FINAL EXAM
A. Superior vena cava CONCEPT/PRINCIPLE.. SVC, IVC, coronary sinus, M. DELFINO, - AUG 2015
B. Coronary sinus anterior cardiac veins and least cardiac vein (aka MD (TOP 10
C. Small cardiac vein Thebesian vein) drain into the right atrium. The - FEB 2015
D. Anterior cardiac vein following drains into the coronary sinus: great MED
E. Thebesian vein cardiac vein, middle cardiac vein, small cardiac BOARDS;
vein, posterior vein of left ventricle. TOPNOTCH
MD FROM
FATIMA)
194 During a difficult dental extraction, patient accidentally Because the right bronchus is the wider and more JEAN PAOLO FINAL EXAM
aspirated a tooth. He was immediately brought to you at the direct continuation of the trachea, foreign bodies M. DELFINO, - AUG 2015
ER. Upon PE, you noted that breath sounds are decreased on tend to enter the right instead of the left bronchus. MD (TOP 10
an area of the chest. There was noted intermittent cough. - FEB 2015
Where is the foreign body probably located? MED
A. trachea BOARDS;
B. R primary bronchus TOPNOTCH
C. L secondary bronchus MD FROM
D. Larynx FATIMA)
E. carina

195 What is the reason for testis descent during infancy? In the male, the testis descends through the pelvis JEAN PAOLO FINAL EXAM
A. Maternal estrogen and inguinal canal during the seventh and eighth M. DELFINO, - AUG 2015
B. Testosterone months of fetal life. The normal stimulus for the MD (TOP 10
C. Temperature descent of the testis is testosterone, which is - FEB 2015
D. pH secreted by the fetal testes.SIMILAR TO PREVIOUS MED
E. Gravity BOARD EXAM CONCEPT/PRINCIPLE. BOARDS;
TOPNOTCH
MD FROM
FATIMA)
196 Lesion in CN III will result to which eye position? CN III supplies all extraocular muscles except JEAN PAOLO FINAL EXAM
A. Abducted and depressed lateral rectus and superior oblique. The eye M. DELFINO, - AUG 2015
B. Adducted and extorted position after injury to CN III is abducted and MD (TOP 10
C. Abducted and extorted depressed. Lesion in CN IV would result to - FEB 2015
D. Adducted and depressed extorted eyeball while lesion in CN VI would result MED
E. A and C to adducted position. BOARDS;
TOPNOTCH
MD FROM
FATIMA)
197 Thiazide diuretic acts on this portion of the nephron? giveaway question last boards JEAN PAOLO FINAL EXAM
A. PCT M. DELFINO, - AUG 2015
B. DCT MD (TOP 10
C. Ascending limb of LOH - FEB 2015
D. Collecting duct MED
E. Collecting tubule BOARDS;
TOPNOTCH
MD FROM
FATIMA)
198 Which structure is not traversed during pericardiocentesis? Layers traversed during pericardiocentesis: JEAN PAOLO FINAL EXAM
A. Rectus sheath subxiphoid approach- skin, fascia, rectus sheath, M. DELFINO, - AUG 2015
B. Rectus abdominis rectus abdominis, fibrous layer of pericardium, MD (TOP 10
C. Serous pericardium parietal layer of visceral pericardium - FEB 2015
D. Endothoracic fascia MED
E. Fibrous layer of pericardium BOARDS;
TOPNOTCH
MD FROM
FATIMA)
199 Goblet cells are present until what structure in the Respiratory epithelium (pseudostratified JEAN PAOLO FINAL EXAM
respiratory system? columnar with goblet cells) is present up to the M. DELFINO, - AUG 2015
A. Trachea level of bronchioles. MD (TOP 10
B. Bronchus - FEB 2015
C. Bronchiole MED
D. Terminal bronchiole BOARDS;
E. Respiratory bronchiole TOPNOTCH
MD FROM
FATIMA)
200 The following are anterior relations of the right kidney Transversus abdominis is at the posterior of the JEAN PAOLO FINAL EXAM
except? kidney, not anterior M. DELFINO, - AUG 2015
A. adrenal MD (TOP 10
B. liver - FEB 2015
C. transversus abdominis MED
D. R colic flexure BOARDS;
E. 2nd part of duodenum TOPNOTCH
MD FROM
FATIMA)
201 This shunt muscle contains the median nerve and the With the deltoid and long head of the triceps, the GRACE DIAGNOSTIC
brachial artery. coracobrachialis serves as a shunt muscle, ARVIOLA, EXAM - FEB
A. Biceps brachii resisting downward dislocation of th e head of the MD (TOP 3 - 2015
B. Brachialis humerus, as when carrying a heavy suitcase. AUG 2014
C. Coracobrachialis MED
D. Deltoid BOARDS;
E. Triceps brachii TOPNOTCH
MD)

TOPNOTCH MEDICAL BOARD PREP ANATOMY SUPEREXAM Page 26 of 94


For inquiries visit www.topnotchboardprep.com.ph or email us at topnotchmedicalboardprep@gmail.com
TOPNOTCH MEDICAL BOARD PREP ANATOMY SUPEREXAM
For inquiries visit www.topnotchboardprep.com.ph or email us at topnotchmedicalboardprep@gmail.com
Item QUESTION EXPLANATION AUTHOR TOPNOTCH
# EXAM
202 The joint that is responsible for thumb opposition is: The CMC joint of the thumb is a saddle joint. Other GRACE DIAGNOSTIC
A. carpometacarpal joint CMC joints are of the plane type of synovial joint. ARVIOLA, EXAM - FEB
B. interphalangeal joint MD (TOP 3 - 2015
C. radiocarpal joint AUG 2014
D. metacarpophalangeal joint MED
E. intercarpal joint BOARDS;
TOPNOTCH
MD)
203 During CABG, the surgeon inserted his finger into the In the embryology of the heart, as the heart tube GRACE DIAGNOSTIC
transverse pericardial sinus. What structures are folds, its venous end moves posterosuperiorly so ARVIOLA, EXAM - FEB
demarcated? that the venous end of the tube lies adjacent to the MD (TOP 3 - 2015
A. SVC, pulmonary trunk, and aorta arterial end, separated only by the transverse AUG 2014
B. IVC, pulmonary veins pericardial sinus. MED
C. Right and left atria BOARDS;
D. Right and left ventricles TOPNOTCH
E. SVC and IVC MD)

204 The oblique vein of Marshall is a remnant of: The oblique vein of the left atrium (of Marshall) is GRACE DIAGNOSTIC
A. Right IVC a small vessel, relatively unimportant postnatally, ARVIOLA, EXAM - FEB
B. Right SVC that descends over the posterior wall of the left MD (TOP 3 - 2015
C. Left SVC atrium and merges with the great cardiac vein to AUG 2014
D. Left IVC form the coronary sinus. This occasionally persists MED
E. Left brachiocephalic vein in adults, replacing or augmenting the right SVC. BOARDS;
**Expect at least one question in embryology.** TOPNOTCH
MD)
205 This space is involved in inguinal hernia repair. The space of Bogros is the anterolateral part of the GRACE DIAGNOSTIC
A. Space of Burns potential space between the transversalis fascia ARVIOLA, EXAM - FEB
B. Space of Bogros and the parietal peritoneum. This provides a plane MD (TOP 3 - 2015
C. Space of Retzius that can be opened without entering the AUG 2014
D. Space of Douglas membranous peritoneal sac. Thus, the risk of MED
E. Space of Nuck contamination is minimized. This sac is used for BOARDS;
placing prostheses when repairing inguinal TOPNOTCH
hernias. MD)
206 What vertebra demarcates the junction of the sigmoid colon The sigmoid colon extends from the iliac fossa to GRACE DIAGNOSTIC
and the rectum? the S3 segment, where it joins the rectum. ARVIOLA, EXAM - FEB
A. L5 MD (TOP 3 - 2015
B. S1 AUG 2014
C. S2 MED
D. S3 BOARDS;
E. S4 TOPNOTCH
MD)

207 During childbirth, which levator ani muscle is most often The pubococcygeus, the main and most medial GRACE DIAGNOSTIC
injured? part of the levator ani, is torn most often during ARVIOLA, EXAM - FEB
A. Puborectalis childbirth. MD (TOP 3 - 2015
B. Pubococcygeus AUG 2014
C. Iliococcygeus MED
D. Coccygeus BOARDS;
E. Sphincter urethra TOPNOTCH
MD)

208 Which structure forms the divide between vagal and pelvic Orad to the left colic flexure, the parasympathetic GRACE DIAGNOSTIC
splanchnic parasympathetic nerves? innervation of the large intestine is vagally- ARVIOLA, EXAM - FEB
A. Hepatic flexure mediated. Aborad to this, it is innervated by pelvic MD (TOP 3 - 2015
B. Splenic flexure splanchnic nerves. AUG 2014
C. Junction between sigmoid colon and rectum MED
D. Junction between rectum and anus BOARDS;
E. Dentate line TOPNOTCH
MD)

209 The Ligament of Treitz corresponds to what vertebra? The ligament of Treitz marks the duodenojejunal GRACE DIAGNOSTIC
A. L1 junction at the level of L2 vertebra, 2-3 cm to the ARVIOLA, EXAM - FEB
B. L2 left of the midline. MD (TOP 3 - 2015
C. L3 AUG 2014
D. L4 MED
E. L5 BOARDS;
TOPNOTCH
MD)
210 What structures form the left sagittal fissure of the liver? The gallblader and IVC form the right sagittal GRACE DIAGNOSTIC
A. Ligamentum teres and ligamentum venosun fissure. ARVIOLA, EXAM - FEB
B. Right and left hepatic lobes MD (TOP 3 - 2015
C. Caudate lobe and quadrate lobe AUG 2014
D. Galbblader and IVC MED
E. Falciform ligament and coronary ligament BOARDS;
TOPNOTCH
MD)
211 Trigeminal neuralgia most commonly affects what division of CN V1 is least frequently involved. GRACE DIAGNOSTIC
cranial nerve V? ARVIOLA, EXAM - FEB
A. V1 MD (TOP 3 - 2015
B. V2 AUG 2014
C. V3 MED
D. V4 BOARDS;
E. None; it has no preferences TOPNOTCH
MD)

TOPNOTCH MEDICAL BOARD PREP ANATOMY SUPEREXAM Page 27 of 94


For inquiries visit www.topnotchboardprep.com.ph or email us at topnotchmedicalboardprep@gmail.com
TOPNOTCH MEDICAL BOARD PREP ANATOMY SUPEREXAM
For inquiries visit www.topnotchboardprep.com.ph or email us at topnotchmedicalboardprep@gmail.com
Item QUESTION EXPLANATION AUTHOR TOPNOTCH
# EXAM
212 Nodose ganglion is associated with what cranial nerve? The nodose ganglion is the inferior ganglion of the GRACE DIAGNOSTIC
A. V vagus nerve that is concerned with the visceral ARVIOLA, EXAM - FEB
B. VII sensory components of the nerve. The superior MD (TOP 3 - 2015
C. IX ganglion, on the other hand, is concerned with the AUG 2014
D. X general sensory component. MED
E. XI BOARDS;
TOPNOTCH
MD)
213 Serous demilunes are seen in: Serous demilunes are found in glands that produce GRACE DIAGNOSTIC
A. Parotid gland both watery and mucoid secretions, i.e. ARVIOLA, EXAM - FEB
B. Sublingual gland submandibular gland. They consist of elongated MD (TOP 3 - 2015
C. Submandibular gland mucus acini that have a crescentic cap of serous AUG 2014
D. Lacrimal gland cells over their ends. MED
E. Sebaceous gland BOARDS;
TOPNOTCH
MD)
214 Synapsis during meiosis occurs in: Synapsis is the event wherein pairs of conjoined GRACE DIAGNOSTIC
A. Prophase chromatids derived from homologous ARVIOLA, EXAM - FEB
B. Metaphase chromosomes come together on the equatorial MD (TOP 3 - 2015
C. Anaphase plate to form groups of four chromatids called AUG 2014
D. Telophase tetrads MED
E. Interphase BOARDS;
TOPNOTCH
MD)
215 Schmidt-Lantermann clefts are formed by: These clefts are formed when Schwann cells GRACE DIAGNOSTIC
A. Cardiac myocytes envelope neurons with myelin. These represent ARVIOLA, EXAM - FEB
B. Endothelial cells thin threads of Schwann cell cytoplasm that MD (TOP 3 - 2015
C. Goblet cells pursue a spiral course from the cell body nearly to AUG 2014
D. Schwann cells the axon. MED
E. B-cells BOARDS;
TOPNOTCH
MD)
216 The acrosome reaction of the sperm involves what receptor The zona pellucida is composed of three GRACE DIAGNOSTIC
of the zona pellucida? glycoproteins: ZP1, ZP2, and ZP3. ARVIOLA, EXAM - FEB
A. ZP1 MD (TOP 3 - 2015
B. ZP2 AUG 2014
C. ZP3 MED
D. ZP4 BOARDS;
E. ZP5 TOPNOTCH
MD)

217 Which epithelium has only a single layer of cells? The trachea is lined with pseudostratified GRACE DIAGNOSTIC
A. Skin columnar epithelium. Pseudostratified implies a ARVIOLA, EXAM - FEB
B. Trachea single layer of cells that only appear as multiple MD (TOP 3 - 2015
C. Urinary bladder layers when viewed under the light microscope. AUG 2014
D. Cornea MED
E. Sweat glands BOARDS;
TOPNOTCH
MD)
218 Herring bodies contain: Herring bodies are found in the posterior GRACE DIAGNOSTIC
A. Oxytocin pituitary. They contain ADH and oxytocin. ARVIOLA, EXAM - FEB
B. Prolactin MD (TOP 3 - 2015
C. Growth Hormone AUG 2014
D. Melatonin MED
E. Aldosterone BOARDS;
TOPNOTCH
MD)
219 Which cells undergo endomitosis? In endomitosis, the DNA undergoes multiple GRACE DIAGNOSTIC
A. Neurons replications without division of the cytoplasm. ARVIOLA, EXAM - FEB
B. Ovum This results in a giant polypoid cell. MD (TOP 3 - 2015
C. RBCs AUG 2014
D. Platelets MED
E. Keratinocytes BOARDS;
TOPNOTCH
MD)
220 High endothelial venules are found in: HEVs are the portal of entry of new blood-borne GRACE DIAGNOSTIC
A. Thymus lymphocytes. ARVIOLA, EXAM - FEB
B. Lymph nodes MD (TOP 3 - 2015
C. Bone marrow AUG 2014
D. Spleen MED
E. All of the above BOARDS;
TOPNOTCH
MD)

TOPNOTCH MEDICAL BOARD PREP ANATOMY SUPEREXAM Page 28 of 94


For inquiries visit www.topnotchboardprep.com.ph or email us at topnotchmedicalboardprep@gmail.com
TOPNOTCH MEDICAL BOARD PREP ANATOMY SUPEREXAM
For inquiries visit www.topnotchboardprep.com.ph or email us at topnotchmedicalboardprep@gmail.com
Item QUESTION EXPLANATION AUTHOR TOPNOTCH
# EXAM
221 Which of the following manifestation is consistent if the mass lesion on the ® sphenoid wing will directly LEAN MIDTERM
patient presents with a ® sphenoid wing meningioma? compress structures at the middle cranial fossa. ANGELO EXAM 1 -
A. Inability to protrude the tongue Skull base openings located at the sphenoid are as SILVERIO, FEB 2015
B. Inability to close the ® eye and wrinkle the ipsilateral follows: optic canal, Superior orbital fissure, MD (TOP 4 -
forehead rotundum, spinsoum, ovale. Hypoglossal nerve AUG 2014
C. Weakness in closing the jaw responsibe for tongue movement is not affected MED
D. increase risk for aspiration since it exits through the hypoglossal canal found BOARDS;
E. nystagmus at the occipital bone.facial nerve as well as the TOPNOTCH
vestibular nerve passes through the internal MD), MD
acoustic meatus located at the petrous part of the
temporal bone. therefore, facial movement and
balance perception is not affected. pt will not have
increase risk for aspiration since the vagus and
glossopharyngeal nerve passing through the
jugular foramen is not affected.
222 which of the following structure will strictly pass through the Abducent nerve is the only cranial nerve that will LEAN MIDTERM
cavernous sinus along with the internal carotid artery? pass through the fenestrations of the cavernous ANGELO EXAM 1 -
A. Abducent nerve sinuses. CN III,IV,V1,and V2 runs forward along SILVERIO, FEB 2015
B. Opthalmic division of trigeminal nerve the lateral wall of the sinus and not through the MD (TOP 4 -
C. Trochlear nerve sinus AUG 2014
D. Oculomotor nerve MED
E. Maxillary nerve BOARDS;
TOPNOTCH
MD), MD
223 A 32 y/o male suffered a skull base fracture secondary MVA mandibular nerve provides motor to the muscles LEAN MIDTERM
causing transection of the (L) mandibular nerve. During of mastication, general sensation to the ant 2/3 of ANGELO EXAM 1 -
recovery, which of the following will the patient unable to the tongue and the lower jaw, and innervation to SILVERIO, FEB 2015
perform? tensor veli palatini. Among the choices, patient MD (TOP 4 -
A. Puff the left cheek will not be able to tense the ipsilateral soft palate. AUG 2014
B. Taste sensation of the left anterior 2/3rd of the tongue buccinator and taste sensation of the anterior MED
C. tense the ipsilateral palate tongue is supplied by the facial nerve. levator veli BOARDS;
D. raises the soft palate during swallowing palatini which raises the soft palate is supplied by TOPNOTCH
E. none of the above the pharyngeal plexus. MD), MD

224 what is the characteristic feature of a 3rd lumbar vertebrae? typical lumbar vertebrae has the ff characteristics: LEAN MIDTERM
A. Short slender transverse process large kidney shaped body, strong cylindical ANGELO EXAM 1 -
B. Superior articular process face laterally pedicles, thick lamina, triangular vertebral SILVERIO, FEB 2015
C. Thick cylindrical pedicles foramen, long slender transverse process, short, MD (TOP 4 -
D. Large round vertebral foramen flat, quadrangular spinous process. Superior AUG 2014
E. Long flat quadrangular spinous process articular process faces medially, and inferior MED
articular process faces laterally. BOARDS;
TOPNOTCH
MD), MD
225 A 56 y/o male underwent a colonoscopy secondary to recent at the distance of 1.5inches of 4cm the scope is LEAN MIDTERM
onset weightloss, change in the stool caliber, and already at the rectal ampulla. At the distance of ANGELO EXAM 1 -
hematochezia. During the procedure, a hemorrhagic 6.5in or 16cm, rectosigmoid junction is reach. SILVERIO, FEB 2015
polypoid mass noted on the anterior wall of the colon 14 cm Therefore, the mass is located at the rectum. MD (TOP 4 -
from the anal verge. where is the exact location of the mass? AUG 2014
A. anal canal MED
B. rectum BOARDS;
C. rectosigmoid TOPNOTCH
D. sigmoid MD), MD
E. cannot be determine

226 Which of the following is true about the liver the centrilobular region of the liver acinus is the LEAN MIDTERM
A. Tha main suspensory structure of the liver is the hepatic most vulnerable to ischemic insult while the ANGELO EXAM 1 -
veins draining to the inferior vena cava periportal region is the most vulnerable to toxin SILVERIO, FEB 2015
B. Periportal regions of the liver acinus is highly vulnerable induced injury. The line of Cantlie functionally MD (TOP 4 -
to ischemic insult divides the liver into right and left in terms of AUG 2014
C. cantlie line separates the IV segment from V and VII blood supply. falciform ligament only demarcates MED
segment the liver anatomically. the caudate lobe is the 1st BOARDS;
D. the falciform ligament divides the liver into left and right liver segment ( posterior segment). TOPNOTCH
functionally in terms of blood supply MD), MD
E. the caudate lobe belongs to the left inferior segment

227 aneurysmal dilatation and thrombosis of the proximal superior mesenteric artery lies behind the neck of LEAN MIDTERM
segment of the superior mesenteric artery will produce the pancreas. It traverses downward crossing the ANGELO EXAM 1 -
which of the following symptoms? 3rd part of the duodenum anteriorly. It provides SILVERIO, FEB 2015
A. gross hematochezia blood supply from the 2nd part of the duodenum MD (TOP 4 -
B. Jaundice secondary to necrosis and inflammation of the up to proximal 2/3 of the transverse colon. AUG 2014
head of the pancreas Referred pain is periumbilical because it supplies MED
C. Upper GI obstruction presenting as nonbilous emesis the midgut derivative. patient will present with BOARDS;
D. compression of the head of the pancreas UGI obstruction however it would be a bilous type TOPNOTCH
E. severe continuous epigastric pain. of emesis since the biliary drainage is proximal to MD), MD
the external obstruction. the head of the pancreas
will least likely to necrosis since there extensive
anastomosis b/w SMA and gastroduodenal artery.

TOPNOTCH MEDICAL BOARD PREP ANATOMY SUPEREXAM Page 29 of 94


For inquiries visit www.topnotchboardprep.com.ph or email us at topnotchmedicalboardprep@gmail.com
TOPNOTCH MEDICAL BOARD PREP ANATOMY SUPEREXAM
For inquiries visit www.topnotchboardprep.com.ph or email us at topnotchmedicalboardprep@gmail.com
Item QUESTION EXPLANATION AUTHOR TOPNOTCH
# EXAM
228 which of the following is a derivative of the 5th pharyngeal thymus gland is derived from 3rd pharyngeal LEAN MIDTERM
pouch or the ultimobranchial body? pouch, same is true with the inferior parathyroid ANGELO EXAM 1 -
A. thymus gland. The superior parathyroid is derived from SILVERIO, FEB 2015
B. Palatine tonsils 4th pharyngeal pouch. The C cells or parafollicular MD (TOP 4 -
C. C cells cells is the only derivative of the 5th pharyngeal AUG 2014
D. Superior parathyroid gland pouch. Palatine tonsils is related to 2nd MED
E. None of the above pharyngeal pouch. BOARDS;
TOPNOTCH
MD), MD
229 which of the following is a functional remnant of the at birth, the proximal portion of the umbilical LEAN MIDTERM
umbilical arteries? arteries remain as the superior vesical artery. ANGELO EXAM 1 -
A. Median umbilical ligament While the distal part will become vestigial SILVERIO, FEB 2015
B. Distal part of the inferior epigastric artery remnant known as the medial umbilical ligament. MD (TOP 4 -
C. Superior vesical artery AUG 2014
D. Distal part of the common iliac artery MED
E. None of the above BOARDS;
TOPNOTCH
MD), MD
230 what layer of the epidermis contains eleidin? stratum lucidum contains refractile droplets of LEAN MIDTERM
A. Stratum spinosum eleidin in the cytoplasm which is responsible for ANGELO EXAM 1 -
B. Stratum germinativum its translucent appearance during microscopy. SILVERIO, FEB 2015
C. Stratum lucidum MD (TOP 4 -
D. Stratum granulosum AUG 2014
E. All of the above MED
BOARDS;
TOPNOTCH
MD), MD
231 which of the following is not true regarding regarding the the arterial vasa vasorum is lesser and extends LEAN MIDTERM
histological difference between an artery and a vein ? only up to tunica media, while in the veins they are ANGELO EXAM 1 -
A. The thickest coat of the artery is the tunica media while more prominent and even extends up to tunica SILVERIO, FEB 2015
that of the vein is the tunica adventitia intima. Expect hard questions in the board exam. MD (TOP 4 -
B. Vasa vasorum is more abundant in the arteries than in AUG 2014
the veins MED
C. the tunica intima of the arteries scalloped or corrugated BOARDS;
during cross section. TOPNOTCH
D. the internal elastic lamina is more prominent in the MD), MD
arteries than in the veins
E. none of the above

232 A 37 y/o male suffered a motor vehicular accident resulting radial nerve innervates the ECRL prior to LEAN MIDTERM
to transection of the deep branch of the radial nerve branching out to superfical and deep branch. The ANGELO EXAM 1 -
secondary to comminuted fracture of the right radius. Which rest of the choices are supplied by the deep branch SILVERIO, FEB 2015
of the following movements will the patient still be able to of the radial nerve. MD (TOP 4 -
perform? AUG 2014
A. extends and abducts the hand at the wrist joint MED
B. extend the distal phalanx of the thumb BOARDS;
C. extends and adducts hand at the wrist joint TOPNOTCH
D. extends fingers and hands MD), MD
E. none of the above

233 a 37 y/o male had suppurative inflammation of the wrist the components of the carpal tunnel are as LEAN MIDTERM
joint secondary to previous open trauma. Wrist radiograph follows: FPL, FDP, FPS, median nerve. The FCR ANGELO EXAM 1 -
shows affectation of the carpal tunnel. Which of the following tendon traverses the wrist joint through a SILVERIO, FEB 2015
structures will not be affected? separate compartment lateral to the carpal tunnel. MD (TOP 4 -
A. Flexor digitorum superficialis tendon. AUG 2014
B. Flexor pollicis longus tendon. C. Flexor carpi radialis MED
tendon D. Flexor digitorum profundus E. none of the above. BOARDS;
TOPNOTCH
MD), MD
234 A 47 y/o female is suffering from an suppurative ischial lesser sciatic foramen provides entrance into the LEAN MIDTERM
bursitis affecting the lesser sciatic foramen. Which of the perineum from the gluteal region. The follwing ANGELO EXAM 1 -
following structures will be affected? structures passes through this foramen: obturator SILVERIO, FEB 2015
A. Inferior gluteal nerve internus tendon, pudendal nerve, internal MD (TOP 4 -
B. Sciatic nerve pudendal artery and vein, obturator internus AUG 2014
C. Posterior cutaneous nerve of the thigh nerve. MED
D. tendon of the obturator internus BOARDS;
E. none of the above TOPNOTCH
MD), MD

235 A 68 y/o female suffering from a chronic osteoarthritis of Quadriceps muscle is composed of Vastus lateralis, LEAN MIDTERM
the ® knee underwent total knee arthroplasty. Which of the intermedius, medialis and rectus femoris. During ANGELO EXAM 1 -
following muscle needs to be strengthen since it is the last to chronic knee disease, alteration of the knee joint SILVERIO, FEB 2015
recover among the quadriceps group? mobility and Limitation of motion causes MD (TOP 4 -
A. Vastus lateralis weakness of the quadriceps. Among the AUG 2014
B. popliteus quadriceps, it I the vastus medialis that is first to MED
C. Vastus medialis atrophy and last to recovery. BOARDS;
D. rectus femoris TOPNOTCH
E. Vastus intermedius MD), MD

TOPNOTCH MEDICAL BOARD PREP ANATOMY SUPEREXAM Page 30 of 94


For inquiries visit www.topnotchboardprep.com.ph or email us at topnotchmedicalboardprep@gmail.com
TOPNOTCH MEDICAL BOARD PREP ANATOMY SUPEREXAM
For inquiries visit www.topnotchboardprep.com.ph or email us at topnotchmedicalboardprep@gmail.com
Item QUESTION EXPLANATION AUTHOR TOPNOTCH
# EXAM
236 A 21 y/o basketball player complains of pain and instability McMurray is performed with the patient is in LEAN MIDTERM
of the (L) knee. Upon your PE. It shows that it is positive for supine position and the test leg is in full flexion. ANGELO EXAM 1 -
Apley and McMurray test. Which structure is primarily The examiner laterally rotates the knee and then SILVERIO, FEB 2015
involved? extend it. If there would be click or pain upon MD (TOP 4 -
A. ACL extension. It is highly suggestive of meniscal tear. AUG 2014
B. PCL Apley test is done in prone with the test leg in 90 MED
C. Medial meniscus knee flexion. examiner applies compresion to the BOARDS;
D. Plica syndrome knee joint while doing rotation. if there is pain it is TOPNOTCH
E. Patella suggestive of meniscal tear. MD), MD

237 A 21 y/o male suffered a monteggia fracture after being hit Monteggia fracture is a fracture of the ulna with LEAN MIDTERM
during a basketball match. Upon your PE, you noted that he concomitant dislocation of the proximal ANGELO EXAM 1 -
cannot perform OK sign and there is extension of th index radioulnar joint. It is usually prone to anterior SILVERIO, FEB 2015
DIP and thumb IP joint. Which of the following is primarily interosseous nerve injury. This nerve supplies MD (TOP 4 -
affected. ? FPL, lateral half FDP, pronator quadratus, thenar AUG 2014
A. radial nerve eminence and lateral two lumbricals.patient will MED
B. Median nerve not be able to flex the DIP of the index and IP of BOARDS;
C. ulnar nerve the thumb. there will be no sensory deficit. TOPNOTCH
D. anterior interosseous nerve MD), MD
E. none of the above

238 Compression of the nerve that passess through the canal of the posterior interosseous branch of the radial LEAN MIDTERM
Frohse will cause which of the following deficit? nerve passes through the canal of frohse formed ANGELO EXAM 1 -
A. Weakness of wrist flexors by the two heads of the supinator muscle. It is a SILVERIO, FEB 2015
B. Numbness and paresthesia along dorsal aspect of the purely motor nerve supplying the extensor MD (TOP 4 -
lateral hand. muscles of the forearm. AUG 2014
C. Wrist drop MED
D. Weakness of elbow extension BOARDS;
E. none of the above TOPNOTCH
MD), MD

239 What structure is an upward extension of the posterior SIMILAR TO PREVIOUS BOARD EXAM LEAN MIDTERM
longitudinal ligament of the spine? CONCEPT/PRINCIPLE. ANGELO EXAM 1 -
A. Tectorial membrane SILVERIO, FEB 2015
B. Apical ligament MD (TOP 4 -
C. Alar ligament AUG 2014
D. Cruciate ligament MED
E. None of the above BOARDS;
TOPNOTCH
MD), MD
240 retrolisthesis or backward extension of T8 vertebra will use this guide in determining which spinal cord is LEAN MIDTERM
affect what level of spinal cord? contiguous with a given vertebra. Cervical ANGELO EXAM 1 -
A. T10 vertebra- add 1; upper thoracic- add 2, T7-T9 - SILVERIO, FEB 2015
B. T11 add 3, tenth thoracic- L1 and L2, 11th thoracic-L3 MD (TOP 4 -
C. T12 and L4, 12th thoracic- L5 segment, 1st lumbar - AUG 2014
D. L1 sacral and coccygeal cord segment. therefore T8 MED
E. None of the above vertebra will affect T11 spinal segment. BOARDS;
TOPNOTCH
MD), MD
241 The thyroid isthmus is known to lie in which area? SIMILAR TO PREVIOUS BOARD EXAM KEVIN MIDTERM 2
A. 2nd to 4th tracheal rings CONCEPT/PRINCIPLE. Isthmus is situated BRYAN LO, EXAM - FEB
B. 3rd to 5th tracheal rings between the 2nd to 4th tracheal rings MD (TOP 7 - 2015
C. midway from the thyroid and cricoid cartilages at C4 AUG 2014
D. C2 to C4 MED
E. C5 to C6 BOARDS;
TOPNOTCH
MD)
242 A 56 year old male patient suffered a motor vehicular SIMILAR TO PREVIOUS BOARD EXAM KEVIN MIDTERM 2
accident, he suffered multiple rib fractures and had CONCEPT/PRINCIPLE, 2nd ICS Mid clavicular for BRYAN LO, EXAM - FEB
ecchymoses along the right hemithorax, chest xray revealed emergency thoracentesis for pneumothorax, use MD (TOP 7 - 2015
blunting of the right costophrenic sulci. The surgeon plans to upper border of the rib to avoid neurovascular AUG 2014
insert a chest tube to drain the fluid at which site? bundle, be sure you're in the proper hemithorax MED
A. 2nd ICS right mid clavicular line upper border of the rib BOARDS;
B. 7th ICS right posterior axillary line upper border of the rib TOPNOTCH
C. 5th ICS right anterior axillary line upper border of the rib MD)
D. 5th ICS left midaxillary line lower border of the rib
E. 7th ICS left midaxillary line lower border of the rib

243 The ovaries are lined by this specific epithelium: ___________, SIMILAR TO PREVIOUS BOARD EXAM KEVIN MIDTERM 2
A. simple cuboidal CONCEPT/PRINCIPLE it is simple cuboidal BRYAN LO, EXAM - FEB
B. simple squamous MD (TOP 7 - 2015
C. simple columnar AUG 2014
D. stratified squamous MED
E. pseudostratified columnar BOARDS;
TOPNOTCH
MD)

TOPNOTCH MEDICAL BOARD PREP ANATOMY SUPEREXAM Page 31 of 94


For inquiries visit www.topnotchboardprep.com.ph or email us at topnotchmedicalboardprep@gmail.com
TOPNOTCH MEDICAL BOARD PREP ANATOMY SUPEREXAM
For inquiries visit www.topnotchboardprep.com.ph or email us at topnotchmedicalboardprep@gmail.com
Item QUESTION EXPLANATION AUTHOR TOPNOTCH
# EXAM
244 The SA and AV node are the primary and secondary SIMILAR TO PREVIOUS BOARD EXAM KEVIN MIDTERM 2
pacemakers of the heart respectively, from which of the CONCEPT/PRINCIPLE, the right coronary artery BRYAN LO, EXAM - FEB
following vessles does their predominant blood supply come supplies both SA and AV nodes MD (TOP 7 - 2015
from? AUG 2014
A. left circumflex MED
B. left anterior descending BOARDS;
C. right coronary artery TOPNOTCH
D. left coronary artery MD)
E. both B and C

245 A patient came in with a pea sized lump non tender on the SIMILAR TO PREVIOUS BOARD EXAM KEVIN MIDTERM 2
wrist, he was assessed to have a ganglion cyst, the surgeon CONCEPT/PRINCIPLE, usually ganglion cysts BRYAN LO, EXAM - FEB
knows that he must carefully dissect it out from this arises from tendon sheaths MD (TOP 7 - 2015
underlying tissue? AUG 2014
A. extensor muscles MED
B. superficial fasia BOARDS;
C. tendon sheath TOPNOTCH
D. synovial membrane MD)
E. deep fascia

246 Which of the following is considered as a direct branch from SIMILAR TO PREVIOUS BOARD EXAM KEVIN MIDTERM 2
the aorta? CONCEPT/PRINCIPLE, direct branches from the BRYAN LO, EXAM - FEB
A. right common carotid aorta are right brachiocephalic, left common MD (TOP 7 - 2015
B. internal jugular carotid, left subclavian, AUG 2014
C. right subclavian MED
D. left subclavian BOARDS;
E. left brachiocephalic TOPNOTCH
MD)

247 56 year old male chronic smoker had a history of weight loss, SIMILAR TO PREVIOUS BOARD EXAM KEVIN MIDTERM 2
body malaise comes for consult due to multiple swellings or CONCEPT/PRINCIPLE, Nasopharyngeal primaries BRYAN LO, EXAM - FEB
neck masses. Which set of lymph nodes should be palpated are the most common subtype to involve the MD (TOP 7 - 2015
and biopsied for suspicion of metastasis of nasopharyngeal posterior cervical lymph node chain AUG 2014
carcinoma MED
A. upper jugular BOARDS;
B. lower jugular TOPNOTCH
C. posterior cervical MD)
D. submandibular
E. middle jugular

248 A 50 year old female patient comes in for a mass near the SIMILAR TO PREVIOUS BOARD EXAM KEVIN MIDTERM 2
nipple areolar complex of the right breast, which of the CONCEPT/PRINCIPLE, the most common sentinel BRYAN LO, EXAM - FEB
following lymph nodes would most likely be affected node involved is still the axillary lymph nodes MD (TOP 7 - 2015
A. superficial pectoral AUG 2014
B. deep pectoral MED
C. internal mammary BOARDS;
D. axillary TOPNOTCH
E. all of the above MD)

249 A farmer swallowed some santol seeds, which area of the GI SIMILAR TO PREVIOUS BOARD EXAM KEVIN MIDTERM 2
tract will these seeds most likely cause obstruction CONCEPT/PRINCIPLE, narrowed area due to the BRYAN LO, EXAM - FEB
A. antral pyloric area ileocecal sphincter where the seeds may lodge MD (TOP 7 - 2015
B. ileocecal junction cause obstruction AUG 2014
C. colorectal junction MED
D. splenic flexure BOARDS;
E. duodenojejunal junction TOPNOTCH
MD)

250 Which of the following bones in the body undergoes SIMILAR TO PREVIOUS BOARD EXAM KEVIN MIDTERM 2
endochondral ossification? CONCEPT/PRINCIPLE, clavicle medial end is BRYAN LO, EXAM - FEB
A. mandible endochondral lateral end is intramembranous MD (TOP 7 - 2015
B. clavicle ossification Endochondral ossification - cartilage is AUG 2014
C. maxilla initially present. MED
D. frontal bone BOARDS;
E. parietal bone TOPNOTCH
MD)

251 Which is responsible for detecting linear (vertical) SIMILAR TO PREVIOUS BOARD EXAM KEVIN MIDTERM 2
deceleration? CONCEPT/PRINCIPLE. Slightly vague since both BRYAN LO, EXAM - FEB
A. utricle utricle and saccule are said to detect linear MD (TOP 7 - 2015
B. saccule acceleration and deceleration. Utricle detects AUG 2014
C. anterior semicircular canals linear acceleration and deceleration. Saccule more MED
D. posterior semicircular canals on the vertical acceleration as in riding an BOARDS;
E. lateral semicircular canals elevator, semicircular canals deal with angular TOPNOTCH
acceleration MD)

TOPNOTCH MEDICAL BOARD PREP ANATOMY SUPEREXAM Page 32 of 94


For inquiries visit www.topnotchboardprep.com.ph or email us at topnotchmedicalboardprep@gmail.com
TOPNOTCH MEDICAL BOARD PREP ANATOMY SUPEREXAM
For inquiries visit www.topnotchboardprep.com.ph or email us at topnotchmedicalboardprep@gmail.com
Item QUESTION EXPLANATION AUTHOR TOPNOTCH
# EXAM
252 A patient presenting with adenocarcinoma of the lower 1/3 SIMILAR TO PREVIOUS BOARD EXAM KEVIN MIDTERM 2
of the esophagus would most likely have metastasis to which CONCEPT/PRINCIPLE. Celiac nodes drain the BRYAN LO, EXAM - FEB
of the following lymph nodes? lower 1/3 of the esophagus MD (TOP 7 - 2015
A. bronchomediastinal AUG 2014
B. celiac MED
C. lower jugular BOARDS;
D. diaphragmatic TOPNOTCH
E. posterior cervical MD)

253 The most common anatomical location of the esophageal SIMILAR TO PREVIOUS BOARD EXAM KEVIN MIDTERM 2
tear in boerhaave's syndrome CONCEPT/PRINCIPLE, exact answer should be left BRYAN LO, EXAM - FEB
A. Right distal esophagus posterolateral distal 1/3 of the esophagus MD (TOP 7 - 2015
B. Right middle 1/3 of the esophagus AUG 2014
C. Left distal esophagus MED
D. Left middle 1/3 of the esophagus BOARDS;
E. A and B TOPNOTCH
MD)

254 A 26 year old male was rushed to the ER following a motor SIMILAR TO PREVIOUS BOARD EXAM KEVIN MIDTERM 2
vehicular accident, patient was noted to have clear to CONCEPT/PRINCIPLE, fracture of cribriform plate BRYAN LO, EXAM - FEB
serosanguinous fluid flowing from the nose positive for of the ethmoid may lead to anosmia, and csf leak MD (TOP 7 - 2015
glucose and with (+) halo sign on gauze pad, which of the through the nose. AUG 2014
following bone has most likely been fractured? MED
A. sphenoidal sinuses BOARDS;
B. petrous portion of the temporal bone TOPNOTCH
C. cribriform plate of the ethmoid MD)
D. frontal sinus of the frontal bone
E. pterygoid plate

255 a 25 year old male athlete was playing basketball when he SIMILAR TO PREVIOUS BOARD EXAM KEVIN MIDTERM 2
experienced sudden inversion of his right foot after landing CONCEPT/PRINCIPLE, in inversion of the foot, BRYAN LO, EXAM - FEB
badly from jumping, which of the following ligaments are most commonly damaged is the anterior MD (TOP 7 - 2015
most likely damaged ? talofibular ligament in the lateral side of the foot AUG 2014
A. anterior tibiotalar ligament MED
B. deltoid ligament BOARDS;
C. talocalcaneal ligament TOPNOTCH
D. anterior talofibular ligament MD)
E. calcaneal fibular ligament

256 A 50 year old male hypertensive patient has been SIMILAR TO PREVIOUS BOARD EXAM KEVIN MIDTERM 2
experiencing chest pains, syncopal attacks and exertional CONCEPT/PRINCIPLE, questions are indeed BRYAN LO, EXAM - FEB
dyspnea. Physical exam revealed a systolic murmur, which of becoming more clinical, triad of syncope chest MD (TOP 7 - 2015
the following valvular pathologies are most likely present? pain exertional dyspnea, systolic murmur is classic AUG 2014
A. mitral regurgitation of aortic stenosis, murmur may radiate to the MED
B. mitral stenosis carotids BOARDS;
C. pulmonic stenosis TOPNOTCH
D. aortic stenosis MD)
E. aortic regurgitation

257 A prostate examination via digital rectal examination was SIMILAR TO PREVIOUS BOARD EXAM KEVIN MIDTERM 2
done for a 50 year old male patient, which of the following CONCEPT/PRINCIPLE, DRE palpates the posterior BRYAN LO, EXAM - FEB
lobes of the prostate can be directly felt by this examination? lobe of the prostate area where most carcinomas MD (TOP 7 - 2015
A. middle lobe arise AUG 2014
B. anterior lobe MED
C. posterior lobe BOARDS;
D. lateral lobes TOPNOTCH
E. C and D MD)

258 The membranous urethra is lined by which lining SIMILAR TO PREVIOUS BOARD EXAM KEVIN MIDTERM 2
epithelium? CONCEPT/PRINCIPLE, what was asked was BRYAN LO, EXAM - FEB
A. transitional prostatic urethra lining transitional epithelium, MD (TOP 7 - 2015
B. pseudostratified columnar membranous urethra is a small (1 or 2 cm) AUG 2014
C. stratified squamous portion passing through the external urethral MED
D. simple columnar sphincter. This is the narrowest part of the urethra BOARDS;
E. both B and C lined by pseudostratified columnar epithelium, TOPNOTCH
spongy or penile urethra last part proximally is MD)
pseudostratified columnar and distally is stratified
squamous

TOPNOTCH MEDICAL BOARD PREP ANATOMY SUPEREXAM Page 33 of 94


For inquiries visit www.topnotchboardprep.com.ph or email us at topnotchmedicalboardprep@gmail.com
TOPNOTCH MEDICAL BOARD PREP ANATOMY SUPEREXAM
For inquiries visit www.topnotchboardprep.com.ph or email us at topnotchmedicalboardprep@gmail.com
Item QUESTION EXPLANATION AUTHOR TOPNOTCH
# EXAM
259 A 30 year old male patient had a motorcycle accident, SIMILAR TO PREVIOUS BOARD EXAM KEVIN MIDTERM 2
physical examination revealed absence of sensation at the CONCEPT/PRINCIPLE, know the dermatome and BRYAN LO, EXAM - FEB
lateral and posterior aspects of bilateral lower legs, this myotome levels MD (TOP 7 - 2015
corresponds to what dermatome level? AUG 2014
A. L2-L3 MED
B. L4-L5 BOARDS;
C. L5-S1 TOPNOTCH
D. S1-S2 MD)
E. both A and B

260 On reading the CT scan of a patient revealed a mass at the SIMILAR TO PREVIOUS BOARD EXAM KEVIN MIDTERM 2
level near the tracheal bifurcation, in determining the level of CONCEPT/PRINCIPLE, choose the best answer, BRYAN LO, EXAM - FEB
the lesion, the doctor is sure that at least part of the mass is near the tracheal bifurcation is at the level of T4- MD (TOP 7 - 2015
at level of T5 AUG 2014
A. T2-T3 MED
B. T3-T4 BOARDS;
C. T4-T5 TOPNOTCH
D. T5-T6 MD)
E. T6-T7

261 All of the following structures pass through the aortic hiatus RAYMUND MIDTERM 3
EXCEPT: MARTIN LI, EXAM - FEB
A. Thoracic duct MD (TOP 1 - 2015
B. Aorta AUG 2014
C. Azygos Vein MED
D. All of the above BOARDS;
E. None of the above TOPNOTCH
MD)

262 A left-sided dominant blood supply of the heart is best Right sided dominant - comes from the RCA; left- RAYMUND MIDTERM 3
characterized by: sided dominant - comes from the LMCA MARTIN LI, EXAM - FEB
A. The posterior 1/3 of the interventricular septum is MD (TOP 1 - 2015
supplied by a branch from the left coronary artery AUG 2014
B. The anterior interventricular artery is a branch of the right MED
coronary artery BOARDS;
C. The posterior interventricular artery comes from the left TOPNOTCH
main coronary artery MD)
D. A and C
E. A and B

263 A 7 year old child is brought to you for consult by his mother. A case of nursemaid's elbow cause by subluxation RAYMUND MIDTERM 3
Upon examination, the child is noted to keep his left forearm of radial head MARTIN LI, EXAM - FEB
pronated and partially flexed close to the body. Which is the MD (TOP 1 - 2015
most likely cause? AUG 2014
A. Supracondylar humeral fracture MED
B. Subluxation of the radial head BOARDS;
C. Surgical humeral neck fracture TOPNOTCH
D. Colles fracture MD)
E. None of the above

264 A patient comes to your clinic with complaints of hoarseness Recurrent laryngeal nerve may be injured during RAYMUND MIDTERM 3
of voice. History revealed that the patient recently ligation of the inferior thyroid artery or section of MARTIN LI, EXAM - FEB
underwent thyroid surgery. Injury most probably occurred the ligament of Berry MD (TOP 1 - 2015
during: AUG 2014
A. Ligation of the inferior thyroid artery MED
B. Ligation of the superior thyroid artery BOARDS;
C. Section of the ligament of Berry TOPNOTCH
D. A or C MD)
E. B or C

265 Which of the following statements regarding abdominal Duodenal papilla in the 2nd part of duodenum. RAYMUND MIDTERM 3
structural relationships is correct? Portal vein is formed behind the pancreas MARTIN LI, EXAM - FEB
A. The portal vein is formed posterior to the 3rd portion of MD (TOP 1 - 2015
the duodenum AUG 2014
B. The superior mesenteric artery passes anterior to the MED
horizontal portion of the duodenum BOARDS;
C. The major duodenal papilla is located in the first part of TOPNOTCH
the duodenum MD)
D. All are correct
E. None are correct

TOPNOTCH MEDICAL BOARD PREP ANATOMY SUPEREXAM Page 34 of 94


For inquiries visit www.topnotchboardprep.com.ph or email us at topnotchmedicalboardprep@gmail.com
TOPNOTCH MEDICAL BOARD PREP ANATOMY SUPEREXAM
For inquiries visit www.topnotchboardprep.com.ph or email us at topnotchmedicalboardprep@gmail.com
Item QUESTION EXPLANATION AUTHOR TOPNOTCH
# EXAM
266 Which of the following abdominal structures is NOT derived Speen is derived from mesoderm RAYMUND MIDTERM 3
from the embryologic endoderm? MARTIN LI, EXAM - FEB
A. Stomach MD (TOP 1 - 2015
B. Liver AUG 2014
C. Spleen MED
D. Pancreas BOARDS;
E. All are derived from the endoderm TOPNOTCH
MD)

267 Which of the following structures forms the majority of the Diaphragmatic surface formed by left ventricle. RAYMUND MIDTERM 3
diaphragmatic surface of the heart? Diaphragmatic border by right ventricle MARTIN LI, EXAM - FEB
A. Rigth ventricle MD (TOP 1 - 2015
B. Left ventricle AUG 2014
C. Right Atrium MED
D. Pulmonary vessels BOARDS;
E. Left atrium TOPNOTCH
MD)

268 A 62 year old woman presents to your clinic complaining of Long thoracic nerve injury usually occurs during RAYMUND MIDTERM 3
right arm weakness. Further examination reveals inability to MRM MARTIN LI, EXAM - FEB
abduct the right arm past horizontal position and protrusion MD (TOP 1 - 2015
of the right scapula in an abnormal position noted when AUG 2014
instructed to push against a wall. Which will most likely be a MED
significant finding in the patient’s history? BOARDS;
A. Right Rotator cuff injury TOPNOTCH
B. Right midshaft humeral fracture MD)
C. Poorly fitted crutches
D. Right modified radical mastectomy
E. History of humeral neck fracture

269 Which of the following structures is NOT transmitted CN III,IV,VI,V1 are transmitted through superior RAYMUND MIDTERM 3
through the superior orbital fissure? orbital fissure MARTIN LI, EXAM - FEB
A. Oculomotor nerve MD (TOP 1 - 2015
B. Mandibular branch of the mandibular nerve AUG 2014
C. Abducens nerve MED
D. Ophthalmic branch of the mandibular nerve BOARDS;
E. None of the above TOPNOTCH
MD)

270 A duodenal ulcer located in the lesser curvature of the Bleeding from left gastric artery due to ulcer in RAYMUND MIDTERM 3
stomach is most likely to cause which of the following lesser curvature. MARTIN LI, EXAM - FEB
complications: MD (TOP 1 - 2015
A. Bleeding from the gastroduodenal artery AUG 2014
B. Bleeding from a branch of the celiac artery MED
C. Bleeding from the left gastroepiploic artery BOARDS;
D. Pancreatitis secondary to perforation TOPNOTCH
E. None of the above MD)

271 A 57 year old man presents at the ER with complaints of Review dermatomal distribution RAYMUND MIDTERM 3
lower back pain. Physical examination findings reveals MARTIN LI, EXAM - FEB
weakened dorsiflexion of the foot and decreased sensation of MD (TOP 1 - 2015
the lateral surface of the leg and dorsum of the foot. Which of AUG 2014
the following nerve root is most likely compressed? MED
A. L2 BOARDS;
B. L5 TOPNOTCH
C. L4 MD)
D. S1
E. S2

272 A 5 year old male patient is noted to have a soft nontender Due to patent processus vaginalis RAYMUND MIDTERM 3
fullness of the left scrotum with associated positive MARTIN LI, EXAM - FEB
transillumination. The underlying predisposing factor for MD (TOP 1 - 2015
this condition is similar to that of: AUG 2014
A. Indirect inguinal hernia MED
B. Direct Inguinal hernia BOARDS;
C. Femoral hernia TOPNOTCH
D. Varicocele MD)
E. All of the above

273 A tracheostomy tube is best inserted in which of the Between 2nd and 3rd tracheal rings is preferred. RAYMUND MIDTERM 3
following locations? Above that there is danger of tracheal stenosis. MARTIN LI, EXAM - FEB
A. Between 1st and 2nd tracheal rings Below that there is danger of hitting isthmus and MD (TOP 1 - 2015
B. Between 2nd and 3rd tracheal rings vessels AUG 2014
C. Between 3rd and 4th tracheal rings MED
D. Between 4th and 5th tracheal rings BOARDS;
E. Between the cricoid and the 1st tracheal ring TOPNOTCH
MD)

TOPNOTCH MEDICAL BOARD PREP ANATOMY SUPEREXAM Page 35 of 94


For inquiries visit www.topnotchboardprep.com.ph or email us at topnotchmedicalboardprep@gmail.com
TOPNOTCH MEDICAL BOARD PREP ANATOMY SUPEREXAM
For inquiries visit www.topnotchboardprep.com.ph or email us at topnotchmedicalboardprep@gmail.com
Item QUESTION EXPLANATION AUTHOR TOPNOTCH
# EXAM
274 Which of the following structures is derived from the Portal vein derived from vitelline vein. Smooth RAYMUND MIDTERM 3
embryologic common cardinal veins? part of right atrium derived from sinus venosus. MARTIN LI, EXAM - FEB
A. superior vena cava MD (TOP 1 - 2015
B. pulmonary veins AUG 2014
C. right atrium MED
D. portal vein BOARDS;
E. None of the above TOPNOTCH
MD)

275 A sports player is noted to have an abnormal posterior Anterior drawer sign - displacement of tibia RAYMUND MIDTERM 3
location of the femur in relation to the tibia upon attempt at anterior to femur or femur located posterior in MARTIN LI, EXAM - FEB
passive displacement. Which structure is injured? relation to the tibia MD (TOP 1 - 2015
A. Anterior cruciate ligament AUG 2014
B. Posterior cruciate ligament MED
C. Medial collateral ligament BOARDS;
D. Lateral collateral ligament TOPNOTCH
E. Medial lemniscus MD)

276 When instructed to perform a pudendal nerve block, which RAYMUND MIDTERM 3
of the following landmarks is most important? MARTIN LI, EXAM - FEB
A. Tip of the ischial spine MD (TOP 1 - 2015
B. Ischeal tuberosity AUG 2014
C. Lesser sciatic foramen MED
D. Greater sciatic foramen BOARDS;
E. Perineal body TOPNOTCH
MD)

277 Your resident asked you to insert a chest tube thoracostomy RAYMUND MIDTERM 3
on a patient presenting with pleural effusion. Being a stellar MARTIN LI, EXAM - FEB
intern, you insert the chest tube on the: MD (TOP 1 - 2015
A. 5th ICS MCL upper border of the rib AUG 2014
B. 5th ICS MCL lower border of the rib MED
C. 8th ICS AAL upper border of the rib BOARDS;
D. 5th ICS AAL upper border of the rib TOPNOTCH
E. 5th ICS PAL lower border of the rib MD)

278 Failure of closure of the urethral folds will cause: Hypospadias - failure of urethral fold closure. RAYMUND MIDTERM 3
A. Epispadias Epispadias - extrophy MARTIN LI, EXAM - FEB
B. Phimosis MD (TOP 1 - 2015
C. Extrophy of the bladder AUG 2014
D. Hypospadias MED
E. None of the above BOARDS;
TOPNOTCH
MD)
279 A 43 year old man was brought to the hospital due to a Middle meningeal artery which is a branch of the RAYMUND MIDTERM 3
vehicular accident. Patient was noted to have loss of maxillary artery usual source of epidural MARTIN LI, EXAM - FEB
consciousness at the site of the accident with a lucid interval hematoma MD (TOP 1 - 2015
immediately after. Currently in the ER, the patient is being AUG 2014
evaluated for deterioration of sensorium. Careful history MED
reveals that the patient hit the part of his head near the BOARDS;
anatomic region called the pterion. CT scan revealed a TOPNOTCH
lentiform-shaped bleed. Which structure is most likely MD)
injured??
A. MCA
B. Bridging veins
C. Branch of the maxillary artery
D. Superior sagittal sinus
E. Branch of the internal carotid artery

280 A 23 y.o. male patient complaining of left testicular mass Testicular drained by paraaortic; prostate drained RAYMUND MIDTERM 3
underwent biopsy and was diagnosed to have testicular by internal iliac nodes; scrotum drained by MARTIN LI, EXAM - FEB
cancer. Which group of lymph nodes will primarily be inguinal nodes MD (TOP 1 - 2015
affected once the cancer spreads? AUG 2014
A. deep inguinal lymph nodes MED
B. paraaortic lymph nodes BOARDS;
C. superficial inguinal lymph nodes TOPNOTCH
D. internal iliac lymph nodes MD)
E. external iliac lymph nodes

281 Pain at the area of the anatomic "snuff box" following a fall The floor of the anatomic snuff box is formed by ERIC ROYD FINAL EXAM
on an outstretched hand may be caused by a fracture of the scaphoid and trapezium. Lateral bundary: TALAVERA, - FEB 2015
which of the following? tendons of EPB and AbdPL. Medial Boundary: MD (TOP 1 -
A. Scaphoid tendon of EPL AUG 2014
B. Capitate MED
C. Hamate BOARDS;
D. Pisiform TOPNOTCH
E. None of the above MD)

TOPNOTCH MEDICAL BOARD PREP ANATOMY SUPEREXAM Page 36 of 94


For inquiries visit www.topnotchboardprep.com.ph or email us at topnotchmedicalboardprep@gmail.com
TOPNOTCH MEDICAL BOARD PREP ANATOMY SUPEREXAM
For inquiries visit www.topnotchboardprep.com.ph or email us at topnotchmedicalboardprep@gmail.com
Item QUESTION EXPLANATION AUTHOR TOPNOTCH
# EXAM
282 Where is the fundus of the gallbladder located? The fundus of the gallbadder is located at the level ERIC ROYD FINAL EXAM
A. At the level of the 10th costal cartilage along the linea of the 9th costal cartilage along the linea TALAVERA, - FEB 2015
semilunaris semilunaris MD (TOP 1 -
B. At the transpyloric plane along the linea alba AUG 2014
C. At the level of the umbilicus MED
D. At the level of the 9th costal cartilage along the linea BOARDS;
semilunaris TOPNOTCH
E. None of the above MD)

283 An individual with tertiary syphilis would have damage Individuals with tertiary syphilis can present with ERIC ROYD FINAL EXAM
involving what structure in the spinal cord? tabes dorsalis which involves damage to the dorsal TALAVERA, - FEB 2015
A. Lateral Corticospinal Tract columns. Indiviuals present with loss of vibration, MD (TOP 1 -
B. Dorsal Column position sense and tactile discrimination. AUG 2014
C. Ventral Spinocerebellar Tract MED
D. Anterior Spinothalamic Tract BOARDS;
E. Lateral Spiothalamic Tract TOPNOTCH
MD)

284 Which of the following structures does not communicate The posterior ethmoidal sinus drain into the ERIC ROYD FINAL EXAM
with the middle meatus? superior meatus. The rest of the choices drain into TALAVERA, - FEB 2015
A. Maxillary Sinus the middle meatus. The nasolacrimal duct drains MD (TOP 1 -
B. Posterior Ethmoidal Sinus into the inferior meatus AUG 2014
C. Frontal Sinus MED
D. Anterior Ethmoidal Sinus BOARDS;
E. None of the above TOPNOTCH
MD)

285 Umbrella or Dome cells are part of the lining epithelium of Umbrella cells are multinucleated superficial cell ERIC ROYD FINAL EXAM
which organ ? of the bladder’s transitional epithelium, which has TALAVERA, - FEB 2015
A. Stomach vacuolated cytoplasm. MD (TOP 1 -
B. Small intestines AUG 2014
C. Uterus MED
D. Liver BOARDS;
E. Bladder TOPNOTCH
MD)

286 Which of the following statements is true regarding the the abdominal aorta enters the abdominal cavity ERIC ROYD FINAL EXAM
abdominal aorta? by piercing the aortic hiatus at the level of TV 12. TALAVERA, - FEB 2015
A. Enters the abdominal cavity at the level of TV 10 It is located to the left of the IVC. It has three MD (TOP 1 -
B. It is located to the right of the inferior vena cava unpaired visceral branches (celiac, superior AUG 2014
C. It terminates at the level of LV 4 mesenteric, inferior mesenteric). It terminates at MED
D. Has four (4) unparied visceral branches the level of LV4 into the common iliac and median BOARDS;
E. Terminal branches are the common iliac and inferior sacral artery TOPNOTCH
phrenic artery MD)

287 A 78 year old male presents with hematuria, urgency, The middle lobe is commonly involved in BPH ERIC ROYD FINAL EXAM
frequency and low back pain. Thorough evaulation resulting in obstruction of the prostatic urethra. TALAVERA, - FEB 2015
eventually lead to a diagnosis of prostate cancer. Which of While the posterior lobe (part of the peripheral MD (TOP 1 -
the following lobes of the prostate is most commonly zone) is commonly involved in carcinomatous AUG 2014
involved? transformation MED
A. Anterior BOARDS;
B. Middle TOPNOTCH
C. Posterior MD)
D. Right Lateral
E. Left Lateral

288 What muscle serves as guide in dividing the axillary artery The tendon of the pectoralis minor divides the ERIC ROYD FINAL EXAM
into three different parts? axillary artery into 3 major parts TALAVERA, - FEB 2015
A. Pectoralis Major MD (TOP 1 -
B. Pectoralis Minor AUG 2014
C. Serratus Anterior MED
D. Teres Major BOARDS;
E. Trapezius TOPNOTCH
MD)

289 Which of the following statements regarding the pancreas is The pancreas is a retroperitoneal organ. Blood ERIC ROYD FINAL EXAM
correct? supply is derived from the celiax and superior TALAVERA, - FEB 2015
A. It is an intraperitoneal organ mesenteric artery. It is located primarily in the MD (TOP 1 -
B. It derives its blood supply from the inferior mesenteric epigastric region. The portal vein is formed behind AUG 2014
artery the neck of the pancreas MED
C. It primarily located in the hypogastic and left BOARDS;
hypochondriac region TOPNOTCH
D. The portal vein is formed behind the tail of the pancreas MD)
E. The major pancreatic duct joins the common bile duct
which drains into the second part of the duodenum

TOPNOTCH MEDICAL BOARD PREP ANATOMY SUPEREXAM Page 37 of 94


For inquiries visit www.topnotchboardprep.com.ph or email us at topnotchmedicalboardprep@gmail.com
TOPNOTCH MEDICAL BOARD PREP ANATOMY SUPEREXAM
For inquiries visit www.topnotchboardprep.com.ph or email us at topnotchmedicalboardprep@gmail.com
Item QUESTION EXPLANATION AUTHOR TOPNOTCH
# EXAM
290 Which of the following structures is most likely to be The esophagus is related posterior to the left ERIC ROYD FINAL EXAM
compressed by an enlargement of the left atrium? atrium, enlargement of the left atrium can cause TALAVERA, - FEB 2015
A. Esophagus compression of the esophagus producing MD (TOP 1 -
B. Thymus symptoms such as dysphagia AUG 2014
C. Thyroid gland MED
D. Larynx BOARDS;
E. Superior vena cava TOPNOTCH
MD)

291 What is the only cranial nerve that exits from the dorsal The trochlear nerve is the only cranial nerve of the ERIC ROYD FINAL EXAM
surface of the brainstem? brainstem that exits from the dorsal surface TALAVERA, - FEB 2015
A. Abducens (CN VI) (particularly from the dorsal surface of the MD (TOP 1 -
B. Trigeminal (CN V) midbrain) AUG 2014
C. Hypoglossal (CN XII) MED
D. Facial (CN VII) BOARDS;
E. Trochlear (CN IV) TOPNOTCH
MD)

292 Brunner's glands are located in the submucosa of what Brunner's glands (or duodenal glands) are ERIC ROYD FINAL EXAM
segment of the gastrointestinal tract? compound tubular submucosal glands found in TALAVERA, - FEB 2015
A. Appendix that portion of the duodenum which is above the MD (TOP 1 -
B. Ileum hepatopancreatic sphincter (Sphincter of Oddi). AUG 2014
C. Duodenum The main function of these glands is to produce a MED
D. Stomach mucus-rich alkaline secretion (containing BOARDS;
E. Jejunum bicarbonate) TOPNOTCH
MD)

293 The fibers of the corticospinal tract passes through what Path of the corticopsinal tract: Motor Cortex ERIC ROYD FINAL EXAM
structure at the level of the midbrain? (Cerebrum) --> corona radiata --> posterior limb TALAVERA, - FEB 2015
A. Cerebral aqueduct of internal capsule --> cerebral peduncle MD (TOP 1 -
B. Superior Colliculus (midbrain)--> basis pontis (pons) --> pyramids AUG 2014
C. Red Nucleus (medulla) --> Decussate at level of lower medulla - MED
D. Medial Longitudinal Fasciculus -> lateral corticopinal tract (spinal cord) BOARDS;
E. Cerebral peduncles TOPNOTCH
MD)

294 A 65 year old chronic smoker presented with right arm pain This is a case of a Pancoast tumor. The said ERIC ROYD FINAL EXAM
and drooping of the right upper eyelid. Diagnostic work up symptoms arise from compression of the adjacent TALAVERA, - FEB 2015
revealed a mass over the apex of the right lung. Which of the structure, particularly the sympathetic chain MD (TOP 1 -
following muscles is affected to account for the finding of ganglia (Horner's Syndrome). The superior tarsal AUG 2014
ptosis over the right eye? muscle or the MULLER's muscle is innervated by MED
A. Lateral rectus sympathetic nerves. Compression of the said BOARDS;
B. Superior Oblique structures by the mass would produce weakness TOPNOTCH
C. Levator palpebrae superioris of the said muscle thus accounting for the ptosis. MD)
D. Superior tarsal LPS is innervated by CN III and is not involved in
E. Medial rectus Horner's syndrome

295 Which of the following structures is not located in the The heart and the pericardium are located in the ERIC ROYD FINAL EXAM
posterior mediastinum? middle mediastinum TALAVERA, - FEB 2015
A. Vagus nerve MD (TOP 1 -
B. Esophagus AUG 2014
C. Pericardium MED
D. Thoracic duct BOARDS;
E. Thoracic aorta TOPNOTCH
MD)

296 A 76 year old male was brought in the ER due to chest pain. A SIMILAR TO PREVIOUS BOARD EXAM ERIC ROYD FINAL EXAM
12L ECG was done which showed ST-segment elevation CONCEPT/PRINCIPLE: V1, V2: septal wall, I, avL: TALAVERA, - FEB 2015
involving leads II, III and avF. Involvement of the said leads high lateral wall, V3-V4: anterior wall, V5-V6: MD (TOP 1 -
signify infarction of which area of the heart? lateral wall, V3r-V4r: RV wall AUG 2014
A. Inferior wall MED
B. Septal wall BOARDS;
C. RV wall TOPNOTCH
D. Lateral Wall MD)
E. Anterior Wall

297 Which of the following statements concerning the larynx is All intrinsic muscles of the larynx are innercated ERIC ROYD FINAL EXAM
INCORRECT? by the recurrent laryngeal nerve except for the TALAVERA, - FEB 2015
A. Sensory innervation is provided by the recurrent cricothyroid which is innervated by the external MD (TOP 1 -
laryngeal and internal laryngeal nerve branch of the superior laryngeal nerve AUG 2014
B. Extends from the tip of the epiglottis to the lower border MED
of the cricoid cartilage BOARDS;
C. The cricothyroid muscle is innervated by the recurrent TOPNOTCH
laryngeal nerve MD)
D. The posterior cricoarytenoid muscle serves to abduct the
vocal folds
E. It can be divided into supraglottic, glottic and infraglottic
areas

TOPNOTCH MEDICAL BOARD PREP ANATOMY SUPEREXAM Page 38 of 94


For inquiries visit www.topnotchboardprep.com.ph or email us at topnotchmedicalboardprep@gmail.com
TOPNOTCH MEDICAL BOARD PREP ANATOMY SUPEREXAM
For inquiries visit www.topnotchboardprep.com.ph or email us at topnotchmedicalboardprep@gmail.com
Item QUESTION EXPLANATION AUTHOR TOPNOTCH
# EXAM
298 Lead V1 is placed in what location/position in the chest? SIMILAR TO PREVIOUS BOARD EXAM ERIC ROYD FINAL EXAM
A. 5th ICS at the left midclavicular line CONCEPT/PRINCIPLE: V1 4th ICS right sternal TALAVERA, - FEB 2015
B. 5th ICS at the left anterior axillary line border, V2 4th ICS left sternal border, V3 halfway MD (TOP 1 -
C. 4th ICS at the left sternal border between V2 and V4, V4 5th ICS left MCL, V5 5th AUG 2014
D. 4th ICS at the right sternal border ICS at left AAL, V6 5th ICS at left MAL, V3R halfway MED
E. 5th ICS at the right midclavicular line between V1 and V4r, V4r 5th ICS at right MCL BOARDS;
TOPNOTCH
MD)
299 Which of the following structures pass anterior to the medial Structures passing anterior to the medial ERIC ROYD FINAL EXAM
malleolus? malleolus: 1. Saphenous nerve 2. Great saphenous TALAVERA, - FEB 2015
A. Tibial nerve vein; Structures passing posterior to the medial MD (TOP 1 -
B. Saphenous nerve malleolus: 1. Tendon of the FDL, 2. Tendon of the AUG 2014
C. Tendon of the flexor digitorum longus FHL, 3. Tibialis posterior, 4. Tibial nerve, 5. MED
D. Tendon of the peroneus longus Posterior tibial artery BOARDS;
E. All of the above TOPNOTCH
MD)

300 Hassall's corpuscles are histologic features of which of the Hassall's corpuscles (or thymic corpuscles ERIC ROYD FINAL EXAM
following lymphoid organs? (bodies)) are structures found in the medulla of TALAVERA, - FEB 2015
A. Spleen the human thymus, formed from eosinophilic type MD (TOP 1 -
B. Lymph Nodes VI epithelial reticular cells arranged concentrically AUG 2014
C. Tonsils MED
D. Thymus BOARDS;
E. All of the above TOPNOTCH
MD)

301 A 57 y/o hypertensive, diabetic male was rushed to ER ipsilateral paralysis of the oculomotor nerve LEAN BACK-UP
secondary to deterioriation of consciousness. Pt was accompanied by contralateral ataxia and ANGELO MIDTERM
successfully stabilized at the ER. Upon secondary PE, Pt is intentional tremors is due to a vascular lesion at SILVERIO, EXAM - FEB
confused and oriented to place and person. his BP the central midbrain area known as Claude MD (TOP 4 - 2015
170/100mmHg, PR 76 bpm, RR 14 cpm. You noted that his syndrome. It affects the oculomotor nucleus, red AUG 2014
(L) eye is fixated in an inferolateral direction with dilated nucleus and the superior cerebellothalamic fibers. MED
nonresponsive pupils. while his (R) arm showed tremors and Weber syndrome is characterized by ipsilateral BOARDS;
incoordination when asked to reach for an object. no oculomotor paralysis and contralateral TOPNOTCH
paralysis was noted on both UE/LE. CT scan was requested hemiplegia. Combination of Weber and Claude MD), MD
however results were pending. what is the most probable syndrome constitute the Benedikt syndrome.
diagnosis? Foville syndrome is the vascular paramedian pons
A. Weber syndrome lesion represented by ipsilateral abducent nerve
B. Benedikt syndrome palsy and contralateral hemiparesis. Wallenberg
C. Claude syndrome syndrome is a lateral medullary syndrome
D. Foville syndrome secondary to occlusion of posterior inferior
E. Wallenberg syndrome cerebellar artery. it manifest with contralateral
hemianesthesia, ipsilateral loss of pain and
thermal sensation in the face, nystagmus, vertigo,
loss of taste from the ipsilateral side of th tonque
and hoarseness with dysphagia. Haines 3rd ed pp
175-190
302 Which of the following neck muscles is not supplied by the cervical plexus provides motor innervation to LEAN BACK-UP
ansa cervicalis ? most of the neck muscles. Ansa cervicalis is the ANGELO MIDTERM
A. geniohyoid union of C1-C3 fibers give innervation to SILVERIO, EXAM - FEB
B. Omohyoid sternohyoid, sternothyroid, and omohyoid. C1 MD (TOP 4 - 2015
C. sternohyoid fibers via hypoglossal nerve supplies the AUG 2014
D. Sternothyroid thyrohyoid and geniohyoid. other cervical plexus MED
E. None of the above branches include the phrenic nerve (C3-C5) BOARDS;
innervating the diaphragm and segmental nerves TOPNOTCH
supplying the prevertebral muscles including the MD), MD
levator scapulae. Snell 9th ed p 619.
303 A 76 y/o male went to your clinic secondary to productive this patient diagnosed with lung cancer on the left LEAN BACK-UP
cough, weight loss and voice changes. He was smoker with a upper lobe accompanied by mediastinal lymph ANGELO MIDTERM
50 pack years and an alcoholic beverage drinker. Xray was nodes can cause compression of the left recurrent SILVERIO, EXAM - FEB
done showing a suspicious mass at the left upper lobe of the laryngeal nerve as it hoops around the aortic arch MD (TOP 4 - 2015
lung. a follow up CT scan showed a probable lung carcinoma and left bronchial area. this will result to paralysis AUG 2014
with mediastinal Lymphadenopathies compressing the left of all the intrinsic laryngeal muscles except for MED
bronchial airway. which of the following laryngeal muscles cricothyroid. the latter is supplied by external BOARDS;
will be spared by this condition? laryngeal nerve. snell 9th ed p 648 TOPNOTCH
A. thyroarytenoid MD), MD
B. cricothyroid
C. posterior cricoarytenoid
D. oblique arytenoid
E. none of the above

TOPNOTCH MEDICAL BOARD PREP ANATOMY SUPEREXAM Page 39 of 94


For inquiries visit www.topnotchboardprep.com.ph or email us at topnotchmedicalboardprep@gmail.com
TOPNOTCH MEDICAL BOARD PREP ANATOMY SUPEREXAM
For inquiries visit www.topnotchboardprep.com.ph or email us at topnotchmedicalboardprep@gmail.com
Item QUESTION EXPLANATION AUTHOR TOPNOTCH
# EXAM
304 A 28 y/o male with a history of poorly controlled asthma and middle meatus is located between inferior and LEAN BACK-UP
Allergic Rhinitis presented to your clinic with a complain of middle nasal concha. It drains the frontal, anterior ANGELO MIDTERM
decrease sense of smell and taste. Upon doing ENT and middle ethmoid, maxillary sinuses. While SILVERIO, EXAM - FEB
examination, you noted a pale grayish polypoid mass located sphenoid sinus drains into the sphenoethmoidal MD (TOP 4 - 2015
between inferior and middle nasal concha. which of the recess superior to superior nasal concha. Posterior AUG 2014
following set of sinuses will most likely obstructed? ethmoidal sinus drains to the superior meatus MED
A. frontal, posterior ethmoid, maxillary sinuses above the middle concha. nasolacrimal duct drains BOARDS;
B. frontal, middle ethmoid, sphenoid, maxillary sinuses to the inferior meatus. Snell 9th ed pp 643-644 TOPNOTCH
C. frontal anterior, posterior and middle ethmoid, maxillary MD), MD
sinuses
D. frontal, anterior ethmoid, maxillary sinuses
E. frontal, ant and middle ethmoid, sphenoid, maxillary
sinuses

305 A 76 y/o female complaining of chronic vague abdominal aortic aneurysm with such length will most likely LEAN BACK-UP
pain was found to have aortic aneurysm extending from T11 affect structures that passes the aortic opening of ANGELO MIDTERM
-to L3 vertebra. What are the other structures will most the diaphragm ( T12 vertebrae). This include SILVERIO, EXAM - FEB
likely be affected by this condition? thoracic duct and azygous veins. Esophageal MD (TOP 4 - 2015
A. thoracic duct opening lies more anterior and it is at the level of AUG 2014
B. vagus nerve T10. it transmits vagus nerve, left gastric vessels MED
C. inferior vena cava and lymphatics. Caval opening lies at the 8th BOARDS;
D. left gastric artery thoracic vertebrae transmits IVC and right phrenic TOPNOTCH
E. righr phrenic nerve nerve. Snell 9th ed p 46. MD), MD

306 A 27 y/o male was sent to ER following an MVA. Pt was the patient is suffering from left tension LEAN BACK-UP
hypotensive, tachycardic and tachypneic along with neck pneumothorax. A needle thoracostomy is needed ANGELO MIDTERM
veins distention and (L) tracheal deviation. There were no inorder to alleviate the pressure in the left pleural SILVERIO, EXAM - FEB
breath sounds noted on the left. What landmark will you use area. by convention, it is best to insert a needle at MD (TOP 4 - 2015
inorder to perform needle thoracostomy? the level of the 2nd ICS either at midclavicular or AUG 2014
A. (L) 2nd ICS 0.5 cm from sternal edge anterior axillary line. Snell 9th ed p 45. MED
B. (L) 2nd ICS midclavicular line BOARDS;
C. (R) 3rd ICS anterior axillary line TOPNOTCH
D. (L) 5th ICS midaxillary line MD), MD
E. none of the above

307 the ductus arteriosus is a derivative of what aortic arch? Aortic arches: 1st - maxillary artery, 2nd- hyoid LEAN BACK-UP
A. (L) 3rd aortic arch and stapedial arteries, 3rd - common carotid ANGELO MIDTERM
B. ® 4th aortic arch artery and the 1st part of internal carotid arteries, SILVERIO, EXAM - FEB
C. (L) 5th aortic arch 4th - (L) arch of aorta between LCCA and LScA, ® - MD (TOP 4 - 2015
D. (L) 6th aortic arch proximal segment of subclavian artery. 5th- AUG 2014
E. ® 2nd aortic arch obliterated, no derivatives. 6th- (L) ductus MED
arteriosus, (R) proximal segment of (R) pulmonary BOARDS;
artery. Langmans embryology 9th ed 255-258. TOPNOTCH
MD), MD
308 Which of the following is not a tributary of the coronary most blood from the heart wall drains into the LEAN BACK-UP
sinus ? right atrium through the coronary sinus, its ANGELO MIDTERM
A. Great cardiac vein tributaries are the great, small, and middle cardiac SILVERIO, EXAM - FEB
B. Middle cardiac vein veins. The anterior cardiac vein drains blood MD (TOP 4 - 2015
C. Anterior cardiac vein directly to the right atrium bypassing the coronary AUG 2014
D. Small cardiac vein sinus. snell 9th ed p 89 MED
E. Both B and C. BOARDS;
TOPNOTCH
MD), MD
309 Which of the following is not a true statement regarding the the deep inguinal ring, a defect in the fascia LEAN BACK-UP
inguinal canal? transversalis, is located midway between ASIS and ANGELO MIDTERM
A. The superior wall is made up by the arching fibers of the pubic tubercle. Snell 9th ed p 127. SILVERIO, EXAM - FEB
Internal oblique and transversus abdominis MD (TOP 4 - 2015
B. The medial side of the posterior wall is reinforced by the AUG 2014
conjoint tendon MED
C. the deep inguinal ring is located at 1.3 cm medial to the BOARDS;
ASIS TOPNOTCH
D. the superficial inguinal ring gives attachment ot the MD), MD
external spermatic fascia.
E. none of the above

310 A 65 y/o male presenting weightloss, anemia and chronic the approximate length of each structures are as LEAN BACK-UP
diarrhea underwent a colonoscopy, a polypoid mass was follows. Anal canal- 1.5 in, rectum -5 in, sigmoid, ANGELO MIDTERM
located at the anterior wall of the colon 40 inches from the 10-15 inch, transverse 15 inch descending colon SILVERIO, EXAM - FEB
anal verge. What part of the colon where the mass is located? 10 in. therefore at 40 inches from the anal verge, MD (TOP 4 - 2015
A. sigmoid the mass is most likely located at the transverse AUG 2014
B. descending colon colon. MED
C. transverse colon BOARDS;
D. hepatic flexure TOPNOTCH
E. ascending colon MD), MD

TOPNOTCH MEDICAL BOARD PREP ANATOMY SUPEREXAM Page 40 of 94


For inquiries visit www.topnotchboardprep.com.ph or email us at topnotchmedicalboardprep@gmail.com
TOPNOTCH MEDICAL BOARD PREP ANATOMY SUPEREXAM
For inquiries visit www.topnotchboardprep.com.ph or email us at topnotchmedicalboardprep@gmail.com
Item QUESTION EXPLANATION AUTHOR TOPNOTCH
# EXAM
311 what is the most dilatable portion of the male urethra? the prostatic urethra is the widest and the most LEAN BACK-UP
A. Prostatic urethra dilatable portion of the entire male urethra. Snell ANGELO MIDTERM
B. Membranous urethra 9th ed p 278 SILVERIO, EXAM - FEB
C. Penile urethra MD (TOP 4 - 2015
D. Navicular fossa of the urethra AUG 2014
E. None of the above MED
BOARDS;
TOPNOTCH
MD), MD
312 the physiological umbilical herniation during embryonic ( SIMILAR TO PREVIOUS BOARD EXAM LEAN BACK-UP
development occur in what age of gestation? CONCEPT/PRINCIPLE). Development of the ANGELO MIDTERM
A. 5th week primary intestinal loop is characterized by rapid SILVERIO, EXAM - FEB
B. 6th week elongation particularly of the cephalic limb. As a MD (TOP 4 - 2015
C. 7th week result of the rapid growth and expansion of the AUG 2014
D. 8th week liver, the abdominal cavity temporarily becomes MED
E. 10th week too small to contain all the intestinal lipps and BOARDS;
they enter the extraembryonic cavity in the TOPNOTCH
umblical cord during 6th week of development. MD), MD
Langmans 9th ed p 307.
313 Which of the following is a branch of the anterior division of upon descent of the internal iliac artery at the LEAN BACK-UP
the internal iliac artery? level of the greater sciatic foramen, it divides into ANGELO MIDTERM
A. Lateral sacral artery anterior and posterior division. The branches of SILVERIO, EXAM - FEB
B. Superior gluteal artery the anterior division are the umbilical artery, MD (TOP 4 - 2015
C. Inferior gluteal artery obturator artery, inferior vesical artery, middle AUG 2014
D. Iliolumbar artery rectal arterym internal pudendal aftery, inferior MED
E. None of the above gluteal artery, uterine and vaginal arteries. while BOARDS;
the posterior division has only three branches: TOPNOTCH
iliolumbar, lateral sacral and superior gluteal MD), MD
artery. Snell 9th ed Regions p 256-257
314 A 27 y/o female was brought to ER secondary to saddle type the contents of the superficial perineal pouch are LEAN BACK-UP
injury due to MVA, pelvic CT scan was requested showing the following: root of the clitoris, bulbospongiosus, ANGELO MIDTERM
hematoma on the superficial perineal pouch. Which of the ischiocavernosus, superficial transverse perineal SILVERIO, EXAM - FEB
following structures will most likely be spared in this muscle, perioneal body and the perineal branch of MD (TOP 4 - 2015
condition? the pudendal nerve. The contents of the deep AUG 2014
A. sphincter urethrae perineal pouch includes the followin: sphincter MED
B. ischiocavernosus muscle urethrae, deep transverse perineal muscle, BOARDS;
C. bulbospongiosus muscle internal pudendal vessels, and dorsal nerve of the TOPNOTCH
D. perineal branch of the pudendal nerve clitoris. Snell 9th ed regions pp 322- 323 MD), MD
E. perineal body

315 A 34 y/o female was brought to ER secondary to a diffuse based on the history, the patient is most likely to LEAN BACK-UP
abdominal pain. she was hypotensive, tachycardic at have a perforated duodenal ulcer. The most ANGELO MIDTERM
presentation. PE showed diffuse rigidity of the abdomen with common site for duodenal ulcer is its 1st part SILVERIO, EXAM - FEB
both rebound and direct tenderness prominent on the which is intraperitoneal structure. CT scan finding MD (TOP 4 - 2015
epigastric area. CT scan was done showing blood collection of blood in the lesser sac indicates that the AUG 2014
in the lesser sac. she was a smoker and alcoholic beverage perforation is most likely at the posterior wall. MED
drinker. she always skip her meals because she want to loose gastroduodenal artery is immediately behind the BOARDS;
weight and that she always take mefenamic acid almost 1st part of the duodenum. affectation of this TOPNOTCH
everyday for her migraine.prior to onset, she claims to have structure can lead to bleeding in the lesser sac, MD), MD
recurrent history of vague epigastric pain whenever she skip and sometimes pancreatitis. Snell 9th ed p 176
meal and that there were occasions of nocturnal awakening
secondary to epigastric pain. what specific vascular structure
will most likely be affected in this setting?
A. superior pancreaticoduodenal artery
B. portal vein
C. gastroduodenal artery
D. (L) gastroepiploic artery
E. all of the above

316 which of the following is true about the schneiderian the schneiderian membrane refers to the LEAN BACK-UP
membrane of the nasal cavity? respiratory portion of the nasal cavity. It is ANGELO MIDTERM
A. Abundant venous plexus in the tunica propria characterized by pseudostratified columnar SILVERIO, EXAM - FEB
B. Tall pseudostratified columnar epithelium without epithelium with goblet cells lining the mucous MD (TOP 4 - 2015
goblet cells membrane. It has a relatively thick basal lamina; AUG 2014
C. No distinct basal lamina contains abundant venous plexuses, and serous/ MED
D. contains bundles of olfactor y nerves in the lamina mucous tubuloacinar glands in the tunica propria. BOARDS;
propria it also has characteristic lymphocytic infiltration in TOPNOTCH
E. all of the above the epithelial layer. other choices refers to the MD), MD
olfactory neuroepithelium. cabral histology p 178

TOPNOTCH MEDICAL BOARD PREP ANATOMY SUPEREXAM Page 41 of 94


For inquiries visit www.topnotchboardprep.com.ph or email us at topnotchmedicalboardprep@gmail.com
TOPNOTCH MEDICAL BOARD PREP ANATOMY SUPEREXAM
For inquiries visit www.topnotchboardprep.com.ph or email us at topnotchmedicalboardprep@gmail.com
Item QUESTION EXPLANATION AUTHOR TOPNOTCH
# EXAM
317 complete transection of the lateral cord of brachial plexus the lateral cord give rise to lateral pectoral nerve ( LEAN BACK-UP
will result to which of the following symptoms? pectoralis major), musculocutaneous nerve ( ANGELO MIDTERM
A. Decrease sensation in the lateral dorsal side of the the coracobrachialis, biceps brachii, brachialis, SILVERIO, EXAM - FEB
hand. sensory to the lateral border of the foreeam) and MD (TOP 4 - 2015
B. Decrease sensation in the lateral side of the forearm lateral root of the median nerve. A- is supplied by AUG 2014
C. inability to supinate the forearm the radial nerve which is a branch of the posterior MED
D. inability to flex the forearm cord. C and D- pt will only have weakness but not BOARDS;
E. all of the above. inability since the supinator muscle is supplied by TOPNOTCH
the radial nerve eventhough the biceps brachii is MD), MD
denervated. on the otherhand, there is still some
flexion of the forearm because of the reverse
action of other forearm muscle as well as the
partial innervation of brachialis by the radial
nerve. ( Snell 9th ed Regions p 354)
318 A 72 y/o male was brought to ER secondary to FOOSH injury the deep branch of radial nerve is a pure motor LEAN BACK-UP
( fall on a outstretched hand). X ray was done showing nerve supplying all the muscles of the posterior ANGELO MIDTERM
displaced compression fracture of the radial head. Physical compartment of the forearm except for the ECRL SILVERIO, EXAM - FEB
examination of the arm reveals findings that are consistent and supinator muscle. There would be no wrist MD (TOP 4 - 2015
with an injury to the deep branch of radial nerve. which of drop since the ECRL is powerful enough to AUG 2014
the following statement is correct. ? promote wrist extension. Snell 9th ed p 431. MED
A. MMT 3/5 in forearm supination BOARDS;
B. decrease sensation on the dorsal lateral side of the hand TOPNOTCH
C. presence of wristdrop MD), MD
D. all of the above
E. none of the above

319 Which of the following structures is not supplied by the the lateral plantar nerve supplies all of the LEAN BACK-UP
lateral plantar nerve? intrinsic muscles of the foot except for abductor ANGELO MIDTERM
A. Adductor hallucis hallucis, flexor digitorum brevis, flexor hallucis SILVERIO, EXAM - FEB
B. Flexor digitorum brevis brevis, and the 1st lumbrical muscle ( all of which MD (TOP 4 - 2015
C. Quadratus plantae are supplied by the medial plantar nerve) Snell p AUG 2014
D. Flexor digiti minimi 676. MED
E. None of the above BOARDS;
TOPNOTCH
MD), MD
320 Retrolisthesis of the 8th thoracic vertebra will affect what relationship of spinal cord segements to vertebral LEAN BACK-UP
corresponding spinal segment? numbers: cervical vertebrae: add 1, upper thoracic ANGELO MIDTERM
A. T8 vertebra (T1-T6): add 2, lower thoracic (T7-T9): SILVERIO, EXAM - FEB
B. T9 add 3, tenth thoracic: L1 and L2 cord segment, MD (TOP 4 - 2015
C. T10 11th thoracic: L3-L4 cord segment, 12 th thoracic: AUG 2014
D. T11 L5 cord segment; 1st lumbar: sacral and coccygeal MED
E. T12 segments. Snell 9th ed p 704 BOARDS;
TOPNOTCH
MD), MD
321 Epithelial lining of the ovaries: SIMILAR TO PREVIOUS BOARD EXAM DEBBIE BACK-UP
A. simple squamous CONCEPT/PRINCIPLE. ROSE MIDTERM
B. stratified squamous TANENGSY, EXAM - FEB
C. simple cuboidal MD (TOP 5 - 2015
D. simple columnar AUG 2014
E. pseudostratified columnar MED
BOARDS;
TOPNOTCH
MD)
322 Tongue papillae with no taste buds and thus serve a Fungiform - mushroom shaped; circumvallate - DEBBIE BACK-UP
mechanical function only: largest, located near the sulcus terminalis; foliate - ROSE MIDTERM
A. fungiform rudimentary in humans; filliform - most abundant TANENGSY, EXAM - FEB
B. circumvallate papillae MD (TOP 5 - 2015
C. foliate AUG 2014
D. filliform MED
BOARDS;
TOPNOTCH
MD)
323 Epithelial lining of lymphangiomas: SIMILAR TO PREVIOUS BOARD EXAM DEBBIE BACK-UP
A. simple squamous CONCEPT/PRINCIPLE. ROSE MIDTERM
B. stratified squamous TANENGSY, EXAM - FEB
C. simple cuboidal MD (TOP 5 - 2015
D. simple columnar AUG 2014
E. psuedostratified columnar MED
BOARDS;
TOPNOTCH
MD)
324 True of the jejunum but not of the ileum, except: DEBBIE BACK-UP
A. makes up the initial 3/5 of the small intestine ROSE MIDTERM
B. has longer vasa recta TANENGSY, EXAM - FEB
C. has 1-2 arterial arcades MD (TOP 5 - 2015
D. has a thicker wall AUG 2014
E. with prominent Peyer's patches MED
BOARDS;
TOPNOTCH
MD)

TOPNOTCH MEDICAL BOARD PREP ANATOMY SUPEREXAM Page 42 of 94


For inquiries visit www.topnotchboardprep.com.ph or email us at topnotchmedicalboardprep@gmail.com
TOPNOTCH MEDICAL BOARD PREP ANATOMY SUPEREXAM
For inquiries visit www.topnotchboardprep.com.ph or email us at topnotchmedicalboardprep@gmail.com
Item QUESTION EXPLANATION AUTHOR TOPNOTCH
# EXAM
325 The lower anal canal, as opposed to the upper anal canal: DEBBIE BACK-UP
A. is supplied by a branch of the inferior mesenteric artery ROSE MIDTERM
B. is drained by the deep inguinal nodes TANENGSY, EXAM - FEB
C. is derived from the ectoderm MD (TOP 5 - 2015
D. is lined by simple columnar epithelium AUG 2014
MED
BOARDS;
TOPNOTCH
MD)
326 The moderator band is located in which chamber of the The moderator band is located in the right DEBBIE BACK-UP
heart? ventricle. ROSE MIDTERM
A. right atrium TANENGSY, EXAM - FEB
B. left atrium MD (TOP 5 - 2015
C. right ventricle AUG 2014
D. left ventricle MED
BOARDS;
TOPNOTCH
MD)
327 True of the right atrium: The right atrium is larger than the left, has thicker DEBBIE BACK-UP
A. smaller than the left atrium walls, and has a sinus venarum, which is a smooth ROSE MIDTERM
B. has thicker walls portion developed from the embryonic sinus TANENGSY, EXAM - FEB
C. has a moderator band venosus. The moderator band is located in the MD (TOP 5 - 2015
D. with a sinus venarum, a coarse muscular portion of the right ventricle. AUG 2014
chamber MED
BOARDS;
TOPNOTCH
MD)
328 GIST is located in what layer of the gastrointestinal tract? DEBBIE BACK-UP
A. mucosa ROSE MIDTERM
B. submucosa TANENGSY, EXAM - FEB
C. muscularis MD (TOP 5 - 2015
D. adventitia AUG 2014
E. serosa MED
BOARDS;
TOPNOTCH
MD)
329 The accessory nerve crosses this triangle of the neck: DEBBIE BACK-UP
A. submental ROSE MIDTERM
B. submandibular TANENGSY, EXAM - FEB
C. carotid MD (TOP 5 - 2015
D. muscular AUG 2014
E. occipital MED
BOARDS;
TOPNOTCH
MD)
330 the lower ureter is supplied by the: Upper ureter - renal artery; middle ureter - DEBBIE BACK-UP
A. renal artery gonadal artery; lower ureter - superior vesical ROSE MIDTERM
B. superior vesical artery artery TANENGSY, EXAM - FEB
C. gonadal artery MD (TOP 5 - 2015
D. middle rectal AUG 2014
E. inferior vesical MED
BOARDS;
TOPNOTCH
MD)
331 Found in the superficial perineal space, except: The membranous urethra is found within the deep DEBBIE BACK-UP
A. bulbospongiosus perineal space. ROSE MIDTERM
B. ischiocavernosus TANENGSY, EXAM - FEB
C. spongy urethra MD (TOP 5 - 2015
D. perineal body AUG 2014
E. membranous urethra MED
BOARDS;
TOPNOTCH
MD)
332 A patient with an upper motor neuron lesion is expected to A person with an upper motor neuron lesion DEBBIE BACK-UP
exhibit: presents with spastic paralysis, increased muscle ROSE MIDTERM
A. flaccid paralysis tone, clonus, (+) Babinski. There is no atrophy or TANENGSY, EXAM - FEB
B. atrophy fasciculation in UMNL. MD (TOP 5 - 2015
C. clonus AUG 2014
D. fasciculations MED
E. reduced muscle tone BOARDS;
TOPNOTCH
MD)
333 Leaf shaped laryngeal cartilage: thyroid - largest laryneal cartilage; cricoid - signet DEBBIE BACK-UP
A. thyroid ring shaped; arytenoid - pyramidal; corniculate - ROSE MIDTERM
B. cricoid conical; cuneiform - club shaped TANENGSY, EXAM - FEB
C. epiglottis MD (TOP 5 - 2015
D. arytenoid AUG 2014
E. corniculate MED
BOARDS;
TOPNOTCH
MD)

TOPNOTCH MEDICAL BOARD PREP ANATOMY SUPEREXAM Page 43 of 94


For inquiries visit www.topnotchboardprep.com.ph or email us at topnotchmedicalboardprep@gmail.com
TOPNOTCH MEDICAL BOARD PREP ANATOMY SUPEREXAM
For inquiries visit www.topnotchboardprep.com.ph or email us at topnotchmedicalboardprep@gmail.com
Item QUESTION EXPLANATION AUTHOR TOPNOTCH
# EXAM
334 Example of a typical rib: Ribs 12 and 11 have a single facet of the heads, are DEBBIE BACK-UP
A. rib 12 short, with no neck or tubercle. Rib 10 articulates ROSE MIDTERM
B. rib 11 with the T10 vertebra only. Other atypical ribs: rib TANENGSY, EXAM - FEB
C. rib 10 1 - broadest, most curved, prominent scalene MD (TOP 5 - 2015
D. rib 9 tubercle; rib 2 - thinner, less curved, has AUG 2014
tuberosity for serratus anterior. MED
BOARDS;
TOPNOTCH
MD)
335 The triangle of Calot: The triangle of Calot contains the cystic artery, is DEBBIE BACK-UP
A. contains the cystic duct bounded by the following: superiorly by the liver, ROSE MIDTERM
B. is bounded by the liver laterally inferiorly by the cystic duct, and medially by the TANENGSY, EXAM - FEB
C. is bounded by the cystic artery inferiorly common hepatic duct. MD (TOP 5 - 2015
D. is bounded by the common hepatic duct medially AUG 2014
MED
BOARDS;
TOPNOTCH
MD)
336 The female pelvis, compared to the male pelvis, has a/n: Female pelvis: borader sacrum, shorter symphysis DEBBIE BACK-UP
A. narrower sacrum pubis, everted ischial tuberosity, shallower iliac ROSE MIDTERM
B. longer symphysis pubis fossa. TANENGSY, EXAM - FEB
C. oval shaped superior aperture MD (TOP 5 - 2015
D. inverted ischial tuberosity AUG 2014
E. deeper iliac fossa MED
BOARDS;
TOPNOTCH
MD)
337 Which part of the adrenals contain pale staining cells in Zona glomerulosa - basophilic arranged in cords of DEBBIE BACK-UP
cords with foamy appearances due to lipid droplet columnar or pyramidal cells. Zona reticularis - ROSE MIDTERM
accumulations? eosinophilic cells with large nucleus arranged in TANENGSY, EXAM - FEB
A. zona glomerulosa cords smaller and less spongy than the cells in MD (TOP 5 - 2015
B. zona fasciculata zona fasciculata. AUG 2014
C. zona reticularis MED
D. adrenal medulla BOARDS;
TOPNOTCH
MD)
338 Fibrocartilage is found in: DEBBIE BACK-UP
A. larynx ROSE MIDTERM
B. epiglottis TANENGSY, EXAM - FEB
C. auditory tube MD (TOP 5 - 2015
D. costal cartilage AUG 2014
E. intervertebral discs MED
BOARDS;
TOPNOTCH
MD)
339 Layer absent in thin skin: DEBBIE BACK-UP
A. corneum ROSE MIDTERM
B. lucidum TANENGSY, EXAM - FEB
C. granulosum MD (TOP 5 - 2015
D. spinosum AUG 2014
E. basale MED
BOARDS;
TOPNOTCH
MD)
340 Thickest layer in veins: DEBBIE BACK-UP
A. tunia intima ROSE MIDTERM
B. tunica media TANENGSY, EXAM - FEB
C. tunica adventitia MD (TOP 5 - 2015
D. none of the above AUG 2014
MED
BOARDS;
TOPNOTCH
MD)
341 The carotid sinus is found in the: The carotid sinus is a localized dilation at the JESSICA MAE BACK-UP
A. Anterior surface of the bifurcation of the carotid artery beginning of the internal carotid artery. SANCHEZ, MIDTERM
B. Posterior surface of the bifurcation of the common carotid The carotid body is a small structure that lies MD (TOP 4 - EXAM - FEB
artery posterior to the bifurcation of the common carotid AUG 2014 2015
C. Beginning of the internal carotid artery artery. MED
D. Beginning of the external carotid artery BOARDS;
Reference: Snell. Clinical Anatomy by Regions, 8th TOPNOTCH
ed. p. 747 MD)
342 The muscles that form the superior boundary of the carotid The boundaries of the carotid triangle are as JESSICA MAE BACK-UP
triangle: follows: SANCHEZ, MIDTERM
A. Posterior belly of digastric -Superiorly by the posterior belly of digastric MD (TOP 4 - EXAM - FEB
B. Superior belly of omohyoid -Posteriorly by the SCM AUG 2014 2015
C. Sternocleidomastoid -Anteriorly by the superior belly of the omohyoid. MED
D. Trapezius BOARDS;
TOPNOTCH
MD)
343 This nerve accompanies the superior thyroid artery: The superior thyroid artery, a branch of the ECA, is JESSICA MAE BACK-UP
A. Superior laryngeal accompanied by the external laryngeal nerve SANCHEZ, MIDTERM
B. External laryngeal which supplies the cricothyroid muscle. MD (TOP 4 - EXAM - FEB
C. Internal laryngeal AUG 2014 2015
D. Recurrent laryngeal Reference: Snell. Clinical Anatomy by Regions, 8th MED
ed. P. 749 BOARDS;
TOPNOTCH
TOPNOTCH MEDICAL BOARD PREP ANATOMY SUPEREXAM Page 44 of 94
For inquiries visit www.topnotchboardprep.com.ph or email us at topnotchmedicalboardprep@gmail.com
TOPNOTCH MEDICAL BOARD PREP ANATOMY SUPEREXAM
For inquiries visit www.topnotchboardprep.com.ph or email us at topnotchmedicalboardprep@gmail.com
Item QUESTION EXPLANATION AUTHOR TOPNOTCH
# EXAM
MD)

344 True of meiosis, except: Meiosis reduces the chromosomes to the haploid JESSICA MAE BACK-UP
A. Creates new set of chromosomes by random distribution number. SANCHEZ, MIDTERM
to daughter cells MD (TOP 4 - EXAM - FEB
B. Bestow each germ cell with the diploid number of AUG 2014 2015
chromosomes MED
C. Provides gamete variabilities BOARDS;
D. Produce daughter cells genetically different from the TOPNOTCH
parent and from each other MD)
345 Which of the following cartilages is characterized by the Reference: Topnotch Histology Handouts JESSICA MAE BACK-UP
absence of a perichondrium? SANCHEZ, MIDTERM
A. Adult hyaline cartilage MD (TOP 4 - EXAM - FEB
B. Fetal hyaline cartilage AUG 2014 2015
C. Fibrocartilage MED
D. Articular cartilage BOARDS;
TOPNOTCH
MD)
346 This structure divides the subclavian artery into 3 parts: Reference: Snell. Clinical Anatomy by Regions, 8th JESSICA MAE BACK-UP
A. First rib ed. P.751 SANCHEZ, MIDTERM
B. Pectoralis minor MD (TOP 4 - EXAM - FEB
C. Scalene anterior AUG 2014 2015
D. Teres major MED
BOARDS;
TOPNOTCH
MD)
347 The deep cervical lymph nodes lie along the: The deep cervical nodes form a vertical chain JESSICA MAE BACK-UP
A. Sternocleidomastoid along the course of the internal jugular vein within SANCHEZ, MIDTERM
B. Common carotid artery the carotid sheath. MD (TOP 4 - EXAM - FEB
C. External jugular vein The superficial cervical nodes lie along the course AUG 2014 2015
D. Internal jugular vein of the external jugular vein on the side of the neck. MED
BOARDS;
Reference: Snell. Clinical Anatomy by Regions, 8th TOPNOTCH
ed. p. 755-756 MD)
348 Tracheostomy tube insertion is usually done at the level of The preferred site for tracheostomy is through the JESSICA MAE BACK-UP
the: second to third ring, with the thyroid isthmus SANCHEZ, MIDTERM
A. Cricothyroid membrane retracted inferiorly. MD (TOP 4 - EXAM - FEB
B. 1st-2nd tracheal rings AUG 2014 2015
C. 2nd-3rd tracheal rings Reference: Snell. Clinical Anatomy by Regions, 8th MED
D. 4th-5th tracheal rings ed. P. 814 BOARDS;
TOPNOTCH
MD)
349 After sustaining a fracture of the mid-humerus following a Reference: Topnotch Anatomy Handouts JESSICA MAE BACK-UP
fall, the patient was unable to extend the wrist. This is SANCHEZ, MIDTERM
consistent with injury to the: MD (TOP 4 - EXAM - FEB
A. Axillary nerve AUG 2014 2015
B. Median nerve MED
C. Radial nerve BOARDS;
D. Ulnar nerve TOPNOTCH
MD)
350 Which of the following is not a characteristic of epithelium in Epithelium may be derived from any of the 3 germ JESSICA MAE BACK-UP
general? layers. SANCHEZ, MIDTERM
A. Avascular MD (TOP 4 - EXAM - FEB
B. Derived from the ectoderm only AUG 2014 2015
C. Capable of metaplasia MED
D. Rests on a basal lamina BOARDS;
TOPNOTCH
MD)
351 What vein accompanies the anterior interventricular artery? Great cardiac vein - anterior interventricular JESSICA MAE BACK-UP
A. Anterior cardiac artery SANCHEZ, MIDTERM
B. Great cardiac Middle cardiac vein - posterior interventricular MD (TOP 4 - EXAM - FEB
C. Middle cardiac artery AUG 2014 2015
D. Small cardiac Small cardiac vein - marginal branch of the RCA MED
BOARDS;
TOPNOTCH
MD)
352 The 6th aortic arch will become the: Aortic arches: JESSICA MAE BACK-UP
A. Common carotid artery I - Maxillary artery SANCHEZ, MIDTERM
B. Maxillary artery II - Hyoid and stapedial arteries MD (TOP 4 - EXAM - FEB
C. Pulmonary artery III - Common carotid, first part of the ICA, ECA AUG 2014 2015
D. Right subclavian artery IV - Left: arch of the aorta; Right: proximal MED
segment of the right subclavian artery BOARDS;
V - regresses TOPNOTCH
VI - proximal part of the pulmonary artery, and the MD)
ductus arteriosus on the left

Reference: Langman's Medical Embryology, 9th
ed. p. 255-258

TOPNOTCH MEDICAL BOARD PREP ANATOMY SUPEREXAM Page 45 of 94


For inquiries visit www.topnotchboardprep.com.ph or email us at topnotchmedicalboardprep@gmail.com
TOPNOTCH MEDICAL BOARD PREP ANATOMY SUPEREXAM
For inquiries visit www.topnotchboardprep.com.ph or email us at topnotchmedicalboardprep@gmail.com
Item QUESTION EXPLANATION AUTHOR TOPNOTCH
# EXAM
353 Melena results from gastrointestinal bleeding found: JESSICA MAE BACK-UP
A. Proximal to the ligament of Treitz SANCHEZ, MIDTERM
B. Distal to the ligament of Treitz MD (TOP 4 - EXAM - FEB
C. At the stomach only AUG 2014 2015
D. At the esophagogastric area only MED
BOARDS;
TOPNOTCH
MD)
354 A 40 year old man sustained a gunshot wound that traversed Reference: Topnotch Anatomy Handouts JESSICA MAE BACK-UP
segments 1 and 4 of the liver. This means that the parts of SANCHEZ, MIDTERM
the liver affected were: MD (TOP 4 - EXAM - FEB
A. Quadrate lobe AUG 2014 2015
B. Caudate lobe MED
C. Both BOARDS;
D. Neither TOPNOTCH
MD)
355 The extent to which neoplastic cells resemble comparable JESSICA MAE BACK-UP
normal cells is referred to as: SANCHEZ, MIDTERM
A. Differentiation MD (TOP 4 - EXAM - FEB
B. Pleomorphism AUG 2014 2015
C. Anaplasia MED
D. Dysplasia BOARDS;
TOPNOTCH
MD)
356 A patient complains of a boil located on her labia majora. Reference: Topnotch Anatomy Handouts JESSICA MAE BACK-UP
Lymphatic spread of the infection would most likely enlarge SANCHEZ, MIDTERM
which nodes? MD (TOP 4 - EXAM - FEB
A. Lumbar nodes AUG 2014 2015
B. Superficial inguinal nodes MED
C. External iliac nodes BOARDS;
D. Internal iliac nodes TOPNOTCH
MD)
357 A 63 year old female patient says that she has pain in her A femoral hernia is more common in women that JESSICA MAE BACK-UP
groin and upper thigh. Upon examination, you palpate a in men. SANCHEZ, MIDTERM
lump located below the inguinal ligament, lateral to its The neck of the sac always lies below and lateral to MD (TOP 4 - EXAM - FEB
attachment to the pubic tubercle. You suspect that this may the pubic tubercle, distinguishing it from an AUG 2014 2015
be a hernia passing through the: inguinal hernia, which lies above and medial to the MED
A. femoral canal pubic tubercle. BOARDS;
B. obturator canal TOPNOTCH
C. adductor hiatus Reference: Snell. Clinical Anatomy by Regions, 8th MD)
D. superficial inguinal ring ed. p. 580
358 The sensory receptor for hearing is located in the: JESSICA MAE BACK-UP
A. Macula SANCHEZ, MIDTERM
B. Ampulla MD (TOP 4 - EXAM - FEB
C. Organ of Corti AUG 2014 2015
D. Saccule MED
BOARDS;
TOPNOTCH
MD)
359 A forearm arterio-venous fistula for hemodialysis access is Radiocephalic AV fistula is the vascular access of JESSICA MAE BACK-UP
usually done by anastomosing the: choice for hemodialysis. SANCHEZ, MIDTERM
A. Radial artery and basilic vein MD (TOP 4 - EXAM - FEB
B. Radial artery and cephalic vein AUG 2014 2015
C. Radial artery and antecubital vein MED
D. Ulnar artery and basilic vein BOARDS;
TOPNOTCH
MD)
360 The aortic valve is best heard clinically at the: JESSICA MAE BACK-UP
A. Right 2nd ICS close to the sternal border SANCHEZ, MIDTERM
B. Left 2nd ICS close to the sternal border MD (TOP 4 - EXAM - FEB
C. Left 5th ICS midclavicular line AUG 2014 2015
D. Xiphisternal junction MED
BOARDS;
TOPNOTCH
MD)
361 Gab is an avid tennis player with a killer serve. In order to do palmaris brevis (Fig. 9.55) is a small muscle that MAIRRE BACK-UP
this, he must have a good grip around the racket. Which arises from the flexor retinaculum and palmar JAMES MIDTERM
muscle improves the grip of the palm especially when aponeurosis and is inserted into the skin of the GADDI, MD EXAM - FEB
holding rounded objects? palm. It is supplied by the superficial branch of the (TOP 4 - AUG 2015
A. Abductor pollicis brevis ulnar nerve. Its function is to corrugate the skin at 2013 MED
B. Flexor pollicis brevis the base of the hypothenar eminence and so BOARDS;
C. Palmaris brevis improve the grip of the palm in holding a rounded TOPNOTCH
D. Extensor pollicis brevis object. Snell 9th pg 397 MD)

APB – Abduction of the thumb
FPB - Flexes MCP joint of the thumb
EPB – Extends MCP joint of the thumb

TOPNOTCH MEDICAL BOARD PREP ANATOMY SUPEREXAM Page 46 of 94


For inquiries visit www.topnotchboardprep.com.ph or email us at topnotchmedicalboardprep@gmail.com
TOPNOTCH MEDICAL BOARD PREP ANATOMY SUPEREXAM
For inquiries visit www.topnotchboardprep.com.ph or email us at topnotchmedicalboardprep@gmail.com
Item QUESTION EXPLANATION AUTHOR TOPNOTCH
# EXAM
362 Marco Reus of Borussia Dortmund recently damaged his Lateral ligament sprains occur in running and MAIRRE BACK-UP
ankle ligaments following a tackle from the outside while on jumping sports. The lateral ligament is injured JAMES MIDTERM
international duty with Germany. Which ligament was most because it is much weaker than the medial liga- GADDI, MD EXAM - FEB
likely involved? ment, and is the ligament that resists inversion at (TOP 4 - AUG 2015
A. Anterior talofibular ligament the talocrural joint. The anterior talofibular 2013 MED
B. Posterior talofibular ligament ligament is most vulnerable and most commonly BOARDS;
C. Calcaneofibular ligament torn during ankle sprains, either partially or TOPNOTCH
D. Anterior tibiotalar ligament completely, resulting in instability of the ankle MD)
joint.
Moore 7th pg 666
363 In order to achieve anesthesia of an intercostal space, an In the chest wall, the neurovascular bundles are MAIRRE BACK-UP
intercostal nerve block can be done by inserting the needle sheltered by the inferior margins of the overlying JAMES MIDTERM
through: ribs. The needle is directed toward the rib near the GADDI, MD EXAM - FEB
A. the superior margin of the rib above the space lower border and the tip comes to rest near the (TOP 4 - AUG 2015
B. the inferior margin of the rib above the space subcostal groove, where the anesthetic is 2013 MED
C. the superior margin of the rib below the space infiltrated around the nerve. Remember that the BOARDS;
D. the inferior margin of the rib below the space order of structures from above downward is TOPNOTCH
E. Between the inferior margin of the rib above the space and intercostal vein, artery, and nerve (VAN) Snell 9th MD)
the superior margin of the rib below the space pg 43

364 A patient with a large fungating tongue mass is schedule to The distance between the incisors to the vocal MAIRRE BACK-UP
undergo operative intervention. The anesthesiologist plans cords measures 5.9in (15cm), between the JAMES MIDTERM
to do nasotracheal intubation. He knows that the distance incisors to the carina 7.9in (20cm), between the GADDI, MD EXAM - FEB
from the external nares to the carina measures: external nares to the carina 11.8in (30cm). Snell (TOP 4 - AUG 2015
A. 15 cm 9th pg 720 2013 MED
B. 20 cm BOARDS;
C. 30 cm TOPNOTCH
D. 40 cm MD)

365 During tracheostomy it is important to note that the thyroid The isthmus is usually anterior to the second and MAIRRE BACK-UP
isthmus is at the level of the: third tracheal rings. Moore 7th pg1018 JAMES MIDTERM
A. Cricoid and 1st tracheal ring GADDI, MD EXAM - FEB
B. 1st and 2nd tracheal rings (TOP 4 - AUG 2015
C. 2nd and 3rd tracheal rings 2013 MED
D. Cricoid and 2nd tracheal rings BOARDS;
TOPNOTCH
MD)
366 Dr. Wi, the resident on duty received a patient involved in a Le Fort I: horizontal fractures of the maxillae, MAIRRE BACK-UP
vehicular crash and upon seeing the patient noted extensive passing superior to the maxillary alveolar process JAMES MIDTERM
maxillofacial injuries. During the subsequent management Le Fort II: passes from the posterolateral parts of GADDI, MD EXAM - FEB
imaging was done which showed a fracture line involving the the maxillary sinuses superomedially through the (TOP 4 - AUG 2015
frontozygomatic sutures, the ethmoid and nasal bones, the infraorbital foramina, lacrimals, or ethmoids to the 2013 MED
superior orbital fissures and the greater wings of the bridge of the nose. BOARDS;
sphenoid. His diagnosis would be? Le Fort III: horizontal fracture that passes through TOPNOTCH
A. Le Fort I the superior orbital fissures and the ethmoid and MD)
B. Le Fort II nasal bones and extends laterally through the
C. Le Fort III greater wings of the sphenoid and the
D. Le Fort IV frontozygomatic sutures.
There is no Le Fort IV. Moore 7th pg 837

367 During left radical nephrectomy, the left kidney is removed The suprarenal glands are supplied by three MAIRRE BACK-UP
together with the left suprarenal gland. As part of the arteries, inferior phrenic artery (superior), JAMES MIDTERM
procedure, the renal artery is ligated. To prevent bleeding of abdominal aorta (middle) and renal artery GADDI, MD EXAM - FEB
the vessels supplying the suprarenal gland, arterial (inferior) Snell 9th pg 215 (TOP 4 - AUG 2015
branch/branches coming from which of the following should 2013 MED
also be ligated? BOARDS;
A. Inferior phrenic artery TOPNOTCH
B. Aorta MD)
C. Splenic artery
D. A and B
E. All of the above

368 True of the rectum: The rectum is primarily retroperitoneal but it is MAIRRE BACK-UP
A. The rectum is a purely retroperitoneal organ and begins as also subperitoneal. The peritoneum covers the JAMES MIDTERM
a continuation of the sigmoid colon anterior and lateral surfaces of the first third of GADDI, MD EXAM - FEB
B. It is covered by a part of peritoneum up to its distal third the rectum and only the anterior surface of the (TOP 4 - AUG 2015
C. The proximal third is covered by peritoneum at its middle third, leaving the lower third devoid of 2013 MED
anterior and lateral surfaces peritoneum. Moore 7th pgs 253, 370; Snell 9th pg BOARDS;
D. All of the above 265 TOPNOTCH
MD)

369 What structure/s pass/es through the diaphragmatic The caval opening lies at the level of T8 and MAIRRE BACK-UP
opening at the level of T12? transmits the inferior vena cava and the terminal JAMES MIDTERM
A. Aorta and Thoracic duct branches of the right phrenic nerve. The GADDI, MD EXAM - FEB
B. Vena cava and phrenic nerve esophageal opening lies at the level of T10 and (TOP 4 - AUG 2015
C. Azygos vein transmits the esophagus, the right and left vagus 2013 MED
D. A and C nerves, the esophageal branches of the left gastric BOARDS;
E. All of the above vessels and the lymphatics of the lower third of TOPNOTCH
the esophagus. The aortic opening lies at the level MD)
of T12 and transmits the aorta, the thoracic duct
TOPNOTCH MEDICAL BOARD PREP ANATOMY SUPEREXAM Page 47 of 94
For inquiries visit www.topnotchboardprep.com.ph or email us at topnotchmedicalboardprep@gmail.com
TOPNOTCH MEDICAL BOARD PREP ANATOMY SUPEREXAM
For inquiries visit www.topnotchboardprep.com.ph or email us at topnotchmedicalboardprep@gmail.com
Item QUESTION EXPLANATION AUTHOR TOPNOTCH
# EXAM
and the azygos vein. Snell 9th pg 46

370 A 56/F diagnosed with endometrial carcinoma confined to The ureter can be damaged during ligation of the MAIRRE BACK-UP
the uterus on TV-UTZ was about to undergo a radical uterine artery during hysterectomy as it passes JAMES MIDTERM
hysterectomy with bilateral salpingo-oophorectomy, the immediately inferior to the uterine artery near the GADDI, MD EXAM - FEB
attending OBGYN wanting to avoid post operative lateral part of the fornix of the vagina and as it (TOP 4 - AUG 2015
complications recalls that the ureter can sustain iatrogenic crosses the uterine artery approximately 2 cms 2013 MED
injury at which of the following locations? superior to the ischial spine. It can also be BOARDS;
A. As it passes immediately inferior to the uterine artery near damaged during oophorectomy when the ovarian TOPNOTCH
the lateral part of the fornix of the vagina artery is ligated since they are close to each other MD)
B. As it crosses the uterine artery approximately 2 cms as they cross the pelvic brim. Moore 7th pg 361
superior to the ischial spine
C. As it crosses the pelvic brim together with the ovarian
artery
D. All of the above

371 A 34/M was on his way home when he was suddenly The patient most likely suffered from an MAIRRE BACK-UP
mugged and was hit at the side of the head with a lead pipe extradural hematoma which usually presents as a JAMES MIDTERM
by unknown assailants. He was briefly unconscious and was brief concussion followed by a lucid interval for a GADDI, MD EXAM - FEB
awakened by concerned bystanders. He then reported the few hours. This then leads to drowsiness and (TOP 4 - AUG 2015
incident to the police and since there were no obvious coma. Extradural hematomas are arterial in origin 2013 MED
external injuries he decided to go home. The following and are usually caused by extravasation of blood BOARDS;
morning, he was found dead by his family. The vessel that from the torn branches of the middle meningeal TOPNOTCH
was most likely responsible for the death of the patient is a artery (specifically the anterior branch which lies MD)
branch of the? immediately on top of the pterion). The middle
A. Middle meningeal artery meningeal artery is a branch of the maxillary
B. Internal carotid artery artery which in itself is a branch of the external
C. External carotid artery carotid artery. Moore 7th pg 876
D. Superior sagittal sinus

372 A 60/M smoker, diabetic, suddenly experienced severe MAIRRE BACK-UP


crushing chest pain. He was brought to the nearest hospital JAMES MIDTERM
and an ECG was done which showed ST elevation of leads GADDI, MD EXAM - FEB
V4-V6. Which vessel was most likely occluded? (TOP 4 - AUG 2015
A. Left circumflex artery 2013 MED
B. Left anterior descending artery BOARDS;
C. Right marginal artery TOPNOTCH
D. Left main coronary artery MD)

Snell 9th pg 89
373 A 32/F underwent total thyroidectomy. Postoperatively the Near the inferior pole of the thyroid gland, the MAIRRE BACK-UP
patient complains of hoarseness. This suggests damage to right recurrent laryngeal nerve is intimately JAMES MIDTERM
the _________ which is closely related to the___________. related to the inferior thyroid artery. Because of GADDI, MD EXAM - FEB
A. Recurrent laryngeal nerve: Superior thyroid artery this the inferior thyroid artery is ligated some (TOP 4 - AUG 2015
B. Recurrent laryngeal nerve: Inferior thyroid artery distance lateral to the thyroid gland. Although the 2013 MED
C. External laryngeal nerve: Superior thyroid artery danger of injuring the left recurrent laryngeal BOARDS;
D. External laryngeal nerve: Inferior thyroid artery nerve during surgery is not as great, owing to its TOPNOTCH
more vertical ascent from the superior MD)
mediastinum, the artery and nerve are also closely
associated near the inferior pole of the thyroid
gland. Hoarseness is the usual sign of unilateral
recurrent nerve injury; however, temporary
aphonia or disturbance of phonation and laryngeal
spasm may occur. These signs usually result from
bruising the recurrent laryngeal nerves during
surgery or from the pressure of accumulated
blood and serous exudate after the operation.
Moore 7th pg 1043

TOPNOTCH MEDICAL BOARD PREP ANATOMY SUPEREXAM Page 48 of 94


For inquiries visit www.topnotchboardprep.com.ph or email us at topnotchmedicalboardprep@gmail.com
TOPNOTCH MEDICAL BOARD PREP ANATOMY SUPEREXAM
For inquiries visit www.topnotchboardprep.com.ph or email us at topnotchmedicalboardprep@gmail.com
Item QUESTION EXPLANATION AUTHOR TOPNOTCH
# EXAM
374 A 7/M fell while from a tree and landed on his outstretched The suprascapular nerve is vulnerable to injury in MAIRRE BACK-UP
hand. He was quickly brought to the emergency room and fractures of the middle third of the clavicle. Injury JAMES MIDTERM
the examining physician noted that the limb is medially to the suprascapular nerve results in loss of lateral GADDI, MD EXAM - FEB
rotated with loss of abduction. There was also a step off rotation of the humerus at the glenohumeral joint. (TOP 4 - AUG 2015
deformity palpable at the middle third of the clavicle. What Consequently the relaxed limb rotates medially 2013 MED
nerve was most likely injured? into the waiter’s tip position. The ability to initiate BOARDS;
A. Subscapular nerve abduction of the limb is also affected. TOPNOTCH
B. Median nerve Injury to the axillary nerve from fracture of the MD)
C. Axillary nerve surgical neck of the humerus or dislocation of the
D. Suprascapular nerve humeral head will produce the same set of deficits.
Moore 7th pg 710 & 1009

375 A 27/F has multiple closed comedones on her nose. She then Inside each cavernous sinus is the internal carotid MAIRRE BACK-UP
proceeded to squeeze them individually. After a few days she artery with its small branches, surrounded by the JAMES MIDTERM
had a severe headache which was followed by high grade carotid plexus of sympathetic nerve(s), and the GADDI, MD EXAM - FEB
fever, periorbital edema and diplopia. A diagnosis of abducent nerve. The oculomotor (CN III) and (TOP 4 - AUG 2015
cavernous sinus thrombosis was made. Which is/are found trochlear (CN IV) nerves, plus two of the three 2013 MED
inside the cavernous sinus? divisions of the trigeminal nerve (CN V1 and V2) BOARDS;
A. Internal carotid artery are embedded in the lateral wall of the sinus. TOPNOTCH
B. Trochlear nerve Moore 7th pg 869 MD)
C. Mandibular nerve
D. A and B
E. All of the above

376 Which of the following is/are TRUE of the inferior vena cava? The inferior vena cava has the following MAIRRE BACK-UP
A. There are two anterior visceral tributaries tributaries: 2 anterior visceral tributaries (hepatic JAMES MIDTERM
B. There are three veins of origin veins), 3 lateral visceral tributaries (right GADDI, MD EXAM - FEB
C. There are three visceral tributaries suprarenal, right renal and right gonadal vein), 5 (TOP 4 - AUG 2015
D. All of the above lateral abdominal wall tributaries (one inferior 2013 MED
E. A and C phrenic vein and four lumbar veins) and 3 veins of BOARDS;
origin (two common iliac veins and the median TOPNOTCH
sacral vein). Snell 9th pg 218 MD)
377 A 30/M has extensive metastatic testicular carcinoma. The preaortic lymph nodes lie around the origins MAIRRE BACK-UP
Metastatic foci would most likely be found where? of the celiac, superior mesenteric, and inferior JAMES MIDTERM
A. Superficial inguinal lymph nodes mesenteric arteries and are referred to as the GADDI, MD EXAM - FEB
B. Deep inguinal lymph nodes celiac, superior mesenteric, and inferior (TOP 4 - AUG 2015
C. Para-aortic lymph nodes mesenteric lymph nodes, respectively. They drain 2013 MED
D. Pre-aortic lymph nodes the lymph from the gastrointestinal tract, BOARDS;
E. All of the above extending from the lower one third of the TOPNOTCH
esophagus to halfway down the anal canal, and MD)
from the spleen, pancreas, gallbladder, and greater
part of the liver.
lateral aortic (para-aortic or lumbar) lymph nodes
drain lymph from the kidneys and suprarenals;
from the testes in the male and from the ovaries,
uterine tubes, and fundus of the uterus in the
female; from the deep lymph
vessels of the abdominal walls; and from the
common iliac nodes. Snell 9th pgs 132 & 220
378 Renal transplantation involves: The iliac fossa on the posterior abdominal wall is MAIRRE BACK-UP
A. End to side anastomosis of the donor renal artery to the the usual site chosen for transplantation of the JAMES MIDTERM
recipient external iliac artery kidney. The fossa is exposed through an incision in GADDI, MD EXAM - FEB
B. End to end anastomosis of the donor renal vein to the the anterior abdominal wall just above the (TOP 4 - AUG 2015
recipient internal iliac vein inguinal ligament. The iliac fossa in front of the 2013 MED
C. Anastomosis of the donor ureter to the recipient bladder iliacus muscle is approached retroperitoneally. BOARDS;
through a ureterocystostomy The renal artery is anastomosed end to end to the TOPNOTCH
D. All of the above internal iliac artery and the renal vein is MD)
E. A and B anastomosed end to side to the external iliac vein.
Ureterocystostomy is then performed by opening
the bladder and providing a wide entrance of the
ureter through the bladder wall. Snell 9th pg 210
379 The following are wholly derived from endoderm EXCEPT Ectoderm derivatives - central nervous system, MAIRRE BACK-UP
A. Greater vestibular glands cornea, retina, and lens, membranous labyrinth of JAMES MIDTERM
B. Prostate gland the inner ear, epidermis, epithelial cells of the GADDI, MD EXAM - FEB
C. Mammary glands sebaceous, sweat, and mammary glands; the (TOP 4 - AUG 2015
D. Bulbourethral glands mucous membrane lining the mouth, nasal 2013 MED
cavities, and paranasal sinuses; the pituitary gland BOARDS;
and the alveoli and ducts of the parotid salivary TOPNOTCH
glands; the mucous membrane of the lower half of MD)
the anal canal; and the terminal parts of the genital
tract and the male urinary tract
Entoderm: epithelial lining of the alimentary tract
from the mouth cavity down to halfway along the
anal canal, thyroid, parathyroid, thymus, liver, and
pancreas, linings of the respiratory tract,
pharyngotympanic tube and middle ear, urinary
bladder, parts of the female and male urethras,
greater vestibular glands, prostate gland,
bulbourethral glands, and vagina.

TOPNOTCH MEDICAL BOARD PREP ANATOMY SUPEREXAM Page 49 of 94


For inquiries visit www.topnotchboardprep.com.ph or email us at topnotchmedicalboardprep@gmail.com
TOPNOTCH MEDICAL BOARD PREP ANATOMY SUPEREXAM
For inquiries visit www.topnotchboardprep.com.ph or email us at topnotchmedicalboardprep@gmail.com
Item QUESTION EXPLANATION AUTHOR TOPNOTCH
# EXAM
380 Which of the following contains a submucosal layer? The wall of the gallbladder does not contain a MAIRRE BACK-UP
A. Gallbladder muscularis mucosae or submucosa. diFiore 12th JAMES MIDTERM
B. Trachea pg 384, the fallopian tubes lack a submucosal layer GADDI, MD EXAM - FEB
C. Fallopian tubes diFiore 12th pg 520, the lamina propria of the (TOP 4 - AUG 2015
D. Larynx laryngeal mucosa blends with the perichondrium 2013 MED
of the thyroid cartilage and there is no distinct BOARDS;
submucosa diFiore 12th pg 398. TOPNOTCH
MD)
381 Which of the following best describes the location of the SIMILAR TO PREVIOUS BOARD EXAM SCOTT BACK-UP
right adrenal gland? CONCEPT/PRINCIPLE. The right adrenal gland is RILEY ONG, MIDTERM
A. Inferior to the right lobe of the liver posterolateral to the inferior vena cava, lateral to MD (TOP 5 - EXAM - FEB
B. Posterolateral to the inferior vena cava the 1st lumbar vertebrae, posterior to the right AUG 2014 2015
C. Anterior to the 1st lumbar vertebra lobe of the liver and supeior to the right kidney. MED
D. Posterior to the right kidney BOARDS;
E. Medial to the spleen TOPNOTCH
MD)

382 Which of the following drains segment I of the liver? SCOTT BACK-UP
A. Middle hepatic vein RILEY ONG, MIDTERM
B. Right hepatic vein MD (TOP 5 - EXAM - FEB
C. Direct venous drainage into the inferior vena cava AUG 2014 2015
D. Direct venous drainage into the common hepatic vein MED
E. Left gastric vein BOARDS;
TOPNOTCH
MD)
383 A 65-year old female who has been bedridden for 3 months SCOTT BACK-UP
presented with symptoms of pneumonia. You suspect an RILEY ONG, MIDTERM
aspiration component for her disease. In which of the MD (TOP 5 - EXAM - FEB
following lung segments will you most likely see the AUG 2014 2015
infiltrates on her chest radiograph? MED
A. Left superior lingular segment BOARDS;
B. Left apicoposterior segment TOPNOTCH
C. Right superior segment MD)
D. Right posterobasal segment
E. Right medial segment

384 The embryonic right 4th aortic arch develops into which of It contributes to the proximal part of the right SCOTT BACK-UP
the following adult structures? subclavian artery. The left 4th aortic arch forms RILEY ONG, MIDTERM
A. Arch of the aorta the arch of the aorta. MD (TOP 5 - EXAM - FEB
B. Right internal carotid artery AUG 2014 2015
C. Right common carotid artery MED
D. Right subclavian artery BOARDS;
E. Right brachiocephalic artery TOPNOTCH
MD)

385 Which of the following is not true about the lesser sac of the The lesser sac is normally collapsed. It can become SCOTT BACK-UP
abdomen? huge and visible when it is filled with fluid (eg. RILEY ONG, MIDTERM
A. The stomach forms its anterior boundary ascites) during disease states. MD (TOP 5 - EXAM - FEB
B. It communicates with the greater sac through the foramen AUG 2014 2015
of Winslow MED
C. The gastrosplenic ligament forms its lateral boundary BOARDS;
D. The lesser omentum transmits the coronary veins which TOPNOTCH
can dilate as varices during portal hypertension MD)
E. The lesser sac is normally filled with fluid and is easily
visualized on CT imaging.

386 During a 12-lead ECG, where should lead V2 be normally SIMILAR TO PREVIOUS BOARD EXAM SCOTT BACK-UP
placed? CONCEPT/PRINCIPLE. RILEY ONG, MIDTERM
A. 4th intercostal space, left parasternal border MD (TOP 5 - EXAM - FEB
B. 5th intercostal space, left parasternal border AUG 2014 2015
C. 4th intercostal space, left midclavicular line MED
D. 5th intercostal space, left midclavicular line BOARDS;
E. 4th intercostal space, right parasternal border TOPNOTCH
MD)

387 The ureters enter the urinary bladder at which of its aspects? SIMILAR TO PREVIOUS BOARD EXAM SCOTT BACK-UP
A. Anterolateral CONCEPT/PRINCIPLE. RILEY ONG, MIDTERM
B. Lateral MD (TOP 5 - EXAM - FEB
C. Posteromedial AUG 2014 2015
D. Posterolateral MED
E. Superomedial BOARDS;
TOPNOTCH
MD)

TOPNOTCH MEDICAL BOARD PREP ANATOMY SUPEREXAM Page 50 of 94


For inquiries visit www.topnotchboardprep.com.ph or email us at topnotchmedicalboardprep@gmail.com
TOPNOTCH MEDICAL BOARD PREP ANATOMY SUPEREXAM
For inquiries visit www.topnotchboardprep.com.ph or email us at topnotchmedicalboardprep@gmail.com
Item QUESTION EXPLANATION AUTHOR TOPNOTCH
# EXAM
388 A patient suffered from myocardial infarction and showed SIMILAR TO PREVIOUS BOARD EXAM SCOTT BACK-UP
signs of hypotension and bradycardia. Which myocardial CONCEPT/PRINCIPLE. RILEY ONG, MIDTERM
wall is most likely affected given this clinical picture. MD (TOP 5 - EXAM - FEB
A. Anterior wall AUG 2014 2015
B. Anteroseptal wall MED
C. Lateral wall BOARDS;
D. Inferior wall TOPNOTCH
E. Posterior wall MD)

389 The thyroid isthmus most commonly lies over which tracheal SIMILAR TO PREVIOUS BOARD EXAM SCOTT BACK-UP
rings? CONCEPT/PRINCIPLE. RILEY ONG, MIDTERM
A. 1st and 2nd MD (TOP 5 - EXAM - FEB
B. 2nd and 3rd AUG 2014 2015
C. 3rd and 4th MED
D. 4th and 5th BOARDS;
E. 5th and 6th TOPNOTCH
MD)

390 Which of the following paranasal sinuses is the last to Present at birth: maxillary and ethmoid sinuses. 3 SCOTT BACK-UP
develop? years old: sphenoid sinus begins to appear. 6 years RILEY ONG, MIDTERM
A. Maxillary sinus old: frontal sinus begins to develop. MD (TOP 5 - EXAM - FEB
B. Ethmoid sinus AUG 2014 2015
C. Sphenoid sinus MED
D. Frontal sinus BOARDS;
E. Both C and D TOPNOTCH
MD)

391 The superior mesenteric vein drains directly into which of The portal vein is formed by the union of the SCOTT BACK-UP
the following structures? superior mesenteric vein and splennic vein. RILEY ONG, MIDTERM
A. Portal vein MD (TOP 5 - EXAM - FEB
B. Celiac vein AUG 2014 2015
C. Inferior vena cava MED
D. Splenic vein BOARDS;
E. Common hepatic vein TOPNOTCH
MD)

392 Which of the following forms the posterior border of the The right ventricle forms its anterior and inferior SCOTT BACK-UP
heart? border. The right atrium forms its right border. RILEY ONG, MIDTERM
A. Right atrium The left ventricle forms its left border. MD (TOP 5 - EXAM - FEB
B. Right ventricle AUG 2014 2015
C. Left atrium MED
D. Left ventricle BOARDS;
E. Apex TOPNOTCH
MD)

393 Which of the following structures is not contained within the The ilioinguinal nerve runs along with but outside SCOTT BACK-UP
spermatic cord? the spermatic cord. RILEY ONG, MIDTERM
A. Vas deferens MD (TOP 5 - EXAM - FEB
B. Ilioinguinal nerve AUG 2014 2015
C. Genital branch of genitofemoral nerve MED
D. Pampiniform plexus BOARDS;
E. Tunica vaginalis TOPNOTCH
MD)

394 Which of the following parts of the urinary bladder is most SIMILAR TO PREVIOUS BOARD EXAM SCOTT BACK-UP
sensitive to stretch? CONCEPT/PRINCIPLE. RILEY ONG, MIDTERM
A. Apex MD (TOP 5 - EXAM - FEB
B. Base AUG 2014 2015
C. Trigone MED
D. Ureteral insertion BOARDS;
E. Urethral opening TOPNOTCH
MD)

395 The stylohyoid muscle is innervated by which of the CN V3: masticator muscles, anterior belly of SCOTT BACK-UP
following nerves? digastric, mylohyoid. CN VII: facial muscles, RILEY ONG, MIDTERM
A. CN V3 stapedius, stylohyoid, posterior belly of digastric. MD (TOP 5 - EXAM - FEB
B. CN VII CN IX: stylopharyngeus. CN XII: intrinsic and AUG 2014 2015
C. CN IX extrinsic tongue muscles (except palatoglossus, MED
D. CN X which is innervated by CN X) BOARDS;
E. CN XII TOPNOTCH
MD)

396 An avulsion fracture at the base of the first proximal phalanx Gamekeeper's thumb: base of 1st proximal SCOTT BACK-UP
is known as: phalanx. Bennett's fracture: base of 1st RILEY ONG, MIDTERM
A. Gamekeeper's thumb metacarpal. Boxer's fracture: neck of 4th and 5th MD (TOP 5 - EXAM - FEB
B. Bennett's fracture metacarpal. Lisfranc fracture: metatarsal fracture AUG 2014 2015
C. Boxer's fracture MED
D. Colles fracture BOARDS;
E. Lisfranc fracture TOPNOTCH
MD)

TOPNOTCH MEDICAL BOARD PREP ANATOMY SUPEREXAM Page 51 of 94


For inquiries visit www.topnotchboardprep.com.ph or email us at topnotchmedicalboardprep@gmail.com
TOPNOTCH MEDICAL BOARD PREP ANATOMY SUPEREXAM
For inquiries visit www.topnotchboardprep.com.ph or email us at topnotchmedicalboardprep@gmail.com
Item QUESTION EXPLANATION AUTHOR TOPNOTCH
# EXAM
397 Which of the following inner ear structures is involved SIMILAR TO PREVIOUS BOARD EXAM SCOTT BACK-UP
during horizontal linear deceleration? CONCEPT/PRINCIPLE. Utricle: horizontal linear RILEY ONG, MIDTERM
A. Utricle acceleration. Saccule: vertical linear acceeleration. MD (TOP 5 - EXAM - FEB
B. Saccule Semicircular canals: angular acceleration. Scala AUG 2014 2015
C. Horizontal semicircular canal media and basilar membrane: involved in hearing MED
D. Scala media function. BOARDS;
E. Basilar membrane TOPNOTCH
MD)

398 Gastrointestinal stromal tumor (GIST), the most common SIMILAR TO PREVIOUS BOARD EXAM SCOTT BACK-UP
mesenchymal tumor of the stomach, most commonly arises CONCEPT/PRINCIPLE. RILEY ONG, MIDTERM
from which of its layers? MD (TOP 5 - EXAM - FEB
A. Mucosa AUG 2014 2015
B. Submucosa MED
C. Muscularis mucosae BOARDS;
D. Muscularis propria TOPNOTCH
E. Serosa MD)

399 What is the lining epithelium of the ovary? SIMILAR TO PREVIOUS BOARD EXAM SCOTT BACK-UP
A. Simple squamous CONCEPT/PRINCIPLE. RILEY ONG, MIDTERM
B. Simple cuboidal MD (TOP 5 - EXAM - FEB
C. Simple columnar AUG 2014 2015
D. Stratified squamous MED
E. Stratified cuboidal BOARDS;
TOPNOTCH
MD)
400 What is the lining epithelium of the prostatic urethra SIMILAR TO PREVIOUS BOARD EXAM SCOTT BACK-UP
A. Simple squamous CONCEPT/PRINCIPLE. RILEY ONG, MIDTERM
B. Stratified squamous MD (TOP 5 - EXAM - FEB
C. Transitional AUG 2014 2015
D. Simple cuboidal MED
E. Stratified cuboidal BOARDS;
TOPNOTCH
MD)
401 What is the most common type of Myoma Uteri? Yes, gyne questions can be seen in anatomy! JOSE CARLO DIAGNOSTIC
A. Submucous SIMILAR TO PREVIOUS BOARD EXAM MASANGKAY EXAM - AUG
B. Pedunculated CONCEPT/PRINCIPLE III, MD (TOP 2014
C. Subserosal 8 - FEB 2014
D. Intramural MED
E. Polypoid BOARDS;
TOPNOTCH
MD)
402 The Dosalis Pedis Artery is SIMILAR TO PREVIOUS BOARD EXAM JOSE CARLO DIAGNOSTIC
A. A continuation of the posterior tibial artery CONCEPT/PRINCIPLE MASANGKAY EXAM - AUG
B. Medial to the adductor longus tendon III, MD (TOP 2014
C. Medial to the medial malleolus 8 - FEB 2014
D. Lateral to the Extensor digitorum longus MED
E. Lateral to the Extensor hallucis longus BOARDS;
TOPNOTCH
MD)
403 Which of the following is derived from the 2nd pharyngeal Meckel's Cartilage is derived from the 1st JOSE CARLO DIAGNOSTIC
arch pharyngeal arch, Hyoid which is derived from the MASANGKAY EXAM - AUG
A. Reichter's Cartilage Reichter's Cartilage is derived from the 2nd III, MD (TOP 2014
B. Meckel's Cartilage pharyngeal arch 8 - FEB 2014
C. Hyoid bone MED
D. A and C BOARDS;
E. All of the above TOPNOTCH
MD)

404 The Cremaster Muscle is a continuation of which of the Ext spermatic fascia derived from Ext. oblique JOSE CARLO DIAGNOSTIC
following? muscle, Darto's from Superficial fascia, Internal MASANGKAY EXAM - AUG
A. External Oblique Muscle spermatic fascia from Transversalis fascia III, MD (TOP 2014
B. Rectus abdominis muscle 8 - FEB 2014
C. Internal Oblique muscle MED
D. Transversalis muscle BOARDS;
E. Transversalis fascia TOPNOTCH
MD)

405 Emphysema is a pulmonary disease where expiration is JOSE CARLO DIAGNOSTIC


active instead of its passive nature. In a patient with MASANGKAY EXAM - AUG
emphysema which of the following is not a muscle used in III, MD (TOP 2014
forced/active expiration? 8 - FEB 2014
A. Internal Intecostal MED
B. External Intercostal BOARDS;
C. Rectus abdominis TOPNOTCH
D. Serratus Posterior Inferior MD)
E. Transverse Thoracis

TOPNOTCH MEDICAL BOARD PREP ANATOMY SUPEREXAM Page 52 of 94


For inquiries visit www.topnotchboardprep.com.ph or email us at topnotchmedicalboardprep@gmail.com
TOPNOTCH MEDICAL BOARD PREP ANATOMY SUPEREXAM
For inquiries visit www.topnotchboardprep.com.ph or email us at topnotchmedicalboardprep@gmail.com
Item QUESTION EXPLANATION AUTHOR TOPNOTCH
# EXAM
406 Which of the following is the reason for the higher tendency SIMILAR TO PREVIOUS BOARD EXAM JOSE CARLO DIAGNOSTIC
of the Sigmoid Colon for Volvolus? CONCEPT/PRINCIPLE MASANGKAY EXAM - AUG
A. Because it is most redundant III, MD (TOP 2014
B. Because It is intraperitoneal 8 - FEB 2014
C. Because it is the narrowest MED
D. Because it is hypermotile BOARDS;
E. Because of its thin wall TOPNOTCH
MD)

407 Which of the following Sinuses is/are present at birth? JOSE CARLO DIAGNOSTIC
A. Frontal MASANGKAY EXAM - AUG
B. Maxillary III, MD (TOP 2014
C. Sphenoid 8 - FEB 2014
D. A and B MED
E. B and C BOARDS;
TOPNOTCH
MD)
408 A patient was rushed to the ED after an automobile accident, A Lucid interval was noted in this patient this is JOSE CARLO DIAGNOSTIC
patient came in with decreased sensorium and multiple frequently associated with the presence of an MASANGKAY EXAM - AUG
abrasions, after a few hours, the patient became conscious epidural hematoma due to a rupture of the middle III, MD (TOP 2014
and coherent which was eventually followed by a coma, on meningeal artery which passes in proximity to the 8 - FEB 2014
physical examination you noted crepitations on the Left pterion. The middle meningeal artery passes thru MED
Pterion area. As a brilliant doctor you knew that there was the Foramen Spinosum BOARDS;
an injured vessel which passes in which foramen of the skull TOPNOTCH
A. Foramen Magnun MD)
B. Foramen Lacerum
C. Foramen Ovale
D. Foramen Spinosum
E. Foramen Rotundum

409 Which of the following structures traverses the Aortic Hiatus The Aorta, Thoracic Duct and Azygous vein enters JOSE CARLO DIAGNOSTIC
of the diaphragm? the aortic hiatus. MASANGKAY EXAM - AUG
A. Thoracic Duct III, MD (TOP 2014
B. Hemiazygous vein 8 - FEB 2014
C. Right Phrenic Nerve MED
D. Vagus Nerve BOARDS;
E. Right Lymphatic Duct TOPNOTCH
MD)

410 A 1-month old female patient was noted to have Rib- Patient is a case of Turner's syndrome, these JOSE CARLO DIAGNOSTIC
notching on X-ray, a chromosomal study was done in this patients have a propensity to develop a Patent MASANGKAY EXAM - AUG
patient revealing a chromosomal count of 45 XO, if this Ductus Arteriosus which presents with rib III, MD (TOP 2014
patient would undergo repair of the said cardiac defect what notching on X-ray. The Left vagus nerve "recurrs" 8 - FEB 2014
structure may be injured? at the aortic arch to become the left recurrent MED
A. Right Phrenic Nerve laryngeal nerve, the Ductus Arteriosus is in BOARDS;
B. Right Vagus Nerve proximity to this structure. The right vagus nerve TOPNOTCH
C. Left Vagus Nerve "recurrs" to become the right recurrent at the MD)
D. Left Phrenic Nerve right subclavian artery.
E. Hemiazygous vein

411 A male patient was rushed to your trauma center after a Flexor carpi ulnaris tendon can be transected in a JOSE CARLO DIAGNOSTIC
massive bleeding secondary to a self-inflicted deep incised deep laceration of the wrist in the ulnar aspect. MASANGKAY EXAM - AUG
wound to the radial aspect of the left wrist, after a recent III, MD (TOP 2014
break-up with his girlfriend. Which of the following 8 - FEB 2014
structures is most likely preserved? MED
A. median nerve BOARDS;
B. flexor carpi radialis tendon TOPNOTCH
C. palmaris longus tendon MD)
D. flexor carpi ulnaris tendon
E. none of the above

412 A prison inmate was rushed to your emergency department from anterior to posterior: Renal vein, Renal JOSE CARLO DIAGNOSTIC
due to an apparent stab wound at the flank on the left artery, renal pelvis. MASANGKAY EXAM - AUG
midscapular line at the level of L2 which of the following III, MD (TOP 2014
structures may be least severed? 8 - FEB 2014
A. Renal vein MED
B. Renal artery BOARDS;
C. Renal pelvis TOPNOTCH
D. Psoas Muscle MD)
E. Quadratus Lumborum muscle

TOPNOTCH MEDICAL BOARD PREP ANATOMY SUPEREXAM Page 53 of 94


For inquiries visit www.topnotchboardprep.com.ph or email us at topnotchmedicalboardprep@gmail.com
TOPNOTCH MEDICAL BOARD PREP ANATOMY SUPEREXAM
For inquiries visit www.topnotchboardprep.com.ph or email us at topnotchmedicalboardprep@gmail.com
Item QUESTION EXPLANATION AUTHOR TOPNOTCH
# EXAM
413 An opera singer underwent a Near Total Thyroidectomy due Injury to the external branch of the superior JOSE CARLO DIAGNOSTIC
to the discovery of an apparent Papillary Thyroid Carcinoma, laryngeal nerve causes paralysis of the MASANGKAY EXAM - AUG
post-surgery, patient was unable to reach high notes, what cricothyroid muscle which tenses and stretches III, MD (TOP 2014
could have been the reason? the vocal cords enabling a person to reach high 8 - FEB 2014
A. Injury to the internal branch of the superior laryngeal notes. MED
nerve BOARDS;
B. Unilateral injury to the recurrent laryngeal nerve TOPNOTCH
C. Bilateral injury to the recurrent laryngeal nerve MD)
D. Injury to the external branch of the superior laryngeal
nerve
E. Injury to the rima glottidis

414 Which of the following is the major blood supply of the The Inferior Parathyroid artery supplies both the JOSE CARLO DIAGNOSTIC
parathyroid glands? superior and inferior parathyroid glands MASANGKAY EXAM - AUG
A. Superior Thyroid Artery III, MD (TOP 2014
B. Inferior Thyroid Artery 8 - FEB 2014
C. Superior Parathyroid artery MED
D. Inferior Parathyroid Artery BOARDS;
E. Middle Thyroid Artery TOPNOTCH
MD)

415 Which of the following cells is responsible for Osteoid SIMILAR TO PREVIOUS BOARD EXAM JOSE CARLO DIAGNOSTIC
formation? CONCEPT/PRINCIPLE MASANGKAY EXAM - AUG
A. Osteocyte III, MD (TOP 2014
B. Osteoblast 8 - FEB 2014
C. Osteoclast MED
D. Langhan's Cell BOARDS;
E. Langerhan's Cell TOPNOTCH
MD)

416 After a prolonged intubation of a COPD patient you decided Tracheostomy is performed at the level of the 2nd JOSE CARLO DIAGNOSTIC
to perform a tracheostomy to lessen the Dead Space and and 4th tracheal rings. MASANGKAY EXAM - AUG
improve ventilation, on what location are you going to III, MD (TOP 2014
perform the tracheostomy? 8 - FEB 2014
A. at the level of C4 vertebra MED
B. in between the thyroid and cricoid cartilage BOARDS;
C. above the thyoid cartilage TOPNOTCH
D. at the 1st to the 4th tracheal rings MD)
E. at the 2nd to the 4th tracheal rings

417 After an automobile accident you realized that the Severance of the Occulomotor nerve causes JOSE CARLO DIAGNOSTIC
occulomotor nerve has been severed due to what diplopia, loss of parallel gaze, fixed and dilated MASANGKAY EXAM - AUG
manifestation of the patient? pupil, loss of light reflex, loss of accomodation, III, MD (TOP 2014
A. Pinpoint pupils Ptosis (Droopy eyelids) (SIMILAR TO PREVIOUS 8 - FEB 2014
B. Droopy eyelids BOARD EXAM CONCEPT/PRINCIPLE) MED
C. Weakness looking down BOARDS;
D. Loss of Corneal reflex TOPNOTCH
E. Horizontal diplopia MD)

418 The melanocytes are seen in which layer of the epidermis? Melanocytes are seen in the basal layer of the JOSE CARLO DIAGNOSTIC
A. S. Corneum epidermis. (SIMILAR TO PREVIOUS BOARD EXAM MASANGKAY EXAM - AUG
B. S. Lucidum CONCEPT/PRINCIPLE) III, MD (TOP 2014
C. S. Granulosum 8 - FEB 2014
D. S. Spinosum MED
E. S. Basale BOARDS;
TOPNOTCH
MD)
419 The oxyphil cells are found in which human structure? The oxyphil cells are seen in the parathyroid JOSE CARLO DIAGNOSTIC
A. Pineal Gland glands still with unknown function. (SIMILAR TO MASANGKAY EXAM - AUG
B. Parathyroid Gland PREVIOUS BOARD EXAM CONCEPT/PRINCIPLE) III, MD (TOP 2014
C. Anterior Pituitary Gland 8 - FEB 2014
D. Posterior Pituitary Gland MED
E. Thyroid Gland BOARDS;
TOPNOTCH
MD)
420 A 35 year-old female mountain climber went under a The thoracodorsal nerve innervates the Latissimus JOSE CARLO DIAGNOSTIC
Modified Radical Mastectomy after being diagnosed with dorsi muscle which elevates the trunk (as if MASANGKAY EXAM - AUG
Breast Cancer. Post-op, patient can move all her limbs, can attempting to climb) this may be a result of a III, MD (TOP 2014
protract her scapula, can laterally and medially rotate her surgical procedure involving the axilla. 8 - FEB 2014
arm, can extend MP joints of all digits and can flex the wrist MED
BUT she can not elevate her trunk. Which nerve may be BOARDS;
injured intra-operatively? TOPNOTCH
A. Long Thoracic nerve MD)
B. Thoracodorsal Nerve
C. Axillary Nerve
D. Lateral Pectoral Nerve
E. Muculocutaneous Nerve

TOPNOTCH MEDICAL BOARD PREP ANATOMY SUPEREXAM Page 54 of 94


For inquiries visit www.topnotchboardprep.com.ph or email us at topnotchmedicalboardprep@gmail.com
TOPNOTCH MEDICAL BOARD PREP ANATOMY SUPEREXAM
For inquiries visit www.topnotchboardprep.com.ph or email us at topnotchmedicalboardprep@gmail.com
Item QUESTION EXPLANATION AUTHOR TOPNOTCH
# EXAM
421 The major blood supply to the parathyroid glands comes There is much controversy to this question. But WEBSTER MIDTERM 1
from: we have to choose, again, the BEST possible ALINDOG, EXAM - AUG
A. Superior thyroid artery answer. Most anatomy books will say that the MD (TOP 3 - 2014
B. Inferior thyroid artery inferior thyroid artery supplies the parathyroids, FEB 2014
C. Superior parathyroid artery but according to Snell 7th edition, the blood comes MED
D. Inferior parathyroid artery from both superior and inferior thyroid arteries. BOARDS;
However, we deem that the inferior thyroid artery TOPNOTCH
is more consistent, and hence the better answer. MD)
Nonetheless, of note, in a study done by Nobori, et
al (PubMed) they have found that 45% of their
subjects had a distinct anastomosing branch
between the superior and inferior thyroid arteries,
supplying also the superior parathyroids.
422 A 25-year old male patient was observed to have recurrent The cavernous sinuses are situated in the middle WEBSTER MIDTERM 1
pyogenic infections of the nasal sinuses. He later on cranial fossa on each side of the body of sphenoid. ALINDOG, EXAM - AUG
developed cavernous sinus thrombosis, a potentially serious They extend from the superior orbital fissure in MD (TOP 3 - 2014
condition because of the many delicate structures that can be front, to the apex of the petrous part of the FEB 2014
affected. Which of the following is least likely to be involved? temporal bone behind. Found within these sinuses MED
A. CN II are the internal carotid artery, CN III and IV, and BOARDS;
B. CN III the ophthalmic and maxillary divisions of CN V. TOPNOTCH
C. CN IV They drain posteriorly into the superior and MD)
D. Internal carotid artery inferior petrosal sinuses and inferiorly into the
E. None of the above. pterygoid venous plexus. They communicate with
the facial vein through the superior ophthalmic
vein.

423 Sounds produced by the aortic valve can be best heard at: Study also some surface anatomy. Pulmonic valve WEBSTER MIDTERM 1
A. 5th ICS, left midclavicular line is best heard at 2nd ICS, left parasternal; mitral ALINDOG, EXAM - AUG
B. 2nd ICS, right parasternal valve at 5th ICS, left MCL; and tricuspid valve at MD (TOP 3 - 2014
C. 2nd ICS, left parasternal the right half of the lower end of the body of FEB 2014
D. Right half of the lower end of the body of sternum sternum. MED
BOARDS;
TOPNOTCH
MD)
424 A perforating ulcer located at the lesser curvature of the Gastric ulcers usually occur within the body of the WEBSTER MIDTERM 1
stomach will most likely erode which branch of the celiac stomach along the lesser curvature above the ALINDOG, EXAM - AUG
artery? incisura angularis. If it perforates, it can erode the MD (TOP 3 - 2014
A. Splenic artery left gastric artery (reviewing the blood supply of FEB 2014
B. Gastroduodenal artery the stomach: Lesser curvature --- right and left MED
C. Left gastric artery gastric arteries; greater curvature --- right and left BOARDS;
D. Left gastroepiploic artery gastroepiploic arteries; fundus --- short gastric TOPNOTCH
artery). Both the left gastric artery and the splenic MD)
artery are branches of the celiac artery.
Gastroduodenal artery comes from the hepatic
artery whereas the left gastroepiploic artery
originates from the gastroduodenal artery.
425 The cell organelle containing oxidative enzymes, WEBSTER MIDTERM 1
synthesizing H2O2 and is involved in beta-oxidation of long ALINDOG, EXAM - AUG
chain fatty acids: MD (TOP 3 - 2014
A. Lysosome FEB 2014
B. Smooth ER MED
C. Mitochondria BOARDS;
D. Peroxisome TOPNOTCH
MD)

426 The most common site of aortic aneurysm: Aneurysms are dilatations of the aorta usually WEBSTER MIDTERM 1
A. Distal to the portion of aorta passing through the resulting from atherosclerosis, which causes ALINDOG, EXAM - AUG
diaphragmatic opening arterial wall weakening. MD (TOP 3 - 2014
B. Below the origin of the renal arteries FEB 2014
C. Along the portion contained within the posterior MED
mediastinum BOARDS;
D. Just distal to the bifurcation of the common iliacs TOPNOTCH
MD)

427 All of the following muscles are important in plantar flexing Gastrocnemius, soleus, and plantaris together WEBSTER MIDTERM 1
the foot at the ankle joint except for: serve as a powerful plantar flexor of the ankle ALINDOG, EXAM - AUG
A. Gastrocnemius joint. They provide the main propulsive force in MD (TOP 3 - 2014
B. Popliteus walking and running. They are supplied by the FEB 2014
C. Soleus tibial nerve, with nerve root coming from S1 and MED
D. Plantaris S2. BOARDS;
TOPNOTCH
MD)

TOPNOTCH MEDICAL BOARD PREP ANATOMY SUPEREXAM Page 55 of 94


For inquiries visit www.topnotchboardprep.com.ph or email us at topnotchmedicalboardprep@gmail.com
TOPNOTCH MEDICAL BOARD PREP ANATOMY SUPEREXAM
For inquiries visit www.topnotchboardprep.com.ph or email us at topnotchmedicalboardprep@gmail.com
Item QUESTION EXPLANATION AUTHOR TOPNOTCH
# EXAM
428 A 22-year old frisbee player was brought to the hospital after The anterior cruciate ligament prevents the WEBSTER MIDTERM 1
sustaining a knee injury. On physical exam, he was noted to posterior displacement of femur on tibia, or ALINDOG, EXAM - AUG
demonstrate a positive anterior drawer sign. This suggests conversely, prevents the anterior displacement of MD (TOP 3 - 2014
that: tibia on femur (anterior drawer test). Option A FEB 2014
A. There is injury to the knee ligament that is attached to refers to the lateral collateral ligament, which is MED
the lateral condyle of the femur above and to the head of the also more flexible and hence less susceptible to BOARDS;
fibula below. injury compared with the medial collateral TOPNOTCH
B. There is injury to the knee ligament that is attached to ligament (option B). Option C simply refers to the MD)
the medial condyle of the femur above and to the medial posterior cruciate ligament, whose stability is
aspect of the tibial shaft below. tested by the posterior drawer test.
C. There is injury to the knee ligament that prevents
anterior displacement of the femur on the tibia.
D. There is injury to the knee ligament that prevents
posterior displacement of the femur on the tibia.

429 Which of the following is not true about the mucous Anal columns are found only in the mucous WEBSTER MIDTERM 1
membrane of the lower half of the anal canal? membrane of the UPPER half of the anal canal. The ALINDOG, EXAM - AUG
A. It is sensitive to pain, temperature, touch and pressure rest of the options are true about the lower anal MD (TOP 3 - 2014
(somatic sensory innervation). canal. It is the pectinate line that indicates the FEB 2014
B. It is thrown into vertical mucous folds called anal level where the two halves join together. MED
columns. BOARDS;
C. The lymph drains downward to the medial group of TOPNOTCH
superficial inguinal nodes. MD)
D. Its blood supply comes directly from the inferior rectal
artery, a branch of the internal pudendal artery.
E. None of these.

430 A normal individual will usually have how many parathyroid It is a "mortal sin" to get wrong in "give-away" WEBSTER MIDTERM 1
glands? questions like this (chances are, everybody will ALINDOG, EXAM - AUG
A. 2 answer them correctly so please always read MD (TOP 3 - 2014
B. 4 CAREFULLY even those questions with obvious FEB 2014
C. 6 answers). Parathyroid glands can be as numerous MED
D. 7 as 7 in a normal individual. BOARDS;
TOPNOTCH
MD)

431 Venous drainage of the posterior ventricular walls including WEBSTER MIDTERM 1
the posterior interventricular septum drain directly to: ALINDOG, EXAM - AUG
A. Great cardiac vein MD (TOP 3 - 2014
B. Smallest cardiac vein FEB 2014
C. Coronary sinus MED
D. Middle cardiac vein BOARDS;
TOPNOTCH
MD)

432 A 46-year old male was rushed to the ER after getting Focused assessment with sonography in trauma WEBSTER MIDTERM 1
involved in a vehicular accident. An anterior abdominal blunt (FAST) is a quick, reliable, non-invasive procedure ALINDOG, EXAM - AUG
injury was suspected. FAST was performed with focus on 4 that can facilitate a timely diagnosis for patients MD (TOP 3 - 2014
areas including the so-called Morrison's pouch. As member with blunt abdominal trauma. It includes views of FEB 2014
of the trauma team you know that this area corresponds to (1) the hepatorenal recess (Morison pouch), (2) MED
the: the perisplenic view, (3) the subxiphoid BOARDS;
A. Splenorenal space pericardial window, and (4) the suprapubic TOPNOTCH
B. Suprapubic space window (Douglas pouch). MD)
C. Hepatorenal space
D. Subxiphoid area

433 This is one of the five terminal nerves of the brachial plexus Must master the brachial (and lumbar) plexus. It is WEBSTER MIDTERM 1
which is also the motor innervation of the anterior formed in the posterior triangle of the neck by the ALINDOG, EXAM - AUG
compartment of the arm, important in flexing the elbow joint union of the anterior rami of C5 to C8 and T1. It MD (TOP 3 - 2014
and supinating the forearm: can be divided into roots, trunks, divisions and FEB 2014
A. Musculocutaneous nerve cords. It has 5 terminal branches namely, the MED
B. Median nerve musculocutaneous nerve (innervating the BOARDS;
C. Radial nerve anterior/flexor compartment of the arm); the TOPNOTCH
D. Ulnar nerve axillary nerve (deltoid and teres minor); the radial MD)
nerve (posterior/extensor compartment of arm
and forearm); the median nerve (anterior/flexor
compartment of the forearm except flexor carpi
ulnaris and flexor digitorum profundus, medial
half); and the ulnar nerve (intrinsic muscles of the
hand).
434 True of utricle and saccule except: The utricle and saccule constitute the static WEBSTER MIDTERM 1
A. Housed by a central cavity known as vestibule labyrinth which functions during the linear ALINDOG, EXAM - AUG
B. Kinetic labyrinth acceleration of the head and with the effects of the MD (TOP 3 - 2014
C. Important for linear acceleration pull of gravity. On the other hand, the semicircular FEB 2014
D. Contain otoliths canals make up the kinetic labyrinth which is MED
E. None of the above important in angular deceleration and BOARDS;
acceleration; it has no otoliths. TOPNOTCH
MD)

TOPNOTCH MEDICAL BOARD PREP ANATOMY SUPEREXAM Page 56 of 94


For inquiries visit www.topnotchboardprep.com.ph or email us at topnotchmedicalboardprep@gmail.com
TOPNOTCH MEDICAL BOARD PREP ANATOMY SUPEREXAM
For inquiries visit www.topnotchboardprep.com.ph or email us at topnotchmedicalboardprep@gmail.com
Item QUESTION EXPLANATION AUTHOR TOPNOTCH
# EXAM
435 Which of the following statements about seminal vesicle is It is the epididymis that can store mature WEBSTER MIDTERM 1
not true? spermatozoa. Structures that can be palpated ALINDOG, EXAM - AUG
A. They are lobulated structures that can store during DRE: rectovesical pouch, full bladder, MD (TOP 3 - 2014
spermatozoa. seminal vesicles, displaced or enlarged ductus FEB 2014
B. They produce secretions added to the seminal fluid and deferentes, membranous part of urethra when MED
important in sperm nourishment. catheterized, and bulbo-urethral glands; ischial BOARDS;
C. Their blood supply comes from the superior vesicle and tuberosity and spine and sacrotuberous ligament; TOPNOTCH
superior rectal arteries. pelvic surface of sacrum and coccyx. In females, MD)
D. They can be palpated during digital rectal exam. vagina, cervix, ostium uteri, body of uterus when
E. All of the above. retroverted, recto-uterine fossa, and,
pathologically, broad ligaments, uterine tubes, and
ovaries. Blood supply of the seminal vesicle comes
from the inferior vesicle and middle rectal
arteries.
436 A 15-year old skateboarder lost balance and fell on his This condition corresponds to a distal lesion of the WEBSTER MIDTERM 1
outstretched right hand causing hyperextension of the wrist median nerve (true also in carpal tunnel ALINDOG, EXAM - AUG
joint . He then manifest numbness and pain over the palmar syndrome) in which there is an altered cutaneous MD (TOP 3 - 2014
aspects of his right thumb, index and middle fingers. Which sensation on the lateral 3 and 1/2 digits of the FEB 2014
of the following mechanisms of injury will most likely hand; a patient may also exhibit weakness in MED
explain the symptoms? opposition of the thumb which remains adducted BOARDS;
A. A fracture of the base of the metacarpal bones and extended as a result, so called "ape" hand. A TOPNOTCH
B. A supracondylar fracture of the humerus with proximal lesion of the median hand meanwhile, MD)
compression of the pronator teres muscle results from supracondylar fracture of the
C. An oblique fracture of the trapezium bone humerus or from compression between the
D. A dislocated lunate bone pronator teres leading to the the condition "hand
of benediction" - with the index and middle fingers
remained extended when attempting to flex.

437 The inferior thyroid vein which receives tributaries from the WEBSTER MIDTERM 1
lower poles of the gland and the isthmus drains directly to: ALINDOG, EXAM - AUG
A. Internal jugular vein MD (TOP 3 - 2014
B. Brachiocephalic vein FEB 2014
C. External jugular vein MED
D. Superior vena cava BOARDS;
TOPNOTCH
MD)

438 Melanocytes, the pigment-producing cells, are most Buzz words: stratum corneum - dead keratinized WEBSTER MIDTERM 1
numerous in this layer of the skin: cells, protection; stratum lucidum - only found in ALINDOG, EXAM - AUG
A. Stratum lucidum thick skin (palm and soles); stratum granulosum - MD (TOP 3 - 2014
B. Stratum granulosum keratohyaline granules; stratum spinosum - FEB 2014
C. Stratum corneum Langerhan cells, SCCA; stratum basale - MED
D. Stratum basale melanocytes, Merkel cells, basal cell CA. BOARDS;
TOPNOTCH
MD)

439 Which of the following anatomic relationships will best WEBSTER MIDTERM 1
locate the gallbladder? ALINDOG, EXAM - AUG
A. It is medial to the ligamentum teres within the falciform MD (TOP 3 - 2014
ligament. FEB 2014
B. It lies superior to the porta hepatis MED
C. It is lateral to the quadrate lobe of the liver. BOARDS;
D. 2/3 of which lies in the left lobe of the liver. TOPNOTCH
MD)

440 In performing the ECG, the V5 lead should be placed over Must know: V1 - 4th ICS, right parasternal area; V2 WEBSTER MIDTERM 1
the: - 4th ICS, left para sternal area; V3 - space between ALINDOG, EXAM - AUG
A. 2nd right ICS, parasternal area V2 and V4; V4 - 5th ICS, left MCL; V5 - 5th ICS, left MD (TOP 3 - 2014
B. 4th left ICS, parasternal area AAL; V6 - 5th ICS, left MAL. FEB 2014
C. 4th left ICS, midclavicular area MED
D. 5th left ICS, lateral to the midclavicular area BOARDS;
TOPNOTCH
MD)
441 A 45 year-old male painter went up to a ladder to paint the Scarpa's or the membranous layer of JULIET MIDTERM 2
roof, however, he slips and fell suffering a straddle injury. superficial fascia forms a tubular sheath over KRISTINE EXAM - AUG
Which of the following structure prevents the spread of the penis and scrotum in males which prevents EVANGELIST 2014
urine inferiorly to the thigh in case of ruptured penile extravasation of urine inferiorly in cases of A, MD (TOP 9
urethra: urethral trauma. In the male, Camper’s fascia is - FEB 2014
A. rectus sheath continued over the penis and outer surface of the MED
B. camper's fascia spermatic cord to the scrotum, where it helps to BOARDS;
C. scarpa's fascia form the dartos. This layer is sufficiently complete TOPNOTCH
D. denonvilier's fascia that fluids escaping from a ruptured vessel or MD)
E. conjoint tendon urethra (blood and/or urine) may accumulate
deep to it.
442 Which is not included in the drainage of the thyroid: The main drainage of the thyroid gland are JULIET MIDTERM 2
A. Superior thyroid vein superior thyroid vein and middle thyroid vein KRISTINE EXAM - AUG
B. Middle thyroid vein draining into internal jugular vein. The inferior EVANGELIST 2014
C. Inferior thyroid vein thyroid vein which drains isthmus and lower poles A, MD (TOP 9
D. Brachiocephalic of the gland which drains into left brachiocephalic - FEB 2014
E. Thyroidea ima vein in the thorax. Thyroidea ima may arise MED
from the brachiocephalic artery or the arch of BOARDS;
the aorta. TOPNOTCH
MD)
TOPNOTCH MEDICAL BOARD PREP ANATOMY SUPEREXAM Page 57 of 94
For inquiries visit www.topnotchboardprep.com.ph or email us at topnotchmedicalboardprep@gmail.com
TOPNOTCH MEDICAL BOARD PREP ANATOMY SUPEREXAM
For inquiries visit www.topnotchboardprep.com.ph or email us at topnotchmedicalboardprep@gmail.com
Item QUESTION EXPLANATION AUTHOR TOPNOTCH
# EXAM
443 The non-ciliated secretory cells which randomly interrupt Clara cells are nonciliated, secretory bronchiolar JULIET MIDTERM 2
the ciliated epithelial lining of the bronchial mucosa are epithelial cells which function as stem cells for KRISTINE EXAM - AUG
called: repair in the bronchioles and can divide into EVANGELIST 2014
A. Kulchitsky cells ciliated or nonciliated bronchiolar cells. A, MD (TOP 9
B. Clara cells Enterochromaffin cells, or Kulchitsky cells, are a - FEB 2014
C. Neuroendocrine cells type of enteroendocrine and neuroendocrine cell MED
D. Type 2 pneumocytes occurring in the epithelial lining the lumen of the BOARDS;
E. Dust cells digestive tract and the respiratory tract. Type 2 TOPNOTCH
pneumocytes are cells secreting surfactant in the MD)
alveoli. Dust cells or alveolar macrophage are in
the lungs that reside on respiratory surfaces and
clean off particles such as dust or microorganisms.
444 A 52 year-old man was rushed to the ER after a car accident. The porous, fragile nature of the ethmoid bone JULIET MIDTERM 2
He was noted to have clear fluid draining from his nose makes it particularly susceptible to fractures. The KRISTINE EXAM - AUG
which is apparently the CSF. The bone which is most likely ethmoid is usually fractured from an upward force EVANGELIST 2014
fractured is the: to the nose. This could occur by hitting the A, MD (TOP 9
A. nasal dashboard in a car crash or landing on the ground - FEB 2014
B. ethmoid after a fall. The ethmoid fracture can produce bone MED
C. frontal fragments that penetrate the cribriform plate. This BOARDS;
D. lacrimal trauma can lead to a leak of cerebral spinal fluid TOPNOTCH
E. zygomatic into the nasal cavity. MD)

445 Which chamber of the heart is most likely enlarged when Left Atrial Enlargement pushes the esophagus JULIET MIDTERM 2
there is narrowing of the thoracic esophagus on Barium toward the spine (as seen on barium swallow) and KRISTINE EXAM - AUG
swallow: is the most sensitive indicator of LAE. EVANGELIST 2014
A. right atrium A, MD (TOP 9
B. right ventricle - FEB 2014
C. left atrium MED
D. left ventricle BOARDS;
E. left auricle TOPNOTCH
MD)

446 A 23 year-old male patient seen at a clinic cannot focus on The ciliary ganglion is a parasympathetic ganglion JULIET MIDTERM 2
near objects but he can move his eyeball normally and see located in the posterior orbit on the lateral side of KRISTINE EXAM - AUG
distant objects clearly. This condition may indicate damage the optic nerve. The postganglionic axons run in EVANGELIST 2014
to: the short ciliary nerves and innervate two eye A, MD (TOP 9
A. Short ciliary nerve and ciliary ganglion muscles including the sphincter pupillae which - FEB 2014
B. Ciliary ganglion and oculomotor nerve constricts the pupil, and the ciliary muscle which MED
C. Oculomotor nerve and long ciliary nerve contracts, releasing tension on the Zonular Fibers, BOARDS;
D. Short and long ciliary nerves and making the lens more convex, resulting to TOPNOTCH
E. Long ciliary nerve and superior cervical ganglion accommodation. MD)

447 A 48 year-old woman was diagnosed of Phyllodes tumor. She Long thoracic nerve supplies serratus anterior. JULIET MIDTERM 2
underwent mastectomy. However, postoperatively, the The nerve is most commonly injured as it courses KRISTINE EXAM - AUG
woman experiences weakness in the ability to protract the superficial to the serratus anterior causing EVANGELIST 2014
scapula with difficulty raising the arm above her head. weakness in the ability to protract the scapula and A, MD (TOP 9
Damage to which of the following nerves is suggested by this difficulty in raising the arm above the head. It also - FEB 2014
finding: causes winging of the scapula. MED
A. Axillary BOARDS;
B. Supraclavicular TOPNOTCH
C. Spinal accessory MD)
D. Long Thoracic
E. Thoracodorsal

448 A 55 year-old male sustained from a motor vehicular Although protected under the bony ribcage, the JULIET MIDTERM 2
accident resulting to fracture of the 9th to 11th ribs left spleen remains the most commonly affected organ KRISTINE EXAM - AUG
posterior. BP is 80/50mmHg, PR=125bpm, RR=22cpm. The in blunt injury to the abdomen in all age groups. EVANGELIST 2014
most likely injured organ is the While some references occasionally document A, MD (TOP 9
A. stomach liver injuries as being more common, blunt - FEB 2014
B. pancreas injuries to the spleen are documented more MED
C. liver frequently as the primary solid organ injury in the BOARDS;
D. spleen abdomen. These injuries are common as a result TOPNOTCH
E. small intestine from motor vehicle crashes, domestic violence, MD)
sporting events, and accidents involving bicycle
handlebars.
449 Massive hemopericardium compromises cardiac and Cardiac tamponade is a life-threatening condition JULIET MIDTERM 2
systemic circulation. In emergency evacuation of the blood, that requires prompt diagnosis and management. KRISTINE EXAM - AUG
the best and safest way to make an incision en route to the Anatomically, pericardiocentesis is carried out EVANGELIST 2014
pericardial activity is via: immediately under the xiphoid process A, MD (TOP 9
A. 5th left ICS lateral to sternum (infrasternal), up and leftwards. - FEB 2014
B. 7th left ICS immediately lateral to sternum MED
C. 6th left ICS 8cm from misdternal line BOARDS;
D. 7th left ICS 8cm from midsternal line TOPNOTCH
E. immediately subxiphoidal MD)

TOPNOTCH MEDICAL BOARD PREP ANATOMY SUPEREXAM Page 58 of 94


For inquiries visit www.topnotchboardprep.com.ph or email us at topnotchmedicalboardprep@gmail.com
TOPNOTCH MEDICAL BOARD PREP ANATOMY SUPEREXAM
For inquiries visit www.topnotchboardprep.com.ph or email us at topnotchmedicalboardprep@gmail.com
Item QUESTION EXPLANATION AUTHOR TOPNOTCH
# EXAM
450 A 65 year-old female, hypertensive, diabetic suddenly The lesion is located in Wernicke's area, which is JULIET MIDTERM 2
developed dizziness and subsequently loss consciousness. the posterior region of the left superior temporal KRISTINE EXAM - AUG
She was rushed to the emergency room, she regained gyrus or the first gyrus of the temporal lobe. EVANGELIST 2014
consciousness. She cannot understand or obey commands, Brodmann's areas 21 and 42 correspond to A, MD (TOP 9
she talks and answers to questions irrelevantly. If an infarct Wernicke's area. With Wernicke's aphasia there is - FEB 2014
is suspected, what Brodmann Area is affected: usually a severe impairment in auditory MED
A. Brodmann Area 4 comprehension. Speech, while fluent, is BOARDS;
B. Brodmann Area 44 and 45 semantically inappropriate and paraphasic. TOPNOTCH
C. Brodmann Area 8 Comprehension and expression tend to be equally MD)
D. Brodmann Area 21, 42 impaired.
E. Brodmann Area 3,1,2

451 In portal hypertension, which of the following veins will not The tributaries of portal vein are the splenic vein, JULIET MIDTERM 2
engorge and is not a collateral circulation: inferior mesenteric vein, superior mesenteric vein, KRISTINE EXAM - AUG
A. Superior rectal left gastric vein, right gastric vein, and cystic veins. EVANGELIST 2014
B. Median sacral Median sacral vein directly drains to left common A, MD (TOP 9
C. Esophageal iliac vein then to inferior vena cava. - FEB 2014
D. Paraumbilical MED
E. Splenic BOARDS;
TOPNOTCH
MD)
452 During a dilatation and curettage in a 23 year-old G1P0 The body of the uterus is related anteriorly to the JULIET MIDTERM 2
patient after an incomplete abortion at 5 weeks AOG, the uterovesical pouch and the superior surface of the KRISTINE EXAM - AUG
instrument accidentally punctured the uterus anteriorly bladder. Perforation is usually caused by a surgical EVANGELIST 2014
hitting this structure: instrument used for scraping and removing A, MD (TOP 9
A. Urinary bladder material from the uterus. The instrument - FEB 2014
B. Uterine artery penetrates through the uterine wall, and rarely, MED
C. Sigmoid colon may migrate into the abdominal cavity where the BOARDS;
D. Ureter bladder may also be perforated. TOPNOTCH
E. Broad ligament MD)

453 A 19 year-old male suffered from a stab wound at the The segment that is seen protruding the patient's JULIET MIDTERM 2
anterior abdomen and was rushed to the ER. A segment of abdomen is the ileum which is in the lower part of KRISTINE EXAM - AUG
the intestine protruded and partially opened. The segment's the cavity and in the pelvis. The ileum receives EVANGELIST 2014
mucosa had few circular folds. It's mesentery was fatty with numerous short terminal vessels that form many A, MD (TOP 9
many vascular arcades. The segment that is seen protruding vascular arcades. At the ileal end of the mesentery, - FEB 2014
the patient's abdomen is: the fat is deposited throughout. MED
A. duodenal bulb BOARDS;
B. distal part of duodenum TOPNOTCH
C. proximal jejunum MD)
D. distal jejunum
E. distal ileum

454 A 12 year-old boy was brought to the ER because of acute During an asthmatic event the muscles JULIET MIDTERM 2
attack of Bronchial Asthma. Symptoms primarily are caused surrounding the bronchioles constrict. The KRISTINE EXAM - AUG
by mucosal inflammation and airway hyperresponsiveness. wheezing sound is caused by the contraction of EVANGELIST 2014
The specific part of the airway involved in the pathogenesis the bronchioles as the air passes through tubes A, MD (TOP 9
of Asthma is: that are almost completely blocked. - FEB 2014
A. trachea MED
B. bronchi BOARDS;
C. bronchioles TOPNOTCH
D. alveolar ducts MD)
E. alveolar sacs

455 A 48 year-old fish vendor was stabbed by an unknown Macula densa cells sense changes in sodium JULIET MIDTERM 2
assailant at the back. He was rushed to the ER with BP 60 chloride level, and will trigger an autoregulatory KRISTINE EXAM - AUG
palpatory, PR=130s, RR=20s. Which particular structure of response to increase or decrease reabsorption of EVANGELIST 2014
the kidney will be stimulated as a result of the above ions and water to the blood in order to alter blood A, MD (TOP 9
findings: volume and return blood pressure to normal. - FEB 2014
A. juxtaglomerular cells Decreased BP is detected initially by Macula MED
B. cells of PCT Densa which results to increased renin release BOARDS;
C. cells of loop of Henle from JG cells. TOPNOTCH
D. macula densa MD)
E. mesangial cells

456 Direct inguinal hernias are found within the Hesselbach's Direct inguinal hernia protrudes through the JULIET MIDTERM 2
triangle. The posterior wall of this triangle where hernias inguinal triangle of Hesselbach that lies between KRISTINE EXAM - AUG
protrude is: the inferior epigastric artery superolaterally, the EVANGELIST 2014
A. Inguinal ligament rectus abdominis medially and the inguinal A, MD (TOP 9
B. External oblique aponeurosis ligament inferiorly. The posterior wall of this - FEB 2014
C. Transversus abdominis aponeurosis triangle where hernias protrude is formed by the MED
D. Internal oblique aponeurosis transversalis fascia. The transversalis fascia forms BOARDS;
E. Transversalis fascia an investing fascial envelope of the abdominal TOPNOTCH
cavity. MD)

TOPNOTCH MEDICAL BOARD PREP ANATOMY SUPEREXAM Page 59 of 94


For inquiries visit www.topnotchboardprep.com.ph or email us at topnotchmedicalboardprep@gmail.com
TOPNOTCH MEDICAL BOARD PREP ANATOMY SUPEREXAM
For inquiries visit www.topnotchboardprep.com.ph or email us at topnotchmedicalboardprep@gmail.com
Item QUESTION EXPLANATION AUTHOR TOPNOTCH
# EXAM
457 A 35 year-old woman has an infected right big toe with A patient may present with an enlarged, painful JULIET MIDTERM 2
swollen lymph nodes. The group of node that is most likely superficial inguinal lymph node that is due to KRISTINE EXAM - AUG
affected is: lymphatic spread of pathogenic organisms that EVANGELIST 2014
A. Femoral entered the body through the legs and big toes. A, MD (TOP 9
B. Superficial inguinal Femoral lymph nodes are ocated in the upper - FEB 2014
C. Deep inguinal thigh portion along the femoral veins, immediately MED
D. Internal iliac below the inguinal lymph nodes which drain from BOARDS;
E. Paravertebral some of the genital parts, buttock, thigh and the TOPNOTCH
medial side of the leg. MD)

458 A 46 year-old woman had crow's feet at lateral angles of her Contraction of the orbicularis oculi muscle is JULIET MIDTERM 2
eyelids. These are due to contraction of: primarily responsible for the clinically observed KRISTINE EXAM - AUG
A. Procerus periorbital crow's feet. The orbicularis oculi EVANGELIST 2014
B. Corrugator supercilli muscle is bordered superolaterally by fibers of the A, MD (TOP 9
C. Frontalis frontalis muscle and medially by the levator - FEB 2014
D. Orbicularis oculi palpebrae muscle. The frontalis muscle and the MED
E. Levator palpebrae superioris corrugator muscle are responsible for the BOARDS;
appearance of wrinkles and vertical frown lines TOPNOTCH
accordingly. The procerus muscle, which overlies MD)
the nasal root, is responsible for a snout-nose
appearance and horizontal frown lines.
459 A 35 year-old man complains of inability to flex the elbow Musculocutaneous nerve supplies the flexor of the JULIET MIDTERM 2
joint. It involves a nerve supplying the muscle which arm which branches from the brachial plexus as KRISTINE EXAM - AUG
branches from the brachial plexus as: C5-C7. C5- Dorsal scapular Nerve, C5-C6- EVANGELIST 2014
A. C5 suprascapular nerve, C6,C7,C8-thoracodorsal A, MD (TOP 9
B. C5-C6 nerve, C8-T1-Ulnar nerve and medial root of - FEB 2014
C. C5-C7 median. MED
D. C6, C7, C8 BOARDS;
E. C8-T1 TOPNOTCH
MD)

460 Pulsation from an artery which is a continuation of anterior Dorsalis pedis artery is a continuation of anterior JULIET MIDTERM 2
tibial artery on the dorsum of the foot can be easily felt. On tibial artery. Medial to this lies the tendon of KRISTINE EXAM - AUG
the medial side of this artery lies the tendon of: extensor hallucis longus. On the lateral side lies EVANGELIST 2014
A. Extensor digitorum brevis the terminal part of the deep peroneal nerve and A, MD (TOP 9
B. Extensor digitorum longus extensor digitorum longus tendons. - FEB 2014
C. Extensor hallusis brevis MED
D. Extensor hallucis longus BOARDS;
E. Inferior extensor retinaculum TOPNOTCH
MD)

461 What is the vein that is used in arterio-venous fistula that is cephalic vein is lateral to the biceps brachii, while LUISA BACK-UP
located lateral to the biceps brachii? basilic vein is medial to the biceps brachii. SARANILLO, MIDTERM
A. Basilic vein MD (TOP 6 - EXAM AUG
B. brachial vein FEB 2014 2014 - FOR
C. Median cubital vein MED INCLUSION
D. Cephalic vein BOARDS; IN THE
E. Axillary vein TOPNOTCH SAMPLEX
MD)

462 In the standard 12 lead ECG what is the anatomic placement 4th ICS Right parasternal border = V1: 4th ICS Left LUISA BACK-UP
of the chest electrode lead V2? parasternal border = V2; 5th ICS Left MCL = V4; SARANILLO, MIDTERM
A. 4th ICS Right parasternal border 5th ICS Right MCL= none MD (TOP 6 - EXAM AUG
B. 4th ICS Left parasternal border FEB 2014 2014 - FOR
C. 5th ICS Right MCL MED INCLUSION
D. 5th ICS Left MCL BOARDS; IN THE
E. None of the choices TOPNOTCH SAMPLEX
MD)

463 A 55 year old male presents with chest pain. ECG shows ST Leads II, III, AVF refers the inferior wall, which is LUISA BACK-UP
segment elevation in lead II, III, AVF. What is the most likely supplied by the Right coronary artery. Septal, SARANILLO, MIDTERM
affected blood vessel supplying the affected part of the anterior, and lateral wall are supplied by the left MD (TOP 6 - EXAM AUG
heart? coronary artery. FEB 2014 2014 - FOR
A. Right coronary artery MED INCLUSION
B. Left marginal artery BOARDS; IN THE
C. left anterior descending artery TOPNOTCH SAMPLEX
D. Left circumflex artery MD)
E. C and D

464 What is the most common type of myoma? Intramural is the most common type, while LUISA BACK-UP
A. Submucosal myoma submucosal is the one frequently associated with SARANILLO, MIDTERM
B. Pedunculated subserosal myoma heavy, prolonged bleeding. MD (TOP 6 - EXAM AUG
C. Subserosal myoma FEB 2014 2014 - FOR
D. Intramural myoma MED INCLUSION
E. Pedunculated submucosal myoma BOARDS; IN THE
TOPNOTCH SAMPLEX
MD)
465 The following are muscles of inspiration except: Internal intercostal depresses the ribs during LUISA BACK-UP
A. Serratus posterior superior exhalation. Other choices elevate the ribs during SARANILLO, MIDTERM
B. External intercostal inspiration. MD (TOP 6 - EXAM AUG
C. Internal intercostal FEB 2014 2014 - FOR
D. Innermost intercostal MED INCLUSION
E. Subcostal BOARDS; IN THE
TOPNOTCH SAMPLEX
MD)
TOPNOTCH MEDICAL BOARD PREP ANATOMY SUPEREXAM Page 60 of 94
For inquiries visit www.topnotchboardprep.com.ph or email us at topnotchmedicalboardprep@gmail.com
TOPNOTCH MEDICAL BOARD PREP ANATOMY SUPEREXAM
For inquiries visit www.topnotchboardprep.com.ph or email us at topnotchmedicalboardprep@gmail.com
Item QUESTION EXPLANATION AUTHOR TOPNOTCH
# EXAM
466 A 25 year old male came in due a stab wound in his The muscular tube from the kidney to the LUISA BACK-UP
abdomen. Upon exploration you noticed a severed blood posterior surface of the bladder is the ureter. It SARANILLO, MIDTERM
vessel supplying the lower part of the the muscular tube has 3 blood supplies: upper part is supplied by MD (TOP 6 - EXAM AUG
from the kidney to the posterior surface of the bladder. What renal artery, the middle part is supplied by the FEB 2014 2014 - FOR
blood vessel is most likely involved? gonadal artery, the lower part is supplied by the MED INCLUSION
A. renal artery superior vesical artery. BOARDS; IN THE
B. testicular artery TOPNOTCH SAMPLEX
C. superior vesical artery MD)
D. iliolumbar artery
E. inferior vesical artery

467 A 65 year old female fall from a height of approximately 10 Fracture of the femoral neck will present with LUISA BACK-UP
step ladder sustaining injuries. Her right leg is shortened and shortened leg and laterally rotated thigh while SARANILLO, MIDTERM
the thigh is laterally rotated. What is the most likely dislocation of the femoral head will present with MD (TOP 6 - EXAM AUG
diagnosis. shortened leg and medially rotated thigh. FEB 2014 2014 - FOR
A. Fracture of the neck of the femur, right MED INCLUSION
B. fracture of the tibia, right BOARDS; IN THE
C. dislocation of the head of the femur, right TOPNOTCH SAMPLEX
D. fracture of the fibula, right MD)
E. none of the choices

468 A surgeon harvested part of the great saphenous vein from Saphenous nerve supplies the skin of the medial LUISA BACK-UP
the right lower extremity of a 30 year old patient. However, aspect of the leg and foot. Obturator nerve SARANILLO, MIDTERM
after the procedure, the patient complained of pain and supplies the skin of the medial thigh. Femoral MD (TOP 6 - EXAM AUG
paresthesia in the medial aspect of the leg and foot. What nerve supplies the antero-medial aspect of the FEB 2014 2014 - FOR
nerve is the most likely affected. thigh. Sciatic nerve supplies the posterior thigh, MED INCLUSION
A. saphenous nerve foot and leg. BOARDS; IN THE
B. obturator nerve TOPNOTCH SAMPLEX
C. femoral nerve MD)
D. sciatic nerve
E. A and B

469 A 6 month old infant has a long and narrow skull with frontal Premature closure of the sagittal suture called LUISA BACK-UP
and occipital expansion was brought in to your clinic by her scaphocephaly presents with long and narrow SARANILLO, MIDTERM
mother for consultation. You explained to the mother that skull with frontal and occipital expansion; MD (TOP 6 - EXAM AUG
the most likely reason for this is the premature closure of the involvement of the coronal suture forms a tower FEB 2014 2014 - FOR
cranial sutures. What suture is involved in this case? skull - a short high skull; premature closure of the MED INCLUSION
A. sagittal suture coronal and lambdoid suture on one side of the BOARDS; IN THE
B. coronal suture skull called plagiocephaly presents with TOPNOTCH SAMPLEX
C. lambdoid suture assymetric skull. MD)
D. coronal and lambdoid suture
E. coronal and sagittal suture

470 The following cranial nerves are both motor and sensory Mixed cranial nerves are vagus, glossopharyngeal, LUISA BACK-UP
except: facial, and trigeminal nerve. Vestibulocochlear SARANILLO, MIDTERM
A. Glossopharyngeal nerve is purely sensory nerve. MD (TOP 6 - EXAM AUG
B. Vagus FEB 2014 2014 - FOR
C. facial MED INCLUSION
D. vestibulocochlear BOARDS; IN THE
E. No exception TOPNOTCH SAMPLEX
MD)

471 A 35 year old male sustained facial laceration due to Facial lacerations tend to gape because the face LUISA BACK-UP
vehicular accident. The wound tend to gape because: has no deep fascia and the subcutaneous tissue is SARANILLO, MIDTERM
A. The face has no deep fascia loose. MD (TOP 6 - EXAM AUG
B. The subcutaneous tissue of the face is loose FEB 2014 2014 - FOR
C. The face has weak muscles MED INCLUSION
D. A and B only BOARDS; IN THE
E. All of the choices TOPNOTCH SAMPLEX
MD)

472 The anal canal is divided into 2 parts by the dentate line. LUISA BACK-UP
What is the lining epithelium of the anal canal above the SARANILLO, MIDTERM
dentate line? MD (TOP 6 - EXAM AUG
A. Simple columnar epithelium FEB 2014 2014 - FOR
B. Simple cuboidal epithelium MED INCLUSION
C. Stratified squamous epithelium BOARDS; IN THE
D. transitional epithelium TOPNOTCH SAMPLEX
E. simple squamous epithelium MD)

TOPNOTCH MEDICAL BOARD PREP ANATOMY SUPEREXAM Page 61 of 94


For inquiries visit www.topnotchboardprep.com.ph or email us at topnotchmedicalboardprep@gmail.com
TOPNOTCH MEDICAL BOARD PREP ANATOMY SUPEREXAM
For inquiries visit www.topnotchboardprep.com.ph or email us at topnotchmedicalboardprep@gmail.com
Item QUESTION EXPLANATION AUTHOR TOPNOTCH
# EXAM
473 A 35 year old female singer underwent a subtotal Superior laryngeal nerve supplies the cricothyroid LUISA BACK-UP
thyroidectomy due to a thyroid mass. After the surgery, she muscle which act as a tensor. Inferior laryngeal SARANILLO, MIDTERM
experienced mild hoarseness of voice and she can not reach nerve supplies the rest of the laryngeal muscles. MD (TOP 6 - EXAM AUG
high notes. What nerve is most likely injured? FEB 2014 2014 - FOR
A. lateral laryngeal nerve MED INCLUSION
B. superior laryngeal nerve BOARDS; IN THE
C. inferior laryngeal nerve TOPNOTCH SAMPLEX
D. phrenic nerve MD)
E. glossopharyngeal nerve

474 In pericardiocentesis, pericardial fluid is apirated between Accumulated pericardial fluid is aspirated LUISA BACK-UP
what layers of the heart? between the parietal layer of the serous SARANILLO, MIDTERM
A. Between fibrous and serous pericardium pericardium and the visceral layer of the MD (TOP 6 - EXAM AUG
B. Between the parietal layer of the serous pericardium and pericardium. FEB 2014 2014 - FOR
the fibrous pericardium MED INCLUSION
C. Between the myocardium and the epicardium BOARDS; IN THE
D. between the parietal layer of the serous pericardium and TOPNOTCH SAMPLEX
the visceral layer of the serous pericardium MD)
E. none of the choices

475 When inserting an NGT, the first esophageal narrowing From the external nares the following are the LUISA BACK-UP
encountered is between the esophagus and the cricoid approximated distance: up to the first esophageal SARANILLO, MIDTERM
cartilage. What is the distance from the external nares up to narrowing which is between the esophagus and MD (TOP 6 - EXAM AUG
this point. the cricoid cartilage is 18cm; up to the 2nd FEB 2014 2014 - FOR
A. 30cm esophageal narrowing between the esophagus and MED INCLUSION
B. 28cm the arch of aorta is 28cm; up to the cardiac orifice BOARDS; IN THE
C. 44 cm of the stomach is 44cm. An additional 12-14cm is TOPNOTCH SAMPLEX
D. 12-14cm added when pushed up to the pylorus. MD)
E. 18cm

476 The blood supply of the appendix comes directly from: LUISA BACK-UP
A. Posterior cecal artery SARANILLO, MIDTERM
B. Ileocolic artery MD (TOP 6 - EXAM AUG
C. Right middle colic artery FEB 2014 2014 - FOR
D. Left middle colic artery MED INCLUSION
E. Jejunoileal artery BOARDS; IN THE
TOPNOTCH SAMPLEX
MD)
477 Which of the following best characterized the parathyroid Usually, there are 4 parathyroid glands. Inferior LUISA BACK-UP
glands? thyroid artery supplies both the superior and SARANILLO, MIDTERM
A. The inferior parathyroid glands are more variable in inferior parathyroid glands. The superior glands MD (TOP 6 - EXAM AUG
location are constant in location, while the inferior glands FEB 2014 2014 - FOR
B. Inferior thyroid artery supplies the inferior parathyroid are more variable in location. MED INCLUSION
gland only BOARDS; IN THE
C. Usually, there are 4 parathyroid glands TOPNOTCH SAMPLEX
D. All of the above MD)
E. A and C only

478 An 18 year old boy had anterior dislocation of the shoulder axillary nerve is injured in anterior dislocation of LUISA BACK-UP
joint while playing basketball. An injury to this nerve is the shoulder joint. It will presents with inability to SARANILLO, MIDTERM
suspected which will present as: abduct the arm, altered sensation in the lateral MD (TOP 6 - EXAM AUG
A. Inability to abduct the arm arm, and weakness in lateral rotation of the arm. FEB 2014 2014 - FOR
B. Inability to adduct the arm MED INCLUSION
C. Weakness in medial rotation of the arm BOARDS; IN THE
D. altered sensation in the medial aspect of the arm TOPNOTCH SAMPLEX
E. all of the above MD)

479 The "yes" and "no" joints are synovial joints without The "yes" joint is the atlanto-occipital joint located LUISA BACK-UP
intervertebral disc. The "yes" joint is located between: between the atlas and occipital condyle, while the SARANILLO, MIDTERM
A. Atlas and occipital condyle "no" joint is the atlanto-axial joint which is MD (TOP 6 - EXAM AUG
B. Atlas and axis between atlas and axis. FEB 2014 2014 - FOR
C. Axis and occipital condyle MED INCLUSION
D. C3 and C4 BOARDS; IN THE
E. None of the choices TOPNOTCH SAMPLEX
MD)

480 What is the anatomic landmark of the cricoid cartilage? C4 =hyoid bone and Common carotid artery LUISA BACK-UP
A. C4 bifurcation. C5= thyroid cartilage. C6 = cricoid SARANILLO, MIDTERM
B. C5 cartilage, start of trachea and esophagus. T2 = MD (TOP 6 - EXAM AUG
C. C6 sternal notch. FEB 2014 2014 - FOR
D. T2 MED INCLUSION
E. C7 BOARDS; IN THE
TOPNOTCH SAMPLEX
MD)

TOPNOTCH MEDICAL BOARD PREP ANATOMY SUPEREXAM Page 62 of 94


For inquiries visit www.topnotchboardprep.com.ph or email us at topnotchmedicalboardprep@gmail.com
TOPNOTCH MEDICAL BOARD PREP ANATOMY SUPEREXAM
For inquiries visit www.topnotchboardprep.com.ph or email us at topnotchmedicalboardprep@gmail.com
Item QUESTION EXPLANATION AUTHOR TOPNOTCH
# EXAM
481 Which of the following cranial nerves will NOT be affected in CN III, IV, V1, V2 and VI and postganglionic fibers ANGELIS FINAL EXAM
cavernous sinus syndrome? en route to the orbit all pass through the ANDREA - AUG 2014
A. CN II cavernous sinus. COCOS, MD
B. CN III (TOP 1 - FEB
C. CN IV 2014 MED
D. CN VI BOARDS;
TOPNOTCH
MD)
482 This type of glia is important for the maintenance of the The astrocyte is important for physical support, ANGELIS FINAL EXAM
blood-brain barrier: repair, K metabolism, removal of excess ANDREA - AUG 2014
A. oligodendroglia neurotransmitters and maintenance of blood- COCOS, MD
B. astrocyte brain barrier. (TOP 1 - FEB
C. microglia 2014 MED
D. Schwann cell BOARDS;
TOPNOTCH
MD)
483 The V4 precordial lead is placed: Several questions on ECG lead placement were ANGELIS FINAL EXAM
A. On the 4th ICS to the left of the sternum asked during our board exam. ANDREA - AUG 2014
B. On the 5th ICS to the left of the sternum COCOS, MD
C. On the 5th ICS at the left midclavicular line (TOP 1 - FEB
D. On the 4th ICS at the left midclavicular line 2014 MED
BOARDS;
TOPNOTCH
MD)
484 This is the most common site of ureteral obstruction: The rest are common sites of obstruction as well. ANGELIS FINAL EXAM
A. Ureteropelvic junction ANDREA - AUG 2014
B. Ureterovesical junction COCOS, MD
C. Pelvic brim (TOP 1 - FEB
D. None of the above 2014 MED
BOARDS;
TOPNOTCH
MD)
485 Gastrulation happens within week: Gastrulation is the phase during which the single- ANGELIS FINAL EXAM
A. 1 layered blastula is reorganized into a trilaminar ANDREA - AUG 2014
B. 2 disc. It happens during week 3. COCOS, MD
C. 3 (TOP 1 - FEB
D. 4 2014 MED
BOARDS;
TOPNOTCH
MD)
486 The following are embryologic derivatives of ectoderm, Eustachian tube is an endodermal derivative. ANGELIS FINAL EXAM
EXCEPT: ANDREA - AUG 2014
A. Parotid gland COCOS, MD
B. Retina (TOP 1 - FEB
C. Melanocytes 2014 MED
D. Eustachian tube BOARDS;
TOPNOTCH
MD)
487 What is the function of Type 2 pneumocytes? Type 2 cells are also the ones which secrete the ANGELIS FINAL EXAM
A. For gas exchange pulmonary surfactant. ANDREA - AUG 2014
B. Degrade toxins COCOS, MD
C. Secrete IgA (TOP 1 - FEB
D. Precursors to type I pneumocytes 2014 MED
BOARDS;
TOPNOTCH
MD)
488 A young fetus of 2 months AOG would be expected to be Erythropoiesis happens in: yolksack for the first 2 ANGELIS FINAL EXAM
producing RBCs via the: months AOG, liver and spleen around 3 months, ANDREA - AUG 2014
A. Yolk sac and bone marrow predominantly from 5 months COCOS, MD
B. Liver up to birth. (TOP 1 - FEB
C. Bone marrow 2014 MED
D. Thymus BOARDS;
TOPNOTCH
MD)
489 The left circumflex coronary artery supplies the: SIMILAR TO PREVIOUS BOARD EXAM ANGELIS FINAL EXAM
A. Anterior 2/3 of interventricular septum CONCEPT/PRINCIPLE. LCX supplies the lateral ANDREA - AUG 2014
B. Posterior walls of the left ventricle and posterior walls of the left ventricle. COCOS, MD
C. Posterior 1/3 of interventricular septum (TOP 1 - FEB
D. Lateral wall of the right ventricle 2014 MED
BOARDS;
TOPNOTCH
MD)
490 Which of the following hormones is secreted by an acidophil? Acidophils secrete GH and prolactin. ANGELIS FINAL EXAM
A. FSH ANDREA - AUG 2014
B. GH COCOS, MD
C. TSH (TOP 1 - FEB
D. LH 2014 MED
BOARDS;
TOPNOTCH
MD)

TOPNOTCH MEDICAL BOARD PREP ANATOMY SUPEREXAM Page 63 of 94


For inquiries visit www.topnotchboardprep.com.ph or email us at topnotchmedicalboardprep@gmail.com
TOPNOTCH MEDICAL BOARD PREP ANATOMY SUPEREXAM
For inquiries visit www.topnotchboardprep.com.ph or email us at topnotchmedicalboardprep@gmail.com
Item QUESTION EXPLANATION AUTHOR TOPNOTCH
# EXAM
491 This ligament contains the portal triad: The hepatoduodenal ligament connects the liver to ANGELIS FINAL EXAM
A. gastrohepatic the duodenum. It contains the portal triad: hepatic ANDREA - AUG 2014
B. hepatosplenic artery, portal vein, and common bile duct. COCOS, MD
C. hepatoduodenal (TOP 1 - FEB
D. falciform 2014 MED
BOARDS;
TOPNOTCH
MD)
492 This/these segment/s of the small intestine contain/s the Brunner's glands are found in the submucosa of ANGELIS FINAL EXAM
Brunner's glands: the duodenum. Several histology questions were ANDREA - AUG 2014
A. duodenum asked during our board exam. COCOS, MD
B. jejunum (TOP 1 - FEB
C. ileum 2014 MED
D. Both A and B BOARDS;
TOPNOTCH
MD)
493 On peripheral blood smear, which cell presents with a large SIMILAR TO PREVIOUS BOARD EXAM ANGELIS FINAL EXAM
kidney-shaped nucleus? CONCEPT/PRINCIPLE. ANDREA - AUG 2014
A. Eosinophil COCOS, MD
B. Monocyte (TOP 1 - FEB
C. B cell 2014 MED
D. T cell BOARDS;
TOPNOTCH
MD)
494 Melanocytes are found in the: Stratum basale is sometimes referred to as ANGELIS FINAL EXAM
A. Stratum basale stratum germinativum, the deepest layer of the ANDREA - AUG 2014
B. Stratum germinativum epidermis. SIMILAR TO PREVIOUS BOARD EXAM COCOS, MD
C. Statum granulosum CONCEPT/PRINCIPLE. (TOP 1 - FEB
D. Both A and B 2014 MED
BOARDS;
TOPNOTCH
MD)
495 The most commonly injured rotator cuff muscle is: Teres major is not part of the rotator cuff muscles. ANGELIS FINAL EXAM
A. Teres minor ANDREA - AUG 2014
B. Teres major COCOS, MD
C. Supraspinatus (TOP 1 - FEB
D. Infraspinatus 2014 MED
BOARDS;
TOPNOTCH
MD)
496 A patient needs a femoral tap for ABG determination. What The acronym from lateral to medial is NAVEL: ANGELIS FINAL EXAM
compartment of the femoral sheath should you aspirate? nerve, artery, vein, empty space and lymphatics. ANDREA - AUG 2014
A. medial The nerve is not included in the femoral sheath. COCOS, MD
B. lateral (TOP 1 - FEB
C. posterior 2014 MED
D. anterior BOARDS;
TOPNOTCH
MD)
497 A woman suffered a traumatic injury to the upper limb which Clawing of the ring and little fingers is a keyphrase ANGELIS FINAL EXAM
results to an inability to spread and extend her fingers, with for lower segment brachial plexus injury. A ANDREA - AUG 2014
noted clawing of the ring and little fingers. Which segments question on brachial plexus was asked during our COCOS, MD
of the brachial plexus would have contributed to the nerve exam. (TOP 1 - FEB
that is damaged? 2014 MED
A. C5 and C6 BOARDS;
B. C6 and C7 TOPNOTCH
C. C7 and C8 MD)
D. C8 and T1

498 A patient suffers an Achilles tendon rupture. Which of the The popliteal tendon does not contribute to the ANGELIS FINAL EXAM
following muscles is NOT affected: formation of the calcaneal or Achilles tendon. ANDREA - AUG 2014
A. soleus COCOS, MD
B. plantaris (TOP 1 - FEB
C. popliteus 2014 MED
D. gastrocnemius BOARDS;
TOPNOTCH
MD)
499 During parotidectomy, the main trunk of the facial nerve was Buccinator is innervated by the facial nerve. The ANGELIS FINAL EXAM
accidentally lacerated. Which of the following muscles will be rest of the choices are innervated by the ANDREA - AUG 2014
affected? trigeminal nerve, specifically V3. COCOS, MD
A. masseter (TOP 1 - FEB
B. buccinator 2014 MED
C. temporalis BOARDS;
D. pterygoids TOPNOTCH
MD)

500 The nasolacrimal duct drains into which recess? SIMILAR TO PREVIOUS BOARD EXAM ANGELIS FINAL EXAM
A. Superior meatus CONCEPT/PRINCIPLE. There is no lateral meatus. ANDREA - AUG 2014
B. Middle meatus COCOS, MD
C. Inferior meatus (TOP 1 - FEB
D. Lateral meatus 2014 MED
BOARDS;
TOPNOTCH
MD)

TOPNOTCH MEDICAL BOARD PREP ANATOMY SUPEREXAM Page 64 of 94


For inquiries visit www.topnotchboardprep.com.ph or email us at topnotchmedicalboardprep@gmail.com
TOPNOTCH MEDICAL BOARD PREP ANATOMY SUPEREXAM
For inquiries visit www.topnotchboardprep.com.ph or email us at topnotchmedicalboardprep@gmail.com
Item QUESTION EXPLANATION AUTHOR TOPNOTCH
# EXAM
501 This nuclei is the most important source of norepinephrine SIMILAR TO PREVIOUS BOARD EXAM JAN BACK-UP
in the cerebral cortex: CONCEPT/PRINCIPLE.
CHARMAINE MIDTERM
A. Caudate nucleus Caudate nucleus-GABA PALOMAR, EXAM AUG
B. Substantia nigra pars compacta Substantia nigra pars compacta-dopamine MD (TOP 9 - 2014
C. Raphe nucleus Raphe nucleus-serotonin FEB 2014
D. Locus ceruleus Locus ceruleus-norepinephrine MED
E. Basal nucleus of Meynert Basal nucleus of Meynert-acetylcholine BOARDS;
TOPNOTCH
MD)
502 A 30-year-old man was allegedly stabbed in his right chest by SIMILAR TO PREVIOUS BOARD EXAM JAN BACK-UP
an unknown assailant as he was going to his car parked 2 CONCEPT/PRINCIPLE. The long thoracic nerve CHARMAINE MIDTERM
blocks away from the bar where he sings.On physical innervates the serratus anterior muscle which is PALOMAR, EXAM AUG
examination, patient is unable to raise his right arm above responsible for drawing the scapula forward MD (TOP 9 - 2014
the horizontal with winging of his right scapula. Which of the around the thoracic wall. A winged scapula is FEB 2014
following nerves is most likely affected? caused by paralysis of the serratus anterior MED
A. Axillary muscle.Snells 8th ed., 441, 434. BOARDS;
B. Thoracodorsal TOPNOTCH
C. Long thoracic MD)
D. Internal intercosta
E. Spinal accesory

503 This is also known as the morrison's pouch: SIMILAR TO PREVIOUS BOARD EXAM JAN BACK-UP
A. Hepatorenal recess CONCEPT/PRINCIPLE. Morrison's pouch- CHARMAINE MIDTERM
B. Splenorenal recess hepatorenal recess; the most posterior cavity in PALOMAR, EXAM AUG
C. Renocolic recess the peritoneal cavity. -Medical eponyms by MD (TOP 9 - 2014
D. Costodiagphragmatic recess Andrew J. Yee, MD, 2007 FEB 2014
MED
BOARDS;
TOPNOTCH
MD)
504 The following muscles are responsible for dorsiflexing the SIMILAR TO PREVIOUS BOARD EXAM JAN BACK-UP
foot, except: CONCEPT/PRINCIPLE. Anterior leg muscles- CHARMAINE MIDTERM
A. Extensor digitorum brevis action: dorsiflexion and extension, nerve: deep PALOMAR, EXAM AUG
B. Tibialis anterior peroneal nerve, muscles: tibialis anterior, extensor MD (TOP 9 - 2014
C. Peroneus longus digitorum longus, peroneus tertius, extensor FEB 2014
D. Extencsor digitorum longus hallucis longus, extensor digitorum brevis. - MED
E. None of the above Topnotch handouts. BOARDS;
Choice C, peroneous longus is found in the lateral TOPNOTCH
compartment of the leg and is responsible for MD)
plantar flexion and evertion
505 The gastric mucosa is lined by which epithelium: SIMILAR TO PREVIOUS BOARD EXAM JAN BACK-UP
A. Simple squamous CONCEPT/PRINCIPLE. -Junquiera and Carneiro, CHARMAINE MIDTERM
B. Simple cuboidal Basic Histology 11th ed., p. 290. PALOMAR, EXAM AUG
C. Simple columnar MD (TOP 9 - 2014
D. Stratified squamous FEB 2014
E. Stratified columnar MED
BOARDS;
TOPNOTCH
MD)
506 Melanocytes are found in which layer of the skin: SIMILAR TO PREVIOUS BOARD EXAM JAN BACK-UP
A. Stratum germinativum CONCEPT/PRINCIPLE. Eumelanin is a dark brown CHARMAINE MIDTERM
B. Stratum granulosum pigment produced by the melanocyte, a PALOMAR, EXAM AUG
C. Stratum corneum specialized cell of the epidermis found beneath or MD (TOP 9 - 2014
D. Stratum basale between the cells of the stratum basale and the FEB 2014
E. Stratum spinosum hair follicles. Melanocytes have rounded bodies MED
from which long irregular extensions branch into BOARDS;
the epidermis, running between the cells of the TOPNOTCH
strata basale and spinosum. -Junquiera and MD)
Carneiro, Basic Histology 11th ed., p. 363
507 The major mechanism of blindness in glaucoma is: SIMILAR TO PREVIOUS BOARD EXAM JAN BACK-UP
A. Optic nerve damage CONCEPT/PRINCIPLE. Vaughan and Ausbury's CHARMAINE MIDTERM
B. Increased intraocular pressure General Ophthalmology 17th ed., p. 214-215. PALOMAR, EXAM AUG
C. Optic disk enlargement MD (TOP 9 - 2014
D. Corneal inflammation FEB 2014
E. Lens opacification MED
BOARDS;
TOPNOTCH
MD)
508 Which of the folling statements regarding tube thoracostomy B. The skin incision is usually made over the JAN BACK-UP
is true? A. intercostal space below the space to be pierced. CHARMAINE MIDTERM
The site of insertion of the tube is at the 4th ICS at the C. The incision through the intercostal space is PALOMAR, EXAM AUG
anterior axillary line. kept close to the upper border of the rib to avoid MD (TOP 9 - 2014
B. The skin incision is usually made over the intercostal injuring the intercostal vessels and nerve. Clinical FEB 2014
space above the space to be pierced. Anatomy by Regions, Snell, 8th ed., 59. MED
C. The incision through the intercostal space is kept close to BOARDS;
the lower border of the rib to avoid injuring the intercostal TOPNOTCH
vessels and nerve. MD)
D. All of the statement are correct.
E. All of the statements are incorrect.

TOPNOTCH MEDICAL BOARD PREP ANATOMY SUPEREXAM Page 65 of 94


For inquiries visit www.topnotchboardprep.com.ph or email us at topnotchmedicalboardprep@gmail.com
TOPNOTCH MEDICAL BOARD PREP ANATOMY SUPEREXAM
For inquiries visit www.topnotchboardprep.com.ph or email us at topnotchmedicalboardprep@gmail.com
Item QUESTION EXPLANATION AUTHOR TOPNOTCH
# EXAM
509 C-shaped organ: SIMILAR TO PREVIOUS BOARD EXAM JAN BACK-UP
A. Esophagus CONCEPT/PRINCIPLE CHARMAINE MIDTERM
B. Duodenum PALOMAR, EXAM AUG
C. Spleen MD (TOP 9 - 2014
D. Pancreas FEB 2014
E. Rectum MED
BOARDS;
TOPNOTCH
MD)
510 Which of the following statements is true regarding the right SIMILAR TO PREVIOUS BOARD EXAM JAN BACK-UP
recurrent laryngeal nerve? CONCEPT/PRINCIPLE. Snell 8th ed., p. 127. The CHARMAINE MIDTERM
A. It hooks around the ligamentum arteriosum and ascends Right recurrent laryngeal nerve arises from the PALOMAR, EXAM AUG
in the groove between the trachea and the esophagus on the right vagus in the neck and hooks around the MD (TOP 9 - 2014
right side. subclavian artery and ascends between the FEB 2014
B. It supplies the cricothyroid muscle which is responsible trachea and esophagus. The left recurrent MED
for tensing the vocal cord. laryngeal nerve arises from the leftvagus trunk as BOARDS;
C. It branches to become the right vagus. the nerve crosses the arch of the aorta. it hooks TOPNOTCH
D. It loops around the right subclavian artery and ascends around the ligamentum arteriosum and ascends in MD)
between the trachea and esophagus. the groove between the trachea and the esophagus
E. All of the statements are true. on the on the left side. it suplies all the muscles
acting on the left vocal cord (except the
cricothyroid muscle, a tensor of the vocal cord,
which is supplied by the external laryngeal branch
of the vagus.
511 Dorsalis pedis pulse can be appreciated: Snell 8th ed., 658 JAN BACK-UP
A. Between the the tendons of flexor digitorum longus and CHARMAINE MIDTERM
flexor hallucis longus PALOMAR, EXAM AUG
B. Between the tendons of extensor hallucis longus and MD (TOP 9 - 2014
extensor digitorum longus FEB 2014
C. Between the anterosuperior iliac spine and the MED
symphysis pubis BOARDS;
D. Midway between the medial and lateral maleoli on the TOPNOTCH
front of the ankle MD)
E. Both B and D are correct

512 A 23 year-old-man was rushed to the hospital after Topnotch handout


JAN BACK-UP
sustaining multiple physical injuries after jumping from a Humeral fracture- and associated nerve injury
CHARMAINE MIDTERM
moving bus. X-ray of the humerus revealed midshaft 1. Surgical neck - axillary nerve
PALOMAR, EXAM AUG
fracture. Which of the following nerves would most likely be 2. midshaft / spiral groove fracture - radial nerve
MD (TOP 9 - 2014
injured? 3. Supracondylar - Median nerve
FEB 2014
A. Ulnar 4. Medial epicondyle - Ulnar nerve MED
B. Radial BOARDS;
C. Median TOPNOTCH
D. Musculocutaneous MD)
E. Axillary

513 SIMILAR TO PREVIOUS BOARD EXAM JAN BACK-UP


CONCEPT/PRINCIPLE CHARMAINE MIDTERM
PALOMAR, EXAM AUG
ECG of a 74 year old-male complaining of chest heaviness MD (TOP 9 - 2014
revealed ST-segment elevation in leads II, III and aVF. This FEB 2014
signifies? MED
A. Inferior wall myocaridal infarction BOARDS;
B. Inferior wall myocaridal ischemia TOPNOTCH
C. Anterolateral wall infarction MD)
D. Anterolateral wall ischemia
E. None of the above
514 An orogastric tube was inserted all the way to the 24-inch Snell 8th ed., p. 130 JAN BACK-UP
mark to a 57 year old male who had an episode of CHARMAINE MIDTERM
hematemesis. Where is the tip of the tube located? PALOMAR, EXAM AUG
A. esophagus MD (TOP 9 - 2014
B. cardia of stomach FEB 2014
C. pylorus MED
D. duodenum BOARDS;
E. Ileum TOPNOTCH
MD)

TOPNOTCH MEDICAL BOARD PREP ANATOMY SUPEREXAM Page 66 of 94


For inquiries visit www.topnotchboardprep.com.ph or email us at topnotchmedicalboardprep@gmail.com
TOPNOTCH MEDICAL BOARD PREP ANATOMY SUPEREXAM
For inquiries visit www.topnotchboardprep.com.ph or email us at topnotchmedicalboardprep@gmail.com
Item QUESTION EXPLANATION AUTHOR TOPNOTCH
# EXAM
515 Which of the following is true of jejunum but not of ileum? In the living, the jejunum can be distinguished JAN BACK-UP
A. Jejunum is longer compared to ileum. from the ileum by the following features: CHARMAINE MIDTERM
B. Jejunum is wider bored, thicker walled and redder than 1. The jejunum lies coiled in the upper part of the PALOMAR, EXAM AUG
the ileum. peritoneal cavity below the left side of the MD (TOP 9 - 2014
C. Aggregations of lymphoid tissue (Peyer's patches) are transverse mesocolon; the ileum is in the lower FEB 2014
present in the mucous membrane of the jejunum. part of the cavity and in the pelvis. MED
D. There are no plicae circulares in the jejunum. 2. The jejunum is wider bored, thicker walled, and BOARDS;
E. Jejunal mesenteric vessels receives numerous short redder than the ileum. The jejunal wall feels TOPNOTCH
terminal vessels that arise from more than 4 arcades. thicker because the permanent infoldings of the MD)
mucous membrane, the plicae circulares, are
larger, more numerous, and closely set in the
jejunum, whereas in the upper part of the ileum
they are smaller and more widely separated and in
the lower part they are absent.
3. The jejunal mesentery is attached to the
posterior abdominal wall above and to the left of
the aorta, whereas the ileal mesentery is attached
below and to the right of the aorta.
4. The jejunal mesenteric vessels form only one or
two arcades, with long and infrequent branches
passing to the intestinal wall. The ileum receives
numerous short terminal vessels that arise from a
series of three or four or even more arcades.
5. At the jejunal end of the mesentery, the fat is
deposited near the root and is scanty near the
intestinal wall. At the ileal end of the mesentery
the fat is deposited throughout so that it extends
from the root to the intestinal wall.
6. Aggregations of lymphoid tissue (Peyer's
patches) are present in the mucous membrane of
the lower ileum along the antimesenteric border.
In the living these may be visible through the wall
of the ileum from the outside.

516 The following are boundaries of the site where breath Triangle of auscultation: lateral-medial border of JAN BACK-UP
sounds are best heard except: scapula, medial- trapezius, inferior- latissimus CHARMAINE MIDTERM
A. Latissimus dorsi dorsi -topnotch handout PALOMAR, EXAM AUG
B. Trapezius MD (TOP 9 - 2014
C. Medial border of scapula FEB 2014
D. External oblique muscle MED
E. None of the above BOARDS;
TOPNOTCH
MD)
517 The following muscles elevate the ribs and increase the Muscles of inspiration (elevate the ribs, Increase JAN BACK-UP
anteroposterior, transverse and vertical diameters of the the AP, transverse and vertical diameters) - CHARMAINE MIDTERM
thoracic cage except: Serratus posterior superior, Levator costarum, PALOMAR, EXAM AUG
A. Serratus posterior inferior External intercostal, Innermost intercostal, MD (TOP 9 - 2014
B. External intercostal Subcostal; Muscles of expiration (depress the ribs) FEB 2014
C. Innermost intercostal - Serratus posterior inferior, Internal intercostal, MED
D. Subcostal And Transversus thoracis - Topnotch handout BOARDS;
E. None of the above TOPNOTCH
MD)

518 The following statements concerning the dermatomes are C6- thumb; C8-small finger JAN BACK-UP
true except: CHARMAINE MIDTERM
A. The C5 dermatome includes the tip of the shoulder on PALOMAR, EXAM AUG
the same side. MD (TOP 9 - 2014
B. The C8 dermatome includes the thumb on the same side. FEB 2014
C. The T10 dermatome lies over the skin of the umbilicus. MED
D. The L4 and L5 dermatome runs along the medial side of BOARDS;
the foot and big toe. TOPNOTCH
E. The S1 dermatome includes the lateral side of the foot MD)
and the small toe.

519 The cremaster muscle raise the testis and the scrotum External spermatic fascia is dervived from the JAN BACK-UP
upward for warmth and for protection againsts injury. The aponeurosis of the external oblique muscle; the CHARMAINE MIDTERM
cremasteric fascia is a derivative of: cremasteric fascia is derived from the internal PALOMAR, EXAM AUG
A. Superficial fascia oblique muscle, and the internal spermatic fascia MD (TOP 9 - 2014
B. External oblique muscle is derived from the fascia transversalis. Snell 8th FEB 2014
C. Transversalis fascia ed., 167. MED
D. Internal oblique muscle BOARDS;
E. Peritoneum TOPNOTCH
MD)

TOPNOTCH MEDICAL BOARD PREP ANATOMY SUPEREXAM Page 67 of 94


For inquiries visit www.topnotchboardprep.com.ph or email us at topnotchmedicalboardprep@gmail.com
TOPNOTCH MEDICAL BOARD PREP ANATOMY SUPEREXAM
For inquiries visit www.topnotchboardprep.com.ph or email us at topnotchmedicalboardprep@gmail.com
Item QUESTION EXPLANATION AUTHOR TOPNOTCH
# EXAM
520 A 30 year old man involved in a car accident was rushed to There were approximately 3 questions on bladder JAN BACK-UP
the ER with severe lower abdominal pain and gross injury and site of blood accumulation in our board CHARMAINE MIDTERM
hematuria. Radiologic evaluation revealed pelvic fracture. exam. ** Urine and PALOMAR, EXAM AUG
Which of the following statements regarding bladder rupture blood escape freely into the peritoneal cavity in MD (TOP 9 - 2014
is not true? intraperitoneal bladder rupture which usually FEB 2014
A. The bladder may rupture intraperitoneally if the occur with injury to the superior wall of a full MED
superior wall of a full bladder was injured. bladder (that has extended up into the abdomen). BOARDS;
B. The bladder may rupture intraperitoneally or -Snell 8th ed., 351. TOPNOTCH
extraperitoneally. MD)
C. Urine and blood escape freely into the peritoneal cavity
in extraperitoneal bladder rupture.
D. Extraperitoneal bladder rupture occurs when bony
fragments pierce the anterior part of the bladder wall below
the level of peritoneal reflection.
E. All of the statements are correct.

521 The foremost example of a saddle joint MIGUEL MIDTERM 1


A) 2nd carpometacarpal RAFAEL EXAM - FEB
B) 2nd metatarsophalangeal RAMOS, MD 2013
C) 1st metatarsophalangeal (TOP 3 - FEB
D) 1st carpometacarpal 2012 MED
BOARDS;
TOPNOTCH
MD)
522 A surgeon tells a medical student to tap the side of the face of MIGUEL MIDTERM 1
a patient who just had thyroid surgery. The surgeon is most RAFAEL EXAM - FEB
worried about damage to which of the following vessels? RAMOS, MD 2013
A) Common carotid artery (TOP 3 - FEB
B) External carotid artery 2012 MED
C) Anterior jugular vein BOARDS;
D) Superior and inferior thyroid artery TOPNOTCH
MD)

523 A 45-year-old woman comes into the ER with abdominal MIGUEL MIDTERM 1
pain that has progressively worsened since the previous RAFAEL EXAM - FEB
night. Imaging reveals that a loop of small intesting has RAMOS, MD 2013
passed through the epiploic foramen into the omental bursa (TOP 3 - FEB
and is constricted by the margins of the foramen. This should 2012 MED
not be surgically relieved because this procedure would risk BOARDS;
cutting the TOPNOTCH
A) Abdominal aorta MD)
B) Hepatic artery
C) Hepatic vein
D) Pancreatic duct

524 The 2nd rib is atypical because of this feature? MIGUEL MIDTERM 1
A) groove for subclavian vessels RAFAEL EXAM - FEB
B) only 1 facet that articulates with a single vertebrae RAMOS, MD 2013
C) tuberosity for serratus anterior (TOP 3 - FEB
D) contains the scalene tubercle 2012 MED
BOARDS;
TOPNOTCH
MD)
525 A 16-year-old teenage boy tried to commit suicide by MIGUEL MIDTERM 1
slashing his wrist after his girlfriend of one week broke up RAFAEL EXAM - FEB
with him. He sustained “suicide cuts” on the lateral side of RAMOS, MD 2013
his wrist, most likely sparing the: (TOP 3 - FEB
A) Radial artery 2012 MED
B) Median nerve BOARDS;
C) Flexor carpi radialis tendon TOPNOTCH
D) Ulnar artery MD)

526 An IVP is performed on a patient to evaluate the function and MIGUEL MIDTERM 1
structure of her kidneys. Examination of the resulting RAFAEL EXAM - FEB
radiographs reveal that the left kidney is normal but that RAMOS, MD 2013
there is a duplication of the ureter and renal pelvis on the (TOP 3 - FEB
right side. Further testing reveals that kidney function is 2012 MED
normal. This variation is a result of abnormal development of BOARDS;
which of the following structures? TOPNOTCH
A) Ureteric bud MD)
B) Metanephric blastema
C) Mesonephric duct
D) Mesonephric tubules

TOPNOTCH MEDICAL BOARD PREP ANATOMY SUPEREXAM Page 68 of 94


For inquiries visit www.topnotchboardprep.com.ph or email us at topnotchmedicalboardprep@gmail.com
TOPNOTCH MEDICAL BOARD PREP ANATOMY SUPEREXAM
For inquiries visit www.topnotchboardprep.com.ph or email us at topnotchmedicalboardprep@gmail.com
Item QUESTION EXPLANATION AUTHOR TOPNOTCH
# EXAM
527 A 12-year-old boy falls from a tree he is climbing, but catches MIGUEL MIDTERM 1
himself on a branch with his right hand. He swings by his RAFAEL EXAM - FEB
right arm and jumps to the ground. Several hours later he RAMOS, MD 2013
presents to the ER with right hand clumsiness. Which of the (TOP 3 - FEB
following structures has he most likely injured? 2012 MED
A) Lower trunk, brachial plexus BOARDS;
B) Axillary nerve TOPNOTCH
C) Musculocutaneous nerve MD)
D) Radial nerve

528 A gastric biopsy is performed on a 42-year-old male. The MIGUEL MIDTERM 1


specimen is observed to have numerous cells with apical RAFAEL EXAM - FEB
membrane-bound secretion granules in the gastric glands. RAMOS, MD 2013
From which area of the stomach was the biopsy taken (TOP 3 - FEB
A) Cardiac region 2012 MED
B) Columns of Morgagni BOARDS;
C) Fundic region TOPNOTCH
D) Pyloric region MD)

529 A 52 y/o male has had a chronic cough with occasional low- MIGUEL MIDTERM 1
volume hemoptysis for the past three weeks. He is a two RAFAEL EXAM - FEB
pack-per-day cigarette smoker, and drinks three to four cans RAMOS, MD 2013
of beer on weekends. PE reveals right-sided face and right (TOP 3 - FEB
arm swelling and engorgement of subcutaneous veins on the 2012 MED
right side of the neck. Which of the following veins is most BOARDS;
likely obstructed in this patient? TOPNOTCH
A) External jugular MD)
B) Subclavian
C) Brachiocephalic
D) Superior vena cava

530 Most of the muscles of the buttocks insert in the MIGUEL MIDTERM 1
A) Quadrate tubercle RAFAEL EXAM - FEB
B) Greater tuberosity RAMOS, MD 2013
C) Lesser trochanter (TOP 3 - FEB
D) Greater trochanter 2012 MED
BOARDS;
TOPNOTCH
MD)
531 After a motor vehicle accident, a patient is brought to the MIGUEL MIDTERM 1
emergency room. Xrays reveal that she has fractures of her RAFAEL EXAM - FEB
left ninth and tenth ribs. She has a rapid heart rate and low RAMOS, MD 2013
blood pressure. Peritoneal lavage reveals free blood in the (TOP 3 - FEB
peritoneal cavity. A surgeon is able to stop the bleeding by 2012 MED
placing a clamp across which of the following structures? BOARDS;
A) Falciform ligament TOPNOTCH
B) Gastrosplenic ligament MD)
C) Splenorenal ligament
D) Hepatoduodenal ligament

532 The glossopharyngeal nerve is transected accidentally MIGUEL MIDTERM 1


during a surgical procedure in a 45 y/o male. Which of the RAFAEL EXAM - FEB
following is most likely lost in this patient? RAMOS, MD 2013
A) Taste sensation from the anterior 2/3 of the tongue (TOP 3 - FEB
B) General sensation from the tonsillar lining 2012 MED
C) Salivary secretion from the submandibular gland BOARDS;
D) Protrusion of the tongue TOPNOTCH
MD)

533 Laceration of the male urethra just inferior to the urogenital MIGUEL MIDTERM 1
diaphragm would likely result in extravasation of urine into RAFAEL EXAM - FEB
all of the following regions except RAMOS, MD 2013
A) The abdominal wall between Scarpa's fascia and deep (TOP 3 - FEB
fascia 2012 MED
B) The anal triangle between superficial fat and deep fascia BOARDS;
C) The penis between the superficial fascia and deep (Buck's) TOPNOTCH
fascia MD)
D) The urogenital triangle between the deep layer of
superficial (Colle's)

TOPNOTCH MEDICAL BOARD PREP ANATOMY SUPEREXAM Page 69 of 94


For inquiries visit www.topnotchboardprep.com.ph or email us at topnotchmedicalboardprep@gmail.com
TOPNOTCH MEDICAL BOARD PREP ANATOMY SUPEREXAM
For inquiries visit www.topnotchboardprep.com.ph or email us at topnotchmedicalboardprep@gmail.com
Item QUESTION EXPLANATION AUTHOR TOPNOTCH
# EXAM
534 A 5-year-old male with a bounding pulse has a thrill best MIGUEL MIDTERM 1
palpated over the upper left sternal edge. A continuous RAFAEL EXAM - FEB
murmur is heard over the area on cardiac auscultation. If RAMOS, MD 2013
surgery is planned, the surgeon should intervene on a (TOP 3 - FEB
derivative of which of the following embryologic structures? 2012 MED
A) Sinus venosus BOARDS;
B) Bulbus cordis TOPNOTCH
C) 4th aortic arch MD)
D) 6th aortic arch

535 In portal hypertension, which of the following veins will not MIGUEL MIDTERM 1
engorge and is not a collateral of the circulation? RAFAEL EXAM - FEB
A) Inferior rectal RAMOS, MD 2013
B) Middle rectal (TOP 3 - FEB
C) Esophageal 2012 MED
D) Hepatic BOARDS;
TOPNOTCH
MD)

536 In performing a cricothyrotomy, an incision should be made MIGUEL MIDTERM 1


at which of the following locations? RAFAEL EXAM - FEB
A) Cricothyroid membrane, at the junction of clavicle and RAMOS, MD 2013
sternum (TOP 3 - FEB
B) Cricothyroid membrane, between the thyroid cartilage 2012 MED
and cricoid cartilage below BOARDS;
C) Thyrohyoid membrane, between the thyroid cartilage and TOPNOTCH
hyoid bone MD)
D) Sternal notch, at the junction of the clavicle and sternum

537 The structure that serves as a landmark in doing pudendal MIGUEL MIDTERM 1
block anesthesia through a transvaginal approach is the RAFAEL EXAM - FEB
A) Ischial spine RAMOS, MD 2013
B) Iliac crest (TOP 3 - FEB
C) Ischial tuberosity 2012 MED
D) Sacral promontory BOARDS;
TOPNOTCH
MD)

538 A neurologic exam of a 34-year-old man reveals a direct and MIGUEL MIDTERM 1
consensual light reflex in his left eye; but neither a direct nor RAFAEL EXAM - FEB
consensual reflex in his right eye. The nerve involved is the RAMOS, MD 2013
A) Right optic nerve (TOP 3 - FEB
B) Left optic nerve 2012 MED
C) Right oculomotor nerve BOARDS;
D) Left oculomotor nerve TOPNOTCH
MD)

539 A wall-climber's primary back muscle is in the following MIGUEL MIDTERM 1


group of muscles as which of the following RAFAEL EXAM - FEB
A) Serratus posterior RAMOS, MD 2013
B) Trapezius (TOP 3 - FEB
C) Splenus cervicis 2012 MED
D) Erector spinae BOARDS;
TOPNOTCH
MD)

540 A 17-year-old football player comes to the emergency MIGUEL MIDTERM 1


department hopping on his right foot. An X-ray of the left leg RAFAEL EXAM - FEB
shows a fracture of the neck of the fibula. Which of the RAMOS, MD 2013
following findings do you most expect on physical (TOP 3 - FEB
examination? 2012 MED
A) Loss of sensation on the sole of the left foot BOARDS;
B) weakened inversion of left foot TOPNOTCH
C) loss of plantarflexion of the left foot MD)
D) Loss of sensation on the dorsum ofthe left foot

TOPNOTCH MEDICAL BOARD PREP ANATOMY SUPEREXAM Page 70 of 94


For inquiries visit www.topnotchboardprep.com.ph or email us at topnotchmedicalboardprep@gmail.com
TOPNOTCH MEDICAL BOARD PREP ANATOMY SUPEREXAM
For inquiries visit www.topnotchboardprep.com.ph or email us at topnotchmedicalboardprep@gmail.com
Item QUESTION EXPLANATION AUTHOR TOPNOTCH
# EXAM
541 A 24/M, gang member, is hit over the right side of his head A classic case of epidural hematoma (head injury + ABDELSIMA FINAL EXAM
with a baseball bat during a brawl. He loses consciousness lucid interval + lens-shaped hemorrhage on CT). R OMAR II, - FEB 2014
for a few minutes, but recovers promptly. One hour later, he The most common artery damaged in epidural MD (TOP 2 -
is found unconscious at his home. The right pupil is fixed and hematoma is the anterior division of the middle AUG 2013
dilated. When he was rushed to the ER, stat CT done showed meningeal artery, a branch of the maxillary artery. MED
lenticular hyperdensity in the temporoparietal area. The BOARDS;
blood vessel which most likely bled in this case, is a direct TOPNOTCH
branch of the: MD - 200
A. Ophthalmic artery QUESTIONS)
B. Maxillary artery AND MARC
C. Internal carotid artery DENVER
D. Middle cerebral artery TIONGSON,
E. Anterior choroidal artery MD (40
QUESTIONS)

542 A 40/F with known tooth infection comes in due to a large, Patient has Ludwig's angina due to an acute ABDELSIMA FINAL EXAM
tender, fluctuant mass occupying the left lower side of her infection of the submandibular fascial space, R OMAR II, - FEB 2014
face and upper neck including the underside of the mouth. commonly secondary to a dental infection. MD (TOP 2 -
On examination, the mass is seen pushing up the floor of the AUG 2013
mouth on the left side. This patient has an acute infection of MED
this fascial space which is usually due to dental infection. BOARDS;
A. Visceral TOPNOTCH
B. Retropharyngeal MD - 200
C. Submandibular QUESTIONS)
D. Masticatory AND MARC
E. Sublingual DENVER
TIONGSON,
MD (40
QUESTIONS)
543 You are performing the neurologic examination on a 45/M, When the patient asked to put out the tongue, the ABDELSIMA FINAL EXAM
diagnosed with a skull base tumor. When you asked him to tongue will deviate toward the paralyzed side. The R OMAR II, - FEB 2014
protrude the tongue, you noted that the tongue deviated to normal genioglossus muscle will pull the MD (TOP 2 -
the left; indicating involvement of the: unaffected side of the tongue forward leaving the AUG 2013
A. Left glossopharyngeal nerve paralyzed side of the tongue stationary MED
B. Right glossopharyngeal nerve BOARDS;
C. Left hypoglossal nerve TOPNOTCH
D. Right hypoglossal nerve MD - 200
E. Right vagus nerve QUESTIONS)
AND MARC
DENVER
TIONGSON,
MD (40
QUESTIONS)
544 Which of the following would necessitate cricothyroidotomy Section of the ELN causes weakness of the voice. ABDELSIMA FINAL EXAM
or tracheostomy? Bilateral complete section of the RLN causes loss R OMAR II, - FEB 2014
A. Section of the external laryngeal nerve of speech and impairment of breathing because MD (TOP 2 -
B. Unilateral partial section of the recurrent laryngeal nerve the glottis is partially closed (vocal fold assumed AUG 2013
C. Bilateral partial section of the recurrent laryngeal nerve position midway between abduction and MED
D. Unilateral complete section of the recurrent laryngeal adduction.) A bilateral partial section of the RLN is BOARDS;
nerve the worst, causing bilateral paralysis of abductors. TOPNOTCH
E. Bilateral complete section of the recurrent laryngeal nerve The vocal cord is drawn together, leading to acute MD - 200
breathlessness and stridor; cricothyroidotomy or QUESTIONS)
tracheostomy would be necessary. AND MARC
DENVER
TIONGSON,
MD (40
QUESTIONS)
545 Knowledge of important airway distances would help in the Incisor teeth to vocal cords: 15 cm; incisor teeth to ABDELSIMA FINAL EXAM
correct performance of procedures such as endotracheal carina: 20 cm; external nares to carina: 30 cm R OMAR II, - FEB 2014
intubation. What is the average distance between the incisor MD (TOP 2 -
teeth and the carina? AUG 2013
A. 15 cm MED
B. 20 cm BOARDS;
C. 25 cm TOPNOTCH
D. 30 cm MD - 200
E. 44 cm QUESTIONS)
AND MARC
DENVER
TIONGSON,
MD (40
QUESTIONS)
546 A 12/M came in due to epistaxis. Most cases of epistaxis ABDELSIMA FINAL EXAM
occur on the anterioinferior septum and involve septal R OMAR II, - FEB 2014
branches of the: MD (TOP 2 -
A. Posterior ethmoidal artery AUG 2013
B. Anterior ethmoidal artery MED
C. Facial artery BOARDS;
D. Sphenopalatine artery TOPNOTCH
E. Greater palatine artery MD - 200
QUESTIONS)
AND MARC
DENVER
TIONGSON,
MD (40

TOPNOTCH MEDICAL BOARD PREP ANATOMY SUPEREXAM Page 71 of 94


For inquiries visit www.topnotchboardprep.com.ph or email us at topnotchmedicalboardprep@gmail.com
TOPNOTCH MEDICAL BOARD PREP ANATOMY SUPEREXAM
For inquiries visit www.topnotchboardprep.com.ph or email us at topnotchmedicalboardprep@gmail.com
Item QUESTION EXPLANATION AUTHOR TOPNOTCH
# EXAM
QUESTIONS)

547 A 40/M, a known case of coronary artery disease, Intercostobrachial nerve communicates with the ABDELSIMA FINAL EXAM
experiences severe crushing pain in the chest referred to the medial cutaneous nerve of the arm. R OMAR II, - FEB 2014
medial side of the arm and armpit. Which nerve is MD (TOP 2 -
responsible for the referred pain to the medial side of the AUG 2013
arm? MED
A. Anterior intercostal nerve BOARDS;
B. Posterior intercostal nerve TOPNOTCH
C. Lateral intercostal nerve MD - 200
D. Intercostobrachial nerve QUESTIONS)
E. Lateral cutaneous nerve AND MARC
DENVER
TIONGSON,
MD (40
QUESTIONS)
548 A patient with carcinoma of the lower third of the esophagus The upper third of the esophagus drains into the ABDELSIMA FINAL EXAM
who underwent surgical removal of the lesion also had the deep cervical; the middle third into the R OMAR II, - FEB 2014
stomach, upper half of duodenum, spleen and omenta mediastinal. MD (TOP 2 -
removed, because all drain commonly into this group of AUG 2013
lymph nodes: MED
A. Deep cervical BOARDS;
B. Mediastinal TOPNOTCH
C. Celiac MD - 200
D. Superior mesenteric QUESTIONS)
E. Inferior mesenteric AND MARC
DENVER
TIONGSON,
MD (40
QUESTIONS)
549 A barium swallow can also be used to assess the size of this If you think about it carefully, the question simply ABDELSIMA FINAL EXAM
chamber of the heart: asks the posteriormost of the chambers. R OMAR II, - FEB 2014
A. Left atrium Remember that the anterior wall of the esophagus MD (TOP 2 -
B. Left ventricle and the posterior wall of the left atrium are in AUG 2013
C. Right atrium close proximity; hence, a barium swallow can be MED
D. Right ventricle used to assess the size of the LA. BOARDS;
TOPNOTCH
MD - 200
QUESTIONS)
AND MARC
DENVER
TIONGSON,
MD (40
QUESTIONS)
550 How many intervertebral discs are there? No IV disc between C1 and C2, sacrum and coccyx. ABDELSIMA FINAL EXAM
A. 21 R OMAR II, - FEB 2014
B. 23 MD (TOP 2 -
C. 25 AUG 2013
D. 27 MED
E. 29 BOARDS;
TOPNOTCH
MD - 200
QUESTIONS)
AND MARC
DENVER
TIONGSON,
MD (40
QUESTIONS)
551 A 24/M, a member of the college varsity swimming team, ABDELSIMA FINAL EXAM
comes in to you complaining of shoulder pain. On R OMAR II, - FEB 2014
examination, you note pain on the shoulder while in the mid- MD (TOP 2 -
range of abduction. You are considering rotator cuff AUG 2013
tendinitis, due to this muscle rubbing on the acromion MED
because of a degenerated subacromial bursa. BOARDS;
A. Supraspinatus TOPNOTCH
B. Infraspinatus MD - 200
C. Teres minor QUESTIONS)
D. Teres major AND MARC
E. Subscapularis DENVER
TIONGSON,
MD (40
QUESTIONS)

TOPNOTCH MEDICAL BOARD PREP ANATOMY SUPEREXAM Page 72 of 94


For inquiries visit www.topnotchboardprep.com.ph or email us at topnotchmedicalboardprep@gmail.com
TOPNOTCH MEDICAL BOARD PREP ANATOMY SUPEREXAM
For inquiries visit www.topnotchboardprep.com.ph or email us at topnotchmedicalboardprep@gmail.com
Item QUESTION EXPLANATION AUTHOR TOPNOTCH
# EXAM
552 An infant was diagnosed to have Erb-Duchenne palsy as a Erb-Duchenne palsy usually affects the (1) ABDELSIMA FINAL EXAM
complication of difficult delivery. The right limb was seen suprascapular nerve causing palsy of the R OMAR II, - FEB 2014
hanging limp, rotated medially and pronated. Which of the supraspinatus (responsible for shoulder MD (TOP 2 -
following muscles is unaffected in Erb-Duchenne palsy? abduction) and infraspinatus (lateral rotation AUG 2013
A. Supraspinatus shoulder); (2) nerve to subclavius (depression of MED
B. Infraspinatus clavicle); (3) musculocutaneous nerve causing BOARDS;
C. Biceps brachii palsy of the biceps, brachialis and coracobrachialis TOPNOTCH
D. Subclavius (flex and supinate elbow); and (4) axillary nerve, MD - 200
E. None of the above supplying the deltoid (shoulder abudciton). QUESTIONS)
AND MARC
DENVER
TIONGSON,
MD (40
QUESTIONS)
553 A 55/F obese, diagnosed case of bilateral knee osteoarthritis, Poorly adjusted crutches may result in axillary ABDELSIMA FINAL EXAM
has been using crutches for a year to help her ambulate. She nerve palsy presenting as weak abduction of the R OMAR II, - FEB 2014
comes in to you complaining of difficulty in abducting her shoulder and lateral rotation of the arm (actions of MD (TOP 2 -
right shoulder. On examination, you note loss of the rounded the deltoid and teres minor, respectively); loss of AUG 2013
contour of the right shoulder and loss of sensation on the sensation on the lower half of the deltoid; and MED
lower half of the right deltoid. Which nerve must have been wasting of the deltoid. BOARDS;
affected? TOPNOTCH
A. Medial antebrachial cutaneous nerve MD - 200
B. Medial cutaneous nerve QUESTIONS)
C. Axillary nerve AND MARC
D. Musculocutaneous nerve DENVER
E. Median nerve TIONGSON,
MD (40
QUESTIONS)

554 Ureteral stones are most likely to lodge at the narrowest part SIMILAR TO PREVIOUS BOARD EXAM ABDELSIMA FINAL EXAM
of the ureter which is at the: CONCEPT/PRINCIPLE. R OMAR II, - FEB 2014
A. Ureteropelvic junction MD (TOP 2 -
B. Ureterovesical junction AUG 2013
C. Point at which the ureter crosses over the iliac vessels MED
D. Pelvic brim BOARDS;
TOPNOTCH
MD - 200
QUESTIONS)
AND MARC
DENVER
TIONGSON,
MD (40
QUESTIONS)
555 You were tasked to perform urethral catheterization on a Things to remember. The external orifice of the ABDELSIMA FINAL EXAM
patient prior to surgery. You note resistance in passing the glans is the narrowest part; the prostatic urethra R OMAR II, - FEB 2014
catheter at the narrowest part of the urethra which is the: is the widest and most dilatable; the part within MD (TOP 2 -
A. External orifice of the glans penis the bulb is subject to chronic inflammation and AUG 2013
B. Membranous urethra stricture formation and is the most common site of MED
C. Bulbar urethra rupture after a perineal blow. BOARDS;
D. Prostatic urethra TOPNOTCH
MD - 200
QUESTIONS)
AND MARC
DENVER
TIONGSON,
MD (40
QUESTIONS)
556 In performing lumbar puncture in an infant, it is important to Spinal cord ends at the lower border of L1 in the ABDELSIMA FINAL EXAM
remember that the spinal cord terminates as low as this adult; and at L3 in the infant. The subarachnoid R OMAR II, - FEB 2014
vertebral level. space extends down as far as the lower border of MD (TOP 2 -
A. L1 S2. AUG 2013
B. L2 MED
C. L3 BOARDS;
D. L4 TOPNOTCH
E. L5 MD - 200
QUESTIONS)
AND MARC
DENVER
TIONGSON,
MD (40
QUESTIONS)
557 In the adult, the notochord gives rise to the: Expect a few (1-2) embryology questions. ABDELSIMA FINAL EXAM
A. Spinal cord R OMAR II, - FEB 2014
B. Vertebral body MD (TOP 2 -
C. Annulus fibrosis AUG 2013
D. Nucleus pulposus MED
E. It has no adult derivatives BOARDS;
TOPNOTCH
MD - 200
QUESTIONS)
AND MARC
DENVER
TIONGSON,
MD (40
QUESTIONS)

TOPNOTCH MEDICAL BOARD PREP ANATOMY SUPEREXAM Page 73 of 94


For inquiries visit www.topnotchboardprep.com.ph or email us at topnotchmedicalboardprep@gmail.com
TOPNOTCH MEDICAL BOARD PREP ANATOMY SUPEREXAM
For inquiries visit www.topnotchboardprep.com.ph or email us at topnotchmedicalboardprep@gmail.com
Item QUESTION EXPLANATION AUTHOR TOPNOTCH
# EXAM
558 A 24/M basketball player lands awkwardly from a rebound, ACL tear. ABDELSIMA FINAL EXAM
and feels that his right knee has hyperextended, bent inward R OMAR II, - FEB 2014
and popped. The knee becomes swollen over the next few MD (TOP 2 -
hours. On PE, you note a positive Lachman test. The patient AUG 2013
has suffered a tear in the ligament which prevents: MED
A. Anterior displacement of the tiba on the femur BOARDS;
B. Posterior displacement of the tibia on the femur TOPNOTCH
C. Adduction of the leg at the knee MD - 200
D. Abduction fo the leg at the knee QUESTIONS)
AND MARC
DENVER
TIONGSON,
MD (40
QUESTIONS)
559 A 35/M, known case of Pott's disease, developed a psoas ABDELSIMA FINAL EXAM
abscess.. On PE, you noted that he had 1+ patellar tendon R OMAR II, - FEB 2014
reflex on the same side and he had difficulty walking. What MD (TOP 2 -
nerve is involved? AUG 2013
A. Femoral nerve MED
B. Obturator nerve BOARDS;
C. Sciatic nerve TOPNOTCH
D. Genitofemoral nerve MD - 200
E. Pudendal nerve QUESTIONS)
AND MARC
DENVER
TIONGSON,
MD (40
QUESTIONS)
560 The germinal epithelium of the ovary is an example of: Other simple cuboidal epithelium include the ABDELSIMA FINAL EXAM
A. Simple squamous epithelium LENS of the eye, the respiratory bronchioles, R OMAR II, - FEB 2014
B. Stratified squamous epithelium thyroid cells, and the pigment layer of the retina. MD (TOP 2 -
C. Simple columnar epithelium AUG 2013
D. Stratified columnar epithelium MED
E. Simple cuboidal epithelium BOARDS;
TOPNOTCH
MD - 200
QUESTIONS)
AND MARC
DENVER
TIONGSON,
MD (40
QUESTIONS)
561 the structure which passes along the borders of the Simons Answer: Recurrent LN can usually be seen within BLAKE MIDTERM 2
triangle is the: the Simons triangle bounded by esophagus, ITA, WARREN EXAM - FEB
a. recurrent laryngeal nerve and common carotid. ANG, MD 2014
b. median nerve (TOP 1 - AUG
c. superficial peroneal nerve 2013 MED
d. greater auricular nerve BOARDS;
TOPNOTCH
MD)

562 the weight of the thyroid gland in a normal adult is: 20g is the average weight of the thyroid gland BLAKE MIDTERM 2
a. 20g WARREN EXAM - FEB
b. 30g ANG, MD 2014
c. 40g (TOP 1 - AUG
d. 45g 2013 MED
BOARDS;
TOPNOTCH
MD)
563 the esophagus is lined by this epithelium BLAKE MIDTERM 2
a. stratified squamous WARREN EXAM - FEB
b. simple columnar ANG, MD 2014
c. stratified cuboidal (TOP 1 - AUG
d. pseudostratified columnar 2013 MED
BOARDS;
TOPNOTCH
MD)
564 the smooth portion of the ventricles of the heart is derived BLAKE MIDTERM 2
from WARREN EXAM - FEB
a. primitive ventricle ANG, MD 2014
b. bulbos cordis (TOP 1 - AUG
c. sinus venosus 2013 MED
d. none of the above BOARDS;
TOPNOTCH
MD)

TOPNOTCH MEDICAL BOARD PREP ANATOMY SUPEREXAM Page 74 of 94


For inquiries visit www.topnotchboardprep.com.ph or email us at topnotchmedicalboardprep@gmail.com
TOPNOTCH MEDICAL BOARD PREP ANATOMY SUPEREXAM
For inquiries visit www.topnotchboardprep.com.ph or email us at topnotchmedicalboardprep@gmail.com
Item QUESTION EXPLANATION AUTHOR TOPNOTCH
# EXAM
565 a patient presents to the trauma center after a vehicular Left 2nd ICS MCL. Although technically anywhere BLAKE MIDTERM 2
crash with tachypnea and hypotension. the trachea is within the chest can be accessed, it is typically WARREN EXAM - FEB
deviated to the right with decrease pulses on both arms and within this area to avoid any major structures. ANG, MD 2014
legs. as an initial/immediate life saving procedure, the (TOP 1 - AUG
physician should access the thorax through 2013 MED
a. right 2nd ICS MCl BOARDS;
b. right 5th ICS MAL TOPNOTCH
c. left 2nd ICS MCL MD)
d. left 2nd ICS MAL

566 blood cell component with the longest half life Adult RBC has half life of 90-120 days BLAKE MIDTERM 2
a. basophil WARREN EXAM - FEB
b. rbc ANG, MD 2014
c. platelet (TOP 1 - AUG
d. neutrophil 2013 MED
BOARDS;
TOPNOTCH
MD)
567 osteoid is predominantly produced by: BLAKE MIDTERM 2
a. osteoblast WARREN EXAM - FEB
b. osteoclast ANG, MD 2014
c. osteocyte (TOP 1 - AUG
d. all of the above 2013 MED
BOARDS;
TOPNOTCH
MD)
568 branchial arch derivative of the stylopharyngeus is: BLAKE MIDTERM 2
a. branchial arch 1 WARREN EXAM - FEB
b. branchial arch 2 ANG, MD 2014
c. branchial arch 3 (TOP 1 - AUG
d. branchial arch 5 2013 MED
BOARDS;
TOPNOTCH
MD)
569 oxyphil cell is found in: BLAKE MIDTERM 2
a. parathyroid gland WARREN EXAM - FEB
b. pituitary gland ANG, MD 2014
c. adrenal gland (TOP 1 - AUG
d. thyroid gland 2013 MED
BOARDS;
TOPNOTCH
MD)
570 the vein that joins the subcostal vein to form the azygos vein BLAKE MIDTERM 2
is: WARREN EXAM - FEB
a. ascending lumbar ANG, MD 2014
b. musculophrenic (TOP 1 - AUG
c. ascending phrenic 2013 MED
d. superior epigastric BOARDS;
TOPNOTCH
MD)

571 fracture of the humeral midshaft produces this abnormality: Radial nerve injury (know the difference of BLAKE MIDTERM 2
a. inability to flex the elbow joint neuropraxia, axonotmesis, etc.) causes inability to WARREN EXAM - FEB
b. numbness of the medial portion of the palm extend the wrist joint. ANG, MD 2014
c. inability to extend wrist joint (TOP 1 - AUG
d. inability to abduct the upper arm 2013 MED
BOARDS;
TOPNOTCH
MD)
572 the epithelial covering of the lens of the eye is: BLAKE MIDTERM 2
a. simple squamous WARREN EXAM - FEB
b. simple cuboidal ANG, MD 2014
c. simple columnar (TOP 1 - AUG
d. pseudostratified cuboidal 2013 MED
BOARDS;
TOPNOTCH
MD)
573 the sternal angle of Louis coincides with this vertebral level BLAKE MIDTERM 2
a. C4 WARREN EXAM - FEB
b. T4 ANG, MD 2014
c. T8 (TOP 1 - AUG
d. C6 2013 MED
BOARDS;
TOPNOTCH
MD)
574 the left gastroepiploic vein drains to the BLAKE MIDTERM 2
a. splenic vein WARREN EXAM - FEB
b. SMV ANG, MD 2014
c. IMV (TOP 1 - AUG
d. Poral vein 2013 MED
BOARDS;
TOPNOTCH
MD)

TOPNOTCH MEDICAL BOARD PREP ANATOMY SUPEREXAM Page 75 of 94


For inquiries visit www.topnotchboardprep.com.ph or email us at topnotchmedicalboardprep@gmail.com
TOPNOTCH MEDICAL BOARD PREP ANATOMY SUPEREXAM
For inquiries visit www.topnotchboardprep.com.ph or email us at topnotchmedicalboardprep@gmail.com
Item QUESTION EXPLANATION AUTHOR TOPNOTCH
# EXAM
575 the major blood supply to the femoral head is BLAKE MIDTERM 2
a. medial circumflex artery WARREN EXAM - FEB
b. lateral circumflex artery ANG, MD 2014
c. feeding artery to the femoral head (TOP 1 - AUG
d. deep femoral artery 2013 MED
BOARDS;
TOPNOTCH
MD)
576 drainage of the nasolacrimal duct is towards the: BLAKE MIDTERM 2
a. superior meatus WARREN EXAM - FEB
b. middle meatus ANG, MD 2014
c. inferior meatus (TOP 1 - AUG
d. oropharynx 2013 MED
BOARDS;
TOPNOTCH
MD)
577 a person was stabbed at the transpyloric area through the BLAKE MIDTERM 2
neck of the pancreas. which of the following structures will WARREN EXAM - FEB
be least affected: ANG, MD 2014
a. SMA (TOP 1 - AUG
b. IMA 2013 MED
c. Splenic vein BOARDS;
d. Portal vein TOPNOTCH
MD)

578 most common site Of ureteral obstruction: Answer: Uteropelvic junction , but the narrowest BLAKE MIDTERM 2
a. ureteropelvic junction portion is ureterovesical junction WARREN EXAM - FEB
b. pelvic brim level ANG, MD 2014
c. ureterovesical junction (TOP 1 - AUG
2013 MED
BOARDS;
TOPNOTCH
MD)
579 the most common hernia in females Indirect hernia is the most common type of hernia BLAKE MIDTERM 2
a. direct in females; males are more involved than females. WARREN EXAM - FEB
b. indirect ANG, MD 2014
c. femoral (TOP 1 - AUG
d. obturator 2013 MED
BOARDS;
TOPNOTCH
MD)
580 craniophayngioma would present with what visual field Bilateral temporal hemianopsia BLAKE MIDTERM 2
defect WARREN EXAM - FEB
a. binasal hemianopsia ANG, MD 2014
b. bitempora hemianopsia (TOP 1 - AUG
c. right/left homonymous hemianopsia 2013 MED
d. right superior quantantanopsia BOARDS;
TOPNOTCH
MD)
581 In endocrine histology, the presence of oxyphil cells will tell Oxyphil cells are found in the parathyoid gland. TIMOTHY MIDTERM 1
us that the tissue specimen came from what organ? There function is still unknown. TANG LEE EXAM - FEB
A. Thyroid gland SAY, MD 2014
B. Parathyroid gland (TOP 4 - AUG
C. Pituitary gland 2013 MED
D. Adrenal gland BOARDS;
E. Gonads TOPNOTCH
MD)

582 Which is the most proximal part of the respiratory The respiratory conducting system in proximal to TIMOTHY MIDTERM 1
conducting system that has absent goblet cells and distal order: trachea, main bronchi, lobar bronchi, TANG LEE EXAM - FEB
submucous glands? terminal bronchioles, respiratory bronchioles, SAY, MD 2014
A. Lobar bronchi alveolar ducts, alveolar sacs, alveoli. Goblet cells (TOP 4 - AUG
B. Segmental bronchi and submucous glands are present up to the 2013 MED
C. Terminal bronchioles segmental bronchi. Smooth muscles are present BOARDS;
D. Respiratory bronchioles up to alveolar ducts. Elastic fibers are present up TOPNOTCH
E. Alveolar ducts to alveoli sacs. MD)

583 A 32 yr old biker fell and sustain lacerations in the right arm Brachioradialis is the only flexor muscle that is TIMOTHY MIDTERM 1
with suspicious fracture. In the ER, X-rays were requested also innervated by the radial nerve. Midshaft TANG LEE EXAM - FEB
which showed fracture in the midshaft of the humerus. fractures of the humerus usually affect the radial SAY, MD 2014
Which of the following muscles would exhibit weakness? nerve. Coracobrachialis and brachialis are (TOP 4 - AUG
A. Coracobrachialis innervated by the musculocutaneous nerve. 2013 MED
B. Brachialis Pronator teres and flexor digitorum profundus are BOARDS;
C. Brachioradialis innervated by the median nerve. TOPNOTCH
D. Pronator teres MD)
E. Flexor digitorum profundus

TOPNOTCH MEDICAL BOARD PREP ANATOMY SUPEREXAM Page 76 of 94


For inquiries visit www.topnotchboardprep.com.ph or email us at topnotchmedicalboardprep@gmail.com
TOPNOTCH MEDICAL BOARD PREP ANATOMY SUPEREXAM
For inquiries visit www.topnotchboardprep.com.ph or email us at topnotchmedicalboardprep@gmail.com
Item QUESTION EXPLANATION AUTHOR TOPNOTCH
# EXAM
584 A patient sustain an injury in the posterior cord of the The posterior cord gives rise to the upper and TIMOTHY MIDTERM 1
brachial plexus. The following actions are compromised lower subscapular nerves, thoracodorsal, axillary TANG LEE EXAM - FEB
EXCEPT? and radial nerves. Supination of the forearm is the SAY, MD 2014
A. Medial rotation of the shoulder main action of biceps brachii which is innervated (TOP 4 - AUG
B. Supination of the forearm by the musculocutaneous nerve. Elevation of the 2013 MED
C. Elevation of the trunk trunk is an action of the latissimus dorsi BOARDS;
D. Extension of the forearm innervated by the thoracodorsal nerve. Radial TOPNOTCH
E. Lateral rotation of the shoulder nerve innervates the posterior compartment of MD)
the arm and forearm (extensors). The muscles of
the shoulder are innervated by the other nerves
mentioned.
585 Which of the following muscles abducts, flexes and laterally Actions: TIMOTHY MIDTERM 1
rotates the thigh at the knee joint? Tensor fascia lata: abducts, medially rotates and TANG LEE EXAM - FEB
A. Sartorius flexes thigh SAY, MD 2014
B. Tensor fascia lata Rectus femoris: flexes the thigh and extends the (TOP 4 - AUG
C. Rectus femoris leg 2013 MED
D. Gracilis Gracilis: adducts thigh, flexes and medially rotate BOARDS;
E. Pectineus the leg TOPNOTCH
Pectineus: adducts, flexes and medially rotate the MD)
thigh
586 A varsity basketball player had an audible pop in his right Anterior drawer sign: Forward sliding of the tibia TIMOTHY MIDTERM 1
knee upon landing from a jump shot. This was accompanied on the femur due to rupture of the anterior TANG LEE EXAM - FEB
by excruciating pain and swelling of the knee joint. To check cruciate ligament SAY, MD 2014
for possible ligamental tear, the rehabilitation medicine Posterior drawer sign: Backward sliding of the (TOP 4 - AUG
specialist asked the patient to lie down and perform the tibia on the femur due to rupture of the posterior 2013 MED
anterior drawer test by doing this maneuver? cruciate ligament BOARDS;
A. Forward sliding of the femur on the tibia due to rupture TOPNOTCH
of either the anterior or posterior cruciate ligament MD)
B. Forward sliding of the femur on the tibia due to rupture
of the posterior cruciate ligament
C. Forward sliding of the tibia on the femur due to rupture
of the posterior cruciate ligament
D. Forward sliding of the femur on the tibia due to rupture
of the anterior cruciate ligament
E. Forward sliding of the tibia on the femur due to rupture
of the anterior cruciate ligament

587 Which is the final draining vessel of the conventional The pathway for aqueous humor outflow are the TIMOTHY MIDTERM 1
aqueous humor outflow pathway? following: trabecular meshwork → Schlemm's TANG LEE EXAM - FEB
A. Orbital veins canal → Collector channels → Aqueous veins → SAY, MD 2014
B. Trabecular meshwork Episcleral veins → Orbital veins → Intracranial (TOP 4 - AUG
C. Schlemm's canal cavernous sinus 2013 MED
D. Episcleral veins BOARDS;
E. Aqueous veins TOPNOTCH
MD)

588 A 60 yr old patient with known atherosclerotic heart disease Review the visual pathway. Vascular and TIMOTHY MIDTERM 1
suddenly complain of an acute onset blurring of vision, in neoplastic (malignant or benign tumours) lesions TANG LEE EXAM - FEB
which stroke was entertained. MRI shows that the lesion from the optic tract, to visual cortex can cause a SAY, MD 2014
involved the cerebral cortex supplied by the left posterior contralateral homonymous hemianopsia. The (TOP 4 - AUG
cerebral artery, what visual field defect is expected? difference is that cortical lesions tend to have 2013 MED
A. Right homonymous hemianopsia without macular macular sparing since the optic tract send signals BOARDS;
sparing coming from the macula to both cortices and they TOPNOTCH
B. Left homonymous hemianopsia without macular sparing have dual blood supply from the MCA (aside from MD)
C. Bitemporal hemianopsia without macular sparing the PCA).
D. Right homonymous hemianopsia with macular sparing
E. Left homonymous hemianopsia with macular sparing

589 An otorhinolaryngologist is examing the tympanic The adult external canal is divided into an outer TIMOTHY MIDTERM 1
membrane of a 20 yr old patient with chronic otitis media. cartilagenous portion in its outer 1/3 and bony TANG LEE EXAM - FEB
He knows that the external auditory canal is divided into a? portion in its inner 2/3. It measures about 2.5 cms SAY, MD 2014
A. 1/2 outer bony portion and 1/2 inner cartilaginous on the whole. (TOP 4 - AUG
portion 2013 MED
B. 1/3 outer cartilaginous portion and 2/3 inner bony BOARDS;
portion TOPNOTCH
C. 1/3 outer bony portion and 2/3 inner cartilaginous MD)
portion
D. 2/3 outer cartilaginous portion and 1/3 inner bony
portion
E. 2/3 outer bony portion and 1/3 inner cartilaginous
portion

TOPNOTCH MEDICAL BOARD PREP ANATOMY SUPEREXAM Page 77 of 94


For inquiries visit www.topnotchboardprep.com.ph or email us at topnotchmedicalboardprep@gmail.com
TOPNOTCH MEDICAL BOARD PREP ANATOMY SUPEREXAM
For inquiries visit www.topnotchboardprep.com.ph or email us at topnotchmedicalboardprep@gmail.com
Item QUESTION EXPLANATION AUTHOR TOPNOTCH
# EXAM
590 A patient had been in a car accident wherein he suffered The pathology of a cape-like loss of sensory TIMOTHY MIDTERM 1
superfical lacerations but the neurologic exam was normal. 6 functions is indicative of syringomyelia or a TANG LEE EXAM - FEB
months later, he complains of loss of pain and temperature central cord syndrome. Anterior cord will result to SAY, MD 2014
sensations in the hands and forearm. This progressed to sensory deficits from a dermatome level down up (TOP 4 - AUG
include mild weakness of hand grip and extension of the to the lower extremities. Brown-Sequard is a 2013 MED
sensory loss to the arm and shoulders. An MRI of the spinal hemisection syndrome. Posterior cord syndrome BOARDS;
cord shows a cyst in a segment of the spinal cord. This will show loss of position sense and vibration. TOPNOTCH
pathology is consistent with? Tabes dorsalis is also a posterior cord syndrome. MD)
A. Anterior cord syndrome
B. Central cord syndrome
C. Brown-Sequard's syndrome
D. Posterior cord syndrome
E. Tabes dorsalis

591 During thyroidectomy, the surgeon accidently ligated the All muscles of phonation are innervated by the TIMOTHY MIDTERM 1
right recurrent laryngeal nerve. Which of the following recurrent larngeal nerve except for the TANG LEE EXAM - FEB
actions on the vocal cord by the muscles of phonation is cricothyroid which is responsible for tensing the SAY, MD 2014
SPARED? vocal cords (TOP 4 - AUG
A. Tenses 2013 MED
B. Abducts BOARDS;
C. Adducts TOPNOTCH
D. Relaxes MD)
E. Opens

592 A athlete had just finsihed running a marathon. He has All muscles of the thoracic wall are responsible for TIMOTHY MIDTERM 1
labored breathing. The following muscles are responsible for inspiration except serratus post inf, internal TANG LEE EXAM - FEB
inspiration EXCEPT? intercostal and tranverse thoracis muscles which SAY, MD 2014
A. External intercostal are responsible for labored expiration. (TOP 4 - AUG
B. Innermost intercostal 2013 MED
C. Serratus posterior superior BOARDS;
D. Serratus posterior inferior TOPNOTCH
E. Subcostal MD)

593 In the embryology of the primitive heart, which of the Review the fate of the fetal heart. Bulbus cordis TIMOTHY MIDTERM 1
following is INCORRECTLY paired? becomes the right ventricle. Sinus venosus (right TANG LEE EXAM - FEB
A. Truncus arteriosus = Pulmonary trunk horn) becomes the right atrium. SAY, MD 2014
B. Bulbus cordis = Right atrium (TOP 4 - AUG
C. Sinus venosus = Coronary sinus 2013 MED
D. Primitive atrium = Left atrium BOARDS;
E. Primitive ventricle = Left Ventricle TOPNOTCH
MD)

594 A 12-yr old boy was eating hurriedly when the school bell In sitting and standing positions, foreign bodies TIMOTHY MIDTERM 1
rang which signaled the end of recess period. He suddenly tend to lodge in the posterior basal BPS of the TANG LEE EXAM - FEB
choked while swallowing the siomai whole without chewing. right lower lobe. Please review the usual location SAY, MD 2014
Heimlick maneuver was done but was unsuccessful to of foreign body aspiration with different positions (TOP 4 - AUG
dislodge the food from the airways. Rigid bronchoscopy was of the body. 2013 MED
done. The siomai is most likely lodge in the? BOARDS;
A. Posterior bronchopulmonary segment of the right TOPNOTCH
superior lobe MD)
B. Posterior bronchopulmonary segment of the left
superior lobe
C. Superior bronchopulmonary segment of the right
inferior lobe
D. Inferior lingular bronchopulmonary segment of the left
superior lobe
E. Posterior basal bronchopulmonary segment of the right
inferior lobe

595 The abdominal cavity is lined by the peritoneum. Which fo For A, the parietal peritonuem should line the TIMOTHY MIDTERM 1
the following is TRUE regarding the peritoneum? internal surface of the wall and the visceral TANG LEE EXAM - FEB
A. The peritoneum is divided into two continuous layers, peritoneum should invest the organs. Parietal is SAY, MD 2014
the parietal peritoneum investing the organs and the visceral sensitive to pressure, pain and heat, with pain (TOP 4 - AUG
peritoneum lining the internal surface of the the abdomino- generally localized. Visceral is sensitive to 2013 MED
pelvic wall. stretching and pain is poorly localized. The BOARDS;
B. The pain fibers subserving the parietal peritoneum is ascending and descending colons are TOPNOTCH
primarily sensitive to stretching. retroperitoneal not the transverse colon. MD)
C. The duodenum, pancreas, and transverse colon are
retroperitoneal organs.
D. Pain in the visceral peritoneum is generally localized.
E. The peritoneal cavity is the space between the parietal
and visceral peritoneum that contains 50 mL of serous fluid
to allow sliding between the layers.

TOPNOTCH MEDICAL BOARD PREP ANATOMY SUPEREXAM Page 78 of 94


For inquiries visit www.topnotchboardprep.com.ph or email us at topnotchmedicalboardprep@gmail.com
TOPNOTCH MEDICAL BOARD PREP ANATOMY SUPEREXAM
For inquiries visit www.topnotchboardprep.com.ph or email us at topnotchmedicalboardprep@gmail.com
Item QUESTION EXPLANATION AUTHOR TOPNOTCH
# EXAM
596 A patient undergone upper GI endoscopy for recurrent The esophagus is 25 cm long but from the incisor, TIMOTHY MIDTERM 1
peptic ulcer disease. At what level of the gastrointestinal 15 cm - UES, 18 cm - thoracic inlet, 25 cm - arch of TANG LEE EXAM - FEB
tract is the scope in at 40 cm from the central incisors? the aorta, 40 cm - LES SAY, MD 2014
A. Lower esophageal sphincter (TOP 4 - AUG
B. Cardia of the stomach 2013 MED
C. Body of the stomach BOARDS;
D. Antrum of the stomach TOPNOTCH
E. Pylorus MD)

597 A trauma surgeon operated on a patient who had a gunshot Jejunum has deeper red color, thick and heavy TIMOTHY MIDTERM 1
wound. In running the bowel, the surgeon differentiates the wall, 2-4 cm in caliber, greater vascularity, long TANG LEE EXAM - FEB
small intestines because he knows that the jejunum has the vasa recta, few and large arcades, less fat, tall and SAY, MD 2014
following characteristics EXCEPT? large plica, and few lymphoid nodules.\ (TOP 4 - AUG
A. Deeper red color 2013 MED
B. Thinner walls BOARDS;
C. Fewer arcades TOPNOTCH
D. Longer vasa recta MD)
E. Larger plica circularis

598 What is TRUE regarding the ligametum teres? The ligamentum teres is a remnant of the left TIMOTHY MIDTERM 1
A. It forms a border of between the quadrate lobe and the umbilical vein. The right umbilical vein obliterates TANG LEE EXAM - FEB
left lobe of the liver. during the fetal period. It is empty and does not SAY, MD 2014
B. It is a remnant of the right umbilical vein. conduct any vessels. The true division of the left (TOP 4 - AUG
C. It contains the portal triad that enters the liver at the and right lobes of the liver is the imaginary line 2013 MED
porta hepatis. passing through the gallbladder and IVC. BOARDS;
D. It divides the liver into a left and right lobe. TOPNOTCH
E. It is a remnant of the ductus venosus MD)

599 In doing pelvic surgery, you noticed that there is a nerve Lateral border of the psoas - iliohypogastric, TIMOTHY MIDTERM 1
found anterior to the psoas muscle, this is probably the? ilioinguinal and femoral nerve TANG LEE EXAM - FEB
A. Iliohypogastric nerve Anterior - genitofemoral SAY, MD 2014
B. Ilioinguinal nerve Medial - obturator (TOP 4 - AUG
C. Genitofemoral nerve 2013 MED
D. Femoral nerve BOARDS;
E. Obturator nerve TOPNOTCH
MD)

600 Which of the following is NOT located in the deep perineal Root of the penis is found in the superficial TIMOTHY MIDTERM 1
space in males? perineal space. TANG LEE EXAM - FEB
A. Root of the penis SAY, MD 2014
B. Bulbourethral gland (TOP 4 - AUG
C. Dorsal nerve of the penis 2013 MED
D. Sphincter urethra BOARDS;
E. Deep transverse perineal muscle TOPNOTCH
MD)

601 Adult venous system is derived from the following Three Pairs of Veins are present in embryo: RACHELLE FINAL EXAM
embryonic structures, EXCEPT: vitelline veins, umbilical veins, common cardinals MENDOZA, - FEB 2013
A. Vitelline vein veins. Common Cardinal Veins collect the MD (TOP 9 -
B. Umbilical vein deoxygenated blood from the body wall. Umbilical AUG 2012
C. Cardinal vein Veins carrying the oxygenated blood from the MED
D. Sinus venosus Placenta. Vitelline veins bring the deoxygenated BOARDS;
E. None of the above blood from the Yolk Sac & gut TOPNOTCH
MD)

602 EG, 47-year old female, known hypertensive and diabetic, Bell's palsy is a peripheral facial nerve palsy, RACHELLE FINAL EXAM
went to your clinic due to sudden onset of left-sided facial usually presenting with inability to wrinkle MENDOZA, - FEB 2013
weakness. On examination, you noted that there was forehead, inability to close eyelid and lagging of MD (TOP 9 -
asymmetry in facial expression with lagging noted on the left facial expression on the side affected. Sensation is AUG 2012
side, blunting of left nasolabial fold and inability to close the typically intact, since facial sensation comes from MED
left eyelid. If you are considering Bell's palsy, which of the CN V (trigeminal). BOARDS;
following is consistent with this finding? TOPNOTCH
A. Loss of sensation on left side of the face MD)
B. Loss of sensation on right side of the face
C. Unable to wrinkle left side of the forehead
D. Able to wrinkle both side of the forehead
E. A and C

603 A 25-year old male presents with severe colicky lumbar pain Since the pain is felt on the lumbar area, the RACHELLE FINAL EXAM
on the right. KUB was done, revealing a radioluscent stone, highest area of anatomic ureteral constriction MENDOZA, - FEB 2013
that is most likely lodged where? should be considered in this patient. MD (TOP 9 -
A. Mid-ureter AUG 2012
B. Point where ureter enters the pelvic brim MED
C. Ureteropelvic junction BOARDS;
D. Point where ureter enters the bladder TOPNOTCH
E. Any of the above MD)

TOPNOTCH MEDICAL BOARD PREP ANATOMY SUPEREXAM Page 79 of 94


For inquiries visit www.topnotchboardprep.com.ph or email us at topnotchmedicalboardprep@gmail.com
TOPNOTCH MEDICAL BOARD PREP ANATOMY SUPEREXAM
For inquiries visit www.topnotchboardprep.com.ph or email us at topnotchmedicalboardprep@gmail.com
Item QUESTION EXPLANATION AUTHOR TOPNOTCH
# EXAM
604 Pringle maneuver involves compression of which of the Pringle manuever is used to control bleeding RACHELLE FINAL EXAM
following? during a liver procedure without compromising MENDOZA, - FEB 2013
A. Portal vein too much of its blood supply. It involves MD (TOP 9 -
B. Bile duct compression of the heaptic arty, as most of the AUG 2012
C. Hepatic artery liver's blood supply would come from the portal MED
D. All of the above vein. BOARDS;
TOPNOTCH
MD)
605 Femoral sheath encloses the following, EXCEPT: The femoral sheath does not enclose the femoral RACHELLE FINAL EXAM
A. Femoral nerve nerve. MENDOZA, - FEB 2013
B. Femoral artery MD (TOP 9 -
C. Femoral vein AUG 2012
D. Lymph vessels MED
E. None of the above BOARDS;
TOPNOTCH
MD)
606 The following structures pass through the esophageal All of these pass through the esophageal aperture, RACHELLE FINAL EXAM
perture of the diaphragm, EXCEPT: alongside esophagus (T10 level) MENDOZA, - FEB 2013
A. Right vagus MD (TOP 9 -
B. Left gastric vesses AUG 2012
C. Esophagus MED
D. Lymphatics from lower 3rd of esophagus BOARDS;
E. None of the above TOPNOTCH
MD)

607 A 26-year old male came from a fist fight and was brought to All EOMs have a common origin at the annulus of RACHELLE FINAL EXAM
your clinic due to difficulty moving the eyeball. MRI revealed zinn except for 2: superior oblique (roof) and MENDOZA, - FEB 2013
isolated rupture of annulus of zinn. Which of the following inferior oblique (floor) MD (TOP 9 -
eye muscles whose origin would remain unaffected? AUG 2012
A. Lateral rectus MED
B. Superior rectus BOARDS;
C. Inferior rectus TOPNOTCH
D. Superior oblique MD)
E. None of the above

608 During thyroidectomy, the first major blood vessel to be The anterior jugular vein descends through the RACHELLE FINAL EXAM
encountered is: midline and joins the opposite vein via the jugular MENDOZA, - FEB 2013
A. Anterior jugular vein notch just above the sternum. This will then join MD (TOP 9 -
B. External jugular vein the external jugular vein deep to the SCM. AUG 2012
C. Internal jjugular vein MED
D. Subclavian vein BOARDS;
E. A and B TOPNOTCH
MD)

609 Most common blood supply of the SA node comes from the: 90% of the time, the SA node is supplied with a RACHELLE FINAL EXAM
A. Left coronary artery nodal branch coming from the right coronary MENDOZA, - FEB 2013
B. Right coronary artery artery. MD (TOP 9 -
C. Left circumflex artery AUG 2012
D. Right marginal artery MED
E. A and C BOARDS;
TOPNOTCH
MD)
610 If the 3rd branchial arch fails to form or mature, which of the The 3rd branchial arch gives rise to the greater RACHELLE FINAL EXAM
following clinical manifestation would be observed? horn of the hyoid, stylopharyngeus muscle and CN MENDOZA, - FEB 2013
A. No facial expression would be appreciated IX. Therefore, if it fails to form or mature to adult MD (TOP 9 -
B. Deafness structures, patient would be expected to present AUG 2012
C. Hoarseness with deficiency of the greater horn of the hyoid MED
D. Decreased secretion of parotid gland and lack of stylopharynegus muscle. And since CN BOARDS;
E. All of the above IX would be absent, there would be also be poor TOPNOTCH
gag reflex and decreased parotid gland secretion. MD)

611 The following layers of the anterior abdominal wall give rise External oblique gives rise to external spermatic RACHELLE FINAL EXAM
to an equivalent covering of scrotal and spermatic fascia, fascia. Internal oblique gives rise to cremasteric MENDOZA, - FEB 2013
EXCEPT: muscle and fascia. Transversalis fascia gives rise to MD (TOP 9 -
A. Subcutaneous fascia internal spermatic fascia. Transversus abdominis AUG 2012
B. External oblique does not contributw to any portion of the scrotum. MED
C. Internal oblique BOARDS;
D. Transversus abdominis TOPNOTCH
E. A and C MD)

612 A 35-year old male presented with enlarged scrotum on the Varicocole is the engorgement of the venous RACHELLE FINAL EXAM
left. Examination of the scrotal content revealed appearance drainage of the testis, the pampiniform plexus. It MENDOZA, - FEB 2013
of "bag of worms." Which of the following clinical usually presents with enlargement of the scrotum MD (TOP 9 -
condition/s is/are considered? and examination would reveal "bag of worms" AUG 2012
A. Varicocoele appearance. Varicocoel is more commonly MED
B. Orchitis observed in the left scrotum, which drains to the BOARDS;
C. Renal carcinoma left renal vein and is subjected to higher pressure TOPNOTCH
D. B and C than the right. Hence, varicocoele is usually MD)
E. A and C associated with renal tumors or other pathologies
that may increase renal vein pressure.

TOPNOTCH MEDICAL BOARD PREP ANATOMY SUPEREXAM Page 80 of 94


For inquiries visit www.topnotchboardprep.com.ph or email us at topnotchmedicalboardprep@gmail.com
TOPNOTCH MEDICAL BOARD PREP ANATOMY SUPEREXAM
For inquiries visit www.topnotchboardprep.com.ph or email us at topnotchmedicalboardprep@gmail.com
Item QUESTION EXPLANATION AUTHOR TOPNOTCH
# EXAM
613 Which enteroendocrine cells produce a compound that I cells produce cholecystokinin, S cells produce RACHELLE FINAL EXAM
promotes insulin secretion and inhibits glucagon secretin, K cells produce gastrin inhinitory peptide MENDOZA, - FEB 2013
production? and L cells produce glucagon-like peptide 1 (GLP MD (TOP 9 -
A. L cells 1). The latter stimulate insulin production and AUG 2012
B. S cells inhibits glucagon secretion. MED
C. K cells BOARDS;
D. I cells TOPNOTCH
E. None of the above MD)

614 An athlete suffered from fracture of his left medial malleolus, Structures running behind the medial malleolus: RACHELLE FINAL EXAM
transecting all structures behind it. Which of the following tibialis poetrior, flexor digitorum longus, posterior MENDOZA, - FEB 2013
will NOT be affected? tibial artery, posterior tibial nerve and flexor MD (TOP 9 -
A. Tibialis posterior hallocis longus. AUG 2012
B. Posterior tibial artery MED
C. Posterior tibial nerve BOARDS;
D. Flexor hallucis longus TOPNOTCH
E. Deep peroneal nerve MD)

615 Tranverse fracture of the humeral mid-shaft would result to: Tranverse fracture of the mid-shaft of the RACHELLE FINAL EXAM
A. Loss of sensation on anterior forearm humerus would injure the deep brachial artery MENDOZA, - FEB 2013
B. Claw hand and the radial nerve. The radial nerve typically MD (TOP 9 -
C. Ape hand supplies posterior compartments of arm and AUG 2012
D. Inability to extend wrist forearm and thereby resposible for allowing MED
E. A and C extension of elbow and wrist. BOARDS;
TOPNOTCH
MD)
616 A stroke patient was found to have a necrotic focus on the Brodmann areas 44 and 45 correspond to Broca's RACHELLE FINAL EXAM
brain, specifically identified as Brodmann areas 44 and 45. speech area. MENDOZA, - FEB 2013
The patient is then expected to: MD (TOP 9 -
A. have difficulty in speaking but can comprehend AUG 2012
B. have difficulty understanding but can speak MED
C. not be able to hear and interpret sounds BOARDS;
D. not be able to associate visual input to other information TOPNOTCH
E. none of the above MD)

617 A trauma patient was found to have total obliteration of CN V-2 (maxillary branch of the trigeminal nerve) RACHELLE FINAL EXAM
foramen rotundum. Which of the following clinical condition passes through foramen rotundum. The maxillary MENDOZA, - FEB 2013
would be a finding in this patient? branch is resposnible for somatic sensory supply MD (TOP 9 -
A. Loss of sensation along the maxillary area of the face of the maxillary area of the face and mouth. CN V- AUG 2012
B. Loss of sensation along the mandibular area of the face 3 passes through foramen ovale, while the middle MED
C. Epidural hemorrhage meningeak artery passes through foramen BOARDS;
D. A and C spinosum. TOPNOTCH
E. B and C MD)

618 The widest and most dilatable portion of the male urethra: Prostatic urethra is the widest and most dilatable. RACHELLE FINAL EXAM
A. Membranous Membranous part is the shortest and least MENDOZA, - FEB 2013
B. Spongy dilatable. The penile/spongy urethra is the MD (TOP 9 -
C. Penile narrowest part of the entire urethre. AUG 2012
D. Prostatic MED
E. B and C BOARDS;
TOPNOTCH
MD)
619 Arthrocentesis is most difficult to carry out in which joint? Arthrocentesis of the ankle is more difficult than RACHELLE FINAL EXAM
A. Knee that of the other joints. The knee is the most MENDOZA, - FEB 2013
B. Elbow common and the easiest joint for the physician to MD (TOP 9 -
C. Ankle aspirate AUG 2012
D. Wrist MED
E. A and C BOARDS;
TOPNOTCH
MD)
620 Histopath cross-section of the gallbladder shows the The gallbladder is lined with tall columnar cells RACHELLE FINAL EXAM
following features, EXCEPT: with microvilli, except the terminal ducts MENDOZA, - FEB 2013
A. Tall columnar epithelium with microvilli (cuboidal epithelium). No muscularis mucosae MD (TOP 9 -
B. Muscularis mucosae can be found. AUG 2012
C. Muscularis MED
D. Adventitia BOARDS;
E. A and C TOPNOTCH
MD)

621 A part of anterior pituitary gland which has a small collar VON ANDRE DIAGNOSTIC
cells around infundibular stalk that mainly contains the MEDINA, MD EXAM - AUG
portal venules. (TOP 4 - FEB 2012
A. Pars Distalis 2012 MED
B. Pars Intermedia BOARDS;
C. Pars Tuberalis TOPNOTCH
D. Pars nervosa MD)
E. None of the above

TOPNOTCH MEDICAL BOARD PREP ANATOMY SUPEREXAM Page 81 of 94


For inquiries visit www.topnotchboardprep.com.ph or email us at topnotchmedicalboardprep@gmail.com
TOPNOTCH MEDICAL BOARD PREP ANATOMY SUPEREXAM
For inquiries visit www.topnotchboardprep.com.ph or email us at topnotchmedicalboardprep@gmail.com
Item QUESTION EXPLANATION AUTHOR TOPNOTCH
# EXAM
622 Which portion or zone of the liver acinus is exposed to blood periportal= exposed to blood high in O2, involved VON ANDRE DIAGNOSTIC
high in oxygen content and nutrients , and is primarily in eclampsia Midzonal= intermediate, involved MEDINA, MD EXAM - AUG
involved in the pathology of preeclampsia /eclampsia? in yellow fever centrilobular= (TOP 4 - FEB 2012
A. Centrilobular exposed to blood low in O2, involved in ischemic 2012 MED
B. Periportal injury, right sided heart failure BOARDS;
C. Midzonal TOPNOTCH
D. Space of Disse MD)
E. Sinusoids

623 The inferior parathyroid glands are derived from which In humans, the superior parathyroid glands are VON ANDRE DIAGNOSTIC
branchial pouch? derived from the 4th branchial pouch while the MEDINA, MD EXAM - AUG
A. 1st inferior parathyroid glands come from 3rd (TOP 4 - FEB 2012
B. 2nd branchial pouch. 2012 MED
C. 3rd BOARDS;
D. 4th TOPNOTCH
E. A and B MD)

624 Diaphragm is the most important muscle of inspiration. embryonic origin


VON ANDRE DIAGNOSTIC
Which embryonic structure gives rise to the central tendon Septum transversum= central tendon MEDINA, MD EXAM - AUG
of the diaphragm which is fused partially with the fibrous Pleuroperitoneal membrane= bulk of muscular (TOP 4 - FEB 2012
pericardium of the heart? part of diaphragm Body wall mesenchyme= 2012 MED
A. Pleuroperitoneal membrane peripheral muscular part of the diaphragm
BOARDS;
B. Dorsal mesentery of the esophagus dorsal mesentery= diphragmatic crura TOPNOTCH
C. Septum transversum MD)
D. Body wall mesenchyme
E. All of the above

625 This structure encloses the kidney and suprarenal gland and renal fascia is also known as gerota's fascia VON ANDRE DIAGNOSTIC
blends with the fascia of IVC and aorta. MEDINA, MD EXAM - AUG
A. Fibrous capsule (TOP 4 - FEB 2012
B. Perirenal fat 2012 MED
C. Gerota's fascia BOARDS;
D. Renal fascia TOPNOTCH
E. C and D MD)

626 A 46 year old patient came to emergency department due to sternal border= right ventricle VON ANDRE DIAGNOSTIC
gunshot wound. During physical examination, it was found MEDINA, MD EXAM - AUG
out that the bullet entered the sternal area. Which chamber (TOP 4 - FEB 2012
of the heart will most probably be injured in this case? 2012 MED
A. Right atrium BOARDS;
B. Right ventricle TOPNOTCH
C. Left ventricle MD)
D. Left atrium
E. Diaphragm

627 An aneurysm of the abdominal aorta at the aortic hiatus of the aortic hiatus of the diaphragm transmits the VON ANDRE DIAGNOSTIC
the diaphragm is most likely to result in the compression of azygous vein and thoracic duct. MEDINA, MD EXAM - AUG
which of the following pairs of structures? (TOP 4 - FEB 2012
A. Vagus nerve and azygous vein 2012 MED
B. Esophagus and vagus nerve BOARDS;
C. Azygous vein and thoracic duct TOPNOTCH
D. Thoracic duct and vagus nerve MD)
E. Inferior vena cava and phrenic nerve

628 During appendectomy performed at McBurney's point, Iliohypogastric nerve runs medially and inferiorly VON ANDRE DIAGNOSTIC
which of the following structures is most likely to be injured? between the internal oblique and transverse MEDINA, MD EXAM - AUG
A. Deep circumflex femoral artery abdominal muscles at Mcburney's point. (TOP 4 - FEB 2012
B. Inferior epigastric artery 2012 MED
C. Iliohypogastric nerve BOARDS;
D. Genitofemoral nerve TOPNOTCH
E. Spermatic cord MD)

629 As the uterine artery passes from the internal iliac artery to the ureter runs under the uterine artery near the VON ANDRE DIAGNOSTIC
the uterus, it crosses which of the following structures that is cervix; thus, the ureter is sometimes mistakenly MEDINA, MD EXAM - AUG
sometimes mistakenly ligated during pelvic surgery? ligated during pelvic surgery (TOP 4 - FEB 2012
A. Ovarian artery 2012 MED
B. Ovarian ligament BOARDS;
C. Uterine tube TOPNOTCH
D. Ureter MD)
E. Round ligament of the uterus

TOPNOTCH MEDICAL BOARD PREP ANATOMY SUPEREXAM Page 82 of 94


For inquiries visit www.topnotchboardprep.com.ph or email us at topnotchmedicalboardprep@gmail.com
TOPNOTCH MEDICAL BOARD PREP ANATOMY SUPEREXAM
For inquiries visit www.topnotchboardprep.com.ph or email us at topnotchmedicalboardprep@gmail.com
Item QUESTION EXPLANATION AUTHOR TOPNOTCH
# EXAM
630 During a game, a 25 year old baseball player receives a VON ANDRE DIAGNOSTIC
severe blow to the head that fractures the optic canal. Which MEDINA, MD EXAM - AUG
of the following pairs of structures is most likely to be (TOP 4 - FEB 2012
damaged? 2012 MED
A. Optic nerve and ophthalmic vein BOARDS;
B. Ophthalmic vein and ophthalmic nerve TOPNOTCH
C. Ophthalmic artery and and optic nerve MD)
D. Ophthalmic nerve and optic nerve
E. Ophthalmic artery and ophthalmic vein

631 What is the lining epithelium of the endocervical canal? the cervix has a simple columnar epithelium, VON ANDRE DIAGNOSTIC
A. Simple columnar except for the external portion which is continous MEDINA, MD EXAM - AUG
B. Simple squamous with the vagina which has a stritified squamous (TOP 4 - FEB 2012
C. Stratified squamous epithelium 2012 MED
D. Cuboidal BOARDS;
E. Pseudostratified columnar TOPNOTCH
MD)

632 Communication of the scala vestibuli and scala tympani the scala vestibuli and the scala tympani are VON ANDRE DIAGNOSTIC
occurs at the: actually one perilymphatic space separated by the MEDINA, MD EXAM - AUG
A. Oval window cochlear duct (scala media). The scala vestibuli (TOP 4 - FEB 2012
B. Round window and tympani communicate with each other at 2012 MED
C. Helicotrema helicotrema. BOARDS;
D. Endolymphatic sac TOPNOTCH
E. None of the above MD)

633 Which of the following is a characteristic of the cornea? the cornea is the transparent anterior portion of VON ANDRE DIAGNOSTIC
A. It represents the anterior portion of the tunica vasculosa the tunica fibrosa, the outer covering of the eye; MEDINA, MD EXAM - AUG
B. It is the anterior transparent portion of the tunica fibrosa thus it forms the anterior wall of the anterior (TOP 4 - FEB 2012
C. It forms the anterior boundary of the posterior chamber chamber of the eye. 2012 MED
of the eye BOARDS;
D. It is devoid of nerve endings TOPNOTCH
E. All of the above MD)

634 In a surgical procedure in which you are to remove the Cystic artery arises mainly from right hepatic VON ANDRE DIAGNOSTIC
gallbladder and therefore you need to ligate the cystic artery. artery MEDINA, MD EXAM - AUG
This artery arises from which of the following? (TOP 4 - FEB 2012
A. Proper hepatic artery 2012 MED
B. Right hepatic artery BOARDS;
C. Left hepatic artery TOPNOTCH
D. Right gastric artery MD)
E. Common hepatic artery

635 Mrs. Succor suffered from an embolic infarct. She came in to VON ANDRE DIAGNOSTIC
the emergency department complaining of difficulty in MEDINA, MD EXAM - AUG
speaking and left arm weakness. Which of the following (TOP 4 - FEB 2012
arteries is most likely affected? 2012 MED
A. Middle cerebral artery BOARDS;
B. Anterior cerebral artery TOPNOTCH
C. Vertebral artery MD)
D. Posterior cerebal artery
E. Basilar artery

636 The following veins drain into the coronary sinus EXCEPT: great cardiac vein, small cardiac vein and middle VON ANDRE DIAGNOSTIC
A. Great cardiac vein cardiac vein drain into the coronary sinus. MEDINA, MD EXAM - AUG
B. Small cardiac vein (TOP 4 - FEB 2012
C. Middle cardiac vein 2012 MED
D. Anterior cardiac vein BOARDS;
E. None of the above TOPNOTCH
MD)

637 Mrs. Jennifer Marikina is suffering from an ovarian VON ANDRE DIAGNOSTIC
malignancy with hepatic metastasis. After 3 days, you MEDINA, MD EXAM - AUG
noticed that her abdomen is getting bigger. On physical (TOP 4 - FEB 2012
examination, there is a positive fluid wave which is highly 2012 MED
suggestive that the patient has ascites. Her attending BOARDS;
physician decided to do a paracentesis. The PREFERRED TOPNOTCH
entry site for paracentesis is? MD)
A. Halfway between the umbilicus and the pubic symphysis
B. Between umbilicus and ASIS, lateral to rectus abdominis
muscle, Left
C. Between umbilicus and ASIS, lateral to the restus
abdominis muscle, Right
D. Between umbilicus and ASIS, medial to the Rectus
abdominis
E. Any of the above

TOPNOTCH MEDICAL BOARD PREP ANATOMY SUPEREXAM Page 83 of 94


For inquiries visit www.topnotchboardprep.com.ph or email us at topnotchmedicalboardprep@gmail.com
TOPNOTCH MEDICAL BOARD PREP ANATOMY SUPEREXAM
For inquiries visit www.topnotchboardprep.com.ph or email us at topnotchmedicalboardprep@gmail.com
Item QUESTION EXPLANATION AUTHOR TOPNOTCH
# EXAM
638 A 58 year old G8P8 underwent TAHBSO with bilateral lymph VON ANDRE DIAGNOSTIC
node dissection for the high grade cervical cancer. Few hours MEDINA, MD EXAM - AUG
post-surgery, she complained that she cannot feel her right (TOP 4 - FEB 2012
inner leg. On your physical examination, impaired right thigh 2012 MED
adduction was observed. The iatrogenically damaged BOARDS;
structure is most likely the: TOPNOTCH
A. Femoral nerve MD)
B. Sciatic nerve
C. Obturator nerve
D. Pudendal nerve
E. Superior gluteal nerve

639 The following structures are parts of the osteon, EXCEPT: VON ANDRE DIAGNOSTIC
A. Osteocytes MEDINA, MD EXAM - AUG
B. Concentric Lamellae (TOP 4 - FEB 2012
C. Canaliculi 2012 MED
D. Volkmann's canal BOARDS;
E. A and B TOPNOTCH
MD)

640 The structure which has a head and a base that are united by SIMILAR TO PREVIOUS BOARD EXAM VON ANDRE DIAGNOSTIC
two limbs and is connected to the oval window is the? CONCEPT/PRINCIPLE MEDINA, MD EXAM - AUG
A. Incus (TOP 4 - FEB 2012
B. Malleus 2012 MED
C. Stapes BOARDS;
D. Stapedius TOPNOTCH
E. Tensor Tympani MD)

641 All of the following are contained in the vertebral canal, Spinal Nerve is located OUTSIDE the vertebral LITO JAY DIAGNOSTIC
EXCEPT? canal (exit throught intervertebral foramen) MACARAIG, EXAM - AUG
A. Spinal cord MD (TOP 8 - 2013
B. Spinal nerve FEB 2013
C. Dorsal nerve roots MED
D. Ventral nerve roots BOARDS;
E. Meninges TOPNOTCH
MD)

642 Which of the following is/are true regarding the "yes" joint? The atlanto-occipital joint is found between the C1 LITO JAY DIAGNOSTIC
A. Atlanto-occipital joint and occipital condyles. It is also called the "yes MACARAIG, EXAM - AUG
B. Atlanto-axial joint joint" because it facilitates extension and flexion of MD (TOP 8 - 2013
C. Joint between C1 and occipital condyles the neck. FEB 2013
D. Joint between C1 and C2 MED
E. choices A and C are both correct BOARDS;
F. choices B and D are both correct TOPNOTCH
MD)

643 The vertebral levels are used as reference points for locating At the level of C4 vertebra, hyoid bone and LITO JAY DIAGNOSTIC
anatomical landmarks. Which among the following choices bifurcation of common carotid artery are found. MACARAIG, EXAM - AUG
is/are found at the level of C4 vertebra? The start of trachea and Cricoid cartilage are MD (TOP 8 - 2013
A. Hyoid bone found at the level of C6 vertebra. FEB 2013
B. Start of trachea MED
C. Cricoid cartilage BOARDS;
D. choices A and B are both correct TOPNOTCH
E. choices B and C are both correct MD)

644 This is a condition which resulted from chronic stress Spondylolisthesis involves degeneration of LITO JAY DIAGNOSTIC
fracture of Pars interarticularis seen as "collar around the pedicles of lumbar vertebra. Osteomyelitis MACARAIG, EXAM - AUG
neck of Scottie dog" on Xray. Usually at the L5 vertebra of involves infection within vertebral bodies. AS MD (TOP 8 - 2013
adolescent athletes. involves the bamboo spine deformity. HNP is the FEB 2013
A. Spondylosis protrusion of Nucleus Pulposus due to a break on MED
B. Spondylolisthesis the annulus fibrosus. BOARDS;
C. Ankylosing spondylitis TOPNOTCH
D. Osteoyelitis MD)
E. Herniation of Nucleus Pulposus

645 During an ER duty, a 24 y/o male patient was brought in due The ideal site for lumbar puncture is 1 level above LITO JAY DIAGNOSTIC
to seizure. History revealed severe ear pain 2 weeks prior to or below the spinous process of L4 vertebra MACARAIG, EXAM - AUG
consult, followed by occasional headaches. Four days PTC, (between L3-L4 or between L4-L5). MD (TOP 8 - 2013
patient complained of fever 38-39 degrees Celsius. Upon PE, FEB 2013
patient had (+) meningeal signs. You are entertaining MED
bacterial meningitis. What vertebral level is ideal to insert BOARDS;
the needle for Lumbar Tap? TOPNOTCH
A. between L1 and L2 MD)
B. between L2 and L3
C. between L3 and L4
D. choices A and B are both correct
E. choices B and C are both correct

TOPNOTCH MEDICAL BOARD PREP ANATOMY SUPEREXAM Page 84 of 94


For inquiries visit www.topnotchboardprep.com.ph or email us at topnotchmedicalboardprep@gmail.com
TOPNOTCH MEDICAL BOARD PREP ANATOMY SUPEREXAM
For inquiries visit www.topnotchboardprep.com.ph or email us at topnotchmedicalboardprep@gmail.com
Item QUESTION EXPLANATION AUTHOR TOPNOTCH
# EXAM
646 A 32 year-old male patient was brought in the ER due to The Right Ventricle is the chamber of the heart LITO JAY DIAGNOSTIC
stabbing. Patient was hypotensive and bleeding profusely that lies directly behind the sternum. MACARAIG, EXAM - AUG
from the chest. Despite all efforts of resuscitation, patient MD (TOP 8 - 2013
subsequently expired. If the wound was located between the FEB 2013
4th and 5th rib Left parasternal line, the chamber of the MED
heart that is most likely affected was? BOARDS;
A. Left Atrium TOPNOTCH
B. Left Ventricle MD)
C. Right Atrium
D. Right Ventricle

647 Volksmann Ischemic Contracture is characterized by severe In a Volksmann ischemic contracture, the fracture LITO JAY DIAGNOSTIC
flexion of the affected arm, wrist and fingers. The following is found at the supracondylar area of humerus and MACARAIG, EXAM - AUG
structures are affected EXCEPT? NOT at the midshaft. MD (TOP 8 - 2013
A. Midshaft of humerus FEB 2013
B. Brachial artery MED
C. Median nerve BOARDS;
D. A and B TOPNOTCH
E. B and C MD)

648 The following are the boundaries of the Anatomical Snuffbox, The first three choices are the ONLY boundaries of LITO JAY DIAGNOSTIC
EXCEPT? the Anatomical Snuffbox. MACARAIG, EXAM - AUG
A. Extensor Pollicis Longus MD (TOP 8 - 2013
B. Extensor Pollicis Brevis FEB 2013
C. Abductor Pollicis Longus MED
D. Abductor Pollicis Brevis BOARDS;
E. There is NO exception TOPNOTCH
MD)

649 These granulocytes are described with "bilobulated nucleus" Eosinophils are characterized by their bilobulated LITO JAY DIAGNOSTIC
and plays roles in fighting viral infections, as proved by the nucleus with numerous pink-orange granules. MACARAIG, EXAM - AUG
RNAses they contain. They are highly involved in Asthma MD (TOP 8 - 2013
A. Basophil pathophysiology and Helminthic infections. They FEB 2013
B. Eosinophil contain RNAses which proves their effectivity MED
C. Neutrophil against viral infections. BOARDS;
D. NK cells TOPNOTCH
E. Lymphocytes MD)

650 While jogging, a 40 y/o male was bumped by a tricycle on the The common peroneal nerve is the root of LITO JAY DIAGNOSTIC
lateral side of his left leg. Patient was unable to evert his left superficial and deep peroneal nerve. If it was MACARAIG, EXAM - AUG
foot, with weakness in plantar flexion but normal affected, signs of deep peroneal nerve lesion must MD (TOP 8 - 2013
dorsiflexion. What nerve was affected? also be present (like weak or no dorsiflexion). The FEB 2013
A. Common Peroneal Nerve superficial peroneal nerve is responsible for MED
B. Superficial Peroneal Nerve eversion and plantar flexion. Tibial nerve is for BOARDS;
C. Deep Peroneal Nerve plantar flexion only. Common fibular nerve is the TOPNOTCH
D. Tibial Nerve other term for common peroneal nerve. MD)
E. Common Fibular Nerve

651 You are assigned in the Neuro-OPD and saw a patient who There are 3 anatomical lobes of the cerebellum. LITO JAY DIAGNOSTIC
presented with gait instability. Patient was NOT able to The anterior lobe (paleo-cerebellum), MACARAIG, EXAM - AUG
perform the "heel-to-shin" movement. You know that Flocculonodular lobe (archi-cerebellum), and the MD (TOP 8 - 2013
Cerebellum is involved. But what lobe? posterior lobe (neo-cerebellum). Anterior lobe FEB 2013
A. Anterior lobe lesions will result to gait instability and patient MED
B. Flocculo-nodular lobe will be unable to perform the heel-to-shin BOARDS;
C. Posterior lobe movement. TOPNOTCH
D. Archi-cerebellum MD)
E. Neo-cerebellum

652 Which among the following spinal cord lesions will present In ALS and Brown-Sequard (hemisection), both LITO JAY DIAGNOSTIC
as both upper and lower motor neuron lesion? the cortico-spinal tract (Upper MN) and the MACARAIG, EXAM - AUG
A. Poliomyelitis anterior horn cells (Lower MN) are affected. MD (TOP 8 - 2013
B. Amyotrophic Lateral Sclerosis FEB 2013
C. Brown-sequard MED
D. Choices A and B are both correct BOARDS;
E. Choices B and C are both correct TOPNOTCH
MD)

653 The muscles of facial expression are derived from? Derivatives of pharyngeal Arch: 1. Muscles of LITO JAY DIAGNOSTIC
A. 1st Pharyngeal Pouch Mastication, 2. Muscles pf Facial expression, 3. MACARAIG, EXAM - AUG
B. 1st Pharyngeal Arch Muscles of deglutition, 4-6. Muscles for Phonation MD (TOP 8 - 2013
C. 2nd Pharyngeal Pouch FEB 2013
D. 2nd Pharyngeal Arch MED
E. None of the above BOARDS;
TOPNOTCH
MD)

TOPNOTCH MEDICAL BOARD PREP ANATOMY SUPEREXAM Page 85 of 94


For inquiries visit www.topnotchboardprep.com.ph or email us at topnotchmedicalboardprep@gmail.com
TOPNOTCH MEDICAL BOARD PREP ANATOMY SUPEREXAM
For inquiries visit www.topnotchboardprep.com.ph or email us at topnotchmedicalboardprep@gmail.com
Item QUESTION EXPLANATION AUTHOR TOPNOTCH
# EXAM
654 The internal carotid artery (ICA) supplies the anterior part of ICA is a derivative of the distal part of the 3rd LITO JAY DIAGNOSTIC
the brain through the Circle of Willis. The ICA is a derivative Aortic Arch. The proximal part will give rise to the MACARAIG, EXAM - AUG
of? Common carotid artey. MD (TOP 8 - 2013
A. Proximal Part of the 3rd Aortic arch FEB 2013
B. Distal Part of the 3rd Aortic Arch MED
C. Right part of the 4th Aortic Arch BOARDS;
D. Left part of the 4th Aortic Arch TOPNOTCH
E. None of the above MD)

655 The genioglossus is an extrinsic muscle of the tongue and is The genioglossus is innervated by CN XII and LITO JAY DIAGNOSTIC
responsible for what movement? protrudes the tongue. MACARAIG, EXAM - AUG
A. elevation MD (TOP 8 - 2013
B. depression FEB 2013
C. protrusion MED
D. retraction BOARDS;
E. None of the above TOPNOTCH
MD)

656 A patient with Mallory Weiss tear came to ER due to severe The Mallory Weiss tear is most commonly located LITO JAY DIAGNOSTIC
hematemesis, pulsating in character. Knowing the diagnosis, at the gastro-esophageal junction. The bleeding is MACARAIG, EXAM - AUG
you already know that the bleeding blood vessel is? described as "pulsating", hence arterial. And the MD (TOP 8 - 2013
A. Middle esophageal artery blood supply of the distal 3rd of esophagus is the FEB 2013
B. Middle esophageal vein Left gstric artery which is a branch of the celiac MED
C. Left gastric artery trunk. BOARDS;
D. Left gastric vein TOPNOTCH
E. Descending aorta MD)

657 Which among the following statements is/are true regarding Jejunum is shorter, thicker, more vascular and LITO JAY DIAGNOSTIC
the difference between Jejunum and Ileum? with long vasa recta. Ileum has more fat, arcades, MACARAIG, EXAM - AUG
A. Jejunum is shorter and lymph nodes. MD (TOP 8 - 2013
B. Ileum has more fat FEB 2013
C. Jejunum is thicker MED
D. B and C only BOARDS;
E. All of the above TOPNOTCH
MD)

658 Which among the following statements is/are true regarding The round ligament of the liver is AKA the LITO JAY DIAGNOSTIC
the round ligament of the liver? Ligamentum Teres. It is a derivative from the left MACARAIG, EXAM - AUG
A. AKA falciform ligament umbilical vein. The right umbilical vein obliterates MD (TOP 8 - 2013
B. AKA ligamentum teres without any derivative in the later stage of life. FEB 2013
C. From the Left umbilical vein MED
D. A and C only BOARDS;
E. B and C only TOPNOTCH
MD)

659 The rectum is divided into three (3) anatomical divisions. the superior rectal artery is from IMA, the middle LITO JAY DIAGNOSTIC
And its inferior part (distal 3rd) is being supplied by a direct rectal artery is from the Internal iliac artery, and MACARAIG, EXAM - AUG
branch of? the inferior rectal artery is from the the internal MD (TOP 8 - 2013
A. Inferior Mesenteric Artery pudendal artery which is also a branch of the FEB 2013
B. Sigmoidal artery internal iliac artery MED
C. Internal pudendal artery BOARDS;
D. A and C only TOPNOTCH
E. All of the above MD)

660 Which among the following statements is/are true regarding A normal adult kidney contains 12 pyramids. 2 LITO JAY DIAGNOSTIC
the anatomy of the kidney? major calyces and 3 minor calyces. MACARAIG, EXAM - AUG
A. 12 pyramids MD (TOP 8 - 2013
B. 2 major calyces FEB 2013
C. 3 minor calyces MED
D. A and C only BOARDS;
E. All of the above TOPNOTCH
MD)

661 Which of the following is bilobed? Basophils and Eosinophils have nuclei that are HAZEL MIDTERM 2
A. neutrophils bilobed. Neutrophils are multinucleated. KAREN RAZ, - AUG 2013
B. basophils MD (TOP 6 -
C. lymphocytes FEB 2013
D. monocytes MED
E. Platelets BOARDS;
TOPNOTCH
MD)

TOPNOTCH MEDICAL BOARD PREP ANATOMY SUPEREXAM Page 86 of 94


For inquiries visit www.topnotchboardprep.com.ph or email us at topnotchmedicalboardprep@gmail.com
TOPNOTCH MEDICAL BOARD PREP ANATOMY SUPEREXAM
For inquiries visit www.topnotchboardprep.com.ph or email us at topnotchmedicalboardprep@gmail.com
Item QUESTION EXPLANATION AUTHOR TOPNOTCH
# EXAM
662 Which of the following statements is true? Bronchioles have lesser cartilage and mucous HAZEL MIDTERM 2
A. Bronchioles have cartilage plates similar to those of glands than bronchi. However, smooth muscle KAREN RAZ, - AUG 2013
bronchi. cells are greater in the distal airways. Bronchioles MD (TOP 6 -
B. Bronchioles have greater amount of smooth muscle than are formed when the airways are reduced to < 1 FEB 2013
bronchi. mm. MED
C. Bronchioles are formed when the airways are reduced to BOARDS;
a diameter of < 1 cm. TOPNOTCH
D. Bronchioles have greater number of mucous glands than MD)
bronchi.
E. None of the above.

663 The following are found inside the lacunae, except? Osteoblast are found outside of lacunar cells in HAZEL MIDTERM 2
A. Osteocytes bone. They are responsible for bone KAREN RAZ, - AUG 2013
B. Osteoclast formation.Osteoblasts that become trapped in the MD (TOP 6 -
C. Osteoblast bone matrix and remain isolated in lacunae FEB 2013
D. Chrondrocyte become osteocytes. MED
E. None of the above BOARDS;
TOPNOTCH
MD)
664 Protein production for extracellular distribution are Protein produced from the RER are transported HAZEL MIDTERM 2
produced in the: out of the cell for ectracellular use. Free ribosomes KAREN RAZ, - AUG 2013
A. Smooth ER produce protein for intracellular use. Smooth ER MD (TOP 6 -
B. Rough ER functions in cholesterol synthesis and all other FEB 2013
C. Free ribosomes food products except complex carbohydrates. MED
D. Nucleus Lysozyme produces enzymes for self - BOARDS;
E. Lysozyme desctruction. TOPNOTCH
MD)

665 In the respiratory tract, what differentiates the terminal Respiratory bronchioles begin where alveoli start HAZEL MIDTERM 2
bronchioles from the respiratory bronchioles? to appear. There are lesser smooth muscle, cilia KAREN RAZ, - AUG 2013
A. The presence of alveoli in respiratory bronchioles and greater amount of elastic fiber. MD (TOP 6 -
B. Presence of greater number of smooth muscle in FEB 2013
respiratory bronchioles MED
C. Presence of greater number of cilia in respiratory BOARDS;
bronchioles TOPNOTCH
D. Lesser elastic fibers in respiratory bronchioles MD)
E. All of the above

666 Cancer of the esophagus presents with early metastasis due Esophagus lacks serosal layer, that is why HAZEL MIDTERM 2
to the absence of which layer? esophageal cancer when diagnosed is usually in KAREN RAZ, - AUG 2013
A. epithelium the advanced stages due to early dissemination MD (TOP 6 -
B. submucosa and metastasis. FEB 2013
C. Muscularis propria MED
D. Lamina propria BOARDS;
E. Serosa TOPNOTCH
MD)

667 A 15- year old triathlethe complains of low back pain which a - spondylolisthesis, b - spondylosis, c - HAZEL MIDTERM 2
is aggravated by activity. Physical examination was normal, ankylosing spondylitis KAREN RAZ, - AUG 2013
no neurological symptoms were noted. After x-ray of his MD (TOP 6 -
lumbar spine, he was diagnosed to have spondylolysis. What FEB 2013
could be the finding which led to this diagnosis? MED
A. Degeneration of pedicles of lumbar vertebrae BOARDS;
B. Formation of bone spurs and osteophytes TOPNOTCH
C. Ossification of annulus fibrosus MD)
D. Scottie dog collar lesion on L5
E. None of the above

668 A patient came to you presenting with fever, nuchal rigidity A lumbar puncture is a puncture into the HAZEL MIDTERM 2
and changes in sensorium. Meningitis was among the subarachnoid space of the spinal cord to obtain KAREN RAZ, - AUG 2013
differentials. Lumbar tap was contemplated. At what level is cerebrospinal fluid (CSF) for clinical MD (TOP 6 -
lumbar tap usually performed? investigation, to remove excess fluid FEB 2013
A. T12 - L1 or inject medication. Lumbar tap is performed MED
B. L1 - L2 between L3 - L4 or L4 and L-5. BOARDS;
C. L3 TOPNOTCH
D. L4 - L5 MD)
E. None of the above

669 A trauma patient presented with difficulty of breathing. CXR CTT insertion is done at the 5th - 6th anterior HAZEL MIDTERM 2
showed blunting of the right costophrenic angle. You decided axillary line and directed inferiorly to drain blood KAREN RAZ, - AUG 2013
to do a CTT insertion to drain the fliud. At what level would or fluid from the thoracic cavity. The tube is MD (TOP 6 -
you insert the CTT tube? directed superiorly to drain pneumothorax. FEB 2013
A. 4th - 5th ICS R Anterior Axillary Line directed superiorly MED
B. 4th -5th ICS R Mid - axillary line directed inferiorly. BOARDS;
C. 5th - 6th ICS R Mid- axillary line directed superiorly TOPNOTCH
D. 5th - 6th ICS R Anterior Axillary Line directed inferiorly MD)
E. None of the above

TOPNOTCH MEDICAL BOARD PREP ANATOMY SUPEREXAM Page 87 of 94


For inquiries visit www.topnotchboardprep.com.ph or email us at topnotchmedicalboardprep@gmail.com
TOPNOTCH MEDICAL BOARD PREP ANATOMY SUPEREXAM
For inquiries visit www.topnotchboardprep.com.ph or email us at topnotchmedicalboardprep@gmail.com
Item QUESTION EXPLANATION AUTHOR TOPNOTCH
# EXAM
670 Amphiarthrosis: Joints are classified based on their mobility and HAZEL MIDTERM 2
A. Symphysis pubis composition. They are classified as synarthrosis KAREN RAZ, - AUG 2013
B. Radio - ulnar joint (fibrous) - immovable, amphiarthrosis MD (TOP 6 -
C. Atlanto - axial joint (cartilagenous) - slightly movable, diarthrosis FEB 2013
D. wrist (synovial) - freely movable. Symphysis pubis is a MED
E. Distal interphalangeal joints fibrocartilagenous joint that is slightly movable. BOARDS;
TOPNOTCH
MD)
671 This muscle is considered the main flexor of the forearm and The muscles innervated by musculocutaneous HAZEL MIDTERM 2
its innervation. nerve includes the following: coracobrachialis KAREN RAZ, - AUG 2013
A. Coracobrachialis muscle, musculocutaneous nerve (flexes and adducts ARM), biceps brachii (flexes MD (TOP 6 -
B. Biceps brachii muscle, musculocuteneous nerve and main supinator of FOREARM), brachialis FEB 2013
C. Brachialis muscle, musculocutaneous nerve (main flexor of the FOREARM). Deltoid muscle is MED
D. Deltoid muscle, musculocutaneous nerve innervated by axillary nerve, a part of the rotator BOARDS;
E. None of the above cuff muscles. TOPNOTCH
MD)

672 Psoas sign is considered positive if there is_____________? psoas sign is a medical sign that indicates HAZEL MIDTERM 2
A. Increased tenderness at the RLQ with palpation of the LLQ irritation to the iliopsoas group of hip flexors in KAREN RAZ, - AUG 2013
B. Pain at the RLQ with hip flexion and internal rotation of the abdomen, and consequently indicates that the MD (TOP 6 -
the leg inflamed appendix is retrocaecal in orientation (as FEB 2013
C. Pain on passive extension of the right thigh the iliopsoas muscle is retroperitoneal). It is MED
D. Pain on slow compression of the abdominal wall with elicited by performing the psoas test by passively BOARDS;
rapid release extending the thigh of a patient lying on his side TOPNOTCH
E. All of the above with knees extended, or asking the patient to MD)
actively flex his thigh at the hip.

673 A complication of radical mastectomy which presents as Injury to the long thoracic nerve produces a HAZEL MIDTERM 2
difficulty in horizontal extension of the upper extermity winged scapula due to paralysis of serratus KAREN RAZ, - AUG 2013
involves injury to the? anterior. Intercostobrachial nerve involvement MD (TOP 6 -
A. Long thoracic nerve causes loss of sensation of the upper inner arm. FEB 2013
B. Intercostobrachial nerve Paralysis of the pectoralis major muscle with MED
C. Medial pectoral nerve weakness of abduction is due to damaged medial BOARDS;
D. Thoracodorsal nerve pectoral nerve. TOPNOTCH
E. Axillary nerve MD)

674 A 60 year old smoker presents with hemoptysis, weight loss, The signs and symptoms presented are due to the HAZEL MIDTERM 2
weakness of the upper limbs, fatigue and difficulty in paraneoplastic syndromes accompanying oat KAREN RAZ, - AUG 2013
swallowing. Electrolyte levels show hyponatremia. What cell/small cell carcinoma of the lung. Lambert - MD (TOP 6 -
could be the possible diagnosis for this patient? Eaton Syndrome presents with muscle weakness, FEB 2013
A. Oat cell CA of the lung fatigue and difficulty in swallowing. SIADH is MED
B. Adenocarcinoma of the lung manifested by hyponatremia (dilutional), and BOARDS;
C. Squamous cell CA ACTH production. Smoking is a major TOPNOTCH
D. PTB predisposing factor in small cell and squamous cell MD)
E. None of the above CA.

675 Lingula is the embryonic counterpart of which lobe of the Lingula is used to denote a projection of the upper HAZEL MIDTERM 2
lung? lobe of the left lung that serves as the homologue KAREN RAZ, - AUG 2013
A. Right lower lobe of the right middle lobe. MD (TOP 6 -
B. Right upper lobe FEB 2013
C. Right middle lobe MED
D. Left Lower lobe BOARDS;
E. Left Upper lobe TOPNOTCH
MD)

676 The inferior diaphragmatic surface of the heart is composed Base/Posterior surface - Left atrium; Apex: LV at HAZEL MIDTERM 2
mainly of the______? 5th ICS MCL; Sternal border: RV; Diaphragmatic KAREN RAZ, - AUG 2013
A. Left atrium surface: LV MD (TOP 6 -
B. Left ventricle FEB 2013
C. Right atrium MED
D. Right Ventricle BOARDS;
E. Base of the heart TOPNOTCH
MD)

677 Leaflets of the valves which separates the right atrium form leaflets of the tricuspid valve: anterior, posterior, HAZEL MIDTERM 2
the right ventricle except: septal. KAREN RAZ, - AUG 2013
A. anterior MD (TOP 6 -
B. septal FEB 2013
C. medial MED
D. posterior BOARDS;
E. None of the above TOPNOTCH
MD)

678 Paracentesis sites: 2 cms below the umbillicus, HAZEL MIDTERM 2


lateral border of the rectus abdominis muscle KAREN RAZ, - AUG 2013
superior and medial to the ASIS MD (TOP 6 -
FEB 2013
MED
BOARDS;
TOPNOTCH
MD)

Most ideal site for paracentesis?


TOPNOTCH MEDICAL BOARD PREP ANATOMY SUPEREXAM Page 88 of 94
For inquiries visit www.topnotchboardprep.com.ph or email us at topnotchmedicalboardprep@gmail.com
TOPNOTCH MEDICAL BOARD PREP ANATOMY SUPEREXAM
For inquiries visit www.topnotchboardprep.com.ph or email us at topnotchmedicalboardprep@gmail.com
Item QUESTION EXPLANATION AUTHOR TOPNOTCH
# EXAM
A. midway between symphysis and umbillicus
B. 1 cm to the left of the lateral border of the rectus
abdominis at the level of the umbillicus
C. Immediately above the symphysis pubis
D. Mc Burney's point
E. above the umbillicus

679 Which of the following associations is correct? HAZEL MIDTERM 2


A. Superficialis fascia : external spermatic fascia KAREN RAZ, - AUG 2013
B. Transversalis fascia : Dartos muscle MD (TOP 6 -
C. Internal Oblique muscle : Cremasteric muscle FEB 2013
D. Peritoneum : none MED
E. None of the above BOARDS;
TOPNOTCH
MD)

680 Which of the ff structures of the renal hilus is most Order from anterior to posterior: Vein, Artery, HAZEL MIDTERM 2
posterior? Pelvis KAREN RAZ, - AUG 2013
A. Renal artery MD (TOP 6 -
B. Renal vein FEB 2013
C. Gerotas fascia MED
D. Perirenal fat BOARDS;
E. Renal pelvis TOPNOTCH
MD)
681 A 15 yo boy complaining of pain in the lower right part of the Answer: B. it is supplied by a long small artery MICHELLE MIDTERM 1
anterior abdominal wall was seen by a physician. Upon that does not anastomose with other arteries JAY - AUG 2013
examination, he was found to have a temperature of 38.3OC. (pp. 234, Snell’s Clinical Anatomy By Regions, 8th FRANCISCO,
He had a furred tongue and was extremely tender in the edition.) Notes: All choices are factors that MD (TOP 9 -
lower right quadrant. The abdominal muscles in that area contribute to the appendix’s predilection to FEB 2013
were found to be firm on palpation and became more spastic infection except choice B which is a factor for MED
when increased pressure was applied. The organ most likely predisposition to perforation. BOARDS;
affected in this case has the following contributory factors TOPNOTCH
why it is predisposed to infection, except; MD)
A. It is a long, narrow, blind-ended tube, which encourages
stasis of large-bowel contents.
B. It is supplied by a long small artery that does not
anastomose with other arteries.
C. It has a large amount of lymphoid tissue in its wall.
D. The lumen has a tendency to become obstructed by
hardened intestinal contents, which leads to further
stagnation of its contents.

682 The following statements concerning the pancreas are Answer: B. The main pancreatic duct opens into MICHELLE MIDTERM 1
correct except which? the third part of the duodenum. JAY - AUG 2013
A. The pancreas receives part of the arterial supply from the Notes: The main pancreatic duct opens into the FRANCISCO,
splenic artery. second part of the duodenum, at about its middle, MD (TOP 9 -
B. The main pancreatic duct opens into the third part of the with the bile duct on the major duodenal papilla. FEB 2013
duodenum. Sometimes, the main duct drains separately into MED
C. The uncinate process of the pancreas projects from the the duodenum. BOARDS;
head of the pancreas. TOPNOTCH
D. The bile duct (CBD) lies posterior to the head of the MD)
pancreas.

683 What part of the ciliary body does the aqueous humor is Answer: D. Pars plicata (Robbins and Cotran MICHELLE MIDTERM 1
formed? Pathologic Basis of Disease, 8th ed.) JAY - AUG 2013
A. Pars optica FRANCISCO,
B. Pars nervosa MD (TOP 9 -
C. Pars pigmentosa FEB 2013
D. Pars plicata MED
BOARDS;
TOPNOTCH
MD)
684 The intercellular spaces in the stratum spinosum of the Answer: A. Membrane-coating granules MICHELLE MIDTERM 1
epidermis contain lipid-containing sheets that are Notes: Membrane-coating granules are present in JAY - AUG 2013
impermeable to water. This material is released from keratinocytes in the stratum spinosum (and FRANCISCO,
A. Membrane-coating granules stratum granulosum). The contents of these MD (TOP 9 -
B. Keratohyalin granules granules are released into the intercellular spaces FEB 2013
C. Langerhans cells to help waterproof the skin. Keratinocytes in the MED
D. Sebaceous glands stratum granulosum also possess keratohyalin BOARDS;
granules; these contain proteins that bind keratin TOPNOTCH
filaments together. MD)

TOPNOTCH MEDICAL BOARD PREP ANATOMY SUPEREXAM Page 89 of 94


For inquiries visit www.topnotchboardprep.com.ph or email us at topnotchmedicalboardprep@gmail.com
TOPNOTCH MEDICAL BOARD PREP ANATOMY SUPEREXAM
For inquiries visit www.topnotchboardprep.com.ph or email us at topnotchmedicalboardprep@gmail.com
Item QUESTION EXPLANATION AUTHOR TOPNOTCH
# EXAM
685 Which of the following is lined by an epithelium containing Answer: B. Terminal bronchiole MICHELLE MIDTERM 1
ciliated cells and Clara cells? Notes: Terminal bronchioles are lined by a simple JAY - AUG 2013
A. Trachea cuboidal epithelium containing ciliated cells and FRANCISCO,
B. Terminal bronchiole Clara cells. Clara cells can divide and regenerate MD (TOP 9 -
C. Intrapulmonary bronchi both cell types. FEB 2013
D. Alveolar duct MED
BOARDS;
TOPNOTCH
MD)
686 Which of the following cells in the inner ear are involved in Answer: A. Hair cells in the maculae MICHELLE MIDTERM 1
detecting movements of the head? Notes: Neuroepithelial hair cells in the maculae of JAY - AUG 2013
A. Hair cells in the maculae the saccule and the utricule detect linear FRANCISCO,
B. Cells of Hensen movement of the head. These cells are connected MD (TOP 9 -
C. Hair cells in the organ of Corti to the vestibular portion of the acoustic nerve. FEB 2013
D. Inner pillar cells MED
BOARDS;
TOPNOTCH
MD)
687 The following statements concerning the stomach are true; Answer: B. The lesser curvature of the stomach MICHELLE MIDTERM 1
except receives its blood supply from the right and JAY - AUG 2013
A. The splenic artery runs along the upper border of the left gastroepiploic arteries FRANCISCO,
pancreas and lies behind the stomach Notes: The greater curvature of the stomach MD (TOP 9 -
B. The lesser curvature of the stomach receives its blood receives its blood supply from the right and left FEB 2013
supply from the right and left gastroepiploic arteries gastroepiploic arteries. MED
C. The lymph drainage from the gastroesophageal junction BOARDS;
passess to the celiac lymph nodes TOPNOTCH
D. The lesser sac lies behind the stomach MD)

688 Ductus/vas deferens is the main content of the spermatic Answer: D. Ilioinguinal nerve MICHELLE MIDTERM 1
cord. Among these choices, which is not found in the Notes: Ilioinguinal nerve travels along/on the JAY - AUG 2013
spermatic cord? superficial part of the spermatic cord. Hernia sac FRANCISCO,
A. Hernia sac may or may not be found inside the spermatic MD (TOP 9 -
B. Genital branch of the genitofemoral nerve cord. FEB 2013
C. Cremasteric muscle fibers MED
D. Ilioinguinal nerve Mnemonics for contents of the spermatic cord: BOARDS;
Piles Don’t Contribute To A Good Sex Life TOPNOTCH
• Pampiniform venous plexus MD)
• Ductus deferens
• Cremasteric muscle
• Testicular Artery
• Artery of the ductus deferens
• Genital branch of the genitofemoral nerve
• Sympathetic nerve fibers
• Lymphatic vessels

689 You asked a patient to look to the left side during a Answer: A. Right medial longitudinal fasciculus MICHELLE MIDTERM 1
neurologic examination. Patient’s right eye cannot look to his Notes: JAY - AUG 2013
left, with (+) left eye nystagmus, but convergence is still • Left abducens nucleus or right cerebral cortex – FRANCISCO,
intact. In order to produce these symptoms, you suspect that neither eye can look left with a slow drift to the MD (TOP 9 -
the lesion is most probably located in the _____________. right FEB 2013
A. Right medial longitudinal fasciculus • Left abducens nerve – left eye can’t look to the MED
B. Left abducens nucleus left BOARDS;
C. Right cerebral cortex • Right medial longitudinal fasciculus – right eye TOPNOTCH
D. Left abducens nerve can’t look left, left eye nystagmus, and MD)
convergence is intact

690 Name the ocular defect if the lesion is found in the right Answer: A. Left homonymous hemianopsia MICHELLE MIDTERM 1
lateral geniculate body in the thalamus Notes: JAY - AUG 2013
A. Left homonymous hemianopsia • Left optic nerve lesion – left eye anopsia (left FRANCISCO,
B. Left nasal and temporal hemianopsia nasal and temporal hemianopsia) MD (TOP 9 -
C. Bitemporal heteronymous hemianopsia • Right calcarine cortex lesion – left homonymous FEB 2013
D. Right nasal hemianopsia hemianopsia MED
• Right LGB lesion – left homonymous BOARDS;
hemianopsia TOPNOTCH
• Optic chiasm lesion – bitemporal heteronymous MD)
hemianopsia
• Right lateral compression of the optic chiasm (as
in aneurysms of the internal carotid artery) – right
nasal hemianopsia
691 What is true regarding lymphatic anatomy? Lymphatic system begins as network of valveless MICHELLE MIDTERM 1
A. The limb lymphatics are valveless. capillaries in the superficial dermis. RBC & bact. JAY - AUG 2013
B. The lymphatic system begins just below the dermis as a enter lymph vessels by separating endothelial cells FRANCISCO,
network of fine capillaries at their junctions. MD (TOP 9 -
C. RBC and bacteria do not enter the lymphatic capillaries FEB 2013
D. Extrinsic factors (muscle contraction, arterial pulsations, MED
respiratory movement and massage)aid in the movement of BOARDS;
lymph flow. TOPNOTCH
E. All of the above. MD)

TOPNOTCH MEDICAL BOARD PREP ANATOMY SUPEREXAM Page 90 of 94


For inquiries visit www.topnotchboardprep.com.ph or email us at topnotchmedicalboardprep@gmail.com
TOPNOTCH MEDICAL BOARD PREP ANATOMY SUPEREXAM
For inquiries visit www.topnotchboardprep.com.ph or email us at topnotchmedicalboardprep@gmail.com
Item QUESTION EXPLANATION AUTHOR TOPNOTCH
# EXAM
692 A 10-yr old girl is brought to her doctor because of Poland syndrome is a congenital defect of about 1 MICHELLE MIDTERM 1
hypoplasia of her right breast. On closer examination, she is in 30,000 births. Hallmark is chest wall anomaly: JAY - AUG 2013
found to have asymmetry of the chest wall itself. Which of partial absence of sternal head of pec.major m., FRANCISCO,
the following is true regarding this syndrome? hypo/aplasia of breast & SC tissue and possible MD (TOP 9 -
A. This affects i in every 10,000 live births. complete absence of pec.minor m. FEB 2013
B. Deformity of the thoracoacromial joint is characteristic of MED
this syndrome. BOARDS;
C. Absence of the sternum is characteristic of this syndrome TOPNOTCH
D. Absence of the sterna head of the pectoralis muscle is a MD)
characteristic of this syndrome.
E. Pectus excavatum is part of this syndrome..

693 Which of the following statements is most accurate Third arch anomalies are rare & appear in lower MICHELLE MIDTERM 1
regarding branchial cleft anomalies? neck. Second branchial arch anomalies are the JAY - AUG 2013
A. Third arch anomalies are common most common & ends at the tonsillar fossa. FRANCISCO,
B. Type II 1st branchial anomalies are the most common MD (TOP 9 -
over-all. FEB 2013
C. The glossopharyngeal nerve is asso. with the 3rd branchial MED
arch. BOARDS;
D. Second arch anomalies end in pyriform sinus. TOPNOTCH
MD)

694 True of liver anatomy: The right lobe consists of posterior and anterior MICHELLE MIDTERM 1
A. The right lobe extends to the umbilical fissure and segments. The left lobe consists of medial JAY - AUG 2013
falciform ligament. (quadrate) and lateral segment divided by FRANCISCO,
B. The left lobe end at the falciform ligament. falciform ligament. The caudate can be considered MD (TOP 9 -
C. The quadrate lobe is a portion of the medial segment of the anatomically independent from both R/L lobes FEB 2013
right lobe. because it receives portal & arterial blood supply MED
D. The lateral segments of the left lobe in the American from both sides. BOARDS;
system consists of segments II & III. TOPNOTCH
MD)

695 All except one drains into the right atrium through the The anterior cardiac vein together w/ smallest MICHELLE MIDTERM 1
coronary sinus: cardiac opens directly into the right atrium. JAY - AUG 2013
FRANCISCO,
A. small cardiac MD (TOP 9 -
B. middle cardiac FEB 2013
C. anterior cardiac MED
D. great cardiac BOARDS;
TOPNOTCH
MD)
696 The muscle and central tendon of the diaphragm develop Pleuroperitoneal membranes-peripheral area of MICHELLE MIDTERM 1
from which of the following embryonic structures? diaphragmatic pleura & upper/lower pleural JAY - AUG 2013
A. Septum transversum surface; Dorsal mesentery - crura FRANCISCO,
B. Pleuroperitoneal membranes MD (TOP 9 -
C. Dorsal mesentery of esophagus FEB 2013
D. Ventral mesentery of esophagus MED
BOARDS;
TOPNOTCH
MD)
697 The blood supply to the thoracic wall mainly comes from the These 9 pairs of post. intercostal arteries arise MICHELLE MIDTERM 1
_________ artery. from the thoracic aorta, the right arteries, having JAY - AUG 2013
A. Lateral thoracic longer course than the left ones. FRANCISCO,
B. Internal thoracic MD (TOP 9 -
C. Posterior intercostals FEB 2013
D. Subcostal MED
BOARDS;
TOPNOTCH
MD)
698 I.M. injections should be given in the upper outer quadrant of *review ana nerve distribution of lower limbs MICHELLE MIDTERM 1
the buttocks to prevent damage to which of the following JAY - AUG 2013
nerves? FRANCISCO,
A. Sciatic MD (TOP 9 -
B. Obturator FEB 2013
C. Superior gluteal MED
D. Lateral femoral cutaneous BOARDS;
TOPNOTCH
MD)
699 This ligament prevents posterior dislocation of the femur at The rectus femoris as part of quadriceps crosses MICHELLE MIDTERM 1
the knee joint. the hip joint anteriorly and its tendon crosses the JAY - AUG 2013
A. Anterior cruciate knee joint anteriorly resulting in flexion of the FRANCISCO,
B. Ischiofemoral thigh and extension of the leg. MD (TOP 9 -
C. Lateral collateral FEB 2013
D. Posterior cruciate MED
BOARDS;
TOPNOTCH
MD)

TOPNOTCH MEDICAL BOARD PREP ANATOMY SUPEREXAM Page 91 of 94


For inquiries visit www.topnotchboardprep.com.ph or email us at topnotchmedicalboardprep@gmail.com
TOPNOTCH MEDICAL BOARD PREP ANATOMY SUPEREXAM
For inquiries visit www.topnotchboardprep.com.ph or email us at topnotchmedicalboardprep@gmail.com
Item QUESTION EXPLANATION AUTHOR TOPNOTCH
# EXAM
700 A neurologic exam of a 34 yr-old man reveals a direct and MICHELLE MIDTERM 1
consensual light reflex in his left eye but neither in his right JAY - AUG 2013
eye. The nerve involved is the: FRANCISCO,
A. right optic nerve MD (TOP 9 -
B. left optic nerve FEB 2013
C. right oculomotor MED
D. left oculomotor BOARDS;
TOPNOTCH
MD)

TOPNOTCH MEDICAL BOARD PREP ANATOMY SUPEREXAM Page 92 of 94


For inquiries visit www.topnotchboardprep.com.ph or email us at topnotchmedicalboardprep@gmail.com
TOPNOTCH MEDICAL BOARD PREP ANATOMY SUPEREXAM
For inquiries visit www.topnotchboardprep.com.ph or email us at topnotchmedicalboardprep@gmail.com

Item # KEY 87 B 175 D 263 B 351 B


ANSWER 88 A 176 E 264 D 352 C
1 E 89 A 177 C 265 B 353 A
2 E 90 A 178 D 266 C 354 C
3 B 91 B 179 A 267 B 355 A
4 A 92 B 180 C 268 D 356 B
5 C 93 C 181 A 269 B 357 A
6 B 94 D 182 D 270 B 358 C
7 E 95 B 183 C 271 B 359 B
8 D 96 C 184 D 272 A 360 A
9 B 97 B 185 B 273 B 361 C
10 A 98 E 186 C 274 A 362 A
11 C 99 C 187 B 275 A 363 B
12 C 100 C 188 A 276 A 364 C
13 A 101 D 189 C 277 D 365 C
14 E 102 D 190 B 278 D 366 C
15 C 103 A 191 E 279 C 367 D
16 C 104 C 192 D 280 B 368 C
17 D 105 B 193 C 281 A 369 D
18 E 106 D 194 B 282 D 370 D
19 C 107 A 195 B 283 B 371 A
20 D 108 B 196 A 284 B 372 A
21 A 109 A 197 B 285 E 373 B
22 C 110 C 198 D 286 C 374 D
23 D 111 E 199 C 287 C 375 D
24 B 112 D 200 C 288 B 376 D
25 A 113 B 201 C 289 E 377 C
26 B 114 C 202 A 290 A 378 C
27 D 115 D 203 A 291 E 379 C
28 D 116 A 204 C 292 C 380 B
29 C 117 B 205 B 293 E 381 B
30 D 118 E 206 D 294 D 382 C
31 C 119 A 207 B 295 C 383 C
32 C 120 C 208 B 296 A 384 D
33 B 121 C 209 B 297 C 385 E
34 B 122 A 210 A 298 D 386 A
35 C 123 E 211 B 299 B 387 D
36 E 124 E 212 D 300 D 388 D
37 E 125 C 213 C 301 C 389 B
38 A 126 B 214 A 302 A 390 D
39 B 127 A 215 D 303 391 A
40 C 128 B 216 C 304 D 392 C
41 C 129 C 217 B 305 A 393 B
42 B 130 D 218 A 306 B 394 C
43 A 131 B 219 D 307 D 395 B
44 B 132 A 220 B 308 C 396 A
45 A 133 B 221 C 309 C 397 B
46 C 134 C 222 A 310 C 398 B
47 A 135 A 223 C 311 A 399 B
48 E 136 B 224 C 312 B 400 C
49 E 137 D 225 B 313 C 401 D
50 C 138 D 226 A 314 A 402 E
51 D 139 D 227 A 315 C 403 D
52 C 140 E 228 C 316 A 404 C
53 C 141 B 229 C 317 B 405 B
54 A 142 A 230 C 318 E 406 A
55 B 143 A 231 B 319 B 407 B
56 E 144 E 232 A 320 D 408 D
57 B 145 B 233 C 321 C 409 A
58 B 146 B 234 D 322 D 410 C
59 C 147 D 235 C 323 A 411 D
60 B 148 B 236 C 324 E 412 A
61 C 149 A 237 D 325 C 413 D
62 A 150 A 238 C 326 C 414 B
63 D 151 E 239 A 327 B 415 A
64 B 152 B 240 B 328 B 416 E
65 B 153 E 241 A 329 E 417 B
66 BONUS 154 A 242 C 330 B 418 E
67 E 155 B 243 A 331 E 419 B
68 D 156 B 244 C 332 C 420 B
69 B 157 B 245 C 333 C 421 B
70 C 158 A 246 D 334 D 422 A
71 B 159 C 247 C 335 D 423 B
72 B 160 D 248 D 336 C 424 C
73 D 161 D 249 B 337 B 425 D
74 E 162 A 250 B 338 E 426 B
75 B 163 B 251 B 339 B 427 B
76 D 164 B 252 B 340 C 428 D
77 C 165 E 253 C 341 C 429 B
78 A 166 A,B 254 C 342 A 430 B
79 D 167 C 255 D 343 B 431 D
80 C 168 A 256 D 344 B 432 C
81 A 169 E 257 C 345 C 433 A
82 D 170 B 258 B 346 C 434 B
83 B 171 A 259 C 347 D 435 A
84 B 172 B 260 C 348 C 436 D
85 A 173 C 261 E 349 C 437 B
86 D 174 B 262 D 350 B 438 D
TOPNOTCH MEDICAL BOARD PREP ANATOMY SUPEREXAM Page 93 of 94
For inquiries visit www.topnotchboardprep.com.ph or email us at topnotchmedicalboardprep@gmail.com
TOPNOTCH MEDICAL BOARD PREP ANATOMY SUPEREXAM
For inquiries visit www.topnotchboardprep.com.ph or email us at topnotchmedicalboardprep@gmail.com
439 C 528 C 617 A
440 D 529 C 618 D
441 C 530 D 619 C
442 E 531 C 620 B
443 B 532 B 621 C
444 B 533 B 622 B
445 C 534 D 623 C
446 A 535 D 624 C
447 D 536 B 625 E
448 D 537 A 626 B
449 E 538 C 627 C
450 D 539 B 628 C
451 B 540 D 629 D
452 A 541 B 630 C
453 E 542 C 631 A
454 C 543 C 632 C
455 D 544 C 633 B
456 E 545 B 634 B
457 B 546 D 635 A
458 D 547 D 636 D
459 C 548 C 637 B
460 D 549 A 638 C
461 D 550 B 639 D
462 B 551 A 640 C
463 A 552 E 641 B
464 D 553 C 642 E
465 C 554 B 643 A
466 C 555 A 644 A
467 A 556 C 645 C
468 A 557 D 646 D
469 A 558 A 647 A
470 D 559 A 648 D
471 D 560 E 649 B
472 A 561 A 650 B
473 B 562 A 651 A
474 D 563 A 652 E
475 E 564 B 653 D
476 A 565 C 654 B
477 E 566 B 655 C
478 A 567 A 656 C
479 A 568 C 657 E
480 C 569 A 658 E
481 A 570 A 659 C
482 B 571 C 660 E
483 C 572 B 661 B
484 A 573 B 662 B
485 C 574 A 663 C
486 D 575 A 664 B
487 D 576 C 665 A
488 A 577 B 666 E
489 B 578 A 667 D
490 B 579 B 668 D
491 C 580 B 669 D
492 A 581 B 670 A
493 B 582 C 671 C
494 D 583 C 672 C
495 C 584 B 673 D
496 B 585 A 674 A
497 D 586 E 675 C
498 C 587 A 676 B
499 B 588 D 677 C
500 C 589 B 678 A
501 D 590 B 679 C
502 C 591 A 680 E
503 A 592 D 681 B
504 C 593 B 682 B
505 C 594 E 683 D
506 D 595 E 684 A
507 A 596 A 685 B
508 A 597 B 686 A
509 B 598 A 687 B
510 D 599 C 688 D
511 E 600 A 689 A
512 B 601 D 690 A
513 A 602 C 691 D
514 D 603 C 692 D
515 B 604 C 693 C
516 D 605 A 694 D
517 A 606 E 695 C
518 B 607 D 696 A
519 D 608 A 697 C
520 C 609 B 698 A
521 D 610 D 699 A
522 D 611 D 700 C
523 B 612 E
524 C 613 A
525 D 614 E
526 A 615 D
527 A 616 A
TOPNOTCH MEDICAL BOARD PREP ANATOMY SUPEREXAM Page 94 of 94
For inquiries visit www.topnotchboardprep.com.ph or email us at topnotchmedicalboardprep@gmail.com
TOPNOTCH MEDICAL BOARD PREP MICROBIOLOGY SUPEREXAM
For inquiries visit www.topnotchboardprep.com.ph or email us at topnotchmedicalboardprep@gmail.com
DEAR TOPNOTCH FRIENDS:

PLEASE FOLLOW THESE INSTRUCTIONS:

1. These questions are previous diagnostic, midterm, and finals exams of Topnotch, almost all of them made by Topnotch Board Exam Topnotchers.
2. Answer this Topnotch Superexam seriously 100-items at a time. Cover the “Explanations” Column. Do not immediately look at the answers from the
answer key. That’s not the correct way of answering sample exams. You need to treat these MCQs as exercises and not as handouts.
3. Time yourself. 1.5 hours per 100-item block.
4. After answering each 100-item block, refer to the Topnotch Answer Key for the correct answers. Please be careful of “frameshift mutations” when
checking your answers – check every 10 items. (the format of the answer key was designed for you to practice against “frameshift mutations”)
5. The Topnotch Superexams are EXERCISES for the actual med boards. They will not appear verbatim in your future exams. More than knowing what’s
the correct answer, it’s more important for you to:
a. Know why the other choices are wrong
b. Know why the other choices were included in the first place
c. Know the explanation to the correct answer
6. Sharpen your mind by answering the Topnotch Superexams. Most of these questions based on past feedback are more difficult than the actual questions
in the med boards. In these exams made by Board Exam Topnotchers, if you’re getting a score of 60/100 , that’s already a good score. More than 80/100
is outstanding.

Item QUESTION EXPLANATION AUTHOR TOPNOTCH
# EXAM
1 Common causes of the common cold include all of the Reovirus may infect the Gastrointestinal and KRISTEL TANHUI DIAGNOSTIC
following EXCEPT: respiratory tract but is not a common cause of (TOP 3 - AUG 2015 EXAM -
A. Respiratory syncytial virus the common cold. MED BOARDS; MARCH
B. Reovirus An example of reovirus is rotavirus. TOPNOTCH MD 2016
C. Coronavirus FROM LA SALLE)
D. Parainfluenza virus Source: Topnotch handout of microbiology
E. Rhinovirus

2 Which of the following is transmitted by a DNA virus? Hepatitis A – Picornavirus KRISTEL TANHUI DIAGNOSTIC
A. Hepatitis A Hepatitis B – Hepadnavirus (TOP 3 - AUG 2015 EXAM -
B. Hepatitis B Hepatitis C – Flavivirus MED BOARDS; MARCH
C. Hepatitis C Hepatitis D – defective virus TOPNOTCH MD 2016
D. Hepatitis D Hepatitis E – Hepevirus/Calicivirus FROM LA SALLE)
E. Hepatitis E All are RNA viruses except Hepatitis B.

Source: Robbins and Cotran Pathologic Basis
of Disease 8th ed p. 844

3 A patient presents with acute diarrhea. Fecal smear was Protozoans are actually classified KRISTEL TANHUI DIAGNOSTIC
done which revealed a ciliated protozoan. Which is the taxonomically according to their motility body (TOP 3 - AUG 2015 EXAM -
infective organism? parts MED BOARDS; MARCH
A. Entamoeba histolytica TOPNOTCH MD 2016
B. Giardia lamblia Groupname (motility) - examples FROM LA SALLE)
C. Escherichia coli Sarcodina (pseudopods) – Entamoeba
D. Balantidium coli histolytica, naegleria fowleri, acanthamoeba
E. Trichomonas castellanii
Zoomastigophora (flagella) – Giardia,
Trypanosoma, Leishmania, Trichomonas
Apicomplexa/Sporozoans (none) –
Plasmodium, Toxoplasma, Cryptosporidium,
Isospora
Ciliophora (cilia) – Balantidium

Source: Jawetz, Melnick and Aldelberg’s
Medical Microbiology back cover

4 Which of the following complement pathway’s activation Classic pathway – IgG or IgM KRISTEL TANHUI DIAGNOSTIC
is mediated by antibodies such as IgG and IgM? Alternative pathway – microbe surface (TOP 3 - AUG 2015 EXAM -
A. Classic polysaccharide molecules MED BOARDS; MARCH
B. Alternative Lectin pathway – mannose and other sugars TOPNOTCH MD 2016
C. Lectin on microbe surface FROM LA SALLE)
D. Mannose
E. All of the above

5 The following acute phase reactants are upregulated in It’s important to know what they do as well! KRISTEL TANHUI DIAGNOSTIC
inflammatory conditions except: Upregulated (TOP 3 - AUG 2015 EXAM -
A. CRP CRP: Opsonin MED BOARDS; MARCH
B. Ferritin Ferritin: sequesters iron TOPNOTCH MD 2016
C. Fibrinogen Fibrinogen: causes coagulation FROM LA SALLE)
D. Hepcidin Hepcidin: prevents release of iron bound by
E. Transferrin ferritin

Downregulated
Albumin
Transferrin: internalized by macrophage to
sequester iron

TOPNOTCH MEDICAL BOARD PREP MICROBIOLOGY SUPEREXAM Page 1 of 90


For inquiries visit www.topnotchboardprep.com.ph or email us at topnotchmedicalboardprep@gmail.com
TOPNOTCH MEDICAL BOARD PREP MICROBIOLOGY SUPEREXAM
For inquiries visit www.topnotchboardprep.com.ph or email us at topnotchmedicalboardprep@gmail.com
Item QUESTION EXPLANATION AUTHOR TOPNOTCH
# EXAM
6 What is the incubation period of malaria? Malaria incubation period: 1wk – 1month! KRISTEL TANHUI DIAGNOSTIC
A. 3-7days (TOP 3 - AUG 2015 EXAM -
B. 10-14days Source: MED BOARDS; MARCH
C. 7-30 days http://www.cdc.gov/malaria/about/disease.h TOPNOTCH MD 2016
D. 14-45 days tml FROM LA SALLE)
E. 2-6 months

7 A 45 year old alcoholic male presents with 4 day history This approach of questioning is sometimes KRISTEL TANHUI DIAGNOSTIC
of fever and cough productive of currant jelly sputum. PE used in the boards. Make sure to study the (TOP 3 - AUG 2015 EXAM -
reveals tachypnea and crackles. A sputum sample was clinical presentation of organisms as well as MED BOARDS; MARCH
taken and the organism was identified. Which of the the other more hardcore microbiology stuff. TOPNOTCH MD 2016
following is most likely? FROM LA SALLE)
A. Gram + Cocci in clusters This is a case of K. pneumonia which is
B. Nonlactose fermenting in McConkey agar associated with alcoholics and has classic
C. Motile at 36 degrees celcius currant jelly quality of sputum and mucoid
D. Mucoid colonies on cultures colonies due to its thick capsule.
E. All of the above
Source: Jawetz, Melnick and Aldelberg’s
Medical Microbiology p215

8 What is the most common etiology of acute Eschericia coli is the most common cause of KRISTEL TANHUI DIAGNOSTIC
uncomplicated bacterial lower urinary tract infection? BOTH complicated and uncomplicated urinary (TOP 3 - AUG 2015 EXAM -
A. Klebsiella tract infection. MED BOARDS; MARCH
B. E. Coli TOPNOTCH MD 2016
C. Proteus Source: Jawetz, Melnick and Aldelberg’s FROM LA SALLE)
D. Pseudomonas Medical Microbiology p751
E. Ureaplasma

9 Characteristics that best describes both Chlamydia and Exists as 2 forms – Chlamydia KRISTEL TANHUI DIAGNOSTIC
Ricketsia: (TOP 3 - AUG 2015 EXAM -
A. Exists as 2 forms Spread by the bite of an infected arthropod – MED BOARDS; MARCH
B. Obligate intracellular parasite Ricketsia TOPNOTCH MD 2016
C. Spread by the bite of an infected arthropod FROM LA SALLE)
D. Resistant to the usual broad spectrum antibiotic Source: Jawetz, Melnick and Aldelberg’s
E. All of the above Medical Microbiology

10 Sputum culture from a patient with pneumonia revealed Source: Topnotch handout on microbiology KRISTEL TANHUI DIAGNOSTIC
beta-hemolytic, catalase-positive, coagulase positive (TOP 3 - AUG 2015 EXAM -
gram positive cocci. The infecting organism is most MED BOARDS; MARCH
probably: TOPNOTCH MD 2016
A. S. aureus FROM LA SALLE)
B. S. epidermidis
C. S. pyogenes
D. S. agalactiae
E. S. pneumoniae

11 A gram-positive cocci was isolated from pustular skin Source: Topnotch handout on microbiology KRISTEL TANHUI DIAGNOSTIC
lesions of a 5 year old boy. If the isolate tested was beta- (TOP 3 - AUG 2015 EXAM -
hemolytic, catalase-negative, bacitracin sensitive, which MED BOARDS; MARCH
of the following organisms would you consider? TOPNOTCH MD 2016
A. S. aureus FROM LA SALLE)
B. S. enteritidis
C. S. pyogenes
D. S. agalactiae
E. S. pneumoniae

12 The following is true of influenza viruses: The diagnostic test of choice for influenza is KRISTEL TANHUI DIAGNOSTIC
A. The diagnostic test of choice is reverse PCR reverse PCR. (TOP 3 - AUG 2015 EXAM -
B. The diagnostic test of choice is immunofluorescence MED BOARDS; MARCH
C. The diagnostic test of choice is ELISA Source: (This is SIMILAR TO PREVIOUS TOPNOTCH MD 2016
D. Influenza A affects only humans and primates. BOARD EXAM CONCEPT/PRINCIPLE) FROM LA SALLE)
E. Gene reassortment produces antigenic shift http://www.ncbi.nlm.nih.gov/pubmed/22528
153

13 Gastric adenocarcinoma is best associated with which Source: Jawetz, Melnick and Aldelberg’s KRISTEL TANHUI DIAGNOSTIC
organism? Medical Microbiology p241 (TOP 3 - AUG 2015 EXAM -
A. Campylobacter jejuni MED BOARDS; MARCH
B. Human papillomavirus TOPNOTCH MD 2016
C. HIV FROM LA SALLE)
D. Enterobacter aerogenes
E. Helicobacter pylori

TOPNOTCH MEDICAL BOARD PREP MICROBIOLOGY SUPEREXAM Page 2 of 90


For inquiries visit www.topnotchboardprep.com.ph or email us at topnotchmedicalboardprep@gmail.com
TOPNOTCH MEDICAL BOARD PREP MICROBIOLOGY SUPEREXAM
For inquiries visit www.topnotchboardprep.com.ph or email us at topnotchmedicalboardprep@gmail.com
Item QUESTION EXPLANATION AUTHOR TOPNOTCH
# EXAM
14 Pila luzonica is the second intermediate host of which Paragonimus – sundathelpusa crab KRISTEL TANHUI DIAGNOSTIC
organism? Opisthorchis – fish (TOP 3 - AUG 2015 EXAM -
A. Paragonimus westermani Fasciola and fasciolopsis – aquatic plants MED BOARDS; MARCH
B. Opisthorchis TOPNOTCH MD 2016
C. Fasciola Source: Topnotch handout on microbiology FROM LA SALLE)
D. Fasciolopsis
E. Echinostoma

15 What is the scientific name of the so-called flesh eating In orthopedics, technically necrotizing fascitis KRISTEL TANHUI DIAGNOSTIC
bacteria which is the MOST COMMON cause of is divided into 3 (TOP 3 - AUG 2015 EXAM -
necrotizing fasciitis? Type I – polymicrobial MED BOARDS; MARCH
A. Streptococcus pyogenes Type II – GAS TOPNOTCH MD 2016
B. Clostridium tetani Type III – gas gangrene (Clostridial) FROM LA SALLE)
C. Clostridium perfringens
D. Staphylococcus aureus GAS is the most common single organism
E. Vibrio vulnificus etiology of necrotizing fasciitis. However, if
they specifically say or describe gas gangrene
then better answer will be Clostridial.

Source: Jawetz, Melnick and Aldelberg’s
Medical Microbiology p. 199 and 753

16 An infant was brought to the emergency room appearing This is a case of floppy baby syndrome or KRISTEL TANHUI DIAGNOSTIC
limp and “floppy”. The mother stated that she has just infant botulism, which is due to ingestion of (TOP 3 - AUG 2015 EXAM -
started to introduce table foods to her baby’s diet. A few Clostridium botulinum spores usually MED BOARDS; MARCH
days prior, she noted her baby to be constipated and to associated with consumption of honey. TOPNOTCH MD 2016
have diminished suckling and crying. This condition was FROM LA SALLE)
most likely caused by: Source: Jawetz, Melnick and Aldelberg’s
A. Spoiled milk consumption Medical Microbiology
B. Improperly cooked chicken
C. Consumption of raw honey
D. Respiratory droplets
E. None of the above

17 A 20 year old female who had a mild fever and cough Source: Jawetz, Melnick and Aldelberg’s KRISTEL TANHUI DIAGNOSTIC
was diagnosed to have pneumonia on chest xray. Gram Medical Microbiology p313 (TOP 3 - AUG 2015 EXAM -
stain of her sputum revealed normal flora. Culture MED BOARDS; MARCH
showed colonies with a “fried egg” appearance. The TOPNOTCH MD 2016
organism responsible for the symptoms in the patient is: FROM LA SALLE)
A. Chlamydia
B. Legionella
C. Ureaplasma
D. Ricketsia
E. M. pneumonia

18 Phlebotomus is the vector of which of the following Malaria – anopheles KRISTEL TANHUI DIAGNOSTIC
protozoan diseases: Trypanosomiasis (African) – Tsetse fly/ (TOP 3 - AUG 2015 EXAM -
A. Trypanosomaiasis Glossina spp MED BOARDS; MARCH
B. Leishmania Trypanosomiasis (American/Chagas) – TOPNOTCH MD 2016
C. Loa loa Kissing bug/reduviid/triatomid FROM LA SALLE)
D. Malaria Loaloa – Chrysops sp/mango fly/deer fly
E. Chagas
Source: topnotch handout on microbiology

19 What is the tissue form of paracoccidioidomycosis? Yeast cells with multiple narrow based bud KRISTEL TANHUI DIAGNOSTIC
A. Yeast cells with multiple narrow based bud/mariner’s Intracellular yeasts - paracoccidiodomycosis (TOP 3 - AUG 2015 EXAM -
wheel appearance Single broad based bud coming from a thick MED BOARDS; MARCH
B. Intracellular yeasts walled yeast cell - blastomycosis TOPNOTCH MD 2016
C. Single broad based bud coming from a thick walled Spherules with endospores - FROM LA SALLE)
yeast cell coccidiodomycosis
D. Spherules with endospores Yeast + pseudohyphae - candida
E. Yeast + pseudohyphae
Source: topnotch handout on microbiology

20 What is the tissue form of candidiasis? Source: topnotch handout on microbiology KRISTEL TANHUI DIAGNOSTIC
A. Yeast cells with multiple narrow based bud (TOP 3 - AUG 2015 EXAM -
B. Intracellular yeasts MED BOARDS; MARCH
C. Single broad based bud coming from a thick walled TOPNOTCH MD 2016
yeast cell FROM LA SALLE)
D. Spherules with endospores
E. Yeast + pseudohyphae

TOPNOTCH MEDICAL BOARD PREP MICROBIOLOGY SUPEREXAM Page 3 of 90


For inquiries visit www.topnotchboardprep.com.ph or email us at topnotchmedicalboardprep@gmail.com
TOPNOTCH MEDICAL BOARD PREP MICROBIOLOGY SUPEREXAM
For inquiries visit www.topnotchboardprep.com.ph or email us at topnotchmedicalboardprep@gmail.com
Item QUESTION EXPLANATION AUTHOR TOPNOTCH
# EXAM
21 Which among the following individuals is most likely to Progression to chronic liver disease is LESTER BRYAN CO MIDTERM 1
progress to chronic liver disease following an acute inversely related to the age of infection with (TOP 10 - AUG 2015 EXAM -
infection? HBV and 90% of neonates can become MED BOARDS; MARCH
A. A 50-year-old who becomes HBsAg positive chronically ill. Rates of HCV infection that TOPNOTCH MD 2016
B. A baby born to chronically active infected HBV mother progresses to chronic liver disease is also FROM UST)
C. A liver transplant patient infected with HCV high, but is less than neonates infected with
D. A recent immigrant who is HEV positive Hepatitis B.
E. A young adult co infected with HBV and HDV

22 Which bacterial gene transfer process would be In transformation, the DNA is extracellular LESTER BRYAN CO MIDTERM 1
inhibited by free extracellular exonucleases? before it is picked up by the competent cells; (TOP 10 - AUG 2015 EXAM -
A. Conjugation during this period, the DNA is subject to the MED BOARDS; MARCH
B. Generalized transduction extracellular exonucleases. Because the DNA TOPNOTCH MD 2016
C. Specialized transduction in generalized and specialized transductions is FROM UST)
D. Transformation protected extracellularly by the virus capsid, it
E. Transposition is not subject to extracellular exonucleases. In
conjunction, the DNA is never outside of a cell.
Transposition is a mechanism of inserting a
transposon into another molecule of DNA and
has no extracellular transport mechanism
associated with it.
23 Which of the following may be present in secondary SIMILAR TO PREVIOUS BOARD EXAM LESTER BRYAN CO MIDTERM 1
syphilis? CONCEPT/PRINCIPLE. There were porbably (TOP 10 - AUG 2015 EXAM -
A. Tabes Dorsalis 10 questions about syphilis in our MED BOARDS; MARCH
B. Gummas Microbiology exam. TOPNOTCH MD 2016
C. Chancre FROM UST)
D. Condylomata Latum
E. Aortic Aneurysm

24 What compound is only found in Gram-positive bacteria? Teichoic and teichuronic acids, which are LESTER BRYAN CO MIDTERM 1
A. Capsule polymers containing ribitol or glycerol, are (TOP 10 - AUG 2015 EXAM -
B. Lipopolysaccharide found only in the cell walls or cell-wall MED BOARDS; MARCH
C. Outer membrane membranes of Gram-positive bacteria. The TOPNOTCH MD 2016
D. Peptidoglycan other choices are present in both Gram- FROM UST)
E. Teichoic acid positive and Gram-negative bacteria.

25 In the following pairs of organisms, which two are Listeria is a Gram-positive rod, whereas LESTER BRYAN CO MIDTERM 1
easiest to distinguish from each other by Gram stain? Proteus is a Gram-negative rod. Clostridium, (TOP 10 - AUG 2015 EXAM -
A. Bacillus and Clostridium Lactobacillus, Corynebacterium, and Bacillus MED BOARDS; MARCH
B. Salmonella and Shigella are all Gram-positive rods, whereas TOPNOTCH MD 2016
C. Haemophilus and Escherichia Haemophilus, Escherichia, Salmonella, and FROM UST)
D. Corynebacterium and Lactobacillus Shigella are all Gram-negative rods.
E. Listeria and Proteus

26 What bacteria can use fermentation pathways but also Facultative anaerobes grow in the presence or LESTER BRYAN CO MIDTERM 1
contain superoxide dismutase? absence of oxygen; a respiratory mode is used (TOP 10 - AUG 2015 EXAM -
A. Obligate aerobes when oxygen is present, and fermentation MED BOARDS; MARCH
B. Obligate anaerobes occurs when it is not. Facultative anaerobes TOPNOTCH MD 2016
C. Facultative anaerobes contain the enzyme superoxide dismutase, FROM UST)
D. Aerobic heterotrophs which aids aerobic growth by preventing the
E. Microaerophiles accumulation of the superoxide ion. Obligate
aerobes do not have fermentative pathways
and require oxygen for growth; obligate
anaerobes lack superoxide dismutase. The
heterotrophs require preformed organic
compounds for growth. Microaerophiles need
oxygen because they cannot ferment or
respire anaerobically. However, they are
poisoned by high concentrations of oxygen.
27 A 5 year old unimmunized boy presented with a 2 day The boy is infected with the mumps virus, a LESTER BRYAN CO MIDTERM 1
history of headache, earache, and swallowing difficulty. single-stranded negative-sense RNA virus. (TOP 10 - AUG 2015 EXAM -
Physical examination showed bilateral swelling of the MED BOARDS; MARCH
parotid glands. The genetic nature of the pathogen TOPNOTCH MD 2016
causing her disease is: FROM UST)
A. Double-stranded DNA
B. Double-stranded RNA
C. Negative-sense RNA
D. Positive-sense RNA
E. Single-stranded DNA
28 A 26-year-old woman presents with high fever, painful SIMILAR TO PREVIOUS BOARD EXAM LESTER BRYAN CO MIDTERM 1
and frequent urination, and left flank pain. The isolate is CONCEPT/PRINCIPLE. Neisseria are (TOP 10 - AUG 2015 EXAM -
a Gram-negative, rod-shaped, facultative anaerobe that diplococci, vibrios are comma shaped, and MED BOARDS; MARCH
is oxidase negative and ferments glucose and lactose. campylobacteria are spiral shaped. Neisseria TOPNOTCH MD 2016
With these data, what genus most likely caused the and vibrios are all oxidase positive. Because FROM UST)
infection? campylobacter is grown
A. Neisseria under unique conditions and temperature (42
B. Proteus C), it is identified by just growing. It has both a
C. Vibrio poorly functioning oxidase and catalase.
D. Campylobacter Proteus and Escherichia are rod shaped and
E. Escherichia both are oxidase negative. Escherichia is the
only one of the two that is a lactose fermenter.

TOPNOTCH MEDICAL BOARD PREP MICROBIOLOGY SUPEREXAM Page 4 of 90


For inquiries visit www.topnotchboardprep.com.ph or email us at topnotchmedicalboardprep@gmail.com
TOPNOTCH MEDICAL BOARD PREP MICROBIOLOGY SUPEREXAM
For inquiries visit www.topnotchboardprep.com.ph or email us at topnotchmedicalboardprep@gmail.com
Item QUESTION EXPLANATION AUTHOR TOPNOTCH
# EXAM
29 A 25-year-old male who has recently started doing With tricuspid valve insufficiency in an IV LESTER BRYAN CO MIDTERM 1
intravenous drugs presents to the emergency drug abuser, the most common cause is (TOP 10 - AUG 2015 EXAM -
department with fever and evidence of mild tricuspid Staphylococcus aureus, which generally MED BOARDS; MARCH
valve insufficiency. As far as he knows, he had no presents as an acute endocarditis. TOPNOTCH MD 2016
previous damage to his heart. What is the most likely FROM UST)
causative agent?
A. Viridans streptococci
B. Coxsackie virus
C. Enterococcus faecalis
D. Pseudomonas aeruginosa
E. Staphylococcus aureus
30 3. A nonnucleoside analog that inhibits herpesvirus DNA Foscarnet inhibits herpesvirus DNA LESTER BRYAN CO MIDTERM 1
replication is: polymerase directly. (TOP 10 - AUG 2015 EXAM -
A. Acyclovir MED BOARDS; MARCH
B. Amantadine TOPNOTCH MD 2016
C. Cytarabine FROM UST)
D. Foscarnet
E. Interferon

31 Which of the following is the principal immunoglobulin SIMILAR TO PREVIOUS BOARD EXAM LESTER BRYAN CO MIDTERM 1
(Ig) in exocrine secretions? CONCEPT/PRINCIPLE. (TOP 10 - AUG 2015 EXAM -
A. IgA MED BOARDS; MARCH
B. IgG TOPNOTCH MD 2016
C. IgM FROM UST)
D. IgD
E. IgE

32 Which cestode does not require an intermediate host to SIMILAR TO PREVIOUS BOARD EXAM LESTER BRYAN CO MIDTERM 1
infect humans? CONCEPT/PRINCIPLE. Our Microbiology exam (TOP 10 - AUG 2015 EXAM -
A. Hymenolopsis nana only had few Parasitology questions. MED BOARDS; MARCH
B. Diphyllobothrium latum TOPNOTCH MD 2016
C. Hymenolopsis dimnuta FROM UST)
D. Diphyllobothrium klebanovski
E. Echinococcus granulosus

33 A patient developed an infection following Staphylococcus epidermidis is noted for its LESTER BRYAN CO MIDTERM 1
catheterization. A biofilm is present on the catheter. ability to secrete biofilms and adhere to (TOP 10 - AUG 2015 EXAM -
What is the most likely causative agent? intravenous lines. Streptococcus mutans (a MED BOARDS; MARCH
A. Enterococcus faecalis Viridans streptococci) is also noted for the TOPNOTCH MD 2016
B. Staphylococcus aureus production of a dextran biofilm that adheres FROM UST)
C. Staphylococcus epidermidis these organisms to dental surfaces and causes
D. Streptococcus agalactiae dental plaque.
E. Streptococcus viridans

34 What is the mechanism of action of aminoglycosides? Aminoglycosides bind to multiple sites on LESTER BRYAN CO MIDTERM 1
A. Damage to the cell membrane both the 30S and 50S ribosomes, thereby (TOP 10 - AUG 2015 EXAM -
B. Inhibit the DNA gyrase preventing the tRNA from forming initiation MED BOARDS; MARCH
C. Inhibit cell wall synthesis complexes. MOA of the other choices pertain TOPNOTCH MD 2016
D. Block initiation complex to: A-polymixins, B-fluoroquinolones, C-beta- FROM UST)
E. Inhibit peptide chain elongation lactams, E-chloramphenicol

35 Repeated Neisseria meningitidis septicemias in an The killing of Neisseria meningitidis LESTER BRYAN CO MIDTERM 1
individual should raise physician awareness of what organisms is primarily dependent on (TOP 10 - AUG 2015 EXAM -
underlying condition? complement-mediated cell lysis. Patients with MED BOARDS; MARCH
A. C5–C8 deficiencies genetic deficiencies in C5 to C8 cannot carry TOPNOTCH MD 2016
B. T-cell dysfunction out complement-mediated lysis of bacterial FROM UST)
C. B-cell dysfunction cells and have repeated septicemias with N.
D. Multiple myeloma meningitidis.
E. Severe neutropenia

36 What additional tissues are frequently involved with Corynebacterium diphtheriae does not invade LESTER BRYAN CO MIDTERM 1
diphtheria infection besides the ‘‘bull neck’’ and the tissues; rather, the exotoxin enters the (TOP 10 - AUG 2015 EXAM -
pseudomembranous pharyngitis? bloodstream and affects tissues, primarily the MED BOARDS; MARCH
A. Skin heart and nerves, causing myocarditis and TOPNOTCH MD 2016
B. Kidneys recurrent laryngeal neuropathy. FROM UST)
C. Heart and nerves
D. Liver and kidneys
E. Ears and sinuses

37 Cellulitis develops in a 26-year-old man after he is bitten Pasteurella multocida, is a dominant organism LESTER BRYAN CO MIDTERM 1
by his girlfriend’s cat. What is the most likely dominant in the cat’s mouth. Bartonella, is the causative (TOP 10 - AUG 2015 EXAM -
organism involved in the infection? agent of cat scratch fever and is less common MED BOARDS; MARCH
A. Bartonella henselae in bites. Calymmatobacterium, TOPNOTCH MD 2016
B. Calymmatobacterium granulomatis causes a sexually transmitted infection, and FROM UST)
C. Pasteurella multocida Toxoplasma is associated with cats but
D. Toxoplasma gondii transmitted by their feces not bites.
E. Clostridium tetani

TOPNOTCH MEDICAL BOARD PREP MICROBIOLOGY SUPEREXAM Page 5 of 90


For inquiries visit www.topnotchboardprep.com.ph or email us at topnotchmedicalboardprep@gmail.com
TOPNOTCH MEDICAL BOARD PREP MICROBIOLOGY SUPEREXAM
For inquiries visit www.topnotchboardprep.com.ph or email us at topnotchmedicalboardprep@gmail.com
Item QUESTION EXPLANATION AUTHOR TOPNOTCH
# EXAM
38 A 36-year-old female drug abuser has an abscess with The organism is most likely an anaerobe that LESTER BRYAN CO MIDTERM 1
draining pus on her jaw where she lost a tooth from did not tolerate transport aerobically. Unless (TOP 10 - AUG 2015 EXAM -
being struck in the face 2 weeks ago. She did not get you ask for specimens to be collected MED BOARDS; MARCH
medical help. You lance the lesion, express some lumpy anaerobically or at least ask the lab to culture TOPNOTCH MD 2016
material with difficulty, and send a swab of the organism an abscess, it will not be cultured FROM UST)
to the lab for Gram stain, culture, and susceptibilities. anaerobically especially when you send it in
The predominant organism in the Gram stain is a on a swab. The clinical case suggests an
filamentous bacterium that is Gram positive, yet nothing organism from the gingival crevices and not
grows in the lab. What is the most likely explanation for Staphylococcus aureus as the cause. Needle
why it did not grow? biopsy with the specimen sent in the syringe
A. It is an obligate aerobe and anaerobic culture should have been done
B. It is microaerophilic instead. Most likely organism is Actinomyces,
C. It is a facultative anaerobe a Gram-positive facultative anaerobe that
D. It is an obligate anaerobe forms filaments and is commonly an oral
E. The lab did not grow it on the correct medium. commensal.
39 A noncompliant HIV-positive patient with a CD4+ cell Mycobacterium avium-intracellulare or LESTER BRYAN CO MIDTERM 1
count of 40/mm3 presents with a pulmonary infection Mycobacterium tuberculosis, both acid-fast (TOP 10 - AUG 2015 EXAM -
caused by an organism that requires 4 weeks to grow on organisms, are the most likely causes of this MED BOARDS; MARCH
Lowenstein-Jensen medium. What is the best descriptor pulmonary infection in a patient with AIDS. TOPNOTCH MD 2016
of the most likely causative agent? Both can be cultured on Lowenstein-Jensen. FROM UST)
A. Acid-fast organism Pneumocystis jiroveci, a dimorphic fungus,
B. Dimorphic fungus cannot be cultured on any medium.
C. Budding yeast
D. Gram-positive coccus
E. Gram-negative rod

40 In June, an 18-year-old man develops a sore throat with Mycoplasma is a common cause of pneumonia LESTER BRYAN CO MIDTERM 1
a fever and a nonproductive cough that develops into in teenagers and young adults. During the (TOP 10 - AUG 2015 EXAM -
pneumonia with a severe, prolonged hacking cough but course of the infection, some cryoagglutinins MED BOARDS; MARCH
little sputum production. Cryoagglutinins are present. He (cold agglutinins) may be formed against red TOPNOTCH MD 2016
is treated appropriately and successfully with blood cells. The antibodies are inactive at FROM UST)
azithromycin. What is the nature of the most likely normal body temperature, but agglutinate red
causative agent? blood cells at 4 C.
A. Acid-fast organism
B. Elementary bodies
C. Bacteria lacking a cell wall
D. Gram-negative rod
E. Gram-positive coccus
41 True of epidemiology of HIV: The Joint United Nations Program on GEORGE MICHAEL MIDTERM 2
A. The majority of infection is by homosexual contact HIV/AIDS estimated that by the end of 2007, a SOSUAN (TOP 5 - EXAM -
B. Many of children infected were from perinatal total of 33 million people worldwide were AUG 2015 MED MARCH
transmission living with HIV/AIDS, the majority having BOARDS; TOPNOTCH 2016
C. AIDS is a homoseually transmitted disease been infected by heterosexual contact. The MD FROM UST)
D. NOTA World Health Organization estimates that of
E. AOTA the 2.7 million new HIV infections each year,
90% are occurring in developing countries. In
those countries, AIDS is overwhelmingly a
heterosexually transmitted disease, and there
are about equal numbers of male and female
cases. from Jawetz Medical Microbiology 25
edition
42 Which does not belong to the Family Enterobacteriaceae: Pseudomonas does not belong to the Family GEORGE MICHAEL MIDTERM 2
A. Escherichia Enterobacteriaceae. SOSUAN (TOP 5 - EXAM -
B. Edwardsiella AUG 2015 MED MARCH
C. Serratia BOARDS; TOPNOTCH 2016
D. Morganella MD FROM UST)
E. Pseudomonas

43 The microorganism causing honey-moon cystitis: 80% of community acquired UTI is caused by GEORGE MICHAEL MIDTERM 2
A. E. coli E. coli. S. saprophyticus is a normal flora of the SOSUAN (TOP 5 - EXAM -
B. S. saprophyticus female perinum and casuses 10-20% of AUG 2015 MED MARCH
C. E. faecalis community acquired UTI. BOARDS; TOPNOTCH 2016
D. Adenovirus MD FROM UST)
E. A. fumigatus

44 The patient was a 22 y/o F with a history of a UTI 4 MacConkey Agar is a selective medium that GEORGE MICHAEL MIDTERM 2
months PTA for which she was treated with oral allows growth of gram negative bacteria. Pink SOSUAN (TOP 5 - EXAM -
ampicillin without complications. Five days prior to this colonies are lactose fermenter. Colonies with AUG 2015 MED MARCH
admission she began to note nausea without vomiting. mucoid appearance means that the bacteria BOARDS; TOPNOTCH 2016
One day later she developed left fl ank pain, fevers, and possess capsule. MD FROM UST)
chills and noted increased urinary frequency. She noted
foul-smelling urine on the day PTA. She presented with a
temperature of 38.8°C, and physical examination showed
left costovertebral angle tenderness. Urinalysis of a
cleancatch urine sample was notable for >50 wbc/hpf , 3
to 10 rbc/hpf, and 3+ bacteria. Urine culture was
subsequently positive for >105 CFU of an organism per
ml. Pink mucoid colonies were noted growing on
MacConkey Agar. Which of the following is true?
A. The microorganism is a nonlactose fermenter.
B. The microorganism possess a capsule.
C. The microorganism is gram positive.
D. AOTA
E. Both A and B

TOPNOTCH MEDICAL BOARD PREP MICROBIOLOGY SUPEREXAM Page 6 of 90


For inquiries visit www.topnotchboardprep.com.ph or email us at topnotchmedicalboardprep@gmail.com
TOPNOTCH MEDICAL BOARD PREP MICROBIOLOGY SUPEREXAM
For inquiries visit www.topnotchboardprep.com.ph or email us at topnotchmedicalboardprep@gmail.com
Item QUESTION EXPLANATION AUTHOR TOPNOTCH
# EXAM
45 A disinfectant effectively kills: Sterilization is killing of both spores and GEORGE MICHAEL MIDTERM 2
A. Spores vegatations of bacteria. SOSUAN (TOP 5 - EXAM -
B. Vegatative parts of bacteria AUG 2015 MED MARCH
C. Virus BOARDS; TOPNOTCH 2016
D. Both B and C MD FROM UST)
E. AOTA

46 Epidemic Hemmorhagic Conjunctivitis is caused by GEORGE MICHAEL MIDTERM 2


A. P. aeruginosa SOSUAN (TOP 5 - EXAM -
B. H. ducreyi AUG 2015 MED MARCH
C. C. albicans BOARDS; TOPNOTCH 2016
D. Adenovirus MD FROM UST)
E. NOTA

47 A 4y/o M who awoke on the day prior to evaluation with Despite the use of penicillin to treat GAS GEORGE MICHAEL MIDTERM 2
a sore throat and fever. His mother had him stay home infections for more than 50 years, this SOSUAN (TOP 5 - EXAM -
from kindergarten and treated him symptomatically organism continues to be uniformly sensitive AUG 2015 MED MARCH
with Tylenol. He slept well but the next day awoke still to this antimicrobial. In penicillin-allergic BOARDS; TOPNOTCH 2016
complaining of sore throat and fever, as well as headache patients, erythromycin and the newer MD FROM UST)
and abdominal pain. He was an only child and neither macrolide antimicrobials clarithromycin and
parent was ill. On physical examination, he was noted to azithromycin are recommended therapeutic
have a fever of 38.4°C. His physical examination was agents for GAS pharyngitis.
significant for a 2+ red anterior pharynx, tonsillar region,
and soft palate. His anterior cervical lymph nodes at the
angle of the mandible were slightly enlarged and tender.
No skin lesions or rashes were seen. He did not have a
cough, runny nose, or conjunctivitis. When the results of
the rapid antigen test were known, the patient was given
a 10-day course of oral amoxicillin. What antimicrobial
resistance problems have been observed with this
organism?
A. The origanism is 20-30% resistant of penicillin
family
B. Erythromycin is given if microbial resistance is
suspected
C. Azithromycin is preferred over clarithromycin
D. Cefuroxime is given in penicllin-allergic patients
E. NOTA
48 A bacterial structure involved in adherence is Ordinary pili, adhesins, and the glycocalyx are GEORGE MICHAEL MIDTERM 2
A. Capsule three bacterial structures that are involved in SOSUAN (TOP 5 - EXAM -
B. LOS adherence. AUG 2015 MED MARCH
C. LPS BOARDS; TOPNOTCH 2016
D. Oridinary pili MD FROM UST)
E. Lipid A

49 Special tissue stain for leprosy bacilli GEORGE MICHAEL MIDTERM 2


A. Trichrome staine SOSUAN (TOP 5 - EXAM -
B. Fite-Faraco AUG 2015 MED MARCH
C. Congo Red BOARDS; TOPNOTCH 2016
D. Silver Methenamine MD FROM UST)
E. NOTA

50 Steam autoclave's setting includes GEORGE MICHAEL MIDTERM 2


A. 121oC, 15 psi, 15 minutes SOSUAN (TOP 5 - EXAM -
B. 101oC, 25 psi, 15 minutes AUG 2015 MED MARCH
C. 125oC, 15 psi, 25 minutes BOARDS; TOPNOTCH 2016
D. 112oC, 10 psi, 10 minutes MD FROM UST)
E. NOTA

51 Which of the following is not a characteristic of this S. epidermidis is Gram positive, coagulase GEORGE MICHAEL MIDTERM 2
bacterium which most commonly forms biofilm on negative, novibiocin sensitive cocci in clusters SOSUAN (TOP 5 - EXAM -
prosthesis: AUG 2015 MED MARCH
A. Gram postitive BOARDS; TOPNOTCH 2016
B. Cocci in clusters MD FROM UST)
C. Novobiocin sensitive
D. Purple on Gram stain
E. Coagulase positive

52 Broad-based budding is characteristic of: GEORGE MICHAEL MIDTERM 2


A. Blastomyces dermatitidis SOSUAN (TOP 5 - EXAM -
B. Paracoccidioides brazilenze AUG 2015 MED MARCH
C. Candida albicans BOARDS; TOPNOTCH 2016
D. Histoplasma capsulatum MD FROM UST)
E. Microsporum canis

TOPNOTCH MEDICAL BOARD PREP MICROBIOLOGY SUPEREXAM Page 7 of 90


For inquiries visit www.topnotchboardprep.com.ph or email us at topnotchmedicalboardprep@gmail.com
TOPNOTCH MEDICAL BOARD PREP MICROBIOLOGY SUPEREXAM
For inquiries visit www.topnotchboardprep.com.ph or email us at topnotchmedicalboardprep@gmail.com
Item QUESTION EXPLANATION AUTHOR TOPNOTCH
# EXAM
53 The method of choice for specimen collection for Cellulose Tape Preparation/Scotch Tape Swab GEORGE MICHAEL MIDTERM 2
recovery of Enterobius vermicularis is the answer. SOSUAN (TOP 5 - EXAM -
A. Direct Fecal Smear AUG 2015 MED MARCH
B. Enzyme Immunoassay BOARDS; TOPNOTCH 2016
C. Tissue biopsy MD FROM UST)
D. Urinalysis
E. NOTA

54 What is a mass of fungal filaments called? Mycelium is a mass of fungal filamnets GEORGE MICHAEL MIDTERM 2
A. Pseudohyphae (Hypha). Septum are the cross-walls SOSUAN (TOP 5 - EXAM -
B. Hyphae AUG 2015 MED MARCH
C. Mycelium BOARDS; TOPNOTCH 2016
D. Septum MD FROM UST)
E. Yeast

55 A 85 y/o M admiited at ICU was on high-lipid fluids and Malassezia furfur is a lipophilic fungus that is GEORGE MICHAEL MIDTERM 2
developed septicemia. What is the most likely causative found on skin. It causes fungemia on SOSUAN (TOP 5 - EXAM -
agent? immunosuppresed individual and on high- AUG 2015 MED MARCH
A. P. aeruginosa lipid intravenous supplements. BOARDS; TOPNOTCH 2016
B. M. furfur MD FROM UST)
C. H. nana
D. E. coli
E. M. kansasii

56 A systemic hypersensitivity caused by Schistosoma is GEORGE MICHAEL MIDTERM 2


called: SOSUAN (TOP 5 - EXAM -
A. Katayama fever AUG 2015 MED MARCH
B. Jock itch BOARDS; TOPNOTCH 2016
C. Swimmers itch MD FROM UST)
D. Kawasaki fever
E. NOTA

57 Sixth Disease is caused by First disease - Rubeola, second disease - GEORGE MICHAEL MIDTERM 2
A. Mumps scarlet fever, third disease - rubella, fourth SOSUAN (TOP 5 - EXAM -
B. GABHS disease - Filatov-Dukes' disease, fifth disease - AUG 2015 MED MARCH
C. HHV-6 erythema infectiosum, sixth disease - roseola BOARDS; TOPNOTCH 2016
D. Rubella (HHV-6) MD FROM UST)
E. Rubeola

58 Leading cause of viral encephalitis in Asia GEORGE MICHAEL MIDTERM 2


A. Japanese B virus SOSUAN (TOP 5 - EXAM -
B. West Nile virus AUG 2015 MED MARCH
C. Sapporo virus BOARDS; TOPNOTCH 2016
D. Dengue virus MD FROM UST)
E. Rubeola

59 Which of the following is H2S producing bacteria? GEORGE MICHAEL MIDTERM 2


A. Proteus SOSUAN (TOP 5 - EXAM -
B. Salmonella AUG 2015 MED MARCH
C. Pseudomonas BOARDS; TOPNOTCH 2016
D. Both A and B MD FROM UST)
E. AOTA

60 Beta-hemolysis is seen on blood agar plate as: Clear zone - beta/complete hemolysis, GEORGE MICHAEL MIDTERM 2
A. Clear zone greenish zone - alpha/partial hemolysis, SOSUAN (TOP 5 - EXAM -
B. Greenish zone AUG 2015 MED MARCH
C. Whitish zone BOARDS; TOPNOTCH 2016
D. Reddish zone MD FROM UST)
E. Pinkish zone

61 All of the statements are correct regarding the antigenic TSST-1 is the prototypical superantigen. TSST- JAN CHRISTIAN MIDTERM 3
structure of Staphycocci EXCEPT? 1 binds to major histocompatibility class FELICIANO (TOP 2 - EXAM -
A. Protein A is a cell wall component that binds to the Fc (MHC) class II molecules NOT class I, yielding AUG 2015 MED MARCH
portion of IgG molecules acting as an impt virulence T-cell stimulation, which promotes the BOARDS; TOPNOTCH 2016
factor protean manifestations of the toxic shock MD FROM UST)
B. Most strains have polysaccharide capsules, which syndrome. All the rest of the statements are
inhibit phagocytosis correct. The current microbiology questions
C. S aureus produces coagulase, an enzyme-like protein are very Jawetz specific lifted verbatim from
that clots plasma. the book.
D. Panton valentine leukocidin, a plasmid mediated toxin
can kill WBC and an important virulence factor in CA-
MRSA infections.
E. TSST-1 is the prototypical superantigen that binds to
MHC class I molecules, yielding T-cell stimulation
62 An organism produces a double hemolysis on egg yolk The organism is Clositridium perfringens and JAN CHRISTIAN MIDTERM 3
agar. This is due to what toxin? produces a double zone of hemolysis due to FELICIANO (TOP 2 - EXAM -
A. Alpha toxin alpha toxin or lecithinase AUG 2015 MED MARCH
B. Beta toxin BOARDS; TOPNOTCH 2016
C. Exotoxin A MD FROM UST)
D. Exotocin B
E. Hyaluronidase

TOPNOTCH MEDICAL BOARD PREP MICROBIOLOGY SUPEREXAM Page 8 of 90


For inquiries visit www.topnotchboardprep.com.ph or email us at topnotchmedicalboardprep@gmail.com
TOPNOTCH MEDICAL BOARD PREP MICROBIOLOGY SUPEREXAM
For inquiries visit www.topnotchboardprep.com.ph or email us at topnotchmedicalboardprep@gmail.com
Item QUESTION EXPLANATION AUTHOR TOPNOTCH
# EXAM
63 What structure is the basis for the Lancefield This carbohydrate is contained in the cell wall JAN CHRISTIAN MIDTERM 3
classification of the streptococci species? of many streptococci and forms the basis of FELICIANO (TOP 2 - EXAM -
A. M protein serologic grouping into Lancefield AUG 2015 MED MARCH
B. Thickness of the peptidoglycan layer classification. The serologic specificity of the BOARDS; TOPNOTCH 2016
C. Polysacharide capsule group- specific carbohydrate is determined by MD FROM UST)
D. Teichoic acids an amino sugar
E. Cell wall carbohydrates

64 This is an example of an obligate intracellular bacteria? Chlamydia and rickettsia are obligate JAN CHRISTIAN MIDTERM 3
A. Mycobacteria intrcellular organisms because they are ATP FELICIANO (TOP 2 - EXAM -
B. Neiserria parasites. The rest of the choices are only AUG 2015 MED MARCH
C. Chlamydia facultatively intracellular. BOARDS; TOPNOTCH 2016
D. Legionella MD FROM UST)
E. Listeria

65 A 40 year old adult came to you due to fever, cough, and The case is referring to bacterial pneumonia JAN CHRISTIAN MIDTERM 3
dyspnea. Chest x ray revealed consolidation of right and S penumoniae is the causative agent in FELICIANO (TOP 2 - EXAM -
lower lobe. Which of the ff statement is FALSE regarding 60% of the cases. The virulence of the AUG 2015 MED MARCH
the most common causative agent of this disease? organism is a function of its capsule. A serum BOARDS; TOPNOTCH 2016
A. It has no Lancefield classification that contains antibodies against the type- MD FROM UST)
B. It is a normal inhabitants of the upper respiratory specific polysaccharide will protect against
tract and 40–70% of humans are at some time carriers of infection. It has no Lancefield classification.
the virulent pneumococci The sputum is similar to the alveolar exudate,
C. A patient's serum that contains antibodies against the being characteristically bloody or rusty
type specific polysaccharide capsule is protected against colored not currant jelly like.
infection.
D. The sputum is similar to the alveolar exudate, being
characteristically currant jelly in appearance
E. Bacteremia from pneumonia has a triad of severe
complications: meningitis, endocarditis, and septic
arthritis
66 You are suspecting a patient with hematuria and Anti DNAse B titer is used to document JAN CHRISTIAN MIDTERM 3
hypertension as a case of PSGN. The test used to antecedent skin infection such as impetigo FELICIANO (TOP 2 - EXAM -
document prior impetigo? while ASO titer is sued to document AUG 2015 MED MARCH
A. ASO titer antecedent pharnyngitis. BOARDS; TOPNOTCH 2016
B. Lim broth test MD FROM UST)
C. Anti DNAse B
D. Dick's test
E. Complete blood count

67 A patient came to you because of a 2 week history of Muramyl dipeptide complexed with mycolic JAN CHRISTIAN MIDTERM 3
cough, weight loss, night sweats and anorexia. What acids can cause granuloma formation while FELICIANO (TOP 2 - EXAM -
statement regarding the causative agent is FALSE? phospholipids induce caseous necrosis. Cord AUG 2015 MED MARCH
A. Mycobacteria cannot be classified as either gram factos is a virulence factor and inhibits WBC BOARDS; TOPNOTCH 2016
positive or gram negative. migration serving as immunolgic adjuvant. MD FROM UST)
B. Acid fastness depends on the integrity of the waxy Mycobacteria cannot be classi- fied as either
envelope. gram positive or gram negative. When stained
C. M tuberculosis and M. bovis are equally pathogenic for by basic dyes, they cannot be decolorized by
humans. alcohol, regardless of treatment with iodine. M
D. Cord formation is correlated with virulence inhibiting tuberculosis and Mycobacterium bovis are
migration of leukocytes and serve as an immunologic equally pathogenic for humans as stated in
adjuvant. Jawetz.
E. Muramyl dipeptide from peptidoglycan complexed
with mycolic acids induce caseous necrosis.
68 An neonate was brought to you with a 2 days history of The most common cause of neontal sepsis is JAN CHRISTIAN MIDTERM 3
poor feeding and lethargy. You are entertaining neonatal Grp B strep or S. agalactiae which are part of FELICIANO (TOP 2 - EXAM -
sepsis. What is the most common causative agent of this the normal vaginal flora and lower AUG 2015 MED MARCH
disease? gastrointestinal tract in women. Group B BOARDS; TOPNOTCH 2016
A. Gram postive cocci in chains, alpha hemolytic, streptococcal infection during the first month MD FROM UST)
optochin senstivie of life may present as fulminant sepsis,
B. Gram positive cocci in clusters, catalase positive meningitis, or respiratory distress syndrome.
C. Gram postive cocci in chains, beta hemolytic, Chocie A refers to S pneumoniae, Choice B is
bacitracin resistant Staph aureus. Choice D is E. coli and choice E is
D. Gran negative rod, lactose fermenting with greenish Listeria. E coli and Lsiteria are als ocommon
metallic sheen on EMB medium cuases of neotal sepsis but the MCC overall is
E. Gram positive rod with tumbling motility Grp B strep.
69 Pertaining to question 48, what is the drug of choice? The drug of chocie for S. agalactiae in neonates JAN CHRISTIAN MIDTERM 3
A. Ampicillin is still Penicillin G. An aminoglycoside may be FELICIANO (TOP 2 - EXAM -
B. Penicillin G added for seriosu infections. Ampicillin given AUG 2015 MED MARCH
C. Chloramphenicol to mothers who are colonized with group B BOARDS; TOPNOTCH 2016
D. Vancomycin streptococci and are in labor prevents MD FROM UST)
E. Meropenem colonization of their infants.

70 The statement that best explains why Penicillin is always SIMILAR TO PREVIOUS BOARD EXAM JAN CHRISTIAN MIDTERM 3
the drug of choice for treponema pallidum? CONCEPT/PRINCIPLE. Most pathogenic FELICIANO (TOP 2 - EXAM -
A. The lack of porin channels in the cell wall bacteria have transposable elements, but T AUG 2015 MED MARCH
B. The genome is well conserved and has no pallidum does not, which suggests that the BOARDS; TOPNOTCH 2016
transposable elements genome is highly conserved and may explain MD FROM UST)
C. Absence of extended spectrum beta lactamases that its continued susceptibility to penicillin. All
inactivates penicillin other choices are erroneous and made up.
D. Inherent point mutation in the trepA gene of the
spirochete
E. Inability to modify penicillinase binding proteinases
(PBP)

TOPNOTCH MEDICAL BOARD PREP MICROBIOLOGY SUPEREXAM Page 9 of 90


For inquiries visit www.topnotchboardprep.com.ph or email us at topnotchmedicalboardprep@gmail.com
TOPNOTCH MEDICAL BOARD PREP MICROBIOLOGY SUPEREXAM
For inquiries visit www.topnotchboardprep.com.ph or email us at topnotchmedicalboardprep@gmail.com
Item QUESTION EXPLANATION AUTHOR TOPNOTCH
# EXAM
71 What is the prototype organism that produces secretory The most well studied organism that produces JAN CHRISTIAN MIDTERM 3
diarrhea due to overactivation of cAMP? secretory diarrhea is vibrio cholerae. The FELICIANO (TOP 2 - EXAM -
A. E. coli eneterotoxin activates Gs and increases levels AUG 2015 MED MARCH
B. Salmonella of intracellular cyclic adenosine BOARDS; TOPNOTCH 2016
C. Entamoeba histolytica monophosphate (cAMP) resulings in MD FROM UST)
D. Rotavirus prolonged hypersecretion of water and
E. Vibrio cholerae electrolytes. Electrolyte-rich diarrhea
occurs— as much as 20–30 L/day

72 Which of the manifestations is part of the clinical picture SIMILAR TO PREVIOUS BOARD EXAM JAN CHRISTIAN MIDTERM 3
of secondary stage of syphilis? CONCEPT/PRINCIPLE. The secondary stage o FELICIANO (TOP 2 - EXAM -
A. Meningitis consist of a red maculopapular rash anywhere AUG 2015 MED MARCH
B. Aortitis on the body, including the hands and feet, and BOARDS; TOPNOTCH 2016
C. Tabes dorsalis moist, pale papules (condylomas) in the MD FROM UST)
D. Soft chancre anogenital region, axillae, and mouth. The
E. Generalized wheals patient may also have syphilitic meningitis,
chorioretinitis, hepatitis, nephritis (immune
complex type), or periostitis.
73 What is the name of hemorrhagic necrosis of skin that Hemorrhagic necrosis of skin occurs often in JAN CHRISTIAN MIDTERM 3
occurs often in sepsis caused by P aeruginosa? sepsis caused by P aeruginosa; the lesions, FELICIANO (TOP 2 - EXAM -
A. Disseminated intravascular coagulatopathy called ecthyma gangrenosum, are surrounded AUG 2015 MED MARCH
B. Ecthyma by erythema and often do not contain pus. BOARDS; TOPNOTCH 2016
C. Ecthyma gangrenosum Ecthyma is a deep form of impetigo and is MD FROM UST)
D. Weil's disease caused by S pyogenes or S aureus. Weil's
E. Ritter's disease disease is the severe form of leptospirosis and
Ritter's disease is known as Stap scalded skin
syndrome.
74 All of the ff bacteria have polysaccharide capsules that The mnemonics for encapsulaed bacteria are JAN CHRISTIAN MIDTERM 3
confer protection against phagocytosis EXCEPT? SHiNE SKiS. Strep pneumonia, H influenzae, FELICIANO (TOP 2 - EXAM -
A. Strep pneumonaie Neisseria mengitidis, E coli, Salmonella, AUG 2015 MED MARCH
B. E. coli Klebsiella and Grp B strep. BOARDS; TOPNOTCH 2016
C. H influenzae MD FROM UST)
D. Neisseria mengitidis
E. All are encapsulated

75 A 6 year old child with several pinkish papular skin This is a case of molluscum contagiosum. The JAN CHRISTIAN MIDTERM 3
lesions with umbilicated centers. What histopathologic HP findings are Henderson Peterson bodies. FELICIANO (TOP 2 - EXAM -
finding is pathognomonic? Guarneri bodies- Smallpox. Cowdry type A- AUG 2015 MED MARCH
A. Guarneri bodies Herpes simplex. Owl's eye nuclear inclusions BOARDS; TOPNOTCH 2016
B. Henderson Peterson bodies is CMV and Councilman's bodies is Hepa B MD FROM UST)
C. Cowdry Type A bodies inffection
D. Owl's eye nuclear inclusions
E. Councilman's bodies

76 What is the name of the neurodegenarative disease that JC Polyoma virus causes progressive JAN CHRISTIAN MIDTERM 3
JC polymoma virus causes in patients with AIDS? multifocal leukoencephalopathy in patients FELICIANO (TOP 2 - EXAM -
A. Subacute combined degeneration with AIDS. Choice A refers to Vit B12 AUG 2015 MED MARCH
B. Subacute sclerosing panencephalitis deficiency. Choice B is a rare complication of BOARDS; TOPNOTCH 2016
C. Amyotropic lateral sclerosis measles. Choice E is an autoimmune disorder. MD FROM UST)
D. Progressive multifocal leukoencephalopathy
E. Acute Disseminated Encephalomyelitis

77 Which of the ff statement regarding influenza virus is Whereas antigenic changes continually occur JAN CHRISTIAN MIDTERM 3
true? within the type A group of influenza viruses FELICIANO (TOP 2 - EXAM -
A. Type A and B appears to be antigenically stable. and to a lesser degree in the type B group, AUG 2015 MED MARCH
B. It has a single-stranded, positive-sense segmented type C appears to be antigenically stable. BOARDS; TOPNOTCH 2016
RNA genome Genome is a Single-stranded RNA, segmented MD FROM UST)
C. The fusion protein of influenza virus binds virus (eight molecules), negative-sense. The
particles to susceptible cells and is the major antigen hemaglutinin protein of influenza virus binds
against which neutralizing antibodies are directed virus particles to susceptible cells and is the
D. Antigenic shift is caused by the accumulation of point major antigen against which neutral- izing
mutations in the gene (protective) antibodies are directed. The NA
E. The neuraminidase helps the virus negotiate through functions at the end of the viral replication
the mucin layer in the respiratory tract cycle and helps the virus negotiate through
the mucin layer in the respiratory tract
78 What is the basis for the classification of influenza SIMILAR TO PREVIOUS BOARD EXAM JAN CHRISTIAN MIDTERM 3
viruses into Type A, B and C? CONCEPT/PRINCIPLE. Antigenic differences FELICIANO (TOP 2 - EXAM -
A. Antigenic differences of the nucleocapsid (NP) and exhibited by two of the internal structural AUG 2015 MED MARCH
matrix (M) proteins proteins, the nucleocapsid (NP) and matrix BOARDS; TOPNOTCH 2016
B. Genetic reassortment of the HA and NA protein spikes (M) proteins, are used to divide influenza MD FROM UST)
C. Inherent variations in the genetic code of the RNA viruses into types A, B, and C.
genome
D. Clinical picture and symptomatology
E. Variabilities in hosts and carriers
79 A patient came to your clinic with discrete, SIMILAR TO PREVIOUS BOARD EXAM JAN CHRISTIAN MIDTERM 3
hypopigmented macules on the chest and upper back for CONCEPT/PRINCIPLE. The clinical picture FELICIANO (TOP 2 - EXAM -
3 months. What is the causative agent? poitns to Pityriasis versicolos. It is a chronic AUG 2015 MED MARCH
A. Microsporum canis mild superficial infection of the stratum BOARDS; TOPNOTCH 2016
B. Trichophyton tonsurans corneum caused by Malassezia globosa, MD FROM UST)
C. Malassezia furfur Malassezia restricta, and other members of
D. Piedraia hortae the Malassezia furfur complex.
E. Hortaea werneckii

TOPNOTCH MEDICAL BOARD PREP MICROBIOLOGY SUPEREXAM Page 10 of 90


For inquiries visit www.topnotchboardprep.com.ph or email us at topnotchmedicalboardprep@gmail.com
TOPNOTCH MEDICAL BOARD PREP MICROBIOLOGY SUPEREXAM
For inquiries visit www.topnotchboardprep.com.ph or email us at topnotchmedicalboardprep@gmail.com
Item QUESTION EXPLANATION AUTHOR TOPNOTCH
# EXAM
80 Pertaining to question no 59. What rapid diagnostic test SIMILAR TO PREVIOUS BOARD EXAM JAN CHRISTIAN MIDTERM 3
can you use to confirm your diagnosis? CONCEPT/PRINCIPLE. Diagnosis is confirmed FELICIANO (TOP 2 - EXAM -
A. 1-2% KOH smear by direct microscopic examination of AUG 2015 MED MARCH
B. Wood's lamp scrapings of infected skin, treated with 10– BOARDS; TOPNOTCH 2016
C. Sabaroud's medium 20% potassium hydroxide (KOH). Short MD FROM UST)
D. CALAS unbranched hyphae and spherical cells are
E. Tzanck smear observed. The lesions also fluoresce under
Wood’s lamp. Culture is not a rapid dx test.
CALAS is for Cryptoccus neofromans and
Tzanck smear is for Herpes simplex
81 1. Where can Ghon's complex be found? ANDREW TIU (TOP 1 FINAL
a. syphilis - AUG 2015 MED EXAM -
b. TB BOARDS; TOPNOTCH MARCH
c. Cat scratch disease MD FROM CIM) 2016
d. leprosy
e. none of the above

82 2. Which of the following is a manifestation of secondary ANDREW TIU (TOP 1 FINAL


syphilis? - AUG 2015 MED EXAM -
a. aortitis BOARDS; TOPNOTCH MARCH
b. gumma MD FROM CIM) 2016
c. meningitis
d. lymphadenopathy
e. none of the above

83 3. Which of the following is the etiologic agent for ANDREW TIU (TOP 1 FINAL
visceral larva migrans? - AUG 2015 MED EXAM -
a. toxocara canis BOARDS; TOPNOTCH MARCH
b. Onchocerca volvulus MD FROM CIM) 2016
c. ancylostoma caninum
d. ancylostoma duodenum
e. Toxoplasma gondii

84 4. A 67 year old alcoholic came in for complaints of ANDREW TIU (TOP 1 FINAL
productive cough with mucoid bloody sputum. Gram - AUG 2015 MED EXAM -
stain revealed large capsules. Which of the following is BOARDS; TOPNOTCH MARCH
the most likely etiologic agent? MD FROM CIM) 2016
a. Streptococci
b. staphylococci
c. Pseudomonas
d. Klebsiella
e. E. Coli
85 5. A 32 year old male came in for painless Jawetz Medical Microbiology 25th edition. p. ANDREW TIU (TOP 1 FINAL
hypopigmented patches over his chest and back with 630 - AUG 2015 MED EXAM -
minimal scaling 3 months prior to consult. Which of the BOARDS; TOPNOTCH MARCH
following is the most likely diagnosis? MD FROM CIM) 2016
a. vitiligo
b. pityriasis versicolor
c. tinea corporis
d. Piedra
e. none of the above
86 6. Which of the following is the diagnostic method of Malassezia species are lipophilic yeasts, and ANDREW TIU (TOP 1 FINAL
choice for the above scenario? most require lipid in the medium for growth. - AUG 2015 MED EXAM -
a. India ink The diagnosis is confirmed by direct BOARDS; TOPNOTCH MARCH
b. 10% KOH microscopic examination of scrapings of MD FROM CIM) 2016
c. biopsy infected skin, treated with 10 - 20% KOH or
d. culture stained with calcofluor white. Short
e. serology unbranched hyphae and spherical cells are
observed
Jawetz Medical Microbiology 25th edition. p.
630
87 7. A 28 year old US navy comes to you for consult and In addition to their function as screening tests, ANDREW TIU (TOP 1 FINAL
wants to be screened for syphilis. Which of the following they can be used to follow the efficacy of - AUG 2015 MED EXAM -
tests is the most appropriate? therapy. The drawbacks to the nontreponemal BOARDS; TOPNOTCH MARCH
a. treponemal antibody tests tests are that they are not very sensitive in MD FROM CIM) 2016
b. non-treponemal antibody tests early syphilis and may not turn positive until a
c. dark field examination few weeks after initial infection.
d. immunofluorescence Jawetz Medical Microbiology 25th edition. p.
e. none of the above 302
88 8. Screening tests turned out positive and the patient The nontreponemal tests can give quantitative ANDREW TIU (TOP 1 FINAL
was treated for syphilis. Which of the following tests results using seiral twofold dilutions. These - AUG 2015 MED EXAM -
would you order to confirm cure? are valuable in establishing a diagnosis and in BOARDS; TOPNOTCH MARCH
a. treponemal antibody tests evaluating the effect of treatment. MD FROM CIM) 2016
b. non-treponemal antibody tests Jawetz Medical Microbiology 25th edition. p.
c. dark field examination 303
d. immunofluorescence
e. none of the above
89 9. Which of the following is NOT true of the etiologic Most pathogenic bacteria have transposable ANDREW TIU (TOP 1 FINAL
agent of syphilis? elements, but T. pallidum does not, which - AUG 2015 MED EXAM -
a. microaerophilic suggests that the genome is highly conserved BOARDS; TOPNOTCH MARCH
b. viable for at least 24 hours in whole blood and may explain its continued susceptibility to MD FROM CIM) 2016
c. transposable elements in the genome penicillin.
d. rotates steadily around their endoflagella Jawetz Medical Microbiology 25th edition. p.
e. reproduce by transverse fission 302

TOPNOTCH MEDICAL BOARD PREP MICROBIOLOGY SUPEREXAM Page 11 of 90


For inquiries visit www.topnotchboardprep.com.ph or email us at topnotchmedicalboardprep@gmail.com
TOPNOTCH MEDICAL BOARD PREP MICROBIOLOGY SUPEREXAM
For inquiries visit www.topnotchboardprep.com.ph or email us at topnotchmedicalboardprep@gmail.com
Item QUESTION EXPLANATION AUTHOR TOPNOTCH
# EXAM
90 10. Which of the following is NOT true of SARS? Jawetz Medical Microbiology 25th edition. p. ANDREW TIU (TOP 1 FINAL
a. immunity develops but is not absolute 575 - AUG 2015 MED EXAM -
b. reinfection with similar strains is uncommon BOARDS; TOPNOTCH MARCH
c. immunity against surface projection antigen is MD FROM CIM) 2016
important for protection
d. more than 95% develop an antibody response to viral
antigens
e. all of the above
91 11. What is the preferred laboratory diagnosis for Jawetz Medical Microbiology 25th edition. p. ANDREW TIU (TOP 1 FINAL
influenza? 546 - AUG 2015 MED EXAM -
a. hemagglutination inhibition test BOARDS; TOPNOTCH MARCH
b. ELISA MD FROM CIM) 2016
c. viral isolation
d. RT - PCR
e. neutralization tests

92 12. Which of the following is responsible for the Gonococcal LPS does not have the long O - ANDREW TIU (TOP 1 FINAL
virulence of gonococci? antigen side chains and is called LOS. - AUG 2015 MED EXAM -
a. Opa proteins Gonococci can express more than one BOARDS; TOPNOTCH MARCH
b. Por protein antigenically different LOS. It causes ciliary MD FROM CIM) 2016
c. pili (fimbriae) loss and mucosal cell death in the fallopian
d. LOS tube explant model.
e. protein III Jawetz Medical Microbiology 25th edition. p.
265
93 13. How long should you observe a patient who came Jawetz Medical Microbiology 25th edition. p. ANDREW TIU (TOP 1 FINAL
from a malaria endemic place? 678 - AUG 2015 MED EXAM -
a. 1 week after the first possible exposure and 2 months BOARDS; TOPNOTCH MARCH
after the last exposure MD FROM CIM) 2016
b. 2 weeks after the first possible exposure and 3 months
after the last exposure
c. 6 months after the first possible exposure and 1 year
after the last exposure
d. 1 month after the first possible exposure and 6
months after the last exposure
e. none of the above
94 14. A 16 year old male came in for one week history of ANDREW TIU (TOP 1 FINAL
productive cough and fever. On gram stain of sputum, a - AUG 2015 MED EXAM -
gram positive lancet shaped diplococci was seen. On BOARDS; TOPNOTCH MARCH
culture, a greenish halo surrounded the colonies and MD FROM CIM) 2016
optochin sensitive. Which of the following is the most
likely etiologic agent?
a. viridans streptococci
b. streptococcus agalactiae
c. streptococcus pneumoniae
d. streptococcus pyogenes
e. streptococcus bovis
95 15. Which of the following is NOT true of influenza Antigenic differences exhibited by two of the ANDREW TIU (TOP 1 FINAL
viruses? internal structural proteins, the nucleocapsid - AUG 2015 MED EXAM -
a. HA protein is the major antigen against which and matrix proteins are used to divide BOARDS; TOPNOTCH MARCH
protective antibodies are directed influenza viruses into types A, B, and C MD FROM CIM) 2016
b. the cleavage of the HA protein is necessary for its Jawetz Medical Microbiology 25th edition. p.
contagiousness 541
c. single stranded, segmented, and negative sense
d. HA protein is used to divide influenza viruses into
types A, B, and C
e. NA protein functions at the end of the viral replication
cycle
96 16. Which of the following reflects a T - cell defect? A - selective IgA deficiency (B cell only) ANDREW TIU (TOP 1 FINAL
a. anaphylaxis to IgA containing products C - SCID (both B and T cell) - AUG 2015 MED EXAM -
b. recurrent viral and fungal infections D - leukocyte adhesion deficiency type 1 BOARDS; TOPNOTCH MARCH
c. recurrent bacterial and protozoal infections (phagocyte dysfunction) MD FROM CIM) 2016
d. delayed separation of umbilical cord E - Wiskott Aldrich Syndrome (both B and T
e. thrombocytopenic purpura and eczema cell)
USMLE first aid 2015 p. 215

97 17. A 35 year old female comes to the emergency room Jawetz Medical Microbiology 25th edition. p. ANDREW TIU (TOP 1 FINAL
with one week history of high grade fever, malaise, 223 - AUG 2015 MED EXAM -
headache, bradycardia, abdominal pain, constipation. On BOARDS; TOPNOTCH MARCH
CBC, leukopenia was noted. As a zoonotic disease, which MD FROM CIM) 2016
of the following periods should you elicit from the
patient's history of exposure?
a. 3-5 days
b. 7-10 days
c. 10-14 days
d. 4 - 6 weeks
e. none of the above
98 18. Culture of the etiologic agent was done of the above Jawetz Medical Microbiology 25th edition. p. ANDREW TIU (TOP 1 FINAL
case. WHich of the following most likely describes the 223 - AUG 2015 MED EXAM -
etiologic agent? BOARDS; TOPNOTCH MARCH
a. hydrogen sulfide production MD FROM CIM) 2016
b. lactose fermenter
c. urease positive
d. indole positive
e. nonmotile

TOPNOTCH MEDICAL BOARD PREP MICROBIOLOGY SUPEREXAM Page 12 of 90


For inquiries visit www.topnotchboardprep.com.ph or email us at topnotchmedicalboardprep@gmail.com
TOPNOTCH MEDICAL BOARD PREP MICROBIOLOGY SUPEREXAM
For inquiries visit www.topnotchboardprep.com.ph or email us at topnotchmedicalboardprep@gmail.com
Item QUESTION EXPLANATION AUTHOR TOPNOTCH
# EXAM
99 19. Which of the following is the treatment of choice of antimicrobial therapy for invasive salmonella ANDREW TIU (TOP 1 FINAL
the above case? infections is with ampicillin, TMP - SMX or a - AUG 2015 MED EXAM -
a. penicillin third generation cephalosporin. BOARDS; TOPNOTCH MARCH
b. aminoglycoside Jawetz Medical Microbiology 25th edition. p. MD FROM CIM) 2016
c. cephalosporin 224
d. tetracycline
e. lincosamide

100 20. After the oathtaking ceremony, the newly licensed Jawetz Medical Microbiology 25th edition. p. ANDREW TIU (TOP 1 FINAL
physician went to a buffet in Cebu and ate a dozen 237 - AUG 2015 MED EXAM -
oysters. The next day he comes to your clinic with BOARDS; TOPNOTCH MARCH
complains of watery stools. Stool culture revealed a MD FROM CIM) 2016
nonlactose fermenter, a positive oxidase test, and
growth on TCBS agar. Which of the following is the
mechanism of action of its drug of choice?
a. inhibits peptidyl transferase
b. inhibits 50S
c. inhibits 30S ribosome
d. inhibits PBP
e. none of the above
101 A 50 y/o female consulted at the OPD due to Cough and Page 14 of Topnotch Handout. Strep. ANGELA PAULINE P. DIAGNOSTIC
fever of 5 days duration. On PE, VS were T: 39.80 C, PR Pneumoniae is gram positive cocci in pairs, CALIMAG-LOYOLA EXAM - AUG
108/min, RR 30/min, with intercostal and subcostal Catalase negative, alpha-hemolytic, bile and (TOP 8 - FEB 2015 2015
retractions. She was then admitted with a clinical optochin sensitive. MED BOARDS;
impression of Community Acquired Pneumonia. Gram TOPNOTCH MD
stain and culture of the sputum revealed Gram positive FROM UST)
cocci in pairs, Catalase negative, alpha-hemolytic, bile
and optochin sensitive. What is the most likely bacterial
causative agent?
A. Strep. Agalactiae
B. Strep. Pyogenes
C. Staph. Aureus
D. Strep. Pneumoniae
E. Staph. Saprophyticus
102 Which of the following viruses present as hemorrhagic SIMILAR TO PREVIOUS BOARD EXAM ANGELA PAULINE P. DIAGNOSTIC
fever:
CONCEPT/PRINCIPLE.Viral hemorrhagic CALIMAG-LOYOLA EXAM - AUG
A. Marburg fevers (VHFs) refer to a group of illnesses that (TOP 8 - FEB 2015 2015
B. Ebola are caused by several distinct families of MED BOARDS;
C. Dengue viruses. VHFs are caused by viruses of four TOPNOTCH MD
D. Lassa distinct families: arenaviruses, filoviruses, FROM UST)
E. All of the above bunyaviruses, and flaviviruses.In general, the
term "viral hemorrhagic fever" is used to
describe a severe multisystem syndrome
(multisystem in that multiple organ systems in
the body are affected). Characteristically, the
overall vascular system is damaged, and the
body's ability to regulate itself is impaired.
These symptoms are often accompanied by
hemorrhage (bleeding); however, the bleeding
is itself rarely life-threatening. While some
types of hemorrhagic fever viruses can cause
relatively mild illnesses, many of these viruses
cause severe, life-threatening disease.
Reference:http://www.cdc.gov/ncidod/dvrd/
spb/mnpages/dispages/vhf.htm
103 A 38 y/o female worked as a teacher in palawan. She Page 52 of Topnotch Handout. A-P. ovale, C- ANGELA PAULINE P. DIAGNOSTIC
now presents with headache, occasional nausea, none, D and E-P. vivax and ovale, CALIMAG-LOYOLA EXAM - AUG
vomiting and periodic fever. Which of the following (TOP 8 - FEB 2015 2015
would fit a diagnosis of Plasmodium falciparum malaria: MED BOARDS;
A. Dividing schizonts with 8 progeny TOPNOTCH MD
B. Banana shaped gametocytes, RBC preference all ages FROM UST)
C. Has a periodicity of malignant quartan
D. Parasites are found inside enlarged RBCs
E. Parasitizes young RBCs the RBCs become slightly
enlarged
104 A 5 y/o male was admitted due to colicky abdominal Page 59 of Topnotch Handout. Principal site of ANGELA PAULINE P. DIAGNOSTIC
pain and vomiting. Patient was noted to be malourished. tissue reaction is the lungs, where CALIMAG-LOYOLA EXAM - AUG
Past medical history revealed sudden onset of wheezing inflammation with an eosinophilic exudate (TOP 8 - FEB 2015 2015
and eosinophilia 3 weeks PTA. 2 hours after admission occurs. Heavy worm burden may contribute to MED BOARDS;
the patient passed out a bolus of worms What is the malnutrition. TOPNOTCH MD
most likely diagnosis: FROM UST)
A. Filariasis
B. Strongyloidiasis
C. Ascariasis
D. Trichinosis
E. Enterobiasis

TOPNOTCH MEDICAL BOARD PREP MICROBIOLOGY SUPEREXAM Page 13 of 90


For inquiries visit www.topnotchboardprep.com.ph or email us at topnotchmedicalboardprep@gmail.com
TOPNOTCH MEDICAL BOARD PREP MICROBIOLOGY SUPEREXAM
For inquiries visit www.topnotchboardprep.com.ph or email us at topnotchmedicalboardprep@gmail.com
Item QUESTION EXPLANATION AUTHOR TOPNOTCH
# EXAM
105 A 35 y/o HIV-positive female presents at the ER with Page 51 of Topnotch Handout. Treatment for ANGELA PAULINE P. DIAGNOSTIC
severe diarrhea and dizziness. Her most recent CD4 Cryptosporidium parvum in CALIMAG-LOYOLA EXAM - AUG
count was 24 cells/mm3. Due to financial constraints she immunocompetent patients is Nitazoxanide, (TOP 8 - FEB 2015 2015
was not able to take any medications related to her HIV this doesn't work in immunocompromised MED BOARDS;
disease. Her diarrhea began about 10 days ago and she patients. For HIV positive patients the TOPNOTCH MD
now has 10 to 15 watery stools per day, abdominal treatment is HAART. FROM UST)
cramping, and nausea. On PE her temperature is 38.1°C,
BP 86/60 mmHg, and minimal diffuse abdominal
tenderness with deep palpation. A modified acid-fast
stain on a stool sample is presumptively identified as
Cryptosporidium species. The recommended treatment
for her is: A.
Lamivudine, Zidovudine and Indinavir
B. Nitazoxanide
C. Paromomycin
D. Azithromycin
E. Paromomycin + Azithromycin
106 Which of the following is a true statement regarding the Page 3 of Topnotch Handout. A- The cellular ANGELA PAULINE P. DIAGNOSTIC
pathogenesis of Prion infection? form of the protein PrPc is encoded by the CALIMAG-LOYOLA EXAM - AUG
A. The cellular form of the protein PrPc is encoded by host's CHROMOSOMAL DNA. B-Abnormal (TOP 8 - FEB 2015 2015
the host's mitochondrial DNA. isoform modify folding of normal prion-like MED BOARDS;
B. Abnormal isoform modify folding of normal prion- proteins found in the body and has a high TOPNOTCH MD
like proteins found in the body and has a high alpha- BETA-SHEET content. D- The new PrPsc is FROM UST)
helix content. taken in by ENDOCYTOSIS and accumulates in
C. PrPsc continues to accumulate as the endosome ENDOSOMES.
contents are transfered to lysosomes.
D. The new PrPsc is taken in by pinocytosis and
accumulates in lysosomes
E. None of the above
107 Which of the following would result in Disseminated SIMILAR TO PREVIOUS BOARD EXAM ANGELA PAULINE P. DIAGNOSTIC
intravascular coagulation?
CONCEPT/PRINCIPLE. Page 8 of Topnotch CALIMAG-LOYOLA EXAM - AUG
A. Activation of Hageman factor Handout. Endotoxin is an integral part of (TOP 8 - FEB 2015 2015
B. ADP-ribosylation Gram Negative bacteria cell walls. Lipid A, the MED BOARDS;
C. Deactivation of Tissue factor toxic component of LPS activates tissue factor TOPNOTCH MD
D. Activation of c3a and c5a in turn activating the coagulation cascade FROM UST)
E. Gram negative sepsis resulting in DIC.

108 Management of generalized tetanus requires the Page 18 of Topnotch Handout. DOC is ANGELA PAULINE P. DIAGNOSTIC
following except:
Metronidazole not Penicillin because CALIMAG-LOYOLA EXAM - AUG
A. Frequent suctioning of oral secretions to prevent Penicillin inhibits glycine activity and hence (TOP 8 - FEB 2015 2015
aspiration. should not be used because it exacerbates MED BOARDS;
B. Surgical wound debridement spasms. TOPNOTCH MD
C. Administration of Tetanus immune globulin FROM UST)
D. Penicillin G intravenously given for 10-14 days
E. Vaccination with Tetanus toxoid
109 A 1 month old male with pertussis was given an Page 22 of handoout. Macrolides, most esp ANGELA PAULINE P. DIAGNOSTIC
antibiotic, after a few days he developed hypertrophic Erythromycin are not given in infants due to CALIMAG-LOYOLA EXAM - AUG
pyloric stenosis. Which of the following drugs could have increased risk of hypertrophic pyloric (TOP 8 - FEB 2015 2015
been given:
stenosis. MED BOARDS;
A. Clarithromycin TOPNOTCH MD
B. Erythromycin FROM UST)
C. TMX-SMP
D. Azithromycin
E. Chloramphenicol

110 A 7 y/o female with high fever, sore throat developed Page 15 of Topnotch Handout. The diagnosis ANGELA PAULINE P. DIAGNOSTIC
diffuse finely papular erythematous rash that feels like of this patient is Scarlet fever and the etiologic CALIMAG-LOYOLA EXAM - AUG
sandpaper, hyperpigmentation of armpits and agent is Strep pyogenes/ Group A Strep. (TOP 8 - FEB 2015 2015
strawberry tongue. Which of the following is the most MED BOARDS;
likely etiologic agent: A. Staph. TOPNOTCH MD
aureus FROM UST)
B. Group A Strep
C. Staph. epidermidis
D. Group B strep
E. None of the above
111 What test is used to determine the susceptibility for the Page 15 of Topnotch Handout. Modified elek ANGELA PAULINE P. DIAGNOSTIC
above diagnosis:
test is for C. diptheria to detect toxigenicity. CALIMAG-LOYOLA EXAM - AUG
A. Dick test Lim broth test is used for pregnant women to (TOP 8 - FEB 2015 2015
B. Modified Elek test test for GBS. String test is used for Giardiasis. MED BOARDS;
C. Lim broth test Rumpel-Leede is a capillary-fragility test. TOPNOTCH MD
D. String test FROM UST)
E. Rumpel-Leede test

112 Which of the following CSF analysis results point to a Page 28 of Topnotch Handout. And page 54 of ANGELA PAULINE P. DIAGNOSTIC
diagnosis of TB meningitis?
pedia Topnotch Handout. In TB CALIMAG-LOYOLA EXAM - AUG
A. Clear, 80 wbc/hpf, protein 45 mg/dL, 70% of blood meningitis,Pressure is usually elevated, 10- (TOP 8 - FEB 2015 2015
sugar 500 WBC mm3; PMNs early but lymphocytes MED BOARDS;
B. Colorless, 3-5 wbc/hpf all lymphocytes, protein 15- predominate. 100-3000 protein (mg/dL), and TOPNOTCH MD
45 mg/dL, 70% of blood sugar glucose <50 mg/dL. FROM UST)
C. Xanthochromic, 100 wbc/hpf predominantly
lymphocytes, protein 500 mg/dL, 45% of blood sugar
D. Turbid, 200 wbc/hpf predominantly neutrophils,
protein 150 mg/dL, 20% of blood sugar
E. None of the above

TOPNOTCH MEDICAL BOARD PREP MICROBIOLOGY SUPEREXAM Page 14 of 90


For inquiries visit www.topnotchboardprep.com.ph or email us at topnotchmedicalboardprep@gmail.com
TOPNOTCH MEDICAL BOARD PREP MICROBIOLOGY SUPEREXAM
For inquiries visit www.topnotchboardprep.com.ph or email us at topnotchmedicalboardprep@gmail.com
Item QUESTION EXPLANATION AUTHOR TOPNOTCH
# EXAM
113 This disease is associated with the appearance of Page 24 of Topnotch Handout. The ANGELA PAULINE P. DIAGNOSTIC
transient maculopapular rashes that blanches on maculopapular rashes are rose spots seen CALIMAG-LOYOLA EXAM - AUG
pressure, usually seen on the chest of 50% of children on during the second week of illness. (TOP 8 - FEB 2015 2015
the 7th to 10th day of fever: MED BOARDS;
A. Roseola infantum TOPNOTCH MD
B. Exanthem subitum FROM UST)
C. Eythema infectiosum
D. Typhoid fever
E. Dengue

114 Which of the following would be positive in the serology SIMILAR TO PREVIOUS BOARD EXAM ANGELA PAULINE P. DIAGNOSTIC
of a patient with Hepatitis B during the window period:
CONCEPT/PRINCIPLE. Page 41 of Topnotch CALIMAG-LOYOLA EXAM - AUG
A. HBs Antigen Handout. Window period- HBsAG, Anti-HBs, (TOP 8 - FEB 2015 2015
B. HBs Antibody HBeAG-negative; Anti-HBc-Positive MED BOARDS;
C. Hbe Antigen TOPNOTCH MD
D. HBc Antibody FROM UST)
E. Both A and B

115 An 8 day old female infant was taken to the ER for Page 44 of Topnotch Handout. The fusion ANGELA PAULINE P. DIAGNOSTIC
breathing and feeding difficulties. She was born full-term protein of RSV causes the cells to fuse forming CALIMAG-LOYOLA EXAM - AUG
via spontaneous vaginal delivery at 39 weeks gestation multinucleated giant cells called syncytia (TOP 8 - FEB 2015 2015
to a 21 y/o mother. The mother stated the infant has had hence giving rise to the name of the MED BOARDS;
nasal congestion since birth, which worsened by day 5. virus.Surface spikes are fusion proteins not TOPNOTCH MD
Nasal discharge described as thin and clear with an hemaglutinins and neuraminadases. It is an FROM UST)
occasional thick white mucus plug removed by bulb enveloped virus with a helical nucleocapsid It
aspirator. And after three days she noted, the infant had has one piece of single-stranded, negative
difficulty feeding due to excessive nasal secretions hence polarity RNA
consult. The infant tested positive for RSV on nasal swab.
Which of the following statements is correct regarding
RSV: A. The
fusion protein of RSV causes the cells to fuse forming
multinucleated giant cells
B. Surface spikes are hemaglutinins and
neuraminadases
C. It is an enveloped virus with a linear nucleocapsid
D. It has one piece of double-stranded, negative
polarity DNA
E. All of the above
116 A 24 y/o female G1P1 (1001) went for a gynecologic Page 52 of Topnotch Handout. A and C are ANGELA PAULINE P. DIAGNOSTIC
check up because she has vaginal itching and burning treatment for PID. B is treatment for CALIMAG-LOYOLA EXAM - AUG
sensation. She also noted watery, foul smelling greenish chlamydia. E is for primary syphillis. (TOP 8 - FEB 2015 2015
vaginal discharge. On Speculum exam, there appears to MED BOARDS;
be punctate hemorrhages on the ectocervix. What is the TOPNOTCH MD
recommended treatment for the patient? FROM UST)
A. Ceftriaxone 250 mg IM in a single dose
B. Doxycycline 100 mg orally twice a day for 7 days
C. Metronidazole 500 mg orally 2 times a day for 14
days
D. Metronidazole 2 grams single dose
E. Benzathine penicillin G 2.4 million units
intramuscularly (IM) in a single dose
117 A 35 y/o female G3P3 (2012), presented to the ER due to Page 39 of Topnotch Handout. This is a case of ANGELA PAULINE P. DIAGNOSTIC
hypogastric pain. On history, 4 months PTC she has a squamous cell carcinoma of the cervix. HPV CALIMAG-LOYOLA EXAM - AUG
noted abnormal vaginal bleeding and foul smelling 16, 18, 31 and 33 are associated with (TOP 8 - FEB 2015 2015
vaginal discharge. On colposcopic exam, a vegetative carcinoma of the cervix, penis and anus. MED BOARDS;
lesion on the cervix was observed, biopsy of the lesion TOPNOTCH MD
was done which showed nests and tongues of malignant FROM UST)
squamous epithelium invading the underlying stroma
there was also noted koilocytic atypia. Which of the
following is associated with the diagnosis:
A. HPV 1 and 4
B. HPV 16 and 18
C. HPV 6 and 11
D. HPV 9 and 11
E. HPV 3 and 14
118 In the bacterial growth curve, it is during this period Page 5 of Topnotch Handout. Phase 3 is the ANGELA PAULINE P. DIAGNOSTIC
when there is accumulation of toxic products and maximum stationary phase where in CALIMAG-LOYOLA EXAM - AUG
exhaustion of nutrients in the medium. Cell division is exhaustion of nutrients or accumulation of (TOP 8 - FEB 2015 2015
still occuring although the growth rate is zero. It is also toxic products occur. There is still cell division MED BOARDS;
in this phase when spores are formed.: A. but the population is constant because TOPNOTCH MD
Lag born=death (zero growth rate). FROM UST)
B. Log
C. Exponential
D. Stationary
E. Decline

TOPNOTCH MEDICAL BOARD PREP MICROBIOLOGY SUPEREXAM Page 15 of 90


For inquiries visit www.topnotchboardprep.com.ph or email us at topnotchmedicalboardprep@gmail.com
TOPNOTCH MEDICAL BOARD PREP MICROBIOLOGY SUPEREXAM
For inquiries visit www.topnotchboardprep.com.ph or email us at topnotchmedicalboardprep@gmail.com
Item QUESTION EXPLANATION AUTHOR TOPNOTCH
# EXAM
119 A 9 month old male developed high fever for three days Page 38 of Topnotch Handout. This is HHV-6 ANGELA PAULINE P. DIAGNOSTIC
but was otherwise playful. On the fourth day, fever was infection, which is from the Herpesviridae CALIMAG-LOYOLA EXAM - AUG
gone but he had maculopapular rashes on the face which family. These viruses are enveloped with (TOP 8 - FEB 2015 2015
rapidly spread all over the body. This common disease is icosahedral nucleocapsid and linear ds-DNA. A MED BOARDS;
caused by an agent which has the following is characteristic of he Parvoviridae. B is for TOPNOTCH MD
characteristic: Reoviruses. C is for Adenoviruses while D is FROM UST)
A. Naked virus with linear ss-DNA characteristic of Coronavirus.
B. Naked virus with linear ds-RNA
C. Naked virus with linear ds-DNA
D. Enveloped virus with helical ss-RNA
E. Enveloped virus with linear ds-DNA
120 Thayer-martin agar, which is a selective medium for N. Page 9 of Topnotch Handout. Bacitracin is not ANGELA PAULINE P. DIAGNOSTIC
gonorrhoeae contains the following antibiotics except:
included in the antibiotics found in Thayer CALIMAG-LOYOLA EXAM - AUG
A. Bacitracin martin. Bacitracin is used in the differentiation (TOP 8 - FEB 2015 2015
B. Colistin of Beta-hemolytic strep. Group A is Bacitracin MED BOARDS;
C. Nystatin sensitive while Group B is Bacitracin resistant. TOPNOTCH MD
D. Vancomycin FROM UST)
E. None of the above

121 The infective stage of schistosoma is: Schistosoma japonicum, a trematode, causes LYNN DARYL MIDTERM 1
A. Embryonated eggs Oriental blood fluke is transmitted through FELICIANO EXAM - AUG
B. Cercaria penetration of cercariae in the skin. VILLAMATER, MD 2015
C. Cystecerci Diagnostic stage: eggs in feces. (TOP 5 - FEB 2015
D. Mastigote MED BOARDS;
E. Trophozoite TOPNOTCH MD
FROM EAC)

122 Schistosomiasis is endemic in the following provinces in The following are areas of endemicity of LYNN DARYL MIDTERM 1
the Philippines EXCEPT: Schistosomiasis: Sorsogon, Samar, Leyte, FELICIANO EXAM - AUG
A. Misamis Oriental Oriental Mindoro, Bohol, all of Mindanao VILLAMATER, MD 2015
B. Oriental Mindoro EXCEPT Misamis Oriental. SIMILAR TO (TOP 5 - FEB 2015
C. Samar PREVIOUS BOARD EXAM CONCEPT MED BOARDS;
D. Sorsogon TOPNOTCH MD
E. Bohol FROM EAC)

123 Malaria can be transmitted through/during: Malaria can be transmitted through a bit of an LYNN DARYL MIDTERM 1
A. Intravenous route infective female Anopheles mosquitoes that FELICIANO EXAM - AUG
B. Transplacental route have been infected through a previous blood VILLAMATER, MD 2015
C. Intrapartum meal taken from an infected person. Other (TOP 5 - FEB 2015
D. Contaminated needle routes of transmission: blood transfusion, MED BOARDS;
E. All of the above organ transplant, shared use of needles or TOPNOTCH MD
syringes, from mother to her unborn infant FROM EAC)
before or during delivery (congenital malaria).
(Source: CDC) SIMILAR TO PREVIOUS BOARD
EXAM CONCEPT
124 A 35-year old prenant woman was about to travel to Mefloquine is the malaria prophylactic drug of LYNN DARYL MIDTERM 1
Palawan. Which of the following prophylactic medication choice for pregnant women. Chloroquine - FELICIANO EXAM - AUG
for Malaria will you prescribe? prophylaxis for areas without resistant P VILLAMATER, MD 2015
A. Chloroquine falciparum, non-pregnant. Doxycycline - (TOP 5 - FEB 2015
B. Doxycyline prophylaxis for areas with multi-drug MED BOARDS;
C. Mefloquine resistant P falciparum, non-pregnant. TOPNOTCH MD
D. Primaquine Primaquine - terminal prophylaxis of P vivax FROM EAC)
E. Quinidine and P ovale prevention. "The question is
prophylaxis. IF the question is treatment of
severe cases of malaria/pregnant, the answer
would be Quinidine. Centers for Disease
Control and Prevention (CDC) consider
chloroquine to be the drug of choice for
malaria prophylaxis during pregnancy for
women traveling to areas where chloroquine-
resistant Plasmodium falciparum has not been
reported. For travel to areas where
chloroquine resistance is present, mefloquine
is the only medication recommended by the
CDC for malaria prophylaxis during
pregnancy. Source: CDC Recently, CDC even
recommend Mefloquine as treatment and
prevention for malaria in pregnancy."
SIMILAR TO PREVIOUS BOARD EXAM
CONCEPT.
125 A 15-year old female male was brought to the hospital by Option A is incorrect. Anti-DNAse B document LYNN DARYL MIDTERM 1
her parents due to sore throat and fever. On physical antecedent skin infection.. It is the ASO titers FELICIANO EXAM - AUG
examination, he has pharyngeal and tonsillar exudates that document antecedent pharyngitis. Option VILLAMATER, MD 2015
and tenderness on the cervical lymph nodes. Throat B is incorrect. Glomerulus in incorrect. The (TOP 5 - FEB 2015
culture done showed gram-positive cocci in chains with immunologic spectrum of this orgnasim MED BOARDS;
complete hemolysis and is catalase negative. Which of causes glomerulonephritis (PSGN) is caused TOPNOTCH MD
the following is true regarding the most likely cause of by M protein that incites immune complex FROM EAC)
this condition? deposition on the glomerular basement
A. Anti-DNAse B titers specificially document membrane. Streptococcal toxic shock
antecedent pharyngitis. syndrome is caused by pyogenic exotoxin A
B. Habitat of this organism include oropharynx, skin, (not exotoxin B).
and glomerulus.
C. The same organism causes toxic shock syndrome
which is due to Exotoxin B
D. It is the most common bacterial cause of sore throat.
E. Options C and D only.

TOPNOTCH MEDICAL BOARD PREP MICROBIOLOGY SUPEREXAM Page 16 of 90


For inquiries visit www.topnotchboardprep.com.ph or email us at topnotchmedicalboardprep@gmail.com
TOPNOTCH MEDICAL BOARD PREP MICROBIOLOGY SUPEREXAM
For inquiries visit www.topnotchboardprep.com.ph or email us at topnotchmedicalboardprep@gmail.com
Item QUESTION EXPLANATION AUTHOR TOPNOTCH
# EXAM
126 A 24-year old G2P1, 37 weeks AOG had a history of Streptococcus agalactiae (Group B LYNN DARYL MIDTERM 1
urinary tract infection and is about to deliver her second streptococcus), a gram-positive cocci, beta- FELICIANO EXAM - AUG
child. Her first child succumbed from neonatal hemolytic, bacitracin-resistant, is the most VILLAMATER, MD 2015
pneumonia and sepsis. Vaginal swab was done, which common cause of neonatal pneumonia, sepsis, (TOP 5 - FEB 2015
showed gram positive organism and eventual growth on and meningitis. All pregnant women should be MED BOARDS;
LIM broth. Which of the following should be given to the screened for GBS colonization at 35-37 weeks TOPNOTCH MD
patient? AOG, and chemoprophylaxis with IV Penicillin FROM EAC)
A. IV Ampicillin 4 hours prior to delivery or Ampicillin 4 hours prior to delivery should
B. IV Chloramphenicol 4 hours prior to delivery be given.
C. IV Metronidazole 2 hours prior to delivery
D. IV Penicillin 6 hours after delivery
E. IV Ceftriaxone 3 hours prior to delivery
127 A 31-year old female presented with grayish vaginal Candidiasis - whitish, curd-like discharge; LYNN DARYL MIDTERM 1
discharge, burning urination, with mild vaginal itching. Trichomoniasis - greenish-yellow , foul- FELICIANO EXAM - AUG
Vaginal pH was noted to be at 6.0 On wet mount, smelling discharge; Gonorrhea and chlamydia VILLAMATER, MD 2015
stipppled epithelial cells appearance being covered by may both present with abnormal vaginal (TOP 5 - FEB 2015
the microorganism was noted. The most likely condition discharge and vaginal discomfort. The MED BOARDS;
described is: presence of alkaline vaginal fluid and presence TOPNOTCH MD
A. Candidiasis of clue cells points to Bacterial vaginosis as FROM EAC)
B. Trichomoniasis the most likely diagnosis. (Note: Similar
C. Bacterial vaginosis question asked in the board exam.)
D. Gonorrhea
E. Chlamydia
128 Which of the following causes hemorrhagic fever? All the options causes hemorrhagic fever. LYNN DARYL MIDTERM 1
A. Marburg SIMILAR TO PREVIOUS BOARD EXAM FELICIANO EXAM - AUG
B. Ebola CONCEPT. VILLAMATER, MD 2015
C. Dengue (TOP 5 - FEB 2015
D. Yellow fever MED BOARDS;
E. All of the above TOPNOTCH MD
FROM EAC)

129 Whichof the following virulence factor present in some Spore - resistance to heat and chemicals; LYNN DARYL MIDTERM 1
bacteria renders protection from opsonization? Granule - site of nutrients and cytoplasm; FELICIANO EXAM - AUG
A. Spore Pilus - attachment and conjugation; Periplasm VILLAMATER, MD 2015
B. Granule - contains hydrolytic enzymes; Polysaccharide (TOP 5 - FEB 2015
C. PIlus capsule - protects agains opsonization and MED BOARDS;
D. Periplasm phagocytosis TOPNOTCH MD
E. Capsule FROM EAC)

130 Which of the following is not true regarding pertussis? Antibiotics such as Erythromycin are shown to LYNN DARYL MIDTERM 1
A. It may present with burst of coughs followed by a eradicate nasopharyngeal carriage of FELICIANO EXAM - AUG
characteristic high-pitched whoop during a long Bordetella pertussis. Immunization against VILLAMATER, MD 2015
inspiratory effort. pertussis is essential for disease prevention. (TOP 5 - FEB 2015
B. Fever is usually minimal throughout the course of All others are characteristics of Pertussis MED BOARDS;
illness. infection. TOPNOTCH MD
C. Older persons are often the source of infection for FROM EAC)
children.
D. Pertussis vaccine eradicates nasopharyngeal
carriage of Bordetella pertussis.
E. It is primarily a toxin-mediated disease
131 Which of the following is the most common and the most Actually both Hepatitis B and C may lead to LYNN DARYL MIDTERM 1
likely implicated organism in liver cirrhosis, primary liver cirrhosis, HCC, and long term carrier or FELICIANO EXAM - AUG
hepatic carcinoma, and long-term carrier of organism? organism. But HCV infection are most likely to VILLAMATER, MD 2015
A. Hepatitis A cause chronic hepatitis in 80% of infected (TOP 5 - FEB 2015
B. Hepatitis B individuals, 1/3 of whom may progress to MED BOARDS;
C. Hepatitis C cirrhosis. Robbins 9th ed., p. 837 SIMILAR TO TOPNOTCH MD
D. Hepatitis D PREVIOUS BOARD EXAM CONCEPT. FROM EAC)
E. Hepatitis E

132 Which of the following consituent of Mycobacterium Muramyl dipeptide complexed with mycolic LYNN DARYL MIDTERM 1
tuberculosis inhibits migration of leukocytes and can acids can cause granuloma formation; FELICIANO EXAM - AUG
serve as an immunologic "adjuvant"? phospholipids induce caseous necrosis. VILLAMATER, MD 2015
A. Mycolic acid Exported repetitive potein (sulfatides) (TOP 5 - FEB 2015
B. Muramyl dipeptide prevents phagosome-lysosomal fusion; MED BOARDS;
C. Repeating sulfatides Tuberculin surface protein elicits delayed TOPNOTCH MD
D. Tuberculin surface protein hypersensitivity. Cord factor, which is the FROM EAC)
E. Cord factor most important virulence factor prevents
leukocyte migration.
133 A 3-year old child presented with fever, petechiae, stiff Neisseria meningitidis is the most common LYNN DARYL MIDTERM 1
neck and positive Kernig and Brudzinski signs. Gram cause of meningitis in ages 2-18 years old. FELICIANO EXAM - AUG
stain of cerebrospinal fluid reveals PMNs, and a few The morphologic description is characteristic VILLAMATER, MD 2015
gram negative cocci. The most likely agent causing the of the organism.This is a case of meningitis. (TOP 5 - FEB 2015
condition is: Most common cause is Neisseria meningitides MED BOARDS;
A. Escherichia coli in this age group. TOPNOTCH MD
B. Pseudomonas aeruginosa FROM EAC)
C. Haemophilus influenzae
D. Neisseria meningitidis
E. Streptococcus agalactiae

TOPNOTCH MEDICAL BOARD PREP MICROBIOLOGY SUPEREXAM Page 17 of 90


For inquiries visit www.topnotchboardprep.com.ph or email us at topnotchmedicalboardprep@gmail.com
TOPNOTCH MEDICAL BOARD PREP MICROBIOLOGY SUPEREXAM
For inquiries visit www.topnotchboardprep.com.ph or email us at topnotchmedicalboardprep@gmail.com
Item QUESTION EXPLANATION AUTHOR TOPNOTCH
# EXAM
134 Patient presented with anorexia, malaise, abdominal This is characteristic of typhoid fever during LYNN DARYL MIDTERM 1
pain and jaundice. Complete blood count showed the second week of illness. During this period, FELICIANO EXAM - AUG
neutropenia and mild thrombocytopenia. What is the culture source can be from urine, bone VILLAMATER, MD 2015
best source of culture specimen for the patient to marrow or the rose spots. Blood and stool can (TOP 5 - FEB 2015
confirm the presence of Samonella? be the culture source during the first week MED BOARDS;
A. Blood and third week of illness respectively. Bone TOPNOTCH MD
B. Stool marrow is the culture source at any given time FROM EAC)
C. Urine during the disease. Abdominal pain and
D. Saliva jaundice are evident on week 2 of the disease.
E. None of the above That’s the clue in the question. Hence, the best
culture source would be Urine and Bone
marrow.
135 A 44 year-year-old gardener complains of persistent Sporothrix schenkii is a dimorphic fungus that LYNN DARYL MIDTERM 1
lesion on the 2nd digit of his left hand which has been lives on vegetaion and occurs most often in FELICIANO EXAM - AUG
persistent for several weeks. On examination, the gardener especially those who prunce roses. VILLAMATER, MD 2015
patient was noted to have mlld fever and nodules on his Sporothrichosis can be transmitted through (TOP 5 - FEB 2015
left forearm. A biopsy of the lesion showed presence of thorn prick. MED BOARDS;
elongated budding yeast. What is the most likely TOPNOTCH MD
etiologic agent? FROM EAC)
A. Blastomyces dermatidis
B. Sporothrix schenckii
C. Histoplasma capsulatum
D. Nocardia asteroides
E. Coccidiodes mycoses
136 A 36-year old female, complains of fever, cough, Morphology of H. influenzae may show flat, LYNN DARYL MIDTERM 1
headache and chest pain. Evidence of pneumonia is seen grayish-brown colonies with diameters of 1-2 FELICIANO EXAM - AUG
by chest radiograph. Gram stain and culture of the mm are present after 24 hours of incubation VILLAMATER, MD 2015
patient's sputum reveals flat, graying brown colonies on chocolate agar. The organism does not (TOP 5 - FEB 2015
with satellite around gram-positive cocci in grape-like grow on sheep blood agar except around MED BOARDS;
clusters. The most likely pathogen responsible for this colonies of staphylocci ("satellite TOPNOTCH MD
pneumonia is: phenomenon"). Identification of organism FROM EAC)
A. Haemophilus influenzae depends in part upon demonstrating the need
B. Legionella pneumophila for Factor X (hemin) and Factor V or
C. Mycoplasma pneumonia nicotinamide adenine nucleotide, yielding the
D. Corynebacterium diphtheriae satellite growth phenomenon.
E. Klebsiella pneumoniae
137 Which of the following is true regarding urine collection Option A is incorrect. Urine specimen for LYNN DARYL MIDTERM 1
for laboratory testing? culture should be submitted immediately or FELICIANO EXAM - AUG
A. Urine specimen for culture should be submitted to may be kept 6-8 hours under refrigeration VILLAMATER, MD 2015
the laboratory within 24 hours of collection at room with no chemical preservatives. Option C is (TOP 5 - FEB 2015
temperature. incorrect. Urine specimen for transport should MED BOARDS;
B. If transportation to the laboratory is go beyond 24 be maintained at 2-8°C (not 10-12) Option D TOPNOTCH MD
hours, chemical preservative should be added and be is incorrect. Laboratory tests for urine FROM EAC)
submitted to the laboratory within 72 hours. specimen no longer than 1 or 2 hours (not 6
C. Urine should be maintained at 10-12°C when hours) should elapse before the tests are done
transporting urine specimen. except if refrigerated. Specimens should be
D. Laboratory tests for urine specimen no longer than 6 transported to the laboratory within 2 hours,
hours should elapse before the tests are done.. but preferably within 30 minutes of collection.
E. Urine specimen should be transported to the (Textbook of Diagnostic Microbiology;
laboratory within 6 hours. Turgeon Clinical Laboratory Science) Similar
question asked in the boards.
138 Finding of sausage-shaped or banana-shaped P. malariae - compact gametocytes; P. vivax - LYNN DARYL MIDTERM 1
gametocytes in peripheral smear is characteristic of? large round, P. falciparum - banana shaped; P. FELICIANO EXAM - AUG
A. Plasmodium malariae ovale - small round VILLAMATER, MD 2015
B. Plasmodium vivax (TOP 5 - FEB 2015
C. Plasmodium falciparum MED BOARDS;
D. Plasmodium ovale TOPNOTCH MD
E. None of the above FROM EAC)

139 The protein responsible for rapid antigenic change in gp120 is the protein that interacts with the LYNN DARYL MIDTERM 1
Human Immunodeficiency Virus is" CD4 receptor and is the one responsible for FELICIANO EXAM - AUG
A. p24 rapid antigenic changes. The rapid VILLAMATER, MD 2015
B. p7 appearance of variants causes difficulty in (TOP 5 - FEB 2015
C. Reverse transcriptase preparation of vaccine against HIV. MED BOARDS;
D. gp120 TOPNOTCH MD
E. gp41 FROM EAC)

140 Which of the following parasite most likely causes Trichinella spiralis is a tissue nematode that LYNN DARYL MIDTERM 1
myocarditis? can be transmitted through ingestion of FELICIANO EXAM - AUG
A. Taenia solium undercooked pork. The organism may be VILLAMATER, MD 2015
B. Strongyloides stercoralis liberated from port by gastric juices, and (TOP 5 - FEB 2015
C. Trichuris trichiura disseminate hematogeneously to striated MED BOARDS;
D. Trichinella spiralis muscle. SIMILAR TO PREVIOUS BOARD EXAM TOPNOTCH MD
E. Echinoccus granulosus CONCEPT. FROM EAC)

141 RM a 29 year old had a blood smear showing banana Banana shaped gametocytes seen in a thin EDWARD HARRY MIDTERM 2
shaped gametocytes, what is the most likely involved blood smear of a person with malaria signifies VALLAJERA, MD EXAM - AUG
species? that the infecting parasite is P. falciparum (TOP 8 - FEB 2015 2015
A. P. malariae MED BOARDS;
B. P. falciparum TOPNOTCH MD
C. P. vivax FROM PERPETUAL
D. P. ovale BINAN)
E. None of the above

TOPNOTCH MEDICAL BOARD PREP MICROBIOLOGY SUPEREXAM Page 18 of 90


For inquiries visit www.topnotchboardprep.com.ph or email us at topnotchmedicalboardprep@gmail.com
TOPNOTCH MEDICAL BOARD PREP MICROBIOLOGY SUPEREXAM
For inquiries visit www.topnotchboardprep.com.ph or email us at topnotchmedicalboardprep@gmail.com
Item QUESTION EXPLANATION AUTHOR TOPNOTCH
# EXAM
142 What makes N. gonorrheae stand when cultured on The presence of the 3 makes Thayer Martin EDWARD HARRY MIDTERM 2
Thayer-Martin Agar Agar selective for the pathogenic Neisseria VALLAJERA, MD EXAM - AUG
A. Oxidase positive cultured from non-sterile sites like the vagina, (TOP 8 - FEB 2015 2015
B. Infused with bacitracin rectum or penile urethra. MED BOARDS;
C. Presence of inhibitors (Vancomycin, Collistin and TOPNOTCH MD
Nystatin) FROM PERPETUAL
D. Catalase positive BINAN)
E. None of the above
143 The virulence factor of TB is otherwise known as? The virulence factor of M. tuberculosis is EDWARD HARRY MIDTERM 2
A. Exotoxin A otherwise known as the cord factor. VALLAJERA, MD EXAM - AUG
B. Endotoxin (TOP 8 - FEB 2015 2015
C. Mycolic acid MED BOARDS;
D. Periplasm TOPNOTCH MD
E. Cord factor FROM PERPETUAL
BINAN)

144 Exotoxin A of Streptococcus causes what? Exotoxin A or streptococcal pyrogenic EDWARD HARRY MIDTERM 2
A. Scarlet Fever exotoxin of Streptococcus pyogenes causes VALLAJERA, MD EXAM - AUG
B. Rheumatic Fever Strep TSS and Scarlet fever, rheumatic fever (TOP 8 - FEB 2015 2015
C. Streptococcal Toxic Shock Syndrome and PSGN are not attributed to the direct MED BOARDS;
D. Post Streptococcal Glomerulonephritis effect of the bacteria but rather a TOPNOTCH MD
E. Necrotizing fasciitis hypersensitivity response. (Jawetz, 26th ed) FROM PERPETUAL
BINAN)

145 What is the treatment when the diarrhea causes This is a case of cholera and the drug of choice EDWARD HARRY MIDTERM 2
abdominal pain and rice watery stools. is tetracycline or azithromycin VALLAJERA, MD EXAM - AUG
A. Tetracycline (TOP 8 - FEB 2015 2015
B. Streptomycin MED BOARDS;
C. Clindamycin TOPNOTCH MD
D. Cefuroxime FROM PERPETUAL
E. Ceftriaxone BINAN)

146 What is the characteristic feature of someone with Bradycardia in the presence of fever is one EDWARD HARRY MIDTERM 2
Salmonella infection? characteristic finding that would warrant you VALLAJERA, MD EXAM - AUG
A. Hypotension to confirm if a patient has typhoid fever. (TOP 8 - FEB 2015 2015
B. Diffuse abdominal pain MED BOARDS;
C. Episodes of watery diarrhea TOPNOTCH MD
D. Alternating episodes of febrile and afebrile episodes FROM PERPETUAL
every 2 or 3 days BINAN)
E. Relative bradycardia despite presence of fever
147 A medical intern does a sputum AFB to a suspected PTB All of the following procedures are correct EDWARD HARRY MIDTERM 2
patient, he does everything right except except that the intern forgot to protect himself VALLAJERA, MD EXAM - AUG
A. He does the procedure without covering his nose and from the airborne particles of PTB (TOP 8 - FEB 2015 2015
mouth MED BOARDS;
B. Asks the patient to breathe deep before coughing TOPNOTCH MD
C. He ensures that the specimen is more of sputum than FROM PERPETUAL
saliva BINAN)
D. Both B and C
E. None of the above
148 When handling a specimen for urine culture, the The 1st stream is discarded when collecting EDWARD HARRY MIDTERM 2
following are correct steps in treating the specimen specimen for urine culture as it usually VALLAJERA, MD EXAM - AUG
except harbors flora from the genital tract and the (TOP 8 - FEB 2015 2015
A. It should be processed in 1 hour if there is no surface epithelium MED BOARDS;
immediately available refrigerator TOPNOTCH MD
B. The 1st stream urine should be included as specimen FROM PERPETUAL
C. It should be collected in clean catch midstream BINAN)
manner
D. It should be stored in a refrigerated container at a
temperature of 18C and can be processed in 24 hours
E. None of the above
149 MJ, a 40 year old man came in to your clinic due to night HIV attaches to CD4 lymphocytes via the EDWARD HARRY MIDTERM 2
sweats, he admits to having illicit sexual affairs, he tested gp120 receptor and also the antigenicity of VALLAJERA, MD EXAM - AUG
(+) on ELISA and (+) also on western blot, you know that this protein changes rapidly, gp41 mediates (TOP 8 - FEB 2015 2015
the causative agent attaches to its target lymphocytes via fusion with host cell, p24 is an important MED BOARDS;
A. p24 serologic marker of infection. P17 is matrix TOPNOTCH MD
B. p7 encoded protein while p7 is a protein for the FROM PERPETUAL
C. gp120 nucleocapsid BINAN)
D. p17
E. gp41

150 What is the most common cause of pneumonia and GBS, E Colie and Listeria are the most EDWARD HARRY MIDTERM 2
sepsis in newborns? common cause of sepsis and pneumonia VALLAJERA, MD EXAM - AUG
A. Group A Beta Hemolytic Streptococci among newborns (TOP 8 - FEB 2015 2015
B. Listeria monocytogenes MED BOARDS;
C. Neisseria gonorrheae TOPNOTCH MD
D. Staphylococcus saprophyticus FROM PERPETUAL
E. Bacillus cereus BINAN)

TOPNOTCH MEDICAL BOARD PREP MICROBIOLOGY SUPEREXAM Page 19 of 90


For inquiries visit www.topnotchboardprep.com.ph or email us at topnotchmedicalboardprep@gmail.com
TOPNOTCH MEDICAL BOARD PREP MICROBIOLOGY SUPEREXAM
For inquiries visit www.topnotchboardprep.com.ph or email us at topnotchmedicalboardprep@gmail.com
Item QUESTION EXPLANATION AUTHOR TOPNOTCH
# EXAM
151 A 38 week AOG pregnant woman tested positive for GBS, Observe the patient until labor commences, it EDWARD HARRY MIDTERM 2
what should you do next? is during this time that antibiotic prophylaxis VALLAJERA, MD EXAM - AUG
A. Give parenteral ampicillin is recommended as giving antibiotics prior to (TOP 8 - FEB 2015 2015
B. Observe the patient onset of labor may be harmful to the MED BOARDS;
C. Repeat the culture pregnancy, repeat cultures are no longer TOPNOTCH MD
D. Test the unborn child for presence of bacteria in the necessary as the patient is already 38 weeks, FROM PERPETUAL
blood well within the recommended screening time BINAN)
E. None of the above of within 5 weeks from delivery (Source: CDC
Guidelines in GBS for Obstetric providers)
152 Which of the following is a cell wall inhibitor? Ceftriaxone is a cephalosporin which EDWARD HARRY MIDTERM 2
A. Azithromycin primarily acts on bacterial cell wall synthesis VALLAJERA, MD EXAM - AUG
B. Clindamycin (TOP 8 - FEB 2015 2015
C. Erythromycin MED BOARDS;
D. Ceftriaxone TOPNOTCH MD
E. Levofloxacin FROM PERPETUAL
BINAN)

153 Which of the following is a bacteriostatic agent? Erythromycin is a bacteriostatic agent, the EDWARD HARRY MIDTERM 2
A. Erythromycin rest are bactericidal VALLAJERA, MD EXAM - AUG
B. Vancomycin (TOP 8 - FEB 2015 2015
C. Meropenem MED BOARDS;
D. Spectinomycin TOPNOTCH MD
E. Ertapenem FROM PERPETUAL
BINAN)

154 What organism is demonstrated by the NSS hanging NSS hanging drop is a test for microbial EDWARD HARRY MIDTERM 2
drop motility, in this case, Trichomonas vaginalis VALLAJERA, MD EXAM - AUG
A. Clostridium infection can be demonstrated moving in an aqueous (TOP 8 - FEB 2015 2015
B. Moniliasis environment inside the drop suspended in a MED BOARDS;
C. Candidiasis cover slip. TOPNOTCH MD
D. Trichomoniasis FROM PERPETUAL
E. Trichinosis BINAN)

155 In a case of an urinary tract infection, the following are The most common organism isolated in EDWARD HARRY MIDTERM 2
the characteristic of the most common organism except: community acquired UTI is E. coli, all of the VALLAJERA, MD EXAM - AUG
A. Endotoxin producing following are the characteristics of E. coli, a (TOP 8 - FEB 2015 2015
B. Contains a thick cell wall gram negative bacteria except a thick cell wall MED BOARDS;
C. Facultative anaerobe TOPNOTCH MD
D. Gram negative FROM PERPETUAL
E. None of the above BINAN)

156 Which the following causes hemorrhagic fever All of the following cause hemorhagic fevers. EDWARD HARRY MIDTERM 2
A. Dengue VALLAJERA, MD EXAM - AUG
B. Ebola (TOP 8 - FEB 2015 2015
C. Marburg MED BOARDS;
D. All of the above TOPNOTCH MD
E. A and B FROM PERPETUAL
BINAN)

157 Which of the following is transmitted via airborne All of the following except herpes simplex are EDWARD HARRY MIDTERM 2
droplets? transported via airborne transmission VALLAJERA, MD EXAM - AUG
A. Varicella (TOP 8 - FEB 2015 2015
B. Mumps MED BOARDS;
C. Herpes simplex TOPNOTCH MD
D. All of the above FROM PERPETUAL
E. A and B only BINAN)

158 Which of the following agents is added in drug resistant Among the choices, amikacin is most effective EDWARD HARRY MIDTERM 2
TB therapy? in TB as resistance rates is still low. VALLAJERA, MD EXAM - AUG
A. Amoxicillin + clavulanate (TOP 8 - FEB 2015 2015
B. Ceftazidime MED BOARDS;
C. Amikacin TOPNOTCH MD
D. Gentamicin FROM PERPETUAL
E. None of the above BINAN)

159 The causative agent of sepsis in patients with sickle cell S. choleraesuis is closely associated with EDWARD HARRY MIDTERM 2
anemia sepsis in patients with sickle cell anemia VALLAJERA, MD EXAM - AUG
A. S. typhi (TOP 8 - FEB 2015 2015
B. S. enteritidis MED BOARDS;
C. S. typhimurium TOPNOTCH MD
D. S. choleraesuis FROM PERPETUAL
E. None of the above BINAN)

160 JRD, a 35 year old male was treated for malaria, after 6 Relapse is the return of a disease after its EDWARD HARRY MIDTERM 2
months, he developed fever and tested positive for apparent cessation (1-6 months due to VALLAJERA, MD EXAM - AUG
malaria parasites. What is this phenomenon? reactivation of hypnozoites, recrudescence is (TOP 8 - FEB 2015 2015
A. Recrudescence recurrence of the symptoms after a temporary MED BOARDS;
B. Relapse abatement of 2-4 weeks. TOPNOTCH MD
C. Reinfection FROM PERPETUAL
D. Reexposure BINAN)
E. None of the above

TOPNOTCH MEDICAL BOARD PREP MICROBIOLOGY SUPEREXAM Page 20 of 90


For inquiries visit www.topnotchboardprep.com.ph or email us at topnotchmedicalboardprep@gmail.com
TOPNOTCH MEDICAL BOARD PREP MICROBIOLOGY SUPEREXAM
For inquiries visit www.topnotchboardprep.com.ph or email us at topnotchmedicalboardprep@gmail.com
Item QUESTION EXPLANATION AUTHOR TOPNOTCH
# EXAM
161 These are obligate intracellular but acellular parasites of SIMILAR TO PREVIOUS BOARD EXAM HAROLD JAY S. MIDTERM 3
plants with naked RNA and causes no human disease. CONCEPT/PRINCIPLE BAYTEC, MD (TOP 10 EXAM - AUG
A. viruses - FEB 2015 MED 2015
B. viroids BOARDS; TOPNOTCH
C. virea MD FROM FEU)
D. prions
E. Spirochetes

162 Which among the following is the smallest bacteria? HAROLD JAY S. MIDTERM 3
A. spirochetes BAYTEC, MD (TOP 10 EXAM - AUG
B. chlamydia - FEB 2015 MED 2015
C. mycoplasma BOARDS; TOPNOTCH
D. mycobacteria MD FROM FEU)
E. borrelia

163 Which of the following structures is unique for gram peptidoglycan is seen in both G+ and G- HAROLD JAY S. MIDTERM 3
positive organisms? organism. LPS and periplasmic space is seen BAYTEC, MD (TOP 10 EXAM - AUG
A. peptidoglycan in G- organism - FEB 2015 MED 2015
B. Mycolic acid BOARDS; TOPNOTCH
C. Teichoic acid MD FROM FEU)
D. LPS
E. Perplasmis space

164 Which among the following organism is a gram positive All gram positive bacteria have no endotoxin HAROLD JAY S. MIDTERM 3
and has endotoxins? except Listeria monocytogenes BAYTEC, MD (TOP 10 EXAM - AUG
A. pneumococcus - FEB 2015 MED 2015
B. strep bovis BOARDS; TOPNOTCH
C. Staph aureus MD FROM FEU)
D. Bacillus anthracis
E. Listeria monocytogenes

165 In gram staining procedure, which of the following is primary stain is crystal violet, mordant is the HAROLD JAY S. MIDTERM 3
used as a mordant? iodine, decolarizing agent is acetone or BAYTEC, MD (TOP 10 EXAM - AUG
A. Crystal violet alcohol, counterstain is safranin - FEB 2015 MED 2015
B. Iodine BOARDS; TOPNOTCH
C. Acetone MD FROM FEU)
D. Alcohol
E. Safranin

166 Dark-field microscopy is used in visualizing which of the darkfield microscopy is for spirochetes, acid HAROLD JAY S. MIDTERM 3
following organism? fast stain is for mycobacteria, silver stain for BAYTEC, MD (TOP 10 EXAM - AUG
A. Mycobacteria legionella, inclusion bodies for chlamydia - FEB 2015 MED 2015
B. mycoplasma BOARDS; TOPNOTCH
C. Legionella MD FROM FEU)
D. Chlamydia
E. Spirochetes

167 The following structure is an essential component of cytoplasmic membrane, ribosome, nucleoid, HAROLD JAY S. MIDTERM 3
bacteria EXCEPT: mesosome and periplasm are the essential BAYTEC, MD (TOP 10 EXAM - AUG
A. Cytoplasmic membrane components of bacteria - FEB 2015 MED 2015
B. ribosome BOARDS; TOPNOTCH
C. nucleoid MD FROM FEU)
D. plasmid
E. None of the above

168 A 3 year old child was seen at the ER complaining of 4 staph aureus is often associated with HAROLD JAY S. MIDTERM 3
days of fever with difficulty of breathing. Xray shows nosocomial pneumonia, necrotizing BAYTEC, MD (TOP 10 EXAM - AUG
pneumatocele of the left lung. Which of the following pneumonia, empyema or pneumatocele, and - FEB 2015 MED 2015
microorganisms is most likely the causative agent? post viral pneumonia. BOARDS; TOPNOTCH
A. Klebsiella pneumoniae MD FROM FEU)
B. Streptococcus pneumoniae
C. Staph aureus
D. Pseudomonas
E. Legionella

169 Ritter syndrome of Scalded Skin Syndrome is exfoliation SIMILAR TO PREVIOUS BOARD EXAM HAROLD JAY S. MIDTERM 3
of skin usually caused by Gram positive bacteria. The CONCEPT/PRINCIPLE. Separation in Ritter BAYTEC, MD (TOP 10 EXAM - AUG
separation of epidermis happens at which layer? happens in stratum granulosum while TEN or - FEB 2015 MED 2015
A. Stratum basale Lyell happens at dermo-epidermal junction BOARDS; TOPNOTCH
B. Stratum granulosum MD FROM FEU)
C. Stratum spinosum
D. Stratum lucidum
E. Stratum corneum

170 A 55 year old female with history of having a prosthetic SIMILAR TO PREVIOUS BOARD EXAM HAROLD JAY S. MIDTERM 3
heart valve was admitted due to persistence of fever for CONCEPT/PRINCIPLE BAYTEC, MD (TOP 10 EXAM - AUG
5 days. Blood culture revealed gram positive cocci in - FEB 2015 MED 2015
clusters which was catalase positve, coagulase negative BOARDS; TOPNOTCH
and novobiocin sensitive. Which of the following is most MD FROM FEU)
likely the causative agent?
A. Staph aureus
B. Staph epidermidis
C. Staph saprophyticus
D. Strep pyogenes
E. Strep viridans

TOPNOTCH MEDICAL BOARD PREP MICROBIOLOGY SUPEREXAM Page 21 of 90


For inquiries visit www.topnotchboardprep.com.ph or email us at topnotchmedicalboardprep@gmail.com
TOPNOTCH MEDICAL BOARD PREP MICROBIOLOGY SUPEREXAM
For inquiries visit www.topnotchboardprep.com.ph or email us at topnotchmedicalboardprep@gmail.com
Item QUESTION EXPLANATION AUTHOR TOPNOTCH
# EXAM
171 A 4 year old male was seen having perioral blisters with kidney function test is imporant since PSGN is HAROLD JAY S. MIDTERM 3
honey-colored crust. Which among the following is the associated with impetigo. BAYTEC, MD (TOP 10 EXAM - AUG
most important diagnostic test to check few weeks after - FEB 2015 MED 2015
treatment of the lesions? BOARDS; TOPNOTCH
A. Urinalysis and Creatinine MD FROM FEU)
B. CBC
C. Liver function test
D. 2D-echo
E. platelet count

172 A group of students came in to your clinic all Bacillus cereus is oftenly linked to eating HAROLD JAY S. MIDTERM 3
complaining of vomiting. Upon probing, you learned that reheated fried rice BAYTEC, MD (TOP 10 EXAM - AUG
they live in the same dormitory and they ate reheated - FEB 2015 MED 2015
Chinese fried rice for lunch few hours prior to consult. BOARDS; TOPNOTCH
Which of the following is most likely the causative agent? MD FROM FEU)
A. Entamoeba hystolytica
B. E coli
C. staphylococcus aureus
D. Bacillus cereus
E. norwalk virus

173 An infant came in to your clinic and you noticed that he Botulism is often associated with bulging HAROLD JAY S. MIDTERM 3
seems floppy but no fever. Further questioning revealed canned goods and honey. BAYTEC, MD (TOP 10 EXAM - AUG
that his parents often mix honey to his milk. Which of the - FEB 2015 MED 2015
following is the most likely the causative agent? BOARDS; TOPNOTCH
A. Staphylococcus MD FROM FEU)
B. Rotavirus
C. Campylobacter jejuni
D. Clostridium perfringes
E. Clostridium botulinum

174 A 50 year old female came in due to cough and fever Legionella causes atypical pneumonia which HAROLD JAY S. MIDTERM 3
associated with confusion and non bloddy diarrhea. Xray can manifest as cough, fever with associated BAYTEC, MD (TOP 10 EXAM - AUG
suggests pneumonia. Which of the following is most diarrhea, confussion, hematuria, and - FEB 2015 MED 2015
likely the causative agent? proteinuria. This is usually treated with BOARDS; TOPNOTCH
A. Klebsiella macrolides MD FROM FEU)
B. Staphylococcus
C. Pneumococcus
D. Legionella
E. H. influenza

175 Which of the following is associated with development of Campylobacter jejuni infection can cause HAROLD JAY S. MIDTERM 3
Guillian Barre syndrome antigenic cross reactivity between BAYTEC, MD (TOP 10 EXAM - AUG
A. C. jejuni oligosaccharide in bacterial capsule and - FEB 2015 MED 2015
B. Klebsiella pneumonia glycosphingolipids on surface of neural tissues BOARDS; TOPNOTCH
C. H. pylori that causes GBS MD FROM FEU)
D. Proteus
E. Ureaplasma

176 Which of the following is the most important virulence cord factor is the most important virulence HAROLD JAY S. MIDTERM 3
factor for the development of Mycobacterium factor and it prevents leukocyte migration BAYTEC, MD (TOP 10 EXAM - AUG
tuberculosis infection? - FEB 2015 MED 2015
A. Sulfatides BOARDS; TOPNOTCH
B. Cord factor MD FROM FEU)
C. Tuberculin surface protein
D. Mycolic acid
E. Protease A

177 In HIV, which of the following functions as attachment to gp41 is for fusion with the host cell. P24 is for HAROLD JAY S. MIDTERM 3
CD4 protein and it also responsible for antigenic the nucleocapsid, p17 is for viral matrix. BAYTEC, MD (TOP 10 EXAM - AUG
variation of the virus? Reverse trascriptase transcribes RNA genome - FEB 2015 MED 2015
A. p24 into DNA BOARDS; TOPNOTCH
B. p17 MD FROM FEU)
C. Reverse transcriptase
D. gp41
E. Gp120

178 In Entamoeba histolytica infection, what is the drug of Metronidazole is only given for mild, HAROLD JAY S. MIDTERM 3
choice for asymptomatic intestinal infection? moderate to severe infection +/- luminal BAYTEC, MD (TOP 10 EXAM - AUG
A. iodoquinol agent. No need to give it if asymptomatic. - FEB 2015 MED 2015
B. paramomycin BOARDS; TOPNOTCH
C. Diloxanide furoate MD FROM FEU)
D. metronidazole
E. Clarithromycin

TOPNOTCH MEDICAL BOARD PREP MICROBIOLOGY SUPEREXAM Page 22 of 90


For inquiries visit www.topnotchboardprep.com.ph or email us at topnotchmedicalboardprep@gmail.com
TOPNOTCH MEDICAL BOARD PREP MICROBIOLOGY SUPEREXAM
For inquiries visit www.topnotchboardprep.com.ph or email us at topnotchmedicalboardprep@gmail.com
Item QUESTION EXPLANATION AUTHOR TOPNOTCH
# EXAM
179 A patient with history of mountain hiking in Palawan SIMILAR TO PREVIOUS BOARD EXAM HAROLD JAY S. MIDTERM 3
came in for consult due to recurrent high grade fever. CONCEPT/PRINCIPLE. Gametocytes of vivax BAYTEC, MD (TOP 10 EXAM - AUG
Further workup showed blood smear with banana- are large rorund. P malariae are compact. P. - FEB 2015 MED 2015
shaped gametocytes. Which of the following species is ovale are small round BOARDS; TOPNOTCH
the most likely causee? MD FROM FEU)
A. P. falciparum
B. P. vivax
C. P. malariae
D. P. ovale
E. P. vivax or P. ovale
180 Which of the following drugs is the drug of choice for SIMILAR TO PREVIOUS BOARD EXAM HAROLD JAY S. MIDTERM 3
Taeniasis? CONCEPT/PRINCIPLE BAYTEC, MD (TOP 10 EXAM - AUG
A. Albendazole - FEB 2015 MED 2015
B. Mebendazole BOARDS; TOPNOTCH
C. Thiabendazole MD FROM FEU)
D. Iodiquinol
E. Praziquantel

181 What eye drops medication will you give to treat a SIMILAR TO PREVIOUS BOARD EXAM JEAN PAOLO M. FINAL EXAM
gonococcal eye infection? CONCEPT/PRINCIPLE.. Silver nitrate drops is DELFINO, MD (TOP - AUG 2015
A. Bacitracin the answer because it is the only antibacterial 10 - FEB 2015 MED
B. Dexamethasone drug given via drops among the choices given. BOARDS; TOPNOTCH
C. Azithromycin MD FROM FATIMA)
D. Erythromycin
E. Silver nitrate drops

182 Laboratory test confirmatory of Streptococcus Although all of the choices are characteristic of JEAN PAOLO M. FINAL EXAM
pneumoniae infection? S.pneumo, the question only asks for a test DELFINO, MD (TOP - AUG 2015
A. Gram positive diplococci that will give a definitive diagnosis.This means 10 - FEB 2015 MED
B. Catalase-negative that the test should only be positive for BOARDS; TOPNOTCH
C. Alpha-hemolytic S.pneumo. That's why culture characteristics MD FROM FATIMA)
D. Optochin sensitive such as bile solubility and optochin sensitivity
E. All of the above are the important laboratory criteria to
consider to confirm the diagnosis.
183 What is the larget roundworm pathogenic to man? Ascaris is the largest roundworm parasitic to JEAN PAOLO M. FINAL EXAM
A. Taenia solium man. DELFINO, MD (TOP - AUG 2015
B. Necator americanus 10 - FEB 2015 MED
C. Enterobius vermicularis BOARDS; TOPNOTCH
D. Ascaris lumbricoides MD FROM FATIMA)
E. Anisakis

184 What microscopic finding in trophozoite is diagnostic of SIMILAR TO PREVIOUS BOARD EXAM JEAN PAOLO M. FINAL EXAM
Entamoeba histolytica infection? CONCEPT/PRINCIPLE.. The detection of E. DELFINO, MD (TOP - AUG 2015
A. Ingested RBC histolytica trophozoites with ingested RBC is 10 - FEB 2015 MED
B. Pus cells diagnostic of amebiasis. Pus cells are seen in BOARDS; TOPNOTCH
C. Glycogen mass bacillary dysentery. Glycogen mass is seen in MD FROM FATIMA)
D. Charcot-Leyden crystal Iodamoeba and in cyst of entamoeba (not
E. Ingested bacteria trophozoite). Charcot-Leyden crystals are also
seen in Entamoeba coli. Ingested bacteria is
seen in Endolimax nana.
185 Schistosomiasis is endemic in the following provinces in In the Philippines, there are 24 provinces JEAN PAOLO M. FINAL EXAM
the Philippines except? endemic to Schistosoma including Sorsogon, DELFINO, MD (TOP - AUG 2015
A. Misamis Occidental Oriental Mindoro, Samar, Leyte, Bohol, and all 10 - FEB 2015 MED
B. Oriental Mindoro provinces in Mindanao except Misamis BOARDS; TOPNOTCH
C. Misamis Oriental Oriental. SIMILAR TO PREVIOUS BOARD MD FROM FATIMA)
D. Sorsogon EXAM CONCEPT/PRINCIPLE.
E. Bohol

186 Patient presented with jaundice and abdominal mass in the most pathognomonic finding of Hydatid JEAN PAOLO M. FINAL EXAM
the right upper quadrant. Imaging studies revealed cyst or Echinococcosis, if demonstrable, is that DELFINO, MD (TOP - AUG 2015
"daughter cyst within a larger cyst". This finding most of daughter cysts within the larger cyst seen in 10 - FEB 2015 MED
likely suggests what infection? imaging studies. BOARDS; TOPNOTCH
A. Entamoeba histolytica MD FROM FATIMA)
B. Staphylococcus aureus
C. E. coli
D. E. granulosus
E. Anaerobic infection

187 A neonate was found to be lethargic, hypothermic and The diagnosis of this case is neonatal sepsis, JEAN PAOLO M. FINAL EXAM
with poor feeding. History revealed prolonged pre-labor most probably secondary to Group B Strep. DELFINO, MD (TOP - AUG 2015
rupture of membrane prior to birth of the patient. What The drug of choice for this is Penicillin G. 10 - FEB 2015 MED
is the drug of choice in this case? BOARDS; TOPNOTCH
A. Ceftriaxone MD FROM FATIMA)
B. Ampicillin
C. Penicillin
D. Cefuroxime
E. Amoxicillin

188 What is the most common cause of neonatal meningitis? SIMILAR TO PREVIOUS BOARD EXAM JEAN PAOLO M. FINAL EXAM
A. Listeria monocytogenes CONCEPT/PRINCIPLE.. Strep agalactiae. DELFINO, MD (TOP - AUG 2015
B. Escherichia coli giveaway question last boards 10 - FEB 2015 MED
C. Staphylococcus aureus BOARDS; TOPNOTCH
D. Streptococcus pyogenes MD FROM FATIMA)
E. Streptococcus agalactiae

TOPNOTCH MEDICAL BOARD PREP MICROBIOLOGY SUPEREXAM Page 23 of 90


For inquiries visit www.topnotchboardprep.com.ph or email us at topnotchmedicalboardprep@gmail.com
TOPNOTCH MEDICAL BOARD PREP MICROBIOLOGY SUPEREXAM
For inquiries visit www.topnotchboardprep.com.ph or email us at topnotchmedicalboardprep@gmail.com
Item QUESTION EXPLANATION AUTHOR TOPNOTCH
# EXAM
189 What bacteria causes marantic endocarditis? Marantic endocarditis is seen in patients with JEAN PAOLO M. FINAL EXAM
A. Staphylococcus aureus Streptococcus bovis infection. It is associated DELFINO, MD (TOP - AUG 2015
B. Streptococcus pyogenes with abdominal malignancy. 10 - FEB 2015 MED
C. Streptococcus sanguis BOARDS; TOPNOTCH
D. Streptococcus bovis MD FROM FATIMA)
E. Enterococcus faecalis

190 26 year old sexually active female presented with The diagnosis for this case is JEAN PAOLO M. FINAL EXAM
unilateral painful inguinal lymphadenopathy. Extensive Lymphogranuloma Venereum which is caused DELFINO, MD (TOP - AUG 2015
enlargement of chains of inguinal nodes are noted above by infection with Chlamydia trachomatis types 10 - FEB 2015 MED
and below the inguinal ligament. What is the most likely L1-L3. this condition starts as a papule or BOARDS; TOPNOTCH
etiologic agent? vesicle which ulcerates and leads to inguinal MD FROM FATIMA)
A. Hemophilus ducreyi lymphadenitis (buboes). (+) Groove sign=
B. Chlamydia trachomatis Extensive enlargement of chains of inguinal
C. Klebsiella granulomatis nodes are noted above and below the inguinal
D. Gardnerella vaginalis ligament.
E. Yersinia enterocolitica
191 30 year old male had pinkish, papular skin lesion. Biopsy Diagnosis is Molluscum contagiosum virus JEAN PAOLO M. FINAL EXAM
of the lesion showed the presence of Henderson- infection. It is characterized by pinkish, DELFINO, MD (TOP - AUG 2015
Peterson bodies. What is the drug of choice? papular skin lesions with an umbilicated 10 - FEB 2015 MED
A. Acyclovir center. Intracytoplasmic eosinophilic BOARDS; TOPNOTCH
B. Foscarnet inclusions (Henderson-Peterson bodies) are MD FROM FATIMA)
C. Cidofovir seen in biopsy of the lesion. The drug of choice
D. Gancyclovir is cidofovir.
E. Maravirox

192 What drug binds to gp41 subunit of viral envelope Enfuvirtide is a fusion inhibitor drug used in JEAN PAOLO M. FINAL EXAM
glycoprotein, preventing fusion of viral and cellular HIV. It binds to gp41 subunit of viral envelope DELFINO, MD (TOP - AUG 2015
membranes and is used in HIV infection? glycoprotein, preventing fusion of viral and 10 - FEB 2015 MED
A. Maravirox cellular membranes BOARDS; TOPNOTCH
B. Delavirdine MD FROM FATIMA)
C. Enfuvirtide
D. Docosanol
E. Fomivirsen

193 Bacillary angiomatosis is secondary to an infection from Bacillary angiomatosis (sometimes called JEAN PAOLO M. FINAL EXAM
what organism? bacillary epithelioid angiomatosis or DELFINO, MD (TOP - AUG 2015
A. Bartonela henselae epithelioid angiomatosis) is a disease of 10 - FEB 2015 MED
B. Brucella abortus severely immunocompromised patients, is BOARDS; TOPNOTCH
C. Klebsiella granulomatis caused by B. henselae or B. quintana, and is MD FROM FATIMA)
D. Pasteurella multocida characterized by neovascular proliferative
E. Shigella sonnei lesions involving the skin and other organs

194 All of the following double stranded viruses are naked All of those in the choices are naked viruses so JEAN PAOLO M. FINAL EXAM
except? what is truly asked in the question is which DELFINO, MD (TOP - AUG 2015
A. Parvovirus among them is not double stranded. 10 - FEB 2015 MED
B. Adenovirus Parvovirus is a naked virus but it is the only BOARDS; TOPNOTCH
C. Papilloma virus DNA virus that is single stranded. Reovirus, on MD FROM FATIMA)
D. Reovirus the other hand, is an RNA virus that is double
E. Polyoma virus stranded.

195 What is the infective stage of Plasmodium to its Definitive hosts of parasite is where the sexual JEAN PAOLO M. FINAL EXAM
definitive host? reproduction takes place. In malaria, the DELFINO, MD (TOP - AUG 2015
A. hypnozoite definitive host is the mosquito and the 10 - FEB 2015 MED
B. sporozoite infective stage is gametocyte. Man is the BOARDS; TOPNOTCH
C. schizont intermediate host and the infective stage is MD FROM FATIMA)
D. gametocyte sporozoite.
E. merozoite

196 Patient with AIDS developed cough (sometimes Aspergillus is an opportunistic fungal infection JEAN PAOLO M. FINAL EXAM
productive), hemoptysis, wheezing, and mild fatigue. so it can cause disease in DELFINO, MD (TOP - AUG 2015
Chest x-ray revealed crescent sign. What is the probable immunocompromised patient like in this case. 10 - FEB 2015 MED
etiology? CXR may reveal new focal infiltrates or BOARDS; TOPNOTCH
A. Mycobacterium tuberculosis nodules. Chest CT may reveal a characteristic MD FROM FATIMA)
B. Paragonimus westermani halo consisting of a mass-like infiltrate
C. Histoplasma capsulatum surrounded by an area of low attenuation. The
D. Aspergilus fumigatus presence of a "crescent sign" on CXR or CT, in
E. Pneumocystis jiroveci which the mass progresses to central
cavitation, is characteristic of invasive
Aspergillus infection but may develop as the
lesions are resolving.
197 On what phase of the bacterial growth curve does Beta- Log or exponential phase is where rapid cell JEAN PAOLO M. FINAL EXAM
Lactam antibiotics act on? division occurs. Beta lactam antibiotics act DELFINO, MD (TOP - AUG 2015
A. Lag phase during this phase 10 - FEB 2015 MED
B. Log phase BOARDS; TOPNOTCH
C. Maximum stationary phase MD FROM FATIMA)
D. Decline phase
E. Death phase

TOPNOTCH MEDICAL BOARD PREP MICROBIOLOGY SUPEREXAM Page 24 of 90


For inquiries visit www.topnotchboardprep.com.ph or email us at topnotchmedicalboardprep@gmail.com
TOPNOTCH MEDICAL BOARD PREP MICROBIOLOGY SUPEREXAM
For inquiries visit www.topnotchboardprep.com.ph or email us at topnotchmedicalboardprep@gmail.com
Item QUESTION EXPLANATION AUTHOR TOPNOTCH
# EXAM
198 70 year old male patient presented with pneumonia The diagnosis is atypical pneumonia JEAN PAOLO M. FINAL EXAM
accompanied by non-bloody diarrhea, hyponatremia and secondary to Legionella pneumophila DELFINO, MD (TOP - AUG 2015
hematuria. His condition is caused by a gram-negative infection. The culture medium used for 10 - FEB 2015 MED
bacterial agent transmitted via aerosol from an Legionella is Charcoal yeast extract. BOARDS; TOPNOTCH
environmental water source. What culture mediium will MD FROM FATIMA)
you use in order to confirm the diagnosis?
A. Bordet-Gengou
B. Barbour-Stoenner-Kelly
C. Charcoal yeast extract
D. Thiosulfate citrate bile salt
E. Xylose-Lysine-Deoxycholate
199 All of the following are oxidase positive except Neisseria, Campylobacter, Pseudomonas and JEAN PAOLO M. FINAL EXAM
A. Neisseria gonorrhea Vibrio cholera are all oxidase positive DELFINO, MD (TOP - AUG 2015
B. Vibrio cholera 10 - FEB 2015 MED
C. Pseudomonas BOARDS; TOPNOTCH
D. Campylobacter jejuni MD FROM FATIMA)
E. None of the above

200 Which of the following virulence factors of Protein A is an immunomodulator which JEAN PAOLO M. FINAL EXAM
Staphylococcus aureus prevents complement activation? prevents complement activation. C5a DELFINO, MD (TOP - AUG 2015
A. Leukocidin peptidase is seen in Strep pyogenes, not staph. 10 - FEB 2015 MED
B. C5a peptidase BOARDS; TOPNOTCH
C. Protein A MD FROM FATIMA)
D. Exotoxin B
E. Alpha toxin

201 A neonate is infected with a gram-positive bacteria tha Tumbling motility of Listeria is observed at GRACE ARVIOLA, MD DIAGNOSTIC
exhibits temperature-dependent motility. The etiologic 22-28 degree Celsius but not at 37 degree (TOP 3 - AUG 2014 EXAM - FEB
agent is: Celsius. MED BOARDS; 2015
A. Bacillus cereus TOPNOTCH MD)
B. Clostridium botulinum
C. E. coli
D. Streptococcus agalactiae
E. Listeria monocytogenes

202 This bacterium is the cause of acute jugular vein septic This is Lemierre's disease. The causative agent GRACE ARVIOLA, MD DIAGNOSTIC
thrombophlebitis associated with infectious is a gram-negative anaerobe. (TOP 3 - AUG 2014 EXAM - FEB
mononucleosis. MED BOARDS; 2015
A. Fusobacterium necrophorum TOPNOTCH MD)
B. Viridans streptococci
C. Staphylococcus aureus
D. Streptococcus mutans
E. Actinomyces spp.

203 Caseous necrosis found in mycobacterial infection is due Muramyl dipeptide complexed with mycolic GRACE ARVIOLA, MD DIAGNOSTIC
to this virulence factor. acids cause granuloma formation. Cord factor (TOP 3 - AUG 2014 EXAM - FEB
A. Cord factor (trehalose-6,6'-dimycolate) inhibits migration MED BOARDS; 2015
B. Mycolic acid of leukocytes, causes chronic granulomas, and TOPNOTCH MD)
C. Muramyl dipeptide serve as an immunologic adjuvant.
D. Phospholipids
E. Trehalose-6,6'-dimycolate

204 During rickettsial infection, this class of antibiotics are Tetracyclines and chloramphenicol inhibit the GRACE ARVIOLA, MD DIAGNOSTIC
avoided as this promotes growth of the bacterial species. growth of rickettsia. (TOP 3 - AUG 2014 EXAM - FEB
A. Macrolides MED BOARDS; 2015
B. Sulfonamides TOPNOTCH MD)
C. Penicillins
D. Chloramphenicol
E. Aminoglycosides

205 Which is a virulence factor of the bacteria that causes This factor is involved in the dissemination of GRACE ARVIOLA, MD DIAGNOSTIC
bubonic plague? the organism from the flea bite injection site. (TOP 3 - AUG 2014 EXAM - FEB
A. Exotoxin A MED BOARDS; 2015
B. Temperature-dependent coagulase TOPNOTCH MD)
C. Factors V and W
D. Type III secretion system
E. Urease

206 The adenovirus receptor shares its affinity with what The host cell receptor for some serotypes is GRACE ARVIOLA, MD DIAGNOSTIC
other virus? CAR (coxsackie-adenovirus receptor). (TOP 3 - AUG 2014 EXAM - FEB
A. measles MED BOARDS; 2015
B. mumps TOPNOTCH MD)
C. rubella
D. coxsackie
E. parvovirus

207 Which group of herpesviruses exhibit fast growth? Herpesviruses are divided into groups A, B, GRACE ARVIOLA, MD DIAGNOSTIC
A. EBV and CMV and C according to their growth rate patterns. (TOP 3 - AUG 2014 EXAM - FEB
B. CMV and HHV 6 Group A viruses have the fastest growth. This MED BOARDS; 2015
C. HSV and VZV includes HSV and VZV. TOPNOTCH MD)
D. HSV and HHV 7
E. HHV 6 and HHV 7

TOPNOTCH MEDICAL BOARD PREP MICROBIOLOGY SUPEREXAM Page 25 of 90


For inquiries visit www.topnotchboardprep.com.ph or email us at topnotchmedicalboardprep@gmail.com
TOPNOTCH MEDICAL BOARD PREP MICROBIOLOGY SUPEREXAM
For inquiries visit www.topnotchboardprep.com.ph or email us at topnotchmedicalboardprep@gmail.com
Item QUESTION EXPLANATION AUTHOR TOPNOTCH
# EXAM
208 The LDL receptor serves as a minor receptor for which ICAM-1 is the major receptor for rhinovirus. GRACE ARVIOLA, MD DIAGNOSTIC
virus? (TOP 3 - AUG 2014 EXAM - FEB
A. Rhinovirus MED BOARDS; 2015
B. Coronavirus TOPNOTCH MD)
C. Influenza virus
D. Rubella virus
E. Hepatitis B virus

209 Which of the following is true about respiratory syncitial Among the paramyxoviruses, only RSV GRACE ARVIOLA, MD DIAGNOSTIC
virus? contains no hemagglutinin nor neuraminidase. (TOP 3 - AUG 2014 EXAM - FEB
A. It belongs in the family Orthomyxoviridae. It commonly affects children less than 5 years MED BOARDS; 2015
B. It has no hemagglutinin nor neuraminidase. old. RSV is a single-stranded RNA virus. TOPNOTCH MD)
C. It is the most common cause of viral pneumonia in
children above 5 years old.
D. It is a double-stranded RNA virus.
E. It is a double-stranded DNA virus.
210 Which is the receptor of lymphocyte-trophic HIV CCR5 is the receptor for macrophage-trophic GRACE ARVIOLA, MD DIAGNOSTIC
viruses? HIV viruses. (TOP 3 - AUG 2014 EXAM - FEB
A. CCR5 MED BOARDS; 2015
B. CXCR4 TOPNOTCH MD)
C. CCR4
D. CXCR5
E. None of the above

211 In patients with histoplasmosis, the presence of Antibodies to m antigen indicate past GRACE ARVIOLA, MD DIAGNOSTIC
antibodies to h antigen indicate: exposure or repeated skin testing. (TOP 3 - AUG 2014 EXAM - FEB
A. Favorable course MED BOARDS; 2015
B. Poor prognosis TOPNOTCH MD)
C. Past exposure
D. Active disease
E. Low infectivity

212 Which protozoal infection is NOT transmitted by a bug Chagas disease is transmitted when an GRACE ARVIOLA, MD DIAGNOSTIC
bite? infected Reduviid bug defecates on a person's (TOP 3 - AUG 2014 EXAM - FEB
A. Malaria skin, not through a bug bite. MED BOARDS; 2015
B. African Sleeping Sickness TOPNOTCH MD)
C. Chagas Disease
D. Babesiosis
E. Kala-azar

213 In the laboratory, a schistosoma with a terminally Laterally located spine: S. mansoni. Barely GRACE ARVIOLA, MD DIAGNOSTIC
located spine was discovered. The patient most likely visible spine: S. japonicum (TOP 3 - AUG 2014 EXAM - FEB
has infection of the ________. MED BOARDS; 2015
A. Urinary bladder TOPNOTCH MD)
B. Liver
C. Intestines
D. Lungs
E. Brain

214 This protozoa can only exist as a trophozoite. Trichomonas has no cyst stage. GRACE ARVIOLA, MD DIAGNOSTIC
A. Naeglaria (TOP 3 - AUG 2014 EXAM - FEB
B. Acanthamoeba MED BOARDS; 2015
C. Trichomonas TOPNOTCH MD)
D. Toxoplasma
E. Giardia

215 A patient who recently traveled to California was The spherule is diagnostic of Coccidioides GRACE ARVIOLA, MD DIAGNOSTIC
admitted due to suspected mycotic encephalitis. The immitis. (TOP 3 - AUG 2014 EXAM - FEB
laboratory identified spherules with a doubly refractile MED BOARDS; 2015
wall. The encephalitis is most likely due to: TOPNOTCH MD)
A. Histoplasmosis
B. Blastomycosis
C. Coccidioidomycosis
D. Paracoccidioidomycosis
E. Aspergillosis

216 Neurocysticercosis is possible for Taenia solium because Intermediate hosts harbor the asexual stage of GRACE ARVIOLA, MD DIAGNOSTIC
___________. the parasite. If humans ingest T. solium eggs, (TOP 3 - AUG 2014 EXAM - FEB
A. Humans can serve as definitive hosts. the cysticerci (larva) encyst in various human MED BOARDS; 2015
B. Humans can serve as intermediate hosts. tissues, including the brain. This condition is TOPNOTCH MD)
C. Humans can serve as paratenic hosts. called cysticercosis.
D. Humans can serve as accidental hosts.
E. Humans can serve as dead-end hosts.

217 Cochin-China diarrhea is caused by: Heavy infection produces intractable, painless, GRACE ARVIOLA, MD DIAGNOSTIC
A. Ascaris intermittent diarrhea characterized by (TOP 3 - AUG 2014 EXAM - FEB
B. Trichuris numeroud episodes of watery and bloody MED BOARDS; 2015
C. Strongyloides stools. TOPNOTCH MD)
D. Capillaria
E. Trichinella

TOPNOTCH MEDICAL BOARD PREP MICROBIOLOGY SUPEREXAM Page 26 of 90


For inquiries visit www.topnotchboardprep.com.ph or email us at topnotchmedicalboardprep@gmail.com
TOPNOTCH MEDICAL BOARD PREP MICROBIOLOGY SUPEREXAM
For inquiries visit www.topnotchboardprep.com.ph or email us at topnotchmedicalboardprep@gmail.com
Item QUESTION EXPLANATION AUTHOR TOPNOTCH
# EXAM
218 Which statement correctly distinguishes T. solium from T. solium: 7-15 lateral uterine branches, (+) GRACE ARVIOLA, MD DIAGNOSTIC
T. saginata? rostellum, (+) accessory ovarian lobe, (-) (TOP 3 - AUG 2014 EXAM - FEB
A. It has no rostellum. vaginal sphincter. It is a cyclophyllidean MED BOARDS; 2015
B. It has fewer uterine branches. cestode. T. saginata: 15-20 uterine branches. TOPNOTCH MD)
C. It is a pseudophyllidean cestode.
D. It has no accessory ovarian lobe.
E. Its vagina has a sphincter.

219 Which is true about cyclophyllidean cestodes? Cyclophyllidean cestodes do not have a GRACE ARVIOLA, MD DIAGNOSTIC
A. D. latum is an example. uterine pore. Apolysis occurs leading to the (TOP 3 - AUG 2014 EXAM - FEB
B. Both eggs and segments may be seen in the feces of discovery of both eggs and segments in the MED BOARDS; 2015
infected patients. feces.The scolex is globular. Only one TOPNOTCH MD)
C. A uterine pore is present; no apolysis occurs. intermediate host is required. D. latum is
D. They require two intermediate hosts. pseudophyllidean.
E. The scolex is spatulate with sucking grooves.

220 Which of the following is INCORRECTLY paired? Barber pole's pattern is for Angiostrongylus GRACE ARVIOLA, MD DIAGNOSTIC
A. Necator : Cutting plate cantonensis. The female worm has uterine (TOP 3 - AUG 2014 EXAM - FEB
B. Enterobius : Cephalic ala tubules which are wound spirally around the MED BOARDS; 2015
C. Capillaria : Peanut-shaped eggs with flat bipolar intestine. TOPNOTCH MD)
plugs
D. Strongyloides : Barber pole's pattern
E. Trichuris : Lemon-shaped eggs with bipolar plugs

221 Which of the following is true about Bacillus anthracis the most common form of infection is LEAN ANGELO MIDTERM
infection? cutaneous type of anthrax. Spores are SILVERIO, MD (TOP 4 EXAM 1 -
A. The most common form of infection is transmitted transmitted via a breakdown in the cutaneous - AUG 2014 MED FEB 2015
via mucous membranes through inhalation. barrier. Individual toxins ( PA, LF, EF) BOARDS; TOPNOTCH
B. the three toxins produced by the bacteria does not individually does not exert virulence. MD), MD
require synergistic cooperation to exert lethal effect. However, the combination of PA and LF
C. the formation of poly D glutamic acid is collectively known as lethal toxin is the major
chromosomally mediated virulence factor and it is the most common
D. The recommended treatment for all forms of anthrax cause of mortality. polyglutamate capsule is
is fluoroquinolone plasmid mediated. the recommended
E. all of the above treatment is ciprofloxacin. Jawets chapter 11.
222 A 34 y/o patient went for consult secondary to a the most common agent responsible for LEAN ANGELO MIDTERM
localized abscess on the left lateral nape area. which of abscesses or other localized suppuration is SILVERIO, MD (TOP 4 EXAM 1 -
the following is the characteristic of the the causative Staph aureus.it is differentiated from - AUG 2014 MED FEB 2015
agent of this infection? streptococci by the presence of a catalase. B- BOARDS; TOPNOTCH
A. It can be differentiated from streptococci using a staph aureus produce 3 major types of MD), MD
catalase test. exotoxin (a,b, gamma). On of its superantigen
B. it does not produce exotoxin. is TSST1 which causes toxic shock syndrome
C. it can cause toxic shock syndrome via direct via direct slimulation of the toxin to MHC2
stimulation of the bacterial cell wall to MHC type 1 molecule leading massive T cell activation.
molecule. staph aureus are only small in numbers in
D. it is the most abundant bacterial component of cutaneous flora. it is staph epidermidis that is
cutaneous flora. the most abundant.
E. all of the above.
223 A 68 y/o male presents with hematochezia, weight loss, aside from endocarditis, streptococcus bovis is LEAN ANGELO MIDTERM
change in stool caliber, and a supraclavicular lymph a common blood isolate in colon cancer SILVERIO, MD (TOP 4 EXAM 1 -
node on the left posterior cervical triangle.During 1st patients. SIMILAR TO PREVIOUS BOARD - AUG 2014 MED FEB 2015
hospital stay, patient went into hypotension, respiratory EXAM CONCEPT/PRINCIPLE BOARDS; TOPNOTCH
distress, and coagulation abnormalities. Which of the MD), MD
following bacteria is commonly associated with this
condition?
A. strep pyogenes
B. Bacteroides fragilis
C. enterococcus faecalis
D. Streptococcus bovis
E. all of the above

224 An 7 y/o male presented to the ER hypotensive and Aschoff bodies and anitsckow cells are LEAN ANGELO MIDTERM
suddenly went to cardiac arrest. history revealed that he pathognomonic myocardial findings found in SILVERIO, MD (TOP 4 EXAM 1 -
had multiple migrating sites of arthritis, serpiginious patients suffering from rheumatic fever. - AUG 2014 MED FEB 2015
rashes on the trunk and fever prior to onset of SIMILAR TO PREVIOUS BOARD EXAM BOARDS; TOPNOTCH
hypotension. Pathology report showed presence os CONCEPT/PRINCIPLE MD), MD
aschoff bodies in the myocardium. Which is the most
likely causative agent for the patient's condition?
A. Strep pneumoniae
B. Staph aureus
C. Strep pyogenes
D. Strep bovis
E. Bacteroides fragilis

225 what is the drug of choice for patients with syphilis the DOC for syphillis is still Penicillin G even if LEAN ANGELO MIDTERM
having type 1 hypersensitivity to penicillin based drugs? the patient has hypersensitivty reaction with SILVERIO, MD (TOP 4 EXAM 1 -
A. Fluoroquinolone that drug. For such cases, the drug must be - AUG 2014 MED FEB 2015
B. Penicillin G given in gradual increments until the patien BOARDS; TOPNOTCH
C. Carbapenem become desensitized with the drug. MD), MD
D. Spectinomycin
E. Azithromycin

TOPNOTCH MEDICAL BOARD PREP MICROBIOLOGY SUPEREXAM Page 27 of 90


For inquiries visit www.topnotchboardprep.com.ph or email us at topnotchmedicalboardprep@gmail.com
TOPNOTCH MEDICAL BOARD PREP MICROBIOLOGY SUPEREXAM
For inquiries visit www.topnotchboardprep.com.ph or email us at topnotchmedicalboardprep@gmail.com
Item QUESTION EXPLANATION AUTHOR TOPNOTCH
# EXAM
226 A 24 y/o male is suffering from multiple bouts of loose the most important treatment for patient with LEAN ANGELO MIDTERM
watery stools. Fecal examination showed curved shaped cholera is fluid replacement and electrolyte SILVERIO, MD (TOP 4 EXAM 1 -
rod bacteria with polar flagellum that produces orange correction through intravenous hydration. - AUG 2014 MED FEB 2015
colonies on TCBS agar. In this case what is the most Administration of tetracycline will only BOARDS; TOPNOTCH
important management? decrease the stool output and the duration of MD), MD
A. IV hydration using D5NSS the disease.
B. prompt treatment with IV tetracycline
C. correction of metabolic acidosis with Bicarbonate
therapy
D. potassium replacement via kalium durule
E. all of the above.
227 a 9 y/o female presented to your clinic with refractory Treatment with oral penicillin does not LEAN ANGELO MIDTERM
fever of 38.9C, headache, and incessant vomitting. You eradicate the carrier state. Rifampin, 600 mg SILVERIO, MD (TOP 4 EXAM 1 -
opted admission however due to lack of funds dediced to orally twice daily for 2 days (or ciprofloxacin - AUG 2014 MED FEB 2015
go home. After a period of 24 hours, She was brought to in adults, 500 mg as a single dose), can often BOARDS; TOPNOTCH
the ER hypotensive with prominent purpura and eradicate the carrier state and serve as MD), MD
petechiae along the lower extremities. which of the chemoprophylaxis for household and other
following is a false statement about her condition? close contacts. Since the appearance of many
A. treatment with oral penicillin can eradicate the sulfonamide-resistant meningococci,
carrier state chemoprophylaxis with sulfonamides is no
B. humans are the only natural host. longer reliable.
C. can be cultured using modified thayer martin medium
D. circulatory collapse is most likely due to adrenal
insuffiency
E. none of the above
228 A 24 y/o male presented to your clinic with history of Neisseria Gonorrhea can only oxidize glucose. LEAN ANGELO MIDTERM
dysuria. Upon physical examination, there was In contrast with other gram negative bacteria. SILVERIO, MD (TOP 4 EXAM 1 -
expression of pus on the penile meatus. Microscopic Its Lipopolysaccharide does not have the - AUG 2014 MED FEB 2015
finding shows multiple diplococci found inside typical O antigen side chain. Thus it is called BOARDS; TOPNOTCH
macrophages. Which of the following is true about his LOS. MD), MD
condition?
A. the bacteria can oxidized by glucose and lactose but
not mannose
B. its surface pili exhibit rapid switching mechanism to
elude antibody detection
C. its lipopolysaccharide is the same with that of the
enteric gram negative bacteria
D. none of the above
E. all of the above
229 which of the following is true about M tuberculosis coinfection of TB patients with measles leads LEAN ANGELO MIDTERM
infection? to suppression of T cell activity leading state SILVERIO, MD (TOP 4 EXAM 1 -
A. Boosted reaction can occur in PPD testing if patient of anergy. IGRA can be use if PPD is equivocal - AUG 2014 MED FEB 2015
is also infected with measles virus particularly if the patient is leaving in BOARDS; TOPNOTCH
B. In cases of equivocal PPD result. Gamma interferon nontuberculous mycobacterium endemic MD), MD
release assay does not increase diagnostic accuracy areas.These assays are based on the host's
C. apical lobe necrosis with multiple hilar immune responses to specific M tuberculosis
lymphadenopathy is a pathologic hallmark of TB antigens ESAT-6 and CFP-10, which are absent
infection even without AFB smear from most nontuberculous mycobacteria and
D. the cord factor produced by TB bacilli inhibits BCG. there are no radiologic hallmarks for TB
migration of leukocytes pulmonary infection. TB disease can be
E. all of the above mimicked by histoplasma or paragonimus
infection.
230 A 7y/o female presents with vomitting, anorexia and an patient is most likely infected with Bordetella LEAN ANGELO MIDTERM
explosive cough accompanied by characteristic whoop pertusis. Clinical presentation exhibit three SILVERIO, MD (TOP 4 EXAM 1 -
upon inhalation. Which of the following is true about the stages of the disease. After an incubation - AUG 2014 MED FEB 2015
disease? period of about 2 weeks, the "catarrhal stage" BOARDS; TOPNOTCH
A. The patient is highly infectious at this point. develops, with mild coughing and sneezing. MD), MD
B. treatment of erythromycin during her stage of the During this stage, large numbers of organisms
disease will significantly alter the clinical presentation are sprayed in droplets, and the patient is
C. absolute lymphocytosis is a characteristic blood highly infectious but not very ill. During the
picture of the disease "paroxysmal" stage, the cough develops its
D. all of the above explosive character and the characteristic
E. none of the above "whoop" upon inhalation. This leads to rapid
exhaustion and may be associated with
vomiting, cyanosis, and convulsions.
Convalescent stage is a slow process
presenting with less cough however it is the
stage were secondary complications occur (
encephalitis, pneumonitis) The white blood
count is high (16,000–30,000/L), with an
absolute lymphocytosis.
231 which of the following structures is unique to gram both cell membrane and peptidoglycan layer LEAN ANGELO MIDTERM
negative bacteria? is present in both gram positive and negative SILVERIO, MD (TOP 4 EXAM 1 -
A. peptidoglycan bacteria. Teichoic acid is unique to gram - AUG 2014 MED FEB 2015
B. Techoic acid positive bacteria. Spores can be seen in certain BOARDS; TOPNOTCH
C. Cell membrane organism such as bacillus and clostridium MD), MD
D. Outer membrane species.
E. spores

TOPNOTCH MEDICAL BOARD PREP MICROBIOLOGY SUPEREXAM Page 28 of 90


For inquiries visit www.topnotchboardprep.com.ph or email us at topnotchmedicalboardprep@gmail.com
TOPNOTCH MEDICAL BOARD PREP MICROBIOLOGY SUPEREXAM
For inquiries visit www.topnotchboardprep.com.ph or email us at topnotchmedicalboardprep@gmail.com
Item QUESTION EXPLANATION AUTHOR TOPNOTCH
# EXAM
232 What structural HIV gene is responsible for the Gag - capsid, nucleocapsid, and matrix LEAN ANGELO MIDTERM
production of nucleocapsid and matrix proteins? proteins. Pol - reverse transcriptase, protease, SILVERIO, MD (TOP 4 EXAM 1 -
A. Gag integrase.Env- Gp120, Gp41 ( surface - AUG 2014 MED FEB 2015
B. Pol glycoproteins) tat -transactivator protein BOARDS; TOPNOTCH
C. Env responsible for upregulation of viral gene MD), MD
D. tat transcription. Nef - negative factor causing
E. nef decreae in MHC 1 molecule expression.

233 In rabies infection, which of the following organ will The organ with the highest titers of virus is LEAN ANGELO MIDTERM
have the highest titer of viral load? the submaxillary salivary gland. Other organs SILVERIO, MD (TOP 4 EXAM 1 -
A. Parotid gland where rabies virus has been found include - AUG 2014 MED FEB 2015
B. Submaxillary gland pancreas, kidney, heart, retina, and cornea. BOARDS; TOPNOTCH
C. Sublingual gland Rabies virus has not been isolated from the MD), MD
D. Minor salivary gland blood of infected persons.
E. Cornea

234 which of the following is true characteristic of a pox LEAN ANGELO MIDTERM
virus? SILVERIO, MD (TOP 4 EXAM 1 -
A. Virions contain more than 100 polypeptides; many - AUG 2014 MED FEB 2015
enzymes are present in core, including transcriptional BOARDS; TOPNOTCH
system MD), MD
B. Cytoplasm is a site of viral replication
C. Vaccinia is the agent use for smallpo vaccination
D. it is the largest and most complex virus and can be
seen in light microscopy
E. all of the above
235 What is the most common complication of measles? the most common life treathening LEAN ANGELO MIDTERM
A. Giant cell pneumonia complication of measles is secondary SILVERIO, MD (TOP 4 EXAM 1 -
B. Aseptic meningitis pneumonia due to bacterial infection. The - AUG 2014 MED FEB 2015
C. Otitis media most common complication is otitis media. BOARDS; TOPNOTCH
D. Acute tonsillopharyngitis MD), MD
E. Encephalitis

236 what is the most common presentation of poliovirus the most common form of infection is LEAN ANGELO MIDTERM
infection? subclinical which comprises about 99% of all SILVERIO, MD (TOP 4 EXAM 1 -
A. subclinical population. Only 1% will present with - AUG 2014 MED FEB 2015
B. Mild disease manifestation. The most common of which are BOARDS; TOPNOTCH
C. Nonparalytic poliomyelitis nonspecific viral symptoms (Mild disease) MD), MD
D. Paralytic poliomyelitis
E. Aseptic meningitis

237 what is the primary mechanism on how human HPV codes for 2 early proteins E6 and E7 LEAN ANGELO MIDTERM
papilloma virus exerts its oncogenetic effect? which inactivates tumor suppressor gene p53 SILVERIO, MD (TOP 4 EXAM 1 -
A. promotes protooncogene activation and Rb leading to malignant transformation. - AUG 2014 MED FEB 2015
B. disruption of ubiquitin proteosome complex BOARDS; TOPNOTCH
C. inactivates tumor suppressor gene p53 and Rb MD), MD
D. incorporates viral DNA adjacent to an exon site
E. all of the above

238 a 26 y/o male presented to the ER with a history of the most common cause of osteomyelitis in LEAN ANGELO MIDTERM
multiple bouts of fever, recent onset of ® knee pain not Sickle cell anemia patient is salmonella SILVERIO, MD (TOP 4 EXAM 1 -
relieved by any position. Past medical history showed enteritidis. SIMILAR TO PREVIOUS BOARD - AUG 2014 MED FEB 2015
that patient was a diagnosed case of sickle cell anemia. EXAM CONCEPT/PRINCIPLE BOARDS; TOPNOTCH
which of the following is the most causative agent for his MD), MD
bone pain ?
A. staph aureus
B. salmonella entertidiis
C. Pseudomonas auriginesa
D. strep pyogenes
E. hemophilus influenza type B
239 A 48 year old male diagnosed with AIDs 2 years ago Aspergilloma occurs when inhaled conidia LEAN ANGELO MIDTERM
presents with productive cough of 2 weeks duration. enter an existing cavity, germinate, and SILVERIO, MD (TOP 4 EXAM 1 -
This was accompanied by night sweats and intermittent produce abundant hyphae in the abnormal - AUG 2014 MED FEB 2015
low grade fever. CXR revealed granuloma formation at pulmonary space. Patients with previous BOARDS; TOPNOTCH
bilateral apical lobes. Three AFB smears happens to be cavitary disease (eg, tuberculosis, sarcoidosis, MD), MD
negative. CT guided biopsy was done revealing fungus emphysema) are at risk. Some patients are
with dimorphism in different temperatures. what is the asymptomatic; others develop cough, dyspnea,
most likely causative agent? weight loss, fatigue, and hemoptysis
A. Histoplasmosis
B. Aspergillus fumigatus
C. paracoccidiodes braziliensis
D. blastomycosis dermatidis
E. paragonimiasis

TOPNOTCH MEDICAL BOARD PREP MICROBIOLOGY SUPEREXAM Page 29 of 90


For inquiries visit www.topnotchboardprep.com.ph or email us at topnotchmedicalboardprep@gmail.com
TOPNOTCH MEDICAL BOARD PREP MICROBIOLOGY SUPEREXAM
For inquiries visit www.topnotchboardprep.com.ph or email us at topnotchmedicalboardprep@gmail.com
Item QUESTION EXPLANATION AUTHOR TOPNOTCH
# EXAM
240 A 56 y/o female went to your clinic secondary to rectal the DOC for trichiuriasis is albendazole LEAN ANGELO MIDTERM
prolapse. HPI revealed that she has chronic bouts on SILVERIO, MD (TOP 4 EXAM 1 -
nonspecific abdominal pain, tenesmus and weight loss. - AUG 2014 MED FEB 2015
Fecalysis revealed presence of barrel shaped egg with BOARDS; TOPNOTCH
polar plugs at the polar ends. what is the drug of choice MD), MD
for this condition ?
A. Mebendazole
B. Praziquantrel
C. Albendazole
D. ivermectin
E. Metronidazole
241 Which of the following fungal species infecting the hair SIMILAR TO PREVIOUS BOARD EXAM KEVIN BRYAN LO, MIDTERM 2
produces a characteristic greenish fluorescence when CONCEPT/PRINCIPLE, microsporum species MD (TOP 7 - AUG EXAM - FEB
hairs are examined under wood's light? do not penetrate the hair hence remain 2014 MED BOARDS; 2015
A. Microsporum canis outside and fluoresces under wood's light TOPNOTCH MD)
B. Trichophyton rubrum
C. Trichophyton mentagrophytes
D. Trichophyton tonsurans
E. Epidermophyton floccosum

242 The most common implicated agent causing secondary SIMILAR TO PREVIOUS BOARD EXAM KEVIN BRYAN LO, MIDTERM 2
bacterial pneumonia following a bout of influenza virus CONCEPT/PRINCIPLE, staph aureus is MD (TOP 7 - AUG EXAM - FEB
infection is? commonly implicated in secondary bacterial 2014 MED BOARDS; 2015
A. Streptococcus pneumonia pneumonia after a bout of influenza TOPNOTCH MD)
B. Staphylococcus aureus
C. Pseudomonas aeruginosa
D. Mycoplasma pneumoniae
E. Haemophilus influenzae

243 Which of the following serves as the snail intermediate SIMILAR TO PREVIOUS BOARD EXAM KEVIN BRYAN LO, MIDTERM 2
host for Paragonimus westermani? CONCEPT/PRINCIPLE, the snail host is MD (TOP 7 - AUG EXAM - FEB
A. Oncomelania quadrasi antemelania asperata, the 2nd intermediate 2014 MED BOARDS; 2015
B. Pila Luzonica host is sundathelphusa or the crustacean TOPNOTCH MD)
C. Antemelania asperata
D. Sundathelphusa philippina
E. none of the above

244 What differentiates the stools from bacillary dysentery SIMILAR TO PREVIOUS BOARD EXAM KEVIN BRYAN LO, MIDTERM 2
from amoebic dysentery? CONCEPT/PRINCIPLE, I researched the MD (TOP 7 - AUG EXAM - FEB
A. the amount of blood is greater in bacillary dysentery answer and usually more PMNs are associated 2014 MED BOARDS; 2015
B. the stool is mucoid in character in amoebic dysentery with bacillary dysentery TOPNOTCH MD)
C. the stools in amoebic dysentery are more foul smelling
D. the amount of neutrophils is greater in bacillary
dysentery
E. no given characteristic can differentiate between the
two
245 Which of the following organisms almost do not invade SIMILAR TO PREVIOUS BOARD EXAM KEVIN BRYAN LO, MIDTERM 2
the bloodstream? CONCEPT/PRINCIPLE, shigella is invasive to MD (TOP 7 - AUG EXAM - FEB
A. shigella the intestinal mucosa but rarely disseminates 2014 MED BOARDS; 2015
B. salmonella to the bloodstream TOPNOTCH MD)
C. aspergillus
D. pseudomonas
E. coagulase negative staphylococci

246 Which of the following drugs can be used in the SIMILAR TO PREVIOUS BOARD EXAM KEVIN BRYAN LO, MIDTERM 2
treatment of Pneumocystis jiroveci pneumonia CONCEPT/PRINCIPLE, pentamidine and MD (TOP 7 - AUG EXAM - FEB
A. ceftriaxone cotrimoxazole can both be used in the 2014 MED BOARDS; 2015
B. pentamidine treatment of pneumocystis carinii pneumonia TOPNOTCH MD)
C. high dose ampicillin sulbactam
D. vancomycin
E. none of the above can be used

247 Which of the following organisms is both gram negative SIMILAR TO PREVIOUS BOARD EXAM KEVIN BRYAN LO, MIDTERM 2
and exotoxin producing? CONCEPT/PRINCIPLE, Bordetella pertussis is MD (TOP 7 - AUG EXAM - FEB
A. Neisseria gonorrhea gram negative exotoxin producing (tracheal 2014 MED BOARDS; 2015
B. Chlamydia trachomatis cytotoxin and others). Diptheria is gram TOPNOTCH MD)
C. Corynebacterium diphtheriae positive, HIB and neisseria and chlamydia are
D. Bordetella pertussis not exotoxin producing
E. Haemophilus influenzae

248 This substance secreted by group A beta hemolytic SIMILAR TO PREVIOUS BOARD EXAM KEVIN BRYAN LO, MIDTERM 2
streptococci is a super antigen associated with toxic CONCEPT/PRINCIPLE asking about MD (TOP 7 - AUG EXAM - FEB
shock syndrome streptococcal exotoxins, pyrogenic exotoxin A 2014 MED BOARDS; 2015
A. Toxic shock syndrome toxin 1 is a superantigen TOPNOTCH MD)
B. Exfoliative toxin
C. M protein
D. pyrogenic exotoxin A
E. alpha toxin

TOPNOTCH MEDICAL BOARD PREP MICROBIOLOGY SUPEREXAM Page 30 of 90


For inquiries visit www.topnotchboardprep.com.ph or email us at topnotchmedicalboardprep@gmail.com
TOPNOTCH MEDICAL BOARD PREP MICROBIOLOGY SUPEREXAM
For inquiries visit www.topnotchboardprep.com.ph or email us at topnotchmedicalboardprep@gmail.com
Item QUESTION EXPLANATION AUTHOR TOPNOTCH
# EXAM
249 20 year old female student presented at the ER with SIMILAR TO PREVIOUS BOARD EXAM KEVIN BRYAN LO, MIDTERM 2
fever, nuchal rigidity, no other focal deficits seen, over CONCEPT/PRINCIPLE, there were a lot of MD (TOP 7 - AUG EXAM - FEB
the course of 24 to 48 hours, there was presence of questions on both neisseria meningitidis and 2014 MED BOARDS; 2015
altered sensorium, appearance of multiple purpuric gonorrhea, rapidly progressive meningitis TOPNOTCH MD)
lesions progressively over the lower extremities and with purpura is characteristic of neisseria
trunk, which is the most probable etiologic agent? meningitidis, usually found on dormitory
A. Streptococcus pneumonia residents and military barracks
B. Neiserria meningitidis
C. Haemophilus influezae
D. Cryptococcal meningitis
E. Herpes simplex meningoencephalitis
250 A 20 year old female patient was treated at the out SIMILAR TO PREVIOUS BOARD EXAM KEVIN BRYAN LO, MIDTERM 2
patient department for gonorrhea, she was given both CONCEPT/PRINCIPLE, there were a lot of MD (TOP 7 - AUG EXAM - FEB
ceftriaxone and azithromycin, azithromycin is given in questions on sexually transmitted infections 2014 MED BOARDS; 2015
these patients for gonorrhea and chlamydia TOPNOTCH MD)
A. to cover for Neisseria gonorrhea
B. to act synergistically with ceftriaxone against
Neisseria gonorrhea
C. to cover for concomitant chlamydia
D. to cover for emerging resistant strains of neisseria
E. to hasten recovery in patients treated for sexually
transmitted infections
251 Which of the following concepts about hepatitis A is SIMILAR TO PREVIOUS BOARD EXAM KEVIN BRYAN LO, MIDTERM 2
false? CONCEPT/PRINCIPLE, hepatitis A virus has no MD (TOP 7 - AUG EXAM - FEB
A. Hepatitis A usually causes asymptomatic disease in chronic form of disease. Also, hepatitis a 2014 MED BOARDS; 2015
the younger age groups compared to older vaccine (available since 1996) has 2 forms - TOPNOTCH MD)
B. hepatitis A is transmitted via fecal oral route inactiavated, and live,attenuated.
C. hepatitis A has an inactivated virus available for
vaccination
D. hepatitis A can progress to a more chronic form of a
disease
E. hepatitis A virus belongs to the family picornaviridae
which is a negative sense RNA virus
252 Which of the following organisms are implicated when SIMILAR TO PREVIOUS BOARD EXAM KEVIN BRYAN LO, MIDTERM 2
stool samples reveal eggs with bipolar plugs CONCEPT/PRINCIPLE, bipolar plugs MD (TOP 7 - AUG EXAM - FEB
A. Enterobius vermicularis trichuriasis 2014 MED BOARDS; 2015
B. Ascaris lumbricoides TOPNOTCH MD)
C. Paragonimus westermani
D. Ancylostoma duodenale
E. Trichuris trichiuria

253 The predominant mechanism or pathophysiology SIMILAR TO PREVIOUS BOARD EXAM KEVIN BRYAN LO, MIDTERM 2
involved in the damage induced by hepatitis B virus on CONCEPT/PRINCIPLE, majority of hepatic MD (TOP 7 - AUG EXAM - FEB
the liver cells: ____________. damage is immune mediated response by the 2014 MED BOARDS; 2015
A. hepatitis B virus cytopathic effect on the liver cells body against the Hep B virus TOPNOTCH MD)
B. immune induced hepatic damage
C. apoptosis induced by the virus on the liver cells
D. hepatitis B incorporates into cell DNA causing damage
E. none of the above, hepatitis B virus remains latent

254 Donor DNA that does not carry information necessary SIMILAR TO PREVIOUS BOARD EXAM KEVIN BRYAN LO, MIDTERM 2
for its own replication must be incorporated with CONCEPT/PRINCIPLE, researched the answer, MD (TOP 7 - AUG EXAM - FEB
recipient DNA in order to be replicated or established in must know other terms, transformation direct 2014 MED BOARDS; 2015
a recipient strain, this process is known as uptake of donor dna, transduction dna is TOPNOTCH MD)
A. recombination carried by phage virus, conjugation transfer of
B. transduction genetic material through sex pilus
C. transformation
D. conjugation
E. gene rearrangement

255 Which is a good alternative antibiotic for pneumococcal SIMILAR TO PREVIOUS BOARD EXAM KEVIN BRYAN LO, MIDTERM 2
pneumonia among those allergic to penicillin which also CONCEPT/PRINCIPLE, macrolides are a good MD (TOP 7 - AUG EXAM - FEB
covers for atypical organisms alternative to cover for penicillin allergic 2014 MED BOARDS; 2015
A. cefuroxime patients with coverage for atypicals as well TOPNOTCH MD)
B. erythromycin
C. clindamycin
D. chloramphenicol
E. ciprofloxacin

256 Which of the following species of schistosoma can cause SIMILAR TO PREVIOUS BOARD EXAM KEVIN BRYAN LO, MIDTERM 2
hepatic fibrosis and portal hypertension CONCEPT/PRINCIPLE, schistosoma mansoni MD (TOP 7 - AUG EXAM - FEB
A. schistosoma mansoni and japonicum both reside in the portal 2014 MED BOARDS; 2015
B. schistosoma japonicum circulation, haematobium causes more TOPNOTCH MD)
C. schistosoma haematobium pathology in the bladder
D. all of the above
E. A and B

TOPNOTCH MEDICAL BOARD PREP MICROBIOLOGY SUPEREXAM Page 31 of 90


For inquiries visit www.topnotchboardprep.com.ph or email us at topnotchmedicalboardprep@gmail.com
TOPNOTCH MEDICAL BOARD PREP MICROBIOLOGY SUPEREXAM
For inquiries visit www.topnotchboardprep.com.ph or email us at topnotchmedicalboardprep@gmail.com
Item QUESTION EXPLANATION AUTHOR TOPNOTCH
# EXAM
257 What is the drug of choice for malarial prophylaxis in SIMILAR TO PREVIOUS BOARD EXAM KEVIN BRYAN LO, MIDTERM 2
pregnant women in chloroquine resistant areas CONCEPT/PRINCIPLE, mefloquine is the drug MD (TOP 7 - AUG EXAM - FEB
A. chloroquine high dose of choice in cholorquine resistant areas for 2014 MED BOARDS; 2015
B. artemether lumefantrine pregnant women TOPNOTCH MD)
C. doxycycline
D. mefloquine
E. primaquine

258 What is the most common etiologic agent causing acute SIMILAR TO PREVIOUS BOARD EXAM KEVIN BRYAN LO, MIDTERM 2
infective endocarditis? CONCEPT/PRINCIPLE, staphylococcus is most MD (TOP 7 - AUG EXAM - FEB
A. Staphylococcus aureus commonly implicated in acute infective 2014 MED BOARDS; 2015
B. Pseudomonas aeruginosa endocarditis especially in IV drug abusers TOPNOTCH MD)
C. Staphylococcus epidermidis
D. group D streptococcus
E. HACEK organisms

259 A 50 year old woman from mindoro with history of long SIMILAR TO PREVIOUS BOARD EXAM KEVIN BRYAN LO, MIDTERM 2
standing swelling of the legs genital and breasts, which CONCEPT/PRINCIPLE, DEC is the drug of MD (TOP 7 - AUG EXAM - FEB
of the following drugs is considered the drug of choice choice for filariasis, a lot of treatment 2014 MED BOARDS; 2015
for her condition? questions in our microbiology exam TOPNOTCH MD)
A. praziquantel
B. ivermectin
C. diethylcarbamazine
D. nifurtimox
E. metronidazole

260 A newborn was delivered uneventful via normal SIMILAR TO PREVIOUS BOARD EXAM KEVIN BRYAN LO, MIDTERM 2
spontaneous delivery, less than 24 hours, he developed CONCEPT/PRINCIPLE, group B strep is MD (TOP 7 - AUG EXAM - FEB
jaundice, unable to breastfeed, irritability, tachypnea, he implicated in neonatal meningitis and sepsis 2014 MED BOARDS; 2015
was admitted and given antibiotics, with assessment of TOPNOTCH MD)
neonatal sepsis which of the following organisms is
implicated?
A. Staphylococcus aureus
B. Streptococcus pyogenes
C. Streptococcus pneumoniae
D. Streptococcus bovis
E. Streptococcus agalactiae
261 A sexually active 26 year old female patient presents Pili/Fimbriae - UTI and gonorrhea RAYMUND MARTIN MIDTERM 3
with recurrent urinary tract infections. Which of the LI, MD (TOP 1 - AUG EXAM - FEB
following virulence factors most likely promotes the 2014 MED BOARDS; 2015
ability of the most common etiologic agent to produce TOPNOTCH MD)
the disease?
A. Capsule
B. Pili
C. Lipopolysaccharide
D. Exotoxin production
E. Techoic acid
262 An alcoholic patient is diagnosed with pneumonia. K. pneuomoniae nonmotile, encapsulated, RAYMUND MARTIN MIDTERM 3
Microbiologic studies identify an encapsulated lactose lactose fermenting, common cause of LI, MD (TOP 1 - AUG EXAM - FEB
fermenting, nonmotile Gram (-) bacteria. What is the pneumonia in alcoholics 2014 MED BOARDS; 2015
most likely etiologic agent? TOPNOTCH MD)
A. Moraxella catarrhalis
B. Haemophilus influenzae
C. Klebsiella pneumoniae
D. Escherichia coli
E. Pseudomonas aeruginosa

263 A patient presenting with diabetic ketoacidosis is noted Descriptive of Rhizopus which is common in RAYMUND MARTIN MIDTERM 3
to have complaints of retro-orbital headache, facial pain, DKA. Aspergillus is septated and branches at LI, MD (TOP 1 - AUG EXAM - FEB
fever, and nasal stuffiness. Necrotic eschar is found on acute angles 2014 MED BOARDS; 2015
the nasal cavity. Biopsy of the area reveals nonseptate, TOPNOTCH MD)
broad-based hyphae that branch at right angles. What is
the most likely causative agent?
A. Rhizopus
B. Aspergillus
C. Histoplasma
D. Coccidioides
E. Candida
264 A tuberculoid type of leprosy is best characterized by: B and C indicate lepromatous type RAYMUND MARTIN MIDTERM 3
A. High cell-mediated immunity LI, MD (TOP 1 - AUG EXAM - FEB
B. Diffuse cutaneous lesions 2014 MED BOARDS; 2015
C. Predominantly Th2 response TOPNOTCH MD)
D. All of these
E. None of these

TOPNOTCH MEDICAL BOARD PREP MICROBIOLOGY SUPEREXAM Page 32 of 90


For inquiries visit www.topnotchboardprep.com.ph or email us at topnotchmedicalboardprep@gmail.com
TOPNOTCH MEDICAL BOARD PREP MICROBIOLOGY SUPEREXAM
For inquiries visit www.topnotchboardprep.com.ph or email us at topnotchmedicalboardprep@gmail.com
Item QUESTION EXPLANATION AUTHOR TOPNOTCH
# EXAM
265 A 33 year old female sex worker presents to your clinic Descriptive of gumma and supported by the RAYMUND MARTIN MIDTERM 3
with complaints of solitary lesions in the perineal area manifestations indicating tertiary syphylis LI, MD (TOP 1 - AUG EXAM - FEB
described as nodular and rubbery with occasional 2014 MED BOARDS; 2015
ulcerations. Further physical examination reveals ataxia, TOPNOTCH MD)
diminished vibratory sensation, and pupil that does not
react to light but responds to accommodation. The
perineal lesions described refer to:
A. Condyloma lata
B. Condyloma acuminatum
C. Chancroid
D. Soft Chancre
E. Gumma

266 A 33 year old man who had an unprotected sex with a Culture medium of choice for Neisseria RAYMUND MARTIN MIDTERM 3
prostitute presents in the ER with complaints of dysuria LI, MD (TOP 1 - AUG EXAM - FEB
and purulent urethral discharge. Gram staining of 2014 MED BOARDS; 2015
discharge reveals numerous intracellular diplococci TOPNOTCH MD)
within neutrophils. In order to culture the etiologic agent
in the laboratory, you will use the following culture
medium:
A. Bordet Gengou
B. Thayer Martin VCN
C. Tellurite agar
D. Thiosulfate citrate bile salt sucrose
E. MacConkey agar

267 A 65 year old patient presents at the ER with dyspnea, S. pneumoniae - alpha hemolytic, bile soluble, RAYMUND MARTIN MIDTERM 3
fever, and purulent sputum production. Chest X-ray catalase negative, optochin sensitive LI, MD (TOP 1 - AUG EXAM - FEB
reveals left lower lobe consolidation. The most common 2014 MED BOARDS; 2015
causative responsible in this age group is best TOPNOTCH MD)
characterized by which characteristic:
A. Bile soluble
B. Catalase positive
C. Optochin resistant
D. Demonstrates complete hemolysis
E. C and D
268 Genetic reassortment plays a major role in the Genetic reassortment occurs in virus with RAYMUND MARTIN MIDTERM 3
generation of epidemiologically significant strains of septated genome such as LI, MD (TOP 1 - AUG EXAM - FEB
influenza virus. Similar to the influenza virus, genetic Orthomyxovirus(Influenza) and 2014 MED BOARDS; 2015
reassortment will also be possible in which of the Reovirus(Rotavirus) TOPNOTCH MD)
following viruses?
A. Rubeola
B. Rubella
C. Respiratory syncytial virus
D. Rotavirus
269 A patient hospitalized for severe electrical burns is noted Ecthyma gangrenosum cause by Pseudomonas RAYMUND MARTIN MIDTERM 3
to have hemorrhagic bullae progressing to a grayish LI, MD (TOP 1 - AUG EXAM - FEB
black eschar with surrounding erythema. The etiologic 2014 MED BOARDS; 2015
agent will most likely have the following characteristic: TOPNOTCH MD)
A. Non-lactose fermenter
B. Produces a fluorescent blue-green pigment
C. Oxidase positive
D. A and B
E. All of these

270 Which of the following helminthic infections will NOT Does not enter the systemic circulation so it RAYMUND MARTIN MIDTERM 3
show eosinophilia? doesn't cause eosinophilia. The rest do. LI, MD (TOP 1 - AUG EXAM - FEB
A. Strongyloides stercoralis 2014 MED BOARDS; 2015
B. Ascaris lumbricoides TOPNOTCH MD)
C. Necator americanus
D. Trichuris trichiura
E. All will cause elevated eosinophil levels

271 A 22 year old sexually active female presents to your Initial body or reticulate body - intracellular; RAYMUND MARTIN MIDTERM 3
clinic complaining of lower abdominal pain, vaginal non-infectious LI, MD (TOP 1 - AUG EXAM - FEB
discharge, and pain upon sexual intercourse. Pelvic 2014 MED BOARDS; 2015
examination reveals exquisite cervical tenderness. You TOPNOTCH MD)
suspect STD but microscopy fails to reveal the
intracellular Gram (-) diplococci that you expected.
However, after incubation in cultured cells and staining
with iodine, you noted presence of cytoplasmic inclusion
bodies. This structure refers to:
A. Initial body
B. Infectious particle
C. Reticulate body
D. A and C
E. A and B

TOPNOTCH MEDICAL BOARD PREP MICROBIOLOGY SUPEREXAM Page 33 of 90


For inquiries visit www.topnotchboardprep.com.ph or email us at topnotchmedicalboardprep@gmail.com
TOPNOTCH MEDICAL BOARD PREP MICROBIOLOGY SUPEREXAM
For inquiries visit www.topnotchboardprep.com.ph or email us at topnotchmedicalboardprep@gmail.com
Item QUESTION EXPLANATION AUTHOR TOPNOTCH
# EXAM
272 A 4 year old pediatric patient with an unknown Diphtheria toxin is obtained via lysogenic RAYMUND MARTIN MIDTERM 3
immunization history presents in the ER with low-grade conversion LI, MD (TOP 1 - AUG EXAM - FEB
fever and sore throat. Physical examination reveals 2014 MED BOARDS; 2015
edematous pharynx with thick gray leathery membrane. TOPNOTCH MD)
The etiologic agent most likely acquired its capability to
produce an exotoxin through which mechanism?
A. Conjugation
B. Transformation
C. Plasmid-mediated transfer
D. Lysogenic conversion
E. Transposon
273 An HbsAg positive mother gives birth to a live baby girl HBIG and Hep B vaccine both given RAYMUND MARTIN MIDTERM 3
via NSVD with no notable complications. To ensure that LI, MD (TOP 1 - AUG EXAM - FEB
baby is protected from hepatitis B, immunization is 2014 MED BOARDS; 2015
given within 12 hours of birth. Which of the following TOPNOTCH MD)
mechanism of immunity results from the intervention
done?
A. active immunity
B. passive immunity
C. active and passive immunity
D. none of the above
274 An 8 year old child presents with irritability, fever, and H. influenzae - requires hemin and NAD; can RAYMUND MARTIN MIDTERM 3
neck rigidity. CSF samples were taken. There was no grow in chocolate agar or blood agar with S. LI, MD (TOP 1 - AUG EXAM - FEB
noted growth on blood agar. But in repeat blood agar aureus 2014 MED BOARDS; 2015
culture containing colonies of S. aureus, growth is noted TOPNOTCH MD)
in areas of hemolysis. This is characteristic of which
microorganism?
A. Streptococcus pneumoniae
B. Neisseria meningitides
C. Haemophilus influenzae
D. Listeria monocytogenes
E. Bordetella pertussis
275 What is the drug of choice for a patient diagnosed with Penicillin is still the drug of choice for RAYMUND MARTIN MIDTERM 3
meningococcal meningitis? meningococcemia LI, MD (TOP 1 - AUG EXAM - FEB
A. Penicillin 2014 MED BOARDS; 2015
B. Ciprofloxacin TOPNOTCH MD)
C. TMP-SMX
D. Ceftriaxone
E. Supportive treatment

276 A 72 year old female presents to your clinic with Lobar consolidation, atypycal symptoms like RAYMUND MARTIN MIDTERM 3
complaints of dyspnea, high grade fever and productive myalgia, abdominal pain, and diarrhea, and LI, MD (TOP 1 - AUG EXAM - FEB
cough. Further history revealed associated symptoms of pulse-temperature dissociation are indicative 2014 MED BOARDS; 2015
headache, myalgia, abdominal pain, and diarrhea. Vital of Legionnaire's disease which is usually TOPNOTCH MD)
signs: BP=140/90, HR=88, RR=28 T=40 degrees Celsius. spread through cruise ship/air-conditoning
Which of the following significant findings would most system
likely be in the patient’s history?
A. Close exposure to a patient being treated with TB
regimen
B. Recent cruise ship travel
C. Hospitalization in the past month
D. History of alcoholism
E. Ingestion of undercooked meat
277 A 10 year old child presents with anemia, acute renal Shiga toxin inhibits 60S ribosomal subunit RAYMUND MARTIN MIDTERM 3
failure, and thrombocytopenia. If this condition results LI, MD (TOP 1 - AUG EXAM - FEB
from an infectious cause that is toxin-mediated most 2014 MED BOARDS; 2015
likely due to consumption of undercooked beef, in what TOPNOTCH MD)
way does the toxin bring about the patient’s
manifestations?
A. Inhibition of elongation factors
B. Inhibition of protein synthesis by binding to 60S
ribosomal subunit
C. Inhibition of protein synthesis by binding to 40S
ribosomal subunit
D. Inhibition of RNA polymerase
E. Inhibition of binding of aminoacyl tRNA to ribosomal
subunit
278 A 45 year old male patient who recently went for a Both mansoni and japonicum live in RAYMUND MARTIN MIDTERM 3
vacation in Leyte presented at the ER complaining of mesenteric venules. Haematobium resides in LI, MD (TOP 1 - AUG EXAM - FEB
fever, malaise, myalgia, and right upper quadrant pain. bladder venules 2014 MED BOARDS; 2015
You are able to elicit a history of wading in fresh water. TOPNOTCH MD)
Schistosomiasis becomes your primary impression.
Being a knowledgeable medical intern, you recall the life
cycle of Schistosoma. The adult trematodes of which of
the following species will you expect to reside in the
mesenteric venules of the GI tract?
A. Schistosoma japonicum
B. Schistosoma haematobium
C. Schistosoma mansoni
D. Schistosoma japonicum and Schistosoma mansoni
E. All of these

TOPNOTCH MEDICAL BOARD PREP MICROBIOLOGY SUPEREXAM Page 34 of 90


For inquiries visit www.topnotchboardprep.com.ph or email us at topnotchmedicalboardprep@gmail.com
TOPNOTCH MEDICAL BOARD PREP MICROBIOLOGY SUPEREXAM
For inquiries visit www.topnotchboardprep.com.ph or email us at topnotchmedicalboardprep@gmail.com
Item QUESTION EXPLANATION AUTHOR TOPNOTCH
# EXAM
279 All of the following bacteria are oxidase positive except: All are oxidase positive RAYMUND MARTIN MIDTERM 3
A. V. cholerae LI, MD (TOP 1 - AUG EXAM - FEB
B. C. jejuni 2014 MED BOARDS; 2015
C. P. aeruginosa TOPNOTCH MD)
D. None of the above
E. All of the above

280 Which of the following characterizes HIV- RAYMUND MARTIN MIDTERM 3


infection/AIDS? LI, MD (TOP 1 - AUG EXAM - FEB
A. Decreased monocyte-macrophage counts 2014 MED BOARDS; 2015
B. CD4/CD8 ratio of 0.5 TOPNOTCH MD)
C. CD4/CD8 ratio of 1.5-2.0
D. Markedly decreased CD8 and NK cell counts
E. B and D

281 What is the causative agent of granulomatosis Granulomatosis infantiseptica (or early onset ERIC ROYD FINAL EXAM
infantiseptica? listeriosis) is caused by L. monocyotgenes. It is TALAVERA, MD (TOP - FEB 2015
A. Chlamydia trachomatis characterized by late miscarriage or birth 1 - AUG 2014 MED
B. Mycobacterium tuberculosis complicated by sepsis, multiorgan abscesses BOARDS; TOPNOTCH
C. Neisseria gonorrhoea and disseminated granulomas MD)
D. Listeria monocytogenes
E. Streptococcus agalactiae

282 A 55 year old male who works as a caretaker in an aviary Given the occupational exposure to bird ERIC ROYD FINAL EXAM
presented with a 6 day history of neck stiffness, (particularly pigeon) feces, the patient has a TALAVERA, MD (TOP - FEB 2015
headache and vomiting. CSF analysis revealed mild risk factor for acquiring Cryptococcus 1 - AUG 2014 MED
elevation of protein levels and a decreased glucose level. neoformans. The lack of response to IV BOARDS; TOPNOTCH
He was started on intravenous ceftriaxone however ceftriaxone makes Streptococcus and MD)
there was no note of clinical improvement. Which of the Neisseria less likely. Meningitis secondary to
following etiologic agents is most likely responsible for naegleria fowleri would mostly present in a
his symptoms? severe fulminant course and would often
A. Streptococcus pneumoniae present with a history of swimming in fresh
B. Neisseria meningitidis water
C. Cryptococcus neoformans
D. Naegleria fowleri
E. None of the above
283 A 22 year old male presented with a 5 day history of This is a most probably a case of severe ERIC ROYD FINAL EXAM
fever and calf pain. He admitted to wading in flood water leptospirosis. Appropriate drugs for severe TALAVERA, MD (TOP - FEB 2015
one week ago. Physical examination reveals a hyperemic leptospirosis include: Pen G, Ceftriaxone, 1 - AUG 2014 MED
conjunctiva and an icteric sclearae. Laboratory findings Ampicillin, Cefotaxime and Azithromycin BOARDS; TOPNOTCH
showed elevated creatinine and thrombocytopenia. MD)
Which of the following is the most appropriate antibiotic
for this case?
A. Metronidazole
B. Ceftriaxone
C. Vancomycin
D. Aztreonam
E. Ciprofloxacin

284 Necrotizing fasciitis caused by Streptococcus pyogenes is Exotoxin A: streptococcal toxic shock ERIC ROYD FINAL EXAM
mediated by which of the following toxins? syndrome, Erythrogenic Toxin: scarlet fever, TALAVERA, MD (TOP - FEB 2015
A. Exotoxin A Alpha Toxin: causes necrotizing fasciitis in 1 - AUG 2014 MED
B. Exotoin B Stapylococcus BOARDS; TOPNOTCH
C. Erythrogenic toxin MD)
D. Alpha toxin
E. Streptolysin O

285 A 27 year old male, diagnosed with AIDS and is currently CMV retinitis is the most important cause of ERIC ROYD FINAL EXAM
on HAART presented with a 2 weeks history of painless, blindness in immunosuppressed individuals. TALAVERA, MD (TOP - FEB 2015
progressive blurring of vision. His latest CD4 count was It is most common if the CD4 count falls below 1 - AUG 2014 MED
noted to be 35/uL. Fundoscopic findings showed areas 50/uL. The characteristic retinal appearance BOARDS; TOPNOTCH
of retinal necrosis, edema and hemorrhage. What is the is that of perivascular hemorrhage and MD)
most probable causative agent in this case? exudate
A. Chlamydia trachomatis
B. Pneumocystis carinii
C. Histoplasma capsulatum
D. Cytomegalovirus
E. Toxoplasma gondii
286 Which of the following causative agents of malaria (with Quartan malarial nephropathy has been ERIC ROYD FINAL EXAM
repeated or chronic infections) has been implicated to implicated with repeated or chronic infections TALAVERA, MD (TOP - FEB 2015
cause nephrotic syndrome? of P. malariae. The histologic appearance is 1 - AUG 2014 MED
A. Plasmodium falciparum that of FSG with splitting of the basement BOARDS; TOPNOTCH
B. Plasmodium malariae membrane It responds poorly to antimalarials, MD)
C. Plasmodium ovale steroids or cytotoxic drugs.
D. Plasmodium vivax
E. Both C and D

TOPNOTCH MEDICAL BOARD PREP MICROBIOLOGY SUPEREXAM Page 35 of 90


For inquiries visit www.topnotchboardprep.com.ph or email us at topnotchmedicalboardprep@gmail.com
TOPNOTCH MEDICAL BOARD PREP MICROBIOLOGY SUPEREXAM
For inquiries visit www.topnotchboardprep.com.ph or email us at topnotchmedicalboardprep@gmail.com
Item QUESTION EXPLANATION AUTHOR TOPNOTCH
# EXAM
287 A 17 year old female presented with a 1 week history of 3rd-4th week is the week of complications in ERIC ROYD FINAL EXAM
fever and abdominal pain. Diagnostic work ups led to a typhoid fever TALAVERA, MD (TOP - FEB 2015
diagnosis of typhoid fever. Life threatening 1 - AUG 2014 MED
complications such as gastrointestinal bleeding and BOARDS; TOPNOTCH
intestinal perforation are most likey to occur at what MD)
week of illness ?
A. 3rd-4th week
B. 5th-6th week
C. 1st-2nd week
D. 4th-5th week
E. None of the above
288 Which of the following trematodes have been implicated S. haematobium are found in the venous ERIC ROYD FINAL EXAM
to cause squamous cell carcinoma of the bladder? plexuses around the urinary bladder and the TALAVERA, MD (TOP - FEB 2015
A. Paragonimus westermanii released eggs traverse the wall of the urine 1 - AUG 2014 MED
B. Clonorchis sinensis bladder causing haematuria and fibrosis of the BOARDS; TOPNOTCH
C. Fasciola hepatica bladder. Studies have shown the relationship MD)
D. Fasciolopsis buski between S. haematobium infection and the
E. Schistosoma haematobium development of squamous cell carcinoma of
the bladder

289 A 35 year old woman with a history of recurrent urinary Struvite precipitates in alkaline urine, forming ERIC ROYD FINAL EXAM
tract infection comes to the ER due to dysuria stones. Struvite stones are potentiated by TALAVERA, MD (TOP - FEB 2015
accompanied by frequency and urgency. Urinalysis was bacterial infection that hydrolyzes urea to 1 - AUG 2014 MED
done which showed an alkaline urine, pyuria and the ammonium and raises urine pH to neutral or BOARDS; TOPNOTCH
presence of numerous struvite crystals. The cause of her alkaline values. Urea-splitting organisms MD)
UTI is likely to be which of the following? include Proteus, Pseudomonas, Klebsiella,
A. Enterobacter aerogenes Staphylococcus, and Mycoplasma.
B. Proteus mirabilis
C. Citrobacter freundii
D. Escherichia coli
E. Serratia marcescens
290 Antigen-antibody complexes are phagocytosed more C3b functions as an opsonin, thereby ERIC ROYD FINAL EXAM
effectively in the presence of which complement enhancing the phagocytosis of bacteria. C3a TALAVERA, MD (TOP - FEB 2015
component? and C5a are anaphylatoxins. C5b6789 form 1 - AUG 2014 MED
A. C3b the membrane attack complex BOARDS; TOPNOTCH
B. C3a MD)
C. C5a
D. C56789 complex
E. None of the above

291 What is the most common cause of acute infective ERIC ROYD FINAL EXAM
endocarditis among known intravenous drug abusers ? TALAVERA, MD (TOP - FEB 2015
A. Viridans streptococci 1 - AUG 2014 MED
B. Enterococcus faecalis BOARDS; TOPNOTCH
C. Staphylococcus aureus MD)
D. Kingella kingae
E. Eikenella corrodens

292 A 25 year old man suffered major injuries to his right leg This is a case of clostridial myonecrosis (gas ERIC ROYD FINAL EXAM
after a motorcycle accident. He was brough to the ER gangrene) caused by c. perfringens. From a TALAVERA, MD (TOP - FEB 2015
after 2 days due to a high grade fever. On examination contaminated wound, the infection spreads in 1 - AUG 2014 MED
he was febrile at 39.6 C and tachycardic at 128 bpm. His 1-3 days to produce crepitation in BOARDS; TOPNOTCH
right leg is swollen and tense, with thin, dark serous fluid subcutaneous tissue and muscle, foul-smelling MD)
draining from the wound. The skin is cool, pale and discharge, fever, hemolysis and eventuallly
shining. Crepitus can also be felt. Which of the following progress to shock and death.
is the most likely causative agent?
A. Clostridium tetani
B. Bacillus anthracis
C. Escherichia coli
D. Clostridium perfringens
E. None of the above

293 The principal reservoir or the "mixing bowl" for the Pigs serve as the mixing bowl within which ERIC ROYD FINAL EXAM
antigenic shift variants of influenza virus appears to be human, avian and swine viruses reassort TALAVERA, MD (TOP - FEB 2015
which of the following animals? 1 - AUG 2014 MED
A. Rodents BOARDS; TOPNOTCH
B. Mosquitoes MD)
C. Dogs
D. Cats
E. Swine

294 A 19 year old female complains of persistent abdominal Giardia lamblia is the most common cause of ERIC ROYD FINAL EXAM
pain, bloatedness and frequent loose stools. She has backpacker's diarrhea. Symptoms would TALAVERA, MD (TOP - FEB 2015
recently returned a month ago from a 3 week hiking and include chronic diarrhea, bloateadness and 1 - AUG 2014 MED
camping trek. Which of the following is most likely characteristic foul smelling stools. BOARDS; TOPNOTCH
responsible for the illness of the patient? MD)
A. Entamoeba coli
B. Trichomonas vaginalis
C. Plasmodium vivax
D. Naegleria fowleri
E. Giardia lamblia

TOPNOTCH MEDICAL BOARD PREP MICROBIOLOGY SUPEREXAM Page 36 of 90


For inquiries visit www.topnotchboardprep.com.ph or email us at topnotchmedicalboardprep@gmail.com
TOPNOTCH MEDICAL BOARD PREP MICROBIOLOGY SUPEREXAM
For inquiries visit www.topnotchboardprep.com.ph or email us at topnotchmedicalboardprep@gmail.com
Item QUESTION EXPLANATION AUTHOR TOPNOTCH
# EXAM
295 The presence of a finding of a "strawberry" cervix points ERIC ROYD FINAL EXAM
to which of the following causative agents? TALAVERA, MD (TOP - FEB 2015
A. Gardnerella vaginalis 1 - AUG 2014 MED
B. Candida albicans BOARDS; TOPNOTCH
C. Trichomonas vaginalis MD)
D. Neisseria gonorrhoea
E. Chlamydia trachomatis

296 Which of the following virus would present with a ERIC ROYD FINAL EXAM
vesicular centrifugal rash described as "a dewdrop on a TALAVERA, MD (TOP - FEB 2015
rose petal appearance" ? 1 - AUG 2014 MED
A. Measles BOARDS; TOPNOTCH
B. Adenovirus MD)
C. Parvovirus
D. Varicella Zoster
E. Coxsackie

297 A 3 year old girl, with an unknown immunization This most likely a case of epiglottitis. ERIC ROYD FINAL EXAM
history, presented to the emergency room due to Symptoms include fever, hoarseness, drooling TALAVERA, MD (TOP - FEB 2015
difficulty of breathing. She has a high grade fever with a preference for tripod position. The 1 - AUG 2014 MED
accompanied by drooling and hoarseness. Auscultation incidence of epiglottitis due to H. influenzae BOARDS; TOPNOTCH
revealed the presence of stridor. She was immediately has been decreasing due to immunization. MD)
intubated and started on intravenous anitbiotics. What is Characteristic radiologic findings inclue the
the most likely causative agent in this case? thumb sign. Endoscopy will show a cherry red
A. Haemophilus influenzae epiglottis Treatment involves airway
B. Escherichia coli protection and antibiotics (ceftriaxone,
C. Bacteroides fragilis ampicillin)
D. Pseudomonas aeruginosa
E. Staphylococcus epidermidis

298 Which of the following forms of extrapulmonary The most infectious patients are those with ERIC ROYD FINAL EXAM
tuberculosis when present confers an individual to cavitary pulmonary TB and laryngeal TB TALAVERA, MD (TOP - FEB 2015
become highly infectious? 1 - AUG 2014 MED
A. Scrofula BOARDS; TOPNOTCH
B. Gastrointestinal TB MD)
C. Laryngeal TB
D. TB meningitis
E. Genitourinary TB

299 Negri bodies, the pathognomonic lesion in rabies, is most SIMILAR TO PREVIOUS BOARD EXAM ERIC ROYD FINAL EXAM
commonly seen in what structure in the CNS? CONCEPT/PRINCIPLE. Negri bodies are most TALAVERA, MD (TOP - FEB 2015
A. Midbrain commonly seen in Purkinje cells of the 1 - AUG 2014 MED
B. Pons cerebellum and in the pyramidal cells of the BOARDS; TOPNOTCH
C. Medulla hippocampus MD)
D. Cerebellum
E. Thalamus

300 A 46 year old female, who is a chronic alcoholic, Pneumonia due to Klebsiella are more ERIC ROYD FINAL EXAM
presented with a 4 day history of cough with a common in alcoholics. They present with a TALAVERA, MD (TOP - FEB 2015
characteristic thick and blood-tinged sputum, fever and "currant jelly" like sputum. Cultures will 1 - AUG 2014 MED
dyspnea. Physical exam showed crackles on both right usually show thick mucoid colonies due to the BOARDS; TOPNOTCH
and left lower lung fields. Sputum GS and CS were done. capsule. Their morphology under gram stain is MD)
The gram stain showed numerous gram negative bacilli. of a gram negative bacilli.
Culture yielded growth of pinkish mucoid colonies in the
MacConkey Agar. Which of the following is the likely
causative agent ?
A. Hemophilus influenzae
B. Staphylococcus aureus
C. Streptococcus pneumoniae
D. Mycoplasma pneumoniae
E. Klebsiella pneumoniae
301 the relative resistance of bacillus anthracis spores to (SIMILAR TO PREVIOUS BOARD EXAM LEAN ANGELO BACK-UP
antibacterial agents are primarily due to what layer of CONCEPT/PRINCIPLE) layers of the SILVERIO, MD (TOP 4 MIDTERM
the endospore ? endospore from innermost to outermost: 1. - AUG 2014 MED EXAM - FEB
A. cortex core - the protoplast containing chromosomes BOARDS; TOPNOTCH 2015
B. Coat and all the necessary organelles. Contains MD), MD
C. exosporium large amount of calcium dipicolinate which
D. Core imparts its heat resistance. 2. spore wall -
E. None innermost spore membrane, contains normal
peptidoglygen components 3. cortex- thickest
layer, highly susceptible to lysosyme and its
autolysis plays a role in spore germination. 4.
coat- keratin like membrane with abundant
disulfide bonds. imparts its impermeabilty to
most antibacterial agents. 5. exosporium-
outermost, function unclear. Jawets 25th ed p
26.
302 In what phase of the microbial growth curve wherein the the phases of growth curve ( in order) 1. lag - LEAN ANGELO BACK-UP
growth rate is constant? zero rate, 2. exponential - constant 3. SILVERIO, MD (TOP 4 MIDTERM
A. Lag maximum stationary - zero 4. decline negative - AUG 2014 MED EXAM - FEB
B. Exponential rate. Jawets 25th ed p 55. BOARDS; TOPNOTCH 2015
C. stationary MD), MD
D. Decline
E. None of the above

TOPNOTCH MEDICAL BOARD PREP MICROBIOLOGY SUPEREXAM Page 37 of 90


For inquiries visit www.topnotchboardprep.com.ph or email us at topnotchmedicalboardprep@gmail.com
TOPNOTCH MEDICAL BOARD PREP MICROBIOLOGY SUPEREXAM
For inquiries visit www.topnotchboardprep.com.ph or email us at topnotchmedicalboardprep@gmail.com
Item QUESTION EXPLANATION AUTHOR TOPNOTCH
# EXAM
303 This type of gene transfer requires the recipient cell to Of all the methods of Gene transfer, only LEAN ANGELO BACK-UP
complete the structure of double stranded DNA by conjugation requires the transfer of only one SILVERIO, MD (TOP 4 MIDTERM
synthesizing the strand that complements the strand strand of DNA to the recipient. Transduction - AUG 2014 MED EXAM - FEB
acquired from the donor? refers to to the phage mediated genetic BOARDS; TOPNOTCH 2015
A. transformation recombination in bacteria. The donor DNA is MD), MD
B. Conjugation carried in a phage coat and is transferred into
C. Transduction the recipient by the mechanism used for
D. gene conversion phage infection. Transformation is the direct
E. none of the above uptake of naked donor DNA by the recipient
cell. gene conversion is the nonreciprocal
transfer of DNA sequence from donor to
recipient. Jawets 25th ed 104-105
304 Which of the following is true about wool sorter disease wool sorter disease is a form of Inhalational LEAN ANGELO BACK-UP
except? anthrax. Its earliest manifestation is SILVERIO, MD (TOP 4 MIDTERM
A. The toxin is made up of three specific proteins: LF, hemorrhagic mediastinitis. Formation of black - AUG 2014 MED EXAM - FEB
PA, EF eschar is a form of cutaneous anthrax. Jawets BOARDS; TOPNOTCH 2015
B. It usually presents with characteristic black eschar 25th ed pp 165-167 MD), MD
C. Culture in blood agar plate will produce a comma
shaped outgrowth from the colony known as medusa
head appearance
D. the recommended treatment for this infection is high
dose ciprofloxacin
E. the presence of poly d glutamate capsule makes the
bacteria virulent.
305 The following are true about staphylococcus aureus? Staphylococcus aureus is catalase positive and LEAN ANGELO BACK-UP
A. It is catalase positive, coagulase negative coagulase positive. The latter as well as the SILVERIO, MD (TOP 4 MIDTERM
B. Panton valentin leukocidin is a form of superantigen ability to ferment mannitol makes S aureus - AUG 2014 MED EXAM - FEB
C. Mechanism of resistance to nafcillin/methicillin is distinct from other staphylococcus species. BOARDS; TOPNOTCH 2015
secondary to production of beta lactamase TSST 1 is the superantigen toxin of S aureus. MD), MD
D. it can ferment mannitol PVL on the otherhand, does not stimulate MHC
E. B and C type II bearing cells but they form pores to
WBC leading cytolysis. resistance to
methicillin is independent of beta lactamase
procuduction. it is secondary to chromosomal
mediated decrease affinity PBP2a. Jawets 25th
ed p 186.
306 Which of the following is a true statement about Mode of transmission of Nocardiosis is via LEAN ANGELO BACK-UP
Nocardiosis? inhalation of the organism coming from soil or SILVERIO, MD (TOP 4 MIDTERM
A. Pulmonary infection are transmissible from person water. It is not infectious and not - AUG 2014 MED EXAM - FEB
to person transmittable to person to person. It is rare for BOARDS; TOPNOTCH 2015
B. It mimics Tuberculosis because it can produce nocardiosis to form granuloma.nocardia is MD), MD
granuloma and caseation necrosis catalase positive. Jawets 25th ed pp 182.
C. It is gram positive, catalase negative and weakly acid
fast
D. the DOC is trimethoprim-sulfamethoxazole
E. all of the above.
307 A 65 y/o male went for consult secondary to 1 week of based on the laboratory results, he is suffering LEAN ANGELO BACK-UP
fever episodes, weight loss and palpitation. Upon PE you from subacute endocarditis secondary to SILVERIO, MD (TOP 4 MIDTERM
noted that he was tachycardic and have holosystolic streptococcus bovis infection. This blood - AUG 2014 MED EXAM - FEB
murmur at the 5th L ICS MCL radiating towards the isolate is most of the time associated with BOARDS; TOPNOTCH 2015
axilla. Echo showed vegetations in the mitral valve. colonic malignancy. Therefore, in adition to MD), MD
Blood culture showed gram positive organism that is high dose intravenous antibiotics, it is prudent
PYR positive, esculin positive and can grow in 6.5% NaCl to perform Colonoscopy as a form of
and bile. what is an ancillary procedure should be done surveillance. Jawets 25th ed p 196.
next ?
A. 12 L ECG and CKMB
B. Chest CT and sputum culture
C. Bone scan
D. Colonoscopy
E. shift antibiotics to vancomycin and or linezolid.
308 A 26 y/o female was brought for consult secondary to 4 Jawets 25th ed p 220-221 LEAN ANGELO BACK-UP
days of diarrheal episodes accompanied by abdominal SILVERIO, MD (TOP 4 MIDTERM
pain. PE showed BP 100/60, PR 97 bpm, RR 21 T 38.3C. - AUG 2014 MED EXAM - FEB
Abdominal examination showed direct tenderness on 4 BOARDS; TOPNOTCH 2015
quadrants. no rebound tenderness or organomegaly MD), MD
noted. fecalysis showed large number of fecal leukocytes
and RBC. culture revealed nonmotile, nonlactose
fermenter, non H2S producer. which of the following
characteristics is true about the causative agent?
A. it does not produce enterotoxin
B. the primary habitat is limited to humans and
primates
C. highly virulent and communicable
D. the basic pathologic process is the invasion of M cells
of the small intestine
E. all of the above

TOPNOTCH MEDICAL BOARD PREP MICROBIOLOGY SUPEREXAM Page 38 of 90


For inquiries visit www.topnotchboardprep.com.ph or email us at topnotchmedicalboardprep@gmail.com
TOPNOTCH MEDICAL BOARD PREP MICROBIOLOGY SUPEREXAM
For inquiries visit www.topnotchboardprep.com.ph or email us at topnotchmedicalboardprep@gmail.com
Item QUESTION EXPLANATION AUTHOR TOPNOTCH
# EXAM
309 A 76 year old male previously diagnosed with the hemorrhagic skin necrosis does not LEAN ANGELO BACK-UP
pneumonia and bronchiectasis is on 7th day of express pus (echtyma gangrenosum) in the SILVERIO, MD (TOP 4 MIDTERM
mechanical ventilation. There were noted recurrence of background of immunocompromised patient - AUG 2014 MED EXAM - FEB
fever and oxygen desaturation. Blood culture was done with prolonged mechanical ventilation is most BOARDS; TOPNOTCH 2015
revealing rod shaped motile organism that is oxidase probably due to Pseudomonas Aureginosa. MD), MD
positive. He was started on intravenous antibiotics, Jawets 25th ed p 229.
however on the 9th hospital stay, there were noted
multiple hemorrhagic necrosis on the skin that does not
express pus. what is the most likely pathogenic organism
involved ?
A. Methicillin Resistant Staphylococcus Aureus
B. Pseudomonas aureginosa
C. Burkholderia cepacia
D. Stenotrophomonas Maltophilia
E. all of the above.
310 A 23 y/o female was brought to ER secondary to 3 days Campylobactor jejuni is a gram negative rods LEAN ANGELO BACK-UP
of fever, abdominal pain and bloody mucoid diarrhea. with comma or gull wing shapes. They are SILVERIO, MD (TOP 4 MIDTERM
Fecalysis done showing gram negative rods with comma motile with a single polar flagellum. It is one of - AUG 2014 MED EXAM - FEB
or gull wing appearance. Which of the medium is the most common cause of gastroenteritis in BOARDS; TOPNOTCH 2015
suitable in isolating this organism ? adults. Skirrows agar is the preferred medium MD), MD
A. Skirrows agar of isolation. Charcoal yeast agar: Legionella
B. Loffler medium pneumophilia; Loffler medium:
C. Charcoal yeast agar corynebacterium diphtheriae, Chocolate agar
D. Chocolate agar with factor V and X with X and V: Hemophilus influenza; TCBS:
E. TCBS agar Vibrio cholera
311 a 9 y/o male brought by his parents secondary to A- bordetella pertussis does not invade blood LEAN ANGELO BACK-UP
intermittent episodes of dry cough followed by rapid vessels. It is only limited in the upper SILVERIO, MD (TOP 4 MIDTERM
inspiration. He states that his seatmate had a prior respiratory tract. B- the cause of the very high - AUG 2014 MED EXAM - FEB
similar symptoms of cough 2 weeks ago. PE showed lymphocytosis (~50,000/ul) is toxin BOARDS; TOPNOTCH 2015
subconjunctival hemorrhage on both eyes, erythematous mediated ( pertussis toxin) and not by MHC MD), MD
nasal turbinates, hyperemic posterior pharyngeal wall dependent lymphocytic activation. secondary
without tonsillar exudates, clear breath sounds on both complications such as otitis media, seizures
lung fields. which of the following is true about his and encephalopathy occurs during the
condition ? convalescent phase. the DOC for pertussis is
A. during this phase, blood cultures would be positive erythromycin. Jawets 25th ed pp 248-249
transient bacteremia.
B. the cause of lymphocytosis is due to the sensitization
of the body to the bacterial filamentous hemagluttinin
C. extrapulmonary complications occur at this phase of
the disease
D. The bacteria can be isolated using Regan lowe
medium
E. all of the above
312 Which of the following is a recommended prophylactic prophylactic medications for close contacts of LEAN ANGELO BACK-UP
medication for close contacts of patients with confirmed N meningitidis infection include the following: SILVERIO, MD (TOP 4 MIDTERM
meningococcemia rifampin, ciprofloxacin and IM ceftriaxone. - AUG 2014 MED EXAM - FEB
A. Cefepime Jawets 25th ed p 270. BOARDS; TOPNOTCH 2015
B. Ceftriaxone MD), MD
C. Cefixime
D. Cefuroxime
E. All of the above

313 The structural feature found in both Treponema the structural feature found in all spirochetes LEAN ANGELO BACK-UP
pallidum and Leptospira interrogans is ? is the axially oriented endoflagella that is SILVERIO, MD (TOP 4 MIDTERM
A. chitin polymers in their peptidoglycan responsible for its corkscrewlike motion. - AUG 2014 MED EXAM - FEB
B. Endoflagella Lippincott micro 3rd ed p 161 BOARDS; TOPNOTCH 2015
C. pili MD), MD
D. Linearly arranged mitochondria
E. Antigenic variation

314 A 43 y/o female went for consult secondary to this is a case of an atypical pneumonia LEAN ANGELO BACK-UP
unrelenting headache, fever, chills and abdominal pain. primarily caused by mycoplasma pneumonia. SILVERIO, MD (TOP 4 MIDTERM
she also complains of nonproductive cough as well as Mycoplasma is unique since it is extremely - AUG 2014 MED EXAM - FEB
otalgia. CBC shows neutrophilia while CXR noted diffuse small zise and their growth on complex but BOARDS; TOPNOTCH 2015
bilateral interstitial infiltrates on all lung fields. she is a cell free media. It does not cell wall, its cell MD), MD
nonsmoker and non alcoholic beverage drinker, her BP membrane is made up of lipoprotein and
is 120/70 mmHg, PR 95bpm, RR 19cpm. Which of the sterols. it produces exotoxin with a similar
following is true about the causative agent of her mechanism as to the bordetella pertussis toxin
condition? .A- is neisseria meningitidis, B- Strep
A. It produces endotoxin (LOS) during autolysis pneumoniae, C-pseudomonas aureginosa
B. Its virulence is due to M protein component of its cell Jawets 25th ed p 313-315
wall
C. sputum culture will yield colonies producing
pyoverdin
D. its cell membrane is contains lipoprotein and sterol
component
E. none of the above

TOPNOTCH MEDICAL BOARD PREP MICROBIOLOGY SUPEREXAM Page 39 of 90


For inquiries visit www.topnotchboardprep.com.ph or email us at topnotchmedicalboardprep@gmail.com
TOPNOTCH MEDICAL BOARD PREP MICROBIOLOGY SUPEREXAM
For inquiries visit www.topnotchboardprep.com.ph or email us at topnotchmedicalboardprep@gmail.com
Item QUESTION EXPLANATION AUTHOR TOPNOTCH
# EXAM
315 A 23 y/o female went for consult secondary to multiple This is a case of lympogranuloma venereum LEAN ANGELO BACK-UP
painful lymphadenopathies along the inguinal area. caused by Chlamydia trachomatis serotype SILVERIO, MD (TOP 4 MIDTERM
some lesion are suppurated with formation of sinus L1-L3. presence of groove sign is - AUG 2014 MED EXAM - FEB
tracts with some fibrosis along the perianal region. characteristic ( cleft made by inguinal BOARDS; TOPNOTCH 2015
Groove sign is positive. Patient is sexually active with 4 ligament bounded above and below by MD), MD
male partners. what is the cause of her condition? lymphadenopathies). B- neisseria does not
A. Chlamydia trachomatis serotype L1 commonly cause suppurative
B. Neisseria Gonorrhea lymphadenopathies in the inguinal region. it is
C. Treponema Pallidum more of endocervicitis and pelvic
D. Hemophilus Ducreyi inflammatory diseases. C- syphilis- painless
E. Kleibsiella Granulomatis papule, macular lesions but not suppurative
lymph nodes. D- painful ragged ulcers with
suppurative lymphadenopathies. E - beefy red
ulcers. Lippincott 3rd ed micro p 371
316 A 10 years old female was brought for consult this is a case of herpangina caused by LEAN ANGELO BACK-UP
secondary cough, colds and febrile episodes for 4 days coksackie virus type A ( member of SILVERIO, MD (TOP 4 MIDTERM
duration. PE showed presence of multiple minute picornaviridae). A- orthomyxoviridae,B- - AUG 2014 MED EXAM - FEB
vesicles along the posterior pharyngeal wall. Tonsils are papovaviridae, D- retroviridae, E- BOARDS; TOPNOTCH 2015
not hyperemic. which of the following characteristics is rhabdoviridae MD), MD
true about the causative agent of this condition?
A. single stranded, negative strand, helical, enveloped,
segmented RNA
B. double stranded circular DNA, icosahedral,
nonenveloped
C. single stranded, positive strand, icosahedral, non
enveloped, nonsegmented RNA
D. single stranded, positive sense linear dual copies of
RNA, enveloped
E. single stranded, negative strand, helical,
nonsegmental RNA, enveloped, contains RNA dependent
RNA polymerase
317 Which of the following cancer is almost always Ebstein Barr Virus is virtually seen in 100% LEAN ANGELO BACK-UP
associated with Ebstein barr virus ? cases of CNS NHL. Other cancers that are SILVERIO, MD (TOP 4 MIDTERM
A. CNS non hodgkin lymphoma associated with this virus are NPCA, burkitts - AUG 2014 MED EXAM - FEB
B. Kaposi sarcoma lymphoma, Hodgkins disease. Kaposi sarcoma BOARDS; TOPNOTCH 2015
C. Hodgkins lymphoma is associated with HHV8. Jawets 25th ed p MD), MD
D. Non african burkitts lymphoma 450-451 (SIMILAR TO PREVIOUS BOARD
E. Angiosarcoma EXAM CONCEPT/PRINCIPLE)

318 Which organ will have the highest titers of rabies virus Jawets 25th ed - 581 LEAN ANGELO BACK-UP
once it spread systemically ? SILVERIO, MD (TOP 4 MIDTERM
A. Pancreas - AUG 2014 MED EXAM - FEB
B. Cervicodorsal skin BOARDS; TOPNOTCH 2015
C. Submaxillary gland MD), MD
D. Parotid gland
E. Sublingual gland

319 What is the DOC for pulmonary paragonimiasis? Markell Medical parasitology 8th ed pp 230 LEAN ANGELO BACK-UP
A. Bithionol SILVERIO, MD (TOP 4 MIDTERM
B. Praziquantrel - AUG 2014 MED EXAM - FEB
C. triclabendazole BOARDS; TOPNOTCH 2015
D. Niclosamide MD), MD
E. Metrifonate

320 which of the following is a tissue nematode? intestinal nematodes - ancylostoma, necator, LEAN ANGELO BACK-UP
A. Strongyloides stercoralis ascaris, enterobius, strongyloides, trichinella, SILVERIO, MD (TOP 4 MIDTERM
B. Wuchereria bancrofti trichiuris. Tissue nematodes- brugia, - AUG 2014 MED EXAM - FEB
C. Ancylostoma duodenale dracunculus, loa loa, onchocerca, toxocara, BOARDS; TOPNOTCH 2015
D. Trichinella spiralis wuchereria. Lippincott 3rd ed p 227 ( MD), MD
E. All of the above SIMILAR TO PREVIOUS BOARD EXAM
CONCEPT/PRINCIPLE)

321 The phlebotomus is the vector for which disease? River blindness - black fly; African sleeping DEBBIE ROSE BACK-UP
A. River blindness sickness - tse tse fly; Chagas disease - reduviid TANENGSY, MD (TOP MIDTERM
B. Filariasis bug 5 - AUG 2014 MED EXAM - FEB
C. Leishmaniasis BOARDS; TOPNOTCH 2015
D. African sleeping sickness MD)
E. Chagas disease

322 What is the most reliable method for diagnosing Specimens are streaked on inhibitory mold DEBBIE ROSE BACK-UP
sporotrichosis? agar on Sobaraud's agar containing TANENGSY, MD (TOP MIDTERM
A. culture antibacterial antibiotics and incubated at 25- 5 - AUG 2014 MED EXAM - FEB
B. microscopy 30C. The identification is confirmed by growth BOARDS; TOPNOTCH 2015
C. serology at 35C and conversion to yeast form. Jawetz MD)
D. any of the above Medical Microbiology 25th edition p.635
E. none of the above

323 Detection of sclerotic bodies is diagnostic of this Chromoblastomycosis is a subcutaneous DEBBIE ROSE BACK-UP
condition: mycotic infection by traumatic inoculation by TANENGSY, MD (TOP MIDTERM
A. dermatophytosis any of the 5 recognized fungal agents residing 5 - AUG 2014 MED EXAM - FEB
B. coccidioidomycosis in soil and vegatation. Jawetz Medical BOARDS; TOPNOTCH 2015
C. mycetoma Microbiology 25th edition p.637 MD)
D. sporotrichosis
E. chromoblastomycosis

TOPNOTCH MEDICAL BOARD PREP MICROBIOLOGY SUPEREXAM Page 40 of 90


For inquiries visit www.topnotchboardprep.com.ph or email us at topnotchmedicalboardprep@gmail.com
TOPNOTCH MEDICAL BOARD PREP MICROBIOLOGY SUPEREXAM
For inquiries visit www.topnotchboardprep.com.ph or email us at topnotchmedicalboardprep@gmail.com
Item QUESTION EXPLANATION AUTHOR TOPNOTCH
# EXAM
324 The most definitive proof that a virus causes human Jawetz Medical Microbiology 25th edition DEBBIE ROSE BACK-UP
cancer is: p.607 TANENGSY, MD (TOP MIDTERM
A. tumor induction in lab animals and transformation of 5 - AUG 2014 MED EXAM - FEB
human cells in culture BOARDS; TOPNOTCH 2015
B. viral infection precedes tumor MD)
C. geographic distribution of viral infection coincides
with that of tumor
D. decrease in tumor incidence by prevention of viral
infection
E. presence of viral markers is higher than in controls
325 Which is not a true statement? All statements are true. Jawetz Medical DEBBIE ROSE BACK-UP
A. Propionibacterium species are normal flora of the Microbiology 25th edition p.276 TANENGSY, MD (TOP MIDTERM
skin, oral cavity, and the large intestine. 5 - AUG 2014 MED EXAM - FEB
B. P. acnes causes the disease acne vulgaris. BOARDS; TOPNOTCH 2015
C. P. acnes is a frequent cause of postsurgical wound MD)
infections especially in those involving insertion of
devices.
D. P. acnes may be a contaminant of blood or CSF
cultures.
E. none of the above
326 Which is not a mechanism of innate immunity? Jawetz Medical Microbiology 25th edition DEBBIE ROSE BACK-UP
A. skin pp.124-126 TANENGSY, MD (TOP MIDTERM
B. phagocytosis 5 - AUG 2014 MED EXAM - FEB
C. complement BOARDS; TOPNOTCH 2015
D. lymphocytes MD)
E. macrophages

327 What is the drug of choice in the treatment of SIMILAR TO PREVIOUS BOARD EXAM DEBBIE ROSE BACK-UP
meningococcal disease? CONCEPT/PRINCIPLE. Jawetz Medical TANENGSY, MD (TOP MIDTERM
A. chloramphenicol Microbiology 25th edition p. 270 5 - AUG 2014 MED EXAM - FEB
B. ceftriaxone BOARDS; TOPNOTCH 2015
C. cefotaxime MD)
D. penicillin G
E. erythromycin

328 What is the most common manifestation of Salmonella Jawetz Medical Microbiology 25th edition DEBBIE ROSE BACK-UP
infection? p.223 TANENGSY, MD (TOP MIDTERM
A. enterocolitis 5 - AUG 2014 MED EXAM - FEB
B. typhoid fever BOARDS; TOPNOTCH 2015
C. bacteremia with focal lesions MD)
D. focal oral infection
E. food poisoning

329 Most microorganisms pathogenic for humans are: Psychrophilic - 15-20C; mesophilic - 30-37C; DEBBIE ROSE BACK-UP
A. psychrophilic thermophilic - 50-60C; hyperthermophilic - TANENGSY, MD (TOP MIDTERM
B. mesophilic above temperature of boiling water. Jawetz 5 - AUG 2014 MED EXAM - FEB
C. thermophilic Medical Microbiology 25th edition p.68 BOARDS; TOPNOTCH 2015
D. hyperthermophilic MD)
E. any of the above

330 True of Pseudomonas aeruginosa? P. aeruginosa is oxidase (+), a non-lactose DEBBIE ROSE BACK-UP
A. oxidase negative fermenter, obligate aerobe. Pyocyanin is non- TANENGSY, MD (TOP MIDTERM
B. lactose fermenter fluorescent and is bluish. 5 - AUG 2014 MED EXAM - FEB
C. grows well at 42C BOARDS; TOPNOTCH 2015
D. facultative anaerobe MD)
E. produces the fluorescent reddish pigment pyocyanin

331 Which of the following is a treponemal antibody test? VDRL - venereal disease research laboratory; DEBBIE ROSE BACK-UP
A. VDRL USR - unheated serum reagin; RPR - rapid TANENGSY, MD (TOP MIDTERM
B. USR plasma reagin; TRUST - toluidine red 5 - AUG 2014 MED EXAM - FEB
C. RPR unheated serum tests; MHA-TP - BOARDS; TOPNOTCH 2015
D. TRUST microhemagluttination Treponema pallidum. MD)
E. MHA-TP Jawetz Medical Microbiology 25th edition
p.302

332 Not true of Chlamydia pneumoniae: C. pneumoniae is rapidly inactivated at room DEBBIE ROSE BACK-UP
A. Serology is the most sensitive method for diagnosis of temperature. Jawetz Medical Microbiology TANENGSY, MD (TOP MIDTERM
its infection. 25th edition pp332-334 5 - AUG 2014 MED EXAM - FEB
B. It grows well at room temperature. BOARDS; TOPNOTCH 2015
C. It may be associated with atherosclerotic coronary MD)
artery and cardiovascular disease.
D. It is susceptible to macrolifes and tetracyclines.
E. Humans are its only known host.

333 The major cause of death in rabies is: Jawetz Medical Microbiology 25th edition DEBBIE ROSE BACK-UP
A. dehydration p.582 TANENGSY, MD (TOP MIDTERM
B. respiratory paralysis 5 - AUG 2014 MED EXAM - FEB
C. hypovolemia BOARDS; TOPNOTCH 2015
D. multi-organ dysfunction MD)
E. myocardial rupture

TOPNOTCH MEDICAL BOARD PREP MICROBIOLOGY SUPEREXAM Page 41 of 90


For inquiries visit www.topnotchboardprep.com.ph or email us at topnotchmedicalboardprep@gmail.com
TOPNOTCH MEDICAL BOARD PREP MICROBIOLOGY SUPEREXAM
For inquiries visit www.topnotchboardprep.com.ph or email us at topnotchmedicalboardprep@gmail.com
Item QUESTION EXPLANATION AUTHOR TOPNOTCH
# EXAM
334 Human diseases associated with B19 parvovirus: Jawetz Medical Microbiology 25th edition p. DEBBIE ROSE BACK-UP
A. Erythema infectiosum 418 TANENGSY, MD (TOP MIDTERM
B. Pure red cell aplasia 5 - AUG 2014 MED EXAM - FEB
C. Transient aplastic crisis BOARDS; TOPNOTCH 2015
D. A & B only MD)
E. all of the above

335 Which HIV protein promotes CD4 degradation? Tat - functions in transactivation; Rev - DEBBIE ROSE BACK-UP
A. Tat expression of viral structural proteins; Nef - TANENGSY, MD (TOP MIDTERM
B. Rev increases viral infectivity, activation of resting 5 - AUG 2014 MED EXAM - FEB
C. Nef T-cells; downregulates expression of CD4- BOARDS; TOPNOTCH 2015
D. Vpr MHC class I; Vpr - increases transport of viral MD)
E. Vpu preintegration complex into nucleus. Jawetz
Medical Microbiology 25th edition p.609

336 The following completely inactivate HIV, except: Jawetz Medical Microbiology 25th edition DEBBIE ROSE BACK-UP
A. 50% ethanol for 10 min at room temperature p.612 TANENGSY, MD (TOP MIDTERM
B. 0.3% H202 for 10 min at room temperature 5 - AUG 2014 MED EXAM - FEB
C. ph 1.0 exposure BOARDS; TOPNOTCH 2015
D. 2.5% Tween 20 MD)
E. 10% household bleach for 10 min at room
temperature

337 Which bacteria produces a green metallic sheen on Eosin DEBBIE ROSE BACK-UP
Methylene Blue agar? TANENGSY, MD (TOP MIDTERM
A. P. aeruginosa 5 - AUG 2014 MED EXAM - FEB
B. E. coli BOARDS; TOPNOTCH 2015
C. P. vulgaris MD)
D. S. dysenteriae
E. S. enteritidis

338 Multi drug resistant tuberculosis is at the very least MDR TB - resistant to INH and rifampin. XDR DEBBIE ROSE BACK-UP
resistant to: (extensively drug resistant) TB - resistant to TANENGSY, MD (TOP MIDTERM
A. INH INH, rifampin, any quinolone, and >/= 3 2nd 5 - AUG 2014 MED EXAM - FEB
B. rifampin line injectable drugs. BOARDS; TOPNOTCH 2015
C. INH, rifampin, any quinolone, and >/= 3 2nd line MD)
injectable drugs
D. INH, and rifampin
E. INH, rifampin, and pyrazinamide
339 A polypeptide obtained from a strain of B. subtilis: DEBBIE ROSE BACK-UP
A. linezolid TANENGSY, MD (TOP MIDTERM
B. quinopristin-dalfopristin 5 - AUG 2014 MED EXAM - FEB
C. polymyxin B BOARDS; TOPNOTCH 2015
D. bacitracin MD)
E. teicoplanin

340 True of Dermatophytosis: Microsporum infects hair and skin. E. DEBBIE ROSE BACK-UP
A. Microsporum infects only skin floccosum infects skin and nails but not hair. TANENGSY, MD (TOP MIDTERM
B. E. floccosum infects skin, hair, and nails Trichophyton species infect hair, skin, and 5 - AUG 2014 MED EXAM - FEB
C. Trichophyton species infect only skin and nails nails. BOARDS; TOPNOTCH 2015
D. Dermatophytes are identified on culture on SDA at MD)
25C
E. none of the above

341 The typical gram reaction and morphology of Nesseriae JESSICA MAE BACK-UP
is: SANCHEZ, MD (TOP 4 MIDTERM
A. Gram negative diplococci - AUG 2014 MED EXAM - FEB
B. Gram positive diplococci BOARDS; TOPNOTCH 2015
C. Gram negative bacilli MD)
D. Gram positive bacilli
342 Which is not true of Neisseria gonorrhoea? Chronic cervicitis or proctitis is usually JESSICA MAE BACK-UP
A. Piliated bacteria appear to be virulent asymptomatic. SANCHEZ, MD (TOP 4 MIDTERM
B. Opa protein expression varies depending on the type - AUG 2014 MED EXAM - FEB
of infection Reference: Jawetz, Medical Microbiology, 25th BOARDS; TOPNOTCH 2015
C. The bacteria attack the mucous membranes of the ed. p. 267 MD)
GUT, eye, rectum and throat
D. Chronic gonococcal cervicitis or proctitis is often
symptomatic
343 Which one of the following statements regarding Chlamydiae are obligate intracellular JESSICA MAE BACK-UP
Chlamydiae is true? parasites, not anaerobes. SANCHEZ, MD (TOP 4 MIDTERM
A. They are obligate anaerobes Subclinical disease is the rule among infected - AUG 2014 MED EXAM - FEB
B. They lack mechanisms for the production of metabolic hosts, and overt disease is the exception. BOARDS; TOPNOTCH 2015
energy and cannot synthesise ATP MD)
C. Chlamydia can be transmitted through sexual contact Reference: Jawetz, Medical Microbiology, 25th
only ed. p. 327
D. Overt disease is usually seen in infected hosts
344 The following diseases are characterised by genital Borrelia causes Lyme disease (Borrelia JESSICA MAE BACK-UP
ulcers and enlarged lymph nodes, except: burgdorferi) and relapsing fever (Borrelia SANCHEZ, MD (TOP 4 MIDTERM
A. Donovanosis recurrentis). These are vector-borne, not - AUG 2014 MED EXAM - FEB
B. Hemophilus ducreyi sexually transmitted, diseases. BOARDS; TOPNOTCH 2015
C. Lymphogranuloma venereum MD)
D. Borrelia

TOPNOTCH MEDICAL BOARD PREP MICROBIOLOGY SUPEREXAM Page 42 of 90


For inquiries visit www.topnotchboardprep.com.ph or email us at topnotchmedicalboardprep@gmail.com
TOPNOTCH MEDICAL BOARD PREP MICROBIOLOGY SUPEREXAM
For inquiries visit www.topnotchboardprep.com.ph or email us at topnotchmedicalboardprep@gmail.com
Item QUESTION EXPLANATION AUTHOR TOPNOTCH
# EXAM
345 This cell wall structure accounts for the positive gram JESSICA MAE BACK-UP
stain reaction: SANCHEZ, MD (TOP 4 MIDTERM
A.Lipid A - AUG 2014 MED EXAM - FEB
B. Peptidoglycan BOARDS; TOPNOTCH 2015
C. Lipopolysaccharide MD)
D. Glycocalyx
346 Salmonella typhi can be differentiated from Shigella spp. Reference: Topnotch Microbiology Handouts JESSICA MAE BACK-UP
by: SANCHEZ, MD (TOP 4 MIDTERM
A. Sulfide production in TSI - AUG 2014 MED EXAM - FEB
B. Not fermenting lactose BOARDS; TOPNOTCH 2015
C. Not utilizing carbon from citrate MD)
D. Not producing indole from tryptophan
347 The following belong to the genera of dermatophytes, JESSICA MAE BACK-UP
except: SANCHEZ, MD (TOP 4 MIDTERM
A. Trichophyton rubrum - AUG 2014 MED EXAM - FEB
B. Microsporum canis BOARDS; TOPNOTCH 2015
C. Epidermophyton floccosum MD)
D. Aspergillus fumigatus
348 Which of the following statements is false regarding Meningitis is the most common manifestation JESSICA MAE BACK-UP
Neisseria? of meningococcemia, but not seen in all cases. SANCHEZ, MD (TOP 4 MIDTERM
A. The important virulence factor of N. meningitides is - AUG 2014 MED EXAM - FEB
the polysaccharide capsule Reference: Jawetz, Medical Microbiology, 25th BOARDS; TOPNOTCH 2015
B. Most gonococci have plasmids which are rare in the ed. p. 269 MD)
meningococci
C. Only piliated gonococci appear to be virulent
D. Meningitis is seen in all cases of meningococcemia
349 A 50 year old male with AIDS consulted his physician for Fungal meningitis would present with low JESSICA MAE BACK-UP
stiffness of the neck, headache with blurred vision, fever, glucose in the CSF. SANCHEZ, MD (TOP 4 MIDTERM
and cough. CSF evaluation and test with India ink - AUG 2014 MED EXAM - FEB
revealed Cryptococcus neoformans. The expected CSF Reference: Jawetz, Medical Microbiology, 25th BOARDS; TOPNOTCH 2015
findings are as follows except: ed. p. 737 MD)
A. High CSF pressure
B. Elevated glucose
C. Elevated proteins
D. Elevated cell count
350 A 45 year old male was noted to be ELISA (+) to HIV. A positive ELISA would still warrant a JESSICA MAE BACK-UP
This result would require: confirmatory test to rule out false-positive SANCHEZ, MD (TOP 4 MIDTERM
A. Treatment with antiretrovirals results. - AUG 2014 MED EXAM - FEB
B. Reporting of the case to the RITM AIDS centre BOARDS; TOPNOTCH 2015
C. Confirmatory test with Western blot Reference: Jawetz, Medical Microbiology, 25th MD)
D. All of the above ed. p. 618
351 In the absence of treatment, a patient with HIV would Reference: Jawetz, Medical Microbiology, 25th JESSICA MAE BACK-UP
have a clinical latency period before turning to AIDS for ed. p. 615 SANCHEZ, MD (TOP 4 MIDTERM
about: - AUG 2014 MED EXAM - FEB
A. 1 year BOARDS; TOPNOTCH 2015
B. 10 years MD)
C. 3-5 years
D. 6 months
352 The ova of this schistosome has a terminal spine: JESSICA MAE BACK-UP
A. Schistosoma hematobium SANCHEZ, MD (TOP 4 MIDTERM
B. Schistosoma mansoni - AUG 2014 MED EXAM - FEB
C. Schistosoma japonicum BOARDS; TOPNOTCH 2015
D. Schistosoma mekongi MD)

353 Autoinfection is responsible for the hyperinfection JESSICA MAE BACK-UP


syndrome of the following parasitic infection: SANCHEZ, MD (TOP 4 MIDTERM
A. Filariasis - AUG 2014 MED EXAM - FEB
B. Ascariasis BOARDS; TOPNOTCH 2015
C. Strongyloidiasis MD)
D. Trichinosis
354 One of the following is not a manifestation of severe P. Relapse occurs in vivax and ovale malaria. JESSICA MAE BACK-UP
falciparum infection: SANCHEZ, MD (TOP 4 MIDTERM
A. Relapse - AUG 2014 MED EXAM - FEB
B. Coma BOARDS; TOPNOTCH 2015
C. Bleeding MD)
D. Diarrhea
355 Circumorbital edema is a classic sign associated with Reference: Topnotch Microbiology Handouts JESSICA MAE BACK-UP
which of the following parasitic infections? SANCHEZ, MD (TOP 4 MIDTERM
A. Trichinella spiralis - AUG 2014 MED EXAM - FEB
B. Enterobius vermicularis BOARDS; TOPNOTCH 2015
C. Ascaris lumbricoides MD)
D. Capillaria philippinensis
356 A 25 year old male mountain climber who spent his Other species would show all forms in a JESSICA MAE BACK-UP
vacation in Palawan for 2 weeks came back with high peripheral blood smear, except for falciparum, SANCHEZ, MD (TOP 4 MIDTERM
fever, chills, and sweating. Presumptive diagnosis is which would only show ring forms and - AUG 2014 MED EXAM - FEB
malaria. If on thick smear, numerous ring forms and gametocytes. BOARDS; TOPNOTCH 2015
gametocytes were appreciated without other stages, one MD)
would think of this Plasmodium species: Reference: Jawetz, Medical Microbiology, 25th
A. vivax ed, p. 677
B. falciparum
C. ovale
D. malariae

TOPNOTCH MEDICAL BOARD PREP MICROBIOLOGY SUPEREXAM Page 43 of 90


For inquiries visit www.topnotchboardprep.com.ph or email us at topnotchmedicalboardprep@gmail.com
TOPNOTCH MEDICAL BOARD PREP MICROBIOLOGY SUPEREXAM
For inquiries visit www.topnotchboardprep.com.ph or email us at topnotchmedicalboardprep@gmail.com
Item QUESTION EXPLANATION AUTHOR TOPNOTCH
# EXAM
357 A 3 year old girl is brought in by her mother because of D-shaped ova are characteristic of Enterobius JESSICA MAE BACK-UP
perianal pruritus. The diagnostic finding most likely to vermicularis SANCHEZ, MD (TOP 4 MIDTERM
be seen on microscopic examination is: - AUG 2014 MED EXAM - FEB
A. Thin-walled ovoid eggs that are flattened on one side BOARDS; TOPNOTCH 2015
and contain a nematode larva MD)
B. Ovoid and have a thick transparent inner shell
covered by albuminous coat
C. Oval or ellipsoid with thin shell usually with embryo at
a four cell stage
D. Barrel shaped with distinctive plug at both sides
358 The following is true of Trichomoniasis: A. Acidic vaginal Vaginal pH is usually maintained in JESSICA MAE BACK-UP
pH in infected females Candidiasis and Trichomoniasis, and elevated SANCHEZ, MD (TOP 4 MIDTERM
B. A female has 100% chance of getting the infection in Bacterial Vaginosis. - AUG 2014 MED EXAM - FEB
from an infected male BOARDS; TOPNOTCH 2015
C. A male has 100% chance of getting infected by an MD)
infected female
D. Can cause neonatal conjunctivitis
359 A sexually active 24 year old woman complains of Reference: Jawetz, Medical Microbiology, 25th JESSICA MAE BACK-UP
vaginal itching and a malodorous purulent discharge. A ed. p. 700 SANCHEZ, MD (TOP 4 MIDTERM
tentative diagnosis of Trichomoniasis is made. To verify - AUG 2014 MED EXAM - FEB
this, one of the following should be included in the BOARDS; TOPNOTCH 2015
workup: MD)
A. Serologic test
B. Fecal smear
C. Wet mount of vaginal fluid
D. ELISA test
360 This drug is used to prevent relapse in malaria infection: JESSICA MAE BACK-UP
A. Chloroquine SANCHEZ, MD (TOP 4 MIDTERM
B. Quinine - AUG 2014 MED EXAM - FEB
C. Primaquine BOARDS; TOPNOTCH 2015
D. Ciprofloxacin MD)

361 Which of the following is/are TRUE of bacterial The spore from innermost to outermost: Core MAIRRE JAMES BACK-UP
endospores EXCEPT? (contains complete nucleus, all protein GADDI, MD (TOP 4 - MIDTERM
A. The spore wall is made up of peptidoglycan except for synthesizing apparatus and energy generating AUG 2013 MED EXAM - FEB
the wall of Bacillus anthracis which is made up of poly D- system; heat resistance attributable to BOARDS; TOPNOTCH 2015
glutamic acid dehydrated state AND calcium dipicolinate), MD)
B. Calcium dipicolinate contributes to the heat resistance Spore wall (innermost layer surrounding the
of the spores inner spore membrane; peptidoglycan layer),
C. Sporulation begins when there is near depletion of Cortex (thickest layer, peptidoglycan layer,
important metabolic substarates with carbon levels extremely sensitive to lysozyme and autolysis
being the most significant factor of the cortex plays a role in spore
D. The cortex is the thickest layer of the spore but it is germination), Coat (keratin like, resistance to
also extremely sensitive to lysozyme chemical agents due to impermeability of this
E. No exception layer), Exosporium (not always present,
composed of CHON, CHO and lipids, function
unclear) Jawetz 26th ed pg 37-38; the
CAPSULE of B. anthracis is the one made up of
poly D-glutamate pg 33
362 A 21/M complained of painful urination with pus per Gram-stained smears of urethral or MAIRRE JAMES BACK-UP
meatus. As the physician, you suspect gonorrhea and in endocervical exudates show gram negative GADDI, MD (TOP 4 - MIDTERM
order to confirm your suspicion you get a urethreal intracellular diplococci but these only give a AUG 2013 MED EXAM - FEB
smear and send it to the laboratory. What diagnostic presumptive diagnosis. Gonorrhea can be BOARDS; TOPNOTCH 2015
exam/s will you order and what do you expect to find? cultured in modified Thayer-Martin medium MD)
A. Gram stained urethral smear showing diplococci Jawetz 26th ed pg 289-290.
within pus cells
B. Culture in chocolate agar with growth of gram
negative diplococci
C. Culture in Thayer-Martin medium with growth of
gram negative diplococci
D. A and C
E. All of the above
363 A 30/M cave tour guide from Sagada came to you with a Histoplasmosis and Paragonimiasis are the MAIRRE JAMES BACK-UP
4 month history of cough, weight loss, progressive most common mimickers of TB. Infection with GADDI, MD (TOP 4 - MIDTERM
weakness and blood streaked sputum. On further Histoplasma results from exposure to large AUG 2013 MED EXAM - FEB
history, he does not smoke and no one from their family inocula of conidia from soil mixed with bird BOARDS; TOPNOTCH 2015
has the same condition. Recent travel history include feces or bat guano in caves (the feces provides MD)
going to Cagayan Province where he tried the local and excellent culture medium for the fungi).
delicacies including raw and fresh crab from Buguey. Infection with Paragonimus is acquired by
Chest xray was initially done showing multiple nodules eating crustacean hosts such as a crayfish or
in the upper lung fields. Based on the above what will freshwater crab (Sundathelphusa philippina).
your differential/s be? What diagnostic exam/s will you Jawetz 26th ed pg 313, 691, 745
order and what result/s will you expect?
A. Paragonimiasis, sputum examination with 10% KOH
revealing the fluke
B. Pulmonary TB, Ziehl-Neelsen stained sputum showing
red rods
C. Histoplasmosis, Giemsa stained blood smear revealing
small ovoid cells within macrophages
D. A and B
E. All of the above

TOPNOTCH MEDICAL BOARD PREP MICROBIOLOGY SUPEREXAM Page 44 of 90


For inquiries visit www.topnotchboardprep.com.ph or email us at topnotchmedicalboardprep@gmail.com
TOPNOTCH MEDICAL BOARD PREP MICROBIOLOGY SUPEREXAM
For inquiries visit www.topnotchboardprep.com.ph or email us at topnotchmedicalboardprep@gmail.com
Item QUESTION EXPLANATION AUTHOR TOPNOTCH
# EXAM
364 Vibrio cholerae can be cultured using Skirrow's: Campylobacter; Bordet-Gengou and MAIRRE JAMES BACK-UP
A. Thiosulfate-citrate-bile-sucrose agar Regan-Lowe agar for Bordatella GADDI, MD (TOP 4 - MIDTERM
B. Skirrow's agar AUG 2013 MED EXAM - FEB
C. Regan-Lowe agar BOARDS; TOPNOTCH 2015
D. Bordet-Gengou agar MD)
E. All of the above

365 A 26/F has recurrent high-grade fever, chills, jaudice and Definitive host - where the sexual stage MAIRRE JAMES BACK-UP
joint pain. Further history revealed travel to Agusan del occurs, in this case for malaria, the mosquito; GADDI, MD (TOP 4 - MIDTERM
Sur. Humans are what type of hosts of this parasite? Intermediate host - where the host harbors AUG 2013 MED EXAM - FEB
A. Definitive host the parasite for a short transition period or for BOARDS; TOPNOTCH 2015
B. Intermediate host a particular developmental stage; Paratenic MD)
C. Paratenic host host - host wherein the parasite does not
D. Reservoir host undergo any further development and are not
E. Accidental host essential to its life cycle. Accumulation of the
parasite may increase chances of transmission
to other hosts, may or may not be included in
the lifecylce of the parasite; Accidental host -
hosts not usually infected by the parasite but
when chanced upon, can actually complete its
life cycle; Reservoir hosts - incidental hosts or
alternative definitive hosts or zoonoses
366 A 22/M came in for multiple nodular lesions in the jaw The patient most likely has cervicofacial MAIRRE JAMES BACK-UP
with multiple draining sinuses. Further history revealed actinomycosis (lumpy jaw) with the GADDI, MD (TOP 4 - MIDTERM
prior dental manipulation a year prior and that the characteristic multiple nodular lesions with AUG 2013 MED EXAM - FEB
symptoms developed after the procedure. On PE you draining sinuses. Sulfur granules can be BOARDS; TOPNOTCH 2015
notice granules present in the exudate. What is your appreciated in the exudate. Treatment MD)
drug of choice? includes prolonged administration of
A. Penicillin penicillin (6-12 months) Jawetz 26th ed pg
B. Erythromycin 298.
C. TMP-SMX
D. Cloxacillin
E. Chloramphenicol
367 A 10/F came in for respiratory distress with marked The signs and symptoms of the patient are MAIRRE JAMES BACK-UP
edema of the neck and severe cervical lymphadenopathy. caused by exotoxin producing GADDI, MD (TOP 4 - MIDTERM
On oral PE, a grayish exudate is seen on the tonsils and Corynebacterium diphtheriae. Diphtheria toxin AUG 2013 MED EXAM - FEB
the pharynx. Which of the following is/are TRUE? inactivates elongation factor 2 which is BOARDS; TOPNOTCH 2015
A. Production of the exotoxin in culture is inhibited by involved in protein synthesis. The production MD)
presence of iron in the medium of the toxin in vitro is largely dependent on
B. The exotoxin inactivates EF-2 causing an arrest in the concentration of iron which is optimal at
protein synthesis 0.14 μg of iron per milliliter. P. aeruginosa
C. Pseudomonas aeruginosa elaborates a similar toxin produces a similar exotoxin. Jawetz 26th ed
which targets EF-2 pg 189.
D. B and C
E. All of the above
368 A 21/F complains of fever, crampy abdominal pain with C jejuni is the most common cause of bacterial MAIRRE JAMES BACK-UP
voluminous stools occasionally being grossly bloody. Lab gastroenteritis (TOPNOTCH) Clinical GADDI, MD (TOP 4 - MIDTERM
tests were done which showing the following: Gram manifestations are acute onset of crampy AUG 2013 MED EXAM - FEB
stain revealed a gram negative rods, catalase and oxidase abdominal pain, profuse diarrhea that may be BOARDS; TOPNOTCH 2015
positive, growth in broth showed that the bacteria were grossly bloody, headache, malaise, and fever. MD)
mostly found on near the top but not at the top most and They are microaerophilic, catalase and oxidase
optimal growth was at 42 °C. The most likely organism positive and gram negative curved rods
isolated is? Jawetz 26th ed pg 259-260 and 262.
A. Vibrio sp.
B. Shigella sp.
C. Salmonella sp.
D. Escherichia coli
E. Campylobacter jejuni
369 Which of the following contain fibers and are unique to Adenoviruses are unique among viruses in MAIRRE JAMES BACK-UP
this class of viruses? that they have a structure called a “fiber” GADDI, MD (TOP 4 - MIDTERM
A. Rhinovirus projecting from the penton bases (Penton AUG 2013 MED EXAM - FEB
B. Coxsackievirus fibers). Jawetz 26th ed pg 457 BOARDS; TOPNOTCH 2015
C. Adenovirus MD)
D. Parvovirus
E. Herpesvirus

370 Autoinfection can occur in which of the following? Autoinfection can occur in all of the choices MAIRRE JAMES BACK-UP
A. Strongyloides stercoralis GADDI, MD (TOP 4 - MIDTERM
B. Capillaria philippinensis AUG 2013 MED EXAM - FEB
C. Enterobius vermicularis BOARDS; TOPNOTCH 2015
D. A and B MD)
E. All of the above

371 An 18/F came in to your clinic complaining of a 3 day MAIRRE JAMES BACK-UP
history of dysuria, increased urinary frequency and GADDI, MD (TOP 4 - MIDTERM
urgency. What is the most likely organism to be isolated AUG 2013 MED EXAM - FEB
if you were to order a urine culture? BOARDS; TOPNOTCH 2015
A. Klebsiella sp. MD)
B. Staphylococcus saprophyticus
C. Proteus mirabilis
D. Escherichia coli
E. Enterobacter sp.

TOPNOTCH MEDICAL BOARD PREP MICROBIOLOGY SUPEREXAM Page 45 of 90


For inquiries visit www.topnotchboardprep.com.ph or email us at topnotchmedicalboardprep@gmail.com
TOPNOTCH MEDICAL BOARD PREP MICROBIOLOGY SUPEREXAM
For inquiries visit www.topnotchboardprep.com.ph or email us at topnotchmedicalboardprep@gmail.com
Item QUESTION EXPLANATION AUTHOR TOPNOTCH
# EXAM
372 HIV gains entry into macrophages by the binding of CXCR 4 chemokine receptor for lymphocyte MAIRRE JAMES BACK-UP
gp120 to which receptor? trophic strains of HIV while CCR 5 chemokine GADDI, MD (TOP 4 - MIDTERM
A. CXCR 4 receptor for macrophage trophic strains of AUG 2013 MED EXAM - FEB
B. CCR 5 HIV. Gp 41 is a fusion peptide of HIV Jawetz BOARDS; TOPNOTCH 2015
C. gp 41 26th ed pg 657. MD)
D. A and B
E. All of the above

373 An 26/F has sudden onset of high fever, vomiting, Toxic shock syndrome is caused by MAIRRE JAMES BACK-UP
rashes, myalgia which rapidly progressed to hypotension superantigens (TSST-1 S aureus, Pyogenic GADDI, MD (TOP 4 - MIDTERM
and multiple organ failure. Which of the following is exotoxin A streptococci) triggering production AUG 2013 MED EXAM - FEB
TRUE? of large amounts of IL-2 and TNF alpha by T BOARDS; TOPNOTCH 2015
A. The superantigen produced by the causative organism lymphocytes. It often occurs within 5 days MD)
triggered the patient's T-cells to produce large amounts after the onset of menses in women who use
of IL-2 and TNF-alpha high-absorbency tampons, but it also occurs in
B. Pyogenic exotoxin A produced by Staphylococcus children and men with staphylococcal wound
aureus and TSST-1 produced by streptococci are infections.Toxic shock syndrome–associated S
examples of superantigens aureus can be found in the vagina, on
C. The causative organism invades the mucosa and tampons, in wounds or other localized
rapidly disseminates throughout the host causing infections, or in the throat but virtually never
massive sepsis in the bloodstream. Jawetz 26th ed pg 157,
D. The above is exclusively as a result of the use of high 202, 203
absorbency tampons
E. None of the above
374 Which of the following is TRUE regarding the cellular The cytochromes and other enzymes and MAIRRE JAMES BACK-UP
structure of bacteria? components of the respiratory chain, GADDI, MD (TOP 4 - MIDTERM
A. Bacteria cannot undergo oxidative phosphorylation including certain dehydrogenases, are located AUG 2013 MED EXAM - FEB
since they lack membrane bound organelles in the cell membrane. The bacterial cell BOARDS; TOPNOTCH 2015
B. Peptidoglycan is unique to Gram positive bacteria that membrane is thus a functional analog of the MD)
is why the Gram negative bacteria are not stained by mitochondrial membrane Jawetz 26th ed pg
crystal violet 21. Peptidoglycan is found in both gram
C. Lipopolysaccharide is only found in Gram negative positive and gram negative bacteria. Listeria, a
bacteria gram positive organism, contains LPS in its
D. The periplasmic space contains hydrolytic enzymes, cell wall (Topnotch).
binding proteins and detoxifying enzymes
E. All of the above
375 During the summer, a 7/M participated in swimming Swimming pool MAIRRE JAMES BACK-UP
classes held at the country club. After a few days he conjunctivitis/Pharyngoconjunctival fever GADDI, MD (TOP 4 - MIDTERM
developed sore throat, cough, fever, eye redness, tearing caused by adenovirus Jawetz 26th ed pg 462. AUG 2013 MED EXAM - FEB
and itchiness. The child is most likely infected with? BOARDS; TOPNOTCH 2015
A. Streptococcus pyogenes MD)
B. Chlamydia trachomatis
C. Adenovirus
D. Herpesvirus
E. Streptococcus pneumonia

376 Which of the following statement/s regarding bacterial The exotoxin of Staphylococcus aureus is heat MAIRRE JAMES BACK-UP
toxins is/are INCORRECT? stable (Topnotch) GADDI, MD (TOP 4 - MIDTERM
A. Exotoxins are found both in Gram positive and Gram AUG 2013 MED EXAM - FEB
negative bacteria BOARDS; TOPNOTCH 2015
B. All exotoxins are heat labile and are rapidly MD)
inactivated by heating above 60 °C
C. The lipid A component of LPS is probably responsible
for its toxicity
D. Exotoxins bind to specific cell receptors while
endotoxins do not
E. All are incorrect
377 Which of the following posses/es reverse transcriptase? Reverse transcriptase found in both MAIRRE JAMES BACK-UP
A. Hepadnaviridae hepadnaviridae and retroviridae Jawetz 26th GADDI, MD (TOP 4 - MIDTERM
B. Retroviridae ed pg 511 and 653. AUG 2013 MED EXAM - FEB
C. Poxviridae BOARDS; TOPNOTCH 2015
D. A and B MD)
E. All of the above

378 Which of the following is TRUE regarding the viral Naked DNA viruses PAPP (Parvovirus, MAIRRE JAMES BACK-UP
envelope? Adenovirus, Polyomavirus, Papillomavirus) GADDI, MD (TOP 4 - MIDTERM
A. Naked non-enveloped viruses include Picornavirus, Naked RNA virus CPR (Calcivirus, AUG 2013 MED EXAM - FEB
Arenavirus, Poxvirus and Paramyxovirus Picornavirus, Reovirus) Topnotch; The BOARDS; TOPNOTCH 2015
B. All enveloped viruses acquire their envelope from the envelope of Herpesvirus is derived from the MD)
plasma membrane nuclear membrane Jawetz 26th ed pg 469.
C. Hepatitis A,B,C,D and E are examples of enveloped Hepatitis A and E are naked pg 507;
viruses Enveloped viruses are much more heat labile
D. Enveloped viruses are more resistant to heat and are and more sensitive to ether solubilization pg
more resistant to ether solubilization 419
E. None of the choices are true

TOPNOTCH MEDICAL BOARD PREP MICROBIOLOGY SUPEREXAM Page 46 of 90


For inquiries visit www.topnotchboardprep.com.ph or email us at topnotchmedicalboardprep@gmail.com
TOPNOTCH MEDICAL BOARD PREP MICROBIOLOGY SUPEREXAM
For inquiries visit www.topnotchboardprep.com.ph or email us at topnotchmedicalboardprep@gmail.com
Item QUESTION EXPLANATION AUTHOR TOPNOTCH
# EXAM
379 A 45/M farmer from Leyte presents with an enlarged Snail fever/Katayama fever occurs after 2-12 MAIRRE JAMES BACK-UP
abdomen, ascites and jaundice. The following is/are weeks following exposure to cercarie; GADDI, MD (TOP 4 - MIDTERM
TRUE of the causative organism: Schistosoma japonicum has only ONE AUG 2013 MED EXAM - FEB
A. An acute illness may occur which is characterized by intermediate host which is Oncomelania BOARDS; TOPNOTCH 2015
headache, fever, chills, diarrhea and eosinophilia 2-12 hupensis quadrasi. The cercarial stage is the MD)
weeks after exposure to the cercaria infective stage, it has no metacercarial stage.
B. The second intermediate host of Schistosoma The adults reside in the superior and inferior
japonicum is Oncomelania hupensis quadrasi mesenteric veins of the SMALL intestine while
C. The metacercariae is the infective stage of the adults of S. mansoni are the ones which reside
organism capable of penetrating the host in the inferior mesenteric veins of the large
D. The adults reside in the inferior mesenteric veins of intestines. Jawetz 26th ed pg 746
the large intestines
E. All of the choices are true
380 A plain cranial CT scan of a 30/M showed multiple ring The patient is infected with Toxoplasma which MAIRRE JAMES BACK-UP
enhancing lesions on the cerebral cortex. On further is most likely due to progessive AIDS. Rapidly GADDI, MD (TOP 4 - MIDTERM
history, he had multiple sexual partners and admitted to multiplying crescentic cells (tachyzoites) AUG 2013 MED EXAM - FEB
taking illegal recreational drugs. What stage of the initiate the acute stage of disease. BOARDS; TOPNOTCH 2015
parasite is most likely inside the lesions? Subsequently, they penetrate nerve cells, MD)
A. Tachyzoite especially of the brain and eye, where they
B. Bradyzoite multiply slowly (as bradyzoites) to form
C. Endozoite quiescent tissue cysts, initiating the chronic
D. Gametozoite stage of disease. Jawetz 26th ed pg 732
E. Trophozoite

381 A 30-year old male with a recent history of travel SIMILAR TO PREVIOUS BOARD EXAM SCOTT RILEY ONG, BACK-UP
developed diarrhea with excessive flatus. Stool exam CONCEPT/PRINCIPLE. This test described is MD (TOP 5 - AUG MIDTERM
was negative. He was then made to swallow a very long known as the "String test" or "Entero-test" for 2014 MED BOARDS; EXAM - FEB
string, with its one end remaining outside the mouth and giardiasis. TOPNOTCH MD) 2015
its other end reaching the bowels. After 4 hours, the
string was withdrawn and examined under the
microscope. Parasite cells were then found attached to
the string. What is your diagnosis?
A. Amoebiasis
B. Giardiasis
C. Cryptosporidium
D. Escherichia coli
E. Campylobacter jejuni
382 A known HBsAg(+) mother was admitted for active SIMILAR TO PREVIOUS BOARD EXAM SCOTT RILEY ONG, BACK-UP
labor. As the pediatrician on-duty, what will your CONCEPT/PRINCIPLE. MD (TOP 5 - AUG MIDTERM
management be upon delivery of her infant? 2014 MED BOARDS; EXAM - FEB
A. Administer active and passive HepB immunization TOPNOTCH MD) 2015
B. Administer active HepB immunization only
C. Test for HBsAg status before administering any HepB
vaccines
D. Administer lamivudine
E. Administer active HepB immunization, then test for
HBsAg status, and give anti-HepB IgG if HBsAg(+)
383 This kidney bean-shaped diplococci is known as the SIMILAR TO PREVIOUS BOARD EXAM SCOTT RILEY ONG, BACK-UP
most common cause of pelvic inflammatory disease. CONCEPT/PRINCIPLE. MD (TOP 5 - AUG MIDTERM
A. Trichominas vaginalis 2014 MED BOARDS; EXAM - FEB
B. Chlamydia trachomatis TOPNOTCH MD) 2015
C. Neisseria meningitidis
D. Neisseria gonorrhoeae
E. Gardnerella vaginais

384 What is the best diagnostic test to confirm a diagnosis of SIMILAR TO PREVIOUS BOARD EXAM SCOTT RILEY ONG, BACK-UP
invasive Entamoeba infection? CONCEPT/PRINCIPLE. In extraintestinal MD (TOP 5 - AUG MIDTERM
A. Wet mount microscopy forms of amoebiasis, serology is the test of 2014 MED BOARDS; EXAM - FEB
B. Stool culture choice as the organism is usually no longer TOPNOTCH MD) 2015
C. Serological assays found in feces. In acute cases of intestinal
D. PCR test of stool sample amoebiasis, wet mount microscopy of stool
E. Indigo dye microscopy samples is preferred due to its ease and cost-
effectiveness, and antibodies may not yet be
present to give a positive serology test.
385 Which of the following phenomenon describes the SIMILAR TO PREVIOUS BOARD EXAM SCOTT RILEY ONG, BACK-UP
process by which bacteria can acquire virulence through CONCEPT/PRINCIPLE. MD (TOP 5 - AUG MIDTERM
the introduction of foreign genetic material via 2014 MED BOARDS; EXAM - FEB
bacteriophage infection? TOPNOTCH MD) 2015
A. Transformation
B. Transduction
C. Conjugation
D. Programmed rearrangement
E. Transudation

386 Which of the following bacteria characteristically SIMILAR TO PREVIOUS BOARD EXAM SCOTT RILEY ONG, BACK-UP
produces a lobar pattern of pneumonia on chest CONCEPT/PRINCIPLE. Lobar pneumonia MD (TOP 5 - AUG MIDTERM
radiographs? (consolidation with air bronchograms, 2014 MED BOARDS; EXAM - FEB
A. Staphylococcus aureus involving multiple lung segments): S. TOPNOTCH MD) 2015
B. Steptococcus pneumoniae pneumoniae, Klebsiella. Bronchopneumonia
C. Streptococcus pyogenes (patchy infiltrates without air bronchograms):
D. Pseudomonas aeruginosa S. aureus, S. pyogenes, Pseudomonas,
E. Mycoplasma pneumoniae anaerobes. Interstitial pneumonia:
Mycoplasma, most viruses.

TOPNOTCH MEDICAL BOARD PREP MICROBIOLOGY SUPEREXAM Page 47 of 90


For inquiries visit www.topnotchboardprep.com.ph or email us at topnotchmedicalboardprep@gmail.com
TOPNOTCH MEDICAL BOARD PREP MICROBIOLOGY SUPEREXAM
For inquiries visit www.topnotchboardprep.com.ph or email us at topnotchmedicalboardprep@gmail.com
Item QUESTION EXPLANATION AUTHOR TOPNOTCH
# EXAM
387 A 43-year old male presented to you with acute onset of SIMILAR TO PREVIOUS BOARD EXAM SCOTT RILEY ONG, BACK-UP
fever, chills and productive cough. Physical examination CONCEPT/PRINCIPLE. Diagnosis: MD (TOP 5 - AUG MIDTERM
revealed bibasal crackles. On further history taking, you Legionnaires' disease (Legionella 2014 MED BOARDS; EXAM - FEB
noted that he is fond of collecting exotic fishes and has a pneumophila infection) TOPNOTCH MD) 2015
large aquarium at home with its own cooling system. To
confirm your diagnosis, which of the following culture
media would you use to isolate the organism from a
sputum sample?
A. Chocolate agar
B. Thioglycollate
C. Stainer-Scholte medium
D. Buffered charcoal yeast extract
E. Eaton medium
388 Which of the following causative agents of pneumonia is SCOTT RILEY ONG, BACK-UP
typically associated with the production of currant-jelly MD (TOP 5 - AUG MIDTERM
sputum? 2014 MED BOARDS; EXAM - FEB
A. Pseudomonas aeruginosa TOPNOTCH MD) 2015
B. Staphylococcus aureus
C. Klebsiella pneumoniae
D. Listeria monocytogenes
E. Streptococcus pyogenes
389 An 8-year old boy presented to you with acute onset of Diagnosis: Corynebacterium diphtheriae with SCOTT RILEY ONG, BACK-UP
fever, cough, sore throat, painful swallowing and Babes-Ernst granules MD (TOP 5 - AUG MIDTERM
hoarseness. Physical examination showed tender 2014 MED BOARDS; EXAM - FEB
cervical lymph nodes and adherent membranes over his TOPNOTCH MD) 2015
tonsils. You then performed a throat swab to confirm
your diagnosis. Microscopic examination of the isolated
organism after culturing your specimen would yield
characteristic metachromatic granules when stained
with which of the following substances?
A. Methylene blue
B. Crystal violet
C. Safranin
D. Carbol fuchsin
E. Periodic acid-Schiff
390 Which of the following common caustive agents of SCOTT RILEY ONG, BACK-UP
neonatal sepsis is a facultative intracellular rod that MD (TOP 5 - AUG MIDTERM
exhibits cold enhancement and a narrow zone of beta- 2014 MED BOARDS; EXAM - FEB
hemolysis when grown in culture media? TOPNOTCH MD) 2015
A. Streptococcus agalactiae
B. Escherichia coli
C. Listeria monocytogenes
D. Legionella pneumophila
E. Neisseria meningitidis

391 A sexually active 18-year old male came to your clinic Diagnosis: Neisseria gonorrhoeae SCOTT RILEY ONG, BACK-UP
due to symptoms of painful urination and penile MD (TOP 5 - AUG MIDTERM
discharge. Gram stain and culture of appropriate 2014 MED BOARDS; EXAM - FEB
specimens showed intracellular diplococci that are able TOPNOTCH MD) 2015
to ferment glucose but not maltose. Which of the
following antibiotics will be your drug of choice for the
eradication of this microorganism?
A. Penicillin G
B. Ceftriaxone
C. Doxycycline
D. Azithromycin
E. Cefuroxime

392 A Filipino went to a desert in Southwestern USA. Upon Diagnosis: Coccidioidomycosis. Choice B SCOTT RILEY ONG, BACK-UP
returning, he developed fever and cough. What would refers to Histoplasmosis. Choice C refers to MD (TOP 5 - AUG MIDTERM
you expect to see in his tissue biopsy? Blastomycosis. Choice D refers to 2014 MED BOARDS; EXAM - FEB
A. Double-walled spherule with endospores Paracoccidioidomycosis. TOPNOTCH MD) 2015
B. Small oval yeasts within macrophages
C. Large broad-based budding yeast cells
D. Large thin-walled multiple budding yeast cells
E. None of the above

393 A 35-year old female who cultivates a variety of roses in Diagnosis: sporotichosis. DOC: itraconazole, SCOTT RILEY ONG, BACK-UP
her backyard presented to you with multiple small skin potassium iodide. MD (TOP 5 - AUG MIDTERM
lesions in her right upper arm and back. The said lesions 2014 MED BOARDS; EXAM - FEB
exhibit ulcerations, erythema, and some pustules. She is TOPNOTCH MD) 2015
afebrile but complains of mild pain in her right elbow.
Given the said history and physical examination, you
decided to start her on which of the following
antimicrobial drug?
A. Oxacillin
B. Amphotericin B
C. Itraconazole
D. Tetracycline
E. Azithromycin

TOPNOTCH MEDICAL BOARD PREP MICROBIOLOGY SUPEREXAM Page 48 of 90


For inquiries visit www.topnotchboardprep.com.ph or email us at topnotchmedicalboardprep@gmail.com
TOPNOTCH MEDICAL BOARD PREP MICROBIOLOGY SUPEREXAM
For inquiries visit www.topnotchboardprep.com.ph or email us at topnotchmedicalboardprep@gmail.com
Item QUESTION EXPLANATION AUTHOR TOPNOTCH
# EXAM
394 Which of the following drugs is used as prophylaxis SIMILAR TO PREVIOUS BOARD EXAM SCOTT RILEY ONG, BACK-UP
against Pneumocystis jirovecii pneumonia? CONCEPT/PRINCIPLE. TMP-SMX and MD (TOP 5 - AUG MIDTERM
A. Pentamidine pentamidine are the drugs used as 2014 MED BOARDS; EXAM - FEB
B. Co-amoxiclav prophylaxis for PCP. TOPNOTCH MD) 2015
C. Rifampicin
D. Benzathine penicillin
E. Suramin

395 Besides retroviruses, which of the following undergoes This is the basis for the use of lamivudine, a SCOTT RILEY ONG, BACK-UP
reverse transcription? nucleoside reverse transcriptase inhibitor, for MD (TOP 5 - AUG MIDTERM
A. Influenza the treatment of Hepatitis B infections. 2014 MED BOARDS; EXAM - FEB
B. Hepatitis B virus TOPNOTCH MD) 2015
C. Respiratory syncytial virus
D. Epstein-Barr virus
E. Human T-lymphotropic virus

396 Which of the following proteins mediates the attachment SCOTT RILEY ONG, BACK-UP
of HIV to the CD4 receptors in helper T cells? MD (TOP 5 - AUG MIDTERM
A. gp41 2014 MED BOARDS; EXAM - FEB
B. gp120 TOPNOTCH MD) 2015
C. p17
D. p24
E. p7

397 Which of the following helminths do not exhibit SCOTT RILEY ONG, BACK-UP
transpulmonary passage? MD (TOP 5 - AUG MIDTERM
A. Ascaris lumbricoides 2014 MED BOARDS; EXAM - FEB
B. Ancylostoma duodenale TOPNOTCH MD) 2015
C. Wuchereria bancrofti
D. Capillaria philippinensis
E. Strongyloides stercoralis

398 Which of the following helminths has eggs described as SIMILAR TO PREVIOUS BOARD EXAM SCOTT RILEY ONG, BACK-UP
barrel-shaped with bipolar plugs, resembling Chinese CONCEPT/PRINCIPLE. Trichuris: Chinese MD (TOP 5 - AUG MIDTERM
lanterns? lantern eggs. Capillaria: peanut-shaped eggs. 2014 MED BOARDS; EXAM - FEB
A. Trichuris trichiura Enterobius: D-shaped eggs. Clonorchis: egg TOPNOTCH MD) 2015
B. Capillaria philippinensis with melon-like ridges and abopercular
C. Enterobius vermicularis protuberance. Schistosoma: eggs with small
D. Clonorchis sinensis hook. Paragonimus: asymmetric eggs with
E. Schistosoma japonicum thin opercular end and thic abopercular end.
399 Which of the following is an intermediate host of SIMILAR TO PREVIOUS BOARD EXAM SCOTT RILEY ONG, BACK-UP
Paragonimus westermani? CONCEPT/PRINCIPLE. The 1st intermediate MD (TOP 5 - AUG MIDTERM
A. Oncomelania hupensis quadrasi host of Paragonimus is the snail Antemelania 2014 MED BOARDS; EXAM - FEB
B. Antemelania asperata asperata. Its 2nd intermediate house is the TOPNOTCH MD) 2015
C. Parafossarulus crab Sundathelphusa philippina.
D. Cyprinid fish
E. None of the above

400 Which of the following helminths can be transmitted via SCOTT RILEY ONG, BACK-UP
inhalation of eggs? MD (TOP 5 - AUG MIDTERM
A. Strongyloides stercoralis 2014 MED BOARDS; EXAM - FEB
B. Loa loa TOPNOTCH MD) 2015
C. Enterobius vermicularis
D. Toxocara canis
E. Ascaris lumbricoides

401 Several Influenza scares in Asia occurred a few years H-stands for Hemagglutinin which binds cell JOSE CARLO DIAGNOSTIC
back, some of its popular strains is Influenza A H1N1. surface receptors to initiate infection of the MASANGKAY III, MD EXAM - AUG
This component of the virus cleaves Sialic Acid to release cell, While N stands for Neuraminidase which (TOP 8 - FEB 2014 2014
progeny virus from the infected cell: cleaves Neuraminidase (Sialic Acid) to release MED BOARDS;
A. Hemagglutinin progeny virus from the infected cell. (SIMILAR TOPNOTCH MD)
B. Neuraminidase TO PREVIOUS BOARD EXAM
C. Hyaluronidase CONCEPT/PRINCIPLE)
D. Sialidase
E. Neuronidase

402 Which of the following bacteria induces virulence by B pertussis toxin increases cAMP by inhibiting JOSE CARLO DIAGNOSTIC
ADP ribosylation? Galpha I; causes whooping cough; inhibits MASANGKAY III, MD EXAM - AUG
A. Bordetella pertussis chemokine receptor, causing lymphocytosis (TOP 8 - FEB 2014 2014
B. Clostridium perfringens MED BOARDS;
C. Bacillus anthracis TOPNOTCH MD)
D. Streptococcus pyogenes
E. All of the above

403 A patient with Hereditary Spherocytosis underwent Splenectomized patients are at high risk for JOSE CARLO DIAGNOSTIC
Splenectomy, which of the following groups of bacteria developing infections/sepsis from MASANGKAY III, MD EXAM - AUG
may most likely cause sepsis if the patient is not encapsulated bacteria which are all positive (TOP 8 - FEB 2014 2014
vaccinated? for the Capsular Swelling test of the Quelling MED BOARDS;
A. Optochin Disk Test (+)bacteria reaction Test TOPNOTCH MD)
B. Novobiocin Sensitivity (+) bacteria
C. Urease (+) bacteria
D. Quellung Reaction (+) bacteria
E. Protease (+) bacteria

TOPNOTCH MEDICAL BOARD PREP MICROBIOLOGY SUPEREXAM Page 49 of 90


For inquiries visit www.topnotchboardprep.com.ph or email us at topnotchmedicalboardprep@gmail.com
TOPNOTCH MEDICAL BOARD PREP MICROBIOLOGY SUPEREXAM
For inquiries visit www.topnotchboardprep.com.ph or email us at topnotchmedicalboardprep@gmail.com
Item QUESTION EXPLANATION AUTHOR TOPNOTCH
# EXAM
404 An unimmunized 2 year-old child came in to your office Catarrhal stage occurs 1-2 weeks after the JOSE CARLO DIAGNOSTIC
and you came up with a diagnosis of Tuspirina, and you infection, it presents with rhinorrhea, malaise, MASANGKAY III, MD EXAM - AUG
knew based on the patient's signs and symptoms that at fever, sneezing, and anorexia it is also the (TOP 8 - FEB 2014 2014
his present stage he is most infectious. At what stage is stage where the patient is MOST infectious. MED BOARDS;
the patient at? TOPNOTCH MD)
A. Incubation
B. Catarrhal
C. Paroxysmal
D. Convalescent
E. Resolved
405 A patient came to you from Pangasinan where his wife Lecithinase is only seen in C. perfringens JOSE CARLO DIAGNOSTIC
told you that he had been infected by a "flesh-eating MASANGKAY III, MD EXAM - AUG
bacteria", you admitted the patient and you submitted a (TOP 8 - FEB 2014 2014
specimen to the Pathology department for MED BOARDS;
Microbiological studies which revealed a "double TOPNOTCH MD)
hemolysis" on Blood agar plate, growth on Egg yolk Agar,
the bacteria is nonmotile with rapidly spreading growth,
and is positive for Lecithinase. What bacteria may be
responsible?
A. Mycobacterium leprae
B. Streptococcus pyogenes
C. Clostridium perfringens
D. Bacillus anthracis
E. Staph aureus
406 A 3 year-old boy was brought to you by his mother due C3 deficience predisposes to recurrent JOSE CARLO DIAGNOSTIC
to recurrent pyogenic infections since he was 6 months pyogenic infections d/t S. aureus, C5-C9 MASANGKAY III, MD EXAM - AUG
old, he has been in and out the hospital since then. What deficiency predisposes to Neisseria infections, (TOP 8 - FEB 2014 2014
Complement may be deficient in this patient? C2 deficiency is usually asymptomatic. MED BOARDS;
A. C1 TOPNOTCH MD)
B. C2
C. C3
D. C4
E. C5-C9

407 After ingestion of almonds, a patient was rushed to your SIMILAR TO PREVIOUS BOARD EXAM JOSE CARLO DIAGNOSTIC
ED due to difficulty of breathing and angioedema, what CONCEPT/PRINCIPLE MASANGKAY III, MD EXAM - AUG
immune mechanism might be occuring in this patient: (TOP 8 - FEB 2014 2014
A. Leukocyte migration causes recruitment of MED BOARDS;
Basophils causing release of IgE and Histamine TOPNOTCH MD)
B. Increased IgE production causes immediate release
of vasoactive amines and other mediators from mast
cells.
C. Antigen-Antibody complexes is created by the
allergen and IgE deposits in multiple sites
D. T-Lymphocytes release IgE from its surface
E. IgE levels in the blood reaches its maximum and
exerts its centrally-acting abilities trigerring the
hypothalamus to induce systemic inflammation.

408 In a patient with Post-Streptococcal Glomerulonephritis SIMILAR TO PREVIOUS BOARD EXAM JOSE CARLO DIAGNOSTIC
which test is most sensitive in documenting an CONCEPT/PRINCIPLE MASANGKAY III, MD EXAM - AUG
antecedent skin infection? (TOP 8 - FEB 2014 2014
A. Anti-Streptolysin O MED BOARDS;
B. C3 titers TOPNOTCH MD)
C. Anti-Streptolysin S
D. Anti-Dnase A
E. Anti-Dnase B

409 A Gram positive bacteria was isolated in a septic The following characteristics of the toxin JOSE CARLO DIAGNOSTIC
immunocompromised patient after ingestion of described is compatible with an endotoxin. MASANGKAY III, MD EXAM - AUG
unpasteurized milk, Bacteriological studies revealed that The only gram positive bacteria with an (TOP 8 - FEB 2014 2014
the toxin from this bacteria is stable at 100 deg C, is a endotoxin is Listeria monocytogenes, it can MED BOARDS;
Lipopolysaccharide, has low antigenicity and a vaccine infect an immunocompromised host by TOPNOTCH MD)
can not be formulated using the toxin itself. Which of the ingesting unpasteurized milk.
following is the probable isolate?
A. Bacillus anthracis
B. Bacillus Cereus
C. Clostridium difficile
D. Clostridium botulinum
E. Listeria monocytogenes

410 A 6 year-old male patient was brought to you with Pneumatocoele in a post-measles pneumonia JOSE CARLO DIAGNOSTIC
severe dyspnea, poor intake, foul-smelling sputum and may be due to Staph aureus. SIMILAR TO MASANGKAY III, MD EXAM - AUG
decreased sensorium, upon your diligent history the PREVIOUS BOARD EXAM (TOP 8 - FEB 2014 2014
patient had an apparent measles infection 2 weeks prior CONCEPT/PRINCIPLE MED BOARDS;
to consult. Chest Xray revealed Pneumatocoele. Which of TOPNOTCH MD)
the following may be the probable cause?
A. Streptococcus pneumonia
B. Haemophilus influenzae
C. Staphylococcus aureus
D. Streptococcus pyogenes
E. Rubeola Virus (reactivation)

TOPNOTCH MEDICAL BOARD PREP MICROBIOLOGY SUPEREXAM Page 50 of 90


For inquiries visit www.topnotchboardprep.com.ph or email us at topnotchmedicalboardprep@gmail.com
TOPNOTCH MEDICAL BOARD PREP MICROBIOLOGY SUPEREXAM
For inquiries visit www.topnotchboardprep.com.ph or email us at topnotchmedicalboardprep@gmail.com
Item QUESTION EXPLANATION AUTHOR TOPNOTCH
# EXAM
411 A 12 month-old male patient was brought to you due to This is a case of Bruton's JOSE CARLO DIAGNOSTIC
severe diarrhea, Fecalysis was done with noted Agammaglobulinemia, where there is a MASANGKAY III, MD EXAM - AUG
abundant fat globules, white blood cells, foul-smelling presence of very low immunoglobulins due to (TOP 8 - FEB 2014 2014
odor and trophozoites with "old-man with glasses-like a virtual absence of B cells due to a tyrosine MED BOARDS;
features", upon your history patient has been kinase mutation, symptoms start at 6 months TOPNOTCH MD)
experiencing recurrent pyogenic bacterial infections, and due to the presence of maternal antibodies
enteroviral infections which all started at 6 months of persisting from birth up to the 6th month of
age. Which of the following maybe deficient in this age. (SIMILAR TO PREVIOUS BOARD EXAM
patient? CONCEPT/PRINCIPLEion)
A. B-Cells
B. C3 Complement
C. T-Cells
D. Both B and T Cells
E. NADPH oxidase activity
412 Which strain of Human Papilloma Virus can cause 1,4-skin and plantar warts; 6, 11- Genital JOSE CARLO DIAGNOSTIC
Carcinoma of the Penis? warts; 16,18,31,33-CA of the cervix penis and MASANGKAY III, MD EXAM - AUG
A. 1 anus (TOP 8 - FEB 2014 2014
B. 4 MED BOARDS;
C. 6 TOPNOTCH MD)
D. 11
E. 31

413 A patient who tested positive for an ELISA test for HBc Antigen can not be serologically JOSE CARLO DIAGNOSTIC
Hepatitis B surface antigen went to your office to determined and can only be detected thru MASANGKAY III, MD EXAM - AUG
undergo a confirmatory test for Hepatitis B and so you liver biopsy. (TOP 8 - FEB 2014 2014
requested a Hepatitis B Profile. Which of the following MED BOARDS;
tests can not be serologically determined? TOPNOTCH MD)
A. HBe Ag
B. HBc Ag
C. Anti-HBe
D. Anti HBc
E. Anti-HBs
414 Plasmodium vivax and Plasmodium ovale are 2 different P. vivax produces 12-24 merozoites while P. JOSE CARLO DIAGNOSTIC
causes of Malaria and are very difficult to distinguish ovale only produces 8 merozoites per cycle. MASANGKAY III, MD EXAM - AUG
from each other, both clinically and diagnostically. (TOP 8 - FEB 2014 2014
Which of the following characteristics may they differ? MED BOARDS;
A. Periodicity TOPNOTCH MD)
B. Preference in the age of RBCs
C. Relapse capability
D. Number of Merozoites
E. Propencity for Cerebral malaria

415 A colleague of yours who just came from Zimbabwe for a This is a case of Chaga's Disease caused by JOSE CARLO DIAGNOSTIC
medical mission had noted fever, periorbital edema, Trypanosoma cruzi, where Nifurtimox is the MASANGKAY III, MD EXAM - AUG
dilated cardiomyopathy, achalasia and drug of choice. (TOP 8 - FEB 2014 2014
hepatosplenomegaly, upon histroy he has been bitten by MED BOARDS;
a reduviid bug while strolling around. What is the drug TOPNOTCH MD)
of choice for the said patient?
A. Melarsoprol
B. Suramin
C. Sulfadiazine
D. Nifurtimox
E. Co-trimoxazole
416 A farmer from Samar, was brought to San Lazaro This is a case of Schistosomiasis by JOSE CARLO DIAGNOSTIC
Hospital due to noted ascites, jaundice, hemorrhoids and Schistosoma japonicum, the diagnostic test of MASANGKAY III, MD EXAM - AUG
esophageal varices, upon history, tiny snails were seen choice is COPT or Circumoval precipitin Test (TOP 8 - FEB 2014 2014
on the streams near their farm. what laboratory lest MED BOARDS;
should you request for? TOPNOTCH MD)
A. Kato-Katz Technique
B. Direct Fecal Smear
C. ELISA
D. Harada-mori Technique
E. COPT
417 An isolate had the following characteristics: Catalase JOSE CARLO DIAGNOSTIC
negative, alpha-hemolysis on blood agar, Optochin MASANGKAY III, MD EXAM - AUG
resistant and Bile Insoluble. What is the probable (TOP 8 - FEB 2014 2014
isolate? MED BOARDS;
A. Staphylococcus aureus TOPNOTCH MD)
B. Staphylococcus saphrophyticus
C. Streptococcus Epidermidis
D. Viridans Streptococci
E. Streptococcus pneumonia
418 Risus sardonicus, Opisthotonus and Trismus are obvious tetanospasmin inhibits release of glycine in JOSE CARLO DIAGNOSTIC
signs if Tetany, what is the pathogenesis for the action of the renshaw cells of the spinal cord which MASANGKAY III, MD EXAM - AUG
teteanospasmin, the toxin of Clostridium tetani? causes continuous muscle contraction hence (TOP 8 - FEB 2014 2014
A. Inhibits GABA in the brain spastic paralysis is noted. MED BOARDS;
B. Inhubits Glycine in the Spinal Cord TOPNOTCH MD)
C. Inhibits Calcium channels directly in the peripheral
nerves
D. Stimulates glutamate secretion in the brain
E. Activates Elongation factor-2 hence inhibiting
protein synthesis

TOPNOTCH MEDICAL BOARD PREP MICROBIOLOGY SUPEREXAM Page 51 of 90


For inquiries visit www.topnotchboardprep.com.ph or email us at topnotchmedicalboardprep@gmail.com
TOPNOTCH MEDICAL BOARD PREP MICROBIOLOGY SUPEREXAM
For inquiries visit www.topnotchboardprep.com.ph or email us at topnotchmedicalboardprep@gmail.com
Item QUESTION EXPLANATION AUTHOR TOPNOTCH
# EXAM
419 In the Philippines, what is the most common cause of E. coli is the most common cause of Neonatal JOSE CARLO DIAGNOSTIC
Neonatal Sepsis? sepsis in the Philippines. Worldwide the most MASANGKAY III, MD EXAM - AUG
A. Streptococcus agalactiae common cause is Strep agalactiae (TOP 8 - FEB 2014 2014
B. Adenovirus MED BOARDS;
C. Escherichia coli TOPNOTCH MD)
D. Listeria monocytogenes
E. Lactobacillus acidophilus

420 In malaria the Anopheles mosquito is a/an? Man is the intermediate host and anopheles JOSE CARLO DIAGNOSTIC
A. Intermediate host mosquito is the definitive host. Man harbors MASANGKAY III, MD EXAM - AUG
B. Definitive host the asexual stage while mosquitoes will (TOP 8 - FEB 2014 2014
C. Paratenic host harbor the sexual stage. MED BOARDS;
D. Reservoir host TOPNOTCH MD)
E. Parasitic host

421 During the 1st week of typhoid fever, the best specimen Blood - 1st week; urine - 2nd week; stool - 3rd WEBSTER ALINDOG, MIDTERM 1
to be utilized for culture is: week; bone marrow - any stage. MD (TOP 3 - FEB EXAM - AUG
A. Blood 2014 MED BOARDS; 2014
B. Urine TOPNOTCH MD)
C. Stool
D. CSF
E. None, no bacilli can be recovered during this period

422 Systemic bacterial infections are notorious for causing Interleukin (IL) 6 is not only an endogenous WEBSTER ALINDOG, MIDTERM 1
elevated fibrinogen, CRP, alpha-2-microglobulin, and pyrogen but also acts to stimulate the liver to MD (TOP 3 - FEB EXAM - AUG
other acute phase reactants. Such may be explained by: synthesize and release the acute phase 2014 MED BOARDS; 2014
A. Elevated IL-1 acting on the hypothalamus reactants - fibrinogen, CRP, microglobulins, TOPNOTCH MD)
B. Elevated IL-3 activating the bone marrow etc. (markers of inflammation).IL-1 is a strong
C. Elevated IL-6 stimulating the liver pyrogen, altering the normal thermal setpoint
D. Elevated IL-8 recruiting macrophages in the hypothalamus. IL-3 activates the
hemoproliferative functions of the BM; IL-8
stimulates the neutrophils to act towards the
site of inflammation.
423 The antimicrobial of choice for Enterococcus infections: The tip here is for you to make a list or table of WEBSTER ALINDOG, MIDTERM 1
A. Trimethoprim-Sulfamethoxazole microorganisms along with their MD (TOP 3 - FEB EXAM - AUG
B. Erythromycin primary/DOC and alternative (just pick the 2014 MED BOARDS; 2014
C. Penicillin second best/appropriate drug). Then do final TOPNOTCH MD)
D. Supportive only (hydration and hygiene) browsing during the break, before the micro
exam. At least 3 questions about the
antibiotics of choice were included in the
previous exam.
424 An anaerobic organism from a necrotic wound of an This is a case of gangrene/myonecrosis of the WEBSTER ALINDOG, MIDTERM 1
extremity demonstrates double hemolysis on blood agar, extremity. C. perfringens elaborates alpha MD (TOP 3 - FEB EXAM - AUG
growth on egg yolk agar, lecithinase (+); non-motile, but toxin, a lecithinase that destroys normal cells 2014 MED BOARDS; 2014
with a rapidly spreading growth pattern on culture. The by cleaving their plasma membranes. Your TOPNOTCH MD)
most likely organism would be: strongest clues here would be the "necrotic
A. Bacteroides fragilis wound," "anaerobic," "double hemolysis on
B. Bacillus anthracis BAP," "non-motility," and "lecithinase (+)." If
C. Escherichia coli you cannot recall your buzz word that will
D. Clostridium perfringens immediately lead you to the right answer, like
in this case "lecithinase (+)" you can do
differential starting from the basic
characterization of bacteria like their Gram
reaction, oxygen requirement, sugar
metabolism, culture growth pattern, etc and
then narrow in until you come up with a
sound guess.
425 The E. coli pathotype that causes hemorrhagic colitis and In the laboratory, aside from serotyping, we WEBSTER ALINDOG, MIDTERM 1
hemolytic uremic syndrome can be characterized by all identify the EHEC strain by its inability to MD (TOP 3 - FEB EXAM - AUG
of the following except: ferment sorbitol in enriched medium. 2014 MED BOARDS; 2014
A. It has the equivalent serotype O157:H7. Serotyping utilizes the antigens K (capsular), TOPNOTCH MD)
B. It is the only strain that does not ferment sorbitol. H (flagellar), O (somatic), and Vi (virulent).
C. It is microangiopathic.
D. It is usually not treated with antibiotics since this
may aggravate the infection.
E. None of these.
426 The definitive diagnosis for HIV infection is carried out The -blots: In Western blot, the antigen of WEBSTER ALINDOG, MIDTERM 1
through this process by which a color reaction reveals interest is a protein; in Northern blot, it is an MD (TOP 3 - FEB EXAM - AUG
the presence of antibodies in the serum after reacting to RNA; and in Southern blot, it is a DNA. PCR is a 2014 MED BOARDS; 2014
viral proteins that are displayed by acrylamide gel rapid but expensive (in the Phils.) TOPNOTCH MD)
electrophoresis and transferred to nitrocellulose: confirmatory test in many infectious diseases,
A. Polymerase chain reaction especially the unconventional types. It
B. Western blot amplifies a single or a few copies of a piece of
C. Enzyme-linked immunosorbent assay DNA across several orders of magnitude,
D. Serologic gel electrophoresis generating thousands to millions of copies of a
particular DNA sequence making specific
identification possible.

TOPNOTCH MEDICAL BOARD PREP MICROBIOLOGY SUPEREXAM Page 52 of 90


For inquiries visit www.topnotchboardprep.com.ph or email us at topnotchmedicalboardprep@gmail.com
TOPNOTCH MEDICAL BOARD PREP MICROBIOLOGY SUPEREXAM
For inquiries visit www.topnotchboardprep.com.ph or email us at topnotchmedicalboardprep@gmail.com
Item QUESTION EXPLANATION AUTHOR TOPNOTCH
# EXAM
427 All of the following correctly describe the Histologic changes in the liver mentioned in WEBSTER ALINDOG, MIDTERM 1
clinicopathologic features of Leptospirosis except: the item are not specific to leptospirosis with MD (TOP 3 - FEB EXAM - AUG
A. After its entry to the bloodstream, the organism hepatic involvement. They are also seen in 2014 MED BOARDS; 2014
disseminates widely into the CNS and the eyes. viral hepatitis and non-alcoholic TOPNOTCH MD)
B. Histologic changes in the liver - ballooning steatohepatitis, among others.
degeneration, acidophilic bodies and cholestasis - are
specific to Leptospirosis
C. The mechanisms for endothelial, hepatic, and renal
damages are unknown but may be immune-mediated.
D. None of the above.

428 The known virulence factor of M. tuberculosis that Sulfatide is also an exported repetitive WEBSTER ALINDOG, MIDTERM 1
inhibits fusion of secondary lysosomes with bacilli- protein. Cord factor on the other hand is the MD (TOP 3 - FEB EXAM - AUG
containing phagosomes within a macrophage, possibly most important virulence factor that 2014 MED BOARDS; 2014
promoting intracellular survival of the organism is: prevents leukocyte migration. It is made up of TOPNOTCH MD)
A. Sulfatide glycolipid molecules that influence the long
B. Cord factor and slender formation of MTB bacilli.
C. Tuberculin surface protein Tuberculin surface protein is responsible for
D. Murein eliciting the delayed hypersensitivity in TB.
Murein simply refers to the peptidoglycan
found in bacterial cell walls.
429 A 32-year old G5P4 mother, who admits to be Give special concentration on OB-GYN WEBSTER ALINDOG, MIDTERM 1
promiscuous, was noted to have herpetic lesions in her infectious. Review complications, and MD (TOP 3 - FEB EXAM - AUG
genital area. All of the following principles can guide immunizations and treatment (indications) of 2014 MED BOARDS; 2014
your management of this patient except for: infectious conditions associated with TOPNOTCH MD)
A. Women with a history of recurrent genital herpes pregnancy (varicella, measles, hepatitis,
can deliver vaginally if active lesions are not present but listeriosis, GBS).
careful monitoring of their newborns is warranted.
B. Despite anti-viral therapy, disseminated infection is
often fatal.
C. Neonatal herpes infection may be silent for up to
several days before disease becomes apparent.
D. None of the above.

430 In prognosticating HIV infection, what is the best CD 4 count is used only to assess the immune WEBSTER ALINDOG, MIDTERM 1
laboratory parameter to be used? status of the patient and not really for MD (TOP 3 - FEB EXAM - AUG
A. Viral load detection using PCR prognosticationg. 2014 MED BOARDS; 2014
B. CD4 count using cytometry TOPNOTCH MD)
C. Antibody quantification using ELISA
D. Antibody quantification using gel electrophoresis

431 True of antibody-antigen interaction: If an antigen has many determinants WEBSTER ALINDOG, MIDTERM 1
A. The size of the antigen-antibody complex is (multivalent) it can form crosslinks with the MD (TOP 3 - FEB EXAM - AUG
determined by the valence of the antigen and the relative corresponding antibodies (principle of 2014 MED BOARDS; 2014
concentration of the antigen and antibody. precipitation), conversely, if it is a univalent it TOPNOTCH MD)
B. The affinity of an antibody molecule reflects the cannot be crosslinked. Moreover, the
tightness of the fit of an antigenic determinant to a single reversibility of the interaction is a result of
antigen-binding site and it is independent of the number many relatively weak noncovalent forces,
of antigenic sites. including hydrophobic and hydrogen bonds,
C. The binding of antibody to antigen is reversible. van der Waals forces and ionic interactions.
D. All of the above.

432 An immunocompromised patient with a CD4 count of This is a case of Cryptosporidium parvum WEBSTER ALINDOG, MIDTERM 1
150 complains of a long-standing non-bloody diarrhea. infection. It is considered an opportunistic MD (TOP 3 - FEB EXAM - AUG
Upon investigation, thick-walled cysts that are acid fast protozoa with thick-walled oocysts that are 2014 MED BOARDS; 2014
were recovered from his stools. This condition of the acid-fast and commonly (but not exclusively) TOPNOTCH MD)
patient can improve when treated with: infects patients with CD4 count less than 200.
A. DEC It is treated with Nitazoxanide, and
B. Metronidazole alternatively with Azithromycin.
C. Ivermectin
D. Nitazoxanide

433 The primary antibiotic of choice for anthrax is: WEBSTER ALINDOG, MIDTERM 1
A. Penicillin G MD (TOP 3 - FEB EXAM - AUG
B. Ciprofloxacin 2014 MED BOARDS; 2014
C. Ceftriaxone TOPNOTCH MD)
D. Erythromycin

TOPNOTCH MEDICAL BOARD PREP MICROBIOLOGY SUPEREXAM Page 53 of 90


For inquiries visit www.topnotchboardprep.com.ph or email us at topnotchmedicalboardprep@gmail.com
TOPNOTCH MEDICAL BOARD PREP MICROBIOLOGY SUPEREXAM
For inquiries visit www.topnotchboardprep.com.ph or email us at topnotchmedicalboardprep@gmail.com
Item QUESTION EXPLANATION AUTHOR TOPNOTCH
# EXAM
434 A broad-based budding yeast was isolated from a There are only 4 primary systemic mycoses WEBSTER ALINDOG, MIDTERM 1
bronchoalveolar lavage sample of an elderly that we should know --- coccidiodomycosis, MD (TOP 3 - FEB EXAM - AUG
immunocompromised patient. To prevent systemic histoplasmosis, paracoccidiodomycosis, and 2014 MED BOARDS; 2014
dissemination of the infection, the patient should be blastomycosis. Among which, it is the TOPNOTCH MD)
treated with: Blastomyces dermatitidis that is characterized
A. Griseofulvin with a broad-based bud in its yeast. It is also
B. Potassium iodide-containing solution the most common cause of fungal pneumonia.
C. Amphotericin B Amphotericin B (a polyene) is the drug of
D. Cotrimoxazole choice in systemic mycosis. It binds with
ergosterol and creates a leak channel that lead
to fungal cell death. Fluconazole, on the other
hand, is the choice to prevent meningeal
involvement because it can penetrate the
blood-brain barrier.
435 This autoantibody is expected to be elevated in Anti-TSH/TSI - autoantibodies correlated with WEBSTER ALINDOG, MIDTERM 1
Hashimoto's thyroiditis, the most common cause of Graves disease (hyperthyroid state) vs anti- MD (TOP 3 - FEB EXAM - AUG
hypothyroidism in the world: TPO, Hashimoto's (hypothyroid). Anti- 2014 MED BOARDS; 2014
A. Anti-thyroid peroxidase glutamate decarboxylase is associated with TOPNOTCH MD)
B. Thyroid stimulating immunoglobulin autoimmune type of DM 1 (1a).
C. Anti-thyroid stimulating hormone
D. Anti-glutamate decarboxylase

436 Of the following, the simplest and most readily available WEBSTER ALINDOG, MIDTERM 1
test by which one can differentiate Staphylococcus from MD (TOP 3 - FEB EXAM - AUG
Streptococcus is: 2014 MED BOARDS; 2014
A. Coagulase TOPNOTCH MD)
B. Catalase
C. Bacitracin sensitivity
D. Optochin sensitivity

437 Small, motile, non-spore-forming, curved, The organism is Campylobacter jejuni - the WEBSTER ALINDOG, MIDTERM 1
microaerophilic Gram negative bacilli that is the most most common cause of bacterial MD (TOP 3 - FEB EXAM - AUG
recognized antecedent cause of GBS, at least in Western gastroenteritis from eating undercooked 2014 MED BOARDS; 2014
countries is: chicken. It causes histologic damage to the TOPNOTCH MD)
A. Escherichia coli mucosal surfaces of jejunum hence "C. jejuni"
B. Pasteurella multocida and is treated with erythromycin (severe
C. Helicobacter pylori cases).
D. Campylobacter jejuni

438 The detectable serologic marker during the window The window phase/period is also known as WEBSTER ALINDOG, MIDTERM 1
phase of hepatitis B infection: equivalence happens when both serological MD (TOP 3 - FEB EXAM - AUG
A. Anti-HBc IgM markers HBsAg and anti-HBs are negative 2014 MED BOARDS; 2014
B. Anti-Hbe IgM (which is due to the fact that, although there TOPNOTCH MD)
C. Anti-HBsAg IgG are anti-HBs antibodies present, they are
D. HBsAg actively bound to the HBsAg). We can usually
use anti-HBcAg IgM instead at this period.

439 What do you call the recurrence of disease due to P. WEBSTER ALINDOG, MIDTERM 1
falciparum or P. malariae, coming from an increase in MD (TOP 3 - FEB EXAM - AUG
numbers of persisting blood stage forms to clinically 2014 MED BOARDS; 2014
detectable levels, and not from persisting liver stage TOPNOTCH MD)
forms?
A. Relapse
B. Recurrence
C. Recrudescence
D. Repeat

440 A blood culture taken from a 32-year old G2P1 on her Other possible buzz words for Listeria WEBSTER ALINDOG, MIDTERM 1
3rd trimester experiencing flu-like illness reveals monocytogenes: "tumbling" motility and cold MD (TOP 3 - FEB EXAM - AUG
colonies that are small, grayish blue with beta hemolysis enhancement. They are acquired from 2014 MED BOARDS; 2014
on blood agar plate; Gram positive; catalase (+); motile unpasteurized milk products and transmitted TOPNOTCH MD)
at room temperature and produce acids from glucose, to the baby either transplacentally or during
trehalose and salicin. The organism involved is: delivery by direct contact. Ampicillin is the
A. Brucella drug of choice.
B. Listeria
C. Bacillus
D. Staphylococcus

441 A 45 year-old post kidney transplant patient developed Aspergillus fumigatus is the most frequent JULIET KRISTINE MIDTERM 2
fever and pneumonia with marked neutropenia. cause of invasive fungal infection in EVANGELISTA, MD EXAM - AUG
Bronchial washing show dichotomously branching immunosuppressed individuals. It causes (TOP 9 - FEB 2014 2014
filamentous fungi. The fungi involved is: invasive infection in the lung, chronic MED BOARDS;
A. Cryptococcus pulmonary infections, allergic TOPNOTCH MD)
B. Nocardia bronchopulmonary aspergillosis.
C. Candida
D. Aspergillus
E. Blastomyces

TOPNOTCH MEDICAL BOARD PREP MICROBIOLOGY SUPEREXAM Page 54 of 90


For inquiries visit www.topnotchboardprep.com.ph or email us at topnotchmedicalboardprep@gmail.com
TOPNOTCH MEDICAL BOARD PREP MICROBIOLOGY SUPEREXAM
For inquiries visit www.topnotchboardprep.com.ph or email us at topnotchmedicalboardprep@gmail.com
Item QUESTION EXPLANATION AUTHOR TOPNOTCH
# EXAM
442 A 2 year-old girl has been having trouble sleeping Pinworm infection is caused by a small, thin, JULIET KRISTINE MIDTERM 2
because of intense perianal itching. Her pediatrician gets white roundworm called Enterobius EVANGELISTA, MD EXAM - AUG
a perianal specimen that night with a scotch tape. The vermicularis. Although pinworom infection (TOP 9 - FEB 2014 2014
parasite causing the symptom is: can affect all people, it most commonly occurs MED BOARDS;
A. Trichuris trichiura among children, institutionalized persons, and TOPNOTCH MD)
B. Enterobius vermicularis household members of persons with pinworm
C. Ancylostoma duodenale infection. A person infected with pinworm is
D. Ascaris lumbricoides often asymptomatic, but itching around the
E. Trichinella spiralis anus is a common symptom. Diagnosis of
pinworm can be done by touching the perianal
skin with transparent tape to collect possible
pinworm eggs around the anus first thing in
the morning.
443 The part of the body where Salmonella typhi resides For a subset of infected individuals, about 3- JULIET KRISTINE MIDTERM 2
making a person chronic carrier: 5%, Salmonella typhi colonizes the gall EVANGELISTA, MD EXAM - AUG
A. Small intestine bladder and remains there long after (TOP 9 - FEB 2014 2014
B. Appendix symptoms subside, serving as a reservoir for MED BOARDS;
C. Blood stream the further spread of the disease. TOPNOTCH MD)
D. Urinary bladder
E. Gall bladder

444 A 25 year-old breastfeeding woman developed redness Mastitis is the inflammation of breast tissue. JULIET KRISTINE MIDTERM 2
and swelling of right breast. On examination, a fluctuant Staphylococcus aureus is the most common EVANGELISTA, MD EXAM - AUG
mass is found. The most probable diagnosis is: etiological organism responsible. S. aureus (TOP 9 - FEB 2014 2014
A. Mastitis caused by S. epidermidis bacteria produce toxins that destroy cell MED BOARDS;
B. Mastitis caused by S. aureus membranes and can directly damage milk- TOPNOTCH MD)
C. Inflammatory breast carcinoma producing tissue. Early stages of mastitis can
D. Intraductal papilloma present with local pain, redness, swelling, and
E. Peau D' Orange Breast warmth, later stages also present with
systemic symptoms like fever and flu-like
symptoms and in rare cases an abscess can
develop.
445 The intermediate host of Schistosoma japonicum in the The intermediate host of Schistosoma JULIET KRISTINE MIDTERM 2
Philippines is: japonicum in the Philippines is the snail EVANGELISTA, MD EXAM - AUG
A. Oncomelania quadrasi Oncomelania quadrasi. (TOP 9 - FEB 2014 2014
B. Pila luzonica MED BOARDS;
C. Thiara snail TOPNOTCH MD)
D. Sundathelpusa philippina
E. Segmentina hemisphaerula

446 A 19 year-old female student nurse has splenomegaly, Infectious mononucleosis is an infectious, JULIET KRISTINE MIDTERM 2
elevated WBC with atypical lymphocytes and (+) widespread viral disease caused by the EVANGELISTA, MD EXAM - AUG
Heterophil antibodies. She probably has: Epstein–Barr virus (EBV), one type of herpes (TOP 9 - FEB 2014 2014
A. Mumps virus. Especially in adolescents and young MED BOARDS;
B. Tuberculosis adults, the disease is characterized by fever, TOPNOTCH MD)
C. Lymphocytic choriomeningitis sore throat and fatigue, along with several
D. Infectious mononucleosis other possible signs and symptoms. The most
E. Rubella commonly used diagnostic criterion is the
presence of 50% lymphocytes with at least
10% atypical lymphocytes (large, irregular
nuclei).
447 Medium used for the culture of Bordetella pertussis is: Regan-Lowe medium (containing charcoal, JULIET KRISTINE MIDTERM 2
A. Lowenstein-Jensen medium blood, and antibiotic) has replaced Bordet- EVANGELISTA, MD EXAM - AUG
B. Loefler's medium Gengou medium as the medium of choice for (TOP 9 - FEB 2014 2014
C. Regan Lowe Medium routine Bordetella pertussis incubation. MED BOARDS;
D. MacConkey medium TOPNOTCH MD)
E. Xylose-Lysine-Deoxycholate medium

448 A 3 year-old child was brought at the out-patient clinic Measles, also known as morbilli, or rubeola is JULIET KRISTINE MIDTERM 2
with symptoms of coryza, conjunctivitis, low-grade fever an infection of the respiratory system, EVANGELISTA, MD EXAM - AUG
and Koplik spots. The causative agent of this disease immune system and skin caused by a virus, (TOP 9 - FEB 2014 2014
belongs to which group of viruses: specifically a paramyxovirus of the genus MED BOARDS;
A. adenovirus Morbillivirus. TOPNOTCH MD)
B. paramyxovirus
C. orthomyxovirus
D. picornavirus
E. herpesvirus

449 A 28 year-old man with a unilateral, painful, tender Lymphogranuloma venereum is primarily a JULIET KRISTINE MIDTERM 2
inguinal lymphadenopathy begins to produce a purulent sexually-transmitted infection of lymphatics EVANGELISTA, MD EXAM - AUG
urethral discharge. The most probable diagnosis is: and lymph nodes. Chlamydia trachomatis is (TOP 9 - FEB 2014 2014
A. Primary syphilis the bacterium responsible for LGV. Clinical MED BOARDS;
B. Chancroid manifestation of LGV among males whose TOPNOTCH MD)
C. Lymphogranuloma venereum primary exposure was genital is unilateral
D. Herpes simplex type 2 lymphadenitis and lymphangitis, often with
E. Granuloma inguinale tender inguinal and/or femoral
lymphadenopathy.
450 Many infections caused by bacteria are initiated by A pilus is a hairlike appendage found on many JULIET KRISTINE MIDTERM 2
bacterial adherence to mucous membranes. The bacteria used to attach to surfaces. EVANGELISTA, MD EXAM - AUG
bacterial structure that mediates adherence is the: Attachment of bacteria to host surfaces is (TOP 9 - FEB 2014 2014
A. Pilus required for colonization during infection or MED BOARDS;
B. Peptidoglycan to initiate formation of a biofilm. TOPNOTCH MD)
C. Flagellum
D. Cilia
E. Endotoxin

TOPNOTCH MEDICAL BOARD PREP MICROBIOLOGY SUPEREXAM Page 55 of 90


For inquiries visit www.topnotchboardprep.com.ph or email us at topnotchmedicalboardprep@gmail.com
TOPNOTCH MEDICAL BOARD PREP MICROBIOLOGY SUPEREXAM
For inquiries visit www.topnotchboardprep.com.ph or email us at topnotchmedicalboardprep@gmail.com
Item QUESTION EXPLANATION AUTHOR TOPNOTCH
# EXAM
451 Septicemic shock from gram negative rods is triggered Many different infections disseminate via the JULIET KRISTINE MIDTERM 2
by: bloodstream, causing systemic febrile EVANGELISTA, MD EXAM - AUG
A. Endotoxin symptoms and septic shock. Endotoxin or (TOP 9 - FEB 2014 2014
B. Exotoxin lipopolysaccharides located in the outer MED BOARDS;
C. Opsonin membrane of the gram negative bacteria TOPNOTCH MD)
D. Exaggerated immune response induces overproduction of cytokines, activates
E. Immune complexes the complement cascade and coagulation
cascade resulting in DIC.
452 Swarming growth is a characteristic of: Proteus mirabilis swarming behavior is JULIET KRISTINE MIDTERM 2
A. Shigella characterized by the development of EVANGELISTA, MD EXAM - AUG
B. Staphylococci concentric rings of growth that are formed as (TOP 9 - FEB 2014 2014
C. Haemophilus cyclic events of swarmer cell differentiation, MED BOARDS;
D. Streptococci swarming migration, and cellular TOPNOTCH MD)
E. Proteus differentiation are repeated during colony
translocation across a surface. This cycle
produces the bull’s eye colony often
associated with cultures of P. mirabilis.
453 The simple staining procedure used in the laboratory to Methylene stain is used in the laboratory to JULIET KRISTINE MIDTERM 2
demonstrate the metachromatic granules of demonstrate the metachromatic granules of EVANGELISTA, MD EXAM - AUG
Corynebacterium diphtheria is: Corynebacterium diphtheria. Safranin is used (TOP 9 - FEB 2014 2014
A. Safranin stain as a counterstain in some staining protocols, MED BOARDS;
B. Methylene stain colouring all cell nuclei red. This is the classic TOPNOTCH MD)
C. Carbol-fuchsin stain counterstain in both Gram stains. Carbol-
D. Crystal violet stain fuchsin is a mixture of phenol and basic
E. India ink stain fuchsin, used in the staining of mycobacteria
as it has an affinity for the mycolic acids found
in their cell membranes. Crystal violet or
gentian violet is the dye is used as a
histological stain and in Gram's method of
classifying bacteria. A common application of
this staining with india ink in the clinical
microbiology laboratory is to confirm the
morphology of the encapsulated yeast
Cryptococcus spp. which cause cryptococcal
meningitis.
454 It is a gram negative, nonmotile bacillus that is most Legionella pneumophila causes Legionnaires' JULIET KRISTINE MIDTERM 2
commonly isolated from cooling towers is: disease. It is contracted by breathing in drops EVANGELISTA, MD EXAM - AUG
A. Haemophilus influenzae of contaminated water. Outbreaks are often (TOP 9 - FEB 2014 2014
B. Legionella pneumophila reported in hotels, cruise ships, and office MED BOARDS;
C. Campylobacter jejuni buildings, where people are exposed to TOPNOTCH MD)
D. Yersinia enterocolitica contaminated droplets from cooling towers
E. Klebsiella pneumoniae and evaporative condensers.

455 It is the predominant facultative organism of the large Escherichia coli is the predominant facultative JULIET KRISTINE MIDTERM 2
intestines of man and is used as an index of fecal organism of the large intestines of man and is EVANGELISTA, MD EXAM - AUG
contamination of a water sample is: used as an index of fecal contamination of a (TOP 9 - FEB 2014 2014
A. Escherichia coli water sample. MED BOARDS;
B. Proteus mirabilis TOPNOTCH MD)
C. Bacteroides fragilis
D. Balatidium coli
E. Clostridium perfringens

456 The class of immunoglobulins important in protecting The class of immunoglobulins important in JULIET KRISTINE MIDTERM 2
the mucosal surface of respiratory, intestinal and protectng the mucosal surface of respiratory, EVANGELISTA, MD EXAM - AUG
genitourinary tracts from pathogenic microorganisms is: intestinal and genitourinary tracts from (TOP 9 - FEB 2014 2014
A. IgM pathogenic microorganisms is IgA. IgM is the MED BOARDS;
B. IgG marker of acute infection, IgG is for chronic TOPNOTCH MD)
C. IgA infection and immunity. IgE is increased in
D. IgE allergic and paracitic infections.
E. IgD

457 The most common etiologic agent of post-transfusion Hepatitis C is the most common etiologic JULIET KRISTINE MIDTERM 2
hepatitis is: agent of posttransfusion hepatitis. EVANGELISTA, MD EXAM - AUG
A. Hepatitis B (TOP 9 - FEB 2014 2014
B. Hepatitis C MED BOARDS;
C. Hepatitis D TOPNOTCH MD)
D. Hepatitis A
E. Hepatitis E

458 Viral nucleic acid may be: Viral nucleic acid may be single-stranded or JULIET KRISTINE MIDTERM 2
A. Single stranded or double stranded double-stranded, RNA or DNA but not both, EVANGELISTA, MD EXAM - AUG
B. RNA or DNA but not both linear or circular. (TOP 9 - FEB 2014 2014
C. Linear or circular MED BOARDS;
D. A and C only TOPNOTCH MD)
E. All of the above

TOPNOTCH MEDICAL BOARD PREP MICROBIOLOGY SUPEREXAM Page 56 of 90


For inquiries visit www.topnotchboardprep.com.ph or email us at topnotchmedicalboardprep@gmail.com
TOPNOTCH MEDICAL BOARD PREP MICROBIOLOGY SUPEREXAM
For inquiries visit www.topnotchboardprep.com.ph or email us at topnotchmedicalboardprep@gmail.com
Item QUESTION EXPLANATION AUTHOR TOPNOTCH
# EXAM
459 A 30 year-old male HIV positive patient was seen at the Cryptococcus neoformans is an oval yeast JULIET KRISTINE MIDTERM 2
Emergency Room due to headache and altered level of with narrow-based bud surrounded by a wide EVANGELISTA, MD EXAM - AUG
consciousness. Findings are compatible with meningitis polysaccharide capsule seen in India ink (TOP 9 - FEB 2014 2014
and CSF showed oval budding on india ink. The most which causes meningitis and encephalitis in MED BOARDS;
probable causative organism is: HIV patients. TOPNOTCH MD)
A. Cryptococcus
B. Toxoplasma
C. Histoplasma
D. Coccidiodes
E. Cryptosporidium
460 The symmetric protein shell that encloses the viral The symmetric protein shell that encloses the JULIET KRISTINE MIDTERM 2
nucleic acid genome is the: viral nucleic acid genome is the capsid. EVANGELISTA, MD EXAM - AUG
A. nucleocapsid Nucleocapsid is nucleic acid genome + capsid. (TOP 9 - FEB 2014 2014
B. capsomer MED BOARDS;
C. capsid TOPNOTCH MD)
D. virion
E. Matrix

461 Cercaria is the infective stage of: Cercaria is the infective stage of all LUISA SARANILLO, BACK-UP
A. Schistosoma japonicum Schistosoma. MD (TOP 6 - FEB MIDTERM
B. Schistosoma mansoni 2014 MED BOARDS; EXAM AUG
C. Schistosoma haematobium TOPNOTCH MD) 2014 - FOR
D. All of the above INCLUSION
E. A and B only IN THE
SAMPLEX

462 What is the intermediate host of Schistosoma Oncomelania quadrasi for Schistosoma, while LUISA SARANILLO, BACK-UP
japonicum? Sundathelphusa philippina and Antemelania MD (TOP 6 - FEB MIDTERM
A. Oncomelania quadrasi asperata for Paragonimus. 2014 MED BOARDS; EXAM AUG
B. Sundathelphusa philippina TOPNOTCH MD) 2014 - FOR
C. Aedes Albopictus INCLUSION
D. Antemelania asperata IN THE
E. None of the choices SAMPLEX

463 The following are human prion diseases except: All of the choices are human prion disease LUISA SARANILLO, BACK-UP
A. Creutzfeldt - Jakob disease except scrapie which is an animal prion MD (TOP 6 - FEB MIDTERM
B. Kuru disease. 2014 MED BOARDS; EXAM AUG
C. Gerstmann-Straussler-Scheinker syndrome TOPNOTCH MD) 2014 - FOR
D. Fatal familial insomnia INCLUSION
E. Scrapie IN THE
SAMPLEX

464 Teichoic acid in the bacterial cell wall is present in a Teichoic acid is present only in cell wall of LUISA SARANILLO, BACK-UP
bacteria with: gram positive bacteria. In gram negative, the MD (TOP 6 - FEB MIDTERM
A. Gram positive only unique characteristic feature of cell wall is the 2014 MED BOARDS; EXAM AUG
B. Gram negative only presence of outer membrane or TOPNOTCH MD) 2014 - FOR
C. Gram positive and gram negative lipopolysaccharide. INCLUSION
D. Bacteria wihout cell wall IN THE
E. Bacteriophage SAMPLEX

465 In complement system, what pathway is activated by There are only 2 pathways in the complement LUISA SARANILLO, BACK-UP
bacteria and other bacterial products? system. Alternative pathway and classical MD (TOP 6 - FEB MIDTERM
A. Alternative pathway pathway. Alternative is activated by bacteria 2014 MED BOARDS; EXAM AUG
B. Classical pathway and bacterial products, while the classical TOPNOTCH MD) 2014 - FOR
C. neither pathway is activated by the complement INCLUSION
D. both products itself. IN THE
E. Common pathway SAMPLEX

466 Stool exam of a 4 year old child revealed a double barrel- The child has Trichuris trichiura, a soil LUISA SARANILLO, BACK-UP
shaped eggs with bipolar plugs. Which statement is transmitted nematode with no MD (TOP 6 - FEB MIDTERM
correct? transpulmonary phase and does not cause 2014 MED BOARDS; EXAM AUG
A. It has no transpulmonary phase significant anemia. The drug of choice for TOPNOTCH MD) 2014 - FOR
B. It causes significant anemia trichuriasis is mebendazole. The blood- INCLUSION
C. The drug of choice for this parasite is thiabendazole sucking nematode is the hookworm which IN THE
D. It is a blood sucking nematode causes significant anemia. SAMPLEX
E. all of the choices

467 What is the enzyme used for bacterial invasion to Collagenase or hyaluronidase facilitates the LUISA SARANILLO, BACK-UP
facilitate spread through subcutaneous tissue? spread through subcutaneous tissue; MD (TOP 6 - FEB MIDTERM
A. Coagulase coagulase accelerates fibrin clot formation; 2014 MED BOARDS; EXAM AUG
B. Collagenase IgA protease allows adherence to mucous TOPNOTCH MD) 2014 - FOR
C. IgA protease membranes; leukocidin destroys neutrophils INCLUSION
D. leucocidin and macrophages. IN THE
E. None of the choices SAMPLEX

468 A 9 month old male infant had recurrent episodes of It is a case of X-linked agammaglobulinemia LUISA SARANILLO, BACK-UP
pyogenic bacterial infections and recurrent history of which is treated with pooled gamma globulin. MD (TOP 6 - FEB MIDTERM
diarrhea. Bone marrow examination shows virtual 2014 MED BOARDS; EXAM AUG
absence of B cells. What is the recommended treatment? TOPNOTCH MD) 2014 - FOR
A. Do not give gamma globulin cause it will cause INCLUSION
anaphylaxis IN THE
B. transplant of thymus SAMPLEX
C. suuportive management
D. pooled gamma globulin
E. antibiotics

TOPNOTCH MEDICAL BOARD PREP MICROBIOLOGY SUPEREXAM Page 57 of 90


For inquiries visit www.topnotchboardprep.com.ph or email us at topnotchmedicalboardprep@gmail.com
TOPNOTCH MEDICAL BOARD PREP MICROBIOLOGY SUPEREXAM
For inquiries visit www.topnotchboardprep.com.ph or email us at topnotchmedicalboardprep@gmail.com
Item QUESTION EXPLANATION AUTHOR TOPNOTCH
# EXAM
469 What is the virulence factor of Staphylococcus aureus All of the choices are virulence factors of S. LUISA SARANILLO, BACK-UP
that prevents complement activation? aureus. Protein A prevents complement MD (TOP 6 - FEB MIDTERM
A. Protein A activation; coagulase forms a fibrin clot; 2014 MED BOARDS; EXAM AUG
B. coagulase leukocidin destroys WBC; catalase detoxifies TOPNOTCH MD) 2014 - FOR
C. leukocidin hydrogen peroxide; hemolysin is toxic to INCLUSION
D. catalase hematopoietic cells. IN THE
E. Hemolysin SAMPLEX

470 Staphylococcus albus is the old name of: Staphylococcus albus is the old name of S. LUISA SARANILLO, BACK-UP
A. Staphylococcus aureus epidermides. MD (TOP 6 - FEB MIDTERM
B. Staphylococcus saprophyticus 2014 MED BOARDS; EXAM AUG
C. Staphylococcus epidermides TOPNOTCH MD) 2014 - FOR
D. Pneumococcus INCLUSION
E. None of the choices IN THE
SAMPLEX

471 A term neonate delivered at home had fever and poor Strep agalactiae or GBS causes UTI in pregnant LUISA SARANILLO, BACK-UP
suck. Upon review of maternal history, the mother said women, and is the most common cause of MD (TOP 6 - FEB MIDTERM
that she had urinary tract infection at 8 months AOG and neonatal sepsis. Other causes of neonatal 2014 MED BOARDS; EXAM AUG
did not complete the prescribed medication. What is the sepsis are: E. coli and L. monocytogenes. TOPNOTCH MD) 2014 - FOR
most likely etiologic agent? INCLUSION
A. Escherichia coli IN THE
B. Streptococcus agalactiae SAMPLEX
C. Listeria monocytogenes
D. Staphylococcus saprophyticus
E. Staphylococcus aureus
472 A 40 year old male farmer came in for cosultation due a It is a characteristic lesion of cutaneous LUISA SARANILLO, BACK-UP
skin lesion with a characteristic malignant pustule with anthrax. The drug of choice is ciprofloxacin. MD (TOP 6 - FEB MIDTERM
central necrosis and subsequent eschar formation. What 2014 MED BOARDS; EXAM AUG
is the drug of choice? TOPNOTCH MD) 2014 - FOR
A. Cloxacillin INCLUSION
B. clindamycin IN THE
C. Vancomycin SAMPLEX
D. clarithromycin
E. ciprofloxacin

473 A 65 year old female patient in the ICU receiving several This is a case of pseudomembranous colitis. LUISA SARANILLO, BACK-UP
antibiotics for 10days developed a nonbloody diarrhea. The cause is the overgrowth of C. difficile due MD (TOP 6 - FEB MIDTERM
What is/are the causative antibiotic/s? to the use of antibiotic such as clindamycin, 2014 MED BOARDS; EXAM AUG
A. Ampicillin ampicillin, 2nd and 3rd generation TOPNOTCH MD) 2014 - FOR
B. Clindamycin cephalosporins. INCLUSION
C. 2nd generation cephalosporins IN THE
D. 3rd generation cephalosporins SAMPLEX
E. all of the choices

474 A 15 year old boy had a 2 week fever and headache, N. meningitis is the most common cause of LUISA SARANILLO, BACK-UP
which later associated with stiff neck. Spinal fluid meningitis among aged 2-18 years. C. MD (TOP 6 - FEB MIDTERM
examination revealed high protein, low glucose, and neoformans is a common in patient with AIDS. 2014 MED BOARDS; EXAM AUG
WBC count of 1000 composed mainly of neutrophils. E. coli is common in neonate. TOPNOTCH MD) 2014 - FOR
What is the most likely etiologic agent? INCLUSION
A. Cryptococcus neoformans IN THE
B. Mycobacterium tuberculosis SAMPLEX
C. Neisseria meningitidis
D. Escherichia coli
475 In the natural history of tuspirina, bacteremia is highest In tuspirina or pertussis, bacteremia starts to LUISA SARANILLO, BACK-UP
in what phase? rise in incubation period, highest in catarrhal MD (TOP 6 - FEB MIDTERM
A. incubation phase, and starts to decline again in 2014 MED BOARDS; EXAM AUG
B. catarrhal paroxysmal to nadir in convalescent phase. TOPNOTCH MD) 2014 - FOR
C. paroxysmal INCLUSION
D. convalescent IN THE
E. A and B SAMPLEX

476 Pontiac fever is cause by: Borrelia burgdorferi- relapsing fever; l. LUISA SARANILLO, BACK-UP
A. Borrelia burgdorferi pneumophila - pontiac fever; S. japonicum - MD (TOP 6 - FEB MIDTERM
B. Legionella pneumophila katayama fever; R. rickettsii - rocky mountain 2014 MED BOARDS; EXAM AUG
C. Schistosoma japonicum spotted fever; C. burnetti - Q fever. TOPNOTCH MD) 2014 - FOR
D. Rickettsia ricketsii INCLUSION
E. Coxiella burnetti IN THE
SAMPLEX

477 A 20 year old swimmer had a yellowish ear discharge. Pseudomonas aeruginosa is the most common LUISA SARANILLO, BACK-UP
Culture of this organism had a fruity odor. What is the cause of otitis externa and chronic MD (TOP 6 - FEB MIDTERM
causative organism? suppurative otitis media. Water sources is its 2014 MED BOARDS; EXAM AUG
A. Streptococcus pneumoniae habitat. It has a fruity or grapelike odor in TOPNOTCH MD) 2014 - FOR
B. Haemophilus influenza type B culture. Other choices are not associated with INCLUSION
C. Haemophilus influenza nontypable water sources. IN THE
D. Pseudomonas aeruginosa SAMPLEX
E. Staphylococcus aureus

TOPNOTCH MEDICAL BOARD PREP MICROBIOLOGY SUPEREXAM Page 58 of 90


For inquiries visit www.topnotchboardprep.com.ph or email us at topnotchmedicalboardprep@gmail.com
TOPNOTCH MEDICAL BOARD PREP MICROBIOLOGY SUPEREXAM
For inquiries visit www.topnotchboardprep.com.ph or email us at topnotchmedicalboardprep@gmail.com
Item QUESTION EXPLANATION AUTHOR TOPNOTCH
# EXAM
478 Mycoplasma pneumoniae produces autoantibodies LUISA SARANILLO, BACK-UP
against Red blood cells called: MD (TOP 6 - FEB MIDTERM
A. Cold agglutinins 2014 MED BOARDS; EXAM AUG
B. Warm agglutinins TOPNOTCH MD) 2014 - FOR
C. hemolysin INCLUSION
D. leukocidin IN THE
E. None of the choices SAMPLEX

479 During summer break, the mother noted a The hypopigmented lesion which become LUISA SARANILLO, BACK-UP
hypopigmented skin lesions on the trunk, upper back prominent after sun exposure is caused by MD (TOP 6 - FEB MIDTERM
and upper extremities of her 6 year old child. If you are Malassezia furfur with a spaghetti and 2014 MED BOARDS; EXAM AUG
going to examine skin scraping of the lesions with a 10 meatball appearance on 10% KOH. A small, TOPNOTCH MD) 2014 - FOR
% KOH, what will you see under the microscope? thin, smooth walled spores is the microconidia INCLUSION
A. small, thin, smooth walled spores of H. capsulatum. Multiple buds in wheel IN THE
B. oval yeast with pseudohyphae configuration is characteristic of P. SAMPLEX
C. multiple buds in wheel configuration brasiliensis. Oval yeast with pseudohyphae is
D. round yeast with broad-based buds characteristic of C. albicans. Round yeast with
E. spaghetti and meatball appearance broad - based bud is characteristic of B.
dermatitidis.
480 What is the protein used by HIV for fusion with the host gp41 is the protein used by HIV for fusion LUISA SARANILLO, BACK-UP
cell? with the host cell; gp120 is for attachment to MD (TOP 6 - FEB MIDTERM
A. gp120 CD4 protein; p7 is the nucleocapsid; p17 is the 2014 MED BOARDS; EXAM AUG
B. gp41 matrix protein; integrase is for integration of TOPNOTCH MD) 2014 - FOR
C. p7 viral DNA into host cell DNA. INCLUSION
D. p17 IN THE
E. Integrase SAMPLEX

481 A 16-day-old infant presents with fever, irritability, poor Most common cause of neonatal infection: E. ANGELIS ANDREA FINAL EXAM
feeding and a bulging fontanelle. Spinal fluid coli (gram negative bacilli), GBS, and Listeria COCOS, MD (TOP 1 - - AUG 2014
demonstrates gram-positive cocci. Which of the (gram positive bacilli). FEB 2014 MED
following is the most likely diagnosis? BOARDS; TOPNOTCH
A. Listeria MD)
B. Group A streptococcus
C. Group B streptococcus
D. Staphylococcus aureus

482 The correct sequence and reagents for Gram stain: The mnemonic is V-I-A-S. ANGELIS ANDREA FINAL EXAM
A. Crystal violet, iodine, alcohol, safranin COCOS, MD (TOP 1 - - AUG 2014
B. Crystal violet, iodine, alcohol, sudan red FEB 2014 MED
C. Crystal violet, alcohol, iodine, safranin BOARDS; TOPNOTCH
D. Crystal violet, alcohol, iodine, sudan red MD)

483 An infant suffers from loose water stools with vomiting Rotavirus which is the most common ANGELIS ANDREA FINAL EXAM
and fever episodes. Which is TRUE regarding the most causative agent for infant diarrhea is a double- COCOS, MD (TOP 1 - - AUG 2014
likely organism? stranded, non-enveloped organism which FEB 2014 MED
A. It can be easily cultured. belongs to the family Reoviridae. BOARDS; TOPNOTCH
B. It is an enveloped organism. MD)
C. Its genetic component is single-stranded.
D. It belongs to family Reoviridae.

484 A febrile child presents with multiple vesicular lesions in The most common strains causing hand, foot ANGELIS ANDREA FINAL EXAM
the mouth and hands. The virus most likely responsible and mouth disease are coxsackie A virus A16 COCOS, MD (TOP 1 - - AUG 2014
for this is: and enterovirus 71. SIMILAR TO PREVIOUS FEB 2014 MED
A. Coxsackie virus A BOARD EXAM CONCEPT/PRINCIPLE. BOARDS; TOPNOTCH
B. Coxsackie virus B MD)
C. Herpes virus
D. Variola virus

485 The drug of choice for strongyloidiasis infection is: Treatment of strongyloidiasis has been ANGELIS ANDREA FINAL EXAM
A. Albendazole traditionally based on thiabendazole despite COCOS, MD (TOP 1 - - AUG 2014
B. Mebendazole its frequent GI side effects. Ivermectin is FEB 2014 MED
C. Thiabendazole currently the drug of choice (however it is not BOARDS; TOPNOTCH
D. Praziquantel among the choices). SIMILAR TO PREVIOUS MD)
BOARD EXAM CONCEPT/PRINCIPLE.
486 On blood agar plate, there is greenish hue surrounding S. pneumonia and viridans streptococci ANGELIS ANDREA FINAL EXAM
the colonies of an unknown bacteria. Which is the most present with partial/incomplete hemolysis COCOS, MD (TOP 1 - - AUG 2014
likely organism? which appears greenish on BAP. The rest of FEB 2014 MED
A. S. pneumonia the choices present with complete RBC BOARDS; TOPNOTCH
B. S. pyogenes hemolysis. MD)
C. S. agalactiae
D. S. aureus

487 The most predominant facultative organism of the large The most predominant organism in the large ANGELIS ANDREA FINAL EXAM
intestine of man used as an index of fecal pollution of intestine is Bacteroides fragilis however it is COCOS, MD (TOP 1 - - AUG 2014
water sample is: an anaerobic organism (not facultative) and it FEB 2014 MED
A. Bacteroides fragilis is not used as index of fecal pollution. BOARDS; TOPNOTCH
B. Escherichia coli MD)
C. Clostridium perfringens
D. Proteus mirabilis

488 Which of the following is FALSE regarding RPR test? The titers of RPR and VDRL do not correlate, ANGELIS ANDREA FINAL EXAM
A. Cardiolipin is the antigen used. although they are both screening tests and can COCOS, MD (TOP 1 - - AUG 2014
B. It is used as a screening test for syphilis. be used to monitor syphilitic activity. FEB 2014 MED
C. Its levels are comparable with VDRL. BOARDS; TOPNOTCH
D. Biological false positives occur. MD)

TOPNOTCH MEDICAL BOARD PREP MICROBIOLOGY SUPEREXAM Page 59 of 90


For inquiries visit www.topnotchboardprep.com.ph or email us at topnotchmedicalboardprep@gmail.com
TOPNOTCH MEDICAL BOARD PREP MICROBIOLOGY SUPEREXAM
For inquiries visit www.topnotchboardprep.com.ph or email us at topnotchmedicalboardprep@gmail.com
Item QUESTION EXPLANATION AUTHOR TOPNOTCH
# EXAM
489 A 30-year-old hard-working driver complains of The usual triple therapy combination for ANGELIS ANDREA FINAL EXAM
recurrent epigastric pain. Which is an appropriate peptic ulcer disease, caused by H. pylori, is: 1 COCOS, MD (TOP 1 - - AUG 2014
combination therapy? proton pump inhibitor, clarithromycin and FEB 2014 MED
A. Amoxicillin, omeprazole, clarithromycin amoxicillin or metronidazole. Quadruple BOARDS; TOPNOTCH
B. Tetracycline, omeprazole, streptomycin therapy includes bismuth, metronidazole, MD)
C. Erythromycin, omeprazole, tetracycline tetracycline, and a proton pump inhibitor.
D. Ranitidine, metronidazole, clarithromycin

490 Which is NOT a characteristic of Mycobacterium Mycobacterium leprae is the causative agent ANGELIS ANDREA FINAL EXAM
tuberculosis? of leprosy or Hansen's disease. COCOS, MD (TOP 1 - - AUG 2014
A. Has a gram-positive reaction FEB 2014 MED
B. Is resistant to acid BOARDS; TOPNOTCH
C. Is resistant to antibodies and complement MD)
D. Is the causative agent of leprosy
491 A patient presents with an expanding bull's eye rash Lyme disease is caused by Borrelia ANGELIS ANDREA FINAL EXAM
with central clearing after having flu-like symptoms. burgdorferia which is transmitted by the tick COCOS, MD (TOP 1 - - AUG 2014
Months after, he presented with facial nerve palsy and Ixodes (also the vector for Babesia). FEB 2014 MED
AV nodal block. The vector for the most likely disease of BOARDS; TOPNOTCH
this patient is? MD)
A. Ixodes sp.
B. Glossina sp.
C. Dermacentor sp.
D. Anopheles sp.
492 The heat labile enterotoxin of ETEC is similar to the toxin ANGELIS ANDREA FINAL EXAM
produced by which of the following organisms? COCOS, MD (TOP 1 - - AUG 2014
A. Shigella dysenteriae FEB 2014 MED
B. Vibrio cholera BOARDS; TOPNOTCH
C. Staphylococcus aureus MD)
D. Salmonella typhi

493 A mass of intertwined hyphal elements is called: ANGELIS ANDREA FINAL EXAM
A. rhizoids COCOS, MD (TOP 1 - - AUG 2014
B. conidia FEB 2014 MED
C. arthrospores BOARDS; TOPNOTCH
D. mycelium MD)

494 Which is NOT an antigen-presenting cell? There were several immunology questions ANGELIS ANDREA FINAL EXAM
A. macrophage during our microbiology board exam. COCOS, MD (TOP 1 - - AUG 2014
B. B cell FEB 2014 MED
C. T cell BOARDS; TOPNOTCH
D. dendritic cell MD)

495 The most specific antibody for the diagnosis of Systemic ANA is the most sensitive. Anti-Sm is more ANGELIS ANDREA FINAL EXAM
lupus erythematosus (SLE) is: specific than anti-dsDNA according to COCOS, MD (TOP 1 - - AUG 2014
A. Anti-nuclear antibody Harrisons; however it does not correlate with FEB 2014 MED
B. Anti-Smooth muscle disease activity. SIMILAR TO PREVIOUS BOARDS; TOPNOTCH
C. Anti-Sm BOARD EXAM CONCEPT/PRINCIPLE Take MD)
D. Anti-dsDNA note that the complete title of the subject is
MICROBIOLOGY AND IMMUNOLOGY.
496 A patient presents with difficulty breathing, shortness of Allergic bronchopulmonary aspergillosis ANGELIS ANDREA FINAL EXAM
breath, and wheezing. There is marked eosinophilia on (ABPA) is a disease where a person's immune COCOS, MD (TOP 1 - - AUG 2014
blood exam. His allergic reaction is most likely due to the system is hypersensitive to Aspergillus spores, FEB 2014 MED
spores of: usually caused by A. fumigatus. Don't be BOARDS; TOPNOTCH
A. Aspergillus niger confused with A. flavus which produces a MD)
B. Aspergillus flavus carcinogenic mycotoxin causing
C. Aspergillus aspergillus hepatocellular carcinoma.
D. Aspergillus fumigatus

497 The infectious viral particle CANNOT be demonstrated Uncoating is also known as the eclipse period. ANGELIS ANDREA FINAL EXAM
or is undetectable during this step of the viral cycle: COCOS, MD (TOP 1 - - AUG 2014
A. adsorption FEB 2014 MED
B. penetration BOARDS; TOPNOTCH
C. uncoating MD)
D. assembly

498 Ian, an Ilocano, is fond of eating raw snails. He is at risk Paragonimus can be acquired through eating ANGELIS ANDREA FINAL EXAM
of acquiring which parasite? raw crayfish and Taenia solium can be COCOS, MD (TOP 1 - - AUG 2014
A. Echinostoma ilocanum acquired from raw pork. Schistosoma FEB 2014 MED
B. Paragonimus westermanii japonicum is from snails but is acquired BOARDS; TOPNOTCH
C. Taenia solium through cutaneous penetration of the cercaria. MD)
D. Schistosoma japonicum
499 Mandy is a pregnant friend who wants to visit Palawan. Mefloquine can be used in pregnant patients. ANGELIS ANDREA FINAL EXAM
She came to your clinic for malarial prophylaxis. Which COCOS, MD (TOP 1 - - AUG 2014
of the following medications can you give? FEB 2014 MED
A. Atovaquone/Proguanil BOARDS; TOPNOTCH
B. Doxycycline MD)
C. Primaquine
D. Mefloquine

TOPNOTCH MEDICAL BOARD PREP MICROBIOLOGY SUPEREXAM Page 60 of 90


For inquiries visit www.topnotchboardprep.com.ph or email us at topnotchmedicalboardprep@gmail.com
TOPNOTCH MEDICAL BOARD PREP MICROBIOLOGY SUPEREXAM
For inquiries visit www.topnotchboardprep.com.ph or email us at topnotchmedicalboardprep@gmail.com
Item QUESTION EXPLANATION AUTHOR TOPNOTCH
# EXAM
500 A premature infant was admitted at the neonatal ICU for The causative organism for the described case ANGELIS ANDREA FINAL EXAM
respiratory distress syndrome. Several days after is most likely Pseudomonas aeruginosa. COCOS, MD (TOP 1 - - AUG 2014
admission, the neonate is noted to be highly febrile with Among the choices, only Ceftazidime has FEB 2014 MED
widespread hemorrhagic skin necrosis. Which coverage for this bacteria. The rest are for BOARDS; TOPNOTCH
medication will address the most likely causative staphylococcal infections. MD)
organism?
A. Clindamycin
B. Vancomycin
C. Ceftazidime
D. Cloxacillin
501 This is considered as the first-line agent for cutaneous Katzung 10th ed., 862. JAN CHARMAINE BACK-UP
and visceral leishmaniasis: PALOMAR, MD (TOP MIDTERM
A. Sodium stibogluconate 9 - FEB 2014 MED EXAM AUG
B. Nitazoxanide BOARDS; TOPNOTCH 2014
C. Suramin MD)
D. Melarsoprol
E. Metronidazole

502 A 39 year old man was rushed to the hospital because of SIMILAR TO PREVIOUS BOARD EXAM JAN CHARMAINE BACK-UP
decreasing sensorium. Condition started with fever and CONCEPT/PRINCIPLE. Harrison's 17th ed., PALOMAR, MD (TOP MIDTERM
headache which doesn't improve with Paracetamol. 913 The following 9 - FEB 2014 MED EXAM AUG
Condition progressively worsened with development of are chemoprophyllactic measures for BOARDS; TOPNOTCH 2014
photophobia and decreasing sensorium. At the ER exposure to Invasive meningococcal disease in MD)
patient was still febrile, GCS=5, with positive Brudzinsky an adult: Rifampin 600mg bid x 2 days;
and Kernig's sign. Petechial rashes were noted on both Ciprofloxacin 500mg orally, single dose;
lower extremities. Which of the following prophyllactic Ofloxacin 400mg orally, single dose;
managements should you do if you were the medical Ceftriaxone 250mg IM, single dose;
intern who intubated the patient? Azithromycin 500mg orally, single dose. There
A. Ciprofloxacin 250mg IM, single dose is no post-exposure prophyllaxis vaccine
B. Rifampin 600mg orally, single dose available.
C. Ciprofloxacin 500mg tab, single dose
D. Get immunized with A, C, Y, W-135 vaccine
immediately
E. Ofloxacin 200mg orally, single dose
503 Least effective drug for the treatment of neonatal sepsis? SIMILAR TO PREVIOUS BOARD EXAM JAN CHARMAINE BACK-UP
A. IV ampicillin CONCEPT/PRINCIPLE PALOMAR, MD (TOP MIDTERM
B. Oral ampicillin 9 - FEB 2014 MED EXAM AUG
C. IV Gentamycin BOARDS; TOPNOTCH 2014
D. IV Amikacin MD)
E. Penicillin G

504 Which of the following diseases is blood-borne? SIMILAR TO PREVIOUS BOARD EXAM JAN CHARMAINE BACK-UP
A. Hepatitis C CONCEPT/PRINCIPLE. Diabetes- lifestyle and PALOMAR, MD (TOP MIDTERM
B. Diabetes genetics play a role in the acquisition of 9 - FEB 2014 MED EXAM AUG
C. Dengue disease; Dengue is arthropod-borne; Syphillis BOARDS; TOPNOTCH 2014
D. Syphillis can be congenital or sexually-transmitted; MD)
E. Pelvic Inflammatory Disease Pelvic inflammatory disease is sexually-
transmitted

505 The best virulence factor Streptococcus pneumonia is? SIMILAR TO PREVIOUS BOARD EXAM JAN CHARMAINE BACK-UP
A. Pilli CONCEPT/PRINCIPLE. Capsule - aka anti- PALOMAR, MD (TOP MIDTERM
B. Exotoxin S phagocytic factor 9 - FEB 2014 MED EXAM AUG
C. Lethal Factor BOARDS; TOPNOTCH 2014
D. Capsule MD)
E. Edema factor

506 What is the drug of choice for Wuchereria bacrofti ? SIMILAR TO PREVIOUS BOARD EXAM JAN CHARMAINE BACK-UP
A. Mebendazole CONCEPT/PRINCIPLE. Katzung 10th ed., 868. PALOMAR, MD (TOP MIDTERM
B. Diethylcarbamazine 9 - FEB 2014 MED EXAM AUG
C. Sodium stibogluconate BOARDS; TOPNOTCH 2014
D. Ivermectin MD)
E. Thiabendazole

507 Which of the following manifestations is evident on Chancre and inguinal lymphadenopathy- JAN CHARMAINE BACK-UP
secondary syphillis? primary syphillis, chancroid- Haemophillus PALOMAR, MD (TOP MIDTERM
A. Condyloma lata ducreyi, condyloma acuminatum- HPV; 9 - FEB 2014 MED EXAM AUG
B. Condyloma acuminatum condyloma lata- papules in the intertriginous BOARDS; TOPNOTCH 2014
C. Chancre areas can enlarge to produce broad, moist, MD)
D. Chancroid pink or gray-white, highly infectious lesions in
E. Inguinal lymphadenopathy 10% of patients with secondary syphillis.

508 Which of these is an incomplete Hepatitis virus? SIMILAR TO PREVIOUS BOARD EXAM JAN CHARMAINE BACK-UP
A. Hepatitis A
CONCEPT/PRINCIPLE PALOMAR, MD (TOP MIDTERM
B. Hepatitis B 9 - FEB 2014 MED EXAM AUG
C. Hepatitis C BOARDS; TOPNOTCH 2014
D. Hepatitis D MD)
E. Hepatitis E

TOPNOTCH MEDICAL BOARD PREP MICROBIOLOGY SUPEREXAM Page 61 of 90


For inquiries visit www.topnotchboardprep.com.ph or email us at topnotchmedicalboardprep@gmail.com
TOPNOTCH MEDICAL BOARD PREP MICROBIOLOGY SUPEREXAM
For inquiries visit www.topnotchboardprep.com.ph or email us at topnotchmedicalboardprep@gmail.com
Item QUESTION EXPLANATION AUTHOR TOPNOTCH
# EXAM
509 What is the infective stage of Schistosoma mansoni to SIMILAR TO PREVIOUS BOARD EXAM JAN CHARMAINE BACK-UP
humans? CONCEPT/PRINCIPLE metacercaria is the PALOMAR, MD (TOP MIDTERM
A. egg infective stage for man for all the trematodes 9 - FEB 2014 MED EXAM AUG
B. miracidium except for the schistosomes for which, BOARDS; TOPNOTCH 2014
C. redia cercaria, is the infective stage. Markell and MD)
D. cercaria Voge's Medical Parasitology, 8th ed., p. 210.
E. metacercaria

510 What is the intermediate host in paragonimiasis? Oncomelania hupensis quadrasi - JAN CHARMAINE BACK-UP
A. Oncomelania hupensis quadrasi intermediate host in Schistosoma japonicum; PALOMAR, MD (TOP MIDTERM
B. Sundathelphusa philippina Cyprinidae - fish intermediate host in 9 - FEB 2014 MED EXAM AUG
C. Cyprinidae Clonorchis sinensis; Parafossarulus- snail BOARDS; TOPNOTCH 2014
D. Parafossarulus intermediate host in Clonorchis MD)
E. None of the above

511 Toxic shock syndrome toxin 1 is to Staphylococcus as to SIMILAR TO PREVIOUS BOARD EXAM JAN CHARMAINE BACK-UP
which toxin is to Streptococcus? CONCEPT/PRINCIPLE. Alpha toxin- PALOMAR, MD (TOP MIDTERM
A. Alpha clostridium perfringens; Erythrogenic toxin- 9 - FEB 2014 MED EXAM AUG
B. Erythrogenic Streptococcal scarlet fever toxin ; exotoxin B- BOARDS; TOPNOTCH 2014
C. Pyogenic exotoxin A Streptococcal necrotizing fasciitis; MD)
D. Exotoxin B Streptococal toxic shock syndrome is clinically
E. None of the above similar but milder than S. aureus TSS which is
due to pyogenic exotoxin A. - TOPNOTCH
handout
512 Which of the following is caused by Cocksackie A virus? group A specific: herpangina(fever, JAN CHARMAINE BACK-UP
A. conjuctivitis sorethroat, tender vesicles in oropharynx); PALOMAR, MD (TOP MIDTERM
B. pericarditis hand-foot-and-mouth disease (vesicular rash 9 - FEB 2014 MED EXAM AUG
C. myocarditis on hands and feet, ulcerations in mouth); BOARDS; TOPNOTCH 2014
D. pleurodynia group B specific: pleurodynia (fever, severe MD)
E. herpangina pleuritic-type chest pain), myocarditis and
pericarditis

513 This is an aerobic, gram-positive, spore-forming rod, Very unique to Bacillus anthracis (the JAN CHARMAINE BACK-UP
with the following virulence factors: a poly-D-glutamate causative agent of anthrax) is its amino acid PALOMAR, MD (TOP MIDTERM
capsule, edema and lethal factor and protective antigen. capsule 9 - FEB 2014 MED EXAM AUG
A. Bacillus anthracis BOARDS; TOPNOTCH 2014
B. Bacillus cereus MD)
C. Clostridium tetani
D. Clostridium botulinum
E. Clostridium perfringens

514 This is a gram positive lancet-shaped cocci in pairs, alpha JAN CHARMAINE BACK-UP
hemolytic, catalse negative, bile and optochin sensitive, PALOMAR, MD (TOP MIDTERM
with polysaccharude capsule exhibiting positive 9 - FEB 2014 MED EXAM AUG
quellung reaction: BOARDS; TOPNOTCH 2014
A. Streptococcus pyogenes MD)
B. Streptococcus agalactiae
C. Streptococcus pneumonia
D. Staphylococcus aureus
E. Staphylococcus epidermidis

515 This bacteria causes a unique colon infection presenting a case of pseudomembranous colitis. - JAN CHARMAINE BACK-UP
initially as 1-2 mm whittish-yellow plaques in the Harrison's 17th ed., p819 PALOMAR, MD (TOP MIDTERM
mucosa that eventually coalesce to form larger plaques 9 - FEB 2014 MED EXAM AUG
that can become confluent over the entire colon wall and BOARDS; TOPNOTCH 2014
is acquired almost exclusively in association with anti- MD)
microbial use and consequent disruption of normal
colonic flora.?
A. Entamoeba hystolytica
B. Clostridium difficile
C. Norwalk virus
D. Shigella dysenteriae
E. Vibrio cholerae

516 This is a gram-negative rod that causes skin and bone Topnotch handout
JAN CHARMAINE BACK-UP
infections associated with human bites and "clenched PALOMAR, MD (TOP MIDTERM
fist" injuries: 9 - FEB 2014 MED EXAM AUG
A. Pasteurella multocida BOARDS; TOPNOTCH 2014
B. Bartonella henselae MD)
C. Eikenella corrodens
D. Ehrlichia chaffeensis
E. Calymmatobacterium granulomatis

517 The following bacteria is properly matched with its Haemophillus influenza requires Factors X JAN CHARMAINE BACK-UP
respective culture medium except: and Y for its growth. PALOMAR, MD (TOP MIDTERM
A. Neisseria gonorrhea from penile discharge : Thayer- 9 - FEB 2014 MED EXAM AUG
Martin Agar BOARDS; TOPNOTCH 2014
B. Haemophilus influenzae : Chocolate agar + Factor X MD)
C. Staphylococcus aureus - Mannitol salt agar
D. Mycoplasma pneumoniae - Eaton Agar
E. Borrelia burgdorferi - Barbour-Stoenner-Kelly Agar

TOPNOTCH MEDICAL BOARD PREP MICROBIOLOGY SUPEREXAM Page 62 of 90


For inquiries visit www.topnotchboardprep.com.ph or email us at topnotchmedicalboardprep@gmail.com
TOPNOTCH MEDICAL BOARD PREP MICROBIOLOGY SUPEREXAM
For inquiries visit www.topnotchboardprep.com.ph or email us at topnotchmedicalboardprep@gmail.com
Item QUESTION EXPLANATION AUTHOR TOPNOTCH
# EXAM
518 Luigi is a 24 year old carpenter came in to your clinic JAN CHARMAINE BACK-UP
because of an severe pain in his right foot. Condition PALOMAR, MD (TOP MIDTERM
started 3 weeks ago when he accidentally stepped on a 9 - FEB 2014 MED EXAM AUG
SIMILAR TO PREVIOUS BOARD EXAM
nail in their construction site.He did not develop fever BOARDS; TOPNOTCH 2014
CONCEPT/PRINCIPLE
but the wound became infected, he could hardly walk MD)
because of the pain. He has already received his 3rd dose
of tetanus toxoid at age 18. What should you give him
now?
A. Tetanus vaccine
B. Tetanus vaccine + Tetanus Immunoglobulin
C. Tetanus vaccine + Tetanus Immunoglobulin + pain-
reliever
D. Tetanus vaccine + NSAIDS + antibiotic
E. Tetanus vaccine + Tetanus Immunoglobulin + pain-
reliever + antibiotic
519 Which of the following is the causative agent of Topnotch handout. JC Polyoma virus only JAN CHARMAINE BACK-UP
progressive multifocal leukoencephalopathy? causes disease in immunocompromised hosts. PALOMAR, MD (TOP MIDTERM
A. Human Papilloma Virus Progressive multifocal leukoencephalopathy 9 - FEB 2014 MED EXAM AUG
B. Parvovirus B19 (PML) is a progressive disorder characterized BOARDS; TOPNOTCH 2014
C. Adenovirus pathologically by multifocal areas of MD)
D. JC Polyoma Virus demyelination of varying size distributed
E. BK Polyoma Virus throughout the brain but sparing the spinal
cord and optic nerves. There are also
characteristic cytologic alterations in both
astrocytes and oligodendrocytes. Patient often
present with homonymous hemianopsia,
mental impairment, weakness and ataxia. -
Harrison's 17th ed., p. 2634-35
520 The following viruses are properly matched with the Jawetz, Melnick, & Adelberg's Medical JAN CHARMAINE BACK-UP
cancer they cause in humans except: Microbiology (Chapter 43) PALOMAR, MD (TOP MIDTERM
A. HPV-testicular cancer 9 - FEB 2014 MED EXAM AUG
B. Human Herpesvirus 4-Hodgkin's disease BOARDS; TOPNOTCH 2014
C. Hepatitis B and C - Hepatoma MD)
D. Human T cell lymphoma virus - Adult T cell leukemia
E. All of the above are correct

521 A 15 year old girl is brought to the physician because of a MIGUEL RAFAEL MIDTERM 1
1 week history if vaginal discharge and a 2 day history of RAMOS, MD (TOP 3 - EXAM - FEB
sore throat and white recurrent candidal infections of FEB 2012 MED 2013
the skin and mucous membranes since childhood. She BOARDS; TOPNOTCH
has a 2 year history of type 1 diabetes mellitys and MD)
thyroditis. PE shows oral and vaginal candidiasis. Which
of the following is the most likely mechanism of her
recurrent candidal infectons?
A) Autoimmune destruction of the thymus
B) Blunting of the inflammatory response from
complement deficiency
C) Deficiency in anticandidal antibodies
D) Impaired cell-mediated immunity

522 A 17 year old boy with mental retardation is brought to MIGUEL RAFAEL MIDTERM 1
the physician because of low grade fever and abdominal RAMOS, MD (TOP 3 - EXAM - FEB
pain for 6 days. His temperature is 37.8 C, pulse is 110 FEB 2012 MED 2013
bpm, respirations are 22 cpm, and blood pressure is BOARDS; TOPNOTCH
120/70 mmHg. Examination shows splinter MD)
hemorrhages under the nails. A grade 2/6 systolic
murmur is heard beast at the upper left sternal border.
There is a systolic ejection click. S1 and S2 are normal.
Abdominal examination reveals splenomegaly.
Laboratories reveals Hb 9.1, leukocytes 30000, platelet
count 928000, ESR 110, Urine blood 2+, urine protein
1+. Which of the following is the most appropriate next
step in management?
A. 24 hour urine collection for measurement of protein
and creatinine concentrations
B. Blood cultures
C. Ultrasonography of the abdomen
D. Broad spectrum antibiotic therapy
523 A 37 year old man comes to the physician because of a 3 MIGUEL RAFAEL MIDTERM 1
day history of diarrhea. He has had six to eight watery RAMOS, MD (TOP 3 - EXAM - FEB
stools daily that occasionally contain streaks f blood. His FEB 2012 MED 2013
wife and children do not have similar symptoms. He has BOARDS; TOPNOTCH
not recently traveled. Two weeks ago, he completed a MD)
course of oral ciprofloxacin for acute prostatitis. He
currently takes no mediations. Vital signs are within
normal range. Results of CBC and serum studies are
within the reference range. Test for the stool for occult
blood is positive. He was started on Metronidazole. What
microorganism was the physician entertaining?
A) Shigella dystenteriae
B) Entamoeba histolytica
C) Clostridium difficile
D) Salmonella enteritides

TOPNOTCH MEDICAL BOARD PREP MICROBIOLOGY SUPEREXAM Page 63 of 90


For inquiries visit www.topnotchboardprep.com.ph or email us at topnotchmedicalboardprep@gmail.com
TOPNOTCH MEDICAL BOARD PREP MICROBIOLOGY SUPEREXAM
For inquiries visit www.topnotchboardprep.com.ph or email us at topnotchmedicalboardprep@gmail.com
Item QUESTION EXPLANATION AUTHOR TOPNOTCH
# EXAM
524 A previously healthy 37 year old man comes to the MIGUEL RAFAEL MIDTERM 1
physician because of fever, generalized muscle aches, RAMOS, MD (TOP 3 - EXAM - FEB
and an ulcer on his neck. The ulcer began 10 days ago as FEB 2012 MED 2013
a painless, itchy papule that enlarged over a 2 day BOARDS; TOPNOTCH
period; small, fluid-filled blisters formed on top of the MD)
papule. The blisters broke down, forming a painless
ulcer covered by a black scab. He is employed as a postal
worker and his hobby is gardening. He also takes care of
several cats at home. Which of the following is the most
likely diagnosis?
A) Cat-scratch fever
B) Cutaneous anthrax
C) Sporotrichosis
D) Streptococcal adenitis

525 A previously healthy 3-month-old girl is brought to the MIGUEL RAFAEL MIDTERM 1
emergency department because of a 3-day history of RAMOS, MD (TOP 3 - EXAM - FEB
grunting and increasing difficulty breathing. She appears FEB 2012 MED 2013
ill. Her temperature is 36.7 C (98 F), pulse is 160/min, BOARDS; TOPNOTCH
and respirations are 76/min. Examination shows MD)
grunting, nasal flaring, and marked intercostal
retractions. Bronchial breath sounds and occasional
bilateral crackles are heard on auscultation. An x-ray
film of the chest shows bilateral, diffuse interstitial
infiltrates and absence of the thymic shadow.
Bronchoalveolar lavage is positive for numerous
Pneumocystis carinii. Which of the following is the most
likely mechanism of these findings?
A) Adenosine deaminase deficiency
B) Consumption of complement
C) Defective opsonization
D) Dysmorphogenesis of the third and fourth pharyngeal
pouches
526 A 42-year-old man comes to the emergency department MIGUEL RAFAEL MIDTERM 1
because of a 2-week history of increasingly severe RAMOS, MD (TOP 3 - EXAM - FEB
headaches and a 2-day history of nausea, vomiting, neck FEB 2012 MED 2013
stiffness, and unsteadiness. He has type 2 diabetes BOARDS; TOPNOTCH
mellitus treated with glyburide. His temperature is 38.1 MD)
C (100.5 F). Funduscopic examination shows bilateral
papilledema. Neurologic examination shows mild
meningismus and diffusely brisk deep tendon reflexes.
He walks with a moderately broad-based gait. He is able
to recall two out of three objects after 5 minutes and
makes several errors on serial sevens. A CT scan of the
head shows no abnormalities. Cerebrospinal fluid
analysis shows a glucose level of 18 mg/dL, a protein
level of 108 mg/dL, and a leukocyte count of 59/mm3
(1% segmented neutrophils and 99% lymphocytes); a
cryptococcal antigen assay is positive. Which of the
following is the most appropriate pharmacotherapy for
this patient?
A) Acyclovir
B) Amphotericin B
C) Itraconazole
D) Penicillin
527 A previously healthy 52-year-old woman comes to the MIGUEL RAFAEL MIDTERM 1
physician because she has had a large pimple on her RAMOS, MD (TOP 3 - EXAM - FEB
right hand for 2 weeks that has failed to heal. She resides FEB 2012 MED 2013
in southeastern USA where she owns a nursery and BOARDS; TOPNOTCH
garden shop. Examination shows a painless red papule MD)
on the hand with several nontender subcutaneous
nodular lesions above it. Which of the following is the
most likely diagnosis?
A) Blastomycosis
B) Candidiasis
C) Coccidioidomycosis
D) Sporotrichosis

528 A 32-year-old woman comes to the physician because of MIGUEL RAFAEL MIDTERM 1
vaginal discharge for 2 weeks. She has been sexually RAMOS, MD (TOP 3 - EXAM - FEB
active with one female partner for 5 years. She has not FEB 2012 MED 2013
been treated with antibiotics over the past 2 years. Her BOARDS; TOPNOTCH
last Pap smear was 6 years ago when she was sexually MD)
active with a male partner. She has not used illicit drugs
or alcohol. Examination shows a grayish vaginal
discharge with a pH greater than 4.5. A wet mount
preparation of the vaginal discharge is most likely to
show which of the following?
A) Budding yeast
B) Clue cells
C) Ferning
D) Leukocytes in sheets

TOPNOTCH MEDICAL BOARD PREP MICROBIOLOGY SUPEREXAM Page 64 of 90


For inquiries visit www.topnotchboardprep.com.ph or email us at topnotchmedicalboardprep@gmail.com
TOPNOTCH MEDICAL BOARD PREP MICROBIOLOGY SUPEREXAM
For inquiries visit www.topnotchboardprep.com.ph or email us at topnotchmedicalboardprep@gmail.com
Item QUESTION EXPLANATION AUTHOR TOPNOTCH
# EXAM
529 A 37-year-old woman comes to the physician because of MIGUEL RAFAEL MIDTERM 1
a 1-day history of throbbing facial pain. She describes RAMOS, MD (TOP 3 - EXAM - FEB
the pain as 7 out of 10 in intensity. Over the past 9 days, FEB 2012 MED 2013
she has had nasal congestion, purulent nasal discharge, BOARDS; TOPNOTCH
sore throat, and a nonproductive cough. She does not MD)
smoke. Her husband and children have had no recent
illness. Her temperature is 38.5 C (101.3 F). Examination
shows congested nasal mucosa and purulent discharge
on the left. There is tenderness to palpation over the left
cheek and no transillumination over the left maxillary
sinus. The tympanic membranes are normal, and there is
no erythema of the throat. Examination shows no
cervical adenopathy. The lungs are clear to auscultation.
Which of the following is the most likely causal
organism?
A) Haemophilus influenzae type b
B) Moraxella catarrhalis
C) Staphylococcus aureus
D) Streptococcus pneumoniae
530 A 3-year-old boy is brought to the physician because of a MIGUEL RAFAEL MIDTERM 1
7-day history of fever and a painful swollen lymph node RAMOS, MD (TOP 3 - EXAM - FEB
in his groin. This is his sixth episode of lymph node FEB 2012 MED 2013
swelling; the previous episodes resolved after drainage BOARDS; TOPNOTCH
and prolonged antibiotic therapy. He also had MD)
pneumonia at the age of 12 months that required chest
tube placement for drainage. A maternal uncle died
during childhood of recurrent infections. The patient is
at the 5th percentile for height and weight. His
temperature is 38.5 C (101.3 F). Examination shows a
warm, tender, erythematous lymph node in the right
inguinal area. There are several healed incisions over the
inguinal area and neck from old drainage sites.
Laboratory studies show:

Hematocrit 35%
Leukocyte count 17,000/mm3
Segmented neutrophils 65%
Bands 10%
Lymphocytes 25%
Platelet count 350,000/mm3

A Gram's stain of the lymph node aspirate shows
numerous segmented neutrophils filled with bacteria;
cultures grow Staphylococcus aureus. Which of the
following is the most likely mechanism for these
findings?
A) Adenosine deaminase deficiency
B) Consumption of complement
C) Impaired phagocytic oxidative metabolism
D) Destruction of CD4+ T lymphocytes

531 A previously healthy 24-year-old woman comes to the Mycoplasma pneumonia MIGUEL RAFAEL MIDTERM 1
physician because of a low-grade fever and a RAMOS, MD (TOP 3 - EXAM - FEB
nonproductive cough for 7 days. She has been able to FEB 2012 MED 2013
continue her daily activities. Her temperature is 37.7 C BOARDS; TOPNOTCH
(99.9 F). A few scattered inspiratory crackles are heard MD)
in the thorax. An x-ray film of the chest shows patchy
infiltrates in both lungs. Which of the following is the
most appropriate initial pharmacotherapy?
A) Amoxicillin
B) Cefaclor
C) Ciprofloxacin
D) Erythromycin

532 A 32-year-old man receiving intensive chemotherapy for Immunosuppression with typical X ray MIGUEL RAFAEL MIDTERM 1
Hodgkin's disease has a temperature of 39 C (102.2 F). pattern RAMOS, MD (TOP 3 - EXAM - FEB
His respirations are 40/min. Widespread crackles are FEB 2012 MED 2013
heard in all lung fields. An x-ray film of the chest shows a BOARDS; TOPNOTCH
diffuse alveolar and interstitial pattern. Which of the MD)
following is the most likely causal organism?
A) Aspergillus species
B) Candida albicans
C) Coccidioides immitis
D) Pneumocystis carinii

TOPNOTCH MEDICAL BOARD PREP MICROBIOLOGY SUPEREXAM Page 65 of 90


For inquiries visit www.topnotchboardprep.com.ph or email us at topnotchmedicalboardprep@gmail.com
TOPNOTCH MEDICAL BOARD PREP MICROBIOLOGY SUPEREXAM
For inquiries visit www.topnotchboardprep.com.ph or email us at topnotchmedicalboardprep@gmail.com
Item QUESTION EXPLANATION AUTHOR TOPNOTCH
# EXAM
533 A sexually active 20-year-old woman has had fever, MIGUEL RAFAEL MIDTERM 1
chills, malaise, and pain of the vulva for 2 days. RAMOS, MD (TOP 3 - EXAM - FEB
Examination shows a vulvar pustule that has ulcerated FEB 2012 MED 2013
and formed multiple satellite lesions. Nodes are palpated BOARDS; TOPNOTCH
in the inguinal and femoral areas. A smear of fluid from MD)
the lesions establishes the diagnosis. Which of the
following is the most likely causal organism?
A) Chlamydia trachomatis
B) Haemophilus ducreyi
C) Neisseria gonorrhoeae
D) Treponema pallidum

534 A 7-year-old girl is brought to the physician because of a MIGUEL RAFAEL MIDTERM 1
2-day history of fever, headache, sore throat, and RAMOS, MD (TOP 3 - EXAM - FEB
swollen glands. She does not have a runny nose, FEB 2012 MED 2013
congestion, or cough. She has no allergies to medications. BOARDS; TOPNOTCH
Her temperature is MD)
38.6C (101.4 F), blood pressure is 100/60 mm Hg, pulse
is 120/min, and respirations are 16/min. Examination
shows a swollen, erythematous oropharynx With
tonsillar exudates. The anterior cervical lymph nodes are
enlarged and tender. No other abnormalities are noted.
Which of the following is the most likely causal
organism?
A) Adenovirus
B) Mycoplasma pneumoniae
C) Group A streptococcus
D) Haemophilus influenzae
535 A previously healthy 57-year-old man comes to the Lyme disease MIGUEL RAFAEL MIDTERM 1
physician because of a nonpruritic rash over both legs RAMOS, MD (TOP 3 - EXAM - FEB
for 1 week and a low-grade fever for 2 days. He recently FEB 2012 MED 2013
returned from a 2-week canoe trip on a river in BOARDS; TOPNOTCH
Minnesota. His temperature is 37.2 C (99 F). MD)
Examination shows a 4 x 6-cm, macular, dark pink, ovoid
lesion on the right posterior hip with central clearing
and a punctate eschar near the center. There are macular
ring lesions with central clearing over the medial tibia
and anterior thighs of the lower extremities. He has no
lymphadenopathy. Which of the following is the most
likely causal organism?
A) Borrelia burgdorferi
B) Brucella melitensis
C) Francisella tularensis
D) Rickettsia rickettsii
536 Two days after admission to the hospital because of a 3- botulism MIGUEL RAFAEL MIDTERM 1
day history of slurred speech, double vision, and RAMOS, MD (TOP 3 - EXAM - FEB
dysphagia, a 24-year-old woman becomes quadriplegic FEB 2012 MED 2013
and requires intubation and mechanical ventilation. Her BOARDS; TOPNOTCH
medical history is unremarkable. One week ago, she MD)
attended a family picnic; several of her family members
have had abdominal cramps
and diarrhea since the picnic. Her temperature is 37C
(98.6 F), blood pressure is 120/80 mm Hg, and pulse is
120/min. Examination shows dry mucous membranes,
large nonreactive pupils, ophthalmoplegia, and profound
facial weakness. There is areflexia, quadriplegia, and no
movement of the palate and tongue. Sensation is normal.
Babinski's sign is absent. Which of the following is the
most appropriate pharmacotherapy?
A) Antitoxin
B) Azathioprine
C) Interferon
D) Pyridostigmine

537 A 2-year-old boy is brought to the physician because of MIGUEL RAFAEL MIDTERM 1
fever and listlessness for 12 hours. He has had recurrent RAMOS, MD (TOP 3 - EXAM - FEB
episodes of pneumonia and otitis media over the past FEB 2012 MED 2013
year. Two maternal uncles died of pneumonia in early BOARDS; TOPNOTCH
childhood. One year ago, he was at the 50th percentile MD)
for height and weight; he is currently at the 25th
percentile for height and 10th percentile for weight. He
appears ill. His temperature is 39 C (102.2 F), blood
pressure is 60/40 mm Hg, pulse is 160/min, and
respirations are 36/min. Examination shows cool and
mottled extremities. A blood culture grows
Streptococcus pneumoniae. Serum IgE, IgG, and IgM
levels are markedly decreased. Which of the following
diagnostic tests is most likely to be abnormal?
A) Candidal skin test
B) Flow cytometry identification of B lymphocytes
C) Nitroblue tetrazolium test
D) T-lymphocyte receptor stimulation by concanavalin A

TOPNOTCH MEDICAL BOARD PREP MICROBIOLOGY SUPEREXAM Page 66 of 90


For inquiries visit www.topnotchboardprep.com.ph or email us at topnotchmedicalboardprep@gmail.com
TOPNOTCH MEDICAL BOARD PREP MICROBIOLOGY SUPEREXAM
For inquiries visit www.topnotchboardprep.com.ph or email us at topnotchmedicalboardprep@gmail.com
Item QUESTION EXPLANATION AUTHOR TOPNOTCH
# EXAM
538 A 57-year-old man with multiple myeloma comes to the MIGUEL RAFAEL MIDTERM 1
physician because of a 12-hour history of fever, sharp RAMOS, MD (TOP 3 - EXAM - FEB
chest pain with deep inspiration, and cough productive FEB 2012 MED 2013
of blood-tinged sputum. His temperature is 38.3 C (101 BOARDS; TOPNOTCH
F), blood pressure is 120/78 mm Hg, pulse is 112/min, MD)
and respirations are 28/min. Crackles are heard at the
right lung base. His hemoglobin level is 9.2 g/dL,
leukocyte count is 2600/mm3, and platelet count is
6,000/mm3. Empiric antibiotics should be directed
against which of the following organisms?
A) Listeria monocytogenes
B) Neisseria meningitidis
C) Pseudomonas aeruginosa
D) Streptococcus pneumoniae

539 An 82-year-old man comes to the physician because of a MIGUEL RAFAEL MIDTERM 1
3-day history of low back pain that radiates to the right RAMOS, MD (TOP 3 - EXAM - FEB
leg. He also has had a lesion over the right shin and FEB 2012 MED 2013
weakness of the right foot. He began taking prednisone 2 BOARDS; TOPNOTCH
weeks ago for acute bronchitis. He has chronic MD)
obstructive pulmonary disease, benign prostatic
hypertrophy, and glaucoma. Examination shows
numerous papular and vesicular lesions over the right
anterior and posterior shin. There is weakness of right
knee flexion, ankle dorsiflexion, plantar flexion, eversion,
and inversion; the right ankle reflex is absent. Sensation
to pinprick and cold is decreased over the right lower
extremity. Which of the following is the most likely
causal organism?
A) Borrelia burgdorferi
B) Treponema pallidum
C) Herpes simplex virus 1
D) Varicella-zoster virus
540 A 2-year-old girl has had fever and bloody diarrhea for MIGUEL RAFAEL MIDTERM 1
10 days. A stool culture obtained 7 days ago grew RAMOS, MD (TOP 3 - EXAM - FEB
Salmonella species sensitive to amoxicillin. A blood FEB 2012 MED 2013
culture was negative. Despite beginning oral amoxicillin BOARDS; TOPNOTCH
therapy 4 days ago, her diarrhea has persisted. Current MD)
examination shows no other abnormalities except for a
temperature of 38.6 C (101.5 F). Which of the following
is the most likely explanation for the failure of
amoxicillin to improve her symptoms?
A) Amoxicillin does not alter the course of Salmonella
enteritidis
B) Amoxicillin has caused pseudomembranous colitis
C) Amoxicillin is absorbed at the level of the jejunum,
leaving no drug to be delivered to the colon
D) Oral amoxicillin is not absorbed into the systemic
circulation in the presence of diarrhea
541 Gram staining of a sample sent to the laboratory Streptococcus pneumoniae is the most ABDELSIMAR OMAR FINAL EXAM
revealed Gram (+) lancet-shaped cocci in pairs. Colonies common cause of otitis media, CAP and II, MD (TOP 2 - AUG - FEB 2014
exhibited partial hemolysis on blood agar. Biochemical sinusitis. 2013 MED BOARDS;
testing reveals that the bacterium is optochin sensitive. TOPNOTCH MD - 200
The isolated bacterium is the most common cause of: QUESTIONS) AND
A. Rheumatic fever MARC DENVER
B. Neonatal meningitis TIONGSON, MD (40
C. Uncomplicated urinary tract infection QUESTIONS)
D. Skin abscess
E. Otitis media

542 35/F presents with a 1-week history of fever, chills and Case of acute infective endocarditis, most ABDELSIMAR OMAR FINAL EXAM
left ankle pain. Patient is a known IV drug abuser. On PE, commonly caused by S. aureus II, MD (TOP 2 - AUG - FEB 2014
vital signs are as follows: BP 110/60, HR 108, RR 18, T 2013 MED BOARDS;
40C. Auscultation reveals a grade 2/4 holosystolic TOPNOTCH MD - 200
murmur best heard at the right upper sternal border. QUESTIONS) AND
Left ankle is visibily swollen and warm. Which of the MARC DENVER
following describes the most likely etiologic agent? TIONGSON, MD (40
A. Gram positive cocci in chain QUESTIONS)
B. Gram positive cocci in clusters
C. Gram positive rod, tennis racket-like in appearance
D. Gram positive rod, non-spore forming, curved, with
characteristic tumbling motility
E. Gram negative diplococci

TOPNOTCH MEDICAL BOARD PREP MICROBIOLOGY SUPEREXAM Page 67 of 90


For inquiries visit www.topnotchboardprep.com.ph or email us at topnotchmedicalboardprep@gmail.com
TOPNOTCH MEDICAL BOARD PREP MICROBIOLOGY SUPEREXAM
For inquiries visit www.topnotchboardprep.com.ph or email us at topnotchmedicalboardprep@gmail.com
Item QUESTION EXPLANATION AUTHOR TOPNOTCH
# EXAM
543 A 1/M infant consults for paroxysms of violent coughing. Regan Lowe medium containing charcoal, ABDELSIMAR OMAR FINAL EXAM
You note cyanosis around his lips during coughing. blood and antibiotics is the medium of choice II, MD (TOP 2 - AUG - FEB 2014
Parents note that the patient had symptoms of the for routine B pertussis incubation. 2013 MED BOARDS;
common colds for a few weeks prior. Patient has not had TOPNOTCH MD - 200
any immunizations since birth. To identify the etiologic QUESTIONS) AND
organism, you take a nasopharyngeal aspirate. On which MARC DENVER
culture medium would the most likely etiologic agent TIONGSON, MD (40
grow? QUESTIONS)
A. Regan-Lowe medium
B. Tellurite
C. Chocolate agar with factors X and V
D. Thayer Martin agar
E. Mannitol salts agar
544 A 30/M who was in a vehicular crash underwent In anatomic asplenia, individuals are prone to ABDELSIMAR OMAR FINAL EXAM
exploratory laparotomy and splenectomy. He would infections with pathogens that normally II, MD (TOP 2 - AUG - FEB 2014
require immunizations against which of the following require opsonization and phagocytosis by 2013 MED BOARDS;
pathogens: macrophages in the spleen; i.e., by TOPNOTCH MD - 200
A. Neisseria meningitidis encapsulated organisms. QUESTIONS) AND
B. Streptococcus pneumoniae MARC DENVER
C. Haemophilus infleunzae TIONGSON, MD (40
D. None of the above QUESTIONS)
E. All of the above

545 A recent study by Albert et al. in the NEJM made the case ABDELSIMAR OMAR FINAL EXAM
for the use of Azithromycin in the prevention of II, MD (TOP 2 - AUG - FEB 2014
exacerbations of COPD. This is because Azithromycin is 2013 MED BOARDS;
active against the MOST common bacterial cause of TOPNOTCH MD - 200
COPD, which is: QUESTIONS) AND
A. Streptococcus pneumoniae MARC DENVER
B. Klebsiella pneumoniae TIONGSON, MD (40
C. Chlamydophila pneumoniae QUESTIONS)
D. Haemophilus influenzae
E. Mycoplasma pneumoniae
546 A 22/M initially complains of bilateral foot and hand The most commonly identified infectious ABDELSIMAR OMAR FINAL EXAM
tingling. The symptoms then rapidly progressed over a trigger in GBS include C jejuni, CMV, EBV and II, MD (TOP 2 - AUG - FEB 2014
week to include lower extremity weakness. He is Mycoplasma pneumoniae 2013 MED BOARDS;
brought to the clinic because he is no longer able to TOPNOTCH MD - 200
ambulate. On PE, motor strength is 0/5 on B lower QUESTIONS) AND
extremities; reflexes are decreased; the rest of the MARC DENVER
neurologic PE was unremarkable. Lumbar puncture TIONGSON, MD (40
done revealed midly elevated protein with no cells and QUESTIONS)
normal glucose. A few weeks prior, patient reported
having "stomach flu." The most likely infectious trigger
of the patient's condition is:
A. Shigella sonnei
B. Escherichia coli
C. Salmonella typhi
D. Enterobacter sp.
E. Campylobacter jejuni
547 Which of the following organisms lack superoxide Question asks us which is anaerobic of the ABDELSIMAR OMAR FINAL EXAM
dismutase and cannot use O2 as a terminal electron four choices II, MD (TOP 2 - AUG - FEB 2014
acceptor? 2013 MED BOARDS;
A. Campylobacter jejuni TOPNOTCH MD - 200
B. Escherichia coli QUESTIONS) AND
C. Shigella sonnei MARC DENVER
D. Pseudomonas aeruginosa TIONGSON, MD (40
E. Clostridium tetani QUESTIONS)

548 23/M with a month-long history of nonproductive cough, Sulfatides inhibit phagosome-lysosomal ABDELSIMAR OMAR FINAL EXAM
intermittent fever and night sweats came in for a consult. fusion; while tuberculin and mycolic acid is II, MD (TOP 2 - AUG - FEB 2014
Sputum sample sent to the local health center was responsible for delayed type hypersensitvitiy. 2013 MED BOARDS;
positive for AFB. Which of the following virulent factors TOPNOTCH MD - 200
of the etiologic agent allow it to inhibit leukocyte QUESTIONS) AND
migration, disrupting mitochondrial respiration and MARC DENVER
oxidative phosphorylation in cells? TIONGSON, MD (40
A. Sulfatides QUESTIONS)
B. Tuberculin
C. Mycolic acid
D. Cord factor
E. Wax D

549 8/M from a rural barangay was brought to the local Cholera exhbits shooting star motility ABDELSIMAR OMAR FINAL EXAM
health center for passing large volumes of loose watery II, MD (TOP 2 - AUG - FEB 2014
"rice water" stools. All of the following statements 2013 MED BOARDS;
describe the most likely etiologic agent except: TOPNOTCH MD - 200
A. Growth on alkaline but not acidic media QUESTIONS) AND
B. Requires a high dose for transmission (>10^7 MARC DENVER
organisms) TIONGSON, MD (40
C. Oxidase positive QUESTIONS)
D. Exhibits a characteristic swarming motlitiy
E. None of the above

TOPNOTCH MEDICAL BOARD PREP MICROBIOLOGY SUPEREXAM Page 68 of 90


For inquiries visit www.topnotchboardprep.com.ph or email us at topnotchmedicalboardprep@gmail.com
TOPNOTCH MEDICAL BOARD PREP MICROBIOLOGY SUPEREXAM
For inquiries visit www.topnotchboardprep.com.ph or email us at topnotchmedicalboardprep@gmail.com
Item QUESTION EXPLANATION AUTHOR TOPNOTCH
# EXAM
550 Septicemic shock is primarily due to which bacterial Lipid A is found in the outer membrane. ABDELSIMAR OMAR FINAL EXAM
structure in Gram negative organisms? II, MD (TOP 2 - AUG - FEB 2014
A. Capsule 2013 MED BOARDS;
B. Periplasmic space TOPNOTCH MD - 200
C. Outer membrane QUESTIONS) AND
D. Inner membrane MARC DENVER
E. Peptidoglycan cell wall TIONGSON, MD (40
QUESTIONS)

551 A 33/M, gardener, comes in to the clinic a few weeks Sporothrix schenckii infection may be treated ABDELSIMAR OMAR FINAL EXAM
after a rose thorn abraded the dorsal surface of his left with itraconazole or KI in milk. II, MD (TOP 2 - AUG - FEB 2014
thumb while gardening. He developed a well- 2013 MED BOARDS;
demarcated ulcerated lesion on the dorsal surface of the TOPNOTCH MD - 200
right thumb and erythematous nodular lesions on his QUESTIONS) AND
right forearm. Which of the following anti-fungals is MARC DENVER
most appropriate for this patient? TIONGSON, MD (40
A. Fluconazole QUESTIONS)
B. Miconazole
C. Amphotericin B
D. Potassium iodide in milk
E. Clotrimazole

552 A 40/M, known case of type I diabetes but poorly Mucormycosis. A = cryptococcus; B = ABDELSIMAR OMAR FINAL EXAM
compliant with medications, presents to the ER with a 3- aspergillus; D = blastomyces; E = II, MD (TOP 2 - AUG - FEB 2014
day history of left facial pain, proptosis, purplish paracoccidioides 2013 MED BOARDS;
discoloration of the periorbital area and hemorrhagic TOPNOTCH MD - 200
exudates from the nose. You suspect a fungal infection. QUESTIONS) AND
Biopsy of infected tissue will reveal: MARC DENVER
A. Oval yeast with narrow-based buds TIONGSON, MD (40
B. Septate hyphae branching at acute angles QUESTIONS)
C. Nonseptate hyphae branching at 90 degree angles
D. Round yeast with broad-based buds
E. Multiple buds in wheel configuration
553 A 28/M consults due to a one-week history of passing Giardiasis. A = cryptosporidium; B = ABDELSIMAR OMAR FINAL EXAM
greasy, foul-smelling, yellowish stools. He also complains Balantidium; C = Trichomonas II, MD (TOP 2 - AUG - FEB 2014
of nausea, flatulence, and abdominal bloating. His male 2013 MED BOARDS;
sexual partner also has the same symptoms. Which of TOPNOTCH MD - 200
the following findings would you expect in this patient? QUESTIONS) AND
A. Acid fast occysts in stool MARC DENVER
B. Ciliated trophozoites and cysts in feces TIONGSON, MD (40
C. Motile trophozoites in methylene blue wet mount; QUESTIONS)
with (+) whiff test in KOH
D. Motile trophozoites with "falling leaf" motility
E. Star-shaped cysts in stool
554 Infections due to Plasmodium vivax and Plasmodium ABDELSIMAR OMAR FINAL EXAM
ovale share a number of similarities. Which of the II, MD (TOP 2 - AUG - FEB 2014
following can one use to differentiate Plasmodium vivax 2013 MED BOARDS;
infection from Plasmodium ovale infection? TOPNOTCH MD - 200
A. Benign tertian periodicity QUESTIONS) AND
B. Presence of persistent hypnozoites MARC DENVER
C. Preference for young RBCs TIONGSON, MD (40
D. Enlarged host cells; with ameoboid trophozoites on QUESTIONS)
blood smear
E. Possibility for relapse
555 A 48/M, a resident of rural Argentina, came home to Chagas disease ABDELSIMAR OMAR FINAL EXAM
Manila for the holidays. During this visit, the patient was II, MD (TOP 2 - AUG - FEB 2014
rushed to the ER after presenting with generalized 2013 MED BOARDS;
edema, exertional dyspnea and palpitations. On PE, you TOPNOTCH MD - 200
note that the: apex beat is displaced to the 6thh ICS QUESTIONS) AND
LAAL; rales are heard on basal lung fields; and jugular MARC DENVER
veins are distended. You suspect a chronic phase disease TIONGSON, MD (40
caused by a parasitic infection; and request blood QUESTIONS)
cultures and PCR to identify infection with:
A. Leishmania donovani
B. Lesihmania braziliensis complex
C. Trypanosoma brucei gambiense
D. Trypanosoma brucei rhodesiense
E. Trypanosoma cruzi
556 A 28/M, known case of AIDS, presents with severe Acid fast cysts? AIDS diarrhea? ABDELSIMAR OMAR FINAL EXAM
diarrhea. Stool exam reveals cysts which were found to Cryptosporidium! Infective stage: thick-walled II, MD (TOP 2 - AUG - FEB 2014
be acid fast. The infective stage in the life cycle of the oocysts 2013 MED BOARDS;
etiologic organism is the: TOPNOTCH MD - 200
A. Sporozoite QUESTIONS) AND
B. Trophozoite MARC DENVER
C. Merozoite TIONGSON, MD (40
D. Macrogamont QUESTIONS)
E. Oocyst

TOPNOTCH MEDICAL BOARD PREP MICROBIOLOGY SUPEREXAM Page 69 of 90


For inquiries visit www.topnotchboardprep.com.ph or email us at topnotchmedicalboardprep@gmail.com
TOPNOTCH MEDICAL BOARD PREP MICROBIOLOGY SUPEREXAM
For inquiries visit www.topnotchboardprep.com.ph or email us at topnotchmedicalboardprep@gmail.com
Item QUESTION EXPLANATION AUTHOR TOPNOTCH
# EXAM
557 According to the Philippine Textbook of Medical ABDELSIMAR OMAR FINAL EXAM
Parasitology, infection with which parasite is the most II, MD (TOP 2 - AUG - FEB 2014
common cause of cor pulmonale in Filipino children? 2013 MED BOARDS;
A. Ascaris lumbricoides TOPNOTCH MD - 200
B. Strongyloides stercoralis QUESTIONS) AND
C. Ancylostoma duodenale MARC DENVER
D. Necator americanus TIONGSON, MD (40
E. Schistosoma japonicum QUESTIONS)

558 This antigen/antibody is generally detectable only after ABDELSIMAR OMAR FINAL EXAM
Hepatitis B virus is no longer detectable and is used to II, MD (TOP 2 - AUG - FEB 2014
suggest a lower risk of transmission. 2013 MED BOARDS;
A. HBsAg TOPNOTCH MD - 200
B. HBsAb QUESTIONS) AND
C. HBcAg MARC DENVER
D. HBeAg TIONGSON, MD (40
E. HBeAb QUESTIONS)

559 A 7/F comes in to your clinic due to a rash. On Erythema infectiosum due to Parvovirus B19, ABDELSIMAR OMAR FINAL EXAM
examination, you note that the patient has bright red a ssDNA, naked virus. II, MD (TOP 2 - AUG - FEB 2014
macules on his cheeks and a lacy, reticulated erythema 2013 MED BOARDS;
over the extremities. Patient has no other symptoms or TOPNOTCH MD - 200
remarkable examination findings. Which of the following QUESTIONS) AND
describes the etiologic agent in this condition? MARC DENVER
A. dsDNA virus TIONGSON, MD (40
B. Enveloped virus QUESTIONS)
C. RNA virus
D. DNA replicates in the cytoplasm
E. None of the above
560 The HIV virus is able to bind to CD4 receptors on host gp120 and gp41 are envelope proteins; and ABDELSIMAR OMAR FINAL EXAM
cells due to the presence of specific surface proteins. are thus products of the Env gene. II, MD (TOP 2 - AUG - FEB 2014
These surface proteins are products of which structural 2013 MED BOARDS;
gene: TOPNOTCH MD - 200
A. Gag QUESTIONS) AND
B. Pol MARC DENVER
C. Env TIONGSON, MD (40
D. Rev QUESTIONS)
E. Nef

561 What is the causative agent of pulmonary tuberculosis? MTB causes PTB BLAKE WARREN MIDTERM 2
a.) Mycobacterium Avium Complex ANG, MD (TOP 1 - EXAM - FEB
b). Mycobacterium Leprae AUG 2013 MED 2014
c.) Mycobacterium kansasii BOARDS; TOPNOTCH
d.) Mycobacterium tuberculosis MD)

562 Which of the following is an enveloped icosahedral RNA Flaviviridae, Togaviridae and retroviridae are BLAKE WARREN MIDTERM 2
virus? all enveloped icosahedra rna viruses. Rubella ANG, MD (TOP 1 - EXAM - FEB
a.) Rubella virus virus is a togavirus. AUG 2013 MED 2014
b.) Rabies virus BOARDS; TOPNOTCH
c.) Varicella virus MD)
d.) Rotavirus

563 Which of the following viruses possess a reverse Hep B and the retroviridae family are the ones BLAKE WARREN MIDTERM 2
transcriptase enzyme? that possesses reverese transcription enzymes ANG, MD (TOP 1 - EXAM - FEB
a.) Parvovirrus for replication. AUG 2013 MED 2014
b.) Reovirus BOARDS; TOPNOTCH
c.) Hepadnavirus MD)
d.) Coxsackie A and B viruses

564 33 year old male has been on a recent travel spree all Answer: The diagnosis of this case is a visceral BLAKE WARREN MIDTERM 2
over Europe and Africa. On returning home to the leishmaniasis with the history of sexual ANG, MD (TOP 1 - EXAM - FEB
Philippines, He had developed spiking fevers. On activity increasing possibility of an AUG 2013 MED 2014
physical examination, there was noted immunocompromised state in which visceral BOARDS; TOPNOTCH
hepatosplenomegaly. Further probing into the patients leishmaniasis is more likely to develop. Choice MD)
history showed that he had been on a number of A reflect a diagnosis of Chagas disease
unprotected random sexual encounters. Laboratory (trypanosomiasis). Choice B is incorrect as
findings revealed pancytopenia and occasional Russel only promastigotes can be seen in the culture
bodies. Which of the following is true? media for leismania. Choice C is incorrect
a.) An acute erythematous lesion at the bite site called a because what is being described is a
chagoma may be seen, with the diease eventually putting trypomastigote. Choice D is correct as
the patient at risk of dilated cardiomyopathy. promastigotes enter the macrophage and
b.) Culture of the infected tissue on an NNN media will developed into unflagellated amastigotes (as
show the presence of amastigotes supporting the described) and are often referred to as LD
diagnosis of Leishmaniasis. (Leishman-Donovan) body.
c.) The hemoflagellates seen on microscopy will show a
posterior kinetoplast giving rise to a full-body length
undulating membrane with a single, anterior free
flagellum.
d.) Macrophages in which the hemoflagellate develop
will show inclusion bodies with one eccentric nucleous
and a dot-like blepharoblast.

TOPNOTCH MEDICAL BOARD PREP MICROBIOLOGY SUPEREXAM Page 70 of 90


For inquiries visit www.topnotchboardprep.com.ph or email us at topnotchmedicalboardprep@gmail.com
TOPNOTCH MEDICAL BOARD PREP MICROBIOLOGY SUPEREXAM
For inquiries visit www.topnotchboardprep.com.ph or email us at topnotchmedicalboardprep@gmail.com
Item QUESTION EXPLANATION AUTHOR TOPNOTCH
# EXAM
565 A 65 year old rich lady presented with cough, shortness The patient above is a case of Lady BLAKE WARREN MIDTERM 2
of breath and weight loss for 1 month. She had no Windermere Syndrome. Voluntary cough ANG, MD (TOP 1 - EXAM - FEB
history of smoking and was very socially active. She suppression has been known to predispose to AUG 2013 MED 2014
recalls to you that there have been numerous times that an infection by MAC, despite BOARDS; TOPNOTCH
she suppresses her cough especially during her very immunocompetence, in the middle lobe and MD)
numerous gallant occasions for fear of embarrassment. lingula.A
Work up showed presence of acid fast bacilli while
imaging revealed a middle lobe infiltrate. What is the
causative agent?
a.) Mycobacterium Avium Complex
b). Mycobacterium Leprae
c.) Mycobacterium kansasii
d.) Mycopacterium tuberculosis

566 A biologist exploring the wilds was bitten by a tick Ixodes tick (bull’s eye rash on bite) is a vector BLAKE WARREN MIDTERM 2
resulting to a bull’s eye red rash at the bite site. She of at least three human pathogens : Borellia, ANG, MD (TOP 1 - EXAM - FEB
consulted to you at the ER. Possible diseases that must Babesia and Anaplasma. AUG 2013 MED 2014
be entertained are the following except: BOARDS; TOPNOTCH
a. Babesiosis MD)
b. Lyme Disease
c. Human Monocytic Ehrlichiosis
d. Human Granulocytic Anaplasmosis

567 Which of the following lab values is consistent with the All Neisseria of clinical importance will show a BLAKE WARREN MIDTERM 2
diagnosis of Neiserria meningitides. positive catelase test. ANG, MD (TOP 1 - EXAM - FEB
a.) catalase positive AUG 2013 MED 2014
b.) non glucose fermenter BOARDS; TOPNOTCH
c.) non-maltose fermenter MD)
d.) oxidase negative

568 These organisms are atypical in the sense that they lack BLAKE WARREN MIDTERM 2
muramic acid in their cell walls. ANG, MD (TOP 1 - EXAM - FEB
a.) Mycoplasma AUG 2013 MED 2014
b.) Legionella BOARDS; TOPNOTCH
c.) Mycobateria MD)
d.) Chlamydia

569 Laboratory work up of a gram positive cocci showed a BLAKE WARREN MIDTERM 2
catalase positive, coagulase negative organism. Further ANG, MD (TOP 1 - EXAM - FEB
work up showed rampant growth around the AUG 2013 MED 2014
Novobiocin antibiotic disc after 16 hours of incubation. BOARDS; TOPNOTCH
The isolated organism is most probably? MD)
a.) Staphylococcus aureus
b.) Streptococcus mutans
c.) Staphylococcus epidemidis
d.) Staphylococcus saprophyticus

570 Patient is a 26 year old female presenting with limited Tuberculous leprosy is mainly TH1 response BLAKE WARREN MIDTERM 2
hypoesthetic areas of the skin. The plaques were also associated and the treatement is 6 months of ANG, MD (TOP 1 - EXAM - FEB
notably hairless. Which of the following is true? dapsone and rifampin. AUG 2013 MED 2014
a.) The humoral response involve is mostly that of the BOARDS; TOPNOTCH
TH2 arm. MD)
b.) Treatment may require the use of clofazimine for 6
months to control the disease.
c.) Management of the patient will involve only dapsone
and rifampin.
d.) The use of dapsone is implicated with the
characteristic discoloration of the skin of treated leprosy
patients.
571 Which of the following is employed by prokaryotes for Cilia is present in eukaryotes (eg Balantidium BLAKE WARREN MIDTERM 2
locomotion? coli which is a protozoan—a eukaryote) ANG, MD (TOP 1 - EXAM - FEB
a.) Cilia Flagella is used by prokaryotes. AUG 2013 MED 2014
b.) Flagella BOARDS; TOPNOTCH
c.) Pseudopodia MD)
d.) Pili

572 Culture of haemophilus can be difficult since they may BLAKE WARREN MIDTERM 2
require additional factors for growth like x factor and V ANG, MD (TOP 1 - EXAM - FEB
factor. Which of the following haemophilus require only AUG 2013 MED 2014
the x factor for growth? BOARDS; TOPNOTCH
a.) H. influenza MD)
b.) H. haemolyticus
c.) H. ducreyi
d.) H. parahaemolyticus

TOPNOTCH MEDICAL BOARD PREP MICROBIOLOGY SUPEREXAM Page 71 of 90


For inquiries visit www.topnotchboardprep.com.ph or email us at topnotchmedicalboardprep@gmail.com
TOPNOTCH MEDICAL BOARD PREP MICROBIOLOGY SUPEREXAM
For inquiries visit www.topnotchboardprep.com.ph or email us at topnotchmedicalboardprep@gmail.com
Item QUESTION EXPLANATION AUTHOR TOPNOTCH
# EXAM
573 Growth of campylobacter can be done on Skirrow’s The antimicrobials used for this agar are BLAKE WARREN MIDTERM 2
medium and Campylobacter agar with 5% sheeps blood cephalotin, trimethoprim, vancomycin, ANG, MD (TOP 1 - EXAM - FEB
and five antimicrobials to suppress the growth of other polymyxin B and amphotericin B. Henrys AUG 2013 MED 2014
organisms from a fecal specimen. Which of the following 22nd ed BOARDS; TOPNOTCH
antibiotics is not used for this agar? MD)
a.) cephalosporin
b.) trimethoprim
c.) vancomycin
d.) polymyxin B

574 Which among the following species of Vibrio causes the vulnificus are acquired from wounds and BLAKE WARREN MIDTERM 2
most severe disease? result to septicemia ANG, MD (TOP 1 - EXAM - FEB
a.) Vibrio parahaemolyticus AUG 2013 MED 2014
b.) Vibrio vulnificus BOARDS; TOPNOTCH
c.) Vibrio cholera MD)
d.) Vibrio mimicus

575 Which among the following filarial worms is noted for its BLAKE WARREN MIDTERM 2
diurnal periodicity and hence should alert the physician ANG, MD (TOP 1 - EXAM - FEB
to the timing of blood extractions for the detection of the AUG 2013 MED 2014
parasites? BOARDS; TOPNOTCH
a.) Wuchererria banfrofti MD)
b.) Brugia malayi
c.) Onchocerca volvulus
d.) Loa loa

576 A single adult ascaris produces a lot of eggs per day such more than 100. Henrys 22nd ed BLAKE WARREN MIDTERM 2
that even a single worm in the duodenum can be ANG, MD (TOP 1 - EXAM - FEB
detected by the usual fecalysis. How many eggs per slide AUG 2013 MED 2014
is indicative of a heavy ascaris infection? BOARDS; TOPNOTCH
a.) More than 20 MD)
b.) More than 50
c.) More than 100
d.) More than 200

577 Rapid testing for presumptive identification of fungi can BLAKE WARREN MIDTERM 2
be done following colonial formation. One of such tests is ANG, MD (TOP 1 - EXAM - FEB
the urease test which if positive is presumptive proof of AUG 2013 MED 2014
infection with: BOARDS; TOPNOTCH
a.) Cryptococcus neoformans MD)
b.) Candida albicans
c.) Malassezia furfur
d.) Saccharomyces cerevisiae

578 Culture for mycobacteria can be done on solid media Answer: Henrys 22nd ed p1151 BLAKE WARREN MIDTERM 2
which are either egg-based (Lowenstein Jensen media) ANG, MD (TOP 1 - EXAM - FEB
or agar-based (Middlebrook). To detect the presence of AUG 2013 MED 2014
growth: BOARDS; TOPNOTCH
a.) the cultures should be checked daily for at least 6 MD)
weeks
b.) the cultures should be checked weekly for at least 6
weeks
c.) the cultures should be checked daily for at least 8
weeks
d.) the cultures should be checked weekly for at least 8
weeks
579 If a patient has a load of about 20 adult Necator Each adult necator consumes BLAKE WARREN MIDTERM 2
americanus worms, what is the expected blood loss per 0.03ml/worm/day while each adult ANG, MD (TOP 1 - EXAM - FEB
day secondary to the hookworm infection? ancylostoma consumes 0.15-0.25/worm/day AUG 2013 MED 2014
a.)0.3 ml BOARDS; TOPNOTCH
b.) 0.6 ml MD)
c.) 0.9 ml
d.) 1.2 ml

580 Patient is a known case of colon cancer. One day he BLAKE WARREN MIDTERM 2
presented to you at ER with high fever, appearing septic. ANG, MD (TOP 1 - EXAM - FEB
Blood cultures were drawn. While waiting for the AUG 2013 MED 2014
results, what is the most probable cause of the sepsis for BOARDS; TOPNOTCH
this patient? MD)
a.) E. coli
b.) Pseudomonas
C.) Klebsiella
d.) S. bovis

TOPNOTCH MEDICAL BOARD PREP MICROBIOLOGY SUPEREXAM Page 72 of 90


For inquiries visit www.topnotchboardprep.com.ph or email us at topnotchmedicalboardprep@gmail.com
TOPNOTCH MEDICAL BOARD PREP MICROBIOLOGY SUPEREXAM
For inquiries visit www.topnotchboardprep.com.ph or email us at topnotchmedicalboardprep@gmail.com
Item QUESTION EXPLANATION AUTHOR TOPNOTCH
# EXAM
581 In the biological world, which organisms and/or The 3 Domains are Eubacteria, Archaea, and TIMOTHY TANG LEE MIDTERM 1
pathogens is not included in the three large domains? Eukarya. Fungi and Protozoa belong to the SAY, MD (TOP 4 - EXAM - FEB
A. Bacteria domain of Eukarya. Viruses are not included AUG 2013 MED 2014
B. Virus in the taxonomy of biological organisms. BOARDS; TOPNOTCH
C. Archaea MD)
D. Fungi
E. Protozoa

582 These are infective pathogens that does not have any Plasmid have DNA. Bacteriopahge is a type of TIMOTHY TANG LEE MIDTERM 1
genetic material in their structure? virus. Prions are protein molecules that infect SAY, MD (TOP 4 - EXAM - FEB
A. Bacteria by misfolding other correctly folded proteins. AUG 2013 MED 2014
B. Plasmid BOARDS; TOPNOTCH
C. Virus MD)
D. Prion
E. Bacteriophage

583 The following are true about bacterial cell walls Endotoxins (Lipopolysaccharide) are usually TIMOTHY TANG LEE MIDTERM 1
EXCEPT? found in Gram-negative cell walls. Only SAY, MD (TOP 4 - EXAM - FEB
A. Gram-positive cell walls have a thicker Listeria monocytogenes has endotoxins among AUG 2013 MED 2014
peptidoglycan layers with teichoic acid found In their all Gram positive bacteria. BOARDS; TOPNOTCH
membranes. MD)
B. Gram-positive cell walls usually contain an
endotoxin in their outer membrane.
C. Certain bacteria like Mycoplasma lack cell walls that
is why β-lactams antibiotics are ineffective against them.
D. Peptidoglycan consists of alternating sugar residues
of N-acetylglucosamine and N-acetylmuramic acid with
attached peptide chains of 3-5 amino acids.
E. Gram-positive cell walls are usually resistant to
decolorizing agents.
584 Which of the following are NOT facultative anaerobes? Streptococcus is a microaerophile. TIMOTHY TANG LEE MIDTERM 1
A. Staphylococcus sp. SAY, MD (TOP 4 - EXAM - FEB
B. Streptococcus sp. AUG 2013 MED 2014
C. Escherichia sp. BOARDS; TOPNOTCH
D. Bacillus sp. MD)
E. All of the above

585 Which of the following is the best test to document Anti-DNAse B has the best sensitivity for TIMOTHY TANG LEE MIDTERM 1
antecedent Group A Streptococcus skin infection? documenting antecedent skin infection among SAY, MD (TOP 4 - EXAM - FEB
A. Anti-streptolysin O all the choices. AUG 2013 MED 2014
B. Anti-DNAse B BOARDS; TOPNOTCH
C. Anti-streptokinase MD)
D. Anti- NADase
E. All of the above

586 A chronically-ill patient was hospitalized for 7 days for This is a classic case of Pseudomembranous TIMOTHY TANG LEE MIDTERM 1
the in-patient treatment of community-acquired colitis caused by Clostridium difficile. The SAY, MD (TOP 4 - EXAM - FEB
pneumonia - moderate risk. He was given a combination treatemnt is oral metronidazole or AUG 2013 MED 2014
of IV antibiotics to treat the infection, however he vancomycin. BOARDS; TOPNOTCH
recently had acute onset of non-bloody diarrhea. MD)
Colonoscopy showed yello-white plaques in the colonic
mucosa. What is the treatment?
A. Add an anti-fungal to the antibiotic regimen
B. Discontinue all antibiotics and increased hydration
C. Give oral vancomycin aside from the IV antibiotics
D. Continue all medications and observe
E. Give steroids while continuing antibiotic therapy

587 What arre the causes of early-onset neonatal sepsis in All are the top 3 causes of early-onset neonatal TIMOTHY TANG LEE MIDTERM 1
the Philippines? sepsis. Group B Strep is S. agalactiae. SAY, MD (TOP 4 - EXAM - FEB
A. Escherichia coli AUG 2013 MED 2014
B. Streptococcus agalactiae BOARDS; TOPNOTCH
C. Listeria monocytogenes MD)
D. Only A and C
E. All of the above

588 Which is NOT TRUE about the differences between the Neissieria meningitides is susceptible to TIMOTHY TANG LEE MIDTERM 1
Neisseria sp.? penicillins while Neisseria gonorrhea is usallly SAY, MD (TOP 4 - EXAM - FEB
A. Neisseria meningitides can ferment both maltose and treated using higher order β-lactams such as AUG 2013 MED 2014
glucose while Neisseria gonorrhea cannot ferment ceftriaxone. BOARDS; TOPNOTCH
maltose. MD)
B. Neisseria meningitides is resistant to penicillin while
Neisseria gonorrhea is usually susceptible to penicillin.
C. Neisseria meningitides causes meningitis while
Neisseria gonoorrhea causes pelvic inflammatory
disease.
D. Neisseria menigitides inhabits the upper respiratory
tract, while Neisseria gonorrhea inhabits the gential tract.
E. All are correct

TOPNOTCH MEDICAL BOARD PREP MICROBIOLOGY SUPEREXAM Page 73 of 90


For inquiries visit www.topnotchboardprep.com.ph or email us at topnotchmedicalboardprep@gmail.com
TOPNOTCH MEDICAL BOARD PREP MICROBIOLOGY SUPEREXAM
For inquiries visit www.topnotchboardprep.com.ph or email us at topnotchmedicalboardprep@gmail.com
Item QUESTION EXPLANATION AUTHOR TOPNOTCH
# EXAM
589 Which of the following is true regarding the time at All are correct. Bone marrow is positive from TIMOTHY TANG LEE MIDTERM 1
presentation and culture source for a patient with 1st to 4th week. SAY, MD (TOP 4 - EXAM - FEB
Typhoid fever? AUG 2013 MED 2014
A. 1st week - Blood BOARDS; TOPNOTCH
B. 2nd week - Urine MD)
C. 3rd week - Stool
D. 4th week - Bone Marrow
E. All of the above

590 What virulence factor in Mycobacterium tuberculosis is Remember: TIMOTHY TANG LEE MIDTERM 1
responsible in preventing phagosome-lysosome fusion? Cord factor - prevents leukocyte migration SAY, MD (TOP 4 - EXAM - FEB
A. Sulfatides Tuberculin - delayed hypersensitivity AUG 2013 MED 2014
B. Cord factor Luciferase - bioluminiscence BOARDS; TOPNOTCH
C. Tuberculin surface protein BCG - vaccine against TB MD)
D. Luciferase
E. Bacillus Calmette-Guerin

591 Which of the following are NOT included in the Mycobacterium tuberculosis complex which TIMOTHY TANG LEE MIDTERM 1
Nontuberculous mycobacterium (NTM) classification? can cause tuberculosis: M. tuberculosis, M. SAY, MD (TOP 4 - EXAM - FEB
A. Mycobacterium chelonae bovis, M. africanum, M. microti and M. canetti. AUG 2013 MED 2014
B. Mycobacterium abscessus M. leprae has a different classification. BOARDS; TOPNOTCH
C. Mycobacterium africanum Mycobacterium avium complex includes M. MD)
D. Mycobacterium fortuitum avium and M. intracellulare
E. All are included The rest of the other mycobacterium species
are classified broadly as NTM.

592 What is TRUE regarding the ocular manifestation of Argyll Robertson pupils aka "Prostitute's TIMOTHY TANG LEE MIDTERM 1
tertiary syphilis? pupil" because it can accommodate (good near SAY, MD (TOP 4 - EXAM - FEB
A. The pupils has good near reflex when viewing reflex) but does not react (no light reflex). AUG 2013 MED 2014
objects at varying distances but has no pupillary light B - describes Adie's pupil BOARDS; TOPNOTCH
reflex. C - describes Marcus Gunn pupil MD)
B. The pupils are large with very slow light reflex D - describes Horner's syndrome
caused by denervation of post-ganglionic supply
C. The pupils react paradoxically to light
D. The pupils are small, accompanied by ptosis and
anhidrosis.
E. The pupils cannot accomodate but can react to light.
593 What inclusion bodies is found in Chlamydial infection? Chlamydial infection - Halberstaedter- TIMOTHY TANG LEE MIDTERM 1
A. Henderson-Patterson bodies Prowazek bodies SAY, MD (TOP 4 - EXAM - FEB
B. Guarnieri bodies AUG 2013 MED 2014
C. Cowdry Type A Cowdry type A - Herpes, CMV, VZV ; type B - BOARDS; TOPNOTCH
D. Cowdry Type B Polio, Adenovirus MD)
E. None of the above Guarnieri - Vaccinia
Henderson - Patterson - Molluscum
contagiosum (poxvirus)
594 A patient with suffering from AIDS has sudden onset Trimethoprim-sulfamethoxazole (TMP-SMX) TIMOTHY TANG LEE MIDTERM 1
difficulty of breathing. Bronchial washing showed PCP is the drug of choice for Pneumocystis jiroveci. SAY, MD (TOP 4 - EXAM - FEB
pneumonia, what is the drug of choice? AUG 2013 MED 2014
A. Amoxicillin BOARDS; TOPNOTCH
B. Metronidazole MD)
C. TMP-SMX
D. Fluconazole
E. Amphotericin B

595 Which component of the immune system is primarily Lymphocytes (B and T cells) are the most TIMOTHY TANG LEE MIDTERM 1
responsible for immunity against viruses? important cells against viruses. Neutrophils SAY, MD (TOP 4 - EXAM - FEB
A. Neutrophils are important against pyogenic bacteria. AUG 2013 MED 2014
B. Macrophages Macrophages phagocytose bacteria and to a BOARDS; TOPNOTCH
C. Lymphocytes lesser extent fungi. Eosinophils are important MD)
D. Eosinophils mediators against parasites. Complement is
E. Complement important for agaisnt Neisseria.

596 Which Human Papilloma Virus serotypes is responsible All are oncogenic strains. 6 and 11 are TIMOTHY TANG LEE MIDTERM 1
for causing cervical cancer? responsible for genital warts. 1-4 are SAY, MD (TOP 4 - EXAM - FEB
A. 16 responsible for skin and plantar warts.\ AUG 2013 MED 2014
B. 18 BOARDS; TOPNOTCH
C. 31 MD)
D. 33
E. All of the above

597 Which Hepatitis virus s a DNA virus? A, C and E are RNA viruses. TIMOTHY TANG LEE MIDTERM 1
A. A D is an incomplete virus. SAY, MD (TOP 4 - EXAM - FEB
B. B AUG 2013 MED 2014
C. C BOARDS; TOPNOTCH
D. D MD)
E. All of the above

TOPNOTCH MEDICAL BOARD PREP MICROBIOLOGY SUPEREXAM Page 74 of 90


For inquiries visit www.topnotchboardprep.com.ph or email us at topnotchmedicalboardprep@gmail.com
TOPNOTCH MEDICAL BOARD PREP MICROBIOLOGY SUPEREXAM
For inquiries visit www.topnotchboardprep.com.ph or email us at topnotchmedicalboardprep@gmail.com
Item QUESTION EXPLANATION AUTHOR TOPNOTCH
# EXAM
598 Which of the following parasites can mebendazole be Although it is the drug of choice only for TIMOTHY TANG LEE MIDTERM 1
used as a treatment regimen? Trichuris, in can still be used as an effective SAY, MD (TOP 4 - EXAM - FEB
A. Ascaris lumbricoides treatment regimen against all nematodes. AUG 2013 MED 2014
B. Trichuris trichiura BOARDS; TOPNOTCH
C. Enterobius vermicularis MD)
D. Strongyloides stercoralis
E. All of the above

599 Which is FALSE regarding malaria infection? Man is only an intermediate host. TIMOTHY TANG LEE MIDTERM 1
A. The greatest morbidity and mortality occurs when SAY, MD (TOP 4 - EXAM - FEB
the infection is due to Plasmodium falciparum. AUG 2013 MED 2014
B. The definitive host is man. BOARDS; TOPNOTCH
C. Relapse is possible when the infection is due to P. MD)
vivax or P ovale.
D. In the Philippines, Palawan has high endemicity and
high chloroquine resistance rates.
E. All are true

600 A 25 yr old male is fond of eating undercooked crabs, he Eating raw crabs is a risk factor fro TIMOTHY TANG LEE MIDTERM 1
also has poor hygiene. He had a 3 week history of Paragonimus westermani infection. SAY, MD (TOP 4 - EXAM - FEB
chronic cough with bloody sputum, the diagnosis is most AUG 2013 MED 2014
likely? BOARDS; TOPNOTCH
A. Histoplasmosis MD)
B. Tuberculosis
C. Paragonimiasis
D. Loeffler's pneumonia
E. Bronchiectasis
601 Which if the following is INCORRECT regarding epidemic Epidemic typhus is transmitted by human RACHELLE FINAL EXAM
typhus? body louse. MENDOZA, MD (TOP - FEB 2013
A. The causative organism is transmitted from rodents to 9 - AUG 2012 MED
human by a tick BOARDS; TOPNOTCH
B. The disease is characterized by a rash MD)
C. The Weil-Felix test can aid in the diagnosis of the
disease
D. The disease is caused by Rickettsia
E. None of the above
602 A 30-year old man, who worked as a volunteer in the CDC recommends that adults who might be at RACHELLE FINAL EXAM
rescue mission during the Habagat, suddenly developed increased risk for leptospirosis be advised to MENDOZA, MD (TOP - FEB 2013
high fever, intense headache, calf pains and consider chemoprophylaxis with doxycycline 9 - AUG 2012 MED
conjunctivitis. He came in for cunsult due to (200 mg orally, weekly), begun 1–2 days BOARDS; TOPNOTCH
development of jaundice. Examination revealed before and continuing through the period of MD)
impaired hepatic and renal functions. What is/are the exposure.
prophylactic regimen/s that should have been given in
this patient?
A. Penicillin G 1.2 MU 1-2 days before exposure then
daily during exposure
B. Penicillin G 1.2 MU 1-2 days after exposure then
continue daily for 1 week
C. Doxycyline 200 mg PO weekly 1-2 days before
exposure then continuing through the period of
exposure
D. Doxycyline 200 mg PO daily 1-2 days before exposure
then continuing through the period of exposure
E. All of the above
603 Antibiotic-associated diarrhea is due to overgrowth of Clostridium perfringes produces alpha toxin RACHELLE FINAL EXAM
this organism. Which of the following is the correct as described, thereby causing gas gangrene. MENDOZA, MD (TOP - FEB 2013
description for this organism? Corynebacterium diphtheriae inhibits 9 - AUG 2012 MED
A. Releases alpha toxin, a lecithinase that cleaves cell eukaryotic elongation factor 2 by ADP BOARDS; TOPNOTCH
membranes ribosylation to inhibiting protein synthesis, MD)
B. Produces exotoxins A and B, which inhibit GTPases, and thereby causing pseudomembranous
leading to apoptosis of enterocytes pharyngitis.
C. Inhibits protein synthesis by ADP ribosylation of
elongation factor 2
D. A and B
E. B and C
604 A 16-year old male living in a college dormitory The causative organism is Neisseria RACHELLE FINAL EXAM
suddenly developed fever, headache, neck rigidity meningitidis. This irganis is gram negative, MENDOZA, MD (TOP - FEB 2013
followed by an episode of generalized seizure. CSF kidney-bean diplococci. Habitat is the URT, 9 - AUG 2012 MED
studies showed gram negative diplococci. Which of the transmission occurs via respiratory droplet, BOARDS; TOPNOTCH
following is/are true regarding this organism? hence there is high carriage among people MD)
A. No significant resistance to penicillin living in close quarters (dormitory). Humans
B. Ferments both glucose and maltose are the only natural host. Treatment of choice
C. Transmission is via respiratory droplets is Pen G as this organism shows no significant
D. All of the above resistance to this drug.
E. B and C
605 Gullain Barre syndrome has been associated with what Campylobacter jejuni is the most common RACHELLE FINAL EXAM
bacterial infection? cause of bacterial gastroenteritis. It is MENDOZA, MD (TOP - FEB 2013
A. Helicobacter pylori associated with Guillain barre syndrome and 9 - AUG 2012 MED
B. Corynebacterium diphtheriae reiter's syndrome BOARDS; TOPNOTCH
C. Klebsiella pneumoniae MD)
D. Campylobacter jejuni
E. None of the above

TOPNOTCH MEDICAL BOARD PREP MICROBIOLOGY SUPEREXAM Page 75 of 90


For inquiries visit www.topnotchboardprep.com.ph or email us at topnotchmedicalboardprep@gmail.com
TOPNOTCH MEDICAL BOARD PREP MICROBIOLOGY SUPEREXAM
For inquiries visit www.topnotchboardprep.com.ph or email us at topnotchmedicalboardprep@gmail.com
Item QUESTION EXPLANATION AUTHOR TOPNOTCH
# EXAM
606 An HIV positive 30-year old male developed cough, fever Nocardia is a genus of weakly staining gram- RACHELLE FINAL EXAM
and dyspnea. Work-up showed lung abscess and sputum positive, catalase-positive, rod-shaped MENDOZA, MD (TOP - FEB 2013
exam showed gram positive, aerobic filments which are bacteria. It forms partially acid-fast beaded 9 - AUG 2012 MED
acid fast. The most probably etiologic organism is: branching filaments (acting as fungi, but being BOARDS; TOPNOTCH
A. Mycobacterium avium-intracellulare truly bacteria). MD)
B. Nocardia
C. Pneumocystis carinii
D. Mycobacterium kansasii
E. A or D

607 Generalized disseases in which the virus spreads in the All herpes viruses travel through the RACHELLE FINAL EXAM
body through the blood stream, affecting multiple bloodstream and affect multiple organs. MENDOZA, MD (TOP - FEB 2013
organs, include all of the following, EXCEPT: 9 - AUG 2012 MED
A. Measles BOARDS; TOPNOTCH
B. Rubella MD)
C. Chlamydia
D. Chickenpox
E. Herpes zoster

608 Which of the following correctly describes the only Parvovirus B 19 is a single-stranded DNA RACHELLE FINAL EXAM
single-stranded DNA virus pathologic to humans? virus, which is naked with icosahedral MENDOZA, MD (TOP - FEB 2013
A. It is an enveloped virus with icosahedral symmetry nucleocapsid. It has only 1 serotype and 9 - AUG 2012 MED
B. The only virus with fiber causes: erythema infectiosum (fifth disease), BOARDS; TOPNOTCH
C. It is the most common cause of congenital aplastic crisis, fetal infections, arthritis and MD)
abnormalities pancytopenia during chronic infection.
D. Causes the fifth disease Adenovirus is the only virus with fiber. CMV is
E. All of the above the most common cause of congenital
abnormalities.
609 Candida albicans is part of the normal flora of URT, GIT Candida albicans causes vulvovaginitis in RACHELLE FINAL EXAM
and female GUT. Which of the following is true immunocompetent host and MENDOZA, MD (TOP - FEB 2013
regarding this organism? pseudomenranous esophagitis in 9 - AUG 2012 MED
A. Causes vulvovaginitis with curd-like discharge in immunocompromised host. It forms germ BOARDS; TOPNOTCH
immunocompetent host tubes in serum and chlamydo-spores in MD)
B. Causes pseudomembranous vulvovaginitis in culture.
immunocompromised host
C. Form germ tubes in culure and chlamydo-spores in
serum
D. A and C
E. B and C
610 A 45-year old female went to Mississippi for her Blastomyces dermatitidis is the most common RACHELLE FINAL EXAM
vacation. 2 weeks after, she develop productive cough, cause of fungal pneumonia. MENDOZA, MD (TOP - FEB 2013
with whitish phlegm. She took consult and was given 9 - AUG 2012 MED
antibiotic, but no relief was noted. She then developed BOARDS; TOPNOTCH
dyspnea as cough persisted for 2 weeks. If you suspect MD)
fungi as the causative agent, which of the following is the
most probable culprit?
A. Coccidioides immitis
B. Histoplasma capsulatum
C. Paracoccidioides brasiliensis
D. Blastomyces dermatitidis
E. A or C

611 During an annual check up, a 34-year old engineer, E.histolytica cysts contain 1-4 nuclei, with RACHELLE FINAL EXAM
asymptomatic, was found to have several cysts in his rounded chromatoid bodies. This is MENDOZA, MD (TOP - FEB 2013
stool specimen. These cyst showed 4 nuclei with microscopically indistinguishable from 9 - AUG 2012 MED
rounded chromatoid bodies. Which of the following E.dispar cysts. E.coli, on the other hand, has a BOARDS; TOPNOTCH
is/are the most probable organism involved? cyst with 8 nuclei with spiculated MD)
A. Entamoeba coli chromatoidal body.
B. Entamoeba histolytica
C. Entamoeba dispar
D. B or C
E. A or C

612 Schistosoma japonicum is an oriental blood fluke that Schistosoma spp. has no second intermediate RACHELLE FINAL EXAM
resides in the mesenteric and portal veins. Which of the host. Its first and only intermediate host is MENDOZA, MD (TOP - FEB 2013
following is its second intermediate host? Oncomelania hupensis quadrasi. 9 - AUG 2012 MED
A. Oncomelania hupensis quadrasi BOARDS; TOPNOTCH
B. Antemelania asperata MD)
C. Sundathelphusa philipina
D. Parafossarulus
E. None of the above

613 Which of the following is/are true regarding pinworm? All of the choices correctly describes RACHELLE FINAL EXAM
A. Caused by Enterobius vermicularis pinworm. MENDOZA, MD (TOP - FEB 2013
B. Transmitted by ingestion and inhalation of eggs 9 - AUG 2012 MED
C. Most common soil-transmitted helminth in developed BOARDS; TOPNOTCH
countries MD)
D. Auto-infection may occur
E. All of the above

TOPNOTCH MEDICAL BOARD PREP MICROBIOLOGY SUPEREXAM Page 76 of 90


For inquiries visit www.topnotchboardprep.com.ph or email us at topnotchmedicalboardprep@gmail.com
TOPNOTCH MEDICAL BOARD PREP MICROBIOLOGY SUPEREXAM
For inquiries visit www.topnotchboardprep.com.ph or email us at topnotchmedicalboardprep@gmail.com
Item QUESTION EXPLANATION AUTHOR TOPNOTCH
# EXAM
614 A 56-year old man, alcoholic, was brought to your clinic Strep pneumoniae is a gram positive cocci, RACHELLE FINAL EXAM
due to expectoration of blood-tinged sputum. lancet-shaped, and commonly presents with MENDOZA, MD (TOP - FEB 2013
Microscopic study of the sputum revealed lancet shaped, rust colored sputum. 9 - AUG 2012 MED
gram positive organisms. Which of the following is the BOARDS; TOPNOTCH
most likely causative agent? MD)
A. Streptococcus pneumoniae
B. Klebsiella pneumoniae
C. Haemophilus influenzae
D. Mycobacterium tuberculosis
E. Pseudomonas aeruginosa
615 A 50-year old abaca farmer in Samar consulted at the The patient has acute filariasis. Presenting RACHELLE FINAL EXAM
rural health center due to fever, cough, nocturnal with filarial fever, acute adenolymphangitis MENDOZA, MD (TOP - FEB 2013
wheezing, with pain and swelling of the groin and axillae. and nocturnal wheezing. This usually 9 - AUG 2012 MED
Which of the following is/are consistent with this clinical progresses to a chronic disease, characterized BOARDS; TOPNOTCH
finding? by lymphedema, causing hydrocoele and/or MD)
A. Usually progresses to lymphedema elephantiasis. Thick blood smear is usally
B. Thick blood smear will show curved or kinky done between 8PM and $am because the
microfilariae kinky or curved microfilariae exhibit
C. Organism exhibits nocturnal periodicity nocturnal periodicity. This is endemic in
D. All of the above Samar.
E. B and C
616 A patient on chronic total parenteral nutrition (TPN) All of the statements correctly describes RACHELLE FINAL EXAM
developed white patches around his chin, trunk and Malassezia furfur. MENDOZA, MD (TOP - FEB 2013
diffusely around his extremities. Which of the following 9 - AUG 2012 MED
is/are INCORRECT regarding this organism? BOARDS; TOPNOTCH
A. Causes superficial skin infection of cosmetic MD)
importance
B. May lead to an invasive infection for this particular
patient
C. Diagnosed by skin scrapings, that will show "spaghetti
and meatball" appearance on 10% KOH
D. Treatment include itraconazole (oral) and
ketoconazole (topical)
E. None of the above
617 A 63-year old man had visited his favorite oyster Such signs and symptoms, given the history of RACHELLE FINAL EXAM
restaurant near a shore. He ate two dozens of oysters. large intake of fresh oysters, will most MENDOZA, MD (TOP - FEB 2013
Two days later, he was admitted because of the abrupt probably point to V.vulnificus as the causative 9 - AUG 2012 MED
onset of chills, fever and lightheadedness. While in the agent. BOARDS; TOPNOTCH
ER, he developed erythematous skin lesions, which MD)
rapidly evolved into hemorrhagic bullae and eventually
ulcers. The microorganism of major concern for this
patient is:
A. Vibrio vulnificus
B. Escherichia coli
C. Salmonella typhi
D. Clostrodium perfringens
E. Group A streptococci
618 An 8-year old boy develops severe sore throat. On Corynebacterium diohtheriae causes RACHELLE FINAL EXAM
examination, a grayish exudate (pseudomembrane) was pseudmebranous pharyngitis. It is described MENDOZA, MD (TOP - FEB 2013
seen over the tonsils and pharynx. The cause of this as a club-shaped gram psotive bacillus that 9 - AUG 2012 MED
boy's pharyngitis is most likely: produces metachromatic granules. They form BOARDS; TOPNOTCH
A. A gram-negative bacillus black colonies on tellurite plate. MD)
B. A single-stranded, positive sense RNA virus
C. A club-shaped gram positive bacillus
D. A double stranded RNA virus
E. A catalse-positive gram positive coccus that grows in
clusters
619 A 48-year old HIV-positive man with CD4 count of 40 JC virus only causes disease in RACHELLE FINAL EXAM
complains of progressive memory loss. Five moths later, immunocompromised hosts. It causes MENDOZA, MD (TOP - FEB 2013
he became paralyzed and died. Autopsy revealed progressive multifocal leukoencephaopathy 9 - AUG 2012 MED
demyelination of many neurons and electron microscopy (PML) in patients with AIDS. This is BOARDS; TOPNOTCH
shows clusters of naked viral particles. What is the most characetrized as a demyelinating disease MD)
likely cause of the disease? affecting oligodendrocytes, characterized by
A. Adenovirus deifcits in speech, coordination and memory.
B. Coxsackie virus B2 JC virus is a double stranded, naked DNA virus
C. Parvovirus B19 with icosahedral nucleocapsid and circular
D. Epstein-Barr virus genome.
E. JC virus
620 The following microorganisms may cause malignancy in Hepatitis B can cause hepatic carcinoma. EBV RACHELLE FINAL EXAM
human host, EXCEPT: can cause Burkitt's lymphoma and MENDOZA, MD (TOP - FEB 2013
A. Cytomegalovirus nasopharyngeal carcinoma. HHV 8 causes 9 - AUG 2012 MED
B. Hepatitis B virus kaposi's sarcoma. HTLV causes adult T-cell BOARDS; TOPNOTCH
C. Epstein-Barr virus leukemia. MD)
D. Human herpesvirus 8 (HHV8)
E. Human T-cell lymphotropic virus (HTLV)

TOPNOTCH MEDICAL BOARD PREP MICROBIOLOGY SUPEREXAM Page 77 of 90


For inquiries visit www.topnotchboardprep.com.ph or email us at topnotchmedicalboardprep@gmail.com
TOPNOTCH MEDICAL BOARD PREP MICROBIOLOGY SUPEREXAM
For inquiries visit www.topnotchboardprep.com.ph or email us at topnotchmedicalboardprep@gmail.com
Item QUESTION EXPLANATION AUTHOR TOPNOTCH
# EXAM
621 A truck driver was involved in a serious accident and Pseudomonas aeruginosa VON ANDRE DIAGNOSTIC
received second and third degree burns over his body. MEDINA, MD (TOP 4 EXAM - AUG
He was placed in the burn unit and on his twelfth - FEB 2012 MED 2012
hospital day, he suddenly developed a wound infection BOARDS; TOPNOTCH
with a bluish green exudate. Treatment with MD)
chlorampenicol and tetracycline was unsuccessful. A
gram negative, motile organism was isolated that was
oxidase positive, did not ferment lactose, sucrose or
glucose but grew on MacConkey's agar and produced a
fruity aroma on that medium. Which of the following
organisms was most likely isolated?
A. Serratia marcescens
B. Clostridium perfringens
C. Pseudomonas aeruginosa
D. Candida albicans
E. Any of the above
622 A group of 25 fifth grade students had lunch at a fast EHEC VON ANDRE DIAGNOSTIC
food restaurant. Most ate hamburgers and French fries MEDINA, MD (TOP 4 EXAM - AUG
prepared quickly for the noontime rush. The next day - FEB 2012 MED 2012
most of the children experienced severe bloody diarrhea BOARDS; TOPNOTCH
and nausea with several requiring hospitalization. Stool MD)
and blood cultures recovered a small, gram-negative,
non-mucoid bacilli which fermented lactose, were indole
positive, produced acid and carbon dioxide but produced
neither hydrogen sulfide nor urease. The organism most
likely responsible for the intestinal infection is:
A. Vibrio cholera
B. Proteus vulgaris
C. Salmonella typhi
D. E. hystolitica
E. Escherichia coli
623 A 4 year old boy presents to the pediatric emergency fluid analysis are consistent with bacterial VON ANDRE DIAGNOSTIC
department with the classic meningitis triad of fever, meningitis (high PMN cells, high protein, low MEDINA, MD (TOP 4 EXAM - AUG
headace and nuchal rigidity. A lumbar puncture is glucose. The MC cause of meningitis aged 6 - FEB 2012 MED 2012
performed and analysis of fluid shows an increase in months-6 years is S. pneumoniae.
BOARDS; TOPNOTCH
polymorphonuclear cells, an increased protein level, and H influenzae type b menigitis is less common MD)
a decreased glucose level. Which of the following is the cause in this age group. since the introduction
most comon cause of mengitis in a child of this age with of Hib vaccine, the incidence of this cause of
this clinical picture? meningitis has greatly decreased.
A. Enteroviruses
B. H. influenza type B
C. HSV
D. Listeria monocytogenes
E. Streptococcus pneumoniae
624 A 50 year old man develops non-bloody watery diarrhea Vibrio cholera causes watery stool VON ANDRE DIAGNOSTIC
while working as an aid worker in a refuge camp in accompanied by abdominal pain, but the MEDINA, MD (TOP 4 EXAM - AUG
Cambodia. He arrived in the area 3 days ago. A stool symptoms are due to dehydration leading to - FEB 2012 MED 2012
smear shows no WBCs but revealed a gram negative electrolyte abnormalities. Cholera toxin BOARDS; TOPNOTCH
curved bacilli, which is oxidase positive. He subsequently causes uncontrolled stimulation of adenylate MD)
develops dehydration and electrolyte abnormalities cyclase rsulting to excess cAMP causes
leading to cardiac and renal failure. Which of the uncontrolled secretion of chloride and water.
following organism is the most likely cause of this man's It is a gram negative curved bacilli and is
enterocolitis? oxidase positive.
A. Clostridium difficile
B. Giardia lamblia
C. H. pylori
D. Salmonella typhi
E. Vibrio cholera
625 A 31 year old pregnant woman comes to her attending ToRCHeS. The Tzanck test is a smear of an VON ANDRE DIAGNOSTIC
physician because of painful vesicular lesions that have opened skin vesicle that detects MEDINA, MD (TOP 4 EXAM - AUG
recently appeared in her genitalia. A positive result on multinucleated giant cells, indicative of HSV.
- FEB 2012 MED 2012
which of the following diagnostic tests would mean that "Tzanck Heaven I Don't Have Herpes!" BOARDS; TOPNOTCH
her baby is at risk for congenital anomalies? MD)
A. Anti-hepatitis B surface antibody test
B. Giemsa stain for cytoplasmic inclusions
C. Monospot test
D. Tzanck smear for multinulceated giant cells
E. Weil-felix test

626 What is the drug of choice for Trichuris trichuria Mebendazole is the DOC for T. trichuria VON ANDRE DIAGNOSTIC
infection? infection. SIMILAR TO PREVIOUS BOARD MEDINA, MD (TOP 4 EXAM - AUG
A. Albendazole EXAM CONCEPT/PRINCIPLE - FEB 2012 MED 2012
B. Mebendazole BOARDS; TOPNOTCH
C. Pyrantel pamoate MD)
D. Metronidazole
E. None of the above

TOPNOTCH MEDICAL BOARD PREP MICROBIOLOGY SUPEREXAM Page 78 of 90


For inquiries visit www.topnotchboardprep.com.ph or email us at topnotchmedicalboardprep@gmail.com
TOPNOTCH MEDICAL BOARD PREP MICROBIOLOGY SUPEREXAM
For inquiries visit www.topnotchboardprep.com.ph or email us at topnotchmedicalboardprep@gmail.com
Item QUESTION EXPLANATION AUTHOR TOPNOTCH
# EXAM
627 A 24 year old call center agent was diagnosed to have EAEC commonly od AIDS patient VON ANDRE DIAGNOSTIC
AIDS. One week after, he experienced persistent watery MEDINA, MD (TOP 4 EXAM - AUG
diarrhea secondary to a bacterial etiology. Which of the - FEB 2012 MED 2012
following is the most likely subtype of E. coli? BOARDS; TOPNOTCH
A. ETEC MD)
B. EPEC
C. EHEC
D. EAEC
E. EIEC

628 A 3 year old female came in due to persistent cough for VON ANDRE DIAGNOSTIC
more than 2 weeks, anorexia and night sweats. Sputum MEDINA, MD (TOP 4 EXAM - AUG
exam was consistent with PTB. In primary complex, - FEB 2012 MED 2012
evidence of radiographically detectable calcification of a BOARDS; TOPNOTCH
subpleural granuloma and its associated lymp node is MD)
called?
A. Ghon's complex
B. Simon's focus
C. Ranke's complex
D. Pastia's lines
E. None of the above
629 A psychotic indigent man with a history of multi- VON ANDRE DIAGNOSTIC
substance abuse has been involuntarily hospitalized for MEDINA, MD (TOP 4 EXAM - AUG
1 week because of persistent diarrhea. Stool sample was - FEB 2012 MED 2012
sent for ova and parasites, revealing numerous granular, BOARDS; TOPNOTCH
spherical, thin walled cyst measuring 10-20 um MD)
diameter. Trichrome stains show up to four nuclei in
most of the cysts. these findings are consistent with an
infection by which of the following organisms?
A. Cryptosporidium parvum
B. Dientamoeba fragilis
C. Giardia lamblia
D. Entamoeba hystolitica
E. Salmonella typhi

630 An 8-month-old, previously healthy, came into your HHV 6= roseola infantum VON ANDRE DIAGNOSTIC
clinic for high grade fever for 3 days duration. Further MEDINA, MD (TOP 4 EXAM - AUG
history revealed that on the 4th day, after the fever was - FEB 2012 MED 2012
resolved, a rose-red macular rashes 2-6 mm in diameter BOARDS; TOPNOTCH
on her trunk and proximal extremities appeared. What is MD)
the most likely etiologic agent in this case.
A. Rubeola
B. Rubella
C. HSV 1
D. HHV 6
E. VZV
631 A 14-year old patient is brought in by his parents Staph aureus grows well in mannitol salt, it is VON ANDRE DIAGNOSTIC
because of sore throat. On P.E., he is febrile and has a a gram positive cocci which is also beta MEDINA, MD (TOP 4 EXAM - AUG
pharyngeal erythema with a tonsillar abscess. A throat hemolytic produces yellow or golden colonies - FEB 2012 MED 2012
culture on sheep blood agar yields colonies of gram on blood agar. BOARDS; TOPNOTCH
positive cocci that are surrounded by a zone of complete MD)
hemolysis. The organism was also plated on mannitol
salt agar; it grew well and caused the medium to turn
gold. Which of the following microorganisms is the most
likely cause of the patient's illness?
A. Corynebacterium diphtheriae
B. Haemophilus influenza
C. Staphylococcus aureus
D. Streptococcus agalactiae
E. Streptococcus pyogenes
632 A 6 year old female came to your cllinic due to high MC cause of herpangina is Coxscakie A VON ANDRE DIAGNOSTIC
grade fever, sore throat and odynophagia. Tender MEDINA, MD (TOP 4 EXAM - AUG
vesicles in her oropharynx were noted. The diagnosis of - FEB 2012 MED 2012
herpangina was made. What is the most likely etiology? BOARDS; TOPNOTCH
A. HSV 1 MD)
B. HSV 2
C. EBV
D. Coxsakie A
E. Coxsackie B

633 The blood culture of a patient with a presumptive Enterococci, gram + cocci, non hemolytic, VON ANDRE DIAGNOSTIC
diagnosis of endocarditis yields a non-hemolytic colonies catalse -, grows in 6.5 % Na Cl. MEDINA, MD (TOP 4 EXAM - AUG
of gram positive cocci. The isolate was catalase negative - FEB 2012 MED 2012
and was noted that it grew in 6.5 % Na Cl and was BOARDS; TOPNOTCH
resistant to penicillin. The isolate is most likely? MD)
A. Enterococci
B. Strep agalactiae
C. Strep bovis
D. Strep mutans
E. Staph aureus

TOPNOTCH MEDICAL BOARD PREP MICROBIOLOGY SUPEREXAM Page 79 of 90


For inquiries visit www.topnotchboardprep.com.ph or email us at topnotchmedicalboardprep@gmail.com
TOPNOTCH MEDICAL BOARD PREP MICROBIOLOGY SUPEREXAM
For inquiries visit www.topnotchboardprep.com.ph or email us at topnotchmedicalboardprep@gmail.com
Item QUESTION EXPLANATION AUTHOR TOPNOTCH
# EXAM
634 A patient with AIDS presents with headaches of VON ANDRE DIAGNOSTIC
increasing severity and mental lethargy. On P.E, MEDINA, MD (TOP 4 EXAM - AUG
macronodlar skin lesions are noted. The CSF is clear, and - FEB 2012 MED 2012
protein and glucose concentrations are within normal BOARDS; TOPNOTCH
levels. On india ink wet mount, encapsulated yeasts can MD)
be seen. Which of the following methods will permit
rapid confirmation of the indentity of the infective
organism?
A. Detection of specific antibodies in the CSF
B. Detection of polysaccharide capsule in the CSF
C. Detection of lipooligosaccharides in the CSF
D. Detection of cell wall antigens in the CSF
E. All of the above
635 Which of the following diseases is caused by a virus that The hepa B virus, the hepadnavirus that VON ANDRE DIAGNOSTIC
uses a reverse transcriptase during replication? causes serum hepatitis, uses a virus encoded MEDINA, MD (TOP 4 EXAM - AUG
A. Infectious hepatitis reverse transcriptase during replication. - FEB 2012 MED 2012
B. Measles BOARDS; TOPNOTCH
C. Serum hepatitis MD)
D. Infectious mononucleosis
E. German measles

636 An alcoholic presents complaining of chest pain, fever, VON ANDRE DIAGNOSTIC
shaking chills, cough and myalgia. He was very cold 2 MEDINA, MD (TOP 4 EXAM - AUG
nights ago and says he felt "poorly" eversince. Her cough - FEB 2012 MED 2012
is producing rust-colored, odorless, mucoid sputum. His BOARDS; TOPNOTCH
temperature on admission is 40 C. His WBC count is MD)
16,000 cells/mm3 with PMN predominance with an
overall left shift. An alpha hemolytic, lancet-shaped,
gram positive diplococcus is isolated on blood agar.
What is the most likely causative agent?
A. Legionella pneumophilia
B. Klebsiella pneumoniae
C. Mycoplasma pneumoniae
D. Neisseria meningitidis
E. Streptococcus pneumoniae
637 Pieces of plasmid or bacterial chromosomal DNA may be genralized transduction, in which an error at VON ANDRE DIAGNOSTIC
mistakenly packaged into the capsid of a bacterial virus. the packaging stage creates a transducing MEDINA, MD (TOP 4 EXAM - AUG
Assuming that no prophage state has existed in the virus - FEB 2012 MED 2012
phage production, what is the process called when virus BOARDS; TOPNOTCH
with bacterial or plasmid DNA infects another MD)
bacterium?
A. Conjugation
B. Lysogeny
C. Transduction
D. Recombinant process
E. Transformation
638 A patient with sickle cell anemia is most likely to have Sickle cell anemia patients have problems VON ANDRE DIAGNOSTIC
repeated septicemias with? with septicemias with encapsulated organism. MEDINA, MD (TOP 4 EXAM - AUG
A. Candida albicans Of those listed only salmonella enteritidis has - FEB 2012 MED 2012
B. Nontypeable H. influenzae a prominent capsule and so is noted for BOARDS; TOPNOTCH
C. Mycobacterium avium-intracellulare causing repeated infections. MD)
D. Salmonella enteritidis
E. Shigella

639 A compromised patient is admitted due to respiratoy This is a case of Invasive aspergillosis. It is VON ANDRE DIAGNOSTIC
distress. She had signs of focal central nervous system found primarily in patients with neutrophil MEDINA, MD (TOP 4 EXAM - AUG
lesions early in the day and is now on the comatose state. count less than 500/mm3. - FEB 2012 MED 2012
The CNS and pulmonary biopsies show dichotomously BOARDS; TOPNOTCH
branching septate hyphae. What is the most likely MD)
underlying condition?
A. CD4 cell count < 200
B. Ketoacidosis diabetes
C. Multiple myeloma
D. Severe neutropenia
E. Sickle cell disease
640 A 16 year old female from a dormitory presents with This is a case of Meningococcemia. G (-) VON ANDRE DIAGNOSTIC
sudden onset of fever of 40 C, severe headache and kidney bean diplococci with large MEDINA, MD (TOP 4 EXAM - AUG
generalized petechial rash. Physical examination reveals polysaccharide capsule, oxidase positive and - FEB 2012 MED 2012
nuchal rigidity. CSF culture was done revealing an ferments maltose and glucose. Complement BOARDS; TOPNOTCH
organism which ferments both maltose and glucose with deficiencies in the late acting complement MD)
large polysaccharide capsule. Which one of the following components (C6-C9) PREDISPOSE TO ILLNESS
is the MOST likely predisposing factor to your diagnosis ( CANNOT FORM MEMBRANE-ATTACK
in this case? COMPLEXES)
A. She is a HIV antibody positive
B. She is deficient in CD8-positive cells
C. She is deficient on one of the late-acting complement
components
D. She is deficient in antigen presenting cells
E. None of the above

TOPNOTCH MEDICAL BOARD PREP MICROBIOLOGY SUPEREXAM Page 80 of 90


For inquiries visit www.topnotchboardprep.com.ph or email us at topnotchmedicalboardprep@gmail.com
TOPNOTCH MEDICAL BOARD PREP MICROBIOLOGY SUPEREXAM
For inquiries visit www.topnotchboardprep.com.ph or email us at topnotchmedicalboardprep@gmail.com
Item QUESTION EXPLANATION AUTHOR TOPNOTCH
# EXAM
641 Which among the following statements is/are true the abnormal forms contain more beta- LITO JAY MACARAIG, DIAGNOSTIC
regarding Prions? sheaths and NOT alpha-chains MD (TOP 8 - FEB EXAM - AUG
A. Prions exist in normal and abnormal forms 2013 MED BOARDS; 2013
B. Abnormal forms contains more alpha-chains and are TOPNOTCH MD)
insoluble in detergents
C. Incubation period may last upto 30 years
D. A and C only
E. All of the above
642 This/these bacterial component/s contain/s the β- Periplasm is the space between the bacterial LITO JAY MACARAIG, DIAGNOSTIC
lactamases which are responsible for drug resistance. plasma and outer membrane and contains the MD (TOP 8 - FEB EXAM - AUG
A. Granule hydrolytic enzymes including the β- 2013 MED BOARDS; 2013
B. Mesosome lactamases. Granules are sites for storage of TOPNOTCH MD)
C. Periplasm nutrients.
D. Endoplasm
E. A and C

643 The largest medically important bacteria is? The largest bacteria is Thiomargarita LITO JAY MACARAIG, DIAGNOSTIC
A. Borrelia burgdorferi namibiensis. But Borrelia burgdorferi is the MD (TOP 8 - FEB EXAM - AUG
B. Listeria monocytogenes largest with medical importance. 2013 MED BOARDS; 2013
C. Leigionella sp. TOPNOTCH MD)
D. Thiomargarita namibiensis
E. Nocardia

644 This the phase in the bacterial growth curve on which The LOG phase is the exponential phase. LITO JAY MACARAIG, DIAGNOSTIC
growth rate is constant. Growth rate is constant but still on a higher MD (TOP 8 - FEB EXAM - AUG
A. Lag phase degree compared to death rate. 2013 MED BOARDS; 2013
B. Log phase TOPNOTCH MD)
C. Stationary phase
D. Decline phase
E. Death phase

645 Bacteria may transfer its genetic information to other conjugation uses bacterial pilus to transfer LITO JAY MACARAIG, DIAGNOSTIC
species of bacteria in different ways. One of which is the plasmids (circular form of DNA, able to MD (TOP 8 - FEB EXAM - AUG
transfer of bacterial DNA through plasmids, that is replicate independently from the bacterial 2013 MED BOARDS; 2013
called? chromosome) TOPNOTCH MD)
A. Conjugation
B. Transduction
C. Transformation
D. A and B only
E. A and C only
646 This is the transfer of bacterial DNA with the use of pilus. conjugation uses bacterial pilus to transfer LITO JAY MACARAIG, DIAGNOSTIC
A. Conjugation plasmids (circular form of DNA, able to MD (TOP 8 - FEB EXAM - AUG
B. Transduction replicate independently from the bacterial 2013 MED BOARDS; 2013
C. Transformation chromosome) TOPNOTCH MD)
D. A and B only
E. A and C only

647 With the advent of technology, viruses can be used to Transduction uses a virus (called LITO JAY MACARAIG, DIAGNOSTIC
transfer bacterial genetic information from one bacterial bacteriophage) to carry genetic information MD (TOP 8 - FEB EXAM - AUG
specie to another. This process is called? from one bacteria to another. 2013 MED BOARDS; 2013
A. Conjugation TOPNOTCH MD)
B. Transduction
C. Transformation
D. A and B only
E. A and C only

648 The normal flora of the anterior nares is the? Carriers of Neisseria menigitidis are mainly LITO JAY MACARAIG, DIAGNOSTIC
A. Staph. aureus detected through their bacterial load in their MD (TOP 8 - FEB EXAM - AUG
B. N. meningitidis nasopharynx. Staphylococcus aureus is the 2013 MED BOARDS; 2013
C. Group A strep main inhabitant of human anterior nares. TOPNOTCH MD)
D. Group B strep
E. A and D only

649 All of the following are bacteriostatic, EXCEPT? In the code "buy AT 30, CELLS at 50", the only LITO JAY MACARAIG, DIAGNOSTIC
A. Doxycycline bactericidals are the A=Aminoglycoside (like MD (TOP 8 - FEB EXAM - AUG
B. Erythromycin Tobramycin, Gentamicin, Kanamycin) and the 2013 MED BOARDS; 2013
C. Clindamycin S=Streptogranins. PREVIOUS BOARD EXAM TOPNOTCH MD)
D. Tobramycin CONCEPT/PRINCIPLE
E. No exception

650 All of the following are bactericidal, EXCEPT? Lincosamides like the Clindamycin are LITO JAY MACARAIG, DIAGNOSTIC
A. Gentamicin bacteriostatics. Gentamicin and Kanamycin MD (TOP 8 - FEB EXAM - AUG
B. Lincosamides are Aminoglycosides and are -cidals. 2013 MED BOARDS; 2013
C. Streptogranins Streptogranins are -cidals as well. ASKED ON TOPNOTCH MD)
D. Kanamycin OUR MICRO BOARDS (not Pharma)
E. No exception

TOPNOTCH MEDICAL BOARD PREP MICROBIOLOGY SUPEREXAM Page 81 of 90


For inquiries visit www.topnotchboardprep.com.ph or email us at topnotchmedicalboardprep@gmail.com
TOPNOTCH MEDICAL BOARD PREP MICROBIOLOGY SUPEREXAM
For inquiries visit www.topnotchboardprep.com.ph or email us at topnotchmedicalboardprep@gmail.com
Item QUESTION EXPLANATION AUTHOR TOPNOTCH
# EXAM
651 A gram stain of an unknown body fluid was shown to Catalase (+) rules out strep species. Coagulase LITO JAY MACARAIG, DIAGNOSTIC
you. You saw gram positive cocci in clusters. Further (-) rules out Staph aureus. Staph epidermidis MD (TOP 8 - FEB EXAM - AUG
studies revealed catalase (+), coagulase (-) and is sensitive to Novobiocin. 2013 MED BOARDS; 2013
novobiocin resistant organisms. You are dealing with? TOPNOTCH MD)
A. Group A strep
B. Group B strep
C. Staph aureus
D. Staph epidermidis
E. Staph saphrophyticus

652 Staphylococcal Scalded Skin Syndrome, AKA Ritter SSSS is caused by Exfoliatin. Protein A LITO JAY MACARAIG, DIAGNOSTIC
disease, is the separation of epidermis at the stratum prevents complement activation during Staph MD (TOP 8 - FEB EXAM - AUG
granulosum. This is caused by what toxin? infection. Alpha toxin causes severe skin 2013 MED BOARDS; 2013
A. Protein A necrosis. Erythrogenic toxin causes the Scarlet TOPNOTCH MD)
B. Alpha toxin fever.
C. AB toxin
D. Erythrogenic toxin
E. Exfoliatin

653 You are given a case of 24 y/o male with Acute Infective Staph aureus is the MCC of Acute IE in NATIVE LITO JAY MACARAIG, DIAGNOSTIC
endocarditis. Upon interview, patient told you he had a valves, while Viridans strep (including MD (TOP 8 - FEB EXAM - AUG
previous operation and a prosthetic valve was implanted sanguis) causes SUBacute IE. 2013 MED BOARDS; 2013
on his heart, which was proven by a grade 3/6 systolic TOPNOTCH MD)
murmur that you heard on your chest PE. What is the
most probable etiologic organism involved?
A. Staph aureus
B. Staph epidermidis
C. Strep Viridans
D. Strep sanguis
E. C and D are both correct
654 During your rounds in the ward, you are given a 72 y/o The history of "currant jelly" sputum, given in LITO JAY MACARAIG, DIAGNOSTIC
male patient who was admitted due to coughing with the case as thick bloody sputum, together with MD (TOP 8 - FEB EXAM - AUG
thick bloody sputum. Patient is a known alcoholic, the alcoholism will give the idea of Klebsiella. 2013 MED BOARDS; 2013
consuming 2 bottles of "lapad" rhum per day for about Which should be strengthened by the results TOPNOTCH MD)
38 years. Culture showed facultative gram(-) rods with of the culture.
large polysaccharide capsule. What organism are you
dealing with?
A. Strep pneumoniae
B. Mycoplasma pneumoniae
C. Leigionella pneumoniae
D. Klebsiella pneumoniae
E. Pseudomonas aeruginosa

655 Dermatophytoses are fungal infections affecting only These are the three most important LITO JAY MACARAIG, DIAGNOSTIC
superficial keratinized structures. This include/s? dermatophytes. MD (TOP 8 - FEB EXAM - AUG
A. Microsporum canis 2013 MED BOARDS; 2013
B. Trichophyton tonsurans TOPNOTCH MD)
C. Epidermophyton floccossum
D. A and C only
E. All of the above

656 Coccidioides immitis is a dimorphic fungus that causes Once inhalated, Arthrospores will become LITO JAY MACARAIG, DIAGNOSTIC
the Valley Fever. It is transmitted via inhalation of? spherules that will be filled with endospores. MD (TOP 8 - FEB EXAM - AUG
A. Arthrospores 2013 MED BOARDS; 2013
B. Endospores TOPNOTCH MD)
C. Spherules
D. A and C only
E. All of the above

657 This is the HIV gene that is responsible for the decrease tat gene is for activation of transcription. Rev LITO JAY MACARAIG, DIAGNOSTIC
in CD4 and class I MHC proteins in patients with AIDS. gene is for transport of mRNA. Vif gene MD (TOP 8 - FEB EXAM - AUG
A. tat enhances hypermutation to escape the 2013 MED BOARDS; 2013
B. rev immune system detection. Vpr facilitates TOPNOTCH MD)
C. nef transport in non-dividing cells.
D. vif
E. Vpr

658 Malaria is the most important parasitic disease in man. It Sporozoites affects the hepatocytes. Schizonts LITO JAY MACARAIG, DIAGNOSTIC
is caused by Plasmodium sp. Through the bite of an infects the RBCs. Gametocytes are the ones MD (TOP 8 - FEB EXAM - AUG
infected female Anopheles flavirostris minimus. On what being harbored by the mosquito during blood 2013 MED BOARDS; 2013
stage on its life cycle does it infects the human RBC? meal. TOPNOTCH MD)
A. schizonts
B. sporozoites
C. gametocytes
D. B and C only
E. All of the above

TOPNOTCH MEDICAL BOARD PREP MICROBIOLOGY SUPEREXAM Page 82 of 90


For inquiries visit www.topnotchboardprep.com.ph or email us at topnotchmedicalboardprep@gmail.com
TOPNOTCH MEDICAL BOARD PREP MICROBIOLOGY SUPEREXAM
For inquiries visit www.topnotchboardprep.com.ph or email us at topnotchmedicalboardprep@gmail.com
Item QUESTION EXPLANATION AUTHOR TOPNOTCH
# EXAM
659 Red tide food poisoning is caused by algae called Saxitoxin is also called the Paralytric toxin. It LITO JAY MACARAIG, DIAGNOSTIC
dinoflagellates. These algae may express different toxins. is heat-stable and the only red tide toxin that MD (TOP 8 - FEB EXAM - AUG
Patients may present with different symptoms can cause respiratory failure 2013 MED BOARDS; 2013
depending on the kind of toxin involved. The most TOPNOTCH MD)
serious among these toxins which can cause respiratory
failure is the?
A. Brevetoxin
B. Domoic toxin
C. Okaaic toxin
D. Saxitoxin
E. Angeloxin
660 Maica Landian, is a 24/F who came in to your clinic due Patient is suffering from Trichomoniasis. The LITO JAY MACARAIG, DIAGNOSTIC
to foul-smelling vaginal discharge. History revealed punctate hemorrhages mentioned is the MD (TOP 8 - FEB EXAM - AUG
coitarche at the age of 13 and currently with her 9th pathognomonic and it is called as strawberry 2013 MED BOARDS; 2013
sexual partner. Speculum exam shows punctate cervix. The recommended treatment is 2grams TOPNOTCH MD)
hemorrhages on the cervix. What is the most Metronidazole given as single dose and treat
appropriate management? the partner as well.
A. Metronidazole 500mg tablet, 4 tablets single dose
B. Metronidazole 500mg tablet, 1 tablet every 8 hrs for
7 days
C. Metronidazole 500mg tablet, 1 tablet every 8 hrs for
14 days
D. B and C only
E. All of the above
661 Cord factor in M. tuberculosis functions in bacterial? Cord factor refers to trehalose dimycolate, a HAZEL KAREN RAZ, MIDTERM 2
A. adherence virulence factor and glycolipid cell-wall MD (TOP 6 - FEB - AUG 2013
B. virulence component of virulent strains 2013 MED BOARDS;
C. Cell envelope of Mycobacterium tuberculosis and closely TOPNOTCH MD)
D. Inhibition of complement deposition related species.It is a surface glycolipid which
E. All of the above blocks macrophage activation by IFN-γ,
induces secretion of TNFα and causes
Mycobacterium tuberculosis to form cords in
vitro. This is the main virulence factor for the
mycobacterium tuberculosis that makes it
resistance to anti-tuberculosis medications.
662 The only gram positive bacteria with endotoxin? L. monocytogenes is unique among Gram- HAZEL KAREN RAZ, MIDTERM 2
A. Staphylococcus positive bacteria in that it might MD (TOP 6 - FEB - AUG 2013
B. streptococcus possess lipopolysaccharide, which serves as 2013 MED BOARDS;
C. clostridium an endotoxin. TOPNOTCH MD)
D. corynebacterium
E. Listeria

663 6 year old female presents with vomiting and fever. E. coli is the most common cause of urinary HAZEL KAREN RAZ, MIDTERM 2
Urinalysis was done revealing 10-15 pus cells. Urine tract infection especially in children. On EMB MD (TOP 6 - FEB - AUG 2013
culture was done which showed a greenish metallic agar, they have a characteristic metallic green 2013 MED BOARDS;
sheen in Eosin Methylene Blue agar. Which of the sheen. TOPNOTCH MD)
following microorganisms is responsible for this
condition?
A. Corynebacterium diphtheriae
B. Staph. epidermidis
C. E. coli
D. Staph. aureus
E. Strep. pneumoniae

664 Which of the following is true about gram positive All the other choices describes the cell wall of HAZEL KAREN RAZ, MIDTERM 2
bacteria? gram negative microorganisms. MD (TOP 6 - FEB - AUG 2013
A. They have thicker cell wall than gram negative 2013 MED BOARDS;
B. They produce endotoxin TOPNOTCH MD)
C. Red on gram stain
D. Periplasmic space is present on the outer cell wall
E. All of the above

665 Which of there is responsible for bacterial motility? Flagellum is a lash-like appendage that HAZEL KAREN RAZ, MIDTERM 2
A. cilia protrudes from the cell body of MD (TOP 6 - FEB - AUG 2013
B. capsule certain prokaryotic andeukaryotic cells. The 2013 MED BOARDS;
C. pili canonical role of the flagellum TOPNOTCH MD)
D. flagella is locomotion but it also often has function as
E. Endospore a sensory organelle, being sensitive to
chemicals and temperatures outside the cell.

666 Which of the following is not an antigen presenting cell? APC's include dendritic cells, B-cell and HAZEL KAREN RAZ, MIDTERM 2
A. None macrophages. MD (TOP 6 - FEB - AUG 2013
B. Polymorphonuclear cells 2013 MED BOARDS;
C. B-cell TOPNOTCH MD)
D. Dendritic cell
E. Macrophage

667 Receptors on CD4+ T-cells recognize: CD4+ cells bind with MHC class II molecules HAZEL KAREN RAZ, MIDTERM 2
A. Peptides associated with MHC class I molecules while CD8+ binds with MHC I. MD (TOP 6 - FEB - AUG 2013
B. Peptides associated with MHC class II molecules 2013 MED BOARDS;
C. NK cells TOPNOTCH MD)
D. Peptides not associated with MHC molecules
E. T-helper cells

TOPNOTCH MEDICAL BOARD PREP MICROBIOLOGY SUPEREXAM Page 83 of 90


For inquiries visit www.topnotchboardprep.com.ph or email us at topnotchmedicalboardprep@gmail.com
TOPNOTCH MEDICAL BOARD PREP MICROBIOLOGY SUPEREXAM
For inquiries visit www.topnotchboardprep.com.ph or email us at topnotchmedicalboardprep@gmail.com
Item QUESTION EXPLANATION AUTHOR TOPNOTCH
# EXAM
668 Quellung reaction is positive in infections caused by? The Quellung reaction is HAZEL KAREN RAZ, MIDTERM 2
A. Strep. pneumoniae a biochemical reaction in MD (TOP 6 - FEB - AUG 2013
B. Klebsiella influenzae which antibodies bind to the bacterial 2013 MED BOARDS;
C. Salmonella capsule of Streptococcus TOPNOTCH MD)
D. A & b only pneumoniae, Klebsiella pneumoniae, Neisseria
E. All of the above meningitidis,Haemophilus influenzae
and Salmonella. If the reaction is positive, the
capsule becomes opaque and appears to
enlarge.
669 Nystatin is a drug used for treatment of diseases caused Nystatin is a polyene antifungal medication to HAZEL KAREN RAZ, MIDTERM 2
by? which many molds and yeast infections are MD (TOP 6 - FEB - AUG 2013
A. Gram negative sensitive, including Candida. 2013 MED BOARDS;
B. Gram positive TOPNOTCH MD)
C. yeast
D. protozoa
E. Viruses

670 Major difference between EHEC and EPEC is/are? Enterohemorrhagic E. coli is a food-borne HAZEL KAREN RAZ, MIDTERM 2
A. EHEC secretes Shiga - like toxin disease primarily acquired from improperly MD (TOP 6 - FEB - AUG 2013
B. EHEC causes hemolytic uremic syndrome cooked ground meat, milk and feacal 2013 MED BOARDS;
C. A and b contaminated vegetables. In most cases, the TOPNOTCH MD)
D. B only illness is self-limiting, but it may lead to a life-
E. None of the above threatening disease including haemolytic
uraemic syndrome (HUS), especially in young
children and the elderly.

671 Which of the following is true of B. anthracis? Anthrax is an acute disease caused by the HAZEL KAREN RAZ, MIDTERM 2
A. It is a gram positive rod with peripheral spores bacterium Bacillus anthracis, gram positive MD (TOP 6 - FEB - AUG 2013
B. It is highly contagious with central spores. Most forms of the disease 2013 MED BOARDS;
C. Inhalational anthrax requires greater number of are lethal, and it affects both humans and TOPNOTCH MD)
spores for infection to occur animals.nthrax does not spread directly from
D. A and b only one infected animal or person to another; it is
E. All of the above spread by spores. These spores can be
transported by clothing or shoes.
672 The following markers are present during the window The immunoglobulin M (IgM) fraction of anti- HAZEL KAREN RAZ, MIDTERM 2
period of Hepa B infection? HBc is used in an important diagnostic assay MD (TOP 6 - FEB - AUG 2013
A. Anti - HBC IgM for acute hepatitis B infection. Before current 2013 MED BOARDS;
B. Anti - Hbe molecular assays were available, it was the TOPNOTCH MD)
C. Both only marker detectable in the window period,
D. none the time between the disappearance of HBsAg
E. HBV DNA and the appearance of anti-HBs. Now anti -
HBe Ab is also detected.

673 Which of the following reflects the correct order of During the process of viral replication, a virus HAZEL KAREN RAZ, MIDTERM 2
events that take place during the multiplication of a induces a living host cell to synthesize the MD (TOP 6 - FEB - AUG 2013
virus?? essential components for the synthesis of new 2013 MED BOARDS;
A. Attachment, penetration, maturation, biosynthesis, viral particles. The particles are then TOPNOTCH MD)
release assembled into the correct structure, and the
B. Penetration, attachment, biosynthesis, maturation, newly formed virions escape from the cell to
release infect other cells.
C. Attachment, penetration, biosynthesis, maturation,
release
D. Attachment, release, biosynthesis, maturation,
penetration
E. None of the above
674 An HIV patient has a CD4 count of < 200. since he is Cotrimoxazole is given prophylactically to HAZEL KAREN RAZ, MIDTERM 2
prone to infection caused by certain microorganisms, decrease susceptibility of HIV patients to MD (TOP 6 - FEB - AUG 2013
this drug should be given prophylactically to decrease Pneumocystis carinii (now jirovechi) 2013 MED BOARDS;
susceptibility to infection caused by opportunistic yeast - pneumonia which commonly develops when TOPNOTCH MD)
like fungus, which causes pneumonia. to CD4 count is < 200. Dose is 15-20 mg/kg in
A. Ceftriaxone 3-4 divided doses for 14 days, based on
B. Chloramphenicol trimethoprim content.
C. Cefepime
D. Cotrimoxazole
E. Rifampicin

675 The botulism intoxication occurs due to? Botulinium toxin or Botox is a neurotoxin HAZEL KAREN RAZ, MIDTERM 2
A. An enterotoxin released from Clostridium botulinum. It MD (TOP 6 - FEB - AUG 2013
B. A neurotoxin inhibits the release of neurotransmitters from 2013 MED BOARDS;
C. A mycotoxin the presynaptic vesicles, thus causing TOPNOTCH MD)
D. A superantigen paralysis.
E. All of the above

676 The species of malaria that affects old RBC's? P. vivax and ovale affects young RBCs, P. HAZEL KAREN RAZ, MIDTERM 2
A. P. falciparum falciparum affects all stages of RBCs. MD (TOP 6 - FEB - AUG 2013
B. P . vivax 2013 MED BOARDS;
C. P. ovale TOPNOTCH MD)
D. P. malariae
E. All of the above

TOPNOTCH MEDICAL BOARD PREP MICROBIOLOGY SUPEREXAM Page 84 of 90


For inquiries visit www.topnotchboardprep.com.ph or email us at topnotchmedicalboardprep@gmail.com
TOPNOTCH MEDICAL BOARD PREP MICROBIOLOGY SUPEREXAM
For inquiries visit www.topnotchboardprep.com.ph or email us at topnotchmedicalboardprep@gmail.com
Item QUESTION EXPLANATION AUTHOR TOPNOTCH
# EXAM
677 Treatment of hypnozoites ? Primaquine is mainly used to treat the P. HAZEL KAREN RAZ, MIDTERM 2
A. Chloroquine vivax or P. ovale malaria. Once the parasite MD (TOP 6 - FEB - AUG 2013
B. Primaquine has been eliminated from the bloodstream, the 2013 MED BOARDS;
C. Arthemeter remaining hypnozoites must be removed from TOPNOTCH MD)
D. Proguanil + Atovaquone the liver and this is done by administering a
E. Doxycycline 14 day course of primaquine.

678 Areas of malarial endemicity in the Philippines include The list includes Agusan del Sur, not Norte. HAZEL KAREN RAZ, MIDTERM 2
the following, except? MD (TOP 6 - FEB - AUG 2013
A. Agusan del Norte 2013 MED BOARDS;
B. Palawan TOPNOTCH MD)
C. Ifugao
D. Kalinga Apayao
E. None of the above

679 The egg of the protozoa which causes rectal prolapse? A - Capillaria philippinensis; B - Taenia sp., C - HAZEL KAREN RAZ, MIDTERM 2
A. Peanut - shaped eggs Toxocara canis, D - Trichuris trichiuria MD (TOP 6 - FEB - AUG 2013
B. With onchosphere and thick striated shell 2013 MED BOARDS;
C. With deep cup -like pits covering the surface of the TOPNOTCH MD)
shell
D. Chinese - Lantern egg/ Bipolar plugs
E. Chinese letters

680 TSST - 1 toxin is an example of ? Toxic shock syndrome toxin (TSST) is HAZEL KAREN RAZ, MIDTERM 2
A. Exfoliatin a superantigen produced by 5 to 25% MD (TOP 6 - FEB - AUG 2013
B. Enterotoxin of Staphylococcus aureus isolates. It 2013 MED BOARDS;
C. Superantigen causes toxic shock syndrome (TSS) by TOPNOTCH MD)
D. Erythrogenic protein stimulating the release of large amounts
E. None of the above of interleukin-1, interleukin-2 and tumour
necrosis factor. In general, the toxin is not
produced by bacteria growing in the blood;
rather, it is produced at the local site of an
infection, and then enters the blood stream.
681 A 21 yo sexually active woman came to your clinic with a Answer: A. Escherichia coli MICHELLE JAY MIDTERM 1
2 day history of urinary frequency with urgency dysuria Notes: E. coli is the most common pathogen in FRANCISCO, MD - AUG 2013
and evidence of blood. She had no prior urinary tract uncomplicated community-acquired UTIs. (TOP 9 - FEB 2013
infection. Laboratory test showed slightly elevated white Other organisms are more prevalent in MED BOARDS;
blood cell count of 10,500/uL. Urinalysis revealed nosocomial or recurrent infections. TOPNOTCH MD)
innumerable white cells. Cultures yielded >105 CFU of a
lactose fermenting gram negative rod. What is the most
probable etiologic agent?
A. Escherichia coli
B. Proteus mirabilis
C. Staphylococcus saprophyticus
D. Klebsiella pneumoniae
682 The following pertains to Coxiella burnetii, except: Answer: B. A rash appears first on the MICHELLE JAY MIDTERM 1
A. Is the etiologic agent of Q fever, which may be acute or extremities then on the trunk FRANCISCO, MD - AUG 2013
chronic Notes: Unlike Rickettsial diseases, there is no (TOP 9 - FEB 2013
B. A rash appears first on the extremities then on the rash involved in Coxiella burnetii infection. MED BOARDS;
trunk Tetracycline is the DOC for uncomplicated TOPNOTCH MD)
C. Is transmitted from animals to man by inhalation infections.
D. Is an obligate intracellular parasite
683 A 45 yo man presents to the ER with fever, chills, nausea Answer: D. Vibrio vulnificus MICHELLE JAY MIDTERM 1
and myalgias. Two days PTC, patient ate raw oysters at a Notes: Vibrio vulnificus is responsible for FRANCISCO, MD - AUG 2013
popular seafood restaurant. Upon admission, he was septicemia after consumption of contaminated (TOP 9 - FEB 2013
febrile and had hemorrhagic, fluid-filled bullous lesions raw oysters. Infections are most severe in MED BOARDS;
in his left leg. The patient had a history of DM, chronic patients with hepatic disease, hematopoietic TOPNOTCH MD)
Hepatitis B, and heavy alcohol consumption. The patient, disease, or chronic renal failure and those
who had a temperature of 39OC, was admitted to the ICU receiving immunosuppressive drugs.
for presumed sepsis and treatment was begun. Blood Mortality in patients with septicemia can be as
cultures drawn on admission and fluid from the bullous high as 50% unless antimicrobial therapy is
leg wound revealed a curved, gram negative rod isolate. started rapidly.
On the 3rd hospital day, DIC developed and patient died.
What is the most probable etiologic agent in this case?
A. Yersinia enterocolitica
B. Plesiomonas shigelloides
C. Aeromonas hydrophila
D. Vibrio vulnificus
684 Intranuclear inclusions were found in epithelial cells Answer: A. Cytomegalovirus MICHELLE JAY MIDTERM 1
from the urine of an infant who was admitted with Notes: Infants usually acquire CMV infection FRANCISCO, MD - AUG 2013
symptoms of low birth weight, jaundice and neurologic before birth or at the time of delivery. These (TOP 9 - FEB 2013
defects. The most likely clinical diagnosis in this case infections may lead to death during the first MED BOARDS;
would be infection from months of life or may result in residual TOPNOTCH MD)
A. Cytomegalovirus neurologic impairment. The virus can be
B. Epstein-Barr virus isolated from several different body fluids
C. Herpes simplex virus with urine being the most commonly
D. Rubella virus examined.

TOPNOTCH MEDICAL BOARD PREP MICROBIOLOGY SUPEREXAM Page 85 of 90


For inquiries visit www.topnotchboardprep.com.ph or email us at topnotchmedicalboardprep@gmail.com
TOPNOTCH MEDICAL BOARD PREP MICROBIOLOGY SUPEREXAM
For inquiries visit www.topnotchboardprep.com.ph or email us at topnotchmedicalboardprep@gmail.com
Item QUESTION EXPLANATION AUTHOR TOPNOTCH
# EXAM
685 An infant was hospitalized with a severe, tender Answer: B. Staphylococcus aureus MICHELLE JAY MIDTERM 1
erythema that started on the face and then spread to the Notes: Scalded Skin Syndrome is the FRANCISCO, MD - AUG 2013
trunk and extremities. The child’s epidermis was loose, dermatitis associated with the effects of the (TOP 9 - FEB 2013
and large areas of skin could be peeled off, leaving the exfoliative toxin produced by strains of MED BOARDS;
sensitive underlying dermis exposed. The condition Staphylococcus aureus. The exfoliatin TOPNOTCH MD)
described is most consistent with a clinical syndrome produced acts in humans to disrupt the
associated with which organism? adhesive forces between cells of the stratum
A. Streptococcus pyogenes granulosum, creating large flaccid bullae. This
B. Staphylococcus aureus syndrome occurs primarily in infants and
C. Bacillus anthracis children, with the primary infection usually
D. Erysipelothrix rhusiopathiae unrelated to the areas where lesions appear.
686 A 40 yo man presents to the Emergency Department Answer: B. Chlamydia psittaci MICHELLE JAY MIDTERM 1
with non-productive cough and rales on physical Notes: Chlamydia psittaci is the etiologic agent FRANCISCO, MD - AUG 2013
examination. Chest-PA suggests bronchopneumonia. of Psittacosis, an infection of birds and (TOP 9 - FEB 2013
Several days PTA, had an onset of fever, severe frontal mammals that is transmissible to humans MED BOARDS;
headache and myalgia. This patient previously worked causing pneumonia-like illness acquired TOPNOTCH MD)
as a zoo janitor. If a tissue biopsy was done on this through inhalation of infectious dropping
patient and revealed Levinthal-Cole-Lillie bodies on from birds, parrots and parakeets. Laboratory
Giemsa stain, which of the following organism is the diagnosis involves detection of inclusion
most likely cause of his condition? bodies called Levinthal-Cole-Lillie bodies
A. Chlamydia trachomatis stained with Giemsa or Macchiavello stain.
B. Chlamydia psittaci These inclusion bodies are more diffuse and
C. Mycoplasma pneumonia irregular in shape containing little glycogen
D. Coxiella burnetii and do not stain recognizably with iodine as
compared to Halberstadter-Prowazek bodies
of Chlamydia trachomatis.
687 What is the unit of structure of fungi? Answer: A. Hyphae (Must know). MICHELLE JAY MIDTERM 1
A. Hyphae FRANCISCO, MD - AUG 2013
B. Mycelium (TOP 9 - FEB 2013
C. Spore MED BOARDS;
D. Conidia TOPNOTCH MD)

688 In the Philippines, thorough cooking of kuhol (Pila Answer: B. Echinostoma ilocanum MICHELLE JAY MIDTERM 1
luzonica) before serving as an exotic native delicacy Notes: Echinostoma ilocanum, the causative FRANCISCO, MD - AUG 2013
prevents infection from this parasite. Heavy infection agent of Echinostomiasis, is confirmed to be (TOP 9 - FEB 2013
may result to mild ulceration of the intestinal mucosa endemic in the Philippines. This intestinal MED BOARDS;
producing bloody diarrhea and abdominal pain. The life- nematode has a unique 2 snail intermediate TOPNOTCH MD)
cycle of this trematode involves 2 snail intermediate hosts. Pila luzonica, the Philippine kuhol, is the
hosts. What parasite is most likely described? 2nd intermediate host.
A. Metagonimus yokogawai *SIMILAR TO PREVIOUS BOARD EXAM
B. Echinostoma ilocanum CONCEPT/PRINCIPLE.
C. Paragonimus westermani *“Philippinized” questions - favorite among
D. Fasciolopsis buski the board examiners
689 Educating the population to undertake activities Answer: C. Misamis Oriental MICHELLE JAY MIDTERM 1
necessary to avert transmission of certain diseases has Notes: In the Philippines, Schistosoma FRANCISCO, MD - AUG 2013
an important role in prevention strategies most japonicum is highly endemic in Sorsogon, (TOP 9 - FEB 2013
especially in certain diseases with high-endemicity. Samar, Leyte, Oriental Mindoro, Bohol and MED BOARDS;
Among the following provinces, Schistosoma japonicum ALL of Mindanao EXCEPT Misamis Oriental. TOPNOTCH MD)
is highly endemic, except in: *SIMILAR TO PREVIOUS BOARD EXAM
A. Zamboanga CONCEPT/PRINCIPLE
B. Cotabato
C. Misamis Oriental
D. Misamis Occidental
690 Steam under pressure is the most dependable moist heat Answer: C. 121OC for 15-20 minutes, 15 MICHELLE JAY MIDTERM 1
method of sterilization for both hospital clinical pounds per square inch FRANCISCO, MD - AUG 2013
laboratories and research facilities. What is the required Notes: (TOP 9 - FEB 2013
temperature and period of time necessary for the • 63-66OC for 30 minutes followed by rapid MED BOARDS;
material to be sterilized to remain in contact with cooling = Pasteurization TOPNOTCH MD)
saturated steam in an autoclave? • 80-100OC for 30 minutes for 3 consecutive
A. 63-66OC for 30 minutes followed by rapid cooling days = Tyndallization
B. 80-100OC for 30 minutes for 3 consecutive days • 121OC for 15-20 minutes, 15 pounds per
C. 121OC for 15-20 minutes, 15 pounds per square inch square inch = Steam under pressure, ex.
D. 180OC for 2-4 hours enough to kill all types of Autoclave
organisms, including the sporeformer • 180OC for 2-4 hours enough to kill all types
of organisms, including the sporeformer = Hot
air oven (dry heat)
• 100OC for 30 minutes adequate to kill
vegetative cell = Boiling
691 Which of the following statements regarding T cells is B cells are from liver and migrate to bone MICHELLE JAY MIDTERM 1
true? marrow. Helper cells (specifically TH2) FRANCISCO, MD - AUG 2013
A. T cells finish development in the thymus and then release interleukins to promote B-cell (TOP 9 - FEB 2013
migrate to the bone marrow. differentiation & maturation. Cytotoxic T cells MED BOARDS;
B. The various types of T cells can be identified by the destroy target cell by recognizing foreign or TOPNOTCH MD)
binding of specific monoclonal antibodies to antigens on modifies self antigen and class I MHC
the cell surface molecules on target cells.
C. Helper T cells can be activated to produce antibodies.
D. Cytotoxic T cells can destroy target cells by
recognizing foreign antigens at the target nucleus.

TOPNOTCH MEDICAL BOARD PREP MICROBIOLOGY SUPEREXAM Page 86 of 90


For inquiries visit www.topnotchboardprep.com.ph or email us at topnotchmedicalboardprep@gmail.com
TOPNOTCH MEDICAL BOARD PREP MICROBIOLOGY SUPEREXAM
For inquiries visit www.topnotchboardprep.com.ph or email us at topnotchmedicalboardprep@gmail.com
Item QUESTION EXPLANATION AUTHOR TOPNOTCH
# EXAM
692 One of these pairs do not match: Brain abscess- Nocardia asteroides MICHELLE JAY MIDTERM 1
A. Histoplasma capsulatum-Fibrosing mediastinitis FRANCISCO, MD - AUG 2013
B. Cryptococcus neoformans- Amphotericin (TOP 9 - FEB 2013
C. Actinomyces israelii- Pelvic Mass MED BOARDS;
D. Candida albicans -Brain abscess TOPNOTCH MD)

693 During an emergency appendectomy, a surgical resident All are potentially transmissible thru the MICHELLE JAY MIDTERM 1
sustains an injury from contaminated hollow-bore exposure described. However, risk for FRANCISCO, MD - AUG 2013
needle with spontaneous bleeding. Which blood-borne acquiring is about 30% for HBV, 3% for HCV & (TOP 9 - FEB 2013
organism is most likely to be transmitted, assuming that 0.3% for HIV. MED BOARDS;
the patient was infected with all of them? TOPNOTCH MD)
A. HIV
B. HBV
C. HCV
D. Plasmodium spp
E. Treponema pallidum

694 An 80 yr-old woman who lives in a nursing home and Most appropriate is metronidazole. If not MICHELLE JAY MIDTERM 1
who just finished a 10-day course of antibiotics has improving, oral vancomycin. If refractory, FRANCISCO, MD - AUG 2013
abdominal pain and profuse diarrhea. Her stool is tested vancomycin enemas. (TOP 9 - FEB 2013
and comes back positive for C.difficile. What is the most MED BOARDS;
appropriate initial management? TOPNOTCH MD)
A. Oral vancomycin
B. IV vancomycin
C. Metronidazole
D. Supportive treatment only
695 Which of the following is associated with megaloblastic MICHELLE JAY MIDTERM 1
anemia? FRANCISCO, MD - AUG 2013
A. E. Coli (TOP 9 - FEB 2013
B. Klebsiella pneumoniae MED BOARDS;
C. P.mirabilis TOPNOTCH MD)
D. P.aeruginosa
E. Diphyllobotrium latum

696 An injured firefighter developed a wound infection, and Pseudomonas aeruginosa is gm-neg, oxidase- MICHELLE JAY MIDTERM 1
culture of the site indicates a gram-negative rod that is positive, aerobic rod that produces a green- FRANCISCO, MD - AUG 2013
oxidized-positive and produces bluish-green pigment. blue pigment, pyocyanin. Frequent cause of (TOP 9 - FEB 2013
The organism was relatively resistant to antibiotics but burn wound infection. MED BOARDS;
susceptible to ticarcillin and tobramycin. The organism TOPNOTCH MD)
is most likely
A. E. Coli
B. Klebsiella pneumoniae
C. P.mirabilis
D. P.aeruginosa
E. Serratia marcescens
697 A 3 yr-old child presents at the physician’s office with Measles or rubeola belong to MICHELLE JAY MIDTERM 1
the symptoms of coryza, conjunctivitis, low-grade fever, paramyxoviridae. FRANCISCO, MD - AUG 2013
and Koplik spots. The causative agent of this disease (TOP 9 - FEB 2013
belongs to which group of viruses? MED BOARDS;
A. Adenovirus TOPNOTCH MD)
B. Herpesvirus
C. Orthomyxovirus
D. Paramyxovirus
E. Picornavirus

698 Viruses have various ways of entering the human body Coronaviruses are inhaled or ingested but MICHELLE JAY MIDTERM 1
and producing disease. Which of the following don’t form ulcers.Insects not impt.in HIV FRANCISCO, MD - AUG 2013
descriptions accurately describe the route and transmission.Inlfuenza viruses do not (TOP 9 - FEB 2013
mechanism for the virus indicated? replicate in lung lymphocytes and mumps MED BOARDS;
A. Coronaviruses enter the GIT thru the mouth and move virus are most often inhaled from TOPNOTCH MD)
into the stomach where they proliferate in mucosal cells respi.secretions of infected individuals.
to produce peptic ulcers.
B. Enteroviruses enter thru the mouth, replicate in the
pharynx and bowel and move via the blood to distant
target organs (CNS).
C. HIV is directly injected into the blood for egg
development.
D. Inlfuenza viruses enter thru the respi.tract, replicate
within lymphocytes in the lung, and move via the
lymphatic vessels to joints and the CNS to produce
muscle aches, stiff joints and fever.
E. Mumps virus enters thru abraded skin in the genital
area and moves into the testicles of males to produce
swelling and sterility.
699 A 25 yr-old gay with AIDS has malabsorption, chronic Cryptosporidium parvum is closely related to MICHELLE JAY MIDTERM 1
abdominal pain, low-grade fever and non-bloody AIDS px. FRANCISCO, MD - AUG 2013
diarrhea. What is an expected finding in fecal smears? (TOP 9 - FEB 2013
A. large numbers of oocysts containing four sporozoites MED BOARDS;
B. free-living amoeba with ingested RBC TOPNOTCH MD)
C. circular protozoa with ciliated cell wall
D. flagellate with falling-leaf motility

TOPNOTCH MEDICAL BOARD PREP MICROBIOLOGY SUPEREXAM Page 87 of 90


For inquiries visit www.topnotchboardprep.com.ph or email us at topnotchmedicalboardprep@gmail.com
TOPNOTCH MEDICAL BOARD PREP MICROBIOLOGY SUPEREXAM
For inquiries visit www.topnotchboardprep.com.ph or email us at topnotchmedicalboardprep@gmail.com
Item QUESTION EXPLANATION AUTHOR TOPNOTCH
# EXAM
700 A newborn boy appears to be lethargic and septic. A Listeria monocytogenes cause sepsis in MICHELLE JAY MIDTERM 1
spinal tap was performed and a Gram stain revealed gm+ newborns. FRANCISCO, MD - AUG 2013
bacilli. Cultures of the spinal fluid on sheep blood agar (TOP 9 - FEB 2013
plates at a temp.of 22-250 C, yielded catalase+ MED BOARDS;
haemolytic rods that had tumbling motion. Penicillin TOPNOTCH MD)
cleared the infection. The most likely organism is:
A. Bacillus cereus
B. Bordetella pertussis
C. Corynebacterium diphtheria
D. Listeria monocytogenes
E. Neisseria meningitides

TOPNOTCH MEDICAL BOARD PREP MICROBIOLOGY SUPEREXAM Page 88 of 90


For inquiries visit www.topnotchboardprep.com.ph or email us at topnotchmedicalboardprep@gmail.com
TOPNOTCH MEDICAL BOARD PREP MICROBIOLOGY SUPEREXAM
For inquiries visit www.topnotchboardprep.com.ph or email us at topnotchmedicalboardprep@gmail.com

Item # ANSWER 88 B 176 B 264 A 352 A


1 B 89 C 177 E 265 E 353 C
2 B 90 B 178 C 266 B 354 A
3 D 91 D 179 A 267 A 355 A
4 A 92 D 180 E 268 D 356 B
5 E 93 A 181 E 269 E 357 A
6 C 94 C 182 D 270 D 358 A
7 D 95 D 183 D 271 D 359 C
8 B 96 B 184 A 272 D 360 C
9 B 97 C 185 C 273 C 361 A
10 A 98 A 186 D 274 C 362 C
11 C 99 C 187 C 275 A 363 E
12 A,E 100 C 188 E 276 B 364 A
13 E 101 D 189 D 277 B 365 B
14 E 102 E 190 B 278 D 366 A
15 A 103 B 191 C 279 D 367 D
16 C 104 C 192 C 280 B 368 E
17 E 105 A 193 A 281 D 369 C
18 B 106 C 194 A 282 C 370 E
19 A 107 E 195 D 283 B 371 D
20 E 108 D 196 D 284 B 372 B
21 B 109 B 197 B 285 D 373 A
22 D 110 B 198 C 286 B 374 D
23 D 111 A 199 E 287 A 375 C
24 E 112 C 200 C 288 E 376 B
25 E 113 D 201 E 289 B 377 D
26 C 114 D 202 A 290 A 378 E
27 C 115 A 203 D 291 C 379 A
28 E 116 D 204 B 292 D 380 B
29 E 117 B 205 B 293 E 381 B
30 D 118 D 206 D 294 E 382 A
31 A 119 E 207 C 295 C 383 D
32 A 120 A 208 A 296 D 384 C
33 C 121 B 209 B 297 A 385 B
34 D 122 A 210 B 298 C 386 B
35 A 123 E 211 D 299 D 387 D
36 C 124 C 212 C 300 E 388 C
37 C 125 D 213 A 301 B 389 A
38 D 126 A 214 C 302 B 390 C
39 A 127 C 215 C 303 B 391 B
40 C 128 E 216 B 304 B 392 A
41 B 129 E 217 C 305 D 393 C
42 E 130 D 218 B 306 D 394 A
43 A 131 C 219 B 307 D 395 B
44 B 132 E 220 D 308 E 396 B
45 D 133 D 221 D 309 B 397 D
46 D 134 C 222 A 310 A 398 A
47 E 135 B 223 D 311 D 399 B
48 D 136 A 224 C 312 B 400 C
49 B 137 B 225 B 313 B 401 B
50 A 138 C 226 A 314 D 402 A
51 E 139 D 227 A 315 A 403 D
52 A 140 D 228 B 316 C 404 B
53 E 141 B 229 D 317 A 405 C
54 C 142 C 230 C 318 C 406 C
55 B 143 E 231 D 319 B 407 B
56 A 144 A or C 232 A 320 B 408 E
57 C 145 A 233 B 321 C 409 E
58 A 146 E 234 E 322 A 410 C
59 D 147 A 235 C 323 E 411 A
60 A 148 B 236 A 324 D 412 E
61 E 149 C 237 C 325 E 413 B
62 A 150 B 238 B 326 D 414 D
63 E 151 B 239 B 327 D 415 D
64 C 152 D 240 C 328 A 416 E
65 D 153 A 241 A 329 B 417 D
66 C 154 D 242 B 330 C 418 B
67 E 155 B 243 C 331 E 419 C
68 C 156 D 244 D 332 B 420 B
69 B 157 E 245 A 333 B 421 A
70 B 158 C 246 B 334 E 422 C
71 E 159 D 247 D 335 E 423 C
72 A 160 B 248 D 336 D 424 D
73 C 161 B 249 B 337 B 425 B
74 E 162 C 250 C 338 D 426 B
75 B 163 C 251 D 339 D 427 B
76 D 164 E 252 E 340 D 428 A
77 E 165 B 253 B 341 A 429 D
78 A 166 E 254 A 342 D 430 A
79 C 167 D 255 B 343 B 431 D
80 B 168 C 256 E 344 D 432 D
81 B 169 B 257 D 345 B 433 B
82 D 170 B 258 A 346 A 434 C
83 A 171 A 259 C 347 D 435 A
84 D 172 D 260 E 348 D 436 B
85 B 173 E 261 B 349 B 437 D
86 B 174 D 262 C 350 C 438 A
87 B 175 A 263 A 351 B 439 C
TOPNOTCH MEDICAL BOARD PREP MICROBIOLOGY SUPEREXAM Page 89 of 90
For inquiries visit www.topnotchboardprep.com.ph or email us at topnotchmedicalboardprep@gmail.com
TOPNOTCH MEDICAL BOARD PREP MICROBIOLOGY SUPEREXAM
For inquiries visit www.topnotchboardprep.com.ph or email us at topnotchmedicalboardprep@gmail.com
440 B 493 D 546 E 599 B 652 E
441 D 494 C 547 E 600 C 653 B
442 B 495 C 548 D 601 A 654 D
443 E 496 D 549 D 602 C 655 E
444 B 497 C 550 C 603 B 656 A
445 A 498 A 551 D 604 D 657 C
446 D 499 D 552 C 605 D 658 A
447 C 500 C 553 D 606 B 659 D
448 B 501 A 554 D 607 C 660 A
449 C 502 C 555 E 608 D 661 B
450 A 503 B 556 E 609 A 662 E
451 A 504 A 557 E 610 D 663 C
452 E 505 D 558 E 611 D 664 A
453 B 506 B 559 E 612 E 665 D
454 B 507 A 560 C 613 E 666 B
455 A 508 D 561 D 614 A 667 B
456 C 509 D 562 A 615 D 668 E
457 B 510 B 563 C 616 E 669 C
458 E 511 C 564 D 617 A 670 C
459 A 512 E 565 A 618 C 671 C
460 C 513 A 566 C 619 E 672 C
461 D 514 C 567 A 620 A 673 C
462 A 515 B 568 A 621 C 674 D
463 E 516 C 569 D 622 E 675 B
464 A 517 B 570 C 623 E 676 D
465 A 518 D 571 B 624 E 677 B
466 A 519 D 572 C 625 D 678 A
467 B 520 E 573 A 626 B 679 D
468 D 521 D 574 B 627 D 680 C
469 A 522 B 575 D 628 C 681 A
470 C 523 C 576 C 629 D 682 B
471 B 524 B 577 A 630 D 683 D
472 E 525 D 578 B 631 C 684 A
473 E 526 B 579 B 632 D 685 B
474 C 527 D 580 D 633 A 686 B
475 B 528 B 581 B 634 B 687 A
476 B 529 D 582 D 635 C 688 B
477 D 530 C 583 B 636 E 689 C
478 A 531 D 584 B 637 C 690 C
479 E 532 D 585 B 638 D 691 B
480 B 533 B 586 C 639 D 692 D
481 C 534 C 587 E 640 C 693 B
482 A 535 A 588 B 641 D 694 C
483 D 536 A 589 E 642 C 695 E
484 A 537 B 590 A 643 A 696 D
485 C 538 D 591 C 644 B 697 D
486 A 539 D 592 A 645 A 698 B
487 B 540 A 593 E 646 A 699 A
488 C 541 E 594 C 647 B 700 D
489 A 542 B 595 C 648 A
490 D 543 A 596 E 649 D
491 A 544 E 597 B 650 B
492 B 545 D 598 E 651 E

TOPNOTCH MEDICAL BOARD PREP MICROBIOLOGY SUPEREXAM Page 90 of 90


For inquiries visit www.topnotchboardprep.com.ph or email us at topnotchmedicalboardprep@gmail.com
TOPNOTCH MEDICAL BOARD PREP PHYSIOLOGY SUPEREXAM
For inquiries visit www.topnotchboardprep.com.ph or email us at topnotchmedicalboardprep@gmail.com
DEAR TOPNOTCH FRIENDS:

PLEASE FOLLOW THESE INSTRUCTIONS:

1. These questions are previous diagnostic, midterm, and finals exams of Topnotch, almost all of them made by Topnotch Board Exam Topnotchers.
2. Answer this Topnotch Superexam seriously 100-items at a time. Cover the “Explanations” Column. Do not immediately look at the answers from the
answer key. That’s not the correct way of answering sample exams. You need to treat these MCQs as exercises and not as handouts.
3. Time yourself. 1.5 hours per 100-item block.
4. After answering each 100-item block, refer to the Topnotch Answer Key for the correct answers. Please be careful of “frameshift mutations” when
checking your answers – check every 10 items. (the format of the answer key was designed for you to practice against “frameshift mutations”)
5. The Topnotch Superexams are EXERCISES for the actual med boards. They will not appear verbatim in your future exams. More than knowing what’s
the correct answer, it’s more important for you to:
a. Know why the other choices are wrong
b. Know why the other choices were included in the first place
c. Know the explanation to the correct answer
6. Sharpen your mind by answering the Topnotch Superexams. Most of these questions based on past feedback are more difficult than the actual questions
in the med boards. In these exams made by Board Exam Topnotchers, if you’re getting a score of 60/100 , that’s already a good score. More than 80/100
is outstanding.

Item QUESTION EXPLANATION AUTHOR TOPNOTCH
# EXAM
1 Renin is a component of the RAAS system, which the The release of renin by the JG cells in the afferent KRISTEL DIAGNOSTIC
body uses to control blood pressure. Which of the arteriole is stimulated by decreased delivery of NaCl to TANHUI EXAM -
following will lead to an increase in the circulating the macula densa as this indicates a decreased GFR and (TOP 3 - AUG MARCH 2016
levels of renin in the blood? thus contracted blood volume. 2015 MED
A. Decreased renal sympathetic nerve activity BOARDS;
B. Increased blood pressure Source: Guyton and Hall Textbook of Medical Physiology TOPNOTCH
C. Decreased NaCl delivery to the macula densa 9th ed p.228-229, 328 MD FROM LA
D. All of the above SALLE)
E. None of the above

2 Antidiuretic hormone acts on which segment of the Antidiuretic hormone causes water conservation by KRISTEL DIAGNOSTIC
nephron to increase permeability to urea? decreasing urine production and increasing urine TANHUI EXAM -
A. Proximal convoluted tubule osmolarity. It has 3 functions in the kidney which brings (TOP 3 - AUG MARCH 2016
B. Thin descending loop of Henle about this effect 2015 MED
C. Distal convoluted tubule - Increased water permeability in the late distal and BOARDS;
D. Inner medullary collecting duct collecting tubules by insertion of aquaporins TOPNOTCH
E. Calyces - Increased activity of the Na-K-2Cl pump in the thick MD FROM LA
asceding loop of henle causing increased osmolarity of SALLE)
the interstitial fluid
- Increased permeability of the inner medullary
collecting duct to urea also causing increased osmolarity
of the interstitial fluid
Together, the latter 2 increases the force which draws
fluid out of the tubules resulting to a more concentrated
urine.

Source: Guyton and Hall Textbook of Medical Physiology
9th ed p 355
3 Cardiac output is not affected by: CO is affected by all of the above KRISTEL DIAGNOSTIC
A. Afterload TANHUI EXAM -
B. Venous return Source: Guyton and Hall Textbook of Medical Physiology (TOP 3 - AUG MARCH 2016
C. Starling’s law of the heart 9th ed p 115, 117 2015 MED
D. Sympathetic tone BOARDS;
E. None of the above TOPNOTCH
MD FROM LA
SALLE)

4 The following are greater in the arteries than in the The veins are the capacitance vessels, which holds most KRISTEL DIAGNOSTIC
veins except: of the vascular blood volume. With sympathetic TANHUI EXAM -
A. Pressure gradient stimulation, there is decreased compliance on the (TOP 3 - AUG MARCH 2016
B. Vascular volume venous circulation promoting increased venous return. 2015 MED
C. Parabolic velocity profile BOARDS;
D. Pulse pressure Source: Guyton and Hall Textbook of Medical Physiology TOPNOTCH
E. None of the above 9th ed p 179 MD FROM LA
SALLE)

5 In the cardiac muscle action potential, the refractory The R-R interval is equivalent to 1 whole heartbeat. KRISTEL DIAGNOSTIC
period: Letter B is just a trick item and is not a real concept TANHUI EXAM -
A. Equals the R-R interval associated with refractory period. (TOP 3 - AUG MARCH 2016
B. Balances off the effects of increased venous return 2015 MED
C. Helps prevent arrhythmias Source: Physiology by Linda Costanzo 3rd ed p133, 136 BOARDS;
D. A and B TOPNOTCH
E. All of the above MD FROM LA
SALLE)

6 According to the West’s zones of the lung, which of Zone 1 - PA>Pa>Pv KRISTEL DIAGNOSTIC
the following decribes zone 2? Zone 2 - Pa>PA>Pv TANHUI EXAM -
A. Pa>Pv>PA Zone 3 - Pa>Pv>PA (TOP 3 - AUG MARCH 2016
B. Pa>PA>Pv 2015 MED
C. PA>Pa>Pv Source: Guyton and Hall Textbook of Medical Physiology BOARDS;
D. Pa>PA<Pv 9th ed p 494 TOPNOTCH
E. Pa>Pv<PA MD FROM LA
SALLE)

TOPNOTCH MEDICAL BOARD PREP PHYSIOLOGY SUPEREXAM Page 1 of 88


For inquiries visit www.topnotchboardprep.com.ph or email us at topnotchmedicalboardprep@gmail.com
TOPNOTCH MEDICAL BOARD PREP PHYSIOLOGY SUPEREXAM
For inquiries visit www.topnotchboardprep.com.ph or email us at topnotchmedicalboardprep@gmail.com
Item QUESTION EXPLANATION AUTHOR TOPNOTCH
# EXAM
7 Which of the following directly stimulates central The drive for respiration is mainly regulated by arterial KRISTEL DIAGNOSTIC
chemoreceptors? PaCO2. However, the central chemoreceptors are only TANHUI EXAM -
A. Increased PaCO2 responsive to changes in CSF pH. (TOP 3 - AUG MARCH 2016
B. Increased CSF PaCO2 Physiologically, the increased CO2 in blood diffuses 2015 MED
C. Decreased arterial pH across the blood brain barrier into the CSF. The CO2 then BOARDS;
D. Decreased CSF pH reacts with water in a reaction catalyzed by carbonic TOPNOTCH
E. Decreased arterial or CSF O2 anhydrase to form carbonic acid causing the decrease in MD FROM LA
pH. SALLE)
A change in blood pH alone will not affect respiration
because H ions cant pass through the BBB.
It is the peripheral receptors which are affected by O2.

Source: Guyton and Hall Textbook of Medical Physiology
9th ed p 528
8 The lung volume during normal inhalation and TV is normally around 500ml. KRISTEL DIAGNOSTIC
exhalation when no extra force is applied is called: TANHUI EXAM -
A. Vital capacity Source: Guyton and Hall Textbook of Medical Physiology (TOP 3 - AUG MARCH 2016
B. Total lung capacity 9th ed p 482 2015 MED
C. Inspiratory capacity BOARDS;
D. Functional residual capacity TOPNOTCH
E. Tidal volume MD FROM LA
SALLE)

9 Juan dela Cruz had a hearty meal of lechon, with lots Gastrocolic reflex is the phenomenon of increased KRISTEL DIAGNOSTIC
of lechon sauce and rice. In a few minutes after all motility in the colon in response to gastric stretch or TANHUI EXAM -
this eating, he suddenly felt the urge to pass a bowel distention. (TOP 3 - AUG MARCH 2016
movement. What is this physiologic phenomenon? 2015 MED
A. Food poisoning Source: Guyton and Hall Textbook of Medical Physiology BOARDS;
B. Vasovagal reflex 9th ed p 797 TOPNOTCH
C. Gastrocolic reflex MD FROM LA
D. Enteroanal reflex SALLE)
E. None of the above

10 The following are true regarding pancreatic Vagal stimulation of the pancreas leads to release of KRISTEL DIAGNOSTIC
secretion except: enzyme-rich secretion. TANHUI EXAM -
A. Of bicarbonate rich fluid is strongly stimulated by (TOP 3 - AUG MARCH 2016
secretin Source: Guyton and Hall Textbook of Medical Physiology 2015 MED
B. Of bicarbonate rich fluid is strongly stimulated by 9th ed p 825 BOARDS;
the vagus TOPNOTCH
C. Of digestive enzymes is strongly stimulated by MD FROM LA
cholecystokinin SALLE)
D. Of digestive enzymes is stimulated by intracellular
production of inositol 1,4,5 – triphosphate
E. None of the above
11 Which of the following is correct? Mean arterial pressure = Cardiac output x total KRISTEL DIAGNOSTIC
A. CO = TPR/MAP peripheral resistance TANHUI EXAM -
B. CO = TPR x MAP (TOP 3 - AUG MARCH 2016
C. TPR = HR X MAP/SV Cardiac output = heart rate x stroke volume 2015 MED
D. TPR = HR X SV/ MAP BOARDS;
E. MAP = HR x SV x TPR Source: Guyton and Hall Textbook of Medical Physiology TOPNOTCH
9th ed p 241 MD FROM LA
SALLE)

12 If the R-R interval was 0.8s in duration what is the The R-R Interval is equivalent to the duration of 1 heart KRISTEL DIAGNOSTIC
heart rate? beat. TANHUI EXAM -
A. 65 This can be paraphrased as, if one heartbeat lasts 0.8s, (TOP 3 - AUG MARCH 2016
B. 70 how many heartbeats are there in 60s? Therefore we can 2015 MED
C. 75 phrase this mathematically as: BOARDS;
D. 80
60s = 0.8x TOPNOTCH
E. 85 60/0.8 = x MD FROM LA
Source: Physiology by Linda Costanzo 3rd ed p136 SALLE)

13 Which of the following substances is not a All of the above are components of the plasma KRISTEL DIAGNOSTIC
components of the plasma membrane? membrane. TANHUI EXAM -
A. Cholesterol (TOP 3 - AUG MARCH 2016
B. Receptor proteins Source: Guyton and Hall Textbook of Medical Physiology 2015 MED
C. Carbohydrates 9th ed p 13 BOARDS;
D. Phospholipids TOPNOTCH
E. None of the above MD FROM LA
SALLE)

14 Which of the ion’s equilibrium potential exerts the The cell membrane has highest permeability for KRISTEL DIAGNOSTIC
greatest effect on determining the cell’s over all potassium so the overall resting membrane is closest to TANHUI EXAM -
resting membrane potential? its equilibrium potential compared to the other ions. (TOP 3 - AUG MARCH 2016
A. Na 2015 MED
B. Cl Source: Guyton and Hall Textbook of Medical Physiology BOARDS;
C. K 9th ed p 60 TOPNOTCH
D. Ca MD FROM LA
E. HCO3- SALLE)

TOPNOTCH MEDICAL BOARD PREP PHYSIOLOGY SUPEREXAM Page 2 of 88


For inquiries visit www.topnotchboardprep.com.ph or email us at topnotchmedicalboardprep@gmail.com
TOPNOTCH MEDICAL BOARD PREP PHYSIOLOGY SUPEREXAM
For inquiries visit www.topnotchboardprep.com.ph or email us at topnotchmedicalboardprep@gmail.com
Item QUESTION EXPLANATION AUTHOR TOPNOTCH
# EXAM
15 Which of the following is not true? Fast pain is conducted by myelinated A fibers but slow KRISTEL DIAGNOSTIC
A. Pain pathways contain exclusively myelinated pain is conducted by unmyelinated C fibers. TANHUI EXAM -
fibers Pain pathways are activated by noxious stimuli (TOP 3 - AUG MARCH 2016
B. Pain pathways may be activated directly by (bradykinin, serotonin, histamine, potassium, acids and 2015 MED
prostaglandins proteolytic enzymes) but prostaglandins simply lower BOARDS;
C. Pain from the viscera may be referred to the skin their threshold for activation. TOPNOTCH
by the same spinal segments MD FROM LA
D. A and B Source: Guyton and Hall Textbook of Medical Physiology SALLE)
E. All of the above 9th ed p 587

16 if a patient with hypercortisolism has an adrenal In a patient with hypercortisolism, if the etiology is a KRISTEL DIAGNOSTIC
tumor, which of the following is the expected finding pituitary secreting ACTH, the expected finding will be TANHUI EXAM -
with the dexamethasone suppression test? not suppressed to low dose dexamethasone suppression (TOP 3 - AUG MARCH 2016
A. Suppressed by low dose and high dose test but suppressed by high dose. If the tumor is a 2015 MED
B. Not suppressed by low dose and suppressed by cortisol secreting tumor in the adrenal cortex, then it will BOARDS;
high dose not be suppressed by both low and high dose TOPNOTCH
C. Suppressed by low dose, not suppressed by high dexamethasone. MD FROM LA
dose SALLE)
D. Not suppressed by low dose and high dose Source: Physiology by Linda Costanzo 3rd ed p413
E. None of the above
17 In a patient who just ran a marathon, some of the During exercise, aside from increased pH in the muscle KRISTEL DIAGNOSTIC
muscle fibers utilize the anaerobic pathway, vasculature, there is also increased CO2 production and TANHUI EXAM -
resulting to an increase in the production of lactic temperature all of which decreases the O2 affinity, (TOP 3 - AUG MARCH 2016
acid and effecting an increase in the local pH. Which increased p50, and a shift of the curve to the right. This 2015 MED
of the following is true regarding the O2- promotes release of oxygen to the muscles. BOARDS;
Hemoglobin curve in the muscle vasculature? TOPNOTCH
A. The curve is shifted to the right Source: Guyton and Hall Textbook of Medical Physiology MD FROM LA
B. The p50 is decreased 9th ed p 518 SALLE)
C. There is increased O2 affinity
D. A and B
E. All of the above

18 How do glucocorticoids result to an increase in the Source: Physiology by Linda Costanzo 3rd ed p415 KRISTEL DIAGNOSTIC
blood pressure? TANHUI EXAM -
A. Decreased TPR (TOP 3 - AUG MARCH 2016
B. Vascular remodeling causing smooth muscle 2015 MED
hypertrophy BOARDS;
C. Permissive action of glucocorticoids to TOPNOTCH
vasoconstrictors like catecholamine MD FROM LA
D. B and C SALLE)
E. All of the above

19 Which of the following brain structures is the master “You need the suprachiasmatic nucleus to determine KRISTEL DIAGNOSTIC
clock of the body and is responsible for circadian when to sleep so you can be super charismatic the next TANHUI EXAM -
rhythm? day” (mnemonic) (TOP 3 - AUG MARCH 2016
A. Suprachiasmatic nucleus 2015 MED
B. Paraventricular nuclei Source Topnotch handout on Physiology BOARDS;
C. Supraoptic nuclei TOPNOTCH
D. Anterior hypothalamus MD FROM LA
E. Central gray area around aqueduct of sylvius SALLE)

20 Which of the following promotes reabsorption of According to Starling’s law, the factors which promote KRISTEL DIAGNOSTIC
fluid in capillaries? filtration are: TANHUI EXAM -
A. Increased capillary oncotic pressure Decreased capillary oncotic pressure (TOP 3 - AUG MARCH 2016
B. Increased capillary hydrostatic pressure Increased capillary hydrostatic pressure 2015 MED
C. Increased interstitial oncotic pressure Increased interstitial oncotic pressure BOARDS;
D. Decreased interstitial hydrostatic pressure Decreased interstitial hydrostatic pressure TOPNOTCH
E. Inflammation Increased vascular permeability (inflammation) MD FROM LA
SALLE)
Note: Do not confuse this with Starling’s law of the heart
which dictates that there is an increased contractility
with increased preload. This ensures that cardiac output
is always approximately equal to the venous return and
prevents accumulation of the blood in the heart.

Source: Guyton and Hall Textbook of Medical Physiology
9th ed p 187

21 In skeletal muscle, which of the following events SIMILAR TO PREVIOUS BOARD EXAM LESTER MIDTERM 1
occurs before depolarization of the T tubules in the CONCEPT/PRINCIPLE. One third of the Physiology exam BRYAN CO EXAM -
mechanism of excitation–contraction coupling? were MCQ questions from BRS Physiology. In the (TOP 10 - MARCH 2016
A. Depolarization of the sarcolemmal membrane mechanism of excitation–contraction coupling, AUG 2015
B. Opening of Ca2+ release channels on the excitation MED
sarcoplasmic reticulum (SR) always precedes contraction. Excitation refers to the BOARDS;
C. Uptake of Ca2+ into the SR by Ca2+-adenosine electrical activation of the muscle cell, which begins with TOPNOTCH
triphosphatase (ATPase) an action potential (depolarization) in the sarcolemmal MD FROM
D. Binding of Ca2+ to troponin C membrane UST)
E. Binding of actin and myosin that spreads to the T tubules. Depolarization of the T
tubules then leads to the release of Ca2+ from the nearby
sarcoplasmic reticulum (SR), followed by an increase in
intracellular Ca2+ concentration, binding of Ca2+ to
troponin C, and then contraction.

TOPNOTCH MEDICAL BOARD PREP PHYSIOLOGY SUPEREXAM Page 3 of 88


For inquiries visit www.topnotchboardprep.com.ph or email us at topnotchmedicalboardprep@gmail.com
TOPNOTCH MEDICAL BOARD PREP PHYSIOLOGY SUPEREXAM
For inquiries visit www.topnotchboardprep.com.ph or email us at topnotchmedicalboardprep@gmail.com
Item QUESTION EXPLANATION AUTHOR TOPNOTCH
# EXAM
22 A new drug is developed that blocks the transporter SIMILAR TO PREVIOUS BOARD EXAM LESTER MIDTERM 1
for H+ secretion in gastric parietal cells. Which of the CONCEPT/PRINCIPLE. H+ secretion by gastric parietal BRYAN CO EXAM -
following transport processes is being inhibited? cells occurs by H+–K+–adenosine triphosphatase (TOP 10 - MARCH 2016
A. Simple diffusion (ATPase), a primary active transporter. AUG 2015
B. Facilitated diffusion MED
C. Primary active transport BOARDS;
D. Cotransport TOPNOTCH
E. Countertransport MD FROM
UST)
23 Following a sympathectomy, a 66-year old man SIMILAR TO PREVIOUS BOARD EXAM LESTER MIDTERM 1
experiences orthostatic hypotension. The CONCEPT/PRINCIPLE. Orthostatic hypotension is a BRYAN CO EXAM -
explanation for this occurrence is decrease in arterial pressure that occurs when a person (TOP 10 - MARCH 2016
A. an exaggerated response of the renin– moves from a supine to a standing position. A person AUG 2015
angiotensin–aldosterone system with a normal baroreceptor mechanism responds to a MED
B. a suppressed response of the renin–angiotensin– decrease in arterial pressure through the vasomotor BOARDS;
aldosterone system center by increasing sympathetic outflow and decreasing TOPNOTCH
C. an exaggerated response of the baroreceptor parasympathetic outflow. The sympathetic component MD FROM
mechanism helps to restore blood pressure by increasing heart rate, UST)
D. a suppressed response of the baroreceptor contractility, total peripheral resistance (TPR), and mean
mechanism
systemic pressure. In a patient who has undergone a
E. none of the above sympathectomy, the sympathetic component of the
baroreceptor mechanism is absent.
24 A 42-year-old woman with severe pulmonary SIMILAR TO PREVIOUS BOARD EXAM LESTER MIDTERM 1
fibrosis is evaluated by her physician and has the CONCEPT/PRINCIPLE. The patient’s arterial blood gases BRYAN CO EXAM -
following arterial blood gases: pH = 7.48, PaO2 = 55 show increased pH, decreased PaO2, and decreased (TOP 10 - MARCH 2016
mm Hg, and PaCO2 = 32 mm Hg. Which statement PaCO2. The decreased PaO2 causes hyperventilation AUG 2015
best explains the observed value of PaCO2? (stimulates breathing) via the peripheral MED
A. The increased pH stimulates breathing via chemoreceptors, but not via the central chemoreceptors. BOARDS;
peripheral chemoreceptors The decreased PacO2 results from hyperventilation TOPNOTCH
B. The increased pH stimulates breathing via central (increased breathing) and causes increased pH, which MD FROM
chemoreceptors inhibits breathing via the peripheral and central UST)
C. The decreased PaO2 inhibits breathing via chemoreceptors.
peripheral chemoreceptors
D. The decreased PaO2 stimulates breathing via
peripheral chemoreceptors
E. The decreased PaO2 stimulates breathing via
central chemoreceptors
25 Which of the following has a much lower SIMILAR TO PREVIOUS BOARD EXAM LESTER MIDTERM 1
concentration in the CSF than in cerebral capillary CONCEPT/PRINCIPLE. Cerebrospinal fluid (CSF) is BRYAN CO EXAM -
blood? similar in composition to the interstitial fluid of the (TOP 10 - MARCH 2016
A. Na+ brain. Therefore, it is similar to an ultrafiltrate of plasma AUG 2015
B. K+ and has a very low protein concentration because large MED
C. Osmolarity protein molecules cannot cross the blood–brain barrier. BOARDS;
D. Protein There are other differences in composition between CSF TOPNOTCH
E. Mg2+ and blood that are created by transporters in the choroid MD FROM
plexus, but the low protein concentration of CSF is the UST)
most dramatic difference.
26 An electrocardiogram (ECG) on a person shows SIMILAR TO PREVIOUS BOARD EXAM LESTER MIDTERM 1
ventricular extrasystoles. The extrasystolic beat CONCEPT/PRINCIPLE. On the extrasystolic beat, pulse BRYAN CO EXAM -
would produce pressure decreases because there is inadequate (TOP 10 - MARCH 2016
A. increased pulse pressure because contractility is ventricular filling time—the ventricle beats “too soon.” AUG 2015
increased As a result, stroke volume decreases. MED
B. increased pulse pressure because heart rate is BOARDS;
increased TOPNOTCH
C. decreased pulse pressure because ventricular MD FROM
filling time is increased UST)
D. decreased pulse pressure because stroke volume
is decreased
E. decreased pulse pressure because the PR interval
is increased
27 After an extrasystole, the next “normal” ventricular SIMILAR TO PREVIOUS BOARD EXAM LESTER MIDTERM 1
contraction produces CONCEPT/PRINCIPLE. The postextrasystolic contraction BRYAN CO EXAM -
A. increased pulse pressure because the contractility produces increased pulse pressure because contractility (TOP 10 - MARCH 2016
of the ventricle is increased is increased. Extra Ca2+ enters the cell during the AUG 2015
B. increased pulse pressure because total peripheral extrasystolic beat. Contractility is directly related to the MED
resistance (TPR) is decreased amount of intracellular Ca2+ available for binding to BOARDS;
C. increased pulse pressure because compliance of troponin C. TOPNOTCH
the veins is decreased MD FROM
D. decreased pulse pressure because the UST)
contractility of the ventricle is increased
E. decreased pulse pressure because TPR is
decreased
28 The ventricles are completely depolarized during SIMILAR TO PREVIOUS BOARD EXAM LESTER MIDTERM 1
which isoelectric portion of the electrocardiogram CONCEPT/PRINCIPLE. The PR segment (part of the PR BRYAN CO EXAM -
(ECG)? interval) and the ST segment are the only portions of the (TOP 10 - MARCH 2016
A. PR interval electrocardiogram (ECG) that are isoelectric. The PR AUG 2015
B. QRS complex interval includes the P wave (atrial depolarization) and MED
C. QT interval the PR segment, which represents conduction through BOARDS;
D. ST segment the atrioventricular (AV) node; during this phase, the TOPNOTCH
E. T wave ventricles are not yet depolarized. The ST segment is the MD FROM
only isoelectric period when the entire ventricle is UST)
depolarized.

TOPNOTCH MEDICAL BOARD PREP PHYSIOLOGY SUPEREXAM Page 4 of 88


For inquiries visit www.topnotchboardprep.com.ph or email us at topnotchmedicalboardprep@gmail.com
TOPNOTCH MEDICAL BOARD PREP PHYSIOLOGY SUPEREXAM
For inquiries visit www.topnotchboardprep.com.ph or email us at topnotchmedicalboardprep@gmail.com
Item QUESTION EXPLANATION AUTHOR TOPNOTCH
# EXAM
29 Receptor potential in a Pacinian corpuscle SIMILAR TO PREVIOUS BOARD EXAM LESTER MIDTERM 1
A. is an all-or-none phenomenon CONCEPT/PRINCIPLE. Receptor potentials in sensory BRYAN CO EXAM -
B. has a stereotypical size and shape receptors (such as the pacinian corpuscle) are not action (TOP 10 - MARCH 2016
C. is the action potential of this sensory receptor potentials and therefore do not have the stereotypical AUG 2015
D. if hyperpolarizing, increases the likelihood of size and shape or the all-or-none feature of the action MED
action potential occurrence potential. Instead, they are graded potentials that vary in BOARDS;
E. if depolarizing, brings the membrane potential size depending on the stimulus intensity. A TOPNOTCH
closer to threshold hyperpolarizing MD FROM
receptor potential would take the membrane potential UST)
away from threshold and decrease the likelihood of
action potential occurrence. A depolarizing receptor
potential would bring the membrane potential toward
threshold and increase the likelihood of action potential
occurrence.
30 A patient has the following arterial blood values: SIMILAR TO PREVIOUS BOARD EXAM LESTER MIDTERM 1
pH = 7.52 CONCEPT/PRINCIPLE. First, the acid–base disorder BRYAN CO EXAM -
PCO2 = 20 mm Hg must be diagnosed. Alkaline pH, low PCO2, and low (TOP 10 - MARCH 2016
[HCO3–] = 16 mEq/L HCO3 – are consistent with respiratory alkalosis. In AUG 2015
Which of the following statements about this patient respiratory alkalosis, the [H+] is decreased and less H+ is MED
is most likely to be bound to negatively charged sites on plasma proteins. As BOARDS;
correct? a result, more Ca2+ is bound to proteins and, therefore, TOPNOTCH
A. He is hypoventilating the ionized [Ca2+] decreases. There is no respiratory MD FROM
B. He has decreased ionized [Ca2+] in blood compensation for primary respiratory disorders. The UST)
C. He has almost complete respiratory compensation patient is hyperventilating, which is the cause of the
D. He has an acid–base disorder caused by respiratory alkalosis. Appropriate renal compensation
overproduction of fixed acid would be decreased reabsorption of HCO3 –, which
E. Appropriate renal compensation would cause his would cause his arterial [HCO3 –] to decrease and his
arterial [HCO3–] to increase blood pH to decrease (become more normal).
31 A patient with a duodenal ulcer is treated SIMILAR TO PREVIOUS BOARD EXAM LESTER MIDTERM 1
successfully with the drug cimetidine. The basis for CONCEPT/PRINCIPLE. Cimetidine is a reversible BRYAN CO EXAM -
cimetidine’s inhibition of gastric H+ secretion is that inhibitor of H2 receptors on parietal cells and blocks H+ (TOP 10 - MARCH 2016
it secretion. Cyclic adenosine monophosphate (cAMP) [the AUG 2015
A. blocks muscarinic receptors on parietal cells second messenger for histamine] levels would be MED
B. blocks H2 receptors on parietal cells expected to decrease, not increase. Cimetidine also BOARDS;
C. increases intracellular cyclic adenosine blocks the action of acetylcholine (ACh) to stimulate H+ TOPNOTCH
monophosphate levels secretion. Omeprazole blocks H+,K+-adenosine MD FROM
D. blocks H+,K+-adenosine triphosphatase triphosphatase (ATPase) directly. UST)
E. enhances the action of acetylcholine on parietal
cells
32 A 24-year-old woman presents to her family CO2 is transported in arterial blood in three forms: as LESTER MIDTERM 1
physician with intractable hiccups. The patient is physically dissolved CO2 (about 5%), in combination BRYAN CO EXAM -
instructed to breathe into and out of a bag in order with the amino groups of Hb as carbaminohemoglobin (TOP 10 - MARCH 2016
to rebreathe exhaled CO2. In the blood, the majority (about 10%), and as bicarbonate ion, that is, (about AUG 2015
of CO2 is transported as which of the following 85%). The amount of CO2 actually carried as carbonic MED
forms? acid, H2CO3, is negligible. Carboxyhemoglobin refers to BOARDS;
A. Bicarbonate the combination of carbon monoxide (CO) and Hb. TOPNOTCH
B. Carbaminohemoglobin MD FROM
C. Carbonic acid UST)
D. Carboxyhemoglobin
E. Dissolved CO2
33 Which of the following explains the suppression of SIMILAR TO PREVIOUS BOARD EXAM LESTER MIDTERM 1
lactation during pregnancy? CONCEPT/PRINCIPLE. Although the high circulating BRYAN CO EXAM -
A. Blood prolactin levels are too low for milk levels of estrogen stimulate prolactin secretion during (TOP 10 - MARCH 2016
production to occur pregnancy, the action of prolactin on the breast is AUG 2015
B. Human placental lactogen levels are too low for inhibited by progesterone and estrogen. After MED
milk production to occur parturition, progesterone and estrogen levels decrease BOARDS;
C. The fetal adrenal gland does not produce dramatically. Prolactin can then interact with its TOPNOTCH
sufficient estriol receptors in the breast, and lactation proceeds if MD FROM
D. Blood levels of estrogen and progesterone are initiated by suckling. UST)
high
E. The maternal anterior pituitary is suppressed
34 A 63-year-old woman has an intractable duodenal The vagus nerve is the primary neural mediator of LESTER MIDTERM 1
ulcer failing all previous treatments. After gastric function. BRYAN CO EXAM -
consultation with a surgeon, a laparoscopic Activation of distension-mediated vago-vagal reflexes in (TOP 10 - MARCH 2016
vagotomy is performed. Subsequently, the patient response to the presence of food in the stomach will (1) AUG 2015
experiences nausea and vomiting after ingestion of a increase gastric compliance (receptive relaxation or MED
mixed meal. Which of the following best explains her accommodation reflex) and promote gastric retention of BOARDS;
symptoms? food, (2) increase the strength of antral peristaltic TOPNOTCH
A. Decreased gastric emptying of liquids contractions necessary for trituration of solids, and (3) MD FROM
B. Decreased gastric emptying of solids increase gastric acid secretion. Sectioning of the vagus UST)
C. Increased gastric emptying of liquids nerve fibers to the antral region of the stomach will
D. Increased gastric emptying of solids decrease the strength of contractions, thereby
E. Increased gastric emptying of solids and liquids prolonging the emptying of solids. The emptying of
liquids will be unaffected.

TOPNOTCH MEDICAL BOARD PREP PHYSIOLOGY SUPEREXAM Page 5 of 88


For inquiries visit www.topnotchboardprep.com.ph or email us at topnotchmedicalboardprep@gmail.com
TOPNOTCH MEDICAL BOARD PREP PHYSIOLOGY SUPEREXAM
For inquiries visit www.topnotchboardprep.com.ph or email us at topnotchmedicalboardprep@gmail.com
Item QUESTION EXPLANATION AUTHOR TOPNOTCH
# EXAM
35 An 83-year-old woman with a history of If a substance disappears from the circulation during its LESTER MIDTERM 1
hypertension presents to her family physician’s passage through the kidney, it usually indicates that it BRYAN CO EXAM -
office with oliguria. Serum creatinine and BUN are has been totally secreted into the nephron, in which case (TOP 10 - MARCH 2016
elevated and a computed tomography (CT) reveals the earance of the substance equals RPF. The clearance AUG 2015
that the patient’s left kidney is hypoplastic. Renal would not equal the GFR (choice c) because the normal MED
function studies are performed to assess the renal filtration fraction is 20%, which would not totally clear BOARDS;
handling of various substances. Substance X is the plasma concentration of substance. None of the TOPNOTCH
injected into an arterial line. All of substance X substance is reabsorbed (choice b) because none of the MD FROM
appears in the urine and none is detected in the substance is detected in the renal vein. If the substance is UST)
renal vein. What do these findings indicate about the bound to plasma proteins, it can be secreted without
renal handling of substance X? being filtered (choice a). Even if it is entirely secreted by
A. It must be filtered by the kidney. the kidney, its urinary concentration may be less than its
B. It must be reabsorbed by the kidney. plasma concentration if the urinary flow rate is very high
C. Its clearance is equal to the GFR. (choice e).
D. Its clearance is equal to the renal plasma flow
(RPF).
E. Its urinary concentration must be higher than its
plasma concentration.
36 A 60-year-old businessman is evaluated by his SIMILAR TO PREVIOUS BOARD EXAM LESTER MIDTERM 1
physician, who determines that his blood pressure is CONCEPT/PRINCIPLE. In this patient, hypertension is BRYAN CO EXAM -
significantly elevated at 185/130 mm Hg. most likely caused by left renal artery stenosis, which led (TOP 10 - MARCH 2016
Laboratory tests reveal an increase in plasma renin to increased renin secretion by the left kidney. The AUG 2015
activity, plasma aldosterone level, and left renal vein increased plasma renin activity causes an increased MED
renin level. His right renal vein renin level is secretion of aldosterone, which increases Na+ BOARDS;
decreased. What is the most likely cause of the reabsorption by the renal distal tubule. The increased TOPNOTCH
patient’s hypertension? Na+ reabsorption leads to increased blood volume and MD FROM
A. Aldosterone-secreting tumor blood pressure. The right kidney responds to the UST)
B. Adrenal adenoma secreting aldosterone and increase in blood pressure by decreasing its renin
cortisol secretion. Right renal artery stenosis causes a similar
C. Pheochromocytoma pattern of results, except that renin secretion from the
D. Left renal artery stenosis right kidney, not the left kidney, is increased.
E. Right renal artery stenosis Aldosterone-secreting tumors cause increased levels of
aldosterone, but decreased plasma renin activity (as a
result of decreased renin secretion by both kidneys).
Pheochromocytoma is associated with increased
circulating levels of catecholamines, which increase
blood pressure by their effects on the heart (increased
heart rate and contractility) and blood vessels
(vasoconstriction); the increase in blood pressure is
sensed by the kidneys and results in decreased plasma
renin activity and aldosterone levels.
37 During which phase of the cardiac cycle is SIMILAR TO PREVIOUS BOARD EXAM LESTER MIDTERM 1
ventricular volume lowest? CONCEPT/PRINCIPLE. Ventricular volume is at its BRYAN CO EXAM -
A. Atrial systole lowest value while the ventricle is relaxed (diastole), just (TOP 10 - MARCH 2016
B. Isovolumetric ventricular contraction before ventricular filling begins. AUG 2015
C. Rapid ventricular ejection MED
D. Reduced ventricular ejection BOARDS;
E. Isovolumetric ventricular relaxation TOPNOTCH
MD FROM
UST)
38 A 49-year-old man in end-stage renal failure is able The osmolality of a substance is the number of osmoles LESTER MIDTERM 1
to perform peritoneal dialysis at home. The per kg of solvent. One osmole (Osm) equals the gram BRYAN CO EXAM -
osmolality of the solution chosen for peritoneal molecular weight of a substance divided by the number (TOP 10 - MARCH 2016
dialysis will determine the rate of ultrafiltration. of free-moving particles that each molecule liberates in AUG 2015
Which of the following statements best characterizes solution. Osmotically active substances in the body are MED
a molecule whose osmolality is zero? dissolved in water, and the density of water is 1. Thus, BOARDS;
A. It will not permeate the membrane. osmolar concentrations can be expressed as osmoles (or TOPNOTCH
B. It can only cross the membrane through the lipid milliosmoles) per liter of water. If the osmolality is zero, MD FROM
bilayer. there are no free-moving particles and, thus, the UST)
C. It causes water to flow across the membrane. molecule is as diffusible as water through the
D. It is as diffusible through the membrane as water. membrane.
E. It must be transported across the membrane by a
carrier.
39 A 55-year-old obese man with type 2 diabetes Increased serum lipid levels may result from increased LESTER MIDTERM 1
mellitus presents for his annual checkup. Serum production, decreased clearance, or both. The principal BRYAN CO EXAM -
lipoprotein analysis done after a 12-hour fast shows lipid abnormality in diabetes is hypertriglyceridemia, (TOP 10 - MARCH 2016
elevated lowdensity lipoprotein (LDL) and very low- which is due to increased VLDLs. VLDL levels are AUG 2015
density lipoproteins (VLDL), elevated triglycerides, increased because of insufficient insulin action in MED
and decreased high-density lipo-protein cholesterol. adipose tissue, which results in decreased VLDL BOARDS;
What contributes to the lipid abnormalities in type 2 clearance as a result of decreased lipoprotein lipase TOPNOTCH
diabetes mellitus? activity. Triglyceride uptake into adipose tissue from MD FROM
A. Hyperglycemia increases triglyceride uptake into plasma lipoproteins requires hydrolysis of triglyceride UST)
adipose tissue. to fatty acids and glycerol by lipoprotein lipase, which is
B. Insulin resistance increases triglyceride uptake bound to the vascular endothelial surface. The activity of
into adipose tissue. lipoprotein lipase varies in reciprocal fashion with that
C. Insulin resistance and hyperglycemia decrease of cytoplasmic hormone-sensitive lipase, and thus is
fatty acid flux to the liver and lipolysis. enhanced by insulin and decreased by catecholamines.
D. Insufficient insulin action in adipose tissue Lipoprotein lipase is present in nearly every tissue and
decreases lipoprotein lipase activity. acts at the capillary surface as it does in adipose tissue.
E. Insufficient insulin action increases LDL receptor
activity.

TOPNOTCH MEDICAL BOARD PREP PHYSIOLOGY SUPEREXAM Page 6 of 88


For inquiries visit www.topnotchboardprep.com.ph or email us at topnotchmedicalboardprep@gmail.com
TOPNOTCH MEDICAL BOARD PREP PHYSIOLOGY SUPEREXAM
For inquiries visit www.topnotchboardprep.com.ph or email us at topnotchmedicalboardprep@gmail.com
Item QUESTION EXPLANATION AUTHOR TOPNOTCH
# EXAM
40 A 16-year-old adolescent boy on the track team asks Phosphorylcreatine is rapidly converted to ATP in LESTER MIDTERM 1
his pediatrician if he can take creatine on a regular muscle. When the metabolic demands exceed the rate at BRYAN CO EXAM -
basis in order to increase his muscle strength prior which ATP can be generated by aerobic metabolism or (TOP 10 - MARCH 2016
to a track meet. Which of the following most likely glycolysis, phosphocreatine can supply the necessary AUG 2015
explains why he wants to take creatine? ATP for a brief period of time. An increase in the MED
A. Creatine delays the metabolism of fatty acids. concentration of phosphorylcreatine in muscle may BOARDS;
B. Creatine increases muscle glycogen concentration. increase the amount of ATP that can be produced and TOPNOTCH
C. Creatine increases plasma glucose concentration. therefore enhance performance. MD FROM
D. Creatine is converted to phosphorylcreatine. UST)
E. Creatine prevents dehydration by increasing
creatinine formation.
41 Reason why tetanic contraction is not observed in GEORGE MIDTERM 2
cardiac muscles MICHAEL EXAM -
A. Longer depolarization SOSUAN MARCH 2016
B. Presence of phase 3 in the action potential (TOP 5 - AUG
C. Less calcium stores 2015 MED
D. Longer refractory period BOARDS;
E. NOTA TOPNOTCH
MD FROM
UST)
42 The blood flow is controlled primarily by the All other choices: controlled by local control GEORGE MIDTERM 2
sympathetic nervous system in the following organs MICHAEL EXAM -
A. Brain SOSUAN MARCH 2016
B. Heart (TOP 5 - AUG
C. Skin 2015 MED
D. Exercising skeletal muscle BOARDS;
E. NOTA TOPNOTCH
MD FROM
UST)
43 Receptors are sensitive to [H+] in the CSF Central chemoreceptor- sensitive to CSF [H+]; peripheral GEORGE MIDTERM 2
A. Central chemoreceptor chemoreceptor - sensitive blood p02 and pCO2 levels MICHAEL EXAM -
B. Peripheral chemoreceptor SOSUAN MARCH 2016
C. J receptor (TOP 5 - AUG
D. Carotid chemoreceptor 2015 MED
E. Aortic chemoreceptor BOARDS;
TOPNOTCH
MD FROM
UST)
44 The hormone with the longest half-life T3 - 1-2 days; T4 7-10 days; PTH, GH and cortisol <1 day GEORGE MIDTERM 2
A. T3 MICHAEL EXAM -
B. T4 SOSUAN MARCH 2016
C. PTH (TOP 5 - AUG
D. GH 2015 MED
E. Cortisol BOARDS;
TOPNOTCH
MD FROM
UST)
45 Which of the following is a step in photoreception in Light converts 11-cis retinal to all-trans retinal, which is GEORGE MIDTERM 2
the rods? converted to such intermediates as metarhodopsin II. MICHAEL EXAM -
A. Light converts all-trans retinal to 11-cis retinal Metarhodopsin II activates a stimulatory G protein SOSUAN MARCH 2016
(B) Metarhodopsin II activates transducin (transducin), which activates a (TOP 5 - AUG
(C) cGMP levels increase phosphodiesterase. Phosphodiesterase breaks down 2015 MED
(D) Rods depolarize cGMP, so intracellular cGMP levels decrease, causing BOARDS;
(E) Glutamate release increases closure of Na+ channels in the photoreceptor cell TOPNOTCH
membrane and hyperpolarization. Hyperpolarization of MD FROM
the photoreceptor cell membrane inhibits release of the UST)
neurotransmitter, glutamate.
46 A cell is most excitable at this phase of action GEORGE MIDTERM 2
potential MICHAEL EXAM -
A. Resting Membrane Potential SOSUAN MARCH 2016
B. Overshoot (TOP 5 - AUG
C. Repolarization 2015 MED
D. Hyperpolarization BOARDS;
E. All phases the same TOPNOTCH
MD FROM
UST)
47 Which of the following is not an endogenous Endotoxcin is an exogenous pyrogen from the cell wall GEORGE MIDTERM 2
pyrogen? (Lipid A) of Gram negative bacteria. MICHAEL EXAM -
A. Interleukin 1 SOSUAN MARCH 2016
B. Interleukin 6 (TOP 5 - AUG
C. Endotoxins 2015 MED
D. Interferon-alpha BOARDS;
E. TNF-alpha TOPNOTCH
MD FROM
UST)
48 Breathing pure oxygen at high altitude causes Breathing pure O2 lowers FiO2 of nitrogen. At higher GEORGE MIDTERM 2
A. Lower fraction of inspired nitrogen altitudes, the partial pressure of the gases decreases. MICHAEL EXAM -
B. Lower fraction of inspired oxygeb SOSUAN MARCH 2016
C. Higher atmospheric barometric pressure (TOP 5 - AUG
D. Decrease in alveolar dead space ventilation 2015 MED
E. AOTA BOARDS;
TOPNOTCH
MD FROM
UST)

TOPNOTCH MEDICAL BOARD PREP PHYSIOLOGY SUPEREXAM Page 7 of 88


For inquiries visit www.topnotchboardprep.com.ph or email us at topnotchmedicalboardprep@gmail.com
TOPNOTCH MEDICAL BOARD PREP PHYSIOLOGY SUPEREXAM
For inquiries visit www.topnotchboardprep.com.ph or email us at topnotchmedicalboardprep@gmail.com
Item QUESTION EXPLANATION AUTHOR TOPNOTCH
# EXAM
49 Calcitonin is secreted by Calcitonin is used as tumor marker in medullary GEORGE MIDTERM 2
A. Medullary carcinoma carcinoma of the thyroid. MICHAEL EXAM -
B. Follicular carcinoma SOSUAN MARCH 2016
C. Papillary carcinoma (TOP 5 - AUG
D. Anaplastic carcinoma 2015 MED
E. NOTA BOARDS;
TOPNOTCH
MD FROM
UST)
50 REM sleep occurs every how many minutes of slow GEORGE MIDTERM 2
wave sleep MICHAEL EXAM -
A. 30 SOSUAN MARCH 2016
B. 60 (TOP 5 - AUG
C. 90 2015 MED
D. 120 BOARDS;
E. NOTA TOPNOTCH
MD FROM
UST)
51 Complement components that are anaphylatoxins, C2a is not an anaphylatoxin; C5a is also chemotactic. GEORGE MIDTERM 2
except MICHAEL EXAM -
A. C2a SOSUAN MARCH 2016
B. C3a (TOP 5 - AUG
C. C4a 2015 MED
D. C5a BOARDS;
E. NOTA TOPNOTCH
MD FROM
UST)
52 T-helper subtype involved in autoimmune diseases TH1- cell-mediated immunity; TH2 - humoral immunity; GEORGE MIDTERM 2
and allergen-induced airway response TH17 - autoimmune diseases and allergen-induced MICHAEL EXAM -
A. TH1 airway response SOSUAN MARCH 2016
B. TH10 (TOP 5 - AUG
C. TH11 2015 MED
D. TH13 BOARDS;
E. TH17 TOPNOTCH
MD FROM
UST)
53 Longest phase of the cardiac cycle Reduced ventricular filling or diastasis GEORGE MIDTERM 2
A. Atrial contraction MICHAEL EXAM -
B. Reduced ventricular ejection SOSUAN MARCH 2016
C. Reduced ventricular filling (TOP 5 - AUG
D. Isovolumetric contraction 2015 MED
E. Isovolumetric relaxation BOARDS;
TOPNOTCH
MD FROM
UST)
54 What initiates the Hering-Breuer Reflex? Hering-Breuer Reflex - decreases respiratory rate by GEORGE MIDTERM 2
A. Lung stretch receptors prolonging expiratory time, initated by lung distention MICHAEL EXAM -
B. Joint and muscle receptors SOSUAN MARCH 2016
C. Irritant receptors (TOP 5 - AUG
D. J receptors 2015 MED
E. Peripheral chemoreceptors BOARDS;
TOPNOTCH
MD FROM
UST)
55 The best source of energy for a 10 km marathon GEORGE MIDTERM 2
runner is MICHAEL EXAM -
A. Substrate level phosphorylation SOSUAN MARCH 2016
B. Oxidative phosphorylation (TOP 5 - AUG
C. Glycogenesis 2015 MED
D. Gluconeogenesis BOARDS;
E. Glycolysis TOPNOTCH
MD FROM
UST)
56 The first step in urine formation is Steps in urine formation: 1. Glomerular filtration, 2. GEORGE MIDTERM 2
A. Glomerular filtration Tubular reabsorptiion, 3. Tubular secretion MICHAEL EXAM -
B. Tubular reabsorption SOSUAN MARCH 2016
C. Tubular secretion (TOP 5 - AUG
D. Glomerular reabsorption 2015 MED
E. NOTA BOARDS;
TOPNOTCH
MD FROM
UST)
57 Under Starling's forces, this favors movement of All the choices favor movement of water into the blood GEORGE MIDTERM 2
fluid out of the blood vessel MICHAEL EXAM -
A. Low capillary hydrostatic force SOSUAN MARCH 2016
B. High interstitial hydrostatic force (TOP 5 - AUG
C. High capillary oncotic pressure 2015 MED
D. Low interstitial oncotic pressure BOARDS;
E. NOTA TOPNOTCH
MD FROM
UST)

TOPNOTCH MEDICAL BOARD PREP PHYSIOLOGY SUPEREXAM Page 8 of 88


For inquiries visit www.topnotchboardprep.com.ph or email us at topnotchmedicalboardprep@gmail.com
TOPNOTCH MEDICAL BOARD PREP PHYSIOLOGY SUPEREXAM
For inquiries visit www.topnotchboardprep.com.ph or email us at topnotchmedicalboardprep@gmail.com
Item QUESTION EXPLANATION AUTHOR TOPNOTCH
# EXAM
58 Which of the following inhibit growth hormone Acute hyperglycemia stimulates somatostatin release by GEORGE MIDTERM 2
secretion the hypothalamus which, in turn, suppresses growth MICHAEL EXAM -
A. Hyperglcyemia hormone secretion from the anterior pituitary gland. SOSUAN MARCH 2016
B. Sleep (TOP 5 - AUG
C. Exercise 2015 MED
D. Arginine BOARDS;
E. NOTA TOPNOTCH
MD FROM
UST)
59 Gastric parietal cells secrete Pepsinogen is secreted by chief cells. GEORGE MIDTERM 2
A. Intrinsic factor MICHAEL EXAM -
B. HCl SOSUAN MARCH 2016
C. Pepsinogen (TOP 5 - AUG
D. Both A and B 2015 MED
E. AOTA BOARDS;
TOPNOTCH
MD FROM
UST)
60 After a fatty meal, the most potent stimulus for GEORGE MIDTERM 2
secretion of enzymes for lipid digestion is MICHAEL EXAM -
A. VIP SOSUAN MARCH 2016
B. CCK (TOP 5 - AUG
C. Gastrin 2015 MED
D. Pancreatic polypepide BOARDS;
E. NOTA TOPNOTCH
MD FROM
UST)
61 This are characteristic of facilitated transporters SIMILAR TO PREVIOUS BOARD EXAM JAN MIDTERM 3
EXCEPT? CONCEPT/PRINCIPLE. Facilitated diffusion is a pssive CHRISTIAN EXAM -
A. It is sterospecific for either the L or D isomer transport that is carrier meidated. It has 3 impt FELICIANO MARCH 2016
B. It is saturable characteristics: Saturation, Stereopecific and is affected (TOP 2 - AUG
C. It is an active transport by competitvie inhibition.Exampels are the GLUT and AA 2015 MED
D. Monosaccharide transport inside a cell is an transporters BOARDS;
example TOPNOTCH
E. Competitive inhibition may occur MD FROM
UST)
62 This is an example of an antiport type of seconary An antiporter, also called exchanger or counter- JAN MIDTERM 3
active transport? transporter is a type of secondary active transport of CHRISTIAN EXAM -
A. GLUT2 in liver two or more different molecules or ions across the FELICIANO MARCH 2016
B. H-K ATPase in stomach plasma membrane in opposite directions. Example are (TOP 2 - AUG
C. SGLT1 in intestines the Na-Ca exhanger in all cells and Na-H exhanger in 2015 MED
D. N-K-2CL in THAL PCT. SGLT1 and N-K-2CL are symporters. BOARDS;
E. Na-H in PCT TOPNOTCH
MD FROM
UST)
63 Which of the ff hormones is not derived from Tryptophan derivatives: Trip Mo Sya Noh? Tryptophan, JAN MIDTERM 3
tryptophan? Melatonin, Serotonin, Niacin/Nicotinic Acid CHRISTIAN EXAM -
A. Melanin FELICIANO MARCH 2016
B. Nicotinic Acid (TOP 2 - AUG
C. Melatonin 2015 MED
D. Serotonin BOARDS;
E. Niacin TOPNOTCH
MD FROM
UST)
64 What is the cause of the long action potential and 2 major differences between the membrane of cardiac JAN MIDTERM 3
plateau phase of the cardiac muscle in contrast to and skeletal muscle accounting for the prolonged AP and CHRISTIAN EXAM -
skeletal muscle? plateau phase in the heart. 1st, the AP of cardiacm uscle FELICIANO MARCH 2016
A. The action potential is caused by opening of both is caused by opening of both Na and Ca channels. The Ca (TOP 2 - AUG
fast Na channels and slow calcium channels channels are slower to open and, even more important, 2015 MED
B. The presence of intercalated disks between remain open for several tenths of a second maintains a BOARDS;
cardiac cells that form permeable gap junctions that prolonged period of depolarization, causing the plateau TOPNOTCH
allow rapid diffusion of ions in the action potential. 2nd: The decreased potassium MD FROM
C. Immediately after the onsent of the action permeability greatly decreases the outflux of positively UST)
potential, the permeability of the cardiac muscle for charged potassium ions during the action potential
potassium ions decreases about fivefold plateau and thereby prevents early return of the action
D. Both A and C potential voltage to its resting level.
E. All of the above
65 What is the major mechanism directly contributing The diffusion potentials alone caused by potassium and JAN MIDTERM 3
to the resting membrane potential? sodium diffusion would give a membrane potential of CHRISTIAN EXAM -
A. Na K ATPase pump about −86 millivolts, almost all of this being determined FELICIANO MARCH 2016
B. Potassium leak channel by potassium diffusion. Then, an additional −4 millivolts (TOP 2 - AUG
C. Sodium leak channel is contributed to the membrane potential by the 2015 MED
D. Innate nernst potential of Na and K continuously acting electrogenic Na+-K+ pump, giving a BOARDS;
E. Na Ca exchanger net membrane potential of −90 millivolts. TOPNOTCH
MD FROM
UST)
66 Which of the ff receptor-2nnd messenger/effector is Alpha 1- Inc IP3 and DAG. Alpha 2- Dec cAMP. Beta 1- Inc JAN MIDTERM 3
correctly paired? cAMP. Beta 2- Inc cAMP. M1- Inc IP3 and DAG. M2-Dec CHRISTIAN EXAM -
A. Alpha 1- Increased cAMP cAMP. M3- Inc IP3 and DAG. H1-Inc IP3 and DAG H2- Inc FELICIANO MARCH 2016
B. Beta 1- Increased IP3 and DAG cAMP (TOP 2 - AUG
C. Beta 2- Increased cAMP 2015 MED
D. M3- Decreased cAMP BOARDS;
E. H2- Increased IP3 and DAG TOPNOTCH
MD FROM
UST)

TOPNOTCH MEDICAL BOARD PREP PHYSIOLOGY SUPEREXAM Page 9 of 88


For inquiries visit www.topnotchboardprep.com.ph or email us at topnotchmedicalboardprep@gmail.com
TOPNOTCH MEDICAL BOARD PREP PHYSIOLOGY SUPEREXAM
For inquiries visit www.topnotchboardprep.com.ph or email us at topnotchmedicalboardprep@gmail.com
Item QUESTION EXPLANATION AUTHOR TOPNOTCH
# EXAM
67 Which statement regarding urine formation is Tubular reabsorption is quantitatively more important JAN MIDTERM 3
correct? than tubular secretion in the formation of urine. Most CHRISTIAN EXAM -
A. The glomerular capillaries have a much higher substances in the plasma, except for proteins are freely FELICIANO MARCH 2016
rate of filtration than most other capillaries because filtered. Sugars such as glucose are also freely filtered. (TOP 2 - AUG
of a high glomerular oncotic pressure The glomerular capillaries have a much higher rate of 2015 MED
B. Endothelial cells of the glomerular capillary filtration than most other capillaries because of a high BOARDS;
membrane are richly endowed with fixed negative glomerular hydrostatic pressure. Although the BM is the TOPNOTCH
charges that hinder passage of plasma proteins main charger barrier of the glomerulus, the endothelium MD FROM
C. Tubular secretion is quantitatively more are also richly endowed with fixed negative charges that UST)
important than tubular reabsorption in the hinder passage of plasma proteins
formation of urine
D. Most substances in the plasma, except for proteins
and sugars, are freely filtered
E. None of the above
68 Which one of the ff is NOT part of the accomodation The accommodation reflex is a reflex action of the eye, in JAN MIDTERM 3
reflex of the eye for near objects? response to focusing on a near object. When moving CHRISTIAN EXAM -
A. Flattening of the lens focus from a distant to a near object, the eyes converge. FELICIANO MARCH 2016
B. Pupillary constriction The ciliary muscle contracts making the lens more (TOP 2 - AUG
C. Relaxation of suspensory ligaments convex, shortening its focal length. The pupil constricts 2015 MED
D. Convegence of eyeballs in order to prevent diverging light rays from hitting the BOARDS;
E. Contraction of ciliary muscle periphery of the retina. TOPNOTCH
MD FROM
UST)
69 Syndrome of Inappropriate ADH will cause what In SIADH, there is water retention with dilution of JAN MIDTERM 3
type of fluid imbalance? electrolytes especially sodium causing a hypoosmotic CHRISTIAN EXAM -
A. Isoosmotic volume expansion type of volume expansion. FELICIANO MARCH 2016
B. Hyperosmotic volume expansion (TOP 2 - AUG
C. Hypoosmotic volume expanson 2015 MED
D. Isoosmotic volume contraction BOARDS;
E. Hyperosmotic volume contraction TOPNOTCH
MD FROM
UST)
70 Cerebral edema is a result of what iatrogenic Mnemonics: Low to high, pons will die; high to low, brain JAN MIDTERM 3
phenomenon? will blow. On the other hand, CPM is due to rapid CHRISTIAN EXAM -
A. Rapid overcorrection of hyponateremia overcorrection of hyponatremia. When hypertonic FELICIANO MARCH 2016
B. Rapid overcorrection of hypernatremia solutions are added too rapidly to correct hyponatremia, (TOP 2 - AUG
C. Rapid overcorrection of hyperkalemia this can outpace the brain’s ability to recapture the 2015 MED
D. Rapid overcorrection of hypokalemia solutes lost from the cells and may lead to osmotic injury BOARDS;
E. None of the above of the neurons that is associated with demyelination, a TOPNOTCH
loss of the myelin sheath from nerves. MD FROM
UST)
71 The transprt maximum of the proximal tubules to The renal threshold is at 200 mg/dL while the transport JAN MIDTERM 3
glucose is seen at what level? maximum is at 375 mg/dL. The splay is the region in CHRISTIAN EXAM -
A. 200 mg/dL between the threshold and the transport maximum and FELICIANO MARCH 2016
B. 250 mg/dL is at 200-375 mg/dL (TOP 2 - AUG
C. 300 mg/dL 2015 MED
D. 350 mg/dL BOARDS;
E. 375 mg/dL TOPNOTCH
MD FROM
UST)
72 Which of the ff statement rearding the loop of Henle Only 25% of Na, K and Cl are reabsorbed here. H ion is JAN MIDTERM 3
is correct? secreted via Na-H countertransport. Descending limb is CHRISTIAN EXAM -
A. 50% of Na and K are reabsorbed impermeable to solutes and is the concentrating FELICIANO MARCH 2016
B. H ion is reabsorbed via Na-H countertransport segment while ascending limb is impermeable to water (TOP 2 - AUG
C. Ca and Mg are absorbed through paracellular and is the diluting segment 2015 MED
transport BOARDS;
D. The descending limb is impermeable to water TOPNOTCH
E. The ascending limb is the concentrating segment MD FROM
UST)
73 The mechanism behind the kidneys's ability to Aside from ADH levels, the countercurrent multiplier by JAN MIDTERM 3
concentrate urine? the Loops of henle CHRISTIAN EXAM -
A. Antidiuretic hormone and countercurrent exchanger by the vasa recta is also FELICIANO MARCH 2016
B. Countercurrent multiplier by the vasa recta needed to concentrate urine (TOP 2 - AUG
C. Countercurrent exchanger by the loops of Henle 2015 MED
D. All of the above BOARDS;
E. None of the above TOPNOTCH
MD FROM
UST)
74 During a physical exam, you asked a patient to smell For noxious/painful odors such has ammonia, the JAN MIDTERM 3
a cotton soaked with ammonia. What cranial nerve trigeminal nerve is stimulated. CHRISTIAN EXAM -
function are you testing? FELICIANO MARCH 2016
A. Olfactory nerve (TOP 2 - AUG
B. Facial nerve 2015 MED
C. Trigeminal nerve BOARDS;
D. Vagus nerve TOPNOTCH
E. Glossopharyngeal nerve MD FROM
UST)
75 The law of the gut is due to what phenomenon? SIMILAR TO PREVIOUS BOARD EXAM JAN MIDTERM 3
A. Peristaltic reflex CONCEPT/PRINCIPLE. . The peristaltic reflex plus the CHRISTIAN EXAM -
B. Local intermittent constrictive contractions anal direction of movement of the peristalsis due to FELICIANO MARCH 2016
C. Receptive relaxation receptive relaxation is called the “law of the gut. When a (TOP 2 - AUG
D. Both A and C segment of the intestinal tract is excited by distention 2015 MED
E. All of the above and thereby initiates peristalsis, the contractile ring BOARDS;
causing the peristalsis normally begins on the orad side TOPNOTCH
of the distended segment and moves toward the MD FROM
distended segment, pushing the intestinal contents in UST)
TOPNOTCH MEDICAL BOARD PREP PHYSIOLOGY SUPEREXAM Page 10 of 88
For inquiries visit www.topnotchboardprep.com.ph or email us at topnotchmedicalboardprep@gmail.com
TOPNOTCH MEDICAL BOARD PREP PHYSIOLOGY SUPEREXAM
For inquiries visit www.topnotchboardprep.com.ph or email us at topnotchmedicalboardprep@gmail.com
Item QUESTION EXPLANATION AUTHOR TOPNOTCH
# EXAM
the anal direction for 5 to 10 centimeters before dying
out

76 This immunoglobulin is covalently linked together IgM is in pentamers and is important in complement JAN MIDTERM 3
with disulfide bonds, mostly as a pentamer and is activation. IgG is in monomers and also activates CHRISTIAN EXAM -
important for complement activation? complement. IgA is in dimers and is secreted in the FELICIANO MARCH 2016
A. IgM respiratory and GI tract (TOP 2 - AUG
B. IgA 2015 MED
C. IgE BOARDS;
D. IgG TOPNOTCH
E. IgD MD FROM
UST)
77 Which of the ff statement regarding the lung zones is SIMILAR TO PREVIOUS BOARD EXAM JAN MIDTERM 3
NOT correct? CONCEPT/PRINCIPLE. The apex of the lung has the CHRISTIAN EXAM -
A. The base is the site of highest ventilation of the highest V/Q ratio while the base of the lung has the site if FELICIANO MARCH 2016
lung highest ventilation and perfusion. (TOP 2 - AUG
B. The base is the site of highest perfusion of the 2015 MED
lung BOARDS;
C. Zone 1 of lungs is usually not seen in normal TOPNOTCH
indivduals MD FROM
D. The base is the site of the highest V/Q ratio UST)
E. An example of a shunt is airway obstruction
78 On the day of the board exams, you skipped taking Migrating motor complexes are waves of electrical JAN MIDTERM 3
breakfast to cram for a samplex in Legal Medicine. activity that sweep through the intestines in a regular CHRISTIAN EXAM -
While taking the test, a loud rumble from your cycle during fasting. The MMC occurs every 90-120 FELICIANO MARCH 2016
abdomen was heard disturbing the other examiners. minutes during the interdigestive phase, and is (TOP 2 - AUG
This is due to what phenomenon? responsible for the rumbling experienced when hungry 2015 MED
A. Slow waves BOARDS;
B. Spike potential TOPNOTCH
C. Migrating myoelectrical complex MD FROM
D. Gastrocolic reflex UST)
E. Peristalsis
79 This is characteristic of type II muscle fiber? SIMILAR TO PREVIOUS BOARD EXAM JAN MIDTERM 3
A. Oxidative CONCEPT/PRINCIPLE. Type II msucle fiber such as the CHRISTIAN EXAM -
B. Larger in size EOMs are larger in size compared to Type I fibers. All the FELICIANO MARCH 2016
C. Low myosin ATPase rest of the choices points to Type I fiber. Please review (TOP 2 - AUG
D. Prolonged contraction duration the table found in your handouts. 2015 MED
E. Red fibers BOARDS;
TOPNOTCH
MD FROM
UST)
80 All of the ff stimulates insulin secretion EXCEPT? SIMILAR TO PREVIOUS BOARD EXAM JAN MIDTERM 3
A. Hyperglycemia CONCEPT/PRINCIPLE. The ff stimulates insulin CHRISTIAN EXAM -
B. Glucagon secretion: Increased blood glucose, FFA and AA, FELICIANO MARCH 2016
C. Cholecystokinin Gastrointestinal hormones (gastrin, cholecystokinin, (TOP 2 - AUG
D. Alpha adrenergic stimulation secretin, gastric inhibitory peptide), glucagon, growth 2015 MED
E. Gastric inhibitory peptide hormone, cortisol, parasympathetic stimulation; BOARDS;
acetylcholine, β-Adrenergic stimulation. ALpha TOPNOTCH
adrenergic stimulation and somastotatin inhibits insulin MD FROM
secretion. UST)
81 1.Which of the following is NOT factor that can alter The normal external pressure is equal to the normal ANDREW FINAL EXAM -
the external pressure on the heart and shift the intrapleural pressure (pressure in the chest cavity) TIU (TOP 1 - MARCH 2016
cardiac output curve? which is -4mmHg. A rise in the intrapleural pressure to - AUG 2015
a. cyclical changes of intrapleural pressure during 2mmHg shifts the entire cardiac output curve to the MED
respiration right by the same amount. The shift occurs because to fill BOARDS;
b. negative and positive pressure breathing the cardiac chambers with blood requres an extra TOPNOTCH
c. opening the thoracic cage 2mmHg right atrial pressure to overcome the increased MD FROM
d. cardiac tamponade pressure on the outside of the heart. CIM)
e. none of the above Guyton and Hall Medical Physiology 12th edition p. 234
82 2. A 58 year old male was admitted for dyspnea and Guyton and Hall Medical Physiology 12th edition p. 260 ANDREW FINAL EXAM -
chest pain radiating to left arm. Admitting TIU (TOP 1 - MARCH 2016
impression was massive MI and patient underwent AUG 2015
coronary revascularization. However, decreased MED
urine output was noted after. WHich of the following BOARDS;
is the most likely mechanism? TOPNOTCH
a. increased GFR MD FROM
b. increased conversion of angiotensinogen to CIM)
angiotensin I
c. activation of the parasympathetic nervous system
d. decreased potassium excretion in urine
e. increased excretion of water and salt by renal
tubules
83 3. A 27 year old male came in for complaints of back atonic bladder cause is crush injury to sacral region. If ANDREW FINAL EXAM -
pain. A month ago, patient had a motorcycle accident the damage is above the sacral region, micturition TIU (TOP 1 - MARCH 2016
and landed on his back. Spinal Cord Injury was reflexes can still occur but no longer controlled by the AUG 2015
suspected and patient is unable to control flow of brain. DUring spinal shock (first few days), micturition MED
urine with only few drops at a time through the reflexes are suppressed. Some patients can still control BOARDS;
urethra. Which of the following is true? urination by stimulating skin in the genital region. TOPNOTCH
a. the level of the spinal cord damage is above the Neurogenic bladder occurs with choice C. MD FROM
sacral region Guyton and Hall Medical Physiology 12th edition p. 310 CIM)
TOPNOTCH MEDICAL BOARD PREP PHYSIOLOGY SUPEREXAM Page 11 of 88
For inquiries visit www.topnotchboardprep.com.ph or email us at topnotchmedicalboardprep@gmail.com
TOPNOTCH MEDICAL BOARD PREP PHYSIOLOGY SUPEREXAM
For inquiries visit www.topnotchboardprep.com.ph or email us at topnotchmedicalboardprep@gmail.com
Item QUESTION EXPLANATION AUTHOR TOPNOTCH
# EXAM
b. micturition reflex contraction cannot occur
c. there is partial damage in the spinal cord or the
brain stem that interrupts most inhibitory signals
d. micturition reflexes cannot be suppressed in
spinal shock
e. none of the above

84 4. Which of the following is not a determinant of Guyton and Hall Medical Physiology 12th edition p. 317 ANDREW FINAL EXAM -
renal blood flow? TIU (TOP 1 - MARCH 2016
a. renal artery pressure AUG 2015
b. renal vein pressure MED
c. resistance in interlobular arteries BOARDS;
d. filtration fraction TOPNOTCH
e. none of the above MD FROM
CIM)
85 5. Which of the following increases ADH secretion? Guyton and Hall Medical Physiology 12th edition p. 357 ANDREW FINAL EXAM -
a. alcohol TIU (TOP 1 - MARCH 2016
b. clonidine AUG 2015
c. haloperidol MED
d. nicotine BOARDS;
e. decreased plasma osmolarity TOPNOTCH
MD FROM
CIM)
86 6. A 34 year old female came in for complaints of Guyton and Hall Medical Physiology 12th edition p. 493 ANDREW FINAL EXAM -
drowning after their banca sank during an island TIU (TOP 1 - MARCH 2016
hopping escapade in Cebu. On ABG: PaO2/ FiO2 ratio AUG 2015
was only 150. Patient was then intubated. Which of MED
the following is true of ventilation - perfusion ratio? BOARDS;
a. if V/Q is below normal, there is shunting of blood TOPNOTCH
b. if V/Q is greater than normal there is dead space MD FROM
c. if V/Q equal to zero, air in the alveolus comes to CIM)
equilibrium with the atmospheric oxygen and
carbon dioxide
d. both A and B
e. both B and C
87 7. An 9 year old female came in for 2 week history of Guyton and Hall Medical Physiology 11th edition p. 530 ANDREW FINAL EXAM -
progressive cough, weight loss, body malaise, TIU (TOP 1 - MARCH 2016
afternoon sweats, and low grade fever. SHe had a AUG 2015
history of exposure to her grandmother, her MED
guardian, who had finished a course of treatment for BOARDS;
PTB. Which of the following is true of tuberculosis in TOPNOTCH
its late stages? MD FROM
a. increased vital capacity CIM)
b. increased total respiratory membrane surface
area
c. reduced thickness of respiratory membrane
d. diminished pulmonary diffusing capacity
e. V/Q ratio is equal to 1
88 8. During a mountain trek in Mount Bulusan, Dr. JCT rhodopsin is a combination of scotopsin and retinal. It is ANDREW FINAL EXAM -
found a child with an opaque cornea and difficulty the cis form of retinal that combine with scotopsin to TIU (TOP 1 - MARCH 2016
seeing at night. Which of the following is true of the form rhodopsin. Metarhodopsin II excites electrical AUG 2015
excitation of rods? changes in the rods and then transmit the visual image MED
a. scotopsin is a combination of rhodopsin and into the CNS. Rod receptor potential is hyperpolarizing. BOARDS;
retinal Guyton and Hall Medical Physiology 11th edition p. 629 TOPNOTCH
b. only the trans form of retinal can bind with MD FROM
scotopsin CIM)
c. when light energy is absorbed by rhodopsin, it
then decomposes to bathorhodopsin
d. metarhodopsin I is the activated rhodopsin that
excites electrical changes in rods
e. rod receptor potential is depolarizing
89 9. Which of the following cells is involved in the as the visual signal progresses farther away from layer ANDREW FINAL EXAM -
detection of line orientation when a line is displaced IV, some neurons respond to lines that are oriented in TIU (TOP 1 - MARCH 2016
laterally or vertically in the visual field? the same direction but not position - specific. AUG 2015
a. X cells Guyton and Hall Medical Physiology 11th edition p. 234 MED
b. Y cells BOARDS;
c. complex cells TOPNOTCH
d. ganglion cells MD FROM
e. horizontal cells CIM)
90 10. Which of the following refers to Place principle low frequency sounds cause activation in apex and high ANDREW FINAL EXAM -
in determination of sound frequency? frequency sounds in base of cochlea. Specific brain TIU (TOP 1 - MARCH 2016
a. low frequency sounds cause maximal activation of neurons are activated by specific sound frequencies. AUG 2015
of basilar membrane near base of cochlea Guyton and Hall Medical Physiology 11th edition p. 656 MED
b. detect sound frequencies by determining BOARDS;
positions along basilar membrane most stimulated TOPNOTCH
c. spatial organization of nerve fibers from cochlea to MD FROM
cerebral cortex CIM)
d. both A and B
e. both C and D

TOPNOTCH MEDICAL BOARD PREP PHYSIOLOGY SUPEREXAM Page 12 of 88


For inquiries visit www.topnotchboardprep.com.ph or email us at topnotchmedicalboardprep@gmail.com
TOPNOTCH MEDICAL BOARD PREP PHYSIOLOGY SUPEREXAM
For inquiries visit www.topnotchboardprep.com.ph or email us at topnotchmedicalboardprep@gmail.com
Item QUESTION EXPLANATION AUTHOR TOPNOTCH
# EXAM
91 11. Which of the following is not a function of the planning complex movements and elaboration of ANDREW FINAL EXAM -
parieto-occipitotemporal association area? thoughts are part of the frontal lobe. TIU (TOP 1 - MARCH 2016
a. elaboration of thoughts Guyton and Hall Medical Physiology 11th edition p. 716 AUG 2015
b. language comprehension MED
c. processing of visual language (reading) BOARDS;
d. analysis of spatial coordinates of the body TOPNOTCH
e. naming objects MD FROM
CIM)
92 12. CDC, a good friend, has been believed to have Process requires 5 - 10 minutes for minimal ANDREW FINAL EXAM -
eidetic memory by all of his classmates being able to consolidation and 1 hour or more for strong TIU (TOP 1 - MARCH 2016
memorize every Bacteria in Jawetz. Which of the consolidation. AUG 2015
following is not likely true of consolidation of Guyton and Hall Medical Physiology 11th edition p. 726 MED
memory? BOARDS;
a. for short term memory to be converted to long TOPNOTCH
term memory, it must be consolidated MD FROM
b. process requires 1 hour of minimal consolidation CIM)
and 2 hours for strong consolidation
c. deep anesthesia or brain concussion can inhibit
consolidation
d. rehearsal enhances transference of short term
into long term memory
e. new memories are codified during consolidation
93 13. Which of the following is NOT true of an increased rate of blood coagulation in SNS response ANDREW FINAL EXAM -
sympathetic nervous system reponse? Guyton and Hall Medical Physiology 11th edition p. 758 TIU (TOP 1 - MARCH 2016
a. capable of mass discharge AUG 2015
b. decreased rate of formation of thrombin MED
c. increased muscle strength BOARDS;
d. decreased renal blood flow TOPNOTCH
e. increased glycolysis MD FROM
CIM)
94 14. Which of the following factors are continually Guyton and Hall Medical Physiology 11th edition p. 786 ANDREW FINAL EXAM -
monitored in the duodenum and can initiate TIU (TOP 1 - MARCH 2016
enterogastric inhibitory reflexes except? AUG 2015
a. degree of distention of duodenum MED
b. acidity of chyme BOARDS;
c. osmolarity of chyme TOPNOTCH
d. amino acids MD FROM
e. none of the above CIM)
95 15. Which of the following is true of pancreatic acid from stomach releases secretin from wall of ANDREW FINAL EXAM -
secretion except? duodenum. vagal stimulation releases enzymes into TIU (TOP 1 - MARCH 2016
a. acids from stomach cause release of CCK acini. AUG 2015
b. Vagal stimulation releases bicarbonate rich fluid Guyton and Hall Medical Physiology 11th edition p. 802. MED
into acini BOARDS;
c. CCK causes secretion of enzyme into acini TOPNOTCH
d. both A and B MD FROM
e. both B and C CIM)
96 16. Which of the following inhibits growth hormone other factors that increase GH secretion: decreased ANDREW FINAL EXAM -
secretion except? glucose and free fatty acids, fasting, stress, excitement, TIU (TOP 1 - MARCH 2016
a. aging exercise, testosterone, estrogen, deep sleep, GHRH AUG 2015
b. obesity Guyton and Hall Medical Physiology 11th edition p. 925 MED
c. trauma BOARDS;
d. somatomedins TOPNOTCH
e. increased free fatty acids MD FROM
CIM)
97 17. Which of the following is an effect of thyroid Guyton and Hall Medical Physiology 11th edition p. 937 ANDREW FINAL EXAM -
hormone except? TIU (TOP 1 - MARCH 2016
a. decreased phospholipids concentrations in plasma AUG 2015
b. decreased requirement for vitamins MED
c. increased blood flow to skin BOARDS;
d. increased heart strength TOPNOTCH
e. exhausting effect of musculoskeletal system MD FROM
CIM)
98 18. Which of the following is the most important in Guyton and Hall Medical Physiology 11th edition p. 950 ANDREW FINAL EXAM -
regulation of aldosterone secretion? TIU (TOP 1 - MARCH 2016
a. angiotensin II levels AUG 2015
b. potassium levels MED
c. sodium concentrations BOARDS;
d. ACTH secretion TOPNOTCH
e. none of the above MD FROM
CIM)
99 19. Which of the following is true of fluoride in the Guyton and Hall Medical Physiology 11th edition p. 994 ANDREW FINAL EXAM -
prevention of caries EXCEPT? TIU (TOP 1 - MARCH 2016
a. small amounts of fluoride in water make enamel AUG 2015
resistant to caries MED
b. fluoride makes enamel harder than usual BOARDS;
c. fluorine ions replace hydroxyl ions in TOPNOTCH
hydroxyapatite crystals which in turn makes enamel MD FROM
less soluble CIM)
d. fluoride promotes deposition of calcium in enamel
pits
e. all of the above

TOPNOTCH MEDICAL BOARD PREP PHYSIOLOGY SUPEREXAM Page 13 of 88


For inquiries visit www.topnotchboardprep.com.ph or email us at topnotchmedicalboardprep@gmail.com
TOPNOTCH MEDICAL BOARD PREP PHYSIOLOGY SUPEREXAM
For inquiries visit www.topnotchboardprep.com.ph or email us at topnotchmedicalboardprep@gmail.com
Item QUESTION EXPLANATION AUTHOR TOPNOTCH
# EXAM
100 20. ESC, Mr. Medlympics 2013, loves to workout Guyton and Hall Medical Physiology 11th edition p. 1060 ANDREW FINAL EXAM -
when he is stressed. He has good foundations on TIU (TOP 1 - MARCH 2016
physiology. Which of the following statements by AUG 2015
ESC is not true regarding changes that occur in MED
hypertrophied muscle? BOARDS;
a. increased number of myofibrils TOPNOTCH
b. 120% increase in mitochondrial enzymes MD FROM
c. 60 - 80% increase in phosphagen metabolic CIM)
system
d. 50% increase in stored glycogen
e. none of the above
101 Albumin has an osmotic coefficient of:
Page 5 of Topnotch Handout A reflection coefficient or ANGELA DIAGNOSTIC
A. 0 osmotic coefficient of ONE means here is no solute PAULINE P. EXAM - AUG
B. 1 penetration, therefore it creates water flow. CALIMAG- 2015
C. 2 LOYOLA
D. 3 (TOP 8 - FEB
E. 4 2015 MED
BOARDS;
TOPNOTCH
MD FROM
UST)
102 An example of secondary active transport:
Page 5 of Topnotch Handout. Secondary active transport ANGELA DIAGNOSTIC
A. Amino acid transport indirectly relies on the Na-K-ATPase pump. It requires PAULINE P. EXAM - AUG
B. H-K ATPase pump in intercalated cells of the Na+. Examples of which are SGLT-1 in the S.I., SGLT-2 in CALIMAG- 2015
kidney the PCT, Na-K-2Cl in the ascending tubule, Na-Ca LOYOLA
C. Proton Pump of the parietal cells in the stomach exchange in almost all cells, Na-H exchange in the PCT (TOP 8 - FEB
D. D-Glucose transport to adipose tissues (kidneys). 2015 MED
E. Na-H exchange in the PCT BOARDS;
TOPNOTCH
MD FROM
UST)
103 Serotonin an inhibitor of pain pathways in the Page 7 of Topnotch Handout. Serotonin is converted to ANGELA DIAGNOSTIC
spinal cord derived from Tryptophan, which is melatonin in the pineal gland. PAULINE P. EXAM - AUG
secreted mainly by the median raphe of the brain CALIMAG- 2015
stem is converted to melatonin in the: LOYOLA
A. Pineal gland (TOP 8 - FEB
B. Locus ceruleus in the pons 2015 MED
C. Substancia nigra pars compacta BOARDS;
D. Ventral tegmental area TOPNOTCH
E. Nucleus basalis of Meynert MD FROM
UST)
104 During repolarization of the nerve action potential, Page 8 of Topnotch Handout. In the nerve action ANGELA DIAGNOSTIC
opening of the potassium gates causes:
potential repolarization, opening of the potassium gates PAULINE P. EXAM - AUG
A. Sodium influx causes potassium efflux, while closure of Na-inactivation CALIMAG- 2015
B. Sodium efflux gates stops sodium influx making the inside of the cell LOYOLA
C. Calcium influx more negative. (TOP 8 - FEB
D. Potassium efflux 2015 MED
E. Potassium influx BOARDS;
TOPNOTCH
MD FROM
UST)
105 The following mechanisms for acclimatization to Page 16 of Topnotch Handout. The mecahnisms for ANGELA DIAGNOSTIC
Low PO2 are correct, except:
acclimatization to low PO2 are: Increased pulmonary PAULINE P. EXAM - AUG
A. Respiratory rate increases by 1.65x ventilation, polycythemia (increased 2,3 BPG shift to the CALIMAG- 2015
B. 2,3 BPG increases shifting the O2-Hgb right of the Hgb curve.), increased diffusing capacity of LOYOLA
dissociation curve to the left oxygen increased vascularity of the peripheral tissues, (TOP 8 - FEB
C. Increased pulmonary capillary blood volume increased ability of cells to use oxygen despite low PO2. 2015 MED
D. Cardiac output increases by 30% BOARDS;
E. Increased cell mitochondria and cellular TOPNOTCH
oxidative enzymes MD FROM
UST)
106 What is the main trigger for the intercalated cells in Page 17 of Topnotch Handout. Hypoxia is the trigger for ANGELA DIAGNOSTIC
the peritubular capillaries to secrete EPO?
EPO secretion. It takes 3-5 days to correct hypoxia by PAULINE P. EXAM - AUG
A. Anemia EPO. CALIMAG- 2015
B. Low Hgb LOYOLA
C. Low Hct (TOP 8 - FEB
D. Hypocarbia 2015 MED
E. Hypoxia BOARDS;
TOPNOTCH
MD FROM
UST)
107 These blood vessels act as control conduits of blood Page 1 of Topnotch Handout part2. Arterioles are the ANGELA DIAGNOSTIC
flow and are the main detrminants of TPR:
control conduits of blood flow and act mainly under the PAULINE P. EXAM - AUG
A. Arteries sympathetic NS control. Alpha 1-vasoconstriction; CALIMAG- 2015
B. Arterioles Beta2-Vasodilatation. LOYOLA
C. Capillaries (TOP 8 - FEB
D. Veins 2015 MED
E. Venules BOARDS;
TOPNOTCH
MD FROM
UST)

TOPNOTCH MEDICAL BOARD PREP PHYSIOLOGY SUPEREXAM Page 14 of 88


For inquiries visit www.topnotchboardprep.com.ph or email us at topnotchmedicalboardprep@gmail.com
TOPNOTCH MEDICAL BOARD PREP PHYSIOLOGY SUPEREXAM
For inquiries visit www.topnotchboardprep.com.ph or email us at topnotchmedicalboardprep@gmail.com
Item QUESTION EXPLANATION AUTHOR TOPNOTCH
# EXAM
108 Ohm's law which is the basis for the formula of SIMILAR TO PREVIOUS BOARD EXAM ANGELA DIAGNOSTIC
BP=CO x TPR states that Flow (Q) is:
CONCEPT/PRINCIPLE. Page 2 of Topnotch Handout PAULINE P. EXAM - AUG
A. Directly proportional to resistance and pressure part2 only. Based on Ohm's law Flow is directly CALIMAG- 2015
difference proportional to pressure difference and indirectly LOYOLA
B. Indirectly proportional to resistance and proportional to resistance. (TOP 8 - FEB
pressure difference 2015 MED
C. Directly proportional to resistance only BOARDS;
D. Directly proportional to pressure difference only TOPNOTCH
E. Indirectly proportional to pressure difference MD FROM
only UST)
109 Which part of the ECG tracing correlates with the SIMILAR TO PREVIOUS BOARD EXAM ANGELA DIAGNOSTIC
plateau of ventricular action potential?
CONCEPT/PRINCIPLE, The part of ecg which corelates PAULINE P. EXAM - AUG
A. T wave with the ventricular repolarization is the T wave. Page 3 CALIMAG- 2015
B. P wave of Topnotch Handout part2. St segment corelates with LOYOLA
C. QRS complex the plateau of ventriular action potential. (TOP 8 - FEB
D. QT interval 2015 MED
E. ST segment BOARDS;
TOPNOTCH
MD FROM
UST)
110 In a patient with hypocalcemia which ECG finding Page 3 of Topnotch Handout part 2. In a patient with ANGELA DIAGNOSTIC
will you see?
hypocalcemia the ecg findings would be a decreased PAULINE P. EXAM - AUG
A. Prolonged QT interval and decreased heart rate heart rate and prolonged QT interval. B-Hyperkalemia; CALIMAG- 2015
B. Low P waves, Peaked T waves C-Hypokalemia; D-STEMI; E Hypokalemia LOYOLA
C. Flat T waves with u waves (TOP 8 - FEB
D. ST segment elevation 2015 MED
E. Inverted T waves BOARDS;
TOPNOTCH
MD FROM
UST)
111 This component of the conductive system of the Page 4 of part 2 Topnotch Handout. SA node-70-80bpm; ANGELA DIAGNOSTIC
heart has the slowest intrinsic firing rate:
AV node-40-60bpm, Bundle of His-40bpm, purkinje PAULINE P. EXAM - AUG
A. Bundle of His fibers-15-20bpm. The intrinsic firing rate is different CALIMAG- 2015
B. Purkinje fibers from the conduction velocity, the AV node has the LOYOLA
C. AV node slowest conduction velocity of 0.01-0.05 m/sec while the (TOP 8 - FEB
D. SA node Bundle of his, purkinje fibers and ventricles hve the 2015 MED
E. ALL of the above have equal intrinsic firing rates fastest at 2-4 m/sec. BOARDS;
TOPNOTCH
MD FROM
UST)
112 The following conditions causes circus movements Page 4 of part 2 Topnotch Handout. Circus movements ANGELA DIAGNOSTIC
except:
occurs when in the propagation of AP around the PAULINE P. EXAM - AUG
A. Dilated cardiomyopathy ventricles, the signal never reaches an area with ARP. CALIMAG- 2015
B. Ischemic heart The causes of circus movements are Long conduction LOYOLA
C. Electrical stimulation of the heart pathway( dilated cardiomyopathy); Decreased (TOP 8 - FEB
D. Administration of epinephrine conduction velocity (ischemic heart, hyperkalemia, 2015 MED
E. Hypokalemia blocked purkinje); Short refractory period (epinephrine, BOARDS;
electrical stimulation). TOPNOTCH
MD FROM
UST)
113 According to the Frank-Starling Mechanism, when Page 5 of part 2 Topnotch Handout. When afterload ANGELA DIAGNOSTIC
Afterload increases:
increases, SV, CO and velocity of sarcomere shortening PAULINE P. EXAM - AUG
A. Stroke volume decreases decreases. CALIMAG- 2015
B. Cardiac output increases LOYOLA
C. The velocity of sarcomere shortening increases (TOP 8 - FEB
D. Stroke volume increases 2015 MED
E. None of the above BOARDS;
TOPNOTCH
MD FROM
UST)
114 In which phase of the cardiac cycle is the incisura of Page 6 of part 2 Topnotch Handout. It is found in the ANGELA DIAGNOSTIC
aortic pressure seen?
isovolumic relaxation part of the cycle. Due to the slight PAULINE P. EXAM - AUG
A. Isovolumic contraction increase in aortic pressure during isovolumic relaxation. CALIMAG- 2015
B. Rapid ventricular ejection LOYOLA
C. Reduced ventricular ejection (TOP 8 - FEB
D. Isovolumic relaxation 2015 MED
E. Rapid ventricular filling BOARDS;
TOPNOTCH
MD FROM
UST)
115 Which is considered as the most potent Page 10 of part 2 Topnotch Handout. According to ANGELA DIAGNOSTIC
vasoconstrictor?
physio books the most potent vasoconstrictor is PAULINE P. EXAM - AUG
A. Prostacyclin vasopressin, according to schwartz however it is CALIMAG- 2015
B. Vasopressin endothelin. LOYOLA
C. Endothelin (TOP 8 - FEB
D. Epinephrine 2015 MED
E. Norepinephrine BOARDS;
TOPNOTCH
MD FROM
UST)

TOPNOTCH MEDICAL BOARD PREP PHYSIOLOGY SUPEREXAM Page 15 of 88


For inquiries visit www.topnotchboardprep.com.ph or email us at topnotchmedicalboardprep@gmail.com
TOPNOTCH MEDICAL BOARD PREP PHYSIOLOGY SUPEREXAM
For inquiries visit www.topnotchboardprep.com.ph or email us at topnotchmedicalboardprep@gmail.com
Item QUESTION EXPLANATION AUTHOR TOPNOTCH
# EXAM
116 This lung volume can not be measured directly by Page 12 of part 2 Topnotch Handout. This is a function of ANGELA DIAGNOSTIC
spirometry but it is important in maintaining the Residual volume. PAULINE P. EXAM - AUG
oxygenation in between breaths: A. Inspiratory CALIMAG- 2015
reserve volume LOYOLA
B. Tidal volume (TOP 8 - FEB
C. Expiratory reserve volume 2015 MED
D. Vital capacity BOARDS;
E. Residual volume TOPNOTCH
MD FROM
UST)
117 Central chemoreceptors found in the ventral Page 16 of part2 Topnotch Handout. ANGELA DIAGNOSTIC
medulla increases respiratory rate by responding PAULINE P. EXAM - AUG
directly to: A. CSF H+ CALIMAG- 2015
B. PaO2 < 70mmHg LOYOLA
C. High PaCO2 (TOP 8 - FEB
D. Low pH 2015 MED
E. All of the above BOARDS;
TOPNOTCH
MD FROM
UST)
118 Cholecystokinin has the following actions except:
Page 18 of part2 Topnotch Handout. D is a function of ANGELA DIAGNOSTIC
A. Stimulates the gallbladder to contract motilin PAULINE P. EXAM - AUG
B. Increases pancreatic HCO3 secretion CALIMAG- 2015
C. Inhibits Gastric emptying LOYOLA
D. Stimulates clearing of GI tract in between meals (TOP 8 - FEB
E. Increases growth of exocrine pancreas 2015 MED
BOARDS;
TOPNOTCH
MD FROM
UST)
119 The percentage of HCL secretion is highest in the:
Page 22 of part 2 Topnotch Handout. The percentage of ANGELA DIAGNOSTIC
A. Cephalic phase HCL acid secretion is highest in the gastric phase at 60%. PAULINE P. EXAM - AUG
B. Intestinal phase 30% only in the cephalic phase, and 10% only in the CALIMAG- 2015
C. Gastric phase intestinal phase. LOYOLA
D. Both A and B (TOP 8 - FEB
E. Same in all phases 2015 MED
BOARDS;
TOPNOTCH
MD FROM
UST)
120 In the excitation-contraction of skelatal muscle the SIMILAR TO PREVIOUS BOARD EXAM ANGELA DIAGNOSTIC
following events take place except:
CONCEPT/PRINCIPLE. Page 3 of part3 Topnotch PAULINE P. EXAM - AUG
A. Depolarization of T tubules Handout. There is an increase iin intracellular calcium CALIMAG- 2015
B. Activation of Dihydroyridine receptor concentration. LOYOLA
C. Decrease in intracellular calcium concentration (TOP 8 - FEB
D. Calcium binds troponin C 2015 MED
E. Tropomyosin moves and allows interaction of BOARDS;
actin and myosin TOPNOTCH
MD FROM
UST)
121 Which of the following is responsible for dull aching Type A delta fibers - responsible for sharp, pricking, LYNN DARYL MIDTERM 1
pain? acute pain while or fast pain, mediated by glutamate FELICIANO EXAM - AUG
A. Typa A-beta fibers while Type C fibers is responsible for dull, aching, slow, VILLAMATE 2015
B. Type A-delta fibers buring, aching, throbbing, and chronic pain. SIMILAR TO R, MD (TOP 5
C. Type A-gamma fibers PREVIOUS BOARD EXAM CONCEPT. - FEB 2015
D. Type B fibers MED
E. Type C fibers BOARDS;
TOPNOTCH
MD FROM
EAC)
122 If the patient's hemoglobin is 15 g/dL, the Each gram of hemoglobin can carry 1.34 mL of oxygen. LYNN DARYL MIDTERM 1
approximate oxygen content would be ____ml/100 15 g x 1.34 ml = 20.1 or ~20 ml/100 ml. SIMILAR TO FELICIANO EXAM - AUG
ml of blood. PREVIOUS BOARD EXAM CONCEPT. VILLAMATE 2015
A. 15 R, MD (TOP 5
B. 20 - FEB 2015
C. 30 MED
D. 45 BOARDS;
E. 60 TOPNOTCH
MD FROM
EAC)
123 A 48-year old, 60 kg, has a tidal volume of 500 ml, Minute ventilation = Tidal volume x breaths/min; LYNN DARYL MIDTERM 1
respiratory rate of 20 breaths/min. PCO2 in his MV=0.5L x 20 breaths/min = 10 L/min FELICIANO EXAM - AUG
arterial blood is 35 mmHg. What is the minute VILLAMATE 2015
ventilation? R, MD (TOP 5
A. 0.125 L/min - FEB 2015
B. 7 L/min MED
C. 10 L/min BOARDS;
D. 35 L/min TOPNOTCH
E. 60 L/min MD FROM
EAC)

TOPNOTCH MEDICAL BOARD PREP PHYSIOLOGY SUPEREXAM Page 16 of 88


For inquiries visit www.topnotchboardprep.com.ph or email us at topnotchmedicalboardprep@gmail.com
TOPNOTCH MEDICAL BOARD PREP PHYSIOLOGY SUPEREXAM
For inquiries visit www.topnotchboardprep.com.ph or email us at topnotchmedicalboardprep@gmail.com
Item QUESTION EXPLANATION AUTHOR TOPNOTCH
# EXAM
124 The approximate blood pressure in the right atrium 0 mmHg - vena cava and RA; 17 mmHg - Capillaries; LYNN DARYL MIDTERM 1
is? 25/8 mmHg in pulmonary artery; 7 mmHg in Pulmonary FELICIANO EXAM - AUG
A. 0 mmHg capillaries; 120/80 Aorta and large arteries. SIMILAR TO VILLAMATE 2015
B. 25/8 mmHg PREVIOUS BOARD EXAM CONCEPT. R, MD (TOP 5
C. 7 mmHg - FEB 2015
D. 17 mmHg MED
E. 120/80 mmHg BOARDS;
TOPNOTCH
MD FROM
EAC)
125 Two-point discrimination is a function of: Pacinian - high frequency vibration; Meissner's - low LYNN DARYL MIDTERM 1
A. Pacinian corpuscle frequency vibration, velocity; Ruffini's - pressure, FELICIANO EXAM - AUG
B. Meissner's corpuscle degree of joint rotation; hair-end organ - movement of VILLAMATE 2015
C. Ruffini's end organ object on skin; Merkel's - location, texture, SIMILAR TO R, MD (TOP 5
D. Hair-end organ PREVIOUS BOARD EXAM CONCEPT - FEB 2015
E. Merkel's disc MED
BOARDS;
TOPNOTCH
MD FROM
EAC)
126 Which of the following is true regarding how sound The base of the basilar membrane near the oval and LYNN DARYL MIDTERM 1
is encoded? round windows is narrow and stiff. It responds best to FELICIANO EXAM - AUG
A. The base of the basilar membrane is wide and high frequencies. The apex of the basilar membrane VILLAMATE 2015
compliant, hence it responds best to low (near the helicotrema) is wide and compliant and R, MD (TOP 5
frequencies. responds best to low frequencies. - FEB 2015
B. The part of the basilar membrane near the MED
helicotroma responds best to low frequencies. BOARDS;
C. The apex of the basilar membrane is narrow and TOPNOTCH
stiff responds best to high and moderate MD FROM
frequencies. EAC)
D. The part of the basilar membrane near the oval
window is wide and compliant.
E. None of the above
127 Which of the following characteristics is shared by Simple and facilitated diffusion does not require LYNN DARYL MIDTERM 1
simple and facilitated diffusion? metabolic energy (non-ATP requiring). Stereospecificity, FELICIANO EXAM - AUG
A. Inhibition of Na+-K+ pump saturation and competition are characteristics of carrier- VILLAMATE 2015
B. Stereospecificity mediated transport. Simple diffusion is the only R, MD (TOP 5
C. Carrier-mediated transport mechanism that is not carrier-mediated. - FEB 2015
D. Does not require metabolic energy Primary active transport requires direct input of MED
E. Saturation metabolic enery in the form of ATP. BOARDS;
TOPNOTCH
MD FROM
EAC)
128 The most potent stimulus to the central LYNN DARYL MIDTERM 1
chemoreceptor center is: FELICIANO EXAM - AUG
A. Hypoxia VILLAMATE 2015
B. Hypercapnea R, MD (TOP 5
C. Acidosis - FEB 2015
D. Hypoventilation MED
E. A and C BOARDS;
TOPNOTCH
MD FROM
EAC)
129 Which of the following responses is NOT mediated Erection - parasympathetic effect LYNN DARYL MIDTERM 1
by sympathetic receptors? FELICIANO EXAM - AUG
A. Erection VILLAMATE 2015
B. Skeletal muscle vasodilation R, MD (TOP 5
C. Dilation of bronchiolar smooth muscle - FEB 2015
D. Mydriasis MED
E. GI Sphinter contraction BOARDS;
TOPNOTCH
MD FROM
EAC)
130 Why is testosterone given for osteoporosis? Testosterone increases bone matrix and causes calcium LYNN DARYL MIDTERM 1
A. It increases the size and strength of the bones. retention. Because of the ability of testosterone to FELICIANO EXAM - AUG
B. It prevents phosphate excretion. increase the size and strength of bones, it is often used in VILLAMATE 2015
C. It inhibits digestion of bone by encouraging older men to treat osteoporosis. (Guyton). SIMILAR TO R, MD (TOP 5
osteoclasts to undergo apoptosis. PREVIOUS BOARD EXAM CONCEPT. - FEB 2015
D. It counteracts the side effects of anti- MED
osteoporotic drug medications. BOARDS;
E. It prevents Vitamin D metabolism and excretion. TOPNOTCH
MD FROM
EAC)
131 Implantation of the blastocyst in the endometrium After reaching the uterus, the developing blastocyst LYNN DARYL MIDTERM 1
occurs how many days after ovulation? usually remains in the uterine cavity an additional 1-3 FELICIANO EXAM - AUG
A. Immediately after fertilization days before it implants in the endometrium; thus, VILLAMATE 2015
B. 3 days implantation ordinarily occurs on about the 5th to 7th R, MD (TOP 5
C. 5-7 days day after ovulation. (Guyton) SIMILAR TO PREVIOUS - FEB 2015
D. 8-10 days BOARD EXAM CONCEPT. MED
E. 2 weeks BOARDS;
TOPNOTCH
MD FROM
EAC)

TOPNOTCH MEDICAL BOARD PREP PHYSIOLOGY SUPEREXAM Page 17 of 88


For inquiries visit www.topnotchboardprep.com.ph or email us at topnotchmedicalboardprep@gmail.com
TOPNOTCH MEDICAL BOARD PREP PHYSIOLOGY SUPEREXAM
For inquiries visit www.topnotchboardprep.com.ph or email us at topnotchmedicalboardprep@gmail.com
Item QUESTION EXPLANATION AUTHOR TOPNOTCH
# EXAM
132 A 45-year old female patient presented with This is a case of aortic regurgitation. When aortic valve LYNN DARYL MIDTERM 1
shortness of breath and and chest pain. Physical fails to close completely, blood flows back into the left FELICIANO EXAM - AUG
examination revealed tachycardia and diastolic ventricle after ejection which increases LV filling, hence VILLAMATE 2015
mumur that is loudest at the right upper sternal increased LV EDV. The increased preload increases the R, MD (TOP 5
border. What is the mechanism characteristic of this force of contraction thus increasing the aortic systolic - FEB 2015
condition? pressure. The aortic diastolic pressure is much lower MED
A. Decreased left ventricular end-diastolic volume than normal because blood more rapidly leaves aorta BOARDS;
B. Decreased force of contraction due to regurgitation. A defining characteristic of AR is an TOPNOTCH
C. Elevation of aortic diastolic pressure increase in aortic pulse pressure. MD FROM
D. Elevation of pulse pressure EAC)
E. Increased afterload
133 A -60 year old woman presented with severe This is a case of congestive heart failure which may LYNN DARYL MIDTERM 1
breathlessness, tachypnea, apprehension, and cause progression to acute pulmonary edema. FELICIANO EXAM - AUG
diaphoresis. Which of the following will be most Pulmonary wedge capillary pressure (PCWP) is used to VILLAMATE 2015
helpful to differentiate pulmonary edema from non- approximate the left ventricular end diastolic pressure. R, MD (TOP 5
cardiogenic cause? High PCWP may indicate left ventricular failure, mitral - FEB 2015
A. Aortic Pressure valve pathology or cardiac insufficinecy. Definite MED
B. Right ventricular pressure differentiation of cardiogenic from non-cardiogenic BOARDS;
C. Central venous pressure pulmonary edema is based on pulmonary capillary TOPNOTCH
D. Pulmonary capillary wedge pressure wedge pressure (PCWP) measurements. MD FROM
E. Mean arterial pressure EAC)
134 Which of the following will cause decrease secretion Aldosterone increases K+secretion. The mechanism LYNN DARYL MIDTERM 1
of K+ in the distal tubule? involves increased sodium entry entry into the cells FELICIANO EXAM - AUG
A. High potassium diet across the luminal membrane and increased pumping of VILLAMATE 2015
B. Metabolic alkalosis Na+ out of the cells by Na+-K+ pump. Stimulation of the R, MD (TOP 5
C. Thiazide diuretics Na+-K+ pump simultaneously increases K uptake into - FEB 2015
D. Hypoaldosteronism the principal cellls, increasing the intracellular K+ MED
E. Treatment with beta agonist concentration and driving force for K+ secretion. BOARDS;
Hypoaldosteronism decreses K" secretion and causes TOPNOTCH
hyperkalemia. MD FROM
EAC)
135 The regulation of body temperature is primarily SIMILAR TO PREVIOUS BOARD EXAM CONCEPT. LYNN DARYL MIDTERM 1
mediated by? FELICIANO EXAM - AUG
A. Skin VILLAMATE 2015
B. Hypothalamus R, MD (TOP 5
C. Thalamus - FEB 2015
D. Medulla MED
E. Pons BOARDS;
TOPNOTCH
MD FROM
EAC)
136 Which of the following would cause an increase in GFR is the filtration across the glomerular capillaries and LYNN DARYL MIDTERM 1
glomerular filtration rate? is affected by Starling forces across the glomerular FELICIANO EXAM - AUG
A. Constriction of efferent arteriole capillaries. Moderate constriction of efferent arteriole VILLAMATE 2015
B. Ureteral stone causes increase in GFR. All other options decreases the R, MD (TOP 5
C. Increased plasma protein GFR. - FEB 2015
D. Constriction of afferent arteriole MED
E. Increased bowman;s space hydrostatic pressure BOARDS;
TOPNOTCH
MD FROM
EAC)
137 The main function of gastrointestinal peristalsis is The main function of peristalsis is to propel the food and LYNN DARYL MIDTERM 1
to: chyme down the lumen of the bowel. Options B and D FELICIANO EXAM - AUG
A. Increase surface area for easy digestion of food are functions of segmentation. SIMILAR TO PREVIOUS VILLAMATE 2015
B. Mixing food or chyme BOARD EXAM CONCEPT. R, MD (TOP 5
C. Move food and chyme down the lumen of the - FEB 2015
bowel MED
D. Ensure contact with intestinal epithelium for BOARDS;
proper absorption. TOPNOTCH
E. All of the above MD FROM
EAC)
138 The following are effects of thyroid hormone Glycogenolysis, gluconeogenesis, and glucose oxidation, LYNN DARYL MIDTERM 1
EXCEPT: driven by demand for ATP are increased. Lipolysis is also FELICIANO EXAM - AUG
A. Maturation of CNS increased. Protein synthesis and degradation are also VILLAMATE 2015
B. Up-regulation of B1-adrenergic receptors in the increased. All other options are effects of thyroid R, MD (TOP 5
heart hormone. - FEB 2015
C. Increases O2 consumption and basal metabolic MED
rate BOARDS;
D. Decreased glycogenolysis and gluconeogenesis TOPNOTCH
E. Increased lipolysis MD FROM
EAC)
139 Which of the following is true regarding Option B is incorrect. Without testosterone, the wolffian LYNN DARYL MIDTERM 1
testosterone? ducts do not differentiate. Option C. Inhibin is produced FELICIANO EXAM - AUG
A. It is the major androgen synthesized and by Sertoli cells which inhibit secreetion of FSH. VILLAMATE 2015
secreted by Leydig cells Testosterone inhibits the secretion of LH by inhibiting R, MD (TOP 5
B. Without testosterone, mullerian ducts are release of GnRH. Testosterone is responsible for - FEB 2015
suppressed and therefore develop into male genital pubertal growth spurt, however, it causes epiphyseal MED
tract. closure, which also causes cessation of pubertal growth BOARDS;
C. Inhibin is produced by Leydig cells, which spurt. (Option A- board exam question) TOPNOTCH
inhibits secretion of testosterone. MD FROM
D. Testosterone prevent epiphyseal closure, hence, EAC)
promotion of pubertal growth spurt.
E. All of the above.

TOPNOTCH MEDICAL BOARD PREP PHYSIOLOGY SUPEREXAM Page 18 of 88


For inquiries visit www.topnotchboardprep.com.ph or email us at topnotchmedicalboardprep@gmail.com
TOPNOTCH MEDICAL BOARD PREP PHYSIOLOGY SUPEREXAM
For inquiries visit www.topnotchboardprep.com.ph or email us at topnotchmedicalboardprep@gmail.com
Item QUESTION EXPLANATION AUTHOR TOPNOTCH
# EXAM
140 Which of the following acts by activation of second Insulin - tyrosine kinase; TRH and Angiotensin II on LYNN DARYL MIDTERM 1
messenger cyclic adenosine monophosphate system vascular smooth muscle - IP3/DAG; Progesterone - FELICIANO EXAM - AUG
of the cell? steroid hormone/cellular receptor VILLAMATE 2015
A. Insulin R, MD (TOP 5
B. TRH - FEB 2015
C. Angiotensin Iin on vascular smooth muscle MED
D. Glucagon BOARDS;
E. Progesterone TOPNOTCH
MD FROM
EAC)
141 1 g of hemoglobin can carry how much oxygen Remember that oxygen carrying capacity of Hgb is 1.34 EDWARD MIDTERM 2
A. 1.34 ml ml of O2/g of Hgb. HARRY EXAM - AUG
B. 1.35 ml VALLAJERA, 2015
C. 1.38 ml MD (TOP 8 -
D. 1.24 ml FEB 2015
E. 1.33 ml MED
BOARDS;
TOPNOTCH
MD FROM
PERPETUAL
BINAN)
142 The cornerstone in the management of pre-renal Hydration is important in pre-renal ARF as it restores EDWARD MIDTERM 2
acute renal failure is? intravascular volume thus restoring renal blood flow and HARRY EXAM - AUG
A. Antibiotics increasing urine output VALLAJERA, 2015
B. Investigating for post-renal causes MD (TOP 8 -
C. Hydration FEB 2015
D. Ruling out masses impinging on the urinary tract MED
E. None of the above BOARDS;
TOPNOTCH
MD FROM
PERPETUAL
BINAN)
143 A person with 15g of hemoglobin can transport this 15 x 1.34 = 20.1 EDWARD MIDTERM 2
much oxygen? HARRY EXAM - AUG
A. 15 VALLAJERA, 2015
B. 20 MD (TOP 8 -
C. 30 FEB 2015
D. 25 MED
E. 35 BOARDS;
TOPNOTCH
MD FROM
PERPETUAL
BINAN)
144 All nucleated cells bear __________ proteins on their All cells of the body bear the MHC I protein on the cell EDWARD MIDTERM 2
surface: membrane and serves as their ID so that they are not HARRY EXAM - AUG
A. MHC 1 identified as foreign cells. VALLAJERA, 2015
B. MHC 2 MD (TOP 8 -
C. MHC 3 FEB 2015
D. MHC 4 MED
E. None of the above BOARDS;
TOPNOTCH
MD FROM
PERPETUAL
BINAN)
145 A call center agent ingests coffee to keep himself Caffeine stimulates the secretion of gastrin EDWARD MIDTERM 2
awake during the night, one of the pharmacologic HARRY EXAM - AUG
effects of coffee is the release of: VALLAJERA, 2015
A. Epinephrine MD (TOP 8 -
B. Glucagon FEB 2015
C. Insulin MED
D. Gastrin BOARDS;
E. None of the above TOPNOTCH
MD FROM
PERPETUAL
BINAN)
146 Which of the following statement/s is/are true? All of the above are true statements EDWARD MIDTERM 2
A. Neutrophils are typically one of the first WBCs to HARRY EXAM - AUG
arrive at a site of infection VALLAJERA, 2015
B. Macrophages contain lysosomes which contain MD (TOP 8 -
digestive enzymes FEB 2015
C. Eosinophils help defend against parasitic MED
infections BOARDS;
D. Natural killer cells attack and kill virus infected TOPNOTCH
cells MD FROM
E. All of the above PERPETUAL
BINAN)
147 Lymph is moved through lymphatic vessels because Lymph moves through the lymphatic system primarily EDWARD MIDTERM 2
of: due to the contraction of the skeletal muscles. HARRY EXAM - AUG
A. Ventricular contraction VALLAJERA, 2015
B. Skeletal Muscle contraction MD (TOP 8 -
C. Presence of cortical sinuses FEB 2015
D. Shunting from the bloodstream MED
E. None of the above BOARDS;
TOPNOTCH
MD FROM
PERPETUAL
TOPNOTCH MEDICAL BOARD PREP PHYSIOLOGY SUPEREXAM Page 19 of 88
For inquiries visit www.topnotchboardprep.com.ph or email us at topnotchmedicalboardprep@gmail.com
TOPNOTCH MEDICAL BOARD PREP PHYSIOLOGY SUPEREXAM
For inquiries visit www.topnotchboardprep.com.ph or email us at topnotchmedicalboardprep@gmail.com
Item QUESTION EXPLANATION AUTHOR TOPNOTCH
# EXAM
BINAN)

148 Which of the following is incorrectly matched IgD has no known function so far EDWARD MIDTERM 2
A. IgM ---> 1st antibody class typically released by HARRY EXAM - AUG
plasma cells VALLAJERA, 2015
B. IgD ---> binds to mast cells and basophils and MD (TOP 8 -
causes histamine release FEB 2015
C. IgG ---> able to activate complement MED
D. IgA ---> found in saliva and tears BOARDS;
E. None of the above TOPNOTCH
MD FROM
PERPETUAL
BINAN)
149 The only antibody that could attack the Rh The Rh blood group in the fetus would be attacked by Rh EDWARD MIDTERM 2
complexes on fetal red blood cells is the: IgG from the mother. HARRY EXAM - AUG
A. IgA VALLAJERA, 2015
B. IgD MD (TOP 8 -
C. IgE FEB 2015
D. IgG MED
E. IgM BOARDS;
TOPNOTCH
MD FROM
PERPETUAL
BINAN)
150 The primary function of the spleen is The function of the spleen is to remove old RBC from the EDWARD MIDTERM 2
A. Filtering of damaged and old erythrocytes from blood stream and store platelets and/or granulocytes HARRY EXAM - AUG
the bloodstream VALLAJERA, 2015
B. Storage of platelets and granulocytes MD (TOP 8 -
C. Blood pressure and volume regulation FEB 2015
D. A and B MED
E. All of the above BOARDS;
TOPNOTCH
MD FROM
PERPETUAL
BINAN)
151 Which of the following would induce the secretion of Oral glucose through the glucose dependent EDWARD MIDTERM 2
insulin better? insulinotropic peptide. HARRY EXAM - AUG
A. Oral glucose VALLAJERA, 2015
B. Parenteral glucose MD (TOP 8 -
C. Intramuscular glucose FEB 2015
D. Both B and C MED
E. All of the above BOARDS;
TOPNOTCH
MD FROM
PERPETUAL
BINAN)
152 Which of the following inhibits the interaction of Calmodulin is a Ca-binding protein that binds to Ca, EDWARD MIDTERM 2
myosin and actin in smooth muscle calmodulin-Ca complex then activates myosin light chain HARRY EXAM - AUG
A. Myosin light chain kinase kinase causing myosin to bind to actin causing sm. VALLAJERA, 2015
B. Caldesmon, calponin Muscle contraction. Without calmodulin or Ca there will MD (TOP 8 -
C. Calmodulin be no interaction between myosin and actin in smooth FEB 2015
D. Troponin I muscle. Caldesmon is the one that inhibits actin-myosin MED
E. None of the above binding. BOARDS;
TOPNOTCH
MD FROM
PERPETUAL
BINAN)
153 Stimulation of the POMC/CART neurons in the POMC/CART neurons are anorexigenic and thus EDWARD MIDTERM 2
arcuate nucleus of the hypothalams stimulates promote energy expenditure and less food intake. HARRY EXAM - AUG
which of the following? VALLAJERA, 2015
A. Increased insulin secretion MD (TOP 8 -
B. Increased food intake FEB 2015
C. Increased energy expenditure MED
D. Both A and B BOARDS;
E. All of the above TOPNOTCH
MD FROM
PERPETUAL
BINAN)
154 It is responsible for the first burst of energy in a During the first 8-10 seconds after activity, the EDWARD MIDTERM 2
physical activity phosphagen energy system is first used. HARRY EXAM - AUG
A. Phosphagen energy system VALLAJERA, 2015
B. Glycogen-lactic acid system MD (TOP 8 -
C. Aerobic system FEB 2015
D. Glycolysis MED
E. None of the above BOARDS;
TOPNOTCH
MD FROM
PERPETUAL
BINAN)

TOPNOTCH MEDICAL BOARD PREP PHYSIOLOGY SUPEREXAM Page 20 of 88


For inquiries visit www.topnotchboardprep.com.ph or email us at topnotchmedicalboardprep@gmail.com
TOPNOTCH MEDICAL BOARD PREP PHYSIOLOGY SUPEREXAM
For inquiries visit www.topnotchboardprep.com.ph or email us at topnotchmedicalboardprep@gmail.com
Item QUESTION EXPLANATION AUTHOR TOPNOTCH
# EXAM
155 Signs and symptoms of high altitude sickness starts Signs of high altitude sickness appear in an unclimatized EDWARD MIDTERM 2
when someone unadapted rapidly ascends to what individual when he reaches 12,000 ft. HARRY EXAM - AUG
altitude? VALLAJERA, 2015
A. 10,000 ft MD (TOP 8 -
B. 12,000 ft FEB 2015
C. 14,000 ft MED
D. 16,000 ft BOARDS;
E. 18,000 ft TOPNOTCH
MD FROM
PERPETUAL
BINAN)
156 Which part of the kidney is very sensitive to changes Interstitial cells of the loop of Henle are sensitive to EDWARD MIDTERM 2
in blood PO2? arterial PO2 HARRY EXAM - AUG
A. Proximal tubule VALLAJERA, 2015
B. Distal tubule MD (TOP 8 -
C. Interstitial Cells of the loop of Henle FEB 2015
D. Collecting duct MED
E. Macula Densa BOARDS;
TOPNOTCH
MD FROM
PERPETUAL
BINAN)
157 These receptors detect high frequency vibration and EDWARD MIDTERM 2
are shaped like onions HARRY EXAM - AUG
A. Merkel's disk VALLAJERA, 2015
B. Iggo Dome receptors MD (TOP 8 -
C. Pacinian corpuscles FEB 2015
D. Ruffini's End Organs MED
E. Meissner's corpuscles BOARDS;
TOPNOTCH
MD FROM
PERPETUAL
BINAN)
158 When light strikes the photoreceptors, it causes Vision is the only sensation wherein the response of the EDWARD MIDTERM 2
what? receptor causes hyperpolarization. HARRY EXAM - AUG
A. Hyperpolarization VALLAJERA, 2015
B. Depolarization MD (TOP 8 -
C. Repolarization FEB 2015
D. Afterpolarization MED
E. None of the above BOARDS;
TOPNOTCH
MD FROM
PERPETUAL
BINAN)
159 Low frequency sounds are analyzed in which region Low frequency sounds are analyzed at the apex of the EDWARD MIDTERM 2
of the inner ear? cochlea or the helicotrema of the cochlea. HARRY EXAM - AUG
A. Near the oval window VALLAJERA, 2015
B. Near the round window MD (TOP 8 -
C. At the apex of the cochlea FEB 2015
D. At the utricle MED
E. None of the above BOARDS;
TOPNOTCH
MD FROM
PERPETUAL
BINAN)
160 Majority of the carbon dioxide in the blood is Majority of CO2 is transported as bicarbonate. EDWARD MIDTERM 2
transported as: HARRY EXAM - AUG
A. Dissolved CO2 VALLAJERA, 2015
B. Carbaminohemoglobin MD (TOP 8 -
C. Carboxyhemoglobin FEB 2015
D. Bicarbonate ions MED
E. None of the above BOARDS;
TOPNOTCH
MD FROM
PERPETUAL
BINAN)
161 Hemoglobin is a protein that can carry oxygen that 1 gram of hemoglobin can carry as much as 1.34 ml of HAROLD JAY MIDTERM 3
will be delivered to different tissues. It is expected oxygen S. BAYTEC, EXAM - AUG
that 12 grams of hemoglobin can carry as much as MD (TOP 10 2015
how many mililiters of oxygen? - FEB 2015
A. 10 MED
B. 12 BOARDS;
C. 14 TOPNOTCH
D. 16 MD FROM
E. 18 FEU)

162 Which of the following is NOT a characteristic or a wave is seen during atrial contraction. V wave is seen HAROLD JAY MIDTERM 3
does NOT happen during atrial contraction? during isovolumic relaxation S. BAYTEC, EXAM - AUG
A. Occurs during the distal third of the diastole MD (TOP 10 2015
B. V wave of atrial pressure is seen - FEB 2015
C. Preceded by p wave in the ECG MED
D. 4th heart sound maybe heard due to atria BOARDS;
contracting against a stiff ventricles TOPNOTCH
E. none of the above MD FROM
FEU)

TOPNOTCH MEDICAL BOARD PREP PHYSIOLOGY SUPEREXAM Page 21 of 88


For inquiries visit www.topnotchboardprep.com.ph or email us at topnotchmedicalboardprep@gmail.com
TOPNOTCH MEDICAL BOARD PREP PHYSIOLOGY SUPEREXAM
For inquiries visit www.topnotchboardprep.com.ph or email us at topnotchmedicalboardprep@gmail.com
Item QUESTION EXPLANATION AUTHOR TOPNOTCH
# EXAM
163 In a normal adult with a heart rate of 70, which diastasis or reduced ventricular filling happens the HAROLD JAY MIDTERM 3
among the phases of cardiac cycle happens the longest during the cardiac cycle S. BAYTEC, EXAM - AUG
longest? MD (TOP 10 2015
A. Diastasis - FEB 2015
B. Isovolumic contraction MED
C. Isovolumic relaxation BOARDS;
D. Reduced ventricular ejection TOPNOTCH
E. Rapid ventricular ejection MD FROM
FEU)
164 Murmurs coming from the Aortic valve can be heard Aortic is heard on 2nd ICS R parasternal border. HAROLD JAY MIDTERM 3
best by putting the stethoscope in which of the Pulmonic is heard on 2nd ICS left parasternal border. 4th S. BAYTEC, EXAM - AUG
following are? ICS left parasternal left parasternal borderj. Mitral is MD (TOP 10 2015
A. 5th ICS left MCL heard best on 5th ICS L MCL - FEB 2015
B. 4th ICS left parasternal border MED
C. 3rd ICS left parasternal border BOARDS;
D. 2nd ICS left parasternal border TOPNOTCH
E. 2nd ICS right parasternal border MD FROM
FEU)
165 Which of the following volume/capacity is not Vital capacity includes all the lung volumes except RV HAROLD JAY MIDTERM 3
included in vital capacity? and capacities which include RV S. BAYTEC, EXAM - AUG
A. IRV MD (TOP 10 2015
B. Tidal volume - FEB 2015
C. ERV MED
D. RV BOARDS;
E. Inspiratory capacity TOPNOTCH
MD FROM
FEU)
166 In a normal human adult, how many ml of air is the normal TD=150ml HAROLD JAY MIDTERM 3
usual anatomic dead space? S. BAYTEC, EXAM - AUG
A. 100 MD (TOP 10 2015
B. 125 - FEB 2015
C. 150 MED
D. 175 BOARDS;
E. 200 TOPNOTCH
MD FROM
FEU)
167 All of the following are considered GI hormones official GI hormones are gastrin, secretin, CCK, GIP, and HAROLD JAY MIDTERM 3
except: motilin S. BAYTEC, EXAM - AUG
A. somatostatin MD (TOP 10 2015
B. gastrin - FEB 2015
C. secretin MED
D. CCK BOARDS;
E. Motilin TOPNOTCH
MD FROM
FEU)
168 Cholecystokinin is secreted in duodenum and G cells secretes gastrin, S cells is for secretin, K cells is HAROLD JAY MIDTERM 3
jejunum primarily for fat absorption. Which of the for GIP, M cells for motilin S. BAYTEC, EXAM - AUG
following cells secretes CCK? MD (TOP 10 2015
A. G cells - FEB 2015
B. S cells MED
C. C cells BOARDS;
D. I cells TOPNOTCH
E. K cells MD FROM
FEU)
169 Which among the following is a characteristic of a fast twitch or type 2 muscles are usually larger, with HAROLD JAY MIDTERM 3
fast twitch muscle as compared to a slow twitch faster SR calcium reuptake, glycolytic, with more active S. BAYTEC, EXAM - AUG
muscle? enzymes for phosphagen and glycogen-lactic acid energy MD (TOP 10 2015
A. They are smaller in size systems, high myosin ATPase, with less mitochondria - FEB 2015
B. More dependent on oxidative metabolism and myoglobin. MED
C. They are usually the red muscles BOARDS;
D. They have less mitochondria and myoglobin TOPNOTCH
E. example of which are anti-gravity muscles MD FROM
FEU)
170 Which among the following will not shorten during HAROLD JAY MIDTERM 3
muscle contraction? S. BAYTEC, EXAM - AUG
A. A band MD (TOP 10 2015
B. Sarcomere - FEB 2015
C. I band MED
D. H zone BOARDS;
E. None of the above TOPNOTCH
MD FROM
FEU)
171 in a motor neuron, which of the following will enter Calcium enters to activate the release of Ach which will HAROLD JAY MIDTERM 3
the presynaptic neuron after the action potential has be released in the synaptic junction S. BAYTEC, EXAM - AUG
reached its terminal? MD (TOP 10 2015
A. K - FEB 2015
B. Ca MED
C. Na BOARDS;
D. Acetylcholline TOPNOTCH
E. Cl MD FROM
FEU)

TOPNOTCH MEDICAL BOARD PREP PHYSIOLOGY SUPEREXAM Page 22 of 88


For inquiries visit www.topnotchboardprep.com.ph or email us at topnotchmedicalboardprep@gmail.com
TOPNOTCH MEDICAL BOARD PREP PHYSIOLOGY SUPEREXAM
For inquiries visit www.topnotchboardprep.com.ph or email us at topnotchmedicalboardprep@gmail.com
Item QUESTION EXPLANATION AUTHOR TOPNOTCH
# EXAM
172 Thyroid hormone primarily regulates how high or HAROLD JAY MIDTERM 3
low an individual's metabolism. Which of the S. BAYTEC, EXAM - AUG
following is the correct sequence of thyroid MD (TOP 10 2015
hormone syntesis - FEB 2015
A. organification--coupling--iodine trapping-- MED
storage BOARDS;
B. iodine trapping--coupling-- organification-- TOPNOTCH
storage MD FROM
C. strorage-- organification-- coupling--iodine FEU)
trapping
D. Iodine trapping-- organificatioin-- coupling--
storage
E. storage-- iodine trapping-- coupline--
organification
173 The earliest stage of RBC that can be seen in a non- reticulocyte is the first stage of RBC that can enter the HAROLD JAY MIDTERM 3
neoplastic blood is blood stream in a non-neoplastic blood. This happens S. BAYTEC, EXAM - AUG
A. reticulocyte specially during hypoxia and anemia MD (TOP 10 2015
B. erythrocyte - FEB 2015
C. Orthochromatic erythroblast MED
D. Basophil erythroblast BOARDS;
E. Proerythroblast TOPNOTCH
MD FROM
FEU)
174 A curious 3 year old male was playing his eyes by The oculocardiac reflex, also known as Aschner HAROLD JAY MIDTERM 3
pressing his thumb on both his eyes. A few seconds phenomenon, Aschner reflex, or Aschner-Dagnini reflex, S. BAYTEC, EXAM - AUG
later, he experienced syncope. The paramedics who is a decrease in pulse rate associated with traction MD (TOP 10 2015
saw him first reported that patient had a very slow applied to extraocular muscles and/or compression of - FEB 2015
heart rate. Which of the following was stimulated to the eyeball. The reflex is mediated by nerve connections MED
the child? between the ophthalmic branch of the trigeminal cranial BOARDS;
A. oculovagal reflex nerve via the ciliary ganglion, and the vagus nerve of the TOPNOTCH
B. oculocardiac reflex parasympathetic nervous system (SIMILAR TO MD FROM
C. vasovagal reflex PREVIOUS BOARD EXAM CONCEPT/PRINCIPLE) FEU)
D. oculotrigemenal reflex
E. none of the above
175 Which of the following component of the opsonization is c3b. Anaphylatoxins are c3a c4a c5a. HAROLD JAY MIDTERM 3
complement system is chemotactic to WBCs? Chemotactic is c5a. MAC is c5b6789. (SIMILAR TO S. BAYTEC, EXAM - AUG
A. c3b PREVIOUS BOARD EXAM CONCEPT/PRINCIPLE) MD (TOP 10 2015
B. c3a - FEB 2015
C. c4a MED
D. c5a BOARDS;
E. C5b5789 TOPNOTCH
MD FROM
FEU)
176 Which of the following is a characteristic of T3 as T3 has faster half-life, less synthesized, less afinity to HAROLD JAY MIDTERM 3
compared to T4? plasma protein, binds more to nuclear receptor, and with S. BAYTEC, EXAM - AUG
A. Faster half-life faster onset of action MD (TOP 10 2015
B. More affinity in binding plasma protein - FEB 2015
C. Higher percentage synthesized MED
D. Slower onset of action BOARDS;
E. None of the above TOPNOTCH
MD FROM
FEU)
177 Which of the following is synthesized by posterior hormones found in the PPG are synthesized in HAROLD JAY MIDTERM 3
pituitary gland? hypothalamus and stored only in PPG S. BAYTEC, EXAM - AUG
A. Growth hormone MD (TOP 10 2015
B. TSH - FEB 2015
C. ADH MED
D. ACTH BOARDS;
E. None of the above TOPNOTCH
MD FROM
FEU)
178 In the pancreas, which of the following cells secrete alpha cells secrete glucagon, beta cell for insulin and HAROLD JAY MIDTERM 3
glucagon? amylin, delta cells for somatostatin and F or pp cell for S. BAYTEC, EXAM - AUG
A. Alpha cell pancreatic polypeptide MD (TOP 10 2015
B. Beta cell - FEB 2015
C. Delta cell MED
D. F cell BOARDS;
E. PP cell TOPNOTCH
MD FROM
FEU)
179 Intercalated discs of cardiac muscles are examples of From Dr.Banzuela: A and D. Interalated disks contain 3 HAROLD JAY MIDTERM 3
what type of junctional complexes? structures: fascia adherens, macula adherens (also S. BAYTEC, EXAM - AUG
A. Macula adherens known as desmosomes) and gap junctions. MD (TOP 10 2015
B. Zonula adherens - FEB 2015
C. Zonula occludens MED
D. Gap junctions BOARDS;
E. Tight junctions TOPNOTCH
MD FROM
FEU)

TOPNOTCH MEDICAL BOARD PREP PHYSIOLOGY SUPEREXAM Page 23 of 88


For inquiries visit www.topnotchboardprep.com.ph or email us at topnotchmedicalboardprep@gmail.com
TOPNOTCH MEDICAL BOARD PREP PHYSIOLOGY SUPEREXAM
For inquiries visit www.topnotchboardprep.com.ph or email us at topnotchmedicalboardprep@gmail.com
Item QUESTION EXPLANATION AUTHOR TOPNOTCH
# EXAM
180 Which of the following factors is common to both factors V and X are common to both pathways HAROLD JAY MIDTERM 3
intrinsic and extrinsic pathway? S. BAYTEC, EXAM - AUG
A. IV MD (TOP 10 2015
B. VII - FEB 2015
C. VIII MED
D. IX BOARDS;
E. X TOPNOTCH
MD FROM
FEU)
181 Patient's tidal volume is 500ml and the respiratory SIMILAR TO PREVIOUS BOARD EXAM JEAN PAOLO FINAL EXAM -
rate is 14. What is the patient's respiratory minute CONCEPT/PRINCIPLE. respiratory minute volume= M. DELFINO, AUG 2015
volume? tidal volume x RR MD (TOP 10
A. 3500 - FEB 2015
B. 4900 MED
C. 7000 BOARDS;
D. 5500 TOPNOTCH
E. 5000 MD FROM
FATIMA)
182 A serum osmolality of 282 is considered as SIMILAR TO PREVIOUS BOARD EXAM JEAN PAOLO FINAL EXAM -
A. hypotonic CONCEPT/PRINCIPLE.. Normal serum osmolality is M. DELFINO, AUG 2015
B. isotonic 275-295 in some books, 280-290 in others. Either way, MD (TOP 10
C. hypertonic 282 is isotonic. - FEB 2015
D. diluted MED
E. concentrated BOARDS;
TOPNOTCH
MD FROM
FATIMA)
183 You are examining a patient's pupil. The room was SIMILAR TO PREVIOUS BOARD EXAM JEAN PAOLO FINAL EXAM -
initially dark then the light was turned on. You noted CONCEPT/PRINCIPLE.. The function tested is pupillary M. DELFINO, AUG 2015
that the patient's pupils constrict. What is primarily light reflex which involves CN II and CN III. MD (TOP 10
tested in this scenario? - FEB 2015
A. cerebrum MED
B. parasympathetic BOARDS;
C. sympathetic TOPNOTCH
D. optic nerve MD FROM
E. medulla FATIMA)

184 What receptor is involved in 2-point discrimination? SIMILAR TO PREVIOUS BOARD EXAM JEAN PAOLO FINAL EXAM -
A. Ruffini's CONCEPT/PRINCIPLE.. (Corrected by Banzuela) - 2PD M. DELFINO, AUG 2015
B. Merkel refers to ability to distinguish between two separate MD (TOP 10
C. Meissner points when stimulated with such, and is a property of - FEB 2015
D. Pacinian mechanoreceptors with SMALL RECEPTIVE FIELDS MED
E. Free nerve endings (Receptive fields type 1). Both Merkel's Discs and BOARDS;
Meissner's Corpuscles utilize small receptive fields, both TOPNOTCH
are found also in high density in the hand.s Both are MD FROM
involved in two point discrimination. However, if you're FATIMA)
made to choose between the two choose Merkel's rather
than Meissner's - merkel's is Slowly-Adapting unlike
Meissner's, and can therefore detect 2 separate points as
long as the stimulus is present.
185 In the normal lungs, the highest airway resistance is the highest airway resistance in a normal lung is found JEAN PAOLO FINAL EXAM -
at the level of in medium sized airways like the bronchiole (because M. DELFINO, AUG 2015
A. bronchioles they are more branched than the small airways) while in MD (TOP 10
B. alveoli disease states, small airways have the highest resistance - FEB 2015
C. alveolar ducts because they are easily occluded MED
D. trachea BOARDS;
E. terminal bronchioles TOPNOTCH
MD FROM
FATIMA)
186 Paraaminohippuric acid is used to measure Renal JEAN PAOLO FINAL EXAM -
Blood Flow because it is M. DELFINO, AUG 2015
A. Freely filtered, not secreted, not reabsorbed MD (TOP 10
B. Freely filtered, secreted, not reabsorbed - FEB 2015
C. Freely filtered, not secreted, partially reabsorbed MED
D. Not filtered, not secreted, not reabsorbed BOARDS;
E. Freely filtered, secreted, partially reabsorbed TOPNOTCH
MD FROM
FATIMA)
187 9 year old male patient was rushed to the emergency Adrenergic receptors: alpha 1 increases IP3/DAG, alpha JEAN PAOLO FINAL EXAM -
room due to difficulty of breathing. Upon 2 decreases cAMP, beta 1 and beta 2 increases cAMP M. DELFINO, AUG 2015
auscultation, wheezes were heard throughout the MD (TOP 10
lung fields. Patient was immediately nebulized with - FEB 2015
a beta-adrenergic agonist. This drug relaxes the MED
airway smooth muscles by BOARDS;
A. increasing cAMP TOPNOTCH
B. decreasing cAMP MD FROM
C. increasing IP3/DAG FATIMA)
D. increasing cGMP
E. activating tyrosine kinase

TOPNOTCH MEDICAL BOARD PREP PHYSIOLOGY SUPEREXAM Page 24 of 88


For inquiries visit www.topnotchboardprep.com.ph or email us at topnotchmedicalboardprep@gmail.com
TOPNOTCH MEDICAL BOARD PREP PHYSIOLOGY SUPEREXAM
For inquiries visit www.topnotchboardprep.com.ph or email us at topnotchmedicalboardprep@gmail.com
Item QUESTION EXPLANATION AUTHOR TOPNOTCH
# EXAM
188 The I cells present in the duodenum and jejunum The hormone produced by I cells is cholecystokinin JEAN PAOLO FINAL EXAM -
produce a hormone that functions to which functions to stimulate GB contraction, sphincter of M. DELFINO, AUG 2015
A. Increase pancreatic HCO3 secretions Oddi relaxation, increase pancreatic enzyme secretion, MD (TOP 10
B. Increase gastric proton secretion inhibits gastric emptying - FEB 2015
C. Relaxes sphincter of Oddi MED
D. Increase insulin secretion BOARDS;
E. Stimulate growth of gastric mucosa TOPNOTCH
MD FROM
FATIMA)
189 Characteristics of extraocular muscle fibers except Extraocular muscles are example of JEAN PAOLO FINAL EXAM -
A. Contraction velocity is fast fast/glycolytic/white type II muscle fibers. Its metabolic M. DELFINO, AUG 2015
B. Metabolic profile is oxidative profile is glycolytic, not oxidative. EOMs are classified as MD (TOP 10
C. Myosin ATPase is high fast twitch muscle. This type of muscle fiber has large - FEB 2015
D. Sarcoplasmic reticulum calcium reuptake is fast diameter (p.401 berne and levy 6th edition). EOMs are MED
E. Large fiber size finest in terms of their motor function/movement, and BOARDS;
not in terms of their size. TOPNOTCH
MD FROM
FATIMA)
190 Patients with liver cirrhosis develop edema and Albumin is the protein synthesized by the liver that JEAN PAOLO FINAL EXAM -
ascites due to determines plasma oncotic pressure. Liver cirrhosis M. DELFINO, AUG 2015
A. Decreased plasma oncotic pressure compromises liver function. Thus, there will be less MD (TOP 10
B. Increased capillary hydrostatic pressure albumin synthesized, less plasma oncotic pressure, - FEB 2015
C. Increased interstitial oncotic pressure leading to displacement of fluids in the interstitial cells MED
D. Decreased interstitial hydrostatic pressure causing edema. BOARDS;
E. Decreased capillary hydrostatic pressure TOPNOTCH
MD FROM
FATIMA)
191 What is the most potent vasoconstrictor substance ADH or Vasopressin is the body's most potent vascular JEAN PAOLO FINAL EXAM -
in the body? constrictor substance M. DELFINO, AUG 2015
A. Norepinephrine MD (TOP 10
B. Epinephrine - FEB 2015
C. Angiotensin MED
D. ADH BOARDS;
E. Dopamine TOPNOTCH
MD FROM
FATIMA)
192 59 year old male with diabetes and hypertension The QRS complex begins slightly before the onset of JEAN PAOLO FINAL EXAM -
came in the ER due to chest pain, described as systole while the T wave occurs slightly before the end of M. DELFINO, AUG 2015
heaviness associated with dyspnea. ECG was done ventricular contraction. Thus, the ST segment coincides MD (TOP 10
which revealed ST segment elevation in Leads V1, with the period of systole wherein there is ventricular - FEB 2015
V2, V3, V4. The ST segment coincides with what contraction and ejection. Isovolumetric contraction MED
phase in the cardiac cycle? coincides with the latter part of the QRS complex only BOARDS;
A. Isovolumetric contraction and not with the ST segment TOPNOTCH
B. Ventricular ejection MD FROM
C. Isovolumetric relaxation FATIMA)
D. Ventricular filling
E. Atrial contraction
193 Vasodilators do not affect what type of blood vessels Capillaries are not affected by vasodilators because they JEAN PAOLO FINAL EXAM -
A. Arteries do not have smooth muscles M. DELFINO, AUG 2015
B. Arterioles MD (TOP 10
C. Capillaries - FEB 2015
D. Venules MED
E. Veins BOARDS;
TOPNOTCH
MD FROM
FATIMA)
194 Which of the following is false regarding saliva At high flow rates, there is not much time for JEAN PAOLO FINAL EXAM -
composition at high flow rates? reabsorption and secretion in the ducts so the M. DELFINO, AUG 2015
A. High Na composition of saliva is the same with that of plasma- Na MD (TOP 10
B. High Cl and Cl are high while K is low (except for HCO3 which is - FEB 2015
C. High K also high) MED
D. High HCO3 BOARDS;
E. None of the above TOPNOTCH
MD FROM
FATIMA)
195 Increased preload is observed in a patient with a Aortic regurgitation causes diastolic murmur with JEAN PAOLO FINAL EXAM -
diastolic murmur. What is the most likely cause? increased preload M. DELFINO, AUG 2015
A. Mitral regurgitation MD (TOP 10
B. Mitral stenosis - FEB 2015
C. Pulmonic regurgitation MED
D. Aortic regurgitation BOARDS;
E. Tricuspid stenosis TOPNOTCH
MD FROM
FATIMA)
196 Which of the following is lower at the base of the V/Q ratio and PaO2 are higher at the apex of the lungs. JEAN PAOLO FINAL EXAM -
lungs than the apex when the patient is standing? M. DELFINO, AUG 2015
A. perfusion MD (TOP 10
B. ventilation - FEB 2015
C. V/Q ratio MED
D. PaCO2 BOARDS;
E. None of the above TOPNOTCH
MD FROM
FATIMA)

TOPNOTCH MEDICAL BOARD PREP PHYSIOLOGY SUPEREXAM Page 25 of 88


For inquiries visit www.topnotchboardprep.com.ph or email us at topnotchmedicalboardprep@gmail.com
TOPNOTCH MEDICAL BOARD PREP PHYSIOLOGY SUPEREXAM
For inquiries visit www.topnotchboardprep.com.ph or email us at topnotchmedicalboardprep@gmail.com
Item QUESTION EXPLANATION AUTHOR TOPNOTCH
# EXAM
197 If both the afferent and efferent arteriole dilates, Both will increase. GFR increases because there is JEAN PAOLO FINAL EXAM -
what will happen to the GFR and RBF? increased glomerular capillary pressure. RBF increases M. DELFINO, AUG 2015
A. Both will increase whenever there is decreased resistance in any of the MD (TOP 10
B. Both will decrease arteriolar vessels. - FEB 2015
C. GFR will increase, RBF will decrease MED
D. GFR will decrease, RBF will increase BOARDS;
E. No change in GFR, RBF will increase TOPNOTCH
MD FROM
FATIMA)
198 The following are effects of Angiotensin II except All the given choices are effects of angiotensin II JEAN PAOLO FINAL EXAM -
A. Stimulate aldosterone release M. DELFINO, AUG 2015
B. Increase Na-H exchange in PCT MD (TOP 10
C. Increase thirst - FEB 2015
D. Vasoconstriction of arterioles MED
E. None of the above BOARDS;
TOPNOTCH
MD FROM
FATIMA)
199 Mother is blood type A and Father is blood type B. JEAN PAOLO FINAL EXAM -
Their children will not possibly have which blood M. DELFINO, AUG 2015
type? MD (TOP 10
A. B - FEB 2015
B. A MED
C. AB BOARDS;
D. O TOPNOTCH
E. None of the above MD FROM
FATIMA)
200 Which of the following is not able to take up glucose Muscle and adipose tissue have insulin dependent JEAN PAOLO FINAL EXAM -
in diabetic patients? transporters of glucose (GLUT4). M. DELFINO, AUG 2015
A. Brain MD (TOP 10
B. RBC - FEB 2015
C. Liver MED
D. Pancreas BOARDS;
E. Muscle TOPNOTCH
MD FROM
FATIMA)
201 Para-amino-hippuric acid is used as a measure of Theoretically, if a substance is completely cleared from GRACE DIAGNOSTIC
renal plasma flow because ____________. the plasma, the clearance rate of that substance is equal ARVIOLA, EXAM - FEB
A. It is only filtered by the renal glomerulus. to the total renal plasma flow. A substance that is MD (TOP 3 - 2015
B. It is both filtered and secreted. completely cleared from the plasma must be both AUG 2014
C. It is both filtered and reabsorbed. filtered and secreted and not reabsorbed. PAH is about MED
D. It is both reabsorbed and secreted. 90% cleared from the plasma and can be used as a BOARDS;
E. It is filtered, reabsorbed, and secreted. measure of renal plasma flow. TOPNOTCH
MD)
202 The threshold for drinking is reached by an increase When the sodium concentration increases only about 2 GRACE DIAGNOSTIC
in sodium concentration of as little as: mEq/L above normal, the thirst mechanism is activated, ARVIOLA, EXAM - FEB
A. 1 mEq/L causing a desire to drink water. MD (TOP 3 - 2015
B. 2 mEq/L AUG 2014
C. 3 mEq/L MED
D. 4 mEq/L BOARDS;
E. 5 mE/q/L TOPNOTCH
MD)

203 Corrective lenses for near-sighted persons consist of In myopia, the image falls in front of the retina. GRACE DIAGNOSTIC
________ lenses because these lenses ________ light rays Corrective lenses should therefore be concave so that ARVIOLA, EXAM - FEB
so that these are focused on the retina. light rays diverge and fall on the retina. MD (TOP 3 - 2015
A. Convex, converge AUG 2014
B. Convex, diverge MED
C. Concave, converge BOARDS;
D. Concave, diverge TOPNOTCH
E. Cylindrical lenses, converge MD)

204 In a person eating a pure carbohydrate diet, the Respiratory exchange ratio is the ratio of carbon dioxide GRACE DIAGNOSTIC
respiratory exchange ratio is expected to be: released with oxygen consumed. The normal ratio is ARVIOLA, EXAM - FEB
A. 1.0 0.825. However, in a pure carbohydrate diet, much more MD (TOP 3 - 2015
B. 0.85 carbon dioxide is released which increases the ratio to AUG 2014
C. 0.7 1.0. When oxygen is metabolized with carbohydrates, MED
D. 0.5 one molecule of CO2 is formed for each molecule of O2 BOARDS;
E. 0.25 consumed. When oxygen reacts with fats, a large share of TOPNOTCH
the oxygen combines with hydrogen atoms from the fats MD)
to form water instead of CO2.
205 Which part of the heart is first to repolarize? The first part of the ventricles to become depolarized is GRACE DIAGNOSTIC
A. Base the left endocardial surface of the septum. The last part ARVIOLA, EXAM - FEB
B. Septum of the heart to become depolarized is the outer walls of MD (TOP 3 - 2015
C. Apex the ventricles near the base of the heart. The greatest AUG 2014
D. Right atria portion of ventricular muscle mass to repolarize first is MED
E. Right ventricle the entire outer surface of the ventricles, especially near BOARDS;
the apex of the heart. The endocardial areas, conversely, TOPNOTCH
normally repolarize last. **So confusing!** MD)

TOPNOTCH MEDICAL BOARD PREP PHYSIOLOGY SUPEREXAM Page 26 of 88


For inquiries visit www.topnotchboardprep.com.ph or email us at topnotchmedicalboardprep@gmail.com
TOPNOTCH MEDICAL BOARD PREP PHYSIOLOGY SUPEREXAM
For inquiries visit www.topnotchboardprep.com.ph or email us at topnotchmedicalboardprep@gmail.com
Item QUESTION EXPLANATION AUTHOR TOPNOTCH
# EXAM
206 Which glucose transporter is insulin-dependent? Sodium-glucose transporters are secondary active co- GRACE DIAGNOSTIC
A. SGLT-1 transport systems that depends on the sodium gradient. ARVIOLA, EXAM - FEB
B. SGLT-2 GLUT-1 and GLUT-3, both present in the brain, does not MD (TOP 3 - 2015
C. GLUT-1 depend on insulin. AUG 2014
D. GLUT-4 MED
E. GLUT-3 BOARDS;
TOPNOTCH
MD)
207 When a nerve action potential reaches a muscle The opening of sodium channels causes more inward GRACE DIAGNOSTIC
membrane, which of the following events take place positive current. This causes the depolarization of the ARVIOLA, EXAM - FEB
in chronological order? muscle membrane. MD (TOP 3 - 2015
A. Opening of sodium channels → Depolarization → AUG 2014
Power stroke → Calcium release from SR MED
B. Depolarization → Opening of sodium channels → BOARDS;
Calcium release from SR → Power stroke TOPNOTCH
C. Opening of sodium channels → Depolarization → MD)
Calcium release from SR → Power stroke
D. Depolarization → Opening of sodium channels →
Power stroke → Calcium release from SR
E. None of the above

208 The latch mechanism of smooth muscles serves to: Once smooth muscle has developed full contraction, the GRACE DIAGNOSTIC
A. Increase the number of cross-bridges between amount of continuing excitation usually can be reduced ARVIOLA, EXAM - FEB
actin and myosin to far less than the initial level, yet the muscle maintains MD (TOP 3 - 2015
B. Increase the power of muscle contraction its full force of contraction. AUG 2014
C. Allow a prolonged relaxed state MED
D. Avoid muscle fatigue BOARDS;
E. Maintain prolonged tonic contraction TOPNOTCH
MD)
209 Which is NOT a diabetogenic hormone? The effects of thyroid hormone on carbohydrate GRACE DIAGNOSTIC
A. Growth hormone metabolism are both anabolic and catabolic as thyroid ARVIOLA, EXAM - FEB
B. Epinephrine hormones cause an overall increase in cellular metabolic MD (TOP 3 - 2015
C. Cortisol enzymes. AUG 2014
D. Thyroid hormone MED
E. Glucagon BOARDS;
TOPNOTCH
MD)
210 Spatial summation results from action potentials Temporal summation results when action potential are GRACE DIAGNOSTIC
that are: successive. ARVIOLA, EXAM - FEB
A. Simultaneous MD (TOP 3 - 2015
B. Successive AUG 2014
C. Alternating MED
D. Decremental BOARDS;
E. Inadequate TOPNOTCH
MD)
211 Sounds of high frequency stimulate basilar fibers The lengths of the basilar fibers increase progressively GRACE DIAGNOSTIC
located near the oval window because in this beginning at the oval window and going from the base of ARVIOLA, EXAM - FEB
location, the basilar fibers are: the cochlea to the apex. The diameters of the fibers, MD (TOP 3 - 2015
A. Short and stiff however, decrease from the oval window to the AUG 2014
B. Short and limber helicotrema, so that their overall stiffness decreases MED
C. Long and limber more than 100-fold. As a result, the stiff, short fibers BOARDS;
D. Long and stiff near the oval window vibrate best at a very high TOPNOTCH
E. None of the above frequency. MD)

212 At the base of the lungs, there is relatively more At the apex, there is relatively more ventilation than GRACE DIAGNOSTIC
perfusion than ventilation. This is: perfusion. This is physiologic dead space. ARVIOLA, EXAM - FEB
A. Physiologic dead space MD (TOP 3 - 2015
B. Physiologic shunt AUG 2014
C. Anatomic dead space MED
D. Anatomic shunt BOARDS;
E. None of the above TOPNOTCH
MD)
213 Which waste product plays a crucial role in Urea is passively reabsorbed from the medullary GRACE DIAGNOSTIC
maintaining the hyperosmolarity of the renal collecting tubules to maintain a hyperosmolar renal ARVIOLA, EXAM - FEB
interstitium? interstitium. This promotes water reabsorption, thus, MD (TOP 3 - 2015
A. Hydrogen ion leading to the formation of a concentrated urine. AUG 2014
B. Urea MED
C. Carbon dioxide BOARDS;
D. Glucose TOPNOTCH
E. Water MD)

214 Cushing's syndrome is ACTH-independent if the The hypercortisolism from an adrenal cause of Cushing's GRACE DIAGNOSTIC
lesion occurs in the: syndrome is not due to an increased ACTH. These ARVIOLA, EXAM - FEB
A. Adrenal patients have, in fact, low or undetectable levels of ACTH. MD (TOP 3 - 2015
B. Pituitary Hence, it is ACTH-independent. AUG 2014
C. Hypothalamus MED
D. Ectopic sites BOARDS;
E. All of the above TOPNOTCH
MD)

TOPNOTCH MEDICAL BOARD PREP PHYSIOLOGY SUPEREXAM Page 27 of 88


For inquiries visit www.topnotchboardprep.com.ph or email us at topnotchmedicalboardprep@gmail.com
TOPNOTCH MEDICAL BOARD PREP PHYSIOLOGY SUPEREXAM
For inquiries visit www.topnotchboardprep.com.ph or email us at topnotchmedicalboardprep@gmail.com
Item QUESTION EXPLANATION AUTHOR TOPNOTCH
# EXAM
215 Which GI hormone receptor belongs in the CCK CCKB is the receptor for gastrin. CCKA is for GRACE DIAGNOSTIC
family of receptors? cholecystokinin. ARVIOLA, EXAM - FEB
A. Gastrin MD (TOP 3 - 2015
B. Secretin AUG 2014
C. Histamine MED
D. Pepsin BOARDS;
E. Insulin TOPNOTCH
MD)
216 The Edinger-Westphal nucleus of the oculomotor This nucleus functions in accommodation of the eye. GRACE DIAGNOSTIC
nerve serves _______ functions. ARVIOLA, EXAM - FEB
A. Sensory MD (TOP 3 - 2015
B. Motor – sympathetic AUG 2014
C. Motor – parasympathetic MED
D. Both sensory and motor BOARDS;
E. Neither sensory nor motor TOPNOTCH
MD)
217 Which is a correct statement regarding Acetylcholine functions both in the parasympathetic and GRACE DIAGNOSTIC
acetylcholine? sympathetic systems. It is degraded by ARVIOLA, EXAM - FEB
A. It binds both to nicotinic and adrenergic acetylcholinesterase. Receptors for acetylcholine are of MD (TOP 3 - 2015
receptors. the nicotinic and muscarinic types. AUG 2014
B. It is usually the hormone released by post- MED
synaptic sympathetic neurons. BOARDS;
C. It is rapidly degraded by pseudocholinesterases. TOPNOTCH
D. Its receptors located in the muscle membrane MD)
are of the nicotinic type.
E. It functions only in the parasympathetic system.
218 In blood vessels, compliance is a function of: A blood vessel is compliant if its volume increases with a GRACE DIAGNOSTIC
A. Pressure and viscosity of blood corresponding increase in pressure. ARVIOLA, EXAM - FEB
B. Pressure and volume MD (TOP 3 - 2015
C. Volume and length of vessel AUG 2014
D. Pressure and length of vessel MED
E. Volume and viscosity of blood BOARDS;
TOPNOTCH
MD)
219 Which hormone accounts for the increase in basal This is the thermogenic effect of progesterone. The GRACE DIAGNOSTIC
body temperature noted around the time of increase in temperature is about 0.5 degree Fahrenheit. ARVIOLA, EXAM - FEB
ovulation? MD (TOP 3 - 2015
A. FSH AUG 2014
B. LH MED
C. Estrogen BOARDS;
D. Progesterone TOPNOTCH
E. Prolactin MD)

220 Capacitation of the spermatozoon occurs in the: Capacitation refers to the collective changes that activate GRACE DIAGNOSTIC
A. Testes the sperm upon coming in contact with the fluids of the ARVIOLA, EXAM - FEB
B. Epididymis female genital tract. These changes make it possible for MD (TOP 3 - 2015
C. Vas deferens the sperm to penetrate the ovum. Maturation occurs in AUG 2014
D. Spongy urethra the epididymis. MED
E. Female genital tract BOARDS;
TOPNOTCH
MD)
221 which of the following will cause hypokalemia Factors that cause shift of K into the cell are the LEAN MIDTERM
secondary to stimulation of potassium uptake by a following: Insulin administration, aldosterone, Beta ANGELO EXAM 1 - FEB
cell? adrenergic stimulation, alkalosis. Thiazide causes SILVERIO, 2015
A. DM type 1 metabolic alkalosis. MD (TOP 4 -
B. 15 km Marathon AUG 2014
C. Propanolol intake MED
D. Thiazide intake BOARDS;
E. all of the above TOPNOTCH
MD), MD
222 Cross section of the sarcomere through the H zone H zone is the center region of the sarcomere bilaterally LEAN MIDTERM
will reveal what structures? adjacent to the M line. It compose of myosin only. ANGELO EXAM 1 - FEB
A. Actin and myosin SILVERIO, 2015
B. Myosin only MD (TOP 4 -
C. Actin only AUG 2014
D. Actin myosin and troponin MED
E. None of the above BOARDS;
TOPNOTCH
MD), MD
223 A lung cancer patient presents to a neurologist The patient is most likely suffering from Lambert Eaton LEAN MIDTERM
compaining of inability to walk after waking up Myasthenic syndrome. This is a common paraneoplastic ANGELO EXAM 1 - FEB
secondary to bilateral leg weakness. He noted that syndrome associated with small cell lung carcinoma. It SILVERIO, 2015
ambulation improves throughout the day. which of attacks the presynaptic voltage gated calcium channels MD (TOP 4 -
the following is the most likely mechanism of the causing decrease release of ach in the synaptic junction. AUG 2014
weakness ? accumulation of ach throughout the day will cause MED
A. antibodies to the Ach receptor improvement in the motor strength. BOARDS;
B. antibodies to the presynaptic vesicular TOPNOTCH
transporters MD), MD
C. antibodies to the presynaptic voltage gated
channel
D. antibodies to the Acetylcholinesterase
E. none of the above.

TOPNOTCH MEDICAL BOARD PREP PHYSIOLOGY SUPEREXAM Page 28 of 88


For inquiries visit www.topnotchboardprep.com.ph or email us at topnotchmedicalboardprep@gmail.com
TOPNOTCH MEDICAL BOARD PREP PHYSIOLOGY SUPEREXAM
For inquiries visit www.topnotchboardprep.com.ph or email us at topnotchmedicalboardprep@gmail.com
Item QUESTION EXPLANATION AUTHOR TOPNOTCH
# EXAM
224 what is the most important controller of coronary SIMILAR TO PREVIOUS BOARD EXAM LEAN MIDTERM
arterial blood flow? CONCEPT/PRINCIPLE. Guyton 11th ed pp 250-251 ANGELO EXAM 1 - FEB
A. prostaglandin SILVERIO, 2015
B. Carbon dioxide MD (TOP 4 -
C. lactate AUG 2014
D. adenosine MED
E. All of the above BOARDS;
TOPNOTCH
MD), MD
225 the largest percentage of blood volume is contain in veins are also known as capacitance vessel. It contains as LEAN MIDTERM
which of the following part of the circulation? much as 2/3 of the blood volume. ANGELO EXAM 1 - FEB
A. capillaries SILVERIO, 2015
B. arteries MD (TOP 4 -
C. veins AUG 2014
D. lungs MED
E. All have the same blood volume BOARDS;
TOPNOTCH
MD), MD
226 In which type of shock is associated with increased during septic shock, there is an overall high basal LEAN MIDTERM
cardiac output? metabolic rate resulting to compensatory vasodilatation ANGELO EXAM 1 - FEB
A. Neurogenic shock of systemic arterioles this leads to dramatic decrease in SILVERIO, 2015
B. Septic shock afterload leading to increase cardiac output. All the other MD (TOP 4 -
C. Anaphylactic shock choices are associated with decrease CO. AUG 2014
D. Hypovolumic shock MED
E. All of the above BOARDS;
TOPNOTCH
MD), MD
227 Which of the following is true regarding EEG wave Alpha waves - awake, quiscent state, beta waves - awake, LEAN MIDTERM
forms ? heightened activity, alert. REM sleep- beta waves. Sleep ANGELO EXAM 1 - FEB
A. Delta waves are seen during stage 4 sleep spindles and K complex are seen during Stage II sleep. SILVERIO, 2015
B. Alpha waves is seen during awake,active and Delta waves are prominent during Stage III and IV sleep. MD (TOP 4 -
during periods of sympathetic surge AUG 2014
C. Beta waves are regular waves seen during MED
quiscent state BOARDS;
D. REM sleep show prominence of sleep spindles TOPNOTCH
an K complexes MD), MD
E. all of the above
228 these concept states the specificity of a specific Guyton 11th ed pp 572-573 LEAN MIDTERM
sensory nerve fiber to transmit only one sensory ANGELO EXAM 1 - FEB
modality? SILVERIO, 2015
A. Labeled line principle MD (TOP 4 -
B. Convergence principle AUG 2014
C. Frequency coding principle MED
D. Specificity coding principle BOARDS;
E. None of the above TOPNOTCH
MD), MD
229 which of the following factors will increase the PR increase in K conductance will result in the LEAN MIDTERM
interval in the ECG? hyperpolarization of AV node leading increase in PR ANGELO EXAM 1 - FEB
A. increased epinephrine interval. Increase sodium, calcium, SILVERIO, 2015
B. Increased K permeability epinephrine,norepinephrine and decrease MD (TOP 4 -
C. Increased calcium conductance parasympathetic will result to an opposite outcome. AUG 2014
D. decrease acetylcholine MED
E. Increase sodium conductance BOARDS;
TOPNOTCH
MD), MD
230 which of the following taste sensation will have the bitter is the most sensitive taste sensation as an LEAN MIDTERM
lowest threshold for stimulation? adaptation to prevention of toxin ingestion. ANGELO EXAM 1 - FEB
A. sweet SILVERIO, 2015
B. salty MD (TOP 4 -
C. Sour AUG 2014
D. None of the above MED
E. Both a and b BOARDS;
TOPNOTCH
MD), MD
231 which of the following will adjust the basic the Dorsal group sets off the basic rhythm of respiration. LEAN MIDTERM
respiratory rhythm by decrease the duration of During periods of exertion and exertion, there is ANGELO EXAM 1 - FEB
inspiration and increase the respiratory rate? overflow of stimulation to the ventral respiratory group SILVERIO, 2015
A. Ventral respiratory group such that it will cause tachypnea and set up a higher MD (TOP 4 -
B. pneumotaxic respiratory rhythm. The apneustic center found in pons AUG 2014
C. apneustic causes an increase in inspiratory duration and lowers MED
D. Dorsal respiratory group the respiratory rate. pneumotaxic center seen also in the BOARDS;
E. None of the above pons provide signal to DRG to set off the rhythm. TOPNOTCH
MD), MD
232 which of the following will decrease the excretion of spirinolactone is an aldosterone antagonist. The action LEAN MIDTERM
potassium at the cortical collecting tubule? of the aldosterone is to increase excretion of potassium ANGELO EXAM 1 - FEB
A. Uncompensated metabolic alkalosis while promoting sodium retention.therefore blocking SILVERIO, 2015
B. hypernatremia this action will lead to opposite result. Hypernatremia MD (TOP 4 -
C. Acetazolamide intake will cause increae Na and fluid load in the tubules AUG 2014
D. Spirinolactone leading to further increase in K secretion. same is true in MED
E. All of the above giving an acetazolamide which is acts on proximal tubule BOARDS;
to inhibit absorption of Na. In acute alkalosis, a decrease TOPNOTCH
in hydrogen will have a lesser inhibitory effect on the Na MD), MD
K pump. therefore, more K is in the extracellular
compartment and will be transported to the tubules.

TOPNOTCH MEDICAL BOARD PREP PHYSIOLOGY SUPEREXAM Page 29 of 88


For inquiries visit www.topnotchboardprep.com.ph or email us at topnotchmedicalboardprep@gmail.com
TOPNOTCH MEDICAL BOARD PREP PHYSIOLOGY SUPEREXAM
For inquiries visit www.topnotchboardprep.com.ph or email us at topnotchmedicalboardprep@gmail.com
Item QUESTION EXPLANATION AUTHOR TOPNOTCH
# EXAM
233 What zone of pulmonary perfusion does the lung the apices of the lung is zone 2, while the mid and basal LEAN MIDTERM
apices belong if the patient is in upright still position lung regions are zone 3. Zone 2 states that only during ANGELO EXAM 1 - FEB
? systole that there would be pulmonary perfusion since at SILVERIO, 2015
A. Zone 1 that time it will exceed the alveolar pressure. Zone 3 MD (TOP 4 -
B. Zone 2 means there is a continuous blood flow since the AUG 2014
C. Zone 3 alveolar capillary pressure is always greater than MED
D. Zone 4 alveolar pressure regardless on what phase of cardiac BOARDS;
E. Zone 5 cycle. Zone 1 happens only if the patient is having severe TOPNOTCH
blood loss or in a positive pressure ventilation. MD), MD
234 at 20th week of gestation, what is predominant site during first trimester, the yolk sac is the predominant LEAN MIDTERM
of RBC production? site of RBC production, it gradually involutes and by the ANGELO EXAM 1 - FEB
A. Yolk sac time of 2nd trimester, the liver will take over with the SILVERIO, 2015
B. liver hematopoeisis. It is only during the third trimester does MD (TOP 4 -
C. Bone marrow the marrow will provide the majority of RBC production. AUG 2014
D. spleen MED
E. Both A and B BOARDS;
TOPNOTCH
MD), MD
235 Digestion of which substances/macronutrients the start of digestion of fats occurs primarily in the LEAN MIDTERM
occurs exclusively in the small intestine? intestine because it needs bile for it to be acted upon by ANGELO EXAM 1 - FEB
A. Triglycerides pancreatic lipase. There is no enzyme of physiologic SILVERIO, 2015
B. Proteins significance is found proximal to the duodenum that can MD (TOP 4 -
C. Cellulose digest fats. Sugars are first acted upon by contact with AUG 2014
D. Starch ptyalin. while proteins are initially digested at the MED
E. None of the above stomach. cellulose is indigestible. CORRECTION: actual BOARDS;
answer is E, not A. Lingual Lipase begins digestion of fats TOPNOTCH
in the stomach. Stomach also causes mechanical MD), MD
digestion of fats.
236 A 32 y/o male was using the threadmill machine for among the organs of the body, the brain and the heart LEAN MIDTERM
about an hour already, which of the following organs will have widest range of autoregulation. Blood flow in ANGELO EXAM 1 - FEB
will have no significant decrease in the blood flow these two organs will be constant even if the person is in SILVERIO, 2015
parameter? strenuous exercise. Visceral organs including the MD (TOP 4 -
A. Small intestine kidneys will have a significant decrease in blood flow at AUG 2014
B. brain this time as well as the non exercising muscle such as the MED
C. kidney forearm. BOARDS;
D. Forearm muscle TOPNOTCH
E. None of the above MD), MD
237 which of the following condition will result to a right central obesity can mechanically translate the heart LEAN MIDTERM
axis deviation in the ECG? superolaterally leading to left axis deviation. Aortic ANGELO EXAM 1 - FEB
A. Central obesity regurgitation and aortic stenosis will result to volume SILVERIO, 2015
B. Chronic bronchitis and pressure overload respectively causing also left axis MD (TOP 4 -
C. Aortic regurgitation deviation. Atrial fibrillation presents with absence of P AUG 2014
D. Aortic stenosis waves and irregularly irregular rhythm. chronic MED
E. Atrial fibrillation bronchitis is in a prolonged state of hypoxia resulting to BOARDS;
pulmonary vasoconstriction. this will lead to increase TOPNOTCH
pulmonary overload and as a compensatory, there would MD), MD
right ventricular hypertrophy.
238 what nephron segment is the primary site of unlike other electrolytes wherein majority are absorbed LEAN MIDTERM
magnesium reabsorption? at the first portion of the PCT. Magnesium absorption ANGELO EXAM 1 - FEB
A. Proximal convoluted tubule 1st portion primarily occurs at ascending limb of henle with a SILVERIO, 2015
B. Descending limb of henle. percentage of approximately 65% MD (TOP 4 -
C. Distal cortical collecting tubule AUG 2014
D. Ascending limb of henle MED
E. Proximal convoluted tubule straight portion BOARDS;
TOPNOTCH
MD), MD
239 In Melzack and Wall Gate Control theory, inhibition according to the gate control theory, A beta fibers LEAN MIDTERM
of pain signal transmission is due to stimulation of responsible for tactile stimulations tends to close the ANGELO EXAM 1 - FEB
which nerve fiber type? transmission of a delta and C fibers at the level of SILVERIO, 2015
A. A alpha substantia gelatinosa MD (TOP 4 -
B. Golgi tendon organ AUG 2014
C. A beta MED
D. A delta BOARDS;
E. Type B fibers TOPNOTCH
MD), MD
240 growth hormone promotes catabolism of which growth hormone promotes oxidation of fatty acids while LEAN MIDTERM
metabolic substrate? decreasing the catabolism of sugars and proteins. ANGELO EXAM 1 - FEB
A. protein SIMILAR TO PREVIOUS BOARD EXAM SILVERIO, 2015
B. Glucose CONCEPT/PRINCIPLE. MD (TOP 4 -
C. Fats AUG 2014
D. All of the above MED
E. A and B only BOARDS;
TOPNOTCH
MD), MD
241 Which of the following conditions could potentially SIMILAR TO PREVIOUS BOARD EXAM KEVIN MIDTERM 2
lead to a coagulation defect in both extrinsic and CONCEPT/PRINCIPLE, blockage to bile flow leads to BRYAN LO, EXAM - FEB
intrinsic pathways? decreased vit K absorption leading to alteration in both MD (TOP 7 - 2015
A. a heritable defect involving von willebrand's extrinsic and intrinsic pathways involving factor 7, 9 AUG 2014
factor MED
B. a mutation in the genes coding for platelet Gp1B BOARDS;
receptor TOPNOTCH
C. a mutation in the genes coding for platelet MD)
Gp2B3A receptor
D. a blockage in bile flow
E. an x linked disorder leading to factor 8 deficiency

TOPNOTCH MEDICAL BOARD PREP PHYSIOLOGY SUPEREXAM Page 30 of 88


For inquiries visit www.topnotchboardprep.com.ph or email us at topnotchmedicalboardprep@gmail.com
TOPNOTCH MEDICAL BOARD PREP PHYSIOLOGY SUPEREXAM
For inquiries visit www.topnotchboardprep.com.ph or email us at topnotchmedicalboardprep@gmail.com
Item QUESTION EXPLANATION AUTHOR TOPNOTCH
# EXAM
242 A 65 year old male patient with long standing SIMILAR TO PREVIOUS BOARD EXAM KEVIN MIDTERM 2
hypertension, occasional intermittent chest CONCEPT/PRINCIPLE, know the starling forces and BRYAN LO, EXAM - FEB
heaviness, with month long history of bilateral understand them by heart, patient has heart failure, MD (TOP 7 - 2015
pedal edema more prominent during the afternoon. increased capillary hydrostatic pressure from fluid AUG 2014
He uses 3 pillows at night during sleep. 5 hrs PTC, overload pushes fluid out into interstitium causing MED
patient developed chest heaviness not releaved by pulmonary edema, causing dyspnea BOARDS;
rest and medications, he suddenly developed TOPNOTCH
dyspnea and was rushed to the hospital. On initial MD)
observation, patient was noted to have distended
neck veins and bilateral crackles were heard on both
lungs. The primary pathophysiologic mechanism
causing his dyspnea is
A. increased glomerular hydrostatic pressure
B. increased interstitial oncotic pressure
C. increased capillary hydrostatic pressure
D. increased capillary oncotic pressure
E. increased interstitial hydrostatic pressure
243 A 15 year old male patient presents at the clinic SIMILAR TO PREVIOUS BOARD EXAM KEVIN MIDTERM 2
saying that he was diagnosed with klinefelter's CONCEPT/PRINCIPLE, grossly male genitalia with BRYAN LO, EXAM - FEB
syndrome which of the following physical findings is female secondary sexual characteristics is klinefelter, E MD (TOP 7 - 2015
expected to be found on examination? is down's syndrome, B is turners, C and D AUG 2014
A. breast development, sparse body and facial hair pseudohermaphroditism MED
and grossly male external genitalia BOARDS;
B. short stature, swelling, broad chest, low hairline, TOPNOTCH
low-set ears, and webbed necks MD)
C. male phenotype with ambiguous genitalia
D. female phenotype with ambiguous genitalia
E. small chin, slanted eyes, poor muscle tone, a flat
nasal bridge, single palmar crease
244 Which of the following lung volumes or capacities SIMILAR TO PREVIOUS BOARD EXAM KEVIN MIDTERM 2
might be increased in severe asthma? CONCEPT/PRINCIPLE, residual volume is increased due BRYAN LO, EXAM - FEB
A. IC inspiratory capacity to air trapping, the rest are decreased MD (TOP 7 - 2015
B. FEV forced expiratory volume AUG 2014
C. IRV inspiratory reserve volume MED
D. TV tidal volume BOARDS;
E. RV residual volume TOPNOTCH
MD)
245 Which of the following lung regions are most SIMILAR TO PREVIOUS BOARD EXAM KEVIN MIDTERM 2
susceptible to barotrauma from over ventilation? CONCEPT/PRINCIPLE, apical alveoli are already partially BRYAN LO, EXAM - FEB
A. apical region distended to begin with, further increase in air pressure MD (TOP 7 - 2015
B. upper lobe may promote barotrauma in these regions first before AUG 2014
C. middle lobe the other regions of the lung MED
D. lower lobe BOARDS;
E. all of the above TOPNOTCH
MD)

246 Comparing the cardiac output of the right and left SIMILAR TO PREVIOUS BOARD EXAM KEVIN MIDTERM 2
heart, which of the following is true? CONCEPT/PRINCIPLE, cardiac output in the left and BRYAN LO, EXAM - FEB
A. cardiac output is higher in the left side due to right heart are the same MD (TOP 7 - 2015
increased systemic vascular resistance AUG 2014
B. cardiac output is lower in the right side due to MED
decreased pulmonary vascular resistance BOARDS;
C. cardiac output in the right side of the heart is the TOPNOTCH
same as the left side MD)
D. cardiac output in the left side of the heart is
doubled due to higher pressure in the aorta
E. cardiac output in the right side of the heart is less
due to the lower pressure in the pulmonary
circulation
247 Which of the following conditions suppress the SIMILAR TO PREVIOUS BOARD EXAM KEVIN MIDTERM 2
secretion of growth hormone? CONCEPT/PRINCIPLE, increased glucose levels inhibit BRYAN LO, EXAM - FEB
A. decreased serum cortisol levels growth hormone secretion MD (TOP 7 - 2015
B. increased dopamine secretion AUG 2014
C. decreased thyroid hormone levels MED
D. increased serum glucose BOARDS;
E. decreased food intake or starvation TOPNOTCH
MD)
248 A 1 year old male was rushed to the hospital due to SIMILAR TO PREVIOUS BOARD EXAM KEVIN MIDTERM 2
bouts of persistent watery diarrhea, patient looks CONCEPT/PRINCIPLE, D5W is a hypotonic solution with BRYAN LO, EXAM - FEB
dry and was lethargic, which of the following fluids tendency to go inside of the dehydrated cells, it will not MD (TOP 7 - 2015
when infused would not increase extracellular fluid replete so much the ECF volume or intravascular volume AUG 2014
volume? MED
A. Plain normal saline solution BOARDS;
B. Plain lactated ringers solution TOPNOTCH
C. D5W MD)
D. D5LR
E. D5NSS

TOPNOTCH MEDICAL BOARD PREP PHYSIOLOGY SUPEREXAM Page 31 of 88


For inquiries visit www.topnotchboardprep.com.ph or email us at topnotchmedicalboardprep@gmail.com
TOPNOTCH MEDICAL BOARD PREP PHYSIOLOGY SUPEREXAM
For inquiries visit www.topnotchboardprep.com.ph or email us at topnotchmedicalboardprep@gmail.com
Item QUESTION EXPLANATION AUTHOR TOPNOTCH
# EXAM
249 A 50 year old male patient was rushed to the ER due SIMILAR TO PREVIOUS BOARD EXAM KEVIN MIDTERM 2
to dull crampy generalized vague abdominal pain, CONCEPT/PRINCIPLE, slow vague visceral pain is BRYAN LO, EXAM - FEB
patient had absence of bowel movement for 3 days transmitted by unmyelinated C fibers in contrast to A MD (TOP 7 - 2015
already with minimal flatus accompanied by delta myelinated fibers which transmit fast localized AUG 2014
vomiting and anorexia, patient had a previous somatic pain, B is preganglionic autonomic, A AB are MED
appendectomy. The abdominal pain felt by the motor fibers BOARDS;
patient is most likely conducted by which nerve TOPNOTCH
fibers? MD)
A. A alpha nerve fibers
B. AB nerve fibers
C. A delta nerve fibers
D. B fibers
E. C fibers
250 17 year old female patient noted to have heavy SIMILAR TO PREVIOUS BOARD EXAM KEVIN MIDTERM 2
menses was noted to be pale with easy fatigability, CONCEPT/PRINCIPLE BRYAN LO, EXAM - FEB
this may be brought about by decreased oxygen MD (TOP 7 - 2015
carrying capacity of the blood, one knows that for AUG 2014
every 1g of Hgb can carry how much oxygen? MED
A. 2.56mL BOARDS;
B. 1.34mL TOPNOTCH
C. 100 mL MD)
D. 50 mL
E. 5.35 mL
251 Which among these is the first step in the process of SIMILAR TO PREVIOUS BOARD EXAM KEVIN MIDTERM 2
urine formation CONCEPT/PRINCIPLE BRYAN LO, EXAM - FEB
A. tubular filtration MD (TOP 7 - 2015
B. tubular secretion AUG 2014
C. glomerular filtration MED
D. tubular reabsorption BOARDS;
E. glomerular feedback TOPNOTCH
MD)
252 A patient who took an overdose of aspirin started SIMILAR TO PREVIOUS BOARD EXAM KEVIN MIDTERM 2
having tinnitus, headache, dizziness or vertigo, CONCEPT/PRINCIPLE: increase temperature, increase BRYAN LO, EXAM - FEB
confusion, tachypnea, metabolic acidosis, nausea, acidity shifts dissociation curve to the right MD (TOP 7 - 2015
vomiting, dehydration/thirst and hypoglycemia, AUG 2014
which of the following physiologic processes is MED
expected to occur BOARDS;
A. shift in hemoglobin dissociation curve to the right TOPNOTCH
B. decreased baroreceptor stimulation MD)
C. increased intracellular influx of potassium
D. decreased renal reabsorption of bicarbonate
E. decreased ADH secretion
253 Which of the following is true regarding the SIMILAR TO PREVIOUS BOARD EXAM KEVIN MIDTERM 2
sympathetic nervous system? CONCEPT/PRINCIPLE, sympathetic has short BRYAN LO, EXAM - FEB
A. It is responsible for accomodation of vision to preganglionic long postganglionic fibers MD (TOP 7 - 2015
near objects AUG 2014
B. it is responsbile for relaxation of urinary and MED
gastrointestinal sphincters BOARDS;
C. it has short preganglionic fibers and long post TOPNOTCH
ganglionic fibers MD)
D. it only utilizes norepinephrine as
neurotransmitter
E. the autonomic ganglia are usually embedded
inside the effector organs
254 Which of the following is false of skeletal muscle SIMILAR TO PREVIOUS BOARD EXAM KEVIN MIDTERM 2
contraction CONCEPT/PRINCIPLE, review skeletal muscle BRYAN LO, EXAM - FEB
A. action potential travels down depolarizing the contraction and NMJ physiology, post synaptic MD (TOP 7 - 2015
terminal axonal bouton promoting release of acetylcholine receptor in the muscle is a nicotinic ligand AUG 2014
Acetylcholine gated ion channel MED
B. during membrane depolarization, the inside of the BOARDS;
cell becomes positively charged TOPNOTCH
C. acetylcholine diffuses across the synaptic cleft MD)
stimulating muscarinic acetylcholine receptors
D. acetylcholinesterase leads to break down of
acetylcholine terminating the synaptic transmission
E. the acetylcholine receptor at the postsynaptic
membrane is a ligand gate ion channel
255 24 year old female patient is pregnant at 8weeks SIMILAR TO PREVIOUS BOARD EXAM KEVIN MIDTERM 2
AOG presents with intractable vomiting, which CONCEPT/PRINCIPLE, can be as simple as this, BRYAN LO, EXAM - FEB
potential acid base disturbance can be seen in this intractable vomiting = metabolic alkalosis MD (TOP 7 - 2015
patient? AUG 2014
A. respiratory acidosis MED
B. compensated metabolic acidosis BOARDS;
C. respiratory alkalosis TOPNOTCH
D. metabolic alkalosis MD)
E. none of the above

TOPNOTCH MEDICAL BOARD PREP PHYSIOLOGY SUPEREXAM Page 32 of 88


For inquiries visit www.topnotchboardprep.com.ph or email us at topnotchmedicalboardprep@gmail.com
TOPNOTCH MEDICAL BOARD PREP PHYSIOLOGY SUPEREXAM
For inquiries visit www.topnotchboardprep.com.ph or email us at topnotchmedicalboardprep@gmail.com
Item QUESTION EXPLANATION AUTHOR TOPNOTCH
# EXAM
256 A neonate was jaundiced at the 8th hour of life, his SIMILAR TO PREVIOUS BOARD EXAM KEVIN MIDTERM 2
mother was RH(-) baby was RH(+), total bilirubin CONCEPT/PRINCIPLE, kernicterus affects the basal BRYAN LO, EXAM - FEB
levels were found to be 25mg/dL at the 2nd day of ganglia MD (TOP 7 - 2015
life, patient was deemed at risk for a CNS AUG 2014
complication due to these conditions which were MED
most likely to affect which part of the brain? BOARDS;
A. basal ganglia TOPNOTCH
B. cerebellum MD)
C. medulla
D. cerebral cortex
E. hypothalamus
257 A 30 year old patient was involved in a motor CO = HR x Stroke volume, stroke volume is amount of KEVIN MIDTERM 2
vehicular accident, he has sustained multiple blood pumped by the heart during each contraction BRYAN LO, EXAM - FEB
fractures over the lower extremities and multiple rib influenced by preload which is represented by left MD (TOP 7 - 2015
fracturs as well. His initial BP was 80/50 HR 138, he ventricular end diastolic volume, afterload and AUG 2014
was assessed to have hypovolemic shock, which of contractility. Decreased afterload increases stroke MED
the following parameters are known to increase volume thereby increasing CO, increased venous BOARDS;
cardiac output? capacitance decreases preload thereby decreasing stroke TOPNOTCH
A. increased venous capacitance volume and CO MD)
B. decreased heart rate
C. decreased afterload
D. decreased inotropy
E. decreased left ventricular end diastolic volume
258 Which of the following conditions increases gastric presence of breakdown products fat, peptides, acid in KEVIN MIDTERM 2
emptying time? the duodenum delays gastric emptying, fatty meal delays BRYAN LO, EXAM - FEB
A. decreased cholecystokinin secretion emptying, increase cholecystokinin delays emptying MD (TOP 7 - 2015
B. decreased intake of fatty foods AUG 2014
C. increased motilin secretion MED
D. presence of fat and peptides in the duodenum BOARDS;
E. none of the above TOPNOTCH
MD)
259 Assuming equal amounts of glucose were provided, because of incretin effect, oral glucose causes increased KEVIN MIDTERM 2
which of the following routes produces a greater insulin secretion BRYAN LO, EXAM - FEB
increase in pancreatic insulin secretion? MD (TOP 7 - 2015
A. oral glucose intake AUG 2014
B. rapid IV glucose infusion MED
C. sugar tablets underneath the tongue BOARDS;
D. A and B produces equal rises in insulin TOPNOTCH
E. none of the above MD)

260 A 30 year old female patient presents with patient most likely has cushings diease, pituitary ACTH KEVIN MIDTERM 2
hypertension, supraclavicular fat pad, multiple most likely increased, non suppression with low dose BRYAN LO, EXAM - FEB
abdominal striae, obesity, red flushed skin, history of dexa, cortisol in serum and urine is expected to be MD (TOP 7 - 2015
visual disturbances specifically involving temporal increased as well AUG 2014
visual fields of both eyes, which of the following MED
laboratory findings is expected? BOARDS;
A. decreased serum free cortisol TOPNOTCH
B. suppression of cortisol secretion following low MD)
dose dexamethasone administration
C. decreased urine free cortisol
D. increased ACTH
E. increased urinary metanephrines
261 The basement membrane of intestinal epithelial cells GLUT 2 is a facilitated diffusion transporter which is RAYMUND MIDTERM 3
requires the GLUT2 transporter for absorption of stereospecific, saturable and competitive. Choice B is MARTIN LI, EXAM - FEB
glucose into circulation. Which of following is true descriptive of competitive inhibition. MD (TOP 1 - 2015
about this kind of transport? AUG 2014
A. This transporter will absorb both D-glucose and MED
L-glucose into the circulation. BOARDS;
B. Other similarly structured solutes such as TOPNOTCH
galactose will competitively inhibit the absorption of MD)
glucose
C. The transport protein has no maximum transport
capability and and is directly proportional to the
gradient at all concentrations
D. This transporter is dependent on ATP
E. None of the above
262 A 22 year old man is noted to have normal Sertoli cells is responsible for secreting the Androgen RAYMUND MIDTERM 3
secondary sex characteristics but has deficient binding protein which traps and maintains testosterone MARTIN LI, EXAM - FEB
testosterone levels in the seminifirous tubules and concentration in the seminiferous tubules. The presence MD (TOP 1 - 2015
epididymis. Which of the following is most likely of normal secondary sex characteristics indicates that AUG 2014
dysfunctional? Leydig cells is functional. MED
A. Leydig cells BOARDS;
B. Anterior pituitary TOPNOTCH
C. Hypothalamus MD)
D. Sertoli cells
E. A an B

TOPNOTCH MEDICAL BOARD PREP PHYSIOLOGY SUPEREXAM Page 33 of 88


For inquiries visit www.topnotchboardprep.com.ph or email us at topnotchmedicalboardprep@gmail.com
TOPNOTCH MEDICAL BOARD PREP PHYSIOLOGY SUPEREXAM
For inquiries visit www.topnotchboardprep.com.ph or email us at topnotchmedicalboardprep@gmail.com
Item QUESTION EXPLANATION AUTHOR TOPNOTCH
# EXAM
263 RBC's in the unoxygenated systemic venous HCO3- is transported out of the RBC in exchange for Cl- RAYMUND MIDTERM 3
circulation will have which of the following in unoxygenated blood. Choices A and C occur in MARTIN LI, EXAM - FEB
characteristics? oxygenated blood. The major form of transport of CO2 is MD (TOP 1 - 2015
A. Has less H+ bound to hemoglobin via HCO3-. AUG 2014
B. The major form of transport of CO2 is via binding MED
to hemoglobin to form carbaminohemoglobin BOARDS;
C. HCO3- enters the RBC's to recombine with H+ to TOPNOTCH
form H2CO3 and which dissociates into CO2 and MD)
H2O
D. Has increased intracellular levels of Cl-
E. All of the above
264 Absence of T-tubules in skeletal muscle cells will T-tubules is responsible for the synchrony of Calcium RAYMUND MIDTERM 3
result in: release. MARTIN LI, EXAM - FEB
A. Asynchronous waveforms and calcium release MD (TOP 1 - 2015
B. Absence of action potential generation AUG 2014
C. Absence of calcium uptake by the sarcoplasmic MED
reticulum BOARDS;
D. Inability of acetylcholine to bind and activate TOPNOTCH
receptors at the neuromuscular junction MD)
E. All of the above
265 Which of the following will increase the contraction Afterload is the load against which the skeletal muscle RAYMUND MIDTERM 3
velocity of a skeletal muscle? must contract and is analogous to the afterload of the MARTIN LI, EXAM - FEB
A. Decreased afterload heart. Decrease in afterload will increase contraction MD (TOP 1 - 2015
B. Decreased myosin-actin overlap velocity. In skeletal muscles, an increase in extracellular AUG 2014
C. Increased extracellular Ca Calcium will lower gradient and decrease contractility. MED
D. All of the above BOARDS;
E. None of the above TOPNOTCH
MD)
266 An exercising athlete will have which of the All are correct. Venous pCO2 (but not arterial pCO2) will RAYMUND MIDTERM 3
following respiratory responses? increase. MARTIN LI, EXAM - FEB
A. Increased oxygen consumption MD (TOP 1 - 2015
B. More evenly distributed V/Q ratio AUG 2014
C. No change in arterial pCO2 and pO2 MED
D. A and B BOARDS;
E. All of the above TOPNOTCH
MD)
267 Glucose and insulin levels were measured one hour The scenario shows high glucose levels despite RAYMUND MIDTERM 3
after a patient is instructed to ingest 75 grams corresponding insulin release. MARTIN LI, EXAM - FEB
glucose which revealed glucose level of 250mg/dL MD (TOP 1 - 2015
accompanied by a corresponding rise in insulin AUG 2014
secretion when compared to baseline. What is most MED
likely the explanation for this finding? BOARDS;
A. Peripheral insulin resistance TOPNOTCH
B. Pancreatic beta cell destruction MD)
C. Insulinoma
D. Physiologic glucose mobilization upon insulin
release
E. A or B
268 A 63 year old presents to the clinic with complaints Constriction of vascular smooth muscles involves RAYMUND MIDTERM 3
of dizziness and lightheadedness upon assuming an stimulation of alpha 1-receptors. The mechanism is via MARTIN LI, EXAM - FEB
upright position. His supine blood pressure Gq, leading to phospholipase C - mediated release of MD (TOP 1 - 2015
measurement is 95/65 and his standing blood diacylglycerol and IP3 AUG 2014
pressure is 80/60. What will you expect to MED
accumulate in the vascular smooth muscle cells in BOARDS;
this patient upon standing. TOPNOTCH
A. cAMP MD)
B. cGMP
C. NO
D. inositol triphosphate
E. acetylcholine
269 Conduction velocity measurements were obtained Fastest to slowest: Purkinje>atria and ventricles > AV RAYMUND MIDTERM 3
from different parts of the heart. The slowest and node MARTIN LI, EXAM - FEB
fastest conduction velocities respectively will be MD (TOP 1 - 2015
found in: AUG 2014
A. Atrial and ventricular muscle cells MED
B. AV node and atrial muscle cells BOARDS;
C. SA node and AV node TOPNOTCH
D. AV node and Purkinje fibers MD)
E. Ventricular muscle cells and SA node
270 Angiotensin II plays an essential role in regulation of Contriction of efferent arteriole will decrease RPF and RAYMUND MIDTERM 3
renal circulation. Its action on the efferent arteriole increase GFR and FF MARTIN LI, EXAM - FEB
will result in which of the following changes? MD (TOP 1 - 2015
A. Decreased GFR, No change in RPF, Decreased AUG 2014
Filtration fraction (FF) MED
B. Increased GFR, Decreased RPF, Increased FF BOARDS;
C. Decreased GFR, Decreased RPF, No change in FF TOPNOTCH
D. Decreased GFR, Decreased RPF, Decreased FF MD)
E. None of the above
271 Unmyelinated nerve fibers is best exemplified by? Unmyelinated: postganglionic autonomic, slow pain RAYMUND MIDTERM 3
A. Preganglionic autonomic fibers MARTIN LI, EXAM - FEB
B. Postganglionic autonomic fibers MD (TOP 1 - 2015
C. A-delta touch and pressure nerve fibers AUG 2014
D. Muscle spindle afferents MED
E. Golgi tendon nerve fibers BOARDS;
TOPNOTCH

TOPNOTCH MEDICAL BOARD PREP PHYSIOLOGY SUPEREXAM Page 34 of 88


For inquiries visit www.topnotchboardprep.com.ph or email us at topnotchmedicalboardprep@gmail.com
TOPNOTCH MEDICAL BOARD PREP PHYSIOLOGY SUPEREXAM
For inquiries visit www.topnotchboardprep.com.ph or email us at topnotchmedicalboardprep@gmail.com
Item QUESTION EXPLANATION AUTHOR TOPNOTCH
# EXAM
MD)

272 The resting membrane potential of an isolated cell is Resting membrane potential approximates that of RAYMUND MIDTERM 3
found to be -70. Which of the following solutes is potassium which is -85 MARTIN LI, EXAM - FEB
most likely responsible for this? MD (TOP 1 - 2015
A. Sodium AUG 2014
B. Potassium MED
C. Calcium BOARDS;
D. Sodium and Calcium TOPNOTCH
E. Sodium and Chloride MD)

273 What is responsible for an HCO3- rich alkaline Secretin release by duodenal cells RAYMUND MIDTERM 3
intestinal content? MARTIN LI, EXAM - FEB
A. Pancreatic ductal stimulation by a duodenal MD (TOP 1 - 2015
hormone AUG 2014
B. CCK-mediated gallbladder contraction MED
C. Sympathetic nerve stimulation BOARDS;
D. Somatostatin-mediated inhibition TOPNOTCH
E. None of the above MD)
274 A 33 year old woman presents to your clinic Though mediated by acteylcholine, the thoracic RAYMUND MIDTERM 3
complaining of excessive sweating in the axillary sympathetic is responsible for sweating. Cervical MARTIN LI, EXAM - FEB
region. Destruction of which of the following symphatetic for head and neck MD (TOP 1 - 2015
structures will prevent the said complaint? AUG 2014
A. Vagus nerve MED
B. Cervical sympathetic ganglion BOARDS;
C. Thoracic sympathetic trunk TOPNOTCH
D. Sacral sympathetic plexus MD)
E. None of the above

275 What is the mechanism by which thyroid hormone Thyroid hormone - nuclear receptors; adrenal steroid RAYMUND MIDTERM 3
act on its target tissues? hormones - cytoplasmic receptors MARTIN LI, EXAM - FEB
A. Gs-mediated activation of protein kinase A MD (TOP 1 - 2015
B. Gq-mediated release of Ca AUG 2014
C. Binding to intracellular cytoplasmic receptor MED
D. Binding to intracellular nuclear receptor BOARDS;
E. Binding to a transmembrane receptor with TOPNOTCH
intrinsic tyrosine kinase activity MD)
276 You assess a 72 year old male patient who comes S3 - rapid ventricular filling; S4- atrial systole RAYMUND MIDTERM 3
into your clinic complaining of chest tightness and MARTIN LI, EXAM - FEB
dyspnea usually precipitated by exertion. Upon MD (TOP 1 - 2015
physical examination, you hear a murmur AUG 2014
immediately after the 2nd heart sound when the MED
patient is in a left lateral position. You conclude that BOARDS;
what you hear is the third heart sound and that it TOPNOTCH
occurs during which part of the cardiac cycle? MD)
A. isovolumic ventricular relaxation
B. atrial systole
C. rapid ventricular filling
D. rapid ventricular ejection
E. reduced ventricular filling
277 The most significant airway resistance is in what Small bronchioles have decreased resistance due to its RAYMUND MIDTERM 3
part of the respiratory system? parallel arrangement MARTIN LI, EXAM - FEB
A. Primary bronchi MD (TOP 1 - 2015
B. Small brionchioles AUG 2014
C. Alveolar ducts MED
D. Trachea BOARDS;
E. Medium-sized bronchi TOPNOTCH
MD)
278 A 53 year old woman who wants to prevent Vit D increases Ca and P levels RAYMUND MIDTERM 3
osteoporotic fractures takes excessive amounts of MARTIN LI, EXAM - FEB
Vitamin D. Which of the following will be expected? MD (TOP 1 - 2015
A. Increased Phosphorus, Decreased Ca, Increased AUG 2014
PTH MED
B. Decreased Phosphorus, Increased Ca, Increased BOARDS;
PTH TOPNOTCH
C. Increased Phosphorus, Increased Ca, Decreased MD)
PTH
D. Increased Calcium and Decreased PTH, No change
in Phophorus
E. None of the above
279 A cardiac patient asks you to discuss to her in great Phase 2 occurs in ventricular, atria, and Purkinje but RAYMUND MIDTERM 3
detail the physiology of cardiac action potential. You NOT in SA node MARTIN LI, EXAM - FEB
tell her that Phase 2 of SA node action potential is: MD (TOP 1 - 2015
A. caused by inward Na current AUG 2014
B. the plateau of the action potential MED
C. due to a transient increase in Ca conductance BOARDS;
D. does not occur TOPNOTCH
E. None of these is correct MD)

TOPNOTCH MEDICAL BOARD PREP PHYSIOLOGY SUPEREXAM Page 35 of 88


For inquiries visit www.topnotchboardprep.com.ph or email us at topnotchmedicalboardprep@gmail.com
TOPNOTCH MEDICAL BOARD PREP PHYSIOLOGY SUPEREXAM
For inquiries visit www.topnotchboardprep.com.ph or email us at topnotchmedicalboardprep@gmail.com
Item QUESTION EXPLANATION AUTHOR TOPNOTCH
# EXAM
280 A patient who presents with hyperventilation at the Dec in pCO2, temp, 2,3 BPG and Inc in pH - shift to the RAYMUND MIDTERM 3
ER will have which of the following changes in the left and Dec in P50 MARTIN LI, EXAM - FEB
hemoglobin-O2 dissociation curve MD (TOP 1 - 2015
A. Shift to the left AUG 2014
B. Decrease in P50 MED
C. Increase in P50 BOARDS;
D. A and B TOPNOTCH
E. A and C MD)

281 Which of the following is true regarding pulmonary the distribution of blood flow in the lungs is affected by ERIC ROYD FINAL EXAM -
blood flow when a person is standing? gravitational effects on arterial hydrostatic pressure. TALAVERA, FEB 2015
A. It is equal between the apex and the base Thus blood flow is greatest in the base, where arterial MD (TOP 1 -
B. It is higher in the apex owing to the effect of hydrostatic pressure is greatest and the difference AUG 2014
gravity on arterial pressure between the arterial and venous pressure is greatest. MED
C. It is higher in the base due to a greater difference BOARDS;
between arterial and venous pressure TOPNOTCH
D. It is lower in the base because that is where MD)
alveolar pressure is greater than arterial pressure
E. None of the above

282 Which of the following gastrointestinal hormones CCK is the most important hormone for digestion and ERIC ROYD FINAL EXAM -
inhibits gastric emptying? absorption of dietary fat. In addition to causing TALAVERA, FEB 2015
A. Gastrin conraction of the gallbladder, it inhibits gastric MD (TOP 1 -
B. Secretin emptying. As a result chyme moves more slowly from AUG 2014
C. Vasoactive intestinal peptide the stomach to the small intestine , thus allowing more MED
D. Cholecystokinin time for fat absorption BOARDS;
E. Gastric inhibitory peptide TOPNOTCH
MD)
283 Which of the following is an INCORRECT statement Due to the sluggish blood flow in polycythemia, most of ERIC ROYD FINAL EXAM -
regarding the effect of polycythemia on the the hemoglobin is in the deoxygenated state. The blue TALAVERA, FEB 2015
circulatory system? color of the deoxygenated Hb masks the red color of the MD (TOP 1 -
A. The increased viscosity promotes a sluggish oxygenated Hb. Therefore a person with polycythemia AUG 2014
blood flow ordinarily has a ruddy complexion with a bluish MED
B. There is a decreased rate of venous return to the (cyanotic) tint to the skin. BOARDS;
heart TOPNOTCH
C. Majority of the hemoglobin is in the oxygenated MD)
state
D. Blood pressure is normal in most individuals
E. All of the above
284 Which of the following conditions would not have an Conn's syndrome is characterized by the overproduction ERIC ROYD FINAL EXAM -
elevation of the plasma renin level? of the mineralocorticoid hormone aldosterone by the TALAVERA, FEB 2015
A. Congestive Heart Failure adrenal glands. This will produce a negative feedback MD (TOP 1 -
B. Conn's syndrome and thus suppress renin secretion. The rest are causes of AUG 2014
C. Renal artery stenosis secondary hyperaldosteronism wherein a decrease in MED
D. Hemorrhagic shock the intravascular volume would stimulate the RAA axis. BOARDS;
E. Liver cirrhosis TOPNOTCH
MD)
285 A patient with chronic asthma is started on Glucocorticoid therapy enhances bone loss by inhibiting ERIC ROYD FINAL EXAM -
glucocorticoid therapy. The treatment may result in bone formation and increasing bone resorption. TALAVERA, FEB 2015
bone loss through what mechanism ? MD (TOP 1 -
A. Suppress vitamin D activation AUG 2014
B. Increase production of osteoblast MED
C. Inhibition of bone formation BOARDS;
D. Inhibition of bone resorption TOPNOTCH
E. Increase calcium absorption from GI tract MD)

286 A 78 year old man visited his physician because he This is a case of Parkinson's disease characterized by ERIC ROYD FINAL EXAM -
finds it difficult to hold his hand steady when resting tremor, rigidity and akinesia. It is caused by TALAVERA, FEB 2015
painting. Physical examination reveals rest tremor destruction of the dopamine secreting neurons within MD (TOP 1 -
and rigidity. The patient's neurological signs are the substantia nigra AUG 2014
most likely related to a lesion within which of the MED
following ? BOARDS;
A. Vestibular nucleus TOPNOTCH
B. Substantia nigra MD)
C. Premotor area
D. Caudate nucleus
E. Hippocampus
287 A 2D-echo was performed on a 66 year old male blood leaks from the aorta into the LV during diastole in ERIC ROYD FINAL EXAM -
with aortic regurgitation to assess ejection fraction patients with AR producing a diastolic murmur. The TALAVERA, FEB 2015
and ventricular hypertrophy. Which of the following rapid flow of blood into the LV during diastole also cause MD (TOP 1 -
would be present in a patient with aortic an increase in end diastolic volume which results in a AUG 2014
regurgitation? larger stroke volume and therefore a larger pulse MED
A. A decrease in diastolic pressure pressure. BOARDS;
B. A decrease in pulse pressure TOPNOTCH
C. Systolic murmur MD)
D. Bradycardia
E. Decrease in heart size
288 The affinity of hemoglobin for oxygen is increased by ERIC ROYD FINAL EXAM -
which of the following? TALAVERA, FEB 2015
A. Metabolic acidosis MD (TOP 1 -
B. Carbon monoxide poisoning AUG 2014
C. Anemia MED
D. Hypoxemia BOARDS;
E. Exercise TOPNOTCH
MD)
TOPNOTCH MEDICAL BOARD PREP PHYSIOLOGY SUPEREXAM Page 36 of 88
For inquiries visit www.topnotchboardprep.com.ph or email us at topnotchmedicalboardprep@gmail.com
TOPNOTCH MEDICAL BOARD PREP PHYSIOLOGY SUPEREXAM
For inquiries visit www.topnotchboardprep.com.ph or email us at topnotchmedicalboardprep@gmail.com
Item QUESTION EXPLANATION AUTHOR TOPNOTCH
# EXAM
289 A 17 year old male who is being treated with Motilin is the GI peptide hormone associated with ERIC ROYD FINAL EXAM -
erythromycin complains of nausea, intestinal initiation of migrating motor complexes during the TALAVERA, FEB 2015
cramping and diarrhea. The side effects are the interdigestive period. The hormone stimulates increased MD (TOP 1 -
result of stimulation of enteric nerves and smooth contractions by a direct action on smooth muscle and by AUG 2014
muscle cells that recognize which hormone? activation of excitatory enteric nerves. Macrolides MED
A. Secretin antibiotics show an ability to excite motilin like BOARDS;
B. Cholecystokinin receptors on enteric nerves and smooth muscle TOPNOTCH
C. Gastrin MD)
D. Motilin
E. Enterogastrone
290 A 64 year old man with a history of renal NSAIDs inhibit synthesis of prostaglandins. PG mediate ERIC ROYD FINAL EXAM -
insufficiency is admitted to the hospital after taking afferent arteriole vasodilation. Therefore the absence of TALAVERA, FEB 2015
a large amount of ibuprofen. His BUN was elevated PG will decrease blood flow through the afferent MD (TOP 1 -
at 150 mg/dL. Which of the following is the most arteriole and thus decrease the GFR, causing elevation of AUG 2014
likely cause of the elevated values? BUN MED
A. An increased synthesis of urea by the liver BOARDS;
B. An increased reabsorption of urea by the TOPNOTCH
proximal tubules MD)
C. A decrease secretion of urea by the distal tubules
D. A decrease in the GFR
E. An increased renal blood flow
291 A 16 year old medical student was brought to the ER Anaphylactic shock is characterized by decreased ERIC ROYD FINAL EXAM -
in shock. Which of the following findings would peripheral resistance and a high cardiac output. TALAVERA, FEB 2015
favor anaphyactic shock rather than hypovolemic Hypovolemic shock is caused by decrease in ECV and MD (TOP 1 -
shock? arterial pressure. Both types of shock, blood flow is AUG 2014
A. Cardiac output is higher than normal shunted away from the heart, decreasing renal blood MED
B. Ventricular contractility is greater than normal flow and GFR leading to an increased Crea. In both BOARDS;
C. Serum creatinine is elevated hypovolemic and anaphylactic the baroreceptor reflex is TOPNOTCH
D. Total peripheral resistance is greater than stimulated which increases HR and ventricular MD)
normal contractility
E. All of the above
292 The NMDA receptor is activated by which of the ERIC ROYD FINAL EXAM -
following neurotransmitters? TALAVERA, FEB 2015
A. Glycine MD (TOP 1 -
B. Acetylcholine AUG 2014
C. Substance P MED
D. Histamine BOARDS;
E. Glutamate TOPNOTCH
MD)
293 Which of the following characteristics is shared by both types of transport occur down an electrochemical ERIC ROYD FINAL EXAM -
simple and facilitated diffusion of glucose? gradient and do not require metabolic energy. TALAVERA, FEB 2015
A. Occurs down an electrochemical gradient Saturability and inhibition of other sugars are MD (TOP 1 -
B. Saturation characteristic only of carrier mediated glucose transport AUG 2014
C. Requires metabolic energy MED
D. Inhibited by the presence of galactose BOARDS;
E. Requires a gradient establised by sodium TOPNOTCH
MD)

294 A 45 year old male consulted his ophthalmologist contracting the ciliary body increases the refractive ERIC ROYD FINAL EXAM -
because of increasing difficulty in reading the power of the eye for near vision. When the ciliary muscle TALAVERA, FEB 2015
newspaper. His vision problem most likley results contracts, it pulls the suspensory ligaments toward the MD (TOP 1 -
from the inability to contract which of the following? cornea, which permits the lens surface to bulge, AUG 2014
A. Iris increasing its refractive power. MED
B. Ciliary body BOARDS;
C. Extraocular muscles TOPNOTCH
D. Pupil MD)
E. None of the above

295 Which of the following is the major factor that Pancreatic bicarbonate secretion into the small intestine ERIC ROYD FINAL EXAM -
protects the duodenal mucosa from damage by is essential for neutralizationof gastric acid emptied into TALAVERA, FEB 2015
gastric acid? the small intestine. Unlike the gastric mucosal lining, the MD (TOP 1 -
A. Endogenous mucosal barrier of the duodenum small intestine does not have a significant endogenous AUG 2014
B. Duodenal bicarbonate secretion mechanism. Upon delivery into the proximal small MED
C. Bicarbonate concentrated in the bile intestine, hydrogen ions stimulate release of secretin BOARDS;
D. Hepatic bicarbonate secretion which stimulates pancreatic bicarbonate secretion. The TOPNOTCH
E. Pancreatic bicarbonate secretion acid ouput of the stomach during a meal is matched MD)
equallly by the pancreatic output of bicarbonate
296 Radiation treatment for a pituitary tumor in an 8 Absence of gonadotropins would cause absence of sexual ERIC ROYD FINAL EXAM -
year old boy resulted in the complete loss of maturation. Growth stunting is expected due to absence TALAVERA, FEB 2015
pituitary function. As a result, the child is likely to of GH . Goiter is not seen in panhypopituitarism since MD (TOP 1 -
experience which of the following signs/symptoms? there would be no TSH present to stimulate excessive AUG 2014
A. Goiter growth of the thyroid follicles. MED
B. Accerlerated growth spurt BOARDS;
C. Absent sexual maturation TOPNOTCH
D. Both B and C MD)
E. None of the above

TOPNOTCH MEDICAL BOARD PREP PHYSIOLOGY SUPEREXAM Page 37 of 88


For inquiries visit www.topnotchboardprep.com.ph or email us at topnotchmedicalboardprep@gmail.com
TOPNOTCH MEDICAL BOARD PREP PHYSIOLOGY SUPEREXAM
For inquiries visit www.topnotchboardprep.com.ph or email us at topnotchmedicalboardprep@gmail.com
Item QUESTION EXPLANATION AUTHOR TOPNOTCH
# EXAM
297 A 68 year old female underwent endoscopic Increased ICP stimulated the cushing's reflex which ERIC ROYD FINAL EXAM -
ventriculostomy to treat her obstructive causes: hypertension, bradycardia and apnea TALAVERA, FEB 2015
hydrocephalus. Intracranial pressure (ICP) MD (TOP 1 -
increased during the procedure. Which of the AUG 2014
following changes in the cardiovascular system can MED
be a result of the increased ICP? BOARDS;
A. A decrease in ventricular contractility TOPNOTCH
B. A decrease in stroke volume MD)
C. A decrease in mean blood pressure
D. A decrease in heart rate
E. A decrease in total peripheral resistance
298 The activity of the central chemoreceptors is The central chemoreceptors are located at or near the ERIC ROYD FINAL EXAM -
primarily stimulated by which of the following? ventral surface of the medulla. They are stimulated to TALAVERA, FEB 2015
A. A decrease in PO2 of blood flowing through the increase ventilation by a decrease in the pH of the ECF. MD (TOP 1 -
brain The pH of this ECF is affected by the PCO2 of the blood AUG 2014
B. An increase in the PCO2 of blood flowing supply to the medullary chemoreceptor as well as by the MED
through the brain CO2 and lactic acid production of the surrounding brain BOARDS;
C. A decrease in the oxygen content of blood tissue. TOPNOTCH
flowing through the brain MD)
D. A decrease in the metabolic rate of the
surrounding brain tissue
E. An increase in the pH of the CSF
299 Which of the following hormones interact with a ERIC ROYD FINAL EXAM -
cytoplasmic receptor and then localizes in the TALAVERA, FEB 2015
nucleus and directs protein and nucleotide MD (TOP 1 -
synthesis? AUG 2014
A. Cortisol MED
B. Epinephrine BOARDS;
C. Insulin TOPNOTCH
D. TRH MD)
E. Luteinizing hormone
300 If the patient is unable to hear high frequency the portion of the basilar membrane vibrated by a sound ERIC ROYD FINAL EXAM -
sounds, the damage to the basilar membrane is depends on the frequency. High frequency sounds TALAVERA, FEB 2015
closest to which of the following? produce a vibration of the basilar membrane at the base MD (TOP 1 -
A. Oval window of the cochlea (near the oval and round window); low AUG 2014
B. Helicotrema frequency sounds produce a vibration of the basilar MED
C. Modiolus membrane at the apex of the cochlea (helicotrema). The BOARDS;
D. Stria vascularis modiulus is the bony center of the cochlea from which TOPNOTCH
E. Spiral ganglion the basilar membrane emerges. the spiral ganglion MD)
contains the cell bodies of the auditory nerve fibers, and
the stria vascularis is the vascular bed located on the
outer wall of the scala media of the cochlea responsible
for endolymph secretion
301 which of the following processes will account for the prolonged opening of the ryanodine receptors will LEAN BACK-UP
excessive uncontrolled muscle rigidity seen in promote excessive Calcium entry in the sarcoplasm ANGELO MIDTERM
Malignant Hyperthermia ? leading to stimulation of sarcoplasmic reticulum to SILVERIO, EXAM - FEB
A. Increase activity of the SERCa pump release excessive amounts of ca for muscular contraction MD (TOP 4 - 2015
B. Delayed closure of the ryanodine receptors in leadin to rigidity. The other choices will lead to decrease AUG 2014
the skeletal muscle ca level in the sarcoplasm. Guyton 11th ed p 89-91 MED
C. decrease level of voltage gated Na and K BOARDS;
channels in the Transverse tubules TOPNOTCH
D. decrease activity of the DHPR MD), MD
E. none of the above
302 what kind of transport process used by non in the presence of insulin, glut 4 receptors are LEAN BACK-UP
exercising skeletal muscles for glucose uptake ? translocated in to sarcoplasmic membrane to promote ANGELO MIDTERM
A. Simple diffusion glucose uptake via facilitated diffusion. Guyton 11th ed p SILVERIO, EXAM - FEB
B. Secondary active transport 50 MD (TOP 4 - 2015
C. Counter or exchange transport AUG 2014
D. Facilitated diffusion MED
E. None of the above BOARDS;
TOPNOTCH
MD), MD
303 which of the following scenario will result to tetanic heart muscle is dependent on extracellular calcium ions LEAN BACK-UP
contraction of the cardiac muscle ? for its contraction. Increasing the latter in an high ANGELO MIDTERM
A. Increased extracellular calcium ions proportion will lead to tetanic contraction of the heart or SILVERIO, EXAM - FEB
B. Extreme hyperthermia ventricular fibrillation. Guyton 11 th ed p 114-115 MD (TOP 4 - 2015
C. Increase catecholamine surge AUG 2014
D. Increase extracellular potassium ions MED
E. All of the above BOARDS;
TOPNOTCH
MD), MD
304 A 45 y/o male, hypertensive an diabetic went for if the origin of cardiac impulse is below the AV node, the LEAN BACK-UP
general check up. PE was unremarkable. 12L ECG action potential will travel towards the atria as well as to ANGELO MIDTERM
was done showing normal sinus rhythm however the ventricles. Since there would be much delay SILVERIO, EXAM - FEB
there were occasional inverted P waves noted after travelling towards the AV node than to the ventricles, MD (TOP 4 - 2015
the QRS complex. What is the most likely reason for inverted P waves will be seen after the QRS complex. AUG 2014
this tracing abnormality? atrial fibrillation leads to no discernible p waves with an MED
A. atrial fibrillation irregularly irregular pattern of QRS complex. Supranodal BOARDS;
B. supra AV nodal Premature contraction premature contraction will have inverted p waves prior TOPNOTCH
C. premature contraction coming from the his to QRS. Know how to read ECG and the basis of each MD), MD
bundle deflections. guyton 11th ed p 150-153
D. atrial premature contraction
E. all of the above

TOPNOTCH MEDICAL BOARD PREP PHYSIOLOGY SUPEREXAM Page 38 of 88


For inquiries visit www.topnotchboardprep.com.ph or email us at topnotchmedicalboardprep@gmail.com
TOPNOTCH MEDICAL BOARD PREP PHYSIOLOGY SUPEREXAM
For inquiries visit www.topnotchboardprep.com.ph or email us at topnotchmedicalboardprep@gmail.com
Item QUESTION EXPLANATION AUTHOR TOPNOTCH
# EXAM
305 increasing the reynolds number or the turbulence in turbulence or shear stress is one of the potent stimulus LEAN BACK-UP
an arteriole will result in the following consequence? for release of Nitric oxide ( formerly known as EDRF) ANGELO MIDTERM
A. Induction of Nitric oxide synthase this is made through upregulation of NOS and increase SILVERIO, EXAM - FEB
B. Decrease prostacylin release levels of cGMP. Guyton 11th ed p 199-200. MD (TOP 4 - 2015
C. Increase renin and angiotensin levels AUG 2014
D. Decrease cGMP MED
E. none of the above BOARDS;
TOPNOTCH
MD), MD
306 the main determinant of blood flow in a non the local metabolic needs of skeletal muscle, cardiac and LEAN BACK-UP
exercising skeletal muscle is? nervous system is the main determinant of its blood flow ANGELO MIDTERM
A. Arginine vasopressin level Guyton 11th ed p 195-196 SILVERIO, EXAM - FEB
B. Angiotensin II MD (TOP 4 - 2015
C. Local Metabolic rate AUG 2014
D. Autonomic nervous system MED
E. adenosine BOARDS;
TOPNOTCH
MD), MD
307 in which level in the vascular system wherein there arterioles are the primary determinant of arterial LEAN BACK-UP
is largest dissipation of arterial pressure during pressure gradient (particulary the arteriole -capillary ANGELO MIDTERM
systole? junction). Guyton 11th ed p 162-163 SILVERIO, EXAM - FEB
A. Aorta MD (TOP 4 - 2015
B. Capillary venule junction AUG 2014
C. arterioles MED
D. Capillary level BOARDS;
E. Medium size capillaries TOPNOTCH
MD), MD
308 which of the following scenario will lead to increasing the efferent arteriole resistance will LEAN BACK-UP
increased peritubular capillary fluid reabsorption? consequently lead to decrease peritubular hydrostatic ANGELO MIDTERM
A. Increase systolic blood pressure pressure resulting to increase in net fluid reabsorption. SILVERIO, EXAM - FEB
B. Increase renal blood flow Use of ACEI will decrease the efferent arteriole MD (TOP 4 - 2015
C. Decrease filtration fraction constriction. Increase in BP will lead to increase in the AUG 2014
D. Intake of enalapril glomerular pressure and peritubular capillary pressure. MED
E. Increase efferent arteriolar constriction decreased filtration fraction increases the peritubular BOARDS;
hydrostatic pressure leading to decrease reabsorption. TOPNOTCH
guyton 11th ed 339-340 MD), MD
309 During Diabetic ketoacidosis, hydrogen ions actively part of the compensatory response of the kidney during LEAN BACK-UP
secreted by the proximal tubule are associated with acidosis is to remove hydrogen ions and consequently ANGELO MIDTERM
which physiologic process? reabsorb bicarbonate ions in the proxima tubule. ( SILVERIO, EXAM - FEB
A. Excretion of ammonium ions SIMILAR TO PREVIOUS BOARD EXAM MD (TOP 4 - 2015
B. Reabsorption of bicarbonate CONCEPT/PRINCIPLE) guyton 11th ed page 306-307 AUG 2014
C. Reabsorption of potassium ions MED
D. reabsorption of phosphate ions BOARDS;
E. all of the above TOPNOTCH
MD), MD
310 which of the following scenario will promote urinary calcium reabsorption is directly proportional to water LEAN BACK-UP
calcium excretion? and sodium reabsorption. Therefore, increase in ANGELO MIDTERM
A. Metabolic acidosis intravascular volume will lead to a decrease in Na,Ca and SILVERIO, EXAM - FEB
B. Increase plasma phosphate levels H20 reabsorption. Guyton 11th ed p 372 MD (TOP 4 - 2015
C. Intravascular Volume expansion AUG 2014
D. Squamous cell carcinoma of the lungs MED
E. None of the above BOARDS;
TOPNOTCH
MD), MD
311 deficiency in FXIII in the serum will lead to which of polymerization of fibrin to form a clot requires the LEAN BACK-UP
the following ? presence of fibrin stabilizing factor (FXIII). The latter is ANGELO MIDTERM
A. Prolonged PTT not required in the intrinsic and extrinsic pathway. SILVERIO, EXAM - FEB
B. Prolonged PT Guyton 11th ed 467-468. MD (TOP 4 - 2015
C. Increase clotting time AUG 2014
D. Decrease clot stability MED
E. None of the above BOARDS;
TOPNOTCH
MD), MD
312 The increase in the alveolar ventilation during during prolonged aerobic exercise, the changes in the LEAN BACK-UP
strenous aerobic exercise such as marathon is mean arterial PO2, PCO2 and pH are negligible. Venous ANGELO MIDTERM
primarily due to which of the following changes? system does not contain any pH or O2 sensors that could SILVERIO, EXAM - FEB
A. cortical collateral impulses to the DRG (dorsal might affect the respiratory drive. Guyton 11th ed page MD (TOP 4 - 2015
respiratory group) 519-520 AUG 2014
B. Decrease in arterial pH MED
C. Decrease in arterial PO2 BOARDS;
D. Decrease in venous PO2 TOPNOTCH
E. All of the above MD), MD
313 Which of the following regions in the brain uses A- dopaminergic neurons, C- enkephalin producing LEAN BACK-UP
serotonin for suppression of pain pathways? neurons, D- olivary nucleus nonspecific, related to ANGELO MIDTERM
A. Ventral tegmental area of tsai auditory system. SILVERIO, EXAM - FEB
B. Nucleus raphes magnus MD (TOP 4 - 2015
C. Periaqueductal gray region AUG 2014
D. Inferior olivary nucleus MED
E. None of the above BOARDS;
TOPNOTCH
MD), MD

TOPNOTCH MEDICAL BOARD PREP PHYSIOLOGY SUPEREXAM Page 39 of 88


For inquiries visit www.topnotchboardprep.com.ph or email us at topnotchmedicalboardprep@gmail.com
TOPNOTCH MEDICAL BOARD PREP PHYSIOLOGY SUPEREXAM
For inquiries visit www.topnotchboardprep.com.ph or email us at topnotchmedicalboardprep@gmail.com
Item QUESTION EXPLANATION AUTHOR TOPNOTCH
# EXAM
314 which is a correct pair of neurotransmitters used in refer to Guyton 11th ed p 600-601 LEAN BACK-UP
fastpain and slow pain respectively at the dorsal ANGELO MIDTERM
horn synapse ? SILVERIO, EXAM - FEB
A. Glutamate/ substance P MD (TOP 4 - 2015
B. Acetycholine/substance P AUG 2014
C. Acetylcholine/cGRP MED
D. Serotonin/cGRP BOARDS;
E. None of the above TOPNOTCH
MD), MD
315 Which of the followng occurs in rods/cones during during phototransduction of light, there is an increase in LEAN BACK-UP
phototransduction of light ? the hydrolysis of cGMP leading to closure of Na channel. ANGELO MIDTERM
A. Decrease phoshodiesterase activity Failure of the Na to enter the rods leads to SILVERIO, EXAM - FEB
B. Increase cGMP hydrolysis hyperpolarization. Guyton 11th ed page 630-631 MD (TOP 4 - 2015
C. Decrease levels of transducin AUG 2014
D. Increase fraction of active voltage gated calcium MED
channels BOARDS;
E. increase fraction of open voltage gated sodium TOPNOTCH
channels MD), MD
316 which of the following is not part of limbic cortex? the components of limbic cortex are the following: LEAN BACK-UP
A. Orbitofrontal gyrus orbitofrontal gyrus, cingulate gyrus, parahippocampal ANGELO MIDTERM
B. Parahippocampal gyrus gyrus, and subcallosal gyrus SILVERIO, EXAM - FEB
C. Paracentral lobule MD (TOP 4 - 2015
D. Cingulate gyrus AUG 2014
E. None of the above MED
BOARDS;
TOPNOTCH
MD), MD
317 damage to which of the following brain region leads Guyton 11th ed p 726 LEAN BACK-UP
to retrograde amnesia ? ANGELO MIDTERM
A. amygdala SILVERIO, EXAM - FEB
B. Dentate gyrus MD (TOP 4 - 2015
C. Hippocampus AUG 2014
D. Subcallosal gyrus MED
E. Thalamus BOARDS;
TOPNOTCH
MD), MD
318 Which of the following ions will increase in during maximal salivary secretion, the rapid transit of LEAN BACK-UP
concentration when salivary secretion is maximally saliva in the duct does not allow sufficient time for Na to ANGELO MIDTERM
stimulated? be reabsorbed. Guyton 11th ed p 794 SILVERIO, EXAM - FEB
A. sodium MD (TOP 4 - 2015
B. potassium AUG 2014
C. bicarbonate MED
D. chloride BOARDS;
E. Ionic levels are constant regardless of secretion TOPNOTCH
rate MD), MD
319 How are triglycerides transported from intstinal cell TAG upon reconstitution inside the intestinal epithelial LEAN BACK-UP
to lacteals? cells will be transported to lacteals as a chylomicron. It ANGELO MIDTERM
A. Micelles will then travel to lymphatic system towards the SILVERIO, EXAM - FEB
B. FFA thoracic duct entering the systemic circulation via the MD (TOP 4 - 2015
C. Chylomicrons subclavian vein. Guyton 11th ed 815-816 AUG 2014
D. LDL MED
E. VLDL BOARDS;
TOPNOTCH
MD), MD
320 feeling of satiety after eating a high fat meal is promotes sateity- increase levels of cholecystokinin, LEAN BACK-UP
secondary to which of the following changes? peptide YY and leptin. Promotes appetite - increase ANGELO MIDTERM
A. Increased cholecystokinin ghrelin and neuropeptide Y. Guyton11th ed p 868-870. SILVERIO, EXAM - FEB
B. Increased ghrelin MD (TOP 4 - 2015
C. Decreased leptin AUG 2014
D. Decreased peptide YY MED
E. A and C BOARDS;
TOPNOTCH
MD), MD
321 This statement describes the mean arterial pressure: MAP = 1/3(S+2D). MAP is determined 40% by systolic DEBBIE BACK-UP
A. one-third of the sum of diastolic and systolic and 60% by diastolic pressure. Guyton Medical ROSE MIDTERM
pressures Physiology 11th edition p.176 TANENGSY, EXAM - FEB
B. average of systolic and diastolic pressures MD (TOP 5 - 2015
C. determined about 40% by diastolic and 60% by AUG 2014
systolic pressure MED
D. average of arterial pressures measured BOARDS;
millisecond by millisecond over a period of time TOPNOTCH
MD)
322 The following are consistent with acute control of Collateral circulation development is a phenomenon of DEBBIE BACK-UP
local blood flow, except: long-term (not acute) local blood flow regulation. Guyton ROSE MIDTERM
A. adenosine accumulation affecting blood flow Medical Physiology 11th edition pp.198-200 TANENGSY, EXAM - FEB
regulation MD (TOP 5 - 2015
B. effect of an increase in concentration of CO2 and AUG 2014
H+ on cerebral vessels MED
C. stretch-induced vascular depolarization, causing BOARDS;
vasoconstriction TOPNOTCH
D. tubuloglomerular feedback MD)
E. development of collateral circulation

TOPNOTCH MEDICAL BOARD PREP PHYSIOLOGY SUPEREXAM Page 40 of 88


For inquiries visit www.topnotchboardprep.com.ph or email us at topnotchmedicalboardprep@gmail.com
TOPNOTCH MEDICAL BOARD PREP PHYSIOLOGY SUPEREXAM
For inquiries visit www.topnotchboardprep.com.ph or email us at topnotchmedicalboardprep@gmail.com
Item QUESTION EXPLANATION AUTHOR TOPNOTCH
# EXAM
323 True regarding the baroreceptor reflex: Signals from the carotid baroreceptors are transmitted DEBBIE BACK-UP
A. Signals from carotid baroreceptors are trasmitted through the Hering's nerves to glossopharyngeal nerves. ROSE MIDTERM
through the vagus nerves. Signals from both carotic and aortic baroreceptors are TANENGSY, EXAM - FEB
B. Signals from aortic baroreceptors are transmitted transmitted to the tractus solitarius of the medulla. The MD (TOP 5 - 2015
to the tractus solitarius of the medulla. carotid baroreceptors are not stimulated at all by AUG 2014
C. The carotic baroreceptors are stimulated by pressures 0-50/60, as they respond progressively more MED
pressures 0-50/60 mmHg. rapidly above these levels and reach a max of 180 BOARDS;
D. The baroreceptors respond much more to a mmHg. The baroreceptors respond much more to a TOPNOTCH
stationary pressure. rapidly changing pressure. When signals have reached MD)
E. When signals have reached the brainstem, the brainstem, secondary signals inhibit the
secondary signals stimulate the vasoconstrictor vasocontrictor center. Guyton Medical Physiology 11th
center. edition pp.209-210
324 What causes the 4th heart sound? B refers to S1. C refers to S2. Guyton Medical Physiology DEBBIE BACK-UP
A. inrush of blood into ventricles after atrial 11th edition p.270 ROSE MIDTERM
contraction TANENGSY, EXAM - FEB
B. closure of the AV valves MD (TOP 5 - 2015
C. closure of the semilunar valves AUG 2014
D. inrush of blood into the great valves after MED
ventricular contraction BOARDS;
E. inrush of blood through stenosed valves TOPNOTCH
MD)
325 True of heart sounds: S2 has a higher frequency than S1, due to the more taut DEBBIE BACK-UP
A. S2 is slightly longer in duration than S1 semilunar valves, and the great elastic coefficient of taut ROSE MIDTERM
B. S2 has a lower frequency than S1 arterial walls. S2 is slightly longer in duration than S1. TANENGSY, EXAM - FEB
C. S1 is caused by closure of the aortic valve Guyton Medical Physiology 11th edition p.270 MD (TOP 5 - 2015
D. S2 is caused by oscillation of blood between walls AUG 2014
of ventricles MED
E. AV valves are more taut than semilunar valves. BOARDS;
TOPNOTCH
MD)
326 Which valve is the second most seriously damaged Mitral valve is the most seriously damaged valve in DEBBIE BACK-UP
in rheumatic fever? rheumatic fever, followed by aortic valve. Guyton ROSE MIDTERM
A. mitral Medical Physiology 11th edition p.271 TANENGSY, EXAM - FEB
B. tricuspid MD (TOP 5 - 2015
C. aortic AUG 2014
D. pulmonic MED
BOARDS;
TOPNOTCH
MD)
327 Not true regarding the glomerular capillary Water, sodium, glucose, and inulin are examples of DEBBIE BACK-UP
membrane? substances with a filterability of 1. Guyton Medical ROSE MIDTERM
A. It is composed of 3 major layers. Physiology 11th edition p.316-317 TANENGSY, EXAM - FEB
B. Albumin is an example of a substance with a MD (TOP 5 - 2015
filterability of 1. AUG 2014
C. The capillary endothelium is endowed with MED
fenestrae. BOARDS;
D. It is much thicker than other capillaries'. TOPNOTCH
E. Both basement membrane and podocytes have MD)
negative charges.
328 Raises GFR, except: Guyton Medical Physiology 11th edition pp.318-320 DEBBIE BACK-UP
A. modest efferent constriction ROSE MIDTERM
B. increased glomerular capillary filtration TANENGSY, EXAM - FEB
coefficient MD (TOP 5 - 2015
C. increased arterial pressure AUG 2014
D. decreased resistance of the afferent arterioles MED
E. none of the above BOARDS;
TOPNOTCH
MD)
329 True of intrinsic pathway for initiating clotting: A & B refer to the extrinsic pathway for initiating DEBBIE BACK-UP
A. begins with a traumatized vascular wall clotting. Intrinsic pathway may also be initiated by the ROSE MIDTERM
B. begins with extravascular tissues that come into exposure of blood to collagen from traumatized vessel TANENGSY, EXAM - FEB
contact with blood wall. Guyton Medical Physiology 11th edition p.462 MD (TOP 5 - 2015
C. begins with trauma to the blood AUG 2014
D. B & C MED
E. all of the above BOARDS;
TOPNOTCH
MD)
330 Not true of the sneeze reflex: Afferent impulses in the sneeze reflex pass through CNV. DEBBIE BACK-UP
A. afferent impulses pass through CNX Guyton Medical Physiology 11th edition p.480 ROSE MIDTERM
B. initiating stimulus is irritation in nasal TANENGSY, EXAM - FEB
passageways MD (TOP 5 - 2015
C. uvula is depressed, as opposed to the cough reflex AUG 2014
D. none of the above MED
E. all of the above BOARDS;
TOPNOTCH
MD)
331 True regarding the Haldane effect: The other statements are regarding Bohr's effect. Guyton DEBBIE BACK-UP
A. quantitatively more important in promoting CO2 Medical Physiology 11th edition p.511 ROSE MIDTERM
transport than is Bohr's effect in promoting O2 TANENGSY, EXAM - FEB
transport MD (TOP 5 - 2015
B. increased delivery of oxygen to tissues occurs AUG 2014
when CO2 and H+ shifts the curve MED
C. CO2 diffuses to the lungs, decreasing the blood BOARDS;
PCO2 and H+ concentration, shifting the curve to the TOPNOTCH
left MD)
D. CO2 in blood increases H2CO3, shifting the curve
TOPNOTCH MEDICAL BOARD PREP PHYSIOLOGY SUPEREXAM Page 41 of 88
For inquiries visit www.topnotchboardprep.com.ph or email us at topnotchmedicalboardprep@gmail.com
TOPNOTCH MEDICAL BOARD PREP PHYSIOLOGY SUPEREXAM
For inquiries visit www.topnotchboardprep.com.ph or email us at topnotchmedicalboardprep@gmail.com
Item QUESTION EXPLANATION AUTHOR TOPNOTCH
# EXAM
to the right
E. When CO2 binds with hemoglobin, O2 is released.

332 True of the ventral respiratory group of neurons: It is the dorsal respiratory group of neurons that DEBBIE BACK-UP
A. participate in the basic rhythmical oscillation that participate in the basic rhythmical oscillation that ROSE MIDTERM
controls respiration controls respiration. The ventral respiratory group is TANENGSY, EXAM - FEB
B. functions in both inspiration and expiration inactive during quiet respiration. Guyton Medical MD (TOP 5 - 2015
C. active during normal quiet respiration Physiology 11th edition p.515 AUG 2014
D. A & B MED
E. B & C BOARDS;
TOPNOTCH
MD)
333 The phase that contributes the most to gastric There is no esophageal phase in gastric secretion. DEBBIE BACK-UP
secretion: Cephalic phase contributes 20%, while the gastric phase ROSE MIDTERM
A. cephalic is responsible for 70% of total gastric secretion. Guyton TANENGSY, EXAM - FEB
B. gastric Medical Physiology 11th edition p.798 MD (TOP 5 - 2015
C. esophageal AUG 2014
D. intestinal MED
BOARDS;
TOPNOTCH
MD)
334 Primary bile acids are combined with what to form Bile acids are conjugated with glycine and taurine. DEBBIE BACK-UP
conjugated bile acids? Guyton Medical Physiology 11th edition p.804 ROSE MIDTERM
A. glutamate TANENGSY, EXAM - FEB
B. glutamine MD (TOP 5 - 2015
C. glycine AUG 2014
D. threonine MED
BOARDS;
TOPNOTCH
MD)
335 The most potent stimulus for causing gallbladder Guyton Medical Physiology 11th edition p.803 DEBBIE BACK-UP
contractions is: ROSE MIDTERM
A. cholecystokinin TANENGSY, EXAM - FEB
B. secretin MD (TOP 5 - 2015
C. gastrin AUG 2014
D. acetylcholine MED
E. histamine BOARDS;
TOPNOTCH
MD)
336 Source/s of flatus: Guyton Medical Physiology 11th edition p.825 DEBBIE BACK-UP
A. swallowed air ROSE MIDTERM
B. as a result of bacterial action in gut TANENGSY, EXAM - FEB
C. diffusion from the blood into the GIT MD (TOP 5 - 2015
D. A & B AUG 2014
E. all of the above MED
BOARDS;
TOPNOTCH
MD)
337 The most numerous cells of the islets of langerhans? DEBBIE BACK-UP
A. alpha cells ROSE MIDTERM
B. beta cells TANENGSY, EXAM - FEB
C. gamma cells MD (TOP 5 - 2015
D. pp cells AUG 2014
MED
BOARDS;
TOPNOTCH
MD)
338 The following is/are factor/s that increase insulin Guyton Medical Physiology 11th edition p.968 DEBBIE BACK-UP
secretion: ROSE MIDTERM
A. leptin TANENGSY, EXAM - FEB
B. fasting MD (TOP 5 - 2015
C. somatostatin AUG 2014
D. obesity MED
E. alpha-adrenergic activity BOARDS;
TOPNOTCH
MD)
339 True of the prostate gland: The seminal vesicle secretes fructose, and contributes DEBBIE BACK-UP
A. secretes fructose the bulk of the semen. Prostate gland secretion is thin ROSE MIDTERM
B. contributes the bulk of the semen and milky. TANENGSY, EXAM - FEB
C. secretes a slightly alkaline fluid MD (TOP 5 - 2015
D. secretion is mucoid AUG 2014
MED
BOARDS;
TOPNOTCH
MD)

TOPNOTCH MEDICAL BOARD PREP PHYSIOLOGY SUPEREXAM Page 42 of 88


For inquiries visit www.topnotchboardprep.com.ph or email us at topnotchmedicalboardprep@gmail.com
TOPNOTCH MEDICAL BOARD PREP PHYSIOLOGY SUPEREXAM
For inquiries visit www.topnotchboardprep.com.ph or email us at topnotchmedicalboardprep@gmail.com
Item QUESTION EXPLANATION AUTHOR TOPNOTCH
# EXAM
340 The only really effective mechanism to prevent body Guyton Medical Physiology 11th edition p.895 DEBBIE BACK-UP
heat control breakdown in severely cold ROSE MIDTERM
environments is: TANENGSY, EXAM - FEB
A. panting MD (TOP 5 - 2015
B. neuronal effector mechanisms AUG 2014
C. behavioral control of body temperature MED
D. sweating BOARDS;
E. piloerection TOPNOTCH
MD)
341 At resting membrane potential, the sodium channel The m gate is also known as the activation gate, and the JESSICA MAE BACK-UP
must have this configuration: h gate is the inactivation gate. In the resting state, the SANCHEZ, MIDTERM
A. m gate open and h gate closed activation (m) gate is closed, and the inactivation gate MD (TOP 4 - EXAM - FEB
B. m gate open and h gate open (h) is open. AUG 2014 2015
C. m gate closed and h gate open MED
D. m gate closed and h gate closed BOARDS;
TOPNOTCH
MD)
342 Binding of epinephrine to a beta adrenergic receptor Epinephrine is both a first messenger and a ligand. JESSICA MAE BACK-UP
leads to generation of cAMP from ATP in the ATP is not the effector protein, but the substrate from SANCHEZ, MIDTERM
presence of adenylyl cyclase. One of the following which cAMP is produced. MD (TOP 4 - EXAM - FEB
statements is correct: Adenylyl cyclase is considered an effector protein. AUG 2014 2015
A. Epinephrine is the first messenger but is not MED
considered as a ligand BOARDS;
B. ATP is the effector protein TOPNOTCH
C. Adenylyl cyclase is an enzyme, hence, can’t be MD)
considered as an effector protein
D. cAMP serves as a chemical relay from the plasma
membrane to the cell’s biochemical machinery
343 A 64 year old male was diagnosed to have heart Inhibition of the Na-K ATPase increases intracellular Na, JESSICA MAE BACK-UP
failure. To increase the heart’s force of contraction, thereby decreasing the action of the Na-Ca exchange SANCHEZ, MIDTERM
you should give a medication that would: which serves to pump Ca out in exchange for Na. This MD (TOP 4 - EXAM - FEB
A. Inhibit the sodium ATPase pump and thereby results in increased intracellular Ca, thereby increasing AUG 2014 2015
inhibiting the sodium-calcium pump contractility. This is the mechanism of action of cardiac MED
B. Lower ECF calcium glycosides given for heart failure. BOARDS;
C. Inhibit both dihydropiridine and ryanodine TOPNOTCH
receptors MD)
D. Stimulate the active calcium pump
344 In smooth muscle contraction, calcium initially binds JESSICA MAE BACK-UP
with: A. Troponin C SANCHEZ, MIDTERM
B. Ryanodine receptor MD (TOP 4 - EXAM - FEB
C. Calmodulin AUG 2014 2015
D. Dihydropiridine receptor MED
BOARDS;
TOPNOTCH
MD)
345 Drugs that increase acetylcholine transmission, such JESSICA MAE BACK-UP
as anti-acetylcholinesterases, promote sympathetic SANCHEZ, MIDTERM
as well as parasympathetic responses because: MD (TOP 4 - EXAM - FEB
A. Ach receptors are located in all effector organs AUG 2014 2015
that receive autonomic innervation MED
B. Ach is the transmitter at all sympathetic and BOARDS;
parasympathetic postganglionic synapses TOPNOTCH
C. Ach is the transmitter at all autonomic ganglionic MD)
synapses
D. Ach is the sole neurotransmitter in the peripheral
nervous system
346 After appendectomy, a patient was unable to void. JESSICA MAE BACK-UP
To relieve his urinary retention, the surgeon should SANCHEZ, MIDTERM
give: MD (TOP 4 - EXAM - FEB
A. Alpha-1 agonist to contract the urinary sphincter AUG 2014 2015
B. M3 antagonist to relax the bladder wall and MED
contract the urinary sphincter BOARDS;
C. Beta-1 agonist to relax the detrusor muscle TOPNOTCH
D. M3 agonist to contract the bladder wall and relax MD)
the urinary sphincter
347 An increase in one of the following parameters will Increasing all of the other choices would result in JESSICA MAE BACK-UP
result to a decrease in stroke volume: increased stroke volume. SANCHEZ, MIDTERM
A. Preload MD (TOP 4 - EXAM - FEB
B. Afterload AUG 2014 2015
C. Contractility MED
D. Venous return BOARDS;
TOPNOTCH
MD)
348 Jugular venous pulsation in a patient with tricuspid a wave - corresponds to right atrial contraction JESSICA MAE BACK-UP
regurgitation will have an increase in: c wave - corresponds to right ventricular contraction, SANCHEZ, MIDTERM
A. a wave causing the closed tricuspid valve to bulge into the right MD (TOP 4 - EXAM - FEB
B. c wave atrium AUG 2014 2015
C. v wave v wave - corresponds to the filling of the right atrium MED
D. x wave during late systole or early diastole during which the BOARDS;
tricuspid valve should still be closed. Tricuspid TOPNOTCH
insufficiency would cause increase in v waves, or in more MD)
severe cases, a c-v wave.

TOPNOTCH MEDICAL BOARD PREP PHYSIOLOGY SUPEREXAM Page 43 of 88


For inquiries visit www.topnotchboardprep.com.ph or email us at topnotchmedicalboardprep@gmail.com
TOPNOTCH MEDICAL BOARD PREP PHYSIOLOGY SUPEREXAM
For inquiries visit www.topnotchboardprep.com.ph or email us at topnotchmedicalboardprep@gmail.com
Item QUESTION EXPLANATION AUTHOR TOPNOTCH
# EXAM
349 During inspiration, this change is expected in right- During inspiration, the fall in intrathoracic pressure JESSICA MAE BACK-UP
sided cardiac events: causes an increase in venous return or preload. SANCHEZ, MIDTERM
A. Increase in preload MD (TOP 4 - EXAM - FEB
B. Decrease in afterload AUG 2014 2015
C. Increase in stroke volume MED
D. Decrease in contractility BOARDS;
TOPNOTCH
MD)
350 During exercise, one of the following occurs: Sympathetic innervation is the primary regulator of JESSICA MAE BACK-UP
A. Sympathetic stimulation will decrease arteriolar blood flow to the skeletal muscle at rest. However, SANCHEZ, MIDTERM
constriction during exercise, local metabolic mechanisms override MD (TOP 4 - EXAM - FEB
B. Afterload increases sympathetic influence, causing local vasodilation to AUG 2014 2015
C. Total peripheral resistance decreases increase the supply of O2 to the active muscle. The MED
D. Posterior pituitary releases vasopressin vasodilation in the skeletal muscles contributes to the BOARDS;
decrease in total peripheral resistance. TOPNOTCH
MD)
Reference: BRS Physiology, 5th ed. p. 95
351 The binding of oxygen with hemoglobin tends to Haldane effect refers to the property of hemoglobin JESSICA MAE BACK-UP
displace carbon dioxide in the lungs. This is known wherein deoxyhemoglobin freely takes up and SANCHEZ, MIDTERM
as the: transports CO2, whereas oxygenated hemoglobin has MD (TOP 4 - EXAM - FEB
A. Dalton’s law reduced affinity for CO2. AUG 2014 2015
B. Bohr effect The Bohr effect, on the other hand, states that an MED
C. Haldane effect increase in blood CO2, which in turn decreases blood pH, BOARDS;
D. Charles law promotes unloading of oxygen or decreased affinity of TOPNOTCH
oxygen for hemoglobin. MD)
352 A diuretic administered to a patient is an The distal tubule has 2 types of cell: the principal and the JESSICA MAE BACK-UP
aldosterone antagonist that blocks the activity of Na intercalated cell. SANCHEZ, MIDTERM
channels found in which cells of the distal tubule and The principal cell is responsible for reabsorbing Na and MD (TOP 4 - EXAM - FEB
cortical collecting tubule? secreting K, and is the site of action of the hormone AUG 2014 2015
A. Intercalated aldosterone. MED
B. Principal The intercalated cell functions in acid-base homeostasis BOARDS;
C. Mesangial and has a H-K ATPase. TOPNOTCH
D. Juxtaglomerular MD)
353 A 65 year old hypertensive patient was noted to JESSICA MAE BACK-UP
have dyspnea, crackles on both lung fields and SANCHEZ, MIDTERM
bipedal edema. The edema is due to: MD (TOP 4 - EXAM - FEB
A. Decreased interstitial colloid oncotic pressure AUG 2014 2015
B. Increased capillary hydrostatic pressure MED
C. Increased capillary colloid oncotic pressure BOARDS;
D. Unchanged intersitial hydrostatic pressure TOPNOTCH
MD)
354 The kidneys regulate acid-base balance by each of The kidneys do not reabsorb H+ in cases of alkalosis, JESSICA MAE BACK-UP
the following mechanisms except: rather, they increase the excretion of HCO3 by SANCHEZ, MIDTERM
A. Reabsorbs filtered HCO3 decreasing its reabsorption along the nephron. MD (TOP 4 - EXAM - FEB
B. Active reabsorption of H+ AUG 2014 2015
C. Excreting titratable acid Reference: Berne, Levy. Physiology, 5th ed. p. 713 MED
D. Acidifying the urine via H+ secretion BOARDS;
TOPNOTCH
MD)
355 The rate of aldosterone secretion would decrease in JESSICA MAE BACK-UP
response to a: SANCHEZ, MIDTERM
A. Fall in renin secretion by the kidney MD (TOP 4 - EXAM - FEB
B. Decrease in tubule fluid sodium concentration at AUG 2014 2015
the macula densa MED
C. Fall in blood pressure BOARDS;
D. Rise in serum potassium TOPNOTCH
MD)
356 A 50 year old male underwent craniotomy. At the JESSICA MAE BACK-UP
recovery room, he was noted to have voluminous SANCHEZ, MIDTERM
urine. You assume that he has a deficiency in: MD (TOP 4 - EXAM - FEB
A. Oxytocin AUG 2014 2015
B. ACTH MED
C. TSH BOARDS;
D. ADH TOPNOTCH
MD)
357 The most important factor in influencing the closure Reference: Berne, Levy. Physiology, 5th ed. p. 431 JESSICA MAE BACK-UP
of the ductus arteriosus is: SANCHEZ, MIDTERM
A. Decrease in adenosine levels MD (TOP 4 - EXAM - FEB
B. Increase in O2 tension AUG 2014 2015
C. Increase in plasma catecholamine levels MED
D. Decrease in prostaglandin levels BOARDS;
TOPNOTCH
MD)
358 An athlete’s muscle is composed predominantly of Type I, or slow twitch fibers, have the capacity for JESSICA MAE BACK-UP
type 1 motor fibers. In which event will he most oxidative phosphorylation, fatigue more slowly, and are SANCHEZ, MIDTERM
likely be a winner? therefore suited for sustained activities and exercises MD (TOP 4 - EXAM - FEB
A. 100 meter race requiring endurance. AUG 2014 2015
B. Weight lifting MED
C. Boxing BOARDS;
D. Marathon TOPNOTCH
MD)

TOPNOTCH MEDICAL BOARD PREP PHYSIOLOGY SUPEREXAM Page 44 of 88


For inquiries visit www.topnotchboardprep.com.ph or email us at topnotchmedicalboardprep@gmail.com
TOPNOTCH MEDICAL BOARD PREP PHYSIOLOGY SUPEREXAM
For inquiries visit www.topnotchboardprep.com.ph or email us at topnotchmedicalboardprep@gmail.com
Item QUESTION EXPLANATION AUTHOR TOPNOTCH
# EXAM
359 The major urinary buffer excreted during metabolic JESSICA MAE BACK-UP
acidosis is: SANCHEZ, MIDTERM
A. Phosphate MD (TOP 4 - EXAM - FEB
B. Ammonia AUG 2014 2015
C. Bicarbonate MED
D. Protein BOARDS;
TOPNOTCH
MD)
360 True of the third heart sound (S3): Rapid flow of blood from the atria to the ventricles JESSICA MAE BACK-UP
A. Appreciated during early diastole during early diastole causes the third heart sound. SANCHEZ, MIDTERM
B. Corresponds to the dicrotic notch of the arterial MD (TOP 4 - EXAM - FEB
waveform Review: Cardiac cycle diagram AUG 2014 2015
C. Due to the closure of the mitral valve MED
D. Coincides with the P wave of the ECG BOARDS;
TOPNOTCH
MD)
361 Which of the following is TRUE regarding CSF is produced at a rate of 0.35mL/min or 21ml/hr. CSF MAIRRE BACK-UP
cerebrospinal fluid (CSF)? has a lower concentration of K+, glucose, and protein but JAMES MIDTERM
A. CSF is produced at a rate of 21 mL/hr by the a greater concentration of Na+ and Cl− than blood does. GADDI, MD EXAM - FEB
choroid plexuses Berne 6th pg 62 (TOP 4 - AUG 2015
B. CSF has a lower concentration of potassium, 2013 MED
glucose and protein compared to blood BOARDS;
C. CSF is reabsorbed by the arachnoid granulations TOPNOTCH
into to the dural sinuses via bulk flow MD)
D. B and C
E. All of the above
362 Tetraethylammonium (TEA) is a poison found in the Berne 6th pg 71 A - tetradotoxin; C - ω-conotoxin; D - MAIRRE BACK-UP
ovaries of certain species of puffer fish. Raw puffer botulinum toxin JAMES MIDTERM
fish is a prized delicacy in Japan and consumption of GADDI, MD EXAM - FEB
improperly prepared puffer fish sushi can be fatal. (TOP 4 - AUG 2015
The reason behind this is that: 2013 MED
A. TEA blocks the sodium voltage gated channels BOARDS;
inhibiting action potential generation TOPNOTCH
B. TEA blocks the potassium voltage gated channels MD)
inhibiting action potential generation
C. TEA blocks the calcium voltage gated channels
inhibiting action potential generation
D. TEA blocks the acetylcholine release preventing
conduction through the NMJ
E. None of the above
363 A 55/M with severe crushing chest pain was rushed In left ventricular failure pulmonary capillary MAIRRE BACK-UP
to the ER. On PE, patient was tachycardic, tachypneic hydrostatic pressure may exceed plasma oncotic JAMES MIDTERM
with coarse crackles heard in all lung fields. ECG pressure. When this occurs, it may cause pulmonary GADDI, MD EXAM - FEB
was done which showed ST elevation in leads V1-V6. edema Berne 6th pg 351 (TOP 4 - AUG 2015
The crackles can be explained by which of the 2013 MED
following alteration/s in the Starling forces? BOARDS;
A. Increased capillary hydrostatic pressure TOPNOTCH
B. Increased plasma oncotic pressure MD)
C. Decreased plasma oncotic pressure
D. Decreased capillary hydrostatic pressure
E. A and C
364 Which of the following limits inspiration? Hering-Breuer reflex is an inspiratory inhibitory reflex MAIRRE BACK-UP
A. Activation of the Hering-Breuer reflex that arises from afferent stretch receptors located in the JAMES MIDTERM
B. Stretching of the smooth muscles of the airways smooth muscles of the airways. Stretching of the GADDI, MD EXAM - FEB
C. Trigerring of the pneumotaxic center which in receptors and results in early exhalation by stimulating (TOP 4 - AUG 2015
turn causes stimulation of the apneustic center the neurons associated with the off-switch phase of 2013 MED
D. A and B inspiratory muscle control. Berne 6th pg 470; BOARDS;
E. All of the above Stimulation of the pneumotaxic center INHIBITS the TOPNOTCH
apneustic center thereby limiting the time for inspiration MD)

365 Proopiomelanocortin is cleaved to produce: ACTH, MSH, γ -Lipotropin and β-endorphin Berne 6th pg MAIRRE BACK-UP
A. MSH 714 JAMES MIDTERM
B. ACTH GADDI, MD EXAM - FEB
C. β-endorphin (TOP 4 - AUG 2015
D. A and B 2013 MED
E. All of the above BOARDS;
TOPNOTCH
MD)
366 The major inhibitory signal for aldosterone release Major stimuli for aldosterone production are a rise in MAIRRE BACK-UP
is: angiotensin II and a rise in serum [K+]. The major JAMES MIDTERM
A. hyperkalemia inhibitory signal is ANP. Berne 6th pg 757 GADDI, MD EXAM - FEB
B. angiotensin II (TOP 4 - AUG 2015
C. hypertension 2013 MED
D. ANP BOARDS;
E. vasopressin TOPNOTCH
MD)
367 The following are TRUE regarding cross-bridge Berne 6th pg 242 MAIRRE BACK-UP
cycling EXCEPT? JAMES MIDTERM
A. The binding of ATP to the myosin head reduces its GADDI, MD EXAM - FEB
affinity to actin (TOP 4 - AUG 2015
B. Partial hydrolysis of ATP causes the displacement 2013 MED
of the myosin head to the positive end of actin BOARDS;
C. The binding of ADP to the myosin head causes an TOPNOTCH
increase in its affinity to actin MD)
D. A and C

TOPNOTCH MEDICAL BOARD PREP PHYSIOLOGY SUPEREXAM Page 45 of 88


For inquiries visit www.topnotchboardprep.com.ph or email us at topnotchmedicalboardprep@gmail.com
TOPNOTCH MEDICAL BOARD PREP PHYSIOLOGY SUPEREXAM
For inquiries visit www.topnotchboardprep.com.ph or email us at topnotchmedicalboardprep@gmail.com
Item QUESTION EXPLANATION AUTHOR TOPNOTCH
# EXAM
E. No exception

368 Testosterone exerts a multitude of effects in the All are effects of testosterone Berne 6th pg 764 MAIRRE BACK-UP
male. These include the following EXCEPT: JAMES MIDTERM
A. Increase in muscle mass and increased bone GADDI, MD EXAM - FEB
growth (TOP 4 - AUG 2015
B. Increased red cell production 2013 MED
C. Maintenance of libido and erectile function BOARDS;
D. Increase in VLDL and LDL and decrease in HDL TOPNOTCH
E. No exception MD)

369 Which of the following is the strongest agonist for Both gastrin and histamine strongly stimulate secretion MAIRRE BACK-UP
H+ secretion of acid by parietal cells Guyton 12th ed 778 but JAMES MIDTERM
A. Gastrin histamine is the strongest agonist of H+ secretion, GADDI, MD EXAM - FEB
B. Histamine whereas gastrin and acetylcholine are much weaker (TOP 4 - AUG 2015
C. Acetylcholine agonists Berne 6th pg 509 2013 MED
D. Secretin BOARDS;
E. A and B TOPNOTCH
MD)
370 Mucus protects the gastric epithelium from the Protection of the gastric epithelium depends on MAIRRE BACK-UP
effects of H+ and pepsins. Which of the following adequate levels of BOTH mucus and HCO3− Berne 6th JAMES MIDTERM
is/are TRUE? pg 511 GADDI, MD EXAM - FEB
A. Epithelial cells are kept at a pH of nearly neutral (TOP 4 - AUG 2015
through the effects of mucus 2013 MED
B. Due to the alkaline nature of mucus, the gastric BOARDS;
epithelium is well protected from acidity even if the TOPNOTCH
secretion of bicarbonate is reduced MD)
C. The mucus layer separates the acidic contents of
the gastric lumen from the bicarbonate rich
secretions of the surface epithelial cells
D. A and C
E. All of the above
371 The following are TRUE regarding G-proteins alpha subunit is the one bound to GTP Berne 6th pg 42- MAIRRE BACK-UP
EXCEPT? 43 JAMES MIDTERM
A. Activation of the receptor promotes a GADDI, MD EXAM - FEB
conformational change in G-protein and exchange of (TOP 4 - AUG 2015
GDP for GTP 2013 MED
B. The beta subunit along with the bound GTP BOARDS;
dissociates from the alpha and gamma subnits TOPNOTCH
C. The subunit associated with GTP goes on to MD)
activate various effector proteins including
adenylate cyclase, phosphodiesterases and
phospholipases
D. RGS proteins facilitate inactivation of signaling by
hydrolysis of GTP to GDP and Pi
E. No exception
372 When light strikes the pigment epithelium: When light is absorbed, the photoisomerization of MAIRRE BACK-UP
A. it causes a reduction in cGMP leading to the rhodopsin activates a G protein called transducin which JAMES MIDTERM
closure of sodium channels and subsequent in turn, activates cGMP phosphodiesterase. This results GADDI, MD EXAM - FEB
hyperpolarization in the, hydrolysis of cGMP to 5′-GMP, and lowers the (TOP 4 - AUG 2015
B. it causes an increase in cAMP leading to opening cGMP concentration in the rod cytoplasm. The reduction 2013 MED
of sodium channels and subsequent depolarization in cGMP leads to closure of the cGMP-gated Na+ BOARDS;
C. it causes an increase in DAG and PIP2 leading to channels, hyperpolarization of the photoreceptor TOPNOTCH
release of calcium and subsequent depolarization membrane, and a reduction in the release of transmitter. MD)
D. None of the above Berne 6th pg 128

373 During the cardiac action potential, the plateau is Phase 0: sodium influx, depolarization; Phase 1: Early MAIRRE BACK-UP
generated by: repolarization, potassium efflux; Phase 2: Plateau, JAMES MIDTERM
A. Influx of calcium into the myocytes calcium influx, continuing potassium efflux; Phase 3: GADDI, MD EXAM - FEB
B. Efflux of sodium out of the myocytes Final repolarization, potassium efflux > potassium influx, (TOP 4 - AUG 2015
C. Efflux of potassim out of the myocytes decreasing calcium influx; Phase 4: Restoration of ionic 2013 MED
D. Influx of sodium into the myocytes concentrations, potassium influx very SLIGHTLY exceed BOARDS;
E. A and C potassium efflux Berne 6th pg 294-299 TOPNOTCH
MD)
374 In the cardiac cycle, the second heart sound is heard S1 isovolumic contraction Berne 6th pg 322 MAIRRE BACK-UP
during: JAMES MIDTERM
A. Isovolumic relaxation GADDI, MD EXAM - FEB
B. Isovolumic contraction (TOP 4 - AUG 2015
C. Rapid ventricular ejection 2013 MED
D. Rapid ventricular filling BOARDS;
E. Atrial contraction TOPNOTCH
MD)

TOPNOTCH MEDICAL BOARD PREP PHYSIOLOGY SUPEREXAM Page 46 of 88


For inquiries visit www.topnotchboardprep.com.ph or email us at topnotchmedicalboardprep@gmail.com
TOPNOTCH MEDICAL BOARD PREP PHYSIOLOGY SUPEREXAM
For inquiries visit www.topnotchboardprep.com.ph or email us at topnotchmedicalboardprep@gmail.com
Item QUESTION EXPLANATION AUTHOR TOPNOTCH
# EXAM
375 During the embryonic period, blood is synthesized AGM/Yolk sac - 3rd week - 3rd month AOG; Liver (plus MAIRRE BACK-UP
by the spleen, LN) - 3rd month AOG - birth; Bone marrow - 4th JAMES MIDTERM
A. Yolk sac month AOG - postnatal life Topnotch Embryonic period GADDI, MD EXAM - FEB
B. Liver starts from the 3rd week after ovulation and fertilization (TOP 4 - AUG 2015
C. Bone marrow lasts 8 weeks Williams 23rd pg 79 or 24th pg 128 2013 MED
D. Lymph nodes BOARDS;
E. None of the above TOPNOTCH
MD)
376 Each hormone uses a specifc signal transduction Vasopressin/ADH V2 cAMP, ADH V1 IP3/DAG; MAIRRE BACK-UP
system to convey its message into the cell. Which Angiotensin II epithelial cells cAMP, Angiotensin II JAMES MIDTERM
hormone/s use/s BOTH the cAMP and the IP3/DAG vascular smooth muscles IP3/DAG; ACTH & CRH cAMP; GADDI, MD EXAM - FEB
pathway for signal transduction? thyroid hormone binds directly to nuclear receptors to (TOP 4 - AUG 2015
A. ACTH increase transcription of specific genes Guyton 12th ed 2013 MED
B. Thyroid hormone pg 889-891 BOARDS;
C. Vasopressin TOPNOTCH
D. CRH MD)
E. A and C

377 When UVB hits the basal layers of the skin, Vitamin D3/cholecalciferol is synthesized via the MAIRRE BACK-UP
conversion of _________ into Vitamin D3 occurs. conversion of 7-dehydrocholesterol by ultraviolet B light JAMES MIDTERM
A. cholesterol (UVB) in the more basal layers of the skin Berne 6th pg GADDI, MD EXAM - FEB
B. 7-dehydrocholesterol 699 (TOP 4 - AUG 2015
C. cholecalciferol 2013 MED
D. hydroxycholecalciferol BOARDS;
E. Vitamin D2 TOPNOTCH
MD)

378 Which of the following is/are TRUE regarding solute Na reabsorption does not occur in the descending thin MAIRRE BACK-UP
transport in the nephron? limb Berne 6th pg 585 Majority of calcium is reabsorbed JAMES MIDTERM
A. Glucose is absorbed in the PCT through SGLT-2 in the PCT via both transcellular and paracellular GADDI, MD EXAM - FEB
B. Sodium is actively transported in alll parts of the pathways pg 630 (TOP 4 - AUG 2015
renal tubule 2013 MED
C. Majority of calcium is reabsorbed in the DCT BOARDS;
transcellularly TOPNOTCH
D. All of the above MD)
379 What allows calcium to be stored at high Calsequestrin is a low-affinity calcium binding protein MAIRRE BACK-UP
concentrations inside the sarcoplasmic reticulum? inside the terminal cisternae. It allows calcium to be JAMES MIDTERM
A. DHPR stored at high concentration and favors a concentration GADDI, MD EXAM - FEB
B. Calsequestrin gradient that facilitates the efflux of calcium from the SR (TOP 4 - AUG 2015
C. RYR into the myoplasm; DHPR dihydropyridine receptor, 2013 MED
D. Troponin C voltage sensor; RYR ryanodine receptor, opening causes BOARDS;
E. None of the above calcium release from the SR Berne 6th pg 239 TOPNOTCH
MD)

380 Which of the following have the capacity to generate D cells: somatostatin; G cells: gastrin; ECL cells: MAIRRE BACK-UP
slow wave activity in the GI tract? histamine; Parietal cells: HCl; ICCs are “pacemaker” cells JAMES MIDTERM
A. D cells that have the capacity to generate the basic electrical GADDI, MD EXAM - FEB
B. G cells rhythm, or slow wave activity Berne 6th pg 511 (TOP 4 - AUG 2015
C. Enterochromaffin-like cells 2013 MED
D. Interstitial cells of Cajal BOARDS;
E. Parietal cells TOPNOTCH
MD)
381 Which layer of the cornea is responsible for Deturgescence refers to a relative state of dehydration, SCOTT BACK-UP
maintaining its deturgescence? which is needed to maintain the transparency of the RILEY ONG, MIDTERM
A. Epithelium cornea. MD (TOP 5 - EXAM - FEB
B. Bowman's layer AUG 2014 2015
C. Stroma MED
D. Descemet's membrane BOARDS;
E. Endothelium TOPNOTCH
MD)
382 A patient presented to you with findings of Diagnosis: Addison Disease. The skin hyperpigmentation SCOTT BACK-UP
hypotension, muscle weakness, weight loss, is due to increased production of melanocyte- RILEY ONG, MIDTERM
hyponatremia and hyperkalemia. On further stimulating hormone (MSH), which share the same MD (TOP 5 - EXAM - FEB
examination, you noted hyperpigmentation of the precursor molecule as ACTH. Aldosterone is deficient in AUG 2014 2015
skin, which included areas not exposed to the sun. this disease. Melatonin is secreted by the pineal gland MED
With your diagnosis in mind, this skin finding is and functions in the maintenance of the biological clock. BOARDS;
most likely due to increased secretion of which of TOPNOTCH
the following substances? MD)
A. Lipotropin
B. Melatonin
C. Cortisol
D. Aldosterone
E. None of the above
383 Cholesterol absorption occurs in which part of the Fats and cholesterol are released by bile salts and SCOTT BACK-UP
gastrointestinal tract? reabsorbed in the jejunum. Bile salts are then RILEY ONG, MIDTERM
A. Stomach reabsorbed in the ileum. Other substances that are MD (TOP 5 - EXAM - FEB
B. Duodenum absorbed in the ileum include vitamins A, D, E, K and AUG 2014 2015
C. Jejunum B12. MED
D. Ileum BOARDS;
E. Both C and D TOPNOTCH
MD)

TOPNOTCH MEDICAL BOARD PREP PHYSIOLOGY SUPEREXAM Page 47 of 88


For inquiries visit www.topnotchboardprep.com.ph or email us at topnotchmedicalboardprep@gmail.com
TOPNOTCH MEDICAL BOARD PREP PHYSIOLOGY SUPEREXAM
For inquiries visit www.topnotchboardprep.com.ph or email us at topnotchmedicalboardprep@gmail.com
Item QUESTION EXPLANATION AUTHOR TOPNOTCH
# EXAM
384 Which of the following is not an inspiratory muscle? SCOTT BACK-UP
A. External intercostal muscles RILEY ONG, MIDTERM
B. Innermost intercostal muscles MD (TOP 5 - EXAM - FEB
C. Sternocleidomastoid AUG 2014 2015
D. Serratus posterior inferior MED
E. Serratus anterior BOARDS;
TOPNOTCH
MD)
385 A marathon runner has been running for 15 minutes. First 8-10 seconds: phosphagen energy system. Next 1.3 SCOTT BACK-UP
At this point, which of the following sources of to 1.6 minutes: glycogen-lactic acid system. Thereafter: RILEY ONG, MIDTERM
energy is most likely sustaining his activity? aerobic system (initially glycogen stores; fats, ketones MD (TOP 5 - EXAM - FEB
A. Cell phosphocreatine and amino acids once glycogen stores are depleted) AUG 2014 2015
B. Anaerobic respiration MED
C. Glycogen BOARDS;
D. Ketone bodies TOPNOTCH
E. Amino acids being shunted to gluconeogenesis MD)

386 Which of the following is the most common cause of SIMILAR TO PREVIOUS BOARD EXAM SCOTT BACK-UP
female infertility? CONCEPT/PRINCIPLE. The most common cause of RILEY ONG, MIDTERM
A. Failure to ovulate female infertility is anovulation, and in which the most MD (TOP 5 - EXAM - FEB
B. Stress common cause is PCOS. AUG 2014 2015
C. Anorexia nervosa MED
D. Prolactinoma BOARDS;
E. Hyperthyroidism TOPNOTCH
MD)
387 Which of the following anterior pituitary hormones SIMILAR TO PREVIOUS BOARD EXAM SCOTT BACK-UP
is the most abundant in the body? CONCEPT/PRINCIPLE. RILEY ONG, MIDTERM
A. Prolactin MD (TOP 5 - EXAM - FEB
B. Growth hormone AUG 2014 2015
C. Adrenocorticotropic hormone MED
D. Follicle-stimulating hormone BOARDS;
E. Thyroid-stimulating hormone TOPNOTCH
MD)
388 ST segment elevation in leads II, III and aVF II, III, aVF --> inferior wall. V1, V2 --> septal wall. V3, V4 - SCOTT BACK-UP
represents myocardial infarction in which wall of -> anterior wall. V5, V6 --> lateral wall. RILEY ONG, MIDTERM
the heart? MD (TOP 5 - EXAM - FEB
A. Inferior wall AUG 2014 2015
B. Anterior wall MED
C. Septal wall BOARDS;
D. Anteroseptal wall TOPNOTCH
E. Lateral wall MD)

389 Which of the following substances is known as the SCOTT BACK-UP


most potent naturally occurring vasocontrictor in RILEY ONG, MIDTERM
the body? MD (TOP 5 - EXAM - FEB
A. Endothelin AUG 2014 2015
B. Bradykinin MED
C. Vasopressin BOARDS;
D. Serotonin TOPNOTCH
E. Prostaglandin F MD)

390 In which of the following organs is blood flow mainly Blood flow in the skin and resting muscles is mainly SCOTT BACK-UP
regulated by the sympathetic nervous system? controlled by the sympathetic nervous system. In RILEY ONG, MIDTERM
A. Skin contrast, blood flow in the brain, heart, kidneys and MD (TOP 5 - EXAM - FEB
B. Brain exercising muscles is mainly under autoregulation and AUG 2014 2015
C. Heart local control. MED
D. Exercising muscles BOARDS;
E. Kidneys TOPNOTCH
MD)

391 A 19-year old male was caught in a gang war and SIMILAR TO PREVIOUS BOARD EXAM SCOTT BACK-UP
was hit in the epigastric area with strong direct CONCEPT/PRINCIPLE. RILEY ONG, MIDTERM
punch. He suddenly developed hypotension, MD (TOP 5 - EXAM - FEB
bradycardia and weak pulses. Which of the following AUG 2014 2015
best explains these findings? MED
A. Gastric perforation and pneumoperitoneum BOARDS;
B. Rupture of the abdominal aorta TOPNOTCH
C. Hepatic contusion and subcapsular hematoma MD)
D. Vaso-vagal reflex stimulation
E. Development of an intraperitoneal hematoma
392 One mechanism by which PTH increases the body's Calcium reabsorption in the loop of Henle occurs via the SCOTT BACK-UP
blood calcium level is by stimulating the kidney to paracellular route. Calcium reabsorption in the early RILEY ONG, MIDTERM
increase calcium reabsorption. At which portion of distal tubule occurs via luminal calcium channels and MD (TOP 5 - EXAM - FEB
the renal tubular system does PTH act? basolateral Na/Ca exchangers. PTH regulates calcium AUG 2014 2015
A. Proximal convoluted tubule reabsorption through these channels. MED
B. Ascending limb of the loop of Henle BOARDS;
C. Early distal tubule TOPNOTCH
D. Late distal tubule MD)
E. Collecting duct

TOPNOTCH MEDICAL BOARD PREP PHYSIOLOGY SUPEREXAM Page 48 of 88


For inquiries visit www.topnotchboardprep.com.ph or email us at topnotchmedicalboardprep@gmail.com
TOPNOTCH MEDICAL BOARD PREP PHYSIOLOGY SUPEREXAM
For inquiries visit www.topnotchboardprep.com.ph or email us at topnotchmedicalboardprep@gmail.com
Item QUESTION EXPLANATION AUTHOR TOPNOTCH
# EXAM
393 The intrinsic and extrinsic pathways of the Common pathway: factor X (Stuart factor) --> factor V SCOTT BACK-UP
coagulation cascade converge at which of the (Labile factor) --> factor II (prothrombin) --> factor XIII RILEY ONG, MIDTERM
following clotting factors? (fibrin-stabilizing factor). *Mnemonic: 10 + 5 - 2 = 13. MD (TOP 5 - EXAM - FEB
A. Labile factor Factor VII (Stable factor) and factor III (tissue factor) are AUG 2014 2015
B. Stable factor from the extrinsic pathway. Factor XII (Hageman factor) MED
C. Stuart factor is from the intrinsic pathway BOARDS;
D. Tissue factor TOPNOTCH
E. Hageman factor MD)

394 Which of the following parameters is increased in The rest of the parameters mentioned are decreased in SCOTT BACK-UP
COPD? COPD. RILEY ONG, MIDTERM
A. Forced vital capacity MD (TOP 5 - EXAM - FEB
B. Forced expiratory volume AUG 2014 2015
C. Functional residual capacity MED
D. Expiratory reserve volume BOARDS;
E. None of the above TOPNOTCH
MD)
395 In type III hypersensitivity reactions, deposition of SIMILAR TO PREVIOUS BOARD EXAM SCOTT BACK-UP
antibody-antigen complexes in basement CONCEPT/PRINCIPLE RILEY ONG, MIDTERM
membranes can elicit inflammation through which MD (TOP 5 - EXAM - FEB
of the following mechanisms? AUG 2014 2015
A. Alternative complement pathway MED
B. Classical complement pathway BOARDS;
C. Cytotoxic T cell activation TOPNOTCH
D. Induced cell apoptosis MD)
E. Eosinophilic degranulation

396 Performing a carotid sinus massage would produce SCOTT BACK-UP


all of the following effects except: RILEY ONG, MIDTERM
A. Decreased heart rate MD (TOP 5 - EXAM - FEB
B. Decreased aortic blood pressure AUG 2014 2015
C. Increased activity and discharge from carotid MED
baroreceptors BOARDS;
D. Activation of the renin-angiotensin-aldosterone TOPNOTCH
system MD)
E. None of the above
397 Which of the following is a known complication of Overcorrection of hypernatremia causes cerebral edema. SCOTT BACK-UP
rapid correction of hypernatremia? Overcorrection of hyponatremia causes central pontine RILEY ONG, MIDTERM
A. Cerebral edema myelinolysis. MD (TOP 5 - EXAM - FEB
B. Central pontine myelinolysis AUG 2014 2015
C. Hypotension MED
D. Rhabdomyolysis BOARDS;
E. All of the above TOPNOTCH
MD)
398 A 60-year old stage IV breast cancer patient is in her Diagnosis: pulmonary embolism. Chest radiograph SCOTT BACK-UP
18th hospital day when she was referred for describes the Westermark sign. Respiratory alkalosis RILEY ONG, MIDTERM
dyspnea and tachypnea. Chest radiograph was secondary to hyperventilation (tachypnea) would be MD (TOP 5 - EXAM - FEB
requested and you noted a new-onset focal area of expected. AUG 2014 2015
oligemia in the right lower lung field. If ABG were be MED
performed on her, what would be your expected BOARDS;
finding? TOPNOTCH
A. Respiratory acidosis MD)
B. Respiratory alkalosis
C. Metabolic acidosis with high anion gap
D. Metabolic acidosis with normal anion gap
E. Metabolic alkalosis
399 Which of the following statements is least accurate The power stroke is driven by the released of hydrolyzed SCOTT BACK-UP
in describing the events occurring during muscle ATP (now in the form of ADP and Pi) from the myosin RILEY ONG, MIDTERM
contraction? head. MD (TOP 5 - EXAM - FEB
A. The H-band and I-zone in the sarcomere shorten AUG 2014 2015
during contraction. MED
B. Calcium is the main ion that facilitates actin- BOARDS;
myosin cross-bridge cycling. TOPNOTCH
C. The hydrolysis of an ATP molecule attached to the MD)
myosin head drives the "power stroke".
D. Sodium influx and potassium efflux both
contribute to the depolarization of the motor
endplate following stimulation by acetylcholine.
E. All of the above statements are true.
400 The transport of calcium ions back to the This process is achieved through CA-ATPase pumps in SCOTT BACK-UP
sarcoplasmic reticulum during muscle relaxation is the sarcoplasmic reticulum (known as SERCA). RILEY ONG, MIDTERM
achieved through which of the following MD (TOP 5 - EXAM - FEB
mechanisms? AUG 2014 2015
A. Simple diffusion MED
B. Facilitated diffusion BOARDS;
C. Primary active transport TOPNOTCH
D. Secondary active transport via symport MD)
E. Secondary active transport via antiport
401 How many oxygen molecules can 1 myoglobin Myoglobin can only bind 1 molecule of oxygen, JOSE CARLO DIAGNOSTIC
molecule bind? Hemoglobin can carry 4 molecules of oxygen MASANGKAY EXAM - AUG
A. 1 III, MD (TOP 2014
B. 2 8 - FEB 2014
C. 3 MED
D. 4 BOARDS;
E. 5 TOPNOTCH
MD)
TOPNOTCH MEDICAL BOARD PREP PHYSIOLOGY SUPEREXAM Page 49 of 88
For inquiries visit www.topnotchboardprep.com.ph or email us at topnotchmedicalboardprep@gmail.com
TOPNOTCH MEDICAL BOARD PREP PHYSIOLOGY SUPEREXAM
For inquiries visit www.topnotchboardprep.com.ph or email us at topnotchmedicalboardprep@gmail.com
Item QUESTION EXPLANATION AUTHOR TOPNOTCH
# EXAM
402 An MRI of a patient who suffered from a Lesions of the right visual cortex posterior to the lateral JOSE CARLO DIAGNOSTIC
deceleration accident revealed a lesion on the right geniculate body will present with left hemianopia with MASANGKAY EXAM - AUG
visual cortex posterior to the lateral geniculate body, macular sparing. Lesions of the optic tract will present III, MD (TOP 2014
which of the following visual defects would be with homonymous hemianopia. 8 - FEB 2014
expected? MED
A. Right homonymous hemianopia BOARDS;
B. Left homonymous hemianopia TOPNOTCH
C. Left hemianopia with macular sparing MD)
D. Right hemianopia with macular sparing
E. Bilateral Hemianopia
403 Which component of the complement system JOSE CARLO DIAGNOSTIC
promotes opsonization? MASANGKAY EXAM - AUG
A. C3a III, MD (TOP 2014
B. C3b 8 - FEB 2014
C. C4a MED
D. C4b BOARDS;
E. C5a TOPNOTCH
MD)
404 On a NORMAL individual, What is the alveolar AV= (TV-Physiologic Dead space) x RR; AV= (500-150) x JOSE CARLO DIAGNOSTIC
ventilation, given the following: Tidal Volume : 22; AV:7,700mL/ 7.7 L (a similar computation-question MASANGKAY EXAM - AUG
500mL, RR:22cpm? was asked in the boards) III, MD (TOP 2014
A. 7.7 L 8 - FEB 2014
B. 8.8 L MED
C. 9.9 L BOARDS;
D. 10.0 L TOPNOTCH
E. 11.0 L MD)

405 Majority of Carbon dioxide is transported in the JOSE CARLO DIAGNOSTIC


body thru which form? MASANGKAY EXAM - AUG
A. HCO3 III, MD (TOP 2014
B. H2CO3 8 - FEB 2014
C. Free CO2 MED
D. CO2 bound to albumin BOARDS;
E. COOH TOPNOTCH
MD)
406 Pulmonary Embolism shows a V/Q ratio of: In Pulmonary embolism there is good ventilation but JOSE CARLO DIAGNOSTIC
A. More than 1 zero perfusion hence V/Q ratio is infinite, there is MASANGKAY EXAM - AUG
B. Zero continuous ventilation with no perfusion. III, MD (TOP 2014
C. Less than 1 8 - FEB 2014
D. Negative 0.8 MED
E. Infinite BOARDS;
TOPNOTCH
MD)
407 Which of the following shifts the Hemoglobin The hemoglobin Dissociation curve shifts to the right in: JOSE CARLO DIAGNOSTIC
association curve to the left: Increased CO2, ACIDOSIS (low pH), Increased 2,3 -BPG, MASANGKAY EXAM - AUG
A. Fever Exercise, Increased Temperature. (similar to SIMILAR III, MD (TOP 2014
B. High pH TO PREVIOUS BOARD EXAM CONCEPT/PRINCIPLE) 8 - FEB 2014
C. Increased 2,3 BPG MED
D. Exercise BOARDS;
E. Increased CO2 TOPNOTCH
MD)
408 Which of the following combination of hormones is Ach, Histamine, and Gastrin are all involved in the JOSE CARLO DIAGNOSTIC
synergistic with each other to promote HCl several mechanisms of HCl secretion. Histamine is the MASANGKAY EXAM - AUG
secretion? most important of the 3. III, MD (TOP 2014
A. VIP, Ach, Histamine 8 - FEB 2014
B. Ach, Histamine, Gastrin MED
C. Secretin, CCK, Ach BOARDS;
D. Motilin, Secretin, Gastrin TOPNOTCH
E. Somatostatin, Gastrin, CCK MD)

409 Basophilic Cells of the Pituitary gland secretes the Basophilic Cells of the Anterior Pituitary Gland secretes JOSE CARLO DIAGNOSTIC
following: the ff: FSH, LH, ACTH, TSH, and MSH MASANGKAY EXAM - AUG
A. FSH III, MD (TOP 2014
B. GH 8 - FEB 2014
C. Prolactin MED
D. A and B BOARDS;
E. All of the above TOPNOTCH
MD)
410 Decreased Acetylcholine levels in the brain is Locus Ceruleus-NE, Serotonin- Raphe Nucleus, JOSE CARLO DIAGNOSTIC
thought to be the reason for developing Alzheimer's Dopamine- Ventral tegmentum of Substantia Nigra, MASANGKAY EXAM - AUG
Disease. Where is Acetylcholine produced in the GABA- Nucleus Accumbens, III, MD (TOP 2014
brain? 8 - FEB 2014
A. Locus Ceruleus of the Pons MED
B. Ventral Tegmentum BOARDS;
C. Basal Nucleus of Meynert TOPNOTCH
D. Nucleus Accumbens MD)
E. Raphe Nucleus

TOPNOTCH MEDICAL BOARD PREP PHYSIOLOGY SUPEREXAM Page 50 of 88


For inquiries visit www.topnotchboardprep.com.ph or email us at topnotchmedicalboardprep@gmail.com
TOPNOTCH MEDICAL BOARD PREP PHYSIOLOGY SUPEREXAM
For inquiries visit www.topnotchboardprep.com.ph or email us at topnotchmedicalboardprep@gmail.com
Item QUESTION EXPLANATION AUTHOR TOPNOTCH
# EXAM
411 While walking at the mall, you incidentally saw your The action potential of the SA node starts at Phase 4 JOSE CARLO DIAGNOSTIC
highschool crush, and your heart started beating followed by a phase 0 then a phase 3. The more steeper MASANGKAY EXAM - AUG
fast. The heart rate is dependent on whate phase of the phase 4 the easier it is to reach the threshold, hence III, MD (TOP 2014
the action potential of the SA node? a faster cardiac rate. 8 - FEB 2014
A. Phase 0 MED
B. Phase 1 BOARDS;
C. Phase 2 TOPNOTCH
D. Phase 3 MD)
E. Phase 4

412 Which of the following is NOT a Sympathetic Erection is a Parasympathetic response. Uterine JOSE CARLO DIAGNOSTIC
response? contraction and relaxation are both sympathetic MASANGKAY EXAM - AUG
A. Mydriasis responses acting on alpha-1 receptor and Beta-2 III, MD (TOP 2014
B. Uterine contraction receptors, respectively. Mydriasis and piloerection are 8 - FEB 2014
C. Uterine relaxation both sympathetic responses MED
D. Piloerection BOARDS;
E. Erection TOPNOTCH
MD)
413 Your lola is sufferring from cataract, bragging in The refractive power of the eye is 59 diopters and 2/3s JOSE CARLO DIAGNOSTIC
front of her amigas, she asks you: "what part of the of this is from the cornea, 1/3 is provided by the lens. MASANGKAY EXAM - AUG
eye is the most important factor in refraction?", you III, MD (TOP 2014
will say that it is the: 8 - FEB 2014
A. Retina MED
B. Lens BOARDS;
C. Uvea TOPNOTCH
D. Cornea MD)
E. Pupil

414 Among which of the following hypoxemic conditions JOSE CARLO DIAGNOSTIC
will manifest as a normal Alveolar-arterial Gradient? MASANGKAY EXAM - AUG
A. High altitude III, MD (TOP 2014
B. V/Q mismatch 8 - FEB 2014
C. Diffusion Limitation MED
D. Right to left shunt BOARDS;
E. None of the above TOPNOTCH
MD)

415 A patient presents to you in the ER with Sinus CN IX/ Glossopharyngeal nerve is responsible for JOSE CARLO DIAGNOSTIC
tachycardia, you immediately performed carotid transmitting the afferent signal from the carotid sinus, MASANGKAY EXAM - AUG
massage to decrease the HR. Which of the following its branch is specifically termed as the Herring's Nerve. III, MD (TOP 2014
is responsible for the transmition of afferent signals 8 - FEB 2014
from the carotid sinus to the medulla. MED
A. CN V BOARDS;
B. CN VII TOPNOTCH
C. CN VIII MD)
D. CN IX
E. CN X
416 This phase of the cell cycle is where the 2nd heart In Isovolumic relaxation the the aortic pressure is JOSE CARLO DIAGNOSTIC
sound is heard, the ventricular pressure decreases greater than the ventricular pressure hence the MASANGKAY EXAM - AUG
but the ventricular volume remains the same semilunar valve closes and the second heart sound is III, MD (TOP 2014
A. Atrial Contraction heard. 8 - FEB 2014
B. Isovolumic Contraction MED
C. Rapid Ventricular Ejection BOARDS;
D. Reduced Ventricular Ejection TOPNOTCH
E. Isovolumic Relaxation MD)

417 A nurse accidentally infused a large amount of JOSE CARLO DIAGNOSTIC


Calcium gluconate into your patient, what will be the MASANGKAY EXAM - AUG
expected cardiac effect? III, MD (TOP 2014
A. Increased Heart rate 8 - FEB 2014
B. Arrhythmia MED
C. Increased Cardiac output BOARDS;
D. Decreased Heart rate TOPNOTCH
E. None of the above MD)

418 Osmotic Diuretics act on all parts of the Nephron JOSE CARLO DIAGNOSTIC
EXCEPT: MASANGKAY EXAM - AUG
A. Proximal Convoluted Tubule III, MD (TOP 2014
B. Loop of Henle 8 - FEB 2014
C. Distal Convoluted Tubules MED
D. Collecting Ducts BOARDS;
E. No exception TOPNOTCH
MD)
419 Glucose Transporters (GLUT) are of what type of GLUTs are passive transport mechanisms, has a downhill JOSE CARLO DIAGNOSTIC
Transport Mechanism? approach and is carrier mediated, which does not use MASANGKAY EXAM - AUG
A. Simple Diffusion metabolic energy, they are also independent of Na III, MD (TOP 2014
B. Facilitated Diffusion gradient. 8 - FEB 2014
C. Primary Active Transport MED
D. Cotransport BOARDS;
E. Counter transport TOPNOTCH
MD)

TOPNOTCH MEDICAL BOARD PREP PHYSIOLOGY SUPEREXAM Page 51 of 88


For inquiries visit www.topnotchboardprep.com.ph or email us at topnotchmedicalboardprep@gmail.com
TOPNOTCH MEDICAL BOARD PREP PHYSIOLOGY SUPEREXAM
For inquiries visit www.topnotchboardprep.com.ph or email us at topnotchmedicalboardprep@gmail.com
Item QUESTION EXPLANATION AUTHOR TOPNOTCH
# EXAM
420 On what phase of the female menstrual cycle is a JOSE CARLO DIAGNOSTIC
patient in, given the following lab results: Estrogen, MASANGKAY EXAM - AUG
FSH and LH: INCREASED; Progesterone: III, MD (TOP 2014
DECREASED? 8 - FEB 2014
A. Menstruation MED
B. Follicular Phase BOARDS;
C. Ovulation TOPNOTCH
D. Luteal Phase MD)
E. Proliferative Phase
421 Which myofibril band does not shorten during *Generally a recall type of exam. H-band (heller) is the WEBSTER MIDTERM 1
contraction? zone of the thick filaments that is not superimposed by ALINDOG, EXAM - AUG
A. H band the thin filaments; whereas A-band (anisotropic) MD (TOP 3 - 2014
B. A band contains the entire length of a single thick filament; and FEB 2014
C. I band finally, I-band (isotropic) is the zone of thin filaments MED
D. All of the above that is not superimposed by thick filaments. During BOARDS;
contraction, the A-band does not change its length, TOPNOTCH
whereas the I-band and the H-band shorten. This also MD)
causes the Z lines to come closer together.
422 Which of the following ECG findings will best suggest The appearance of Q waves (i.e. significant Q - wider and WEBSTER MIDTERM 1
an old inferior wall myocardial infarction? deeper waves) indicates that irreversible myocardial cell ALINDOG, EXAM - AUG
A. Q wave in the beginning of QRS complex in lead I death has occurred. Its presence is diagnostic of MI. MD (TOP 3 - 2014
B. Q wave in leads II, III, avF Leads II, III, avF assess the posterior wall territory. J FEB 2014
C. J point elevation in leads II, III, avF point is the place where ST segment takes off from the MED
D. All of the above QRS complex and said to have ho pathologic implications BOARDS;
whatsoever. TOPNOTCH
MD)

423 Osmotic pressure is defined as the amount of WEBSTER MIDTERM 1


pressure required to stop movement of water ALINDOG, EXAM - AUG
through a semi-permeable membrane. Which of the MD (TOP 3 - 2014
following is not true about osmotic pressure? FEB 2014
A. It is determined by the number of particles per MED
unit volume of fluid. BOARDS;
B. It is determined by the mass of particles per unit TOPNOTCH
volume of fluid. MD)
C. The molar concentration of the solution dictates
the osmotic pressure.
D. None of the above.
424 How much body water is lost in the feces during Must memorize several values; they are asked. Water WEBSTER MIDTERM 1
prolonged heavy exercise? loss from the skin and feces remain constant even during ALINDOG, EXAM - AUG
A. 50 ml/day prolonged heavy exercise, set at 350 and 100 ml/day, MD (TOP 3 - 2014
B. 100 ml/day respectively. On the other hand, loss from respiration FEB 2014
C. 250 ml/day increases from 350 to 650 ml/day; sweat - from 100 to MED
D. 400 ml/day 5000 ml/day. Urine finally decreases from 1400 to as BOARDS;
little as 500 ml/day. TOPNOTCH
MD)
425 Which of the following will decrease the glomerular Hydraulic conductivity influences the filtration WEBSTER MIDTERM 1
filtration rate? coefficient and hence the GFR. It refers to the ease with ALINDOG, EXAM - AUG
A. An increase in the hydraulic conductivity of the which substances can pass through the glomerular MD (TOP 3 - 2014
glomerulus capillary membrane. Increasing the conductivity, FEB 2014
B. An increase in the glomerular capillary osmotic therefore will not decrease the GFR. Decreasing the MED
pressure sympathetic effect on afferent arteriole would mean BOARDS;
C. A decrease in the sympathetic activity to the dilatation and an increase in glomerular hydrostatic TOPNOTCH
afferent renal vasculature pressure and hence increased GFR. Moderate increase in MD)
D. A moderate increase in angiotensin II acting on angiotensin II will cause efferent arteriole constriction
efferent arterioles and will contribute to increased ccapillary hydrostatic
pressure. It should be noted however that in excessive
stimulation by angiotensin II, the severe constriction will
eventually increase the glomerular capillary oncotic
pressure (trapped protein and ions) and therefore will
diminish GFR.
426 What is the last nucleated stage found in Reticulocyte, on the other hand, is the earliest RBC stage WEBSTER MIDTERM 1
erythropoiesis? found in peripheral blood. It is already anucleated. ALINDOG, EXAM - AUG
A. Basophilic erythroblast MD (TOP 3 - 2014
B. Polychromatophilic erythroblast FEB 2014
C. Orthochromatic erythroblast MED
D. Reticulocyte BOARDS;
TOPNOTCH
MD)
427 What is the endothelial cell molecule responsible for Please refer to Acute and Chronic Inflammation Chapter WEBSTER MIDTERM 1
the adhesion, arrest and transmigration of all of Robbins (Unit 1, Chapter 2, Table 2-1, p. 54, 7th ed). It ALINDOG, EXAM - AUG
leukocytes? was very helpful! =) MD (TOP 3 - 2014
A. P-selectin FEB 2014
B. VCAM MED
C. ICAM BOARDS;
D. Integrins TOPNOTCH
MD)

TOPNOTCH MEDICAL BOARD PREP PHYSIOLOGY SUPEREXAM Page 52 of 88


For inquiries visit www.topnotchboardprep.com.ph or email us at topnotchmedicalboardprep@gmail.com
TOPNOTCH MEDICAL BOARD PREP PHYSIOLOGY SUPEREXAM
For inquiries visit www.topnotchboardprep.com.ph or email us at topnotchmedicalboardprep@gmail.com
Item QUESTION EXPLANATION AUTHOR TOPNOTCH
# EXAM
428 Which of the following platelet/coagulation studies Appreciate the purpose of the different lab tests WEBSTER MIDTERM 1
results are most consistent with von Willebrand's including: BLEEDING TIME - a qualitative test for ALINDOG, EXAM - AUG
disease? platelet function (we mean, does it adhere, aggregate MD (TOP 3 - 2014
A. NORMAL bleeding time, platelet count, PT and well?), while PLATELET COUNT - obviously, a FEB 2014
PTT quantitative test. PT is a coagulation test that assesses MED
B. NORMAL bleeding time, platelet count, PT; both the extrinsic (factors VII, III/tissue factor) and the BOARDS;
PROLONGED PTT common (factors V, X, II, I, XIII) pathways - mnemonic: TOPNOTCH
C. PROLONGED bleeding time; NORMAL platelet PeT. While PTT assesses both the intrinsic (HMWK, MD)
count and PT; PROLONGED PTT factors XII, XI, VIII, IX) and the common pathways -
D. PROLONGED bleeding time; LOW platelet count; mnemonic: PiTT. In von Willebrand disease, there is
PROLONGED PT and PTT problem with vWf which is important both in platelet
adhesion and in intrinsic pathway
(stabilizer/vehicle/cofactor of factor VIII) hence
prolonged BT and PTT.
429 Speech is a complex mechanism that makes use of WEBSTER MIDTERM 1
the respiratory system, cerebral cortex, and the ALINDOG, EXAM - AUG
articulation and phonation structures of the mouth MD (TOP 3 - 2014
and nasal cavities. Its mechanical function phonation FEB 2014
is particularly achieved through the: MED
A. Tongue BOARDS;
B. Larynx TOPNOTCH
C. Lips MD)
D. Epiglottis

430 Zone 2, the predominant pulmonary blood flow type Option A refers to Zone 1, while option C refers to Zone WEBSTER MIDTERM 1
in the lung apices, is characterized by: 3. Normally, the lungs have only zones 2 and 3 blood ALINDOG, EXAM - AUG
A. Alveolar air pressure that is greater than arterial flow - zone 2 (intermittent blood flow) in the apices, and MD (TOP 3 - 2014
pressure during the entire cardiac cycle zone 3 (continuous flow) in all lower areas. FEB 2014
B. Alveolar air pressure that is lesser than arterial MED
pressure during systole but greater only during BOARDS;
diastole TOPNOTCH
C. Arterial pressure and pulmonary capillary MD)
pressure remaining greater than alveolar air
pressure at all times
D. None of the above

431 Vasopressin secretion is triggered by: Vasopressin or ADH is water-regulating hormone WEBSTER MIDTERM 1
A. LOW blood volume; LOW blood pressure; HIGH (water-conserving). It responds to low blood/plasma ALINDOG, EXAM - AUG
urine osmolality volume, low BP, and high plasma osmolality --- all MD (TOP 3 - 2014
B. HIGH blood volume; HIGH blood alcohol; LOW suggesting water depletion. FEB 2014
plasma osmolality MED
C. HIGH blood pressure; HIGH body alcohol; HIGH BOARDS;
urine osmolality TOPNOTCH
D. LOW blood volume; LOW blood pressure; HIGH MD)
plasma osmolality
432 Which of the following will cause increased K WEBSTER MIDTERM 1
secretion? ALINDOG, EXAM - AUG
A. Hyperaldosteronism MD (TOP 3 - 2014
B. Alkalosis FEB 2014
C. Luminal anions MED
D. All of the above BOARDS;
E. None of the above TOPNOTCH
MD)
433 All of the following will shift the oxygen dissociation *Should you fail to master this topic, you may not take WEBSTER MIDTERM 1
curve to the right except: the physio exam =). The O2 dissociation curve (which is, ALINDOG, EXAM - AUG
A. Increased 2,3-bisphosphoglycerate by the way, sigmoidal in shape) shifts to the right MD (TOP 3 - 2014
B. Hypercarbia (meaning hemoglobin has lower affinity to O2 and FEB 2014
C. Fever releases more O2 to tissues) happens during C-A-B-E-T: MED
D. Alkalosis high Carbon dioxide; Acidosis or low pH or high H ions; BOARDS;
E. None of the above increased 2,3-Bisphosphoglycerate; Exercise; and high TOPNOTCH
Temperature or fever. MD)
434 Pathogenesis of Alzheimer's disease is believed to Option A is Parkinson's; option C is Huntington's. At least WEBSTER MIDTERM 1
involve the destruction of: know their basic pathophysio, useful also. ALINDOG, EXAM - AUG
A. Dopamine-secreting cell bodies in the substantia MD (TOP 3 - 2014
nigra pars compacta FEB 2014
B. Acetylcholine-producing neurons in many parts MED
of the brain BOARDS;
C. GABA-producing neurons in caudate nucleus TOPNOTCH
D. Norepinephrine-secreting neurons in locus MD)
cereleus
435 What is the importance of secondary peristaltic Primary and secondary peristalsis are exhibited by the WEBSTER MIDTERM 1
waves in GI functions? esophagus. Primary peristalsis is the wave that begins in ALINDOG, EXAM - AUG
A. It mediates what is known as the gastrocolic the pharynx and spreads into the esophagus during the MD (TOP 3 - 2014
reflex. pharyngeal stage of swallowing. If it fails to move into FEB 2014
B. It delays gastric emptying time promoting the stomach all the food that have gone to the esophagus, MED
mixing of foods until they form chyme. secondary peristalsis results from distention of the BOARDS;
C. It is responsible for the mass movements of the esophagus itself by the retained food and continue until TOPNOTCH
colon. all of them reach the stomach. MD)
D. It mediates esophageal contraction until all
retained foods are emptied into the stomach.

TOPNOTCH MEDICAL BOARD PREP PHYSIOLOGY SUPEREXAM Page 53 of 88


For inquiries visit www.topnotchboardprep.com.ph or email us at topnotchmedicalboardprep@gmail.com
TOPNOTCH MEDICAL BOARD PREP PHYSIOLOGY SUPEREXAM
For inquiries visit www.topnotchboardprep.com.ph or email us at topnotchmedicalboardprep@gmail.com
Item QUESTION EXPLANATION AUTHOR TOPNOTCH
# EXAM
436 Pancreatic secretion is stimulated by: WEBSTER MIDTERM 1
A. Cholecystokinin ALINDOG, EXAM - AUG
B. Secretin MD (TOP 3 - 2014
C. Acetylcholine FEB 2014
D. All of these MED
BOARDS;
TOPNOTCH
MD)
437 This serves as the feeding center of the brain, Option A is the satiety center; stimulation of which WEBSTER MIDTERM 1
destruction of which results in progressive inanition results in aphagia (refuses to eat), and conversely, its ALINDOG, EXAM - AUG
- marked weight loss, muscle weakness and destruction leads to voracious eating. MD (TOP 3 - 2014
decreased metabolism: FEB 2014
A. Ventromedial nuclei of hypothalamus MED
B. Lateral nuclei of hypothalamus BOARDS;
C. Anteroventral walls of the 3rd ventricle TOPNOTCH
D. Posteromedial walls of the 3rd ventricle MD)

438 Which of the following is effected by somatotropin? Somatotropin is GH. Its tissue effector are the insulin- WEBSTER MIDTERM 1
A. Enhances amino acid reuptake and protein like growth factors (IGFs) or somatomedins, particularly ALINDOG, EXAM - AUG
synthesis by cells IGF 1 or somatomedin C. They are synthesized from the MD (TOP 3 - 2014
B. Stimulates the transcription of DNA to RNA for liver FEB 2014
translation MED
C. May cause excessive mobilization of fat from the BOARDS;
adipose tissue TOPNOTCH
D. All of the above MD)
439 This refers to the extra osmotic pressure caused by WEBSTER MIDTERM 1
cations, primarily Na and K, held in the plasma by ALINDOG, EXAM - AUG
the proteins -- contributing to the total plasma MD (TOP 3 - 2014
colloid osmotic pressure: FEB 2014
A. Bohr effect MED
B. Scavenger system principle BOARDS;
C. Bainbridge TOPNOTCH
D. Donnan effect MD)
440 The specific ability to pump iodide actively to the Oxidation of iodide ion is considered as the first essential WEBSTER MIDTERM 1
interior of the thyroid cell from its basal membrane step in thyroid hormone synthesis because it requires ALINDOG, EXAM - AUG
is known as: that iodide be converted to the oxidized ions before MD (TOP 3 - 2014
A. Organification combining directly with amino acid tyrosine. In contrast, FEB 2014
B. Iodide trapping organification refers to the binding of iodine with the MED
C. Oxidation of iodide ion thyroglobulin molecule. BOARDS;
D. Deiodination TOPNOTCH
MD)

441 Among the following cellular organelles, this Mitochondria are self-replicative, which means that one JULIET MIDTERM 2
organelle has the greatest potential for self- mitochondrion can form a second one, a third one, and KRISTINE EXAM - AUG
replication so on, whenever there is a need in the cell for increased EVANGELIST 2014
A. Lysosome amounts of ATP. A, MD (TOP 9
B. Golgi complex - FEB 2014
C. Mitochondria MED
D. Cell membrane BOARDS;
E. Ribosome TOPNOTCH
MD)
442 A 24 year-old woman was diagnosed with Pyridostigmine is an indirect-acting cholinomimetic. It JULIET MIDTERM 2
myasthenia gravis. She was prescribed with inhibits acetylcholinesterase which increases levels of KRISTINE EXAM - AUG
Pyridostigmine and she noted increased muscle acetylcholine at the motor end plates resulting to EVANGELIST 2014
strength when she took the prescribed medicine. increased muscle strength. A, MD (TOP 9
The basis for this improvement is increased in the: - FEB 2014
A. Amount of acetylcholine destroyed in the motor MED
end plates BOARDS;
B. Levels of acetylcholine at the motor end plates TOPNOTCH
C. Number of acetylcholine receptors in the motor MD)
end plate
D. Amount of norepinehrine released from the
motor nerves
E. Amount of norepinephrine receptors in the
motor end plates
443 The resting membrane potential is determined The resting membrane potential is about -90 millivolts JULIET MIDTERM 2
largely by: which is determined largely by potassium efflux through KRISTINE EXAM - AUG
A. Sodium influx through the voltage-gated the Na-K leak channels. The channels are more EVANGELIST 2014
channels permeable to potassium than to sodium, normally about A, MD (TOP 9
B. Sodium influx through the Na-K leak channels 100 times as permeable. This differential in permeability - FEB 2014
C. Potassium efflux throught the voltage-gated is exceedingly important in determining the level of the MED
channels normal RMP. BOARDS;
D. Potassium efflux through the Na-K leak channels TOPNOTCH
E. Sodium and potassium exchange by ATPase MD)
444 Which of the following forces causes osmosis of fluid Osmosis is the spontaneous net movement of solvent JULIET MIDTERM 2
outwards through the capillary membrane? through a selectively permeable membrane such as a cell KRISTINE EXAM - AUG
A. Interstitial colloid osmotic pressure membrane, into a region of higher solute concentration. EVANGELIST 2014
B. Positive interstitial fluid pressure Interstitial colloid osmotic pressure is an osmotic A, MD (TOP 9
C. Plasma colloid osmotic pressure pressure exerted by proteins which tends to pull water - FEB 2014
D. Plasma oncotic pressure into interstitial space. It opposes the plasma colloid MED
E. All of the above osmotic pressure. BOARDS;
TOPNOTCH
MD)

TOPNOTCH MEDICAL BOARD PREP PHYSIOLOGY SUPEREXAM Page 54 of 88


For inquiries visit www.topnotchboardprep.com.ph or email us at topnotchmedicalboardprep@gmail.com
TOPNOTCH MEDICAL BOARD PREP PHYSIOLOGY SUPEREXAM
For inquiries visit www.topnotchboardprep.com.ph or email us at topnotchmedicalboardprep@gmail.com
Item QUESTION EXPLANATION AUTHOR TOPNOTCH
# EXAM
445 Tetrodotoxin block or inhibit: Tetrodotoxin is a potent neurotoxin from a pufferfish JULIET MIDTERM 2
A. Sodium channel which inhibits the firing of action potentials in nerves by KRISTINE EXAM - AUG
B. Potassium channel binding to the voltage-gated sodium channels in nerve EVANGELIST 2014
C. Sodium-potassium pump cell membranes and blocking the passage of sodium ions. A, MD (TOP 9
D. Na-K leak channel - FEB 2014
E. Calcium channel MED
BOARDS;
TOPNOTCH
MD)
446 Thromboembolic conditions in humans is mainly Thromboembolic conditions is mainly caused by JULIET MIDTERM 2
caused by: roughened endothelial surface of vessels. Endothelial KRISTINE EXAM - AUG
A. Presence of thrombomodulin in the vascular dysfunction as an altered phenotype impairs EVANGELIST 2014
system vasoreactivity or induces a surface that is thrombogenic A, MD (TOP 9
B. Roughened endothelial surface of vessels or abnormally adhesive to inflammatory cells. - FEB 2014
C. Presence of glycocalyx on the inner surface of Thrombomodulin is a glycoprotein present in plasma MED
endothelium membrane of endothelial cells that binds thrombin, an BOARDS;
D. Increased blood flow throught the vessels additional regulatory mechanism in coagulation. TOPNOTCH
E. All of the above MD)
447 Sinoatrial Node is the pacemaker of a normal heart Sinoatrial Node is virtually the pacemaker of a normal JULIET MIDTERM 2
because: heart because it has the highest frequency of impulse KRISTINE EXAM - AUG
A. It has the fastest velocity of impulse conduction generation. The sinus node controls the beat of the heart EVANGELIST 2014
B. It has both sympathetic and parasympathetic because its rate of rhythmical discharge is faster than A, MD (TOP 9
innervations that of any other part of the heart. - FEB 2014
C. It has the highest frequency of impulse MED
generation BOARDS;
D. It has the capability of spontaneous diastolic TOPNOTCH
depolarization MD)
E. It has the strongest capacity to stimulate the
myocardium
448 Pulmonary circulation has a lower pressure due to Pulmonary ciculation has a lower pressure due to JULIET MIDTERM 2
A. Lesser vessel diameter dilatable vessels resulting to low resistance based on KRISTINE EXAM - AUG
B. Less viscous blood Poiseuille's equation. Recruitment or opening of EVANGELIST 2014
C. High resistance vessels previously closed blood vessels also contribute to low A, MD (TOP 9
D. Dilatable vessels pressure. With low BP, some small blood vessels at the - FEB 2014
E. High blood velocity top of the lung may be closed, but these vessels will open MED
when the BP increases. BOARDS;
TOPNOTCH
MD)
449 Blood Pressure measurement is one of the vital signs Blood Pressure measurement is one of the vital signs JULIET MIDTERM 2
monitored in patients primarily with cardiovascular monitored in patients primarily with cardiovascular and KRISTINE EXAM - AUG
and renal conditions. Korotkoff sounds are used to renal conditions. Korotkoff sounds are used to identify EVANGELIST 2014
identify the systolic and diastolic measurements. the systolic and diastolic measurements. Auscultatory A, MD (TOP 9
Auscultatory gap is best described as disappearance gap is best described as disappearance of Korotkoff - FEB 2014
of Korotkoff sounds: sounds at the level above the diastolic pressure. MED
A. at the level of the diastolic pressure BOARDS;
B. below the level of the diastolic pressure TOPNOTCH
C. at the level above the diastolic pressure MD)
D. at the level above the systolic pressure
E. at the level of the systolic pressure
450 True of the Rh hemolytic disease of the newborn: Rh hemolytic disease of the newborn occurs when JULIET MIDTERM 2
A. First pregnancy is the most severely affected preformed maternal antibodies cross the placenta to KRISTINE EXAM - AUG
B. Mother is Rh positive, while fetus is Rh negative react with fetal Rh+ cells. Subsequent pregnancies are EVANGELIST 2014
C. After an erythroblastic child is born, future severely affected compared to first child. The mother is A, MD (TOP 9
children are certainly with disease Rh negative while the fetus is Rh positive. After an - FEB 2014
D. Preformed maternal antibodies cross the erythroblastic child is born, future children may be MED
placenta to react with fetal Rh+ cells protected from the disease by administration of Rhogam. BOARDS;
E. Subsequent pregnancies are protected from TOPNOTCH
maternal antibodies MD)
451 One of the following is not a physiologic effect of Physiologic effects of testosterone are: growth and JULIET MIDTERM 2
testosterone: differentiation of internal and external genitalia, KRISTINE EXAM - AUG
A. Increased RBC production increased bone and muscle mass, calcium deposition, EVANGELIST 2014
B. Skeletal and muscle growth epiphyseal closure, RBC production, increased BMR and A, MD (TOP 9
C. Maturation of internal and external genitalia pubertal growth spurt. - FEB 2014
D. Decreased calcium deposition in the bones MED
E. None of the above BOARDS;
TOPNOTCH
MD)
452 Adrenergic receptors in smooth muscle and glands Adrenergic receptors in smooth muscle and glands are JULIET MIDTERM 2
are: Alpha 1 and Beta 2. KRISTINE EXAM - AUG
A. Alpha 1 and beta 1 EVANGELIST 2014
B. Alpha 1 and beta 2 A, MD (TOP 9
C. Alpha 2 and beta 1 - FEB 2014
D. Alpha 2 and beta 2 MED
E. All of the above BOARDS;
TOPNOTCH
MD)
453 Three basic renal processes except: The three basic renal processes are glomerular filtration, JULIET MIDTERM 2
A. Glomerular filtration tubular reabsorption and tubular secretion. KRISTINE EXAM - AUG
B. Tubular reabsorption EVANGELIST 2014
C. Tubular secretion A, MD (TOP 9
D. Glomerular excretion - FEB 2014
E. All of the above MED
BOARDS;
TOPNOTCH
MD)

TOPNOTCH MEDICAL BOARD PREP PHYSIOLOGY SUPEREXAM Page 55 of 88


For inquiries visit www.topnotchboardprep.com.ph or email us at topnotchmedicalboardprep@gmail.com
TOPNOTCH MEDICAL BOARD PREP PHYSIOLOGY SUPEREXAM
For inquiries visit www.topnotchboardprep.com.ph or email us at topnotchmedicalboardprep@gmail.com
Item QUESTION EXPLANATION AUTHOR TOPNOTCH
# EXAM
454 A neurotransmitter that is synthesized from Serotonin is a monoamine neurotransmitter that is JULIET MIDTERM 2
tryptophan, converted to melatonin and is involved biochemically derived from tryptophan. It is found in KRISTINE EXAM - AUG
in mood and sleep is found in great amounts in: great amounts in median raphe of brainstem which is EVANGELIST 2014
A. Locus ceruleus of pons involved in regulation of mood, appetite and sleep. A, MD (TOP 9
B. Substancia nigra - FEB 2014
C. Median raphe of brainstem MED
D. Postganglionic neurons BOARDS;
E. Nucleus basalis of Meynert TOPNOTCH
MD)
455 The association area/s of the central cortex that The association area of the central cortex that provides a JULIET MIDTERM 2
provide/s a high level of interpretive meaning from high level of interpretive meaning from the surrounding KRISTINE EXAM - AUG
the surrounding sensory areas is/are the: sensory areas is the parieto-occipitotemporal EVANGELIST 2014
A. Corpus callosum association area. A, MD (TOP 9
B. Prefrontal association area - FEB 2014
C. Parieto-occipitotemporal association area MED
D. Secondary somatic sensory area BOARDS;
E. All of the above TOPNOTCH
MD)
456 Parathyroid hormone maintains normal ionized Parathyroid hormone maintains normal ionized serum JULIET MIDTERM 2
serum calcium concentration by the action on the calcium concentration by the action on the bone which KRISTINE EXAM - AUG
bone: increases formation of new osteoclasts and increases EVANGELIST 2014
A. Inhibits bone mineralization osteoblast-initiated recruitment of osteocytes. A, MD (TOP 9
B. Increases formation of new osteoclasts - FEB 2014
C. Increases osteoblast-initiated recruitment of MED
osteocytes BOARDS;
D. A to C TOPNOTCH
E. B and C MD)
457 The auditory function of the middle ear ossicles is to: The auditory function of the middle ear ossicles is to JULIET MIDTERM 2
A. Convert mechanical energy to sound to amplify the sound. KRISTINE EXAM - AUG
electrochemical energy EVANGELIST 2014
B. Detect the direction and intensity of sound A, MD (TOP 9
C. Filter high frequency sound - FEB 2014
D. Amplify the sound MED
E. All of the above BOARDS;
TOPNOTCH
MD)
458 The decrease in oxygen affinity of hemoglobin with The decrease in oxygen affinity of hemoglobin with high JULIET MIDTERM 2
high concentration of carbon dioxide is called: concentration of carbon dioxide is called Bohr effect. KRISTINE EXAM - AUG
A. Chloride shift Haldane effect is the decrease in the amount of EVANGELIST 2014
B. Haldane effect carbaminohemoglobin due to increased oxygenation. A, MD (TOP 9
C. Bohr effect Chloride shift is the exchange of chloride and - FEB 2014
D. Physiologic shunting bicarbonate in the RBC. MED
E. Fick's Law BOARDS;
TOPNOTCH
MD)
459 In sudden ascent of scuba divers to the surface from If a diver has been beneath the sea long enough that JULIET MIDTERM 2
deeper ocean bodies, this is the main cause of the large amounts of nitrogen have dissolved in his body and KRISTINE EXAM - AUG
symptoms: the diver then suddenly comes back to the surface of the EVANGELIST 2014
A. Oxygen toxicity sea, significant quantities of nitrogen bubbles can A, MD (TOP 9
B. Carbon dioxide excess develop in the body fluids and can cause minor or - FEB 2014
C. Increased nitrogen bubbles serious damage in almost any area of the body, MED
D. Metabolic alkalosis depending on the number and sizes of bubbles formed. BOARDS;
E. Severe respiratory acidosis TOPNOTCH
MD)
460 The role of hydrochloric acid in gastric juice is to: When stimulated, the parietal cells secrete an acid JULIET MIDTERM 2
A. Activate pepsinogen to pepsin solution that has a pH of 0.8 demonstrating an extreme KRISTINE EXAM - AUG
B. Protect the gastric mucosa acidity. When pepsinogen is first secreted, it has no EVANGELIST 2014
C. Open pyloric sphincter digestive activity. As soon as it comes in contact with A, MD (TOP 9
D. Prevent esophageal reflux hydrochloric acid, it is activated to form pepsin because - FEB 2014
E. All of the above it only functions as a proteolytic enzyme in a highly MED
acidic medium. BOARDS;
TOPNOTCH
MD)
461 In Cushing syndrome, dorsocervical fat pad is due to: dorsocervical fat pad is due to faster fat generation than LUISA BACK-UP
A. Increase gluconeogenesis its metabolism in some tissues. Other choices are just SARANILLO, MIDTERM
B. Increase lipolysis effect of cortisol but will not specifically cause MD (TOP 6 - EXAM AUG
C. Increase protein catabolism in muscles dorsocervical fat pad. FEB 2014 2014 - FOR
D. Fat generation is faster than metabolism MED INCLUSION IN
E. Increase sodium retention BOARDS; THE SAMPLEX
TOPNOTCH
MD)
462 Hormones and neurotransmitters are synthesized Proteins bound for the cell membrane, lysosomes, and LUISA BACK-UP
by: outside of the cell such as hormones and SARANILLO, MIDTERM
A. Ribosomes of the RER neurotransmitters are synthesized by the ribosomes of MD (TOP 6 - EXAM AUG
B. Smooth endoplasmic reticulum the RER. Free floating ribosomes synthesized proteins FEB 2014 2014 - FOR
C. Free floating ribosomes bound for the cytoplasm and mitochondria. SER MED INCLUSION IN
D. Golgi apparatus synthesized lipids. Golgi apparatus is for tagging and BOARDS; THE SAMPLEX
E. Lysosomes packaging. Lysosomes for autolysis. TOPNOTCH
MD)

TOPNOTCH MEDICAL BOARD PREP PHYSIOLOGY SUPEREXAM Page 56 of 88


For inquiries visit www.topnotchboardprep.com.ph or email us at topnotchmedicalboardprep@gmail.com
TOPNOTCH MEDICAL BOARD PREP PHYSIOLOGY SUPEREXAM
For inquiries visit www.topnotchboardprep.com.ph or email us at topnotchmedicalboardprep@gmail.com
Item QUESTION EXPLANATION AUTHOR TOPNOTCH
# EXAM
463 Osmolarity is defined as: Osmolarity is defined as osmoles/liter of water. LUISA BACK-UP
A. Osmoles/L of water Osmolality is defined as osmoles/kilogram of water. SARANILLO, MIDTERM
B. Osmoles/kg of water MD (TOP 6 - EXAM AUG
C. Osmoles/pound of water FEB 2014 2014 - FOR
D. Osmoles/gallon of water MED INCLUSION IN
E. None of the choices BOARDS; THE SAMPLEX
TOPNOTCH
MD)
464 A 40 year old man was brought in to the emergency tetanus and botulism utilized the retrograde axonal LUISA BACK-UP
department with trismus, rigidity and photophobia. transport that is from the axon terminal to the cell body. SARANILLO, MIDTERM
What is the axonal transport utilized by the MD (TOP 6 - EXAM AUG
organism causing the disease? FEB 2014 2014 - FOR
A. Anterograde transport MED INCLUSION IN
B. Orthograde transport BOARDS; THE SAMPLEX
C. retrograde transport TOPNOTCH
D. primary transport MD)
E. none of the choices

465 The "happy hormone" involved in mood and sleep is serotonin is the happy hormone involved in mood and LUISA BACK-UP
mainly produced by: sleep which is mainly secreted by the median raphe of SARANILLO, MIDTERM
A. Pineal gland the brainstem. Pineal gland produce melatonin; MD (TOP 6 - EXAM AUG
B. Median raphe of the brainstem tuberomammillary nucleus of the hypothalamus secretes FEB 2014 2014 - FOR
C. Tuberomammillary nucleus of the hypothalamus mainly histamine; nucleus basalis of Meynert secretes MED INCLUSION IN
D. Nucleus basalis of Meynert mainly acetylcholine; substantia nigra secretes BOARDS; THE SAMPLEX
E. Substantia nigra dopamine TOPNOTCH
MD)
466 When you walk straight and change direction In general, there are 3 fiber types - A, B, C. Type A is the LUISA BACK-UP
suddenly without falling into the ground, fastest which is further subdivided into 4. SARANILLO, MIDTERM
maintaining balance, what nerve fiber type is used? Proprioception, and motor use the type A alpha fiber. MD (TOP 6 - EXAM AUG
A. A Type B fiber is used by all preganglionic autonomic FEB 2014 2014 - FOR
B. B sytem. Type C is for slow pain, temperature, and MED INCLUSION IN
C. C postganglionic sympathetic. BOARDS; THE SAMPLEX
D. D TOPNOTCH
E. E MD)

467 A 50 year old male American travels to the jet lag is due to alteration of circadian rhythm when one LUISA BACK-UP
Philippines to visit his girlfiend. When he arrived, he travels across time zones. It is treated with melatonin SARANILLO, MIDTERM
had a jet lag. What is the treatment? and sunlight exposure MD (TOP 6 - EXAM AUG
A. Place him in a dark room for at least 24 hrs FEB 2014 2014 - FOR
B. anxiolytic drugs MED INCLUSION IN
C. melatonin and/or sunlight exposure BOARDS; THE SAMPLEX
D. tell him to go back to his place of origin TOPNOTCH
E. just observe MD)

468 During your duty in Baguio, you noted the there are 3 thermal receptors: warmth, cold, and pain LUISA BACK-UP
temperature at 12 deg. C. What thermal receptor is receptors. Pain receptor is activated at temperature SARANILLO, MIDTERM
mainly activated at this temperature? below 15deg. C and above 43 deg. C. MD (TOP 6 - EXAM AUG
A. Pain receptor FEB 2014 2014 - FOR
B. Warmth receptor MED INCLUSION IN
C. Cold receptor BOARDS; THE SAMPLEX
D. freezing receptor TOPNOTCH
E. Cold and freezing receptors MD)

469 The brain is a relatively small organ but is highly 15% to the brain; 5% to the heart; 5% to the skin; 25% LUISA BACK-UP
metabolic. How many percent of blood flows to the each to the kidney, GIT, and skeletal muscle. SARANILLO, MIDTERM
brain? MD (TOP 6 - EXAM AUG
A. 25% FEB 2014 2014 - FOR
B. 20% MED INCLUSION IN
C. 5% BOARDS; THE SAMPLEX
D. 15% TOPNOTCH
E. 10% MD)

470 Patient complained of dyspnea and peripheral LUISA BACK-UP


edema. You suspect for a possible cardiac problem. SARANILLO, MIDTERM
You request for 2D echo. What parameter is MD (TOP 6 - EXAM AUG
considered as the marker for cardiac function or FEB 2014 2014 - FOR
contractility? MED INCLUSION IN
A. Cardiac output BOARDS; THE SAMPLEX
B. Ejection fraction TOPNOTCH
C. stroke volume MD)
D. End diastolic volume
E. heart rate
471 A 24 year old male came in for an annual PE. You 1st heart sound is heard during isovolumic contraction. LUISA BACK-UP
perform auscultation of the heart, and you heard the 2nd heart sound is heard during isovolumic relaxation. SARANILLO, MIDTERM
first heart sound which is heard in what phase of the 3rd heart sound is heard during rapid inflow. 4th heart MD (TOP 6 - EXAM AUG
cardiac cycle? soound maybe heard during atrial systole FEB 2014 2014 - FOR
A. Isovolumic contraction MED INCLUSION IN
B. Isovolumic relaxation BOARDS; THE SAMPLEX
C. rapid inflow TOPNOTCH
D. Ejection MD)
E. Atrial systole

TOPNOTCH MEDICAL BOARD PREP PHYSIOLOGY SUPEREXAM Page 57 of 88


For inquiries visit www.topnotchboardprep.com.ph or email us at topnotchmedicalboardprep@gmail.com
TOPNOTCH MEDICAL BOARD PREP PHYSIOLOGY SUPEREXAM
For inquiries visit www.topnotchboardprep.com.ph or email us at topnotchmedicalboardprep@gmail.com
Item QUESTION EXPLANATION AUTHOR TOPNOTCH
# EXAM
472 A 19 year old male lacerated his wrist cutting a macula densa is part of the JG apparatus. It is found in LUISA BACK-UP
major blood vessel was bleeding profusely. Heart the distal tubule of the kidney. SARANILLO, MIDTERM
rate was 120bpm and BP was 80/60mmHg. The MD (TOP 6 - EXAM AUG
decreased BP is detected by macula densa which is FEB 2014 2014 - FOR
found in: MED INCLUSION IN
A. Proximal tubule BOARDS; THE SAMPLEX
B. Collecting ducts TOPNOTCH
C. distal tubule MD)
D. loop of henle
E. vasa recta
473 Which statement is true during skeletal muscle During skeletal muscle contraction, there is no LUISA BACK-UP
contraction? shortening of actin and myosin, what happens is just SARANILLO, MIDTERM
A. There is shortening of the sarcomere, actin and sliding of actin and myosin, what actually shortens is the MD (TOP 6 - EXAM AUG
myosin sarcomere. FEB 2014 2014 - FOR
B. There is shortening of sarcomere, but not of MED INCLUSION IN
actin and myosin BOARDS; THE SAMPLEX
C. There is shortening of actin and myosin but not TOPNOTCH
of sarcomere MD)
D. There is shortening of actin and myosin, and
lengthening of sarcomere
E. There is shortening of sarcomere, and
lengthening of actin and myosin
474 What energy system is used when you play physical activities which last for 10 sec. use phosphagen LUISA BACK-UP
badminton? energy. Those activities lasting for 1.6min like SARANILLO, MIDTERM
A. Phosphagen energy system badminton use glycogen-lactic acid. Those lasting for MD (TOP 6 - EXAM AUG
B. Glycogen-lactic acid system more than 2 min or more use aerobic system. FEB 2014 2014 - FOR
C. Aerobic system MED INCLUSION IN
D. All of the above BOARDS; THE SAMPLEX
E. A and C TOPNOTCH
MD)
475 In a patient with hypovolemic shock, what part of PCT is the workhorse of the nephron which is higly LUISA BACK-UP
the kidney is the most susceptible to hypoxia? metabolic, thus considered to be the most susceptible to SARANILLO, MIDTERM
A. Collecting duct hypoxia. MD (TOP 6 - EXAM AUG
B. Loop of henle, descending limb FEB 2014 2014 - FOR
C. Distal convoluted tubule MED INCLUSION IN
D. Proximal convoluted tubule BOARDS; THE SAMPLEX
E. Loop of henle, ascending limb TOPNOTCH
MD)

476 A 52 year old male patient came in due to dyspnea. normal ABG result: pH=7.35-7.45; H+ =40meq/L; LUISA BACK-UP
ABG result showed: pH of 7.2, pCO2 of 50mmHg, H+ pCO2=40mmHg; HCO3=24meq/L. In acidosis, pH is SARANILLO, MIDTERM
of 45meq/L, and HCO3 of 30meq/L. What is the acid below normal, and H+ is above normal; the cause is MD (TOP 6 - EXAM AUG
- base abnormality? respiratory if pCO2 is above normal, and as renal FEB 2014 2014 - FOR
A. Respiratory acidosis compensation, HCO3 is also above normal; it is metabolic MED INCLUSION IN
B. Metabolic acidpsis in nature if HCO3 is below normal, and as pulmonary BOARDS; THE SAMPLEX
C. Respiratory alkalosis compensation pCO2 is also below normal. When the pH TOPNOTCH
D. metabolic alkalosis is above normal, it is alkalosis. MD)
E. normal result, no abnormality

477 Bone marrow starts to form blood cells at what age? bone marrow starts to form blood cells at 4th month of LUISA BACK-UP
A. 3rd week of fetal development fetal development and become the major hematopoeitic SARANILLO, MIDTERM
B. 4th week of fetal development organ postnatally. MD (TOP 6 - EXAM AUG
C. 3rd month of fetal development FEB 2014 2014 - FOR
D. 4th month of fetal development MED INCLUSION IN
E. At birth BOARDS; THE SAMPLEX
TOPNOTCH
MD)
478 What is the life span of platelet? The average life span of platelet is 7-10days LUISA BACK-UP
A. 7-9 days SARANILLO, MIDTERM
B. 11-12 days MD (TOP 6 - EXAM AUG
C. 13-14 days FEB 2014 2014 - FOR
D. 3-5 days MED INCLUSION IN
E. 21 days BOARDS; THE SAMPLEX
TOPNOTCH
MD)
479 What is the clotting factor IV? clotting factor IV is calcium. LUISA BACK-UP
A. Labile factor SARANILLO, MIDTERM
B. Conversion factor MD (TOP 6 - EXAM AUG
C. Calcium FEB 2014 2014 - FOR
D. Hageman factor MED INCLUSION IN
E. Magnesium BOARDS; THE SAMPLEX
TOPNOTCH
MD)
480 What happens if after puberty, the male is castrated? If male is castrated after puberty, there is loss of body LUISA BACK-UP
A. Increase in body hair hair, musculature, thick bones, and decreased libido. SARANILLO, MIDTERM
B. Increased libido Erection still happens but without ejaculation. MD (TOP 6 - EXAM AUG
C. Can not erect FEB 2014 2014 - FOR
D. Can not ejaculate MED INCLUSION IN
E. All choices are correct BOARDS; THE SAMPLEX
TOPNOTCH
MD)

TOPNOTCH MEDICAL BOARD PREP PHYSIOLOGY SUPEREXAM Page 58 of 88


For inquiries visit www.topnotchboardprep.com.ph or email us at topnotchmedicalboardprep@gmail.com
TOPNOTCH MEDICAL BOARD PREP PHYSIOLOGY SUPEREXAM
For inquiries visit www.topnotchboardprep.com.ph or email us at topnotchmedicalboardprep@gmail.com
Item QUESTION EXPLANATION AUTHOR TOPNOTCH
# EXAM
481 A 40-year-old male was stabbed by an assailant. He Hypovolemic shock, like in massive hemorrhage, will ANGELIS FINAL EXAM -
was found to have continuous bleeding. At the ER, increase renin activity and increase conversion of ANDREA AUG 2014
his bp was found to be 80/60 mmHg. Which of the angiotensin I to angiotensin II. Urine osmolality will COCOS, MD
following is expected in this patient? increase because the body's tendency is to conserve (TOP 1 - FEB
A. Decrease in renin activity water (due to increased aldosterone). 2014 MED
B. Increase in urine osmolality BOARDS;
C. Increased conversion of angiotensin II to TOPNOTCH
angiotensin I MD)
D. None of the above
482 A previously normal newborn in a community This case describes an infant with a ductal-dependent ANGELIS FINAL EXAM -
hospital is noted at 14 hours of life to be cyanotic. cyanotic congenital heart lesion. The ductus arteriosus ANDREA AUG 2014
She is placed on facemask but she remains cyanotic typically closes in the first few hours causing the COCOS, MD
and her pulse oximetry reading does not change. cyanosis of this neonate. Prostaglandin will help keep (TOP 1 - FEB
There are clear bilateral breath sounds and she has the ductus patent until a definitive procedure can be 2014 MED
no murmur. You are concerned about a congenital performed. BOARDS;
heart disease. You should initiate which of the TOPNOTCH
following? MD)
A. Indomethacin infusion
B. Saline infusion
C. Prostaglandin E1 infusion
D. Adenosine infusion
483 Which among the anterior pituitary hormone is Among the pituitary hormones only prolactin is under ANGELIS FINAL EXAM -
under tonic inhibitory control? tonic inhibition. ANDREA AUG 2014
A. FSH COCOS, MD
B. LH (TOP 1 - FEB
C. PRL 2014 MED
D. GH BOARDS;
TOPNOTCH
MD)
484 A 67-year-old patient presents with fever and Inadequate oxygenation caused by the low PO2 results ANGELIS FINAL EXAM -
respiratory distress. She was noted to have multiple to increased lactic acid causing metabolic acidosis. The ANDREA AUG 2014
infiltrates on chest radiograph. ABG on room air low PCO2 reflects hyperventilation, secondary to the COCOS, MD
shows pH of 7.1, PO2 35 mmHg, and PaCO2 28 respiratory difficulty, and is a respiratory compensation (TOP 1 - FEB
mmHg. These values indicate? for metabolic acidosis. 2014 MED
A. metabolic acidosis, respiratory alkalosis and BOARDS;
hypoxia TOPNOTCH
B. respiratory acidosis, metabolic alkalosis and MD)
hypoxia
C. primary respiratory alkalosis
D. acidosis with compensatory hypoventilation
485 A 20-year-old athlete is accidentally hit in the Trauma can cause pancreatitis. No diagnostic test is ANGELIS FINAL EXAM -
abdomen by a baseball bat. Over the 24 hours, he completely accurate but serum lipase is more specific for ANDREA AUG 2014
develops fever, abdominal pain radiating to the back pancreatitis compared to serum amylase. COCOS, MD
and persistent vomiting. The abdomen is tender (TOP 1 - FEB
with decreased bowel sounds. Which of the 2014 MED
following tests is most likely to confirm the BOARDS;
diagnosis? TOPNOTCH
A. serum lipase MD)
B. serum amylase
C. serum bilirubin
D. electrolyte panel
486 There is increased steroid production in the All these organs are steroid-producing. SIMILAR TO ANGELIS FINAL EXAM -
following organ/s: PREVIOUS BOARD EXAM CONCEPT/PRINCIPLE ANDREA AUG 2014
A. adrenals COCOS, MD
B. testes (TOP 1 - FEB
C. ovary 2014 MED
D. all of the above BOARDS;
TOPNOTCH
MD)
487 A patient in a coma with severe brain injury has This patient likely has developed diabetes insipidus, a ANGELIS FINAL EXAM -
developed very dilute urine with an output of common complication of severe head trauma, due to a ANDREA AUG 2014
100ml/h. This condition can be managed by which deficiency in secretion of antidiuretic hormone. COCOS, MD
of the following? (TOP 1 - FEB
A. Fluid restriction 2014 MED
B. Giving insulin BOARDS;
C. Giving synthetic vasopression intranasally TOPNOTCH
D. Replacement with D5water and normal saline MD)
488 A 4-year-old child came into your clinic with blue The basic defect of osteogenesis imperfecta is an ANGELIS FINAL EXAM -
sclerae and a history of multiple fractures with abnormality in the production and composition of the ANDREA AUG 2014
minimal trauma. The expected serum concentration matrix of the bone. Both calcium and phosphate COCOS, MD
of calcium and phosphate is: concentrations remain normal. SIMILAR TO PREVIOUS (TOP 1 - FEB
A. Low phosphate, normal calcium BOARD EXAM CONCEPT/PRINCIPLE. 2014 MED
B. Normal phosphate, normal calcium BOARDS;
C. Normal phosphate, low calcium TOPNOTCH
D. High phosphate, low calcium MD)

489 This type of RTA is due to a defect in proximal tubule Type 1 is known as distal RTA which is due to a defect in ANGELIS FINAL EXAM -
bicarbonate reabsorption. Untreated patients have a the collecting tubule's ability to excrete hydrogen ions. ANDREA AUG 2014
urine pH of <5.5: Type 4 is due to hypoaldosteronism or the lack of COCOS, MD
A. Type 1 collecting tubule response to aldosterone. There is no (TOP 1 - FEB
B. Type 2 Type 3 RTA. 2014 MED
C. Type 3 BOARDS;
D. Type 4 TOPNOTCH
MD)

TOPNOTCH MEDICAL BOARD PREP PHYSIOLOGY SUPEREXAM Page 59 of 88


For inquiries visit www.topnotchboardprep.com.ph or email us at topnotchmedicalboardprep@gmail.com
TOPNOTCH MEDICAL BOARD PREP PHYSIOLOGY SUPEREXAM
For inquiries visit www.topnotchboardprep.com.ph or email us at topnotchmedicalboardprep@gmail.com
Item QUESTION EXPLANATION AUTHOR TOPNOTCH
# EXAM
490 The hormone which is responsible for a positive HCG is produced by the syncytiotrophoblast. ANGELIS FINAL EXAM -
pregnancy test is secreted by: ANDREA AUG 2014
A. the syncytiotrophoblast COCOS, MD
B. the cytotrophoblast (TOP 1 - FEB
C. the yolk sac 2014 MED
D. the blastula BOARDS;
TOPNOTCH
MD)
491 A castrated male is expected to: Castration, the removal of testicles where testosterone is ANGELIS FINAL EXAM -
A. Have no erection produced, in an adult male results in partial loss of his ANDREA AUG 2014
B. Have complete loss of libido sex drive or libido (but not complete). However, he can COCOS, MD
C. Still have an orgasm still have sex, erection and orgasm. SIMILAR TO (TOP 1 - FEB
D. Have reversal of secondary sexual PREVIOUS BOARD EXAM CONCEPT/PRINCIPLE. 2014 MED
characteristics BOARDS;
TOPNOTCH
MD)
492 During exercise, all are expected, EXCEPT: With exercise, there is vasodilation of apical capillaries, ANGELIS FINAL EXAM -
A. Increase in respiratory rate resulting in a V/Q ratio that approaches 1. ANDREA AUG 2014
B. V/Q ratio approaches zero COCOS, MD
C. Increase in CO2 production (TOP 1 - FEB
D. Blood pH decreases 2014 MED
BOARDS;
TOPNOTCH
MD)
493 In the lungs, oxygenation of hemoglobin promotes Contrast this to Bohr effect which happens in the ANGELIS FINAL EXAM -
dissociation of H+ from hemoglobin. This shifts peripheral tissues. There is unloading of O2 due to ANDREA AUG 2014
equilibrium toward CO2 formation, therefore CO2 is increased hydrogen from tissue metabolism. SIMILAR COCOS, MD
released from RBC. This is known as the: TO PREVIOUS BOARD EXAM CONCEPT/PRINCIPLE (TOP 1 - FEB
A. Haldane effect 2014 MED
B. Gibbs Donnan effect BOARDS;
C. Bohr effect TOPNOTCH
D. Boyle effect MD)

494 The c wave in jugular venous pulse is secondary to: The c wave reflects RV contraction (closed tricuspid ANGELIS FINAL EXAM -
A. Atrial contraction valve bulging into atrium). ANDREA AUG 2014
B. Atrial relaxation COCOS, MD
C. RV contraction (TOP 1 - FEB
D. Blood flow from RA to RV 2014 MED
BOARDS;
TOPNOTCH
MD)
495 The T wave in the electrocardiogram represents: The QRS complex represents: ventricular depolarization; ANGELIS FINAL EXAM -
A. Ventricular repolarization P wave: atrial depolarization; atrial repolarization is ANDREA AUG 2014
B. Atrial repolarization buried under the QRS complex. This is a must-know COCOS, MD
C. Ventricular depolarization concept. (TOP 1 - FEB
D. Atrial depolarization 2014 MED
BOARDS;
TOPNOTCH
MD)
496 The following are absorbed in the ileum, EXCEPT: Iron is absorbed mainly in the duodenum and upper ANGELIS FINAL EXAM -
A. Bile salts jejunum. ANDREA AUG 2014
B. Vitamin B12 COCOS, MD
C. Vitamin K (TOP 1 - FEB
D. Iron 2014 MED
BOARDS;
TOPNOTCH
MD)
497 This hormone increases pancreatic bicarbonate and Secretin increases pancreatic bicarbonate and bile ANGELIS FINAL EXAM -
bile secretion: secretion, while it decreases gastric acid secretion. ANDREA AUG 2014
A. Gastrin COCOS, MD
B. Secretin (TOP 1 - FEB
C. Cholecystokinin 2014 MED
D. VIP BOARDS;
TOPNOTCH
MD)
498 A decrease in ESR is expected in: A decrease in ESR is expected in polycythemia, sickle cell ANGELIS FINAL EXAM -
A. SLE anemia, congestive heart failure and ANDREA AUG 2014
B. URTI hypofibrinogenemia. COCOS, MD
C. Sickle cell anemia (TOP 1 - FEB
D. Pregnancy 2014 MED
BOARDS;
TOPNOTCH
MD)
499 Vitamin K deficiency results in a decrease in the Vitamin-K dependent factors are: factors X, IX, VII, II, ANGELIS FINAL EXAM -
following, EXCEPT: (10972) protein C and S. ANDREA AUG 2014
A. Factor II COCOS, MD
B. Protein C (TOP 1 - FEB
C. Factor I 2014 MED
D. Factor X BOARDS;
TOPNOTCH
MD)

TOPNOTCH MEDICAL BOARD PREP PHYSIOLOGY SUPEREXAM Page 60 of 88


For inquiries visit www.topnotchboardprep.com.ph or email us at topnotchmedicalboardprep@gmail.com
TOPNOTCH MEDICAL BOARD PREP PHYSIOLOGY SUPEREXAM
For inquiries visit www.topnotchboardprep.com.ph or email us at topnotchmedicalboardprep@gmail.com
Item QUESTION EXPLANATION AUTHOR TOPNOTCH
# EXAM
500 Which is TRUE regarding Type 1 muscle fibers? Type 1 muscle fibers are slow twitch red fibers resulting ANGELIS FINAL EXAM -
A. They are fast twitch fibers. from increased mitochondria and myoglobin ANDREA AUG 2014
B. They have increased mitochondrial concentration. COCOS, MD
concentration. (TOP 1 - FEB
C. They have decreased myoglobin concentration. 2014 MED
D. They are known as white fibers. BOARDS;
TOPNOTCH
MD)
501 Which of the following coincides with the second IM platinum p42 JAN BACK-UP
heart sound? CHARMAINE MIDTERM
A. Closure of mitral valve and tricuspid valves PALOMAR, EXAM AUG
B. Closure of the aortic and pulmonic valves MD (TOP 9 - 2014
C. Flow of blood during rapid ventricular filling FEB 2014
D. Late diastole or atrial systole (slow ventricular MED
filling) BOARDS;
E. None of the above TOPNOTCH
MD)
502 Which of the following correctly depicts the formula BP=CO x TPR; CO= HR x SV JAN BACK-UP
for blood pressure? CHARMAINE MIDTERM
A. Blood pressure= cardiac output x heart rate PALOMAR, EXAM AUG
B. Blood pressure= heart rate X cardiac output MD (TOP 9 - 2014
C. Blood pressure= stroke volume X total FEB 2014
peripheral resistance X heart rate MED
D. Blood pressure= cardiac output x stroke volume BOARDS;
E. Blood pressure= total peripheral resistance x TOPNOTCH
stroke volume MD)
503 Continued production of this estrogen depends on JAN BACK-UP
the living fetus, hence is used as a marker of fetal CHARMAINE MIDTERM
well-being: PALOMAR, EXAM AUG
A. Estrone MD (TOP 9 - 2014
B. Estradiol FEB 2014
C. Estriol MED
D. All of the above BOARDS;
E. None of the above TOPNOTCH
MD)
504 Which of the following statements regarding B. Depolarization is making the mambrane potential JAN BACK-UP
depolarization is true? more negative. - Hyperpolarization CHARMAINE MIDTERM
A. Depolarization is making the membrane C. Depolarization is the membrane potential in which PALOMAR, EXAM AUG
potential less negative. occurrence of action potential is inevitable. - Threshold MD (TOP 9 - 2014
B. Depolarization is making the mambrane D. Depolarization is the period in which a new action FEB 2014
potential more negative. potential can be elicited. - no answer MED
C. Depolarization is the membrane potential in E. Depolarization is the period in which a new action BOARDS;
which occurrence of action potential is inevitable. potential can not be elicited. -Refractory period TOPNOTCH
D. Depolarization is the period in which a new MD)
action potential can be elicited.
E. Depolarization is the period in which a new
action potential can not be elicited.
505 Which of the following nerve fiber type conducts Type C- slowest, non-myelinated JAN BACK-UP
action potential the fastest? CHARMAINE MIDTERM
A. Type Aα PALOMAR, EXAM AUG
B. Type Aβ MD (TOP 9 - 2014
C. Type B FEB 2014
D. Type C MED
E. None of the above BOARDS;
TOPNOTCH
MD)
506 This lobe of the brain is responsible for hearing, Frontal- motor, personality, calculation Parietal- JAN BACK-UP
vestibular processing, recognition of faces and Somatosensory Occipital-Vision CHARMAINE MIDTERM
storage of memory? PALOMAR, EXAM AUG
A. Frontal lobe MD (TOP 9 - 2014
B. Parietal lobe FEB 2014
C. Temporal lobe MED
D. Occipital lobe BOARDS;
E. None of the above TOPNOTCH
MD)
507 Lesions in this part of the brain causes loss of pre- Thalamic lesions-cause retrograde amnesia JAN BACK-UP
existing memories or retrograde amnesia: Hippocampal lesions- cause anterograde amnesia CHARMAINE MIDTERM
A. Thalamus PALOMAR, EXAM AUG
B. Hippocampus MD (TOP 9 - 2014
C. Amygdala FEB 2014
D. Hypothalamus MED
E. None of the above BOARDS;
TOPNOTCH
MD)
508 These brain waves predominate in a person who is A. Alpha waves- awake, eyes closed; relaxing person JAN BACK-UP
doing his thesis at 10am in the morning: B. Beta waves- awake, eyes open; busy and alert CHARMAINE MIDTERM
A. Alpha waves C. Theta waves-brain disorders and degenerative brain PALOMAR, EXAM AUG
B. Beta waves states MD (TOP 9 - 2014
C. Theta waves D. Delta waves-deep sleep and organic brain disease FEB 2014
D. Delta waves and infants MED
E. None of the above BOARDS;
TOPNOTCH
MD)

TOPNOTCH MEDICAL BOARD PREP PHYSIOLOGY SUPEREXAM Page 61 of 88


For inquiries visit www.topnotchboardprep.com.ph or email us at topnotchmedicalboardprep@gmail.com
TOPNOTCH MEDICAL BOARD PREP PHYSIOLOGY SUPEREXAM
For inquiries visit www.topnotchboardprep.com.ph or email us at topnotchmedicalboardprep@gmail.com
Item QUESTION EXPLANATION AUTHOR TOPNOTCH
# EXAM
509 This tactile receptor is an onion-like structure in the B. Ruffini's end organs- detects pressure and joint JAN BACK-UP
skin and deep fascia that detects high-frequency rotation CHARMAINE MIDTERM
vibration: C. Merkel's Disc- for localizing touch sensation and to PALOMAR, EXAM AUG
A. Pacinian corpuscle determine texture MD (TOP 9 - 2014
B. Ruffini's end organs D. Meissner's corpuscle-found in non-hairy skin as FEB 2014
C. Merkel's Disc finger tips; for low-frequency vibration MED
D. Meissner's corpuscle E. Free nerve endings-found in the skin; touch and BOARDS;
E. Free nerve endings pressure TOPNOTCH
MD)
510 These blood vessels are also known as capacitance Veins are easily distensible. It holds more than 60% of JAN BACK-UP
vessels because of its function as reservoir of blood: blood at any time. CHARMAINE MIDTERM
A. Arterioles PALOMAR, EXAM AUG
B. Arteries MD (TOP 9 - 2014
C. Veins FEB 2014
D. Capilaries MED
E. None of the above BOARDS;
TOPNOTCH
MD)
511 What does Tall T waves in contiguous leads in an B. Hypercalcemia- shortened QT interval JAN BACK-UP
ECG tracing imply? C. Hypokalemia- flat T waves CHARMAINE MIDTERM
A. Hyperkalemia D. Hypocalcemia- Prolonged QT interval PALOMAR, EXAM AUG
B. Hypercalcemia MD (TOP 9 - 2014
C. Hypokalemia FEB 2014
D. Hypocalcemia MED
E. None of the above BOARDS;
TOPNOTCH
MD)
512 The following are part of the Cushing reflex in Cushing reflex: HPN, Bradycardia, Irregular respiration JAN BACK-UP
response to increased intracranial pressure, except: CHARMAINE MIDTERM
A. Irregular respiration PALOMAR, EXAM AUG
B. Bradycardia MD (TOP 9 - 2014
C. Hypotension FEB 2014
D. All of the above MED
E. None of the above BOARDS;
TOPNOTCH
MD)
513 The following vasocative mediators are vasodilators, Serotonin- is released as a result of blood vessel damage; JAN BACK-UP
except: it causes arteriolar vasoconstriction; implicated in CHARMAINE MIDTERM
A. Serotonin migraineq PALOMAR, EXAM AUG
B. Prostacyclin MD (TOP 9 - 2014
C. Nitric oxide FEB 2014
D. Histamine MED
E. None of the above BOARDS;
TOPNOTCH
MD)
514 The following are true of slow-twitch or red muscles, fast-twitch/ white muscle=glycolytic; slow-twitch/red JAN BACK-UP
except: muscle=oxidative CHARMAINE MIDTERM
A. Smaller in size PALOMAR, EXAM AUG
B. Glycolytic in metabolic profile MD (TOP 9 - 2014
C. Slower Calcium reuptake by sarcoplasmic FEB 2014
reticulum MED
D. Has more numerous mitochondria BOARDS;
E. For endurance activities TOPNOTCH
MD)
515 What is the function of Troponin I? SIMILAR TO PREVIOUS BOARD EXAM JAN BACK-UP
A. Attaches troponin complex to tropomyosin CONCEPT/PRINCIPLE: Troponin T- attaches troponin CHARMAINE MIDTERM
B. Inhibits actin-myosin binding complex to tropomyosin; Troponin I-inhibits actin- PALOMAR, EXAM AUG
C. Inhibits calcium binding protein myosin binding; Troponin C- calcium binding protein MD (TOP 9 - 2014
D. All of the above FEB 2014
E. None of the above MED
BOARDS;
TOPNOTCH
MD)
516 One of the manifestations of Decompression Chokes-dyspnea; Caisson's-Chronic decompression JAN BACK-UP
sickness caused by sudden formation of nitrogen sickness CHARMAINE MIDTERM
bubbles that blocks blood vessels from rapid ascent PALOMAR, EXAM AUG
is pain in the joints and muscles. What is its other MD (TOP 9 - 2014
name? FEB 2014
A. Chokes MED
B. Turns BOARDS;
C. Bends TOPNOTCH
D. Caisson's Disease MD)
E. All of the above
517 The following statements are true regarding the Right is lower (because of the liver) JAN BACK-UP
kidneys, except: CHARMAINE MIDTERM
A. Left kidney is lower than the right kidney. PALOMAR, EXAM AUG
B. Glomerular capillaries are the only capillaries MD (TOP 9 - 2014
that lead to artetioles. FEB 2014
C. Kidneys produce calcitriol, renin and MED
erythropoietin. BOARDS;
D. Peritubular capillaries participate in TOPNOTCH
countercurrent exchange. MD)
E. None of the above

TOPNOTCH MEDICAL BOARD PREP PHYSIOLOGY SUPEREXAM Page 62 of 88


For inquiries visit www.topnotchboardprep.com.ph or email us at topnotchmedicalboardprep@gmail.com
TOPNOTCH MEDICAL BOARD PREP PHYSIOLOGY SUPEREXAM
For inquiries visit www.topnotchboardprep.com.ph or email us at topnotchmedicalboardprep@gmail.com
Item QUESTION EXPLANATION AUTHOR TOPNOTCH
# EXAM
518 This refers to the intrinsic ability of the tubules to Glomerulotubular balance- percentage of solute JAN BACK-UP
increase their reabsorption rate in response to reabsorbed is held constant Tubuloglomerular CHARMAINE MIDTERM
increased tubular load: feedback-constant sodium load delivered to distal tubule PALOMAR, EXAM AUG
A. Glomerulotubular balance MD (TOP 9 - 2014
B. Tubuloglomerular feedback FEB 2014
C. Gradient-time transport MED
D. Renal blood flow BOARDS;
E. None of the above TOPNOTCH
MD)
519 The following promotes entry of potassium into Factors that shift potassium out of cells: insulin JAN BACK-UP
cells, except: deficiency, addison's, B-blockers, acidosis, cell lysis, CHARMAINE MIDTERM
A. Insulin strenous exercise, increased ECF osmolality PALOMAR, EXAM AUG
B. Aldosterone MD (TOP 9 - 2014
C. Beta agonists FEB 2014
D. Alkalosis MED
E. None of the above BOARDS;
TOPNOTCH
MD)
520 What is the last nucleated stage of RBC? Orthochromatic erythroblast-last nucleated stage JAN BACK-UP
A. Proerythroblast CHARMAINE MIDTERM
B. Basophilic eryhtroblast PALOMAR, EXAM AUG
C. Polychromatophilic erythroblast MD (TOP 9 - 2014
D. Orthochromatic erythroblast FEB 2014
E. Reticulocyte MED
BOARDS;
TOPNOTCH
MD)
521 An 87-year-old nursing home resident with MIGUEL MIDTERM 1
dementia, Alzheimer's type, is admitted to the RAFAEL EXAM - FEB
hospital because of progressive lethargy and RAMOS, MD 2013
decreased appetite for 3 days. She had a flu-like (TOP 3 - FEB
illness followed by a deep cough 1 week ago. Over 2012 MED
the past 10 months, she has been hospitalized once BOARDS;
for bacterial pneumonia. She is responsive only to TOPNOTCH
painful stimuli. Her temperature is 38.6 C (101.5 F), MD)
blood pressure is 110/60 mm Hg, pulse is 123/min
and regular, and respirations are 28/min.
Examination shows dry mucous membranes. There
is no adenopathy. Crackles are heard in the right
lung base. An x-ray film of the chest shows an
infiltrate at the right lung base. The remainder of the
examination shows no abnormalities. Which of the
following is the most likely predisposing factor for
this patient's pneumonia?
A ) Decreased airway elasticity
B ) Decreased baroreflex
C ) Decreased gag reflex
D ) Decreased thyroid function
522 A 72-year-old man comes to the physician because MIGUEL MIDTERM 1
of a 7-month history of leg weakness and dry eyes RAFAEL EXAM - FEB
and mouth. He also has had a 10.4-kg (23-lb) weight RAMOS, MD 2013
loss over the past 4 months despite no change in (TOP 3 - FEB
appetite. He has smoked one and a half packs of 2012 MED
cigarettes daily for 50 years. He drinks 4 oz of BOARDS;
alcohol daily. He has peptic ulcer disease and TOPNOTCH
emphysema. Medications include cimetidine, MD)
theophylline, and low-dose prednisone. Examination
shows mild ptosis. He has a barrel-shaped chest.
Breath sounds are distant. There is moderate
weakness of the proximal muscles of the lower
extremities. Reflexes are absent. He has difficulty
rising from a chair. Sensory examination shows no
abnormalities. An x-ray film shows a hyperinflated
chest and a 3 x 4-cm mass in the right hilum. His
neurologic findings are most likely due to a lesion
involving which of the following?
A) Choline acetyltransferase
B) Acetylcholinesterase
C) Postynaptic neuromuscular junction
D) Presynaptic neuromuscular junction

TOPNOTCH MEDICAL BOARD PREP PHYSIOLOGY SUPEREXAM Page 63 of 88


For inquiries visit www.topnotchboardprep.com.ph or email us at topnotchmedicalboardprep@gmail.com
TOPNOTCH MEDICAL BOARD PREP PHYSIOLOGY SUPEREXAM
For inquiries visit www.topnotchboardprep.com.ph or email us at topnotchmedicalboardprep@gmail.com
Item QUESTION EXPLANATION AUTHOR TOPNOTCH
# EXAM
523 A 37-year-old woman comes to the physician MIGUEL MIDTERM 1
because of progressive shortness of breath over the RAFAEL EXAM - FEB
past 5 years; she now has fatigue and shortness of RAMOS, MD 2013
breath with mild exertion. She has a history of mitral (TOP 3 - FEB
stenosis secondary to rheumatic fever at the age of 2012 MED
15 years. She was asymptomatic until 5 years ago BOARDS;
when she developed severe shortness of breath TOPNOTCH
during pregnancy. She was treated with diuretics, MD)
low-sodium diet, and bed rest, and she was able to
deliver the baby at term. Her only medication is
hydrochlorothiazide. Her temperature is 37 C (98.6
F), blood pressure is 110/80 mm Hg, pulse is
100/min and regular, and respirations are 26/min.
Cardiac examination shows an obvious opening snap
in S2. A grade 3/6, late diastolic murmur is heard at
the apex. A right ventricular lift is palpated along the
left sternal border. Which of the following is most
likely increased in this patient?
A) Blood flow to the lower lung fields
B) Pulmonary artery pressure
C) Left-to-right shunt of blood
D) Left ventricular end-diastolic pressure
524 A 52-year-old scuba diver has a malfunctioning MIGUEL MIDTERM 1
oxygen tank at 80 feet below sea level. Panicking RAFAEL EXAM - FEB
about this occurrence, she made every effort to RAMOS, MD 2013
immediately resurface. Upon reaching the surface, (TOP 3 - FEB
she had severe difficulty of breathing. Which among 2012 MED
the following explains why this happened to her? BOARDS;
A) Nitrogen microbubbles plugging the pulmonary TOPNOTCH
capillaries MD)
B) Swallowing excess amounts of sea water
C) Nitrogen narcosis in the central nervous system
D) Free radical damage to the pulmonary
vasculature
525 A newborn infant is born with 5-alpha reductase MIGUEL MIDTERM 1
deficiency. Examination of his blood would reveal RAFAEL EXAM - FEB
decreased levels of which among the following RAMOS, MD 2013
compounds? (TOP 3 - FEB
A) Androstenedione 2012 MED
B) Dihydrotestosterone BOARDS;
C) Estradiol TOPNOTCH
D) Pregnenolone MD)

526 Which among the following physiologic principles MIGUEL MIDTERM 1


pertains to iodine-induced hyperthyroidism, RAFAEL EXAM - FEB
typically presenting in a patient with endemic goiter RAMOS, MD 2013
who relocates to an iodine-abundant geographical (TOP 3 - FEB
area? 2012 MED
A) Wolff-Chaikoff effect BOARDS;
B) Frank-Starling mechanism TOPNOTCH
C) Jod-Basedow phenomenon MD)
D) Bainbridge reflex

527 Aldosterone synthase is only present in one portion MIGUEL MIDTERM 1


of the adrenal gland. This explains why this is the RAFAEL EXAM - FEB
only portion capable of producing aldosterone, a RAMOS, MD 2013
mineralocorticoid hormone that is essential for life. (TOP 3 - FEB
Which among the following is the site of aldosterone 2012 MED
production? BOARDS;
A) Zona glomerulosa TOPNOTCH
B) Zona fasciculate MD)
C) Zona reticularis
D) Adrenal cortex

528 Without a countercurrent exchanger, the solutes MIGUEL MIDTERM 1


pumped into the renal medulla by the RAFAEL EXAM - FEB
countercurrent multiplier system would be rapidly RAMOS, MD 2013
dissipated. Which among the following structures is (TOP 3 - FEB
responsible for this phenomenon in human 2012 MED
nephrons? BOARDS;
A) Loop of Henle TOPNOTCH
B) Proximal convoluted tubule MD)
C) Vasa recta
D) Distal convoluted tubule

529 Which of the following refers to the intrinsic ability MIGUEL MIDTERM 1
of tubules to increase their reabsorption rate in RAFAEL EXAM - FEB
response to increased tubular load? RAMOS, MD 2013
A) Tubuloglomerular Feedback (TOP 3 - FEB
B) Glomerulotubular Balance 2012 MED
C) Countercurrent Multiplication BOARDS;
D) Local Autoregulation TOPNOTCH
MD)

TOPNOTCH MEDICAL BOARD PREP PHYSIOLOGY SUPEREXAM Page 64 of 88


For inquiries visit www.topnotchboardprep.com.ph or email us at topnotchmedicalboardprep@gmail.com
TOPNOTCH MEDICAL BOARD PREP PHYSIOLOGY SUPEREXAM
For inquiries visit www.topnotchboardprep.com.ph or email us at topnotchmedicalboardprep@gmail.com
Item QUESTION EXPLANATION AUTHOR TOPNOTCH
# EXAM
530 A 45-year-old man who underwent bowel surgery MIGUEL MIDTERM 1
developed pernicious anemia as a consequence. RAFAEL EXAM - FEB
Which among the following will most likely be RAMOS, MD 2013
observed in this patient? (TOP 3 - FEB
A) Low dietary intake of ferrous sulfate 2012 MED
B) Hypertrophy of gastric mucosa BOARDS;
C) Excess levels of pyridoxine in the blood TOPNOTCH
D) Decreased production of hydrochloric acid by MD)
gastric parietal cells
531 A 34-year-old man is suffering from gastrointestinal MIGUEL MIDTERM 1
complications. On examination of his RAFAEL EXAM - FEB
gastrointestinal fluids, gastric fluids were noted to RAMOS, MD 2013
have increased [H+], bile had decreased [HCO3] and (TOP 3 - FEB
pancreatic fluids had decreased [HCO3]. Which 2012 MED
among the following GIT hormones is most likely BOARDS;
deficient in this patient? TOPNOTCH
A) Cholecystokinin MD)
B) Gastrin
C) Glucose-dependent insulinotropic peptide (GIP)
D) Secretin
532 Respiratory distress developed in a neonate born at MIGUEL MIDTERM 1
30 weeks age of gestation. On chest radiograph, RAFAEL EXAM - FEB
areas of atelectasis were seen. Which among the RAMOS, MD 2013
following represents the dominant component of the (TOP 3 - FEB
deficient pulmonary surfactant in this patient? 2012 MED
A) Phosphatidylcholine BOARDS;
B) Phosphatidylserine TOPNOTCH
C) Phosphatidylinositol MD)
D) Phosphatidylpalmitate

533 The affinity of oxygen for hemoglobin increases in MIGUEL MIDTERM 1


which of the following situations? RAFAEL EXAM - FEB
A) Hypothermia RAMOS, MD 2013
B) Metabolic acidosis (TOP 3 - FEB
C) Exercise 2012 MED
D) Chronic hypoxemia BOARDS;
TOPNOTCH
MD)
534 Following a sympathectomy, a 66-year-old man MIGUEL MIDTERM 1
experiences orthostatic hypotension. Which among RAFAEL EXAM - FEB
the following may explain this occurrence? RAMOS, MD 2013
A) An exaggerated response of the renin- (TOP 3 - FEB
angiotensin-aldosterone system 2012 MED
B) A suppressed response of the renin-angiotensin- BOARDS;
aldosterone system TOPNOTCH
C) A suppressed response of the baroreceptor MD)
mechanism
D) An exaggerated response of the baroreceptor
mechanism
535 A 65-year-old female who had a traumatic brain MIGUEL MIDTERM 1
injury seems to be able to understand the written RAFAEL EXAM - FEB
and spoken word but cannot create the correct RAMOS, MD 2013
sounds to be able to speak a word that is (TOP 3 - FEB
recognizable. She most likely has damage to which 2012 MED
area of the brain? BOARDS;
A) Wernicke’s area TOPNOTCH
B) Broca’s area MD)
C) Angular gyrus
D) Arcuate fasciculus
536 The myelin sheath of nerves is produced by MIGUEL MIDTERM 1
supportive cells like oligodendrocytes in the central RAFAEL EXAM - FEB
nervous system and Schwann cells in the peripheral RAMOS, MD 2013
nervous system. The following are consequences of (TOP 3 - FEB
myelination of nerve fibers EXCEPT? 2012 MED
A) Decrease in nonselective diffusion of ions across BOARDS;
the axonal membrane TOPNOTCH
B) Increase in energy required to maintain ion MD)
gradients across the membrane
C) Increase in the velocity of nerve impulses along
the axon
D) Generation of action potentials only at nodes of
Ranvier
537 Which among the following transport mechanisms is MIGUEL MIDTERM 1
limited by a saturable transport maxima? RAFAEL EXAM - FEB
A) Facilitated diffusion RAMOS, MD 2013
B) Simple diffusion (TOP 3 - FEB
C) Osmosis 2012 MED
D) Filtration BOARDS;
TOPNOTCH
MD)

TOPNOTCH MEDICAL BOARD PREP PHYSIOLOGY SUPEREXAM Page 65 of 88


For inquiries visit www.topnotchboardprep.com.ph or email us at topnotchmedicalboardprep@gmail.com
TOPNOTCH MEDICAL BOARD PREP PHYSIOLOGY SUPEREXAM
For inquiries visit www.topnotchboardprep.com.ph or email us at topnotchmedicalboardprep@gmail.com
Item QUESTION EXPLANATION AUTHOR TOPNOTCH
# EXAM
538 Skeletal muscle contraction will cease and relaxation MIGUEL MIDTERM 1
will commence when calcium ions are pumped back RAFAEL EXAM - FEB
into which of the following structures? RAMOS, MD 2013
A) Endoplasmic reticulum (TOP 3 - FEB
B) Transverse tubules 2012 MED
C) Neuromuscular junction BOARDS;
D) Sarcoplasmic reticulum TOPNOTCH
MD)

539 A 22-year-old man is brought to the emergency MIGUEL MIDTERM 1


room after a traffic accident causing a traumatic RAFAEL EXAM - FEB
brain injury. Within several hours he begins eating RAMOS, MD 2013
objects such as paper, is unable to maintain (TOP 3 - FEB
attention, and displays increased sexual activity. A 2012 MED
diagnostic MRI reveals bilateral amygdala lesions. BOARDS;
Which among the following best explains this TOPNOTCH
patient's condition? MD)
A) Parkinson's disease
B) Kluver-Bucy syndrome
C) Gerstmann's syndrome
D) Brown-Sequard syndrome

540 Why is atrial repolarization not visible in a normal MIGUEL MIDTERM 1


ECG tracing? RAFAEL EXAM - FEB
A) Because it is buried in the concurrent QRS RAMOS, MD 2013
complex (TOP 3 - FEB
B) Because it is isoelectric and hence cannot be seen 2012 MED
C) Because it is of too low voltage to be detected by BOARDS;
the ECG TOPNOTCH
D) Because it occurs together with ventricular MD)
repolarization
541 A 26 year primigravid patient is about to deliver her Oxytocin is produced in the magnocellular cells of the ABDELSIMA FINAL EXAM -
baby. During your labor watch you notice that the paraventricular nuclei R OMAR II, FEB 2014
uterine contractions occur at >20 minute interval. MD (TOP 2 -
You decided to do nipple massage to stimulate the AUG 2013
release of a hormone which would improve uterine MED
contrations. Which of the following stuctures/areas BOARDS;
in the hypothalamus releases this hormone? TOPNOTCH
A. Gigantocellular Cells in Paraventricular Nuclei MD - 200
B. Magnocellular Cells in Paraventricular Nuclei QUESTIONS)
C. Magnocellular cells in the Supraoptic Nuclei AND MARC
D. Gigantocellular cells in the supraoptic nuclei DENVER
E. None of the Above TIONGSON,
MD (40
QUESTIONS)
542 Which of the following CORRECTLY describes the A - distal tubule B- increases the permeability to urea ABDELSIMA FINAL EXAM -
function of vasopressin in the following organ R OMAR II, FEB 2014
systems? MD (TOP 2 -
A. Kidney: increases the water permeability in the AUG 2013
proximal tubule and collecting duct which allows MED
reabsorption and excretion of a more concentrated BOARDS;
urine TOPNOTCH
B. Kidney: decreases the permeability of the inner MD - 200
medullary portion of the collecting duct to urea QUESTIONS)
C. Central Nervous System: implicated in motor AND MARC
movements DENVER
D. Cardiovascular: increases peripheral vascular TIONGSON,
resistence therefore increasing blood pressure MD (40
E. All of the above QUESTIONS)
543 A young soccer player had an accidental collision commotio cordis - v fib blunt trauma just before the peak ABDELSIMA FINAL EXAM -
with his opponent during the game. After the of t wave R OMAR II, FEB 2014
collision, the player suddenly dropped down dead MD (TOP 2 -
due to a fatal arrythmia. This condition is triggered AUG 2013
by a a blunt trauma to the heart during the MED
susceptible phase of the heart. This MOST BOARDS;
COMMONLY occurs if the blunt trauma occurs: TOPNOTCH
A. before the peak of the p wave MD - 200
B. before the peak of the r wave QUESTIONS)
C. before the start of the qrs complex AND MARC
D. before the peak of the t wave DENVER
E. before the peak of the u wave TIONGSON,
MD (40
QUESTIONS)
544 An average individual has a respiratory rate of 16 Va = RR x (VT-VD) ABDELSIMA FINAL EXAM -
breaths per minute. His measured tidat volume is R OMAR II, FEB 2014
500ml. What is the alveolar ventilation of this MD (TOP 2 -
patient? AUG 2013
A. 7L MED
B. 6.5L BOARDS;
C. 6L TOPNOTCH
D. 5.5L MD - 200
E. 5L QUESTIONS)
AND MARC
DENVER
TIONGSON,
MD (40

TOPNOTCH MEDICAL BOARD PREP PHYSIOLOGY SUPEREXAM Page 66 of 88


For inquiries visit www.topnotchboardprep.com.ph or email us at topnotchmedicalboardprep@gmail.com
TOPNOTCH MEDICAL BOARD PREP PHYSIOLOGY SUPEREXAM
For inquiries visit www.topnotchboardprep.com.ph or email us at topnotchmedicalboardprep@gmail.com
Item QUESTION EXPLANATION AUTHOR TOPNOTCH
# EXAM
QUESTIONS)

545 Hypoventilation causes the oxy-hemoglobin curve to ABDELSIMA FINAL EXAM -


shift to the: R OMAR II, FEB 2014
A. Right MD (TOP 2 -
B. Left AUG 2013
C. Upward MED
D. Downward BOARDS;
E. Change TOPNOTCH
MD - 200
QUESTIONS)
AND MARC
DENVER
TIONGSON,
MD (40
QUESTIONS)
546 Which of the following decreases Cobalamin Ascorbic acid decreases the absorption of cobalamin ABDELSIMA FINAL EXAM -
(vitamin b12) in the body? R OMAR II, FEB 2014
A. Vitamin A MD (TOP 2 -
B. Vitamin D AUG 2013
C. Vitamin C MED
D. Vitamin E BOARDS;
E. Vitamin K TOPNOTCH
MD - 200
QUESTIONS)
AND MARC
DENVER
TIONGSON,
MD (40
QUESTIONS)
547 Which of the following describes the effect on All of the above are ECG findings in hyperkalemia ABDELSIMA FINAL EXAM -
hyperkalemia on a patient's electrocardiogram? R OMAR II, FEB 2014
A. Reduction of size of the P wave MD (TOP 2 -
B. Development of peaked T waves AUG 2013
C. Widening of the QRS complex MED
D. Evolution to a sinusoidal shape BOARDS;
E. All of the above TOPNOTCH
MD - 200
QUESTIONS)
AND MARC
DENVER
TIONGSON,
MD (40
QUESTIONS)
548 A patient with congestive heart failure with EF<40% Spironolactone - inhibits Na absorption and K excretion ABDELSIMA FINAL EXAM -
was prescribed with a drug that causes by inhibiting aldosterone R OMAR II, FEB 2014
gynecomastia. This drug affects one of the transport MD (TOP 2 -
mechanisms in the kidney. Which of the following is AUG 2013
affected by this drug? MED
A. Inhibit H+ secretion and HCO3- reabsorption BOARDS;
B. Inhibit Na+Cl- co-transport TOPNOTCH
C. Inhibit Na+K+Cl- co-transport MD - 200
D. Inhibit Na+ reabsorption and K+ excretion QUESTIONS)
E. None of the above AND MARC
DENVER
TIONGSON,
MD (40
QUESTIONS)
549 A 23 year old male is suffering from a large tumor When the afferent and efferent nerves are both ABDELSIMA FINAL EXAM -
located at the cauda equina. Which of the following destroyed, as they may be by tumors of the cauda equina R OMAR II, FEB 2014
describes the effect of a compressive tumor to the or filum terminale, the bladder is flaccid and distended MD (TOP 2 -
bladder? for a while. Gradually, however, the muscle of the AUG 2013
A. The bladder becomes initially flaccid and "decentralized bladder" becomes active, with many MED
distended contraction waves that expel dribbles of urine out of the BOARDS;
B. The bladder may become active and expels urine urethra. The bladder becomes shrunken and the bladder TOPNOTCH
C. The bladder may be shrunked wall hypertrophied. MD - 200
D. The bladder may hypertrophy QUESTIONS)
E. All of the above AND MARC
DENVER
TIONGSON,
MD (40
QUESTIONS)

TOPNOTCH MEDICAL BOARD PREP PHYSIOLOGY SUPEREXAM Page 67 of 88


For inquiries visit www.topnotchboardprep.com.ph or email us at topnotchmedicalboardprep@gmail.com
TOPNOTCH MEDICAL BOARD PREP PHYSIOLOGY SUPEREXAM
For inquiries visit www.topnotchboardprep.com.ph or email us at topnotchmedicalboardprep@gmail.com
Item QUESTION EXPLANATION AUTHOR TOPNOTCH
# EXAM
550 An individual was brought into the emergency ABDELSIMA FINAL EXAM -
department following a motorbike collision. He was R OMAR II, FEB 2014
conscious and reports no loss of consciousness after MD (TOP 2 -
the incident. There is no physical evidence of any AUG 2013
head trauma. However, upon examination of this MED
individual, the following sensory deficits are noted: BOARDS;
loss of fine touch and vibration sense in both the left TOPNOTCH
arm and leg including the hand and foot, loss of pain MD - 200
and temperature sense in the right arm and leg QUESTIONS)
including the hand and foot. In the description AND MARC
above, one can RULE OUT a lesion in which of the DENVER
following areas of the nervous system? TIONGSON,
A. Midbrain MD (40
B. Medulla QUESTIONS)
C. Brainstem
D. Cortex
E. Peripheral nerves
551 A 24 year old female presents with inappropriate ABDELSIMA FINAL EXAM -
eating and she puts everything in her mouth. Patient R OMAR II, FEB 2014
also has difficulty in recognizing individuals. You are MD (TOP 2 -
suspecting a bilateral lesion in one of the structures AUG 2013
of the limbic system. The structure involved in this MED
case is also important in? BOARDS;
A. Comprehension of auditory and visual function TOPNOTCH
B. Transfer of information between 2 hemisphere MD - 200
C. Consolidation of memory QUESTIONS)
D. Recognizing severe pain and fear AND MARC
E. None of the above DENVER
TIONGSON,
MD (40
QUESTIONS)
552 The human brain has the ability to determine which ABDELSIMA FINAL EXAM -
stimulus is important and should always be R OMAR II, FEB 2014
recognized and which stimulus can be ignored. A MD (TOP 2 -
perfect example is that after wearing your clothes AUG 2013
for quite sometime, your body do recognize the MED
"feel" of your clothes to your skin. This process is BOARDS;
best explained by which of the following concepts? TOPNOTCH
A. Habituation MD - 200
B. Sensitization QUESTIONS)
C. Classical conditioning AND MARC
D. Operant conditioning DENVER
E. Learning TIONGSON,
MD (40
QUESTIONS)
553 Circadian rhythms are highly influenced by the Tryptophan ABDELSIMA FINAL EXAM -
external environment. It is mainly regulated by a R OMAR II, FEB 2014
hormone which is stimulated by darkness. From MD (TOP 2 -
which of the following amino acids is this hormone AUG 2013
synthesized from? MED
A. Tyrosine BOARDS;
B. Serotonin TOPNOTCH
C. Melanin MD - 200
D. Glycine QUESTIONS)
E. Tryptophan AND MARC
DENVER
TIONGSON,
MD (40
QUESTIONS)
554 The stucture of the limbic system which receives ABDELSIMA FINAL EXAM -
neuronal signals from all portions of the cortices is R OMAR II, FEB 2014
involved in which of the following functions? MD (TOP 2 -
A. Tranquility AUG 2013
B. Voluntary Muscle movement MED
C. Severe Pain BOARDS;
D. Rewards Center TOPNOTCH
E. None of the above MD - 200
QUESTIONS)
AND MARC
DENVER
TIONGSON,
MD (40
QUESTIONS)
555 Fast pain, usually described as sharp and localized, is ABDELSIMA FINAL EXAM -
carried by which of the following? R OMAR II, FEB 2014
A. small, unmyelinated C fibers MD (TOP 2 -
B. small, myelinated A-delta fibers AUG 2013
C. small, unmyelinated C fibers MED
D. large, myelinated A-beta fibers BOARDS;
E. large, unmyelinated C fibers TOPNOTCH
MD - 200
QUESTIONS)
AND MARC
DENVER
TIONGSON,
MD (40

TOPNOTCH MEDICAL BOARD PREP PHYSIOLOGY SUPEREXAM Page 68 of 88


For inquiries visit www.topnotchboardprep.com.ph or email us at topnotchmedicalboardprep@gmail.com
TOPNOTCH MEDICAL BOARD PREP PHYSIOLOGY SUPEREXAM
For inquiries visit www.topnotchboardprep.com.ph or email us at topnotchmedicalboardprep@gmail.com
Item QUESTION EXPLANATION AUTHOR TOPNOTCH
# EXAM
QUESTIONS)

556 43 year old patient who presents with malar rash, Cushing's syndrome ABDELSIMA FINAL EXAM -
anemia, chronic kidney disease and positive ANA R OMAR II, FEB 2014
presented is being treated with prednisone. Which MD (TOP 2 -
of the following is/are known potential problem/s AUG 2013
of chronic prednisone/steroid use? MED
A. Hypertension BOARDS;
B. Hyperkalemia TOPNOTCH
C. Thickening of the skin MD - 200
D. Bone formation QUESTIONS)
E. Hypercalcemia AND MARC
DENVER
TIONGSON,
MD (40
QUESTIONS)
557 A 73 year old diabetic came in at the E.R. For HHS ABDELSIMA FINAL EXAM -
decrease in sensorium. Initial CBG revealed "Hi." The R OMAR II, FEB 2014
E.R. Physician suspected a possible diabetic MD (TOP 2 -
emergency in this case. On History, it was noted that AUG 2013
the patient had severe diarrhea for the past few MED
days. Which of the following Blood Gas is consistent BOARDS;
with the patient's condition? TOPNOTCH
A. pH 7.32 pCO2 18 HCO3 12 MD - 200
B. pH 7.32 pCO2 80 HCO3 32 QUESTIONS)
C. pH 7.36 pCO2 35 HCO3 20 AND MARC
D. pH 7.47 pCO2 25 HCO3 28 DENVER
E. pH 7.40 pCO2 40 HCO3 22 TIONGSON,
MD (40
QUESTIONS)
558 Growth hormone will DECREASE which of the ABDELSIMA FINAL EXAM -
following: R OMAR II, FEB 2014
A. Uptake of glucose into the cell MD (TOP 2 -
B. Mobilization of fat from adipose tissue AUG 2013
C. Amino acid transport into the cell MED
D. Transcription of DNA to RNA BOARDS;
E. All of the above TOPNOTCH
MD - 200
QUESTIONS)
AND MARC
DENVER
TIONGSON,
MD (40
QUESTIONS)
559 Which of the following is a physiologic action of ABDELSIMA FINAL EXAM -
progesterone: R OMAR II, FEB 2014
A. Promote proliferative changes in the MD (TOP 2 -
endometrium AUG 2013
B. Increased frequency and intensity of uterine MED
contraction BOARDS;
C. Stimulate the development of lobules and alveoli TOPNOTCH
in the breast MD - 200
D. Increased osteoblastic activity QUESTIONS)
E. None of the above AND MARC
DENVER
TIONGSON,
MD (40
QUESTIONS)
560 A 36 year old male was diagnosed with AML. He is ABDELSIMA FINAL EXAM -
admitted for blood transfusion due to low R OMAR II, FEB 2014
hemoglobin. The NOD called your attention because MD (TOP 2 -
the patient is suffering from fever. Transfusion was AUG 2013
immediately stopped. Upon attending to the patient MED
and looking at the chart, you noted that the patient BOARDS;
has a blood type "O neg". You then checked the blood TOPNOTCH
transfused and you noted that it was type "O neg" as MD - 200
well. Which among the following best explains the QUESTIONS)
incident? AND MARC
A. Preformed antibodies against donor RBCs DENVER
B. undetected allo-Abs against minor antigens TIONGSON,
C. Abs againts donor WBCs and cytokines released in MD (40
blood product QUESTIONS)
D. IgA deficiency with IgA abs
E. Rh incompatibility

TOPNOTCH MEDICAL BOARD PREP PHYSIOLOGY SUPEREXAM Page 69 of 88


For inquiries visit www.topnotchboardprep.com.ph or email us at topnotchmedicalboardprep@gmail.com
TOPNOTCH MEDICAL BOARD PREP PHYSIOLOGY SUPEREXAM
For inquiries visit www.topnotchboardprep.com.ph or email us at topnotchmedicalboardprep@gmail.com
Item QUESTION EXPLANATION AUTHOR TOPNOTCH
# EXAM
561 When molecules or ions move uphill against a Active transport is the process of moving substances BLAKE MIDTERM 2
concentration gradient, the process is called uphill against concentration gradient. WARREN EXAM - FEB
A. simple diffusion ANG, MD 2014
B. facilitated diffusion (TOP 1 - AUG
C. osmosis 2013 MED
D. active transport BOARDS;
TOPNOTCH
MD)

562 Parkinson’s disease results from loss of Dopamine deficiency in the basal ganglia characterize BLAKE MIDTERM 2
A. cell bodies of GABA-secreting neurons in the Parkinsons disease WARREN EXAM - FEB
caudate nucleus ANG, MD 2014
B. Ach-secreting neurons in the many parts of the (TOP 1 - AUG
brain 2013 MED
C. dopamine due to destruction of substantia nigra BOARDS;
D. serotonin due to destruction of the brainstem TOPNOTCH
MD)
563 Causes of true visceral pain include: All of the above cause visceral pain BLAKE MIDTERM 2
A. Ischemia WARREN EXAM - FEB
B. Spasm of hollow viscus ANG, MD 2014
C. Chemical stimuli (TOP 1 - AUG
D. All of the above 2013 MED
BOARDS;
TOPNOTCH
MD)
564 Mechanism involved when heat is lost thru Radiation is the mechanism of loss of heat through BLAKE MIDTERM 2
electromagnetic rays is by: noncontiguous surfaces WARREN EXAM - FEB
A. convection ANG, MD 2014
B. radiation (TOP 1 - AUG
C. evaporation 2013 MED
D. conduction BOARDS;
TOPNOTCH
MD)
565 The attraction of the white blood cells to the site of BLAKE MIDTERM 2
injury is called WARREN EXAM - FEB
A. diapedesis ANG, MD 2014
B. margination (TOP 1 - AUG
C. chemotaxis 2013 MED
D. phagocytosis BOARDS;
TOPNOTCH
MD)
566 The process of gas exchange in the lungs is by Answer: Simple diffusion allows the freely permeable BLAKE MIDTERM 2
A. simple diffusion oxygen gas to penetrate through the alveolar wall from WARREN EXAM - FEB
B. facilitated diffusion high to low concentration gradient. ANG, MD 2014
C. endocytosis (TOP 1 - AUG
D. filtration 2013 MED
BOARDS;
TOPNOTCH
MD)
567 The following have direct effects on the respiratory Both CO2 and H+ exert their effects in the Respiratory BLAKE MIDTERM 2
center in the medulla center WARREN EXAM - FEB
A. Carbon Dioxide ANG, MD 2014
B. Oxygen (TOP 1 - AUG
C. Hydrogen ions 2013 MED
D. A and C BOARDS;
TOPNOTCH
MD)
568 The heart vector extends mainly in the same Lead II, based on Einthoven’s triangle BLAKE MIDTERM 2
direction as the axis of Lead WARREN EXAM - FEB
A. I ANG, MD 2014
B. II (TOP 1 - AUG
C. III 2013 MED
D. AVL BOARDS;
TOPNOTCH
MD)
569 True for Diabetics : Answer: In DM2, FFA is increased in both plasma levels BLAKE MIDTERM 2
A. have decreased plasma amino acid concentrations and its usage. A, B, and D are not effects of insulin WARREN EXAM - FEB
B. have low amounts of acetyl-coA deficiency ANG, MD 2014
C. increases the use of free fatty acids in most tissues (TOP 1 - AUG
D. increases protein synthesis 2013 MED
BOARDS;
TOPNOTCH
MD)
570 Severe vasoconstriction of efferent arterioles will Answer: By virtue of Gibbs-Donnan phenomenon, severe BLAKE MIDTERM 2
cause: efferent arteriolar vasoconstriction causes decreases WARREN EXAM - FEB
a. Increase in glomerular filtration rate GFR. However, slight to moderate resistance increase in ANG, MD 2014
b. Decrease in tubular reabsorption efferent arterioles consequently increases GFR. (TOP 1 - AUG
c. Decrease in glomerular filtration rate 2013 MED
d. Increase in renal plasma flow BOARDS;
TOPNOTCH
MD)

TOPNOTCH MEDICAL BOARD PREP PHYSIOLOGY SUPEREXAM Page 70 of 88


For inquiries visit www.topnotchboardprep.com.ph or email us at topnotchmedicalboardprep@gmail.com
TOPNOTCH MEDICAL BOARD PREP PHYSIOLOGY SUPEREXAM
For inquiries visit www.topnotchboardprep.com.ph or email us at topnotchmedicalboardprep@gmail.com
Item QUESTION EXPLANATION AUTHOR TOPNOTCH
# EXAM
571 The following will cause increased thirst except: Answer: Increase in serum osmolarity, decrease in BV, BLAKE MIDTERM 2
A. Increased osmolarity INCREASE angiotensin and dehydration all activate the WARREN EXAM - FEB
B. Decreased blood volume thirst mechanism. ANG, MD 2014
C. Decreased angiotensin (TOP 1 - AUG
D. Dryness of mouth 2013 MED
BOARDS;
TOPNOTCH
MD)
572 A deep sea welder sought consultation to physician Answer: Nitrogen comprises the majority of compressed BLAKE MIDTERM 2
because of difficulty of breathing after work. Which air inhaled from the tank. It is the primary gas being WARREN EXAM - FEB
of the following will be the most likely cause of his dissolved at high pressures and is the one responsible ANG, MD 2014
symptom? Pulmonary capillaries are blocked by air for decompression sickness with rapid ascent. (TOP 1 - AUG
bubbles primarily composed of 2013 MED
A. Oxygen BOARDS;
B. Nitrogen TOPNOTCH
C. Helium MD)
D. Carbon Dioxide

573 Destruction of parietal cells as seen in chronic Answer: Instrinsic Factor is produced by the same cell BLAKE MIDTERM 2
gastritis is accompanied by decreased production of that secretes HCl. Atrophy of these cells cause reduction WARREN EXAM - FEB
HCl and in B12 absorption in the ileum. ANG, MD 2014
A. intrinsic factor (TOP 1 - AUG
B. pepsin 2013 MED
C. bicarbonate BOARDS;
D. gastrin TOPNOTCH
MD)

574 Which of the following substances would have a Answer: Under normal conditions, glucose is fully BLAKE MIDTERM 2
renal clearance of 0? reabsorbed in the PCT. All the others are secreted in the WARREN EXAM - FEB
A. inulin urine. ANG, MD 2014
B. para – aminohippuric acid (TOP 1 - AUG
C. glucose 2013 MED
D. creatinine BOARDS;
TOPNOTCH
MD)
575 In which of the following nephron segments is the In the descending loop, there is exclusivity in that only BLAKE MIDTERM 2
tubular filtrate always hyperosmotic? water is permeable while solutes are contained within WARREN EXAM - FEB
A. proximal tubule this segment causing hyperosmotic state in the area. ANG, MD 2014
B. descending loop of Henle (TOP 1 - AUG
C. distal tubule 2013 MED
D. medullary collecting duct BOARDS;
TOPNOTCH
MD)
576 This condition is LEAST likely to stimulate red cell Answer: ESRD causes anemia secondary to reduction or BLAKE MIDTERM 2
production: total absence of EPO secretion. Anemia is universal in WARREN EXAM - FEB
A. Pulmonary disease CKD stage 4. ANG, MD 2014
B. Cardiac failure (TOP 1 - AUG
C. End-stage renal disease 2013 MED
D. Hemorrhagic disease BOARDS;
TOPNOTCH
MD)
577 An unknown red cell solution that agglutinates with Answer: The presence of both A and B antigens are BLAKE MIDTERM 2
the addition of antisera A and antisera B has the responsible for reacting with both anti-A and anti-B test WARREN EXAM - FEB
following phenotype: serum. ANG, MD 2014
A. Blood type O (TOP 1 - AUG
B. Blood type A 2013 MED
C. Blood type B BOARDS;
D. Blood type AB TOPNOTCH
MD)

578 The following condition/s may have depressed Answer: Post-hepatic cirrhosis causes reduction in BLAKE MIDTERM 2
levels of prothrombin resulting to excessive synthesis of vitamin K dependent clotting factors, with WARREN EXAM - FEB
bleeding: prothrombin among them. ANG, MD 2014
A. Bile duct obstruction (TOP 1 - AUG
B. Hemophilia 2013 MED
C. Von Willebrand’s disease BOARDS;
D. Hemolytic uremic syndrome TOPNOTCH
MD)

579 What part of the brain is the principal source of the BLAKE MIDTERM 2
hormone Serotonin? WARREN EXAM - FEB
A.) Raphe nucleus ANG, MD 2014
B.) Basal nuclei (TOP 1 - AUG
C.)cerebellum 2013 MED
D.)pineal gland BOARDS;
TOPNOTCH
MD)
580 In an ECG tracing, the P wave corresponds to: Answer: Atrial depolarization is the upstroke seen in P BLAKE MIDTERM 2
A.)opening of the AV valves and ventricular wave associated almost immediately with mechanical WARREN EXAM - FEB
contraction contraction of the atrium ANG, MD 2014
B.) Atrial repolarization (TOP 1 - AUG
C.) Blood flow from the atria to the ventricles 2013 MED
D.) Atrial contraction BOARDS;
TOPNOTCH
MD)

TOPNOTCH MEDICAL BOARD PREP PHYSIOLOGY SUPEREXAM Page 71 of 88


For inquiries visit www.topnotchboardprep.com.ph or email us at topnotchmedicalboardprep@gmail.com
TOPNOTCH MEDICAL BOARD PREP PHYSIOLOGY SUPEREXAM
For inquiries visit www.topnotchboardprep.com.ph or email us at topnotchmedicalboardprep@gmail.com
Item QUESTION EXPLANATION AUTHOR TOPNOTCH
# EXAM
581 What is the main intracellular anion found in the Chloride ion is the main extracellular anion, organic TIMOTHY MIDTERM 1
body? anions such as porteins are the main intracellular anion. TANG LEE EXAM - FEB
A. Cl- SAY, MD 2014
B. HCO3- (TOP 4 - AUG
C. Organic anions 2013 MED
D. CN- BOARDS;
E. Histones TOPNOTCH
MD)
582 During the fasting state, the counter-regulatory Glucagon acts throught the cAMP pathway using G- TIMOTHY MIDTERM 1
hormones predominate to counter the effects of protein coupled receptors. TANG LEE EXAM - FEB
insulin, the main physiologic antagonist of insulin SAY, MD 2014
acts through this receptor? (TOP 4 - AUG
A. G-protein coupled receptors 2013 MED
B. Intracellular receptors BOARDS;
C. Ion-Channel linked receptors TOPNOTCH
D. Tyrosine kinase receptors MD)
E. Serine/Threonine kinase receptors
583 At resting membrane potential both the Na N gate= K gates; M gate=Na activation gate; H gates=Na TIMOTHY MIDTERM 1
activation gates and the K gates are closed while the inactivation gates TANG LEE EXAM - FEB
Na inactivation gates are open. During Choice A= Resting State SAY, MD 2014
depolarization, which gates are opened? Choice B= Depolarization (TOP 4 - AUG
A. N gates closed; M gates closed; H gates open Choice C= Repolarization 2013 MED
B. N gates closed; M gates open; H gates open D= Undershoot; E = Does not exist physiologically BOARDS;
C. N gates open; M gates open; H gates closed TOPNOTCH
D. N gates open; M gates closed; H gates closed MD)
E. N gates open; M gates open; H gates open

584 Folding a piece of paper into a paper crane uses The motor cortex is responsible for movement but TIMOTHY MIDTERM 1
coordinated movements that is planned by the planning and execution of complex tasks require input TANG LEE EXAM - FEB
cerebral cortex. Which part of the cortex is from the supplementary motor cortex. The premotor SAY, MD 2014
responsible for planning of these complex tasks? cortex keeps track of the location of the trunk and limbs (TOP 4 - AUG
A. Primary motor cortex in relation to the task. It plays a role in the spatial 2013 MED
B. Premotor cortex guidance of movement. The posterior parietal cortex BOARDS;
C. Supplementary motor cortex receives input from the somatosensory cortex, auditory TOPNOTCH
D. Posterior parietal cortex and visual cortex to know the localization of the object MD)
E. Somatosensory cortex and the body in space, and it sends these signals to the
supplementary and premotor cortex.
585 In the representation known as Traquair's Island of The dark dim pit represents the eye's anatomic TIMOTHY MIDTERM 1
Vision, it describes an island of vision in a sea of blindspot which is the optic disk. TANG LEE EXAM - FEB
darkness representing the visual field of an eye. SAY, MD 2014
There is a dark dim pit within the island which is (TOP 4 - AUG
represented by what structure in the eye? 2013 MED
A. Peipheral retina BOARDS;
B. Macula TOPNOTCH
C. Fovea MD)
D. Optic disk
E. Pupillary aperture
586 A patient bleeding profusely was rushed to the ER. A The question is asking where secretion occurred. ADH is TIMOTHY MIDTERM 1
powerful vasoconstrictor in response to the produced in the Supraoptic nucleus, stored and secreted TANG LEE EXAM - FEB
hemorrhage is being secreted by what organ? as needed by the posterior pituitary. SAY, MD 2014
A. Supraoptic nucleus of the hypothalamus (TOP 4 - AUG
B. Paraventricular nucleus of the hypothalamus 2013 MED
C. Anterior Pituitary BOARDS;
D. Posterior Pituitary TOPNOTCH
E. Pineal gland MD)

587 Which of the following is NOT a characteristic of Fast-twitch muscles usually have less mitochondria TIMOTHY MIDTERM 1
Fast-twitch muscle fibers in comparison to Slow- because they rely mostly in the glycolytic rather than the TANG LEE EXAM - FEB
twitch muscle fibers? oxidative pathway. SAY, MD 2014
A. Higher Myosin ATPase activity (TOP 4 - AUG
B. Extraocular muscles are examples 2013 MED
C. More mitochondria BOARDS;
D. Higher Creatine phosphate levels TOPNOTCH
E. All are correct about fast-twitch muscles MD)

588 Which of the following proteins connects myosin to Titin is the largest protein and tethers porteins to the Z TIMOTHY MIDTERM 1
the Z lines? lines. TANG LEE EXAM - FEB
A. Titin SAY, MD 2014
B. Dystrophin (TOP 4 - AUG
C. Actinin 2013 MED
D. Desmin BOARDS;
E. Vimentin TOPNOTCH
MD)
589 Which of the following parameters should increase Poiseuille's Law states that Blood flow is directly TIMOTHY MIDTERM 1
to decrease the blood flow? proportional to pressure difference, radius of the vessel TANG LEE EXAM - FEB
A. Change in pressure and inversely proportional to fliud viscosity and length SAY, MD 2014
B. Diameter of the vessel of the vessel (determinants of resistance). So to decrease (TOP 4 - AUG
C. Fluid viscosity flow, we should increase the determinants of resistance. 2013 MED
D. Turbulent flow BOARDS;
E. Temperature TOPNOTCH
MD)

TOPNOTCH MEDICAL BOARD PREP PHYSIOLOGY SUPEREXAM Page 72 of 88


For inquiries visit www.topnotchboardprep.com.ph or email us at topnotchmedicalboardprep@gmail.com
TOPNOTCH MEDICAL BOARD PREP PHYSIOLOGY SUPEREXAM
For inquiries visit www.topnotchboardprep.com.ph or email us at topnotchmedicalboardprep@gmail.com
Item QUESTION EXPLANATION AUTHOR TOPNOTCH
# EXAM
590 In the cardiac action potential, phase 2 is Phase 2 of the cardiac action potential is the plateau TIMOTHY MIDTERM 1
predominantly attributed to which electrolyte? phase, the primary electrolyte responsible for plateauing TANG LEE EXAM - FEB
A. Na+ influx of the action potential is Ca influx occuring together with SAY, MD 2014
B. Na+ efflux K efflux. K efflux however, is the main electrolyte (TOP 4 - AUG
C. Ca+2 influx responsible for the downsloping of phase 3. 2013 MED
D. K+ influx BOARDS;
E. K+ efflux TOPNOTCH
MD)
591 In a normal kidney, subtances are filtered, Most electrolytes are filtered and partially reabsorbed. TIMOTHY MIDTERM 1
reabsorbed and secreted through the tubules. Which Glucose and AA are fully reabsorbed. Vitamin C is TANG LEE EXAM - FEB
subtances are filtered and partially reabsorbed? secreted. Creatinine is filtered, slightly secreted and not SAY, MD 2014
A. Glucose reabsorbed. (TOP 4 - AUG
B. Amino acid 2013 MED
C. Vitamin C BOARDS;
D. Sodium TOPNOTCH
E. Creatinine MD)

592 In a normal kidney, the arterioles are regulated to Dilating the efferent arteriole would increase RBF while TIMOTHY MIDTERM 1
control the blood flow inspite of changes in blood decreasing GFR. TANG LEE EXAM - FEB
pressure. Which mechanism of vascular regulation SAY, MD 2014
would increase renal blood flow at the cost of (TOP 4 - AUG
glomerular filtration rate? 2013 MED
A. Dilating the afferent arteriole BOARDS;
B. Dilating the efferent arteriole TOPNOTCH
C. Constricting the afferent arteriole MD)
D. Constricting the efferent arteriole
E. Severe constriction of the efferent arteriole
593 Acid-base regulation is important to maintain Patient has a pH of 7.46 (not between 7.35-7.45), so it is TIMOTHY MIDTERM 1
normal pH for the functioning of all enzymes with partially compensated and alkalotic. An increase in TANG LEE EXAM - FEB
only a narrow range compatible to life. ABG is bicarbonate would mean it is metabolic while an SAY, MD 2014
important in monitoring the acid-base balance in decrease in carbon dioxide would mean it is respiratory. (TOP 4 - AUG
certain patient populations. ABG results of pH=7.46; Pls review acid-base derangements, including if it is 2013 MED
PaO2=80 mmHg; HCO3=29 mEq/L; PaCO2=51mmHg; uncompensated, partially or fully compensated. BOARDS;
SaO2=95% are compatible with what abnormality? TOPNOTCH
A. Uncompensated metabolic alkalosis MD)
B. Uncompensated respiratory alkalosis
C. Partially compensated respiratory alkalosis
D. Parially compensated metabolic alkalosis
E. Partially compensated respiratory acidosis

594 What is responsible for normal expiration? Normal expiration is a passive process that is caused by TIMOTHY MIDTERM 1
A. The negative pressure of the lungs pushes air the elastic recoil forces of the lungs and chest wall. TANG LEE EXAM - FEB
towards the outside. SAY, MD 2014
B. The diaphragm pushes the lungs upwards (TOP 4 - AUG
initiating expiration. 2013 MED
C. It is an active process that is initiated by the BOARDS;
distensibility of the lung. TOPNOTCH
D. Abdominal muscles are recruited for quiet MD)
expiration.
E. The elastic recoil of the chest wall force air out.
595 The central chemoreceptors of the central nervous Central chemoreceptors react to increase in carbon TIMOTHY MIDTERM 1
system are located on the ventrolateral medullary dioxide (hypercapnia) and not to hypoxemia, which is TANG LEE EXAM - FEB
surface. They are necessary in controlling responsible for increase firing of the peripheral SAY, MD 2014
ventilation, which stimulus will trigger the central chemoreceptors. (TOP 4 - AUG
chemoreceptors? 2013 MED
A. Hypercapnia BOARDS;
B. Hypoxemia TOPNOTCH
C. Hyperoxemia MD)
D. Hyperventilation
E. Hypoosmolality
596 A 25 year old woman is enjoying herself to a buffet, Histamine and Gastrin would increase stomach acid TIMOTHY MIDTERM 1
celebrating, after having passed the board secretion. Secretin and cholecystokinin are active only TANG LEE EXAM - FEB
examination. She is enjoying herself to a well- once chyme enters the duodenum. Motilin is active only SAY, MD 2014
deserved sumptuous meal, which of the following GI during fasting and PP has no biological function. (TOP 4 - AUG
hormones are active a few minutes after having the 2013 MED
first bite? BOARDS;
A. Secretin TOPNOTCH
B. Cholecystokinin MD)
C. Pancreatic polypetide
D. Motilin
E. Histamine
597 What is the composition of the pancreatic fluid with Pancreatic fluid consist of sodium and potassium ions TIMOTHY MIDTERM 1
regards to plasma? that is the same composition of plasma, bicarbonate TANG LEE EXAM - FEB
A. Na+=plasma; K+=plasma; HCO3->plasma; Cl- higher than plasma and chloride ion lower than plasma SAY, MD 2014
<plasma at any flow rates. At low flow rates, the pancreas (TOP 4 - AUG
B. Na+>plasma; K+<plasma; HCO3->plasma; Cl- secretes an isotonic fluid consists mainly of Na and Cl, at 2013 MED
<plasma higher flow rates, the pancreas secretes an isotonic fluid BOARDS;
C. Na+>plasma; K+>plasma; HCO3->plasma; Cl- that is composed mainly of Na and HCO3-. TOPNOTCH
>plasma MD)
D. Na+>plasma; K+=plasma; HCO3->plasma; Cl-
>plasma
E. Na+=plasma; K+=plasma; HCO3->plasma; Cl-
>plasma

TOPNOTCH MEDICAL BOARD PREP PHYSIOLOGY SUPEREXAM Page 73 of 88


For inquiries visit www.topnotchboardprep.com.ph or email us at topnotchmedicalboardprep@gmail.com
TOPNOTCH MEDICAL BOARD PREP PHYSIOLOGY SUPEREXAM
For inquiries visit www.topnotchboardprep.com.ph or email us at topnotchmedicalboardprep@gmail.com
Item QUESTION EXPLANATION AUTHOR TOPNOTCH
# EXAM
598 Patient with Graves' Disease have overactive thyroid Thyroid hormones bound to nuclear receptors. Thyroid TIMOTHY MIDTERM 1
hormone production due to activating anti-TSH hormone receptors regulate gene expression by binding TANG LEE EXAM - FEB
receptor antibody in the cell membranes. The to hormone response elements (HREs) in DNA SAY, MD 2014
thyroid hormone initiates its action through the use (TOP 4 - AUG
of? 2013 MED
A. Transmembrane receptors BOARDS;
B. Tyrosine kinase receptors TOPNOTCH
C. Mitochondrial membrane receptors MD)
D. Intracytoplasmic receptors
E. Nuclear receptors
599 Which condition is characterized by increasing These are characteristics of primary aldosteronism, an TIMOTHY MIDTERM 1
aldosterone levels and extracellular fluid volume, example of which is Conn's syndrome. TANG LEE EXAM - FEB
but with a decreased in renin levels. There is also no SAY, MD 2014
change in sodium urinary excretion? (TOP 4 - AUG
A. Congestive heart failure 2013 MED
B. Ectopic ACTH secretion BOARDS;
C. Conn's syndrome TOPNOTCH
D. SIADH MD)
E. Cushing's syndrome

600 In the embryological development, what factor is The most important factor in the development of the TIMOTHY MIDTERM 1
responsible for the normal physiological female external genitalia is the absence of testosterone. TANG LEE EXAM - FEB
development of the female external genitalia? The chromosome X and Y is responsible for the SAY, MD 2014
A. The presence of two X chromosomes. development of the gonads. The mullerian inhibiting (TOP 4 - AUG
B. The absence of testosterone. factor is responsible for development of internal 2013 MED
C. The absence of the Y chromosome gentalia. BOARDS;
D. The presence of at least one X chromosome, in TOPNOTCH
the abscence of the Y chromosome MD)
E. The absence of the mullerian inhiibiting factor
601 The lysosome contains the following bactericidal Answer: C. The lysosomes contain bactericidal agents RACHELLE FINAL EXAM -
agents, except: that can kill phagocytized bacteria before they can cause MENDOZA, FEB 2013
A. Lysozyme cellular damage. These agents include (1) lysozyme, MD (TOP 9 -
B. Lysoferrin which dissolves the bacterial cell membrane; (2) AUG 2012
C. Lysolecithin lysoferrin, which binds iron and other substances before MED
D. Acid hydrolase they can promote bacterial growth; and (3) acid at a pH BOARDS;
E. None of the above. of about 5.0, which activates the hydrolases and TOPNOTCH
inactivates bacterial metabolic systems. (Guyton 11th MD)
ed, page 20).
602 What is/are the BEST index/indices of left Answer: A. The best indices of left ventricular preload RACHELLE FINAL EXAM -
ventricular pre-load? are ventricular EDV and EDP. Less reliable indices MENDOZA, FEB 2013
A. Ventricular end-diastolic volume include left atrial pressure, pulmonary venous pressure, MD (TOP 9 -
B. Pulmonary wedge pressure pulmonary wedge pressure. (Kaplan Step 1, lecture AUG 2012
C. Left atrial pressure notes, page71). MED
D. Pulmonary venous pressure BOARDS;
E. B and C TOPNOTCH
MD)
603 Premature ventricular contractions (PVCs) causes Answer: D. PVCs cause specific effects in ECG, as follows: RACHELLE FINAL EXAM -
the following ECG changes, EXCEPT: 1. The QRS complex is usually considerably prolonged. MENDOZA, FEB 2013
A. QRS complex prolongation The reason is that the impulse is conducted mainly MD (TOP 9 -
B. Large QRS complexes through slowly conducting muscle of the ventricles AUG 2012
C. T-wave inversion immediately after a premature rather than through the Purkinje system. 2. The QRS MED
beat complex has a high voltage for the following reasons: BOARDS;
D. None of the above when the normal impulse passes through the heart, it TOPNOTCH
E. A and C passes through both ventricles nearly simultaneously; MD)
consequently, in the normal heart, the depolarization
waves of the two sides of the heart—mainly of opposite
polarity to each other—partially neutralize each other in
the electrocardiogram.When a PVC occurs, the impulse
almost always travels in only one direction, so that there
is no such neutralization effect, and one entire side or
end of the ventricles is depolarized ahead of the other;
this causes large electrical potentials. 3. After almost all
PVCs, the T wave has an electrical potential polarity
exactly opposite to that of the QRS complex, because the
slow conduction of the impulse through the cardiac
muscle causes the muscle fibers that depolarize first also
to repolarize first. (Guyton, 11th ed, Chapter 13, page
151).
604 The principle that in the spinal cord the dorsal roots Answer: A. The principle that in the spinal cord the RACHELLE FINAL EXAM -
are sensory and the ventral roots are motor is dorsal roots are sensory and the ventral roots are motor MENDOZA, FEB 2013
known as the: is known as the Bell-Magendie law (Ganong, 2003). MD (TOP 9 -
A. Bell-Magendie law AUG 2012
B. Maestrini law MED
C. Frank-Starling law BOARDS;
D. Polyvagal principle TOPNOTCH
E. Muller law MD)

TOPNOTCH MEDICAL BOARD PREP PHYSIOLOGY SUPEREXAM Page 74 of 88


For inquiries visit www.topnotchboardprep.com.ph or email us at topnotchmedicalboardprep@gmail.com
TOPNOTCH MEDICAL BOARD PREP PHYSIOLOGY SUPEREXAM
For inquiries visit www.topnotchboardprep.com.ph or email us at topnotchmedicalboardprep@gmail.com
Item QUESTION EXPLANATION AUTHOR TOPNOTCH
# EXAM
605 The basic rhythm of respiration is generated mainly Answer: B. The basic rhythm of respiration is generated RACHELLE FINAL EXAM -
in: mainly in the dorsal respiratory group of neurons. The MENDOZA, FEB 2013
A. Ventral respiratory group, medulla neurons of the ventral respiratory group remain almost MD (TOP 9 -
B. Dorsal respiratory group, medulla totally inactive during normal quiet respiration. AUG 2012
C. Pneumotaxic center, pons Therefore, normal quiet breathing is caused only by MED
D. All of the above repetitive inspiratory signals from the dorsal respiratory BOARDS;
E. A and B only group transmitted mainly to the diaphragm, and TOPNOTCH
expiration results from elastic recoil of the lungs and MD)
thoracic cage. They are especially important in
providing the powerful expiratory signals to the
abdominal muscles during very heavy expiration. The
function of the pneumotaxic center is primarily to limit
inspiration. (Guyton, page 514).
606 A normal 25-year old man has a respiratory rate of Answer: B. Computation is 500ml (normal tidal volume) RACHELLE FINAL EXAM -
14 breaths per minute. What is his alveolar – 150ml (normal anatomic dead space, assuming that MENDOZA, FEB 2013
ventilation? this subject has 0 physiologic dead space) = 350ml x 14 = MD (TOP 9 -
A. 6 L 4,900 ml, rounded off to 5L. AUG 2012
B. 5 L MED
C. 4 L BOARDS;
D. 7 L TOPNOTCH
MD)
607 Which of the following statements is TRUE regarding Answer: D. Voltage-gated Na channels are opened RACHELLE FINAL EXAM -
voltage-gated sodium ion channels? during depolarization. Na-K ATPase pump is the one MENDOZA, FEB 2013
A. They are opened when the membrane is pumping 3 Na out and 2 K in. There is virtually MD (TOP 9 -
hyperpolarized. minimal/negligible conductance of sodium in a resting AUG 2012
B. They pump 3 sodium ions out of the cell and 2 membrane. MED
potassium ions inwards. BOARDS;
C. They display a high conductance in the resting TOPNOTCH
membrane. MD)
D. They are virtually closed when the cell is at rest.
608 The triggering mechanism for the genesis of REM RACHELLE FINAL EXAM -
sleep depends on neurons that release: Answer: A. Physiologically, certain neurons in the brain MENDOZA, FEB 2013
A. Acetylcholine stem, known as REM sleep-on cells, (located in the MD (TOP 9 -
B. Serotonin pontine tegmentum), are particularly active during REM AUG 2012
C. Norepinephrine sleep and are probably responsible for its occurrence. MED
D. Dopamine The release of certain neurotransmitters, the BOARDS;
monoamines(norepinephrine, serotonin and histamine), TOPNOTCH
is completely shut down during REM. Acetylcholine is MD)
released in high levels as a result of wakefulness and
alertness, but it is also found in high levels during REM
sleep. Its lowest levels have been found in slow wave
sleep when there is no cortical arousal, but relaxation.
People that have been unfortunate enough to be exposed
to organophosphates insecticides, which are ACh
agonists, spend more time in REM sleep than individuals
who have not been exposed to these toxins. All this
evidence suggests that ACh is involved in controlling
REM sleep as well as arousal.
609 BB, 35-year old male office worker on his pre- Answer is C. The patient described has acromegaly, RACHELLE FINAL EXAM -
employment check-up, showed marked enlargement which is a result of increased growth hormone release MENDOZA, FEB 2013
of bone of the extremities and significant coarsening from the anterior pituitary gland after the epiphyseal MD (TOP 9 -
of his facial feature. His jaw was likewise enlarged plates have already closed. In over 90 percent of AUG 2012
and protruding. This condition is most commonly acromegaly patients, the overproduction of growth MED
associated with a tumor located in: hormones is caused by a benign tumor of the pituitary BOARDS;
A. Posterior pituitary gland, called an adenoma. TOPNOTCH
B. Adrenal glands MD)
C. Anterior Pituitary
D. Lungs
610 What will be the result of prolonged diastole? Answer: D. Prolonged diastole permits more time for RACHELLE FINAL EXAM -
A. Increase heart rate ventricular filling, hence better filling of the ventricle. MENDOZA, FEB 2013
B. Reduced peripheral resistance Prolonging the diastole will result in decreased heart MD (TOP 9 -
C. Reduced stroke volume rate and increased stroke volume. Duration of diastole AUG 2012
D. Better filling of the ventricles has no effect on peripheral resistance. MED
E. A and D BOARDS;
TOPNOTCH
MD)
611 The sympathetic nervous system modulates the RACHELLE FINAL EXAM -
output of the pacemaker cells of the heart by: Answer: A. Sympathetic activation, which releases MENDOZA, FEB 2013
A. increasing cAMP causing a steep rise in phase 4 norepinephrine (NE), increases pacemaker rate by MD (TOP 9 -
B. decreasing the activity of L type Ca++ channels decreasing gK+ and increasing slow inward gCa++ and AUG 2012
and lowering the threshold gNa+; the pacemaker current (If) is enhanced. These MED
C. increasing the activity of the rectifying K+ changes increase the slope of phase 4 so that the BOARDS;
channels in phase 3 pacemaker potential more rapidly reaches the threshold TOPNOTCH
D. increasing the activity of slow Ca++ channels in for action potential generation. MD)
phase 2
E. All of the above
612 12. The hormone secreted mainly by the corpus Answer: A. Corpus luteum secretes progesterone to RACHELLE FINAL EXAM -
luteum is: start the luteal (secretory) phase of the menstrual cycle. MENDOZA, FEB 2013
A. Progesterone MD (TOP 9 -
B. Testosterone AUG 2012
C. Estrogen MED
D. Inhibin BOARDS;
E. None of the above TOPNOTCH
MD)

TOPNOTCH MEDICAL BOARD PREP PHYSIOLOGY SUPEREXAM Page 75 of 88


For inquiries visit www.topnotchboardprep.com.ph or email us at topnotchmedicalboardprep@gmail.com
TOPNOTCH MEDICAL BOARD PREP PHYSIOLOGY SUPEREXAM
For inquiries visit www.topnotchboardprep.com.ph or email us at topnotchmedicalboardprep@gmail.com
Item QUESTION EXPLANATION AUTHOR TOPNOTCH
# EXAM
613 Following massive hemorrhage during delivery, a RACHELLE FINAL EXAM -
29-year old female was unable to breastfed due to Answer: D. The patient is suffering from postpartum MENDOZA, FEB 2013
absence of milk secretion. Which of the following is pituitary necrosis secondary to massive haemorrhage MD (TOP 9 -
most likely to be associated with this clinical (Sheehan’s syndrome). Therefore, it is expected that all AUG 2012
picture? hormones produced by the pituitary gland will be MED
A. Symptoms of hyperthyroidism decreased. There will be hypothyroidism, decreased BOARDS;
B. Elevated prolactin secretion prolactin secretion (reason for inability to produce milk) TOPNOTCH
C. Cushing’s disease and decreased gonadotropin secretion. MD)
D. Decreased gonadotropin secretion
E. All of the above
614 Which of the following is/are one of the principal Answer: E. The principal means by which acclimatization RACHELLE FINAL EXAM -
means by which acclimatization to high altitudes comes about are (1) a great increase in pulmonary MENDOZA, FEB 2013
come about? ventilation, (2) increased numbers of red blood cells, (3) MD (TOP 9 -
A. Lower than normal pulmonary ventilation increased diffusing capacity of the lungs, (4) increased AUG 2012
B. Decreased number of red blood cells vascularity of the peripheral tissues, and (5) increased MED
C. Reduced diffusing capacity of the lungs ability of the tissue cells to use oxygen despite low PO2. BOARDS;
D. Decreased vascularity of peripheral tissues (Guyton 11th ed, page 539) TOPNOTCH
E. None of the above MD)

615 The deficiency of what trace element is associated RACHELLE FINAL EXAM -
with skin lesions, weakened immune system, and Answer D. Patients with zinc deficiency will present MENDOZA, FEB 2013
hypogonadal dwarfism? with skin ulcer, depressed immune response and MD (TOP 9 -
A. Copper hypogonadal dwarfism. AUG 2012
B. Chromium MED
C. Cobalt BOARDS;
D. Zinc TOPNOTCH
E. Iron MD)

616 In the sarcomere, which of the following keep/s the Answer: A. The side-by-side relationship between the RACHELLE FINAL EXAM -
myosin and actin in place? myosin and actin filaments is difficult to maintain. This is MENDOZA, FEB 2013
A. Titin achieved by a large number of filamentous molecules of MD (TOP 9 -
B. Z disc a protein called titin (Guyton, 11th ed). AUG 2012
C. Myofibrils MED
D. All of the above BOARDS;
E. A and B only TOPNOTCH
MD)
617 DM, 45 year old female, went to you as her physician RACHELLE FINAL EXAM -
complaining of weight gain, even though her diet Answer: B. The patient has a condition called Cushing’s MENDOZA, FEB 2013
was controlled. Upon physical examination, you syndrome. This is manifested by abnormally elevated MD (TOP 9 -
found out that she has elevated blood pressure, levels of cortisol. AUG 2012
purple striae on the trunk, growth of fat pads along MED
the collar bone and hirsutism. Laboratory revealed BOARDS;
elevated blood sodium levels and low blood TOPNOTCH
potassium levels. Increased production of which MD)
hormone would explain her symptoms?
A. Insulin
B. Cortisol
C. Glucagon
D. Aldosterone
E. Testosterone
618 A newborn male was held at the NICU because he RACHELLE FINAL EXAM -
did not pass out meconium after 48 hours of life. Answer: D. Both the myenteric (Auerbach) plexus and MENDOZA, FEB 2013
Plain abdominal radiographs showed distended the submucosal (Meissner) plexus are absent from the MD (TOP 9 -
bowel loops with a paucity of air in the rectum. muscular layer of the bowel wall in Hirschprung’s AUG 2012
Absence of which of the following is/are expected to disease. MED
be seen in the biopsy of the affected segment? BOARDS;
A. Meissner’s plexus TOPNOTCH
B. Auerbach’s plexus MD)
C. Myenteric plexus
D. All of the above
E. A and B only
619 Which of the following is primarily utilized by an RACHELLE FINAL EXAM -
athlete during the first 8-10 seconds of a 100-meter Answer: C. The phosphagen energy system can provide MENDOZA, FEB 2013
run? maximal muscle power for 8 to 10 seconds, almost MD (TOP 9 -
A. Glycogen-lactic acid system enough for the 100-meter run. The glycogen-lactic acid AUG 2012
B. Aerobic system system can provide 1.3 to 1.6 minutes of maximal muscle MED
C. Phosphagen energy system activity in addition to the 8 to 10 seconds provided by BOARDS;
D. A and B the phosphagen system, although at somewhat reduced TOPNOTCH
E. All of the above muscle power. Aerobic system is required for prolonged MD)
athletic activity.
620 The concentrating mechanism of the nephron RACHELLE FINAL EXAM -
depends upon the maintenance of a gradient of Answer: E. The concentrating mechanism depends upon MENDOZA, FEB 2013
increasing osmolality along the medullary pyramids. the maintenance of a gradient of increasing osmolality MD (TOP 9 -
Which of the following is/are true regarding the along the medullary pyramids. This gradient is produced AUG 2012
production of this gradient? by the operation of the loops of Henle as countercurrent MED
A. It is produced by the operation of the loops of multipliers and maintained by the operation of the vasa BOARDS;
Henle as countercurrent exchanger recta as countercurrent exchangers. (Guyton, 11th ed) TOPNOTCH
B. It is maintained by the operation of the vasa recta MD)
as countercurrent multiplier
C. It is generated by insertion of aquaporins at
collecting tubules as stimulated by vasopressin
D. A and B only
E. None of the above

TOPNOTCH MEDICAL BOARD PREP PHYSIOLOGY SUPEREXAM Page 76 of 88


For inquiries visit www.topnotchboardprep.com.ph or email us at topnotchmedicalboardprep@gmail.com
TOPNOTCH MEDICAL BOARD PREP PHYSIOLOGY SUPEREXAM
For inquiries visit www.topnotchboardprep.com.ph or email us at topnotchmedicalboardprep@gmail.com
Item QUESTION EXPLANATION AUTHOR TOPNOTCH
# EXAM
621 A 35 year old male from a nearby city was a All of the choices are consequences of PTB infection. VON ANDRE DIAGNOSTIC
diagnosed case of pulmonary tuberculosis . Lesion MEDINA, MD EXAM - AUG
from this kind of infection is usually walled off by a (TOP 4 - FEB 2012
fibrous tissue but if it fails, abscess cavities and 2012 MED
multiple areas of fibrosis will develop causing which BOARDS;
of the following consequences? TOPNOTCH
A. Reduced vital capacity MD)
B. Reduced membrane surface area
C. Increased thickness of the surface membrane
D. Abnormal V/Q ratio
E. All of the above
622 Jackie Casimiro, a full time teacher, is currently Hunger pangs are strongest in the 3rd-4th day. SIMILAR VON ANDRE DIAGNOSTIC
involved in a hunger strike joined by various TO PREVIOUS BOARD EXAM CONCEPT/PRINCIPLE MEDINA, MD EXAM - AUG
teachers as a protest for the low compensation given (TOP 4 - FEB 2012
to them by the government. Common to hunger 2012 MED
strikes ,hunger contraction causes hunger pangs BOARDS;
especially in young healthy individual. Hunger pangs TOPNOTCH
are usually strongest when? MD)
A. 1st 12 hours
B. 36-48 hours
C. on the 3rd-4th day
D. on the 2nd week
E. None of the above choices
623 All of the following lung volume and lung capacities VON ANDRE DIAGNOSTIC
cannot be measured clinically except? MEDINA, MD EXAM - AUG
A. TV (TOP 4 - FEB 2012
B. ERV 2012 MED
C. RV BOARDS;
D. FRC TOPNOTCH
E. TLC MD)

624 Which characteristic or component is shared by an elevation of intracellular calcium is common to VON ANDRE DIAGNOSTIC
skeletal muscle and smooth muscle? mechanism of excitation-contraction coupling in skeletal MEDINA, MD EXAM - AUG
A. Thick and thin filaments arranged in sarcomeres muscles. (TOP 4 - FEB 2012
B. Troponin 2012 MED
C. Elevation of intracellular Ca for excitation- BOARDS;
contraction coupling TOPNOTCH
D. Spontaneous depolarization of the membrane MD)
potential
E. High degree of electrical coupling between cells
625 Which of the following structures has a primary output of prkinje cells from the cerebellar cortex to deep VON ANDRE DIAGNOSTIC
function to coordinate rate, range, force and cerebellar nuclei is inhibitory. This output modulates MEDINA, MD EXAM - AUG
direction of movement? movement and is responsible for coordination. (TOP 4 - FEB 2012
A. Primary motor cortex 2012 MED
B. Premotor cortex and supplementary motor BOARDS;
cortex TOPNOTCH
C. Prefrontal cortex MD)
D. Basal ganglia
E. Cerebellum
626 All of the following are muscles for inspiration VON ANDRE DIAGNOSTIC
except? MEDINA, MD EXAM - AUG
A. External Intercostals (TOP 4 - FEB 2012
B. SCM 2012 MED
C. Anterior Serrati BOARDS;
D. Scalene TOPNOTCH
E. Internal Intercostals MD)

627 Slow waves as an electrical activity of GI smooth Slow waves are slowest in the stomach and fastest in the VON ANDRE DIAGNOSTIC
muscles are actually not true action potentials. It is a small intestines MEDINA, MD EXAM - AUG
cyclic opening of calcium channel (depolarization) (TOP 4 - FEB 2012
and opening of Na channel (repolarization) which 2012 MED
bring undulating changes in RMP by the use of a BOARDS;
pace maker (Interstitial cells of Cajal). These waves TOPNOTCH
are found to be slowest in which part of the GI tract? MD)
A. Esophagus
B. Stomach
C. Small intestines
D. Colon
E. Rectum

628 Stimulation of gastric acid secretion is VON ANDRE DIAGNOSTIC


potentiated/initiated by all of the following EXCEPT: MEDINA, MD EXAM - AUG
A. Histamine (TOP 4 - FEB 2012
B. Gastrin 2012 MED
C. Acetylcholine BOARDS;
D. Secretin TOPNOTCH
MD)

TOPNOTCH MEDICAL BOARD PREP PHYSIOLOGY SUPEREXAM Page 77 of 88


For inquiries visit www.topnotchboardprep.com.ph or email us at topnotchmedicalboardprep@gmail.com
TOPNOTCH MEDICAL BOARD PREP PHYSIOLOGY SUPEREXAM
For inquiries visit www.topnotchboardprep.com.ph or email us at topnotchmedicalboardprep@gmail.com
Item QUESTION EXPLANATION AUTHOR TOPNOTCH
# EXAM
629 Jennifer Arcilla is a professional mountain climber at 10, 000 feet: most impt effect is Decreased mental VON ANDRE DIAGNOSTIC
and is used to changes brought about by high proficiency, the rest of the choices can be observed at MEDINA, MD EXAM - AUG
altitude. Normally an unacclimatized person can 12,000 feet. (TOP 4 - FEB 2012
remain conscious up to 50 % of its arterial oxygen is 2012 MED
saturated. At 10, 000 feet, arterial oxygenation is at BOARDS;
90 % and drops rapidly as one ascends. What is the TOPNOTCH
most important observed effect to an individual at MD)
this level?
A. Drowsiness
B. Lassitude
C. Headache
D. Decreased mental proficiency
E. Euphoria
630 All of the following are characteristics of A-D are correct. End diastolic volume instead of End VON ANDRE DIAGNOSTIC
isovolumetric ventricular contraction phase of the Systolic Volume is involved in this phase. MEDINA, MD EXAM - AUG
cardiac cycle , EXCEPT: (TOP 4 - FEB 2012
A. It is a period between closure of the AV valves 2012 MED
and opening of Semilunar valves BOARDS;
B. There is an abrupt increase in the ventricular TOPNOTCH
pressure more than the atrial pressure MD)
C. All valves are closed in this phase
D. In this phase, the 1st heart sound is produced
E. End Systolic Volume is involved
631 In one physiology class, you are tasked to study the VON ANDRE DIAGNOSTIC
cardiac cycle along with the heart sounds and its MEDINA, MD EXAM - AUG
origin. Which of the following pair of heart sounds (TOP 4 - FEB 2012
and its origin is INCORRECT? 2012 MED
A. S1- closure of AV valves BOARDS;
B. S2- closure of SL valves TOPNOTCH
C. S3- rapid filling phase (flow of blood from atria MD)
to ventricle)
D. S4- atrial systole
E. All of the above choices are correct
632 Which of the following phases of SA nodal Phase 4- slow conduction, inward Na current is VON ANDRE DIAGNOSTIC
conduction is responsible for its automaticity? respinsible for SA nodal automaticity MEDINA, MD EXAM - AUG
A. Phase O-upstroke of action potential (TOP 4 - FEB 2012
(depolarization brought about by increase Ca 2012 MED
conductance) BOARDS;
B. Phase 3-repolarization (outward K current) TOPNOTCH
C. Phase 4- slow depolarization (inward Na MD)
current)
D. Phase 5- fast Ca current
E. None of the above
633 All of the following are vasoconstrictors, EXCEPT: Bradykinin and histamine are vasodilators VON ANDRE DIAGNOSTIC
A. Serotonin MEDINA, MD EXAM - AUG
B. Endothelin (TOP 4 - FEB 2012
C. Bradykinin 2012 MED
D. Histamine BOARDS;
E. C and D TOPNOTCH
MD)

634 A 56 year old female was admitted due to VON ANDRE DIAGNOSTIC
uncontrolled DM Type 2. Further work ups revealed MEDINA, MD EXAM - AUG
that her kidney function is already impaired (TOP 4 - FEB 2012
significantly and she is then already anemic due to 2012 MED
lack of production of erythropoietin brought about BOARDS;
by kidney damage. Where is erythropoietin TOPNOTCH
produced in a normal kidney? MD)
A. Renal Papilla
B. Fibrous capsule
C. Macula Densa
D. Peritubular Capillaries (Interstitial cells)
E. Lacis cells
635 Which of the following would cause an increase in VON ANDRE DIAGNOSTIC
both glomerular filtration rate and renal plasma MEDINA, MD EXAM - AUG
flow? (TOP 4 - FEB 2012
A. hyperproteinemia 2012 MED
B. A ureteral stone BOARDS;
C. Dilation of the afferent arteriole TOPNOTCH
D. Dilation of the efferent arteriole MD)
E. Constriction of the afferent arteriole

636 A man presents with hypertension and hypokalemia. hypertension, hypokalemia, metabolic alkaLOSIS , VON ANDRE DIAGNOSTIC
Measurement of his arterial blood gases reveal a pH ELEVATED ALDOSTERONE AND LOW RENIN ARE ALL MEDINA, MD EXAM - AUG
of 7.5 and a calculated HCO3 of 32 mEq/L. His serum CONSISTENT TO CONN'S SYNDROME. (TOP 4 - FEB 2012
cortisol and VMA are normal, his serum aldosterone 2012 MED
is increased and his plasma renin activity is BOARDS;
decreased. Which of the following is the most likely TOPNOTCH
cause of his hypertension? MD)
A. Cushing's syndrome
B. Cushings's disease
C. Conn's syndrome
D. Renal artery stenosis
E. Pheochromocytoma

TOPNOTCH MEDICAL BOARD PREP PHYSIOLOGY SUPEREXAM Page 78 of 88


For inquiries visit www.topnotchboardprep.com.ph or email us at topnotchmedicalboardprep@gmail.com
TOPNOTCH MEDICAL BOARD PREP PHYSIOLOGY SUPEREXAM
For inquiries visit www.topnotchboardprep.com.ph or email us at topnotchmedicalboardprep@gmail.com
Item QUESTION EXPLANATION AUTHOR TOPNOTCH
# EXAM
637 Propagation of the action potential through the heart slowest= Av node. Fastest= purkinje fibers VON ANDRE DIAGNOSTIC
is slowest in which of the following? MEDINA, MD EXAM - AUG
A. SA node (TOP 4 - FEB 2012
B. AV node 2012 MED
C. Purkinje fibers BOARDS;
D. Ventricular muscle TOPNOTCH
E. None of the above MD)

638 Among the following organs, which is most VON ANDRE DIAGNOSTIC
vulnerable to hypoglycemia because of its MEDINA, MD EXAM - AUG
dependence on the circulating glucose for energy? (TOP 4 - FEB 2012
A. Liver 2012 MED
B. Kidney BOARDS;
C. Skeletal Muscle TOPNOTCH
D. Brain MD)
E. Pancreas

639 Change/s in the lung volumes of capacities of VON ANDRE DIAGNOSTIC


patients with restrictive lung disease: MEDINA, MD EXAM - AUG
A. Decrease in TLC (TOP 4 - FEB 2012
B. Decrease in residual volume 2012 MED
C. Decrease in Functional Capacity BOARDS;
D. Decrease in Expiratory reserve volume TOPNOTCH
E. All of the above MD)

640 A 23 year old female was admitted due to DHF. Her VON ANDRE DIAGNOSTIC
attending physician requested for bleeding MEDINA, MD EXAM - AUG
parameters including PT, PTT, CT and BT. Bleeding (TOP 4 - FEB 2012
time is a test of? 2012 MED
A. Intrinsic pathway BOARDS;
B. Extrinsic pathway TOPNOTCH
C. Platelet Function MD)
D. A and B
E. All of the above
641 The least common of granulocytes but very Basophils are the least common among the granulocytes. LITO JAY DIAGNOSTIC
important in immediate hypersensitivity reactions In the tissues, they are called Mast cells and are MACARAIG, EXAM - AUG
because it releases Histamine. responsible for release of Histamine during Type 1 MD (TOP 8 - 2013
A. Basophil Hypersensitivity reaction. FEB 2013
B. Eosinophil MED
C. Neutrophil BOARDS;
D. NK cells TOPNOTCH
E. Lymphocytes MD)

642 This is the most powerful structure of the optical Fovea is the center of central vision. Uvea is the mi- LITO JAY DIAGNOSTIC
system. vascular layer of the eye. Retina serves as the "film" of MACARAIG, EXAM - AUG
A. Cornea the eye. The Lens keeps the images focused. MD (TOP 8 - 2013
B. Fovea FEB 2013
C. Uvea MED
D. Retina BOARDS;
E. Lens TOPNOTCH
MD)
643 The heart recieves its blood supply during the During diastole, the negative pressure at the LV have a LITO JAY DIAGNOSTIC
Incisura. Which statement/s is/are true regarding suction effect on the aortic valve, causing it to close. The MACARAIG, EXAM - AUG
the Incisura? closure of the aortic valve (pulling of the cusps together, MD (TOP 8 - 2013
A. It occurs during Systole and away from the wall of aorta) is the oppurtunity for FEB 2013
B. It occurs during Diastole the coronary arteries (whose openings are on the walls MED
C. During Ventricular contraction of the base of the aorta) to recieve blood. Because during BOARDS;
D. During Ventricular relaxation systole, the cusps of the aortic valve cover them. TOPNOTCH
E. A and C MD)
F. B and D

644 This hormone initiates the process of FSH stimulates the sertoli cells and initiates the LITO JAY DIAGNOSTIC
spermatogenesis. spermatogenesis. MACARAIG, EXAM - AUG
A. LH MD (TOP 8 - 2013
B. FSH FEB 2013
C. Testosterone MED
D. Inhibin BOARDS;
E. TSH TOPNOTCH
MD)
645 This hormone is directly responsible for Testosterone directly stimulates the semiferous tubules LITO JAY DIAGNOSTIC
spermatogenesis. to produce sperm cells. MACARAIG, EXAM - AUG
A. LH MD (TOP 8 - 2013
B. FSH FEB 2013
C. Testosterone MED
D. Inhibin BOARDS;
E. TSH TOPNOTCH
MD)

TOPNOTCH MEDICAL BOARD PREP PHYSIOLOGY SUPEREXAM Page 79 of 88


For inquiries visit www.topnotchboardprep.com.ph or email us at topnotchmedicalboardprep@gmail.com
TOPNOTCH MEDICAL BOARD PREP PHYSIOLOGY SUPEREXAM
For inquiries visit www.topnotchboardprep.com.ph or email us at topnotchmedicalboardprep@gmail.com
Item QUESTION EXPLANATION AUTHOR TOPNOTCH
# EXAM
646 The residual volume is the amount of gas remaining The residual volume is NOT an oxygen reserve. The LITO JAY DIAGNOSTIC
in th lungs after maximal exhalation. What is the lungs can NEVER store oxygen!! It prevents complete MACARAIG, EXAM - AUG
function of this lung volume? collapse of the lungs after maximal exhalation and MD (TOP 8 - 2013
A. Prevents collapse of the lungs provides oxygen in between breaths. FEB 2013
B. Acts as oxygen reserve for hypoxic states MED
C. Provides the oxygen needed in between breaths BOARDS;
D. A and C only TOPNOTCH
E. All of the above MD)

647 This is an onion-like mechanoreceptor present on LITO JAY DIAGNOSTIC


skin surface which primarily function is to detect MACARAIG, EXAM - AUG
vibration and pressure. MD (TOP 8 - 2013
A. Paccinian Corpuscle FEB 2013
B. Merkel cells MED
C. Meissner's corpuscle BOARDS;
D. A and B TOPNOTCH
E. A and C MD)

648 Aldosterone is very important in controlling All the choices given are done by Aldosterone. BUT with LITO JAY DIAGNOSTIC
peripheral blood pressure. It execute this function regards to BP control, only choices A and B are MACARAIG, EXAM - AUG
by applicable MD (TOP 8 - 2013
A. Upregulating basolateral Na/K pump, which FEB 2013
pumps sodium out of cell MED
B. Causing reabsorption of sodium BOARDS;
C. Secreting Potassium TOPNOTCH
D. A and B only MD)
E. All of the above
649 The first heart sound is created by the closure of? The Atriventricular valves are the Mitral and Tricuspid LITO JAY DIAGNOSTIC
A. AV Valves valves. MACARAIG, EXAM - AUG
B. Semilunar Valves MD (TOP 8 - 2013
C. Mitral and Tricuspid Valves FEB 2013
D. A and C MED
E. B and C BOARDS;
TOPNOTCH
MD)
650 If the SA node is the pacemaker of the heart, what is LITO JAY DIAGNOSTIC
the pacemaker of peristalsis? MACARAIG, EXAM - AUG
A. Interstitial cells of Cajal MD (TOP 8 - 2013
B. Meissner's plexus FEB 2013
C. Submucosal plexus MED
D. B and C only BOARDS;
E. All of the above TOPNOTCH
MD)
651 Bilirubin conjugation occurs at the liver. Which Conjugated - Dubin and Rotor LITO JAY DIAGNOSTIC
among the choices is/are jaundice due to conjugated MACARAIG, EXAM - AUG
bilirubin? MD (TOP 8 - 2013
A. Dubin-Johnson Syndrome FEB 2013
B. Rotor syndrome MED
C. Criggler-Najjar Syndrome BOARDS;
D. A and B only TOPNOTCH
E. A and C only MD)

652 Which of the following statements is/are true LITO JAY DIAGNOSTIC
regarding Acetylcholine? MACARAIG, EXAM - AUG
A. Degraded into Choline and acetate after the MD (TOP 8 - 2013
action potential FEB 2013
B. Acetylcholinesterase acts as inhibitor for its MED
degradation BOARDS;
C. The amount of re-uptake is more than that of TOPNOTCH
being degraded MD)
D. A and C only
E. All of the above
653 The irregular curvature of the cornea may result in LITO JAY DIAGNOSTIC
an error of refraction called MACARAIG, EXAM - AUG
A. presbyopia MD (TOP 8 - 2013
B. myopia FEB 2013
C. Near-sightedness MED
D. Far-sightedness BOARDS;
E. Astigmatism TOPNOTCH
MD)
654 The opercular/triangular parts of the inferior frontal LITO JAY DIAGNOSTIC
gyrus is also called MACARAIG, EXAM - AUG
A. Broca's area MD (TOP 8 - 2013
B. Brodmann 44,45 FEB 2013
C. Auditory association area MED
D. A and B BOARDS;
E. A and C TOPNOTCH
MD)
655 This intracellular organelle is responsible for beta- LITO JAY DIAGNOSTIC
oxidation of long chain fatty-acids. MACARAIG, EXAM - AUG
A. Lysosome MD (TOP 8 - 2013
B. Peroxisome FEB 2013
C. Golgi body MED
D. Mitochondrai BOARDS;
E. Endoplasmic reticulum TOPNOTCH
MD)
TOPNOTCH MEDICAL BOARD PREP PHYSIOLOGY SUPEREXAM Page 80 of 88
For inquiries visit www.topnotchboardprep.com.ph or email us at topnotchmedicalboardprep@gmail.com
TOPNOTCH MEDICAL BOARD PREP PHYSIOLOGY SUPEREXAM
For inquiries visit www.topnotchboardprep.com.ph or email us at topnotchmedicalboardprep@gmail.com
Item QUESTION EXPLANATION AUTHOR TOPNOTCH
# EXAM
656 This is a structural cytoskeleton that is the link LITO JAY DIAGNOSTIC
between the extracellular matrix, cytoplasm and MACARAIG, EXAM - AUG
nucleus. This also serves as tumor marker. MD (TOP 8 - 2013
A. Microfilament FEB 2013
B. Intermediate filament MED
C. Microtubule BOARDS;
D. A and B TOPNOTCH
E. A and C MD)

657 These are cells found in the respiratory tract that is LITO JAY DIAGNOSTIC
capable of metabolizing airborne toxins MACARAIG, EXAM - AUG
A. Basal cells MD (TOP 8 - 2013
B. Goblet cells FEB 2013
C. Clara cells MED
D. Type I pneumocytes BOARDS;
E. Type II pneumocytes TOPNOTCH
MD)

658 This shifts the hemoglobin oxygen dissociation curve LITO JAY DIAGNOSTIC
to the left MACARAIG, EXAM - AUG
A. Carbon dioxide MD (TOP 8 - 2013
B. Alkalosis FEB 2013
C. 2,3-DPG MED
D. Exercise BOARDS;
E. Increase in temperature TOPNOTCH
MD)
659 The increase of stroke volume during exercise is LITO JAY DIAGNOSTIC
primarily due to an increase in MACARAIG, EXAM - AUG
A. Venous return MD (TOP 8 - 2013
B. Heart rate FEB 2013
C. Peripheral resistance MED
D. B and C BOARDS;
E. All of the above TOPNOTCH
MD)
660 A 57 year old patient had an acute myocardial LITO JAY DIAGNOSTIC
infarction. BP was 70/40 and cardiac output is 1/3 MACARAIG, EXAM - AUG
of normal. The central venous should be MD (TOP 8 - 2013
A. Unchanged FEB 2013
B. Increased due to decrease peripheral resistance MED
C. Decreased due to decrease peripheral resistance BOARDS;
D. Increased due to decrease venous return TOPNOTCH
E. Decrease due to decrease venous return MD)

661 This best describes gap junctions: A gap junction or nexus is a specialized intercellular HAZEL MIDTERM 2 -
A. They are attachments between cells that permit connection between the cytoplasm of two cells. B and c KAREN RAZ, AUG 2013
intercellular communication describes a tight junction. MD (TOP 6 -
B. Intracellular pathway for solutes FEB 2013
C. May be "tight" or "leaky" MED
D. Loosely attached to basement membrane BOARDS;
E. All of the above TOPNOTCH
MD)
662 Na/K ATPase pump is an example of what type of Primary active transport, also called direct active HAZEL MIDTERM 2 -
cellular transport? transport, directly uses energy to transport molecules KAREN RAZ, AUG 2013
A. Facilitated diffusion across a membrane. MD (TOP 6 -
B. Primary active transport FEB 2013
C. Simple Diffusion MED
D. Secondary active transport BOARDS;
E. Symport TOPNOTCH
MD)
663 This is true of action potentials: Depolarization makes the membrane potential less HAZEL MIDTERM 2 -
A. Depolarization makes the membrane potential negative allowing it to reach threshold to produce an KAREN RAZ, AUG 2013
more negative action potential. Repolarization returns the RMP near MD (TOP 6 -
B. Hyperpolarization makes the membrane threshold making it more negative. Hyperpolarization FEB 2013
potential less negative further makes the RMP more negative. RMP of skeletal MED
C. The RMP of skeletal muscles is approximately - muscles = -95mV BOARDS;
95mV TOPNOTCH
D. Repolarization closes the activation gates of the MD)
sodium channel
E. all of the above
664 During muscular contraction these regions in the Both I band and H band shorten during muscle HAZEL MIDTERM 2 -
sarcomere shorten: contractions. KAREN RAZ, AUG 2013
A. A band MD (TOP 6 -
B. I band FEB 2013
C. H band MED
D. Z line BOARDS;
E. B and c TOPNOTCH
MD)
665 This is true of the ANS except: Receptors for the sympathetic nervous system: alpha HAZEL MIDTERM 2 -
A. Parasympathetic ganglia are located near the and beta KAREN RAZ, AUG 2013
effector organs MD (TOP 6 -
B. Preganglionic neurons of the sympathetic NS FEB 2013
originate from the thoracolumbar region MED
C. Ganglionic neurotransmitters of both PANS and BOARDS;
SANS is Ach TOPNOTCH
D. SANS receptors at the effector organs is MD)
primarily muscarinic
TOPNOTCH MEDICAL BOARD PREP PHYSIOLOGY SUPEREXAM Page 81 of 88
For inquiries visit www.topnotchboardprep.com.ph or email us at topnotchmedicalboardprep@gmail.com
TOPNOTCH MEDICAL BOARD PREP PHYSIOLOGY SUPEREXAM
For inquiries visit www.topnotchboardprep.com.ph or email us at topnotchmedicalboardprep@gmail.com
Item QUESTION EXPLANATION AUTHOR TOPNOTCH
# EXAM
E. None of the above

666 Activation of alpha 2 receptors of the sympathetic alpha 2 receptors are inhibitory G-protein coupled HAZEL MIDTERM 2 -
nervous system causes? receptors that when stimulated, causes inhibition of KAREN RAZ, AUG 2013
A. Increased cAMP adenylate cyclase, decreasing cAMP. MD (TOP 6 -
B. Decreased cAMP FEB 2013
C. Increased IP3/DAG MED
D. Opening of the Na/K channels BOARDS;
E. None of the above TOPNOTCH
MD)
667 Ménière's disease is primarily due to_______? Ménière's disease is idiopathic, but it is believed to be HAZEL MIDTERM 2 -
A. Structural defect of the ear linked to endolymphatic hydrops, an excess of fluid in KAREN RAZ, AUG 2013
B. Excess perilymph production the inner ear. It is thought that endolymphatic fluid MD (TOP 6 -
C. Excess endolymph production bursts from its normal channels in the ear and flows into FEB 2013
D. Paralysis of the stereocilia other areas, causing damage. MED
E. Lack of cupula BOARDS;
TOPNOTCH
MD)

668 In active adults, this type of Eeg tracing alpha - relaxed, closed eyes, beta - active, alert, theta - HAZEL MIDTERM 2 -
predominates: drowsiness, gamma - mplicated in creating the unity of KAREN RAZ, AUG 2013
A. alpha conscious perception, delta - adult slow - wave sleep MD (TOP 6 -
B. beta FEB 2013
C. theta MED
D. gamma BOARDS;
E. Delta TOPNOTCH
MD)
669 The fourth heart sound is due to: In healthy adults, there are two normal heart sounds HAZEL MIDTERM 2 -
A. Closure of AV valves often described as a lub and a dub (or dup), that occur in KAREN RAZ, AUG 2013
B. Closure of SL valves sequence with each heartbeat. These are the first heart MD (TOP 6 -
C. Rapid ventricular filling against SL valve sound (S1) and second heart sound (S2), produced by FEB 2013
D. Contraction of atrium the closing of the AV valves and semilunar valves MED
E. None of the above respectively. S3 is produced by rapid ventricular filling. BOARDS;
S4 is produced by atrial contraction. TOPNOTCH
MD)
670 The Frank - Starling Law states that: The Frank–Starling law of the heart states that the stroke HAZEL MIDTERM 2 -
A. An increase in SV and CO occurs in response to volume of the heart increases in response to an increase KAREN RAZ, AUG 2013
decreased HR in the volume of blood filling the heart (the end diastolic MD (TOP 6 -
B. An increase in SV and CO occurs in response to volume) when all other factors remain constant. FEB 2013
decreased TPR MED
C. An increase in SV and CO occurs in response to BOARDS;
increased HR TOPNOTCH
D. an increased in SV and CO occurs in response to MD)
increased venous return
E. all of the above
671 Which of the following can be measured by Residual volume, functional residual capacity and total HAZEL MIDTERM 2 -
spirometry? lung capacity cannot be measured by spirometry. KAREN RAZ, AUG 2013
A. Residual volume MD (TOP 6 -
B. Tidal Volume FEB 2013
C. Functional Residual Capacity MED
D. Total Lung Capacity BOARDS;
E. All of the above TOPNOTCH
MD)
672 Which of the following causes increased oxygen All the other factors causes shift of theO2-hemoglobin HAZEL MIDTERM 2 -
delivery capacity of hemoglobin? dissociation curve to the left, which increases affinity, KAREN RAZ, AUG 2013
A. Decreased temperature thereby decreasing delivery capacity. MD (TOP 6 -
B. Increased pH FEB 2013
C. Increased hydrogen concentration MED
D. Decreased pCO2 BOARDS;
E. All of the above TOPNOTCH
MD)
673 Which of the following causes a shift of potassium Factors that causes intracellular shift of potassium HAZEL MIDTERM 2 -
into cells resulting in hypokalemia? includes: Insulin, B- agonists, Alkalosis, Hyposmolarity. KAREN RAZ, AUG 2013
A. exercise MD (TOP 6 -
B. acidosis FEB 2013
C. B-antagonists MED
D. insulin BOARDS;
E. All of the above TOPNOTCH
MD)
674 The following statements correctly describes serum Anion gap represents po4, citrate, sulfate and protein, HAZEL MIDTERM 2 -
anion gap: chloride is not included. KAREN RAZ, AUG 2013
A. Represents unmeasured anions in serum MD (TOP 6 -
B. Includes PO4, citrate, sulfate, chloride and FEB 2013
protein MED
C. The normal value for the anion gap is 12 +/-4. BOARDS;
D. A and C TOPNOTCH
E. A and B MD)

TOPNOTCH MEDICAL BOARD PREP PHYSIOLOGY SUPEREXAM Page 82 of 88


For inquiries visit www.topnotchboardprep.com.ph or email us at topnotchmedicalboardprep@gmail.com
TOPNOTCH MEDICAL BOARD PREP PHYSIOLOGY SUPEREXAM
For inquiries visit www.topnotchboardprep.com.ph or email us at topnotchmedicalboardprep@gmail.com
Item QUESTION EXPLANATION AUTHOR TOPNOTCH
# EXAM
675 This structures primarily controls gastro-intestinal Auerbach's plexus (or myenteric plexus) provides motor HAZEL MIDTERM 2 -
smooth muscle motility? innervation to both layers of the tunica muscularis, KAREN RAZ, AUG 2013
A. Auerbach's plexus (myenteric) having both parasympathetic and sympathetic input, MD (TOP 6 -
B. Meissuer's Plexus whereas Meissner's plexus has only parasympathetic FEB 2013
C. Submuscosal Plexus fibers and provides secretomotor innervation to the MED
D. Migrating motor complex / MMC mucosa nearest the lumen of the gut. BOARDS;
E. None of the Above TOPNOTCH
MD)
676 True of cholecystokinin / CCK, except? The presence of fatty acids and/or certain amino acids in HAZEL MIDTERM 2 -
A. Stimulates contraction of the GB the chyme entering the duodenum is the greatest KAREN RAZ, AUG 2013
B. Prolings Gastric emptying time stimulator of CCK release. MD (TOP 6 -
C. Release is stimulated by triglycerides FEB 2013
D. Stimulates pancreatic enzyme secretion MED
E. None of the Above BOARDS;
TOPNOTCH
MD)
677 Which of the following is a direct action of growth All other functions are actions of GH through IGF. HAZEL MIDTERM 2 -
hormone? KAREN RAZ, AUG 2013
A. Linear Growth MD (TOP 6 -
B. Lean body mass FEB 2013
C. Organ size MED
D. Glucose uptake into cells (diabetogenic) BOARDS;
E. None of the Above TOPNOTCH
MD)
678 Describe the relationship between volume of gas and Boyle's law - relation bet pressure and volume; Gay - HAZEL MIDTERM 2 -
temperature? Lussac's - Pressure ant temperature; Avogadro's law - KAREN RAZ, AUG 2013
A. Boyle's Law Volume and molar composition MD (TOP 6 -
B. Charles' Law FEB 2013
C. Gay-Lussac's Law MED
D. Avogadro's Law BOARDS;
E. None of the Above TOPNOTCH
MD)
679 Direct effect of excess T3 in Grave's ---, except? exophthalmos is due to the autoimmune effect of grave's HAZEL MIDTERM 2 -
A. Tachycardia disease KAREN RAZ, AUG 2013
B. Hyperthermia MD (TOP 6 -
C. Exophthalmos FEB 2013
D. Tremors MED
E. None of the Above BOARDS;
TOPNOTCH
MD)
680 This is the largest zone in the adrenal cortex? The zona fasciculata constitutes the middle zone of the HAZEL MIDTERM 2 -
A. Z. glomerulosa adrenal cortex, sitting directly beneath the zona KAREN RAZ, AUG 2013
B. Z. fasciculata glomerulosa. Constituent cells are organized into MD (TOP 6 -
C. Z. reticularis bundles or "fascicles" FEB 2013
D. Z. pellucida MED
E. None of the Above BOARDS;
TOPNOTCH
MD)
681 During spermatogenesis, sperm cells require several Answer: A. Epididymis (pp. 975, Guyton & Hall: MICHELLE MIDTERM 1 -
days to mature in which part of the male Textbook of Medical Physiology, 12th ed.) JAY AUG 2013
reproductive system? Notes: The sperm require several days to pass through FRANCISCO,
A. Epididymis the 6-meter-long tubule of the epididymis. After 18-24 MD (TOP 9 -
B. Rete testis hours, they develop the capability of motility. FEB 2013
C. Efferent ductules *SIMILAR TO PREVIOUS BOARD EXAM MED
D. Vas deferens CONCEPT/PRINCIPLE BOARDS;
TOPNOTCH
MD)
682 A 55 yo male patient was immediately brought into Answer: B. Hallucinations (pp. 954, Guyton & Hall: MICHELLE MIDTERM 1 -
the ED due to hypoglycemic manifestations after Textbook of Medical Physiology, 12th ed.) JAY AUG 2013
administration of insulin. Blood glucose taken Notes: FRANCISCO,
revealed 40 mg/100ml. Among the following • 50-70 mg/100ml: hallucinations, extreme nervousness, MD (TOP 9 -
choices, these symptoms are most likely manifested trembles all over, breaks out in a sweet. FEB 2013
by this patient; except • 20-50 mg/100ml and lower: clonic seizures, LOC, coma MED
A. Clonic seizures *SIMILAR TO PREVIOUS BOARD EXAM BOARDS;
B. Hallucinations CONCEPT/PRINCIPLE TOPNOTCH
C. State of Coma MD)
D. Loss of consciousness
683 A young college student had nausea, vomiting, visual Answer: C. Inhibit release of acetylcholine from MICHELLE MIDTERM 1 -
disorders, and muscular paralysis after eating presynaptic membranes. JAY AUG 2013
canned tuna fish. The probable diagnosis is botulism, Notes: The toxin from Clostridium botulinum inhibits the FRANCISCO,
caused by ingestion of the Clostridium botulinum release of acetylcholine, the neurotransmitter at MD (TOP 9 -
toxin. The physiological effect of this toxin is to myoneural junctions. As a result, motor nerve impulses FEB 2013
A. Bind to and thus inactivate acetylcholine cannot be transmitted across the junction, and muscle MED
receptors at myoneural junctions cells are not stimulated to contract. BOARDS;
B. Prevent release of calcium ions from the TOPNOTCH
sarcoplasmic reticulum, thus inhibiting muscle MD)
contraction
C. Inhibit release of acetylcholine from presynaptic
membranes
D. Inhibit hydrolysis of adenosine triphosphate
during the contraction cycle

TOPNOTCH MEDICAL BOARD PREP PHYSIOLOGY SUPEREXAM Page 83 of 88


For inquiries visit www.topnotchboardprep.com.ph or email us at topnotchmedicalboardprep@gmail.com
TOPNOTCH MEDICAL BOARD PREP PHYSIOLOGY SUPEREXAM
For inquiries visit www.topnotchboardprep.com.ph or email us at topnotchmedicalboardprep@gmail.com
Item QUESTION EXPLANATION AUTHOR TOPNOTCH
# EXAM
684 Characteristics of humoral immunity include the Answer: A. The body develops circulating antibodies MICHELLE MIDTERM 1 -
following: which are globin molecules in the blood that are JAY AUG 2013
A. The body develops circulating antibodies which capable of attacking the invading agent FRANCISCO,
are globin molecules in the blood that are capable of MD (TOP 9 -
attacking the invading agent FEB 2013
B. Achieved through the formation of large numbers MED
of activated lymphocytes designed to destroy foreign BOARDS;
agent TOPNOTCH
C. T lymphocytes produce immunoglobulins MD)
D. All of these
685 The extrinsic mechanism for initiating the formation Answer: C. Activation of factor XI MICHELLE MIDTERM 1 -
of prothrombin activator begins with a traumatized Notes: Factor XI – Intrinsic pathway JAY AUG 2013
vascular wall or extravascular tissues and occurs FRANCISCO,
according to the following steps; except MD (TOP 9 -
A. Release of tissue factor FEB 2013
B. Activation of factor X MED
C. Activation of factor XI BOARDS;
D. Effect of activated factor X to form prothrombin TOPNOTCH
activator MD)
686 One of the following is not a depolarization wave Answer: C. T wave MICHELLE MIDTERM 1 -
A. P wave Notes: T wave - repolarization JAY AUG 2013
B. QRS complex FRANCISCO,
C. T wave MD (TOP 9 -
D. None of these FEB 2013
MED
BOARDS;
TOPNOTCH
MD)
687 Some varieties of cabbage block peroxidase enzyme Answer: B. Propylthiouracil MICHELLE MIDTERM 1 -
required for iodination of tyrosine leading to growth JAY AUG 2013
of glandular tissues and forming a goiter. This effect FRANCISCO,
is similar to which anti-thyroid substance? MD (TOP 9 -
A. Thiocyanate FEB 2013
B. Propylthiouracil MED
C. Inorganic iodides BOARDS;
D. Both B and C TOPNOTCH
MD)
688 Capacitation of the spermatozoa include the Answer: A. Membrane of the sperm becomes less MICHELLE MIDTERM 1 -
following changes; except permeable to calcium ions JAY AUG 2013
A. Membrane of the sperm becomes less permeable Notes: Membrane of the sperm becomes more FRANCISCO,
to calcium ions permeable to calcium ions. MD (TOP 9 -
B. While the spermatozoa remains in the fluid of the FEB 2013
male genital ducts, they were continually exposed to MED
many floating vesicles from the semeniferous BOARDS;
tubules containing cholesterol TOPNOTCH
C. The uterine and the fallopian tube fluids wash MD)
away the various inhibitory factors that has
suppressed sperm activity in the male genital ducts
D. None of these
689 What is the main difference between voltage-gated Answer: B. Voltage-gated are activated by membrane MICHELLE MIDTERM 1 -
and ligand-gated ion channels? potential changes whereas ligand-gated channels are JAY AUG 2013
A. Voltage-gated channels are located in cardiac cells by chemical messenger binding FRANCISCO,
whereas ligand-gated are in neuronal cells MD (TOP 9 -
B. Voltage-gated are activated by membrane FEB 2013
potential changes whereas ligand-gated channels are MED
by chemical messenger binding BOARDS;
C. Ligand-gated are found in resting conformational TOPNOTCH
state MD)
D. Ligand-gated channels contain an a-subunit
protein
690 A 45 yo female has severe scarring of the lungs or Answer: C. V/Q mismatch MICHELLE MIDTERM 1 -
pulmonary fibrosis and underwent a left JAY AUG 2013
pneumonectomy several years earlier because of FRANCISCO,
recurrent infections. She was admitted to the ICU MD (TOP 9 -
with respiratory failure with hypoxemia caused by FEB 2013
severe bronchitis. On the ventilator with FiO2 of MED
0.35, her ABG shows Pa O2 = 80mmHg, PaCO2 = BOARDS;
60mmHg, pH = 7.34 with an RR = 25 cpm, VT = TOPNOTCH
350mL. What is the most likely cause of this MD)
patient’s hypoxemia?
A. Decreased FiO2
B. Diffusion abnormality
C. V/Q mismatch
D. Shunt
691 After completion of an endurance training program Answer: A. Increased biceps cross-sectional area MICHELLE MIDTERM 1 -
(3-5x/wk, 30-60min/day for 3 months at 60% Notes: Increased biceps cross-sectional area is a result of JAY AUG 2013
VO2max), all of the following will occur in skeletal resistance training, not endurance. FRANCISCO,
muscle; except MD (TOP 9 -
A. Increased biceps cross-sectional area FEB 2013
B. Increased mitochondrial density MED
C. Angiogenesis BOARDS;
D. Increased utilization of fats TOPNOTCH
MD)

TOPNOTCH MEDICAL BOARD PREP PHYSIOLOGY SUPEREXAM Page 84 of 88


For inquiries visit www.topnotchboardprep.com.ph or email us at topnotchmedicalboardprep@gmail.com
TOPNOTCH MEDICAL BOARD PREP PHYSIOLOGY SUPEREXAM
For inquiries visit www.topnotchboardprep.com.ph or email us at topnotchmedicalboardprep@gmail.com
Item QUESTION EXPLANATION AUTHOR TOPNOTCH
# EXAM
692 The excitatory cells of the SA and AV nodes transmit MICHELLE MIDTERM 1 -
electrochemical stimuli via which of the following? JAY AUG 2013
A. Desmosomes FRANCISCO,
B. Gap junctions MD (TOP 9 -
C. Basement mambranes FEB 2013
D. Tight junction MED
BOARDS;
TOPNOTCH
MD)
693 A 36 y.o man is evaluated at the office because of Answer: D MICHELLE MIDTERM 1 -
complaints of fatigue, weight gain and irritability. G proteins have intracellular portion. First messenger- JAY AUG 2013
Routine lab tests are performed and TSH level was extracellular; second messenger- intracellular.cAMP FRANCISCO,
found to be elevated. Patient is concerned about his activates protein kinase A. MD (TOP 9 -
condition and the relationship between FEB 2013
hypothyroidism and his symptoms. Which of the MED
following statements regarding second messenger BOARDS;
system is true? TOPNOTCH
A. Most receptor proteins are completely MD)
extracellular.
B. Both the first messenger and second messenger
mediators of cell signalling function within the cell
cytoplasm.
C. Adenylate cyclase stimulates the conversion of
cAMP to ATP.
D. IP3 greatly increases cytoplasmic calcium
concentrations.
694 A 34 yr-old male is brought into the emergency ADH acts to increase renal water conservation MICHELLE MIDTERM 1 -
room, having been found unconscious in his secondary to dehydration JAY AUG 2013
apartment. Apparently he has been in this state for FRANCISCO,
2 days. Labs for blood & urine are consistent with MD (TOP 9 -
elevated antidiuretic hormone. Which of these will FEB 2013
directly stimulate ADH in this patient? MED
A. Angiotensinogen BOARDS;
B. Extracellular fluid osmolality increase TOPNOTCH
C. Temperature decrease MD)
D. Thyroid hormone
E. Volume increase
695 A 45 yr-old male was suffering from unstable angina Cardiac output is calculated by multiplying heart rate by MICHELLE MIDTERM 1 -
and is undergoing cardiac testing. At rest. HR is 70 stroke volume. Stroke volume is the difference between JAY AUG 2013
bpm. His left ventricular end diastolic volume is EDV & ESV. So, 150-80 = 70 ml. Stroke volume. 70 ml x FRANCISCO,
estimated to be 150 ml while his end systolic volume 70 bpm = 4900 ml/min. MD (TOP 9 -
is 80 ml. What is his estimated cardiac output? FEB 2013
A. 3900 ml/min MED
B. 4400 ml/min BOARDS;
C. 4900 ml/min TOPNOTCH
D. 5400 ml/min MD)
E. 5900 ml/min
696 A 23 yr-old female has serum electrolytes tested as Insulin stimulates the uptake of K into the cells via Na+ MICHELLE MIDTERM 1 -
part of a routine physical. The lab results reveal mild K+ ATPase. JAY AUG 2013
hypokalemia. Which of the following will promote FRANCISCO,
movement of extracellular potassium into the MD (TOP 9 -
intracellular fluid compartment and cause FEB 2013
hypokalemia? MED
A. ECF hyperosmolality BOARDS;
B. IV administration of a beta-adrenergic blocker TOPNOTCH
C. IV insulin MD)
D. Metabolic acidosis
E. Physical exercise
697 Normal metabolism by the body generates large The vast majority of metabolic acid excretion is in the MICHELLE MIDTERM 1 -
quantities of acid. In spite of this, normal blood pH is form of volatile acid carbon dioxide which is removed JAY AUG 2013
slightly alkaline 7.4 This extracellular fluid alkalinity via the lungs. Much smaller quantities of non-volatile FRANCISCO,
is maintained primarily by the body’s removal of acids must be excreted in the urine. MD (TOP 9 -
which of these? FEB 2013
A. Ammonia MED
B. Carbon dioxide BOARDS;
C. Ketoacids such as acetoacetic acid TOPNOTCH
D. Lactic acid MD)
E. Titrable acids such as phosphoric acid
698 Which of the following is an adaptive response to At high elevation the decreased atmospheric pressure MICHELLE MIDTERM 1 -
moving from sea level to higher elevation. means there is less available O2. Hyperventilation helps JAY AUG 2013
A. Bronchial relaxation to bring alveolar and thus arterial O2 levels back FRANCISCO,
B. Decreased cardiac output towards normal. MD (TOP 9 -
C. Decreased circulating levels of EPO FEB 2013
D. Decreased levels of 2,3 DPG in RBC MED
E. Hyperventilation BOARDS;
TOPNOTCH
MD)

TOPNOTCH MEDICAL BOARD PREP PHYSIOLOGY SUPEREXAM Page 85 of 88


For inquiries visit www.topnotchboardprep.com.ph or email us at topnotchmedicalboardprep@gmail.com
TOPNOTCH MEDICAL BOARD PREP PHYSIOLOGY SUPEREXAM
For inquiries visit www.topnotchboardprep.com.ph or email us at topnotchmedicalboardprep@gmail.com
Item QUESTION EXPLANATION AUTHOR TOPNOTCH
# EXAM
699 During a marathon attempt a runner collapses and is Acute dehydration results in decreased plasma volume, MICHELLE MIDTERM 1 -
admitted with severe dehydration. Which of the cardiac output and arterial pressure which leads to low JAY AUG 2013
following is most likely to occur in this patient? firing rate of baroreceptors. FRANCISCO,
A. Decreased firing of baroreceptors MD (TOP 9 -
B. Decresed plasma osmolality FEB 2013
C. High renal water excretion MED
D. Low plasma ADH levels BOARDS;
E. Low water permeability of collecting duct tubular TOPNOTCH
cells MD)
700 GABA is an amino acid that functions as If the equilibrium potential of the ion (Cl) is more MICHELLE MIDTERM 1 -
neurotransmitter in the CNS. GABA typically causes negative than the RMP, increasing conductance of that JAY AUG 2013
increased chloride conductance and functions as an ion will hyperpolarize the membrane.If and only if the FRANCISCO,
inhibitory neurotransmitter. Assume that the RMP equals equilibrium potential for an ion will an MD (TOP 9 -
equilibrium potential for Chloride in particular cell increase in the conductance to that ion not cause a FEB 2013
is -80 mV and that application of GABA inhibits the change in membrane potential. MED
cell without any change in resting membrane BOARDS;
potential, what is the RMP of the cell? TOPNOTCH
A. +80 mV MD)
B. 0 mV
C. -70 mV
D. -80mV
E. -90 mV

TOPNOTCH MEDICAL BOARD PREP PHYSIOLOGY SUPEREXAM Page 86 of 88


For inquiries visit www.topnotchboardprep.com.ph or email us at topnotchmedicalboardprep@gmail.com
TOPNOTCH MEDICAL BOARD PREP PHYSIOLOGY SUPEREXAM
For inquiries visit www.topnotchboardprep.com.ph or email us at topnotchmedicalboardprep@gmail.com

Item # ANSWER 88 C 176 A 264 A 352 B


1 C 89 C 177 E 265 A 353 B
2 D 90 D 178 A 266 E 354 B
3 E 91 A 179 A, D 267 A 355 A
4 B 92 B 180 E 268 D 356 D
5 C 93 B 181 C 269 D 357 B
6 B 94 E 182 B 270 B 358 D
7 D 95 D 183 D 271 B 359 B
8 E 96 C 184 B and C 272 B 360 A
9 C 97 B 185 A 273 A 361 E
10 B 98 B 186 B 274 C 362 B
11 E 99 B 187 A 275 D 363 A
12 C 100 E 188 C 276 C 364 D
13 E 101 B 189 B 277 E 365 E
14 C 102 E 190 A 278 C 366 D
15 D 103 A 191 D 279 D 367 C
16 D 104 D 192 B 280 D 368 E
17 A 105 B 193 C 281 C 369 B
18 C 106 E 194 C 282 D 370 D
19 A 107 B 195 D 283 C 371 B
20 A 108 D 196 C 284 B 372 A
21 A 109 E 197 A 285 C 373 E
22 C 110 A 198 E 286 B 374 A
23 D 111 B 199 E 287 A 375 A
24 D 112 E 200 E 288 B 376 C
25 D 113 A 201 B 289 D 377 B
26 D 114 D 202 B 290 D 378 A
27 A 115 B 203 D 291 A 379 B
28 D 116 E 204 A 292 E 380 D
29 E 117 A 205 C 293 A 381 E
30 B 118 D 206 D 294 B 382 E
31 B 119 C 207 C 295 E 383 C
32 A 120 C 208 E 296 C 384 D
33 D 121 E 209 D 297 D 385 C
34 B 122 B 210 A 298 B 386 A
35 D 123 C 211 A 299 A 387 B
36 D 124 A 212 B 300 A 388 A
37 E 125 E 213 B 301 B 389 C
38 D 126 B 214 A 302 D 390 A
39 D 127 D 215 A 303 A 391 D
40 D 128 B 216 C 304 C 392 C
41 D 129 A 217 D 305 A 393 C
42 C 130 A 218 B 306 C 394 C
43 A 131 C 219 D 307 C 395 B
44 B 132 D 220 E 308 E 396 D
45 B 133 D 221 D 309 B 397 A
46 A 134 D 222 B 310 C 398 B
47 C 135 B 223 C 311 D 399 C
48 A 136 A 224 D 312 A 400 C
49 A 137 C 225 C 313 B 401 A
50 C 138 D 226 B 314 A 402 C
51 A 139 A 227 A 315 B 403 B
52 E 140 D 228 A 316 C 404 A
53 C 141 A 229 B 317 E 405 A
54 A 142 C 230 D 318 A 406 E
55 B 143 B 231 B 319 C 407 B
56 A 144 A 232 D 320 A 408 B
57 E 145 D 233 B 321 D 409 A
58 A 146 E 234 B 322 E 410 C
59 D 147 B 235 E 323 B 411 E
60 B 148 B 236 B 324 A 412 E
61 C 149 D 237 B 325 A 413 D
62 E 150 D 238 D 326 A 414 A
63 A 151 A 239 C 327 B 415 D
64 D 152 B 240 C 328 E 416 E
65 B 153 C 241 D 329 C 417 B
66 C 154 A 242 C 330 A 418 C
67 B 155 B 243 A 331 A 419 B
68 A 156 C 244 E 332 B 420 C
69 C 157 C 245 A 333 B 421 B
70 B 158 A 246 C 334 C 422 B
71 E 159 C 247 D 335 A 423 B
72 C 160 D 248 C 336 E 424 B
73 A 161 D 249 E 337 B 425 B
74 C 162 B 250 B 338 D 426 C
75 D 163 A 251 C 339 C 427 C
76 A 164 E 252 A 340 C 428 C
77 D 165 D 253 C 341 C 429 B
78 C 166 C 254 C 342 D 430 B
79 B 167 A 255 D 343 A 431 D
80 D 168 D 256 A 344 C 432 D
81 E 169 D 257 C 345 C 433 D
82 B 170 A 258 D 346 D 434 B
83 B 171 B 259 A 347 B 435 D
84 D 172 D 260 D 348 C 436 D
85 D 173 A 261 B 349 A 437 B
86 D 174 B 262 D 350 C 438 D
87 D 175 D 263 D 351 C 439 D
TOPNOTCH MEDICAL BOARD PREP PHYSIOLOGY SUPEREXAM Page 87 of 88
For inquiries visit www.topnotchboardprep.com.ph or email us at topnotchmedicalboardprep@gmail.com
TOPNOTCH MEDICAL BOARD PREP PHYSIOLOGY SUPEREXAM
For inquiries visit www.topnotchboardprep.com.ph or email us at topnotchmedicalboardprep@gmail.com
440 B 529 B 618 D
441 C 530 D 619 C
442 B 531 D 620 E
443 D 532 A 621 E
444 A 533 A 622 C
445 A 534 C 623 A
446 B 535 A 624 C
447 C 536 B 625 E
448 D 537 A 626 E
449 C 538 D 627 B
450 D 539 B 628 D
451 D 540 A 629 D
452 B 541 B 630 E
453 D 542 D 631 E
454 C 543 D 632 C
455 C 544 D 633 E
456 E 545 A 634 D
457 D 546 C 635 C
458 C 547 E 636 C
459 C 548 D 637 B
460 A 549 E 638 D
461 D 550 D 639 E
462 A 551 D 640 C
463 A 552 A 641 A
464 C 553 E 642 A
465 B 554 C 643 F
466 A 555 B 644 B
467 C 556 A 645 C
468 A 557 C 646 D
469 D 558 A 647 A
470 B 559 C 648 D
471 A 560 C 649 D
472 C 561 D 650 A
473 B 562 C 651 D
474 B 563 D 652 A
475 D 564 B 653 E
476 A 565 C 654 D
477 D 566 A 655 B
478 A 567 D 656 B
479 C 568 B 657 C
480 D 569 C 658 B
481 B 570 C 659 A
482 C 571 C 660 E
483 C 572 B 661 A
484 A 573 A 662 B
485 A 574 C 663 C
486 D 575 B 664 E
487 C 576 C 665 D
488 B 577 D 666 B
489 B 578 A 667 C
490 A 579 A 668 B
491 C 580 D 669 D
492 B 581 C 670 D
493 A 582 A 671 B
494 C 583 B 672 C
495 A 584 C 673 D
496 D 585 D 674 D
497 B 586 D 675 A
498 C 587 C 676 C
499 C 588 A 677 D
500 B 589 C 678 B
501 B 590 C 679 C
502 C 591 D 680 B
503 C 592 B 681 A
504 A 593 D 682 B
505 A 594 E 683 C
506 C 595 A 684 A
507 A 596 E 685 C
508 B 597 A 686 C
509 A 598 E 687 B
510 C 599 C 688 A
511 A 600 B 689 B
512 C 601 C 690 C
513 A 602 A 691 A
514 B 603 D 692 B
515 B 604 A 693 D
516 C 605 B 694 B
517 A 606 B 695 C
518 A 607 D 696 C
519 E 608 A 697 B
520 D 609 C 698 E
521 C 610 D 699 A
522 D 611 A 700 D
523 B 612 A
524 A 613 D
525 B 614 E
526 C 615 D
527 A 616 A
528 C 617 B
TOPNOTCH MEDICAL BOARD PREP PHYSIOLOGY SUPEREXAM Page 88 of 88
For inquiries visit www.topnotchboardprep.com.ph or email us at topnotchmedicalboardprep@gmail.com
TOPNOTCH MEDICAL BOARD PREP LEGAL MEDICINE AND JURIS SUPEREXAM
For inquiries visit www.topnotchboardprep.com.ph or email us at topnotchmedicalboardprep@gmail.com
DEAR TOPNOTCH FRIENDS:

PLEASE FOLLOW THESE INSTRUCTIONS:

1. These questions are previous diagnostic, midterm, and finals exams of Topnotch, almost all of them made by Topnotch Board Exam Topnotchers.
2. Answer this Topnotch Superexam seriously 100-items at a time. Cover the “Explanations” Column. Do not immediately look at the answers from the
answer key. That’s not the correct way of answering sample exams. You need to treat these MCQs as exercises and not as handouts.
3. Time yourself. 1.5 hours per 100-item block.
4. After answering each 100-item block, refer to the Topnotch Answer Key for the correct answers. Please be careful of “frameshift mutations” when
checking your answers – check every 10 items. (the format of the answer key was designed for you to practice against “frameshift mutations”)
5. The Topnotch Superexams are EXERCISES for the actual med boards. They will not appear verbatim in your future exams. More than knowing what’s
the correct answer, it’s more important for you to:
a. Know why the other choices are wrong
b. Know why the other choices were included in the first place
c. Know the explanation to the correct answer
6. Sharpen your mind by answering the Topnotch Superexams. Most of these questions based on past feedback are more difficult than the actual questions
in the med boards. In these exams made by Board Exam Topnotchers, if you’re getting a score of 60/100 , that’s already a good score. More than 80/100
is outstanding.

Item QUESTION EXPLANATION AUTHOR TOPNOTCH
# EXAM
1 Medical service is referred to in the physician-patient The professional fee is the cause or the KRISTEL TANHUI DIAGNOSTIC
relationship as: consideration. (TOP 3 - AUG 2015 EXAM - MARCH
A. Object of the contract The nature of the relationship is consensual MED BOARDS; 2016
B. Cause or consideration and fiduciary. TOPNOTCH MD
C. Consent FROM LA SALLE)
D. Professional fee Source: Topnotch handout on legal medicine.
E. All of the above

2 The preliminary examination of the Physician licensure Source: Topnotch handout on legal medicine. KRISTEL TANHUI DIAGNOSTIC
examination in the Philippines includes: (TOP 3 - AUG 2015 EXAM - MARCH
A. Anatomy, Physiology, Biochemistry only MED BOARDS; 2016
B. Anatomy, Physiology, Biochemistry, Microbiology TOPNOTCH MD
only FROM LA SALLE)
C. Anatomy, Physiology, Biochemistry, Microbiology,
Pathology only
D. Anatomy, Physiology, Biochemistry, Pathology and
Pharmacology only
E. Anatomy, Physiology, Biochemistry, Microbiology,
Pathology, Pharmacology only

3 Who comprises the board of medicine? Source: Topnotch handout on legal medicine. KRISTEL TANHUI DIAGNOSTIC
A. 1 chairman and 4 members (TOP 3 - AUG 2015 EXAM - MARCH
B. 1 chairman and 5 members MED BOARDS; 2016
C. 1 chairman and 6 members TOPNOTCH MD
D. 2 co-chairs and 4 members FROM LA SALLE)
E. 2 co-chairs and 6 members

4 The nature of the physician-patient relationship is/are: Source: Topnotch handout on legal medicine. KRISTEL TANHUI DIAGNOSTIC
A. Fiduciary (TOP 3 - AUG 2015 EXAM - MARCH
B. Consensual MED BOARDS; 2016
C. Serendipitous TOPNOTCH MD
D. A and B FROM LA SALLE)
E. All of the above

5 Which of the following is the site of remote action of Postmortem findings of epicardial KRISTEL TANHUI DIAGNOSTIC
arsenics and thus will have pathologic findings on hemorrhage is suggestive of arsenic (TOP 3 - AUG 2015 EXAM - MARCH
autopsy? poisoning. MED BOARDS; 2016
A. Peripheral nerve TOPNOTCH MD
B. Mucous membrane Source: Topnotch handout on legal medicine. FROM LA SALLE)
C. Liver
D. Heart
E. GIT

6 Fingerprints appear in the fetus as early as: Source: Topnotch handout on legal medicine. KRISTEL TANHUI DIAGNOSTIC
A. 2nd month (TOP 3 - AUG 2015 EXAM - MARCH
B. 3rd month MED BOARDS; 2016
C. 4th month TOPNOTCH MD
D. 5th month FROM LA SALLE)
E. 6th month

TOPNOTCH MEDICAL BOARD PREP LEGAL MEDICINE AND JURIS SUPEREXAM Page 1 of 86
For inquiries visit www.topnotchboardprep.com.ph or email us at topnotchmedicalboardprep@gmail.com
TOPNOTCH MEDICAL BOARD PREP LEGAL MEDICINE AND JURIS SUPEREXAM
For inquiries visit www.topnotchboardprep.com.ph or email us at topnotchmedicalboardprep@gmail.com
Item QUESTION EXPLANATION AUTHOR TOPNOTCH
# EXAM
7 Which of the following is not helpful in distinguishing Parts of the skeleton which are useful in KRISTEL TANHUI DIAGNOSTIC
the sex of the skeleton? determining the sex of the skeleton: Pelvis, (TOP 3 - AUG 2015 EXAM - MARCH
A. Pelvis sternum, skull, humerus, femur. The pelvis is MED BOARDS; 2016
B. Sternum the most informative. TOPNOTCH MD
C. Skull FROM LA SALLE)
D. Humerus Source: Topnotch handout on legal medicine.
E. 3rd Proximal Phalanges

8 A method of identification which utilizes The rogue’s gallery and photographic profile KRISTEL TANHUI DIAGNOSTIC
anthropometrics: both utilize pictures or sketches for (TOP 3 - AUG 2015 EXAM - MARCH
A. Rougue’s gallery identification. MED BOARDS; 2016
B. Bertillion system Portrait parle is a verbal, accurate and TOPNOTCH MD
C. Portrait parle picturesque description of the person FROM LA SALLE)
D. Photographic profile identified.
E. All of the above
Source: Topnotch handout on legal medicine.

9 On postmortem examination, cutis galina or Cutis galina – submerged in water KRISTEL TANHUI DIAGNOSTIC
washerwoman’s hands were noted on the body. This Cutis galina – submerged in water…. (TOP 3 - AUG 2015 EXAM - MARCH
signifies: MED BOARDS; 2016
A. The cause of death is drowning Source: Topnotch handout on legal medicine. TOPNOTCH MD
B. The occupation of the deceased involved water FROM LA SALLE)
exposure
C. The corpse has been submerged in water for a long
period of time
D. The deceased had a chronic skin disorder prior to
death
E. The deceased died of anaphylaxis
10 On changing the position position of a cadaver, the Kinds of portmortem lividity KRISTEL TANHUI DIAGNOSTIC
location of lividity does not shift. Which form of - Hypostatic: Early stage, blood is still fluid (TOP 3 - AUG 2015 EXAM - MARCH
suggilation is this? within the blood vessels, any change in MED BOARDS; 2016
A. Disinterment position of the body leads to the formation of TOPNOTCH MD
B. Hypostatic lividity in another place FROM LA SALLE)
C. Osmotic - Diffusion: Later stage, blood has coagulated
D. Diffusion and changes in position will not change the
E. Algor mortis location of lividity

Source: Topnotch handout on legal medicine.
11 The most prominent sign of the death is: Algor mortis is a progressive fall of the body KRISTEL TANHUI DIAGNOSTIC
A. Algor mortis temperature. It is the most prominent sign of (TOP 3 - AUG 2015 EXAM - MARCH
B. Rigor mortis death. MED BOARDS; 2016
C. Cadaveric spasm Algor mortis – most prominent sign of death TOPNOTCH MD
D. Negative respiration FROM LA SALLE)
E. Negative heart rate and BP Source: Topnotch handout on legal medicine.

12 The instantaneous stiffening of a cadaver postmortem Rigor mortis, cadaveric rigidity, and death KRISTEL TANHUI DIAGNOSTIC
an example of which is a victim tightly grasping a stiffening are synonymous. It is universal and (TOP 3 - AUG 2015 EXAM - MARCH
weapon is called: its onset is around 3-6 hours postmortem. MED BOARDS; 2016
A. Cadaveric spasm Cadaveric spasm may or may not occur. Its TOPNOTCH MD
B. Rigor mortis onset is instantaneous FROM LA SALLE)
C. Cadaveric rigidity
D. Death stiffening Source: Topnotch handout on legal medicine.
E. Algor mortis

13 What makes up the primer? Source: Topnotch handout on legal medicine. KRISTEL TANHUI DIAGNOSTIC
A. Antimony, Copper, Lead (TOP 3 - AUG 2015 EXAM - MARCH
B. Antimony, Barium, Lead MED BOARDS; 2016
C. Copper and Lead TOPNOTCH MD
D. Antimony and Lead FROM LA SALLE)
E. Lead only

14 Which test used to determine live birth involves Fodere’s Test/Hydrostatic Test – observation KRISTEL TANHUI DIAGNOSTIC
submerging the lungs in water and observing for of floating lungs (TOP 3 - AUG 2015 EXAM - MARCH
flotation: Wredin’s Test – The middle ear of a child, MED BOARDS; 2016
A. Fodere’s Test before birth is filled with gelatinous TOPNOTCH MD
B. Wredin’s Test embryonic connective tissues. After birth, FROM LA SALLE)
C. Breslau’s Test this disappears.
D. Icard’s Test Breslau’s Test – floating organs (stomach,
E. Magnus Test intestines)

Icards and Magnus Test are for identifying
circulation in a cadaver and is not for
identification of live birth.

Source: Topnotch handout on legal medicine.

TOPNOTCH MEDICAL BOARD PREP LEGAL MEDICINE AND JURIS SUPEREXAM Page 2 of 86
For inquiries visit www.topnotchboardprep.com.ph or email us at topnotchmedicalboardprep@gmail.com
TOPNOTCH MEDICAL BOARD PREP LEGAL MEDICINE AND JURIS SUPEREXAM
For inquiries visit www.topnotchboardprep.com.ph or email us at topnotchmedicalboardprep@gmail.com
Item QUESTION EXPLANATION AUTHOR TOPNOTCH
# EXAM
15 Dr. Juan dela Cruz witnessed a stabbing incident while Ordinary witness describes only what he KRISTEL TANHUI DIAGNOSTIC
doing his groceries. He was called upon in court to perceived without providing interpretation (TOP 3 - AUG 2015 EXAM - MARCH
describe what he saw and heard during the incident. of giving professional opinions. MED BOARDS; 2016
What form of witness is he appearing for? TOPNOTCH MD
A. Expert witness Expert witness provides interpretation and FROM LA SALLE)
B. Professional witness opinions regarding facts according to his/her
C. Ordinary witness field of expertise.
D. Medical witness
E. Civilian witness Source: Topnotch handout on legal medicine.

16 Dr. John Doe was appalled by something the judge said Direct contempt of court: misbehavior in the KRISTEL TANHUI DIAGNOSTIC
during a hearing wherein he was a witness. He hits the presence of or near a court/judge. (TOP 3 - AUG 2015 EXAM - MARCH
judge of the face. He may be cited for: MED BOARDS; 2016
A. Direct contempt of court Indirect contempt of court: misbehavior TOPNOTCH MD
B. Indirect contempt of court outside the court (ex. Not showing up for a FROM LA SALLE)
C. Disrespect for the court hearing when there is no good excuse)
D. Homicide
E. Murder Source: Topnotch handout on legal medicine.

17 A beautician got ahold of some clindamycin topical Source: Topnotch handout on legal medicine. KRISTEL TANHUI DIAGNOSTIC
ointment and has been prescribing it to her clients for (TOP 3 - AUG 2015 EXAM - MARCH
treating acne. Which of the following applies? MED BOARDS; 2016
A. She is performing a cosmetic function as a beautician TOPNOTCH MD
B. She can invoke her right of occupation to make a FROM LA SALLE)
living and support herself
C. This is freedom of action
D. She is illegally practicing medicine
E. None of the above

18 Who is authorized to perform an autopsy? Source: Topnotch handout on legal medicine. KRISTEL TANHUI DIAGNOSTIC
A. Medical officers of law enforment agencies (TOP 3 - AUG 2015 EXAM - MARCH
B. Health officers of local health departments MED BOARDS; 2016
C. Medical staff members of accredited hospitals TOPNOTCH MD
D. A and B only FROM LA SALLE)
E. All of the above

19 An IQ range of which is exempting from criminal 0-20: Idiot – exempt (functionally equivalent KRISTEL TANHUI DIAGNOSTIC
liability: to <2y/o) (TOP 3 - AUG 2015 EXAM - MARCH
A. 0-20 20-40: Imbecile – exempt (2-7y/o) MED BOARDS; 2016
B. 20-40 40-70: Moron – mitigating (7-12y/o) TOPNOTCH MD
C. 40-70 FROM LA SALLE)
D. A and B My mnemonic for this is idiot, imbecile and
E. All of the above moron are alphabetically arranged.

Source: Topnotch handout on legal medicine.

20 Paraplegic patient promised to pay Dr. Juan dela Cruz Contingent, package deal fee and fee splitting KRISTEL TANHUI DIAGNOSTIC
P1M if he can make him walk again. This form of are unethical. (TOP 3 - AUG 2015 EXAM - MARCH
professional fee is called: - Contingent: payment after results. Unethical MED BOARDS; 2016
A. Contingent fee because doctor should not be gambling TOPNOTCH MD
B. Package deal fee - Package deal and fee splitting (referral fee FROM LA SALLE)
C. Fee splitting sharing) are unethical because the practice of
D. Simple contractual fee medicine is not a business venture
E. Retainers fee
Simple contractual fee and retainers fee are
ethical.
- Simple contractual fee: fee for service
- Retainers fee: fee for hours of duty
regardless of how many services rendered
during that period of time

Source: Topnotch handout on legal medicine.
21 After graduating from medical school, you begin using LESTER BRYAN CO MIDTERM 1
the title MD after your name. You are (TOP 10 - AUG 2015 EXAM - MARCH
A. Commiting malpractice MED BOARDS; 2016
B. Commiting fraud TOPNOTCH MD
C. Academically entitled to use MD FROM UST)
D. Illegally practicing medicine
E. Academically entitled to use MD after passing the
revalida

TOPNOTCH MEDICAL BOARD PREP LEGAL MEDICINE AND JURIS SUPEREXAM Page 3 of 86
For inquiries visit www.topnotchboardprep.com.ph or email us at topnotchmedicalboardprep@gmail.com
TOPNOTCH MEDICAL BOARD PREP LEGAL MEDICINE AND JURIS SUPEREXAM
For inquiries visit www.topnotchboardprep.com.ph or email us at topnotchmedicalboardprep@gmail.com
Item QUESTION EXPLANATION AUTHOR TOPNOTCH
# EXAM
22 What is the study of identification of fingerprints by LESTER BRYAN CO MIDTERM 1
comparison? (TOP 10 - AUG 2015 EXAM - MARCH
A. Dactyloscopy MED BOARDS; 2016
B. Dactylography TOPNOTCH MD
C. Poroscopy FROM UST)
D. Dermatoglyphics
E. Graphology

23 Disinternment is also known as LESTER BRYAN CO MIDTERM 1


A. Embalming (TOP 10 - AUG 2015 EXAM - MARCH
B. Euthanasia MED BOARDS; 2016
C. Exhumation TOPNOTCH MD
D. Disembowelment FROM UST)
E. Dissection

24 Manceras children are children born to LESTER BRYAN CO MIDTERM 1


A. Live-in partners (TOP 10 - AUG 2015 EXAM - MARCH
B. Incestuous couples MED BOARDS; 2016
C. Homosexual couples TOPNOTCH MD
D. Prostitutes FROM UST)
E. Families below poverty line

25 A physician who stubbornly refuses to go to court when Direct contempt occurs when a person LESTER BRYAN CO MIDTERM 1
summoned may be cited for: speaks words or commits acts in the (TOP 10 - AUG 2015 EXAM - MARCH
A. Disobedience presence of the court or a judge acting MED BOARDS; 2016
B. Dereliction of civic duty judicially or when a person resists or TOPNOTCH MD
C. Direct contempt interferes with the lawful authority of the FROM UST)
D. Indirect contempt court in its presence or so near the court or
E. May not be cited judge as to interrupt or hinder judicial
proceedings.

26 Wood alcohol is otherwise known as: LESTER BRYAN CO MIDTERM 1


A. Propylene glycol (TOP 10 - AUG 2015 EXAM - MARCH
B. Isopropyl alcohol MED BOARDS; 2016
C. Ethylene glycol TOPNOTCH MD
D. Ethyl alcohol FROM UST)
E. Methyl alcohol

27 Kimberly Chu. sues Dr. Balu for mental anguish, social Actual or compensatory damages simply LESTER BRYAN CO MIDTERM 1
humiliation and loss of sleep after a botched facelift make good or replace the loss caused by the (TOP 10 - AUG 2015 EXAM - MARCH
operation. What type of damages can Ms. Chu recover wrong. Nominal damages vindicate or MED BOARDS; 2016
from this situation? recognize the injured party’s right to a TOPNOTCH MD
A. Actual damages property that has been violated or invaded, FROM UST)
B. Compensatory damages with minimal or no compensation. Exemplary
C. Exemplary damages or corrective damages are intended to serve
D. Moral damages as a deterrent to serious wrongdoings.
E. Nominal damages

28 Jejo scratched Mar in the face in an altercation. You LESTER BRYAN CO MIDTERM 1
attended to the injury that resolved in less than 10 days (TOP 10 - AUG 2015 EXAM - MARCH
but a resultant keloid scar formed. The legal MED BOARDS; 2016
classification of this injury is: TOPNOTCH MD
A. Serious physical injuries FROM UST)
B. Less serious injuries
C. Slight injuries
D. Mutilation
E. Scarring injury

29 If the parents are type A and type B, which of the SIMILAR TO PREVIOUS BOARD EXAM LESTER BRYAN CO MIDTERM 1
following would be the blood group of their children? CONCEPT/PRINCIPLE. Both A and B alleles (TOP 10 - AUG 2015 EXAM - MARCH
A. Types O, A and B are dominant over O. O is a recessive allele. MED BOARDS; 2016
B. Types A, B, AB and O As a result, individuals who have an AO TOPNOTCH MD
C. Types A and B genotype will still have an A phenotype. FROM UST)
D. Types A, B and AB People who are type O have OO genotypes.
E. Type AB In other words, they inherited a recessive O
allele from both parents. The A and B alleles
are codominant. Therefore, if an A is
inherited from one parent and a B from the
other, the phenotype will be AB. A parent
with phenotype A may have genotype AA or
AO, a parent with phenotype B may have
genotype BB or BO. Therefore, they may
produce children with O phenotype (OO
genotype).
30 A verbal and accurate description of the person LESTER BRYAN CO MIDTERM 1
identified is termed (TOP 10 - AUG 2015 EXAM - MARCH
A. Anthropometry MED BOARDS; 2016
B. Bertillon System TOPNOTCH MD
C. Verbal Drawing FROM UST)
D. Portrait Parle
E. Oral description

TOPNOTCH MEDICAL BOARD PREP LEGAL MEDICINE AND JURIS SUPEREXAM Page 4 of 86
For inquiries visit www.topnotchboardprep.com.ph or email us at topnotchmedicalboardprep@gmail.com
TOPNOTCH MEDICAL BOARD PREP LEGAL MEDICINE AND JURIS SUPEREXAM
For inquiries visit www.topnotchboardprep.com.ph or email us at topnotchmedicalboardprep@gmail.com
Item QUESTION EXPLANATION AUTHOR TOPNOTCH
# EXAM
31 Type of medical evidence which is addressed to the LESTER BRYAN CO MIDTERM 1
senses of the court? (TOP 10 - AUG 2015 EXAM - MARCH
A. Autoptic evidence MED BOARDS; 2016
B. Corpus delicti evidence TOPNOTCH MD
C. Documentary evidence FROM UST)
D. Physical evidence
E. Circumstantial evidence

32 This finding in death is not consistent and may or may LESTER BRYAN CO MIDTERM 1
not appear on a person who died: (TOP 10 - AUG 2015 EXAM - MARCH
A. Rigor mortis MED BOARDS; 2016
B. Algor mortis TOPNOTCH MD
C. Putrefaction FROM UST)
D. Cadaveric spasm
E. Decomposition

33 The following medical fees are considered immoral or LESTER BRYAN CO MIDTERM 1
unethical under the Code of Medical Ethics EXCEPT? (TOP 10 - AUG 2015 EXAM - MARCH
A. Contingent fee MED BOARDS; 2016
B. Dichotomous fee TOPNOTCH MD
C. Retainer fee FROM UST)
D. Fee splitting
E. Straight fee

34 If a patient died or became incapacitated, the claim for LESTER BRYAN CO MIDTERM 1
medical fees shall be borne the following persons in the (TOP 10 - AUG 2015 EXAM - MARCH
order of: MED BOARDS; 2016
A. Spouse, parents, children, siblings TOPNOTCH MD
B. Spouse, parents, siblings, children FROM UST)
C. Spouse, children, parents, siblings
D. Children, spouse, siblings, parents
E. Children, spouse, parents, siblings

35 Which of the following is NOT an inherent right of the LESTER BRYAN CO MIDTERM 1
physician? (TOP 10 - AUG 2015 EXAM - MARCH
A. Choose patients MED BOARDS; 2016
B. Exemption from execution of instruments and library TOPNOTCH MD
C. Avail of hospital services FROM UST)
D. Limit the practice of medicine
E. Determine the appropriate management and
procedures for his patients

36 For a child to be legitimate, he must be born in a lawful LESTER BRYAN CO MIDTERM 1


wedlock or within these number of days after the (TOP 10 - AUG 2015 EXAM - MARCH
dissolution of marriage: MED BOARDS; 2016
A. 120 days TOPNOTCH MD
B. 180 days FROM UST)
C. 300 days
D. 100 days
E. 365 days

37 Saturnine gout pertains to: LESTER BRYAN CO MIDTERM 1


A. Pseudogout (TOP 10 - AUG 2015 EXAM - MARCH
B. Gout caused by excess thiazide use MED BOARDS; 2016
C. Gout caused by cyclosporine toxicity TOPNOTCH MD
D. Gout caused by lead poisoning FROM UST)
E. Gout in people with Lesch-Nyhan syndrome

38 A man marries his stepdaughter and they conceive a LESTER BRYAN CO MIDTERM 1
child. The child is considered: (TOP 10 - AUG 2015 EXAM - MARCH
A. adulterous MED BOARDS; 2016
B. natural child by legal fiction TOPNOTCH MD
C. natural child by presumption FROM UST)
D. legitimated
E. Natural child by wedlock

39 Principle applied when the medical fee is not specified: LESTER BRYAN CO MIDTERM 1
A. "Assumpsit on quanthum merit" (TOP 10 - AUG 2015 EXAM - MARCH
B. "lex Loci contract" MED BOARDS; 2016
C. "Dura lex sed Lex" TOPNOTCH MD
D. "Sine Qua now" FROM UST)
E. "Quis custodiet ipsos custodes"

TOPNOTCH MEDICAL BOARD PREP LEGAL MEDICINE AND JURIS SUPEREXAM Page 5 of 86
For inquiries visit www.topnotchboardprep.com.ph or email us at topnotchmedicalboardprep@gmail.com
TOPNOTCH MEDICAL BOARD PREP LEGAL MEDICINE AND JURIS SUPEREXAM
For inquiries visit www.topnotchboardprep.com.ph or email us at topnotchmedicalboardprep@gmail.com
Item QUESTION EXPLANATION AUTHOR TOPNOTCH
# EXAM
40 A patient was found dead in his room with multiple stab LESTER BRYAN CO MIDTERM 1
wounds. On autopsy, you saw prominent superficial (TOP 10 - AUG 2015 EXAM - MARCH
veins with reddish discoloration on his flanks. This MED BOARDS; 2016
finding is called? TOPNOTCH MD
A. Postmortem lividity FROM UST)
B. Maceration
C. Marbolization
D. Cadaveric lividity
E. Adipocere formation

41 Which of the following laws is enforced by state? Law which is not enforced by the state: GEORGE MICHAEL MIDTERM 2
A. Moral Law Natural Law (e.g law of gravity, law of SOSUAN (TOP 5 - EXAM - MARCH
B. Natural Law nature), Divine Law (e.g. 10 commandments), AUG 2015 MED 2016
C. Divine Law Moral Law Law which is BOARDS; TOPNOTCH
D. Law of Gravity enforced by state: Substantive Law (e.g. law MD FROM UST)
E. Adjective Law om property rights),
Procedural/Adjective/Remedial Law (law on
criminal procedure)
42 The following are the functions of the Board of Functions of Board of Medicine: 1. To GEORGE MICHAEL MIDTERM 2
Medicine EXCEPT: determine and to prepare contents of the SOSUAN (TOP 5 - EXAM - MARCH
A. Determine and prepare contents of licensure licensure examination 2. Promulgate rules AUG 2015 MED 2016
examination and regulations for the proper conduct of the BOARDS; TOPNOTCH
B. Administering and conducting the licensure examinations, corrections and registrations MD FROM UST)
examination in accordance to its rules and regulations 3. Administer oath 4. Investigate violations,
C. Promulgate rules and regulations for the proper issues subpoena 5. Issue Certificate of
conduct of the examinations, corrections and registration 6. Suspend, revoke or reissue
registrations Certificate of Registration
D. Administer oath Administering and conducting licensure
E. Issue Certificate of Registration examination: Function of PRC
43 Which of the following is an Inherent Right of a Inherent Rights: 1. To choose patients 2. GEORGE MICHAEL MIDTERM 2
physician Limit the practice 3. Determine the SOSUAN (TOP 5 - EXAM - MARCH
A. Avail of hospital services appropriate management 4. Avail of hospital AUG 2015 MED 2016
B. Right of exemption from execution of instruments services Incidental Rights: BOARDS; TOPNOTCH
and library 1. Right of way while responding to call of MD FROM UST)
C. To hold certain public or private offices emergency 2. Right of exemption from
D. Right to compensation execution of instruments and library 3.
E. Right to membership in medical societies Determine appropriate management
procedures 4. Avail of hospital services
44 Which of the following fees is considered ethical? Ethical fees: Simple contractual fee; Retainer GEORGE MICHAEL MIDTERM 2
A. Commission Fee fee Unethical fees: Contingent SOSUAN (TOP 5 - EXAM - MARCH
B. Retainer Fee fee, Dichotomous/commission/fee splitting; AUG 2015 MED 2016
C. Fee Splitting Straight fee BOARDS; TOPNOTCH
D. Straight Fee MD FROM UST)
E. Contingent Fee

45 Revocation of license is a sanction imposed against a Prison/fine: Criminal liability; Damages: Civil GEORGE MICHAEL MIDTERM 2
doctor when he is guilty of what liability? liability; Reprimand, Suspension, Revocation: SOSUAN (TOP 5 - EXAM - MARCH
A. Criminal liability Administrative Liability AUG 2015 MED 2016
B. Administrative liability BOARDS; TOPNOTCH
C. Civil liability MD FROM UST)
D. No liability
E. AOTA

46 Dr. X operated on a patient and inadvertently left Doctrine of Res Ipsa Loquitor (Common GEORGE MICHAEL MIDTERM 2
behind a needle. Which doctrine is applicable in this knowledge doctrine) is the nature of the SOSUAN (TOP 5 - EXAM - MARCH
case? wrongful act is suggestive of negligence AUG 2015 MED 2016
A. Captain of the ship BOARDS; TOPNOTCH
B. Doctrine of Forseeabiity MD FROM UST)
C. Fellow Servant
D. Res Ipsa Loquitor
E. Ostensible agent

47 The following are conditions for award of moral Conditions for award of moral damages: 1. GEORGE MICHAEL MIDTERM 2
damages EXCEPT: There must be an injury 2. There must be a SOSUAN (TOP 5 - EXAM - MARCH
A. There must be an injury culpable act 3. Wrong doing act us the AUG 2015 MED 2016
B. There must be a culpable act proximate cause of the injury 4. Award of BOARDS; TOPNOTCH
C. Award of damages is based on Article 2217 of the damages is based on Article 2219 of the civil MD FROM UST)
Civil code code
D. Wrong doing act is the proximate cause of the
injury
E. NOTA
48 Immediate transplantation should be done after death Immediate transplantation: kidney, heart, GEORGE MICHAEL MIDTERM 2
to ascertain success on which of the following organs? lung and liver Transplanation SOSUAN (TOP 5 - EXAM - MARCH
A. Kidney may be done several hours after removal AUG 2015 MED 2016
B. Cornea from the donor: Cornea, skin, bone, blood BOARDS; TOPNOTCH
C. Blood vessels vessels MD FROM UST)
D. AOTA
E. NOTA

TOPNOTCH MEDICAL BOARD PREP LEGAL MEDICINE AND JURIS SUPEREXAM Page 6 of 86
For inquiries visit www.topnotchboardprep.com.ph or email us at topnotchmedicalboardprep@gmail.com
TOPNOTCH MEDICAL BOARD PREP LEGAL MEDICINE AND JURIS SUPEREXAM
For inquiries visit www.topnotchboardprep.com.ph or email us at topnotchmedicalboardprep@gmail.com
Item QUESTION EXPLANATION AUTHOR TOPNOTCH
# EXAM
49 Type of asphyxial death characterize by failure of Anoxia death: failure of the arterial blood to GEORGE MICHAEL MIDTERM 2
arterial blood to become naturally saturated with become naturallly saturated with oxygen
SOSUAN (TOP 5 - EXAM - MARCH
oxygen. Anemic anoxia death: due to decrease AUG 2015 MED 2016
A. Histotoxic Anoxia capacity of blood to carry oxygen Stagnant BOARDS; TOPNOTCH
B. Stagnant anoxia anoxia: due to failure of circulation
MD FROM UST)
C. Anoxia death Histotoxic anoxia: failure of cellular oxidation
D. Anemic anoxia death process
E. NOTA

50 One of the following is not helpful in determining the Following bones are helpful in determining GEORGE MICHAEL MIDTERM 2
sex of the skeleton the skeleton sex: skull, sternum, pelvis, SOSUAN (TOP 5 - EXAM - MARCH
A. Femur humerus, femur AUG 2015 MED 2016
B. Humerus BOARDS; TOPNOTCH
C. Pelvis MD FROM UST)
D. Sternum
E. NOTA

51 Dactyloscopy is: Dactyloscopy: Art of comparing fingerprints GEORGE MICHAEL MIDTERM 2


A. Art of comparing fingerprints for identification for identification SOSUAN (TOP 5 - EXAM - MARCH
B. Art and study of recording fingerprints as a means AUG 2015 MED 2016
of identification BOARDS; TOPNOTCH
C. Art of changing fingerprints MD FROM UST)
D. The study of the pores on the papillary or friction
ridges of the skin for the purpose of identification
E. NOTA

52 Which of the following is TRUE in estimating how long a Methods of estimating how long a person has GEORGE MICHAEL MIDTERM 2
person has been dead from cooling of the body? been dead from cooling of the body: 1. When SOSUAN (TOP 5 - EXAM - MARCH
A. When the body temperature is normal at the time the body temperature is normal at the time of AUG 2015 MED 2016
of death, the average rate of fall of the temperature death, the average rate of fall of the BOARDS; TOPNOTCH
during the second hour is one-half the difference of the temperature during the first hour is one-half MD FROM UST)
body temperature and that of air the difference of the temperature and that of
B. During the next two hours, the temperature fall is air 2. During the next two hours, the
1/3 of the previous rate, and during the succeeding two temperature fall is 1/2 of the previous rate,
hours, it is 1/2 of the last mentioned rate and during the succeeding two hours, it is
C. As a general rule the body attains the temperature 1/2 of the last mentioned rate 3. As a general
of the surrounding air from 8-10 hours after death in rule the body attains the temperature of the
tropical countries surrounding air from 12-15hours after death
D. To make an approximate estimate of the duration in tropical countries 4. To make an
of death from the body temperature the ff formula is approximate estimate of the duration of
used: (Rectal temperature) 98.4 - (Normal Temp)/1.5 death from the ff formula: (Normal temp)
E. NOTA 98.4 - (Rectal temp)/1.5

53 Which of the following tests determines whether the GEORGE MICHAEL MIDTERM 2
specimen is of human blood or not: SOSUAN (TOP 5 - EXAM - MARCH
A. Acetone-hemin of Wagenhaar Test AUG 2015 MED 2016
B. Puramine Test BOARDS; TOPNOTCH
C. Takayama test MD FROM UST)
D. Precipitin Test
E. Berberio's Test

54 Which of the following is a characteristic of exit Exit wounds: Size bigger except in contact GEORGE MICHAEL MIDTERM 2
wounds? fire, shape is variable, abrasion collar absent, SOSUAN (TOP 5 - EXAM - MARCH
A. Abrasion collar present products of combustion absent, edges are AUG 2015 MED 2016
B. Products of combustion present everted and punch out BOARDS; TOPNOTCH
C. Size is bigger in cases of contact fire MD FROM UST)
D. Edges are inverted
E. Variable shape

55 Cadaveric spasm is also known as: GEORGE MICHAEL MIDTERM 2


A. Rigor Mortis SOSUAN (TOP 5 - EXAM - MARCH
B. Postmortem muscular irritability AUG 2015 MED 2016
C. Instantaneous rigidity BOARDS; TOPNOTCH
D. Postmortem rigidity MD FROM UST)
E. AOTA

56 Killing a child less than 3 days old is called? GEORGE MICHAEL MIDTERM 2
A. Parricide SOSUAN (TOP 5 - EXAM - MARCH
B. Murder AUG 2015 MED 2016
C. Infanticide BOARDS; TOPNOTCH
D. Abortion MD FROM UST)
E. NOTA

TOPNOTCH MEDICAL BOARD PREP LEGAL MEDICINE AND JURIS SUPEREXAM Page 7 of 86
For inquiries visit www.topnotchboardprep.com.ph or email us at topnotchmedicalboardprep@gmail.com
TOPNOTCH MEDICAL BOARD PREP LEGAL MEDICINE AND JURIS SUPEREXAM
For inquiries visit www.topnotchboardprep.com.ph or email us at topnotchmedicalboardprep@gmail.com
Item QUESTION EXPLANATION AUTHOR TOPNOTCH
# EXAM
57 Method of conducting a search in which in which the Strip Method: area is blocked out in the form GEORGE MICHAEL MIDTERM 2
area is blocked out in the form of a rectangle in which of rectangle. The searcher proceeds slowly at SOSUAN (TOP 5 - EXAM - MARCH
the searcer proceeds slowly at the same pace along the the same pace along the path parallel to the AUG 2015 MED 2016
path parallel to the one side of the rectangle? to the side of the rectangle.
BOARDS; TOPNOTCH
A. Zone Grid Method: the searchers will traverse first MD FROM UST)
B. Wheel parallel to the base and then parallel to the
C. Spiral side
D. Strip Spiral Method: the searchers follow each
E. Grid other in the path in the spiral manner
beginning from the center towards the
outside or vice versa Wheel Method:
the searchers gather at the center and
proceed outwards along radii or spokes
Zone Method: Whole area is divided into
subdivision or quadrants and search is made
in the individual quadrants
58 these are the damages imposed on a physician as a GEORGE MICHAEL MIDTERM 2
punishment and to serve as an example for the medical SOSUAN (TOP 5 - EXAM - MARCH
profession AUG 2015 MED 2016
A. Liquidated damages BOARDS; TOPNOTCH
B. Temperate damages MD FROM UST)
C. Compensatory damages
D. Nominal damages
E. Corrective damages

59 TRUE of simple seduction EXCEPT: Simpe Seduction: 1, The offended party over GEORGE MICHAEL MIDTERM 2
A. The offended party over 12 years old but less than 12 years old but less than 18 years of age 2. SOSUAN (TOP 5 - EXAM - MARCH
18 years old Offended party must be single or a widow of AUG 2015 MED 2016
B. Offended party must be single or a widow of good good reputation 3. Sexual Intercourse with BOARDS; TOPNOTCH
reputation the offended party 4. deceit MD FROM UST)
C. Sexual intercourse with the offended party
D. Deceit
E. The offender commits the act by abuse of authority

60 Requires a MD to produce documents or papers in his Subpoena ad testificandum: MD is required GEORGE MICHAEL MIDTERM 2
possession to appear before a trial or investigation
SOSUAN (TOP 5 - EXAM - MARCH
A. Subpoena ad testificandum Subpoena duces tecum: MD is required to AUG 2015 MED 2016
B. Subpoena duces tecum produce some documents which are under BOARDS; TOPNOTCH
C. Subpoena duces tecum et ad testefecandum his control or possession that are pertinent MD FROM UST)
D. Injunction to the issues of the controversy
E. NOTA Subpoena duces tecum et ad testificandum:
combination of 2
61 This type of law pescribes the right and obligations of SIMILAR TO PREVIOUS BOARD EXAM JAN CHRISTIAN MIDTERM 3
persons in their daily relation with each other and with CONCEPT/PRINCIPLE FELICIANO (TOP 2 - EXAM - MARCH
society. AUG 2015 MED 2016
A. Natural law BOARDS; TOPNOTCH
B. Substantive law MD FROM UST)
C. Procedural law
D. Public law
E. Divine law

62 One of the ff is not a function of the Board of Medicine Administering and conducting the licensure JAN CHRISTIAN MIDTERM 3
A. Determining and preparing contents of the licnesure exam is a function of the Professional FELICIANO (TOP 2 - EXAM - MARCH
exams Regulatory Commision and not the Board of AUG 2015 MED 2016
B. Promulgaging rules and rgulation for the proer Medicine BOARDS; TOPNOTCH
conduct of the examinations MD FROM UST)
C. Administering the licensure examinations
D. Administering the oath
E. Investigating violations, issues and subpoenas

63 Which one of the following is an act that constitutes the A registered nurse can take a patient's BP but JAN CHRISTIAN MIDTERM 3
practice of Medicine? if one gives advices or precribes treatment FELICIANO (TOP 2 - EXAM - MARCH
A. Any medical student enrolled in the medical school one is ocnsdered practice of Medicine and is AUG 2015 MED 2016
under direct supervision of a physician therefore illegal. All other acts does not BOARDS; TOPNOTCH
B. Registered nurse who takes blood pressure of constitute practice of Medicine MD FROM UST)
patients and gives advices to lower blood pressure
C. Registered optometrist engaged in mechanical
examination of the eye
D. A lay person who gives CPR to a stranger who had a
heart attack
E. A grandmother who gives salabat to his grandchild
who has cough

TOPNOTCH MEDICAL BOARD PREP LEGAL MEDICINE AND JURIS SUPEREXAM Page 8 of 86
For inquiries visit www.topnotchboardprep.com.ph or email us at topnotchmedicalboardprep@gmail.com
TOPNOTCH MEDICAL BOARD PREP LEGAL MEDICINE AND JURIS SUPEREXAM
For inquiries visit www.topnotchboardprep.com.ph or email us at topnotchmedicalboardprep@gmail.com
Item QUESTION EXPLANATION AUTHOR TOPNOTCH
# EXAM
64 ALl of the ff persons can have limited practice without Med students who have completed the first 4 JAN CHRISTIAN MIDTERM 3
any certificate of registration EXCEPT? years of Medicine, graduates of Medicine and FELICIANO (TOP 2 - EXAM - MARCH
A. MD's from other countries called for consultation and RN may be given limited and special AUG 2015 MED 2016
exclusively for specific cases authorization by the secretary of Health BOARDS; TOPNOTCH
B. Foreign MDs employed as exhange professors in durign epidemics or national emergencies MD FROM UST)
special branches of Medicine only.
C. Commisioned Medical officers of the US armed forces
D. Graduates of medicine during ROTC training
E. Plastic surgeons from the US form Operation Smile
with proper authorization

65 A patient follows the advice of his physician fully In a guidance cooperation relation- patient is JAN CHRISTIAN MIDTERM 3
because he trusts that his doctor knows best. What kind conscious, seeks advice and hel and is willing FELICIANO (TOP 2 - EXAM - MARCH
of psychological pattern of MD-patient relationship is to cooperate. MD is in a position of trust. AUG 2015 MED 2016
being described? BOARDS; TOPNOTCH
A. Guidance-cooperation relation MD FROM UST)
B. Activity-passivity relation
C. Mutual paricipation relation
D. Fraternity-trust relation
E. Camarederie-respect relation

66 The ff are duties and obligations imposed on the The MD-Patient relationship does NOT imply JAN CHRISTIAN MIDTERM 3
physician in the MD patient relationship any promise that the treatment will benefit FELICIANO (TOP 2 - EXAM - MARCH
A. He should possess the knowledge and skill of which the patient not does not promise that the AUG 2015 MED 2016
an average MD is expected. treatment will not harm the patient i.e. BOARDS; TOPNOTCH
B. He should use use such skill and knowledge with cancer chemotherapy MD FROM UST)
ordinary care and diligence
C. He is obliged to exercise the best judgement
D. He has a duty to observe the utmost good faith
E. He has a duty that the treatment will benefit and not
harm the patient

67 One of the ff is an incidental right of a physician and not SIMILAR TO PREVIOUS BOARD EXAM JAN CHRISTIAN MIDTERM 3
an inherent right? CONCEPT/PRINCIPLE FELICIANO (TOP 2 - EXAM - MARCH
A. To choose patients AUG 2015 MED 2016
B. Limit one's practice BOARDS; TOPNOTCH
C. Exemption from execution of instruments and library MD FROM UST)
D. Determine the appropriate management
E. Avail of hospital services

68 Dr. Octavius works in Oscorp Industries as a company Retainer fee is measured by the space of time JAN CHRISTIAN MIDTERM 3
doctor. He treats the employyes of the patients and not by the quality and quantitiy of FELICIANO (TOP 2 - EXAM - MARCH
exclusively during office hours and does it poorly. What medical services rendered. AUG 2015 MED 2016
kind of medical fee is he under? BOARDS; TOPNOTCH
A. Simple contractual fee MD FROM UST)
B. Straight fee
C. Contingent fee
D. Dichotomous fee
E. Retainer fee

69 All of the ff are instances wherein the MD cannot SIMILAR TO PREVIOUS BOARD EXAM JAN CHRISTIAN MIDTERM 3
recover the professional fee EXCEPT? CONCEPT/PRINCIPLE FELICIANO (TOP 2 - EXAM - MARCH
A. When the patient died due to an honest mistake by AUG 2015 MED 2016
the doctor BOARDS; TOPNOTCH
B. When there is an agreement that the service is MD FROM UST)
gratitious
C. Doctors in government charity hospitals, health
centers and rural health units
D. Those covered in Medicare
E. When there is expressed contract to cure and the MD
fails to comply with such agreement
70 What is the evidence needed for conviction of a Criminal- Guilt beyond reasonable doubt; JAN CHRISTIAN MIDTERM 3
physician in an administrative case Civil- Prepoderence of evidence; FELICIANO (TOP 2 - EXAM - MARCH
A. Guilt beyon reasonable doubt Adminstrative- Substantial evidence AUG 2015 MED 2016
B. Preopenderence of evidence BOARDS; TOPNOTCH
C. Substantial evidence MD FROM UST)
D. Autoptotic evidence
E. Circumstantial evidence

71 The pathologist diagnosed a tissue as being malignant Doctrine of ostensible agent states that JAN CHRISTIAN MIDTERM 3
when it was infact benign leading to a accidental pathologists, radiologiss and FELICIANO (TOP 2 - EXAM - MARCH
mastectomy. The patient filed charges to the anesthesiologists are employees and at the AUG 2015 MED 2016
pathologist and the hospital as well. Filing of charges same time indepedendent contractors of the BOARDS; TOPNOTCH
agaisnt the hospital is according to what doctrine? hospital. MD FROM UST)
A. Borrowed servant doctrine
B. Captain of the ship doctrine
C. Doctrine of Res Ipsa Loquitur
D. Doctrine of Ostensible Agent
E. Doctrine of Vicarious liability

TOPNOTCH MEDICAL BOARD PREP LEGAL MEDICINE AND JURIS SUPEREXAM Page 9 of 86
For inquiries visit www.topnotchboardprep.com.ph or email us at topnotchmedicalboardprep@gmail.com
TOPNOTCH MEDICAL BOARD PREP LEGAL MEDICINE AND JURIS SUPEREXAM
For inquiries visit www.topnotchboardprep.com.ph or email us at topnotchmedicalboardprep@gmail.com
Item QUESTION EXPLANATION AUTHOR TOPNOTCH
# EXAM
72 After incision and drainage in the OPD, the pateint It is also known as the doctrine of common JAN CHRISTIAN MIDTERM 3
failed to take his antbiotics and developed sepsis. He fault. It is conduct on the part of the plaintiff, FELICIANO (TOP 2 - EXAM - MARCH
cannot recover damages from the doctor under what contributing as a legal cause of the harm that AUG 2015 MED 2016
doctrine? he has sufferred which falls below the BOARDS; TOPNOTCH
A. Doctrine of Contributory negligence standrard to which he is req'd to conform for MD FROM UST)
B. Doctrine of Continuing negligence his own protection.
C. Doctrine of Assumption of risk
D. Doctrine of last clear chance
E. Doctrine of foreseeability

73 Doctor X maliciously harmed the patient with intent There should have been the word EXCEPT in JAN CHRISTIAN MIDTERM 3
because of a personal grudge. Other than the usual the question. Exemplary or corrective FELICIANO (TOP 2 - EXAM - MARCH
damages, he was asked by the court to pay additional damages are monetary compensation over AUG 2015 MED 2016
damages because of the oppresive nature of the act. and abive actual or compensatory damages BOARDS; TOPNOTCH
What are these damages? awarded as punishment or deterrent bec of MD FROM UST)
A. Actual damages the wanton, reckless, malicious or oprresive
B. Compensatory damages nature of the wrong committed.
C. Moral damages
D. Corrective damages
E. Nominal damages

74 It is the art of identification by comparison of SIMILAR TO PREVIOUS BOARD EXAM JAN CHRISTIAN MIDTERM 3
fingerprints? CONCEPT/PRINCIPLE FELICIANO (TOP 2 - EXAM - MARCH
A. Dactyloscopy AUG 2015 MED 2016
B. Dactylography BOARDS; TOPNOTCH
C. Poroscopy MD FROM UST)
D. Graphology
E. Gonioscopy

75 All of the ff bones can be used to determine the sex of a To determine the sex: Pelvis, skull, sternum, JAN CHRISTIAN MIDTERM 3
human skeleton EXCEPT? femur and humerus; to determine the height: FELICIANO (TOP 2 - EXAM - MARCH
A. Pelvis Femur, humerus, tibia and radius AUG 2015 MED 2016
B. Skull BOARDS; TOPNOTCH
C. Femur MD FROM UST)
D. Sternum
E. Tibia

76 Which of the ff is a microscopic examination to Guaiacum test is a chemical examinaiton of JAN CHRISTIAN MIDTERM 3
determine rpesence of blood? the blood not a microscopic exam. FELICIANO (TOP 2 - EXAM - MARCH
A. Berberio's test AUG 2015 MED 2016
B. Florence test BOARDS; TOPNOTCH
C. Gonzales test MD FROM UST)
D. Guaiacum test
E. Teichmann's test

77 It is the complete, persistent and continuous cessation SIMILAR TO PREVIOUS BOARD EXAM JAN CHRISTIAN MIDTERM 3
of the vital functions of the brain, heart and lungs which CONCEPT/PRINCIPLE FELICIANO (TOP 2 - EXAM - MARCH
maintain life and health? AUG 2015 MED 2016
A. Molecular death BOARDS; TOPNOTCH
B. Cellular death MD FROM UST)
C Brain death
D. Somatic death
E. Permanent death

78 Which of the statements is true regarding post-mortem it is alsko known as rigor mortis/cadaveric JAN CHRISTIAN MIDTERM 3
rigidity? rigiditiy. It occurs 3-6 hours after death and FELICIANO (TOP 2 - EXAM - MARCH
A. It is also known as heat stiffening occurs early in thr aged and newborn. AUG 2015 MED 2016
B. Usually the whole body becomes stiff in 12 hours Cadaveric spasm not rigor mortis can be used BOARDS; TOPNOTCH
C. It occurs immediately after death to determine the nature of the crime. MD FROM UST)
D. Later onset in the aged and the newborn
E. Can be used to determine the nature of the crime

79 All of the statements regarding post mortem lividity is Answer is ABCD. I apologize, the word JAN CHRISTIAN MIDTERM 3
correct EXCEPT? EXCEPT should have been omitted in the FELICIANO (TOP 2 - EXAM - MARCH
A. It is the breaking down of the complex proteins of the question. Cadaveric lividity or Livor mortis is AUG 2015 MED 2016
body into simpler components the manner through which blood BOARDS; TOPNOTCH
B. Blood moves away from the dependent part of the accumulates in the msot depended portion fo MD FROM UST)
body the body. Color is uniform but isi suualy
C. It can appear elevated from the rest of the skin greensih at the start fo the decomposing
because of blood accumulation process. The color of the body can guide the
D. Color is heterogenous and becomes brownish red at examiner in the manner of death.
the start of decomposition
E. It can indicate the manner of death

TOPNOTCH MEDICAL BOARD PREP LEGAL MEDICINE AND JURIS SUPEREXAM Page 10 of 86
For inquiries visit www.topnotchboardprep.com.ph or email us at topnotchmedicalboardprep@gmail.com
TOPNOTCH MEDICAL BOARD PREP LEGAL MEDICINE AND JURIS SUPEREXAM
For inquiries visit www.topnotchboardprep.com.ph or email us at topnotchmedicalboardprep@gmail.com
Item QUESTION EXPLANATION AUTHOR TOPNOTCH
# EXAM
80 During an altercation, the nagging wife accidentally Prison correctional in its medium and JAN CHRISTIAN MIDTERM 3
sprayed sulfuric acid on the jobless husband's face maximum pd- loss of speech or the power to FELICIANO (TOP 2 - EXAM - MARCH
henceforth losing his sight on the left eye. The wife can hear, smell, loss of one eye, a hand, foot, an AUG 2015 MED 2016
be charged with what offense? arm or leg. BOARDS; TOPNOTCH
A. Prison mayor MD FROM UST)
B. Prison correctional in its medium and maximum pd
C. Prison correctional in its minimum and medium pd
D. Arresto mayor
E. Destierro

81 1. Which of the following organs is NOT included in Blood levels are utilized for alcohol, ANDREW TIU (TOP 1 FINAL EXAM -
examination for suspected barbiturate poisoning? pesticide, antibiotic, kerosine, and gasoline. - AUG 2015 MED MARCH 2016
a. stomach Legal Medicine Solis p. 715 BOARDS; TOPNOTCH
b. blood MD FROM CIM)
c. liver
d. kidney
e. urine

82 2. In M.J. Cuenco Avenue, a motorcyclist was hit by a 10 A - no signs of mental impairment or ANDREW TIU (TOP 1 FINAL EXAM -
wheeler truck. On examination of the driver, he face incoordination of movement and difficulty of - AUG 2015 MED MARCH 2016
was noted to be flushed with eyeballs congested. He is speech BOARDS; TOPNOTCH
observed to be reckless, with difficulty in articulation, C - confused, irregular behavior and MD FROM CIM)
talkative and argumentative. Which of the following uncontrollable, thick speech and
degrees of alcohol intoxication is present in the driver? incoordinated
a. slight inebriation D - disoriented, walking impossible, marked
b. moderate inebriation motor incoordination
c. drunk Legal Medicine Solis p. 696
d. very drunk
e. coma
83 3. Which of the following refers to children conceived Legal Medicine Solis p. 598 ANDREW TIU (TOP 1 FINAL EXAM -
by prostitutes? - AUG 2015 MED MARCH 2016
a. sacrilegous children BOARDS; TOPNOTCH
b. incestuous children MD FROM CIM)
c. manceres
d. adulterous children
e. natural children

84 4. How long shall a woman remarry following the death Legal Medicine Solis p. 592 ANDREW TIU (TOP 1 FINAL EXAM -
of her husband or annulment of their marriage if she is - AUG 2015 MED MARCH 2016
currently pregnant? BOARDS; TOPNOTCH
a. 180 days MD FROM CIM)
b. 240 days
c. 300 days
d. 365 days
e. none of the above
85 5. Mrs. CTM is a prominent Gynecologic Oncologist in ANDREW TIU (TOP 1 FINAL EXAM -
Cebu. She was on her way to a hospital to attend a - AUG 2015 MED MARCH 2016
clinico-pathological conference as she is one of the BOARDS; TOPNOTCH
speakers. While driving along Maxilom Avenue, she MD FROM CIM)
beat the red light and counter flowed the traffic on the
other side. Which of the following is TRUE?
a. she has no right to disobey traffic rules
b. as a doctor, she is given priority and right of way
c. it is an emergency and her drivers license will not be
confiscated
d. she can beat the red light but not counter flow the
traffic on the other side
e. none of the above
86 6. Which of the following is an inherent right of the Inherent rights include: choose patients, ANDREW TIU (TOP 1 FINAL EXAM -
physician? limit practice, determine appropriate - AUG 2015 MED MARCH 2016
A. Getting paid for medical services procedure, avail hospital services BOARDS; TOPNOTCH
B. Perform laparoscopic surgery Basics of Philippine Medical Jurisprudence MD FROM CIM)
C. Being elected as president of PMA cebu chapter and Ethics 2010 p.84
D. Have clinic hours on TThS at Aventus Clinic
E. Beat a red light In traffic when responding to an
emergency
87 7. which of the following occurs among those working heat cramps - involuntary spasmodic painful ANDREW TIU (TOP 1 FINAL EXAM -
in ill - ventilated places with dry and high temperatures contraction of muscles due to dehydration - AUG 2015 MED MARCH 2016
or direct exposure to sunlight? and excessive loss of chlorides by sweating. BOARDS; TOPNOTCH
a. heat cramps also seen in laborers working in rooms with MD FROM CIM)
b. heat exhaustion high temperature and with profused
c. heat stroke perspiration
d. heat collapse heat exhaustion/ collapse/ syncope - due to
e. heat syncope heart failure caused by heat and precipitated
by muscular exertion and warm clothing
Legal Medicine Solis p. 397
88 8. Which of the following is not construed to be a Basics of Philippine Medical Jurisprudence ANDREW TIU (TOP 1 FINAL EXAM -
practice of Medicine? and Ethics 2010 p.67 - AUG 2015 MED MARCH 2016
A. Advertising and offering services by means of cards BOARDS; TOPNOTCH
and letterheads MD FROM CIM)
B. Masseur applying massage
C. Hospital which furnishes accommodations for
patients
D. Mother who recommends to helper a household
remedy
E. Rendering emergency medical care

TOPNOTCH MEDICAL BOARD PREP LEGAL MEDICINE AND JURIS SUPEREXAM Page 11 of 86
For inquiries visit www.topnotchboardprep.com.ph or email us at topnotchmedicalboardprep@gmail.com
TOPNOTCH MEDICAL BOARD PREP LEGAL MEDICINE AND JURIS SUPEREXAM
For inquiries visit www.topnotchboardprep.com.ph or email us at topnotchmedicalboardprep@gmail.com
Item QUESTION EXPLANATION AUTHOR TOPNOTCH
# EXAM
89 9. Which of the following is not true of the physician - Basics of Philippine Medical Jurisprudence ANDREW TIU (TOP 1 FINAL EXAM -
patient relationship? and Ethics 2010 p.91 - AUG 2015 MED MARCH 2016
A. Patient's part of the agreement is liability for BOARDS; TOPNOTCH
payment Of services MD FROM CIM)
B. Based on mutual consent
C. Contract deemed not perfected when there is
agreement between the offer and the acceptance
D. Physician obliged to attend the patient as long as
attention is required
E. Deaf mutes who cannot write cannot give consent
90 10. Which of the following refers to a neighbor uttering Basics of Philippine Medical Jurisprudence ANDREW TIU (TOP 1 FINAL EXAM -
maliciously the bad habits of a physician in a fiesta and Ethics 2010 p.208 - AUG 2015 MED MARCH 2016
gathering in Bohol? BOARDS; TOPNOTCH
A. Defamation MD FROM CIM)
B. Libel
C. Slander
D. Usurpation
E. None of the above
91 11. Which of the following refers to when an A- doing of an act ANDREW TIU (TOP 1 FINAL EXAM -
obstetrician performs cesarean sections on all of her B- performing an act wholly wrongful - AUG 2015 MED MARCH 2016
patients? C- improper performance of some lawful act BOARDS; TOPNOTCH
A. Feasance D- failure to do something which should have MD FROM CIM)
B. Malfeasance been done
C. Misfeasance Basics of Philippine Medical Jurisprudence
D. Nonfeasance and Ethics 2010 p.113
E. Disfeasance
92 12. Which of the following is the most important Basics of Philippine Medical Jurisprudence ANDREW TIU (TOP 1 FINAL EXAM -
element in determining an employee from an and Ethics 2010 p.138 - AUG 2015 MED MARCH 2016
independent contractor? BOARDS; TOPNOTCH
A. Selection and engagement of the employee MD FROM CIM)
B. Payment of wages
C. Power of dismissal
D. Control of conduct
E. NOTA
93 13. Which of the following refers to damages imposed A-by way of example or correction for the ANDREW TIU (TOP 1 FINAL EXAM -
for pecuniary loss but amount cannot be provided with public good - AUG 2015 MED MARCH 2016
certainty? C- agreed upon contract BOARDS; TOPNOTCH
A. Exemplary D-mental anguish etc MD FROM CIM)
B. Temperate E- supported by evidence on record
C. Liquidated Basics of Philippine Medical Jurisprudence
D. Moral and Ethics 2010 p.160
E. Actual
94 14. According to the anti-sexual harassment act of Basics of Philippine Medical Jurisprudence ANDREW TIU (TOP 1 FINAL EXAM -
1995, which of the following has the duty to prevent or and Ethics 2010 p.205 - AUG 2015 MED MARCH 2016
deter the acts of sexual harassment? BOARDS; TOPNOTCH
A. Employees MD FROM CIM)
B. Employers
C. Security guards
D. Administration
E. B and D
F. A and C
95 15. Which of the following does not apply to privilege Basics of Philippine Medical Jurisprudence ANDREW TIU (TOP 1 FINAL EXAM -
communication? and Ethics 2010 p.233 - AUG 2015 MED MARCH 2016
A. Patient's will BOARDS; TOPNOTCH
B. Malpractice cases MD FROM CIM)
C. Illegal purpose
D. Personal injury suit by patient
E. AOTA

96 16. Which of the following is TRUE about right of access Basics of Philippine Medical Jurisprudence ANDREW TIU (TOP 1 FINAL EXAM -
to medical record? and Ethics 2010 p.270 - AUG 2015 MED MARCH 2016
A. Patient can possess the original copy of the record BOARDS; TOPNOTCH
B. Insurance service providers have access to the MD FROM CIM)
records
C. The hospital is not the owner of the record but the
patient
D. Doctor cannot have a photocopy of the record
E. NOTA
97 17. Which of the following is not an element of Basics of Philippine Medical Jurisprudence ANDREW TIU (TOP 1 FINAL EXAM -
informed consent? and Ethics 2010 p.420 - AUG 2015 MED MARCH 2016
A. Comprehension BOARDS; TOPNOTCH
B. Voluntariness MD FROM CIM)
C. Sufficient information
D. Competency
E. NOTA

98 18. Which of the following is Not an element of a felony? Should be revised penal code ANDREW TIU (TOP 1 FINAL EXAM -
A. Must be external acts Basics of Philippine Medical Jurisprudence - AUG 2015 MED MARCH 2016
B. Punishable by civil law and Ethics 2010 p.173 BOARDS; TOPNOTCH
c. Performed By means of culpa or fault MD FROM CIM)
D. Deceit
E. NOTA

TOPNOTCH MEDICAL BOARD PREP LEGAL MEDICINE AND JURIS SUPEREXAM Page 12 of 86
For inquiries visit www.topnotchboardprep.com.ph or email us at topnotchmedicalboardprep@gmail.com
TOPNOTCH MEDICAL BOARD PREP LEGAL MEDICINE AND JURIS SUPEREXAM
For inquiries visit www.topnotchboardprep.com.ph or email us at topnotchmedicalboardprep@gmail.com
Item QUESTION EXPLANATION AUTHOR TOPNOTCH
# EXAM
99 19. Which of the following is not an exempting Basics of Philippine Medical Jurisprudence ANDREW TIU (TOP 1 FINAL EXAM -
circumstance from criminal liability? and Ethics 2010 p.178 - AUG 2015 MED MARCH 2016
A. 13 years old BOARDS; TOPNOTCH
B. Manic patient killing a doctor during a relapse of his MD FROM CIM)
psychosis
C. 17 years old female shooting his father dead because
of her hatred
D. Patient died in the ward due to undiagnosed
ruptured aneurysm
E. NOTA
100 20. Which of the following is res ipsa loquitor not Basics of Philippine Medical Jurisprudence ANDREW TIU (TOP 1 FINAL EXAM -
applicable? and Ethics 2010 p.143 - AUG 2015 MED MARCH 2016
A. Treatment did not produce desired results BOARDS; TOPNOTCH
B. Knocking out a tooth during intubation MD FROM CIM)
C. Leaving an epidural catheter which resulted to
infection
D. Lacerations on scalp of neonate during cesarean
section
E. NOTA
101 During an epidemic or national emergencies the Page 3 of Topnotch Handout. Med students ANGELA PAULINE P. DIAGNOSTIC
following may be given limited and special who have completed the first 4 years of CALIMAG-LOYOLA EXAM - AUG
authorization by the Secretary of Health to practice medicine, graduates of medicine, RNs may be (TOP 8 - FEB 2015 2015
medicine except:
given limited and special authorization by the MED BOARDS;
A. Post Graduate Medical interns secretary of health during epidemics or TOPNOTCH MD
B. Registered nurses national emergencies. FROM UST)
C. Medical students who have completed the first 4
years of medicine
D. Underboard graduates of medicine
E. None of the above

102 A physician-patient relationship is seen in which of the SIMILAR TO PREVIOUS BOARD EXAM ANGELA PAULINE P. DIAGNOSTIC
following circumstances:
CONCEPT/PRINCIPLE. Page 3 of Topnotch CALIMAG-LOYOLA EXAM - AUG
A. Pre-employment PE where the patient was given a Handout. B to E are instances where there is (TOP 8 - FEB 2015 2015
prescription no MD-patient relationship. MED BOARDS;
B. A surgeon performing an autopsy TOPNOTCH MD
C. Relative asking casual questions during a reunion FROM UST)
D. PE for determining eligibilty for an insurance
E. A physician examining a suspect upon appointment
by the court

103 In the classical classification of mental deficiency, an Page 54 of Topnotch Handout. In the classical ANGELA PAULINE P. DIAGNOSTIC
imbecile has an IQ of:
classification Idiot- 0-20, Imbecile 21-40, CALIMAG-LOYOLA EXAM - AUG
A. 0 to 20 Feeble minded 41-70. In the Politically (TOP 8 - FEB 2015 2015
B. 21 to 40 correct classification Profound <20, Severe MED BOARDS;
C. 52 to 67 20-35, Moderate 36-51, Mild 52-67. TOPNOTCH MD
D. 41 to 70 FROM UST)
E. 36 to 51

104 A 23 y/o male suffered several injuries due to a Page 29 of Topnotch Handout. The age of ANGELA PAULINE P. DIAGNOSTIC
vehicular accident, The color of his contusions should contusion can be appreciated from its color CALIMAG-LOYOLA EXAM - AUG
appear _________ after 4-5 days:
changes. Red-purple soon after its complete (TOP 8 - FEB 2015 2015
A. Red development, Green in 4-5 days, yellow in 7- MED BOARDS;
B. Purple 10 days, gradually disappears on the 14-15th TOPNOTCH MD
C. Green day. FROM UST)
D. Yellow
E. Starting to disappear

105 Mr. P who works as a professional tennis player was Page 29 of Topnotch Handout. Serious ANGELA PAULINE P. DIAGNOSTIC
assaulted by one of his opponents after he beat the physical injuries resulting in 1) loss of the CALIMAG-LOYOLA EXAM - AUG
latter in a tournament. Due to the assault he was use of speech or the power to hear or to (TOP 8 - FEB 2015 2015
confined for one week and he became blind in one eye. smell, or loss of an eye, a hand, a foot, an arm MED BOARDS;
The penalty for such injury is: A. or a leg; 2) loss of the use of any such TOPNOTCH MD
Arresto mayor member, 3) becomes incapacitated for the FROM UST)
B. Prison mayor work in which he was therefore habitually
C. Prison correcional in its minimum and medium engaged is punishable by PRISON
period CORRECIONAL IN IT MEDIUM AND
D. Prison correcional in its medium and maximum MAXIMUM PERIODS.
period
E. None of the above
106 Doctor L, was charged by the relatives of a former Page 6 of Topnotch Handout. Criminal ANGELA PAULINE P. DIAGNOSTIC
patient before the Public Prosecutor's office with liability is when the act or omission of MD CALIMAG-LOYOLA EXAM - AUG
Homicide resulting from Gross Negligence and Reckless constitute a crime. May be done with (TOP 8 - FEB 2015 2015
Imprudence, He is most likely being charged with:
deliberate intent or on account of MED BOARDS;
A. Administrative liability imprudence, negligence, lack of foresight or TOPNOTCH MD
B. Civil liability lack of skill. FROM UST)
C. Criminal liability
D. None of the above
E. All of the above

TOPNOTCH MEDICAL BOARD PREP LEGAL MEDICINE AND JURIS SUPEREXAM Page 13 of 86
For inquiries visit www.topnotchboardprep.com.ph or email us at topnotchmedicalboardprep@gmail.com
TOPNOTCH MEDICAL BOARD PREP LEGAL MEDICINE AND JURIS SUPEREXAM
For inquiries visit www.topnotchboardprep.com.ph or email us at topnotchmedicalboardprep@gmail.com
Item QUESTION EXPLANATION AUTHOR TOPNOTCH
# EXAM
107 Burking is classified as what type of asphyxia:
Page 46 of Topnotch Handout. Burking is ANGELA PAULINE P. DIAGNOSTIC
A. By strangulation done when the assailant kneels or sits on the CALIMAG-LOYOLA EXAM - AUG
B. By hanging chest of the victim and with his/her hands (TOP 8 - FEB 2015 2015
C. By suffocation closes the nostrils and mouth of the victim. MED BOARDS;
D. By submersion This is classified under TOPNOTCH MD
E. By compression compression/traumatic/crush asphyxia. FROM UST)

108 Uranism is a form of sexual perversion where a man Page 50 of Topnotch Handout. Uranism is ANGELA PAULINE P. DIAGNOSTIC
gains sexual gratification by:
sexual gratification attained by fingering, CALIMAG-LOYOLA EXAM - AUG
A. rubbing his sex organ against the body parts of fondling with the breast or licking parts of (TOP 8 - FEB 2015 2015
another the body. A is Frottage, C is Mixoscopia, D is MED BOARDS;
B. fingering, fondling with the breast or licking parts Transvestism, E is Bestiality. TOPNOTCH MD
of the body FROM UST)
C. watching couples undress or during sexual activity
D. wearing female apparel
E. engaging in sexual intercourse with animals
109 The following statements are true regarding the Page 38 of Topnotch Handout. The entrance ANGELA PAULINE P. DIAGNOSTIC
characteristics of a gunshot wound entrance except :
woundis usually oval or round depending CALIMAG-LOYOLA EXAM - AUG
A. Smaller in size compared with the bullet upon the angle of approach of the bullet. (TOP 8 - FEB 2015 2015
B. The edges are inverted MED BOARDS;
C. It does not manifest any definite shape TOPNOTCH MD
D. The paraffin test may be positive FROM UST)
E. Tattooing and smudging are present when the gun
is fired within 1-15 cm

110 Once you have passed the Physician licensure exam, Page 4 of Topnotch Handout. A-D are ANGELA PAULINE P. DIAGNOSTIC
the following are considered as your inherent rights as inherent rights of a physician. While the right CALIMAG-LOYOLA EXAM - AUG
a physician except:
to compensation is only an incidental right. (TOP 8 - FEB 2015 2015
A. Avail of hospital services MED BOARDS;
B. Determine the appropriate management TOPNOTCH MD
procedures for your patient FROM UST)
C. Limit your practice within a geographical boundary
D. To choose your patients
E. Right to compensation

111 Doctor X feels that his patient is being uncooperative Page 4 of Topnotch Handout. This is an ANGELA PAULINE P. DIAGNOSTIC
and difficult while his patient feels that Doctor X is example of mutual participation relation CALIMAG-LOYOLA EXAM - AUG
unsympathetic. This type of psychological pattern of wherein the patient thinks that he is (TOP 8 - FEB 2015 2015
MD-patient relationship is an example of:
juridically equal to his physician. The nature MED BOARDS;
A. Guidance-cooperation relation of this relationship is on a negotiated TOPNOTCH MD
B. Activity-Passivity relation agreement. FROM UST)
C. Mutual Participation relation
D. Fiduciary relation
E. Consensual relation

112 Doctor G who is a resident physician in a government SIMILAR TO PREVIOUS BOARD EXAM ANGELA PAULINE P. DIAGNOSTIC
hospital always asks patients in need of a CT scan to go CONCEPT/PRINCIPLE. Page 5 of Topnotch CALIMAG-LOYOLA EXAM - AUG
and have it done in a certain diagnostic clinic outside of Handout. Dichotomous/commission/fee (TOP 8 - FEB 2015 2015
the hospital because he claims that it is cheaper and splitting- sharing of fee with another MED BOARDS;
results are faster. But in truth he is just after the physician, laboratory or drug company, not TOPNOTCH MD
monetary fee being given to him by the owner of the based on services performed. FROM UST)
said diagnostic clinic who is also a physician.This type
of medical fee is called:
A. Straight fee
B. Retainer fee
C. Contractual fee
D. Dichotomous fee
E. Contingent fee

113 This person who comes within the premises of the Page 8 of Topnotch Handout. A licensee is ANGELA PAULINE P. DIAGNOSTIC
hospital, who has no contractual relations with the neither a customer, servant or trespasser. He CALIMAG-LOYOLA EXAM - AUG
hospital and is within the hospital premises for his own has no contratual relation with the hospital. (TOP 8 - FEB 2015 2015
interest or convenience is termed as a:
He is permitted, expressly or impliedly to be MED BOARDS;
A. Business visitor within the hospital premises for his own TOPNOTCH MD
B. Invitee interest or convenience. His presence is FROM UST)
C. Trespasser merely tolerated.
D. Licensee
E. Patient

TOPNOTCH MEDICAL BOARD PREP LEGAL MEDICINE AND JURIS SUPEREXAM Page 14 of 86
For inquiries visit www.topnotchboardprep.com.ph or email us at topnotchmedicalboardprep@gmail.com
TOPNOTCH MEDICAL BOARD PREP LEGAL MEDICINE AND JURIS SUPEREXAM
For inquiries visit www.topnotchboardprep.com.ph or email us at topnotchmedicalboardprep@gmail.com
Item QUESTION EXPLANATION AUTHOR TOPNOTCH
# EXAM
114 Patient A had a hysterectomy due to a uterine tumor, Page 6 of Topnotch Handout. Criminal ANGELA PAULINE P. DIAGNOSTIC
however during the procedure her ureter was liability is when the act or omission of MD CALIMAG-LOYOLA EXAM - AUG
unintentionally cut by Doctor B. She wants to file a case constitute a crime. May be done with (TOP 8 - FEB 2015 2015
for gross negligence against Doctor B. What is the deliberate intent or on account of MED BOARDS;
degree of proof/evidence needed to convict Doctor B of imprudence, negligence, lack of foresight or TOPNOTCH MD
a crime?
lack of skill. Guilt beyond reasonable doubt is FROM UST)
A. Substantial evidence needed for a conviction.
B. Preponderance of evidence
C. Guilt beyond reasonable doubt
D. Any of the above
E. None of the above

115 There are several rules used by the court to determine Page 54 of Topnotch Handout. This is known ANGELA PAULINE P. DIAGNOSTIC
whether an accused is exempt from criminal liability, as the Durham rule. Irresistable impluse rule CALIMAG-LOYOLA EXAM - AUG
One such rule states that the accused is not criminally states that a person is considered insane (TOP 8 - FEB 2015 2015
responsible if his act was the product of a mental when mental disease has rendered him MED BOARDS;
disease or mental defect. This rule is known as:
incapable of restraining himself, although he TOPNOTCH MD
A. Durham rule understands what he is doing and knows it is FROM UST)
B. Irresistable impulse rule wrong. Currens rule states if it is proven that
C. Currens rule at the time of commiting the prohibited act
D. Delusion rule the defendant as a result of mental disease or
E. McNaghten's rule defect lacked substantial capacity to conform
his conduct to the requirementsof the law
which he has allegedly violated. Delusion
rule- a person is not responsible for his act if
he is suffering from delusion although he
knows hat the act is wrong. McNaghten's
rule- A defense on the ground of insanity can
be proven that at the time of committing the
act the accused was laboring under such
defect of reason or from a disease of the mind
as not to know the nature and quality of the
act he was doing and if he did know, he did
not know that what he was doing was wrong.
116 Doctor C, a balikbayan physician can be allowed to Page 3 of Topnotch Handout. The only ANGELA PAULINE P. DIAGNOSTIC
practice medicine in the Philippines provided provisions are A,B,D and E. Membership to CALIMAG-LOYOLA EXAM - AUG
that_____________, except: A. He pays the proper PMA is only an incidental right. Membership (TOP 8 - FEB 2015 2015
professional license fee with the PRC may be vonluntary or involuntary. MED BOARDS;
B. He should register with the PRC Involuntary when membership is imposed as TOPNOTCH MD
C. He is a member of PMA a condition for the enjoyment of a right. FROM UST)
D. He is of good standing prior to departure from the
Philippines and in their adopted country
E. None of the above

117 Mr X a patient with ESRD is scheduled for a kidney SIMILAR TO PREVIOUS BOARD EXAM ANGELA PAULINE P. DIAGNOSTIC
transplant, he will receive a kidey from a living CONCEPT/PRINCIPLE. Page 9 of Topnotch CALIMAG-LOYOLA EXAM - AUG
unrelated human donor. This kind of transplant is Handout. An unrelated human donor to (TOP 8 - FEB 2015 2015
called:
human recipient is called MED BOARDS;
A. Iso transplantation homeotransplantation. TOPNOTCH MD
B. Homeotransplantation FROM UST)
C. Heterotransplantation
D. Autotransplantation
E. None of the above

118 Autoptic or real evidence is defined as evidence which SIMILAR TO PREVIOUS BOARD EXAM ANGELA PAULINE P. DIAGNOSTIC
is:
CONCEPT/PRINCIPLE. Page 13 of Topnotch CALIMAG-LOYOLA EXAM - AUG
A. perceived by the senses Handout. Autoptic or real evidence- (TOP 8 - FEB 2015 2015
B. part of the body of the crime that proves that the perceived by the senses- sight, hearing, taste, MED BOARDS;
crime existed smell and touch. TOPNOTCH MD
C. a testimony given orally by a witness under oath FROM UST)
D. a written record of evidence given orally and then
transcribed
E. an article/material found in connection with the
investigation and assists the investigator in locating the
suspect
119 Mr P went home and caught his daughter who is just 16 Page 25 of Topnotch Handout. This is a case ANGELA PAULINE P. DIAGNOSTIC
y/o commiting sexual intercourse with a 25 y/o man at of physical injuries afflicted under CALIMAG-LOYOLA EXAM - AUG
their house, as a result of his rage he assaulted them execptional circumstances specifically (TOP 8 - FEB 2015 2015
both and inflicted serious physical injuries on both the "surprise of a daughter" The parents with MED BOARDS;
seducer and his daughter. The seducer brought the case respect to their daughter under 18 y/o, and TOPNOTCH MD
to court. Under the law Mr. P may be given the their seducers, while the daughter is living FROM UST)
following punishment: with her parents who shall kill any or both of
A. Prison Mayor them in the act or immediately there after, or
B. Prison correcional in its minimum and medium shall inflict upon them serious physical
period injury, shall suffer the penalty of DESTIERRO.
C. Prison correcional in its medium and maximum As long as the following requisites are met: 1)
period the daughter is below 18; 2) the daughter is
D. Destierro
living with the parents; 3) Parents caught her
E. Since the physical injuries were inflicted due to by surprise commiting sexual intercourse
exceptional circumstances, he shall be exempt from with the seducer; 4) Killing was done at the
punishment very act of sexual intercourse or immediately
thereafter.

TOPNOTCH MEDICAL BOARD PREP LEGAL MEDICINE AND JURIS SUPEREXAM Page 15 of 86
For inquiries visit www.topnotchboardprep.com.ph or email us at topnotchmedicalboardprep@gmail.com
TOPNOTCH MEDICAL BOARD PREP LEGAL MEDICINE AND JURIS SUPEREXAM
For inquiries visit www.topnotchboardprep.com.ph or email us at topnotchmedicalboardprep@gmail.com
Item QUESTION EXPLANATION AUTHOR TOPNOTCH
# EXAM
120 Ms H was allegedly found hanging in her condo unit by Page 44 of Topnotch Handout. A and C are ANGELA PAULINE P. DIAGNOSTIC
her boyfriend. As a medico-legal officer you wanted to post mortem findings in strangulation. B and CALIMAG-LOYOLA EXAM - AUG
determine if this was indeed a suicide by hanging or a D are found in both hanging and (TOP 8 - FEB 2015 2015
homicide by strangulation with ligature. Which of the strangulation. E is foud only in hanging. MED BOARDS;
following findings would show that the cause of death TOPNOTCH MD
was by hanging? FROM UST)
A. The hyoid bone is spared
B. Tardieu spots are found in the subpleural and
subcardial areas
C. The level of the ligature is below the larynx
D. The right side of the heart and the big blood
vessels connected with it are distended with blood
E. The direction of the ligature is an inverted V and
the knot is found at the apex.

121 Which of the following type of law is not enforced by Laws which is not enforced by the state: LYNN DARYL MIDTERM 1
the state? Natural law, Divine law and Moral law. FELICIANO EXAM - AUG
A. Substansive law SIMILAR TO PREVIOUS BOARD EXAM VILLAMATER, MD 2015
B. Remedial law CONCEPT. (TOP 5 - FEB 2015
C. International law MED BOARDS;
D. Commercial law TOPNOTCH MD
E. Moral law FROM EAC)

122 Who among the following is/are qualified to practice Option A. Balikbayan physicians should LYNN DARYL MIDTERM 1
medicine in the Philippines? register with PRC and pay the proper FELICIANO EXAM - AUG
A. Balikbayan Physicians, even without PRC professional license fee, as well as the VILLAMATER, MD 2015
registration as long as they are professional corresponding income tax before they can be (TOP 5 - FEB 2015
practitioners in good standing prior to their departure qualified to practice medicine in the MED BOARDS;
from the Philippines and in their adopted country Philippines. Option C. Foreign physician TOPNOTCH MD
B. Medical students under training and under direct should secure authorization first from the FROM EAC)
supervision of licensed MD Board of Medicine. Option D. Medical
C. Foreign physicians of a medical and surgical students who have completed the first 4
mission, even without securing authorization, given the years of medicine and graduates of medicine
charitable nature of their practice may be given limited and special
D. All medical students who are graduates of medicine authorization by SECRETARY OF HEALTH
during epidemics during epidemics or national emergencies.
E. All of the above

123 Patient-physician relationship is not terminated when? All others are conditions or situations when LYNN DARYL MIDTERM 1
A. Patient recovers from the condition being treated. physician-patient relationship is terminated. FELICIANO EXAM - AUG
B. Patient consented from withdrawal of the physician VILLAMATER, MD 2015
from the contract. (TOP 5 - FEB 2015
C. Physician considers his further service will no MED BOARDS;
longer be beneficial to the patient TOPNOTCH MD
D. Physician abandons the patient FROM EAC)
E. Physician dies

124 Juan just graduated from a medical school and After conferment of the degree Doctor of LYNN DARYL MIDTERM 1
immediately used the title MD after his name. He is: Medicine, one has the legal right to use MD FELICIANO EXAM - AUG
A. Commiting malpractice after his name although a holder of MD VILLAMATER, MD 2015
B. Illegally practicing medicine degree does not qualify a person to (TOP 5 - FEB 2015
C. Academically entitled to used MD legitimately practice medicine. He must first MED BOARDS;
D. Administratively liable pass the board exams and hold a valid TOPNOTCH MD
E. Qualified to legitimately practice medicine certificate of registration. FROM EAC)

125 In which of the following instances where there is Commencement of relationship: Physician LYNN DARYL MIDTERM 1
commencement of a physician-patient relationship? accepts offer by undertaking treatment of the FELICIANO EXAM - AUG
A. In a mall, a patient met her physician and made a patient; invites an offer by establishing VILLAMATER, MD 2015
casual consultation. availability, makes an offer by arriving for (TOP 5 - FEB 2015
B. One night, a patient called a physician by phone for treatment. MED BOARDS;
consultation. The physician told the patient to take TOPNOTCH MD
aspirin that night and see him the following day. FROM EAC)
C. A patient called the phsycian's clinic to make an
appontment. The secretary asked what is wrong and
the patient related her complaints.
D. While passing the emergency room, the physician
noted that the patient is very pale. He then asked the
nurse to inform the resident physician to see and
examine the patient immediately.
E. All of the above.

126 Under the Dangerous Drug Act, marijuana is classified SIMILAR TO PREVIOUS BOARD EXAM LYNN DARYL MIDTERM 1
as: CONCEPT. FELICIANO EXAM - AUG
A. Sedative VILLAMATER, MD 2015
B. Hypnotic (TOP 5 - FEB 2015
C. Prohibited drug MED BOARDS;
D. A stimulant TOPNOTCH MD
E. Hallucinogenic FROM EAC)

TOPNOTCH MEDICAL BOARD PREP LEGAL MEDICINE AND JURIS SUPEREXAM Page 16 of 86
For inquiries visit www.topnotchboardprep.com.ph or email us at topnotchmedicalboardprep@gmail.com
TOPNOTCH MEDICAL BOARD PREP LEGAL MEDICINE AND JURIS SUPEREXAM
For inquiries visit www.topnotchboardprep.com.ph or email us at topnotchmedicalboardprep@gmail.com
Item QUESTION EXPLANATION AUTHOR TOPNOTCH
# EXAM
127 Dr. J, an American surgeon, is in the Philippines on Physicians from other countries can have LYNN DARYL MIDTERM 1
holiday He joined a group of Filipino surgeons in a limited practices without any certificate of FELICIANO EXAM - AUG
medical and surgical mission performing operations for registration provided that that secure VILLAMATER, MD 2015
the poor and the marginalized. Which of the following authorization from the Board of medicine. (TOP 5 - FEB 2015
is correct? SIMILAR TO PREVIOUS BOARD EXAM MED BOARDS;
A. He can invoke reciprocity as the basis for his CONCEPT TOPNOTCH MD
performing surgery in the Philippines. FROM EAC)
B. He is illegally practicing medicine in the country.
C. He can practice even without a certificate of
registration and authorization from the Board of
Medicine.
D. He needs to pass the local board exams before he
can practice.
E. He can practice in the country as long as he is in
good moral and professional standing in his country.
128 Which one of the following is an inherent right of a The answer is A. All other options are LYNN DARYL MIDTERM 1
physician? incidental rights of a physician. Other FELICIANO EXAM - AUG
A. To avail of hospital services. inherent rights of a physician include the ff: VILLAMATER, MD 2015
B. To hold certain public offices choose patients, limit practice, and determine (TOP 5 - FEB 2015
C. To charge professional fees the appropriate management and MED BOARDS;
D. To be exempted from execution of instruments and procedures. TOPNOTCH MD
library FROM EAC)
E. To be a member in medical societies.
129 A 38-year old female had breast surgery for cancer of Doctrine of Assumption of Risk - anyone who LYNN DARYL MIDTERM 1
the breast with metastasis. She had consented in consents to the application of a procedure FELICIANO EXAM - AUG
writing to undergo a trial of a new chemotherapeutic involving certain risks cann recover damages VILLAMATER, MD 2015
modality. However, patient continued to deteriorate for injury resulting from inherent or ordinary (TOP 5 - FEB 2015
and sued the physician. The physician was absolved risks in the procedure. MED BOARDS;
under the doctrine of: TOPNOTCH MD
A. Continuing negligence FROM EAC)
B. Foreseeability
C. Assumption of risk
D. Contributory negligence
E. Common knowledge

130 A general practicioner referred a patient to a specialist. Dichotomous/commission/fee splitting - LYNN DARYL MIDTERM 1
After the treatment, the patient paid the specialist who sharing a fee with another physician, tipsters; FELICIANO EXAM - AUG
in turn gave a share to the general practitioner. What is Contingent-fee depends on success of VILLAMATER, MD 2015
this called? treatment; Straight fee/Pakyaw system (TOP 5 - FEB 2015
A. Contingent fee MED BOARDS;
B. Straight fee TOPNOTCH MD
C. Retainer's fee FROM EAC)
D. Dichotomous Fee
E. Professional fee

131 Dr. A performed an autopsy without proper SIMILAR TO PREVIOUS BOARD EXAM LYNN DARYL MIDTERM 1
authorization from the next of kin or competent CONCEPT. FELICIANO EXAM - AUG
authorities. What is his liability? VILLAMATER, MD 2015
A. Civil liability only (TOP 5 - FEB 2015
B. Criminal liability only MED BOARDS;
C. Both civil and criminal liability TOPNOTCH MD
D. Administrative liability only FROM EAC)
E. Administrative and civil liability

132 A physician issued a medical certificate in favor of a LYNN DARYL MIDTERM 1


friend who was absent from work attesting the latter FELICIANO EXAM - AUG
was sick when in fact he was not. The physician under VILLAMATER, MD 2015
the law is liable: (TOP 5 - FEB 2015
A. Civilly MED BOARDS;
B. Administratively TOPNOTCH MD
C. Morally FROM EAC)
D. Criminally
E. Both administratively and criminally

133 The hospital is almost full and the only vacant beds are SIMILAR TO PREVIOUS BOARD EXAM LYNN DARYL MIDTERM 1
in the ward. An infectious case was admitted in the CONCEPT FELICIANO EXAM - AUG
ward resulting to contamination of other patients. The VILLAMATER, MD 2015
patient who admitted the patient is liable under the (TOP 5 - FEB 2015
Doctrine of: MED BOARDS;
A. Last clear chance TOPNOTCH MD
B. Continuing negligence FROM EAC)
C. Contributory negligence
D. Foreseeability
E. Captain of the ship

TOPNOTCH MEDICAL BOARD PREP LEGAL MEDICINE AND JURIS SUPEREXAM Page 17 of 86
For inquiries visit www.topnotchboardprep.com.ph or email us at topnotchmedicalboardprep@gmail.com
TOPNOTCH MEDICAL BOARD PREP LEGAL MEDICINE AND JURIS SUPEREXAM
For inquiries visit www.topnotchboardprep.com.ph or email us at topnotchmedicalboardprep@gmail.com
Item QUESTION EXPLANATION AUTHOR TOPNOTCH
# EXAM
134 Patient was being managed as a case of Diabetes In mutual participation relation, patient LYNN DARYL MIDTERM 1
mellitus. The physician encouraged lifestyle changes thinks that he is juridically equal to the MD FELICIANO EXAM - AUG
and prescribed him with oral anti-hyperglycemic and that the nature of relationship is on a VILLAMATER, MD 2015
medications and insulin. The patient inquires the negotiated agreement. They are appropriate (TOP 5 - FEB 2015
physician, "are those tablets really necessary? What is in the treatment of chronic illness where MED BOARDS;
the pattern of relationship between the physician and patients are required to carry out much of TOPNOTCH MD
the patient? the treatment program themselves with FROM EAC)
A. Mutual participation occassional medical consultation.
B. Active-passive
C. Guidance-participation
D. Consensual
E. None of the above

135 In a malpractice case, Dr. X was ordered by the court to SIMILAR TO PREVIOUS BOARD EXAM LYNN DARYL MIDTERM 1
produce at trial the patient's charts, laboratory and x- CONCEPT FELICIANO EXAM - AUG
ray results under his possession and control. The VILLAMATER, MD 2015
process served on him to do is called (TOP 5 - FEB 2015
A. Ordinary subpoena MED BOARDS;
B. Subpoena duces tecum TOPNOTCH MD
C. Subpoena ad testificandum FROM EAC)
D. Court summons
E. All of the above

136 A physician is considered as an ordinary witness: A physician can be both an ordinary and LYNN DARYL MIDTERM 1
A. When he is asked to inform the court of the result of expert witness. As an ordinary witness, the FELICIANO EXAM - AUG
the physical examination done to the patient. physician is allowed only to state the cacts VILLAMATER, MD 2015
B. When the chairman of Department of Surgery is which comes to his own perception. (TOP 5 - FEB 2015
asked to give his inference regarding the incident. MED BOARDS;
C. When asked about questions regarding his personal TOPNOTCH MD
qualifications as a witness. FROM EAC)
D. When asked to answer outside of the issue in
consideration of the court.
E. When he is asked to provide inferences regarding
the case.

137 As a result of the deformity he sustained from a facelift Exemplary/Corrective damages - as LYNN DARYL MIDTERM 1
operation, Mary sued Dr. X for mental anguish, social punishment or deterrence because of FELICIANO EXAM - AUG
humiliation, and moral shock. The type of damages she wanton, reckless, malicious, or oppressive VILLAMATER, MD 2015
can recover as a result of these is: nature of wrong committed; Actual damage - (TOP 5 - FEB 2015
A. Exemplary damages adequate compensation for pecuniar; MED BOARDS;
B. Moral damages Nominal damage - invasion of righ or breach TOPNOTCH MD
C. Actual damages of du; Temperate - more than nominal but FROM EAC)
D. Nominal damages less than compensatory when court finds
E. Temperate Damage that some pecuniary loss has been suffered
but its amount cannot be proved with
certainty
138 Maria and Juana had fight. Maria scratched Juana on the SIMILAR TO PREVIOUS BOARD EXAM LYNN DARYL MIDTERM 1
face. The doctor attended the injury that resolved in CONCEPT FELICIANO EXAM - AUG
less than 10 days, however a keloid scar formed. The VILLAMATER, MD 2015
legal classification of this injury is: (TOP 5 - FEB 2015
A. Mutilation MED BOARDS;
B. Slight physical injury TOPNOTCH MD
C. Less serious injury FROM EAC)
D. Serious physical injury
E. Physical injury inflicted in tumutous affray

139 Its main function is the transformation of mechanical SIMILAR TO PREVIOUS BOARD EXAM LYNN DARYL MIDTERM 1
energy by the hit of the firing pin on the percussion cap CONCEPT FELICIANO EXAM - AUG
to chemical energy? VILLAMATER, MD 2015
A. Primer (TOP 5 - FEB 2015
B. Powder MED BOARDS;
C. Bullet TOPNOTCH MD
D. Projectile FROM EAC)
E. Ricochet

140 In which of the following is consent necessary? Consent is not necessary in cases of extreme LYNN DARYL MIDTERM 1
A. Consultation in an out-patient basis emergencies, compulsory procedure by law, FELICIANO EXAM - AUG
B. Compulsory procedure by law, e.g. vaccination and waiver on part of the patient. SIMILAR VILLAMATER, MD 2015
C. In extreme emergencies TO PREVIOUS BOARD EXAM CONCEPT (TOP 5 - FEB 2015
D. Waiver on the part of the patient MED BOARDS;
E. All of the above TOPNOTCH MD
FROM EAC)

TOPNOTCH MEDICAL BOARD PREP LEGAL MEDICINE AND JURIS SUPEREXAM Page 18 of 86
For inquiries visit www.topnotchboardprep.com.ph or email us at topnotchmedicalboardprep@gmail.com
TOPNOTCH MEDICAL BOARD PREP LEGAL MEDICINE AND JURIS SUPEREXAM
For inquiries visit www.topnotchboardprep.com.ph or email us at topnotchmedicalboardprep@gmail.com
Item QUESTION EXPLANATION AUTHOR TOPNOTCH
# EXAM
141 You are assigned at the forensics laboratory, you know Clostridium perfringens is the most EDWARD HARRY MIDTERM 2
that the most commonly implicated organism in decay commonly implicated microbe in decay of VALLAJERA, MD EXAM - AUG
of human remains is human remains. C. welchii is the former (TOP 8 - FEB 2015 2015
A. Clostridium perfringens name of Clostridium perfringens and the MED BOARDS;
B. Clostridium welchii newer name is preferred over the former. TOPNOTCH MD
C. Bacillus anthracis (CHANGED BY DR.BANZUELA: C. welchii and FROM PERPETUAL
D. Clostridium tetani perfringens can both be answers to this BINAN)
E. Staphylococcus aureus question)

142 MSP, a 50 year old male politician charged with plunder It is based on pure imagination with no basis EDWARD HARRY MIDTERM 2
is currently detained at a detention facility, the patient of the fact. Factitious malingering is an VALLAJERA, MD EXAM - AUG
prior to detention is healthy. The patient complains of exagerration of an already present symptom. (TOP 8 - FEB 2015 2015
unbearable nape pain, you performed a thorough MED BOARDS;
history and PE and ordered all the necessary diagnostic TOPNOTCH MD
tests. All of which are normal. What is this patient FROM PERPETUAL
showing? BINAN)
A. Fictitious malingering
B. Factitious malingering
C. Manchausen syndrome
D. None of the above
E. Both A and B

143 Dr. HK was given a subpoena to appear to court but Dr. HK can be cited for indirect contempt for EDWARD HARRY MIDTERM 2
since he's a busy cardiologist, he wasn't able to attend failing to respond to a subpoena VALLAJERA, MD EXAM - AUG
the hearing, the court may cite Dr. HK for: (TOP 8 - FEB 2015 2015
A. Direct contempt MED BOARDS;
B. Disrespect of the court TOPNOTCH MD
C. Indirect contempt FROM PERPETUAL
D. Misprioritizing his obligations BINAN)
E. None of the above

144 The body vested by law to have general supervision and Through the Board of Medicine, the PRC or EDWARD HARRY MIDTERM 2
regulation over the practice of medicine is Professional Regulation Commission is the VALLAJERA, MD EXAM - AUG
A. Commission on Higher Education body vested by law to supervise and regulate (TOP 8 - FEB 2015 2015
B. Professional Regulation Commission the practice of medicine in the Philippines. MED BOARDS;
C. Department of Education TOPNOTCH MD
D. Department of Health FROM PERPETUAL
E. Philippine Medical Association BINAN)

145 You noted a person affixing MD after his name but has He is academically entitle to affix MD after his EDWARD HARRY MIDTERM 2
not yet taken the boards, you know that affixing MD name if he has graduated from medical VALLAJERA, MD EXAM - AUG
after his name is: school (TOP 8 - FEB 2015 2015
A. Illegally practicing medicine MED BOARDS;
B. Falsely using the title MD TOPNOTCH MD
C. Academically entitled to affix MD FROM PERPETUAL
D. Committing fraud BINAN)
E. None of the above

146 Dr. X has written an offensive article about his patient, Slander is oral defamation while libel is EDWARD HARRY MIDTERM 2
patient Y decided to charge Dr. X, what would patient Y written defamation of another person VALLAJERA, MD EXAM - AUG
charge Dr. X with? (TOP 8 - FEB 2015 2015
A. Slander MED BOARDS;
B. Libel TOPNOTCH MD
C. Malicious mischief FROM PERPETUAL
D. Acts of lasciviousness BINAN)
E. None of the above

147 A physician stands on trial for a civil case, the burden of A preponderance of evidence is needed to EDWARD HARRY MIDTERM 2
proof needed by the prosecution to convict the convict one in a civil case. VALLAJERA, MD EXAM - AUG
physician is: (TOP 8 - FEB 2015 2015
A. Guilt beyond reasonable doubt MED BOARDS;
B. Substantial evidence TOPNOTCH MD
C. Circumstantial evidence FROM PERPETUAL
D. Preponderance of evidence BINAN)
E. None of the above

148 Dr. A lost in a civil case and was ordered by the court to EDWARD HARRY MIDTERM 2
pay in compensation for the pecuniary loss suffered by VALLAJERA, MD EXAM - AUG
his patient, this is: (TOP 8 - FEB 2015 2015
A. Actual damages MED BOARDS;
B. Moral damages TOPNOTCH MD
C. Liquidated damages FROM PERPETUAL
D. Exemplary damages BINAN)
E. None of the above

TOPNOTCH MEDICAL BOARD PREP LEGAL MEDICINE AND JURIS SUPEREXAM Page 19 of 86
For inquiries visit www.topnotchboardprep.com.ph or email us at topnotchmedicalboardprep@gmail.com
TOPNOTCH MEDICAL BOARD PREP LEGAL MEDICINE AND JURIS SUPEREXAM
For inquiries visit www.topnotchboardprep.com.ph or email us at topnotchmedicalboardprep@gmail.com
Item QUESTION EXPLANATION AUTHOR TOPNOTCH
# EXAM
149 The following are signs of death except EDWARD HARRY MIDTERM 2
A. Cessation of respiration VALLAJERA, MD EXAM - AUG
B. Algor mortis (TOP 8 - FEB 2015 2015
C. Cessation of circulation MED BOARDS;
D. Loss of consciousness TOPNOTCH MD
E. All of the above FROM PERPETUAL
BINAN)

150 A 13 year old girl was raped by her teacher, the teacher The teacher is liable for rape as the crime EDWARD HARRY MIDTERM 2
is liable for was actually consumated. VALLAJERA, MD EXAM - AUG
A. Simple seduction (TOP 8 - FEB 2015 2015
B. Consented abduction MED BOARDS;
C. Qualified seduction TOPNOTCH MD
D. Acts of lasciviousness FROM PERPETUAL
E. None of the above BINAN)

151 This test determines whether a newborn child took a Wredin is for middle ear, Fodere is testing EDWARD HARRY MIDTERM 2
breath before he died by dipping the stomach in water the lungs, Florence test is testing for the VALLAJERA, MD EXAM - AUG
and if the stomach floats, then breathing took place presence of semen, Katayama test is testing (TOP 8 - FEB 2015 2015
A. Breslau's test for carboxyhemoglobin in blood MED BOARDS;
B. Wredin's test TOPNOTCH MD
C. Fodere's test FROM PERPETUAL
D. Florence test BINAN)
E. Katayama test

152 JE was hit in the face by her friend causing a deformity JE can be charged with serious physical EDWARD HARRY MIDTERM 2
on her friend's face, her friend can charge JE with injury as it resulted in a disfiguration of her VALLAJERA, MD EXAM - AUG
A. Slight physical injury friend's face (TOP 8 - FEB 2015 2015
B. Less serious physical injury MED BOARDS;
C. Serious physical injury TOPNOTCH MD
D. Mutilation FROM PERPETUAL
E. None of the above BINAN)

153 A test used to determine the presence of powder Lung's or paraffin test determines the EDWARD HARRY MIDTERM 2
residues on the suspect's hands or the site of gunshot presence of nitrates on the site to be tested VALLAJERA, MD EXAM - AUG
wound entrance but can be falsely positve if the subject (TOP 8 - FEB 2015 2015
A. Lung's test handled fertilizers, cosmetics or cigarettes or MED BOARDS;
B. Fodere's test other nitrogen containing compounds TOPNOTCH MD
C. Breslau's test FROM PERPETUAL
D. Walker's test BINAN)
E. Harrison and Gilroy test

154 For a child to be considered legitimate, the child must All are correct and are definitions fulfilled for EDWARD HARRY MIDTERM 2
be born: a child to be considered legitimate VALLAJERA, MD EXAM - AUG
A. in 180 days following the celebration of marriage (TOP 8 - FEB 2015 2015
B. within 300 days from the dissolution of marriage MED BOARDS;
C. in the preceding 300 days before the child's birth, TOPNOTCH MD
there is no physical impossibility of the husband having FROM PERPETUAL
contact with the wife within the first 120 days BINAN)
D. Both A and B
E. All of the above

155 The following are inherent rights of physician's except: EDWARD HARRY MIDTERM 2
A. To choose patients VALLAJERA, MD EXAM - AUG
B. Hold public or private offices (TOP 8 - FEB 2015 2015
C. Avail of hospital services MED BOARDS;
D. Limit the practice TOPNOTCH MD
E. Determine appropriate management procedures FROM PERPETUAL
BINAN)

156 A kind of compensation wherein the physician is paid Simple contractual stipulates the nature of EDWARD HARRY MIDTERM 2
by space of time and not by the quality or quantity of the procedure, contingent fee depends on the VALLAJERA, MD EXAM - AUG
the service rendered success or failure of the treatment, (TOP 8 - FEB 2015 2015
A. Contingent fee dichotomous fee or fee splitting with other MED BOARDS;
B. Simple contractual fee physicians, laboratory or drug company, TOPNOTCH MD
C. Retainer fee straight fee is like the package system. FROM PERPETUAL
D. Dichotomous fee BINAN)
E. Straight fee

157 A radiologist was sued for wrongly interpreting a chest Because the hospital chose them as their EDWARD HARRY MIDTERM 2
x-ray finding costing the patient her right lung, the radiologist, the hospital is also liable for the VALLAJERA, MD EXAM - AUG
hospital is also reliable for the actions of the radiologist action of the radiologist. (TOP 8 - FEB 2015 2015
through this doctrine MED BOARDS;
A. Doctrine of ostensible agent TOPNOTCH MD
B. Borrowed servant doctrine FROM PERPETUAL
C. Captain of the ship doctrine BINAN)
D. Res Ipsa Loquitor
E. None of the above

TOPNOTCH MEDICAL BOARD PREP LEGAL MEDICINE AND JURIS SUPEREXAM Page 20 of 86
For inquiries visit www.topnotchboardprep.com.ph or email us at topnotchmedicalboardprep@gmail.com
TOPNOTCH MEDICAL BOARD PREP LEGAL MEDICINE AND JURIS SUPEREXAM
For inquiries visit www.topnotchboardprep.com.ph or email us at topnotchmedicalboardprep@gmail.com
Item QUESTION EXPLANATION AUTHOR TOPNOTCH
# EXAM
158 A resident was assisting a surgeon in an ex lap when the The surgeon is liable under the doctrine of EDWARD HARRY MIDTERM 2
resident accidentally injured the left kidney, the the captain of the ship as well as the resident VALLAJERA, MD EXAM - AUG
surgeon as well as the resident is liable under this (TOP 8 - FEB 2015 2015
doctrine MED BOARDS;
A. Borrowed servant doctrine TOPNOTCH MD
B. Doctrine of ostensible agent FROM PERPETUAL
C. Res ipsa loquitur BINAN)
D. Doctrine of assumption of risk
E. None of the above

159 A physician may not be issued a certificate of The physician is still under trial and EDWARD HARRY MIDTERM 2
registration if he/she has commited the following therefore is presumed innocent until proven VALLAJERA, MD EXAM - AUG
except: guilty (TOP 8 - FEB 2015 2015
A. was undergoing trial for a criminal offense involving MED BOARDS;
moral turpitude TOPNOTCH MD
B. was declared to be of unsound mind FROM PERPETUAL
C. was found guilty of immoral or dishonorable conduct BINAN)
after investigation of the Board of Medicine
D. was convicted by any court for a criminal offense
E. None of the above

160 A physician can apply for reinstatement of his/her A physician can apply for reinstatement upon EDWARD HARRY MIDTERM 2
license if it has been revoked for how long? recommendation of the board after 2 years VALLAJERA, MD EXAM - AUG
A. 1 year for reasons of equity and justice. (TOP 8 - FEB 2015 2015
B. 2 years MED BOARDS;
C. 3 years TOPNOTCH MD
D. 4 years FROM PERPETUAL
E. 5 years BINAN)

161 The microorganism that plays a dominant role in SIMILAR TO PREVIOUS BOARD EXAM HAROLD JAY S. MIDTERM 3
decomposition CONCEPT/PRINCIPLE BAYTEC, MD (TOP 10 EXAM - AUG
A. Bacillus mesenterious - FEB 2015 MED 2015
B. Bacillus coli BOARDS; TOPNOTCH
C. Clostridium welchi MD FROM FEU)
D. Bacilus proteus vulgaris
E. Bacillus clausi

162 This finding in death is not consistent and may or may HAROLD JAY S. MIDTERM 3
not appear on a person who died BAYTEC, MD (TOP 10 EXAM - AUG
A. Rigor mortis - FEB 2015 MED 2015
B. Cadaveric spasm BOARDS; TOPNOTCH
C. Algor mortis MD FROM FEU)
D. putrefaction
E. None of the above

163 Which of the following findings suggests suicide HAROLD JAY S. MIDTERM 3
happened to the victim? BAYTEC, MD (TOP 10 EXAM - AUG
A. Indication of struggle - FEB 2015 MED 2015
B. Presence of hesitation cuts BOARDS; TOPNOTCH
C. Multiple wounds in different parts of the body MD FROM FEU)
D. NO history of depression
E. None of the above

164 What is the source when blood is dark red in color and HAROLD JAY S. MIDTERM 3
has low oxygen content in a crime scene? BAYTEC, MD (TOP 10 EXAM - AUG
A. child - FEB 2015 MED 2015
B. menstrual BOARDS; TOPNOTCH
C. elderly MD FROM FEU)
D. venous
E. Arterial

165 Algor mortis refers to: HAROLD JAY S. MIDTERM 3


A. irritability BAYTEC, MD (TOP 10 EXAM - AUG
B. Softening of the body - FEB 2015 MED 2015
C. Hardening of the body BOARDS; TOPNOTCH
D. Loss of elasticity MD FROM FEU)
E. Cooling of the body

166 The basis to pronounce a person dead is: HAROLD JAY S. MIDTERM 3
A. Molecular death BAYTEC, MD (TOP 10 EXAM - AUG
B. Apparent death - FEB 2015 MED 2015
C. Cellular death BOARDS; TOPNOTCH
D. Somatic death MD FROM FEU)
E. None of the above

TOPNOTCH MEDICAL BOARD PREP LEGAL MEDICINE AND JURIS SUPEREXAM Page 21 of 86
For inquiries visit www.topnotchboardprep.com.ph or email us at topnotchmedicalboardprep@gmail.com
TOPNOTCH MEDICAL BOARD PREP LEGAL MEDICINE AND JURIS SUPEREXAM
For inquiries visit www.topnotchboardprep.com.ph or email us at topnotchmedicalboardprep@gmail.com
Item QUESTION EXPLANATION AUTHOR TOPNOTCH
# EXAM
167 In determining the sex of the skeleton, the following HAROLD JAY S. MIDTERM 3
bones must be studied EXCEPT: BAYTEC, MD (TOP 10 EXAM - AUG
A. mandible - FEB 2015 MED 2015
B. sternum BOARDS; TOPNOTCH
C. pelvis MD FROM FEU)
D. femur
E. None of the above

168 Which among the following is an incidental right of a others are inherennt rights of a physician HAROLD JAY S. MIDTERM 3
physician? BAYTEC, MD (TOP 10 EXAM - AUG
A. Right to choose patient - FEB 2015 MED 2015
B. Right to limit practice in a private clinic or hospital BOARDS; TOPNOTCH
C. Right to hold certain public or private offices MD FROM FEU)
D. Right to determine the appropriate management or
procedures
E. right to avail hospital services

169 Which of the following Medical Fees is considered simple contractual fee and retainer fee are HAROLD JAY S. MIDTERM 3
ethical? the only medical fees considered ethical. BAYTEC, MD (TOP 10 EXAM - AUG
A. Simple contractual fee contingent fee is unethical. the entry on the - FEB 2015 MED 2015
B. Contingent fee handout was corrected by Dr Rebosa during BOARDS; TOPNOTCH
C. Dichotomous fee his lecture. simple contractual and retainer MD FROM FEU)
D. Fee splitting are the only ethical fees. SIMILAR TO
E. Straight fee PREVIOUS BOARD EXAM
CONCEPT/PRINCIPLE
170 A newly licensed doctor was hired by a HMO company HAROLD JAY S. MIDTERM 3
and is paid 180/hour regardless of how many patients BAYTEC, MD (TOP 10 EXAM - AUG
he can handle at that time. This fee is called: - FEB 2015 MED 2015
A. Simple contractual BOARDS; TOPNOTCH
B. Retainer MD FROM FEU)
C. contingent
D. dichotomous
E. Straight

171 A radiologist performed a radiation therapy however SIMILAR TO PREVIOUS BOARD EXAM HAROLD JAY S. MIDTERM 3
the patient suffered severe burns on the skin. The CONCEPT/PRINCIPLE BAYTEC, MD (TOP 10 EXAM - AUG
hospital will also be liable under this doctrine - FEB 2015 MED 2015
A. Borrowed servant BOARDS; TOPNOTCH
B. Captain of the ship MD FROM FEU)
C. Ostensible agent
D. Assumption of risk
E. none of the above

172 For an applicant to be qualified to take the Physician This is actually the #521 in compilation of HAROLD JAY S. MIDTERM 3
Licensure Examination of the Philippines, the applicant legal med question. Dr Rebosa's answer is BAYTEC, MD (TOP 10 EXAM - AUG
must meet the following requirements EXCEPT: citizen of the Philippines since a citizen of - FEB 2015 MED 2015
A. Shall be of sound mind another country can also apply provided that BOARDS; TOPNOTCH
B. At least 21 years old his/her country has reciprocity with ours. MD FROM FEU)
C. Citizen of the Philippines With regard to B, it can also be accepted since
D. Holder of MD degree applicants who are 19-21 y/o can already
E. none of the above take the preliminary examination provided
they satisfy the other requirements. (page 2
legal med topnotch handout)
173 Ina, a clerk, was tasked to be with the patient at the OR HAROLD JAY S. MIDTERM 3
while waiting for surgery. She was confident that BAYTEC, MD (TOP 10 EXAM - AUG
nothing wrong will happen to the patient and left the - FEB 2015 MED 2015
patient at the OR alone to have a coffee. After several BOARDS; TOPNOTCH
minutes, patient fell asleep and fell from the OR table MD FROM FEU)
and he suffered contusions and lacerations. Dr. Diaz
who is the surgeon can also be charged for negligence
under this doctrine.
A. Ostensible agent
B. Captain of the Ship
C. Barrowed Servant
D. calculated risk
E. surgeon can not be charge of negligence because
the act of the clerk is grossly slacking.

174 What is the liability of a physician who performed an SIMILAR TO PREVIOUS BOARD EXAM HAROLD JAY S. MIDTERM 3
autopsy without proper authorization from the next of CONCEPT/PRINCIPLE x5 BAYTEC, MD (TOP 10 EXAM - AUG
kin or competent authorities - FEB 2015 MED 2015
A. Civil liability BOARDS; TOPNOTCH
B. Criminal liability MD FROM FEU)
C. Administrative liability
D. A and C
E. All of the above

TOPNOTCH MEDICAL BOARD PREP LEGAL MEDICINE AND JURIS SUPEREXAM Page 22 of 86
For inquiries visit www.topnotchboardprep.com.ph or email us at topnotchmedicalboardprep@gmail.com
TOPNOTCH MEDICAL BOARD PREP LEGAL MEDICINE AND JURIS SUPEREXAM
For inquiries visit www.topnotchboardprep.com.ph or email us at topnotchmedicalboardprep@gmail.com
Item QUESTION EXPLANATION AUTHOR TOPNOTCH
# EXAM
175 A patient wants to file a complaint against a physician HAROLD JAY S. MIDTERM 3
for immorality. Where should he file his complaint? BAYTEC, MD (TOP 10 EXAM - AUG
A. Regional Trial Court - FEB 2015 MED 2015
B. Prosecutors office BOARDS; TOPNOTCH
C. DOH MD FROM FEU)
D. PMA
E. PRC

176 While driving in your town, you witnessed a shooting HAROLD JAY S. MIDTERM 3
incident by a group of armed men. You were BAYTEC, MD (TOP 10 EXAM - AUG
subpoenaed to appear in court as a/an _____________ - FEB 2015 MED 2015
witness. BOARDS; TOPNOTCH
A. expert MD FROM FEU)
B. hostile
C. reluctant
D. ordinary
E. Professional

177 In a botched up surgery, the chairman of the surgery HAROLD JAY S. MIDTERM 3
department of a medical center was called in to testify BAYTEC, MD (TOP 10 EXAM - AUG
on the procedure. He is giving testimony as a _________ - FEB 2015 MED 2015
witness. BOARDS; TOPNOTCH
A. precipient MD FROM FEU)
B. expert
C. hostile
D. defense
E. Ordinary

178 Under the doctrine the hospital owner may be liable for SIMILAR TO PREVIOUS BOARD EXAM HAROLD JAY S. MIDTERM 3
the mistakes of the residents: CONCEPT/PRINCIPLE BAYTEC, MD (TOP 10 EXAM - AUG
A. Barrowed servant - FEB 2015 MED 2015
B. Captain of the ship BOARDS; TOPNOTCH
C. Vicariuos liability MD FROM FEU)
D. Fellow servant
E. Assumption of risk

179 The license of Dr HK was suspended indefinitely. After SIMILAR TO PREVIOUS BOARD EXAM HAROLD JAY S. MIDTERM 3
how long can he apply for reinstatement? CONCEPT/PRINCIPLE BAYTEC, MD (TOP 10 EXAM - AUG
A. 3 years - FEB 2015 MED 2015
B. 6 months BOARDS; TOPNOTCH
C. 1 year MD FROM FEU)
D. 2 years
E. 5 years

180 A patient of a rare disease takes an experimental drug SIMILAR TO PREVIOUS BOARD EXAM HAROLD JAY S. MIDTERM 3
despite unknown side effects: CONCEPT/PRINCIPLE BAYTEC, MD (TOP 10 EXAM - AUG
A. Doctrine of informed consent - FEB 2015 MED 2015
B. Doctrine of foreseeability BOARDS; TOPNOTCH
C. Assumption of risk MD FROM FEU)
D. Contributory negligence
E. None of the above

181 A physician who deliberately refuses to obey a SIMILAR TO PREVIOUS BOARD EXAM JEAN PAOLO M. FINAL EXAM -
subpoena issued by a court without any justifiable CONCEPT/PRINCIPLE.. penalty for indirect DELFINO, MD (TOP AUG 2015
reason is guilty of indirect contempt and can have a contempt: 30000 pesos or 6 months 10 - FEB 2015 MED
penalty of imprisonment. Penalty for direct contempt: BOARDS; TOPNOTCH
A. 200 pesos 200 pesos or 1-2 days imprisonment MD FROM FATIMA)
B. 1-2 days imprisonment
C. 6 months imprisonment
D. 1 month imprisonment
E. 10,000 pesos

182 What organ is affected by crush syndrome? Crush syndrome is characterized by JEAN PAOLO M. FINAL EXAM -
A. kidney secondary kidney changes in crush injuries. DELFINO, MD (TOP AUG 2015
B. spleen Edema and anuria follow a crush. 10 - FEB 2015 MED
C. liver BOARDS; TOPNOTCH
D. pancreas MD FROM FATIMA)
E. stomach

183 In this psychological pattern of doctor-patient Mutual participation relation is when the JEAN PAOLO M. FINAL EXAM -
relationship, the doctor feels that the patient is patient thinks that he is juridically equal to DELFINO, MD (TOP AUG 2015
uncooperative and difficult while the patient feels that the doctor and that the nature of the 10 - FEB 2015 MED
the doctor is unsympathetic relationship is on a negotiated agreement. BOARDS; TOPNOTCH
A. Activity-Passivity MD FROM FATIMA)
B. Guidance-cooperation relation
C. Mutual participation relation
D. All of the above
E. None of the above

TOPNOTCH MEDICAL BOARD PREP LEGAL MEDICINE AND JURIS SUPEREXAM Page 23 of 86
For inquiries visit www.topnotchboardprep.com.ph or email us at topnotchmedicalboardprep@gmail.com
TOPNOTCH MEDICAL BOARD PREP LEGAL MEDICINE AND JURIS SUPEREXAM
For inquiries visit www.topnotchboardprep.com.ph or email us at topnotchmedicalboardprep@gmail.com
Item QUESTION EXPLANATION AUTHOR TOPNOTCH
# EXAM
184 Mother does not intend to hurt her child. When she is JEAN PAOLO M. FINAL EXAM -
drunk, she periodically batters son. She becomes DELFINO, MD (TOP AUG 2015
remorseful afterwards and provides proper care in 10 - FEB 2015 MED
between her drinking spree. What type of child abuser BOARDS; TOPNOTCH
is the mother? MD FROM FATIMA)
A. Occasional child abuser
B. Constant child abuser
C. Intermittent child abuser
D. One-time child abuser
E. None of the above

185 Disorientation, mental confusion, dizziness, sensory 100mg%- loss of critical judgment, 150- JEAN PAOLO M. FINAL EXAM -
disturbances exaggerated emotional state suggests that 300mg%- Disorientation, mental confusion, DELFINO, MD (TOP AUG 2015
the alcohol blood level is? dizziness, sensory disturbances, 400mg%- 10 - FEB 2015 MED
A. 200mg% anesthetic, unconscious BOARDS; TOPNOTCH
B. 400mg% MD FROM FATIMA)
C. 100mg%
D. 20mg%
E. 50mg%

186 Confidentiality of information in the course of a doctor- SIMILAR TO PREVIOUS BOARD EXAM JEAN PAOLO M. FINAL EXAM -
patient relationship may be breached in the following CONCEPT/PRINCIPLE.. Confidentiality of DELFINO, MD (TOP AUG 2015
instances? information in the course of a doctor-patient 10 - FEB 2015 MED
A. When disclosure is necessary to serve the best relationship may be breached when BOARDS; TOPNOTCH
interest of justice disclosure is necessary to serve the best MD FROM FATIMA)
B. When disclosure of information will serve both interest of justice
public and private health and safety
C. When disclosure is needed in a civil case to
determine the moral character of a patient
D. When information are being asked by police
authorities
E. All of the above

187 A doctor's wife filed an administrative complaint before Immoral or dishonorable conduct is a ground JEAN PAOLO M. FINAL EXAM -
the PRC against her husband who was living with a for personal disqualification that may cause DELFINO, MD (TOP AUG 2015
mistress for having abandoned her and her 3 children. the revocation os a physician's certificate of 10 - FEB 2015 MED
The doctor's immoral acts are grounds for revoking his registration BOARDS; TOPNOTCH
registration certificate and classified as? MD FROM FATIMA)
A. Criminal act
B. Unprofessional conduct
C. Unethical conduct
D. Personal disqualification
E. Civil liability

188 It concerns the state in its sovereign or political Public laws are concerned with the state in JEAN PAOLO M. FINAL EXAM -
capacity to enforce or regulate the exercise of a right, its political and sovereign capacity. Examples DELFINO, MD (TOP AUG 2015
duty or obligation imposed on the subject of public law are criminal law, international 10 - FEB 2015 MED
A. law on obligation and contracts law and political law BOARDS; TOPNOTCH
B. criminal law MD FROM FATIMA)
C. law on torts and damages
D. private law
E. commercial law

189 Marijuana is classified as what type of drug in the SIMILAR TO PREVIOUS BOARD EXAM JEAN PAOLO M. FINAL EXAM -
Dangeous Drug Act? CONCEPT/PRINCIPLE.. Hallucinogens/ DELFINO, MD (TOP AUG 2015
A. Sedative psychomimetic drugs: marijuana, LSD, MDA, 10 - FEB 2015 MED
B. Hypnotics MDMA, Phencyclidine, etc. are examples of BOARDS; TOPNOTCH
C. Prohibited drug prohibited drugs MD FROM FATIMA)
D. Stimulant
E. Dangerous drug

190 Man was found guilty of large-scale estafa and was to be JEAN PAOLO M. FINAL EXAM -
imprisoned. He suddenly became insane. Which should DELFINO, MD (TOP AUG 2015
take place? 10 - FEB 2015 MED
A. He should be sent to prison BOARDS; TOPNOTCH
B. His imprisonment should be deferred until he MD FROM FATIMA)
recovers
C. He should undergo another trial upon recovery of
sanity
D. He should be pardoned
E. None of the above
191 When the color of the contusion changes to green, the 2-4 days- red/purple, 4-5 days- green, 7-10 JEAN PAOLO M. FINAL EXAM -
contusion's estimated age is? days- yellow, 14-15 days- disappears DELFINO, MD (TOP AUG 2015
A. 2-4 days 10 - FEB 2015 MED
B. 4-5 days BOARDS; TOPNOTCH
C. 14-15 days MD FROM FATIMA)
D. 7-10 days
E. 24 hours

TOPNOTCH MEDICAL BOARD PREP LEGAL MEDICINE AND JURIS SUPEREXAM Page 24 of 86
For inquiries visit www.topnotchboardprep.com.ph or email us at topnotchmedicalboardprep@gmail.com
TOPNOTCH MEDICAL BOARD PREP LEGAL MEDICINE AND JURIS SUPEREXAM
For inquiries visit www.topnotchboardprep.com.ph or email us at topnotchmedicalboardprep@gmail.com
Item QUESTION EXPLANATION AUTHOR TOPNOTCH
# EXAM
192 The art of identification of fingerprints by comparison JEAN PAOLO M. FINAL EXAM -
is known as DELFINO, MD (TOP AUG 2015
A. endoscopy 10 - FEB 2015 MED
B. dactyloscopy BOARDS; TOPNOTCH
C. poroscopy MD FROM FATIMA)
D. graphology
E. dactylography

193 Fingerprint may persist if not removed at the scene for JEAN PAOLO M. FINAL EXAM -
how long? DELFINO, MD (TOP AUG 2015
A. days 10 - FEB 2015 MED
B. weeks BOARDS; TOPNOTCH
C. months MD FROM FATIMA)
D. years
E. hours

194 How soon after drowning does the dead body float? JEAN PAOLO M. FINAL EXAM -
A. Within 24 hours DELFINO, MD (TOP AUG 2015
B. After 36 hours 10 - FEB 2015 MED
C. After 48 hours BOARDS; TOPNOTCH
D. After 60 hours MD FROM FATIMA)
E. After 72 hours

195 The body vested by law to have general supervision and Professional Regulatory Commission is the JEAN PAOLO M. FINAL EXAM -
regulation over the practice of medicine is? body vested by law to have general DELFINO, MD (TOP AUG 2015
A. Commission on Higher Education supervision and regulation over the practice 10 - FEB 2015 MED
B. Department of Education of medicine BOARDS; TOPNOTCH
C. Professional Regulatory Commission MD FROM FATIMA)
D. Board of Medical Education
E. Association of Philippine Medical Colleges

196 Which of the following kinds of medical fees is In straight fee (aka Pakyaw system), the JEAN PAOLO M. FINAL EXAM -
unethical? amount of medical fee is dependent on what DELFINO, MD (TOP AUG 2015
A. Simple contractual fee will be the remaining balance when all of the 10 - FEB 2015 MED
B. Retainer fee other expenses have been paid. This is BOARDS; TOPNOTCH
C. Contingent fee unethical. Another unethical fee is the MD FROM FATIMA)
D. Straight fee dichotomous fee or fee-
E. All of the above splitting.DR.BANZUELA: According to
Dr.Rebosa, contingent fee is also unethical.
Correct answer for 96 is E
197 What is the kind of transplantation wherein the donor SIMILAR TO PREVIOUS BOARD EXAM JEAN PAOLO M. FINAL EXAM -
and recipient are of the same species but are unrelated? CONCEPT/PRINCIPLE.. transplantation from DELFINO, MD (TOP AUG 2015
A. Heterotransplantation a human donor to an unrelated human 10 - FEB 2015 MED
B. Homeotransplantation recipient is known as homeotransplantation BOARDS; TOPNOTCH
C. Isotransplantation MD FROM FATIMA)
D. Autotransplantation
E. Xenotransplantation

198 The height can be approximated using the following Height can be estimated using the following JEAN PAOLO M. FINAL EXAM -
bones except? bone remains of a dead person: mnemonics DELFINO, MD (TOP AUG 2015
A. Femur "FR HT" (for height) Femur, Radius, 10 - FEB 2015 MED
B. Radius Humerus, Tibia BOARDS; TOPNOTCH
C. Humerus MD FROM FATIMA)
D. Tibia
E. Ulna

199 An applicant in a shipping company sued the physician Instances when there is no doctor-paitent JEAN PAOLO M. FINAL EXAM -
of a pre-employment company for negligence due to the relationship- page 3 topnotch legal med DELFINO, MD (TOP AUG 2015
latter's refusal to prescribe an antidiabetic drug for the handouts 10 - FEB 2015 MED
"increased blood sugar" in his FBS. Which among the BOARDS; TOPNOTCH
following is a valid defense to be invoked by the MD FROM FATIMA)
physician?
A. right to choose patients
B. right to limit practice
C. absence of physician-patient relationship
D. absence of consideration
E. all of the above

200 What is the liability of a physician who performed an SIMILAR TO PREVIOUS BOARD EXAM JEAN PAOLO M. FINAL EXAM -
autopsy without proper authorization from the next of CONCEPT/PRINCIPLE.. the liability of a DELFINO, MD (TOP AUG 2015
kin or competent authorities? physician who performed an autopsy without 10 - FEB 2015 MED
A. Civil liability only proper authorization from the next of kin or BOARDS; TOPNOTCH
B. Criminal liability only competent authorities is both civil and MD FROM FATIMA)
C. Both civil and criminal liability criminal according to Dr. Rebosa =) Ideally, it
D. Administrative liability should be all of the above.However, there's
E. None of the above no 'all of the above' in the choices so the best
answer would still be letter C.

TOPNOTCH MEDICAL BOARD PREP LEGAL MEDICINE AND JURIS SUPEREXAM Page 25 of 86
For inquiries visit www.topnotchboardprep.com.ph or email us at topnotchmedicalboardprep@gmail.com
TOPNOTCH MEDICAL BOARD PREP LEGAL MEDICINE AND JURIS SUPEREXAM
For inquiries visit www.topnotchboardprep.com.ph or email us at topnotchmedicalboardprep@gmail.com
Item QUESTION EXPLANATION AUTHOR TOPNOTCH
# EXAM
201 Criminal liability of physicians is based on: Administrative liability: Medical Act of 1959. GRACE ARVIOLA, MD DIAGNOSTIC
A. Medical Act of 1959 Civil liability: Civil code. (TOP 3 - AUG 2014 EXAM - FEB
B. Revised Penal Code MED BOARDS; 2015
C. Civil Code TOPNOTCH MD)
D. Hospital Rules and Regulations
E. State Rules

202 Which doctrine can potentially nullify the doctrine of GRACE ARVIOLA, MD DIAGNOSTIC
Res Ipsa Loquitur? (TOP 3 - AUG 2014 EXAM - FEB
A. Doctrine of Continuing Negligence MED BOARDS; 2015
B. Doctrine of Ostensible Agents TOPNOTCH MD)
C. Doctrine of Contributory Negligence
D. Doctrine of Calculated Risk
E. Common Fault Doctrine

203 Which is NOT a requirement in the case of simple The woman must be a virgin in the case of GRACE ARVIOLA, MD DIAGNOSTIC
seduction? qualified seduction. (TOP 3 - AUG 2014 EXAM - FEB
A. The woman is 12 to 18 years old. MED BOARDS; 2015
B. The woman if of good moral standing. TOPNOTCH MD)
C. The woman is single or widow.
D. The woman is a virgin.
E. Seduction is by means of deceit.

204 What amount of evidence must be presented in order to Criminal case: guilt beyond reasonable doubt. GRACE ARVIOLA, MD DIAGNOSTIC
convict a physician in a civil case? Administrative case: Substantial evidence. (TOP 3 - AUG 2014 EXAM - FEB
A. Guilt beyond reasonable doubt MED BOARDS; 2015
B. Preponderance of evidence TOPNOTCH MD)
C. Substantial evidence
D. Physical evidence
E. Adequate evidence

205 Dr. Reyes lost an administrative case filed against him Criminal case: Imprisonment. Civil case: GRACE ARVIOLA, MD DIAGNOSTIC
by his wife. He would be penalized as ________. Damages. (TOP 3 - AUG 2014 EXAM - FEB
A. Imprisonment MED BOARDS; 2015
B. Damages TOPNOTCH MD)
C. Suspension or revocation of his license
D. Community service
E. Destierro

206 The National Health Insurance Act is also known as: This was approved on February 14, 1995 by GRACE ARVIOLA, MD DIAGNOSTIC
A. RA 7578 President Fidel Ramos. (TOP 3 - AUG 2014 EXAM - FEB
B. RA 7875 MED BOARDS; 2015
C. RA 9241 TOPNOTCH MD)
D. RA 9142
E. RA 1995

207 PhilHealth's goal of universal coverage or Kalusugan GRACE ARVIOLA, MD DIAGNOSTIC


Pangkalahatan is defined as _____ of the Philippine (TOP 3 - AUG 2014 EXAM - FEB
population. MED BOARDS; 2015
A. 70% TOPNOTCH MD)
B. 75%
C. 80%
D. 85%
E. 90%

208 The doctor-patient relationship is fiduciary in that it GRACE ARVIOLA, MD DIAGNOSTIC


implies: (TOP 3 - AUG 2014 EXAM - FEB
A. Mutual respect MED BOARDS; 2015
B. Mutual consent TOPNOTCH MD)
C. Mutual trust
D. Mutual benefits
E. Meeting of two minds

209 Reinstatement of one's license may be applied for after GRACE ARVIOLA, MD DIAGNOSTIC
a period of how many years? (TOP 3 - AUG 2014 EXAM - FEB
A. 1 year MED BOARDS; 2015
B. 2 years TOPNOTCH MD)
C. 3 years
D. 4 years
E. 5 years

210 Medical records of outpatients should be kept for at Inpatients: 15 years. GRACE ARVIOLA, MD DIAGNOSTIC
least: (TOP 3 - AUG 2014 EXAM - FEB
A. 2 years MED BOARDS; 2015
B. 5 years TOPNOTCH MD)
C. 10 years
D. 15 years
E. 20 years

TOPNOTCH MEDICAL BOARD PREP LEGAL MEDICINE AND JURIS SUPEREXAM Page 26 of 86
For inquiries visit www.topnotchboardprep.com.ph or email us at topnotchmedicalboardprep@gmail.com
TOPNOTCH MEDICAL BOARD PREP LEGAL MEDICINE AND JURIS SUPEREXAM
For inquiries visit www.topnotchboardprep.com.ph or email us at topnotchmedicalboardprep@gmail.com
Item QUESTION EXPLANATION AUTHOR TOPNOTCH
# EXAM
211 Sexual deviation described as rubbing of one's genital Tribadism: lesbianism. Irrumation: fellatio. GRACE ARVIOLA, MD DIAGNOSTIC
part to another person's body. Algolagnia: sado-masochism. Uranism: (TOP 3 - AUG 2014 EXAM - FEB
A. Tribadism fingering, fondling. Rubbing: frottage. MED BOARDS; 2015
B. Irrumation TOPNOTCH MD)
C. Algolagnia
D. Uranism
E. Frottage

212 A male physician accused of a criminal act hysterically Direct contempt is any inappropriate GRACE ARVIOLA, MD DIAGNOSTIC
shouted against the witness during a court hearing. The behavior done within or immediately near (TOP 3 - AUG 2014 EXAM - FEB
physician may be cited for: the court. MED BOARDS; 2015
A. Direct contempt TOPNOTCH MD)
B. Indirect contempt
C. Prision mayor
D. Prision correcional
E. Arresto mayor

213 Fingerprints may be collected from a murder victim GRACE ARVIOLA, MD DIAGNOSTIC
with washerwoman's skin by injection of: (TOP 3 - AUG 2014 EXAM - FEB
A. Water MED BOARDS; 2015
B. Glycerin TOPNOTCH MD)
C. Salt solution
D. NaOH
E. Oil

214 The onset of decomposition usually starts at _________ GRACE ARVIOLA, MD DIAGNOSTIC
after death. (TOP 3 - AUG 2014 EXAM - FEB
A. 8-16 hours MED BOARDS; 2015
B. 16-24 hours TOPNOTCH MD)
C. 24-48 hours
D. 48-72 hours
E. 72-96 hours

215 Decomposition is retarded among stillborns because: GRACE ARVIOLA, MD DIAGNOSTIC


A. It has less weight. (TOP 3 - AUG 2014 EXAM - FEB
B. It has a greater percentage of fat. MED BOARDS; 2015
C. It is sterile. TOPNOTCH MD)
D. It has a small surface area.
E. The bones are not fully calcified.

216 A couple whose blood types are B and AB are expecting They will only have children with blood type GRACE ARVIOLA, MD DIAGNOSTIC
a child. Their child will NOT have a blood type of: A, B, and AB. (TOP 3 - AUG 2014 EXAM - FEB
A. A MED BOARDS; 2015
B. B TOPNOTCH MD)
C. AB
D. O
E. None; all are possible

217 In asphyxia, the right ventricle is _______ and the left GRACE ARVIOLA, MD DIAGNOSTIC
ventricle is __________. (TOP 3 - AUG 2014 EXAM - FEB
A. Empty, also empty MED BOARDS; 2015
B. Empty, dilated TOPNOTCH MD)
C. Dilated, Empty
D. Dilated, also dilated
E. None of the above

218 Which is NOT a test for gunpowder residues? Takayama test is a test for hemoglobin. GRACE ARVIOLA, MD DIAGNOSTIC
A. Walker's test (TOP 3 - AUG 2014 EXAM - FEB
B. Gonzales test MED BOARDS; 2015
C. Diphenylamine test TOPNOTCH MD)
D. Takayama test
E. Lung's test

219 Oil of vitriol is: GRACE ARVIOLA, MD DIAGNOSTIC


A. HCl (TOP 3 - AUG 2014 EXAM - FEB
B. H2SO4 MED BOARDS; 2015
C. NaOH TOPNOTCH MD)
D. HCN
E. CO

220 Odd and Even Rule applies to: GRACE ARVIOLA, MD DIAGNOSTIC
A. Lacerations (TOP 3 - AUG 2014 EXAM - FEB
B. Gunpowder residues MED BOARDS; 2015
C. Vehicular accidents TOPNOTCH MD)
D. Retained bullets
E. Drowning

TOPNOTCH MEDICAL BOARD PREP LEGAL MEDICINE AND JURIS SUPEREXAM Page 27 of 86
For inquiries visit www.topnotchboardprep.com.ph or email us at topnotchmedicalboardprep@gmail.com
TOPNOTCH MEDICAL BOARD PREP LEGAL MEDICINE AND JURIS SUPEREXAM
For inquiries visit www.topnotchboardprep.com.ph or email us at topnotchmedicalboardprep@gmail.com
Item QUESTION EXPLANATION AUTHOR TOPNOTCH
# EXAM
221 Which of the following is a false statement regarding it should be between collateral relatives by LEAN ANGELO MIDTERM
incestuous and void marriages from the beginning? blood within the 4th civil degree SILVERIO, MD (TOP EXAM 1 - FEB
A. Between the legitimate children of the adopter and 4 - AUG 2014 MED 2015
adopted BOARDS; TOPNOTCH
B. Between adopting mother/father and the adopted MD), MD
C. Between collateral relatives by blood within the 3rd
civil degree
D. between ascendants and descendants
E. none of the above

222 which of the following is true about Drug Habituation? drug habituation is defined as the desire to LEAN ANGELO MIDTERM
A. It involves both psychological and physical have continuous use of the drug but with the SILVERIO, MD (TOP EXAM 1 - FEB
dependence capacity to refrain physically from using it. 4 - AUG 2014 MED 2015
B. The detrimental effect is primarily on the individual The desire is merely psychical and not BOARDS; TOPNOTCH
C. Strong tendency to increase the dose compulsive and it is primarily detrimental MD), MD
D. An overpowering desire or need to continue taking only to the individual and not to the society.
the drug or to otain it by any means there is little tendency to increase the dose.
E. all are correct all the other choices defines the term Drug
addiction.
223 what is the most likely blood alcohol of the person 50mg%- increase self confidence, decrease LEAN ANGELO MIDTERM
when he/she presents with general apathy, inertia, inhibition, 100mg%- mental confusion, loss SILVERIO, MD (TOP EXAM 1 - FEB
impaired response to stimuli, impaired conscious either of critical judgement, incompetency, 150- 4 - AUG 2014 MED 2015
lethargic or stuporous decrease sense of pain, slurred speech, BOARDS; TOPNOTCH
A. 50mg% exaggerated emotions, disorientation. MD), MD
B. 100mg% 300mg%- apathy, general inertia, stuporous.
C. 150mg% 400mg%- comatose state.
D. 300mg%
E. 400mg%

224 which of the following is ethical in terms of payment of contingent fee - unethical since it depends on LEAN ANGELO MIDTERM
medical fees ? the success or failure of the treatment SILVERIO, MD (TOP EXAM 1 - FEB
A. Dichotomous fee instituted. No fees will be received for efforts 4 - AUG 2014 MED 2015
B. Straight fee until a result is attained. Dichotomous fee - BOARDS; TOPNOTCH
C. Contingency fee unethical, sharing a fee with another MD), MD
D. Retainer fee physician, laboratory, or drug company, not
E. All are unethical. based on the services performed. Straight fee
aka pakyaw system- unethical, the amount is
dependent on what will be remaining after all
expenses had been subtracted.retainer fee is
ethical and it relates with MD working a
company of HMOs- medical fee is measured
by the time and space not by the quality or
quantity of medical services rendered.
225 which of the following methods in approximating the it should be 8x the length of the head. LEAN ANGELO MIDTERM
height of a person is false? SILVERIO, MD (TOP EXAM 1 - FEB
A. 12x the length of the head 4 - AUG 2014 MED 2015
B. Distance between suprasternal notch and the BOARDS; TOPNOTCH
symphysis pubis is about 1/3 of the height MD), MD
C. The distance between the base of the skull to the
coccyx is about 44% of the height
D. distance between the tips of the middle fingers of
both hands with the arms extended laterally
E. none of the above

226 true about fingerprinting except? It is formed during the 4th month of LEAN ANGELO MIDTERM
A. It is the most valuable method of identification gestation SILVERIO, MD (TOP EXAM 1 - FEB
B. Formed during 6th month of gestation 4 - AUG 2014 MED 2015
C. It reappears consistently the same as before even BOARDS; TOPNOTCH
after finger injury or burn MD), MD
D. It is not the same even in identical twins
E. the chances of having the same fingerprint is about
1 in 64billion.

227 characterized by compulsive desire of a person to rub partialism- the person has special affinity to LEAN ANGELO MIDTERM
his sex organ against some parts of the body of another certain part of the female body. Uranism - SILVERIO, MD (TOP EXAM 1 - FEB
? sexual gratification attained by fingering, 4 - AUG 2014 MED 2015
A. Partialism fondling with the breast, licking part of the BOARDS; TOPNOTCH
B. Frottage body. Sodomy-sexual act through the anus of MD), MD
C. uranism another human being. Troilism- group of 3
D. sodomy person having sex.
E. Troilism

228 imbecile is in what IQ range? Idiot - 0-20, feeble minded - 41-70 LEAN ANGELO MIDTERM
A. 0-20 SILVERIO, MD (TOP EXAM 1 - FEB
B. 21-40 4 - AUG 2014 MED 2015
C. 41-70 BOARDS; TOPNOTCH
D. 70-100 MD), MD
E. None of the above

TOPNOTCH MEDICAL BOARD PREP LEGAL MEDICINE AND JURIS SUPEREXAM Page 28 of 86
For inquiries visit www.topnotchboardprep.com.ph or email us at topnotchmedicalboardprep@gmail.com
TOPNOTCH MEDICAL BOARD PREP LEGAL MEDICINE AND JURIS SUPEREXAM
For inquiries visit www.topnotchboardprep.com.ph or email us at topnotchmedicalboardprep@gmail.com
Item QUESTION EXPLANATION AUTHOR TOPNOTCH
# EXAM
229 which of the following is not true regarding the to practice medicine in the philippines, the LEAN ANGELO MIDTERM
prerequisite to practice medicine? law requires that you must be a filipino SILVERIO, MD (TOP EXAM 1 - FEB
A. At least 21 years of age citizen ( regardless whether you are a natural 4 - AUG 2014 MED 2015
B. Natural born filipino born or naturalized) or a citizen of foreign BOARDS; TOPNOTCH
C. Holder of a certificate of registration country in which there is a reciprocity b/w MD), MD
D. Not convicted by any court on criminal offense the two countries
involving moral turpitude
E. none of the above

230 A pathologist mistakenly exchanged the report from a the hospital can be liable since pathologist as LEAN ANGELO MIDTERM
frozen section from one person to another leading to well as radiologist and anesthesiologist SILVERIO, MD (TOP EXAM 1 - FEB
unneccessary surgery. The hospital will be liable although independent contractors are also 4 - AUG 2014 MED 2015
according what principle or doctrine? employees of the hospital. BOARDS; TOPNOTCH
A. Doctrine of contributory negligence MD), MD
B. Doctrine of bad result rule
C. captain of the ship doctrine
D. Doctrine of ostensible agent
E. all of the above

231 This refers to monetary sum awarded to the plaintiff in actual or compensatory damage is an LEAN ANGELO MIDTERM
an action where there is no substantial loss or injury to adequate compensation only for pecuniary SILVERIO, MD (TOP EXAM 1 - FEB
be compensated but where the law recognizes a loss suffered by a person as he has duly 4 - AUG 2014 MED 2015
technical invasion of the right of the plaintiff or a proved. Liquidated damages are those agreed BOARDS; TOPNOTCH
breach of the defendants duty? by the parties to a contract, to be paid in case MD), MD
A. Nominal damages of breech thereof. Temperate or moderate
B. Compensatory damages damages is given when the court finds that
C. Liquidated damages some pecuniary loss has been suffered but its
D. Temperate damages amount cannot be proved with certainty.
E. Actual damages

232 which of the following will not make the person persons not disqualified from becoming a LEAN ANGELO MIDTERM
disqualified from becoming a witness? witness: parties or othe persons interested in SILVERIO, MD (TOP EXAM 1 - FEB
A. Tender age the outcome of the case. Persons who have 4 - AUG 2014 MED 2015
B. Husband against his wife been convicted of a crime. Persons on BOARDS; TOPNOTCH
C. Unsound mind account of his opinion on matters of religious MD), MD
D. Person convicted of a crime belief.
E. Descendants of a

233 Serious physical injuries resulting the blindness of one prison correcional in medium and maximum LEAN ANGELO MIDTERM
eye is a ground for what kind of punishment against the periods includes the loss of the use of SILVERIO, MD (TOP EXAM 1 - FEB
offendant? speech, or the power to hear or to smell or 4 - AUG 2014 MED 2015
A. Prison mayor loss of an eye, a hand, a foot, an arm or a leg BOARDS; TOPNOTCH
B. Arresto mayor which result to incapacitation for the work MD), MD
C. Prison correcional in medium and maximum he/she was therefore habitually engaged.
periods loss of both eyes is a ground for prison
D. Prison correcional in medium and minimum mayor.
periods
E. reclusion perpetua
234 It refers to asphyxia by closing of the external trottling is by application of constricting LEAN ANGELO MIDTERM
respiratory orifices either by the use of the hand or by force to the neck using the hands. Choking is SILVERIO, MD (TOP EXAM 1 - FEB
some other means? impaction of a foreign body in the respiratory 4 - AUG 2014 MED 2015
A. throttling passages. Strangulation by ligature is BOARDS; TOPNOTCH
B. smothering compression of the neck by means of ligature MD), MD
C. Choking which is tightened by a force other than the
D. Ligature strangulation weight of the body.
E. None of the above

235 the legal definition of virginity refers to which of the virginity refers to the condition of a female LEAN ANGELO MIDTERM
following choices? who has not experienced sexual intercourse SILVERIO, MD (TOP EXAM 1 - FEB
A. Refers to a female and whose genital organs have not been 4 - AUG 2014 MED 2015
B. Should be virtuous altered by carnal connection. The latters BOARDS; TOPNOTCH
C. Must have an intact hymen refers to even slightest penetration of sexual MD), MD
D. Can either be a male or a female organ. it is not neccessary means that
E. Must have full penetration of sexual organs ruptured hymen is always a result of carnal
knowledge. Letter B specifically refers to
virgo intacta.
236 it refers to illegitimate chuldren who were conceived Spurius is a general term for illegitimacy LEAN ANGELO MIDTERM
specifically by prostitutes? secondary to adultery, sacrilegious ( born of SILVERIO, MD (TOP EXAM 1 - FEB
A. Sacrilegious parents who have been ordained in sacris). 4 - AUG 2014 MED 2015
B. spurius Incestious( out of incestious marriage), and BOARDS; TOPNOTCH
C. Adulterous manceras ( conceived by prostitute). MD), MD
D. Manceres
E. Incestuous

TOPNOTCH MEDICAL BOARD PREP LEGAL MEDICINE AND JURIS SUPEREXAM Page 29 of 86
For inquiries visit www.topnotchboardprep.com.ph or email us at topnotchmedicalboardprep@gmail.com
TOPNOTCH MEDICAL BOARD PREP LEGAL MEDICINE AND JURIS SUPEREXAM
For inquiries visit www.topnotchboardprep.com.ph or email us at topnotchmedicalboardprep@gmail.com
Item QUESTION EXPLANATION AUTHOR TOPNOTCH
# EXAM
237 which of the following is an incidental right of a medical incidental rights of a doctor are as follows: LEAN ANGELO MIDTERM
doctor? right of way during a response to emergency SILVERIO, MD (TOP EXAM 1 - FEB
A. Right to compensation call, right to exemption from execution of 4 - AUG 2014 MED 2015
B. Right to avail of hospital services instruments and library, to compensation, to BOARDS; TOPNOTCH
C. Right to choose patients hold public or private offices, to membership MD), MD
D. Right to limit practice in medical societies. All the other choices are
E. Right to determine appropriate management inherent rights of a physician.
procedures
238 what is a prerequisite for conviction in a case of civil Criminal - guilt beyond reasonable doubt; LEAN ANGELO MIDTERM
liability of a physician? administrative liability - substantial evidence. SILVERIO, MD (TOP EXAM 1 - FEB
A. Guilt beyond reasonable doubt 4 - AUG 2014 MED 2015
B. Preponderance of evidence BOARDS; TOPNOTCH
C. Substantial evidence MD), MD
D. Any of the above
E. None of the above

239 this refers to an object or substances which may be a autoptic or real evidence refers to evidences LEAN ANGELO MIDTERM
part of the body of the crime which proves that the that is actually perceived using the five SILVERIO, MD (TOP EXAM 1 - FEB
crime existed? sensations. Associative evidence is a form of 4 - AUG 2014 MED 2015
A. Autoptic evidence physical evidence that links suspect to the BOARDS; TOPNOTCH
B. Corpus delicti evidence crime. MD), MD
C. Associative evidence
D. Real evidence
E. A or D

240 which of the following bones cannot be used in bones that can be used in determining the LEAN ANGELO MIDTERM
determining the sex of the individual/victim? sex of a victim are the following: skull, pelvis, SILVERIO, MD (TOP EXAM 1 - FEB
A. Humerus humerus, femur, and sternum. 4 - AUG 2014 MED 2015
B. Radius BOARDS; TOPNOTCH
C. Sternum MD), MD
D. Skull
E. Pelvis

241 Which of the following statements regarding death is Solis page 111 KEVIN BRYAN LO, MIDTERM 2
true? MD (TOP 7 - AUG EXAM - FEB
A. the civil personality of a natural person is not 2014 MED BOARDS; 2015
extinguished by death TOPNOTCH MD)
B. the criminal liability of a person is extinguished by
death
C. death of a partner is not a cause for dissolution of
partnership agreement
D. state of suspended animation is an example of
somatic or clinical death
E. none of the above
242 Which of the following regarding an antemortem clot is solis page 131, post mortem clot forms layers KEVIN BRYAN LO, MIDTERM 2
correct? or lines of zahn, chicken fat layering MD (TOP 7 - AUG EXAM - FEB
A. clot is homogenous 2014 MED BOARDS; 2015
B. lines of zahn is present TOPNOTCH MD)
C. chicken fat layer present
D. soft in consistency
E. surface of the blood vessels are smooth and healthy
after clot removal

243 The microorganism that plays an important and Solis page 146 KEVIN BRYAN LO, MIDTERM 2
dominant role n decomposition of a dead body is MD (TOP 7 - AUG EXAM - FEB
A. Bacteroides fragilis 2014 MED BOARDS; 2015
B. Clostridium perfringens TOPNOTCH MD)
C. Clostridium welchii
D. E. Coli
E. Enterobacteriacae

244 Which method of conducting a search of the crime solis page 162 KEVIN BRYAN LO, MIDTERM 2
scene wherein the area is blocked out in the form of a MD (TOP 7 - AUG EXAM - FEB
rectangle and the searcher proceeds slowly at the same 2014 MED BOARDS; 2015
pace along the path parallel to one side of the rectangle TOPNOTCH MD)
A. spiral method
B. wheel method
C. strip method
D. zone method
E. circular method

245 All of the following are conditions in which an autopsy solis page 165 autopsy important topic, upon KEVIN BRYAN LO, MIDTERM 2
can be performed on a dead body EXCEPT written request of police authorities MD (TOP 7 - AUG EXAM - FEB
A. whenever required by special laws 2014 MED BOARDS; 2015
B. upon order of a competent court, mayor and or fiscal TOPNOTCH MD)
C. upon written request by the attending physician
D. when the nearest of kin shall request in writing
E. none of the above

TOPNOTCH MEDICAL BOARD PREP LEGAL MEDICINE AND JURIS SUPEREXAM Page 30 of 86
For inquiries visit www.topnotchboardprep.com.ph or email us at topnotchmedicalboardprep@gmail.com
TOPNOTCH MEDICAL BOARD PREP LEGAL MEDICINE AND JURIS SUPEREXAM
For inquiries visit www.topnotchboardprep.com.ph or email us at topnotchmedicalboardprep@gmail.com
Item QUESTION EXPLANATION AUTHOR TOPNOTCH
# EXAM
246 Which of the following characteristics of the gunshot burns up to 6 inches, smoke up to 12 inches, KEVIN BRYAN LO, MIDTERM 2
wound will NOT be present when a pistol is fired at a gun powder stipplings up to 24 inches MD (TOP 7 - AUG EXAM - FEB
distance of 10 inches? 2014 MED BOARDS; 2015
A. smudging TOPNOTCH MD)
B. tattoing
C. soot
D. burning
E. contusion collar

247 Which of the following statements are false regarding paraffin test always negative in an exit KEVIN BRYAN LO, MIDTERM 2
gun shot wounds? wound, the number of entrance and exit MD (TOP 7 - AUG EXAM - FEB
A. the number of entrance wounds may be lesser than wound may not always be the same, bullet 2014 MED BOARDS; 2015
the number of exit wounds may lodge on natural orifices or enter via TOPNOTCH MD)
B. the number of exit wounds may be lesser than the natural orifices
number of entrance wounds
C. a contusion collar may be present in an entrance
wound
D. paraffin test is always negative in an exit wound
E. all the above statements are true

248 All of the above descriptions are consistent with direction of the ligature mark in hanging is KEVIN BRYAN LO, MIDTERM 2
strangulation as a result of hanging or via suicide usually inverted V shaped with apex as the MD (TOP 7 - AUG EXAM - FEB
EXCEPT site of the knot 2014 MED BOARDS; 2015
A. hyoid bone is frequently injured TOPNOTCH MD)
B. direction of the ligature mark is usually horizontal
C. ligature is usually at the level of the hyoid bone
D. ligature groove is deepest opposite the site of the
know
E. vertebral injury is frequently observed
249 Which of the following is the type of child abuser intermiitent child abuser - parents who KEVIN BRYAN LO, MIDTERM 2
wherein the parents supposedly mean well but their periodically batter a child with periods of MD (TOP 7 - AUG EXAM - FEB
attempts at rearing their children result in permanent proper care, one time abuser - never repeat 2014 MED BOARDS; 2015
injury or death of their children the act, constant child abuser is a parent who TOPNOTCH MD)
A. intermittent abuser actually hates his or her child
B. one time abuser
C. constant child abuser
D. ignorant abuser
E. psychotic abuser

250 What is the term given to include women who have had true physical virginity hymen is intact, KEVIN BRYAN LO, MIDTERM 2
previous sexual intercourse or even habitually but had opening barely admit the tip of finger, false MD (TOP 7 - AUG EXAM - FEB
not given birth? physical virginity hymen intact but orifice is 2014 MED BOARDS; 2015
A. true physical virginity wide, demi virginity woman who permits any TOPNOTCH MD)
B. false physical virginity form of sexual liberties as long as they
C. demi-virginity abstain from rupturing the hymen
D. virgo intacta
E. moral virginity

251 Which of the following is NOT considered an absolute a person whose country's existing laws KEVIN BRYAN LO, MIDTERM 2
qualification for a candidate to apply for the complete permit citizens of the Philippines to practice MD (TOP 7 - AUG EXAM - FEB
board examination in medicine? medicine under the same rules and 2014 MED BOARDS; 2015
A. he must be a filipino citizen regulations can be allowed by virtue of TOPNOTCH MD)
B. he must be of good moral character reciprocity
C. must be of sound mind
D. must have completed internship
E. should be a holder of a degree of doctor of medicine

252 The following are considered NOT participating in the household remedies are not included in KEVIN BRYAN LO, MIDTERM 2
practice of medicine EXCEPT practice of medicine, any person who MD (TOP 7 - AUG EXAM - FEB
A. any medical student or intern under the supervision examines and prescribes or treats a patient is 2014 MED BOARDS; 2015
of a registered physician without supervision of a registed physician is TOPNOTCH MD)
B. a duly registered masseur of physiotherapist applies practicing medicine
massage upon prescription or order from a physician
C. a clinical psychologist or mental hygienist who is
under direct supervision by a physician
D. A person who administers paracetamol to a patient
with fever
E. a nurse who takes the blood pressure and prescribes
the appropriate treatment

253 Which of the following conditions are to which the contributory negligence cannot apply where KEVIN BRYAN LO, MIDTERM 2
doctrine of contributory negligence may NOT apply? the patient is mentally ill, semiconscious, MD (TOP 7 - AUG EXAM - FEB
A. failure to give the physician an accurate medical heavily sedated or of advanced age 2014 MED BOARDS; 2015
history TOPNOTCH MD)
B. A post surgical patient din not follow the physician's
take home instructions and medications
C. a semiconscious patient rolls over from her stretcher
and breaks her hip
D. leaving a hospital against the advice of the attending
physician
E. failure to seek further medical assistance if
symptoms persisted despite warnings from the
physician
TOPNOTCH MEDICAL BOARD PREP LEGAL MEDICINE AND JURIS SUPEREXAM Page 31 of 86
For inquiries visit www.topnotchboardprep.com.ph or email us at topnotchmedicalboardprep@gmail.com
TOPNOTCH MEDICAL BOARD PREP LEGAL MEDICINE AND JURIS SUPEREXAM
For inquiries visit www.topnotchboardprep.com.ph or email us at topnotchmedicalboardprep@gmail.com
Item QUESTION EXPLANATION AUTHOR TOPNOTCH
# EXAM
254 Which of the following elements are included in all are requisites for res ipsa loquitur KEVIN BRYAN LO, MIDTERM 2
fulfilling the conditions necessary for application of the MD (TOP 7 - AUG EXAM - FEB
doctrine of res ipsa loquitur? 2014 MED BOARDS; 2015
A. the accident would not occur without negligence TOPNOTCH MD)
B. the agency must be in the control of the defendant
C. there was no contributing conduct by the plaintiff
D. all of the above
E. only A and B

255 These are damages paid for by virtue of the amount moral damages from mental anguish, KEVIN BRYAN LO, MIDTERM 2
agreed upon by the parties to be paid in case of breach physical suffering ,fright and moral shock, MD (TOP 7 - AUG EXAM - FEB
of contract temperate or moderate - more than nominal 2014 MED BOARDS; 2015
A. moral damages less than compensatory, when there is some TOPNOTCH MD)
B. liquidated damages pecuniary loss but the amount cannot be
C. temperate damages proved with certainty page 374 solis
D. moderate damages
E. nominal damages

256 Which of the following tests can be used to determine all of the above are the same as paraffin test KEVIN BRYAN LO, MIDTERM 2
the presence of gunpower residues on the hand or site MD (TOP 7 - AUG EXAM - FEB
of the wound entrance? 2014 MED BOARDS; 2015
A. dermal nitrate test TOPNOTCH MD)
B. paraffin test
C. diphenylamine test
D. all of the above
E. only A and B

257 Which of the following terms refers to natural children all are under illegitimate children except KEVIN BRYAN LO, MIDTERM 2
acknowledged by the father or the mother separately if legitimate and legitimated children. Natural MD (TOP 7 - AUG EXAM - FEB
the acknowledging parent was legally competent to children proper- born out of wedlock but 2014 MED BOARDS; 2015
contract marriate at the time of conception? parents have no impediment to marry each TOPNOTCH MD)
A. legitimated children other, by legal fiction children born of void
B. legitimate children mariages or born of voidable marriages,
C. natural children proper legitimate those born in lawful wedlock or
D. natural children by legal fiction within 300days after dissolution of marriage
E. natural children by presumption

258 Which among the following statements regarding the criminal act is almost always in the home KEVIN BRYAN LO, MIDTERM 2
infanticide is false? but not always MD (TOP 7 - AUG EXAM - FEB
A. it is most often committed by the mother 2014 MED BOARDS; 2015
B. the criminal act is always committed in the home TOPNOTCH MD)
C. the crime scene shows no disturbance, witnesses and
or noise or outcry
D. trauma applied is minimal
E. if the newborn child found dead was born dead,
burden of proof that a living child has been killed is on
the prosecution

259 All of the following are chemical examinations in luminescence test is a physical test KEVIN BRYAN LO, MIDTERM 2
detection of blood and blood stains EXCEPT? MD (TOP 7 - AUG EXAM - FEB
A. Luminescence test 2014 MED BOARDS; 2015
B. Guaiacum test TOPNOTCH MD)
C. Kastle-Meyer test
D. Benzidine test
E. none of the above

260 Which of the following tests determines whether the review the different tests KEVIN BRYAN LO, MIDTERM 2
specimen is of human blood or not MD (TOP 7 - AUG EXAM - FEB
A. Precipitin test 2014 MED BOARDS; 2015
B. Aetone-haemin of Wagenhaar test TOPNOTCH MD)
C. puramine test
D. berberio's test
E. none of the above

261 What refers to the art of identification of fingerprints by RAYMUND MARTIN MIDTERM 3
comparison? LI, MD (TOP 1 - AUG EXAM - FEB
A. Graphology 2014 MED BOARDS; 2015
B. Endoscopy TOPNOTCH MD)
C. Poroscopy
D. Dactyloscopy
E. Dactylography

262 What is the most prominent sign of death? RAYMUND MARTIN MIDTERM 3
A. Cessation of heart action LI, MD (TOP 1 - AUG EXAM - FEB
B. Cessation of respiration 2014 MED BOARDS; 2015
C. Progressive fall of body temperature TOPNOTCH MD)
D. Loss of power to move

TOPNOTCH MEDICAL BOARD PREP LEGAL MEDICINE AND JURIS SUPEREXAM Page 32 of 86
For inquiries visit www.topnotchboardprep.com.ph or email us at topnotchmedicalboardprep@gmail.com
TOPNOTCH MEDICAL BOARD PREP LEGAL MEDICINE AND JURIS SUPEREXAM
For inquiries visit www.topnotchboardprep.com.ph or email us at topnotchmedicalboardprep@gmail.com
Item QUESTION EXPLANATION AUTHOR TOPNOTCH
# EXAM
263 A person lost an eye after being physically assaulted. He Any injury with loss of body part or RAYMUND MARTIN MIDTERM 3
was hospitalized hor 10 days. The offender is liable for? deformity is considered serious regardless of LI, MD (TOP 1 - AUG EXAM - FEB
A. Serious physical injury days of confinement/treatment 2014 MED BOARDS; 2015
B. Less serious physical injury TOPNOTCH MD)
C. Slight physical injury
D. Mutilation

264 The crime of qualified seduction can be committed on a RAYMUND MARTIN MIDTERM 3
woman above the age of 18 when: LI, MD (TOP 1 - AUG EXAM - FEB
A. The sexual act was done deceitfully 2014 MED BOARDS; 2015
B. The sexual act was committed with force or TOPNOTCH MD)
intimidation
C. The offender is the father or brother of the victim
D. The woman is a prostitute.

265 A baby born alive has a legal responsibility. One of the RAYMUND MARTIN MIDTERM 3
tests used to determine life before death of a fetus is to LI, MD (TOP 1 - AUG EXAM - FEB
float the lungs in the water or what is called as: 2014 MED BOARDS; 2015
A. Wrendin's TOPNOTCH MD)
B. Hydrostatic
C. Icard's
D. Breslau's

266 If the husband is impotent yet her wife gave birth to a RAYMUND MARTIN MIDTERM 3
healthy baby while in lawful wedlock. Which of the LI, MD (TOP 1 - AUG EXAM - FEB
following is true? 2014 MED BOARDS; 2015
A. Proof of impotence will overthrow presumption of TOPNOTCH MD)
legitimacy
B. The child's legitimacy cannot be questioned
C. The child upon birth can be declared illegitimate
D. None of the above
E. All of the above
267 Which of the following constitutes parricide? RAYMUND MARTIN MIDTERM 3
A. When a man kills his common law wife LI, MD (TOP 1 - AUG EXAM - FEB
B. A moron who has three wives and kills the last 2014 MED BOARDS; 2015
married to him TOPNOTCH MD)
C. When a married woman kills his illegitimate
grandfather
D. When a son kills his illigitimate father
268 Disorientation, mental confusion, dizziness, sensory RAYMUND MARTIN MIDTERM 3
disturbances, exaggerated emotional state suggests that LI, MD (TOP 1 - AUG EXAM - FEB
the blood alcohol level is: 2014 MED BOARDS; 2015
A. 400 mg% TOPNOTCH MD)
B. 150-300 mg%
C. 80-100 mg%
D. 500 mg%
E. <80 mg%

269 A confession is different from admission because the RAYMUND MARTIN MIDTERM 3
latter is: LI, MD (TOP 1 - AUG EXAM - FEB
A. statement of guilt 2014 MED BOARDS; 2015
B. statement of fact which does not directly involve an TOPNOTCH MD)
acknowledgement of guilt
C. expressed acknowledgement of the truth of his guilt
as to the crime charged
D. All of the above
E. None of the above

270 Patient prmised to pay his doctor if he is cured of his RAYMUND MARTIN MIDTERM 3
cancer. This refers to: LI, MD (TOP 1 - AUG EXAM - FEB
A. Contingent fee 2014 MED BOARDS; 2015
B. Simple contractual fee TOPNOTCH MD)
C. Packaged deal fee
D. Retainers fee
E. None of the above

271 A fiduciary physician-patient relationship is based on: RAYMUND MARTIN MIDTERM 3


A. law LI, MD (TOP 1 - AUG EXAM - FEB
B. court order 2014 MED BOARDS; 2015
C. mutual trust TOPNOTCH MD)
D. mutual concent
E. All of the above

272 Burden of evidence needed to convict a physician for RAYMUND MARTIN MIDTERM 3
civil liability LI, MD (TOP 1 - AUG EXAM - FEB
A. Substantial evidence 2014 MED BOARDS; 2015
B. Preponderance of evidence TOPNOTCH MD)
C. Proof beyond reasonable doubt
D. Circumstantial evidence

TOPNOTCH MEDICAL BOARD PREP LEGAL MEDICINE AND JURIS SUPEREXAM Page 33 of 86
For inquiries visit www.topnotchboardprep.com.ph or email us at topnotchmedicalboardprep@gmail.com
TOPNOTCH MEDICAL BOARD PREP LEGAL MEDICINE AND JURIS SUPEREXAM
For inquiries visit www.topnotchboardprep.com.ph or email us at topnotchmedicalboardprep@gmail.com
Item QUESTION EXPLANATION AUTHOR TOPNOTCH
# EXAM
273 Which of the following is an administrative liability of a RAYMUND MARTIN MIDTERM 3
hospital? LI, MD (TOP 1 - AUG EXAM - FEB
A. Failing to place sideboards on a bed after they were 2014 MED BOARDS; 2015
deemed necessary TOPNOTCH MD)
B. Administering the wrong blood product
C. Improperly administering a hypodermic injection
D. Keeping a hot water bottle too long on a patient's
body
E. None of the above
274 As a result of deformity sustained from an operation, a RAYMUND MARTIN MIDTERM 3
doctor was sued by his patient for mental anguish and LI, MD (TOP 1 - AUG EXAM - FEB
social humiliation. The type of damage she can recover 2014 MED BOARDS; 2015
is? TOPNOTCH MD)
A. Actual damage
B. Exemplary damage
C. Compensatory damage
D. Moral damage

275 The doctrine which will bar a patient from recovering RAYMUND MARTIN MIDTERM 3
damages because he assents to the risks of injury: LI, MD (TOP 1 - AUG EXAM - FEB
A. Doctrine of assumption of risk 2014 MED BOARDS; 2015
B. Contributory negligence TOPNOTCH MD)
C. Doctrine of continuing negligence
D. Doctrine of sole responsibility

276 Which of the following psychological patterns of a RAYMUND MARTIN MIDTERM 3


doctor-patient relationship is present if the patient LI, MD (TOP 1 - AUG EXAM - FEB
seeks help and is ready and willing to participate? 2014 MED BOARDS; 2015
A. Activity-passivity TOPNOTCH MD)
B. Mutual participation
C. Guidance-Cooperation
D. Superior knowledge
E. None of the above

277 Objects or substances which maybe a part of the body RAYMUND MARTIN MIDTERM 3
of the crime refers to: LI, MD (TOP 1 - AUG EXAM - FEB
A. Testimonial evidence 2014 MED BOARDS; 2015
B. Associative evidence TOPNOTCH MD)
C. Tracing evidence
D. Experimental evidence
E. Corpus delicti evidence

278 What is the estimated duration of cadaveric rigidity in tropical summer - 18-36; tropical cold RAYMUND MARTIN MIDTERM 3
tropical countries during summer? weather 24-48; temperate climate - 2-3 days LI, MD (TOP 1 - AUG EXAM - FEB
A. 18-36 hours 2014 MED BOARDS; 2015
B. 24-48 hours TOPNOTCH MD)
C. 48-72 hours
D. a short time immediately after death
E. None of the above

279 A person who inflicts serious physical injury on another A - Imbecility/total blindness/impotency - RAYMUND MARTIN MIDTERM 3
causing physical deformity is punishable by: prison mayor; B - Loss of hearing, LI, MD (TOP 1 - AUG EXAM - FEB
A. Prison mayor sight, smell, speech, arm, leg - prison 2014 MED BOARDS; 2015
B. Prison correcional in its medium or maximum correccional medium or maximum
TOPNOTCH MD)
periods C - Deformity/loss of other members of body
C. Prison correcional in its minimum or medium - prison correccional minimum or medium
periods D - <90 days incapacitated - arresto mayor
D. Arresto mayor
E. None of the above
280 Which of the following is true regarding the rights of Unrestricted correspondence; Free from RAYMUND MARTIN MIDTERM 3
patients when confined to the hospital? restraints except if there is justifiable reason, LI, MD (TOP 1 - AUG EXAM - FEB
A. Right to receive visitors and relatives within the right to be discharged 2014 MED BOARDS; 2015
limits prescribed by the hospital TOPNOTCH MD)
B. Correspondence that is restricted by the hospital
C. Right to be free from mechanical restraints with
absolutely no exceptions
D. All of the above

281 The privilege to practice medicine in the Philippines is The police power of the state has the right to ERIC ROYD FINAL EXAM -
regulated by what entity/body? regulate the practice of medicine TALAVERA, MD (TOP FEB 2015
A. Board of Medicine 1 - AUG 2014 MED
B. Police power of the State BOARDS; TOPNOTCH
C. Judicial body of the State MD)
D. Professional Regulatory Commission
E. Philippine Medical Association

TOPNOTCH MEDICAL BOARD PREP LEGAL MEDICINE AND JURIS SUPEREXAM Page 34 of 86
For inquiries visit www.topnotchboardprep.com.ph or email us at topnotchmedicalboardprep@gmail.com
TOPNOTCH MEDICAL BOARD PREP LEGAL MEDICINE AND JURIS SUPEREXAM
For inquiries visit www.topnotchboardprep.com.ph or email us at topnotchmedicalboardprep@gmail.com
Item QUESTION EXPLANATION AUTHOR TOPNOTCH
# EXAM
282 Which of the following test would determine whether ERIC ROYD FINAL EXAM -
the blood and semen obtained from the crime scene is TALAVERA, MD (TOP FEB 2015
of human origin? 1 - AUG 2014 MED
A. Precipitin test BOARDS; TOPNOTCH
B. Takayama's test MD)
C. Florence's test
D. Benzidin's test
E. Fodere's test

283 A physician was asked by the court to describe the Merely description without inference of the ERIC ROYD FINAL EXAM -
findings in his physical examination of his patient, who possible explanation for the injuries or TALAVERA, MD (TOP FEB 2015
was a victim in a hazing incident. The physician in court diagnosis would mean that he is merely 1 - AUG 2014 MED
is acting as a/an? acting as an ordinary witness. BOARDS; TOPNOTCH
A. Expert witness MD)
B. Ordinary witness
C. Special witness
D. Both A and B
E. None of the above

284 A patient with malignant melanoma undergoes SIMILAR TO PREVIOUS BOARD EXAM ERIC ROYD FINAL EXAM -
experimental treatment despite lack of knowledge of CONCEPT/PRINCIPLE. TALAVERA, MD (TOP FEB 2015
the possible adverse effects. The researchers are not 1 - AUG 2014 MED
liable for any untoward event that may occur due to BOARDS; TOPNOTCH
what doctrine? MD)
A. Captain of the ship doctrine
B. Doctrine of sole responsibility
C. Doctrine of forseeability
D. Doctrine of contributory negligence
E. Assumption of risk

285 An OFW who came from Sierra Leone returned to the for infectious - buried not later than 12 hours ERIC ROYD FINAL EXAM -
Philippines and died due to complications of Ebola TALAVERA, MD (TOP FEB 2015
virus. If he is to remain unembalmed he must be buried 1 - AUG 2014 MED
not later than how many hours? BOARDS; TOPNOTCH
A. 6 hours MD)
B. 12 hours
C. 20 hours
D. 24 hours
E. 48 hours

286 A 65 year old male who underwent exploratory Res ipsa loquitur: the thing speaks for itself ERIC ROYD FINAL EXAM -
laparotomy complained of abdominal pain 24 hours TALAVERA, MD (TOP FEB 2015
post-op. Imaging studies revealed that a mosquito 1 - AUG 2014 MED
forcep and multiple gauzes were retained in the BOARDS; TOPNOTCH
patient's abdominal cavity. The surgeon is liable by MD)
application of what doctrine?
A. Doctrine of vicarious liability
B. Doctrine of continuing negligence
C. Doctrine of contributory negligence
D. Doctrine of res ipsa loquitur
E. Doctrine of direct negligence

287 Which of the following tests is used to determine if the ERIC ROYD FINAL EXAM -
fetus was born alive by allowing to float the lungs in TALAVERA, MD (TOP FEB 2015
water? 1 - AUG 2014 MED
A. Hydrostatic Test BOARDS; TOPNOTCH
B. Icard’s test MD)
C. Breslau’s test
D. Wrendin's test
E. Precipitin Test

288 A 37 year old man was hit by a speeding van in the legs ERIC ROYD FINAL EXAM -
and was thrown 15 feet from impact, subsequently TALAVERA, MD (TOP FEB 2015
hitting the pavement. As the attending physician, apart 1 - AUG 2014 MED
from the fractures on both legs, you noted abrasions, BOARDS; TOPNOTCH
contusions and lacerations in the abdomen, flanks and MD)
chest. These injuries are referred to as what?
A. Run over injuries
B. Primary impact
C. Secondary impact
D. Contrecoup injuries
E. Coup injuries

TOPNOTCH MEDICAL BOARD PREP LEGAL MEDICINE AND JURIS SUPEREXAM Page 35 of 86
For inquiries visit www.topnotchboardprep.com.ph or email us at topnotchmedicalboardprep@gmail.com
TOPNOTCH MEDICAL BOARD PREP LEGAL MEDICINE AND JURIS SUPEREXAM
For inquiries visit www.topnotchboardprep.com.ph or email us at topnotchmedicalboardprep@gmail.com
Item QUESTION EXPLANATION AUTHOR TOPNOTCH
# EXAM
289 A 25 year old female, who was a victim of a home SIMILAR TO PREVIOUS BOARD EXAM ERIC ROYD FINAL EXAM -
invasion, sustained multiple stab wounds and is rapildy CONCEPT/PRINCIPLE. TALAVERA, MD (TOP FEB 2015
deteriorating due to massive blood loss. She made a 1 - AUG 2014 MED
statement to the attending physician at the emergency BOARDS; TOPNOTCH
room in referrence to the individual who inflicted the MD)
injury on her. Her statement is best considered as
what?
A. Privillege communication
B. Dying declaration
C. Inadmissible evidence
D. Hearsay evidence
E. None of the above

290 You evaluated a 68 year old woman with 4-month ERIC ROYD FINAL EXAM -
history of chest pains and fainting spells that you feel TALAVERA, MD (TOP FEB 2015
would benefit from a cardiac catheterization. After 1 - AUG 2014 MED
informing her fully of the benefits, consequences and BOARDS; TOPNOTCH
risks of the procedure, she decided with full MD)
understanding to forego the intervention. What is the
ethical approach that you should do?
A. Respect her choice
B. Explore reasons for her decision and try to convince
her again
C. Refer her to a psychiatrist
D. Continue with the procedure nevertheless even if
without her consent
E. Talk to the family and have them convince her to
undergo the procedure

291 For a child to be legitimate, he must be born in a lawful ERIC ROYD FINAL EXAM -
wedlock or within how many number of days after the TALAVERA, MD (TOP FEB 2015
dissolution of marriage? 1 - AUG 2014 MED
A. 110 days BOARDS; TOPNOTCH
B. 230 days MD)
C. 250 days
D. 300 days
E. 365 days

292 A physician refused to attend on several occasion of a ERIC ROYD FINAL EXAM -
court hearing citing as reason that he has several out- TALAVERA, MD (TOP FEB 2015
patients to examine. Which of the following is he liable 1 - AUG 2014 MED
for? BOARDS; TOPNOTCH
A. Direct contempt of the court MD)
B. Indirect contempt of the court
C. Disrespect for the court
D. No liability, as his reason is considered qualified and
valid
E. Both A and B
293 Confidentiality of information in the course of a doctor- Confidentiality may only be breached in the ERIC ROYD FINAL EXAM -
patient relationship may be breached in which of the best interest of justice and public safety. TALAVERA, MD (TOP FEB 2015
following instances? 1 - AUG 2014 MED
A. When disclosure is necessary to serve the best BOARDS; TOPNOTCH
interest of justice MD)
B. When disclosure of information will serve the
private health sector and pharmaceutical companies
C. When disclosure is needed in a civil case to
determine the moral character of a patient
D. Both A and C
E. All of the above

294 What type of injury is incurred when a blow to the ERIC ROYD FINAL EXAM -
forehead causes contusion of the eyeball due to fracture TALAVERA, MD (TOP FEB 2015
of the bone at the roof of the orbit? 1 - AUG 2014 MED
A. Coup BOARDS; TOPNOTCH
B. Contre-coup MD)
C. Coup contre-coup
D. Locus minoris resistencia
E. None of the above

TOPNOTCH MEDICAL BOARD PREP LEGAL MEDICINE AND JURIS SUPEREXAM Page 36 of 86
For inquiries visit www.topnotchboardprep.com.ph or email us at topnotchmedicalboardprep@gmail.com
TOPNOTCH MEDICAL BOARD PREP LEGAL MEDICINE AND JURIS SUPEREXAM
For inquiries visit www.topnotchboardprep.com.ph or email us at topnotchmedicalboardprep@gmail.com
Item QUESTION EXPLANATION AUTHOR TOPNOTCH
# EXAM
295 The hospital is almost full and the only vacant beds are Contamination of other patients is foreseen ERIC ROYD FINAL EXAM -
in the ward. An infectious case was admitted in the when one admits an infectious case in the TALAVERA, MD (TOP FEB 2015
ward resulting to contamination of other patients. The ward filled with patients who does not have 1 - AUG 2014 MED
physician who admitted the patient is liable under what the same infection. BOARDS; TOPNOTCH
doctrine? MD)
A. Captain of the ship
B. Contributory Negligence
C. Superior knowledge
D. Assumption of risk
E. Foreseeability

296 Which of the following findings in death is not Cadaveric spasm or instantaneous rigor is ERIC ROYD FINAL EXAM -
consistently present and may not appear on a person not always present and is not part of the 3 TALAVERA, MD (TOP FEB 2015
who has just died? stages of muscles after death. 1 - AUG 2014 MED
A. Cadaveric spasm BOARDS; TOPNOTCH
B. Primary flaccidity MD)
C. Rigor mortis
D. Secondary flaccidity
E. None of the above

297 What is the burden of evidence needed to convict a ERIC ROYD FINAL EXAM -
physician in a criminal case? TALAVERA, MD (TOP FEB 2015
A. Substantial evidence 1 - AUG 2014 MED
B. Circumstantial evidence BOARDS; TOPNOTCH
C. Guilt beyond reasonable doubt MD)
D. Preponderance of evidence
E. None of the above

298 Which of the following is an incidental right of a The rest of the choices are inherent rights of ERIC ROYD FINAL EXAM -
physician? a physician TALAVERA, MD (TOP FEB 2015
A. Right to exemption from execution of instruments 1 - AUG 2014 MED
and library BOARDS; TOPNOTCH
B. Right to choose patients MD)
C. Right to avail of hospital services
D. Right to limit the practice of medicine
E. None of the above
299 A woman attacked her co-worker in the office after a Slight: 1-9 days ERIC ROYD FINAL EXAM -
heated arguement. A physican attended to the injury Less serious: 10-29 days TALAVERA, MD (TOP FEB 2015
that resolved within 3 days. After a week, the victim Serious: >30days; or with scar, deformity, 1 - AUG 2014 MED
developed multiple linear scars over the forehead and loss of organ BOARDS; TOPNOTCH
malar area where the wounds were initially inflicted. MD)
What is the legal classification of this injury?
A. Slight physical injury
B. Less serious physical injury
C. Serious physical injury
D. Mutilation
E. Physical injuries inflicted in a tumultuous affair

300 What is the term used to refer to physical injuries ERIC ROYD FINAL EXAM -
which develop on parts of the body near bony tissues TALAVERA, MD (TOP FEB 2015
that are deep seated and caused by forcible impact of 1 - AUG 2014 MED
hard blunt objects? BOARDS; TOPNOTCH
A. Abrasion MD)
B. Contusion
C. Hematoma
D. Laceration
E. Incision

301 Which of the following is an inherent right of a inherent rights of the physician: 1. to choose LEAN ANGELO BACK-UP
physician ? patients, however should respond to any in SILVERIO, MD (TOP MIDTERM
A. To choose patients in a non emergency cases cases of emergency. 2. to limit the practice. 3. 4 - AUG 2014 MED EXAM - FEB
B. To hold certain public or private offices to determine the appropriate management BOARDS; TOPNOTCH 2015
C. Right to have appropriate compensation procedures. 4. to avail of hospital services. MD), MD
D. Right of way while responding to call of emergency Incidental rights of the physician: 1. right of
E. all of the above way during emergency calls 2. right of
exemption from execution of instruments
and library 3. to hold cetain public and
private offices, 4. to have appropriate
compensation 5. to membership in medical
societies. Topnotch handout
302 the kind of medical compensation given to doctors retainer fee - measureed by the space of time LEAN ANGELO BACK-UP
working in a private/public companies or health and not by the quality or quantity of medical SILVERIO, MD (TOP MIDTERM
maintenance organizations? services rendered. Contractual fee - contract 4 - AUG 2014 MED EXAM - FEB
A. Contractual fee stipulates the nature of procedure, it states BOARDS; TOPNOTCH 2015
B. Contingent fee the value of such medical services either MD), MD
C. Retainer fee verbval or in writing. commision fee or fee
D. Straight fee splitting - sharing of fee with another
E. Commission fee physician, laboratory or drug company, not
baed on services
performed(unethical).contingent fee - fee
depends on the success or failure of the
treatment instituted (unethical). straight fee (
pakyaw system) - the amount of medical fee
is dependent on what will be the remaining
TOPNOTCH MEDICAL BOARD PREP LEGAL MEDICINE AND JURIS SUPEREXAM Page 37 of 86
For inquiries visit www.topnotchboardprep.com.ph or email us at topnotchmedicalboardprep@gmail.com
TOPNOTCH MEDICAL BOARD PREP LEGAL MEDICINE AND JURIS SUPEREXAM
For inquiries visit www.topnotchboardprep.com.ph or email us at topnotchmedicalboardprep@gmail.com
Item QUESTION EXPLANATION AUTHOR TOPNOTCH
# EXAM
balance (unethical). topnotch handout.

303 what is the minimum requirement for conviction if the immorable of dishonourable conduct is a LEAN ANGELO BACK-UP
physician is charged with immoral or dishonourable ground for administrative investigation. The SILVERIO, MD (TOP MIDTERM
conduct towards his/her patient under the medical act requirement for conviction is a substantial 4 - AUG 2014 MED EXAM - FEB
of 1959? evidence on proving the charge agains the BOARDS; TOPNOTCH 2015
A. Guilt beyond reasonable doubt physician. The venue for trial is at the PRC MD), MD
B. Preponderance of evidence board of medicine. While the penalty may
C. Substantial evidence range from reprimand, suspension or
D. Circumstantial evidence revocation of license. civil liabilities -
E. none of the above peponderance of evidence while criminal
liabilities is of guilty beyond reasonable
doubt.
304 which of the following will not disqualify a person as a persons not disqualified from becoming a LEAN ANGELO BACK-UP
witness? witness:1. parties or other persons interested SILVERIO, MD (TOP MIDTERM
A. Persons who have been convicted of a crime in the outcome of the case 2. persons who 4 - AUG 2014 MED EXAM - FEB
B. Descendants testifying against his/her parents have been convicted of a crime. 3. persons on BOARDS; TOPNOTCH 2015
C. Husband testifying against a criminal by her wife account of his opinion on matters of religious MD), MD
against a unrelated party belief.
D. person of tender age
E. all of the above

305 this is a type of medical evidence which is perceived A- doing experiment inorder to confirm LEAN ANGELO BACK-UP
through the use of sight, hearing, taste, smell and touch? his/her allegation in front of the court. C- use SILVERIO, MD (TOP MIDTERM
A. Experimental evidence of letters, figures,or marks intended to be 4 - AUG 2014 MED EXAM - FEB
B. Autoptic evidence used for the purpose of recording. D-objects BOARDS; TOPNOTCH 2015
C. Documentary evidence or substances which may be a part of the MD), MD
D. Corpus delicti evidence body of the crime. E- this are physical
E. Tracing evidence evidences that assist an investigator in
locating a suspect ex. ship or aircraft
manifest.
306 which of the following is not correct about the methods the approximate height of the person is about LEAN ANGELO BACK-UP
in approximating the height of a person? 8x the length of the head SILVERIO, MD (TOP MIDTERM
A. 9x the length of the head 4 - AUG 2014 MED EXAM - FEB
B. The distance betw+een suprasternal notch and the BOARDS; TOPNOTCH 2015
symphysis pubis is about 1/3 of the height MD), MD
C. Length of the forearm measured from the tip of the
olecranon process to the tip of the middle finger is 5/19
of the height
D. the distance from the base of the skull to the coccyx
is about 44% of the height
E. none of the above

307 what is the minimum approximate age of a person if 9 y/o -12 permanent teeth; 11 y/o -20 LEAN ANGELO BACK-UP
his/her 3rd molar root is completely calcified? permanent teeth, 13y/o - 28 permanent SILVERIO, MD (TOP MIDTERM
A. 20 teeth, 25 y/o - calcification of 3rd molar root 4 - AUG 2014 MED EXAM - FEB
B. 22 is complete. BOARDS; TOPNOTCH 2015
C. 25 MD), MD
D. 27
E. 30

308 this refers to qualified seduction? A- simple seduction. B- rape, D- consented LEAN ANGELO BACK-UP
A. Single or widow of good reputation over 12 y/o but abduction. SILVERIO, MD (TOP MIDTERM
under 18 y/o committed y means of deceit 4 - AUG 2014 MED EXAM - FEB
B. Having carnal knowledge to a woman under 12 y/o BOARDS; TOPNOTCH 2015
age MD), MD
C. To a virgin >12 y/o but under 18 y/o committed by
any person in public office, priest, or teacher,
D. to a virgin >12 but under 18y/o carried out with
her consent and with lewd design
E. none of the above

309 which of the following is a form of microscopic chemical examination - ammonia saline LEAN ANGELO BACK-UP
examination of blood or blood stains? extract test, guaiacum test, schombein test, SILVERIO, MD (TOP MIDTERM
A. Precipitin test phenolphthalein test, leucomalachite green 4 - AUG 2014 MED EXAM - FEB
B. Wagenhaar test test. Microscopic examination - takayama BOARDS; TOPNOTCH 2015
C. Schombein test test, teichmann blood crystal test, and MD), MD
D. Phenolpthalein test wagenhaar test. Biological examination -
E. All of the above precipitin test and blood groupings

TOPNOTCH MEDICAL BOARD PREP LEGAL MEDICINE AND JURIS SUPEREXAM Page 38 of 86
For inquiries visit www.topnotchboardprep.com.ph or email us at topnotchmedicalboardprep@gmail.com
TOPNOTCH MEDICAL BOARD PREP LEGAL MEDICINE AND JURIS SUPEREXAM
For inquiries visit www.topnotchboardprep.com.ph or email us at topnotchmedicalboardprep@gmail.com
Item QUESTION EXPLANATION AUTHOR TOPNOTCH
# EXAM
310 During cadaveric rigidity, what is the minimum time initiation of rigidity starts at the range of 3-6 LEAN ANGELO BACK-UP
needed for the whole body to become stiff? hours of death, it becomes total rigidity at a SILVERIO, MD (TOP MIDTERM
A. 10 hours least of 12 hours. Duration of rigidity in 4 - AUG 2014 MED EXAM - FEB
B. 12 hours temperate countries is about 48-72 hours BOARDS; TOPNOTCH 2015
C. 14 hours while in tropical countries is 24-48 hours. MD), MD
D. 16 hours
E. 18 hours

311 what is the approximate duration needed for a red to green color - 4-5 days , red to yellow LEAN ANGELO BACK-UP
contusion to change from red to green? color - 7-10 days .( SIMILAR TO PREVIOUS SILVERIO, MD (TOP MIDTERM
A. 2-4 days BOARD EXAM CONCEPT/PRINCIPLE) 4 - AUG 2014 MED EXAM - FEB
B. 4-5 days BOARDS; TOPNOTCH 2015
C. 6-8 days MD), MD
D. 8-10 days
E. None of the above

312 what is the maximum penalty applied to a person who prison correcional in its medium and LEAN ANGELO BACK-UP
is charged with serious physical injuries resulting for maximum periods - 1. loss of the use of SILVERIO, MD (TOP MIDTERM
the victim to loose one eye? speech or the power to hear or to smell or 4 - AUG 2014 MED EXAM - FEB
A. Prison correcional in its minimum or medium loss of an eye, a hand, a fooot, an arm or a leg BOARDS; TOPNOTCH 2015
periods 2. loss of the used of any such member. 3. MD), MD
B. Arresto mayor becomes incapacitated for the work in which
C. Prsion correcional in its maximum periods he was therefore habitually engaged. Prison
D. prison mayor mayor - results to imbecility, impotency, and
E. reclusion perpetua total blindness ( should be both eyes).

313 What type of asphyxia does cyanide poisoning belongs Histotoxic death- refers to asphyxia LEAN ANGELO BACK-UP
to? secondary to failire of cellular oxidative SILVERIO, MD (TOP MIDTERM
A. Anoxic death processes: cyanide, alcohol overdose and 4 - AUG 2014 MED EXAM - FEB
B. Anemic anoxic death carbon monoxide. Anoxic death - associated BOARDS; TOPNOTCH 2015
C. Stagnant anoxic death with the failure of the arterial blood to MD), MD
D. Histotoxic anoxic dath become normally saturated with oxygen e.g.
E. None of the above air passage obstruction, high altitude,
congenital heart disease (R-->L shunt).
anemic anoxic death- refers to decrease
capacity of the blood to carry oxygen e.g.
severe hemorrhage, CO ( also histotoxic),
anemia. stagnant anoxic death - failure of
circulation ( Myocardial infarction)
314 this refers to asphyxia by forcefully occluding the Burking - any form of traumatic asphyxia for LEAN ANGELO BACK-UP
external respiratory orifices either by the use of hand the purpose or intent to sold the body to SILVERIO, MD (TOP MIDTERM
or by some other means? medical schools for dissection. Trottling - use 4 - AUG 2014 MED EXAM - FEB
A. smothering of hand to apply constricting force to the BOARDS; TOPNOTCH 2015
B. choking neck. Choking - impaction of foreign body in MD), MD
C. Trottling the respiratory passageways.
D. Burking
E. None of the above

315 definition of virginity applies to which of the following On legal point, the term virginity refers to a LEAN ANGELO BACK-UP
statement? female who has not experienced sexual SILVERIO, MD (TOP MIDTERM
A. Refers to a female intercourse and whose genital organs have 4 - AUG 2014 MED EXAM - FEB
B. Refers to both sexes. not been altered by carnal connection. C- BOARDS; TOPNOTCH 2015
C. Refers to a state of not knowing the nature of sexual refers to moral virginity. MD), MD
life and not having experienced sexual intercourse
D. All of the above

316 Refers to the compulsive desire of a person to rub his uranism - sexual gratification attained by LEAN ANGELO BACK-UP
sexua lorgan against some parts of the body of another? fingering, fondling with the breast, licking SILVERIO, MD (TOP MIDTERM
A. Frottage part of the body. Partialism - the person has a 4 - AUG 2014 MED EXAM - FEB
B. Partialism special affinity to certain part of the female BOARDS; TOPNOTCH 2015
C. Mixoscopia body. Sodomy - sexual act through anus of MD), MD
D. Sodomy another human being. mixoscopia - sexual
E. Uranism pleasure is attain by watching couple undress
or during sexual activity.

317 This refers to an examination of peripheral circulation Magnus test - ligature is applied around the LEAN ANGELO BACK-UP
wherein a fluorescent solution is injected to the base of the finger with moderate tightness. In SILVERIO, MD (TOP MIDTERM
circulation to see whether the latter will spread to all a living person, there appears a bloodless 4 - AUG 2014 MED EXAM - FEB
over body or not ? zone at the site of application. If such ligature BOARDS; TOPNOTCH 2015
A. Diaphanous test is applied on a dead man, there is no change MD), MD
B. Icards test in color. Diaphanus test - the fingers are
C. Magnus test spread wide and the finger webs are viewed
D. Winslows test through a strong lightt in the living, the finger
E. none of the above webs appear red but yellow in dead. Winslow
test - theres is no movement of the image
formed by reflecting artificial light on the
water or mercury contained ina saucer and
placed on chest or abdomen if respiration is
not taking place.

TOPNOTCH MEDICAL BOARD PREP LEGAL MEDICINE AND JURIS SUPEREXAM Page 39 of 86
For inquiries visit www.topnotchboardprep.com.ph or email us at topnotchmedicalboardprep@gmail.com
TOPNOTCH MEDICAL BOARD PREP LEGAL MEDICINE AND JURIS SUPEREXAM
For inquiries visit www.topnotchboardprep.com.ph or email us at topnotchmedicalboardprep@gmail.com
Item QUESTION EXPLANATION AUTHOR TOPNOTCH
# EXAM
318 How many hours it usually takes for a body to float LEAN ANGELO BACK-UP
secondary to drowning ? SILVERIO, MD (TOP MIDTERM
A. 16 hours 4 - AUG 2014 MED EXAM - FEB
B. 18 hours BOARDS; TOPNOTCH 2015
C. 22 hours MD), MD
D. 24 hours
E. 48 hours

319 refers to illegitimacy when a child is conceived by Spurious children refers to the general term LEAN ANGELO BACK-UP
parents who are in a second civil degree? illegitimate children. It can be classified into SILVERIO, MD (TOP MIDTERM
A. Spurious 4 kinds: adulterous, sacrilegious -parents 4 - AUG 2014 MED EXAM - FEB
B. Manceras who are ordained by sacris, incestious - BOARDS; TOPNOTCH 2015
C. Sacrilegious blood relationship, manceras- conceived by MD), MD
D. Incestuous prostitutes
E. Adulterous

320 A 29 y/o medical technologist went for consult mutual participation - patient thinks that he LEAN ANGELO BACK-UP
secondary to chronic cough. based on the history and is juridicially equal to the MD and that the SILVERIO, MD (TOP MIDTERM
PE, you diagnosed her to have post nasal drip. After nature of relationship is on a negotiated 4 - AUG 2014 MED EXAM - FEB
hearing your diagnosis, she started searching the agreement. Guidance cooperation- patient is BOARDS; TOPNOTCH 2015
internet in front of you and states that you dont know conscious ,seeks help and ready to cooperate. MD), MD
what postnasal drip really is. what is the nature of MD Activity-passivity- patient is unable to
patient relationship in this case? contribute activity ( patient is unconscious).
A. activity-passivity fiduciary - states that the relationship is
B. Guidance cooperation based on mutual trust and confidence.
C. mutual participation
D. fiduciary
E. expressed contract

321 Carnal knowledge means: DEBBIE ROSE BACK-UP


A. knowledge about the nature of sexual life TANENGSY, MD (TOP MIDTERM
B. rupture of the hymen 5 - AUG 2014 MED EXAM - FEB
C. slightlest penetration of the female sexual organ by BOARDS; TOPNOTCH 2015
the male sex organ MD)
D. Full penetration is necessary.

322 When a blow to the forehead causes contusion of the DEBBIE ROSE BACK-UP
eyeball due to a fracture of the bone at the roof of the TANENGSY, MD (TOP MIDTERM
orbit, this type of injury is called: 5 - AUG 2014 MED EXAM - FEB
A. coup BOARDS; TOPNOTCH 2015
B. contre-coup MD)
C. coup contre-coup
D. locus minoris resistencia

323 Dactylography is: Choice D pertains to dactyloscopy. DEBBIE ROSE BACK-UP


A. the study of the pores on the papillary or friction TANENGSY, MD (TOP MIDTERM
ridges of the skin for the purpose of identification 5 - AUG 2014 MED EXAM - FEB
B. arte of changing fingerprints BOARDS; TOPNOTCH 2015
C. art and study of recording fingerprints as a means of MD)
identification
D. art of comparing fingerprints for identification

324 If the person died as a result of a communicable disease DEBBIE ROSE BACK-UP
and is unembalmed, the body should be buried not later TANENGSY, MD (TOP MIDTERM
than: 5 - AUG 2014 MED EXAM - FEB
A. 12 hours BOARDS; TOPNOTCH 2015
B. 20 hours MD)
C. 24 hours
D. 48 hours

325 A wound may be indirectly fatal by reason of: Choices B, C, and D are direct causes of death DEBBIE ROSE BACK-UP
A. scarring effect from wounds. TANENGSY, MD (TOP MIDTERM
B. hemorrhage 5 - AUG 2014 MED EXAM - FEB
C. mechanical injuries to vital organs BOARDS; TOPNOTCH 2015
D. shock MD)

326 According to Dupuytren's classification, burns resulting fifth degree burns - involving the deep fascia DEBBIE ROSE BACK-UP
in charring of the limb and involving bone and muscle is and muscles which may result to severe TANENGSY, MD (TOP MIDTERM
considered to be: scarring or deformity 5 - AUG 2014 MED EXAM - FEB
A. fifth degree burns BOARDS; TOPNOTCH 2015
B. sixth degree burns MD)
C. seventh degree burns
D. eighth degree burns

TOPNOTCH MEDICAL BOARD PREP LEGAL MEDICINE AND JURIS SUPEREXAM Page 40 of 86
For inquiries visit www.topnotchboardprep.com.ph or email us at topnotchmedicalboardprep@gmail.com
TOPNOTCH MEDICAL BOARD PREP LEGAL MEDICINE AND JURIS SUPEREXAM
For inquiries visit www.topnotchboardprep.com.ph or email us at topnotchmedicalboardprep@gmail.com
Item QUESTION EXPLANATION AUTHOR TOPNOTCH
# EXAM
327 Kunkel's test is carried out on a blood sample to DEBBIE ROSE BACK-UP
determine the presence of: TANENGSY, MD (TOP MIDTERM
A. hydrogen sulfide 5 - AUG 2014 MED EXAM - FEB
B. hydrogen cyanide BOARDS; TOPNOTCH 2015
C. carbon monoxide MD)
D. nitrates

328 An expert medical witness has the following DEBBIE ROSE BACK-UP
qualification: TANENGSY, MD (TOP MIDTERM
A. duly licensed medical practitioner 5 - AUG 2014 MED EXAM - FEB
B. duly licensed medical practitioner with at least 5 BOARDS; TOPNOTCH 2015
years of active practice MD)
C. duly licensed medical practitioner who has
specialized in a particular branch of medicine
D. duly licensed medical practitioner who is a specialist
in the line along which he is testifying

329 A physician who fails to comply to a subpoena issued by DEBBIE ROSE BACK-UP
the court may be guilty of: TANENGSY, MD (TOP MIDTERM
A. administrative sanction 5 - AUG 2014 MED EXAM - FEB
B. direct contempt of the court BOARDS; TOPNOTCH 2015
C. indirect contempt of the court MD)
D. criminal act of ommission

330 In which of the following is there a physician-patient DEBBIE ROSE BACK-UP


relationship? TANENGSY, MD (TOP MIDTERM
A. physical examination conducted for insurance 5 - AUG 2014 MED EXAM - FEB
purposes with prescription BOARDS; TOPNOTCH 2015
B. physician appointed by trial courts to examine an MD)
accused
C. autopsy examinations
D. in casual consultations during conferences
331 A statement made by a dying patient to the attending DEBBIE ROSE BACK-UP
physician in reference to the persons who inflicted TANENGSY, MD (TOP MIDTERM
injury on him is known as: 5 - AUG 2014 MED EXAM - FEB
A. privileged communication BOARDS; TOPNOTCH 2015
B. dying declaration MD)
C. hearsay evidence
D. inadmissible evidence

332 The declaration of a patient who knows that he is dying DEBBIE ROSE BACK-UP
from cancer that his compadre told him that he TANENGSY, MD (TOP MIDTERM
(compadre) killed his wife is: 5 - AUG 2014 MED EXAM - FEB
A. privileged communication BOARDS; TOPNOTCH 2015
B. dying declaration MD)
C. hearsay evidence
D. testimonial evidence
E. deposition

333 Which one of the following medical fees is considered DEBBIE ROSE BACK-UP
unethical? TANENGSY, MD (TOP MIDTERM
A. professional fees 5 - AUG 2014 MED EXAM - FEB
B. retainer's fees BOARDS; TOPNOTCH 2015
C. contingent fees MD)
D. contractual fees
334 The medical records department refused to disclose the DEBBIE ROSE BACK-UP
clinical records of a patient. The patient may seek TANENGSY, MD (TOP MIDTERM
redress through: 5 - AUG 2014 MED EXAM - FEB
A. mandamus BOARDS; TOPNOTCH 2015
B. court injunction MD)
C. subpoena duces tecum
D. TRO

335 the following are rights of patients, except: DEBBIE ROSE BACK-UP
A. right to give consent to diagnostic and treatment TANENGSY, MD (TOP MIDTERM
procedures 5 - AUG 2014 MED EXAM - FEB
B. right to religious belief BOARDS; TOPNOTCH 2015
C. right to avail hospital services MD)
D. right to compensation
336 A form of sexual deviation whereby pain is the principal DEBBIE ROSE BACK-UP
factor for satisfaction, either as a recipient of inflictor, is TANENGSY, MD (TOP MIDTERM
called: 5 - AUG 2014 MED EXAM - FEB
A. algolagnia BOARDS; TOPNOTCH 2015
B. pedophilia MD)
C. fetishism
D. bestiality
E. uranism

TOPNOTCH MEDICAL BOARD PREP LEGAL MEDICINE AND JURIS SUPEREXAM Page 41 of 86
For inquiries visit www.topnotchboardprep.com.ph or email us at topnotchmedicalboardprep@gmail.com
TOPNOTCH MEDICAL BOARD PREP LEGAL MEDICINE AND JURIS SUPEREXAM
For inquiries visit www.topnotchboardprep.com.ph or email us at topnotchmedicalboardprep@gmail.com
Item QUESTION EXPLANATION AUTHOR TOPNOTCH
# EXAM
337 In slight physical injuries, the period of medical DEBBIE ROSE BACK-UP
attendance is not more than: TANENGSY, MD (TOP MIDTERM
A. 9 days 5 - AUG 2014 MED EXAM - FEB
B. 15 days BOARDS; TOPNOTCH 2015
C. 20 days MD)
D. 60 days

338 Which of the following is a test for the presence of DEBBIE ROSE BACK-UP
powder residues on clothing: TANENGSY, MD (TOP MIDTERM
A. Gonzales' test 5 - AUG 2014 MED EXAM - FEB
B. Walker's test BOARDS; TOPNOTCH 2015
C. Paraffin test MD)
D. Lung's test

339 A missing person is presumed dead after how many DEBBIE ROSE BACK-UP
years? TANENGSY, MD (TOP MIDTERM
A. 3 5 - AUG 2014 MED EXAM - FEB
B. 5 BOARDS; TOPNOTCH 2015
C. 7 MD)
D. 9
E. 11

340 In contrast to ante-mortem clot, post-mortem clot is: DEBBIE ROSE BACK-UP
A. firm TANENGSY, MD (TOP MIDTERM
B. soft 5 - AUG 2014 MED EXAM - FEB
C. homogeneous BOARDS; TOPNOTCH 2015
D. uniform MD)

341 Dactylography is: Choice A - poroscopy JESSICA MAE BACK-UP


A. The study of the pores on the papillary or friction Choice D - dactyloscopy SANCHEZ, MD (TOP MIDTERM
ridges of the skin for the purpose of identification 4 - AUG 2014 MED EXAM - FEB
B. Art of changing fingerprints BOARDS; TOPNOTCH 2015
C. Art and study of recording fingerprints as a means of MD)
identification
D. Art of comparing fingerprints for identification
342 The following are inherent rights of a physician, except: Reference: Topnotch Legal Med Handouts JESSICA MAE BACK-UP
A. To choose their patients SANCHEZ, MD (TOP MIDTERM
B. Limit their practice 4 - AUG 2014 MED EXAM - FEB
C. To be compensated BOARDS; TOPNOTCH 2015
D. Avail of hospital services MD)

343 A common law husband who kills his common law wife If the legal wife was killed, the charge would JESSICA MAE BACK-UP
with whom he has a 2 day-old child can be prosecuted be for parricide. SANCHEZ, MD (TOP MIDTERM
for: 4 - AUG 2014 MED EXAM - FEB
A. Infanticide BOARDS; TOPNOTCH 2015
B. Parricide MD)
C. Homicide
D. Impossible crime
344 It is an expressed acknowledgment by the accused in a Confession involves guilt. Admission would JESSICA MAE BACK-UP
criminal case of the truth of his guilt as to the crime be attesting to the fact only, but not to guilt. SANCHEZ, MD (TOP MIDTERM
charged: 4 - AUG 2014 MED EXAM - FEB
A. Confession BOARDS; TOPNOTCH 2015
B. Admission MD)
C. Interrogation
D. All of the above
345 The following are prerequisites to the practice of Minimum age for practice is 21 years old JESSICA MAE BACK-UP
Medicine, except: SANCHEZ, MD (TOP MIDTERM
A. Age of at least 19 years old 4 - AUG 2014 MED EXAM - FEB
B. Must be a holder of a certificate of registration BOARDS; TOPNOTCH 2015
C. Must not have been convicted by any court of MD)
competent jurisdiction of any offense involving moral
turpitude
D. Proper educational background
346 The presence of a weapon which is highly grasped by JESSICA MAE BACK-UP
the hand of a victim of a shooting incident is an example SANCHEZ, MD (TOP MIDTERM
of: 4 - AUG 2014 MED EXAM - FEB
A. Rigor mortis BOARDS; TOPNOTCH 2015
B. Death stiffening MD)
C. Cadaveric spasm
D. Cadaveric rigidity
347 Permission to disinter the remains of a person who 3 years for those who died of non-dangerous JESSICA MAE BACK-UP
died of a dangerous communicable disease maybe communicable disease. SANCHEZ, MD (TOP MIDTERM
granted after a burial period of: If with special permit, remains may be 4 - AUG 2014 MED EXAM - FEB
A. 4 years disinterred anytime except in cases of BOARDS; TOPNOTCH 2015
B. 5 years dangerous communicable disease. MD)
C. 6 years
D. 7 years Reference: Topnotch Legal Med Handouts
348 Dr. X, the MHO of Municipality X, shall perform an Choice A - should be requested by nearest of JESSICA MAE BACK-UP
autopsy on a dead body when: kin only, not any relative SANCHEZ, MD (TOP MIDTERM
A. Requested by any relative of the victim Choice B - it should be a written, not just 4 - AUG 2014 MED EXAM - FEB
B. Verbally directed by the police verbal request by the police BOARDS; TOPNOTCH 2015
C. Upon order of the mayor Choice D - if ordered by the provincial or city MD)
D. Ordered by the Provincial Hospital Medical Director fiscal

TOPNOTCH MEDICAL BOARD PREP LEGAL MEDICINE AND JURIS SUPEREXAM Page 42 of 86
For inquiries visit www.topnotchboardprep.com.ph or email us at topnotchmedicalboardprep@gmail.com
TOPNOTCH MEDICAL BOARD PREP LEGAL MEDICINE AND JURIS SUPEREXAM
For inquiries visit www.topnotchboardprep.com.ph or email us at topnotchmedicalboardprep@gmail.com
Item QUESTION EXPLANATION AUTHOR TOPNOTCH
# EXAM
Reference: Topnotch Legal Med Handouts

349 The most prominent sign of death is: JESSICA MAE BACK-UP
A. Progressive fall of the body temperature SANCHEZ, MD (TOP MIDTERM
B. Cessation of heart action and circulation 4 - AUG 2014 MED EXAM - FEB
C. Cessation of respiration BOARDS; TOPNOTCH 2015
D. Insensibility of body or loss of power to move MD)

350 After incision and drainage of an abscess at the OPD, the JESSICA MAE BACK-UP
patient failed to take his antibiotics and developed SANCHEZ, MD (TOP MIDTERM
sepsis. The doctor cannot be held liable under which 4 - AUG 2014 MED EXAM - FEB
doctrine? BOARDS; TOPNOTCH 2015
A. Doctrine of contributory negligence MD)
B. Doctrine of continuing negligence
C. Doctrine of assumption of risk
D. Doctrine of forseeability
351 For a dying declaration to be admissible in court, which JESSICA MAE BACK-UP
statement is not valid? SANCHEZ, MD (TOP MIDTERM
A. The declarant was not the victim 4 - AUG 2014 MED EXAM - FEB
B. The declaration must be with regard to the BOARDS; TOPNOTCH 2015
impending death MD)
C. The declarant was conscious of his impending death
D. The declarant was in full possession of his mental
faculties when he made the declaration
352 The following manner of death should be autopsied, JESSICA MAE BACK-UP
except: SANCHEZ, MD (TOP MIDTERM
A. Suicides 4 - AUG 2014 MED EXAM - FEB
B. Death unattended by a physician BOARDS; TOPNOTCH 2015
C. Accidental death MD)
D. Death occurring in a natural manner
353 Accidental death of a young child by suffocation, either JESSICA MAE BACK-UP
from the pressure of the beddings and pillows or from SANCHEZ, MD (TOP MIDTERM
the pressure of the unconscious mother is called: 4 - AUG 2014 MED EXAM - FEB
A. Battering BOARDS; TOPNOTCH 2015
B. Overlaying MD)
C. Burking
D. Mugging
354 The type of asphyxial death when oxygen is delivered to Anoxic - failure of blood to be saturated with JESSICA MAE BACK-UP
the tissues but cannot be utilised properly due to failure oxygen SANCHEZ, MD (TOP MIDTERM
of cellular oxidative processes is: A. Anoxic Anemic anoxic - decreased capacity to carry 4 - AUG 2014 MED EXAM - FEB
B. Histotoxic oxygen BOARDS; TOPNOTCH 2015
C. Anemic anoxic Stagnant anoxic - failure of circulation MD)
D. Stagnant anoxic

355 This type of criminal offender commits a crime with Reference: Topnotch Legal Med Handouts JESSICA MAE BACK-UP
motive or intention and with full possession of their SANCHEZ, MD (TOP MIDTERM
mental faculties: 4 - AUG 2014 MED EXAM - FEB
A. Ordinary offenders BOARDS; TOPNOTCH 2015
B. Rational offenders MD)
C. Professional offenders
D. Non-emotional offenders
356 When the color of a contusion changes to green, the Red/purple - 2-4 days JESSICA MAE BACK-UP
estimated age is: Yellow - 7-10 days SANCHEZ, MD (TOP MIDTERM
A. 2-4 days Disappears - 14-15 days 4 - AUG 2014 MED EXAM - FEB
B. 4-5 days BOARDS; TOPNOTCH 2015
C. 14-15 days MD)
D. 7-10 days
357 This type of injury need not be reported to police JESSICA MAE BACK-UP
authorities under PD 169: SANCHEZ, MD (TOP MIDTERM
A. Serious physical injuries 4 - AUG 2014 MED EXAM - FEB
B. Mutilation BOARDS; TOPNOTCH 2015
C. Slight physical injuries MD)
D. Less serious physical injuries
358 A male intern had sexual intercourse with a female JESSICA MAE BACK-UP
patient with her consent. Under what circumstance SANCHEZ, MD (TOP MIDTERM
would this be considered rape? 4 - AUG 2014 MED EXAM - FEB
A. The intern promised to marry her BOARDS; TOPNOTCH 2015
B. The patient is feeble minded MD)
C. The patient is his wife from whom he is legally
separated
D. The patient is above 18 years of age
359 A branch or division of law which deals with the rules JESSICA MAE BACK-UP
concerning pleadings, practices and procedures in all SANCHEZ, MD (TOP MIDTERM
courts of the Philippines: 4 - AUG 2014 MED EXAM - FEB
A. Remedial law BOARDS; TOPNOTCH 2015
B. Civil law MD)
C. Criminal law
D. Special law

TOPNOTCH MEDICAL BOARD PREP LEGAL MEDICINE AND JURIS SUPEREXAM Page 43 of 86
For inquiries visit www.topnotchboardprep.com.ph or email us at topnotchmedicalboardprep@gmail.com
TOPNOTCH MEDICAL BOARD PREP LEGAL MEDICINE AND JURIS SUPEREXAM
For inquiries visit www.topnotchboardprep.com.ph or email us at topnotchmedicalboardprep@gmail.com
Item QUESTION EXPLANATION AUTHOR TOPNOTCH
# EXAM
360 The following are justifiable grounds to refuse JESSICA MAE BACK-UP
admission of patients in a hospital, except: SANCHEZ, MD (TOP MIDTERM
A. Patient is chronically ill 4 - AUG 2014 MED EXAM - FEB
B. Patient’s condition is contagious and a risk to other BOARDS; TOPNOTCH 2015
patients MD)
C. All accommodations are filled
D. Patient is critically ill and needs emergent care
361 Tin and Tonbart had a child out of lawful wedlock. After Legitimation is defined as the remedy or MAIRRE JAMES BACK-UP
3 years, they decided to finally get married. The child process by which a child born out of lawful GADDI, MD (TOP 4 - MIDTERM
becomes? wedlock and are therefore considered AUG 2013 MED EXAM - FEB
A. Illegitimate since the child was born before the illegitimate are by fiction of law considered BOARDS; TOPNOTCH 2015
parents were married legitimate by subsequent valid marriage of MD)
B. Legitimated since there was subsequent marriage of the parents Topnotch pg 57
the parents
C. Adopted by the father since the child was born before
the parents were married and in effect only the child of
the mother prior to marriage
D. Sacrilegious since the child was born before the
parents were married
E. Manceres since the child was born before the parents
were married

362 Dr. Bon was seen by his neighbor at the Hyatt The neighbor most likely plans to file an MAIRRE JAMES BACK-UP
accompanied by a woman other than his wife. The administrative case against the doctor since GADDI, MD (TOP 4 - MIDTERM
neighbor was appalled and decided to file a case against he believes that the doctor's conduct is AUG 2013 MED EXAM - FEB
him since the neighbor believes that his actions are a unethical and unprofessional and only BOARDS; TOPNOTCH 2015
disgrace to his profession. What level of evidence is substantial evidence is needed. For civil MD)
needed for this case? cases: preponderance of evidence, for
A. Substantial evidence criminal cases: guilt beyond reasonable
B. Preponderance of evidence doubt Topnotch pg 7
C. Guilt beyond reasonable doubt
D. Considerable evidence
E. Significant evidence

363 A 16/F college student was failing her course in Math. Seduction of virgin over 12 years and under MAIRRE JAMES BACK-UP
After the last departmental exam, she approached her 18 comitted by any person in public GADDI, MD (TOP 4 - MIDTERM
professor so that she can drop the course and avoid a authority, teacher, entrusted with the AUG 2013 MED EXAM - FEB
failing grade. The professor proposed another solution, education or custody of a woman - qualified BOARDS; TOPNOTCH 2015
he will promise an "uno" if she agrees to have sex with seduction Topnotch pg 19 MD)
him. The professor can be charged with?
A. Simple seduction
B. Qualified seduction
C. Rape
D. Consented abduction
E. Acts of lasciviousness

364 Nino married Emma which happened to be his own Since Nino married his own stepdaughter, MAIRRE JAMES BACK-UP
stepdaughter. The had a child following the their marriage is considered void from the GADDI, MD (TOP 4 - MIDTERM
consummation of their marriage. The child is start. Therefore the child and all their future AUG 2013 MED EXAM - FEB
considered? children would be considered natural by BOARDS; TOPNOTCH 2015
A. Legitimate legal fiction Topnotch 58 MD)
B. Sacriligious
C. Natural by legal fiction
D. Spurious
E. Natural by presumption

365 Which of the following statements are TRUE? MAIRRE JAMES BACK-UP
A. Impotency if proven will overthrown the GADDI, MD (TOP 4 - MIDTERM
presumption of legitimacy AUG 2013 MED EXAM - FEB
B. Impotency can be grounds for annulment BOARDS; TOPNOTCH 2015
C. Impotence can be used as a defense of the defendant MD)
when accused of rape
D. A and C
E. All are true

366 A patient promised Dr. Joseph 1 million pesos if he can Contingent fee since it depends on the MAIRRE JAMES BACK-UP
completely reconstruct his jaw. The professional fee to success or failure of the treatment Topnotch GADDI, MD (TOP 4 - MIDTERM
be collected is: pg 5 AUG 2013 MED EXAM - FEB
A. Retainer fee BOARDS; TOPNOTCH 2015
B. Contingent fee MD)
C. Contractual fee
D. Straight fee
E. Dichotomous fee

TOPNOTCH MEDICAL BOARD PREP LEGAL MEDICINE AND JURIS SUPEREXAM Page 44 of 86
For inquiries visit www.topnotchboardprep.com.ph or email us at topnotchmedicalboardprep@gmail.com
TOPNOTCH MEDICAL BOARD PREP LEGAL MEDICINE AND JURIS SUPEREXAM
For inquiries visit www.topnotchboardprep.com.ph or email us at topnotchmedicalboardprep@gmail.com
Item QUESTION EXPLANATION AUTHOR TOPNOTCH
# EXAM
367 A surgeon finished doing an appendectomy in record Doctrine of common knowledge or res ipsa MAIRRE JAMES BACK-UP
time. After the surgery, the patient complained of loquitur - nature of the wrongful act is GADDI, MD (TOP 4 - MIDTERM
persistent abdominal pain. Upon xray, a thin stripe was suggestive of negligence such as when a AUG 2013 MED EXAM - FEB
seen inside the abdominal cavity indicating that a piece foregin body is left unintentionally during BOARDS; TOPNOTCH 2015
of surgical gauze was left inside the patient. The closing of an incision MD)
surgeon is liable by virtue of which doctrine?
A. Doctrine of continuing negligence
B. Doctrine of common knowledge
C. Captain of the ship doctrine
D. Doctrine of Vicarious liability
E. None of the above

368 A 34/M was involved in a bar fight. Unfortunately the Serious injuries involving the loss of the use MAIRRE JAMES BACK-UP
patient sustained multiple injuries to head which of speech, power to hear or loss of an eye, a GADDI, MD (TOP 4 - MIDTERM
culminated in the loss of his right eye. The person hand, a foot, an arm or a leg warrants a AUG 2013 MED EXAM - FEB
responsible would receive the penalty of? penalty of prison correcional in its medium BOARDS; TOPNOTCH 2015
A. Prison mayor and maximum periods. Imbecility, impotency MD)
B. Prison correcional or blindness (total or involvement of both
C. Arresto mayor eyes) warrants a penalty of prison mayor
D. Reclusion temporal Topnotch pg 32
E. Arresto menor

369 Test used to detect unburnt poweder in clothing: Gonzales'/Paraffin/Diphenylamine test MAIRRE JAMES BACK-UP
A. Walker's test checks for the presence of power residues on GADDI, MD (TOP 4 - MIDTERM
B. Gonzales' test the dorsum of the hand AUG 2013 MED EXAM - FEB
C. Paraffin test BOARDS; TOPNOTCH 2015
D. Icard's test MD)
E. None of the above

370 A feeble minded individual has an IQ of Idiot 0-20; Imbecile 21-40; Feeble minded MAIRRE JAMES BACK-UP
A. 0-20 41-70 GADDI, MD (TOP 4 - MIDTERM
B. 41-70 AUG 2013 MED EXAM - FEB
C. 71-90 BOARDS; TOPNOTCH 2015
D. 21-40 MD)
E. 31-50

371 Which of the following can be used to determine if an MAIRRE JAMES BACK-UP
infant was born alive or was still born? GADDI, MD (TOP 4 - MIDTERM
A. Fodere's test AUG 2013 MED EXAM - FEB
B. Werdin's test BOARDS; TOPNOTCH 2015
C. Breslau's test MD)
D. A and C
E. All of the above

372 Which of the following is/are TRUE regarding MAIRRE JAMES BACK-UP
infanticide? GADDI, MD (TOP 4 - MIDTERM
A. It involves the killing of a child less than 3 days old AUG 2013 MED EXAM - FEB
B. It is most often committed by the mother BOARDS; TOPNOTCH 2015
C. The crime can be committed through omission, MD)
neglect or comission
D. A and C
E. All of the above

373 The following is/are TRUE regarding death by The presence of washewoman's hands and MAIRRE JAMES BACK-UP
drowning feet is not diagnostic of drowning. It only GADDI, MD (TOP 4 - MIDTERM
A. The presence of washerwoman's hands and feet means that the body has remained in the AUG 2013 MED EXAM - FEB
proves that the cause of death is by drowning water for some time; The whitish foam that BOARDS; TOPNOTCH 2015
B. The whitish foam that accumulates in the mouth and accumulates in the mouth and nostrils is MD)
nostrils is called edema aquosum called champignon d'ocume; Tardieu spots
C. The average time required for death in drowning is are found in deaths caused by asphyxia
between 2-5 minutes through strangulation by ligature
D. Tardieu's spots can be found beneath the
conjunctiva, face and neck
E. All of the above are true

TOPNOTCH MEDICAL BOARD PREP LEGAL MEDICINE AND JURIS SUPEREXAM Page 45 of 86
For inquiries visit www.topnotchboardprep.com.ph or email us at topnotchmedicalboardprep@gmail.com
TOPNOTCH MEDICAL BOARD PREP LEGAL MEDICINE AND JURIS SUPEREXAM
For inquiries visit www.topnotchboardprep.com.ph or email us at topnotchmedicalboardprep@gmail.com
Item QUESTION EXPLANATION AUTHOR TOPNOTCH
# EXAM
374 A 50/M was brought to the ER with multiple stab MAIRRE JAMES BACK-UP
wounds in the abdomen. While attending to the GADDI, MD (TOP 4 - MIDTERM
injuries, the patient grabed the physician's coat AUG 2013 MED EXAM - FEB
signaling him to come closer. The patient then BOARDS; TOPNOTCH 2015
proceeded to narrate that the person responsible for MD)
his predicament was his neighbor. He then
subsequently expired. In order for his dying declaration
to admissible in court the which of the following must
be satisfied?
A. The declaration was made under consciousness of
impending death
B. The declaration made refers to the circumstances
surrounding the declarant's death
C. The patient dies thereafter
D. The declarant was a competent witness
E. All of the above

375 A woman who permits any form of sexual act so long as Virgo intacta - truly virtuous woman; Moral MAIRRE JAMES BACK-UP
the hymen remains unruptured is referred to as a? virginity - state of not knowing the nature of GADDI, MD (TOP 4 - MIDTERM
A. Virgo intacta sexual life and not having experienced sexual AUG 2013 MED EXAM - FEB
B. True virgin relations BOARDS; TOPNOTCH 2015
C. Morally virgin MD)
D. Demi virgin
E. None of the above

376 A 15/F was raped by his next door neighbor. She was Superficial - not more than half of the width MAIRRE JAMES BACK-UP
brought to you for medicolegal examination. Upon of the hymen; Complete - reaching the base; GADDI, MD (TOP 4 - MIDTERM
inspection of the vaginal introitus, you noted that the Compound/complicated - beyond the base AUG 2013 MED EXAM - FEB
laceration is more than half of the width of the hymen BOARDS; TOPNOTCH 2015
but not reaching the base. You classify this as? MD)
A. Superficial
B. Deep
C. Complete
D. Compound
E. Complicated

377 Which of the following is/are TRUE regarding rape? The anti rape law was amended by RA 8353 MAIRRE JAMES BACK-UP
A. Rape is committed when the woman is under 12 in 1997 and has reclassified rape as a crime GADDI, MD (TOP 4 - MIDTERM
years of age against persons and not just as a crime AUG 2013 MED EXAM - FEB
B. An impotent man can be charged with rape as long as against chastity BOARDS; TOPNOTCH 2015
there is carnal knowledge MD)
C. Force, threat or intimidation was used
D. It is classfied as a crime against persons
E. All of the above

378 A neurosurgeon came home early from work when he If a legally married person caught his partner MAIRRE JAMES BACK-UP
found his wife in bed with another man. In his rage he in the act of sexual intercourse with another GADDI, MD (TOP 4 - MIDTERM
took out his gun and shot them both resulting in their man and killed or inflicted serious injury to AUG 2013 MED EXAM - FEB
deaths. He shall suffer the penalty of? any or both of them shall suffer the penalty of BOARDS; TOPNOTCH 2015
A. Reclusion perpetua destierro MD)
B. Reclusion temporal
C. Destierro
D. Prison mayor
E. Death penalty

379 A 26/F recevied multiple bruises from his husband. On 2-4 days purple; 4-5 days green; 7-10 days MAIRRE JAMES BACK-UP
physical examination, you notice that the color of the yellow; 14-15 days gradually disappearing GADDI, MD (TOP 4 - MIDTERM
bruises are mostly purple. You estimate the age of the AUG 2013 MED EXAM - FEB
bruises to be? BOARDS; TOPNOTCH 2015
A. 2-4 days MD)
B. 4-5 days
C. 7-10 days
D. 2 weeks
E. 4 weeks

380 Accumulation of blood in the most dependent portions Livor mortis/Post mortem lividity/Cadaveric MAIRRE JAMES BACK-UP
of the body with the blood coagulated inside the blood lividity - Accumulation of blood in the most GADDI, MD (TOP 4 - MIDTERM
vessels. dependent portions of the body, two kinds: AUG 2013 MED EXAM - FEB
A. Marbolization hypostatic lividity - blood still in fluid form; BOARDS; TOPNOTCH 2015
B. Hypostatic lividity diffusion lividity - blood coagluated inside; MD)
C. Diffusion lividity marbolization - pprominence of the
D. Adipocere formation superficial veins during decomposition which
E. None of the above develops on both flanks of the abdomen, root
of the neck and shoulder
381 This test is used to determine whether semen is of SCOTT RILEY ONG, BACK-UP
human origin or not: MD (TOP 5 - AUG MIDTERM
A. Acid phosphatase 2014 MED BOARDS; EXAM - FEB
B. Berberio's TOPNOTCH MD) 2015
C. Florence
D. Precipitin

TOPNOTCH MEDICAL BOARD PREP LEGAL MEDICINE AND JURIS SUPEREXAM Page 46 of 86
For inquiries visit www.topnotchboardprep.com.ph or email us at topnotchmedicalboardprep@gmail.com
TOPNOTCH MEDICAL BOARD PREP LEGAL MEDICINE AND JURIS SUPEREXAM
For inquiries visit www.topnotchboardprep.com.ph or email us at topnotchmedicalboardprep@gmail.com
Item QUESTION EXPLANATION AUTHOR TOPNOTCH
# EXAM
382 Which one of the following is the chemical test used to SCOTT RILEY ONG, BACK-UP
determine if the stain is of blood origin or not? MD (TOP 5 - AUG MIDTERM
A. Marquis test 2014 MED BOARDS; EXAM - FEB
B. Van Urk test TOPNOTCH MD) 2015
C. Benzidine test
D. Zwikkers test

383 In the ordinary method of human identification SCOTT RILEY ONG, BACK-UP
applicable to the living person, the "facies" is a MD (TOP 5 - AUG MIDTERM
characteristic not easily changed. Which appearance of 2014 MED BOARDS; EXAM - FEB
the face is indicative of approaching death? TOPNOTCH MD) 2015
A. Leonine facies
B. Mongolian facies
C. Hippocratic facies
D. Myxedemic facies

384 A common law husband who kills his common law wife For him to be prosecuted for parricide, he SCOTT RILEY ONG, BACK-UP
with whom he has a 2-day old child can be prosecuted must be legally married to his wife ("legal MD (TOP 5 - AUG MIDTERM
for: spouse"). 2014 MED BOARDS; EXAM - FEB
A. Infanticide TOPNOTCH MD) 2015
B. Parricide
C. Homicide
D. Impossible crime

385 It is an expressed acknowledgement by the accused in a Admission: acknowledgement of facts but not SCOTT RILEY ONG, BACK-UP
criminal case of the truth of his guilt as the crime guilt MD (TOP 5 - AUG MIDTERM
charged. 2014 MED BOARDS; EXAM - FEB
A. Confession TOPNOTCH MD) 2015
B. Admission
C. Interrogation
D. All of the above

386 The presence of weapon which is highly grasped by the Cadaveric spasm is also known as SCOTT RILEY ONG, BACK-UP
hand of a victim of a shooting incident is an example of: instantaneous rigor. Cadaveric rigidity is also MD (TOP 5 - AUG MIDTERM
A. Rigor mortis known as rigor mortis 2014 MED BOARDS; EXAM - FEB
B. Death stiffening TOPNOTCH MD) 2015
C. Cadaveric spasm
D. Cadaveric rigidity

387 Permission to disinter remains of persons who died of 3 years if non-dangerous communicable SCOTT RILEY ONG, BACK-UP
dangerous communicable diseases may be granted disease MD (TOP 5 - AUG MIDTERM
after a burial period of: 2014 MED BOARDS; EXAM - FEB
A. 4 years TOPNOTCH MD) 2015
B. 5 years
C. 6 years
D. 7 years

388 Dr. B, the MHO of a municipality B, shall perform an Autopsy is done following the written order SCOTT RILEY ONG, BACK-UP
autopsy on a dead body when: of the following: court,fiscal, mayor, or police MD (TOP 5 - AUG MIDTERM
A. Requested by any relative of the victim authority. 2014 MED BOARDS; EXAM - FEB
B. Verbally directed by the police TOPNOTCH MD) 2015
C. Upon order by the mayor
D. Ordered by the Provincial Hospital Medical Director

389 It is the prominence of superficial vveins with reddish SCOTT RILEY ONG, BACK-UP
discoloration due to the process of decomposition MD (TOP 5 - AUG MIDTERM
which develops on both flanks of the abdomen: 2014 MED BOARDS; EXAM - FEB
A. Putrefaction TOPNOTCH MD) 2015
B. Marbolization
C. Adipose formation
D. Maceration

390 When a blow to the forehead cause contusion of the This came out in our Board exam like three SCOTT RILEY ONG, BACK-UP
eyeball due to fracture of the bone at the roof of the times… :p MD (TOP 5 - AUG MIDTERM
orbit, what is the type of injury incurred? 2014 MED BOARDS; EXAM - FEB
A. Coup TOPNOTCH MD) 2015
B. Contre-coup
C. Coup contre-coup
D. Locus minoris resistencia

391 The liability of a school teacher enticing a 16-year old Qualified seduction: virgin >12 and <18 years SCOTT RILEY ONG, BACK-UP
girl who is a virgin into having sexual intercourse with old; committed by a person in authority. MD (TOP 5 - AUG MIDTERM
him is: Simple seduction: single or widow >12 and 2014 MED BOARDS; EXAM - FEB
A. Statutory rape <19 years old; done by deceit. TOPNOTCH MD) 2015
B. Simple seduction
C. Qualified seduction
D. Acts of lasciviousness

TOPNOTCH MEDICAL BOARD PREP LEGAL MEDICINE AND JURIS SUPEREXAM Page 47 of 86
For inquiries visit www.topnotchboardprep.com.ph or email us at topnotchmedicalboardprep@gmail.com
TOPNOTCH MEDICAL BOARD PREP LEGAL MEDICINE AND JURIS SUPEREXAM
For inquiries visit www.topnotchboardprep.com.ph or email us at topnotchmedicalboardprep@gmail.com
Item QUESTION EXPLANATION AUTHOR TOPNOTCH
# EXAM
392 For a child to be legitimate, he must be born in lawful Legitimacy: after 180 days following the SCOTT RILEY ONG, BACK-UP
wedlock within these number of days after the celebration of marriage, or within 300 days MD (TOP 5 - AUG MIDTERM
dissolution of the preivous marriage: following the dissolution of marriage. 2014 MED BOARDS; EXAM - FEB
A. 120 days TOPNOTCH MD) 2015
B. 180 days
C. 300 days
D. 280 days

393 The children of a prostitute are called: SCOTT RILEY ONG, BACK-UP
A. Incestuous children MD (TOP 5 - AUG MIDTERM
B. Sacrilegious children 2014 MED BOARDS; EXAM - FEB
C. Immoral children TOPNOTCH MD) 2015
D. Manceras children

394 This prevents an individual from being admitted to the Minimum age requirement to practice SCOTT RILEY ONG, BACK-UP
practice of medicine medicine is 21 years. MD (TOP 5 - AUG MIDTERM
A. Age 19 years 2014 MED BOARDS; EXAM - FEB
B. Passed the board examination on his third retake TOPNOTCH MD) 2015
C. Holder of a valid Certificate of Registration issued by
the Board of Medicine
D. Must not have been convicted of any offense
involving moral turpitude

395 Which one of the following is measured by the space of SCOTT RILEY ONG, BACK-UP
time provided by the physician and not by the quality MD (TOP 5 - AUG MIDTERM
and quantity of medical services rendered? 2014 MED BOARDS; EXAM - FEB
A. Retainer fee TOPNOTCH MD) 2015
B. Package deal fee
C. Contractual fee
D. Simple fee

396 The hospital is almost full and the only vacant beds are SCOTT RILEY ONG, BACK-UP
in the ward. An infectious case was admitted in the MD (TOP 5 - AUG MIDTERM
ward resulting to contamination of other patients. The 2014 MED BOARDS; EXAM - FEB
physician who admitted the patient is liable under what TOPNOTCH MD) 2015
doctrine?
A. Foreseeability
B. Last clear chance
C. Continuing negligence
D. Contributory negligence

397 The Medical Records Department refused to disclose SCOTT RILEY ONG, BACK-UP
the clinical records of a patient. The patient may seek MD (TOP 5 - AUG MIDTERM
redress through: 2014 MED BOARDS; EXAM - FEB
A. Mandamus TOPNOTCH MD) 2015
B. Court injuction
C. Certoriari
D. TRO

398 Operating without consent constitute an assualt, and an SCOTT RILEY ONG, BACK-UP
assault, even without specific proof of injury resulting MD (TOP 5 - AUG MIDTERM
from it, entitles the plaintiff to this type of damages: 2014 MED BOARDS; EXAM - FEB
A. Liquidated damages TOPNOTCH MD) 2015
B. Nominal damages
C. Temperate damages
D. Moderate damages

399 Patient was aware that physician was drink yet he SCOTT RILEY ONG, BACK-UP
allowed the doctor to inject him. Which doctrine is MD (TOP 5 - AUG MIDTERM
applicable? 2014 MED BOARDS; EXAM - FEB
A. Continuing negligence TOPNOTCH MD) 2015
B. Vicarious liability
C. Res ipsa loquitor
D. Captain of the ship

400 Patient decided to leave the hospital against medical SCOTT RILEY ONG, BACK-UP
advice. Which doctrine is applicable? MD (TOP 5 - AUG MIDTERM
A. Continuing negligence 2014 MED BOARDS; EXAM - FEB
B. Res ipsa loquitor TOPNOTCH MD) 2015
C. Vicarious liability
D. Contributory negligence

401 Which of the following medical fees is considered JOSE CARLO DIAGNOSTIC
unethical? MASANGKAY III, MD EXAM - AUG
A. Professional fees (TOP 8 - FEB 2014 2014
B. Retainer's fees MED BOARDS;
C. Dichotomous fees TOPNOTCH MD)
D. Contractual fees
E. None of the above

TOPNOTCH MEDICAL BOARD PREP LEGAL MEDICINE AND JURIS SUPEREXAM Page 48 of 86
For inquiries visit www.topnotchboardprep.com.ph or email us at topnotchmedicalboardprep@gmail.com
TOPNOTCH MEDICAL BOARD PREP LEGAL MEDICINE AND JURIS SUPEREXAM
For inquiries visit www.topnotchboardprep.com.ph or email us at topnotchmedicalboardprep@gmail.com
Item QUESTION EXPLANATION AUTHOR TOPNOTCH
# EXAM
402 Dactylography is: SIMILAR TO PREVIOUS BOARD EXAM JOSE CARLO DIAGNOSTIC
A. The study of the pores on the papillary or friction CONCEPT/PRINCIPLE MASANGKAY III, MD EXAM - AUG
ridges of the skin for the purpose of identification (TOP 8 - FEB 2014 2014
B. Art and study of recording ingerprints as a means of MED BOARDS;
identification TOPNOTCH MD)
C. Art of changing fingerprints
D. Art of studying fingerprints in relation to the
bearer's characteristics
E. Art of comparing fingerprints for identification

403 Which of the following is the chemical test to determine JOSE CARLO DIAGNOSTIC
if the stain is of blood origin or not: MASANGKAY III, MD EXAM - AUG
A. Marquis Test (TOP 8 - FEB 2014 2014
B. Van Urk Test MED BOARDS;
C. Benzidine Test TOPNOTCH MD)
D. Zwikker's Test
E. Ilenburger's Test

404 Which of the following is a Schombein Test? SIMILAR TO PREVIOUS BOARD EXAM JOSE CARLO DIAGNOSTIC
A. Van Deen Test CONCEPT/PRINCIPLE MASANGKAY III, MD EXAM - AUG
B. Benzidine Test (TOP 8 - FEB 2014 2014
C. Teichmann's Test MED BOARDS;
D. Icard's Test TOPNOTCH MD)
E. Hemin Crystal Test

405 When the body surface of a corpse is pressed it leaves a JOSE CARLO DIAGNOSTIC
flatened area due to: MASANGKAY III, MD EXAM - AUG
A. Precipitated skin proteins (TOP 8 - FEB 2014 2014
B. Absence of blood MED BOARDS;
C. Absence of connective tissue TOPNOTCH MD)
D. Loss of elasticity of the skin
E. Absence of edema

406 Algor mortis refers to: SIMILAR TO PREVIOUS BOARD EXAM JOSE CARLO DIAGNOSTIC
A. Irritability CONCEPT/PRINCIPLE MASANGKAY III, MD EXAM - AUG
B. Cooling of the body (TOP 8 - FEB 2014 2014
C. Softening of the body MED BOARDS;
D. Hardening of the body TOPNOTCH MD)
E. Discoloration of the body

407 The body should be buried within how many hours if JOSE CARLO DIAGNOSTIC
death occurred due to a dangerous communicable MASANGKAY III, MD EXAM - AUG
disease? (TOP 8 - FEB 2014 2014
A. 8 hrs MED BOARDS;
B. 10 hrs TOPNOTCH MD)
C. 12 hrs
D. 24 hrs
E. 36 hrs

408 The basis to pronounce a person dead: JOSE CARLO DIAGNOSTIC


A. Molecular death MASANGKAY III, MD EXAM - AUG
B. Apparent death (TOP 8 - FEB 2014 2014
C. Cellular death MED BOARDS;
D. Organic death TOPNOTCH MD)
E. Somatic Death

409 Death by Carbon monoxide poisoning is: JOSE CARLO DIAGNOSTIC


A. Anoxic MASANGKAY III, MD EXAM - AUG
B. Histotoxic (TOP 8 - FEB 2014 2014
C. Anemic anoxic MED BOARDS;
D. Stagnant anoxic TOPNOTCH MD)
E. Pulmotoxic

410 Carnal knowledge means: SIMILAR TO PREVIOUS BOARD EXAM JOSE CARLO DIAGNOSTIC
A. Rupture of the hymen CONCEPT/PRINCIPLE MASANGKAY III, MD EXAM - AUG
B. Knowledge about the nature of sexual life (TOP 8 - FEB 2014 2014
C. Full penetration is necessary MED BOARDS;
D. Slightest penetration of the female sexual organ by TOPNOTCH MD)
the male sex organ
E. Craving for meat

TOPNOTCH MEDICAL BOARD PREP LEGAL MEDICINE AND JURIS SUPEREXAM Page 49 of 86
For inquiries visit www.topnotchboardprep.com.ph or email us at topnotchmedicalboardprep@gmail.com
TOPNOTCH MEDICAL BOARD PREP LEGAL MEDICINE AND JURIS SUPEREXAM
For inquiries visit www.topnotchboardprep.com.ph or email us at topnotchmedicalboardprep@gmail.com
Item QUESTION EXPLANATION AUTHOR TOPNOTCH
# EXAM
411 A female that permits all forms of sexual liberty except SIMILAR TO PREVIOUS BOARD EXAM JOSE CARLO DIAGNOSTIC
rupture of hymen: CONCEPT/PRINCIPLE MASANGKAY III, MD EXAM - AUG
A. Demi-virgin (TOP 8 - FEB 2014 2014
B. Morally virgin MED BOARDS;
C. Virgo intacta TOPNOTCH MD)
D. False physical virginity
E. Tru physical virginity

412 Disorientation, mental confusion, dizziness, sensory JOSE CARLO DIAGNOSTIC


disturbances exagerrated emotional state suggests that MASANGKAY III, MD EXAM - AUG
the blood alcohol level is (TOP 8 - FEB 2014 2014
A. 150-300 mg% MED BOARDS;
B. 300-500 mg% TOPNOTCH MD)
C. 100 - 150 mg%
D. 80-100 mg%
E. 60 - 80 mg%

413 A physician becomes what type of witness when he An expert witness states his opinion JOSE CARLO DIAGNOSTIC
states his physical findings and diagnosis in court: according to his profession or specialization. MASANGKAY III, MD EXAM - AUG
A. Expert witness (TOP 8 - FEB 2014 2014
B. Ordinary witness MED BOARDS;
C. Hostile witness TOPNOTCH MD)
D. Reluctant witness
E. Special Witness

414 The right to regulate the practice of medicine by the SIMILAR TO PREVIOUS BOARD EXAM JOSE CARLO DIAGNOSTIC
state is based on: CONCEPT/PRINCIPLE MASANGKAY III, MD EXAM - AUG
A. Police power (TOP 8 - FEB 2014 2014
B. Power of eminent domain MED BOARDS;
C. Medical Act of 1959 TOPNOTCH MD)
D. Constitution
E. Professional Regulation Commission

415 A patient who had an eclamptic seizure is in coma and JOSE CARLO DIAGNOSTIC
is being treated by a tea of physicians. What is the MASANGKAY III, MD EXAM - AUG
pattern of relationship between the physicians and the (TOP 8 - FEB 2014 2014
patient? MED BOARDS;
A. Active-Passive TOPNOTCH MD)
B. Mutual Participation
C. Guidance cooperation
D. Consensual
E. No relationship

416 A Civil case is to be filed by a patient against a doctor JOSE CARLO DIAGNOSTIC
for recovery of damages, what is the minimum evidence MASANGKAY III, MD EXAM - AUG
needed for conviction? (TOP 8 - FEB 2014 2014
A. Guilt beyond reasonable doubt MED BOARDS;
B. Preponderance of evidence TOPNOTCH MD)
C. Substantial Evidence
D. Gross negligence
E. Minimal reasonable evidence

417 An indigent patient is sufferring from Hepatic JOSE CARLO DIAGNOSTIC


Encephalopathy, but due to financial restraints his MASANGKAY III, MD EXAM - AUG
relatives requested to be discharged against medical (TOP 8 - FEB 2014 2014
advice. What Doctrine is applicable? MED BOARDS;
A. Doctrine of Continuing negligence TOPNOTCH MD)
B. Doctrine of Contributory negligence
C. Doctrine of Assumption of risk
D. Doctrine of last clear chance
E. Doctrine of Res Ipsa Liquitor

418 The type of damages given to "teach the public a JOSE CARLO DIAGNOSTIC
lesson": MASANGKAY III, MD EXAM - AUG
A. Moral (TOP 8 - FEB 2014 2014
B. Actual MED BOARDS;
C. Compensatory TOPNOTCH MD)
D. Exemplary
E. Liquidated

419 After an alleged fight of 2 women over their common even if the period of medical attendance JOSE CARLO DIAGNOSTIC
boyfriend, one of them sustained a deep laceration in required only a few days, if the injury MASANGKAY III, MD EXAM - AUG
the face causing a permanent scar, the patient is given a sustained caused a permanent severe (TOP 8 - FEB 2014 2014
7 day-medical attendance. What type of injury is this? deformity, or caused a psychological MED BOARDS;
A. Serious physical injuries disorder, the injury is considered to be TOPNOTCH MD)
B. Less serious physical injuries Serious.
C. Slight physical injuries
D. Mutilation
E. Defense Injuries

TOPNOTCH MEDICAL BOARD PREP LEGAL MEDICINE AND JURIS SUPEREXAM Page 50 of 86
For inquiries visit www.topnotchboardprep.com.ph or email us at topnotchmedicalboardprep@gmail.com
TOPNOTCH MEDICAL BOARD PREP LEGAL MEDICINE AND JURIS SUPEREXAM
For inquiries visit www.topnotchboardprep.com.ph or email us at topnotchmedicalboardprep@gmail.com
Item QUESTION EXPLANATION AUTHOR TOPNOTCH
# EXAM
420 Instances when the MD cannot recover PF EXCEPT: JOSE CARLO DIAGNOSTIC
A. When there is an agreement that the service is MASANGKAY III, MD EXAM - AUG
gratuitous (TOP 8 - FEB 2014 2014
B. Waiver on the part of the MD to the PF even if there MED BOARDS;
has been a previous agreement that the service is for a TOPNOTCH MD)
fee
C. Those not covered by the HMO
D. Medical services rendered by industrial or
company MD to their respective employees
E. MDs in charity hospitals, health centers, rural
health untis, asylums, tenements to indigent patients.
421 This law is not enforced by the State: *Remember that all subjects have just the WEBSTER ALINDOG, MIDTERM 1
A. Substantive law same bearing in your total score, so NEVER MD (TOP 3 - FEB EXAM - AUG
B. Remedial law take legal med for granted. Lectures by 2014 MED BOARDS; 2014
C. Public law lawyer-Drs. Rebosa and Rebosa have covered TOPNOTCH MD)
D. Moral law at least 70% of the questions in our batch. It
doesn't require much memorization, but
more of hardcore comprehension, hence one
of the easiest and most fun. Make sure you'd
get a high score in legal med. =)
422 Which of the following is not a pre-requisite to practice Remember that the State allows foreign WEBSTER ALINDOG, MIDTERM 1
medicine in the Philippines? doctors to practice medicine in the country MD (TOP 3 - FEB EXAM - AUG
A. Age of at least 21 years granted by reciprocity (with compliance to 2014 MED BOARDS; 2014
B. Natural born Filipino the requirements prescribed in Section 8 of TOPNOTCH MD)
C. Proper medical education the Medical Act) or endorsement.
D. Certificate of registration (PRC)

423 A patient was denied of access to his clinical records by Subpoena duces tecum requires the WEBSTER ALINDOG, MIDTERM 1
a hospital. To compel the hospital to do disclosure, the physician to produce documents under his MD (TOP 3 - FEB EXAM - AUG
patient may file for: possession relevant to the case. Whereas 2014 MED BOARDS; 2014
A. Subpoena duces tecum subpoena ad testificandum requires the TOPNOTCH MD)
B. Temporary restraining order physician to appear and testify before a court
C. Mandamus or in an investigation.
D. Court injunction

424 A physician feels that his new patient is stubborn, Option B is a relationship in which the WEBSTER ALINDOG, MIDTERM 1
uncooperative and difficult. On the other hand, the patient is unable to contribute to any related MD (TOP 3 - FEB EXAM - AUG
patient, who believes that he is juridically equal to his activity (say unconscious or in a vegetative 2014 MED BOARDS; 2014
doctor, sees his attending as unsympathetic and state). In oprtion C, the patient is conscious TOPNOTCH MD)
arrogant. The psychological pattern of doctor-patient and rational but seeks help and is purely
relationship in this case is: cooperative with the physician. He has
A. Mutual participation complete trust with the MD.
B. Activity-passivity
C. Guidance-cooperation
D. Negligent relation

425 What is the burden of proof needed to convict a *Must know! For a criminal case, it is guilt WEBSTER ALINDOG, MIDTERM 1
physician for a civil liability? beyond reasonable doubt; and for MD (TOP 3 - FEB EXAM - AUG
A. Guilt beyond reasonable doubt administrative, it is substantial evidence. 2014 MED BOARDS; 2014
B. Preponderance of evidence TOPNOTCH MD)
C. Substantial evidence
D. Testimonial evidence

426 A community doctor was caught in a crossfire between WEBSTER ALINDOG, MIDTERM 1
local authorities and an armed criminal group. In such MD (TOP 3 - FEB EXAM - AUG
circumstance, he may not respond to an emergency if: 2014 MED BOARDS; 2014
A. He is 10 meters away from the site of emergency. TOPNOTCH MD)
B. He has different political affiliation.
C. He is a government employee and therefore his
service is limited only to the military.
D. There is threat to his life.

427 During a surgical procedure, the assisting medical Under the captain of ship doctrine, the WEBSTER ALINDOG, MIDTERM 1
intern accidentally cut a major vessel instead of the primary surgeon is held accountable to any MD (TOP 3 - FEB EXAM - AUG
suture. Should the event resulted in an uncorrectable hapless or negligent event that happens 2014 MED BOARDS; 2014
complication, the surgeon may be held liable under the inside his OR. Ostensible agent principle is TOPNOTCH MD)
principle of: not applicable is neither an employee nor an
A. Ostensible agent independent contractor of the hospital, much
B. Borrowed servant in the same way with the borrowed servant
C. Captain of the ship doctrine. The bad result rule presumes that a
D. Bad result rule doctor is innocent of a negligent act until
proven otherwise because it is realized prior
to the event that a bad result "may actually
happen" and in such case res ipsa loquitor is
non-applicable.

TOPNOTCH MEDICAL BOARD PREP LEGAL MEDICINE AND JURIS SUPEREXAM Page 51 of 86
For inquiries visit www.topnotchboardprep.com.ph or email us at topnotchmedicalboardprep@gmail.com
TOPNOTCH MEDICAL BOARD PREP LEGAL MEDICINE AND JURIS SUPEREXAM
For inquiries visit www.topnotchboardprep.com.ph or email us at topnotchmedicalboardprep@gmail.com
Item QUESTION EXPLANATION AUTHOR TOPNOTCH
# EXAM
428 Egay, a well-trained and internationally acclaimed There are 2 fundamental elements to this WEBSTER ALINDOG, MIDTERM 1
beautician, treats acne and scars of his clients by case: 1. Egay is not a physician and is MD (TOP 3 - FEB EXAM - AUG
supplying them with an innovative oral and topical therefore not qualified to practice medicine; 2014 MED BOARDS; 2014
solutions. Which of the following is true about this and 2. Any act of seeing and treating - TOPNOTCH MD)
case? especially if unproven, innovative, not OTC -
A. Egay can be sued for malpractice. a human person for a disease condition
B. Egay should be commended for performing a new (including that of the skin) constitute
medical discovery. practice of medicine. Enlighted by these 2, we
C. Egay can be held liable for illegal practice of say that Egay can be charged of illegal
medicine. practice of medicine.
D. Egay's actions are legitimate and scientific.

429 Under this doctrine the hospital owner may be held WEBSTER ALINDOG, MIDTERM 1
legally responsible for the mistakes of its nursing staff: MD (TOP 3 - FEB EXAM - AUG
A. Fellow servant doctrine 2014 MED BOARDS; 2014
B. Borrowed servant doctrine TOPNOTCH MD)
C. Common knowledge doctrine
D. Vicarious liability doctrine

430 This refers to the post-mortem finding in which blood Post-mortem lividity/ suggilation/livor WEBSTER ALINDOG, MIDTERM 1
has gravitated to the most dependent parts of the body mortis happens when the capillaries of a MD (TOP 3 - FEB EXAM - AUG
but still inside the the blood vessels in its fluid form: dead person are distended with blood and 2014 MED BOARDS; 2014
A. Tardeau spots they coalesced with one another until the TOPNOTCH MD)
B. Hypostatic lividity whole dependent area becomes dull-red or
C. Diffusion lividity purplish. The other kind of lividity, the
D. Algor mortis diffusion type (option C) appears during the
later stage when the blood has already
coagulated inside the vessels.

431 Which of the following will differentiate cadaveric In contrast, rigor mortis starts to appear 3-6 WEBSTER ALINDOG, MIDTERM 1
spasm from rigor mortis? hours after death and involves all muscles of MD (TOP 3 - FEB EXAM - AUG
A. Cadaveric spasm happens immediately after death. the body. It is a natural phenomenon, 2014 MED BOARDS; 2014
B. It is a natural phenomenon. meaning it happens constantly in all dead TOPNOTCH MD)
C. It involves all muscles of the body. persons, and is utilized to approximate the
D. All of the above. time of death.

432 Which of these bones is most useful in determining the To determine the sex: pelvis, skull, sternum, WEBSTER ALINDOG, MIDTERM 1
sex of the deceased? femur, humerus. To determine the height: MD (TOP 3 - FEB EXAM - AUG
A. Femur femur, humerus, tibia, radius. 2014 MED BOARDS; 2014
B. Pelvis TOPNOTCH MD)
C. Sternum
D. Skull

433 Disinterment of remains of a person who died of non- If died of a communicable disease, WEBSTER ALINDOG, MIDTERM 1
communicable cause may be permitted only after a disinterment is permitted after 5 years. MD (TOP 3 - FEB EXAM - AUG
burial period of: 2014 MED BOARDS; 2014
A. 3 years TOPNOTCH MD)
B. 5 years
C. 7 years
D. 9 years

434 What is the estimated age of a contusion if it has *FAQ. Red to purple - soon after complete WEBSTER ALINDOG, MIDTERM 1
changed its color to green? development; green - 4-5 days; yellow - 7-10 MD (TOP 3 - FEB EXAM - AUG
A. 2-3 days days. 2014 MED BOARDS; 2014
B. Hours after injury TOPNOTCH MD)
C. 4-5 days
D. 7-10 days

435 All of the following findings are consistent with an exit Must know! In contrast, an entrance wound WEBSTER ALINDOG, MIDTERM 1
gunshot wound except: is smaller, its edges are inverted, it is MD (TOP 3 - FEB EXAM - AUG
A. Size is bigger than the caliber of the bullet commonly round or oval depending on the 2014 MED BOARDS; 2014
B. Edges of the wound are everted bullet trajectory, and contusion collar and TOPNOTCH MD)
C. It has irregular shape contact ring are present.
D. Presence of contusion collar

TOPNOTCH MEDICAL BOARD PREP LEGAL MEDICINE AND JURIS SUPEREXAM Page 52 of 86
For inquiries visit www.topnotchboardprep.com.ph or email us at topnotchmedicalboardprep@gmail.com
TOPNOTCH MEDICAL BOARD PREP LEGAL MEDICINE AND JURIS SUPEREXAM
For inquiries visit www.topnotchboardprep.com.ph or email us at topnotchmedicalboardprep@gmail.com
Item QUESTION EXPLANATION AUTHOR TOPNOTCH
# EXAM
436 A receptionist was given an extra fee by a physician Fee splitting is unethical, same with WEBSTER ALINDOG, MIDTERM 1
after the former endorsed a patient to his clinic. The contingent and straight or pakyaw fees. MD (TOP 3 - FEB EXAM - AUG
physician is guilty of committing: Simple contractual and retainer fees, on the 2014 MED BOARDS; 2014
A. Pakyaw system other hand, are ethical. TOPNOTCH MD)
B. Simple contractual payment
C. Fee splitting
D. Retainer

437 This refers to the prominence of superficial veins with WEBSTER ALINDOG, MIDTERM 1
reddish discoloration during decomposition which MD (TOP 3 - FEB EXAM - AUG
appears on both flanks of the abdomen, root of the neck 2014 MED BOARDS; 2014
and shoulders: TOPNOTCH MD)
A. Marbolization
B. Cadaveric lividity
C. Angiolysis
D. All of the above

438 A laceration that involves more than half of the width of Defloration means laceration of the hymen as WEBSTER ALINDOG, MIDTERM 1
the hymen but does not reach its base is called: a result of sexual intercourse. Superficial: MD (TOP 3 - FEB EXAM - AUG
A. Superficial does not go beyond 1/2 of the whole width of 2014 MED BOARDS; 2014
B. Deep the hymen; deep: if more than half but not TOPNOTCH MD)
C. Complete reaching the base; complete: if it reaches the
D. Compound base; and compound/complicated: if beyond
the base.

439 Carnal knowledge is: WEBSTER ALINDOG, MIDTERM 1


A. A required element in rape cases MD (TOP 3 - FEB EXAM - AUG
B. Refers to the slightest penetration in the sexual 2014 MED BOARDS; 2014
organ of a woman by the penis TOPNOTCH MD)
C. An act of man having a sexual bodily connection
with a woman
D. All of the above

440 A person having an IQ of 0 to 20 is considered to be WEBSTER ALINDOG, MIDTERM 1


a/an: MD (TOP 3 - FEB EXAM - AUG
A. Idiot 2014 MED BOARDS; 2014
B. Imbecile TOPNOTCH MD)
C. Feeble-minded
D. Stupid

441 It is the art and study of recording fingerprints as a Dactylography is the art and study of JULIET KRISTINE MIDTERM 2
means of identification: recording fingerprints as a means of EVANGELISTA, MD EXAM - AUG
A. Poroscopy identification. Poroscopy is the study of the (TOP 9 - FEB 2014 2014
B. Dactylography pores found on the papillary or friction MED BOARDS;
C. Dactyloscopy ridges of the skin for purposes of TOPNOTCH MD)
D. Graphology identification. Dactyloscopy is the art of
E. Endoscopy identification by comparison of fingerprints.

442 Barr bodies observed in scraping of the mucous Barr bodies observed in scraping of the JULIET KRISTINE MIDTERM 2
mebranes of this part of the body is used to determine mucous mebranes of the mouth is used to EVANGELISTA, MD EXAM - AUG
the sex of the body: determine the sex of the body. (TOP 9 - FEB 2014 2014
A. Ears MED BOARDS;
B. Nose TOPNOTCH MD)
C. Mouth
D. Vagina
E. Anal canal

443 This is a medical piece of evidence made known or Autoptic or Real Evidence is a medical piece JULIET KRISTINE MIDTERM 2
addressed to the senses of the court, not limited to the of evidence made known or addressed to the EVANGELISTA, MD EXAM - AUG
sense of vision, but is intended to the sense of hearing, senses of the court, not limited to the sense of (TOP 9 - FEB 2014 2014
taste, smell and touch: vision, but is intended to the sense of hearing, MED BOARDS;
A. Autoptic Evidence taste, smell and touch. TOPNOTCH MD)
B. Experimental Evidence
C. Documentary Evidence
D. Corpus Evidence
E. Physical Evidence

444 Mr. X was found dead in his room by his son with a Cadaveric spasm is the instantaneous rigidity JULIET KRISTINE MIDTERM 2
weapon highly grasped by his hand after a shooting of the muscles which occurs at the moment EVANGELISTA, MD EXAM - AUG
incident. This is an example of: od death due to extreme nervous tension, (TOP 9 - FEB 2014 2014
A. Rigor mortis exhaustion and injury to the nervous system MED BOARDS;
B. Death stiffening or injury to the chest. Stage of postmortem TOPNOTCH MD)
C. Cadaveric spasm rigidity, cadaveric rigidity, death stiffening or
D. Cadaveric rigidity rigor mortis occurs 3 to 6 hours after death
E. Postmortem rigidity when muscles gradually stiffen.

TOPNOTCH MEDICAL BOARD PREP LEGAL MEDICINE AND JURIS SUPEREXAM Page 53 of 86
For inquiries visit www.topnotchboardprep.com.ph or email us at topnotchmedicalboardprep@gmail.com
TOPNOTCH MEDICAL BOARD PREP LEGAL MEDICINE AND JURIS SUPEREXAM
For inquiries visit www.topnotchboardprep.com.ph or email us at topnotchmedicalboardprep@gmail.com
Item QUESTION EXPLANATION AUTHOR TOPNOTCH
# EXAM
445 The most prominent sign of death is: The progressive fall of the body temperature JULIET KRISTINE MIDTERM 2
A. Progressive fall of the body temperature is one of the most prominent signs of death EVANGELISTA, MD EXAM - AUG
B. Cessation of respiration due to cessation of the metabolic processes (TOP 9 - FEB 2014 2014
C. Cessation of heart action and circulation inside the body. MED BOARDS;
D. Insensibility of body loss of power to move TOPNOTCH MD)
E. Change in the color of the fingers

446 The drug used as a truth serum in detecting deception Truth serum is a procedure which does not JULIET KRISTINE MIDTERM 2
is: make someone tell the truth and the thing EVANGELISTA, MD EXAM - AUG
A. Meperidine administered is not a serum but is actually a (TOP 9 - FEB 2014 2014
B. Hyoscine hydrobromide drug. In the test, hyoscine hydrobromide is MED BOARDS;
C. Sodium amytal given hypodermically in repeated doses until TOPNOTCH MD)
D. Hydroxyzine a state of delirium is induced.
E. Sodium pentothal

447 Exhumation is also known as: The deceased buried may be raised or JULIET KRISTINE MIDTERM 2
A. Interment disinterred upon the lawful order of the EVANGELISTA, MD EXAM - AUG
B. Ground burial proper authorities. The body is exhumed for (TOP 9 - FEB 2014 2014
C. Embalming the purpose of performing postmortem MED BOARDS;
D. Cremation examination. TOPNOTCH MD)
E. Disinterment

448 It is a type of asphyxial death when oxygen is delivered Histotoxic anoxic death is due to failure of JULIET KRISTINE MIDTERM 2
to the tissues but cannot be utilized properly due to the cellular oxidative process, although the EVANGELISTA, MD EXAM - AUG
failure of cellular oxidative process is: oxygen is delivered to the tissues, it cannot (TOP 9 - FEB 2014 2014
A. Anoxic be utilized properly. MED BOARDS;
B. Histotoxic TOPNOTCH MD)
C. Anemic anoxic
D. Stagnant anoxic
E. Typical anoxic

449 A 50 year-old man was driving his motorcycle home Contusion is the effusion of blood into the JULIET KRISTINE MIDTERM 2
from a drinking session with his friend when he tissues underneath the skin on account of the EVANGELISTA, MD EXAM - AUG
suddenly crashed with a car. Physical injuries noted on rupture of the blood vessels as a result of the (TOP 9 - FEB 2014 2014
parts of the body which are deep-seated, caused by application of blunt force or violence. MED BOARDS;
forcible impact on hard blunt objects are: TOPNOTCH MD)
A. Abrasion
B. Hematoma
C. Contusion
D. Lacerated wound
E. Concussion

450 Mr. M killed their baby who is a day-old because he was Parricide is any person who shall kill his JULIET KRISTINE MIDTERM 2
suspecting that his wife was having an extramarital father, mother or child whether legitimate or EVANGELISTA, MD EXAM - AUG
affair with his coworker, Mr. B. The crime committed by illegitimate, ascendants or descendants. (TOP 9 - FEB 2014 2014
Mr. M is: Infanticide is imposed upon any person who MED BOARDS;
A. Murder shall kill any child less than three days of age. TOPNOTCH MD)
B. Parricide According to Dr. Rebosa, Parricide-due to
C. Homicide filiation.
D. Infanticide
E. Accidental

451 Dr. J was given subpoena from a court in Quezon City. Indirect contempt is the failure to appear to JULIET KRISTINE MIDTERM 2
He refused to attend on several occasions in a court court despite 3 subpoenas. Direct contempt EVANGELISTA, MD EXAM - AUG
hearing. His reason is that he has lots of patients to is failure to comply with a subpoena inside or (TOP 9 - FEB 2014 2014
attend to in his clinic in Quezon City. He may be cited near the courtroom. MED BOARDS;
for: TOPNOTCH MD)
A. Direct contempt of court
B. Indirect contempt of court
C. Disrespect of the court
D. Unduly prioritizing his patients
E. None. He has right to refuse

452 The reporting of less serious and serious physical The reporting of less serious and serious JULIET KRISTINE MIDTERM 2
injuries by a physician according to Presidential Decree physical injuries by a physician according to EVANGELISTA, MD EXAM - AUG
169 is: Presidential Decree 169 is mandatory. (TOP 9 - FEB 2014 2014
A. Mandatory MED BOARDS;
B. Voluntary TOPNOTCH MD)
C. Every end of the month
D. Every quarter
E. Case to case

453 The right to regulate the practice of medicine by the The right to regulate the practice of medicine JULIET KRISTINE MIDTERM 2
State is based on: by the State is based on Police Power. EVANGELISTA, MD EXAM - AUG
A. Power of eminent domain (TOP 9 - FEB 2014 2014
B. Medical Act of 1959 MED BOARDS;
C. Constitution TOPNOTCH MD)
D. PD 169
E. Police Power

TOPNOTCH MEDICAL BOARD PREP LEGAL MEDICINE AND JURIS SUPEREXAM Page 54 of 86
For inquiries visit www.topnotchboardprep.com.ph or email us at topnotchmedicalboardprep@gmail.com
TOPNOTCH MEDICAL BOARD PREP LEGAL MEDICINE AND JURIS SUPEREXAM
For inquiries visit www.topnotchboardprep.com.ph or email us at topnotchmedicalboardprep@gmail.com
Item QUESTION EXPLANATION AUTHOR TOPNOTCH
# EXAM
454 The hospital owner may be liable for the mistakes of The hospital owner (employer) may be liable JULIET KRISTINE MIDTERM 2
the resident physicians is under the doctrine of: for the mistakes of the resident physicians EVANGELISTA, MD EXAM - AUG
A. Captain of the ship (employee) is under the doctrine of vicarious (TOP 9 - FEB 2014 2014
B. Borrowed servant doctrine liability. MED BOARDS;
C. Fellow servant doctrine TOPNOTCH MD)
D. Vicarious liability
E. Doctrine of ostensible agent

455 Damages imposed to Dr. K as punishment and to serve Exemplary or corrective damages are JULIET KRISTINE MIDTERM 2
as an example or correction for the medical profession: monetary compensation awarded as EVANGELISTA, MD EXAM - AUG
A. Nominal damages punishment or deterrence, because of (TOP 9 - FEB 2014 2014
B. Compensatory damages wanton, reckless, malicious or oppressive MED BOARDS;
C. Exemplary damages nature of the wrong committed. It is imposed TOPNOTCH MD)
D. Temperate damages by way of example or correction for the
E. Moral damages public good.

456 It is the direct physical connection between the Doctrine of proximate cause is the direct JULIET KRISTINE MIDTERM 2
wrongful act of the physician and the injury suffered by physical connection between the wrongful EVANGELISTA, MD EXAM - AUG
the patient: act of the physician and the injury suffered by (TOP 9 - FEB 2014 2014
A. Sine Qua now the patient. MED BOARDS;
B. Doctrine of proximate cause TOPNOTCH MD)
C. Substantial factor test
D. Doctrine of forseeability
E. Dura Lox and Lex

457 Creditors of a doctor may ask the court to levy the Exemptions include: medical instruments JULIET KRISTINE MIDTERM 2
latter's property except: and medical books not pre-medical books. EVANGELISTA, MD EXAM - AUG
A. His resthouse in Baguio (TOP 9 - FEB 2014 2014
B. His Macbook Pro MED BOARDS;
C. His premedical books amounting to 1M TOPNOTCH MD)
D. His medical equipment worth 500,000
E. None of the above

458 Dr. Y left a forceps in the abdominal cavity after an Doctrine of res ipsa loquitur is also known as JULIET KRISTINE MIDTERM 2
explore lap to a patient suffering from stab wound. The common knowledge doctrine. The nature of EVANGELISTA, MD EXAM - AUG
applicable legal principle in this case is: the wrongful act is suggestive of negligence (TOP 9 - FEB 2014 2014
A. Doctrine of vicarious liability such as some foreign bodies that are left MED BOARDS;
B. Doctrine of contributory negligence unintentionally during closing of an incision. TOPNOTCH MD)
C. Doctrine of res ipsa loquitur
D. Doctrine of forseeability
E. Doctrine of Continuing negligence

459 Mr. Sy is a patient who promised to pay Dr. G two Contingent fees depends on the success or JULIET KRISTINE MIDTERM 2
million pesos if he is cured for his cancer. The medical failure of the treatment insituted. It is EVANGELISTA, MD EXAM - AUG
fee to be collected which is considered unethical is: unethical in the practice of medicine. (TOP 9 - FEB 2014 2014
A. Contractual fees MED BOARDS;
B. Contingent fees TOPNOTCH MD)
C. Retainer's fees
D. Professional fees
E. None of the above

460 Dr. T was sanctioned by the Board of Medicine due to Administrative liability is given to physicians JULIET KRISTINE MIDTERM 2
his unprofessional conduct. He is guilty of what by the Board of Medicine on the grounds of EVANGELISTA, MD EXAM - AUG
liability? unprofessional, unethical conduct, (TOP 9 - FEB 2014 2014
A. Civil liability immorality, gross negligence, ignorance, MED BOARDS;
B. Administrative liability incompetence, etc. TOPNOTCH MD)
C. Criminal liability
D. No liability
E. All of the above

461 The hospital cannot practice medicine because of the all of the choices are characteristics of LUISA SARANILLO, BACK-UP
following except: hospital which cannot practice medicine MD (TOP 6 - FEB MIDTERM
A. Cannot be subjected to licensure exam except it is a natural person. Hospital is a 2014 MED BOARDS; EXAM AUG
B. Cannot be regulated juridical person TOPNOTCH MD) 2014 - FOR
C. It is a natural person INCLUSION IN
D. It is created by law and attributes are provided by THE SAMPLEX
the law
E. No exception

462 When the secretary of the doctor is in the lobby of the dichotomous fee or commision or fee LUISA SARANILLO, BACK-UP
hospital calling for more patients to come in the clinic, splitting is the sharing of fee not based on the MD (TOP 6 - FEB MIDTERM
and in return the physician give her a share of what was services performed. It requires services of a 2014 MED BOARDS; EXAM AUG
earned. It is called: person who may act as an agent to solicit TOPNOTCH MD) 2014 - FOR
A. Retainer fee patients. INCLUSION IN
B. Straight fee THE SAMPLEX
C. Simple contractual fee
D. contingent fee
E. dichotomous fee

TOPNOTCH MEDICAL BOARD PREP LEGAL MEDICINE AND JURIS SUPEREXAM Page 55 of 86
For inquiries visit www.topnotchboardprep.com.ph or email us at topnotchmedicalboardprep@gmail.com
TOPNOTCH MEDICAL BOARD PREP LEGAL MEDICINE AND JURIS SUPEREXAM
For inquiries visit www.topnotchboardprep.com.ph or email us at topnotchmedicalboardprep@gmail.com
Item QUESTION EXPLANATION AUTHOR TOPNOTCH
# EXAM
463 A 45 year old male, married was brought to the ER consent must be obtained from (in order): LUISA SARANILLO, BACK-UP
unconscious, if the following persons are present, who patient > legal spouse > descendants / MD (TOP 6 - FEB MIDTERM
will give the consent for any procedures performed to children > ascendants/ parents > collaterals/ 2014 MED BOARDS; EXAM AUG
him? siblings > nearest relative > legal guardian > TOPNOTCH MD) 2014 - FOR
A. Mother attending doctor INCLUSION IN
B. 22 year old son THE SAMPLEX
C. brother
D. mistress
E. Uncle

464 What is the evidence needed to in a criminal case? guilt beyond reasonable doubt is needed for LUISA SARANILLO, BACK-UP
A. Preponderance of evidence conviction in a criminal case; in a civil case - MD (TOP 6 - FEB MIDTERM
B. Substantial evidence it is preponderance of evidence; in 2014 MED BOARDS; EXAM AUG
C. A and B Administrative case - it is substantial TOPNOTCH MD) 2014 - FOR
D. Guilt beyond reasonable doubt evidence. INCLUSION IN
E. All of the above THE SAMPLEX

465 The following are the inherent rights of a physician right to compensation is an incidental right. LUISA SARANILLO, BACK-UP
except: The rest of the choices are the inherent right MD (TOP 6 - FEB MIDTERM
A. compensation of a physician 2014 MED BOARDS; EXAM AUG
B. Choose patient TOPNOTCH MD) 2014 - FOR
C. Limit the practice INCLUSION IN
D. Determine the appropriate management THE SAMPLEX
E. Avail of hospital services

466 In determining the sex of the skeleton, the following are all of the choices are used in determining the LUISA SARANILLO, BACK-UP
used except: sex of the skeleton MD (TOP 6 - FEB MIDTERM
A. femur 2014 MED BOARDS; EXAM AUG
B. humerus TOPNOTCH MD) 2014 - FOR
C. pelvis INCLUSION IN
D. skull THE SAMPLEX
E. No exception

467 Schombein's test is also known as: Schombein's test or vandeen dyes. LUISA SARANILLO, BACK-UP
A. Benzidine test Teichmann's or hemin crystal. MD (TOP 6 - FEB MIDTERM
B. Vandeens dye 2014 MED BOARDS; EXAM AUG
C. Hemin crystal TOPNOTCH MD) 2014 - FOR
D. Teichmann's test INCLUSION IN
E. Icard's test THE SAMPLEX

468 It is an expressed acknowledgement by the accused in a confession is the acknowledgement of guilt, LUISA SARANILLO, BACK-UP
criminal case of the truth of his guilt as the crime while admission is acknowledgement of fault MD (TOP 6 - FEB MIDTERM
charged. but not guilt. 2014 MED BOARDS; EXAM AUG
A. Interrogation TOPNOTCH MD) 2014 - FOR
B. admission INCLUSION IN
C. confession THE SAMPLEX
D. declaration
E. None of the choices

469 What is the most prominent sign of death? LUISA SARANILLO, BACK-UP
A. Algor mortis MD (TOP 6 - FEB MIDTERM
B. Rigor mortis 2014 MED BOARDS; EXAM AUG
C. Livor mortis TOPNOTCH MD) 2014 - FOR
D. Muscle rigidity INCLUSION IN
E. Instantaneous rigor THE SAMPLEX

470 One of the following death is not autopsied. LUISA SARANILLO, BACK-UP
A. suicide MD (TOP 6 - FEB MIDTERM
B. Accidental death 2014 MED BOARDS; EXAM AUG
C. Death unattended by the physician TOPNOTCH MD) 2014 - FOR
D. Death occuring in a natural manner INCLUSION IN
E. None of the choices THE SAMPLEX

471 A person died of rabies infection should be burried not if the cause of death is communicable, it LUISA SARANILLO, BACK-UP
later than: should be burried not later than 12 hrs. MD (TOP 6 - FEB MIDTERM
A. 72 hrs 2014 MED BOARDS; EXAM AUG
B. 48 hrs TOPNOTCH MD) 2014 - FOR
C. 24 hrs INCLUSION IN
D. 12 hrs THE SAMPLEX
E. None of the choices

472 In contrast to a post-mortem clot, the ante-mortem clot postmortem clot is soft, while ante-mortem LUISA SARANILLO, BACK-UP
is: clot is firm MD (TOP 6 - FEB MIDTERM
A. soft 2014 MED BOARDS; EXAM AUG
B. firm TOPNOTCH MD) 2014 - FOR
C. homogenous INCLUSION IN
D. uniform THE SAMPLEX
E. None of the choices

TOPNOTCH MEDICAL BOARD PREP LEGAL MEDICINE AND JURIS SUPEREXAM Page 56 of 86
For inquiries visit www.topnotchboardprep.com.ph or email us at topnotchmedicalboardprep@gmail.com
TOPNOTCH MEDICAL BOARD PREP LEGAL MEDICINE AND JURIS SUPEREXAM
For inquiries visit www.topnotchboardprep.com.ph or email us at topnotchmedicalboardprep@gmail.com
Item QUESTION EXPLANATION AUTHOR TOPNOTCH
# EXAM
473 tardieu spots are associated with: LUISA SARANILLO, BACK-UP
A. hanging MD (TOP 6 - FEB MIDTERM
B. choking 2014 MED BOARDS; EXAM AUG
C. Manual strangulation TOPNOTCH MD) 2014 - FOR
D. Ligature strangulation INCLUSION IN
E. All of the choices THE SAMPLEX

474 After drowning, how soon does the dead body float? LUISA SARANILLO, BACK-UP
A. After 2 days MD (TOP 6 - FEB MIDTERM
B. After 4 days 2014 MED BOARDS; EXAM AUG
C. After 3 days TOPNOTCH MD) 2014 - FOR
D. Within 1 day INCLUSION IN
E. After 1 week THE SAMPLEX

475 Pugilistic position of a boxer ia associated with: LUISA SARANILLO, BACK-UP


A. drowning MD (TOP 6 - FEB MIDTERM
B. burns 2014 MED BOARDS; EXAM AUG
C. asphyxia TOPNOTCH MD) 2014 - FOR
D. Gunshot wound INCLUSION IN
E. Suicide THE SAMPLEX

476 When the color of the contusion changes to green, the LUISA SARANILLO, BACK-UP
estimated age of contusion is: MD (TOP 6 - FEB MIDTERM
A. 4-5 days 2014 MED BOARDS; EXAM AUG
B. 2-3 days TOPNOTCH MD) 2014 - FOR
C. 7-10days INCLUSION IN
D. 14 days THE SAMPLEX
E. 21 days

477 What is the IQ of a moron? 0-20=idiot; 20-40 = imbecile; 40-70= moron; LUISA SARANILLO, BACK-UP
A. 0-20 70-80= borderline deficiency; 100=normal MD (TOP 6 - FEB MIDTERM
B. 20-40 average 2014 MED BOARDS; EXAM AUG
C. 40-70 TOPNOTCH MD) 2014 - FOR
D. 70-80 INCLUSION IN
E. More than 100 THE SAMPLEX

478 An erroneous perception wtihout an external object of LUISA SARANILLO, BACK-UP


stimulus is called: MD (TOP 6 - FEB MIDTERM
A. daydreaming 2014 MED BOARDS; EXAM AUG
B. hallucination TOPNOTCH MD) 2014 - FOR
C. illusion INCLUSION IN
D. fantasy THE SAMPLEX
E. Delusion

479 Under the Dangerous drug act, marijuana is classified LUISA SARANILLO, BACK-UP
as: MD (TOP 6 - FEB MIDTERM
A. sedative 2014 MED BOARDS; EXAM AUG
B. hypnotic TOPNOTCH MD) 2014 - FOR
C. prohibited INCLUSION IN
D. regulated THE SAMPLEX
E. Stimulant

480 A surgeon is called to testify in the court, and is asked LUISA SARANILLO, BACK-UP
about his opinion on the surgical procedure performed MD (TOP 6 - FEB MIDTERM
on the patient he is not attending. He is called as: 2014 MED BOARDS; EXAM AUG
A. Expert witness TOPNOTCH MD) 2014 - FOR
B. Ordinary witness INCLUSION IN
C. Defense witness THE SAMPLEX
D. Hostile witness
E. percipient witness

481 This chemical test is used to determine if the stain is of In contrast, precipitin test is a test used to ANGELIS ANDREA FINAL EXAM -
blood origin or not: determine whether the blood is of human COCOS, MD (TOP 1 - AUG 2014
A. Benzidine test origin or not. Be sure to be familiar with the FEB 2014 MED
B. Marquis test other names of chemical tests and their uses. BOARDS; TOPNOTCH
C. Van Urk test SIMILAR TO PREVIOUS BOARD EXAM MD)
D. Precipitin test CONCEPT/PRINCIPLE.

482 Dactyloscopy is: Choice A is the definition of dactylography ANGELIS ANDREA FINAL EXAM -
A. The art and study of recording fingerprints as a (keyword: recording of fingerprints) while D COCOS, MD (TOP 1 - AUG 2014
means of identification is the definition of poroscopy. FEB 2014 MED
B. The art of changing fingerprints BOARDS; TOPNOTCH
C. The art of identification of fingerprints by MD)
comparison
D. The study of pores on the papillary or friction
ridges of the skin for the purpose of identification

TOPNOTCH MEDICAL BOARD PREP LEGAL MEDICINE AND JURIS SUPEREXAM Page 57 of 86
For inquiries visit www.topnotchboardprep.com.ph or email us at topnotchmedicalboardprep@gmail.com
TOPNOTCH MEDICAL BOARD PREP LEGAL MEDICINE AND JURIS SUPEREXAM
For inquiries visit www.topnotchboardprep.com.ph or email us at topnotchmedicalboardprep@gmail.com
Item QUESTION EXPLANATION AUTHOR TOPNOTCH
# EXAM
483 Maimai was stabbed at the abdomen. Days after she SIMILAR TO PREVIOUS BOARD EXAM ANGELIS ANDREA FINAL EXAM -
developed peritonitis was admitted at a hospital and CONCEPT/PRINCIPLE COCOS, MD (TOP 1 - AUG 2014
died of pneumonia. What is the immediate cause of FEB 2014 MED
death? BOARDS; TOPNOTCH
A. Stab wound MD)
B. peritonitis
C. pneumonia
D. hospital

484 Algor mortis refers to: ANGELIS ANDREA FINAL EXAM -


A. Cooling of the body COCOS, MD (TOP 1 - AUG 2014
B. Softening of the body FEB 2014 MED
C. Hardening of the body BOARDS; TOPNOTCH
D. Foul smell of the body MD)

485 When the color of the contusion changes to green, the The usual evoluation of a contusion: 2-4 ANGELIS ANDREA FINAL EXAM -
contusion's estimated age is: days: red, 4-5 days: green, 7-10 days: yellow, COCOS, MD (TOP 1 - AUG 2014
A. 2-4 days 14-15 days: resolution of contusion FEB 2014 MED
B. 4-5 days BOARDS; TOPNOTCH
C. 7-10 days MD)
D. 14-15 days

486 The primer is usually made up of: ANGELIS ANDREA FINAL EXAM -
A. Lead and antimony COCOS, MD (TOP 1 - AUG 2014
B. Lead, barium and copper FEB 2014 MED
C. Lead and copper BOARDS; TOPNOTCH
D. Lead, barium and antimony MD)

487 Jc was admitted for 3 days and sustained a scar on the A permanent damage is considered under ANGELIS ANDREA FINAL EXAM -
face after being physically assaulted. The offender is serious physical injury. COCOS, MD (TOP 1 - AUG 2014
liable for: FEB 2014 MED
A. mutilation BOARDS; TOPNOTCH
B. Slight physical injury MD)
C. Less serious physical injury
D. Serious physical injury

488 The laceration involving more than half of the hymenal Superficial is less than half, deep is more than ANGELIS ANDREA FINAL EXAM -
height but does not reach the hymenal base is classified half, complete would include the base and COCOS, MD (TOP 1 - AUG 2014
as: complicated would involve the surrounding FEB 2014 MED
A. superficial structures. Two questions were asked using BOARDS; TOPNOTCH
B. deep this concept alone in our board exam. MD)
C. complete
D. complicated

489 Nikki is married to Edward, however, she had a one- Both the wife and the man he slept with, who ANGELIS ANDREA FINAL EXAM -
night stand with Jobee who knew her to be married. new her to be married, can be held liable for COCOS, MD (TOP 1 - AUG 2014
Jobee can be held liable for: adultery. FEB 2014 MED
A. infidelity BOARDS; TOPNOTCH
B. concubinage MD)
C. adultery
D. marital impropriety

490 Which of the following marriages shall be considered ANGELIS ANDREA FINAL EXAM -
void from the beginning? COCOS, MD (TOP 1 - AUG 2014
A. Between the adopting father or mother and the FEB 2014 MED
adopted BOARDS; TOPNOTCH
B. Between the legitimate children of the adopter and MD)
the adopted
C. Between the stepfather and the stepdaughter
D. All of the above

491 What should be Ryan's IQ level if he is to be exempted An idiot (0-20) is exempted from criminal ANGELIS ANDREA FINAL EXAM -
from criminal liability. liability. An imbecile has an IQ of 20-40 while COCOS, MD (TOP 1 - AUG 2014
A. 0-20 feeble-minded/moron has 40-70 IQ level. FEB 2014 MED
B. 20-40 BOARDS; TOPNOTCH
C. 40-70 MD)
D. 70-80

492 Wood alcohol is otherwise known as: ANGELIS ANDREA FINAL EXAM -
A. Methyl alcohol COCOS, MD (TOP 1 - AUG 2014
B. Ethylene glycol FEB 2014 MED
C. Isopropyl alcohol BOARDS; TOPNOTCH
D. Ethyl alcohol MD)

493 This is composed of laws which are produced by the ANGELIS ANDREA FINAL EXAM -
country's legislation and which are defined, confided COCOS, MD (TOP 1 - AUG 2014
and incorporated by the law-making body: FEB 2014 MED
A. Statutory law BOARDS; TOPNOTCH
B. Common law MD)
C. Customary law
D. Remedial law

TOPNOTCH MEDICAL BOARD PREP LEGAL MEDICINE AND JURIS SUPEREXAM Page 58 of 86
For inquiries visit www.topnotchboardprep.com.ph or email us at topnotchmedicalboardprep@gmail.com
TOPNOTCH MEDICAL BOARD PREP LEGAL MEDICINE AND JURIS SUPEREXAM
For inquiries visit www.topnotchboardprep.com.ph or email us at topnotchmedicalboardprep@gmail.com
Item QUESTION EXPLANATION AUTHOR TOPNOTCH
# EXAM
494 Physical evidence recovered from the crime scene such ANGELIS ANDREA FINAL EXAM -
as a gun with fingerprints of the suspect is called: COCOS, MD (TOP 1 - AUG 2014
A. Circumstantial evidence FEB 2014 MED
B. Associative evidence BOARDS; TOPNOTCH
C. Tracing evidence MD)
D. Corpus delicti

495 Web appeared in the court hearing wearing shorts and ANGELIS ANDREA FINAL EXAM -
slippers. He can be cited for: COCOS, MD (TOP 1 - AUG 2014
A. Direct contempt of court FEB 2014 MED
B. Indirect contempt of court BOARDS; TOPNOTCH
C. Disrespect of court MD)
D. Nothing

496 Dr. Jinette, a moonlighter, was called in to testify what An ordinary witness tells the court his/her ANGELIS ANDREA FINAL EXAM -
happened during the night of the incident when the own perceptions or his/her own testimony. COCOS, MD (TOP 1 - AUG 2014
victim was admitted at the emergency room. She is An expert witness has special knowledge or FEB 2014 MED
giving a testimony as a/an: skill gained by education, or training and BOARDS; TOPNOTCH
A. Expert witness gives explanations to enlighten the court. MD)
B. Ordinary witness What happened during the night of the
C. Defense witness incident is something that the doctor has
D. Expert and Ordinary witness perceived.

497 Dax had his PRC license suspended indefinitely. After ANGELIS ANDREA FINAL EXAM -
how long can he apply for reinstatement? COCOS, MD (TOP 1 - AUG 2014
A. 2 years FEB 2014 MED
B. 3 years BOARDS; TOPNOTCH
C. 4 years MD)
D. 5 years

498 Jack, a GP, referred a patient to Ray, a specialist. After SIMILAR TO PREVIOUS BOARD EXAM ANGELIS ANDREA FINAL EXAM -
treatment, the patient paid Ray who in turn gave a CONCEPT/PRINCIPLE. COCOS, MD (TOP 1 - AUG 2014
share to Jack. What is this called? FEB 2014 MED
A. Contingent fee BOARDS; TOPNOTCH
B. Retainer's fee MD)
C. Simple contractual fee
D. Dichotomous fee

499 The burden of evidence needed to convict a physician SIMILAR TO PREVIOUS BOARD EXAM ANGELIS ANDREA FINAL EXAM -
for criminal malpractice is: CONCEPT/PRINCIPLE. COCOS, MD (TOP 1 - AUG 2014
A. Substantial evidence FEB 2014 MED
B. Preponderance of evidence BOARDS; TOPNOTCH
C. Proof beyond reasonable doubt MD)
D. Circumstantial evidence

500 The hospital is full and the only vacant beds are in the The physician should know better--this is the ANGELIS ANDREA FINAL EXAM -
ward. An infectious case was admitted in the ward main thought under the doctrine of COCOS, MD (TOP 1 - AUG 2014
resulting to contamination of other patients. The foreseeability. Make sure to be familiar with FEB 2014 MED
physician who admitted the patient is liable under what all the doctrines. BOARDS; TOPNOTCH
doctrine? MD)
A. Foreseeability
B. Last clear chance
C. Continuing negligence
D. Contributory negligence

501 Doctor X performed cesarian section on patient Eli JAN CHARMAINE BACK-UP
when her pregnancy is normal and uncomplicated. This PALOMAR, MD (TOP MIDTERM
is referred to as: 9 - FEB 2014 MED EXAM AUG
A. Over treatment BOARDS; TOPNOTCH 2014
B. Alternative treatment MD)
C. Ideal treatment
D. Preventive measure
E. Active management

502 The following facts should warrant suspicion of a case JAN CHARMAINE BACK-UP
of child abuse, except: PALOMAR, MD (TOP MIDTERM
A. Parents delay in seeking medical care. 9 - FEB 2014 MED EXAM AUG
B. Child frequently misbehaves and cries often BOARDS; TOPNOTCH 2014
C. History given by parents are vague and not MD)
consistent
D. Physical examination reveals bruises, fractures,
lacerations with different degrees of healing
E. None of the above

TOPNOTCH MEDICAL BOARD PREP LEGAL MEDICINE AND JURIS SUPEREXAM Page 59 of 86
For inquiries visit www.topnotchboardprep.com.ph or email us at topnotchmedicalboardprep@gmail.com
TOPNOTCH MEDICAL BOARD PREP LEGAL MEDICINE AND JURIS SUPEREXAM
For inquiries visit www.topnotchboardprep.com.ph or email us at topnotchmedicalboardprep@gmail.com
Item QUESTION EXPLANATION AUTHOR TOPNOTCH
# EXAM
503 Reporting the examination and treatment of serious JAN CHARMAINE BACK-UP
physical injuries by a physician is: PALOMAR, MD (TOP MIDTERM
A. Mandatory 9 - FEB 2014 MED EXAM AUG
B. Optional BOARDS; TOPNOTCH 2014
C. Case to case basis MD)
D. Once every year
E. Once every quarter

504 There is negligence in which of the following acts: JAN CHARMAINE BACK-UP
A. Not giving antibiotics in a patient with a few hours PALOMAR, MD (TOP MIDTERM
fever with no other signs and symptoms 9 - FEB 2014 MED EXAM AUG
B. Not doing a skin test before administering IV BOARDS; TOPNOTCH 2014
oxytocin MD)
C. Not giving tetanus toxoid to a patient who stepped
on a nail
D. Giving of prescriptions using the generic name of
the drugs only
E. None of the above
505 Post-operative abdominal x-ray revealed a mosquito JAN CHARMAINE BACK-UP
forceps in the film. Which doctrine is applicable? PALOMAR, MD (TOP MIDTERM
A. Doctrine of vicarious liability 9 - FEB 2014 MED EXAM AUG
B. Doctrine of continuing negligence BOARDS; TOPNOTCH 2014
C. Doctrine of contributory negligence MD)
D. Doctrine of res ipsa loquitur
E. Doctrine of direct negligence

506 Luisa was refused to see her in-hospital records. She JAN CHARMAINE BACK-UP
may request access to her file by filing a petition in the PALOMAR, MD (TOP MIDTERM
court for: 9 - FEB 2014 MED EXAM AUG
A. Mandamus BOARDS; TOPNOTCH 2014
B. Temporary restraining order MD)
C. Presidential decree
D. Right to medical records by patient
E. Injunction

507 All of the following are considered hospital employees, JAN CHARMAINE BACK-UP
except: PALOMAR, MD (TOP MIDTERM
A. House anesthesiologist 9 - FEB 2014 MED EXAM AUG
B. Resident physicians BOARDS; TOPNOTCH 2014
C. Staff nurse MD)
D. Orderlies
E. Junior interns

508 Which of the following medical fees are considered JAN CHARMAINE BACK-UP
unethical? PALOMAR, MD (TOP MIDTERM
A. Dichotomous fees 9 - FEB 2014 MED EXAM AUG
B. Contractual fees BOARDS; TOPNOTCH 2014
C. Professional fees MD)
D. Retainer's fees
E. None of the above

509 Which of the following adverstisements is unethical? JAN CHARMAINE BACK-UP


A. Obstetrician and Gynecologist PALOMAR, MD (TOP MIDTERM
B. Consultation hours: 10am to 4pm 9 - FEB 2014 MED EXAM AUG
C. Infertility Specialist BOARDS; TOPNOTCH 2014
D. Trained at John Hopkins Hospital MD)
E. None of the above

510 Doctor Y gave an overdose of a drug to a patient causing JAN CHARMAINE BACK-UP
his demise. Doctor may be charged for: PALOMAR, MD (TOP MIDTERM
A. Immorality 9 - FEB 2014 MED EXAM AUG
B. Gross Negligence BOARDS; TOPNOTCH 2014
C. Incompetence MD)
D. Stupidity
E. Murder

511 Patient A was mistakenly operated on without his JAN CHARMAINE BACK-UP
consent when it was patient B who was supposed to PALOMAR, MD (TOP MIDTERM
undergo surgery. Even though no harm was done to 9 - FEB 2014 MED EXAM AUG
him, Patient A may recover what kind of damages? BOARDS; TOPNOTCH 2014
A. Liquidated MD)
B. Exemplary
C. Nominal
D. Moderate
E. None of the above

TOPNOTCH MEDICAL BOARD PREP LEGAL MEDICINE AND JURIS SUPEREXAM Page 60 of 86
For inquiries visit www.topnotchboardprep.com.ph or email us at topnotchmedicalboardprep@gmail.com
TOPNOTCH MEDICAL BOARD PREP LEGAL MEDICINE AND JURIS SUPEREXAM
For inquiries visit www.topnotchboardprep.com.ph or email us at topnotchmedicalboardprep@gmail.com
Item QUESTION EXPLANATION AUTHOR TOPNOTCH
# EXAM
512 A physician who stubbornly refuses to go to court when JAN CHARMAINE BACK-UP
summoned may be cited for? PALOMAR, MD (TOP MIDTERM
A. Disobedience 9 - FEB 2014 MED EXAM AUG
B. Direct contempt BOARDS; TOPNOTCH 2014
C. Indirect contempt MD)
D. Reluctancy
E. Circumstantial

513 A genius has an IQ of? JAN CHARMAINE BACK-UP


A. >140 PALOMAR, MD (TOP MIDTERM
B. >130 9 - FEB 2014 MED EXAM AUG
C. 110-120 BOARDS; TOPNOTCH 2014
D. 90-110 MD)
E. 80-90

514 Manceras children are children born to? JAN CHARMAINE BACK-UP
A. Politicians PALOMAR, MD (TOP MIDTERM
B. Live-in partners 9 - FEB 2014 MED EXAM AUG
C. Royalty BOARDS; TOPNOTCH 2014
D. Families below poverty line MD)
E. Prostitutes

515 Jona is considered legitimate daughter of Hans and Ola JAN CHARMAINE BACK-UP
because Jona was born within how many days from the PALOMAR, MD (TOP MIDTERM
dissolution of the previous marriage? 9 - FEB 2014 MED EXAM AUG
A. 50 days BOARDS; TOPNOTCH 2014
B. 100 days MD)
C. 200 days
D. 280 days
E. 300 days

516 Disinterment is also known as: JAN CHARMAINE BACK-UP


A. Embalming PALOMAR, MD (TOP MIDTERM
B. Exhumation 9 - FEB 2014 MED EXAM AUG
C. Euthanasia BOARDS; TOPNOTCH 2014
D. Parricide MD)
E. Homicide

517 Washer woman's hands and feet or Cutis Galina only JAN CHARMAINE BACK-UP
signifies that the body has been in the water for some PALOMAR, MD (TOP MIDTERM
time. This statement is: 9 - FEB 2014 MED EXAM AUG
A. True BOARDS; TOPNOTCH 2014
B. False MD)
C. Cannot be ascertained
D. Probable

518 The most prominent sign of death is? JAN CHARMAINE BACK-UP
A. Cessation of pulse rate PALOMAR, MD (TOP MIDTERM
B. Drop in BP 9 - FEB 2014 MED EXAM AUG
C. Flat line on ECG BOARDS; TOPNOTCH 2014
D. Algor mortis MD)
E. Rigor mortis

519 What do you call the instantaneous rigidity of muscles JAN CHARMAINE BACK-UP
which occurs at the moment of death? PALOMAR, MD (TOP MIDTERM
A. Rigor mortis 9 - FEB 2014 MED EXAM AUG
B. Algor mortis BOARDS; TOPNOTCH 2014
C. Cadaveric spasm MD)
D. Molecular death
E. None of the above

520 Tete de negri is observed in which of the following? JAN CHARMAINE BACK-UP
A. Drowning PALOMAR, MD (TOP MIDTERM
B. Suffocation 9 - FEB 2014 MED EXAM AUG
C. Carbon monoxide poisoning BOARDS; TOPNOTCH 2014
D. Arsenic poisoning MD)
E. Head trauma

TOPNOTCH MEDICAL BOARD PREP LEGAL MEDICINE AND JURIS SUPEREXAM Page 61 of 86
For inquiries visit www.topnotchboardprep.com.ph or email us at topnotchmedicalboardprep@gmail.com
TOPNOTCH MEDICAL BOARD PREP LEGAL MEDICINE AND JURIS SUPEREXAM
For inquiries visit www.topnotchboardprep.com.ph or email us at topnotchmedicalboardprep@gmail.com
Item QUESTION EXPLANATION AUTHOR TOPNOTCH
# EXAM
521 A 3-year-old girl is brought to the physician after her MIGUEL RAFAEL MIDTERM 1
mother noted blood on her underpants. Examination RAMOS, MD (TOP 3 - EXAM - FEB
shows genital condylomata acuminata in the perineal, FEB 2012 MED 2013
peri-introital, labial, and anal areas. Some of the BOARDS; TOPNOTCH
pedunculated condylomata appear to have caused the MD)
bleeding. She has no visible intravaginal condylomata
or vaginal or anal tears. Her mother has a palmar wart
on her hand but no history of condylomata acuminata.
Her mother has a boyfriend who does not live with
them and who has never been left alone with the girl.
They live with the mother's 27-year-old brother who
only baby-sits the children when they are asleep. Which
of the following is the most appropriate next step in
management?
A) Psychiatric assessment of the mother
B) DNA typing of the mother's palmar wart for
papillomavirus
C) Treatment of the mother's palmar wart
D) Vaginal, anal, and throat cultures for Chlamydia
trachomatis and Neisseria gonorrhea in the child

522 A 27-year-old primigravid woman at 38 weeks' MIGUEL RAFAEL MIDTERM 1


gestation is admitted in labor. Her pregnancy has been RAMOS, MD (TOP 3 - EXAM - FEB
uncomplicated, and a routine prenatal visit 2 days ago FEB 2012 MED 2013
showed no abnormalities. On admission, fetal heart BOARDS; TOPNOTCH
tones cannot be heard. Ultrasonography shows little MD)
amniotic fluid, fetal edema, and no evidence of a fetal
heartbeat. After 1 hour, she delivers a 3175-g (7-lb)
stillborn infant; examination of the infant shows no
obvious abnormalities except for mild edema. The
placenta and membranes appear normal. Which of the
following is the most appropriate immediate course of
action?
A) Notify the hospital liability department
B) Obtain consent for fetal organ donation from the
parents
C) Recommend autopsy of the infant
D) Tell the mother not to worry since she can get
pregnant again

523 A 57-year-old man is brought to the physician by his MIGUEL RAFAEL MIDTERM 1
wife because his skin has appeared yellow for 3 weeks. RAMOS, MD (TOP 3 - EXAM - FEB
Examination shows jaundice and scleral icterus. His FEB 2012 MED 2013
total serum bilirubin level is 8 mg/dL with a direct BOARDS; TOPNOTCH
component of 6.2 mg/dL. A CT scan of the abdomen MD)
shows a large lesion in the head of the pancreas. When
the results are initially discussed, the patient says that
he does not want to hear the report, and his wife agrees
to abide by his wishes. Which of the following is the
most appropriate course of action?
A) Withhold the results as the patient wishes
B) Contact the patient's children to discuss the results
C) Consult with the hospital ethics committee
D) Insist on telling the patient the results

524 A 15-year-old girl is brought to the MIGUEL RAFAEL MIDTERM 1


emergencydepartment by her 20-year-old sister RAMOS, MD (TOP 3 - EXAM - FEB
because of a 1-week history of FEB 2012 MED 2013
fatigue, nausea, and abdominal pain. Menarche was at BOARDS; TOPNOTCH
the age of 12 years, and her last menstrual period was 3 MD)
weeks ago. She has not had vaginal discharge. She is
sexually active, and she and her partner use condoms
inconsistently.
Her temperature is 37 C (98.6 F), blood pressure is
110/60 mm Hg, and pulse is 95/min. Abdominal
examination shows mild bilateral lower quadrant
tenderness. Before a pelvic examination and a
pregnancy test can be performed in this patient,
consent must be obtained from which of the following?
A) The court
B) The patient
C) The patient's parent
D) The patient's sister

TOPNOTCH MEDICAL BOARD PREP LEGAL MEDICINE AND JURIS SUPEREXAM Page 62 of 86
For inquiries visit www.topnotchboardprep.com.ph or email us at topnotchmedicalboardprep@gmail.com
TOPNOTCH MEDICAL BOARD PREP LEGAL MEDICINE AND JURIS SUPEREXAM
For inquiries visit www.topnotchboardprep.com.ph or email us at topnotchmedicalboardprep@gmail.com
Item QUESTION EXPLANATION AUTHOR TOPNOTCH
# EXAM
525 A 72-year-old man is brought to the physician by his MIGUEL RAFAEL MIDTERM 1
daughter because of painless jaundice for 1 month. His RAMOS, MD (TOP 3 - EXAM - FEB
wife died 10 years ago, and his daughter is his only FEB 2012 MED 2013
child. Before examining the patient, the daughter asks BOARDS; TOPNOTCH
to speak privately with the physician and asks that she MD)
be given the results of any tests. She specifically
requests that he not be given any "bad news." The
patient is alert. His vital signs are within normal limits.
Examination shows scleral icterus and jaundice. There
is mild abdominal tenderness on palpation. He is
oriented to person, place, and time. A CT scan of the
abdomen shows a pancreatic mass with bile duct
obstruction and probable metastatic lesions in the liver.
Which of the following is the most appropriate next
step?
A) Abide by the daughter's wishes
B) Ask the patient if he wishes to discuss his test
results, preferably with his daughter present
C) Tell the daughter it is a legal requirement to tell the
patient any and all results of medical testing
D) Consult with the hospital attorney

526 A mentally competent 76-year-old man is in the in a terminally ill patient, the priority is pain MIGUEL RAFAEL MIDTERM 1
terminal stage of multiple myeloma. He is unable to control RAMOS, MD (TOP 3 - EXAM - FEB
move and requires 24-hour nursing care. Increasing FEB 2012 MED 2013
doses of narcotics are needed to control severe pain. As BOARDS; TOPNOTCH
a result, when he is pain-free, respiratory function is MD)
impaired and consciousness is clouded. The patient
says he cannot live with this degree of pain and asks to
be given a lethal injection of pain medication. Which of
the following is the most appropriate step regarding the
pain medication?
A) Reduce the dosage so as not to impair respiration
B) Administer the dosage necessary to control pain
despite respiratory impairment
C) Administer the dosage necessary to control pain and
add a centrally acting stimulant
D ) Appeal to the family to convince the patient to
tolerate a bit more pain

527 An 82-year-old woman is brought to the emergency MIGUEL RAFAEL MIDTERM 1


department 30 minutes after collapsing at a shopping RAMOS, MD (TOP 3 - EXAM - FEB
center. She was intubated at the scene. On admission, FEB 2012 MED 2013
she is comatose and requires mechanical ventilation. BOARDS; TOPNOTCH
Her pulse is 120/min, respirations are 12/min, and MD)
blood pressure is 90/60 mm Hg. She does not respond
to painful stimuli. Thirty minutes later, her
granddaughter comes to the hospital and shows the
physician an advance directive signed by the patient
requesting no intubation or “artificial means to prolong
life.” Which of the following is the most appropriate
next step in management?
A) Contact the hospital ethics committee
B) Continue mechanical ventilation and monitor closely
in the coronary care unit
C) Obtain nuclear brain blood flow scan to determine
brain death
D) Extubate the patient and provide supportive care
only

528 Patient was aware that the physician was drunk yet he MIGUEL RAFAEL MIDTERM 1
allowed the doctor to inject him: RAMOS, MD (TOP 3 - EXAM - FEB
A) Doctrine of continuing negligence FEB 2012 MED 2013
B) Vicarious liability BOARDS; TOPNOTCH
C) Res ipsa loquitor MD)
D) Captain of the ship

529 Which of the following characteristics of a person is not MIGUEL RAFAEL MIDTERM 1
easily changed? RAMOS, MD (TOP 3 - EXAM - FEB
A) Hair pattern FEB 2012 MED 2013
B) Places frequented BOARDS; TOPNOTCH
C) Speech, gait and mannerism MD)
D) Voice

TOPNOTCH MEDICAL BOARD PREP LEGAL MEDICINE AND JURIS SUPEREXAM Page 63 of 86
For inquiries visit www.topnotchboardprep.com.ph or email us at topnotchmedicalboardprep@gmail.com
TOPNOTCH MEDICAL BOARD PREP LEGAL MEDICINE AND JURIS SUPEREXAM
For inquiries visit www.topnotchboardprep.com.ph or email us at topnotchmedicalboardprep@gmail.com
Item QUESTION EXPLANATION AUTHOR TOPNOTCH
# EXAM
530 What is the study of identification of fingerprints by MIGUEL RAFAEL MIDTERM 1
comparison? RAMOS, MD (TOP 3 - EXAM - FEB
A) Dactyloscopy FEB 2012 MED 2013
B) Dactylography BOARDS; TOPNOTCH
C) Graphology MD)
D) Poroscopy
531 Which of the following is the chemical test used to MIGUEL RAFAEL MIDTERM 1
determine if the stain is of blood origin or not? RAMOS, MD (TOP 3 - EXAM - FEB
A) Benzidine test FEB 2012 MED 2013
B) Marquis test BOARDS; TOPNOTCH
C) Icards test MD)
D) Zwikkers test

532 What is the pathophysiology behind the flattened area MIGUEL RAFAEL MIDTERM 1
when the body surface of a corpse is pressed? RAMOS, MD (TOP 3 - EXAM - FEB
A) Absence of edema FEB 2012 MED 2013
B) Loss of elasticity of the skin BOARDS; TOPNOTCH
C) Precipitated skin proteins MD)
D) Absence of blood

533 A physician must appear in response to a subpoena if < 50 km MIGUEL RAFAEL MIDTERM 1
his place of residence is how many kilometers from the RAMOS, MD (TOP 3 - EXAM - FEB
court issuing the subpoena? FEB 2012 MED 2013
A) < 80 km BOARDS; TOPNOTCH
B) <70 km MD)
C) <60 km
D) < 50 km

534 A doctor is impotent yet his wife gave birth to a healthy MIGUEL RAFAEL MIDTERM 1
baby while in lawful wedlock. Which is true? RAMOS, MD (TOP 3 - EXAM - FEB
A) The child’s legitimacy cannot be questioned FEB 2012 MED 2013
B) Proof of impotence will overthrow presumption of BOARDS; TOPNOTCH
legitimacy MD)
C) The child upon birth can be declared if formal
D) None of the above

535 A mother does not intend to hurt her child. When she is MIGUEL RAFAEL MIDTERM 1
drunk, she periodically batters her son. She becomes RAMOS, MD (TOP 3 - EXAM - FEB
remorseful afterwards and provides proper care in FEB 2012 MED 2013
between her drinking sprees. What type of child abuser BOARDS; TOPNOTCH
is the mother? MD)
A) Occasional child abuser
B) Intermittent child abuser
C) Constant child abuser
D) One time child abuser

536 A baby, born alive, has a legal personality. One of the MIGUEL RAFAEL MIDTERM 1
tests used to determine life before death of a fetus is to RAMOS, MD (TOP 3 - EXAM - FEB
float the lungs in water is FEB 2012 MED 2013
A) Wrendin’s test BOARDS; TOPNOTCH
B) Icard’s test MD)
C) Breslau’s test
D) Hydrostatic test

537 The liability of a professor enticing a 16 year old girl, MIGUEL RAFAEL MIDTERM 1
who is a virgin, into having sexual intercourse with him RAMOS, MD (TOP 3 - EXAM - FEB
is FEB 2012 MED 2013
A) Statutory rape BOARDS; TOPNOTCH
B) Simple seduction MD)
C) Qualified seduction
D) Act of lasciviousness

538 Physical injuries which develop on parts of the body MIGUEL RAFAEL MIDTERM 1
near bony tissues that are deep seated caused by RAMOS, MD (TOP 3 - EXAM - FEB
forcible impact of hard blunt objects are usually FEB 2012 MED 2013
A) Abrasion BOARDS; TOPNOTCH
B) Contusion MD)
C) Hematoma
D) Lacerated wound

539 What is another term for the accidental death of a MIGUEL RAFAEL MIDTERM 1
young child by suffocation either from the pressure of RAMOS, MD (TOP 3 - EXAM - FEB
beddings and pillows or from pressure of the FEB 2012 MED 2013
unconscious drunk mother? BOARDS; TOPNOTCH
A) Battering MD)
B) Overlaying
C) Burking
D) Mugging

TOPNOTCH MEDICAL BOARD PREP LEGAL MEDICINE AND JURIS SUPEREXAM Page 64 of 86
For inquiries visit www.topnotchboardprep.com.ph or email us at topnotchmedicalboardprep@gmail.com
TOPNOTCH MEDICAL BOARD PREP LEGAL MEDICINE AND JURIS SUPEREXAM
For inquiries visit www.topnotchboardprep.com.ph or email us at topnotchmedicalboardprep@gmail.com
Item QUESTION EXPLANATION AUTHOR TOPNOTCH
# EXAM
540 An idiot is exempted from criminal liability.What is the MIGUEL RAFAEL MIDTERM 1
IQ of an idiot? RAMOS, MD (TOP 3 - EXAM - FEB
A) 0-20 FEB 2012 MED 2013
B) 10-30 BOARDS; TOPNOTCH
C) 20-40 MD)
D) 50-70

541 According to the Medical Act of 1959, which of the Pedia-lyte is a household remedy. ABDELSIMAR OMAR FINAL EXAM -
following does NOT constitute an act of medicine? II, MD (TOP 2 - AUG FEB 2014
A. A person who has neither graduated from medical 2013 MED BOARDS;
school nor has taken the board exam uses the title of TOPNOTCH MD - 200
MD after his name. QUESTIONS) AND
B. A person physically examines another and diagnoses MARC DENVER
and prescribes medications; but refuses to take TIONGSON, MD (40
professional fees. QUESTIONS)
C. A physician appearing on Salamat Doc diagnoses and
treats a patient consulting over the phone.
D. A mother gives Pedia-Lyte to her daughter suffering
from diarrhea.
E. None of the above

542 According to the Medical Act of 1959, which of the ABDELSIMAR OMAR FINAL EXAM -
following is NOT a ground for suspension or revocation II, MD (TOP 2 - AUG FEB 2014
of registration certificate? 2013 MED BOARDS;
A. Physician knowingly issues a false medical certificate TOPNOTCH MD - 200
B. Physician is addicted to alcoholic beverages QUESTIONS) AND
C. Physician issues business cards proclaiming that he MARC DENVER
graduated from Harvard Medical School and trained in TIONGSON, MD (40
Brigham and Women's Hospital QUESTIONS)
D. Physician spreads rumor derogatory to the character
of another physician.
E. None of the above

543 The following are inherent rights of the physician D is an incidental right ABDELSIMAR OMAR FINAL EXAM -
except: II, MD (TOP 2 - AUG FEB 2014
A. Right to choose patient 2013 MED BOARDS;
B. Right to limit practice TOPNOTCH MD - 200
C. Right to determine appropriate management QUESTIONS) AND
procedures MARC DENVER
D. Right of way while responding to call of emergencies TIONGSON, MD (40
E. Right to avail of hospital services QUESTIONS)
544 An industrial physician charges the company he is ABDELSIMAR OMAR FINAL EXAM -
working for according to hours spent at clinic and not II, MD (TOP 2 - AUG FEB 2014
by the quantity of medical services rendered. Which of 2013 MED BOARDS;
the following statements is correct? TOPNOTCH MD - 200
A. The physician charges a contingent fee which is QUESTIONS) AND
considered unethical. MARC DENVER
B. The physician charges a contractual fee which is TIONGSON, MD (40
considered ethical. QUESTIONS)
C. The physician charges a retainer fee which is
considered ethical.
D. The physician charges a straight fee which is
considered unethical.
E. The physician charges a straight fee which is
considered ethical.

545 An actress sustained facial deformities after undergoing ABDELSIMAR OMAR FINAL EXAM -
a face lift. The physician is sued because the patient has II, MD (TOP 2 - AUG FEB 2014
suffered 'anguish, social humiliation, and moral shock.' 2013 MED BOARDS;
The type of damages that can be recovered as a result of TOPNOTCH MD - 200
these is termed: QUESTIONS) AND
A. Actual damages MARC DENVER
B. Nominal damages TIONGSON, MD (40
C. Moral damages QUESTIONS)
D. Compensatory damages
E. Liquidated damages

546 A physician may do a procedure even when it is against ABDELSIMAR OMAR FINAL EXAM -
the patient's wishes if: II, MD (TOP 2 - AUG FEB 2014
A. The procedure is essential to maintain the patient's 2013 MED BOARDS;
well being TOPNOTCH MD - 200
B. The procedure is deemed safe by the medical QUESTIONS) AND
community MARC DENVER
C. The patient's parents consent to it TIONGSON, MD (40
D. The procedure is required by law QUESTIONS)

TOPNOTCH MEDICAL BOARD PREP LEGAL MEDICINE AND JURIS SUPEREXAM Page 65 of 86
For inquiries visit www.topnotchboardprep.com.ph or email us at topnotchmedicalboardprep@gmail.com
TOPNOTCH MEDICAL BOARD PREP LEGAL MEDICINE AND JURIS SUPEREXAM
For inquiries visit www.topnotchboardprep.com.ph or email us at topnotchmedicalboardprep@gmail.com
Item QUESTION EXPLANATION AUTHOR TOPNOTCH
# EXAM
547 Physical injury is considered slight if the victim is ABDELSIMAR OMAR FINAL EXAM -
incapacitated for how long: II, MD (TOP 2 - AUG FEB 2014
A. 6 - 12 hours 2013 MED BOARDS;
B. 12 - 24 hours TOPNOTCH MD - 200
C. 1 - 9 days QUESTIONS) AND
D. 2 weeks MARC DENVER
E. 1 month TIONGSON, MD (40
QUESTIONS)

548 After being hit by a truck in the legs, a man was thrown ABDELSIMAR OMAR FINAL EXAM -
10 feet from impact hitting concrete pavement. As a II, MD (TOP 2 - AUG FEB 2014
result, patient sustained abrasions, contusions and 2013 MED BOARDS;
lacerations in other parts of the body aside from TOPNOTCH MD - 200
sustaining leg fractures. The abrasiions, contusions and QUESTIONS) AND
lacerations are termed: MARC DENVER
A. Run over injuries TIONGSON, MD (40
B. Hit and run injuries QUESTIONS)
C. Secondary impact injuries
D. Primary impact injuries

549 A 28/M embraces and kisses a woman against her will. ABDELSIMAR OMAR FINAL EXAM -
He may be charged with: II, MD (TOP 2 - AUG FEB 2014
A. Rape 2013 MED BOARDS;
B. Simple seduction TOPNOTCH MD - 200
C. Qualified seduction QUESTIONS) AND
D. Assault and battery MARC DENVER
E. Acts of lasciviousness TIONGSON, MD (40
QUESTIONS)

550 A physician who appeared in court was asked about the ABDELSIMAR OMAR FINAL EXAM -
injuries sustained by a victim of a vehicular crash. He II, MD (TOP 2 - AUG FEB 2014
then described in detail the abrasions and contusions 2013 MED BOARDS;
sustained by the patient. TOPNOTCH MD - 200
A. The physician is providing documentary evidence in QUESTIONS) AND
court. MARC DENVER
B. The physician is acting as an ordinary witness. TIONGSON, MD (40
C. The physician is acting as an expert witness. QUESTIONS)
D. The physician is testifying on privileged informaiton.
551 The following are characteristics of an enrance wound ABDELSIMAR OMAR FINAL EXAM -
EXCEPT: II, MD (TOP 2 - AUG FEB 2014
A. Wound size is usually smaller than an exit wound 2013 MED BOARDS;
B. Usually inverted TOPNOTCH MD - 200
C. Usually oval or round depending on the angle of QUESTIONS) AND
approach of the bullet MARC DENVER
D. May cause tattooing TIONGSON, MD (40
E. None of the above QUESTIONS)

552 The following are characteristics of medico-legal C is a feature of pathologic autopsy ABDELSIMAR OMAR FINAL EXAM -
autopsy except: II, MD (TOP 2 - AUG FEB 2014
A. The purose is to correlate tissue changes with a 2013 MED BOARDS;
criminal act TOPNOTCH MD - 200
B. Emphasis is placed on the effect of wrongful act on QUESTIONS) AND
the body MARC DENVER
C. Requires consent of the next of kin TIONGSON, MD (40
D. Provides a factual, objective medical report QUESTIONS)
E. None of the above
553 A person who died of complications of PTB and is ABDELSIMAR OMAR FINAL EXAM -
unembalmed must be buried not later than ___. II, MD (TOP 2 - AUG FEB 2014
A. 6 hours 2013 MED BOARDS;
B. 12 hours TOPNOTCH MD - 200
C. 20 hours QUESTIONS) AND
D. 24 hours MARC DENVER
E. 48 hours TIONGSON, MD (40
QUESTIONS)

554 In carbon monoxide poisoning, oxygen is delivered to ABDELSIMAR OMAR FINAL EXAM -
the tissues but cannot be utilized properly due to II, MD (TOP 2 - AUG FEB 2014
failure of celular oxidative processes. This type of 2013 MED BOARDS;
asphyxial death is termed: TOPNOTCH MD - 200
A. Anoxic QUESTIONS) AND
B. Histotoxic MARC DENVER
C. Stagnant TIONGSON, MD (40
D. Anemic QUESTIONS)
E. Molecular

555 A man who had carnal knowledge of a woman through ABDELSIMAR OMAR FINAL EXAM -
force, threat or intimidation is charged with rape. If II, MD (TOP 2 - AUG FEB 2014
convicted, he shall be punished by: 2013 MED BOARDS;
A. Prison correccional TOPNOTCH MD - 200
B. Prison mayor QUESTIONS) AND
C. Reclusion temporal MARC DENVER
D. Reclusion perpetua TIONGSON, MD (40
E. Death penalty QUESTIONS)

TOPNOTCH MEDICAL BOARD PREP LEGAL MEDICINE AND JURIS SUPEREXAM Page 66 of 86
For inquiries visit www.topnotchboardprep.com.ph or email us at topnotchmedicalboardprep@gmail.com
TOPNOTCH MEDICAL BOARD PREP LEGAL MEDICINE AND JURIS SUPEREXAM
For inquiries visit www.topnotchboardprep.com.ph or email us at topnotchmedicalboardprep@gmail.com
Item QUESTION EXPLANATION AUTHOR TOPNOTCH
# EXAM
556 Which of the following is a requisite for a physician to ABDELSIMAR OMAR FINAL EXAM -
have his license suspended or revoked? II, MD (TOP 2 - AUG FEB 2014
A. Substantial evidence 2013 MED BOARDS;
B. Preponderance of evidence TOPNOTCH MD - 200
C. Guilt beyond reasonable doubt QUESTIONS) AND
D. Real evidence MARC DENVER
E. Autoptic evidence TIONGSON, MD (40
QUESTIONS)

557 A patient who was denied access to his or her medical ABDELSIMAR OMAR FINAL EXAM -
record may compel the custodian to allow him or her to II, MD (TOP 2 - AUG FEB 2014
see the documents by filing a petition in court for: 2013 MED BOARDS;
A. Subpoena duces tecum TOPNOTCH MD - 200
B. Injunction QUESTIONS) AND
C. Certoriari MARC DENVER
D. Temporary restraining order TIONGSON, MD (40
E. Mandamus QUESTIONS)

558 A patient expresses his wishes to leave against medical ABDELSIMAR OMAR FINAL EXAM -
advice. The physician, believing that he is acting in the II, MD (TOP 2 - AUG FEB 2014
patient's best interest, forces the patient to remain. The 2013 MED BOARDS;
physician may be charged with: TOPNOTCH MD - 200
A. Assault and battery QUESTIONS) AND
B. Malpractice MARC DENVER
C. Negligence TIONGSON, MD (40
D. Breach of duty QUESTIONS)
E. False imprisonment

559 A patient with pancreatic cancer undergoes ABDELSIMAR OMAR FINAL EXAM -
experimental treatment despite lack of knowledge of II, MD (TOP 2 - AUG FEB 2014
the possible side effects. The researchers are not liable 2013 MED BOARDS;
for any untoward event that may occur due to this TOPNOTCH MD - 200
doctrine. QUESTIONS) AND
A. Doctrine of informed consent MARC DENVER
B. Assumption of risk TIONGSON, MD (40
C. Doctrine of forseeability QUESTIONS)
D. Doctrine of contributory negligence
E. Doctrine of sole responsibility

560 After graduating from medical school, you begin using SIMILAR TO PREVIOUS BOARD EXAM ABDELSIMAR OMAR FINAL EXAM -
the title MD after your name. You are: CONCEPT/PRINCIPLE II, MD (TOP 2 - AUG FEB 2014
A. Committing malpractice 2013 MED BOARDS;
B. Committing fraud TOPNOTCH MD - 200
C. Illegally practicing medicine QUESTIONS) AND
D. Academically entitled to use MD MARC DENVER
TIONGSON, MD (40
QUESTIONS)
561 1. The following are acts considered as immoral or BLAKE WARREN MIDTERM 2
dishonorable conduct under the Code of Medical Ethics: ANG, MD (TOP 1 - EXAM - FEB
a. Referral Fee AUG 2013 MED 2014
b. Insanity BOARDS; TOPNOTCH
c. Simple fee MD)
d. Conviction of a crime

562 A gauze was left inside the abdominal cavity after an BLAKE WARREN MIDTERM 2
operation. The applicable legal principle in this case ANG, MD (TOP 1 - EXAM - FEB
is: AUG 2013 MED 2014
a. Doctrine of vicarious liability BOARDS; TOPNOTCH
b. Doctrine of res ipsa loquitur MD)
c. Doctrine of contributory negligence
d. Doctrine of foreseeability

563 Medical malpractice is an offense that is filed to which BLAKE WARREN MIDTERM 2
regulatory body? ANG, MD (TOP 1 - EXAM - FEB
a. Philippine Medical Association AUG 2013 MED 2014
b. The Board of Medicine BOARDS; TOPNOTCH
c. Department of Health MD)
d. Board of Medical Education

TOPNOTCH MEDICAL BOARD PREP LEGAL MEDICINE AND JURIS SUPEREXAM Page 67 of 86
For inquiries visit www.topnotchboardprep.com.ph or email us at topnotchmedicalboardprep@gmail.com
TOPNOTCH MEDICAL BOARD PREP LEGAL MEDICINE AND JURIS SUPEREXAM
For inquiries visit www.topnotchboardprep.com.ph or email us at topnotchmedicalboardprep@gmail.com
Item QUESTION EXPLANATION AUTHOR TOPNOTCH
# EXAM
564 Before admission to the hospital, Mr. Tindoc was BLAKE WARREN MIDTERM 2
informed of the absence of window bars to prevent a ANG, MD (TOP 1 - EXAM - FEB
psychiatric patient from jumping. AUG 2013 MED 2014
a. Assumption of risk BOARDS; TOPNOTCH
b. Res ipsa loquitor MD)
c. Continuing negligence
d. Common knowledge

565 Pregnancy may be a ground for the annulment of BLAKE WARREN MIDTERM 2
marriage when: ANG, MD (TOP 1 - EXAM - FEB
a. Wife conceals the fact the child is that of another man AUG 2013 MED 2014
b. Husband at the time of marriage knows the child is BOARDS; TOPNOTCH
not his MD)
c. Wife conceals the pregnancy caused by her husband
d. The husbands files for annulment 10 years from the
discovery the child is not his

566 Which cannot be sued and thus vicarious liability does BLAKE WARREN MIDTERM 2
not apply: ANG, MD (TOP 1 - EXAM - FEB
a. Government hospitals with proprietary functions AUG 2013 MED 2014
b. Charity hospitals BOARDS; TOPNOTCH
c. Private hospitals MD)
d. Public hospitals with governmental functions

567 Tardieu spots are seen during the autopsy, what was BLAKE WARREN MIDTERM 2
the cause of death? ANG, MD (TOP 1 - EXAM - FEB
a. Hanging AUG 2013 MED 2014
b. Manual strangulation BOARDS; TOPNOTCH
c. Ligature strangulation MD)
d. Choking

568 A whitish foam in the mouth and nostrils as a sign BLAKE WARREN MIDTERM 2
of drowning is: ANG, MD (TOP 1 - EXAM - FEB
a. Emphysema AUG 2013 MED 2014
b. edema aquasum BOARDS; TOPNOTCH
c. tache noire MD)
d. champignon d'ocume

569 Presidential Decree 169 mandates all doctors to BLAKE WARREN MIDTERM 2
report all cases involving: ANG, MD (TOP 1 - EXAM - FEB
a. slight and less physical injury AUG 2013 MED 2014
b. less serious and serious physical injuries BOARDS; TOPNOTCH
c. mutilation MD)
d. child abuse cases

570 Injury that may be found in areas of the skull offering BLAKE WARREN MIDTERM 2
the least resistance: ANG, MD (TOP 1 - EXAM - FEB
a. Locus minoris resitencia AUG 2013 MED 2014
b. Countre-coup injuries BOARDS; TOPNOTCH
c. Remote injuries MD)
d. Coup injuries

571 A test finding wherein the middle ear of a child, before BLAKE WARREN MIDTERM 2
birth is filled with gelatinous, embryonic connective ANG, MD (TOP 1 - EXAM - FEB
tissues but disappears after birth. AUG 2013 MED 2014
a. Fodere’s BOARDS; TOPNOTCH
b. Wredin’s MD)
c. Winslo’s
d. Breslau’s

572 Fingerprints are formed in the fetus during pregnancy BLAKE WARREN MIDTERM 2
in the: ANG, MD (TOP 1 - EXAM - FEB
a. Second month AUG 2013 MED 2014
b. Third month BOARDS; TOPNOTCH
c. Fourth week MD)
d. Fourth month

TOPNOTCH MEDICAL BOARD PREP LEGAL MEDICINE AND JURIS SUPEREXAM Page 68 of 86
For inquiries visit www.topnotchboardprep.com.ph or email us at topnotchmedicalboardprep@gmail.com
TOPNOTCH MEDICAL BOARD PREP LEGAL MEDICINE AND JURIS SUPEREXAM
For inquiries visit www.topnotchboardprep.com.ph or email us at topnotchmedicalboardprep@gmail.com
Item QUESTION EXPLANATION AUTHOR TOPNOTCH
# EXAM
573 Test for determining whether semen is of human origin BLAKE WARREN MIDTERM 2
or not is: ANG, MD (TOP 1 - EXAM - FEB
a. Precipitin AUG 2013 MED 2014
b. Takayama BOARDS; TOPNOTCH
c. Florence MD)
d. Benzidine

574 Disorientation, mental confusion, dizziness, sensory BLAKE WARREN MIDTERM 2


disturbances exaggerated emotional state suggest that ANG, MD (TOP 1 - EXAM - FEB
the blood alcohol level is: AUG 2013 MED 2014
a. 150 - 300mg% BOARDS; TOPNOTCH
b. 400 - 500 mg% MD)
c. 100 - 120 mg%
d. 80 mg %

575 A child is legitimate if he/she was born in lawful BLAKE WARREN MIDTERM 2
wedlock or within how many days after the dissolution ANG, MD (TOP 1 - EXAM - FEB
of marriage: AUG 2013 MED 2014
a. 320 days BOARDS; TOPNOTCH
b. 280 days MD)
c. 300 days
d. 360 days

576 A gunshot victim was brought to Dr. Tindoc for BLAKE WARREN MIDTERM 2
management. Dr. Tindoc refused to treat the patient ANG, MD (TOP 1 - EXAM - FEB
since he is a pathologist. Which one of the following is AUG 2013 MED 2014
applicable? BOARDS; TOPNOTCH
a. Dr. Tindoc has the right to choose his patients. MD)
b. It is illegal to refuse rendering medical service.
c. Dr. Tindoc is liable for damages.
d. Dr. Tindoc must manage the gunshot victim.

577 Burden of evidence needed to convict a physician for BLAKE WARREN MIDTERM 2
criminal malpractice: ANG, MD (TOP 1 - EXAM - FEB
a. Substantial evidence AUG 2013 MED 2014
b. Circumstantial evidence BOARDS; TOPNOTCH
c. Proof beyond reasonable doubt MD)
d. Preponderance of evidence

578 The body should be buried within how many hours if BLAKE WARREN MIDTERM 2
the death is due to dangerous communicable disease ANG, MD (TOP 1 - EXAM - FEB
a. 6 hours AUG 2013 MED 2014
b. 12 hours BOARDS; TOPNOTCH
c. 24 hours MD)
d. 48 hours

579 The form of evidence given by the attending physician BLAKE WARREN MIDTERM 2
of a dying patient who related to him that his wife ANG, MD (TOP 1 - EXAM - FEB
stabbed him. AUG 2013 MED 2014
a. hearsay evidence BOARDS; TOPNOTCH
b. dying declaration MD)
c. testimonial evidence
d. deposition

580 These are damages imposed on a physician as BLAKE WARREN MIDTERM 2


punishment and to serve as an example or ANG, MD (TOP 1 - EXAM - FEB
correction for the medical profession: AUG 2013 MED 2014
a. Nominal damages BOARDS; TOPNOTCH
b. Compensatory damages MD)
c. Temperate damages
d. Exemplary damages

581 A member of the PMA is required a total of how many Every PMA member is required to earn his TIMOTHY TANG LEE MIDTERM 1
CME units per year to maintain membership with good credit a minimum of 100 CME Units per fiscal SAY, MD (TOP 4 - EXAM - FEB
standing? year. (From Section 38: Required CME credit AUG 2013 MED 2014
A. 10 units of the PMA Code for Continuing Medical BOARDS; TOPNOTCH
B. 25 Education) PDF content is downloadable MD)
C. 50 from PMA website.
D. 75
E. 100

TOPNOTCH MEDICAL BOARD PREP LEGAL MEDICINE AND JURIS SUPEREXAM Page 69 of 86
For inquiries visit www.topnotchboardprep.com.ph or email us at topnotchmedicalboardprep@gmail.com
TOPNOTCH MEDICAL BOARD PREP LEGAL MEDICINE AND JURIS SUPEREXAM
For inquiries visit www.topnotchboardprep.com.ph or email us at topnotchmedicalboardprep@gmail.com
Item QUESTION EXPLANATION AUTHOR TOPNOTCH
# EXAM
582 Which of the following is not a prerequisite to the Being a citizen is not a requirement to TIMOTHY TANG LEE MIDTERM 1
practice of Medicine in the Philippines? practice medicine in the Philippines. SAY, MD (TOP 4 - EXAM - FEB
A. At least 21 years old AUG 2013 MED 2014
B. Citizen of the Philippines BOARDS; TOPNOTCH
C. Medical school diploma MD)
D. Board exam passer
E. Holder of certificate of registration

583 The privilege to practice Medicine is regulated by what The police power of the state has the right to TIMOTHY TANG LEE MIDTERM 1
entity? regulate the practice of medicine SAY, MD (TOP 4 - EXAM - FEB
A. Board of Medicine AUG 2013 MED 2014
B. Police power of the State BOARDS; TOPNOTCH
C. Judicial body of the State MD)
D. Professional Regulatory Commission
E. Philippine Medical Association

584 Dr. AD was asked by the court to describe the findings Merely description without inference of the TIMOTHY TANG LEE MIDTERM 1
in his physical examination of Patient BL, who was a possible explanation for the injuries or SAY, MD (TOP 4 - EXAM - FEB
victim in a shoot-out. Dr. AD in court is acting as a? diagnosis would mean that he is merely AUG 2013 MED 2014
A. Ordinary witness acting as an ordinary witness. BOARDS; TOPNOTCH
B. Expert witness MD)
C. Both as ordinary and expert witness
D. Special witness
E. None of the above

585 A case was filed against the surgeon who left a foreign The intern cannot be imposed any TIMOTHY TANG LEE MIDTERM 1
object inside the patient's abdomen after performing reprimand, suspension or revocation because SAY, MD (TOP 4 - EXAM - FEB
cholecystectomy. The residents, interns and nurses he does not possess a license, therefore he is AUG 2013 MED 2014
assisting in the operation are also included in the only liable criminally and civilly. BOARDS; TOPNOTCH
lawsuit. The nature of the liability against the intern MD)
would be?
A. Administrative
B. Civil
C. Administrative and Civil
D. Criminal and Civil
E. Criminal, Civil and Administrative

586 What is the burden of proof needed for a physician to Criminal - proof beyond reasonable doubt TIMOTHY TANG LEE MIDTERM 1
be civilly liable? Civil - Preponderance of evidence SAY, MD (TOP 4 - EXAM - FEB
A. Proof beyond reasonable doubt Administrative - Substantial evidence AUG 2013 MED 2014
B. Preponderance of evidence BOARDS; TOPNOTCH
C. Extensive evidence MD)
D. Substantial evidence
E. Ample evidence

587 Dr. FF was subpoenaed to act as an expert witness in a The doctor may not appear in any courts he TIMOTHY TANG LEE MIDTERM 1
criminal case held in the regional trial court of the Bicol was subpoenaed provided it is greater than SAY, MD (TOP 4 - EXAM - FEB
region, 120 km away from his residence. He did not 100 km from his residence. AUG 2013 MED 2014
comply because he has a lot of patients to see in his BOARDS; TOPNOTCH
clinic, he may be cited for? MD)
A. Direct contempt
B. Indirect contempt
C. Both direct and indirect contempt
D. Unduly prioritizing his practice
E. None

588 The following medical fees are considered immoral or Simple contractual and retainer fee is ethical. TIMOTHY TANG LEE MIDTERM 1
unethical under the Code of Medical Ethics EXCEPT? SAY, MD (TOP 4 - EXAM - FEB
A. Contingent fee AUG 2013 MED 2014
B. Dichotomous fee BOARDS; TOPNOTCH
C. Retainer fee MD)
D. Fee splitting
E. Straight fee

589 The hospital is almost full and the only vacant beds are Contamination of other patients is foreseen TIMOTHY TANG LEE MIDTERM 1
in the ward. An infectious case was admitted in the when one admits an infectious case in the SAY, MD (TOP 4 - EXAM - FEB
ward resulting to contamination of other patients. The ward filled with patients who does not have AUG 2013 MED 2014
physician who admitted the patient is liable under what the same infection. BOARDS; TOPNOTCH
doctrine? MD)
A. Vicarious liability
B. Res Ipsa Loquitor
C. Superior knowledge
D. Foreseeability
E. Continuing negligence

TOPNOTCH MEDICAL BOARD PREP LEGAL MEDICINE AND JURIS SUPEREXAM Page 70 of 86
For inquiries visit www.topnotchboardprep.com.ph or email us at topnotchmedicalboardprep@gmail.com
TOPNOTCH MEDICAL BOARD PREP LEGAL MEDICINE AND JURIS SUPEREXAM
For inquiries visit www.topnotchboardprep.com.ph or email us at topnotchmedicalboardprep@gmail.com
Item QUESTION EXPLANATION AUTHOR TOPNOTCH
# EXAM
590 Which of the following is NOT an inherent right of the Exemption from execution of instruments TIMOTHY TANG LEE MIDTERM 1
physician? and library is an incidental right. SAY, MD (TOP 4 - EXAM - FEB
A. Choose patients AUG 2013 MED 2014
B. Exemption from execution of instruments and BOARDS; TOPNOTCH
library MD)
C. Avail of hospital services
D. Limit the practice of medicine
E. Determine the appropriate management and
procedures for his patients
591 If a patient died or become incapacitated to do so, the The order is spouse, descendants, TIMOTHY TANG LEE MIDTERM 1
claim for medical fee shall be made from the following ascendants, siblings. SAY, MD (TOP 4 - EXAM - FEB
persons in order? AUG 2013 MED 2014
A. Spouse, parents, children, siblings BOARDS; TOPNOTCH
B. Spouse, parents, siblings, children MD)
C. Spouse, children, parents, siblings
D. Spouse, siblings, parents, children
E. Spouse, children, siblings, parents

592 A physician who acquire his license after passing the Renewal is done every 3 years. TIMOTHY TANG LEE MIDTERM 1
board exam needs to renew it at what time interval? SAY, MD (TOP 4 - EXAM - FEB
A. Every 2 years AUG 2013 MED 2014
B. Every 3 years BOARDS; TOPNOTCH
C. Every 4 years MD)
D. Every 5 years
E. Every 6 years

593 Which of the following organs putrefy late? Putrefy Early: Brain, Stomach and intestines; TIMOTHY TANG LEE MIDTERM 1
A. Brain Lining of the trachea and larynx; spleen, liver; SAY, MD (TOP 4 - EXAM - FEB
B. Gravid Uterus gravid uterus. AUG 2013 MED 2014
C. Spleen Putrefy Late: Esophagus; Diaphragm; Heart; BOARDS; TOPNOTCH
D. Liver Lungs; Kidneys; Urinary Bladder; Prostate; MD)
E. Lungs Uterus (not pregnant)

594 The patient realized that his physicain was grossly 3 types: TIMOTHY TANG LEE MIDTERM 1
intoxicated at the time he was going to give the patient Antecedent: Problem SAY, MD (TOP 4 - EXAM - FEB
an injection. The patient allowed the physician to Contemporaneous: while the esophagoscope AUG 2013 MED 2014
proceed in spite of the physician's conditions. The was being applied the patient made a violent BOARDS; TOPNOTCH
patient was injured. The court will held the patient move without warning which resulted to the MD)
liable for? puncture of the esophagus.
A. Antecedent contributory negligence Subsequent: Patient sustained a wound
B. Contemporaneous contributory negligence which the physician sutured and gave a
C. Subsequent contributory negligence prescription for tetanus anti0toxin. He
D. Concurrent contributory negligence instructed the patient to go immediately to
E. No liability the drug store and have it filled and bring it
back to the office for an injetion. The patient
took the prescription and went home.
595 The test used to determine whether semen is of human Precipitin test or biological test of Farnum TIMOTHY TANG LEE MIDTERM 1
origin or not is? will test whether the semen is human origin SAY, MD (TOP 4 - EXAM - FEB
A. Florence test or not. AUG 2013 MED 2014
B. Berberio's test BOARDS; TOPNOTCH
C. Farnum's test MD)
D. Acid phosphatase test
E. Ganguli's method

596 The finding in death is not consistent and may not Cadaveric spasm or instantaneous rigor is TIMOTHY TANG LEE MIDTERM 1
appear on a person who died? not always present and is not part of the 3 SAY, MD (TOP 4 - EXAM - FEB
A. Instantaneous rigor stages of muscles after death. AUG 2013 MED 2014
B. Primary flaccidity BOARDS; TOPNOTCH
C. Rigor mortis MD)
D. Cadaveric rigidity
E. Secondary flaccidity

597 A woman scratched another in the face after a fight. The Slight: 1-9 days TIMOTHY TANG LEE MIDTERM 1
doctor attended the injury that resolved in 10 days Less serious: 10-29 days SAY, MD (TOP 4 - EXAM - FEB
without any sequalae. The legal classification of the Serious: >30days; or with scar, deformity, AUG 2013 MED 2014
injury is? loss of organ BOARDS; TOPNOTCH
A. Slight physical injury MD)
B. Less serious physical injury
C. Serious physical injury
D. Mutilation
E. Physical injuries inflicted in a tumultuous affray

598 The entrance wound in a gunshot can be differentiated Underlying tissues are not protruding in an TIMOTHY TANG LEE MIDTERM 1
from the exit wound with the following characteristics entrance wound. SAY, MD (TOP 4 - EXAM - FEB
EXCEPT? AUG 2013 MED 2014
A. Appears to be smaller than the missile owing to the BOARDS; TOPNOTCH
elasticity of the tissues MD)
B. Usually oval or round depedning upon the angle of
approach
C. Edges are inverted
D. Underlying tissues may be seen protruding from
the wound

TOPNOTCH MEDICAL BOARD PREP LEGAL MEDICINE AND JURIS SUPEREXAM Page 71 of 86
For inquiries visit www.topnotchboardprep.com.ph or email us at topnotchmedicalboardprep@gmail.com
TOPNOTCH MEDICAL BOARD PREP LEGAL MEDICINE AND JURIS SUPEREXAM
For inquiries visit www.topnotchboardprep.com.ph or email us at topnotchmedicalboardprep@gmail.com
Item QUESTION EXPLANATION AUTHOR TOPNOTCH
# EXAM
E. Tattoing or smudging may be present

599 A doctor, who is unmarried had an illicit relation with Since both are unmarried, it is just TIMOTHY TANG LEE MIDTERM 1
his unmarried secretray. He is guilty of? immorality. SAY, MD (TOP 4 - EXAM - FEB
A. Adultery AUG 2013 MED 2014
B. Concubinage BOARDS; TOPNOTCH
C. Infidelity MD)
D. Sodomy
E. Immorality

600 Dr. SS has illicit affairs with an 11 yr old prostitute, The woman is below 12 years old, so the TIMOTHY TANG LEE MIDTERM 1
wherein there is deceit and abuse of power. The physician is quilty of rape. SAY, MD (TOP 4 - EXAM - FEB
physicain can be charged with? AUG 2013 MED 2014
A. Rape BOARDS; TOPNOTCH
B. Qualified Seduction MD)
C. Simple Seduction
D. Ordinary Seduction
E. Immorality

601 ES just received the news that she passed the physician ES is still not a holder of a license. She should RACHELLE FINAL EXAM -
licensure exam. She then immediately set up her own have waited for the oath taking and be MENDOZA, MD (TOP FEB 2013
clinic and started seeing patients even before the oath handed her license certificate before 9 - AUG 2012 MED
taking. ES can be sued for: practicing medicine. Until then, she is still BOARDS; TOPNOTCH
A. Illegal practice of medicine considered illegally practicing medicine. MD)
B. Malpractice
C. As long as no damage is incurred by her patients, she
has no liability
D. Administrative liability

602 A 4-year old child was suffering from diarrhea. A non- Pedialyte 45 is approved as a household RACHELLE FINAL EXAM -
doctor neighbor told the mother to give Pedialyte 45. remedy by BFAD for diarrhea. Therefore, no MENDOZA, MD (TOP FEB 2013
Which describes appropriately her action? practice of medicine has been committed. 9 - AUG 2012 MED
A. Illegal practice of medicine BOARDS; TOPNOTCH
B. Malpractice MD)
C. Acting against false pretenses
D. Not considered an act constituting practice of
medicine

603 A test to prove live birth that involves checking the Breslau test - aka stomach bowel test or RACHELLE FINAL EXAM -
middle ear of a child. Before birth, this is filled with floatation test. If the organs float in water MENDOZA, MD (TOP FEB 2013
gelatinous embryonic connective tissues but disappears and air bubbles are liberated, breathing has 9 - AUG 2012 MED
after birth. This test is called: taken place. Fodere test - hydrostatic test. BOARDS; TOPNOTCH
A. Wredin Gettler's test - test used to determine MD)
B. Breslau chloride content of blood in the R and L
C. Fodere ventricles, used to determine if death
D. Winslo occured by drowning, or if death occured in
fresh water or salt water
604 Virginity is not part of the inclusion of which of the Ordinary qualified seduction requires that RACHELLE FINAL EXAM -
following: the victim is a virgin. Simple seduction and MENDOZA, MD (TOP FEB 2013
A. Qualified seduction incestuous qualified seduction do not include 9 - AUG 2012 MED
B. Simple seduction virginity as a requirement. As long as the BOARDS; TOPNOTCH
C. Qualified incestuous seduction victim is of good reputation and the crime is MD)
D. A and B committed by means of deceit.
E. B and C

605 This can be a ground for legal action which is an act of Doctrine of contributory negligence - RACHELLE FINAL EXAM -
omission, which amounts to want of care on the part of doctrine of common fault. Conduct on the MENDOZA, MD (TOP FEB 2013
the complainant, which concurring with the defendat's part of the plaintiff contributing as a legal 9 - AUG 2012 MED
negligence is the proximate cause of the injury. cause to the harm he has suffered, which falls BOARDS; TOPNOTCH
A. Doctrine of continuing negligence below the standard to which he is required to MD)
B. Doctrine of contributory negligence conform. Doctrine of continuing negligence -
C. Res ipsa loquitor if the MD after a prolonged treatment of the
D. Doctrine of ostensible agent patient fails to investigate non response or
the MD ignores an early warning with
regards to the treatment of the patient
resulting to harm. Res ipsa loquitor - the
nature of the wrongful act is suggestive of
negligence (foreign bodies left). Doctrine of
ostensible agent - pathologst, radiologist and
anesthesiologist can be considred employees
of the hospital.

TOPNOTCH MEDICAL BOARD PREP LEGAL MEDICINE AND JURIS SUPEREXAM Page 72 of 86
For inquiries visit www.topnotchboardprep.com.ph or email us at topnotchmedicalboardprep@gmail.com
TOPNOTCH MEDICAL BOARD PREP LEGAL MEDICINE AND JURIS SUPEREXAM
For inquiries visit www.topnotchboardprep.com.ph or email us at topnotchmedicalboardprep@gmail.com
Item QUESTION EXPLANATION AUTHOR TOPNOTCH
# EXAM
606 Medical malpractice is an offense that falls under the The PRC under the Board of Medicine RACHELLE FINAL EXAM -
jurisdiction of: handles all administrative cases filed against MENDOZA, MD (TOP FEB 2013
A. Philippine Medical Association doctors, inclusive of which are malparactice 9 - AUG 2012 MED
B. Board of Medicine cases. BOARDS; TOPNOTCH
C. Department of Health MD)
D. NBI

607 If the person died as a result of communicable disease A person who died with non-communicable RACHELLE FINAL EXAM -
and is unembalmed, it should be buried not later than: causes and is unembalmed should be buried MENDOZA, MD (TOP FEB 2013
A. 12 hours within 48hours. 9 - AUG 2012 MED
B. 20 hours BOARDS; TOPNOTCH
C. 24 hours MD)
D. 48 hours

608 The following describes the lungs if respiration takes There should be crepitation present when RACHELLE FINAL EXAM -
place after birth, EXCEPT: the lung tissue is compressed between the MENDOZA, MD (TOP FEB 2013
A. lungs fill the thoracic cavity and overlapping the thumb and the finger. 9 - AUG 2012 MED
ehart and thymus gland BOARDS; TOPNOTCH
B. surface is covered with mosaic of expanded air MD)
vesicles, giving a marble appearance
C. lungs are voluminous, with rounded edges and pink
mottled color
D. when squeezed between the finger and the thumb,
they do not crepitate

609 Mr. A was married to Ms. B, who is his own step Natural children - born outside a lawful RACHELLE FINAL EXAM -
daughter. The marriage is considered void from the wedlock. Natural by legal fiction - children MENDOZA, MD (TOP FEB 2013
start, hence a child born thereafter is considered: born of void marriages or voidable marriages 9 - AUG 2012 MED
A. adulterous after the decress of annulment. Natural BOARDS; TOPNOTCH
B. natural by legal fiction children by presumption - natural children MD)
C. natural child by presumption acknowledged by the father or mother
D. legitimated separately. Spurious children - illegitimate
children who are not considered natural
(adulterous, sacrilegious, incestuous,
manceres)
610 Confidentiality of information in the course of a doctor- Confidentiality may only be breached in the RACHELLE FINAL EXAM -
patient relationship may be breached in the following best interest of justice and public safety. MENDOZA, MD (TOP FEB 2013
instances: 9 - AUG 2012 MED
A. when disclosure is necessary to serve the best BOARDS; TOPNOTCH
interest of justice MD)
B. when disclosure of information will serve both the
public and private health and safety
C. when disclosure is needed in a civil case to determine
the moral character of a patient
D. when information are being asked by police
authorities
E. all of the above
611 A physician may perform a diagnostic or therapeutic Physician can only proceed with a procedure RACHELLE FINAL EXAM -
procedure without the consent of the patient or his without consent during acute emergencies, MENDOZA, MD (TOP FEB 2013
relatives wherein there is no enough time for an 9 - AUG 2012 MED
A. when the physician believes the procedure is informed consent to be discussed and the life BOARDS; TOPNOTCH
necessary of the patient may immediately be at stake. MD)
B. when the patient is unconscious
C. in an acute emergency when there is no time to
discuss the situation
D. all of the above
612 Dr. B refused to attend on several occasion a court Direct contempt - misbehavious inside the RACHELLE FINAL EXAM -
hearing citing as reason that he has several out-patients courtroom. Indirect contempt - refusing to MENDOZA, MD (TOP FEB 2013
to examine. He may be liable for: attend to court after a subpeona has been 9 - AUG 2012 MED
A. Direct contempt of the court issued, without a valid reason. Failure to BOARDS; TOPNOTCH
B. Indirect contempt of the court obey a subpeona is a ground for indirect MD)
C. Disrespect for the court contempt of the court.
D. No liability, as his reason is considered qualified and
valid

613 A 45-year old man, who works as a painter in a Chronic lead poisoning can lead to formation RACHELLE FINAL EXAM -
shipyard for 10 years now, presents with gouty tophi of tophaceous gout. MENDOZA, MD (TOP FEB 2013
and arthritis. He is probably suffering from: 9 - AUG 2012 MED
A. Hyperuricemia BOARDS; TOPNOTCH
B. Hydrocyanic poisoning MD)
C. Arsenic poisoning
D. Lead poisoning

TOPNOTCH MEDICAL BOARD PREP LEGAL MEDICINE AND JURIS SUPEREXAM Page 73 of 86
For inquiries visit www.topnotchboardprep.com.ph or email us at topnotchmedicalboardprep@gmail.com
TOPNOTCH MEDICAL BOARD PREP LEGAL MEDICINE AND JURIS SUPEREXAM
For inquiries visit www.topnotchboardprep.com.ph or email us at topnotchmedicalboardprep@gmail.com
Item QUESTION EXPLANATION AUTHOR TOPNOTCH
# EXAM
614 Examination of presence of peripheral circulation to Magnus test - application of ligature around RACHELLE FINAL EXAM -
confirm death includes a test wherein a solution of the base of a finger with moderate tightness. MENDOZA, MD (TOP FEB 2013
fluorescent dye will be injected subcutaneously. If Diaphanous test - the fingers are spread wide 9 - AUG 2012 MED
circulation is present, the dye will spread all over the and the fingers are viewed through a strong BOARDS; TOPNOTCH
body and the whole skin will have a greenish-yellow light. Teichmann's test is a test use dto MD)
discoloration. this test is called: determine if blood is human or not.
A. Icard's test
B. Diaphanous test
C. Magnus test
D. Teichmann's test

615 One of the following is NO longer true of rape under RA Offended party may be any person of any RACHELLE FINAL EXAM -
8353: gender. MENDOZA, MD (TOP FEB 2013
A. Offender may be any person 9 - AUG 2012 MED
B. Offended party must be female BOARDS; TOPNOTCH
C. Insertion of the penis into another person's mouth or MD)
anus
D. Insertion of any instrument into the genital or anal
orifice of another person
616 Which of the following laws mandates al doctors to PD 603/RA 7610 - anti child abuse law, RACHELLE FINAL EXAM -
report all cases of less serious and serious physical mandates doctors to report all cases within MENDOZA, MD (TOP FEB 2013
injuries? 48 hours from knowledge. PD 541 - gives 9 - AUG 2012 MED
A. PD 603 former Filipino doctors right to practice in BOARDS; TOPNOTCH
B. RA 7610 the Philippines again, without revoking their MD)
C. PD 189 current citizenship.
D. PD 541
E. PD 169

617 The court orderd Dr. R to produce th skull of the victim , Autoptic or real evidence - evidence made RACHELLE FINAL EXAM -
who was killed by a blow on the head caused by a blunt known or addressed to the senses (sight, MENDOZA, MD (TOP FEB 2013
weapon. The skull is what type of evidence? smell, hearing, taste, touch) of the court. 9 - AUG 2012 MED
A. Testimonial evidence Testimonial evidence - giving a testimony on BOARDS; TOPNOTCH
B. Autoptic evidence matters that the witnessed perceived. MD)
C. Real evidence
D. A and B
E. B and C

618 The following bones are used in determination of sex of Bones used to identify sex: pelvis, skull, RACHELLE FINAL EXAM -
the victim, EXCEPT: sternum, femur, humerus. Bones used to MENDOZA, MD (TOP FEB 2013
A. Pelvis estimate height: femur, humerus, tibia, 9 - AUG 2012 MED
B. Skull radius. BOARDS; TOPNOTCH
C. Sternum MD)
D. Humerus
E. Femur

619 Characteristics of menstrual blood, EXCEPT: These are all characteristics of menstrual RACHELLE FINAL EXAM -
A. Does not clot blood MENDOZA, MD (TOP FEB 2013
B. Acidic in reaction 9 - AUG 2012 MED
C. Presence of vaginal epithelial cells BOARDS; TOPNOTCH
D. Numerous deoderlein's bacillus MD)
E. No exception

620 The following are characteristics of human hair, Human hair has the following characteristics: RACHELLE FINAL EXAM -
EXCEPT: The medulla consists of fine grains of air MENDOZA, MD (TOP FEB 2013
A. Humans have large sacks of air network network, invisible hair cells without 9 - AUG 2012 MED
B. Hair cells are easily visible even without treatment treatment in water, I (relation bet diameter BOARDS; TOPNOTCH
with water. of medulla and diameter of whole hair) value MD)
C. Cortex looks like a fairly thin hollow cylinder lower than 0.3, fuzz without medulla. Cortex
D. Thick scales protruding looks like thick muff, pigments are in the
E. All are incorrect/exception form of fine grains. Cuticle is composed of
thin scales not protruding, covering one
another about 4/5.
621 Patient was aware that the physician was drunk yet he VON ANDRE DIAGNOSTIC
allowed the doctor to inject him: MEDINA, MD (TOP 4 EXAM - AUG
A. Doctrine of continuing negligence - FEB 2012 MED 2012
B. Vicarious liability BOARDS; TOPNOTCH
C. Res ipsa loquitor MD)
D. Captain of the ship
E. Doctrine of informed consent

622 Which of the following characteristics of a person is not VON ANDRE DIAGNOSTIC
easily changed? MEDINA, MD (TOP 4 EXAM - AUG
A. Hair pattern - FEB 2012 MED 2012
B. Places frequented BOARDS; TOPNOTCH
C. Speech, gait and mannerism MD)
D. Voice
E. All of the above

TOPNOTCH MEDICAL BOARD PREP LEGAL MEDICINE AND JURIS SUPEREXAM Page 74 of 86
For inquiries visit www.topnotchboardprep.com.ph or email us at topnotchmedicalboardprep@gmail.com
TOPNOTCH MEDICAL BOARD PREP LEGAL MEDICINE AND JURIS SUPEREXAM
For inquiries visit www.topnotchboardprep.com.ph or email us at topnotchmedicalboardprep@gmail.com
Item QUESTION EXPLANATION AUTHOR TOPNOTCH
# EXAM
623 The art of identification of fingerprints by comparison VON ANDRE DIAGNOSTIC
is known as: MEDINA, MD (TOP 4 EXAM - AUG
A. Dactyloscopy - FEB 2012 MED 2012
B. Dactylography BOARDS; TOPNOTCH
C. Graphology MD)
D. Poroscopy
E. A and B

624 Which of the following is the chemical test used to VON ANDRE DIAGNOSTIC
determine if the stain is of blood origin or not? MEDINA, MD (TOP 4 EXAM - AUG
A. Benzidine test - FEB 2012 MED 2012
B. Marquis test BOARDS; TOPNOTCH
C. Van urk test MD)
D. Zwikkers test
E. Icards test

625 When the body surface of corpse is pressed, it leaves a VON ANDRE DIAGNOSTIC
flattened area due to: MEDINA, MD (TOP 4 EXAM - AUG
A. Absence of edema - FEB 2012 MED 2012
B. Loss of elasticity of the skin BOARDS; TOPNOTCH
C. Precipitated skin proteins MD)
D. Absence of blood
E. Age of the body

626 A physician must appear in response to a subpoena if < 50 km VON ANDRE DIAGNOSTIC
his place of residence is how many kilometers from the MEDINA, MD (TOP 4 EXAM - AUG
court issuing the subpoena? - FEB 2012 MED 2012
A. < 80 km BOARDS; TOPNOTCH
B. <70 km MD)
C. <60 km
D. < 50 km
E. Must appear no matter what the distance

627 For a child to be legitimate, he must be born in a lawful 300 days VON ANDRE DIAGNOSTIC
wedlock or within these number of days after the MEDINA, MD (TOP 4 EXAM - AUG
dissolution of marriage: - FEB 2012 MED 2012
A. 120 days BOARDS; TOPNOTCH
B. 180 days MD)
C. 300 days
D. 360 days
E. 365 days

628 The comprehensive study of a dead body performed by autopsy VON ANDRE DIAGNOSTIC
a trained physician employing recognized procedures MEDINA, MD (TOP 4 EXAM - AUG
and techniques is known as: - FEB 2012 MED 2012
A. Post mortem examination BOARDS; TOPNOTCH
B. Autopsy MD)
C. Medical Examination
D. Post mortem dissection
E. Any of the above choices

629 Cases when trauma or disease kill quickly that there is immediate cause of death VON ANDRE DIAGNOSTIC
no opportunity for sequel or complication to develop is MEDINA, MD (TOP 4 EXAM - AUG
known as: - FEB 2012 MED 2012
A. Proximate cause of death BOARDS; TOPNOTCH
B. Secondary cause of death MD)
C. Immediate cause of death
D. Underlying cause of death
E. Complete cause of death

630 The following are characteristics of medicolegal VON ANDRE DIAGNOSTIC


autopsy EXCEPT? MEDINA, MD (TOP 4 EXAM - AUG
A. Consent of relatives are not needed - FEB 2012 MED 2012
B. Correlates tissue changes to the criminal act BOARDS; TOPNOTCH
C. Notation of all abnormal findings MD)
D. Must be specific for the purpose of determining
whether it is in relation to the criminal act
E. All of the above choices

631 If the parents are type A and type B, which of the A, B and AB VON ANDRE DIAGNOSTIC
following would be the blood group of their children. MEDINA, MD (TOP 4 EXAM - AUG
A. Type O, A and B - FEB 2012 MED 2012
B. Type A, B and AB BOARDS; TOPNOTCH
C. Type A and B MD)
D. Types A, B, AB and O
E. Any of the above choices

TOPNOTCH MEDICAL BOARD PREP LEGAL MEDICINE AND JURIS SUPEREXAM Page 75 of 86
For inquiries visit www.topnotchboardprep.com.ph or email us at topnotchmedicalboardprep@gmail.com
TOPNOTCH MEDICAL BOARD PREP LEGAL MEDICINE AND JURIS SUPEREXAM
For inquiries visit www.topnotchboardprep.com.ph or email us at topnotchmedicalboardprep@gmail.com
Item QUESTION EXPLANATION AUTHOR TOPNOTCH
# EXAM
632 Which of the following is NOT a requisite of the VON ANDRE DIAGNOSTIC
privileged information between the physician and is MEDINA, MD (TOP 4 EXAM - AUG
patient? - FEB 2012 MED 2012
A. That the information is confidential and if disclosed, BOARDS; TOPNOTCH
will tend to blacken the character of the patient MD)
B. That the person against whom the privelege is
claimed is one duly authorized to practice medicine,
surgery or obstetrics
C. That the privelege is claimed in a criminal case not
in any civil proceedings
D. That the physician acquired the information while
he was attending the patient in his professional
capacity
E.
633 A statement made by a dying patient to the attending VON ANDRE DIAGNOSTIC
physician in reference to the persons who inflicted MEDINA, MD (TOP 4 EXAM - AUG
injury on him is known as: - FEB 2012 MED 2012
A. Privileged communication BOARDS; TOPNOTCH
B. Dying declaration MD)
C. Hearsay evidence
D. Inadmissible evidence
E. None of the above

634 This prevents an individual from being admitted to the VON ANDRE DIAGNOSTIC
practice of medicine: MEDINA, MD (TOP 4 EXAM - AUG
A. Age of 19 yrs - FEB 2012 MED 2012
B. Passed the board examination BOARDS; TOPNOTCH
C. Holder of a valid certificate of registration issued by MD)
the board of medical education
D. Must not have been convicted of any offense
involving moral turpitude
E. All of the above

635 Which of the following is measured by the space and retainers fee VON ANDRE DIAGNOSTIC
time provided by the physician and not by the quality MEDINA, MD (TOP 4 EXAM - AUG
and quantity of medical services rendered: - FEB 2012 MED 2012
A. Retainers fee BOARDS; TOPNOTCH
B. Package deal fee MD)
C. Contractual fee
D. Simple fee
E. Professional fee

636 Principle applied when the medical fee is not specified: VON ANDRE DIAGNOSTIC
A. "Assumpsit on quanthum merit" MEDINA, MD (TOP 4 EXAM - AUG
B. "lex Loci contract" - FEB 2012 MED 2012
C. "Dura lex sed Lex" BOARDS; TOPNOTCH
D. "Sine Qua now" MD)
E. None of the above

637 Confidentiality of information in the course of a doctor- VON ANDRE DIAGNOSTIC


patient relationship may be breached in the following MEDINA, MD (TOP 4 EXAM - AUG
instances: - FEB 2012 MED 2012
A. When disclosure is necessary to serve the best BOARDS; TOPNOTCH
interest of justice MD)
B. When disclosure of infornation will serve both
public and private health safety
C. When disclosure is needed in a civil case to
determine the moral character of a patient
D. When information are being asked by police
authorities
E. All of the above

638 Burden of evidence needed to convict a physician for VON ANDRE DIAGNOSTIC
criminal malpractice: MEDINA, MD (TOP 4 EXAM - AUG
A. Substantial evidence - FEB 2012 MED 2012
B. Proof beyond reasonable doubt BOARDS; TOPNOTCH
C. Preponderance of evidence MD)
D. Circumstantial evidence
E. Any of the above choices

639 An otorhinolaryngologist was forced by a terrorist Assumption of risk VON ANDRE DIAGNOSTIC
group member to do a caesarian section on his wife. MEDINA, MD (TOP 4 EXAM - AUG
The physician agreed on the condition that he will be - FEB 2012 MED 2012
paid PhP 1 M. The physician maybe held liable for the BOARDS; TOPNOTCH
injuries sustained under the doctrine of: MD)
A. Foreseeability
B. Continuing negligence
C. Assumption of risk
D. Contributory negligence
E. None of the above

TOPNOTCH MEDICAL BOARD PREP LEGAL MEDICINE AND JURIS SUPEREXAM Page 76 of 86
For inquiries visit www.topnotchboardprep.com.ph or email us at topnotchmedicalboardprep@gmail.com
TOPNOTCH MEDICAL BOARD PREP LEGAL MEDICINE AND JURIS SUPEREXAM
For inquiries visit www.topnotchboardprep.com.ph or email us at topnotchmedicalboardprep@gmail.com
Item QUESTION EXPLANATION AUTHOR TOPNOTCH
# EXAM
640 When a surgeon operates on the patient without a Nominal damage VON ANDRE DIAGNOSTIC
consent, this constitues an assault and even without MEDINA, MD (TOP 4 EXAM - AUG
proof of injury resulting, the plaintiff may recover what - FEB 2012 MED 2012
type of damage? BOARDS; TOPNOTCH
A. Nominal MD)
B. Liquidated
C. Exemplary
D. Moral
E. Moderate

641 Dr. Jaw Orski was found dead on his clinic with multiple LITO JAY MACARAIG, DIAGNOSTIC
stab wounds. On autopsy, you saw prominent MD (TOP 8 - FEB EXAM - AUG
superficial veins with reddish discoloration on his 2013 MED BOARDS; 2013
flanks. This is called? TOPNOTCH MD)
A. Putrefaction
B. Maceration
C. Marbolization
D. Cadaveric lividity
E. Adipocere formation

642 Mrs. Kath Ang-Isip is a chinese national who was found LITO JAY MACARAIG, DIAGNOSTIC
soaked with her own blood in her store. If you are MD (TOP 8 - FEB EXAM - AUG
entertaining suicide, you should find 2013 MED BOARDS; 2013
A. Indications of struggle TOPNOTCH MD)
B. Hesitation cuts
C. Tentative incisions
D. A and B
E. B and C

643 This finding in death is not consistent and may or may LITO JAY MACARAIG, DIAGNOSTIC
not appear on a person who died MD (TOP 8 - FEB EXAM - AUG
A. Rigor mortis 2013 MED BOARDS; 2013
B. Algor mortis TOPNOTCH MD)
C. Putrefaction
D. Cadaveric spasm
E. Decomposition

644 In ballistics, this transforms mechanical energy by the LITO JAY MACARAIG, DIAGNOSTIC
hit of the firing pin on the percussion cap to chemical MD (TOP 8 - FEB EXAM - AUG
energy. 2013 MED BOARDS; 2013
A. Primer TOPNOTCH MD)
B. Powder
C. Bullet
D. Projectile
E. None

645 This is a head injury that is found on areas of the skull LITO JAY MACARAIG, DIAGNOSTIC
offering the least protection. MD (TOP 8 - FEB EXAM - AUG
A. Coup 2013 MED BOARDS; 2013
B. Contre-coup TOPNOTCH MD)
C. Coup contre-coup
D. Locus minoris Resistencia
E. Remote injuries

646 Which of the following is a test for the presence of LITO JAY MACARAIG, DIAGNOSTIC
powder residues on clothings? MD (TOP 8 - FEB EXAM - AUG
A. Gonzales' test 2013 MED BOARDS; 2013
B. Walker's test TOPNOTCH MD)
C. Paraffin test
D. Lung's test
E. None

647 You were walking along the street of Delli-Cado when a LITO JAY MACARAIG, DIAGNOSTIC
car accidently hit you on your left leg. The bumper- MD (TOP 8 - FEB EXAM - AUG
fracture of the leg bones in your case is a consequence 2013 MED BOARDS; 2013
of TOPNOTCH MD)
A. Run over injuries
B. Primary impact
C. Secondary impact
D. Hit and run
E. Coup injuries

TOPNOTCH MEDICAL BOARD PREP LEGAL MEDICINE AND JURIS SUPEREXAM Page 77 of 86
For inquiries visit www.topnotchboardprep.com.ph or email us at topnotchmedicalboardprep@gmail.com
TOPNOTCH MEDICAL BOARD PREP LEGAL MEDICINE AND JURIS SUPEREXAM
For inquiries visit www.topnotchboardprep.com.ph or email us at topnotchmedicalboardprep@gmail.com
Item QUESTION EXPLANATION AUTHOR TOPNOTCH
# EXAM
648 A man was hit by a truck in the legs and was thrown 10 LITO JAY MACARAIG, DIAGNOSTIC
feet from impact, subsequently hitting the pavement. As MD (TOP 8 - FEB EXAM - AUG
the attending physician, you noted abrasions, 2013 MED BOARDS; 2013
contusions, and lacerations in other parts of his body TOPNOTCH MD)
aside from leg fractures. Those injuries are called?
A. Run over injuries
B. Primary impact
C. Secondary impact
D. Hit and run
E. Coup injuries

649 The absence of smoke, soiling, burning, or powder LITO JAY MACARAIG, DIAGNOSTIC
tattoing is suggestive of this type of gunshit wound. MD (TOP 8 - FEB EXAM - AUG
A. Contact wound 2013 MED BOARDS; 2013
B. Close-range wound TOPNOTCH MD)
C. Long-range wound
D. Mid-range wound
E. None

650 When a surgeon operates on a patient without consent, LITO JAY MACARAIG, DIAGNOSTIC
this constitutes an assault. And even without proof of MD (TOP 8 - FEB EXAM - AUG
injury, the plaintiff may recover what type of damage? 2013 MED BOARDS; 2013
A. Nominal TOPNOTCH MD)
B. Moderate
C. Liquidated
D. Exemplary
E. Moral

651 Being a physician, you are given a right to limit your LITO JAY MACARAIG, DIAGNOSTIC
practice by the dictate of your conscience and religious MD (TOP 8 - FEB EXAM - AUG
beliefs. Hence, you may refuse to do which of the 2013 MED BOARDS; 2013
following? TOPNOTCH MD)
A. Tubal ligation
B. IUD insertion
C. Vasectomy
D. A and C only
E. All of the above

652 The following fees are ethical and accepted in practice, Contingent fee is adding amount of payment LITO JAY MACARAIG, DIAGNOSTIC
EXCEPT? depending on the level of satisfactory of the MD (TOP 8 - FEB EXAM - AUG
A. Professional fee result of treatment. (eg. Patient MUST give 2013 MED BOARDS; 2013
B. Retainer's fee additional one million pesos if no scar is TOPNOTCH MD)
C. Contractual fee visible). This is unethical.
D. Contingent fee
E. No excemption

653 If a baby was born alive, that baby has the right to LITO JAY MACARAIG, DIAGNOSTIC
possess legal personality. Which among the following is MD (TOP 8 - FEB EXAM - AUG
a test to prove live birth (given that the child eventually 2013 MED BOARDS; 2013
died) by aloowing the fetal lungs to float in water? TOPNOTCH MD)
A. Wredin's test
B. Icard's test
C. Breslau's test
D. Hydrostatic test
E. None

654 If a person died as a result of communicable disease, LITO JAY MACARAIG, DIAGNOSTIC
and is unembalmed, the body should be buried not later MD (TOP 8 - FEB EXAM - AUG
than? 2013 MED BOARDS; 2013
A. 12 hours TOPNOTCH MD)
B. 20 hours
C. 24 hours
D. 48 hours
E. 72 hours

655 In cases of natural death (without any suspicion of foul- LITO JAY MACARAIG, DIAGNOSTIC
play and cause of death is certainly from natural cause), MD (TOP 8 - FEB EXAM - AUG
certificate of death should be provided within 2013 MED BOARDS; 2013
A. 12 hours TOPNOTCH MD)
B. 20 hours
C. 24 hours
D. 48 hours
E. 72 hours

TOPNOTCH MEDICAL BOARD PREP LEGAL MEDICINE AND JURIS SUPEREXAM Page 78 of 86
For inquiries visit www.topnotchboardprep.com.ph or email us at topnotchmedicalboardprep@gmail.com
TOPNOTCH MEDICAL BOARD PREP LEGAL MEDICINE AND JURIS SUPEREXAM
For inquiries visit www.topnotchboardprep.com.ph or email us at topnotchmedicalboardprep@gmail.com
Item QUESTION EXPLANATION AUTHOR TOPNOTCH
# EXAM
656 In medical school curriculum, this is the study of LITO JAY MACARAIG, DIAGNOSTIC
licensure laws, regulatory laws, and physician-patient MD (TOP 8 - FEB EXAM - AUG
relationship. 2013 MED BOARDS; 2013
A. Legal Medicine TOPNOTCH MD)
B. Bioethics
C. Legal Ethics
D. Medical Jurisprudence
E. Medical Ethics

657 An HIV patient voluntarily takes experimental drugs LITO JAY MACARAIG, DIAGNOSTIC
despite of unknown side effects subsequently MD (TOP 8 - FEB EXAM - AUG
developed ischemic necrosis of the penis and have to be 2013 MED BOARDS; 2013
cut. What doctrine is applicable? TOPNOTCH MD)
A. Res Ipsa Loquitur
B. Doctrine of informed consent
C. Doctrine of foreseeability
D. Doctrine of Contributory negligence
E. Assumption of the Risk doctrine

658 A man graduated from a Medical school immediately LITO JAY MACARAIG, DIAGNOSTIC
used the title MD after his name even without passing MD (TOP 8 - FEB EXAM - AUG
the Philippine Physician Licensure Examination. This 2013 MED BOARDS; 2013
man is TOPNOTCH MD)
A. Commiting malpractice
B. Illegally practicing medicine
C. Falsely using MD after his name
D. Academically entitled to use MD
E. Has no right to do so, and is unethical

659 A fiduciary physician-patient relationship is based on LITO JAY MACARAIG, DIAGNOSTIC


A. Law MD (TOP 8 - FEB EXAM - AUG
B. Court order 2013 MED BOARDS; 2013
C. Mutual trust TOPNOTCH MD)
D. Mutual fund
E. Mutual consent

660 A statement made by a dying patient to the attending LITO JAY MACARAIG, DIAGNOSTIC
physician in referrence to the persons who inflicted MD (TOP 8 - FEB EXAM - AUG
injury on him is known as? 2013 MED BOARDS; 2013
A. Privillege communication TOPNOTCH MD)
B. Dying declaratin
C. Dying admiration
D. Hearsay evidence
E. Inadmissible evidence

661 Legal medicine is defined as? A and B - Medical Jurisprudence, C - Forensic HAZEL KAREN RAZ, MIDTERM 2 -
A. Knowledge of law in relation to practice of Medicine MD (TOP 6 - FEB AUG 2013
medicine 2013 MED BOARDS;
B. Concerned w/ rights, duties and obligations of TOPNOTCH MD)
physician w/ reference to phys-px relation
C. Application of medical science to elucidate legal
problems
D. Branch of medicine which deals with application of
medical knowledge to the purpose of law and
administration of justice
E. None of the Above
662 Type of medical evidence which is addressed to the Autopic evidence – the term refers to HAZEL KAREN RAZ, MIDTERM 2 -
senses of the court:? material evidence, presented in court. MD (TOP 6 - FEB AUG 2013
A. Physical evidence 2013 MED BOARDS;
B. Corpus adicti evidence TOPNOTCH MD)
C. Documentary evidence
D. Autopic evidence
E. None of the Above

663 Most efficient method of evidence preservation? HAZEL KAREN RAZ, MIDTERM 2 -
A. Photographs MD (TOP 6 - FEB AUG 2013
B. Drawing 2013 MED BOARDS;
C. Embalming TOPNOTCH MD)
D. Refrigiration
E. None of the Above

664 Truth serum is composed of:? HHB is a derivative of scopolamine. Because HAZEL KAREN RAZ, MIDTERM 2 -
A. Amylnitrite of a number of undesirable side effects, MD (TOP 6 - FEB AUG 2013
B. Na amytal scopolamine was shortly disqualified as a 2013 MED BOARDS;
C. Na penthotal truth drug. TOPNOTCH MD)
D. Hyoscine Hyobromide
E. None of the Above

TOPNOTCH MEDICAL BOARD PREP LEGAL MEDICINE AND JURIS SUPEREXAM Page 79 of 86
For inquiries visit www.topnotchboardprep.com.ph or email us at topnotchmedicalboardprep@gmail.com
TOPNOTCH MEDICAL BOARD PREP LEGAL MEDICINE AND JURIS SUPEREXAM
For inquiries visit www.topnotchboardprep.com.ph or email us at topnotchmedicalboardprep@gmail.com
Item QUESTION EXPLANATION AUTHOR TOPNOTCH
# EXAM
665 Expressed admission of guilt in a criminal case? In the law of criminal evidence, a confession HAZEL KAREN RAZ, MIDTERM 2 -
A. Expression is a statement by a suspect in crime which is MD (TOP 6 - FEB AUG 2013
B. Confession adverse to that person. 2013 MED BOARDS;
C. Declaration TOPNOTCH MD)
D. Swearing
E. None of the Above

666 Almond eyes, pale complexion, prominent cheekbones? HAZEL KAREN RAZ, MIDTERM 2 -
A. Hippocratic Facies - approaching death Hippocratic Facies - approaching death MD (TOP 6 - FEB AUG 2013
B. Leonine Facies - Leprosy, Elephanthiasis Leonine Facies - Leprosy, Elephanthiasis 2013 MED BOARDS;
C. Mongolian Facies Myxedemic Fascies - Hypothyroidism TOPNOTCH MD)
D. Myxedemic Fascies - Hypothyroidism
E. None of the Above

667 Verbal, accurate and picturesque description of the Portrait Parle is a French term referring to a HAZEL KAREN RAZ, MIDTERM 2 -
person identified? picture of a suspect in both front and profile MD (TOP 6 - FEB AUG 2013
A. Anthropometry views. Modernization has led this to be our 2013 MED BOARDS;
B. Bertillon System "mug shot" of a suspect. TOPNOTCH MD)
C. Drawing
D. Portrait Parle
E. None of the Above

668 Art and study of recording fingerprints as a means of The scientific study of fingerprints, which is HAZEL KAREN RAZ, MIDTERM 2 -
identification? known as dactylography, is used as a MD (TOP 6 - FEB AUG 2013
A. dactyloscopy technique of crime detection by practically 2013 MED BOARDS;
B. Locard's Method every modern law enforcement agency. TOPNOTCH MD)
C. Dactylography
D. Poroscopy
E. None of the Above

669 Used to differentiate male from female skeleton HAZEL KAREN RAZ, MIDTERM 2 -
except:? MD (TOP 6 - FEB AUG 2013
A. Pelvis 2013 MED BOARDS;
B. Tibia TOPNOTCH MD)
C. Sternum
D. Femur
E. None of the Above

670 Chemical examination used to ID blood of blood stains, Solubility test is an example of a physical test, HAZEL KAREN RAZ, MIDTERM 2 -
except:? not chemical MD (TOP 6 - FEB AUG 2013
A. Benzedine test 2013 MED BOARDS;
B. Guiacum Test TOPNOTCH MD)
C. Solubility Test
D. Kastle - Meyer Test (aka Pnenolphthalein Test)
E. None of the Above

671 Post-mortem Lividity? Livor mortis is a settling of the blood in the HAZEL KAREN RAZ, MIDTERM 2 -
A. Livor Mortis lower (dependent) portion of the body, MD (TOP 6 - FEB AUG 2013
B. Rigor Mortis causing a purplish red discoloration of the 2013 MED BOARDS;
C. Cadavenic Spasm skin. When the heart stops functioning and is TOPNOTCH MD)
D. Putrifaction no longer agitating the blood, heavy red
E. None of the Above blood cells sink through the serum by action
of gravity.

672 Instances wherein it is required to do autopsy except:? An autopsy—also known as a post-mortem HAZEL KAREN RAZ, MIDTERM 2 -
A. Violence examination, necropsy (particularly as to MD (TOP 6 - FEB AUG 2013
B. Accidents non-human bodies), autopsia cadaverum, or 2013 MED BOARDS;
C. Suicide obduction—is a highly specialized surgical TOPNOTCH MD)
D. Natural Death procedure that consists of a thorough
E. None of the Above examination of a corpse to determine the
cause and manner of death and to evaluate
any disease or injury that may be present. It
is usually performed by a specialized medical
doctor called a pathologist.
673 Exit round for bullets? Exit wounds - as we have already mentioned HAZEL KAREN RAZ, MIDTERM 2 -
A. Large, Star shaped - are usually larger than the entrance wound. MD (TOP 6 - FEB AUG 2013
B. With tatooing of smudging Exit wounds will often bleed profusely as 2013 MED BOARDS;
C. Everted Edges they are larger but entrance wounds can TOPNOTCH MD)
D. Inverted Edges sometimes look only like small holes - unless
E. None of the Above the weapon is fired at close proximity to the
victim.

674 Born out of lawful wedlock? In common law, legitimacy is the status of a HAZEL KAREN RAZ, MIDTERM 2 -
A. Legitimate child born to parents who are legally married MD (TOP 6 - FEB AUG 2013
B. Illegitimate to each other; and of a child conceived before 2013 MED BOARDS;
C. Legitimated the parents receive a legal divorce. TOPNOTCH MD)
D. Adopted Conversely, illegitimacy (or bastardy) is the
E. All of the above status of a child born outside marriage.

TOPNOTCH MEDICAL BOARD PREP LEGAL MEDICINE AND JURIS SUPEREXAM Page 80 of 86
For inquiries visit www.topnotchboardprep.com.ph or email us at topnotchmedicalboardprep@gmail.com
TOPNOTCH MEDICAL BOARD PREP LEGAL MEDICINE AND JURIS SUPEREXAM
For inquiries visit www.topnotchboardprep.com.ph or email us at topnotchmedicalboardprep@gmail.com
Item QUESTION EXPLANATION AUTHOR TOPNOTCH
# EXAM
675 Laws not enforced by the state, except? Remedial Law is that which prescribes the HAZEL KAREN RAZ, MIDTERM 2 -
A. Remedial Law manner of enforcing Legal rights and claims. MD (TOP 6 - FEB AUG 2013
B. Natural Law 2013 MED BOARDS;
C. Divine Law TOPNOTCH MD)
D. Moral Law
E. None of the above

676 Requirements for admission to practice medicine in the 21 years of age HAZEL KAREN RAZ, MIDTERM 2 -
Philippines, except? Passed the board exam MD (TOP 6 - FEB AUG 2013
A. 21 years of age Holder of certificate of registration 2013 MED BOARDS;
B. Passed the board exam TOPNOTCH MD)
C. Natural born citizen of the Philippines
D. Holder of certificate of registration
E. None of the above

677 An american surgeon on vacation joined a medical HAZEL KAREN RAZ, MIDTERM 2 -
mission in which he performed circumcision for MD (TOP 6 - FEB AUG 2013
children in Samar. He is_____: 2013 MED BOARDS;
A. Practicing medicine TOPNOTCH MD)
B. Illegally practicing medicine
C. Committing medical malpractice
D. A and c
E. None of the above

678 Grounds for reprimand, suspension, and revocation of HAZEL KAREN RAZ, MIDTERM 2 -
license includes the following except: MD (TOP 6 - FEB AUG 2013
A. Immoral conduct 2013 MED BOARDS;
B. Dishonorable conduct TOPNOTCH MD)
C. Fraud
D. Insanity
E. Sleeping while on duty

679 When a physician is found guilty of a criminal act, he HAZEL KAREN RAZ, MIDTERM 2 -
may be punished with? MD (TOP 6 - FEB AUG 2013
A. Imprisonment 2013 MED BOARDS;
B. Revocation of license TOPNOTCH MD)
C. suspension
D. fined
E. A and D

680 Doctrine of vicarious liability? HAZEL KAREN RAZ, MIDTERM 2 -


A. Responsibility of a person for negligible or Liability of hospital for negligent acts of their MD (TOP 6 - FEB AUG 2013
wrongful act of another ostensible agents - ostensible agent doctrine 2013 MED BOARDS;
B. Liability of hospital for negligent acts of their Liability of a surgeon for wrongful acts of TOPNOTCH MD)
ostensible agents those who are under his control - captain of
C. Liability of a surgeon for wrongful acts of those who the ship doctrine
are under his control The thing speaks of itself - res ipsa loquitour
D. The thing speaks of itself
E. None of the above

681 If the parents are type A and type B, which of the Answer: B. Types A, B, AB and O MICHELLE JAY MIDTERM 1 -
following would be the blood group of their children? Notes: FRANCISCO, MD AUG 2013
A. Types O, A and B Genotype --> Phenotype (TOP 9 - FEB 2013
B. Types A, B, AB and O AB -----------> AB MED BOARDS;
C. Types A and B AA & AO ----> A TOPNOTCH MD)
D. Types A, B and AB BB & BO ----> B
OO -----------> O
Type AO x BO = will produce Genotypes AB,
BO, AO, OO = Phenotypes AB, A, B, O

682 Dr. Datu performed a surgery on Mr. Kim despite not MICHELLE JAY MIDTERM 1 -
having the proper training. Negligently, he nicked the FRANCISCO, MD AUG 2013
colon and cut the ureter while removing the appendix. (TOP 9 - FEB 2013
Mr. Kim asked Dr. Datu for monetary compensation. MED BOARDS;
The nature of liability against Dr. Datu would be: TOPNOTCH MD)
A. Civil
B. Administrative
C. Criminal
D. Ethical

TOPNOTCH MEDICAL BOARD PREP LEGAL MEDICINE AND JURIS SUPEREXAM Page 81 of 86
For inquiries visit www.topnotchboardprep.com.ph or email us at topnotchmedicalboardprep@gmail.com
TOPNOTCH MEDICAL BOARD PREP LEGAL MEDICINE AND JURIS SUPEREXAM
For inquiries visit www.topnotchboardprep.com.ph or email us at topnotchmedicalboardprep@gmail.com
Item QUESTION EXPLANATION AUTHOR TOPNOTCH
# EXAM
683 You evaluated a 65 yo man with 3-month history of MICHELLE JAY MIDTERM 1 -
chest pains and fainting spells that you feel would merit FRANCISCO, MD AUG 2013
cardiac catheterization. After informing him fully of the (TOP 9 - FEB 2013
benefits, consequences, risks, etc., he was able to MED BOARDS;
demonstrate that he understands all of these and TOPNOTCH MD)
refuses the intervention. What should you do?
A. Respect his choice
B. Explore reasons for his decision and try to convince
him again
C. Consider the discussion ended
D. Continue with the procedure nevertheless

684 Which of the following statements is/are true of faith MICHELLE JAY MIDTERM 1 -
healing in relation to the practice of medicine: FRANCISCO, MD AUG 2013
A. If a person acted pursuance of his religious belief and (TOP 9 - FEB 2013
the act is in accordance with the tenets of his church, it MED BOARDS;
is deemed to be part of his religious freedom TOPNOTCH MD)
B. The Medical Act of 1959 exempts faith healing from
the definition of the acts which constitute practice of
medicine
C. The faith healer can practice even without a
certification of registration from the Board of Medicine
D. He can invoke his right to livelihood as the basis for
his performing faith healing

685 A 35 yo old lingerie model came to your clinic with chief MICHELLE JAY MIDTERM 1 -
complaint of foul-smelling vaginal discharge. FRANCISCO, MD AUG 2013
Examination revealed gram-negative diplococcic. She (TOP 9 - FEB 2013
told you that the treatment should be kept in strict MED BOARDS;
confidentiality. Furthermore, she revealed that she got TOPNOTCH MD)
her illness from a promiscuous lover. She placed rat
poison in his lover’s tomato juice before she left to see
you. You promised that you will keep it between the
two of you. The following morning, you learned that
your patient’s lover died of internal hemorrhage which
can be attributed to a rat poison. What will you do if
summoned to testify on the death of your patient’s
lover?
A. You will raise the defense of a privileged
communication.
B. You will raise the right of the patient for privacy.
C. You will raise the issue of your right to limit practice,
not in a medico-legal practice.
D. You will testify.

686 A bullet that has penetrated the skin and hits bony MICHELLE JAY MIDTERM 1 -
tissue along its course may deflect and change its FRANCISCO, MD AUG 2013
direction. Such change of trajectory is called: (TOP 9 - FEB 2013
A. Russian roulette MED BOARDS;
B. Ricochet of bullet TOPNOTCH MD)
C. Bullet sporting movement
D. Missile disintegrating movement

687 A 34 yo female came to your clinic with chief complaint MICHELLE JAY MIDTERM 1 -
of severe chest pain. History revealed an abrupt onset FRANCISCO, MD AUG 2013
after reading a text message from her husband’s former (TOP 9 - FEB 2013
girlfriend. Diagnostics revealed essentially normal MED BOARDS;
findings; however physical examination revealed TOPNOTCH MD)
costochondral tenderness which is converted to a more
serious disability or exaggeration of the real complaint.
This is known as:
A. Fortuitous malingering
B. Simulated malingering
C. Factitious malingering
D. Fictitious malingering

688 In the ordinary method of human identification MICHELLE JAY MIDTERM 1 -


applicable to the living person, the “facies” is a FRANCISCO, MD AUG 2013
characteristic not easily changed. Which appearance of (TOP 9 - FEB 2013
the face indicative of approaching death? MED BOARDS;
A. Leonine facies TOPNOTCH MD)
B. Mongolian facies
C. Myxedemic facies
D. Hippocratic facies

TOPNOTCH MEDICAL BOARD PREP LEGAL MEDICINE AND JURIS SUPEREXAM Page 82 of 86
For inquiries visit www.topnotchboardprep.com.ph or email us at topnotchmedicalboardprep@gmail.com
TOPNOTCH MEDICAL BOARD PREP LEGAL MEDICINE AND JURIS SUPEREXAM
For inquiries visit www.topnotchboardprep.com.ph or email us at topnotchmedicalboardprep@gmail.com
Item QUESTION EXPLANATION AUTHOR TOPNOTCH
# EXAM
689 Annabelle scratched Nadia in the face in an altercation. MICHELLE JAY MIDTERM 1 -
Dr. Guttier attended to the injury that resolved in less FRANCISCO, MD AUG 2013
than 10 days but a resultant keloid scar formed. The (TOP 9 - FEB 2013
legal classification of this injury is: MED BOARDS;
A. Serious physical injuries TOPNOTCH MD)
B. Less serious injuries
C. Slight injuries
D. Mutilation

690 As a result of the deformity she sustained from facelift MICHELLE JAY MIDTERM 1 -
operation, Ethel B. sued Dr. Belu for mental anguish, FRANCISCO, MD AUG 2013
social humiliation and moral shock. What type of (TOP 9 - FEB 2013
damage can Ethel B. recover from this situation? MED BOARDS;
A. Actual damages TOPNOTCH MD)
B. Compensatory damages
C. Exemplary damages
D. Moral damages

691 The following are acts considered as immoral or MICHELLE JAY MIDTERM 1 -
dishonourable conduct under the Code of Medical FRANCISCO, MD AUG 2013
Ethics: (TOP 9 - FEB 2013
A. Fee splitting MED BOARDS;
B. Insanity TOPNOTCH MD)
C. Conviction of crime
D. Drug addiction

692 A 50 y.o female had breast surgery for CA of breast with MICHELLE JAY MIDTERM 1 -
metastasis. She consented in writing to undergo a trial FRANCISCO, MD AUG 2013
of a new chemotherapeutic modality. The patient (TOP 9 - FEB 2013
continued to deteriorate and sued the physician. The MED BOARDS;
case was absolved under the doctrine of: TOPNOTCH MD)
A. Continuing negligence
B. Common knowledge
C. Assumption of risk
D. Contributory negligence

693 A patient-physician relationship does not contemplate MICHELLE JAY MIDTERM 1 -


one of the ff. promises or guaranties: FRANCISCO, MD AUG 2013
A. Treatment will be beneficial (TOP 9 - FEB 2013
B. Treatment will not be injurious to the patient MED BOARDS;
C. Treatment will result in definite outcome TOPNOTCH MD)
D. Treatment will be gratuitous

694 The chemical test to determine if the stain is of blood MICHELLE JAY MIDTERM 1 -
origin or not. FRANCISCO, MD AUG 2013
A. Marquis test (TOP 9 - FEB 2013
B. Van Urk test MED BOARDS;
C. Benzidine test TOPNOTCH MD)
D. Zwickers test

695 Considered as a prominent sign of death: MICHELLE JAY MIDTERM 1 -


A. Progressive fall of body temperature FRANCISCO, MD AUG 2013
B. Cessation of heart action and circulation (TOP 9 - FEB 2013
C. Cessation of respiration MED BOARDS;
D. Insensibility of body loss of the power to move TOPNOTCH MD)

696 The right of a citizen of foreign country to be allowed MICHELLE JAY MIDTERM 1 -
to practice medicine in the Philippines is based on: FRANCISCO, MD AUG 2013
A. Reciprocity provisions of the medical law of the (TOP 9 - FEB 2013
Philippines MED BOARDS;
B. International law, inasmuch as the Phil. has adopted TOPNOTCH MD)
the generally accepted
C. International convention like W.H.O.
D. Executive agreement between the heads of state

697 All but one type of injury need not be reported to police MICHELLE JAY MIDTERM 1 -
authorities under PD 169: FRANCISCO, MD AUG 2013
A. Serious physical injuries (TOP 9 - FEB 2013
B. Multiple physical injuries MED BOARDS;
C. Slight physical injuries TOPNOTCH MD)
D. Less serious physical injuries

698 Weapons, fingerprints and garments left at the crime MICHELLE JAY MIDTERM 1 -
scene are examples of what type of evidence? FRANCISCO, MD AUG 2013
A. Real (TOP 9 - FEB 2013
B. Ordinary MED BOARDS;
C. Corpus delicti TOPNOTCH MD)
D. Associative

TOPNOTCH MEDICAL BOARD PREP LEGAL MEDICINE AND JURIS SUPEREXAM Page 83 of 86
For inquiries visit www.topnotchboardprep.com.ph or email us at topnotchmedicalboardprep@gmail.com
TOPNOTCH MEDICAL BOARD PREP LEGAL MEDICINE AND JURIS SUPEREXAM
For inquiries visit www.topnotchboardprep.com.ph or email us at topnotchmedicalboardprep@gmail.com
Item QUESTION EXPLANATION AUTHOR TOPNOTCH
# EXAM
699 Dr. Bu Tan Deng, the MHO of a known municipality in MICHELLE JAY MIDTERM 1 -
the outskirts of Central Visayas, shall perform an FRANCISCO, MD AUG 2013
autopsy on a dead body when: (TOP 9 - FEB 2013
A. Requested by any relative of the family MED BOARDS;
B. Verbally directed by Chief of Police TOPNOTCH MD)
C. Upon order of the Mayor
D. Ordered by the Provincial Medical Health Director

700 When a blow to the forehead causes contusion of the MICHELLE JAY MIDTERM 1 -
eyeball due to fracture of the bone at the roof of the FRANCISCO, MD AUG 2013
orbit, the type injury incurred is: (TOP 9 - FEB 2013
A. Coup MED BOARDS;
B. Contre-coup TOPNOTCH MD)
C. Coup contre-coup
D. Locus minoris resistencia

TOPNOTCH MEDICAL BOARD PREP LEGAL MEDICINE AND JURIS SUPEREXAM Page 84 of 86
For inquiries visit www.topnotchboardprep.com.ph or email us at topnotchmedicalboardprep@gmail.com
TOPNOTCH MEDICAL BOARD PREP LEGAL MEDICINE AND JURIS SUPEREXAM
For inquiries visit www.topnotchboardprep.com.ph or email us at topnotchmedicalboardprep@gmail.com

Item # ANSWER 88 A 176 D 264 C 352 D


1 A 89 C 177 B 265 B 353 B
2 B 90 C 178 C 266 A 354 B
3 B 91 B 179 D 267 D 355 B
4 D 92 D 180 C 268 B 356 B
5 D 93 B 181 C 269 B 357 C
6 C 94 E 182 A 270 A 358 B
7 E 95 E 183 C 271 C 359 A
8 B 96 B 184 C 272 B 360 D
9 C 97 E 185 A 273 A 361 B
10 D 98 B 186 A 274 D 362 A
11 A 99 C 187 D 275 A 363 B
12 A 100 A 188 B 276 C 364 C
13 B 101 E 189 C 277 E 365 E
14 A 102 A 190 B 278 A 366 B
15 C 103 B 191 B 279 C 367 B
16 A 104 C 192 B 280 A 368 B
17 D 105 D 193 D 281 B 369 A
18 E 106 C 194 A 282 A 370 B
19 D 107 E 195 C 283 B 371 E
20 A 108 B 196 E 284 E 372 E
21 C 109 C 197 B 285 B 373 C
22 A 110 E 198 E 286 D 374 E
23 C 111 C 199 C 287 A 375 D
24 D 112 D 200 C 288 C 376 B
25 D 113 D 201 B 289 B 377 E
26 E 114 C 202 D 290 A 378 C
27 D 115 A 203 D 291 D 379 A
28 A 116 C 204 B 292 B 380 C
29 B 117 B 205 C 293 A 381 D
30 D 118 A 206 B 294 D 382 C
31 A 119 D 207 D 295 E 383 C
32 D 120 E 208 C 296 A 384 C
33 C 121 E 209 B 297 C 385 A
34 C 122 B 210 C 298 A 386 C
35 B 123 D 211 E 299 C 387 B
36 C 124 C 212 A 300 B 388 C
37 D 125 B 213 B 301 A 389 B
38 B 126 C 214 C 302 C 390 D
39 A 127 B 215 C 303 C 391 C
40 C 128 A 216 D 304 A 392 C
41 E 129 C 217 C 305 B 393 D
42 B 130 D 218 D 306 A 394 A
43 A 131 C 219 B 307 C 395 A
44 B 132 E 220 D 308 C 396 A
45 B 133 D 221 C 309 B 397 A
46 D 134 A 222 B 310 B 398 B
47 C 135 B 223 D 311 B 399 C
48 A 136 A 224 D 312 C 400 D
49 C 137 B 225 A 313 D 401 C
50 E 138 D 226 B 314 A 402 B
51 A 139 A 227 B 315 A 403 C
52 E 140 A 228 B 316 A 404 A
53 D 141 A,B 229 B 317 B 405 D
54 E 142 A 230 D 318 D 406 B
55 C 143 C 231 A 319 D 407 C
56 C 144 B 232 D 320 C 408 E
57 D 145 C 233 C 321 C 409 C
58 E 146 B 234 B 322 D 410 D
59 E 147 D 235 A 323 C 411 A
60 B 148 A 236 D 324 A 412 A
61 B 149 D 237 A 325 A 413 A
62 C 150 E 238 B 326 B 414 A
63 B 151 A 239 B 327 C 415 A
64 D 152 C 240 B 328 A 416 B
65 A 153 A 241 B 329 C 417 B
66 E 154 E 242 A 330 A 418 D
67 C 155 B 243 C 331 B 419 A
68 E 156 C 244 C 332 C 420 C
69 A 157 A 245 C 333 C 421 D
70 C 158 A, E 246 D 334 A 422 B
71 D 159 A 247 D 335 D 423 C
72 A 160 B 248 B 336 A 424 A
73 ABCD 161 C 249 D 337 A 425 B
74 A 162 B 250 D 338 B 426 D
75 E 163 B 251 A 339 C 427 C
76 E 164 D 252 E 340 B 428 C
77 D 165 E 253 C 341 C 429 D
78 B 166 D 254 D 342 C 430 B
79 ABCD 167 A 255 B 343 C 431 A
80 B 168 C 256 D 344 A 432 B
81 B 169 A 257 C 345 A 433 A
82 B 170 B 258 B 346 C 434 C
83 C 171 C 259 A 347 B 435 D
84 C 172 B, C 260 A 348 C 436 C
85 A 173 B 261 D 349 A 437 A
86 D 174 E 262 C 350 A 438 B
87 C 175 E 263 A 351 A 439 D
TOPNOTCH MEDICAL BOARD PREP LEGAL MEDICINE AND JURIS SUPEREXAM Page 85 of 86
For inquiries visit www.topnotchboardprep.com.ph or email us at topnotchmedicalboardprep@gmail.com
TOPNOTCH MEDICAL BOARD PREP LEGAL MEDICINE AND JURIS SUPEREXAM
For inquiries visit www.topnotchboardprep.com.ph or email us at topnotchmedicalboardprep@gmail.com
440 A 529 C 618 D
441 B 530 A 619 E
442 C 531 A 620 E
443 A 532 B 621 C
444 C 533 D 622 C
445 A 534 B 623 A
446 B 535 A 624 A
447 E 536 D 625 B
448 B 537 C 626 D
449 C 538 B 627 C
450 B 539 B 628 B
451 B 540 A 629 C
452 A 541 D 630 C
453 E 542 E 631 D
454 D 543 D 632 C
455 C 544 C 633 B
456 B 545 C 634 A
457 D 546 D 635 A
458 C 547 C 636 A
459 B 548 C 637 A
460 B 549 E 638 B
461 C 550 B 639 C
462 E 551 E 640 A
463 B 552 C 641 C
464 D 553 B 642 E
465 A 554 B 643 D
466 E 555 D 644 A
467 B 556 A 645 D
468 C 557 E 646 B
469 A 558 E 647 B
470 D 559 B 648 C
471 D 560 D 649 C
472 B 561 A 650 A
473 A 562 B 651 E
474 D 563 B 652 D
475 B 564 A 653 D
476 A 565 A 654 A
477 C 566 D 655 D
478 B 567 A 656 D
479 C 568 D 657 E
480 A 569 B 658 D
481 A 570 A 659 C
482 C 571 B 660 B
483 C 572 D 661 D
484 A 573 A 662 D
485 B 574 A 663 A
486 D 575 C 664 D
487 D 576 D 665 B
488 B 577 C 666 C
489 C 578 B 667 D
490 D 579 B 668 C
491 A 580 D 669 B
492 A 581 E 670 C
493 A 582 B 671 A
494 D 583 B 672 D
495 A 584 A 673 C
496 B 585 D 674 B
497 A 586 B 675 A
498 D 587 E 676 C
499 C 588 C 677 B
500 A 589 D 678 E
501 A 590 B 679 E
502 B 591 C 680 A
503 A 592 B 681 B
504 C 593 E 682 A
505 D 594 A 683 A
506 A 595 C 684 A
507 E 596 D 685 D
508 A 597 B 686 B
509 D 598 D 687 D
510 B 599 E 688 D
511 C 600 A 689 A
512 C 601 A 690 D
513 A 602 D 691 A
514 E 603 A 692 C
515 E 604 E 693 D
516 B 605 B 694 C
517 A 606 B 695 A
518 D 607 A 696 A
519 C 608 D 697 C
520 A 609 B 698 C
521 D 610 A 699 C
522 C 611 C 700 D
523 A 612 B
524 B 613 D
525 B 614 A
526 B 615 B
527 D 616 E
528 C 617 E
TOPNOTCH MEDICAL BOARD PREP LEGAL MEDICINE AND JURIS SUPEREXAM Page 86 of 86
For inquiries visit www.topnotchboardprep.com.ph or email us at topnotchmedicalboardprep@gmail.com
TOPNOTCH MEDICAL BOARD PREP PATHOLOGY SUPEREXAM
For inquiries visit www.topnotchboardprep.com.ph or email us at topnotchmedicalboardprep@gmail.com
DEAR TOPNOTCH FRIENDS:

PLEASE FOLLOW THESE INSTRUCTIONS:

1. These questions are previous diagnostic, midterm, and finals exams of Topnotch, almost all of them made by Topnotch Board Exam Topnotchers.
2. Answer this Topnotch Superexam seriously 100-items at a time. Cover the “Explanations” Column. Do not immediately look at the answers from the
answer key. That’s not the correct way of answering sample exams. You need to treat these MCQs as exercises and not as handouts.
3. Time yourself. 1.5 hours per 100-item block.
4. After answering each 100-item block, refer to the Topnotch Answer Key for the correct answers. Please be careful of “frameshift mutations” when
checking your answers – check every 10 items. (the format of the answer key was designed for you to practice against “frameshift mutations”)
5. The Topnotch Superexams are EXERCISES for the actual med boards. They will not appear verbatim in your future exams. More than knowing what’s
the correct answer, it’s more important for you to:
a. Know why the other choices are wrong
b. Know why the other choices were included in the first place
c. Know the explanation to the correct answer
6. Sharpen your mind by answering the Topnotch Superexams. Most of these questions based on past feedback are more difficult than the actual questions
in the med boards. In these exams made by Board Exam Topnotchers, if you’re getting a score of 60/100 , that’s already a good score. More than 80/100
is outstanding.

Item QUESTION EXPLANATION AUTHOR TOPNOTCH
# EXAM
1 Which of the following is least likely to facilitate TGF - transforming growth factor. KRISTEL TANHUI DIAGNOSTIC
chemotaxis: (TOP 3 - AUG 2015 EXAM - MARCH
A. LTB4 Questions in the boards can sometimes be phrased as MED BOARDS; 2016
B. IL8 “least likely or most likely”, so try to accustom TOPNOTCH MD
C. C5a yourself to choosing the best answer in such FROM LA SALLE)
D. TGF circumstances.
E. N-formylmethionine
Source: Robbins and Cotran Pathologic Basis of
Disease 8th ed p. 50
2 The following are anaphylotoxins The anaphylatoxins are components of the KRISTEL TANHUI DIAGNOSTIC
A. C3a complement system which are involved in (TOP 3 - AUG 2015 EXAM - MARCH
B. C4a anaphylaxis. Source: Robbins and Cotran Pathologic MED BOARDS; 2016
C. C5a Basis of Disease 8th ed p. 57 TOPNOTCH MD
D. A and C only FROM LA SALLE)
E. All of the above

3 A 56 year old hypertensive male presented with Infarcts in the brain result to liquefactive necrosis KRISTEL TANHUI DIAGNOSTIC
left sided hemiparesis on waking up. A CT scan while those in all other organs except the brain exhibit (TOP 3 - AUG 2015 EXAM - MARCH
was done which revealed an infarct in the coagulative necrosis. MED BOARDS; 2016
distribution of the MCA. Which of the following TOPNOTCH MD
is the expected gross pathological finding in the Source: Robbins and Cotran Pathologic Basis of FROM LA SALLE)
brain? Disease 8th ed p. 15
A. Coagulative necrosis
B. Caseous necrosis
C. Liquefactive necrosis
D. Fibrinoid necrosis
E. Gangrenous necrosis
4 A 45 year old woman presents with recurrent This is a case of CML. It is associated with 9:22 KRISTEL TANHUI DIAGNOSTIC
infections and on PE was found to have marked translocation and may be treated with imatinib (TOP 3 - AUG 2015 EXAM - MARCH
splenomegaly. Her leukocyte count is increased mesylate. MED BOARDS; 2016
to 300,000. The differential count reveals the Basophilia is a rare finding. It is strongly indicative of TOPNOTCH MD
presence of myeloblasts and promyelocytes, CML. FROM LA SALLE)
with predominance of myelocytes,
metamyelocytes, bands and segmented There are a couple of conditions that cause increased
neutrophils. Basophils are also increased in WBC, either a leukemia or an infection. The increase in
number. The patient is not anemic. Leukocyte WBC count due to an infection is called a leukemoid
alkaline phosphatase is decreased. Which of the reaction. This can be differentiated from CML via the
following describes a major characteristic of leukocyte alkaline phosphatase test. In a leukemoid
this disorder? reaction LAP is increased.
A. 9:22 translocation
B. Expansion of mature B lymphocytes within Source: Topnotch handout on Pathology.
multiple lymph nodes
C. Hypogammaglobulinemia
D. Neoplastic cells exhibiting hair-like
filamentous projections
E. Peak incidence occurs at 65 years

5 A 50 year old man seeks consult due to a This is a case of Mycosis fungoides, which is a T cell KRISTEL TANHUI DIAGNOSTIC
pruritic rash which he has had over the past 8 lymphoma of the skin. Atypical CD4+ T cells with (TOP 3 - AUG 2015 EXAM - MARCH
months. On PE, there were erythematous, cerebriform nuclei are found on biopsy. The disorder MED BOARDS; 2016
eczematoid patches and raised plaques may remain confined to the skin for several years. TOPNOTCH MD
distributed asymmetrically over the chest and When the neoplastic cells invade the skin and become FROM LA SALLE)
abdomen. On biopsy of the lesions, atypical systemic, this is called Sezary syndrome. Sezary
CD4+ T cells with cerebriform nuclei were syndrome is the leukemic form of this cutaneous T cell
found. What is a possible outcome in the course lymphoma and is characterized by the combination of
of this condition? skin lesions and circulating neoplastic cells.
A. Acute leukemia
B. Myelofibrosis Acute leukemia and myelofibrosis are courses in the
C. Sezary Syndrome natural history of the myeloproliferative syndromes.
D. A and B only
E. All of the above Source: Topnotch handout on Pathology

TOPNOTCH MEDICAL BOARD PREP PATHOLOGY SUPEREXAM Page 1 of 99


For inquiries visit www.topnotchboardprep.com.ph or email us at topnotchmedicalboardprep@gmail.com
TOPNOTCH MEDICAL BOARD PREP PATHOLOGY SUPEREXAM
For inquiries visit www.topnotchboardprep.com.ph or email us at topnotchmedicalboardprep@gmail.com
Item QUESTION EXPLANATION AUTHOR TOPNOTCH
# EXAM
6 A 70 year male presented with a 12 hour 0-4hrs: None to Variable waviness of the fiber KRISTEL TANHUI DIAGNOSTIC
history of anginal chest pain. The stat troponin I 1-3 days: neutrophilic infiltrate (TOP 3 - AUG 2015 EXAM - MARCH
and ECG studies confirm the diagnosis of acute 3-7 days: macrophage infiltrate MED BOARDS; 2016
myocardial infarction. Unfortunately, he 1-2 wks: granulation tissue TOPNOTCH MD
succumbed to a fatal arrythmia during his 30 >2 mos: dense collagenous scar FROM LA SALLE)
minute minute stay in the ER before a definitive
intervention could be applied. What are the Source: Robbins and Cotran Pathologic Basis of
expected microscopic findings on LM for his Disease 8th ed p. 550
heart?
A. Dense collagenous scar
B. Variable waviness of the fiber
C. Coagulation necrosis with neutrophilic
infiltrate
D. Beginning disintegration of muscle fibers
with macrophage infiltrate
E. None, it’s too early for histopathologic
changes to occur.
7 Which of the following findings is an unlikely Trea-bark appearance of the ascending aorta is a KRISTEL TANHUI DIAGNOSTIC
finding in malignant hypertension? characteristic of tertiary syphilis. (TOP 3 - AUG 2015 EXAM - MARCH
A. Multiple punctate hemorrhage on the surface Source: Robbins and Cotran Pathologic Basis of MED BOARDS; 2016
of both kidneys Disease 8th ed p. 950 TOPNOTCH MD
B. Trea-bark appearance of the ascending aorta FROM LA SALLE)
C. Fibrinoid necrosis of arterioles
D. Onion skinning of arterioles
E. None of the above

8 A 65 year old female presents with fever, This is a case of Giant cell arteritis. Along with KRISTEL TANHUI DIAGNOSTIC
headache and diplopia. On palpation, involvement of the branches of the carotid artery, (TOP 3 - AUG 2015 EXAM - MARCH
tenderness is noted along the course of the there is a close association with polymyalgia MED BOARDS; 2016
temporal artery. A biopsy would most likely rheumatica. TOPNOTCH MD
confirm which of the following diagnosis? FROM LA SALLE)
A. Takayasu arteritis Takayasu arteritis is pulseless disease. It is more
B. Retinoblastoma common in women of childbearing age. Initial
C. Giant cell arteritis symptoms are usually nonspecific, including fatigue,
D. Kaposi sarcoma weight loss, and fever. With progression, vascular
E. Katayama disease symptoms appear and dominate the clinical picture,
including reduced blood pressure and weaker pulses
in the upper extremities; ocular disturbances,
including visual defects, retinal hemorrhages, and
total blindness; and neurologic deficits. Involvement
of the more distal aorta may lead to claudication of the
legs; pulmonary artery involvement may cause
pulmonary hypertension. Narrowing of the coronary
ostia may lead to myocardial infarction, and
involvement of the renal arteries leads to systemic
hypertension in roughly half of patients.

Source: Robbins and Cotran Pathologic Basis of
Disease 8th ed p. 512
9 A 50 year old male was recently diagnosed with Tumor necrosis factor or cachectin is secreted by KRISTEL TANHUI DIAGNOSTIC
PTB. On physical examination, he appears activated macrophages and is also responsible for the (TOP 3 - AUG 2015 EXAM - MARCH
emaciated. Which of the following is also called cachexia in cancer patients. MED BOARDS; 2016
cachectin and is responsible for the weight loss TOPNOTCH MD
and wasting noted in the patient? Source: Robbins and Cotran Pathologic Basis of FROM LA SALLE)
A. IL1 Disease 8th ed p. 320
B. IL2
C. PAF
D. TGF-beta
E. TNF

10 Pneumoconiosis refers to nonneoplastic lung Silicosis – upper lobes, eggshell calcification, PTB KRISTEL TANHUI DIAGNOSTIC
reaction to inhalation of mineral dusts (TOP 3 - AUG 2015 EXAM - MARCH
encountered in the work place. Which Source: Robbins and Cotran Pathologic Basis of MED BOARDS; 2016
pneumoconiosis is characterized by discrete Disease 8th ed p. 699 TOPNOTCH MD
pale to blackened nodules in the upper zones of FROM LA SALLE)
the lungs. Radiographically it presents as
eggshell calcifications. This disease may be
progressive even if patient is no longer exposed
and it has also been associated with increased
susceptibility to PTB.
A. Coal workers’ pneumoconiosis
B. Silicosis
C. Asbestosis
D. Byssinosis
E. Sarcoidosis

TOPNOTCH MEDICAL BOARD PREP PATHOLOGY SUPEREXAM Page 2 of 99


For inquiries visit www.topnotchboardprep.com.ph or email us at topnotchmedicalboardprep@gmail.com
TOPNOTCH MEDICAL BOARD PREP PATHOLOGY SUPEREXAM
For inquiries visit www.topnotchboardprep.com.ph or email us at topnotchmedicalboardprep@gmail.com
Item QUESTION EXPLANATION AUTHOR TOPNOTCH
# EXAM
11 A 40 yr old male presents with fever and cough Patient has bacterial pneumonia. Streptococcus is a KRISTEL TANHUI DIAGNOSTIC
of 5 days duration. PE revealed increased popular agent for causing lobar pneumonia. (TOP 3 - AUG 2015 EXAM - MARCH
bronchial breath sounds over the lower Stages of inflammatory response in lobar pneumonia: MED BOARDS; 2016
segment of the right lobe posteriorly. Chest - Congestion: red, heavy, boggy lung TOPNOTCH MD
xray showed lobar consolidation on the right - Red hepatization: massive, confluent exudation with FROM LA SALLE)
lower lobe and culture was positive for neutrophils, red cells and fibrin
pneumonococcus. Which is the prominent - Gray hepatization
inflammatory cells of this exudate? - Resolution
A. Platelets
B. Basophils Source: Robbins and Cotran Pathologic Basis of
C. Eosinophils Disease 8th ed p.712
D. Neutrophils
E. Macrophage

12 19 year old male presents with acute onset When you study for patho and all basic sciences, try to KRISTEL TANHUI DIAGNOSTIC
hematuria, oliguria and periorbital edema make sure you know how the disease will present (TOP 3 - AUG 2015 EXAM - MARCH
which is worse in the morning and gradually clinically cause they like to correlate things. The MED BOARDS; 2016
improves through the day. On PE, blood hardcore micro and biochem or patho knowledge will TOPNOTCH MD
pressure is elevated and urinalysis shows trace only get you halfway cause sometimes they will not FROM LA SALLE)
proteinuria, and many RBCs. ASOT is negative give you the diagnosis.
and DNAse is positive. Which is the expected
finding in light microscopy for this patient? This is a case of PSGN. ASOT may be negative if the
A. Diffuse endocapillary proliferation nephritic strain comes from a skin infection because
B. Subepithelial humps skin lipids bind to streptolysin O. DNAse is the most
C. Normal appearing sensitive test for skin infection with Group A
D. Dense deposits streptococcus. The patient in this case presented with
E. Extracapillary proliferation of crescents the classic nephritic syndrome of hypertension,
hematuria and oliguria.
A and B are both PSGN but B is an electron microscopy
finding.

Source: Robbins and Cotran Pathologic Basis of
Disease 8th ed p.918
13 Patient sought consult for chest pain and Don’t just focus on the electron and LM findings and KRISTEL TANHUI DIAGNOSTIC
hemoptysis associated with hematuria, and sacrifice immunofluorescence. SIMILAR TO PREVIOUS (TOP 3 - AUG 2015 EXAM - MARCH
signs and symptoms of uremia. His blood BOARD EXAM CONCEPT/PRINCIPLE. MED BOARDS; 2016
pressure is elevated and he has grade III TOPNOTCH MD
bipedal edema. As the clinician in charge, you A rule of thumb is if the pathophysiology involves FROM LA SALLE)
suspect that he is suffering from an immune complex deposition, it would usually present
autoimmune disease in which antibodies with a granular pattern. If the pathophysiology
against type IV collagen attack the basement involves antibodies against basement membranes, the
membrane of the lungs and kidneys. On pattern is usually linear.
immunofluorescence renal biopsy would most
probably reveal: This is actually a case of Goodpasture syndrome which
A. Granular IgG and C3 in GBM and mesangium involves antibodies against the basement membrane
B. Linear IgG and C3 of the lungs and the kidneys and the findings on
C. Negative immunofluorescent studies show linear IgG and C3.
D. Focal IgM or C3
E. IgA in the mesangium Source: Robbins and Cotran Pathologic Basis of
Disease 8th ed p. 709, 918

14 Patent urachus is a risk factor for which type of Transitional cell carcinoma is associated with KRISTEL TANHUI DIAGNOSTIC
carcinoma of the bladder? smoking. (TOP 3 - AUG 2015 EXAM - MARCH
A. Transitional cell carcinoma Squamous cell carcinoma is associated with chronic MED BOARDS; 2016
B. Squamous cell carcinoma irritation and S. haematobium infection. TOPNOTCH MD
C. Adenocarcinoma FROM LA SALLE)
D. Clear cell carcinoma Source: Robbins and Cotran Pathologic Basis of
E. Sarcoma Disease 8th ed p. 979

15 In which of the following nutritional deficiency Kwashiorkor is a protein deficiency while marasmus a KRISTEL TANHUI DIAGNOSTIC
is hepatic steatosis an expected finding? balanced deficiency of all macronutrients. (TOP 3 - AUG 2015 EXAM - MARCH
A. Vitamin A deficiency MED BOARDS; 2016
B. Vitamin E deficiency Source: Robbins and Cotran Pathologic Basis of TOPNOTCH MD
C. Copper deficiency Disease 8th ed p. 429 FROM LA SALLE)
D. Kwashiorkor
E. Marasmus

16 Which of the following is not a characteristic of Benign ulcers may have mucosal margins which KRISTEL TANHUI DIAGNOSTIC
a benign peptic ulcer? overhang the base slightly but is usually level with the (TOP 3 - AUG 2015 EXAM - MARCH
A. Sharply punched out defect surrounding mucosa. Heaped up margins are more MED BOARDS; 2016
B. Heaped-up margins characteristic of malignant ulcers. TOPNOTCH MD
C. Hemorrhage and fibrin deposition in the FROM LA SALLE)
gastric serosa Source: Robbins and Cotran Pathologic Basis of
D. Perforation Disease 8th ed p. 780
E. Malignant transformation is very rare

TOPNOTCH MEDICAL BOARD PREP PATHOLOGY SUPEREXAM Page 3 of 99


For inquiries visit www.topnotchboardprep.com.ph or email us at topnotchmedicalboardprep@gmail.com
TOPNOTCH MEDICAL BOARD PREP PATHOLOGY SUPEREXAM
For inquiries visit www.topnotchboardprep.com.ph or email us at topnotchmedicalboardprep@gmail.com
Item QUESTION EXPLANATION AUTHOR TOPNOTCH
# EXAM
17 Which of the following is the least likely Napkin ring constrictions are characteristic of left KRISTEL TANHUI DIAGNOSTIC
characteristic of right-sided colonic sided colonic malignancy. Right sided lesions present (TOP 3 - AUG 2015 EXAM - MARCH
malignancy? as polypoid, exophytic masses. Left sided lesions MED BOARDS; 2016
A. Adenocarcinoma present clinically as obstruction while right sided ones TOPNOTCH MD
B. Napkin ring constriction and luminal present as anemia. FROM LA SALLE)
narrowing
C. Desmoplastic response Source: Robbins and Cotran Pathologic Basis of
D. Liver metastasis Disease 8th ed p. 824
E. None of the above
18 Which of the following is the most common Most common cause KRISTEL TANHUI DIAGNOSTIC
cause of chronic pancreatitis? Acute pancreatitis – gallstones (TOP 3 - AUG 2015 EXAM - MARCH
A. Gallstones Chronic pancreatitis – alcoholism MED BOARDS; 2016
B. Alcoholism TOPNOTCH MD
C. Hypercholesterolemia Source: Robbins and Cotran Pathologic Basis of FROM LA SALLE)
D. Smoking Disease 8th ed p. 896
E. Hereditary predisposition

19 A 22 year old male consults for ptosis and This is a case of myasthenia gravis KRISTEL TANHUI DIAGNOSTIC
diplopia which usually begins to occur in the (TOP 3 - AUG 2015 EXAM - MARCH
late afternoon and improves upon waking up Antibodies to presynaptic Ca channel preventing MED BOARDS; 2016
after a long good sleep. Recently he also noted release of Acetylcholine – Lambert Eaton myasthenic TOPNOTCH MD
dysphagia and muscle weakness which also syndrome (paraneoplastic syndrome) FROM LA SALLE)
improves with rest. What is the
pathophysiology of his condition? Source: Robbins and Cotran Pathologic Basis of
A. Antibodies to presynaptic Ca channel Disease 8th ed p. 1275
preventing release of Acetylcholine
B. Antibodies to Acetylcholine receptors
C. Autoimmune peripheral demyelination
D. Inflammation of the spinal cord
E. Patient appears to be faking it. He should be
referred to psych.
20 HSV 1 is the most common cause of viral Accustom yourself to answering question phrased as KRISTEL TANHUI DIAGNOSTIC
encephalitis. Which is the least likely expected most likely or least likely. Examiners may want to (TOP 3 - AUG 2015 EXAM - MARCH
pathological finding? phrase it that way. MED BOARDS; 2016
A. The encephalitis most severely affects the TOPNOTCH MD
frontal lobe. HSV1 encephalitis classically most severely affects the FROM LA SALLE)
B. The infection is necrotizing and often temporal lobe. It most commonly presents as
hemorrhagic in the most severely affected alterations in mood, behavior and memory.
regions
C. Cowdry type A intranuclear viral inclusion
bodies may be found
D. All of the above
E. None of the above
21 A 4-year-old boy presents with recurrent joint SIMILAR TO PREVIOUS BOARD EXAM LESTER BRYAN CO MIDTERM 1
pain involving the knees and hips. He had CONCEPT/PRINCIPLE. Classic hemophilia (factor VIII (TOP 10 - AUG 2015 EXAM - MARCH
always bruised easily, and recently the parents deficiency) is an abnormality of the intrinsic pathway MED BOARDS; 2016
had seen blood in his urine. A presumptive of coagulation proximal to the final common pathway, TOPNOTCH MD
diagnosis of classic hemophilia (hemophilia A) which begins at factor X → Xa activation. This defect FROM UST)
is made, and coagulation blood tests are leads to a prolonged APTT. The other laboratory tests
performed. Which of the following is the most listed remain normal, because the bleeding time is a
likely set of findings of coagulation screening measure of platelet plug formation, the PT is a
tests? measure of the extrinsic pathway of coagulation, and
A. Normal bleeding time, platelet count, and the thrombin time is an assay of the conversion of
thrombin time; prolonged PT and APTT fibrinogen to fibrin. The presumptive diagnosis is
B. Normal bleeding time, platelet count, confirmed by specific
thrombin time, and APTT; prolonged PT factor VIII assay.
C. Normal bleeding time, platelet count,
thrombin time, and PT; prolonged APTT
D. Normal platelet count and thrombin time;
prolonged bleeding time, PT, and APTT
E. Prolonged bleeding time, PT, APTT, and
thrombin time; decreased platelet count
22 A 9-year-old girl is diagnosed with acute The most common cause of death that occurs during LESTER BRYAN CO MIDTERM 1
rheumatic fever. Instead of recovering as acute rheumatic fever is cardiac failure secondary to (TOP 10 - AUG 2015 EXAM - MARCH
expected, her condition worsens, and she dies. myocarditis. MED BOARDS; 2016
Which of the following is the most likely cause TOPNOTCH MD
of death? FROM UST)
A. Central nervous system involvement
B. Endocarditis
C. Myocarditis
D. Pericarditis
E. Streptococcal sepsis

TOPNOTCH MEDICAL BOARD PREP PATHOLOGY SUPEREXAM Page 4 of 99


For inquiries visit www.topnotchboardprep.com.ph or email us at topnotchmedicalboardprep@gmail.com
TOPNOTCH MEDICAL BOARD PREP PATHOLOGY SUPEREXAM
For inquiries visit www.topnotchboardprep.com.ph or email us at topnotchmedicalboardprep@gmail.com
Item QUESTION EXPLANATION AUTHOR TOPNOTCH
# EXAM
23 A 25-year-old man presents with hematuria, SIMILAR TO PREVIOUS BOARD EXAM LESTER BRYAN CO MIDTERM 1
periorbital edema, hypertension, and CONCEPT/PRINCIPLE. The clinical description is that (TOP 10 - AUG 2015 EXAM - MARCH
hemoptysis. He has also experienced nausea, of Goodpasture syndrome (antiglomerular basement MED BOARDS; 2016
vomiting, fever, and chills. Serologic testing is membrane disease), caused by antibodies directed TOPNOTCH MD
positive for antiglomerular basement against antigens in the glomerular and pulmonary FROM UST)
membrane antibodies. Which of the following is alveolar basement membranes. Because antigens are
the classic histologic finding in this renal an intrinsic component of the basement membrane,
disease?
labeled antibodies “paint” the surface of the basement
A. Linear immunofluorescence membrane, resulting in the characteristic linear
B. “Lumpy-bumpy” immunofluorescence immunofluorescent pattern characteristic of this
C. “Spike and dome” appearance of the disorder. “Lumpy-bumpy” immunofluorescence is
glomerular basement membrane very coarse, granular immunofluorescence found in
D. Subendothelial immune complex deposition poststreptococcal immune complex deposit disease.
E. Tram-track appearance of the glomerular The tram-track appearance is seen in
basement membrane on electron microscopy membranoproliferative glomerulonephritis. The
“spike and dome” appearance is seen in membranous
glomerulonephritis. Subendothelial immune complex
deposition is seen in lupus nephropathy.
24 A 19-year-old young woman who emigrated Inflammation and stenosis of branches of the aortic LESTER BRYAN CO MIDTERM 1
from Taiwan 8 years ago presents with fever, arch is known as Takayasu arteritis, or “pulseless (TOP 10 - AUG 2015 EXAM - MARCH
malaise, myalgias, and arthritis and “coldness” disease.” It most commonly occurs in young Asian MED BOARDS; 2016
in her upper extremities. She has a weak radial females. Buerger disease usually affects young Jewish TOPNOTCH MD
pulse bilaterally, and a magnetic resonance males and involves the arteries of the extremities. The FROM UST)
angiogram demonstrates nearly 75% stenosis disease is exacerbated by smoking and can lead to
of the main arteries originating from the aorta. gangrene of the extremities. Kawasaki disease affects
She likely has which of the following the branches of the coronary arteries. Raynaud
rheumatologic conditions? disease is due to vasospasm of small vessels of the
A. Buerger disease fingers and toes, leading to cyanosis and pallor of the
B. Kawasaki disease affected tissues. Temporal arteritis is usually
C. Raynaud disease encountered in older patients and affects the branches
D. Takayasu arteritis of the carotid artery, most commonly the temporal
E. Temporal arteritis artery.
25 A 45-year-old man presents with involuntary This is a case of Huntington disease, which is an LESTER BRYAN CO MIDTERM 1
facial grimaces and movements of the fingers. autosomal dominant, fatal, progressive degeneration (TOP 10 - AUG 2015 EXAM - MARCH
His mother had had similar symptoms and atrophy of the striatum (caudate nucleus and MED BOARDS; 2016
beginning at about the same age. Her disorder putamen). The disorder is characterized by an TOPNOTCH MD
had progressed to dancing movements, increased number of trinucleotide (CAG) repeats in FROM UST)
writhing of the arms and legs, and eventually the HD (huntingtin) gene on the short arm of
coma and death. His maternal grandfather had chromosome 4. Degeneration of the upper and lower
had a similar disorder but at an age older than motor neurons is characteristic of ALS. Dopamine
the mother. Which of the following is most depletion and depigmentation of the substantia nigra
characteristic of this disease? is characteristic of Parkinson disease. Neurofibrillary
A. Degeneration of upper and lower motor tangles and amyloid plaques are found in Alzheimer
neurons disease. Pick bodies can be found in Pick disease,
B. Dopamine depletion and depigmentation of which clinically resembles Alzheimer disease.
the substantia nigra
C. Increased number of trinucleotide repeats in
a gene on chromosome 4
D. Neurofibrillary tangles and amyloid plaques
in the cerebral cortex
E. Pick bodies, characterized by round
intracytoplasmic inclusions consisting of
neurofilaments
26 A 14-year-old girl presents with prolonged von Willebrand disease, a disorder transmitted by LESTER BRYAN CO MIDTERM 1
bleeding from wounds and minor trauma and autosomal modes of inheritance (both dominant and (TOP 10 - AUG 2015 EXAM - MARCH
severe menorrhagia. Family history reveals that recessive) is the most common hereditary bleeding MED BOARDS; 2016
her father also has prolonged bleeding from disorder. There are many variants, all marked by TOPNOTCH MD
wounds and minor trauma, as does her brother. either qualitative or quantitative deficiencies of vWF. FROM UST)
Which of the following is the most likely
mechanism of this patient’s disorder?
A. Absence of platelet glycoprotein IIb-IIIa
B. Antiplatelet antibodies reacting with platelet
surface glycoproteins
C. Deficiency of factor VIII
D. Deficiency of factor IX
E. Deficiency of vWF
27 A 42-year-old man is seen because of a long Cardiomyopathies are noninflammatory myocardial LESTER BRYAN CO MIDTERM 1
history of slowly developing congestive heart disorders that are not associated with coronary artery (TOP 10 - AUG 2015 EXAM - MARCH
failure. His blood pressure is normal. Coronary obstruction, hypertension, valvular disease, congenital MED BOARDS; 2016
artery angiography reveals no vascular disease. heart disease, or infectious disease. They are most TOPNOTCH MD
No heart murmurs are heard. The white blood often characterized by otherwise unexplained FROM UST)
cell count, differential, and erythrocyte ventricular dysfunction, such as cardiac failure,
sedimentation rate are normal. The most likely ventricular enlargement, or ventricular arrhythmias.
diagnosis is
A. carcinoid heart disease
B. cardiomyopathy
C. coarctation of the aorta
D. constrictive pericarditis
E. myocardial infarction

TOPNOTCH MEDICAL BOARD PREP PATHOLOGY SUPEREXAM Page 5 of 99


For inquiries visit www.topnotchboardprep.com.ph or email us at topnotchmedicalboardprep@gmail.com
TOPNOTCH MEDICAL BOARD PREP PATHOLOGY SUPEREXAM
For inquiries visit www.topnotchboardprep.com.ph or email us at topnotchmedicalboardprep@gmail.com
Item QUESTION EXPLANATION AUTHOR TOPNOTCH
# EXAM
28 A 28-year-old woman complains of fatigue, SIMILAR TO PREVIOUS BOARD EXAM LESTER BRYAN CO MIDTERM 1
dyspnea, and malaise. She also notes that her CONCEPT/PRINCIPLE. Paroxysmal nocturnal (TOP 10 - AUG 2015 EXAM - MARCH
urine has been reddishbrown, particularly with hemoglobinuria results in an acquired MED BOARDS; 2016
the first void of the morning. Subsequent intracorpuscular defect in the ability to synthesize GPI TOPNOTCH MD
studies confirm that she has paroxysmal anchors required for appropriate placement of FROM UST)
nocturnal hemoglobinuria. Which of the complement regulatory proteins on the surface of red
following best describes the defect leading to blood cells. Functional deficiency of such proteins as
this condition? CD55 and CD59 renders the cells sensitive to
A. Anti-intrinsic factor antibodies complement-mediated lysis. Anti-intrinsic factor
B. Deficiency of the intracellular structural antibodies are seen in pernicious anemia. Spectrin is
protein spectrin deficient in hereditary spherocytosis. Ineffective
C. Impaired synthesis of the cell-surface GPI erythropoiesis is seen in megaloblastic anemia due to
anchor folate or vitamin B12 deficiencies. Substitution of
D. Ineffective erythropoiesis valine for glutamic acid in the β-globin gene underlies
E. Substitution of a valine for a glutamate the defect in sickle cell anemia.
residue in the β-globin gene
29 A 45-year-old man presents with abdominal Berry aneurysms, which occur in 10% to 15% of LESTER BRYAN CO MIDTERM 1
pain and hypertension. On physical patients with adult polycystic kidney disease, are (TOP 10 - AUG 2015 EXAM - MARCH
examination, he is found to have an abdominal small saccular lesions that develop at the site of MED BOARDS; 2016
mass. Further workup confirms the diagnosis of congenital weakness of cerebral arteries, especially TOPNOTCH MD
adult polycystic kidney disease. Which of the those of the circle of Willis. Rupture of these FROM UST)
following vascular complications is associated aneurysms is the most common cause of subarachnoid
with this condition? hemorrhage. Arteriovenous fistulas are often
A. Arteriovenous fistula secondary to trauma. Dissecting aneurysm is
B. Atherosclerotic aneurysm associated with hypertension or with diseases
C. Berry aneurysm affecting the vascular media, most notably Marfan
D. Dissecting aneurysm syndrome. Syphilitic (luetic) aneurysm is associated
E. Luetic aneurysm with tertiary syphilis.
30 A 55-year-old man presents with a large, black- Malignant melanoma arises from melanocytes or LESTER BRYAN CO MIDTERM 1
colored, asymmetric skin lesion with ill-defined nevus cells, is most often associated with excessive (TOP 10 - AUG 2015 EXAM - MARCH
borders on his back. He reports a family history sun exposure, and is most common in fair-skinned MED BOARDS; 2016
of malignant melanoma. Which of the following persons. Of the clinical variants of malignant TOPNOTCH MD
clinical variants of malignant melanoma has the melanoma, nodular melanoma has the worst FROM UST)
poorest prognosis? prognosis. Malignant melanomas have a better
A. Lentigo maligna melanoma prognosis when characterized by a long period of
B. Superficial spreading melanoma radial (superficial) growth, as opposed to early
C. Nodular melanoma vertical growth (as in nodular melanoma).
D. Acral-lentiginous melanoma
E. uveal melanoma
31 When ordering academic attire for a recent A mosaic pattern of bone caused by increases in both LESTER BRYAN CO MIDTERM 1
graduation, a 65-year-old university professor osteoblastic and osteoclastic activity is characteristic (TOP 10 - AUG 2015 EXAM - MARCH
is surprised to find that his hat size has of Paget disease of bone (osteitis deformans). Serum MED BOARDS; 2016
increased. Shortly thereafter, in a routine alkaline phosphatase is markedly increased. Hearing TOPNOTCH MD
checkup, serum alkaline phosphatase activity is loss is common (from narrowing of the auditory FROM UST)
found to be markedly elevated. Serum calcium foramen and compression of the eighth cranial nerve),
and phosphorus are normal. Examination and an increase in hat size due to frontal bossing is
reveals enlargement of the skull with frontal often noted.
bossing and enlarged maxilla, and hearing loss
is evident. Which of the following abnormalities
is associated with the bone disorder suggested
by these findings?
A. Brown tumor of bone
B. Defective calcification of osteoid matrix
C. Mosaic pattern of bone
D. Polyostotic fibrous dysplasia with severe
deformity
E. Subperiosteal hemorrhage and osteoporosis
32 A 3-year-old African-American man with a Sickle cell anemia is the most common hereditary LESTER BRYAN CO MIDTERM 1
history since early childhood of severe anemia anemia in persons of African lineage. Leg ulcers and (TOP 10 - AUG 2015 EXAM - MARCH
requiring many transfusions has nonhealing leg recurring painful crises are characteristic. In sickle cell MED BOARDS; 2016
ulcers and recurrent periods of abdominal and anemia, in contrast to sickle cell trait, sickle cells are TOPNOTCH MD
chest pain. These signs and symptoms are most often seen on the peripheral blood smear. FROM UST)
likely to be associated with which of the
following laboratory abnormalities?
A. Decreased erythropoietin
B. Increased erythrocyte osmotic fragility
C. Schistocytes
D. Sickle cells on peripheral blood smear
E. Teardrop-shaped cells
33 For the past week, a 65-year-old woman has Pseudomembranous colitis is caused by overgrowth of LESTER BRYAN CO MIDTERM 1
been treated for a severe infection with broad- C. difficile. This organism produces exotoxin that (TOP 10 - AUG 2015 EXAM - MARCH
spectrum antibiotics, and she had recovered induces necrosis of the superficial mucosa, leading to MED BOARDS; 2016
well. Over the past day, however, she has pseudomembrane formation. The bacteria itself does TOPNOTCH MD
developed foul-smelling, voluminous, greenish, not invade the mucosa. This condition most often FROM UST)
watery diarrhea, as well as abdominal pain and occurs in patients with a history of broad-spectrum
fever. Which of the following is the mechanism antibiotic use, because elimination of normal
associated with this condition? intestinal flora promotes overgrowth of C. difficile.
A. Aggregation of bacterial colonies on the
lumen, forming pseudomembranes
B. Bacterial release of exotoxin, inducing
necrosis of the mucosa
C. Physical invasion of bacteria into the
superficial mucosa, leading to
pseudomembrane formation
D. Selective killing of C. difficile bacteria by
antibiotics

TOPNOTCH MEDICAL BOARD PREP PATHOLOGY SUPEREXAM Page 6 of 99


For inquiries visit www.topnotchboardprep.com.ph or email us at topnotchmedicalboardprep@gmail.com
TOPNOTCH MEDICAL BOARD PREP PATHOLOGY SUPEREXAM
For inquiries visit www.topnotchboardprep.com.ph or email us at topnotchmedicalboardprep@gmail.com
Item QUESTION EXPLANATION AUTHOR TOPNOTCH
# EXAM
E. Spread of the previous infection to the colon

34 A 50-year-old woman with a 20-year history of SIMILAR TO PREVIOUS BOARD EXAM LESTER BRYAN CO MIDTERM 1
type 2 diabetes mellitus presents with CONCEPT/PRINCIPLE. Diabetic nephropathy (TOP 10 - AUG 2015 EXAM - MARCH
proteinuria, hypoalbuminemia, edema, and manifests clinically as the nephrotic syndrome; MED BOARDS; 2016
hyperlipidemia. She has not monitored her however, this syndrome is compounded by renal TOPNOTCH MD
serum glucose levels over the past several failure and hypertension. Ultrastructural changes FROM UST)
years. What is the classic morphologic finding include a marked increase in the thickness of the
in this condition? glomerular basement membrane and mesangial
A. Crescentic formation in glomeruli on light accumulation of glycosylated basement membranelike
microscopy material. Light microscopy findings include diffuse
B. Intramembranous and epimembranous glomerulosclerosis (a diffuse increase in mesangial
immune complex deposits in the glomerular matrix) and nodular glomerulosclerosis (nodular
basement membrane on electron microscopy accumulations of mesangial matrix).
C. Nodular accumulations of mesangial matrix
on light microscopy
D. Sclerosis within capillary tufts that involves
only some glomeruli and only parts of affected
glomeruli on light microscopy
E. Wire-loop abnormalities from immune
complex deposits and thickening of the
glomerular basement membrane on light
microscopy
35 A 23-year-old woman presents with cervical SIMILAR TO PREVIOUS BOARD EXAM LESTER BRYAN CO MIDTERM 1
and mediastinal lymphadenopathy. Biopsy of a CONCEPT/PRINCIPLE. The diagnosis is Hodgkin (TOP 10 - AUG 2015 EXAM - MARCH
cervical lymph node reveals a nodular lymphoma, nodular sclerosing subtype. This form of MED BOARDS; 2016
appearance with fibrous bands, effacement of Hodgkin lymphoma differs from other forms of TOPNOTCH MD
the lymph node architecture, and numerous classical Hodgkin lymphoma in being the most FROM UST)
lacunar cells. Which of the following is common in young women, having a relatively
characteristic of this disorder? favorable clinical course, and having little association
A. Benign neoplasm with EBV infection. Lacunar cells are considered a
B. Frequent association with EBV infection Reed-Sternberg cell variant, and the diagnosis of NS
C. Most often a complication of human can be based on the finding of fibrous bands and
immunodeficiency virus infection lacunar cells.
D. Peak incidence in early childhood
E. Relatively favorable clinical course
36 A 15-year-old boy presents with a pathologic In young patients, bone malignancies showing LESTER BRYAN CO MIDTERM 1
fracture following a minor injury on the soccer prominent cartilaginous differentiation are almost (TOP 10 - AUG 2015 EXAM - MARCH
field. The area of fracture is surrounded by a assuredly chondroblastic osteosarcomas, rather than MED BOARDS; 2016
large tumor which shows marked chondrosarcomas. Conventional chondrosarcomas TOPNOTCH MD
pleomorphism, high mitotic activity, and occur almost exclusively in older patients. FROM UST)
extensive cartilaginous differentiation on
microscopy. The most likely diagnosis is
A. giant cell tumor
B. osteochondroma
C. chondrosarcoma
D. osteosarcoma
E. Ewing sarcoma
37 A 45-year-old woman presents to her primary Primary biliary cirrhosis is an autoimmune condition LESTER BRYAN CO MIDTERM 1
care physician with jaundice, pruritus, and that typically presents in middle-aged women. The (TOP 10 - AUG 2015 EXAM - MARCH
periocular and intradigital xanthomas. Her itching and hypercholesterolemia are secondary to MED BOARDS; 2016
laboratory results indicate a significantly severe obstructive jaundice. Leptospirosis is a TOPNOTCH MD
increased alkaline phosphatase as well as a condition caused by a treponemal bacterium that FROM UST)
positive test for antimitochondrial antibodies. results in jaundice, renal failure, and hemorrhagic
The most likely cause of her symptoms is phenomena. Macronodular cirrhosis is usually a result
A. leptospirosis of hepatitis B or hepatitis C infection. Primary
B. macronodular cirrhosis sclerosing cholangitis is associated with ulcerative
C. primary biliary cirrhosis colitis and with an increased incidence of
D. primary sclerosing cholangitis cholangiocarcinoma. Secondary biliary cirrhosis is
E. secondary biliary cirrhosis caused by extrahepatic biliary obstruction.
38 The chest radiograph of a 23-year-old medical Cavitation occurs only in secondary tuberculosis. Both LESTER BRYAN CO MIDTERM 1
student reveals a calcified cavitary pulmonary primary and secondary tuberculosis are characterized (TOP 10 - AUG 2015 EXAM - MARCH
lesion. The tuberculin test is positive, but by caseating granulomas, often with Langhans giant MED BOARDS; 2016
sputum smears and cultures are negative for cells, which heal by scarring and calcification. The skin TOPNOTCH MD
Mycobacterium tuberculosis. A presumptive test result for tuberculin sensitivity is positive in both FROM UST)
diagnosis of secondary tuberculosis is made. If forms.
further studies, including a biopsy, were
performed, which of the following
findings would justify the diagnosis of
secondary tuberculosis, as contrasted to
primary tuberculosis?
A. Calcification
B. Caseating granulomas
C. Cavitation
D. Langhans giant cells

TOPNOTCH MEDICAL BOARD PREP PATHOLOGY SUPEREXAM Page 7 of 99


For inquiries visit www.topnotchboardprep.com.ph or email us at topnotchmedicalboardprep@gmail.com
TOPNOTCH MEDICAL BOARD PREP PATHOLOGY SUPEREXAM
For inquiries visit www.topnotchboardprep.com.ph or email us at topnotchmedicalboardprep@gmail.com
Item QUESTION EXPLANATION AUTHOR TOPNOTCH
# EXAM
E. Positive tuberculin test result

39 A 40-year-old woman presents with a painless Approximately 80% to 90% of salivary gland tumors LESTER BRYAN CO MIDTERM 1
mass anterior to her left ear. The mass had originate in the parotid gland and, of these, (TOP 10 - AUG 2015 EXAM - MARCH
been slowly enlarging over the past year. The approximately 70% are pleomorphic adenomas. The MED BOARDS; 2016
mass is firm and nontender. Computed term “mixed tumor” properly applies to this benign TOPNOTCH MD
tomography and magnetic resonance imaging tumor, which often demonstrates myxoid and FROM UST)
reveal a well-circumscribed, homogeneous cartilage-like elements in addition to stellate or
mass within the left parotid gland. Biopsy fusiform epithelial cells. Complete surgical resection is
reveals anastomosing strands of stellate and difficult because of the tumor’s proximity to the facial
fusiform epithelial cells embedded in a myxoid nerve, and, thus, recurrence is frequent.
stroma. Which of the following is a
characteristic of the lesion?
A. It is also called papillary cystadenoma
lymphomatosum.
B. It is most often localized to the
submandibular gland.
C. It is the most common malignant salivary
gland tumor.
D. Recurrence often takes place after surgical
resection.
E. Surgical resection should not be performed,
because this condition is usually already
metastatic on diagnosis.
40 A 70-year-old man presents with severe bone The diagnosis is multiple (plasma cell) myeloma, a LESTER BRYAN CO MIDTERM 1
pain and frequent respiratory infections. Serum neoplastic proliferation of malignant plasma cells (TOP 10 - AUG 2015 EXAM - MARCH
protein electrophoresis demonstrates an M (mature B cells, not T cells). Death is often caused by MED BOARDS; 2016
protein spike in the gamma region. renal insufficiency caused by myeloma kidney. The TOPNOTCH MD
Radiographs of the skull, long bones, and spine average age of presentation is approximately 70 years FROM UST)
demonstrate multiple “punched-out” lesions, of age. IgM myeloma is very uncommon. Both the
and bone marrow aspiration demonstrates neoplastic cells and the serum protein spike are
large numbers of neoplastic plasma cells. Which monoclonal rather than polyclonal, and the
of the following statements is true of this monoclonal spike protein is most frequently an IgG or
disorder? an IgA.
A. Although this patient presents at 70 years of
age, the average age of presentation is 50 years
of age.
B. Renal insufficiency is a common cause of
death.
C. The M spike is most often an IgM.
D. The M spike is most often polyclonal in
nature.
E. This disorder is the most common T-cell
neoplasm.
41 Spongiotic dermatitis is the histologic pattern Contact dermatitis is a type of spongiotic dermatitis GEORGE MICHAEL MIDTERM 2
for which of the following dermatoses? with edema and perivascular lymphocytic infiltrate. SOSUAN (TOP 5 - EXAM - MARCH
A. Discoid lupus erythematosus AUG 2015 MED 2016
B. Contact dermatitis BOARDS; TOPNOTCH
C. Psoriasis MD FROM UST)
D. Erythema nodosum
E. Erythema multiforme

42 Oral lesions may be seen secondary to fungal Special histochemical stains (Grocott-Gomori GEORGE MICHAEL MIDTERM 2
pulmonary infections which spread by methenamine silver (GMS), or periodic acid-Schiff SOSUAN (TOP 5 - EXAM - MARCH
hematogenously or by direct inoculation of (PAS) highlight fungi in sections. AUG 2015 MED 2016
infected sputum. Which of the following maybe BOARDS; TOPNOTCH
used to stain these fungal elements MD FROM UST)
A. Gomori methenamine stain
B. PAS
C. Gram's stain
D. Both A and B
E. AOTA
43 A 72 y/o patient with diabetes complains of Malignant otitis externa is most commonly associated GEORGE MICHAEL MIDTERM 2
severe pain and bloody discharge of the right with Pseudomonas aeruginosa, which is a gram- SOSUAN (TOP 5 - EXAM - MARCH
ear. Which of the following would a Gram stain negative bacillus. AUG 2015 MED 2016
of material from surgical debridement be BOARDS; TOPNOTCH
expected to show? MD FROM UST)
A. Gram-positive cocci
B. Gram-negative cocci
C. Gram-positive bacilli
D. Gram-negative bacilli
E. Septate, branching hyphae

TOPNOTCH MEDICAL BOARD PREP PATHOLOGY SUPEREXAM Page 8 of 99


For inquiries visit www.topnotchboardprep.com.ph or email us at topnotchmedicalboardprep@gmail.com
TOPNOTCH MEDICAL BOARD PREP PATHOLOGY SUPEREXAM
For inquiries visit www.topnotchboardprep.com.ph or email us at topnotchmedicalboardprep@gmail.com
Item QUESTION EXPLANATION AUTHOR TOPNOTCH
# EXAM
44 Which of the following is incorrectly matched? All of them are correct GEORGE MICHAEL MIDTERM 2
A. Bronchopneumonia: Patchy multifocal SOSUAN (TOP 5 - EXAM - MARCH
consolidation AUG 2015 MED 2016
B. Lobar pneumonia: Lobar consolidation BOARDS; TOPNOTCH
C. Atypical pneumonia: Inflammation within MD FROM UST)
alveolar interstitium
D. AOTA
E. NOTA
45 Leading cause of blood transfusion related TRALI is currently the leading cause of transfusion GEORGE MICHAEL MIDTERM 2
mortality mortality. Donor antibodies to recipient leukocyte SOSUAN (TOP 5 - EXAM - MARCH
A. Transfusion related acute lung injury antigens activate complement and encourage AUG 2015 MED 2016
B. Graft versus host reaction granulocytes to aggregate within the pulmonary BOARDS; TOPNOTCH
C. Hemolytic transfusion reaction microvasculature. The diagnosis is based on clinical MD FROM UST)
D. Sepsis findings that are similar to ARDS but arise within 6
E. Anaphylaxis hours after transfusion. TRALI has a 5–10% mortality
rate, but with interim ventilatory support, most
symptoms resolve within
96 hours.
46 Paradoxical embolus is usually seen among In patients with cardiac septal defects, an embolus GEORGE MICHAEL MIDTERM 2
patients with: may detour into the left heart as a paradoxical SOSUAN (TOP 5 - EXAM - MARCH
A. Tetralogy of Fallot embolus to organs and the brain. AUG 2015 MED 2016
B. Transposition of great vessels BOARDS; TOPNOTCH
C. Cardiac septal defect MD FROM UST)
D. Tricuspid atresia
E. NOTA

47 A 13 y/o F girl with asthma seeks medical Charcot-Leyden crystals are a sign of eosinophilic GEORGE MICHAEL MIDTERM 2
attention at her pediatrician’s office degranulation, SOSUAN (TOP 5 - EXAM - MARCH
complaining of shortness of breath. What and Curschmann spirals are a sign of excess mucus in AUG 2015 MED 2016
would sputum cytology tests reveal? sputum of asthma patients. BOARDS; TOPNOTCH
A. Fungal hyphae MD FROM UST)
B. Acid-fast bacilli
C. Keratin pearls
D. Charcot-Leyden crystals
E. Atypical squamous epithelial cells
48 This type of gastritis is characterized by auto- Type A - Autoimmune gastritis (10%), autoantibodies GEORGE MICHAEL MIDTERM 2
antibodies to gastric parietal cells and intrinsic are made against parietal cells and intrinsic factor, SOSUAN (TOP 5 - EXAM - MARCH
factor. gland atrophy leads to pernicious anemia; Type B - AUG 2015 MED 2016
A. Type A Helicobacter pylori infection (90%), the most common BOARDS; TOPNOTCH
B. Type B cause of gastritis, Increased risk of peptic ulcers and MD FROM UST)
C. Type AB carcinoma
D. Type C
E. Type O

49 A 32 y/o F has progressive dyspnea, cough, and Polyarteritis nodosa typically does not involve the GEORGE MICHAEL MIDTERM 2
fever. A chest radiograph shows bilateral pulmonary vessels. SOSUAN (TOP 5 - EXAM - MARCH
infiltrates. She also notes dark brown urine. AUG 2015 MED 2016
Which of the following statements is FALSE? BOARDS; TOPNOTCH
A. Low levels of serum complement may be MD FROM UST)
seen
B. Blood cultures should be performed
C. Serum ANCA should be performed
D. The differential diagnosis includes systemic
lupus erythematosus
E. Polyarteritis nodosa is the most likely
diagnosis
50 The leather-bottle appearance of the gastric Linitis plastica refers to a thickened “leather-bottle” GEORGE MICHAEL MIDTERM 2
adenocarcinoma is characteristic of this appearance of the stomach caused by diffuse SOSUAN (TOP 5 - EXAM - MARCH
histologic type. infiltration of the gastric wall by a poorly AUG 2015 MED 2016
A. Intestinal differentiated signet-ring type of adenocarcinoma. BOARDS; TOPNOTCH
B. Diffuse MD FROM UST)
C. Nodular
D. Esophageal
E. Squamous

51 An elderly woman complains of fatigue, anemia, 10% of colon cancers occur in the rectum. Anal GEORGE MICHAEL MIDTERM 2
and bright red blood in stool. Which of the carcinoma is usually human papillomavirus– related SOSUAN (TOP 5 - EXAM - MARCH
following is the MOST LIKELY diagnosis? and occurs in younger patients; endometriosis causing AUG 2015 MED 2016
A. Rectal adenocarcinoma rectal bleeding would not be expected in a BOARDS; TOPNOTCH
B. Anal squamous cell carcinoma postmenopausal patient. MD FROM UST)
C. Endometriosis
D. AOTA
E. NOTA

52 True of HBsAg: HBsAg (surface antigen) provides the first evidence of GEORGE MICHAEL MIDTERM 2
A. First evidence of infection infection and appears in the serum before symptoms. SOSUAN (TOP 5 - EXAM - MARCH
B. Appears in the serum after symptoms AUG 2015 MED 2016
C. Co-incide with the elevation of ALT BOARDS; TOPNOTCH
D. AOTA MD FROM UST)
E. Both A and C

TOPNOTCH MEDICAL BOARD PREP PATHOLOGY SUPEREXAM Page 9 of 99


For inquiries visit www.topnotchboardprep.com.ph or email us at topnotchmedicalboardprep@gmail.com
TOPNOTCH MEDICAL BOARD PREP PATHOLOGY SUPEREXAM
For inquiries visit www.topnotchboardprep.com.ph or email us at topnotchmedicalboardprep@gmail.com
Item QUESTION EXPLANATION AUTHOR TOPNOTCH
# EXAM
53 Hallmark of malignant transformation Anaplasia or lack of differentiation is the hallmark of GEORGE MICHAEL MIDTERM 2
A. Keratin pearls malignant transformation. SOSUAN (TOP 5 - EXAM - MARCH
B. Dedifferentiation AUG 2015 MED 2016
C. Lack of differentiation BOARDS; TOPNOTCH
D. Metastasis MD FROM UST)
E. Skip lesions

54 The leading causative agent of injection drug In hepatitis C, chronic disease occurs in 85% of GEORGE MICHAEL MIDTERM 2
use hepatitis: patients. The leading cause is injection drug use. SOSUAN (TOP 5 - EXAM - MARCH
A. Hepatitis A Cirrhosis will develop in 20% of patients within 20 AUG 2015 MED 2016
B. Hepatitis B years. BOARDS; TOPNOTCH
C. Hepatitis C MD FROM UST)
D. Hepatitis D
E. Both B and C

55 Hepatocellular carcinoma with the best Fibrolamellar variant of HCC may have a better GEORGE MICHAEL MIDTERM 2
prognosis prognosis than conventional HCC. It usually occurs in SOSUAN (TOP 5 - EXAM - MARCH
A. Hepatitis B variant young adults without a history of prior liver disease. It AUG 2015 MED 2016
B. Fibrolamellar variant is characterized by oncocytic-like hepatocytes BOARDS; TOPNOTCH
C. Scirrhous type (abundant intracellular mitochondria) infiltrating MD FROM UST)
D. Mucinous type fibrous stroma.
E. NOTA

56 This pathology is characterized by distinctive The decription stated is of a "tram track," GEORGE MICHAEL MIDTERM 2
combination of mesangial and endothelial characteristic of MPGN. SOSUAN (TOP 5 - EXAM - MARCH
proliferation, along AUG 2015 MED 2016
with thickening and duplication of the capillary BOARDS; TOPNOTCH
basement membrane MD FROM UST)
A. Amyloidosis
B. Diabetic nephropathy
C. Membranoproliferative glomerulonephritis
D. IgA nephropathy
E. Post-streptococcal glomerulonephritis
57 This pathological process is characterized by Malignant hypertension is a rapid disease results in GEORGE MICHAEL MIDTERM 2
acute vascular lesions include fibrinoid necrosis acute vascular lesions and parenchymal injury SOSUAN (TOP 5 - EXAM - MARCH
of renal arteries and referred AUG 2015 MED 2016
arterioles, with onion skin lesions to as “malignant nephrosclerosis.” BOARDS; TOPNOTCH
A. Renal artery stenosis MD FROM UST)
B. Malignant hypertension
C. Interstitial nephritis
D. Hypertensive nephrosclerosis
E. Vasculitis

58 Crescentic glomerulonephritis is most ANCAs (anti-neutrophil cytoplasmic antibodies) are GEORGE MICHAEL MIDTERM 2
associated with: associated with pauci-immune crescentic SOSUAN (TOP 5 - EXAM - MARCH
A. p-ANCA glomerulonephritis. HIV is associated with focal AUG 2015 MED 2016
B. HIV segmental glomerulosclerosis; cryoglobulin and C3 BOARDS; TOPNOTCH
C. Cytomegalovirus nephritic factor may be associated with different types MD FROM UST)
D. Cryoglobulin of membranoproliferative glomerulonephritis.
E. Nephritic factor Cytomegalovirus infection causes tubulointerstitial
nephritis and is seen in immunocompromised
patients.
59 Pretibial myxedema is characterized by: Pretibial myxedema is secondary to hyaluronic acid GEORGE MICHAEL MIDTERM 2
A. Hyaluronic acid deposition in the deposition and lymphocytic infiltrates within the SOSUAN (TOP 5 - EXAM - MARCH
subcutaneous tissue dermis. AUG 2015 MED 2016
B. Lymphocytic inflitrates in the dermis BOARDS; TOPNOTCH
C. Chondroitin sulfate deposition within the MD FROM UST)
dermis
D. Both A and B
E. AOTA
60 This is a disorder characterized by peripheral Refetoff syndrome is a disorder of peripheral GEORGE MICHAEL MIDTERM 2
resistance to circulating thyroid hormone resistance to circulating thyroid hormone. SOSUAN (TOP 5 - EXAM - MARCH
A. Refetoff syndrome AUG 2015 MED 2016
B. Zellweger syndrome BOARDS; TOPNOTCH
C. Reiter syndrome MD FROM UST)
D. Thyroid dysplasmocytic syndrome
E. NOTA

61 What is the interluekin that is vital for IL-8 is the major chemoatactic factor for neutrophils. JAN CHRISTIAN MIDTERM 3
neutrophil chemotaxis? IL-1 is mostly resposnsible for fever. IL-2 stimutales FELICIANO (TOP 2 - EXAM - MARCH
A. IL-1 grwoth of T cells. IL-6 stimulates acute phase protein AUG 2015 MED 2016
B. IL-2 production and IL-10 modulates the immune response BOARDS; TOPNOTCH
C. IL-6 MD FROM UST)
D. IL-8
E. IL-10

62 What cytokine mediates septic shock and plays SIMILAR TO PREVIOUS BOARD EXAM JAN CHRISTIAN MIDTERM 3
a role in cancer cachexia? CONCEPT/PRINCIPLE/ TNF alpha is believed to be the FELICIANO (TOP 2 - EXAM - MARCH
A. TNF alpha cytokine that mediates septic shock and is implicated AUG 2015 MED 2016
B. IL-1 in cachexia of malignancy. BOARDS; TOPNOTCH
C. Inteferon alpha MD FROM UST)
D. Inferterferon gamma
E. Lipopolysaccharide

TOPNOTCH MEDICAL BOARD PREP PATHOLOGY SUPEREXAM Page 10 of 99


For inquiries visit www.topnotchboardprep.com.ph or email us at topnotchmedicalboardprep@gmail.com
TOPNOTCH MEDICAL BOARD PREP PATHOLOGY SUPEREXAM
For inquiries visit www.topnotchboardprep.com.ph or email us at topnotchmedicalboardprep@gmail.com
Item QUESTION EXPLANATION AUTHOR TOPNOTCH
# EXAM
63 This is considered an irreversible histologic Nuclear pyknosis, karyorrhesis and karyolysis are JAN CHRISTIAN MIDTERM 3
manifestation of cellular injury considered irreversible changes of cell injury, The rest FELICIANO (TOP 2 - EXAM - MARCH
A. Cellular swelling of the choices are reversible. AUG 2015 MED 2016
B. Nuclear chromatin clumping BOARDS; TOPNOTCH
C. Nuclear pyknosis MD FROM UST)
D. Ribosomal detachment
E. Membrane blebbing

64 Which statement refers to dystrophic Dystrophic calcification is ca deposition in abnormal JAN CHRISTIAN MIDTERM 3
calcification rather than metastatic tissues usually due to necrosis. It is seen in TB, FELICIANO (TOP 2 - EXAM - MARCH
calcification? infarcts, thrombus, schistosomiasis, congenital CMV, AUG 2015 MED 2016
A. Diffuse and widespread toxoplasmosis and psamomma bodies. The rest of the BOARDS; TOPNOTCH
B. Ca deposition in normal tissues choices refers to metastatic calcification. MD FROM UST)
C. Patients are usually not normocalcemic
D. Occurs in patients on long term
hemodialysis
E. Associated with thrombus and infarcts
65 Which of the ff mechanisms regarding Diapedesis uses PECAM-1 not VCAM (utilized in tight JAN CHRISTIAN MIDTERM 3
leukocyte extravasatation is incorrectly paired? binding together with ICAM). All the other choices are FELICIANO (TOP 2 - EXAM - MARCH
A. Margination and rolling- Selectins correct AUG 2015 MED 2016
B. Margination and rolling- GlyCAM-1 BOARDS; TOPNOTCH
C. Tight-binding- ICAM-1 MD FROM UST)
D. Diapededis- VCAM-1
E. Migration- IL-8

66 Cigarette smoking is carcinogenic to the ff SIMILAR TO PREVIOUS BOARD EXAM JAN CHRISTIAN MIDTERM 3
organs EXCEPT? CONCEPT/PRINCIPLE. Cigarrete smoking is assocated FELICIANO (TOP 2 - EXAM - MARCH
A. Colon with maligancies in the ff organs: bladder, cervix, AUG 2015 MED 2016
B. Bladder esophagus, kidne, larynx, lung, pancreas. Colon and BOARDS; TOPNOTCH
C. Pancreas breast malignancies usually does not have smoking as MD FROM UST)
D. Kidney a risk factor/
E. Cervix

67 Which of the ff statements regarding neoplasia Benign neoplasms and well-differentiated carcinomas JAN CHRISTIAN MIDTERM 3
is INCORRECT? of endocrine glands frequently secrete hormones FELICIANO (TOP 2 - EXAM - MARCH
A. Tumor is said to be benign when its gross character tic of their origin. Well-differentiated AUG 2015 MED 2016
and microscopic appearances are considered squamous cell carcinomas of the epidermis synthesize BOARDS; TOPNOTCH
relatively innocent keratin, and well-differentiated hepatocellular MD FROM UST)
B. Environmental influences not genetic factors carcinomas elaborate bile. All other statements are
appear to be the dominant risk factors for most correct
cancers.
C. Hamartomas are disorganized but benign
masses composed of cells indigenous to the
involved site.
D. Benign neoplasms and poorly differentiated
carcinomas of endocrine glands frequently
secrete hormones characteristic of their origin
E. Once the tumor cells breach the basement
membrane, the tumor is said to be invasive.
68 Overall, what is the most commonly mutated Point mutations of RAS family genes constitute the JAN CHRISTIAN MIDTERM 3
proto-oncogene seen in approximately 15% to most common type of abnormality involving proto- FELICIANO (TOP 2 - EXAM - MARCH
20% of all human tumors? oncogenes in human tumors. Approximately 15% to AUG 2015 MED 2016
A. RET 20% of all human tumors express mutated RAS BOARDS; TOPNOTCH
B. BCL-2 proteins, but in some types of cancers the frequency of MD FROM UST)
C. RAS RAS mutations is much higher. p53 and Rb are tumor
D. p53 suprresor genes not proto-oncogene
E. Rb

69 Which of these statements regarding True enough, aortas of infants can exhibit fatty JAN CHRISTIAN MIDTERM 3
atherosclerosis is correct? streaks, and such lesions are present in virtually FELICIANO (TOP 2 - EXAM - MARCH
A. It literally means “hardening of the arteries" all adolescents, even those without known risk AUG 2015 MED 2016
B. Fatt streaks are present in virtually all factors. Arteriosclerosis literally means “hardening of BOARDS; TOPNOTCH
adolescents even without risk factors the arteries”. Family history and not age is the most MD FROM UST)
C. Age is the most important independent risk important independent risk factor for atherosclerosis.
factor for atherosclerosis. lipid-filled macrophages are called foam cells. Collagen
D. Because the modified lipoproteins cannot be is the major structural component of the fibrous cap,
completely degraded, chronic ingestion leads to and accounts for its mechanical strength and stability.
the formation of lipid-filled macro- phages
called fatty streaks
E. Fibrinogen is the major structural
component of the fibrous cap, and accounts for
its mechanical strength and stability
70 Pertaining to the previous question, where is In descending order, the most extensively involved JAN CHRISTIAN MIDTERM 3
the most common site of atherosclerosis? vessels are the lower abdominal aorta, the coronary FELICIANO (TOP 2 - EXAM - MARCH
A. Abdominal aorta arteries, the popliteal arteries, the internal carotid AUG 2015 MED 2016
B. Coronorary artery arteries, and the vessels of the circle of Willis. BOARDS; TOPNOTCH
C. Internal carotid artery MD FROM UST)
D. Thoracic aorta
E. Popliteal artery

TOPNOTCH MEDICAL BOARD PREP PATHOLOGY SUPEREXAM Page 11 of 99


For inquiries visit www.topnotchboardprep.com.ph or email us at topnotchmedicalboardprep@gmail.com
TOPNOTCH MEDICAL BOARD PREP PATHOLOGY SUPEREXAM
For inquiries visit www.topnotchboardprep.com.ph or email us at topnotchmedicalboardprep@gmail.com
Item QUESTION EXPLANATION AUTHOR TOPNOTCH
# EXAM
71 A 70 yr old male had severe crushing chest pain SIMILAR TO PREVIOUS BOARD EXAM JAN CHRISTIAN MIDTERM 3
and died 3 hours later before he was brought to CONCEPT/PRINCIPLE. The gross and microscopic FELICIANO (TOP 2 - EXAM - MARCH
the ER. Cause of death is myocardial infarction. appearance of an infarct depends on the duration of AUG 2015 MED 2016
What do you expect to see on light microscopy survival of the patient following the MI. Light BOARDS; TOPNOTCH
on time of death? microcopy findings: Less than 30 mins- None; 30 MD FROM UST)
A. Early coagulation necrosis mins-4 hours- Usually none but variable waviness of
B. Marginal contraction band necrosis fibers at border; 4-12 hours- Early coag necrosis; 12-
C. Myocyte hypereosinophilia 24 hrs- pyknosis; myocyte hypereosniphilia and
D. Waviness of fibers marginal contraction band necrosis
E. Pyknosis of nuclei
72 A 50 yr old patient had routine checkup and the The case points to possible mitral valve prolapse JAN CHRISTIAN MIDTERM 3
only finding is a midsystolic click on wherein the underlying pathology is myxomatous FELICIANO (TOP 2 - EXAM - MARCH
auscultation. You are suspecting a valvular degeneration. The chordae tendineae become AUG 2015 MED 2016
defect. Which of the following pathologic elongated and can rupture to produce acute valvular BOARDS; TOPNOTCH
changes is most likely present in the valve? incompetence. MD FROM UST)
A. Destructive vegetations
B Dystrophic calcification
C Fibrinoid necrosis
D Myxomatous degeneration
E Rheumatic fibrosis
73 A patient came to you complaining of chronic Answer is A. Typo error. Choice should have been JAN CHRISTIAN MIDTERM 3
back pain. Radiagraphic exams shows multple Serum IgA. This is highly suggestive of multitple FELICIANO (TOP 2 - EXAM - MARCH
punched out lytic bone lesions. Which lab myeloma. IgG or IgA M proteins are almaot always AUG 2015 MED 2016
abnomrality is most likely? found in multiple myeloma. Additionaly lab criteria is BOARDS; TOPNOTCH
A. Serum IgM M protein is hypercalcemia, monocloonal urinary light chains MD FROM UST)
B. Hypocalemia and rolouex formation.
C. Increased serum alkaline phpsphatase
D. Marked splenomegaly
E. Polyclonal urinary light chains
74 This pneumoconioses is seen as Eggshell calcification of hilar lymph nodes is JAN CHRISTIAN MIDTERM 3
radiographically as an eggshell calcification of suggestive of silicosis. FELICIANO (TOP 2 - EXAM - MARCH
hilar lymph nodes described as stained glass in AUG 2015 MED 2016
appearance? BOARDS; TOPNOTCH
A. Asbestosis MD FROM UST)
B. Berylloisis
C. Coal worker's pneumoconioses
D. Silicosis
E. Hypersenstivity pneumonitis
75 This statement is correct regarding esophageal Some serotypes of Helicobacter pylori are associated JAN CHRISTIAN MIDTERM 3
cancer. with decreased risk of esophageal adenocarcinoma, FELICIANO (TOP 2 - EXAM - MARCH
A. Adenocarcinoma is more common because they cause gastric atrophy, which in turn AUG 2015 MED 2016
worldwide leads to reduced acid secretion and reflux, and BOARDS; TOPNOTCH
B. Half of squamous cell carcinomas occur in reduced incidence of Barrett esophagus. Squamous CA MD FROM UST)
the lower third of the esophagus is most common and occurs at middle third of
C. Reduced rates of Helicobacter pylori esophagus. AdenoCA produces mucin and is often the
infection may be a factor in the increasing intestinal type morphology. It is 7 times more
incidence of esophageal adenocarcinoma common in males than females.
D. Squamous cell CA most commonly produce
mucin and form glands often with intestinal
type morphology
E. Esophageal adenocarcinoma occurs most
frequently in Caucasians and is equally
distributed among genders
76 On endoscopy of a 14 year old boy complaining Grossly, the polyps are large and pedunculated with a JAN CHRISTIAN MIDTERM 3
of bleeding per rectum, a colonic mass was lobulated contour. Histologic examination FELICIANO (TOP 2 - EXAM - MARCH
seen. Histologic examination demonstrates a demonstrates a characteristic arborizing network of AUG 2015 MED 2016
characteristic arborizing network of connective connective tissue, smooth muscle, lamina propria, and BOARDS; TOPNOTCH
tissue, smooth muscle, lamina propria, and glands lined by normal-appearing intestinal MD FROM UST)
glands lined by normal-appearing intestinal epithelium. The arborization and presence of smooth
epithelium. What is the diagnosis? muscle intermixed with lamina propria are helpful in
A. Familial adenomatous polyposis distinguishing polyps of Peutz-Jeghers syndrome from
B. Peutz-Jeghers syndrome juvenile polyps.
C. Juvenile polyposis
D. Lynch syndrome
E. Colon Adenocarcinoma
77 What is the most important characteristic of Size is the most important characteristic that JAN CHRISTIAN MIDTERM 3
adenomatous polyps that best correlates with correlates with risk of malignancy. Although villous FELICIANO (TOP 2 - EXAM - MARCH
risk of malignancy? adenomas contain foci of invasion more frequently AUG 2015 MED 2016
A. Morphology (tubular or villous) than tubular adenomas, villous architecture alone BOARDS; TOPNOTCH
B. Degree of differentiation does not increase cancer risk when polyp size is MD FROM UST)
C. Size considered.
D. Presence of stalk (pedunculated or sessile)
E. APC proto-oncogene mutation

78 A patient is suspected of having alcholic liver SIMILAR TO PREVIOUS BOARD EXAM JAN CHRISTIAN MIDTERM 3
disease possible alcohol hepatitis. You expect to CONCEPT/PRINCIPLE. Do not rely on buzzwords. FELICIANO (TOP 2 - EXAM - MARCH
see what finding in liver biopsy? Understand the morhpology. Choice a refers to AUG 2015 MED 2016
A. Clumped, amorphous, eosinophilic material Mallory bodies and is characertistic of Alcoholic BOARDS; TOPNOTCH
in ballooned hepatocytes made up of tangled hepatitis. Choice B refrs to autoimmune hepatitis. MD FROM UST)
skeins of intermediate filaments Choice C refers to chronic HepB infection. Choice D
B. Plasma cell predominance in the refers to cholestatis. Choice E refers to Primary
mononuclear inflammatory infiltrates sclerosing cholangitis
C. Ground-glass hepatocytes
D. Extensive feathery degeneration of
periportal hepatocytes
E. Circumferential “onion skin” fibrosis around
TOPNOTCH MEDICAL BOARD PREP PATHOLOGY SUPEREXAM Page 12 of 99
For inquiries visit www.topnotchboardprep.com.ph or email us at topnotchmedicalboardprep@gmail.com
TOPNOTCH MEDICAL BOARD PREP PATHOLOGY SUPEREXAM
For inquiries visit www.topnotchboardprep.com.ph or email us at topnotchmedicalboardprep@gmail.com
Item QUESTION EXPLANATION AUTHOR TOPNOTCH
# EXAM
an increasingly atrophic duct lumen

79 A child was brought to your clinic complaining SIMILAR TO PREVIOUS BOARD EXAM JAN CHRISTIAN MIDTERM 3
of hematuria. He had sore throat 5 days prior to CONCEPT/PRINCIPLE.The case most likely points to FELICIANO (TOP 2 - EXAM - MARCH
consult. What do you expect to find on electron IgA nephropathy. PSGN usually appears 1 to 4 weeks AUG 2015 MED 2016
microscopy? after a streptococcal infection of the pharynx or skin BOARDS; TOPNOTCH
A. discrete, amorphous, electron-dense deposits (impetigo). Choice A refers to PSGN, Choice B is MD FROM UST)
on the epithelial side of the membrane, often miniman change disease. Choice C refers to MPGN
having the appearance of “humps Type 1. Choice E is RPGN
B. uniform and diffuse effacement of foot
processes
C. double contour” or “tram-track” appearance
D. presence of electron-dense deposits
predominantly in the mesangium
E. proliferation of parietal cells forming
crescents
80 What is the microscopic finding in diabetic SIMILAR TO PREVIOUS BOARD EXAM JAN CHRISTIAN MIDTERM 3
glomerulonephropathy? CONCEPT/PRINCIPLE. The most important FELICIANO (TOP 2 - EXAM - MARCH
A. Mesangial expansion glomerular lesions are capillary basement membrane AUG 2015 MED 2016
B. GBM thickening thickening, diffuse mesangial sclerosis, and nodular BOARDS; TOPNOTCH
C. Eosinophilic nodular glomerulosclerosiss glomerulosclerosis aka Kimmelstiel-Wilson disease. MD FROM UST)
D. Renal atherosclerosis Renal atherosclerosis and arteriolosclerosis constitute
E. All of the above part of the macrovascular disease in diabetics.

81 1. A 58 year old male presented with left sided liquefactive necrosis is characterized by digestion of ANDREW TIU (TOP 1 FINAL EXAM -
weakness associated with headache and dead cells resulting in transmoration of tissue into a - AUG 2015 MED MARCH 2016
vomiting 4 hours prior to admission. Patient is liquid viscous mass. for unknown reasons, hypoxic BOARDS; TOPNOTCH
a known hypertensive and currently takes death of cells within CNS often manifests as such. It is MD FROM CIM)
Losartan. What is the most probable pattern of also seen in bacterial and fungal infections.
tissue necrosis seen in this patient? Coagulative necrosis - architecture of dead tissues is
a. gangrenous necrosis preserved for a span of at least some days.
b. coagulative necrosis Gangrenous necrosis - usually applied to a limb that
c. liquefactive necrosis has lost its blood supply and has undergone typically
d. fat necrosis coagulative necrosis involving multiple tissue planes.
e. none of the above Fat necrosis refers to focal areas of fat destruction
from release of activated pancreatic lipases into
pancreas and peritoneal cavity.
(robbins 8th edition p.16)
82 2. A 20 year old male was bitten by a red ant on contraction of endothelial cells resulting in increased ANDREW TIU (TOP 1 FINAL EXAM -
the dorsum of his foot. After 30 minutes, he endothelial spaces is the most common mechanism of - AUG 2015 MED MARCH 2016
noticed his foot was swollen, erythematous, and vascular leakage and is elicited by histamine, BOARDS; TOPNOTCH
itchy. What is the most likely mechanism for the bradykinin, leukotrienes, substance P, etc. It is called MD FROM CIM)
symptoms? the immediate transient response because it occurs
a. increased transcytosis rapidly after exposure to the mediator. (Robbins 8th
b. endothelial injury edition p. 47)
c. retraction of endothelial cells
d. leukocyte mediated vascular injury
e. none of the above
83 3. After 1 day, patient noted increased pruritus, contraction of endothelial cells resulting in increased ANDREW TIU (TOP 1 FINAL EXAM -
swelling, and erythema over the dorsum of his endothelial spaces is the most common mechanism of - AUG 2015 MED MARCH 2016
foot after repeatedly scratching it. What is now vascular leakage and is elicited by histamine, BOARDS; TOPNOTCH
the most likely mechanism for the symptoms? bradykinin, leukotrienes, substance P, etc. It is called MD FROM CIM)
a. increased transcytosis the immediate transient response because it occurs
b. endothelial injury rapidly after exposure to the mediator.
c. retraction of endothelial cells (Robbins 8th edition p. 47)
d. leukocyte mediated vascular injury
e. none of the above
84 4. A 28 year old female, CRB, presents 1 month rim or peripheral staining pattern are most indicative ANDREW TIU (TOP 1 FINAL EXAM -
history of easy fatigability, knee pain, mouth of antibodies to dsDNA. Homogeneous or diffuse - AUG 2015 MED MARCH 2016
sores, and tea colored urine. Past medical usually reflects antibodies to chromatin, histones, and BOARDS; TOPNOTCH
history and family history was unremarkable. occasionally dsDNA. Speckled pattern is least specific MD FROM CIM)
On workup, the physician noted a positive ANA and is the most commonly observed. This includes Sm
and anti - dsDNA. Which of the following antigen, RNP, SS-A, SS-B.
patterns of nuclear fluorescence most likely (RObbins 8th edition p. 214)
suggests the type of antibody present in the
patient?
a. homogeneous
b. rim
c. speckled
d. nucleolar
e. none of the above

TOPNOTCH MEDICAL BOARD PREP PATHOLOGY SUPEREXAM Page 13 of 99


For inquiries visit www.topnotchboardprep.com.ph or email us at topnotchmedicalboardprep@gmail.com
TOPNOTCH MEDICAL BOARD PREP PATHOLOGY SUPEREXAM
For inquiries visit www.topnotchboardprep.com.ph or email us at topnotchmedicalboardprep@gmail.com
Item QUESTION EXPLANATION AUTHOR TOPNOTCH
# EXAM
85 5. Patient CRB was then started on steroids. robbins 8th edition p. 213 ANDREW TIU (TOP 1 FINAL EXAM -
Creatinine was noted to be slightly elevated on - AUG 2015 MED MARCH 2016
follow up. Which of the following is the BOARDS; TOPNOTCH
hallmark of her disease? MD FROM CIM)
a. malar rash
b. glomerulonephritis
c. production of autoantibodies
d. normocytic normochromic anemia
e. none of the above
86 6. A 2 year old female presented with 12 week robbins 8th edition p. 249 ANDREW TIU (TOP 1 FINAL EXAM -
history of recurrent, fever, cough, 15% weight - AUG 2015 MED MARCH 2016
loss, rash, and generalized lymphadenopathy. BOARDS; TOPNOTCH
Patient was started with antibiotics without MD FROM CIM)
relief of symptoms. Biopsy of the lymph node
was done which showed follicles depleted of
cells and the organized network of follicular
dendritic cells is disrupted. What is the most
likely diagnosis?
a. hodgkin's lymphoma
b. AIDS
c. ALL
d. Miliary TB
e. none of the above
87 7. A 56 year old farmer comes to you with benzene - leukemia, hodgkins lymphoma (principal ANDREW TIU (TOP 1 FINAL EXAM -
complaints of a painful chronic skin ulcer. component of light oil); beryllium - lung cancer - AUG 2015 MED MARCH 2016
Biopsy was done which revealed squamous cell (missile fuel and space vehicles); chromium - lung BOARDS; TOPNOTCH
carcinoma. On further workup, patient also has cancer (metal alloys, paints, pigments, preservatives) MD FROM CIM)
pulmonary nodules on the right middle lobe Robbins 8th edition p. 274
and left upper lobe. Which of the following
agents is most likely responsible?
a. benzene
b. beryllium
c. chromium
d. arsenic
e. none of the above
88 8. JCT, 5 year old child presents with fever, neck Mumps encephalitis causes perivenous demyelination ANDREW TIU (TOP 1 FINAL EXAM -
mass, and decreased level of consciousness. and perivascular mononuclear cuffing. Aseptic - AUG 2015 MED MARCH 2016
Past medical history includes pneumonia at 8 meningitis is the most common extrasalivary gland BOARDS; TOPNOTCH
months old and German measles at 2 years old. complication of mumps infection occuring in 10% of MD FROM CIM)
Biopsy of neck mass showed a edematous cases.
interstitium diffusely infiltrated with Robbins 8th edition p. 250
macrophages, lymphocytes, and plasma cells.
Which of the following is the most likely
etiologic agent?
a. SSPE
b. mumps
c. polio
d. EBV
e. none of the above
89 9. A 72 year old male chronic smoker presented Components of cigarette smoke particularly ANDREW TIU (TOP 1 FINAL EXAM -
with abdominal pain and bone pains at night. polycyclic hydrocarbons and nitrosamines are potent - AUG 2015 MED MARCH 2016
Serum calcium was noted to be elevated. Which carcinogens in animals and likely to be directly BOARDS; TOPNOTCH
of the following is the organ specific carcinogen involved in the development of lung cancer in humans. MD FROM CIM)
in tobacco smoke? Robbins 8th edition p. 411-412
a. tar
b. formaldehyde
c. polycyclic aromatic hydrocarbons
d. nicotine
e. none of the above
90 10. A previously healthy neonate presented Robbins 8th edition p.456 ANDREW TIU (TOP 1 FINAL EXAM -
with tachypnea, retractions, and cyanosis. CRP - AUG 2015 MED MARCH 2016
was elevated. What is the most likely diagnosis? BOARDS; TOPNOTCH
a. RDS type I MD FROM CIM)
b. RDS type II
c. ARDS
d. BPD
e. none of the above
91 11. Which of the following refers to a plaque Atherosclerotic plaques have 3 principal components: ANDREW TIU (TOP 1 FINAL EXAM -
witha superficial fibrous cap composed of 1) cells 2) ECM 3)intracellular and extracellular lipid. - AUG 2015 MED MARCH 2016
smooth muscle cells and relaively dense Robbins 8th edition p. 502 BOARDS; TOPNOTCH
collagen with a lipid core? MD FROM CIM)
a. atherosclerotic plaque
b. fatty streak
c. all of the above
d. none of the above
e. none of the above
92 12. A 58 year old retired Chinese teacher came In acute cor pulmonale, there is marked dilation of ANDREW TIU (TOP 1 FINAL EXAM -
in for multiple injuries secondary to fall in the right ventricle without hypertrophy. The rest of the - AUG 2015 MED MARCH 2016
bathroom. X ray was done and showed a choices refer to chronic cor pulmonale. BOARDS; TOPNOTCH
femoral fracture. ORIF was done and 48 hours Robbins 8th edition p. 560 MD FROM CIM)
post -op, patient was noted to be dyspneic,
tachycardic with O2 sats 68%. Despite
resuscitative efforts, patient died. What is the
most likely autopsy findings of the heart?
a. thickened right ventricular wall
b. dilated right ventricular wall
TOPNOTCH MEDICAL BOARD PREP PATHOLOGY SUPEREXAM Page 14 of 99
For inquiries visit www.topnotchboardprep.com.ph or email us at topnotchmedicalboardprep@gmail.com
TOPNOTCH MEDICAL BOARD PREP PATHOLOGY SUPEREXAM
For inquiries visit www.topnotchboardprep.com.ph or email us at topnotchmedicalboardprep@gmail.com
Item QUESTION EXPLANATION AUTHOR TOPNOTCH
# EXAM
c. disappareance of fat in the ventricular wall
d. regurgitation and fibrous thickening of
tricuspid valve
e. none of the above

93 13. An 8 year old male came for complaints of B - RHD, C - NBTE, D - Libmann sacks endocarditis ANDREW TIU (TOP 1 FINAL EXAM -
exertional dyspnea, recurrent high grade fever, robbins 8th edition p. 567 - AUG 2015 MED MARCH 2016
subcutaneous nodules in the pulps of the digits, BOARDS; TOPNOTCH
and tea colored urine. A year ago, patient was MD FROM CIM)
noted to have recurrent sore throat however no
consult was done and no medications were
taken. Which of the following is the hallmark
seen in this case?
a. friable bulky destructive vegetations
b. small warty vegetations along the lines of
closure
c. small bland vegetations along the lines of
closure
d. medium sized vegetations on either sides of
the valve leaflets
e. none of the above
94 14. One week prior to consult, an 18 year old Nodes involved in acute lymphadenitis are enlarged ANDREW TIU (TOP 1 FINAL EXAM -
nursing student came in for complaints of and painful. When pyogenic organisms are the cause, - AUG 2015 MED MARCH 2016
impacted wisdom tooth on the right side of the the centers of the follicles may undergo necrosis. BOARDS; TOPNOTCH
mandible. After 24 hours, a painful neck mass Choices A, B, and D refer to chronic nonspecific MD FROM CIM)
was noted. Which of the following morphology lymphadenitis.
of the neck mass may undergo necrosis and pus (robbins 8th edition p.595)
formation?
a. follicular hyperplasia
b. paracortical hyperplasia
c. acute lymphadenitis
d. reticular hyperplasia
e. none of the above
95 15. What stage of inflammatory response of the It is characterized by vascular engorgement, intra- ANDREW TIU (TOP 1 FINAL EXAM -
lung is described when the lung is heavy, boggy, alveolar fluid with few neutrophils and often the - AUG 2015 MED MARCH 2016
and red? presence of numerous bacteria. In red hepatization, BOARDS; TOPNOTCH
a. congestion the lobe now appears red, firm, and airless with a liver MD FROM CIM)
b. red hepatization - like consistency.
c. gray hepatization (robbins 8th edition p. 713)
d. resolution
e. none of the above
96 16. A 13 year old female presented with A - goodpasture's syndrome; B - MPGN type 1; D - ANDREW TIU (TOP 1 FINAL EXAM -
headache and decreased urine output after a 4 MPGN type 2; E - IgA nephropathy - AUG 2015 MED MARCH 2016
week history of skin infection. Which of the Robbins 8th edition p.921 BOARDS; TOPNOTCH
following is the most likely morphology of the MD FROM CIM)
kidney?
a. linear GBM fluorescence for Ig and
complement
b. discrete subendothelial electron dense
deposits
c. granular deposits of IgG, IgM, and C3 in the
mesangium and along the GBM
d. GBM transformed into an irregular ribbon
like, electron dense structure
e. mesangial deposition of IgA
97 17. An 17 year old female presented with B - uniform diffuse thickening of the glomerular ANDREW TIU (TOP 1 FINAL EXAM -
elevated blood pressure, tea colored urine, and capillary wall - AUG 2015 MED MARCH 2016
pallor. Kidney biopsy was done which revealed C - glomeruli show mesangial widening and BOARDS; TOPNOTCH
glomeruli being large, hypercellular, and having endocapillary proliferation MD FROM CIM)
an accentuated "lobular appearance". Which of D - crescents which are proliferation of parietal cells
the following is the most likely diagnosis? and migration of monocytes and macrophages into the
a. membranoproliferative glomerulonephritis urinary space
b. membranous nephropathy E - collapse of capillary loops, increase in matrix, and
c. IgA nephropathy segmental deposition of plasma proteins aong
d. rapid crescentic glomerulonephritis capillary wall (hyalinosis)
e. focal segmental glomerulosclerosis Robbins 8th edition p. 929
98 18. A 63 year old female, ECT, came in for Robbins 8th edition p. 945 ANDREW TIU (TOP 1 FINAL EXAM -
complaints of bilateral knee pain. ECT has been - AUG 2015 MED MARCH 2016
self medicating Ibuprofen for 5 years. For the BOARDS; TOPNOTCH
past week, ECT has been having recurrent MD FROM CIM)
headaches, vague abdominal pain, and easy
fatigability. Which of the following occurs first
in renal damage in analgesic nephropathy?
a. cortical tubulointerstitial nephritis
b. papillary necrosis
c. ischemic kidney injury
d. toxic kidney injury
e. none of the above

TOPNOTCH MEDICAL BOARD PREP PATHOLOGY SUPEREXAM Page 15 of 99


For inquiries visit www.topnotchboardprep.com.ph or email us at topnotchmedicalboardprep@gmail.com
TOPNOTCH MEDICAL BOARD PREP PATHOLOGY SUPEREXAM
For inquiries visit www.topnotchboardprep.com.ph or email us at topnotchmedicalboardprep@gmail.com
Item QUESTION EXPLANATION AUTHOR TOPNOTCH
# EXAM
99 19. Which of the following nipple discharges robbins 8th edition p. 1068 ANDREW TIU (TOP 1 FINAL EXAM -
would most likely signify malignancy? - AUG 2015 MED MARCH 2016
a. bloody BOARDS; TOPNOTCH
b. serous MD FROM CIM)
c. milky
d. both a and b
e. none of the above

100 20. A 14 year old female presented with a Yes during our exam, it was female and not male but ANDREW TIU (TOP 1 FINAL EXAM -
breast mass. Biopsy was done which showed the description was really referring to gynecomastia. - AUG 2015 MED MARCH 2016
dense collagenous connective tissue and A - delicate cellular, and often myxoid stroma which BOARDS; TOPNOTCH
marked micropapillary epithelial hyperplasia of resembles normal intralobular stroma. B - MD FROM CIM)
the duct lining. Which of the following is the nonproliferative changes which includes cysts,
most likely diagnosis? fibrosis, and adenosis. D - bulbous protrusions due to
a. fibroadenoma presence of nodules of proliferating stroma covered
b. fibrocystic disease by epithelium. E - multiple branching fibrovascular
c. gynecomastia cores each having a connective tissue axis lined by
d. phylloides tumor luminal and myoepithelial cells
e. Papilloma Robbins 8th edition p. 1093
101 Characteristic cytologic feature of medullary Page 73 of Topnotch Handout. Medullary thyroid ANGELA PAULINE P. DIAGNOSTIC
thyroid cancer: carcinoma has polygonal to spindle shaped cells, CALIMAG-LOYOLA EXAM - AUG
A. Orphan annie acellular amyloid deposits, and C-cell hyperplasia. (TOP 8 - FEB 2015 2015
B. Vacuolated nucleus MED BOARDS;
C. Amyloid stroma TOPNOTCH MD
D. Ground glass FROM UST)
E. Pleomorphic giant cells

102 Which of the following is a calcitonin secreting Page 73 of Topnotch Handout. Medully thyroid ANGELA PAULINE P. DIAGNOSTIC
tumor of the thyroid gland? carcinoma is a neuroendocrine neoplasma derived CALIMAG-LOYOLA EXAM - AUG
A. Papillary CA from the parafollicular cells or c-cells which secrete (TOP 8 - FEB 2015 2015
B. Follicular CA calcitonin and is important in diagnosis. MED BOARDS;
C. Medullary CA TOPNOTCH MD
D. Hurthle cell CA FROM UST)
E. Anaplastic CA

103 23y/o female consults because of bipedal Page 125 of Topnotch Handout. The most common ANGELA PAULINE P. DIAGNOSTIC
edema & passing out of tea colored urine. On and severe type of Lupus nephritis is Type IV: Diffuse CALIMAG-LOYOLA EXAM - AUG
PE, her BP is 160/100. she has +2 bipedal proliferative GN which has a wire-loop capillary (TOP 8 - FEB 2015 2015
edema. The urinalysis showed +4 protein, 50- appearance. MED BOARDS;
60RBC/hpf, 0-1 WBC/hpf and occasional red TOPNOTCH MD
cell casts. Serum creatinine is elevated 2mg/dL FROM UST)
from a previously normal level of 0.9 mg/dL 1
week ago. If this is a case of SLE, which of the
following is the most common and severe
histopathologic type?
A. Mesangial GN
B. Focal proliferative GN
C. Membranous GN
D. Difffuse proliferative GN
E. Cresenteric GN
104 A 40 y/o female was found to have a diastolic Page 112 of hand out. Mitral stenosis causes an ANGELA PAULINE P. DIAGNOSTIC
rumble at the apex. The chest x-ray showed increase in LA size due to restriction of blood flow CALIMAG-LOYOLA EXAM - AUG
uplifting of the left main stem bronchus, from the left atrium to the left ventricle as a result of a (TOP 8 - FEB 2015 2015
retrosternal fullness, and dilated main narrowed mitral passage. Murmurs associated with MED BOARDS;
pulmonary artery. What is the most likely MS are opening snap, accentuated S1, and diastolic TOPNOTCH MD
diagnosis? A. rumble. Plain film chest xray findings are FROM UST)
Mitral Stenosis cardiomegaly, double right heart border (enlarged left
B. Mitral Regurgitation atrium and normal right atrium), prominent left atrial
C. Aortic Stenosis appendage and splaying of the subcarinal angle (>120
D. Pulmonic Stenosis degrees).
E. Aortic Regurgitation
105 What is the expected thyroid function test in Page 71 of Topnotch Handout. Primary ANGELA PAULINE P. DIAGNOSTIC
primary hyperthyroidism? hyperthyroidism is the term used when the pathology CALIMAG-LOYOLA EXAM - AUG
A. decreased TSH, increased T4
is within the thyroid gland. Secondary (TOP 8 - FEB 2015 2015
B. decreased TSH, decreased T4
hyperthyroidism is the term used when the thyroid MED BOARDS;
C. increased TSH, increased T4
gland is stimulated by excessive thyroid-stimulating TOPNOTCH MD
D. increased TSH, decreased T4 hormone (TSH) in the circulation.The diagnosis of FROM UST)
E. None of the above hyperthyroidism is confirmed by blood tests that
show a decreased thyroid-stimulating hormone (TSH)
level and elevated T4 and T3 levels. A low TSH level
typically indicates that the pituitary gland is being
inhibited or "instructed" by the brain to cut back on
stimulating the thyroid gland, having sensed increased
levels of T4 and/or T3 in the blood.
106 What can generally differentiate between Page 115of Topnotch Handout. Asthma and COPD are ANGELA PAULINE P. DIAGNOSTIC
asthma and COPD in spirometry? both obstructive lung diseases. On spirometry FEV1, CALIMAG-LOYOLA EXAM - AUG
a. FEV1 FVC and FEV1/FVC are decreased in both how ever (TOP 8 - FEB 2015 2015
b. reversibility asthma is a reversible condition. MED BOARDS;
c. PEF TOPNOTCH MD
d. TLC FROM UST)
E. FVC

TOPNOTCH MEDICAL BOARD PREP PATHOLOGY SUPEREXAM Page 16 of 99


For inquiries visit www.topnotchboardprep.com.ph or email us at topnotchmedicalboardprep@gmail.com
TOPNOTCH MEDICAL BOARD PREP PATHOLOGY SUPEREXAM
For inquiries visit www.topnotchboardprep.com.ph or email us at topnotchmedicalboardprep@gmail.com
Item QUESTION EXPLANATION AUTHOR TOPNOTCH
# EXAM
107 Ranson’s criteria is used in acute panceatitis Page 101 of Topnotch Handout. Ranson's criteria is a ANGELA PAULINE P. DIAGNOSTIC
for: criteria for prognostication on admission and for the CALIMAG-LOYOLA EXAM - AUG
A. Diagnosis
first 48 hours. (TOP 8 - FEB 2015 2015
B. Monitor abscess formation MED BOARDS;
C. Predict possible etiology TOPNOTCH MD
D. Decide surgical intervention FROM UST)
E. Prognostication

108 Which collagen type is commonly found in the Page 8 of Topnotch Handout. Epidermolysis bullosa is ANGELA PAULINE P. DIAGNOSTIC
dermoepidermal junction and is usually caused by genetic defects (or mutations) within the CALIMAG-LOYOLA EXAM - AUG
defective in Epidermolysis bullosa: A. human COL7A1 gene encoding the protein type VII (TOP 8 - FEB 2015 2015
Type I collagen (collagen VII). MED BOARDS;
B. Type 3 TOPNOTCH MD
C. Type 5 FROM UST)
D. Type 7
E. Type 9
109 In a patient with pulmonary embolism the Page 11 of Topnotch Handout. The most common ECG ANGELA PAULINE P. DIAGNOSTIC
most common ECG manifestation that should manifestation is a sinus tachycardia. Right ventricular CALIMAG-LOYOLA EXAM - AUG
be expected is? A. strain pattern or S1Q3T3 is found in only 6% of (TOP 8 - FEB 2015 2015
Incomplete right bundle branch block patients. MED BOARDS;
B. Sinus tachycardia TOPNOTCH MD
C. Right ventricular strain pattern FROM UST)
D. S1Q3T3
E. Junctional rhythm
110 A patient diagnosed with drug-induced lupus Page 20 of Topnotch Handout. A patient with drug ANGELA PAULINE P. DIAGNOSTIC
would most likely be positive for which induced lupus will be positive to anti-histone. A- CALIMAG-LOYOLA EXAM - AUG
autoantibody? A. Sjorgren syndrome, B-CREST syndrome, D-SLE, E- (TOP 8 - FEB 2015 2015
Anti-SS-A Primary biliary cirrhosis MED BOARDS;
B. Anti-centromere TOPNOTCH MD
C. Anti-histone FROM UST)
D. Anti-dsDNA
E. Anti-mitochondrial
111 A 59 y/o male, received a blood group identical Page 23 of Topnotch Handout. Hyperacute rejection ANGELA PAULINE P. DIAGNOSTIC
living unrelated kidney graft. During surgery no occurs within minutes to hours due to preformed CALIMAG-LOYOLA EXAM - AUG
abnormalities occurred. Four hours after the antibodies. It is a Type II hypersensitibity reaction. (TOP 8 - FEB 2015 2015
transplantation, it was noted that diuresis Morphological features: thrombotic occlusion of MED BOARDS;
suddenly decreased. Upon repeat laparotomy capillaries and fibrinoid necrosis occurs in arterial TOPNOTCH MD
the transplanted kidney showed signs of walls. FROM UST)
hyperacute rejection and had to be removed.
Which pathological examination findings are
consistent with hyperacute rejection?
A. Thrombosis of capillaries and fibrinoid
necrosis in arterial walls
B. Presence of interstitial mononuclear cell
infiltration and edema
C. Necrotizing vasculitis withendothelial cell
necrosis
D. Interstitial fibrosis and tubular atrophy
with loss of renal parenchyma
E. Neutrophilic infiltration, deposition of Ig,
complement and fibrin
112 A 32-day-old female infant was admitted due to Page 25 of Topnotch Handout. This is a case of ANGELA PAULINE P. DIAGNOSTIC
delayed umbilical cord detachment and Leukocyte adhesion deficiency syndrome it is an CALIMAG-LOYOLA EXAM - AUG
omphalitis. After admission, CBC revealed autosomal recessive disease involving a mutation in (TOP 8 - FEB 2015 2015
severe leukocytosis, and there was poor clinical integrins hence defective adhesion LFA-1 proteins on MED BOARDS;
response to several kinds of antibiotics. She is the surface of phagocytes. It is commonly manifested TOPNOTCH MD
diagnosed to have aan immunodefiency disease bu severe pyogenic infections in infancy and delayed FROM UST)
specifically a phagocyte disorder. What is the cord separation.
most probable pathophysiologic mechanism of
this disease?
A. Mutation in WASP gene for actin filament
assembly
B. Mutation in DNA repair enzymes
C. Lack of NADPH oxidase activity
D. Failure of phagolysosomal fusion
E. Defective LFA-1 proteins
113 Which electrolyte abnormality is not usually Page 29 of Topnotch Handout. Tumor lysis syndrome ANGELA PAULINE P. DIAGNOSTIC
found in tumor lysis syndrome?
is an oncologic emergency that is caused by massive CALIMAG-LOYOLA EXAM - AUG
A. Hypocalcemia tumor cell lysis with the release of large amounts of (TOP 8 - FEB 2015 2015
B. Hypercalcemia potassium, phosphate, and nucleic acids into the MED BOARDS;
C. Hyperphosphatemia systemic circulation. TOPNOTCH MD
D. Hyperuricemia FROM UST)
E. Hyperkalemia

114 Early chronic myeloid leukemia (CML) and Page 30 of Topnotch Handout. CML must be ANGELA PAULINE P. DIAGNOSTIC
leukemoid reaction (LR) sometimes show differentiated from leukemoid reactions in which CALIMAG-LOYOLA EXAM - AUG
similar histological pictures. To differentiate there is a marked increase in myeloid elements (TOP 8 - FEB 2015 2015
between a leukemoid reaction and CML the secondary to infection, chronic inflammation and MED BOARDS;
following should be requested:
other causes. Both present with extreme leukocytosis TOPNOTCH MD
A. Neutrophil alkaline phosphatase however CML has low NAP and CRP. Cytogenetic FROM UST)
B. C Reactive Protein testing will determine the presence of the BCR-ABL
C. Cytogenetic testing gene.
D. A and B only
E. All of the above

TOPNOTCH MEDICAL BOARD PREP PATHOLOGY SUPEREXAM Page 17 of 99


For inquiries visit www.topnotchboardprep.com.ph or email us at topnotchmedicalboardprep@gmail.com
TOPNOTCH MEDICAL BOARD PREP PATHOLOGY SUPEREXAM
For inquiries visit www.topnotchboardprep.com.ph or email us at topnotchmedicalboardprep@gmail.com
Item QUESTION EXPLANATION AUTHOR TOPNOTCH
# EXAM
115 This type of Hodgkin's lymphoma which has an Page 32 of Topnotch Handout. Refer to the table on ANGELA PAULINE P. DIAGNOSTIC
intermediate prognosis, has a highly significant types of hodgkin's lymphoma. Mixed cellularity CALIMAG-LOYOLA EXAM - AUG
association with EBV infection and numerous involved lymph nodes are diffusely effaced by a (TOP 8 - FEB 2015 2015
R-S cells in a mixed inflammatory background heterogenous cellular infiltrate. Plentiful RS cells MED BOARDS;
that obliterates the normal architecture:
admixed with lymphocytes. It has an intermediate TOPNOTCH MD
A. Lymphocyte depleted prognosis and EBV-infected in 70% of cases. FROM UST)
B. Lymphocyte predominant
C. Lymphocyte rich
D. Mixed cellularity
E. Nodular sclerosis
116 Cold agglutinin disease is a form of Page 38 of Topnotch Handout. IgM antibodies ANGELA PAULINE P. DIAGNOSTIC
autoimmune hemolytic anemia caused by cold- generally cause cold agglutinin disease. M-Malamig! CALIMAG-LOYOLA EXAM - AUG
reacting autoantibodies. Autoantibodies bind to (TOP 8 - FEB 2015 2015
the erythrocyte membrane leading to MED BOARDS;
premature erythrocyte destruction. Which TOPNOTCH MD
antibody is commonly involved? FROM UST)
A. IgM
B. IgG
C. IgA
D. IgD
E. IgE
117 Incomplete excision of a dentigerous cyst may Page 42 of Topnotch Handout. Dentigerous cyst ANGELA PAULINE P. DIAGNOSTIC
result in a neoplastic transformation, originates around the crown of an unerupted tooth, CALIMAG-LOYOLA EXAM - AUG
specifically into a/an: A. often associated with an impacted third molar. (TOP 8 - FEB 2015 2015
Odontoma Complete excision is curative, however incomplete MED BOARDS;
B. Ameloblastoma excision may result to recurrence or neoplastic TOPNOTCH MD
C. Basal cell carcinoma transformation into an ameloblastoma or a squamous FROM UST)
D. Cholesteatoma cell carcinoma.
E. Paraganglioma
118 A primary melanoma located in which part of SIMILAR TO PREVIOUS BOARD EXAM ANGELA PAULINE P. DIAGNOSTIC
the body has the worst prognosis?
CONCEPT/PRINCIPLE. Page 44 of Topnotch Handout. CALIMAG-LOYOLA EXAM - AUG
A. Sole Anatomic location of the primary melanoma is an (TOP 8 - FEB 2015 2015
B. Palm important independent predictor of SLN status and MED BOARDS;
C. Scalp prognosis. Patients with primary melanomas of the TOPNOTCH MD
D. Chest head/neck and trunk have a worse prognosis than FROM UST)
E. Back primary melanomas of other anatomic locations.

119 Penile carcinoma in situ has a strong SIMILAR TO PREVIOUS BOARD EXAM ANGELA PAULINE P. DIAGNOSTIC
association with:
CONCEPT/PRINCIPLE. Page 52 of Topnotch Handout. CALIMAG-LOYOLA EXAM - AUG
A. HPV 6 Penile Carcinoma in situ is strongly associated with (TOP 8 - FEB 2015 2015
B. HPV 11 HPV 16 infection. MED BOARDS;
C. HPV 16 TOPNOTCH MD
D. HPV 18 FROM UST)
E. HPV 31

120 Psammoma bodies are frequently encountered Page 59 of Topnotch Handout. PSaMMoma bodies are ANGELA PAULINE P. DIAGNOSTIC
in the following conditions except?
encountered in A-D. CALIMAG-LOYOLA EXAM - AUG
A. Papillary thyroid CA
(TOP 8 - FEB 2015 2015
B. Serous cystadeno CA MED BOARDS;
C. Meningioma TOPNOTCH MD
D. Mesothelioma FROM UST)
E. Medullary thyroid CA

121 Which of the following is an effect of PAF? Platelet activating factor is a phospholipid-dervided LYNN DARYL MIDTERM 1
A. Vasodilation mediator that has multiple inflammatory effects. It FELICIANO EXAM - AUG
B. Bronchoconstriction cause platelet aggregation, vasoconstriction, VILLAMATER, MD 2015
C. Decreased vascular permeability bronchoconstriction, increased venular permeability, (TOP 5 - FEB 2015
D. Decreased platelet aggregation increased leukocyte adhesion to endothelium and MED BOARDS;
E. Decreased leukocyte adhesion chemotaxis. (Robbin's) PAF causes TOPNOTCH MD
bronchoconstriction and also vasodilation in low FROM EAC)
doses. SIMILAR TO PREVIOUS BOARD EXAM
CONCEPT.
122 Cell-derived mediators of inflammation, in Options B and C are properties of plasma-derived LYNN DARYL MIDTERM 1
contrast with plasma-derived mediators mediators of inflammation. Option D is incorrect. FELICIANO EXAM - AUG
A. Are normally sequestered in granule and Both types of mediators can act on one or few target VILLAMATER, MD 2015
can be rapidly secreted by granule exocytosis in cells. SIMILAR TO PREVIOUS BOARD EXAM CONCEPT. (TOP 5 - FEB 2015
response to stimulus. MED BOARDS;
B. Are produced mainly in the liver. TOPNOTCH MD
C. Are inactive precursors that must be FROM EAC)
activated, usually by a series of proteolytic
cleavages to acquire their biologic properties.
D. Act only in one or few target cell types.
E. None of the above.
123 The following presents with granulomatous Histoplasma mimics tuberculosis. All except E would LYNN DARYL MIDTERM 1
inflammation EXCEPT: show granulomatous type of inflammation in the FELICIANO EXAM - AUG
A. Histoplasmosis affected organs. SIMILAR TO PREVIOUS BOARD EXAM VILLAMATER, MD 2015
B. Tuberculosis CONCEPT. (TOP 5 - FEB 2015
C. Sarcoidosis MED BOARDS;
D. Schistomiasis TOPNOTCH MD
E. Molluscum contagiosum FROM EAC)

TOPNOTCH MEDICAL BOARD PREP PATHOLOGY SUPEREXAM Page 18 of 99


For inquiries visit www.topnotchboardprep.com.ph or email us at topnotchmedicalboardprep@gmail.com
TOPNOTCH MEDICAL BOARD PREP PATHOLOGY SUPEREXAM
For inquiries visit www.topnotchboardprep.com.ph or email us at topnotchmedicalboardprep@gmail.com
Item QUESTION EXPLANATION AUTHOR TOPNOTCH
# EXAM
124 What is the mechanism of edema in nephrotic Increased permeability to plasma proteins resulting LYNN DARYL MIDTERM 1
syndrome? from either structural or physicochemical alteration FELICIANO EXAM - AUG
A. Glomerular injury allows protein to escape from the plasma into the VILLAMATER, MD 2015
B. Increased capillary permeability to proteins urinary space. Massive proteinuria depletes serum (TOP 5 - FEB 2015
C. Decreased protein absorption albumin resulting in hypoalbuminemia, and thus MED BOARDS;
D. Tubulointerstitial disorder decreased colloid osmotic pressure of the blood with TOPNOTCH MD
E. Increased hydrostatic pressure subsequent accumulation of fluid in the interstitial FROM EAC)
tissues. Sodium and water retention due to
compensatory secretion of aldosterone and
stimulation of sympathetic system also contributes
and aggravates the edema. SIMILAR TO PREVIOUS
BOARD EXAM CONCEPT.
125 A 24-year old male patient presented with This is a morphologic description of Tay-Sachs LYNN DARYL MIDTERM 1
cherry-red spots in the macula. Morphology of Disease, a deficiency of hexosaminidase. SIMILAR TO FELICIANO EXAM - AUG
the brain shows neurons ballooned with PREVIOUS BOARD EXAM CONCEPT. VILLAMATER, MD 2015
cytoplasmic vacuoles. What enzyme is deficient (TOP 5 - FEB 2015
in this condition? MED BOARDS;
A. Alpha 1,4-glucosidase TOPNOTCH MD
B. Sphingomyelinase FROM EAC)
C. Arylsulfatase
D. Iduronidase
E. Hexosaminidase
126 A 48-year old male patient who underwent Hyperacute rejection occurs few minutes to few hours LYNN DARYL MIDTERM 1
kidney transplant suddenly developed bloody after transplant. Option A describes Acute cellular FELICIANO EXAM - AUG
urine few hours after the procedure. rejection. Options C and D are mophologic findings VILLAMATER, MD 2015
Morphologic changes in this pattern of rejection found in acute humoral rejection. Option E describes (TOP 5 - FEB 2015
will reveal: the morphology of kidney in chronic rejection. MED BOARDS;
A. Extensive interstitial mononuclear cell TOPNOTCH MD
infiltration and edema FROM EAC)
B. Thrombotic occlusion of capillaries and
fibrinoid necrosis
C. Necrotizing vasculitis with endothelial cell
necrosis
D. Neutrophilic infiltration and deposition of
immunoglobulin, complement, and fibrin
E. Interstitial fibrosis and tubular atrophy
with loss of renal parenchyma.
127 What is the hallmark of tissue repair? SIMILAR TO PREVIOUS BOARD EXAM CONCEPT. LYNN DARYL MIDTERM 1
A. Vasoconstriction FELICIANO EXAM - AUG
B. Blood clot formation VILLAMATER, MD 2015
C. Granulation tissue (TOP 5 - FEB 2015
D. Tissue remodeling MED BOARDS;
E. Wound contraction TOPNOTCH MD
FROM EAC)

128 The first step in phagocytosis is: Steps in phagocytosis: 1. Recognition and attachment; LYNN DARYL MIDTERM 1
A. Formation of phagocytic vacuole 2. Engulfment and formation of phagocytic vacuole, 3. FELICIANO EXAM - AUG
B. Degradation of ingested material Killing and degradation of ingested material. SIMILAR VILLAMATER, MD 2015
C. Recognition of particle TO PREVIOUS BOARD EXAM CONCEPT. (TOP 5 - FEB 2015
D. Attachment MED BOARDS;
E. Engulfment TOPNOTCH MD
FROM EAC)

129 An 18-year old male patient presents with easy The diagnosis of AML is based on the presence of at LYNN DARYL MIDTERM 1
fatigability, fever, and cutaneous bleeding. least 20% myeloid blasts in the bone marrow. Robbins FELICIANO EXAM - AUG
Bone marrow biopsy showed 40% myeloblast. 9th ed., p. 613 . The most common manifestation of VILLAMATER, MD 2015
What is the most likely diagnosis? AML include fever, easy fatigability and bleeding. (TOP 5 - FEB 2015
A. ALL SIMILAR TO PREVIOUS BOARD EXAM CONCEPT. MED BOARDS;
B. AML TOPNOTCH MD
C. CML FROM EAC)
D. Burkitt's lymphoma
E. Adult T cell lymphoma
130 Morphologic finding/s in alcoholic hepatitis The rest are features of viral hepatitis. SIMILAR TO LYNN DARYL MIDTERM 1
include: PREVIOUS BOARD EXAM CONCEPT. FELICIANO EXAM - AUG
A. Councilman bodies VILLAMATER, MD 2015
B. Hepatocyte swelling and necrosis (TOP 5 - FEB 2015
C. Lymphoid aggregates within portal tracts MED BOARDS;
D. Hepatocyte apoptosis TOPNOTCH MD
E. All of the above FROM EAC)

131 The most common cause of sudden cardiac SIMILAR TO PREVIOUS BOARD EXAM CONCEPT. LYNN DARYL MIDTERM 1
death in Myocardial infaction is FELICIANO EXAM - AUG
A. Congestive heart failure VILLAMATER, MD 2015
B. Ventricular fibrillation (TOP 5 - FEB 2015
C. Pulmonary edema MED BOARDS;
D. Acute pericarditis TOPNOTCH MD
E. Ventricular rupture FROM EAC)

132 Juxta-articular osteopenia is characteristic of SIMILAR TO PREVIOUS BOARD EXAM CONCEPT. LYNN DARYL MIDTERM 1
A. Systemic lupus erythematosus FELICIANO EXAM - AUG
B. Rheumatoid arthritis VILLAMATER, MD 2015
C. Osteoarthritis (TOP 5 - FEB 2015
D. Ankylosing spondylitis MED BOARDS;
E. Gouty arthritis TOPNOTCH MD
FROM EAC)

TOPNOTCH MEDICAL BOARD PREP PATHOLOGY SUPEREXAM Page 19 of 99


For inquiries visit www.topnotchboardprep.com.ph or email us at topnotchmedicalboardprep@gmail.com
TOPNOTCH MEDICAL BOARD PREP PATHOLOGY SUPEREXAM
For inquiries visit www.topnotchboardprep.com.ph or email us at topnotchmedicalboardprep@gmail.com
Item QUESTION EXPLANATION AUTHOR TOPNOTCH
# EXAM
133 True of female hemophilia carrier Hemophilia is an X-linked recessive disorders wherein LYNN DARYL MIDTERM 1
A. Both X chromosomes are defective heterozygous female or female carriers does not FELICIANO EXAM - AUG
B. Decrease Factor VIII express full phenotypic change because of paired VILLAMATER, MD 2015
C. 25% of her offspring are affected normal allele, with random inactivation of one of the X (TOP 5 - FEB 2015
D. One of the X chromosome shows chromosome leading to variability. Only 1 of the X MED BOARDS;
abnormality chromosome is abnormal. Decrease in Factor VIII is a TOPNOTCH MD
E. Transmits disease to half her sons and half manifestation of affected male offspring. Option C is FROM EAC)
her daughters characteristic of AR while Option E is a property of X-
Linked Dominant disorders.
134 An 8-year old male presented with sunburn- This is a case of Staphylococcal scalded-skin syndrome LYNN DARYL MIDTERM 1
like rash that spread over the entire body and or Ritter disease caused by S. aureus. It is FELICIANO EXAM - AUG
evolves into fragile bullae. Desquamation of distinguished from toxic epidermal necrolysis or VILLAMATER, MD 2015
epidermis follows at the level of granulosa Lyell’s disease which is secondary to drug (TOP 5 - FEB 2015
layer. This is most likely caused by: hypersensitivity and causes desquamation at the level MED BOARDS;
A. Staphylococcus aureus of epidermal-dermal junction. TOPNOTCH MD
B. Streptococcus pyogenes FROM EAC)
C. Viridans streptococcus
D. Drug hypersensitivity
E. Pseudomonas aeruginosa
135 A 62-year old female presented with fever and Pseudomonas causes necrotizing pneumonia, LYNN DARYL MIDTERM 1
cough for the past few days. She also had vasculitis accompanied by thrombosis and FELICIANO EXAM - AUG
necrotizing oval lesion on her extremities. The hemorrhage. It proliferates widely, penetrating VILLAMATER, MD 2015
most likely etiology is: deeply into the veins and spreads hematogenously. (TOP 5 - FEB 2015
A. Staphylococcus aureus Ecthyma gangrenosum, well-demarcated necrotic and MED BOARDS;
B. Streptococcus pyogenes hemorrhagic oval skin lesion, often appear. TOPNOTCH MD
C. Clostridium perfringens FROM EAC)
D. Bacillus anthracis
E. Pseudomonas aeruginosa
136 The most likely renal pathology in multiple Renal insufficiency in multiple myeloma can be due to LYNN DARYL MIDTERM 1
myeloma is: Bence Jones proteins which are directly toxic to FELICIANO EXAM - AUG
A. Tubulo-interstitial nephritis epithelial cell, and accumulation of light chains which VILLAMATER, MD 2015
B. Membranoproliferative glomerulonephritis results to tubulo-interstitial nephritis. Other causes (TOP 5 - FEB 2015
C. Acute glomerulonephritis include hypercalcemia nad hyperuricemia. MED BOARDS;
D. Focal segmental glomerulosclerosis TOPNOTCH MD
E. Tubular necrosis FROM EAC)

137 A 45-year old male, smoker, presented with Chronic cough, copious sputum in a smoker points to LYNN DARYL MIDTERM 1
cough, copious sputum, and progressive chronic bronchitis as the diagnosis. Option A - FELICIANO EXAM - AUG
dyspnea on exertion for the last two years. On emphysema. Option C - Bronchiectasis. Option D - VILLAMATER, MD 2015
physical examination, he has wheezes all over Asthma; Option E - Idiopathic interstitial fibrosis. (TOP 5 - FEB 2015
his lung fields, and cyanotic lips. Morphologic SIMILAR TO PREVIOUS BOARD EXAM CONCEPT. MED BOARDS;
findings of the lungs in this condition will most TOPNOTCH MD
likely show a/an: FROM EAC)
A. Abnormally large alveoli separated by thin
septa and deformed respiratory bronchioles.
B. Enlargement of the mucus-secreting glands
of the bronchi and hyperemia and edema of the
mucus membranes.
C. Dilatation of bronchi and bronchioles
caused by destruction of the muscle and elastic
tissue.
D. Thickening of the basement membrane,
edema and inflammatory infiltrate in the
bronchial walls.
E. Cobblestone pleural surfaces and patchy
interstitial fibrosis varying in intensity.
138 The most common malignancy of the stomach: SIMILAR TO PREVIOUS BOARD EXAM CONCEPT. LYNN DARYL MIDTERM 1
A. Carcinoid FELICIANO EXAM - AUG
B. Lymphoma VILLAMATER, MD 2015
C. Adenocarcinoma (TOP 5 - FEB 2015
D. Squamous cell carcinoma MED BOARDS;
E. Gastrointestinal stromal tumor TOPNOTCH MD
FROM EAC)

139 A 57-year old, obese male was brought to the This is a case of acute myocardial infarction. LYNN DARYL MIDTERM 1
hospital because of sudden onset of chest pain, Myocardial necrosis begins at approximately 30 FELICIANO EXAM - AUG
characterized as squeezing, accompanied by minutes after coronary occlusion. SIMILAR TO VILLAMATER, MD 2015
diaphoresis and nausea. Neutrophils were PREVIOUS BOARD EXAM CONCEPT. (TOP 5 - FEB 2015
noted to be elevated. What is the MED BOARDS;
pathophysiologic mechanism of his condition? TOPNOTCH MD
A. Tissue necrosis FROM EAC)
B. Alveolar edema
C. Infection
D. Inflammation of airways
E. B and C
140 Thrombosis of what vessel will most most Right coronary artery (30-40%) infarct involves the LYNN DARYL MIDTERM 1
likely involve the posterior portion of the inferior/posterior wall of LV, posterior portion of FELICIANO EXAM - AUG
ventricular septum? ventricular septum, inferior/posterior RV free wall in VILLAMATER, MD 2015
A. Left anterior descending coronary artery some cases; LADA involves the anterior wall of LV (TOP 5 - FEB 2015
B. Left circumflex coronary artery near apex, anterior portion of ventricular septum and MED BOARDS;
C. Marginal branch of left circumflex artery apex circumferentially; Left circumflex artery involves TOPNOTCH MD
D. Right coronary artery the lateral wall of left ventricle except the apex. FROM EAC)
E. Left main coronary artery (Robbin's)

TOPNOTCH MEDICAL BOARD PREP PATHOLOGY SUPEREXAM Page 20 of 99


For inquiries visit www.topnotchboardprep.com.ph or email us at topnotchmedicalboardprep@gmail.com
TOPNOTCH MEDICAL BOARD PREP PATHOLOGY SUPEREXAM
For inquiries visit www.topnotchboardprep.com.ph or email us at topnotchmedicalboardprep@gmail.com
Item QUESTION EXPLANATION AUTHOR TOPNOTCH
# EXAM
141 A 30 year old male admitted for dengue fever Increased capillary permeability is the EDWARD HARRY MIDTERM 2
on the 7th day of illness develops pleural pathophysiologic mechanism of pleural effusion in VALLAJERA, MD EXAM - AUG
effusion, what is the explanation for the pleural dengue fever more than decreased platelets (TOP 8 - FEB 2015 2015
effusion? MED BOARDS;
A. Decreased plasma oncotic pressure TOPNOTCH MD
B. Increased capillary permeability FROM PERPETUAL
C. Increased capillary hydrostatic pressure BINAN)
D. Decreased platelets
E. None of the above
142 JS, a 70 year old male who was a smoker of 40 The major (primary bronchus) is the most common EDWARD HARRY MIDTERM 2
pack years was diagnosed with squamous cell site of development of squamous cell carcinoma of the VALLAJERA, MD EXAM - AUG
carcinoma of the lung, which among the lung (TOP 8 - FEB 2015 2015
following is the most common site of origin of MED BOARDS;
this lesion? TOPNOTCH MD
A. Trachea FROM PERPETUAL
B. Secondary bronchus BINAN)
C. Primary bronchus
D. Terminal bronchioles
E. None of the above
143 RLDL, a 40 year old female was a diagnosed Diffuse proliferative GN is the most common as well as EDWARD HARRY MIDTERM 2
case of SLE, she had a renal biopsy done and the most severe form of glomerulonephritis VALLAJERA, MD EXAM - AUG
results were brought to you, you know that the (TOP 8 - FEB 2015 2015
most common type of renal lesion of lupus has: MED BOARDS;
A. Mesangial lupus glomerulonephritis TOPNOTCH MD
B. Diffuse proliferative glomerulonephritis FROM PERPETUAL
C. Membranous glomerulopathy BINAN)
D. Focal proliferative glomerulopathy
E. None of the above

144 What is the cytokine that stimulates collagen TGF-B stimulates collagen synthesis, IL-1 stimulates EDWARD HARRY MIDTERM 2
synthesis fever as well as TNF, VEGF causes angiogenesis, PDGF VALLAJERA, MD EXAM - AUG
A. TNF causes collagenase secretion. (TOP 8 - FEB 2015 2015
B. IL-1 MED BOARDS;
C. VEGF TOPNOTCH MD
D. TGF-B FROM PERPETUAL
E. PDGF BINAN)

145 MIV, a 48 year old female underwent Pap Koilocytic atypia is a characteristic finding in the pap EDWARD HARRY MIDTERM 2
smear, which of the following cytologic findings smear which suggests HPV infection VALLAJERA, MD EXAM - AUG
suggest the presence of HPV infection (TOP 8 - FEB 2015 2015
A. Atypical ductal cells MED BOARDS;
B. Diffuse thickening of the basement TOPNOTCH MD
membrane FROM PERPETUAL
C. Koilocytic atypia BINAN)
D. A and B
E. None of the above
146 RF, a 30 year old male underwent lymph node Starry sky pattern is seen in Burkitt's lymphoma EDWARD HARRY MIDTERM 2
biopsy, the result showed a starry sky pattern, which is associated with EBV infection VALLAJERA, MD EXAM - AUG
you know that his malignancy is associated (TOP 8 - FEB 2015 2015
with? MED BOARDS;
A. Hepatitis B infection TOPNOTCH MD
B. Cytomegalovirus FROM PERPETUAL
C. Herpes zoster BINAN)
D. Epstein Barr virus
E. Infection with viruses belonging to filoviridae
family
147 Which among the following is important in the HIV invades the CD4 cells which are also responsible EDWARD HARRY MIDTERM 2
pathology of HIV infection? for the cell mediated immunity VALLAJERA, MD EXAM - AUG
A. CD8 (TOP 8 - FEB 2015 2015
B. CD4 MED BOARDS;
C. Macrophages TOPNOTCH MD
D. Dendritic cells FROM PERPETUAL
E. Neutrophils BINAN)

148 A patient was diagnosed with melanoma, one of The number of skin dendritic cells is not a prognostic EDWARD HARRY MIDTERM 2
the following is a prognostic factor for factor for melanoma VALLAJERA, MD EXAM - AUG
melanoma except (TOP 8 - FEB 2015 2015
A. tumor depth MED BOARDS;
B. presence of tumor infiltrating lymphocytes TOPNOTCH MD
C. number of dendritic cells FROM PERPETUAL
D. location of the tumor BINAN)
E. gender

149 BB, a 70 year old chronic alcoholic presents to Mallory bodies are eosinophilic cytoplasmic clumps in EDWARD HARRY MIDTERM 2
you with changes in sensorium, you requested a hepatocytes signifying liver injury, councilman bodies VALLAJERA, MD EXAM - AUG
liver biopsy, which of the following would you are eosinophilic globules seen in acute hepatitis, Negri (TOP 8 - FEB 2015 2015
expect to see: bodies are associated with rabies while psamomma MED BOARDS;
A. Councilman bodies bodies are associated with meningioma, prolactinoma, TOPNOTCH MD
B. Mallory bodies ovarian serous cystadenoma and papillary thyroid CA. FROM PERPETUAL
C. Negri bodies BINAN)
D. Psamomma bodies
E. None of the above

TOPNOTCH MEDICAL BOARD PREP PATHOLOGY SUPEREXAM Page 21 of 99


For inquiries visit www.topnotchboardprep.com.ph or email us at topnotchmedicalboardprep@gmail.com
TOPNOTCH MEDICAL BOARD PREP PATHOLOGY SUPEREXAM
For inquiries visit www.topnotchboardprep.com.ph or email us at topnotchmedicalboardprep@gmail.com
Item QUESTION EXPLANATION AUTHOR TOPNOTCH
# EXAM
150 MBDM, a 29 year old male underwent a tissue AML presents with >20% of myeloblasts in the bone EDWARD HARRY MIDTERM 2
section that showed 40% myeloid cells, what is marrow. VALLAJERA, MD EXAM - AUG
the diagnosis? (TOP 8 - FEB 2015 2015
A. CLL MED BOARDS;
B. AML TOPNOTCH MD
C. ALL FROM PERPETUAL
D. Mantle cell lymphoma BINAN)
E. Non-Hodgkins lymphoma

151 Most common bone involved in osteoporosis The vertebra are the most commonly affected as it is a EDWARD HARRY MIDTERM 2
A. Pelvis weight bearing bone. VALLAJERA, MD EXAM - AUG
B. Tibia (TOP 8 - FEB 2015 2015
C. Femur MED BOARDS;
D. Vertebra TOPNOTCH MD
E. Ribcage FROM PERPETUAL
BINAN)

152 What is the most common cause of nephritic EDWARD HARRY MIDTERM 2
syndrome in children? VALLAJERA, MD EXAM - AUG
A. Hepatitis B (TOP 8 - FEB 2015 2015
B. Intake of toxic substances MED BOARDS;
C. Antecedent GABHS infection TOPNOTCH MD
D. Viral infections FROM PERPETUAL
E. None of the above BINAN)

153 Which of the following lesions is more prone to Lobular carcinoma in situ is a fertile ground for breast EDWARD HARRY MIDTERM 2
develop breast CA cancer, proliferation without atypia and proliferation VALLAJERA, MD EXAM - AUG
A. Proliferative atypia with atypia is hyperplasia, fibroadenoma is a benign (TOP 8 - FEB 2015 2015
B. Proliferation without atypia condition. MED BOARDS;
C. Carcinoma in situ TOPNOTCH MD
D. Fibroadenoma FROM PERPETUAL
E. None of the above BINAN)

154 Which among the following is involved in Degree of differentiation or tissue grading is not EDWARD HARRY MIDTERM 2
staging a particular malignancy except needed in the staging in most malignancies VALLAJERA, MD EXAM - AUG
A. Presence of metastasis (TOP 8 - FEB 2015 2015
B. Lymph node involvement MED BOARDS;
C. Degree of differentiation TOPNOTCH MD
D. Size of the mass FROM PERPETUAL
E. None of the above BINAN)

155 KB, a 32 year old female fond of using beauty Type IV or delayed hypersensitivity develops usually EDWARD HARRY MIDTERM 2
products tried a new product on her face, 5 after 48 hours on application of offending agent on the VALLAJERA, MD EXAM - AUG
days later, she noticed redness on the areas of skin of the patient. (TOP 8 - FEB 2015 2015
application of the new product, what is the MED BOARDS;
most likely explanation: TOPNOTCH MD
A. IgE mediated immune reaction FROM PERPETUAL
B. Cytotoxin mediated BINAN)
C. Immunoglobulin-antigen complex
D. Cell mediated hypersensitivity
E. None of the above
156 What is the mechanism of DIC in All of the above are the mechanism of DIC EDWARD HARRY MIDTERM 2
meningococcemia? VALLAJERA, MD EXAM - AUG
A. Massive intravascular coagulation occuring (TOP 8 - FEB 2015 2015
everywhere in the circulation MED BOARDS;
B. Endothelial dysfunction TOPNOTCH MD
C. Depletion of clotting factors FROM PERPETUAL
D. All of the above BINAN)
E. None of the above
157 A 2 week old infant had been undergoing The most likely cause is septic shock due to systemic EDWARD HARRY MIDTERM 2
treatment for sepsis when the patient went into vasodilation. VALLAJERA, MD EXAM - AUG
hypotension and died, what is the most likely (TOP 8 - FEB 2015 2015
explanation for the infant's death? MED BOARDS;
A. DIC TOPNOTCH MD
B. Sepsis FROM PERPETUAL
C. Thrombocytopenia BINAN)
D. B and C
E. None of the above
158 The classic anaphylactic reaction is due to: SIMILAR TO PREVIOUS BOARD EXAM EDWARD HARRY MIDTERM 2
A. Th1 CONCEPT/PRINCIPLE. Th2 subset of helper T cells are VALLAJERA, MD EXAM - AUG
B. Th2 overactive in patients who develop the classic (TOP 8 - FEB 2015 2015
C. NK cell anaphylactic reaction MED BOARDS;
D. Macrophage TOPNOTCH MD
E. None of the above FROM PERPETUAL
BINAN)

159 Among the following adaptations of the body to Metaplasia is defined as the change from one adult cell EDWARD HARRY MIDTERM 2
injury, which of the following is a fertile ground type into another and is a fertile ground for VALLAJERA, MD EXAM - AUG
for neoplasia? malignancies, examples of malignancies originating (TOP 8 - FEB 2015 2015
A. Hyperplasia from metaplastic change include Barett's esophagus MED BOARDS;
B. Atrophy leading to esophageal adenoCA, cervical CA from TOPNOTCH MD
C. Hypertrophy squamous metaplasia of cervical columnar cells, SQCA FROM PERPETUAL
D. Metaplasia of the lung due to squamous metaplasia of respiratory BINAN)
E. None of the above epithelium.

TOPNOTCH MEDICAL BOARD PREP PATHOLOGY SUPEREXAM Page 22 of 99


For inquiries visit www.topnotchboardprep.com.ph or email us at topnotchmedicalboardprep@gmail.com
TOPNOTCH MEDICAL BOARD PREP PATHOLOGY SUPEREXAM
For inquiries visit www.topnotchboardprep.com.ph or email us at topnotchmedicalboardprep@gmail.com
Item QUESTION EXPLANATION AUTHOR TOPNOTCH
# EXAM
160 What is the most common type of gastric Gastric adenoCA is the most common type of gastric EDWARD HARRY MIDTERM 2
malignancy? malignancy VALLAJERA, MD EXAM - AUG
A. Lymphoma (TOP 8 - FEB 2015 2015
B. Gastric adenocarcinoma MED BOARDS;
C. Carcinoid tumor TOPNOTCH MD
D. Gastric adenoma FROM PERPETUAL
E. Pleomorphic adenoma BINAN)

161 This can be a pathologic or physiologic process hypertrophy is increase in size of cells. Atrophy is HAROLD JAY S. MIDTERM 3
which is described as the increase in the decrease in number and size of the cell. Metaplasia is BAYTEC, MD (TOP 10 EXAM - AUG
number of cells in an organ. replacement of one differentiated type to another. - FEB 2015 MED 2015
A. Hypertrophy Anaplasia happens when cells lose their morphologic BOARDS; TOPNOTCH
B. Hyperplasia characteristic of a mature cells. MD FROM FEU)
C. Atrophy
D. Metaplasia
E. Anaplasia

162 In Rheumatic Heart Disease, at which Aschoff bodies can be found in all layers of the heart in HAROLD JAY S. MIDTERM 3
layer/layers of the heart can you find Aschoff RF and RHD. SIMILAR TO PREVIOUS BOARD EXAM BAYTEC, MD (TOP 10 EXAM - AUG
bodies? CONCEPT/PRINCIPLE - FEB 2015 MED 2015
A. pericardium BOARDS; TOPNOTCH
B. myocardium MD FROM FEU)
C. endocardium
D. B and C
E. All layers
163 Valvular vegetations characrerized as small, IE are large irregular masses on the valve cusps that HAROLD JAY S. MIDTERM 3
warty vegetations along the lines of closure of can extend onto the chordae. NBTE/marantic BAYTEC, MD (TOP 10 EXAM - AUG
the valve leaflets are usually seen in what endocarditis are small bland vegetations usually - FEB 2015 MED 2015
disease? attached at the line of closure. LSE are small or BOARDS; TOPNOTCH
A. RHD medium sized vegetation on either or both sides of the MD FROM FEU)
B. Infective endocarditis valve leaflets
C. Non bacterial thrombotic endocarditis
D. Libman sacks endocarditis
E. marantic endocarditis
164 In Libman-Sacks Disease, which of the following In Robbins, Mitral and Tricuspid. In Medscape, Mitral HAROLD JAY S. MIDTERM 3
valve is usually affected? and Aortic. BAYTEC, MD (TOP 10 EXAM - AUG
A. Tricuspid - FEB 2015 MED 2015
B. Pulmonary BOARDS; TOPNOTCH
C. Mitral MD FROM FEU)
D. Aortic
E. All valves are equally involve

165 Which of the following caspase is an example of caspases 8, 9, 10 are initiators while caspases 6 and 3 HAROLD JAY S. MIDTERM 3
executioner in apoptosis? are executioners BAYTEC, MD (TOP 10 EXAM - AUG
A. 8 - FEB 2015 MED 2015
B. 9 BOARDS; TOPNOTCH
C. 10 MD FROM FEU)
D. 6
E. 5

166 Wire-loop capillaries are seen in which type of SIMILAR TO PREVIOUS BOARD EXAM HAROLD JAY S. MIDTERM 3
kidney disease? CONCEPT/PRINCIPLE BAYTEC, MD (TOP 10 EXAM - AUG
A. Lupus nephritis - FEB 2015 MED 2015
B. RPGN BOARDS; TOPNOTCH
C. Diabetic nephropathy MD FROM FEU)
D. Polycystic kidney disease
E. Acute tubular necrosis

167 What is the most common and the most severe type IV or the diffuse proliferative glomerulonephritis HAROLD JAY S. MIDTERM 3
type of Lupus nephropathy? is the most common and most severe type. BAYTEC, MD (TOP 10 EXAM - AUG
A. Type I - FEB 2015 MED 2015
B. Type II BOARDS; TOPNOTCH
C. Type III MD FROM FEU)
D. Type IV
E. Type V

168 Minamata disease is an environmental concern Mercury poisoning is the one associated with HAROLD JAY S. MIDTERM 3
because it causes cerebral palsy, deafness, Minamata disease. BAYTEC, MD (TOP 10 EXAM - AUG
blindness, mental retardation and other major - FEB 2015 MED 2015
CNS defects in children in utero. Which of the BOARDS; TOPNOTCH
following elements is the cause of the disease? MD FROM FEU)
A. arsenic
B. lead
C. mercury
D. cadnium
E. chromium
169 All of the following cancers are strongly cancers of the lung, larynx, esophagus, pancreas, HAROLD JAY S. MIDTERM 3
associated with smoking except: bladder, oral cavity are strongly associated with BAYTEC, MD (TOP 10 EXAM - AUG
A. Oral cavity smoking. - FEB 2015 MED 2015
B. pancreas BOARDS; TOPNOTCH
C. esophagus MD FROM FEU)
D. bladder
E. Breast

TOPNOTCH MEDICAL BOARD PREP PATHOLOGY SUPEREXAM Page 23 of 99


For inquiries visit www.topnotchboardprep.com.ph or email us at topnotchmedicalboardprep@gmail.com
TOPNOTCH MEDICAL BOARD PREP PATHOLOGY SUPEREXAM
For inquiries visit www.topnotchboardprep.com.ph or email us at topnotchmedicalboardprep@gmail.com
Item QUESTION EXPLANATION AUTHOR TOPNOTCH
# EXAM
170 Among the types of Hodgkin's lymphoma, this lymphocyte depleted has the highest association with HAROLD JAY S. MIDTERM 3
has the highest association with EBV and also EBV and also has the poorest prognosis.Nodular BAYTEC, MD (TOP 10 EXAM - AUG
has the poorest prognosis. sclerosis is the most common and has excellent - FEB 2015 MED 2015
A. Lymphocyte-depleted prognosis BOARDS; TOPNOTCH
B. Lymphocyte rich MD FROM FEU)
C. Lymphocyte predominant
D. Nodular sclerosis
E. Mixed cellularity

171 Chromosome translocations are important in 9:22 is associated with CML. 14:18 is associated with HAROLD JAY S. MIDTERM 3
diagnosing and prognosticating certain types of Follicular lymphoma. 11:14 is associated with mantle BAYTEC, MD (TOP 10 EXAM - AUG
cancers. Which of the following may be seen in cell lymphoma. 15:17 is AML M3 - FEB 2015 MED 2015
Burkitt's lymphoma? BOARDS; TOPNOTCH
A. t9:22 MD FROM FEU)
B. T8:14
C. T14:18
D. T11:14
E. T15:17
172 Which of the following types of vasculitis is p-ANCA is associated with microscopic polyangitis HAROLD JAY S. MIDTERM 3
highly associated with c-ANCA or the antibodies and Churgstraus while c-ANCA is for Wegener BAYTEC, MD (TOP 10 EXAM - AUG
against proteinase-3? Granulomatosis - FEB 2015 MED 2015
A. PAN BOARDS; TOPNOTCH
B. Takayasu arteritis MD FROM FEU)
C. Wegener Granulomatosis
D. Microscopic polyangitis
E. Churg-Strauss syndrome

173 A patient came in for second opinion about his 30% of patients with PAN are HBsAg positive. HAROLD JAY S. MIDTERM 3
newly diagnosed polyarteritis nodosa. All of the pathology topnotch handout page 106. PAN does not BAYTEC, MD (TOP 10 EXAM - AUG
following statements are true about this disease affect the lungs, affects mainly young adults, usually - FEB 2015 MED 2015
EXCEPT involves medium sized arteries, very responsive to BOARDS; TOPNOTCH
A. Does NOT affect lungs steroid therapy and cyclophosphamide, and 30% of MD FROM FEU)
B. Affects mainly young adults patients are HbsAg positive.
C. Usually involves medium sized arteries
D. NOT associated with hepatitis B
E. very responsive to steroid therapy and
cyclophospahmide
174 Which among the following arteries is HAROLD JAY S. MIDTERM 3
commonly affected in Kawasaki disease? BAYTEC, MD (TOP 10 EXAM - AUG
A. Abdominal Aorta - FEB 2015 MED 2015
B. Thoracic aorta BOARDS; TOPNOTCH
C. Arch of the aorta MD FROM FEU)
D. Pulmonary artery
E. Coronary arteries

175 Which among the following medications is/are Steroids is contraindicated because it promote HAROLD JAY S. MIDTERM 3
essential in treating a patient with Kawasaki rupture of coronary vessels and aggration of the BAYTEC, MD (TOP 10 EXAM - AUG
disease? disease. - FEB 2015 MED 2015
A. ASA BOARDS; TOPNOTCH
B. IVIg MD FROM FEU)
C. Steroids
D. A and B
E. All of the above

176 This is a primary malignant small round-cell HAROLD JAY S. MIDTERM 3


tumor of the bone and soft tissue which usually BAYTEC, MD (TOP 10 EXAM - AUG
involve the mutation of a gene in chromosome - FEB 2015 MED 2015
22. Homer-Wright rosettes can be seen BOARDS; TOPNOTCH
histologically and onion-skin appearance can be MD FROM FEU)
seen radiographically on patients with this
disease.
A. Ewing sarcoma
B. Osteosarcoma
C. Chondrosarcoma
D. Giant Cell Tumor
E. Fibrous Dysplasia
177 Reiter syndrome is an autoimmune disease these three are the triad of Reiter syndrome HAROLD JAY S. MIDTERM 3
which is usually triggered by infections caused BAYTEC, MD (TOP 10 EXAM - AUG
by Shigella, Salmonella, Yersinia, and - FEB 2015 MED 2015
Chlamydia. Which among the following is/are BOARDS; TOPNOTCH
component of the disease? MD FROM FEU)
A. urethritis
B. arthritis
C. uveitis
D. A and B
E. All of the above
178 In infective endocarditis, what is the most HAROLD JAY S. MIDTERM 3
organism involved in patients with native but BAYTEC, MD (TOP 10 EXAM - AUG
previously damaged heart valves? - FEB 2015 MED 2015
A. Strep viridans BOARDS; TOPNOTCH
B. Staph epidermidis MD FROM FEU)
C. Strep bovis
D. Staph aureus
E. Enterococcus

TOPNOTCH MEDICAL BOARD PREP PATHOLOGY SUPEREXAM Page 24 of 99


For inquiries visit www.topnotchboardprep.com.ph or email us at topnotchmedicalboardprep@gmail.com
TOPNOTCH MEDICAL BOARD PREP PATHOLOGY SUPEREXAM
For inquiries visit www.topnotchboardprep.com.ph or email us at topnotchmedicalboardprep@gmail.com
Item QUESTION EXPLANATION AUTHOR TOPNOTCH
# EXAM
179 Which of the following conditions will cause decrease in plasma proteins will decrease oncotic HAROLD JAY S. MIDTERM 3
edema? pressure within the vessel which can cause edema BAYTEC, MD (TOP 10 EXAM - AUG
A. Increase oncotic pressure in the vessels - FEB 2015 MED 2015
B. Decrease plasma volume in blood vessels BOARDS; TOPNOTCH
C. Decrease oncotic pressure outside the MD FROM FEU)
vessels
D. Decrease in plasma proteins
E. None of the above
180 Which of the following diseases is an X-linked Alport syndrome and vitamin D resistant rickets are HAROLD JAY S. MIDTERM 3
dominant disorder? the only most commonly associated with X linked BAYTEC, MD (TOP 10 EXAM - AUG
A. G6PD deficiency dominant genetic transmission. - FEB 2015 MED 2015
B. Hemophilia BOARDS; TOPNOTCH
C. Ehlers Danlos syndrome MD FROM FEU)
D. Marfan syndrome
E. Alport syndrome

181 What is the most common tumor in the SIMILAR TO PREVIOUS BOARD EXAM JEAN PAOLO M. FINAL EXAM -
stomach? CONCEPT/PRINCIPLE. DELFINO, MD (TOP AUG 2015
A. adenocarcinoma 10 - FEB 2015 MED
B. GIST BOARDS; TOPNOTCH
C. carcinoid MD FROM FATIMA)
D. adenoma
E. lymphoma

182 True of Luetic aneurysm? All are true regarding Syphilitic/Luetic aneurysm JEAN PAOLO M. FINAL EXAM -
A. Inflammation begins in the tunica DELFINO, MD (TOP AUG 2015
adventitia 10 - FEB 2015 MED
B. With characteristic tree-barking BOARDS; TOPNOTCH
appearance MD FROM FATIMA)
C. Involvement of aorta favors development of
superimosed atheromatosisof the aortic root
D. All of the above
E. A and C
183 Microscopically, the earliest change of systemic JEAN PAOLO M. FINAL EXAM -
Hypertensive Heart Disease is? DELFINO, MD (TOP AUG 2015
A. Increase in transverse diameter of 10 - FEB 2015 MED
myocytes BOARDS; TOPNOTCH
B. Irregular cellular enlargement MD FROM FATIMA)
C. Irregular nuclear enlargement
D. Interstitial fibrosis
E. All of the above
184 In acute Rheumatic Fever, inflammation and SIMILAR TO PREVIOUS BOARD EXAM JEAN PAOLO M. FINAL EXAM -
Aschoff bodies are commonly found in which CONCEPT/PRINCIPLE.. During acute RF, diffuse DELFINO, MD (TOP AUG 2015
layer of the heart? inflammation and Aschoff bodies may be found in any 10 - FEB 2015 MED
A. pericardium of the 3 layers of the heart- hence the lesion is called a BOARDS; TOPNOTCH
B. myocardium PANCARDITIS. MD FROM FATIMA)
C. endocardium
D. All of the above
E. A and B

185 What is the valve most commonly affected in SIMILAR TO PREVIOUS BOARD EXAM JEAN PAOLO M. FINAL EXAM -
Libman-Sacks endocarditis? CONCEPT/PRINCIPLE.. In SLE, mitral and tricuspid DELFINO, MD (TOP AUG 2015
A. Mitral valvulitis with small, sterile vegetations, called 10 - FEB 2015 MED
B. Tricuspid Libman-Sacks endocarditis is ocassionally BOARDS; TOPNOTCH
C. Aortic encountered. according to harrison, MD FROM FATIMA)
D. Pulmonic "The characteristic endocardial lesions of SLE are
E. A and B verrucous valvular abnormalities known as Libman-
Sacks endocarditis . They most often are located on
the left-sided cardiac valves, particularly on the
ventricular surface of the posterior mitral leaflet, and
are made up almost entirely of fibrin. " So yes, the
correct answer is mitral valve.
186 What is the most severe form of α-thalassemia Hydrops fetalis is the most severe form of α- JEAN PAOLO M. FINAL EXAM -
which is caused by deletion of all four α-globin thalassemia. it is caused by deletion of all four α- DELFINO, MD (TOP AUG 2015
genes? globin genes. In the fetus, excess γ-globin chains form 10 - FEB 2015 MED
A. αThalassemia trait tetramers (hemoglobin Barts) that have such a high BOARDS; TOPNOTCH
B. Silent carrier state affinity for oxygen that they deliver little to tissues. MD FROM FATIMA)
C. Hydrops fetalis
D. Hemoglobin H disease
E. A and D

187 A 58 year old male presented with easy CLL and SLL differ only in the degree of peripheral JEAN PAOLO M. FINAL EXAM -
fatigability, anorexia and weight loss. On PE, blood lymphocytosis. Most affected patients have DELFINO, MD (TOP AUG 2015
there were noted lymphadenopathies and sufficient lymphocytosis to fulfill the diagnostic 10 - FEB 2015 MED
hepatosplenomegaly. CBC revealed requirement for CLL (absolute lymphocyte count BOARDS; TOPNOTCH
leukocytosis, with absolute lymphocyte count >4000 per mm3). In this condition, lymph nodes are MD FROM FATIMA)
3000 per mm3. There were numerous small, diffusely effaced by an infiltrate of predominantly
round lymphocytes with scant cytoplasm which small lymphocytes 6 to 12 μm in diameter with round
are frequently disrupted in the process of to slightly irregular nuclei, condensed chromatin, and
making smears. What is the diagnosis? scant cytoplasm. Some of these cells are usually
A. ALL disrupted in the process of making smears, producing
B. SLL so-called smudge cells.
C. CLL
D. CML
E. Multiple Myeloma

TOPNOTCH MEDICAL BOARD PREP PATHOLOGY SUPEREXAM Page 25 of 99


For inquiries visit www.topnotchboardprep.com.ph or email us at topnotchmedicalboardprep@gmail.com
TOPNOTCH MEDICAL BOARD PREP PATHOLOGY SUPEREXAM
For inquiries visit www.topnotchboardprep.com.ph or email us at topnotchmedicalboardprep@gmail.com
Item QUESTION EXPLANATION AUTHOR TOPNOTCH
# EXAM
188 What is the most common lesion of the salivary Mucocele is the most common lesion of the salivary JEAN PAOLO M. FINAL EXAM -
gland? glands and it results from either blockage or rupture DELFINO, MD (TOP AUG 2015
A. Sialolithiasis of a salivary gland duct, with consequent leakage of 10 - FEB 2015 MED
B. Pelomorphic adenoma saliva into the surrounding connective tissue stroma. BOARDS; TOPNOTCH
C. Warthin tumor MD FROM FATIMA)
D. Mucocele
E. Sialadenitis

189 Acute hepatitis is characterized by what With acute hepatitis, hepatocyte injury takes the form JEAN PAOLO M. FINAL EXAM -
morphologic feature? of diffuse swelling (“ballooning degeneration”;), so the DELFINO, MD (TOP AUG 2015
A. Swollen hepatocytes with irregulaly cytoplasm looks empty and contains only scattered 10 - FEB 2015 MED
clumped cytoplasmic organelles and large clear eosinophilic remnants of cytoplasmic organelles. BOARDS; TOPNOTCH
spaces Feathery degeneration is retention of biliary material MD FROM FATIMA)
B. Presence of feathery degeneration causing foamy appearance of hepatocytes seen in
C. Deposition of fibrous tissue in the portal cholestatic liver injury. C and D are charcteristics of
tracts and periportal septa chronic hepatitis.
D. Bridging inflammation and necrosis
E. All of the above
190 True statement about Crigler-Najjar Syndrome Crigler-Najjar Syndrome type I is an autosomal JEAN PAOLO M. FINAL EXAM -
type I recessive condition wherein there is ABSENT UGT1A1 DELFINO, MD (TOP AUG 2015
A. Liver morphology is normal activity causing indirect hyperbilirubinemia. Liver 10 - FEB 2015 MED
B. There is decreased UGT1A1 enzyme pathology is normal and it is fatal in the neonatal BOARDS; TOPNOTCH
activity period. MRP2 mutation is seen in Dubin-Johnson MD FROM FATIMA)
C. It is generally mild with occasional Syndrome
kernicterus
D. There is mutation in MRP2
E. Hyperbilirubinemia is of direct type
191 Hepatitis B carrier state is most commonly In endemic regions such as Africa and Southeast Asia, JEAN PAOLO M. FINAL EXAM -
acquired via what mode of transmission? spread of Hepatitis B from an infected mother to a DELFINO, MD (TOP AUG 2015
A. Heterosexual transmission neonate during birth (vertical transmission) is 10 - FEB 2015 MED
B. Needle-stick injuries common. These neonatal infections often lead to a BOARDS; TOPNOTCH
C. Vertical transmission carrier state for life. MD FROM FATIMA)
D. Blood transfusion
E. Homosexual transmission

192 Variant of Renal Cell Carcinoma which is made SIMILAR TO PREVIOUS BOARD EXAM JEAN PAOLO M. FINAL EXAM -
up of pale eosinophilic cells, often with a CONCEPT/PRINCIPLE.. Chromophobe renal carcinoma DELFINO, MD (TOP AUG 2015
perinuclear halo, arranged in solid sheets with is made up of pale eosinophilic cells, often with a 10 - FEB 2015 MED
a concentration of the largest cells around perinuclear halo, arranged in solid sheets with a BOARDS; TOPNOTCH
blood vessels? concentration of the largest cells around blood MD FROM FATIMA)
A. Clear cell CA vessels.
B. Papillary CA
C. Chromophobe CA
D. Collecting Duct CA
E. Urothelial CA
193 This agent predisposes susceptible populations The uses of cadmium include yellow pigments and JEAN PAOLO M. FINAL EXAM -
to develop prostate cancer? phosphors; found in solders; used in batteries and as DELFINO, MD (TOP AUG 2015
A. Nickel alloy and in metal platings and coatings. It is 10 - FEB 2015 MED
B. Chromium associated with prostate cancer. Nickel is associated BOARDS; TOPNOTCH
C. Cadmium with nose, lung cancer; Benzene- Leukemia, Hodgkin MD FROM FATIMA)
D. Vinyl chloride lymphoma; Chromium- lungs; Vinyl chloride-
E. Benzene Angiosarcoma, liver.

194 Metastasis unequivocally marks a tumor as All cancers metastasize except glioma and basal cell JEAN PAOLO M. FINAL EXAM -
malignant. What cancer does not metastasize? carcinoma. Lipoma does not metastasize. But the DELFINO, MD (TOP AUG 2015
A. hepatoma question asks what "cancer" does not metastasize. 10 - FEB 2015 MED
B. seminoma Lipoma is benign so it should be ruled out from the BOARDS; TOPNOTCH
C. lipoma choices. Glioma is the correct answer MD FROM FATIMA)
D. glioma
E. meningioma

195 True of chronic bronchitis except Although the numbers of goblet cells increase slightly, JEAN PAOLO M. FINAL EXAM -
A. Grossly, there may be hyperemia, swelling the major change is in the size of the mucous gland DELFINO, MD (TOP AUG 2015
and edema of the mucus membranes (hyperplasia). This increase can be assessed by the 10 - FEB 2015 MED
B. The ratio of the number of mucus glands to ratio of the thickness of the mucous gland layer to the BOARDS; TOPNOTCH
the thickness of the wall is increased thickness of the wall between the epithelium and the MD FROM FATIMA)
C. There is excessive mucinous to cartilage (Reid index).
mucopurulent secretions layering the epithelial
surfaces
D. The major change is in the size of the
mucus glands
E. All are true
196 20 year old male presented with history of Diagnosis for this case is Churg-Strauss Syndrome. It JEAN PAOLO M. FINAL EXAM -
allergic rhinitis, asthma and recurrent sinusitis. is a small-vessel necrotizing vasculitis classically DELFINO, MD (TOP AUG 2015
An autoimmune etiology is suspected. What is associated with asthma, allergic rhinitis, lung 10 - FEB 2015 MED
the most likely autoantibody involved in this infiltrates, peripheral hypereosinophilia, and BOARDS; TOPNOTCH
condition? extravascular necrotizing granulomas. The MD FROM FATIMA)
A. Anti-myeloperoxidase Ab autoantibody implicated in this condition is the anti-
B. Anti-proteinase 3 Ab myeloperoxidase antibody and p-ANCA. B-Wegener's;
C. Anti-endomysial Ab C- Celiac Disease; D- primary biliary cirrhosis; E-
D. Anti-mitochondrial Ab Crohn's disease.
E. Anti-saccharomyces Ab

TOPNOTCH MEDICAL BOARD PREP PATHOLOGY SUPEREXAM Page 26 of 99


For inquiries visit www.topnotchboardprep.com.ph or email us at topnotchmedicalboardprep@gmail.com
TOPNOTCH MEDICAL BOARD PREP PATHOLOGY SUPEREXAM
For inquiries visit www.topnotchboardprep.com.ph or email us at topnotchmedicalboardprep@gmail.com
Item QUESTION EXPLANATION AUTHOR TOPNOTCH
# EXAM
197 What is the most common site of ectopic Aberrantly situated, or ectopic, pancreatic tissue is JEAN PAOLO M. FINAL EXAM -
pancreas? found in about 2% of careful routine postmortem DELFINO, MD (TOP AUG 2015
A. duodenum examinations. The favored sites for ectopia are the 10 - FEB 2015 MED
B. jejunum stomach and duodenum, followed by the jejunum, BOARDS; TOPNOTCH
C. ileum Meckel diverticula, and ileum. MD FROM FATIMA)
D. Meckel's diverticulum
E. spleen

198 8 year old male patient abruptly develops Diagnosis is PSGN. The electron microscopy finding in JEAN PAOLO M. FINAL EXAM -
malaise, fever, nausea, oliguria, and hematuria this condition is subepithelial humps. A- minimal DELFINO, MD (TOP AUG 2015
2 weeks after recovery from a sore throat. The change disease; B- MPGN; C- IgA nephropathy; D- 10 - FEB 2015 MED
patient has red cell casts in the urine, mild Membranous GN BOARDS; TOPNOTCH
proteinuria, periorbital edema, and MD FROM FATIMA)
hypertension. What is the expected electron
microscopy finding in this case?
A. Loss of foot processes
B. Subendothelial deposits
C. Mesangial and paramesangial dense
deposits
D. Subepithelial deposits
E. Subepithelial humps
199 In the heart, grossly apparent bands of The terms steatosis and fatty change describe JEAN PAOLO M. FINAL EXAM -
yellowed myocardium alternating with bands abnormal accumulations of triglycerides within DELFINO, MD (TOP AUG 2015
of darker, red-brown, uninvolved myocardium parenchymal cells. This is most often seen in the liver 10 - FEB 2015 MED
is known as tigered effect. This is due to and heart. In the heart, fat deposits create grossly BOARDS; TOPNOTCH
accumulation of what material in the cardiac apparent bands of yellowed myocardium alternating MD FROM FATIMA)
cells? with bands of darker, red-brown, uninvolved
A. calcium myocardium (tigered effect)
B. glycogen
C. triglyceride
D. cholesterol
E. lipofuscin
200 What is the immunologically mediated Acute Rheumatic Fever is a Type 2 hypersensitivity JEAN PAOLO M. FINAL EXAM -
pathologic lesion seen in Acute Rheumatic reaction. It is an antibody mediated reaction causing DELFINO, MD (TOP AUG 2015
Fever? phagocytosis and lysis of cells; inflammation; in some 10 - FEB 2015 MED
A. Vascular dilation, edema, smooth muscle diseases, functional derangements without cell or BOARDS; TOPNOTCH
contraction tissue injury. A- Type 1; B- type 3; D- type 4 MD FROM FATIMA)
B. Necrotizing vasculitis
C. Phagocytosis and cell lysis
D. Perivascular cellular infiltrates
E. B and C
201 The gene involved in Williams syndrome is: Williams syndrome: supravalvular aortic stenosis, GRACE ARVIOLA, MD DIAGNOSTIC
A. Fibrillin hypercalcemia, cognitive abnormalities, and hallmark (TOP 3 - AUG 2014 EXAM - FEB
B. Collagen facial anomalies. MED BOARDS; 2015
C. Elastin TOPNOTCH MD)
D. Reticulin
E. Chondroitin

202 Anti-centromere antibodies are present in: CREST syndrome: calcinosis, Raynaud's phenomenon, GRACE ARVIOLA, MD DIAGNOSTIC
A. Sjogren syndrome esophageal dysmotility, sclerodactyly, and (TOP 3 - AUG 2014 EXAM - FEB
B. SLE telangiectasia. MED BOARDS; 2015
C. Wegener's granulomatosis TOPNOTCH MD)
D. CREST syndrome
E. Churg-Strauss syndrome

203 The presence of Heinz bodies and bite cells in a Heinz bodies: RBCs with denatured hemoglobin. Bite GRACE ARVIOLA, MD DIAGNOSTIC
patient having hemolytic anemia strongly cells results when splenic macrophages pluck out (TOP 3 - AUG 2014 EXAM - FEB
suggests: these inclusions. MED BOARDS; 2015
A. Sickle cell anemia TOPNOTCH MD)
B. G6PD deficiency
C. Alpha thalassemia
D. Multiple myeloma
E. Hereditary spherocytosis

204 Nocturnal pain relieved by aspirin intake is The pain is probably caused by excessive GRACE ARVIOLA, MD DIAGNOSTIC
characteristic of: prostaglandin E2 (PGE2) production by the (TOP 3 - AUG 2014 EXAM - FEB
A. Osteomyelitis proliferating osteoblasts. MED BOARDS; 2015
B. Osteoid osteoma TOPNOTCH MD)
C. Chondroma
D. Osteosarcoma
E. Ossifying fibroma

205 What is the most important factor in the Location in the jejunum or ileum carries the worst GRACE ARVIOLA, MD DIAGNOSTIC
prognosis of GI carcinoid? prognosis. (TOP 3 - AUG 2014 EXAM - FEB
A. Size MED BOARDS; 2015
B. Nuclear grade TOPNOTCH MD)
C. Location
D. Metastasis
E. Presence of other tumors

TOPNOTCH MEDICAL BOARD PREP PATHOLOGY SUPEREXAM Page 27 of 99


For inquiries visit www.topnotchboardprep.com.ph or email us at topnotchmedicalboardprep@gmail.com
TOPNOTCH MEDICAL BOARD PREP PATHOLOGY SUPEREXAM
For inquiries visit www.topnotchboardprep.com.ph or email us at topnotchmedicalboardprep@gmail.com
Item QUESTION EXPLANATION AUTHOR TOPNOTCH
# EXAM
206 Which condition is most likely to give rise to In hereditary tyrosinemia, almost 40% of patients GRACE ARVIOLA, MD DIAGNOSTIC
hepatocellular carcinoma? develop the tumor despite adequate dietary control. (TOP 3 - AUG 2014 EXAM - FEB
A. Hereditary tyrosinemia However, this condition is extremely rare. MED BOARDS; 2015
B. Chronic Hepatitis B TOPNOTCH MD)
C. Chronic Hepatitis C
D. Chronic Alcoholism
E. Autoimmune hepatitis

207 Mucocutaneous lymph node syndrome AKA Kawasaki disease because it presents with GRACE ARVIOLA, MD DIAGNOSTIC
preferentially affects the: conjunctival and oral erythema and erosion, edema of (TOP 3 - AUG 2014 EXAM - FEB
A. Temporal artery the hands and feet, erythema of the palms and soles, a MED BOARDS; 2015
B. Aortic arch desquamative rash, and cervical lymph node TOPNOTCH MD)
C. Renal arteries involvement. Approximately 20% of untreated
D. Coronary arteries patients develop cardiovascular sequela involving the
E. Tibial and radial arteries coronary arteries.

208 A person with hypothyroidism and Pendred syndrome is caused by a mutation in the GRACE ARVIOLA, MD DIAGNOSTIC
sensorineural deafness most likely has an SLC26A4 gene whose product, pendrin, is an anion (TOP 3 - AUG 2014 EXAM - FEB
abnormality of the: transporter expressed on the apical surface of MED BOARDS; 2015
A. H-P-O axis thyrocytes and in the inner ear. TOPNOTCH MD)
B. Thyroglobulin
C. Anion transporter
D. Iodide channel
E. Peripheral deiodinases

209 What is the most common clinical Primary hyperparathyroidism is the most common GRACE ARVIOLA, MD DIAGNOSTIC
manifestation among patients with Sipple manifestation in MEN 1 (Wermer syndrome). (TOP 3 - AUG 2014 EXAM - FEB
Syndrome? MED BOARDS; 2015
A. Pheochromocytoma TOPNOTCH MD)
B. Medullary carcinoma of the thyroid
C. Parathyroid adenoma
D. Neuroganglioma
E. Pituitary adenoma
210 Coffin lid appearance is seen in calculi Struvite stones are composed of magnesium GRACE ARVIOLA, MD DIAGNOSTIC
composed of: ammonium phosphate. (TOP 3 - AUG 2014 EXAM - FEB
A. Calcium oxalate MED BOARDS; 2015
B. Cystine TOPNOTCH MD)
C. Magnesium ammonium phosphate
D. Uric acid
E. Calcium carbonate

211 Which are considered vascular phenomena in Osler nodes and Roth spots are immunologic GRACE ARVIOLA, MD DIAGNOSTIC
infective endocarditis? phenomena. Osler nodes are subcutaneous nodules in (TOP 3 - AUG 2014 EXAM - FEB
A. Janeway lesions and Osler nodes the pulp of the digits. Roth spots are retinal MED BOARDS; 2015
B. Splinter hemorrhages and Roth spots hemorrhages in the eyes. Splinter hemorrhages are TOPNOTCH MD)
C. Janeway lesions and Roth spots micro-thromoboemboli. Janeway lesions are
D. Splinter hemorrhages and Janeway lesions erythematous or hemorrhagic nontender lesions on
E. Osler nodes and Roth spots the palms or soles.

212 Which heavy metal plays a role in the treatment AML with the t(15,17) (promyelocytic leukemia) is GRACE ARVIOLA, MD DIAGNOSTIC
of relapsing acute promyelocytic leukemia? treated with pharmacologic doses of ATRA (all-trans (TOP 3 - AUG 2014 EXAM - FEB
A. Arsenic retinoic acid) combined with conventional MED BOARDS; 2015
B. Lead chemotherapy, or more recently, with arsenic salts, TOPNOTCH MD)
C. Copper which appear to cause PML-RARa to be degraded.
D. Mercury
E. Zinc

213 Which characteristic of a tumor will make it Poorly vascularized, poorly oxygenated, and a central GRACE ARVIOLA, MD DIAGNOSTIC
less radiosensitive? location makes a tumor less radiosensitive. (TOP 3 - AUG 2014 EXAM - FEB
A. Peripheral location MED BOARDS; 2015
B. Highly oxygenated TOPNOTCH MD)
C. Poorly vascularized
D. A and B
E. B and C

214 Subacute combined degeneration of the spinal The combined degenration of both ascending and GRACE ARVIOLA, MD DIAGNOSTIC
cord is seen in deficiency of which vitamin? descending tracts of the spinal cord is characteristic of (TOP 3 - AUG 2014 EXAM - FEB
A. B1 vitamin B12 deficiency. MED BOARDS; 2015
B. B2 TOPNOTCH MD)
C. B3
D. B5
E. B12

215 A 38 year old male patient presents with a Anti-Saccharomyces antibodies are present in Crohn's GRACE ARVIOLA, MD DIAGNOSTIC
longstanding history of abdominal pain and disease. (TOP 3 - AUG 2014 EXAM - FEB
intermittent diarrhea. Imaging studies showed MED BOARDS; 2015
cobblestone appearance of the colon with TOPNOTCH MD)
ulcers that were sporadically located. Biopsy
revealed non-caseating granulomas. What
antibodies might this patient potentially have?
A. p-ANCA
B. c-ANCA
C. Anti-Saccharomyces antibodies
D. Antibodies to gliadin
E. Anti-mitochondrial antibodies

TOPNOTCH MEDICAL BOARD PREP PATHOLOGY SUPEREXAM Page 28 of 99


For inquiries visit www.topnotchboardprep.com.ph or email us at topnotchmedicalboardprep@gmail.com
TOPNOTCH MEDICAL BOARD PREP PATHOLOGY SUPEREXAM
For inquiries visit www.topnotchboardprep.com.ph or email us at topnotchmedicalboardprep@gmail.com
Item QUESTION EXPLANATION AUTHOR TOPNOTCH
# EXAM
216 What fusion gene is carried in the Philadelphia CML is distinguished from other myeloproliferative GRACE ARVIOLA, MD DIAGNOSTIC
chromosome of CML? disorders by the presence of a chimeric BCR-ABL gene (TOP 3 - AUG 2014 EXAM - FEB
A. C-myc and N-myc derived from portions of the BCR gene on MED BOARDS; 2015
B. BCR-ABL chromosome 22 and the ABL gene on chromosome 9. TOPNOTCH MD)
C. BRCA1 and BRCA2
D. NOD2
E. JAK/STAT

217 A 42 year old female presents with a 5 year The histologic hallmark is the presence of dysohesive GRACE ARVIOLA, MD DIAGNOSTIC
history of palpable breast mass. Biopsy showed infiltrating tumor cells, often arranged in a single file (TOP 3 - AUG 2014 EXAM - FEB
signet ring cells arranged in an Indian file pattern or in loose clusters or sheets. Signet ring MED BOARDS; 2015
pattern. The mass is most likely: cellscontaining an intracytoplasmic mucin droplet are TOPNOTCH MD)
A. Invasive ductal carcinoma common.
B. Invasive lobular carcinoma
C. Paget's disease of the breast
D. Carcinoma in situ
E. Medullary carcinoma of the breast
218 The chest x-ray of a cyanotic infant revealed an This is transposition of the great arteries. Choice A is GRACE ARVIOLA, MD DIAGNOSTIC
egg-shaped heart. Which statement is correct? TAPVC, choice B is TOF, choice D is endocardial (TOP 3 - AUG 2014 EXAM - FEB
A. The pulmonary veins empty into the right cushion defect, and choice E is coarctation of the aorta. MED BOARDS; 2015
atrium. TOPNOTCH MD)
B. The underlying pathology is obstruction of
the right ventricular outflow tract.
C. This condition is common among infants of
diabetic mothers.
D. The infant probably has Down's syndrome.
E. The aorta is constricted at a site just distal
to the ligamentum arteriosum.
219 A patient with retroperitoneal fibrosis is also at Retroperitoneal fibrosis is associated with Reidel GRACE ARVIOLA, MD DIAGNOSTIC
risk of having: thyroiditis, PSC, and right-sided varicocele. (TOP 3 - AUG 2014 EXAM - FEB
A. Left-sided varicocele MED BOARDS; 2015
B. Hashimoto's thyroiditis TOPNOTCH MD)
C. Crohn's disease
D. Primary sclerosing cholangitis
E. Whipple's disease

220 This subtype of Hodgkin's lymphoma is In LP, the Reed-Sternberg cells have a distinctive B- GRACE ARVIOLA, MD DIAGNOSTIC
considered non-classical. cell immunophenotype that differs from that of the (TOP 3 - AUG 2014 EXAM - FEB
A. Nodular sclerosis classical types. MED BOARDS; 2015
B. Lymphocyte-predominant TOPNOTCH MD)
C. Lymphocyte-rich
D. Lymphocyte-depleted
E. Mixed cellularity

221 Which of the following is true regarding dense dense deposit disease (MPGN type II) - is a primary LEAN ANGELO MIDTERM
deposit disease? type MPGN associated with activation of the SILVERIO, MD (TOP 4 EXAM 1 - FEB
A. Characterized by immune complex deposits alternative pathway. This is based on the diminished - AUG 2014 MED 2015
in the glomerular basement membrance and serum levels of factor B and properdin along with BOARDS; TOPNOTCH
activiation of classical pathway normal C1 an C4 levels. Ultrastructurally, type II MD), MD
B. Highly responsive to treatment with MPGN is characterized by deposition of dense material
immunosuppresive drugs such as pulse along the GBM proper. Natural history of patients with
cyclophosphamide combined with steroids this disease showed refractory to combined pulse
C. Diminished serum levels of factor B and cyclophosphamide and steroids. there is also high
properdin incidence of recurrence among transplant patient
D. Dense materials are primarily deposited on compared to type I MPGN . Robbins 8th ed pp 928-
the podocyte-GBM junction 929
E. Less recurrence among allograft recipients
compared to Type I MPGN
222 54 y/o male patient went for an OPD consult A 54 y/o patient complaining of nocturia and LEAN ANGELO MIDTERM
secondary to a chief complaint of urinary frequency along with constellation of clinical findings SILVERIO, MD (TOP 4 EXAM 1 - FEB
frequency and nocturia. PMHx: CABG-2 years such as on diabetic and on insulin therapy, - AUG 2014 MED 2015
ago. FM: (+) DM (+) Htn both parents. he is on hypertensive, (+) Family history, obese, bipedal BOARDS; TOPNOTCH
insulin therapy since he was 45 y/o; other edema without any overt clinical evidence of infection MD), MD
medications include the ff: losartan, metoprolol, is most likely suffering from DM nephropathy. renal
ACEI. pertinent physical examination showed morphologic changes include the following.
the following data: BP 140/80, PR 98 bpm, T Widespread thickening of the capillary basement
37.1C; BMI: 34 kg/m2, soft nontender membrane. diffuse increase of mesangial matrix
abdomen, no suprapubic tenderness, no CVA secondary to GBM thickening with minimal mesangial
tenderness, (+) bipedal edema. which of the cell proliferation. Presence of PAS positive nodules (
following is a correct morphologic Kimmelsteil Wilson nodules) along the periphery of
characteristics behind his most plausible cause the glomerulus. diffuse hyaline arteriolosclerosis
of urinary complaint? affecting both afferent and efferent arterioles.
A. presence of focal thickening of the Robbins 8th ed pp 1140-1141
glomerular capillary basement membrane
B. presence of mesangial proliferation
secondary to hyperplasia of the mesangial cells
C. PAS negative nodular lesions noted on the
periphery of the glomerulus.
D. presence of hyaline arteriolosclerosis
affecting both afferent and efferent arterioles
E. All of the above

TOPNOTCH MEDICAL BOARD PREP PATHOLOGY SUPEREXAM Page 29 of 99


For inquiries visit www.topnotchboardprep.com.ph or email us at topnotchmedicalboardprep@gmail.com
TOPNOTCH MEDICAL BOARD PREP PATHOLOGY SUPEREXAM
For inquiries visit www.topnotchboardprep.com.ph or email us at topnotchmedicalboardprep@gmail.com
Item QUESTION EXPLANATION AUTHOR TOPNOTCH
# EXAM
223 A 45 y/o male patient brought to ER due to the rise of the serum creatinine and presence of LEAN ANGELO MIDTERM
massive hematochezia, he is a known case of oliguria after an hypotensive episode is suggestive of SILVERIO, MD (TOP 4 EXAM 1 - FEB
liver cirrhosis secondary to Chronic Hep B ischemic type AKI. Morphologic changes include - AUG 2014 MED 2015
infection. Clinical findings are of the following: patchy necrosis primarily along the the straight BOARDS; TOPNOTCH
lethargic, BP 60 mmHg palpatory, PR 145 bpm, portion of the PCT and ascending limb of henle, MD), MD
pale palpebral conjunctiva, icteric sclera, cold eosinophilic cast composed of tamm horsfall protein
clammy extremities. he was successfully are noted along the ascending limb and the distal
stabilized after undergoing endoscopic band tubules. In contrast, toxin mediated AKI is manifested
ligation along with blood transfusion and by diffuse damage along the whole length of the PCT.
intravenous hydration. during his hospital stay, based on lab findings and having oliguria, patient is
there was a noted increase in the serum still on maintenance phase. It is the recovery phase
creatinine from 1.1 -2.1 mg/dl accompanied by that is associated with increase urinary volume,
oliguria. which of the following is true hypokalemia and susceptibility to infection. Robbins
regarding the cause of the oliguria? 8th ed pp 937-938
A. the most affected nephron segment is
proximal segment of the proximal convoluted
tubule
B. he is at the stage wherein there is increased
risk for generalized infection and electrolyte
abnormalities like hypokalemia
C. cell swelling, vacuolization and
tubulorrhexis primarily along the descending
limb of henle
D. Eosinophilic hyaline cast noted along the
ascending limb of henle and Distal tubules
E. all of the above
224 A 66 y.o male went for consult secondary to CLL is the most common leukemia of adults in the LEAN ANGELO MIDTERM
recent onset of weight loss accompanied by western countries. It is distinguished to SLL only by SILVERIO, MD (TOP 4 EXAM 1 - FEB
anorexia and easy fatigability. PE findings absolute lymphocytic count of >4000/mm3. - AUG 2014 MED 2015
showed normotensive, tachycardic, pale Morphologic characteristics include diffuse BOARDS; TOPNOTCH
palpebral conjunctiva, enlarged lymph nodes effacement of the LN along with loose aggregrates of MD), MD
on bilateral cervical, axillary, inguinal regions. atypical larger lymphocytes which collectively called
Traube space is obliterated and liver edge is proliferation centers. unlike most leukemias, it is rare
5cm from right subcostal margin. CBC showed for CLL to undergo chromosomal translocation.
normocytic anemia, thrombocytopenia and furthermore, CLL also has an unknown mechanism
lymphocytosis. A diagnosis of chronic that disrupts the normal immune function resulting to
lymphocytic leukemia is entertain, which of the a decrease in antibody production. CLL is also prone
following is consistent regarding the diagnosis? to undergo Richter transformation wherein there is
A. there is diffuse effacement of lymph nodes transformation of the primary cancer to DLBCL.
by predominantly small lymphocytes along
with loose aggregrates of larger activated
lymphocytes.
B. disruption of normal immune function
accompanied with hypergammaglobulinemia
C. prolymphocytic transformation to diffuse B
cell lymphoma is rare
D. There is a high incidence of chromosomal
translocation
E. all of the above.
225 Which of the following factors is associated ALL has one of the greatest prognosis since it is highly LEAN ANGELO MIDTERM
with worse prognosis in Acute Lymphoblastic responsive to chemotherapy, however there are some SILVERIO, MD (TOP 4 EXAM 1 - FEB
Leukemia? parameters associated with worse prognosis. These - AUG 2014 MED 2015
A. Presence of philadelphia chromosome are the following: age under 2 at presentation, BOARDS; TOPNOTCH
B. Peripheral blood blast count of 80,000 presentation at adulthood or adolescence, blast count MD), MD
C. Presentation of symptoms at 5 years of age of more than 100,000; presence of phidelphia
D. presence of chromosomal translocation t chromosome. On the otherhand, favorable prognostic
(12,21) factors include the ff: an age 2-10 years old, a low
E. hyperploidy white cell count, hyperploidy, trisomy of chromosome
4,7,and 10, presence of t(12,21) Robbins 8th ed pp
603
226 A 72 y/o male went for consult secondary to 3 expect clinical scenarios in your board exam. An LEAN ANGELO MIDTERM
days of fever accompanied by productive elderly patient presenting with recurrent pneumonia, SILVERIO, MD (TOP 4 EXAM 1 - FEB
cough. History revealed that he had multiple low back pain and polyuria supported with lab - AUG 2014 MED 2015
episodes of pneumonia within the last 3 years evidence of pancytopenia and radiographic finding of BOARDS; TOPNOTCH
.aside from the primary complaint, he also had compression fractures of lesions on vertebral column MD), MD
polyuria, continuous low back pain even at rest is consistent with multiple myeloma. features of MM
and constipation. PE are normal except for the stems from the effect of plasmacytic growth on axial
ff: pale conjunctiva, bibasilar crackles, skeleton, production of excessive immunoglobulins
hypotonic bowel sounds, paravertebral and alteration of humoral immunity. Because of the
tenderness, smooth, non nodular slightly factors produced by plasma cells, it causes the
enlarged prostate, weak bilateral LE strength activation of osteoclast leading to bone resorption and
2/5. Radiographic examination showed attendant hypercalcemia. Definitive diagnosis is made
pneumonia of bilateral lower lobes, with only by doing bone marrow examination. Patient
incidental finding of thoracic compression usually suffers renal insufficiency secondary to bence
fractures of t5-t6 t8-t9 level and multiple jones proteinuria. Infection is the most common cause
lucencies along the thoracic vertebral bodies. of death among this patient secondary to abnormal
CBC showed normocytic normochromic immunoglobulin production and decreased number of
anemia, leukopenia, thrombocytopenia. UA white blood cells. MGUS and not MM is the most
revealed massive protenuria. which of the common plasma cell dyscrasia
following is consistent with the most possible
primary diagnosis
A. patient is suffering from hypocalcemia and
hypoglobulinemia
B. it is the most common plasma cell dyscrasia
C. definitive diagnosis can be established only
through clinical and radiographic findings

TOPNOTCH MEDICAL BOARD PREP PATHOLOGY SUPEREXAM Page 30 of 99


For inquiries visit www.topnotchboardprep.com.ph or email us at topnotchmedicalboardprep@gmail.com
TOPNOTCH MEDICAL BOARD PREP PATHOLOGY SUPEREXAM
For inquiries visit www.topnotchboardprep.com.ph or email us at topnotchmedicalboardprep@gmail.com
Item QUESTION EXPLANATION AUTHOR TOPNOTCH
# EXAM
D. Infection is the most common cause of
death
E. all of the above

227 Which of the following is not an immune Multiple sclerosis is an example of type IV LEAN ANGELO MIDTERM
complex mediated disease? hypersensitivity reaction. Other type III or immune SILVERIO, MD (TOP 4 EXAM 1 - FEB
A. Systemic Lupus Erythematosus complex mediated reaction aside from the choices are - AUG 2014 MED 2015
B. Multiple sclerosis serum sickness, arthus reaction and reactive bacterial BOARDS; TOPNOTCH
C. Polyarteritis nodosa arthritis. MD), MD
D. Poststreptococcal glomerulonephritis
E. None of the above

228 What is the most serious complication of the most serious complication of chronic tuberculous LEAN ANGELO MIDTERM
Tuberculous meningitis? meningitis is arachnoid fibrosis leading to SILVERIO, MD (TOP 4 EXAM 1 - FEB
A. Tuberculoma formation hydrocephalus and obliterative endarteritis leading to - AUG 2014 MED 2015
B. Fibrinous basal exudates leading to cranial brain infarction. This is SIMILAR TO PREVIOUS BOARDS; TOPNOTCH
nerve palsies BOARD EXAM CONCEPT/PRINCIPLE. MD), MD
C. Choroid plexus involvement leading to
diffuse meningoencephalitis
D. Obliterative endarteritis
E. none of the above
229 A 42 y/o male presents with right abdominal Von hippel lindau disease is a type of familial tumor LEAN ANGELO MIDTERM
mass associated with gross hematuria. He also syndrome characterized by renal cell carcinoma, SILVERIO, MD (TOP 4 EXAM 1 - FEB
had episodes of diaphoresis and refractory pheochromocytoma and cerebellar - AUG 2014 MED 2015
hypertension. Imaging showed renal mass and hemangioblastoma. Sipple syndrome aka MEN type BOARDS; TOPNOTCH
adrenal medullary tumor. What is your primary IIA is a triad of parathyroid hyperplasia, medullary MD), MD
diagnosis? carcinoma, pheochromocytoma. Tuberous sclerosis is
A. Sipple syndrome associated with renal angiomyolipoma, retinal
B. Tuberous sclerosis hamartoma,cortical tubers, cardiac rhabdomyoma,
C. Von hippel lindau syndrome shagreen patches and ash leaf patch. Gorlin
D. Li Fraumeni syndrome syndrome.Li Fraumeni syndrome is caused by p53
E. None of the above mutation and associated with sarcomas, breast cancer,
adrenal cortical tumors, leukemia and gliomas.
230 What is the most striking histologic finding in SIMILAR TO PREVIOUS BOARD EXAM LEAN ANGELO MIDTERM
Desquamative Interstitial Pneumonia CONCEPT/PRINCIPLE last aug 2014. taken verbatim SILVERIO, MD (TOP 4 EXAM 1 - FEB
secondary to smoking? Robbins 8th ed pp 704 - AUG 2014 MED 2015
A. Thickened alveolar septa due to sparse BOARDS; TOPNOTCH
inflammatory lymphocytic infiltrate MD), MD
B. Mild interstitial fibrosis
C. Necrotic type II pneumocytes
D. accumulation of a large number of
macrophages with brown pigment cytoplasmic
bodies.
E. none of the above
231 An 18 yo female patient presents with chronic Menetrier disease is characterized by diffuse LEAN ANGELO MIDTERM
epigastric pain associated with weight loss, hyperplasia of the foveolar epithelium of the body and SILVERIO, MD (TOP 4 EXAM 1 - FEB
diarrhea, and peripheral edema. Upon fundus of the stomach. It is secondary to proliferation - AUG 2014 MED 2015
endoscopy, menetrier disease was given as a of mucous neck cells and not the gastric connective BOARDS; TOPNOTCH
diagnosis. Which of the following is true tissue. The hyperplasia is secondary to the excessive MD), MD
regarding the diagnosis? secretion of TGF alpha. although it is a self limiting
A. there is an irregular enlargement of the condition, it has an increased risk for the development
gastric rugal folds secondary to hyperplasia of of gastric adenocarcinoma.
gastric connective tissue
B. associated with excessive secretion of
platelet derived growth factor
C. it is a progressive unremmittng condition
however no risk for gastric adenocarcinoma
D. characterized by hyperplasia of foveolar
mucous neck cells
E. all of the above
232 Which of the following pathogenic organism Rotavirus is the most common cause of severe LEAN ANGELO MIDTERM
can cause diarrhea by selectively destroying childhood diarrhea. It affects only the apical and mid SILVERIO, MD (TOP 4 EXAM 1 - FEB
apical mature enterocytes of the small villous enterocytes leading to loss of absorptive - AUG 2014 MED 2015
intestine? capacity of the small intestine. All the other virus BOARDS; TOPNOTCH
A. Norovirus shows nonspecific affectation of different population MD), MD
B. adenovirus of enterocytes. Robbins 8th ed pp 804-805.
C. Rotavirus
D. Norwalk virus
E. All of the above

TOPNOTCH MEDICAL BOARD PREP PATHOLOGY SUPEREXAM Page 31 of 99


For inquiries visit www.topnotchboardprep.com.ph or email us at topnotchmedicalboardprep@gmail.com
TOPNOTCH MEDICAL BOARD PREP PATHOLOGY SUPEREXAM
For inquiries visit www.topnotchboardprep.com.ph or email us at topnotchmedicalboardprep@gmail.com
Item QUESTION EXPLANATION AUTHOR TOPNOTCH
# EXAM
233 A 61 y/o male presents with hesitancy, dysuria, Benign Prostatic hyperplasia is characterized by LEAN ANGELO MIDTERM
and nocturia. BPH is entertained. Which of the hyperplasia of the stromal and epithelial cells along SILVERIO, MD (TOP 4 EXAM 1 - FEB
following is true regarding the diagnosis? ther prostatic periurethral zone. The hallmark is the - AUG 2014 MED 2015
A. This is secondary to hypertrophy of the presence of nodularity. The compression of the BOARDS; TOPNOTCH
stromal and epithelial cells adjacent normal prostate by the nodules creates a MD), MD
B. the microscopic hallmark is nodularity pseudocapsular plane. the pathogenesis behind the
C. presence of true capsule creates a plane hyperplasia is not secondary to increase mitosis but
between the normal tissue and prostatic rather impaired cell death.
nodules
D. the cause of the nodularity is secondary to
increase mitosis of the epithelial cells
E. all of the above
234 what is the most serious consequence of Alkaptonuria is an autosomal recessive disorder LEAN ANGELO MIDTERM
alkaptonuria? secondary to lack of homogentisic oxidase which SILVERIO, MD (TOP 4 EXAM 1 - FEB
A. Renal failure converts homogentisic acid to methylacetoacetic acid. - AUG 2014 MED 2015
B. Hypertrophic cardiomyopathy It causes black discoloration of the urine if it allows to BOARDS; TOPNOTCH
C. Arthropathy stand on ambient air. It accumulates throughout the MD), MD
D. Liver failure body causing blue to black pigmentation of soft tissue
E. None of the above such as ears, nose and face. however, it also causes
pigmentation of the articular cartilages causing its loss
of resiliency and fibrillation. there is no association
b/w alkaptonuria and cardiomyopathy,liver failure
and renal failure.
235 Which of the following describes the resolution B- congestion, C- red hepatization, D- gray LEAN ANGELO MIDTERM
stage of lobar pneumonia hepatization. Robbins 8th ed pp 712-713 SILVERIO, MD (TOP 4 EXAM 1 - FEB
A. Progressive enzymatic digestion of alveolar - AUG 2014 MED 2015
exudates producing granular semifluid debris BOARDS; TOPNOTCH
ingested by macrophage MD), MD
B. Vascular dilatation with intraalveolar fluid
and presence of numerous bacteria
C. massive confluent exudation with
neutrophils, rbc, and alveolar fibrin
D. progressive disintegration of RBC and
presence of fibrinosuppurative exudates
E. none of the above
236 A 28 y/o female veterinarian presents with 3 the most likely impression regarding this case is LEAN ANGELO MIDTERM
week history dry cough accompanied by low histoplasmosis. TB is set aside because of the absence SILVERIO, MD (TOP 4 EXAM 1 - FEB
grade fever and night sweats. CXR revealed of positive sputum smear along with negative PPD. - AUG 2014 MED 2015
cavitary lesion noted on bilateral lung apices. Histoplasmosis clinical and morphological BOARDS; TOPNOTCH
AFB showed negative for 3 specimen. PPD is presentation greatly mimicks that of TB. H. MD), MD
also negative. Which of the following is true capsulatum is internalized by macrophage after
regarding the most plausible diagnosis in this opsonization. there is also presence of caseation
case? necrosis. fulminant disemminated histoplasmosis can
A. the organism is internalized primarily by occur in immunocompromised state. This patient is
NK cells and atypical lymphocytes after also veterinarian which most likely expose to bird
opsonization with antibody droppings.
B. there is absence of caseation necrosis
C. produces concentric calcification of the
lesion ( tree bark appearance) when drug
control is achieved
D. rarely become disseminated even in
immunocompromised state.
E. all are correct.
237 A 59 y/o female presents with rapidly enlarging based on the rapidity of symptoms, the most likely LEAN ANGELO MIDTERM
anterior neck mass accompanied with diagnosis is anaplastic carcinoma. B- Medullary SILVERIO, MD (TOP 4 EXAM 1 - FEB
proggressive of dyspnea, dysphagia and carcinoma. C- papillary carcinoma. D- follicular - AUG 2014 MED 2015
hoarseness of 1 month duration. What would carcinoma BOARDS; TOPNOTCH
be the most consistent pathologic finding if MD), MD
biopsy is done to the mass?
A. large pleomorphic multinucleated giant
cells with fusiform cells
B. spindle cells with amyloid deposits on
adjacent stroma
C. finely dispersed chromatin giving a
optically clear or empty appearance of the
nucleus
D. uniform cells forming small follicles
containing colloid substance
E. None of the above.
238 A 45 y/o male presents with a rapidly growing based on the characteristics, this is most likely a case LEAN ANGELO MIDTERM
nodular skin lesion on the forehead with noted of basal cell carcinoma. A- squamous cell carcinoma, c- SILVERIO, MD (TOP 4 EXAM 1 - FEB
telangiectasia on top of the lesion. If biopsy was seborrheic keratoses, d- trichoepithelioma, e- lichen - AUG 2014 MED 2015
done, which of the following will be consistent planus BOARDS; TOPNOTCH
with the most plausible diagnosis? MD), MD
A. Highly anaplastic with necrosis and
presence of polygonal cells with numerous
areas of keratinization
B. basaloid cells with hyperchromatic nuclei
with palisading alignment in the periphery of
the tumor
C. hyperkeratosis with horn cyst
D. basaloid cell with with hair like
differentiation
E. presence of civatte bodies

TOPNOTCH MEDICAL BOARD PREP PATHOLOGY SUPEREXAM Page 32 of 99


For inquiries visit www.topnotchboardprep.com.ph or email us at topnotchmedicalboardprep@gmail.com
TOPNOTCH MEDICAL BOARD PREP PATHOLOGY SUPEREXAM
For inquiries visit www.topnotchboardprep.com.ph or email us at topnotchmedicalboardprep@gmail.com
Item QUESTION EXPLANATION AUTHOR TOPNOTCH
# EXAM
239 A 49 y/o female presented to the ER secondary Pagets disease or Osteitis deformans is a rare skeletal LEAN ANGELO MIDTERM
to right tibial fracture after a fall from a disease characterized by haphazard mosiaic pattern of SILVERIO, MD (TOP 4 EXAM 1 - FEB
standing height, Xray revealed middle lamellar bone formation. Clinically it affects the axial - AUG 2014 MED 2015
transverse fracture of the right tibia with noyed skeleton leading to compression of multiple spinal and BOARDS; TOPNOTCH
enlarged, sclerotic irregular thickening of both cranial nerve exits. Bony overgrowth of skull base also MD), MD
cortical and cancellous bone. further physical causes different cranial nerve palsies. they also
examination noted weakness of bilateral hip prominence of facial bones presenting as leontiasis
flexors and knee flexors, sensory deficit on L3- ossea.
S1 dermatome bilateral. patient presents with
prominent zygoma and supraorbital ridge with
frontal bossing. she also has lateral rectus palsy
on (R), (L) peripheral facial palsy. which of the
following is the most likely diagnosis of the
patient?
A. Pagets disease
B. early onset idiopathic osteoporosis
C. osteomalacia
D. multiple enchondromatosis
E. Osteopetrosis
240 What is the most common malignancy of the basal cell carcinoma is the most common malignancy LEAN ANGELO MIDTERM
eyelid? of the eyelid with predilection on the lower lid and SILVERIO, MD (TOP 4 EXAM 1 - FEB
A. Squamous cell carcinoma medial canthus. This is followed by sebaceous - AUG 2014 MED 2015
B. Basal cell carcinoma carcinoma and squamous cell carcinoma. Robbins 8th BOARDS; TOPNOTCH
C. Sebaceous carcinoma ed p 1348 MD), MD
D. lymphangioma
E. Hemangioma

241 In pathologic examination of a brain of a person SIMILAR TO PREVIOUS BOARD EXAM KEVIN BRYAN LO, MIDTERM 2
who died from rabies which of the following CONCEPT/PRINCIPLE, in our exam we had to choose MD (TOP 7 - AUG EXAM - FEB
areas of the brain where the pathognomonic between cerebellum or hippocampus, both answers 2014 MED BOARDS; 2015
negri bodies will be found? are correct and should be either cerebellum or TOPNOTCH MD)
A. cerebrum hippocampus
B. cerebellum
C. hippocampus
D. thalamus
E. B and C
242 a 65 year old male patient presents with a SIMILAR TO PREVIOUS BOARD EXAM KEVIN BRYAN LO, MIDTERM 2
chronic history of decreasing urinary stream, CONCEPT/PRINCIPLE MD (TOP 7 - AUG EXAM - FEB
dribbling of urine and difficulty in voding, What 2014 MED BOARDS; 2015
is the pathophysiologic process involved in a TOPNOTCH MD)
person diagnosed to have BPH?
A. hypertrophy
B. hyperplasia
C. neoplasia
D. metaplasia
E. none of the above
243 What percentage of patients with hepatitis B SIMILAR TO PREVIOUS BOARD EXAM KEVIN BRYAN LO, MIDTERM 2
eventually proceed to have hepatocellular CONCEPT/PRINCIPLE, almost exact choices, page 96 MD (TOP 7 - AUG EXAM - FEB
carcinoma? of topnotch handout in the diagram, I did not get this 2014 MED BOARDS; 2015
A. 6-15% as well during our exam :) TOPNOTCH MD)
B. 12-20%
C. >30%
D. <5%
E. 2-8%

244 A 60 year old patient with longstanding type 2 SIMILAR TO PREVIOUS BOARD EXAM KEVIN BRYAN LO, MIDTERM 2
DM recently underwent renal transplantation CONCEPT/PRINCIPLE, A is hyperacute, B is acute, C is MD (TOP 7 - AUG EXAM - FEB
of 1 kidney for end stage renal disease chronic 2014 MED BOARDS; 2015
secondary to diabetic nephropathy, after a TOPNOTCH MD)
period of 4 weeks after transplantation, he
developed sudden weight gain, bilateral pedal
edema, pain or tenderness near the right flank
area and fever, histologic examination would
most likely reveal:
A. thrombotic occlusion of capillaries and
fibrinoid necrosis
B. extensive interstitial mononuclear cell
infiltration and edema
C. vascular changes, interstitial fibrosis and
tubular atrophy with loss of renal parenchyma
D. A and B
E. all of the above
245 40 year old male patient presented with SIMILAR TO PREVIOUS BOARD EXAM KEVIN BRYAN LO, MIDTERM 2
repeated episodes of bloody stools. Workup CONCEPT/PRINCIPLE, no granulomas, no strictures, MD (TOP 7 - AUG EXAM - FEB
and evaluation revealed multiple superficial limited to the colon, submucosal inflammation, no skip 2014 MED BOARDS; 2015
broad based ulcers, with diffuse inflammation lesions TOPNOTCH MD)
limited to the colon, biopsy revealed
inflammation limited only to the submucosal
area, which of the following is the most likely
condition?
A. Crohn's disease
B. bacillary dysentery
C. amoebic colitis
D. ulcerative colitis
E. none of the above

TOPNOTCH MEDICAL BOARD PREP PATHOLOGY SUPEREXAM Page 33 of 99


For inquiries visit www.topnotchboardprep.com.ph or email us at topnotchmedicalboardprep@gmail.com
TOPNOTCH MEDICAL BOARD PREP PATHOLOGY SUPEREXAM
For inquiries visit www.topnotchboardprep.com.ph or email us at topnotchmedicalboardprep@gmail.com
Item QUESTION EXPLANATION AUTHOR TOPNOTCH
# EXAM
246 50 year old male chronic smoker presents with SIMILAR TO PREVIOUS BOARD EXAM KEVIN BRYAN LO, MIDTERM 2
chronic cough dyspnea and sputum production. CONCEPT/PRINCIPLE, panacinar is with alpha 1 MD (TOP 7 - AUG EXAM - FEB
He also has bouts of easy fatigability. Patient antitrypsin deficiency 2014 MED BOARDS; 2015
also presents with increased anteroposterior TOPNOTCH MD)
diameter of the chest wall. Patient most likely
has this type of emphysema
A. panacinar
B. centrilobular
C. paraseptal
D. alveolar
E. none of the above
247 A 56 year old male presenting with recent onset SIMILAR TO PREVIOUS BOARD EXAM KEVIN BRYAN LO, MIDTERM 2
of bipedal edema, BP was 120/80 Heart rate of CONCEPT/PRINCIPLE, most questions were cases, MD (TOP 7 - AUG EXAM - FEB
90. Does not recall any history of recent wordy, 2 step cases. A lot of renal pathology. 2014 MED BOARDS; 2015
infections but has been said to have multiple Subendothelial, granular pattern nephrotic syndrome TOPNOTCH MD)
sexual partners as well. Over the past few is membranoproliferative type
weeks, he noticed his urine to be foamy and
bubbly, and would have puffy eyelids especially
on waking in the morning. Further examination
revealed urine protein of +3, RBCs 0-2 WBC 0-
1, pus cells (+1), no casts and crystals, urine
specific gravity of 1.015, urine pH of 5.5. A renal
biopsy was eventually done revealing large
hypercellular glomeruli, thickened glomerular
basement membrane and increased mesangial
matrix. Immunofluorescence revealed granular
pattern of C3, IgG and C1q, electron microscopy
revealed subendothelial deposits, which of the
following is the most likely finding?
A. focal segmental glomerulosclerosis
B. membranous glomerulonephritis
C. rapidly progressive glomerulonephritis
D. membranoproliferative glomerulonephritis
E. minimal change disease
248 What is the most common etiologic factor in SIMILAR TO PREVIOUS BOARD EXAM KEVIN BRYAN LO, MIDTERM 2
acute myeloid leukemia? CONCEPT/PRINCIPLE, vague question, a lot of genetic MD (TOP 7 - AUG EXAM - FEB
A. radiation exposure mutations like down syndrome are commonly 2014 MED BOARDS; 2015
B. benzene associated with acute leukemias, all the others are also TOPNOTCH MD)
C. genetic mutation common causes of leukemias but less common.
D. carcinogen exposure
E. previous treatment for leukemia

249 What is the pathophysiologic mechanism in SIMILAR TO PREVIOUS BOARD EXAM KEVIN BRYAN LO, MIDTERM 2
which patients with protein energy CONCEPT/PRINCIPLE, malnutrition with low protein MD (TOP 7 - AUG EXAM - FEB
malnutrition sometimes have fatty liver levels leads to low synthesis of necessary proteins like 2014 MED BOARDS; 2015
especially with protein malnutrition? globulins and lipoprotein transporters like LDL HDL TOPNOTCH MD)
A. increased mobilization of fat stores from VLDL that transport cholesterol and trigylcerides thus
adipose tissue fats are continuously stored in the liver and not
B. increased glucose stored as fat distributed properly to the tissues
C. increased endogenous synthesis of
triglycerides
D. impaired transport of fats
E. all of the above
250 A 50 year old male with long standing history of SIMILAR TO PREVIOUS BOARD EXAM KEVIN BRYAN LO, MIDTERM 2
chronic alcoholism presents with jaundice, CONCEPT/PRINCIPLE, eosinophilic clumps are MD (TOP 7 - AUG EXAM - FEB
palmar erythema, distended abdomen, spider mallory bodies with both macro micronodular 2014 MED BOARDS; 2015
nevi near in the upper anterior chest wall. What cirrhosis in the setting of long standing alcoholism and TOPNOTCH MD)
is the most likely histologic finding present in hallmarks of cirrhosis, D is chronic hepatitis,C is acute
the liver? hepatitis B is cholestatic change
A. councilman bodies
B. fine foamy appearance with feathery
degeneration and accumulation of bile pigment
plugs in dilated canaliculi
C. apoptosis bridging necrosis, ballooning
degeneration, interface hepatitis, ground glass
hepatocyes
D. hepatocyte apoptosis bridging necrosis and
fibrosis, deposition of fibrous tissue,
E. hepatocyte swelling necrosis, eosinophilic
clumps in hepatocytes, both micro and
macronodular cirrhosis
251 What pathological entity is characterized by SIMILAR TO PREVIOUS BOARD EXAM KEVIN BRYAN LO, MIDTERM 2
widespread small vessel vasculitis, interstitial CONCEPT/PRINCIPLE, diffuse skin fibrosis and organ MD (TOP 7 - AUG EXAM - FEB
and perivascular fibrosis of the skin and changes including esophageal dysmotility are due to 2014 MED BOARDS; 2015
multiple organs, progressive atrophy and small vessel vasculitis and fibrosis TOPNOTCH MD)
collagenous replacement of alimentary canal
especially the esophagus, clawlike deformities
and limited range of motion of joints and
fingers, raynauds phenomenon, fibrinoid
necrosis of renal arterioles
A. wegeners granulomatosis
B. systemic lupus erythematosus
C. systemic sclerosis
D. churg strauss
E. polyarteritis nodosa

TOPNOTCH MEDICAL BOARD PREP PATHOLOGY SUPEREXAM Page 34 of 99


For inquiries visit www.topnotchboardprep.com.ph or email us at topnotchmedicalboardprep@gmail.com
TOPNOTCH MEDICAL BOARD PREP PATHOLOGY SUPEREXAM
For inquiries visit www.topnotchboardprep.com.ph or email us at topnotchmedicalboardprep@gmail.com
Item QUESTION EXPLANATION AUTHOR TOPNOTCH
# EXAM
252 A 30 year old male patient with 9 months SIMILAR TO PREVIOUS BOARD EXAM KEVIN BRYAN LO, MIDTERM 2
history of chronic cough with sputum CONCEPT/PRINCIPLE - obliterative endarteritis cause MD (TOP 7 - AUG EXAM - FEB
production accompanied by low grade the multiple microinfarcts from CNS TB 2014 MED BOARDS; 2015
afternoon fevers, night sweats, weight loss TOPNOTCH MD)
suddenly presents with fever, altered
sensorium and nuchal rigidity. Cranial CT scan
shows basal enhancement and multiple
infarcts, what is the pathophysiologic
mechanism of these infarcts?
A. thrombotic occlusion of the vessels causes
multiple infarcts
B. embolic seeding from extrapulmonary
tuberculotic sites
C. vasospasm induced by hemorrhage in the
CNS
D. obliterative endarteritis produced by CNS
infection
E. accelerated atherosclerotic changes
253 A 16 year old female patient presents with 2 B symptoms, mediastinal mass, contiguous growth KEVIN BRYAN LO, MIDTERM 2
months history of easy fatigability, body most likely hodgkin's lymphoma MD (TOP 7 - AUG EXAM - FEB
malaise, afternoon fevers, night sweats and 2014 MED BOARDS; 2015
weight loss. There was noted occasional cough TOPNOTCH MD)
with multiple palpable cervical lymph nodes
noted which prompted an initial assessment of
pulmonary tuberculosis, however, a chest xray
revealed an incidental mediastinal mass with
no apical lesions noted. What is the most likely
consideration?
A. primary bronchogenic carcinoma
B. hodgkin's lymphoma
C. non hodgkin's lymphoma
D. pulmonary tuberculoma
E. metastatic pulmonary carcinoma
254 50 year old postmenopausal female patient SIMILAR TO PREVIOUS BOARD EXAM KEVIN BRYAN LO, MIDTERM 2
presenting with a history of vertebral CONCEPT/PRINCIPLE, punched out lesions and MD (TOP 7 - AUG EXAM - FEB
compression fractures and hip fracture was multiple pathological fractures 2014 MED BOARDS; 2015
worked up due to repeated complains of leg TOPNOTCH MD)
and extremity pains. She was noted to have 4
episodes of flu over the past 2 months. Work
ups revealed mild anemia, slightly elevated
serum calcium and multiple lytic bone lesions
were noted over the ribs and some punched out
defects were seen in the skull xrays, which of
the following conditions does this patient most
likely have
A. osteoporosis
B. multiple myeloma
C. waldenstrom's macroglobulinemia
D. myelodysplastic syndrome
E. osteomalacia
255 35 year old female presents with an SIMILAR TO PREVIOUS BOARD EXAM KEVIN BRYAN LO, MIDTERM 2
erythematous rash on the cheeks aggravated by CONCEPT/PRINCIPLE, most specific and indicates a MD (TOP 7 - AUG EXAM - FEB
exposure to sunlight, she has been having a probable renal involvement of disease 2014 MED BOARDS; 2015
history of repeated recurrent joint pains and TOPNOTCH MD)
swelling over the ankles and wrists, she has
some oral apthous ulcers accompanied by
headaches and easy fatigability. She came in for
consult due to a recent onset of bipedal edema
accompanied by tea colored urine. Initial work
ups were requested which revealed urine
protein of +2, urine RBC of 5-10, urine WBC of
0-1, pus cells of +1, rbc casts +1, no crystals.
ANA titers were also elevated. What is the
autoantibody specific to this disease which also
associated with renal pathological involvement
A. anti-smith
B. ANA
C. anti-dsDNA
D. anti-histone
E. Anti-Ro
256 In relation to the question above, if a kidney SIMILAR TO PREVIOUS BOARD EXAM KEVIN BRYAN LO, MIDTERM 2
biopsy is done, what is the most CONCEPT/PRINCIPLE, a lot of multiple stem questions MD (TOP 7 - AUG EXAM - FEB
pathognomonic expected finding? 2014 MED BOARDS; 2015
A. nonspecific mesangial proliferation TOPNOTCH MD)
B. tram track appearance of the basement
membrane
C. spike and dome appearance
D. chicken wire loop appearance
E. focal segmental glomerulosclerosis

TOPNOTCH MEDICAL BOARD PREP PATHOLOGY SUPEREXAM Page 35 of 99


For inquiries visit www.topnotchboardprep.com.ph or email us at topnotchmedicalboardprep@gmail.com
TOPNOTCH MEDICAL BOARD PREP PATHOLOGY SUPEREXAM
For inquiries visit www.topnotchboardprep.com.ph or email us at topnotchmedicalboardprep@gmail.com
Item QUESTION EXPLANATION AUTHOR TOPNOTCH
# EXAM
257 In relation to question above, this disease SIMILAR TO PREVIOUS BOARD EXAM KEVIN BRYAN LO, MIDTERM 2
manifestation belongs to what type of immune CONCEPT/PRINCIPLE, immune complex type 3, ABO MD (TOP 7 - AUG EXAM - FEB
hypersensitivity? incompatibility also came out as type 2 2014 MED BOARDS; 2015
A. type 1 hypersensitivity TOPNOTCH MD)
B. type 2
C. type 3
D. type 4
E. non of the above

258 Which of the following cytokines secreted by T SIMILAR TO PREVIOUS BOARD EXAM KEVIN BRYAN LO, MIDTERM 2
helper lymphocytes leads to direct strong CONCEPT/PRINCIPLE from boards, IL1 and IL6 are MD (TOP 7 - AUG EXAM - FEB
activation of macrophages? pyrogens and initiate inflammation and fever, TNF 2014 MED BOARDS; 2015
A. IL1 alpha also promotes inflammation and cachexia, TGF TOPNOTCH MD)
B. TNF alpha beta inhibits inflammation and promotes cell repair
C. TNF beta
D. IFN gamma
E. IL-6

259 20 year old patient developed paraparesis. 2 SIMILAR TO PREVIOUS BOARD EXAM KEVIN BRYAN LO, MIDTERM 2
weeks after developing a diarrheal episode, CONCEPT/PRINCIPLE, GBS albuminocytologic MD (TOP 7 - AUG EXAM - FEB
patient started developing gradual ascending dissociation 2014 MED BOARDS; 2015
paralysis. No other sensory and CN deficits TOPNOTCH MD)
were seen, lumbar puncture was done, which of
the following is an expected finding?
A. oligoclonal bands IgG, CSF pleocytosis
B. increased CSF protein, WBCs 0-1
C. increased CSF protein, neutrophils 5000
D. increased CSF protein, lymphocytes 500
E. normal CSF protein, normal glucose WBCs 0-
1
260 30 year old female patient presenting with SIMILAR TO PREVIOUS BOARD EXAM KEVIN BRYAN LO, MIDTERM 2
nonspecific pneumonitis, cough, intermittent CONCEPT/PRINCIPLE, asthmatic manifestations, MD (TOP 7 - AUG EXAM - FEB
bouts of bronchial asthma with long standing pulmonary, skin, other involved organs but spares the 2014 MED BOARDS; 2015
history of allergic rhinitis. Presents with renal vessels TOPNOTCH MD)
nodular skin lesions, biopsy of skin lesions
reveal capillary vasculitis with granulomas and
eosinophilic necrosis. There was no noted renal
involvement, the most likely pathological
condition in this case is?
A. polyarteritis nodosa
B. churg strauss
C. wegeners granulomatosis
D. microscopic polyangitis
E. giant cell arteritis
261 All of the following are ultrastructural changes Karyorrhexis is nuclear fragmentation which is RAYMUND MARTIN MIDTERM 3
of reversible cell injury except: irreversible LI, MD (TOP 1 - AUG EXAM - FEB
A. Dilation of the endoplasmic reticulum 2014 MED BOARDS; 2015
B. Cell membrane blebbing TOPNOTCH MD)
C. Karyorrhexis
D. Detachment of polysomes
E. None of the above

262 A patient was diagnosed with Paget's disease Although the use of glucocorticoids may predispose to RAYMUND MARTIN MIDTERM 3
and has been treated with bisphophonates. osteoporotic fracture. The description indicates LI, MD (TOP 1 - AUG EXAM - FEB
Currently, he presents at your clinic with left Codman's triangle and is indicative of osteosarcoma. 2014 MED BOARDS; 2015
hip pain that is not relieved by over the counter Paget's disease is a predisposing factor to TOPNOTCH MD)
pain relievers? He has history of taking low osteosarcoma
dose steroids for his asthma. Imaging reveals
lytic and sclerotic changes with periosteal
elevation and reactive new bone formation.
What is the most likely diagnosis?
A. Osteopetrosis
B. Glucocorticoid-induced osteoporosis
C. Osteoarthritis
D. Osteosarcoma
E. Rheumatoid arthritis
263 Which type of small vessel vasculitis is Churg-Strauss spares the kidney and is characterized RAYMUND MARTIN MIDTERM 3
characterized by eosinophil-rich by association with asthma and eosinophilia LI, MD (TOP 1 - AUG EXAM - FEB
granulomatous inflammation involving the 2014 MED BOARDS; 2015
respiratory tract and is associated with asthma TOPNOTCH MD)
and blood eosinophilia with sparing of the
kidneys?
A. Polyarteritis nodosa
B. Wegener granulomatosis
C. Takayasu arteritis
D. Leukocytoclastic vasculitis
E. Churg-Strauss syndrome
264 Which if the following cytokines has an IL-12 - Th1; IL-4 - Th2; IL10 -suppression of RAYMUND MARTIN MIDTERM 3
important role in IFN-γ production in Th1- inflammation LI, MD (TOP 1 - AUG EXAM - FEB
mediated chronic inflammation? 2014 MED BOARDS; 2015
A. IL-12 TOPNOTCH MD)
B. IL-10
C. IL-4
D. IL-2
E. TNF-α

TOPNOTCH MEDICAL BOARD PREP PATHOLOGY SUPEREXAM Page 36 of 99


For inquiries visit www.topnotchboardprep.com.ph or email us at topnotchmedicalboardprep@gmail.com
TOPNOTCH MEDICAL BOARD PREP PATHOLOGY SUPEREXAM
For inquiries visit www.topnotchboardprep.com.ph or email us at topnotchmedicalboardprep@gmail.com
Item QUESTION EXPLANATION AUTHOR TOPNOTCH
# EXAM
265 A patient with Duchenne muscular dystrophy is Allelic heterogeneity refers to different mutations at RAYMUND MARTIN MIDTERM 3
a muscular disorder involving the dystrophin the same locus. Variable expressivity is the same LI, MD (TOP 1 - AUG EXAM - FEB
gene that usually leads to a male patient being mutation but with different phenotypic 2014 MED BOARDS; 2015
wheelchair-bound during his teenage years. In characteristics. Anticipation occurs in trinucleotide TOPNOTCH MD)
contrast, Becker muscular dystrophy is a milder repeats and is characterized by an ealier and more
but similar form of disease wherein afflicted severe onset of disease in suceeding generations.
patients can usually still walk at the same age Incomplete penetrance means some patients with the
period. This is an example of: disease causing mutation may not present signs or
A. Pleiotropy symptoms; usually in autosominal dominant.
B. Variable expressivity Pleiotropy occurs in mitochondrial disorders.
C. Anticipation
D. Incomplete penetrance
E. Allelic heterogeneity
266 A 34 year old male smoker presents to your Thromboangiitis obliterans or Buerger disease usually RAYMUND MARTIN MIDTERM 3
clinic with leg pain. History is unremarkable in male smokers less than 35 years old. LI, MD (TOP 1 - AUG EXAM - FEB
except for a 15 year history of smoking 2014 MED BOARDS; 2015
consuming 1 pack per day. Pain worsens with TOPNOTCH MD)
exercise and is also not totally relieved by rest.
Biopsy of the affected leg reveals segmental
vasculitis of small to medium vessels
accompanied with luminal thrombosis and
microabscesses extending into contiguous veins
and nerve. What is the most likely diagnosis?
A. Thromboangiitis obliterans
B. Polyarteritis nodosa
C. Peripheral arterial atherosclerosis
D. Microscopic polyangiitis
E. Infectious vasculitis
267 A 32 year old male patient claims that he has Thymoma - associated with myasthenia gravis; Small RAYMUND MARTIN MIDTERM 3
difficulty with prolonged chewing of food and cell Ca - associated with Lambert-Eaton (improves LI, MD (TOP 1 - AUG EXAM - FEB
that he experiences diplopia when watching TV with prolonged stimulation) 2014 MED BOARDS; 2015
for long periods of time. What is neoplasm most TOPNOTCH MD)
associated with this condition?
A. Small cell lung Ca
B. Thymoma
C. Hodgkin's lymphoma
D. Renal Cell Ca
E. Breast Ca
268 A 10 year old child presents to your clinic with A is type 1 hypersensitivity; B is type II, C is type IV, D RAYMUND MARTIN MIDTERM 3
complaints of dark urine, periorbital edema, is type III LI, MD (TOP 1 - AUG EXAM - FEB
malaise, and body weakness. You elicit a history 2014 MED BOARDS; 2015
of throat infection 3 weeks prior. You expect TOPNOTCH MD)
which of the following pathological lesion for
this type of hypersensitivity reaction:
A. Vascular dilation edema, smooth muscle
contraction
B. Phagocytosis and lysis of cells
C. Perivascular cellular infiltrates, edema, and
granuloma formation
D. Inflammation with associated necrotizing
vasculitis
E. None of these
269 An young athlete dies suddenly and Hypertrophic cardiomyopathy is most commonly RAYMUND MARTIN MIDTERM 3
unexpectedly. Autopsy reveals massive caused by a mutation in myosin heavy chain LI, MD (TOP 1 - AUG EXAM - FEB
myocardial hypertrophy with disproportionate 2014 MED BOARDS; 2015
thickening of the ventricular septum. What is TOPNOTCH MD)
the most likely pathologic etiology of this
condition?
A. Previous viral myocardial infection
B. Atherosclerotic blockage of coronary arteries
C. Radiation-induced myocardial fibrosis
D. Mutation of gene encoding beta-myosin
heavy chain
E. None of the above
270 You are assigned to assist a transplant surgeon A and D is acute rejection; C is hyperacute rejection RAYMUND MARTIN MIDTERM 3
who will be performing a kidney transplant. In LI, MD (TOP 1 - AUG EXAM - FEB
order to prepare yourself for possible questions 2014 MED BOARDS; 2015
that you may be asked during the surgery, you TOPNOTCH MD)
recall your pathology knowledge.You know that
chronic rejection will be characterized by:
A. Extensive interstitial mononuclear cell
infiltration and edema
B. Dominated by vascular changes, interstitial
fibrosis
C. Occurs due to preformed antibodies is the
circulation
D. Takes the form of necrotizing vasculitis with
endothelial cell necrosis
E. A and B

TOPNOTCH MEDICAL BOARD PREP PATHOLOGY SUPEREXAM Page 37 of 99


For inquiries visit www.topnotchboardprep.com.ph or email us at topnotchmedicalboardprep@gmail.com
TOPNOTCH MEDICAL BOARD PREP PATHOLOGY SUPEREXAM
For inquiries visit www.topnotchboardprep.com.ph or email us at topnotchmedicalboardprep@gmail.com
Item QUESTION EXPLANATION AUTHOR TOPNOTCH
# EXAM
271 Which of following characteristics indicate A, B, and C is indicative of non-neoplastic proliferation RAYMUND MARTIN MIDTERM 3
neoplastic/malignant lymph node LI, MD (TOP 1 - AUG EXAM - FEB
proliferation? 2014 MED BOARDS; 2015
A. Marked variation in the shape and size of the TOPNOTCH MD)
follicles
B. Presence of phagocytic macrophages and
recognizable light and dark zones
C. Preservation of the lymph node architecture
D. Marked by monoclonal proliferation
E. All of these
272 A patient presents to your clinic with chronic Crohn's disease is most common in the ileum though it RAYMUND MARTIN MIDTERM 3
diarrhea and perianal abscess occasionally may occur anywhere in the GI tract LI, MD (TOP 1 - AUG EXAM - FEB
associated with low-grade fever. She had a 2014 MED BOARDS; 2015
prior history of intestinal fistula formation TOPNOTCH MD)
which was managed conservatively. Physical
examination reveals mouth ulcers with no
other significant findings. What is the most
commonly affected site in patients with this
condition?
A. Ileum
B. Sigmoid colon
C. Jejunum
D. Stomach
E. Duodenum
273 A mother brings to you her 1 year old child due This refers to Bruton's which is an X-linked RAYMUND MARTIN MIDTERM 3
to recurrent respiratory bacterial infections immunodeficiency syndrome LI, MD (TOP 1 - AUG EXAM - FEB
and otitis media. You suspect an 2014 MED BOARDS; 2015
immunodeficiency syndrome due to failure of TOPNOTCH MD)
preB cells to undergo Ig gene rearrangement
and to develop into mature B cells. The
mutation responsible is encoded in which
chromosome?
A. Chromosome 17
B. Chromosome 7
C. Chromosome 16
D. Chromosome 22
E. Chromosome X
274 A female patient presenting with right breast Charateristic description of invasive lobular Ca RAYMUND MARTIN MIDTERM 3
mass but with no other associated symptoms LI, MD (TOP 1 - AUG EXAM - FEB
undergoes biopsy which revealed presence of 2014 MED BOARDS; 2015
dyscohesive infiltrating tumor cells, often TOPNOTCH MD)
arranged in single file or in loose clusters or
sheets with associated signet-ring cells
containing an intracytoplasmic mucin droplet.
What is the diagnosis?
A. Invasive ductal carcinoma
B. Ductal carcinoma in situ
C. Paget's disease
D. Invasive lobular carcinoma
E. Medullary carcinoma
275 A patient complains of polydipsia and polyuria. ADH levels are adequate but kidney does not respond RAYMUND MARTIN MIDTERM 3
Laboratory tests reveal a dilute urine and in nephogenic type LI, MD (TOP 1 - AUG EXAM - FEB
hypernatremia. If this condition is a nephogenic 2014 MED BOARDS; 2015
type of diabetes insipidus, what will be TOPNOTCH MD)
expected during water deprivation test?
A. Decreased ADH levels
B. Elevated ADH levels
C. Elevated urine osmolality
D. Correction of hypernatremia
E. Low serum osmolality
276 An infant born term to G1P1 mother dies. Prior Characteristic description of cytomegalovirus in utero RAYMUND MARTIN MIDTERM 3
to death, the infant was noted to have infection. LI, MD (TOP 1 - AUG EXAM - FEB
intrauterine growth retardation, jaundice, 2014 MED BOARDS; 2015
hepatosplenomegaly, anemia, and bleeding due TOPNOTCH MD)
to thrombocytopenia. Periventricular
calcification is observed on imaging and
histology reveals prominent intranuclear
basophilic inclusions surrounded by a clear
halo. In utero infection is suspected. Which is
the most likely etiologic agent?
A. Toxoplasma
B. Herpes virus
C. Cytomegalovirus
D. Rubella
E. Treponema pallidum

TOPNOTCH MEDICAL BOARD PREP PATHOLOGY SUPEREXAM Page 38 of 99


For inquiries visit www.topnotchboardprep.com.ph or email us at topnotchmedicalboardprep@gmail.com
TOPNOTCH MEDICAL BOARD PREP PATHOLOGY SUPEREXAM
For inquiries visit www.topnotchboardprep.com.ph or email us at topnotchmedicalboardprep@gmail.com
Item QUESTION EXPLANATION AUTHOR TOPNOTCH
# EXAM
277 A patient presents with chronic cough CD4 lymphocytes and IFN- gamma are responsible for RAYMUND MARTIN MIDTERM 3
associated with night sweats, weight loss, and granulomatous reaction (type IV) LI, MD (TOP 1 - AUG EXAM - FEB
body malaise. Sputum microscopy reveals acid 2014 MED BOARDS; 2015
fast bacilli and X-ray reveals right upper lobe TOPNOTCH MD)
density? Granulomatous reaction which is
typical of this condition is brought about by
which of the following:
A. CD8 lymphocytes, Humoral immune system
B. CD4 lymphocytes, IFNγ
C. CD4 lymphocytes, IL-4
D. CD8 lymophocytes, NK cells
E. B cells and macrophages
278 A 16 year old African American man presents to Autosplenectomy occurs in early childhood in sickle RAYMUND MARTIN MIDTERM 3
the emergency room severe chest pain, fever, cell anemia LI, MD (TOP 1 - AUG EXAM - FEB
tachypnea, and cough. Imaging reveals lung 2014 MED BOARDS; 2015
infiltrates and ABG shows hypoxemia. Further TOPNOTCH MD)
probing reveals that patient had previous
episodes of painful crises and anemia.
Peripheral smear reveals target cells and
characteristically shaped erythrocytes. An Hb
electrophoresis is ordered but patient
deteriorates and dies. Autopsy examination of
the spleen will most likely demonstrate:
A. Red pulp expansion
B. Extramedullary hematopoiesis
C. Small fibrotic atrophy
D. Congestive changes and increase in volume
E. A and B
279 Baretts esophagus is what type of abnormal Intestinal metaplasia - replacement with simple RAYMUND MARTIN MIDTERM 3
cell/tissue change? columnar with goblet cells that predispose to LI, MD (TOP 1 - AUG EXAM - FEB
A. Dysplasia adenocarcinoma 2014 MED BOARDS; 2015
B. Metaplasia TOPNOTCH MD)
C. Anaplasia
D. Neoplastic change
E. Necrosis

280 A 30 year old woman presents to your clinic Most common form of kidney disease in SLE is diffuse RAYMUND MARTIN MIDTERM 3
complaining of a butterfly rash over the face, proliferative (type IV). B is focal proliferative (III). C is LI, MD (TOP 1 - AUG EXAM - FEB
fever, joint pains, and photosensitivity. membranous (V). D and E are mesangial (I and II) 2014 MED BOARDS; 2015
Prelimionary laboratory tests reveal positive TOPNOTCH MD)
ANA and thrombocytopenia. The pathologic
picture of the most common form of renal
disease associated with this condition is:
A. >50% of glomeruli exhibiting crescent
formation, fibrinoid necrosis, proliferation of
endothelial and mesangial cells
B. <50% of glomeruli exhibiting crescent
formation, fibrinoid necrosis, proliferation of
endothelial and mesangial cells
C. characterized by diffuse thickening of the
capillary walls
D. slight mesangial cell proliferation and
immune complex deposition
E. moderate mesangial cell proliferation and
immune complex deposition
281 A 65 year old woman with a heavy smoking Dx: Chronic Bronchitis. Hallmark is marked ERIC ROYD FINAL EXAM -
history presents with chronic productive cough hyperplasia of bronchial submucosal glands and TALAVERA, MD (TOP FEB 2015
that has been present for 3 consecutive months bronchial smooth muscle hypertrophy (with 1 - AUG 2014 MED
over the past 2 years. On PE, she is noted to be LYMPHOCYTIC) infiltrates.Choice A pertains to BOARDS; TOPNOTCH
overweight with a bluish tinge to her skin. Asthma, Choice B is bronchiectasis, Choice C is MD)
Auscultation revealed rhonchi on both lung emphysema, Choice E is ARDS
fields. Which of the following is the most likely
histologic finding in this patient's airways?
A. Bronchial smooth muscle hypertrophy with
proliferation of eosinophils
B. Permanent bronchial dilation, which is
filled with mucus and neutrophils
C. Dilation of air spaces with destruction of
alveolar walls
D. Hyperplasia of mucus secreting submucosal
glands
E. Diffuse alveolar damage with leakage of
protein rich fluid into the alveolar spaces
282 A newborn infant who presented with cyanosis The clinical consequence primarily depends on the ERIC ROYD FINAL EXAM -
was diagnosed to have Tetralogy of Fallot severity of the subpulmonary stenosis as this TALAVERA, MD (TOP FEB 2015
(TOF), which of the following features of TOF determines the direction of blood flow. If the stenosis 1 - AUG 2014 MED
would primarily determine the severity of the is mild, the abnormality resembles an isolated VSD BOARDS; TOPNOTCH
disease process? and the shunt may be left to right without cyanosis MD)
A. Ventricular Septal Defect (so called pink tetralogy). As the obstruction
B. Right Ventricular Hypertrophy increases, the greater is the resitance to the RV
C. Subpulmonary stenosis outflow, producing right to left shunting and cyanosis
D. Over riding of the aorta
E. None of the above

TOPNOTCH MEDICAL BOARD PREP PATHOLOGY SUPEREXAM Page 39 of 99


For inquiries visit www.topnotchboardprep.com.ph or email us at topnotchmedicalboardprep@gmail.com
TOPNOTCH MEDICAL BOARD PREP PATHOLOGY SUPEREXAM
For inquiries visit www.topnotchboardprep.com.ph or email us at topnotchmedicalboardprep@gmail.com
Item QUESTION EXPLANATION AUTHOR TOPNOTCH
# EXAM
283 Which of the following is true regarding cancer There is some correlation between tumor burden and ERIC ROYD FINAL EXAM -
cachexia? severity of cachexia. The BMR is increased depsite TALAVERA, MD (TOP FEB 2015
A. There is no correlation between tumor reduced food intake. There is currently no satisfactory 1 - AUG 2014 MED
burden and the severity of cachexia treatment to cancer cachexia other than removal of BOARDS; TOPNOTCH
B. There is a redued basal metabolic rate the underlying cause, the tumor. MD)
accompied by reduced food intake
C. The weight loss seen results equally from
loss of fat and lean muscle
D. Satisfactory treatment can be achieved by
giving total parenteral nutrition alone
E. All of the above
284 A 72 year old male presented with progressive Dx: (most likely) Pancreatic CA. Weight loss, anorexia ERIC ROYD FINAL EXAM -
weight loss, anorexia and abdominal pain and malaise are signs of advanced disease. Majority TALAVERA, MD (TOP FEB 2015
radiating to the back. On PE, the gallbladder (60%) arise in the pancreatic head. Only 15% arise in 1 - AUG 2014 MED
was noted to be enlarged. An abdominal CT the body and 5% in the tail. Obstructive jaundice is BOARDS; TOPNOTCH
scan was done which demonstrated a mass in associated with most cases of carcinoma involving the MD)
the pancreatic head. Which of the following pancreatic head. CA 19-9 are useful in following
statements is true regarding the probable patient response but are too non specific and lack the
disease of this patient? sensitivity for screening.
A. Weight loss, anorexia and generalized
malaise are early signs of the disease
B. Abdominal pain is usually the first
symptom
C. The most common location is in the body of
the pancreas
D. Jaundice is more prominent when there is
involvement of the tail of the pancreas
E. CA 19-9 is a specific and sensitive marker
and can be used to screen for this disease
285 Which of the following tumors of the CNS Meningiomas are predominantly benign tumors of ERIC ROYD FINAL EXAM -
express progesterone receptors which may adult usually attached to the dura, that arise from the TALAVERA, MD (TOP FEB 2015
result in rapid growth during pregnancy? meningthelial cell of the arachnoid. Meningiomas are 1 - AUG 2014 MED
A. Meningioma usually slow growing lesions that present with vague BOARDS; TOPNOTCH
B. Oligodendroglioma non localizing sign , however due to presence of MD)
C. Astrocytoma progesterone receptors there can be rapid growth
D. Ependymoma during pregnancy
E. Medulloblastoma

286 A 78 year old female presented with severe Dx: Multiple Myeloma. Bone marrow aspiration would ERIC ROYD FINAL EXAM -
bone pain, pallor and frequent respiratory tract yield an increase number of plasma cells (>30%) TALAVERA, MD (TOP FEB 2015
infections. Laboratory studies showed 1 - AUG 2014 MED
hypercalcemia and an elevated creatinine. BOARDS; TOPNOTCH
Radiographs of the skull, long bones and spine MD)
demonstrated multiple punch out lesions.
Which of the following statement is FALSE
regarding this disorder?
A. Bone marrow aspiration would
demonstrate an increased number of T cells
B. Neurologic manifestations such as lethargy
and confusion are also present due to the
hypercalcemia
C. Cellular immunity is relatively unaffected
D. Perfomance of protein electrophoresis
would yield an abnormal spike (M spike) which
is most often an IgG
E. Renal failure and infections are major
causes of death from this disease
287 A 13 year old boy was brought to the ER due to the clinical findings are consistent with primary ERIC ROYD FINAL EXAM -
progressive weakness, easy fatigability and adrenocortical insufficiency or addison disease. 70% TALAVERA, MD (TOP FEB 2015
weight loss for the past 4 months. In addition, of cases are autoimmune but recently the most 1 - AUG 2014 MED
he has recently presented with nausea, frequent cause was TB. Hyperpigmentation is due to BOARDS; TOPNOTCH
vomiting and abdominal discomfort. On PE his compensatory hypothalamic production of MD)
blood pressure was noted to be markedly proopiomelanocortin, the precursor peptide of both
decreased; in addition there was note of corticotropin and MSH.
increased pigmentation over the creases of his
skin. What is the most likey diagnosis of this
case?
A. Secondary hyperaldosteronism
B. Cushing syndrome
C. Osteitis fibrosa cystica
D. Pheochromocytoma
E. Addison disease
288 Which of the following interleukins has a IL with predominantly anti inflammatory functions: ERIC ROYD FINAL EXAM -
predominant anti-inflammatory functions? IL-4, IL-3, IL-9, IL-10, IL-11, IL-13 ans IL-19 TALAVERA, MD (TOP FEB 2015
A. IL-1 1 - AUG 2014 MED
B. IL-2 BOARDS; TOPNOTCH
C. IL-5 MD)
D. IL-10
E. IL-8

TOPNOTCH MEDICAL BOARD PREP PATHOLOGY SUPEREXAM Page 40 of 99


For inquiries visit www.topnotchboardprep.com.ph or email us at topnotchmedicalboardprep@gmail.com
TOPNOTCH MEDICAL BOARD PREP PATHOLOGY SUPEREXAM
For inquiries visit www.topnotchboardprep.com.ph or email us at topnotchmedicalboardprep@gmail.com
Item QUESTION EXPLANATION AUTHOR TOPNOTCH
# EXAM
289 A 26 year old male presented with hematuria, Dx: Goodpasture syndrome. Anti-GBM antibodies are ERIC ROYD FINAL EXAM -
periorbital edema, hypertension and directed against both glomerular and alveolar TALAVERA, MD (TOP FEB 2015
hemoptysis. Serological testing was positive for basement membranes which would account for the 1 - AUG 2014 MED
anti-glomerular basement membrane bodies. nephritic syndrome an hemoptysis seen. Choice A is BOARDS; TOPNOTCH
Which of the following is the classic histologic seen in post streptococcal glomerulonephritis. Choice MD)
finding of this disease? C is seen in membranous GN. Choice D is seen in lupus
A. Lumpy bumpy immunofluorescence nephropathy and MPGN while Choice E is seen in
B. Linear immunofluorescence MPGN
C. Spike and dome appearance of the
glomerular basement membrane
D. Subenothelial immune complex deposition
E. Tram track appearance of the glomerular
basement membrane on electron microscopy
290 Which of the following statements is true SCID affects both cell mediated and humoral response. ERIC ROYD FINAL EXAM -
regarding Severe Combined Immunodeficiency It more commonly affects boys than girls. The most TALAVERA, MD (TOP FEB 2015
(SCID)? common form is inherited in a X-linked manner and 1 - AUG 2014 MED
A. It is a syndrome with a characteristic defect involves a common mutation in the common-gamma BOARDS; TOPNOTCH
involving only the humoral response chain of cytokine receptors. The remaininf cases are MD)
B. Majority are inherited in an autosomal inherited as AR, for which the most common cause is
recessive manner ADA deficiency. BMT is the mainstay of treatment and
C. It is the first human disease in which gene it is the first human disease in which gene therapy has
therapy has been successful been successful
D. It more commonly affects girls than boys
E. Deficiency of adenosine deaminase enzyme
(ADA) is seen in the X-linked variant of SCID
291 A 58 year old male, non-smoker, is referred for Dx: Polycythemia vera. The disorder is characterized ERIC ROYD FINAL EXAM -
evaluation of marked erythrocytosis and by prominent erythrocytosis, moderate granulocytosis TALAVERA, MD (TOP FEB 2015
splenomegaly. A CBC showed a marked and thrombocytosis. Because of the hyperviscoity and 1 - AUG 2014 MED
elevation of the RBC, WBC and platelet count. sludging of blood, there is frequet association with BOARDS; TOPNOTCH
Blood uric acid level was also elevated. Oxygen thrombosis or hemorrhagic phenomenon. Marked MD)
saturation was normal at 98%. Which of the splenomegaly and a decreased EPO are other classic
following is characteristic of this disorder ? characteristics. Cushing syndrome and hypoxia are
A. Frequent association with thrombosis or associated with secondary polycythemia which
hemorrhagic phenomenon characterized by an increased EPO. About 3% of
B. Secondary to increased EPO production patients terminate in Acute leukemia not CML
C. It is most often secondary to hypoxia
D. Usually terminates into chronic
myelogenous leukemia
E. A manifestation of Cushing syndrome
292 Which of the following features of liver Hyperestrogenism produces local vasodilation in the ERIC ROYD FINAL EXAM -
cirrhosis is not associated with impaired skin which could account for palmar erythema and TALAVERA, MD (TOP FEB 2015
estrogen metabolism and consequent spider angiomata. It also causes gynecomastia and 1 - AUG 2014 MED
hyperestrogenism ? testicular atrophy among males. Hemorrhoids are BOARDS; TOPNOTCH
A. Palmar erythema primarily due to the formation of portosystemic MD)
B. Internal hemorrhoids shunts.
C. Gynecomastia
D. Testicular atrophy
E. Spider angiomata
293 A 68 year old woman fell and sustained a pelvic Fat embolism syndrome occurs 2-3 days after severe ERIC ROYD FINAL EXAM -
fracture. Due to financial constraints she opted fracture injury and includes progressive CNS TALAVERA, MD (TOP FEB 2015
to just take pain medications for the moment. dysfunction and severe respiratory insufficiency. 1 - AUG 2014 MED
After a couple of days she developed a rapidly Thrombocytopenia is common and petechial BOARDS; TOPNOTCH
progresive respiratory failure which eventually hemorrhage can result from obstruction of the MD)
led to her death. On autopsy, there was note of microvasculature by embolic fat droplets. Respiratory
numerous petechiae over the conjunctiva and insufficiency may be due to injury to pulmonary
chest wall. What could have probably caused microvessels with leakage of fluid into the alveoli
the death of this woman? resulting in ARDS.
A. Saddle embolus
B. Acute tubular necrosis
C. Epidural hematoma
D. Bladder rupture
E. Fat embolization
294 Which of the the following features would point The inflammation in UC is limited to the ERIC ROYD FINAL EXAM -
more to Crohn's disease rather than Ulcerative mucosa/submucosal area. Skip lesions are common in TALAVERA, MD (TOP FEB 2015
colitis ? crohn's disease. 1 - AUG 2014 MED
A. Diffuse colonic involvement BOARDS; TOPNOTCH
B. Marked pseudopolyps MD)
C. Transmural inflammation
D. Toxic megacolon
E. Absence of non caseating granuloma

295 An 18 year old male presented with swelling of Osteoclastoma or giant cell tumor involve both the ERIC ROYD FINAL EXAM -
the left knee, an X-ray was done which showed epihyses and metaphyses. The majority arise in the TALAVERA, MD (TOP FEB 2015
lytic lesions over the said area. A biopsy was knee. The typical location causes arthritis like 1 - AUG 2014 MED
done which showed an abundance of symptoms. Biopsy of the tumor will reveal an BOARDS; TOPNOTCH
multinucleated giant cells with background of abundance of multinucleated giant cells with MD)
mononuclear stromal cells. What is the most background of mononuclear stromal cells
likely diagnosis for this case?
A. Osteoclastoma
B. Osteosarcoma
C. Ewing's tumor
D. Chondrosarcoma
E. Osteochondroma

TOPNOTCH MEDICAL BOARD PREP PATHOLOGY SUPEREXAM Page 41 of 99


For inquiries visit www.topnotchboardprep.com.ph or email us at topnotchmedicalboardprep@gmail.com
TOPNOTCH MEDICAL BOARD PREP PATHOLOGY SUPEREXAM
For inquiries visit www.topnotchboardprep.com.ph or email us at topnotchmedicalboardprep@gmail.com
Item QUESTION EXPLANATION AUTHOR TOPNOTCH
# EXAM
296 A 33 year old woman sought consult for post Risk factors for Cervical CA: all of the aforementioned ERIC ROYD FINAL EXAM -
coital bleeding. Which of the following risk choices + male partner with multiple sexual partners, TALAVERA, MD (TOP FEB 2015
factors would point to cervical carcinoma as the persistent infection with HPV 16 or 18, 1 - AUG 2014 MED
underlying cause? Immunosuppression, certain HLA subtypes, use of oral BOARDS; TOPNOTCH
A. Multiple sexual partners contraceptives MD)
B. Young age at first intercourse
C. High parity
D. Smoking
E. All of the above
297 A 56 year old male presented with a 2 day Dx: acute prostatitis. Biopsy of a man with prostatitis ERIC ROYD FINAL EXAM -
history of fever, chills and dysuria. On PE, the is contraindicated as it may lead to sepsis TALAVERA, MD (TOP FEB 2015
prostate was noted to be exquisitely tender and 1 - AUG 2014 MED
boggy. Which of the following statements is BOARDS; TOPNOTCH
FALSE regarding this condition? MD)
A. The diagnosis should be established by
biopsy which would show focal areas of
necrosis with diffuse edema
B. The causative agens are similar to those
that cause UTI
C. The organisms are usually implanted by
intraprostatic reflux of urine from the posterior
urethra.
D. Organisms can seed the prostate by
lymphohematogenous routes from a distant
foci of infection
E. None of the above
298 Which of the following features is more The presence of increased storage of iron in marrow ERIC ROYD FINAL EXAM -
suggestive of anemia of chronic disease rather macrophages, a high serum ferritin level and a TALAVERA, MD (TOP FEB 2015
than iron deficiency anemia? reduced total iron binding capacity readily rules out 1 - AUG 2014 MED
A. Low serum ferritin level IDA as the cause BOARDS; TOPNOTCH
B. Low MCV MD)
C. Low TIBC
D. High serum iron
E. None of the above

299 A 62 year old male, known hypertensive, Dx: Aortic dissection. The dissection is usually ERIC ROYD FINAL EXAM -
presented to the ER due to an acute onset of initiated by a tear of the intimal layer of the blood TALAVERA, MD (TOP FEB 2015
chest pain. He described the chest pain as vessel. Most serious complications occur with 1 - AUG 2014 MED
having a "tearing" quality and radiating to the dissection that involves the aorta from the aortic valve BOARDS; TOPNOTCH
back. 12L ECG showed no findings of ischemia to the aortic arch. More than 90% of cases of aortic MD)
or infarction Which of the following statements dissection are men aged 40-60 with antecedent
is true regarding the most likely condition ? hypertension. A recognizable medial damage seems to
A. It is usally initiated by a tear over the be neither a pre requiste for dissection nor a
adventitia of the blood vessel guarantee that dissection is imminent.
B. The most serious complications occur when
the involvement is distal to the subclavian
artery
C. More than 90% of patients presenting with
this condition have an underlying connective
tissue disorder
D. A damage to the tunica media is a pre
requiste for the the disease condition to occur
E. Majoirty of cases are found within 10 cm of
the the aortic valve
300 A 45 year old woman presented with areas of Dx: Acanthosis nigricans. Acanthosis nigricans is a ERIC ROYD FINAL EXAM -
velvety hyperpigmentation involving the axilla brown to black, poorly defined, velvety TALAVERA, MD (TOP FEB 2015
and posterior surface of the neck. The said hyperpigmentation of the skin. It is usually found in 1 - AUG 2014 MED
areas initially started as smaller macules but body folds, such as the posterior and lateral folds of BOARDS; TOPNOTCH
have now progressed to form palpable plaques. the neck, the armpits, groin, navel, forehead, and other MD)
Which of the following is an important areas. It typically occurs in individuals younger than
association of this skin lesion? age 40, and is associated with obesity or
A. Viral infection endocrinopathies, and is also indicative of visceral
B. Visceral malignancy malignancy such as CA of the lungs, breast, stomach or
C. Asthma uterus
D. Seizure disorders
E. None of the above
301 Which of the following is true about the intrinsic or mitochondrial pathway is the major LEAN ANGELO BACK-UP
intrinsic pathway of apoptosis ? mechanism of apoptosis in the mammalian cells. The SILVERIO, MD (TOP 4 MIDTERM
A. The upregulation of Bcl-2 regulates the role of Bcl 2 ( in the presence of a growth signal) is to - AUG 2014 MED EXAM - FEB
mitochondrial permeability preventing the regulate the permeability of the mitochondrial BOARDS; TOPNOTCH 2015
leakage of cytochrome aa3 membranes limiting the leakage of cytochrome c.once MD), MD
B. Activation of Bax and Bak forms oligomers stress or damage was done to the cell, there is
that create a hole in the mitochondrial upregulation of Bax and Bak. this in turn inhibits the
membrane Bcl2 and forms oligomers creating holes to the
C. caspase 3 is the critical initiating caspase mitochondria leading to release of cytochrome c.
for apoptosis cytosolic cytochrome c binds to Apaf 1 forming
D. upregulation of Smac/DIABLO activates the apoptosome to activate caspase 9. the critical initiator
physiologic inhibitors of apoptosis of caspase activation. Other factors such as
E. all of the above Smac/DIABLO inhibits the inhibitors of apoptosis
including caspase 3. Robbins 8th ed p 29.

TOPNOTCH MEDICAL BOARD PREP PATHOLOGY SUPEREXAM Page 42 of 99


For inquiries visit www.topnotchboardprep.com.ph or email us at topnotchmedicalboardprep@gmail.com
TOPNOTCH MEDICAL BOARD PREP PATHOLOGY SUPEREXAM
For inquiries visit www.topnotchboardprep.com.ph or email us at topnotchmedicalboardprep@gmail.com
Item QUESTION EXPLANATION AUTHOR TOPNOTCH
# EXAM
302 various experimental models states that the sirtuins have histone deacetylase activity and are LEAN ANGELO BACK-UP
most important way of promoting cellular thought to promote the expression of several genes SILVERIO, MD (TOP 4 MIDTERM
longevity is through caloric restriction. Which whose products increase longevity. These products - AUG 2014 MED EXAM - FEB
of the family of proteins is responsible for such include proteins that increase metabolic activity, BOARDS; TOPNOTCH 2015
effect? reduce apoptosis, stimulate protein folding, and MD), MD
A. Insulin Growth factor inhibit the harmful effects of oxygen free radicals.
B. telomerase Robbins 8th ed p 41.
C. sirtuins
D. transforming growth factor beta
E. none of the above
303 Which of the followingl nuclear ultrastructural when a cell is exposed to transient heat, it undergoes LEAN ANGELO BACK-UP
changes can be seen in the affected cell, when cellular swelling. These are the ultrastructural SILVERIO, MD (TOP 4 MIDTERM
the latter is exposed to transient heat ? changes of reversible cell injury: Plasma membrane - - AUG 2014 MED EXAM - FEB
A. disaggregation blebbing, blunting and loss of microvilli, mitochondria BOARDS; TOPNOTCH 2015
B. Appearance of amorphous densities - swelling, appearance of small amorphous densities, MD), MD
C. Myelin figures ER- dilation, polysome detachment, myelin figures.
D. Blebbing nuclear 0 disaggregation of granular and fibirllar
E. All of the above elements.Robbins 8th ed p 14.

304 the following proteins are leukocyte molecules P selectin is expressed in the endothelium during LEAN ANGELO BACK-UP
responsible for endothelial adhesion except? leukocyte adhesion. Other endothelial molecutes are SILVERIO, MD (TOP 4 MIDTERM
A. integrin E -selectin, Glycam 1, ICAM 1, VCAM 1. Robbins 8th ed - AUG 2014 MED EXAM - FEB
B. Sialyl Lewis X modified proteins p 49. BOARDS; TOPNOTCH 2015
C. L selectin MD), MD
D. P selectin
E. None of the above

305 what type of substance accumulate in Niemann cholesterol and cholesterol esters accumulate in the LEAN ANGELO BACK-UP
pick disease Type C? following conditions: atherosclerosis, xanthomas, SILVERIO, MD (TOP 4 MIDTERM
A. protein cholesterolosis of gallbladder, and Niemann pick - AUG 2014 MED EXAM - FEB
B. TAG disease type C. page 34-35. BOARDS; TOPNOTCH 2015
C. cholesterol MD), MD
D. calcium
E. Glycogen

306 A 65 y/o male went for consult secondary, this is a case of diffuse Large B cell lymphoma. Its LEAN ANGELO BACK-UP
fever, chronic malaise, and weight loss. Upon characteristics are anaplastic relatively large cells SILVERIO, MD (TOP 4 MIDTERM
physical examination, multiple with diffuse pattern of growth. Its cytogenetic, gene - AUG 2014 MED EXAM - FEB
lymphadenophathies were noted on the expression, profiling and immunohistochemical is BOARDS; TOPNOTCH 2015
oropharyngeal area. hepatosplenomegaly is heterogenous. However, 30% presents with BCL6 MD), MD
also noted. Biopsy of the lymph node showed dysregulation. A- follicular lymphoma. C- ( starry sky
generalized effacement of sinusoid structure by pattern) Burkitts lymphoma. D- mantle cell
sheets of large cells 4-5x the size of a normal lymphoma. Robbins 8th ed - 606-608
lymphocyte. which of the following is true
about the diagnosis ?
A. t(14,18) translocation is the hallmark for
this condition
B. BCL6 dysregulation is common in around
30% of the cases
C. it exhibits a high mitotic index with
numerous apoptotic cells interspersed with
macrophages
D. it is correlated with cyclin D1
overexpression
E. none of the above
307 what is the most striking histologic finding in (SIMILAR TO PREVIOUS BOARD EXAM LEAN ANGELO BACK-UP
desquamative interstitial pneumonia ( CONCEPT/PRINCIPLE) Robbins 8th ed p 704 SILVERIO, MD (TOP 4 MIDTERM
smoking related interstitial disease)? - AUG 2014 MED EXAM - FEB
A. Multiple macrophages containing dusty BOARDS; TOPNOTCH 2015
brown cytoplasmic pigments MD), MD
B. Thickening of alveolar septa
C. presence of lamellar bodies in the
macrophage
D. presence of plump cuboidal pneumocytes
along the septa
E. massive interstitial fibrosis
308 A 48 y/o male complains of intermittent low (SIMILAR TO PREVIOUS BOARD EXAM LEAN ANGELO BACK-UP
grade fever, fatigue and weight loss of CONCEPT/PRINCIPLE). This is a case of AML. Robbins SILVERIO, MD (TOP 4 MIDTERM
approximately 1 month duration. Physical 8th ed page 621-624 - AUG 2014 MED EXAM - FEB
examination showed BP 100/60, PR 105 bpm, BOARDS; TOPNOTCH 2015
PP 25 cpm, pale palpebral conjunctiva, palatal MD), MD
and cutaneous petechia.liver edge was palpated
4 cm below the right subcostal margin.blood
smear showed large pleomorphic cells with
multiple nucleoli and cytoplasmic needle like
azurophilic granules. which of the following is
true about his condition ?
A. t(8,21) balanced translocation imparts a
favorable prognosis
B. aberrant tyrosine kinase activation is a
universal feature of the disease
C. diagnosis requires at least 20% blast in the
bone marrow
D. all of the above
E. none of the above

TOPNOTCH MEDICAL BOARD PREP PATHOLOGY SUPEREXAM Page 43 of 99


For inquiries visit www.topnotchboardprep.com.ph or email us at topnotchmedicalboardprep@gmail.com
TOPNOTCH MEDICAL BOARD PREP PATHOLOGY SUPEREXAM
For inquiries visit www.topnotchboardprep.com.ph or email us at topnotchmedicalboardprep@gmail.com
Item QUESTION EXPLANATION AUTHOR TOPNOTCH
# EXAM
309 a 78 y/o male complains of sever chronic low this is a case of multiple myeloma. Serum M protein is LEAN ANGELO BACK-UP
back pain accompanied by weakness and almost always greater than 3gm/dl. ( asymptomatic SILVERIO, MD (TOP 4 MIDTERM
lethargy. PE reveals pale palpebral conjunctiva, patients with < 3gm/dl M protein and without skeletal - AUG 2014 MED EXAM - FEB
bilateral basal crackles and hepatomegaly. lesion is called MGUS). IgG rearrangement is universal BOARDS; TOPNOTCH 2015
Lumbar Xray showed multiple osteolytic lesion in MM. myeloma cells produce factors that activated MD), MD
scattered around the vertebral bodies. which of RANKL which in turn stimulates osteoclastic activity.
the following is consistent about his condition? (SIMILAR TO PREVIOUS BOARD EXAM
A. serum M protein level is 2.5 g/dl CONCEPT/PRINCIPLE)
B. rearrangements involving the Ig heavy chain
is rare
C. proliferation and survival of neoplastic cells
are dependent on cytokines (IL6)
D. RANKL upregulation causes apoptosis of
osteoblast
E. all of the above.
310 A 45 y/o obese female complains of the difference between reflux and eosinophilic LEAN ANGELO BACK-UP
retrosternal pain aggravated after a high fat esophagitis is that the latter is characterized by SILVERIO, MD (TOP 4 MIDTERM
meal. She also noted frequent regurgitation of abundance of intraepithelial eosinophils. Robbins 8th - AUG 2014 MED EXAM - FEB
sour tasting gastric contents especially at the ed p770. ( SIMILAR TO PREVIOUS BOARD EXAM BOARDS; TOPNOTCH 2015
middle of her sleep.pastmedical history shows CONCEPT/PRINCIPLE) MD), MD
that she is asthmatic, hypertensive and diabetic.
EGD was done and biopsy was performed in the
lower esophageal region showing mild number
of intraepithelial eosinophils and basal zone
hyperplasia. what is the diagnosis?
A. chemical esophagitis
B. reflux esophagitis
C. eosinophilic esophagitis
D. hiatal hernia
E. none of the above
311 which of the following is a consistent finding in Systemic sclerosis is associated with anti Scl70 while LEAN ANGELO BACK-UP
scleroderma except? CREST or limited scleroderma is correlated to SILVERIO, MD (TOP 4 MIDTERM
A. Anticentromere antibody is correlated to anticentromere antibody. Intimal proliferation of - AUG 2014 MED EXAM - FEB
CREST syndrome . vascular tissue with progressive fibrosis is BOARDS; TOPNOTCH 2015
B. Intimal vascular proliferation is the most characteristic of this disease. joint destruction is not MD), MD
consistent finding common in systemic sclerosis ( distinguishing feature
C. Renal abnormalities resembles that of a against RA). Robbins 8th ed p 224
malignant hypertension
D. synovial inflammation and joint destruction
is similar to that of RA.
312 which of the following is true about the children diagnosed at <18 months of age generally has LEAN ANGELO BACK-UP
neuroblastoma ? a good prognosis regardless of the stage. N myc SILVERIO, MD (TOP 4 MIDTERM
A. Children younger than 18months have a amplication is related to Neuroblastoma not c myc. - AUG 2014 MED EXAM - FEB
worst prognosis regardless of stage 90% of neuroblastoma secretes catecholamines BOARDS; TOPNOTCH 2015
B. C myc amplification is the most important regardless of location. Robbins 8th ed p 475-476 MD), MD
genetic abnormality used in risk stratification
C. only neuroblastoma cells that are located in
the adrenal medulla produces catecholamines
D. presence of Homer wright pseuodorosettes
E. all of the above
313 What is the most characteristic lesion of HIV HIV is highly associated with a variant of FSGS known LEAN ANGELO BACK-UP
associated nephropathy ? as collapsing glomerulopathy. It is characterized by SILVERIO, MD (TOP 4 MIDTERM
A. Capillary wall hyalinosis retraction or collapse of the entire glomerulus and - AUG 2014 MED EXAM - FEB
B. Duplication of the basement membrane hypertrophy and proliferation of visceral epithelial BOARDS; TOPNOTCH 2015
C. Retraction of the entire glomerulus cells. Robbins 8th ed p 926 MD), MD
D. Diffuse mesangial proliferation
E. Fibrin crescents

314 What is the most serious complication of the most serious complication of chronic tuberculous LEAN ANGELO BACK-UP
Tuberculous meningitis? meningitis is arachnoid fibrosis leading to SILVERIO, MD (TOP 4 MIDTERM
A. Tuberculoma formation hydrocephalus and obliterative endarteritis leading to - AUG 2014 MED EXAM - FEB
B. Fibrinous basal exudates leading to cranial brain infarction. This is a SIMILAR TO PREVIOUS BOARDS; TOPNOTCH 2015
nerve palsies BOARD EXAM CONCEPT/PRINCIPLE. MD), MD
C. Choroid plexus involvement leading to
diffuse meningoencephalitis
D. Obliterative endarteritis
E. none of the above
315 which of the following morphologic changes 30% of polyarteritis nodosa is associated with LEAN ANGELO BACK-UP
consistent with hepatitis B associated Hepatitis B virus infection. PAN is a segmental SILVERIO, MD (TOP 4 MIDTERM
vasculitis? transmural necrotizing inflammation of small and - AUG 2014 MED EXAM - FEB
A. Pulmonary circulation is commonly medium sized arteries typically involving the renal BOARDS; TOPNOTCH 2015
affected and visceral vessels sparing the pulmonary MD), MD
B. Focal transmural necrotizing lesions of all circulation. all stages of activity coexist in different
stages of activity vessels. this is unlike microscopic polyangitis wherein
C. Associated with peripheral all lesions are at the same stage of activity and
hypereosinophilia pulmonary circulation is commonly affected.
D. granulomatous inflammation with elastic hypereosinophilia is common in churge strauss
lamina fragmentation syndrome while elastic lamina fragmentation is
E. all of the above characteristic of giant cell arteritis. Robbins 8th ed
513-515

TOPNOTCH MEDICAL BOARD PREP PATHOLOGY SUPEREXAM Page 44 of 99


For inquiries visit www.topnotchboardprep.com.ph or email us at topnotchmedicalboardprep@gmail.com
TOPNOTCH MEDICAL BOARD PREP PATHOLOGY SUPEREXAM
For inquiries visit www.topnotchboardprep.com.ph or email us at topnotchmedicalboardprep@gmail.com
Item QUESTION EXPLANATION AUTHOR TOPNOTCH
# EXAM
316 A 63 y/o male went for consult secondary to this is a case of lichen planus. It is a form of interface LEAN ANGELO BACK-UP
skin lesion noted on his forehead. PE showed dermatitis with characteristic necrolytic basal layer at SILVERIO, MD (TOP 4 MIDTERM
multiple pruritic, violaceous polygonal papule the tip of dermal papilla ( civatte bodies). B-mycoses - AUG 2014 MED EXAM - FEB
with lacelike pattern. Which of the morphologic fungoides, C- psoriasis, D- pemphigus vulgaris BOARDS; TOPNOTCH 2015
changes is consistent with the diagnosis? Robbins 8th ed p 1192 MD), MD
A. civatte bodies at the basal epidermis
B. pautrier microabscesses
C. thinned out stratum granulosum
D. acantholytic blisters
E. all of the above
317 A 25 y/o female complains of a constant left Robbins 8th ed page 1224. osteoid osteoma are by LEAN ANGELO BACK-UP
midthigh pain most severe during at sleep. She definition less than 2cm in greatest dimension SILVERIO, MD (TOP 4 MIDTERM
claims that taking effectively relieved of her occuring more on the appendicular skeleton. It usually - AUG 2014 MED EXAM - FEB
symptoms. Femoral xray done showing 2cm affects teenagers with 2:1 ratio predilection for BOARDS; TOPNOTCH 2015
subperiosteal lesion located at the midshaft women. 50% arise in the femur. Most common MD), MD
with dense sclerotic background. which of the symptom of this condition is severe nocturnal pain
following is the most likely diagnosis? effectively relieved by aspirin. the pain is secondary to
A. osteoblastoma the production of prostaglandin E2 by the
B. osteoma proliferating osteoblast. morphology will show a
C. osteoid osteoma tremendous amount of reactive bone formation
D. osteochondroma around the tumor ( nidus).
E. none of the above
318 what is the hallmark feature in diagnosing diagnosis of parathyroid carcinoma based on cytologic LEAN ANGELO BACK-UP
parathyroid carcinoma? detail is unreliable, and invasion of surrounding SILVERIO, MD (TOP 4 MIDTERM
A. Nuclear atypia tissues and metastasis are the only reliable criteria. - AUG 2014 MED EXAM - FEB
B. Follicular formation of oxyphil cells (SIMILAR TO PREVIOUS BOARD EXAM BOARDS; TOPNOTCH 2015
C. Local invasion CONCEPT/PRINCIPLE) Robbins 8th pp 1127-1128 MD), MD
D. Mitotic index
E. Capsular invasion

319 what major fibril protein responsible for A - primary amyloidosis, C- systemic senile LEAN ANGELO BACK-UP
secondary amyloidosis on a chronic kidney amyloidosis, D- medullary carcinoma E-alzheimers SILVERIO, MD (TOP 4 MIDTERM
disease patient on prolonged hemodialysis ? disease. Robbins 8th ed p 252 - AUG 2014 MED EXAM - FEB
A. Amyloid light chain BOARDS; TOPNOTCH 2015
B. Beta microglobulin MD), MD
C. transthyretin
D. calcitonin
E. Amyloid precursor protein

320 A 32 y/o male was brought to ER secondary to SIMILAR TO PREVIOUS BOARD EXAM LEAN ANGELO BACK-UP
personality changes. He appears to be CONCEPT/PRINCIPLE. Arboviral encephalitis is SILVERIO, MD (TOP 4 MIDTERM
combative, confused and agitated. 30mins at characterized with neuronophagia and microgial - AUG 2014 MED EXAM - FEB
the ER, patient experienced status epilepticus nodules. Robbins 8th ed p 1302. BOARDS; TOPNOTCH 2015
which despite all efforts he eventually perished. MD), MD
Biopsy of the brain revealed single cell necrosis
with neuronophagia. what is the primary
diagnosis?
A. Herpes simplex encephalitis
B. CMV encephalitis
C. Arboviral encephalitis
D. HIV encephalitis
E. HSV encephalitis
321 True of cancer cachexia: Weight loss in cancer cachexia results equally from a DEBBIE ROSE BACK-UP
A. Weight loss results more from loss of muscle loss of muscle and of fat. It is NOT caused by the TANENGSY, MD (TOP MIDTERM
than of fat. nutritional demands of the tumor. BMR is increased in 5 - AUG 2014 MED EXAM - FEB
B. It is caused by nutritional demands of the patients with cancer. It is suspected that TNF BOARDS; TOPNOTCH 2015
tumor. produced by macrophages mediates cachexia. Robbins MD)
C. It has no satisfactory treatment other than & Cotran Pathologic Basis of Disease 8th edition, p.320
removal of the underlying cause.
D. Basal metabolic rate is decreased in patients
with cancer.
E. It is suspected that Il-4 produced by
macrophages mediates cachexia.
322 A paraneoplastic syndrome characterized by Trosseau phenomenon is a migratory DEBBIE ROSE BACK-UP
gray-black patches of verrucous thrombophlebitis associated with cancer of pancreas TANENGSY, MD (TOP MIDTERM
hyperkeratosis: or lungs. Robbins & Cotran Pathologic Basis of Disease 5 - AUG 2014 MED EXAM - FEB
A. carcinoid syndrome 8th edition, p.321-322 BOARDS; TOPNOTCH 2015
B. trousseau phenomenon MD)
C. dermatomyositis
D. acanthosis nigricans
E. hypertrophic osteoarthropathy

323 Causes of non-immune fetal hydrops, except: Robbins & Cotran Pathologic Basis of Disease 8th DEBBIE ROSE BACK-UP
A. high output heart failure edition, p.461 TANENGSY, MD (TOP MIDTERM
B. turner syndrome 5 - AUG 2014 MED EXAM - FEB
C. parvovirus B19 infection BOARDS; TOPNOTCH 2015
D. maternal Rh isoimmunization MD)
E. CMV infection

TOPNOTCH MEDICAL BOARD PREP PATHOLOGY SUPEREXAM Page 45 of 99


For inquiries visit www.topnotchboardprep.com.ph or email us at topnotchmedicalboardprep@gmail.com
TOPNOTCH MEDICAL BOARD PREP PATHOLOGY SUPEREXAM
For inquiries visit www.topnotchboardprep.com.ph or email us at topnotchmedicalboardprep@gmail.com
Item QUESTION EXPLANATION AUTHOR TOPNOTCH
# EXAM
324 This condition is characterized by a sharply Churg-Strauss syndrome - granulomas with DEBBIE ROSE BACK-UP
segmental acute and chronic vasculitis of geographic patterns of central necrosis and vasculitis. TANENGSY, MD (TOP MIDTERM
medium sized and small arteries, Takayasu arteritis - histologic appearance is 5 - AUG 2014 MED EXAM - FEB
predominantly of the extremities: indistinguishable from temporal arteritis. Polyarteritis BOARDS; TOPNOTCH 2015
A. thromboangiitis obliterans nodosa - segmental transmural necrotizing MD)
B. Churg-Strauss syndrome inflammation of small and medium arteries. Temporal
C. Takayasu arteritis arteritis - nodular intimal thickening reducing luminal
D. polyarteritis nodosa diameter, granulomatous inflammation leading to
E. temporal arteritis elastic lamina fragmentation. Robbins & Cotran
Pathologic Basis of Disease 8th edition, p.512-514
325 The most common primary tumor of the heart: The most frequent primary tumor of the heart in DEBBIE ROSE BACK-UP
A. lipoma children, on the other hand, is rhabdomyosarcoma. TANENGSY, MD (TOP MIDTERM
B. myxoma Robbins & Cotran Pathologic Basis of Disease 8th 5 - AUG 2014 MED EXAM - FEB
C. rhabdomyoma edition, p.584 BOARDS; TOPNOTCH 2015
D. sarcoma MD)
E. papillary fibroelastoma

326 The most specific morphologic finding in Spherocytosis is distinctive but not pathognomonic of DEBBIE ROSE BACK-UP
hereditary spherocytosis: Hereditary Spherocytosis. Robbins & Cotran TANENGSY, MD (TOP MIDTERM
A. reticulocytosis Pathologic Basis of Disease 8th edition, p.643 5 - AUG 2014 MED EXAM - FEB
B. hemosiderosis BOARDS; TOPNOTCH 2015
C. spherocytosis MD)
D. extramedullary hematopoiesis
E. cholelithiasis

327 The most common hereditary disease von Willebrand factor deficiency, on the other hand, is DEBBIE ROSE BACK-UP
associated with life threatening bleeding: the most common inherited bleeding disorder of TANENGSY, MD (TOP MIDTERM
A. von Willebrand factor deficiency humans. Robbins & Cotran Pathologic Basis of Disease 5 - AUG 2014 MED EXAM - FEB
B. hemophilia A 8th edition, p.672 BOARDS; TOPNOTCH 2015
C. hemophilia B MD)
D. Bernard Soulier syndrome
E. Glanzmann Thrombasthenia

328 The major condition/s associated with Airway remodeling is a histologic finding in bronchial DEBBIE ROSE BACK-UP
bronchiectasis: asthma. Robbins & Cotran Pathologic Basis of Disease TANENGSY, MD (TOP MIDTERM
A. obstruction 8th edition, p.692 5 - AUG 2014 MED EXAM - FEB
B. infection BOARDS; TOPNOTCH 2015
C. airway remodeling MD)
D. A & B
E. all of the above

329 True of typhoid fever, except: The oval ulcers in typhoid fever are oriented along the DEBBIE ROSE BACK-UP
A. Humans are its sole reservoir. axis of the ileum. Robbins & Cotran Pathologic Basis of TANENGSY, MD (TOP MIDTERM
B. Gallbladder colonization is associated with Disease 8th edition, p.801 5 - AUG 2014 MED EXAM - FEB
the chornic carrier state. BOARDS; TOPNOTCH 2015
C. Morphologically oval ulcers perpendicular to MD)
the axis of the ileum are seen.
D. Liver, bone marrow, lymph nodes may show
typhoid nodules.
E. Patients with sickle cell disease are
susceptible to Salmonella osteomyelitis.
330 A patient presents with diarrhea, weight loss, PAS(+) macrophages are also seen in Intestinal TB, DEBBIE ROSE BACK-UP
malabsorption, along with arthritis and fever. but it is also usually AFB(+). Robbins & Cotran TANENGSY, MD (TOP MIDTERM
Biopsy of the small intestine reveals Pathologic Basis of Disease 8th edition, p.803 5 - AUG 2014 MED EXAM - FEB
accumulation of distended, foamy macrophages BOARDS; TOPNOTCH 2015
containing PAS(+), diastase resistant granules. MD)
Rod-shaped bacilli can be identified by electron
microscopy. What is the most likely diagnosis?
A. Whipple disease
B. intestinal TB
C. lactase deficiency
D. pseudomembranous colitis
331 The disease that is most likely to give rise to Robbins & Cotran Pathologic Basis of Disease 8th DEBBIE ROSE BACK-UP
hepatocellular carcinoma: edition, p.878 TANENGSY, MD (TOP MIDTERM
A. hepatitis C 5 - AUG 2014 MED EXAM - FEB
B. hepatitis B BOARDS; TOPNOTCH 2015
C. non-alcoholic steatohepatitis MD)
D. hereditary tyrosinemia
E. alpha1-antitrypsin deficiency

332 The most common congenital anomaly of the Pancreas divisum is the failure of fusion of fetal duct DEBBIE ROSE BACK-UP
pancreas: systems of dorsal and ventral pancreatic primordia. TANENGSY, MD (TOP MIDTERM
A. agenesis Robbins & Cotran Pathologic Basis of Disease 8th 5 - AUG 2014 MED EXAM - FEB
B. annular pancreas edition, p.892 BOARDS; TOPNOTCH 2015
C. ectopic pancreas MD)
D. pancreas divisum
333 Most pancreatic cancers arise from: Head - origin of 60% of pancreatic cancer; body - 15%; DEBBIE ROSE BACK-UP
A. pancreatic head tail - 5%. Robbins & Cotran Pathologic Basis of Disease TANENGSY, MD (TOP MIDTERM
B. pancreatic body 8th edition, p.902 5 - AUG 2014 MED EXAM - FEB
C. pancreatic tail BOARDS; TOPNOTCH 2015
D. diffusely involving the entire gland MD)

TOPNOTCH MEDICAL BOARD PREP PATHOLOGY SUPEREXAM Page 46 of 99


For inquiries visit www.topnotchboardprep.com.ph or email us at topnotchmedicalboardprep@gmail.com
TOPNOTCH MEDICAL BOARD PREP PATHOLOGY SUPEREXAM
For inquiries visit www.topnotchboardprep.com.ph or email us at topnotchmedicalboardprep@gmail.com
Item QUESTION EXPLANATION AUTHOR TOPNOTCH
# EXAM
334 Foot process effacement associated with In Minimal change disease, the prinicipal lesion is in DEBBIE ROSE BACK-UP
normal glomeruli by light microscopy makes the visceral epithelial cells which show a uniform and TANENGSY, MD (TOP MIDTERM
this diagnosis: diffuse effacement of foot processes. Robbins & Cotran 5 - AUG 2014 MED EXAM - FEB
A. membranous nephropathy Pathologic Basis of Disease 8th edition, p.925 BOARDS; TOPNOTCH 2015
B. minimal change disease MD)
C. PSGN
D. MPGN
E. Berger disease

335 A 27-year-old female with hypertension was Choice A is seen in males, diabetics, advanced age. DEBBIE ROSE BACK-UP
found to have renal artery stenosis. What is the Choice B in malignant HPN (fibronoid necrosis TANENGSY, MD (TOP MIDTERM
expected morphologic finding on examination? described). Choice C (the answer) refers to 5 - AUG 2014 MED EXAM - FEB
A. a concentrically placed atheromatous plaque fibromuscular dysplasia, ocuring in females, in their BOARDS; TOPNOTCH 2015
with superimposed thrombosis 3rd-4th decades. Choice D describes onion skinning MD)
B. eosinophilic granular change in blood vessel seen in malignant HPN. Robbins & Cotran Pathologic
wall staining (+) for fibrin Basis of Disease 8th edition, p.951
C. fibromuscular thickening involving the media
of artery
D. intimal thickening caused by proliferation of
elongated, concentrically arranged smooth
muscle cells
336 Consistent with benign ulcers, except: SIMILAR TO PREVIOUS BOARD EXAM DEBBIE ROSE BACK-UP
A. oval, sharply punched out defect CONCEPT/PRINCIPLE. Robbins & Cotran Pathologic TANENGSY, MD (TOP MIDTERM
B. mucosal margin with slight overhang from Basis of Disease 8th edition, p.780-781 5 - AUG 2014 MED EXAM - FEB
base BOARDS; TOPNOTCH 2015
C. hemorrhage and fibrin deposition on gastric MD)
serosa
D. heaped up margins
E. thin layer of fibrinoid deposit at base
underlaid by predominantly neutrophilic
inflammatory infiltrate
337 True of ulcerative colitis, except: The earliest lesion in Crohn's disease is an aphthous DEBBIE ROSE BACK-UP
A. normal serosal surface of colon ulcer. Robbins & Cotran Pathologic Basis of Disease TANENGSY, MD (TOP MIDTERM
B. toxic megacolon may complicate 8th edition, p.811-812 5 - AUG 2014 MED EXAM - FEB
C. earliest lesion is an aphthous ulcer BOARDS; TOPNOTCH 2015
D. backwash ileitis MD)
E. absence of granulomas

338 A 21-year-old male presenting with recurrent Robbins & Cotran Pathologic Basis of Disease 8th DEBBIE ROSE BACK-UP
colicky abdominal pain from GI obstruction and edition, p.818 TANENGSY, MD (TOP MIDTERM
transient intussesceptions. On physical 5 - AUG 2014 MED EXAM - FEB
examination, dark blue to brown maculesa are BOARDS; TOPNOTCH 2015
noted around his mouth, eyes, nostrils, buccal MD)
mucosa. What is the most likely diagnosis?
A. Cronkhite Canada syndrome
B. Cowden syndrome
C. Bannayan-Ruvalbaca-Riley syndrome
D. Peutz-Jeghers syndrome
339 The main regulatory factor for iron absorption? Robbins & Cotran Pathologic Basis of Disease 8th DEBBIE ROSE BACK-UP
A. ferritin edition, p.862 TANENGSY, MD (TOP MIDTERM
B. hepcidin 5 - AUG 2014 MED EXAM - FEB
C. transferrin BOARDS; TOPNOTCH 2015
D. serum Fe MD)
E. TIBC

340 What refers to de novo formation of blood Angiogenesis/neovascularization is new vessel DEBBIE ROSE BACK-UP
vessels during embryogenesis? formation in the mature organism. Arteriogenesis TANENGSY, MD (TOP MIDTERM
A. angiogenesis involved remodeling of existing arteries in response to 5 - AUG 2014 MED EXAM - FEB
B. neovascularization chronic changes in pressure and flow. Robbins & BOARDS; TOPNOTCH 2015
C. arteriogenesis Cotran Pathologic Basis of Disease 8th edition, p.489 MD)
D. vasculogenesis
341 A 56 year old hypertensive and diabetic male JESSICA MAE BACK-UP
complained of chest pains and was brought to SANCHEZ, MD (TOP 4 MIDTERM
the emergency room. ECG showed ST segment - AUG 2014 MED EXAM - FEB
elevation and lab work-up showed elevated BOARDS; TOPNOTCH 2015
troponin and CK-MB. What type of necrosis is MD)
expected in the cardiac muscle?
A. Coagulation
B. Liquefactive
C. Gangrenous
D. Enzymatic fat
342 A 50 year old male alcoholic presents with liver This patient presents with signs of chronic liver JESSICA MAE BACK-UP
failure. He has spider angiomata and testicular disease. The pathologic hallmark of chronic liver SANCHEZ, MD (TOP 4 MIDTERM
atrophy. A liver biopsy would reveal which of disease is the presence of fibrosis. - AUG 2014 MED EXAM - FEB
the following? BOARDS; TOPNOTCH 2015
A. Ballooning degeneration of hepatocytes MD)
B. Disorganized liver cells with Councilman
bodies
C. Regenerating hepatic nodules surrounded by
extensive fibrous tissue
D. Severe congestion with centrilobular atrophy

TOPNOTCH MEDICAL BOARD PREP PATHOLOGY SUPEREXAM Page 47 of 99


For inquiries visit www.topnotchboardprep.com.ph or email us at topnotchmedicalboardprep@gmail.com
TOPNOTCH MEDICAL BOARD PREP PATHOLOGY SUPEREXAM
For inquiries visit www.topnotchboardprep.com.ph or email us at topnotchmedicalboardprep@gmail.com
Item QUESTION EXPLANATION AUTHOR TOPNOTCH
# EXAM
343 An 18 year old girl sustained a gaping wound in JESSICA MAE BACK-UP
the thigh which was not sutured. The healed SANCHEZ, MD (TOP 4 MIDTERM
wound showed an elevated scar which did not - AUG 2014 MED EXAM - FEB
go beyond the original margin of the wound. BOARDS; TOPNOTCH 2015
Which of the following abnormalities in repair MD)
occurred in her case?
A. Fibromatosis
B. Deficient wound contraction
C. Failure of collagen maturation
D. Excessive granulation tissue formation
344 A 59 year old woman had loss of consciousness This patient had an ischemic stroke. For unknown JESSICA MAE BACK-UP
that persisted for over an hour. When she reasons, hypoxic death of cells within the CNS often SANCHEZ, MD (TOP 4 MIDTERM
became arousable, she cannot speak nor move manifests as liquefactive necrosis. - AUG 2014 MED EXAM - FEB
her right arm or leg. A cerebral angiogram BOARDS; TOPNOTCH 2015
revealed an occlusion to her left middle Reference: Robbins, Pathologic Basis of Disease, 8th MD)
cerebral artery. Months later, a CT scan shows a ed. p. 15
large 5 cm cystic area in her left parietal lobe
cortex. This CT finding is most likely the
consequence of resolution from which of the
following cellular events?
A. Apoptosis
B. Atrophy
C. Coagulation necrosis
D. Liquefactive necrosis
345 A 15 year old girl had episodes of sneezing with Allergic reactions and atopic diseases, such as allergic JESSICA MAE BACK-UP
watery eyes and runny nose for the past 2 rhinitis, are examples of type I hypersensitivity SANCHEZ, MD (TOP 4 MIDTERM
weeks. On physical examination, she has red, reactions which are mediated by substances released - AUG 2014 MED EXAM - FEB
swollen nasal mucosa. She has had similar from mast cells, such as histamine. BOARDS; TOPNOTCH 2015
episodes in the summer, when the amount of MD)
pollen in the air is high. Her symptoms are most
likely to be mediated by the release of which of
the following chemical mediators?
A. Complement C3b
B. Histamine
C. Platelet activating factor
D. Immunoglobulin G
346 Classic polyarteritis nodosa spares the blood Reference: Topnotch Pathology Handouts JESSICA MAE BACK-UP
vessels in which of the following organs? SANCHEZ, MD (TOP 4 MIDTERM
A. Spleen - AUG 2014 MED EXAM - FEB
B. Kidneys BOARDS; TOPNOTCH 2015
C. Heart MD)
D. Lungs
347 A 12 year old girl was brought to a Diagnosis is Rheumatic Fever. Based on the Jones JESSICA MAE BACK-UP
paediatrician because of joint pains. This was Criteria, fever and joint pains are only minor criteria SANCHEZ, MD (TOP 4 MIDTERM
accompanied by moderate grade fever. On PE, (migratory polyarthritis is a major criterion), and - AUG 2014 MED EXAM - FEB
the paediatrician noted diastolic murmurs and evidence of prior Strep infection, though a required BOARDS; TOPNOTCH 2015
friction rub. The patient had a history of criterion, is not considered as one of the five major MD)
pharyngitis 10 weeks prior to consultation. The criteria.
major criterion seen in this patient that would
help in your diagnosis is: A. Carditis
B. Fever
C. Joint pains
D. Previous history of pharyngitis
348 A 30 year old pedicab driver complains of PE findings suggest consolidation. Choice B is JESSICA MAE BACK-UP
productive cough, fever, and dyspnea. He was consistent with pathologic findings of consolidation. SANCHEZ, MD (TOP 4 MIDTERM
diagnosed with Bacterial Pneumonia. On PE, - AUG 2014 MED EXAM - FEB
dullness on percussion over the left lung field BOARDS; TOPNOTCH 2015
was elicited. What microscopic finding can MD)
explain these signs and symptoms?
A. Interstitial edema and interstitial capillary
vessel congestion
B. Intraalveolar leukocyte accumulation with
red cell exudation
C. Minimal fluid accumulation in pleural
cavities with fibrin deposition
D. Pulmonary cavitary formation with
intracavitary necrotic debris accumulation
349 A 60 year old patient with an 80 pack year The most likely diagnosis in this case is Squamous cell JESSICA MAE BACK-UP
smoking history was seen at the ER because of carcinoma. Keratin pearls are seen in this type of SANCHEZ, MD (TOP 4 MIDTERM
productive cough, weight loss, and an episode cancer. - AUG 2014 MED EXAM - FEB
of hemoptysis. A chest X-ray showed a 6 cm BOARDS; TOPNOTCH 2015
mass on the middle lobe. Bronchoscopy MD)
revealed an endophytic mass almost
obstructing the segmental bronchus. Which
pathology is associated with the most likely
diagnosis for this case?
A. Keratin pearl formation
B. Mucin lakes
C. Psammoma body formation
D. Giant cell formation

TOPNOTCH MEDICAL BOARD PREP PATHOLOGY SUPEREXAM Page 48 of 99


For inquiries visit www.topnotchboardprep.com.ph or email us at topnotchmedicalboardprep@gmail.com
TOPNOTCH MEDICAL BOARD PREP PATHOLOGY SUPEREXAM
For inquiries visit www.topnotchboardprep.com.ph or email us at topnotchmedicalboardprep@gmail.com
Item QUESTION EXPLANATION AUTHOR TOPNOTCH
# EXAM
350 Which of the following is an essential Reference: Robbins, Pathologic Basis of Disease, 8th JESSICA MAE BACK-UP
component in the definition of Barrett ed. p. 770 SANCHEZ, MD (TOP 4 MIDTERM
esophagus? - AUG 2014 MED EXAM - FEB
A. Columnar epithelium BOARDS; TOPNOTCH 2015
B. Intestinal type epithelium with goblet cells MD)
C. Fundic type gastric epithelium
D. Cardiac type gastric epithelium
351 A 5 year old boy with stunted growth has JESSICA MAE BACK-UP
worsening headaches for 2 months. A cranial SANCHEZ, MD (TOP 4 MIDTERM
CT scan reveals a 1.5 cm mass expanding the - AUG 2014 MED EXAM - FEB
sella turcica. The mass is cystic with scattered BOARDS; TOPNOTCH 2015
calcification. Which of the following is the most MD)
likely diagnosis?
A. Neuroblastoma
B. Craniopharyngioma
C. Pituitary adenoma
D. Mature teratoma
352 In benign hypertension, the arterioles of the Onion skinning and fibrinoid necrosis are associated JESSICA MAE BACK-UP
kidney would show: with malignant hypertension. SANCHEZ, MD (TOP 4 MIDTERM
A. Arteriolitis - AUG 2014 MED EXAM - FEB
B. Onion-skinning BOARDS; TOPNOTCH 2015
C. Fibrinoid necrosis MD)
D. Hyaline thickening
353 A 5 year old boy is brought to the paediatrician The most common cause of nephrotic syndrome in JESSICA MAE BACK-UP
because of periorbital and bipedal edema. children is Minimal Change Disease. A renal biopsy SANCHEZ, MD (TOP 4 MIDTERM
Urinalysis showed the following: sugar - would show no changes in light microscopy, but may - AUG 2014 MED EXAM - FEB
negative; protein - 4+; RBC - 0-1/hpf; show effacement of podocytes on electron BOARDS; TOPNOTCH 2015
leucocytes - 0-1/lpf. Blood chemistry shows: microscopy. MD)
albumin - 2g/dL; cholesterol - 1.8 mmol/L.
Light microscopy of a kidney biopsy of this
patient would most likely show:
A. Normocellular glomeruli
B. Glomeruli with endocapillary
hypercellularity
C. Glomeruli with extracapillary
hypercellularity
D. Hypocellular glomeruli
354 A 20 year old female was diagnosed with SLE 5 Wire loop lesions seen in lupus nephritis reflect active JESSICA MAE BACK-UP
years ago. A renal biopsy is performed because disease. SANCHEZ, MD (TOP 4 MIDTERM
she presents now with nephrotic and nephritic - AUG 2014 MED EXAM - FEB
syndromes. Light microscopy of the biopsy Reference: Robbins, Pathologic Basis of Disease, 8th BOARDS; TOPNOTCH 2015
would most likely show: ed. p. 219 MD)
A. Spike and dome appearance
B. Tram-track basement membrane
C. Wire-loop lesions
D. Normocellular glomeruli
355 A 10 year old boy consulted a physician Reference: Robbins, Pathologic Basis of Disease, 8th JESSICA MAE BACK-UP
because of tea coloured urine and puffiness of ed. p. 919 SANCHEZ, MD (TOP 4 MIDTERM
the eyelids, especially noted in the morning. - AUG 2014 MED EXAM - FEB
Blood pressure is 140/90. These symptoms BOARDS; TOPNOTCH 2015
were noted 3 weeks after he had fever and sore MD)
throat. Urinalysis showed: protein - 4+; sugar -
negative; leucocytes - 0-1/hpf; RBC - 10-
20/hpf; red cell cast - 1-2/lpf; granular cast - 0-
1/lpf; waxy cast - 0-1/lpf. Electron microscopy
of this patient’s renal biopsy would most likely
show:
A. Electron dense deposits with fingerprint
appearance
B. Subepithelial humps
C. No electron dense deposits
D. Subendothelial humps and spikes
356 A 5 year old boy was admitted for high grade CSF findings are characteristic of Tuberculous JESSICA MAE BACK-UP
fever with nuchal rigidity on PE. CSF meningitis. Viral meningitis would present with SANCHEZ, MD (TOP 4 MIDTERM
examination revealed moderate pleocytosis moderately elevated protein, and usually normal - AUG 2014 MED EXAM - FEB
with lymphocyte predominance, with increased glucose levels. Bacterial meningitis presents with BOARDS; TOPNOTCH 2015
protein and decreased glucose. What is the neutrophil predominance. MD)
most likely diagnosis?
A. Acute bacterial meningitis
B. Viral meningitis
C. Tuberculous meningitis
D. Subarachnoid hemorrhage
357 A 25 year old woman with recent onset of a Reference: Robbins, Pathologic Basis of Disease, 8th JESSICA MAE BACK-UP
major depressive disorder ingests an entire ed. p. 416 SANCHEZ, MD (TOP 4 MIDTERM
bottle of acetaminophen. She becomes - AUG 2014 MED EXAM - FEB
progressively obtunded over the next 8 hours. BOARDS; TOPNOTCH 2015
Which of the following microscopic findings is MD)
most likely to be present in her liver 3 days
following ingestion?
A. Normal histology
B. Extensive necrosis
C. Severe steatosis
D. Bridging fibrosis

TOPNOTCH MEDICAL BOARD PREP PATHOLOGY SUPEREXAM Page 49 of 99


For inquiries visit www.topnotchboardprep.com.ph or email us at topnotchmedicalboardprep@gmail.com
TOPNOTCH MEDICAL BOARD PREP PATHOLOGY SUPEREXAM
For inquiries visit www.topnotchboardprep.com.ph or email us at topnotchmedicalboardprep@gmail.com
Item QUESTION EXPLANATION AUTHOR TOPNOTCH
# EXAM
358 A 9 year old girl has a firm well-circumscribed Diagnosis is Thyroglossal duct cyst. JESSICA MAE BACK-UP
midline nodule on her neck that moves SANCHEZ, MD (TOP 4 MIDTERM
upwards with protrusion of the tongue. FNAB - AUG 2014 MED EXAM - FEB
shows mucus and benign epithelial cells. Which BOARDS; TOPNOTCH 2015
is a correct statement about her condition? MD)
A. The serum TSH is probably increased
B. This is due to cystic dilatation of an
embryologic remnant
C. This is a very common condition especially
among Asians
D. Papillary carcinoma is never associated with
it
359 A 30 year old motorcycle driver develops JESSICA MAE BACK-UP
polyuria and polydipsia following a motor SANCHEZ, MD (TOP 4 MIDTERM
vehicular accident. His condition is most - AUG 2014 MED EXAM - FEB
probably the result of which of the following? BOARDS; TOPNOTCH 2015
A. Deficiency of vasopressin MD)
B. Excess of growth hormone
C. Deficiency of insulin
D. Excess of PTH
360 Which of the following is usually produced by Acidophilic cells: GH, prolactin JESSICA MAE BACK-UP
an acidophilic pituitary adenoma? Basophilic cells: FSH, LH, ACTH, TSH SANCHEZ, MD (TOP 4 MIDTERM
A. TSH - AUG 2014 MED EXAM - FEB
B. FSH/LH BOARDS; TOPNOTCH 2015
C. Growth Hormone MD)
D. ACTH
361 Which of the following is/are the hallmark/s of There are 8 hallmarks of cancer: Selfsufficiency in MAIRRE JAMES BACK-UP
cancer? growth signals, Insensitivity to growthinhibitory GADDI, MD (TOP 4 - MIDTERM
A. Evasion of apoptosis signals, Altered cellular metabolism, Evasion of AUG 2013 MED EXAM - FEB
B. Metastasis apoptosis, Limitless replicative potential BOARDS; TOPNOTCH 2015
C. Invasion (immortality), Sustained angiogenesis, Ability to MD)
D. Sustained angiogenesis invade and metastasize and Ability to evade the host
E. All of the above immune response. Robbins 9th pg 282-283

362 As the pathologist tasked to autopsy a 10/M There is widespread neuronal degeneration and an MAIRRE JAMES BACK-UP
who recently died due to rabies, you know that inflammatory reaction that is most severe in the GADDI, MD (TOP 4 - MIDTERM
Negri bodies can be found in the? brainstem. The basal ganglia, spinal cord, and dorsal AUG 2013 MED EXAM - FEB
A. Cerebral cortex root ganglia may also be involved. Negri bodies are BOARDS; TOPNOTCH 2015
B. Cerebellum cytoplasmic, round to oval, eosinophilic inclusions MD)
C. Pons found in the pyramidal neurons of the hippocampus
D. Medulla and Purkinje cells of the cerebellum. Robbins 9th pg
E. None of the above 1277

363 Which of the following is TRUE regarding Anti-apoptotic proteins include BCL-2 BCL-xL and MAIRRE JAMES BACK-UP
apoptosis? MCL1 while pro-apoptotic proteins include BAX and GADDI, MD (TOP 4 - MIDTERM
A. The intrinsic pathway is initiated by the BAK. Sensors of cellular stress and damage which are AUG 2013 MED EXAM - FEB
release of cytochrome c into the cytoplasm the regulators between the two groups include BAD BOARDS; TOPNOTCH 2015
leading to activation of caspase 9 BIM BID Puma and Noxa Robbins 9th pg 53-56 MD)
B. Apoptosis is inhibited by anti-apoptotic
proteins such as BAX and BAK
C. Triggering of the death receptor pathway
leads to the activation of caspase 8 and 10
D. A and C
E. All of the above
364 A 60/F who has recurrent fever, fatigue, weight Sarcoidosis is a multisystem disease of unknown MAIRRE JAMES BACK-UP
loss, cough and night sweats came to you for etiology; the diagnostic histopathologic feature is the GADDI, MD (TOP 4 - MIDTERM
consult. You ordered chest xray which showed presence of noncaseating granulomas in various AUG 2013 MED EXAM - FEB
enlarged hilar lymph nodes and multiple 1-2 tissues. Clinically, It may be discovered unexpectedly BOARDS; TOPNOTCH 2015
cm non-cavitating lesions. Biopsy of the lesions on routine chest films as bilateral hilar adenopathy. It MD)
was done showing non-caseating granulomata may presents with shortness of breath, cough, chest
with fibrosis. The most probable diagnosis is: pain, hemoptysis and constitutional signs and
A. Lung carcinoma symptoms (fever, fatigue, weight loss, anorexia, night
B. Pulmonary tuberculosis sweats). Robbins 9th pg 693-694
C. Mycobacterium avium-intracellulare
infection
D. Sarcoidosis
E. Silicosis
365 A 60/M presents with seizures and left sided The histologic appearance of glioblastoma is similar to MAIRRE JAMES BACK-UP
weakness. MRI was done which showed a large anaplastic astrocytoma with the additional features of GADDI, MD (TOP 4 - MIDTERM
heterogenous mass on the right cerebral necrosis and vascular/endothelial cell AUG 2013 MED EXAM - FEB
hemisphere. The patient was then referred to proliferation.Tumor cells collect along the edges of the BOARDS; TOPNOTCH 2015
Dr. Karl F who then proceeded to remove the necrotic regions, producing a histologic pattern MD)
mass as best as he could. As the pathologist, you referred to as pseudo-palisading. Robbins 9th pg
examined sections of the mass and you noted 1308
multiple foci of necrosis, hemorrhage and
pseudo-palisading tumor cells. What is your
diagnosis?
A. Oligodendroglioma
B. Glioblastoma multiforme
C. Anaplastic astrocytoma
D. Ependymoma
E. Pilocytic astrocytoma

TOPNOTCH MEDICAL BOARD PREP PATHOLOGY SUPEREXAM Page 50 of 99


For inquiries visit www.topnotchboardprep.com.ph or email us at topnotchmedicalboardprep@gmail.com
TOPNOTCH MEDICAL BOARD PREP PATHOLOGY SUPEREXAM
For inquiries visit www.topnotchboardprep.com.ph or email us at topnotchmedicalboardprep@gmail.com
Item QUESTION EXPLANATION AUTHOR TOPNOTCH
# EXAM
366 A previously well 60/M presents with sudden Angiodysplasia, a lesion with malformed submucosal MAIRRE JAMES BACK-UP
painless massive LGIB and he was then and mucosal blood vessels, occurs most often in the GADDI, MD (TOP 4 - MIDTERM
immediately brought to a local hospital. In the cecum or right colon and usually presents after the AUG 2013 MED EXAM - FEB
hospital, the bleeding was still massive and sixth decade of life.The lesions are characterized by BOARDS; TOPNOTCH 2015
persistent. Sigmoid colonoscopy was done ectatic nests of tortuous veins, venules, and capillaries. MD)
which failed to reveal the site of bleeding. The The vascular channels may be separated from the
only finding was that there were multiple intestinal lumen by only the vascular wall and limited
outpouchings in the sigmoid colon. Due to the injury may therefore result in significant bleeding.
difficulty in localizing and in controlling the Robbins 9th pg 780-781; Symptoms of diverticular
bleeding, total colectomy was done. As the disease include intermittent cramping, continuous
pathologist evaluating the specimen, you found lower abdominal discomfort, constipation, distention,
multiple outpouchings in the sigmoid and or a sensation of never being able to completely empty
tortuous and ectatic submucosal and mucosal the rectum. Occasionally there may be minimal
vessels in the cecum. What is your diagnosis? chronic or intermittent blood loss, and, rarely, massive
A. Diverticulosis hemorrhage. pg 804
B. Diverticulitis
C. Angiodysplasia
D. Ulcerative colitis
E. Mesenteric ischemia
367 A 45/M was diagnosed to have HNPCC. What is In HNPCC DNA mismatch repair deficiency causes MAIRRE JAMES BACK-UP
the underlying molecular pathology which led mutations to accumulate in microsatellite repeats, a GADDI, MD (TOP 4 - MIDTERM
to the development of the patient's condition? condition referred to as microsatellite instability . AUG 2013 MED EXAM - FEB
A. DNA mismatch repair deficiency Robbins 9th pg 812. Choice B-D refers to the BOARDS; TOPNOTCH 2015
B. APC gene mutation at 5q21 adenoma-carcinoma sequence with the last step being MD)
C. K-RAS gene mutation at 12p12 TP53 17q13 mutation pg 811. Loss-of-function
D. Loss of heterozygosity at 18q21 mutations in the gene STK11 are present in
E. Loss of function of the gene STK11 approximately half of individuals with familial Peutz-
Jeghers syndrome pg 806
368 Which of the following is/are small round blue Other examples include rhabdomyosarcoma, MAIRRE JAMES BACK-UP
cell tumors? medulloblastoma, small cell lung carcinoma, small-cell GADDI, MD (TOP 4 - MIDTERM
A. Neuroblastoma lymphom AUG 2013 MED EXAM - FEB
B. Wilm's Tumor BOARDS; TOPNOTCH 2015
C. Ewing sarcoma MD)
D. A and C
E. All of the above

369 A 35-year-old woman had a firm nodule Leiomyosarcomas are uncommon malignant MAIRRE JAMES BACK-UP
palpable on the dome of the uterus six years neoplasms that are thought to arise from myometrial GADDI, MD (TOP 4 - MIDTERM
ago on routine examination. The nodule has or endometrial stromal precursor cells, rather than AUG 2013 MED EXAM - FEB
slowly increased in size and is now about twice from degenration of leiomyomas. Leiomyomas are BOARDS; TOPNOTCH 2015
the size it was when first discovered. She is sharply circumscribed with low mitotic index in MD)
asymptomatic. She opted to have total contrast to leiomyosarcomas Robbins 9th pg 1020
hysterectomy done. As the pathologist, you
found that the uterus was grossly distorted and
asymmetric. The mass measures 14 x 10 x 12
cm located at the posterior midcorpus. Sections
of the mass showed a whorled white surface
with multiple large areas of necrosis and ill-
defined borders. Which of the following is most
likely?
A. Adenocarcinoma
B. Leiomyosarcoma
C. Adenomyosis
D. Leiomyoma
E. Metastasis
370 Which of the following is/are responsible for Macrophages produce IL-1, 6, 8,12, and TNF alpha; all MAIRRE JAMES BACK-UP
the production IL-1? T cells produce IL-2, 3; TH1 cells produce IFN gamma; GADDI, MD (TOP 4 - MIDTERM
A. Macrophage TH2 cells produce IL-4,5,10 Robbins 9th pg 198 AUG 2013 MED EXAM - FEB
B. TH1 cell BOARDS; TOPNOTCH 2015
C. TH2 cell MD)
D. B cells
E. B and C

371 A 35/M presents with the classic triad of This classic triad or Charcot's neurologic triad is MAIRRE JAMES BACK-UP
scanning speech, intention tremor and associated with multiple sclerosis. MS is a type IV GADDI, MD (TOP 4 - MIDTERM
nystagmus. What type of hypersensitivity hypersensitivity reaction Robbins 9th pg 209 AUG 2013 MED EXAM - FEB
reaction is responsible for the patient's BOARDS; TOPNOTCH 2015
disorder? MD)
A. Type I
B. Type II
C. Type III
D. Type IV
E. The illness described is not caused by a
hypersensitivity reaction
372 A 34/F with lupus successfully gave birth to a Anti-Ro/SS-A and Anti-La/SS-B are associated with MAIRRE JAMES BACK-UP
live baby girl. On PE, the neonate was congenital heart block and neonatal lupus Robbins 9th GADDI, MD (TOP 4 - MIDTERM
bradycardic but otherwise asymptomatic with pg 219 AUG 2013 MED EXAM - FEB
no signs of distress. On ECG, there was BOARDS; TOPNOTCH 2015
prolongation of the PR interval followed by a MD)
dropped beat. What marker/s is/are associated
with this condition?
A. Anti-Ro
B. Anti-CCP
C. Anti-dsDNA
D. Anti-Sm
E. C and D

TOPNOTCH MEDICAL BOARD PREP PATHOLOGY SUPEREXAM Page 51 of 99


For inquiries visit www.topnotchboardprep.com.ph or email us at topnotchmedicalboardprep@gmail.com
TOPNOTCH MEDICAL BOARD PREP PATHOLOGY SUPEREXAM
For inquiries visit www.topnotchboardprep.com.ph or email us at topnotchmedicalboardprep@gmail.com
Item QUESTION EXPLANATION AUTHOR TOPNOTCH
# EXAM
373 C3 convertase splits C3 into two distinct All three pathways of complement activation lead to MAIRRE JAMES BACK-UP
fragments, C3a and C3b. C3 convertase is the formation of an active enzyme called the C3 GADDI, MD (TOP 4 - MIDTERM
formed through which pathway/s? conver- tase, which splits C3 into two functionally AUG 2013 MED EXAM - FEB
A. Classical distinct frag- ments, C3a and C3b. Robbins 9th pg 88 BOARDS; TOPNOTCH 2015
B. Alternative MD)
C. Lectin
D. A and B
E. All of the above

374 A 10/M, diagnosed to have mental retardation, CTG - myotonic dystrophy; GAA - Friedreich ataxia; MAIRRE JAMES BACK-UP
underwent karyotypic analysis. The was no CAG - Huntington disease Robbins 9th pg 168-169 GADDI, MD (TOP 4 - MIDTERM
aneuploidy present but a discontinuity of AUG 2013 MED EXAM - FEB
staining was seen in the long arm of the X BOARDS; TOPNOTCH 2015
chromosome. Expansion of what type of MD)
trinucleotide repeat sequence can be expected
from the patient?
A. CTG
B. CGG
C. GAA
D. CAG
E. CCC
375 What disorder is due to a single gene mutation The mutation results in a glutamine to arginine MAIRRE JAMES BACK-UP
causing resistance to cleavage and inactivation substitution at amino acid residue 506 that renders GADDI, MD (TOP 4 - MIDTERM
by protein C resulting in a hypercoaguable factor V resistant to cleavage and inactivation by AUG 2013 MED EXAM - FEB
state? protein C. Robbins 9th pg 123-124 BOARDS; TOPNOTCH 2015
A. Factor V Leiden MD)
B. Sickle cell anemia
C. Protein C deficiency
D. Protein S deficiency
E. Antithrombin III deficiency
376 A 34/F with recurrent headaches was Acute hemorrhage into an adenoma is associated with MAIRRE JAMES BACK-UP
diagnosed to have a suprasellar tumor 2 clinical evidence of rapid enlargement of the lesion, a GADDI, MD (TOP 4 - MIDTERM
months ago. While walking she suddenly situation appropriately termed pituitary apoplexy. AUG 2013 MED EXAM - FEB
complained of blindness and was subsequently Robbins 9th pg 1075 BOARDS; TOPNOTCH 2015
rushed to the hospital. In the ER she was MD)
vomiting and complained of a severe headache.
What is your diagnosis?
A. Pituitary microadenoma
B. Craniopharyngioma
C. Pituitary apoplexy
D. Pituitary macroadenoma
E. Prolactinoma
377 A 18/M presents with periorbital edema, B - FSGS; C - MCD; D - Alport syndrome Robbins 9th MAIRRE JAMES BACK-UP
proteinuria and hematuria. You suspect MPGN pg 917, 919, 920, 924 GADDI, MD (TOP 4 - MIDTERM
type 1 for this case. If you were to do kidney AUG 2013 MED EXAM - FEB
biopsy, you would expect to find? BOARDS; TOPNOTCH 2015
A. Large and hypercellular glomeruli with MD)
thickened and duplicated GBM
B. Sclerotic segments with collapse of capillary
loops, increase in matrix and segmental
deposition of plasma proteins along the
capillary wall
C. Uniform and diffuse effacement of foot
processes in the visceral epithelial cells
D. GBM shows irregular foci of thickening
alternating with thinning and splitting and
lamination of the lamina densa
E. None of the above
378 Which of the following best describes the Robbins 9th pg 793 Choice A describes C. difficile pg MAIRRE JAMES BACK-UP
pathology of a Rotavirus infection? 791 Choice B describes Shigella pg 788 Choice D GADDI, MD (TOP 4 - MIDTERM
A. Pseudomembrane formation made up of an describes Salmonella pg 789 AUG 2013 MED EXAM - FEB
adherent layer of inflammatory cells BOARDS; TOPNOTCH 2015
B. The mucosa of the left colon is hemorrhagic MD)
and ulcerated, and pseudomembranes may be
present
C. Selective infection and destruction of mature
enterocytes in the small intestine with the villus
surface repopulated by immature secretory
cells
D. Enlargement of Peyer patches in the terminal
ileum with oval ulcers that may perforate
E. None of the above
379 The characteristic histologic finding/s of The other characteristic histologic findings of asthma, MAIRRE JAMES BACK-UP
asthma include/s? collectively called airway remodeling, include GADDI, MD (TOP 4 - MIDTERM
A. Thickening of the airway wall thickening of airway wall, subbasement membrane AUG 2013 MED EXAM - FEB
B. Hypertrophy of the bronchial wall muscle fibrosis (due to deposition of type I and III collagen), BOARDS; TOPNOTCH 2015
C. Increase in the size of the submucosal glands increased vascularity, increase in the size of the MD)
and number of airway goblet cells submucosal glands and number of airway goblet cells
D. A and B and hypertrophy and/or hyperplasia of the bronchial
E. All of the above wall muscle Robbins 9th pg 682

TOPNOTCH MEDICAL BOARD PREP PATHOLOGY SUPEREXAM Page 52 of 99


For inquiries visit www.topnotchboardprep.com.ph or email us at topnotchmedicalboardprep@gmail.com
TOPNOTCH MEDICAL BOARD PREP PATHOLOGY SUPEREXAM
For inquiries visit www.topnotchboardprep.com.ph or email us at topnotchmedicalboardprep@gmail.com
Item QUESTION EXPLANATION AUTHOR TOPNOTCH
# EXAM
380 Choristomas are: Choice A refers to hamartomas Robbins 9th pg 473 MAIRRE JAMES BACK-UP
A. Excessive focal overgrowths of cells and Choice B - choriocarcinoma Choice C - teratoma GADDI, MD (TOP 4 - MIDTERM
tissues native to the organ where it occurs AUG 2013 MED EXAM - FEB
B. Malignant germ cell tumors composed of BOARDS; TOPNOTCH 2015
syncitiotrophoblasts and cytotrophoblasts MD)
C. Helter-skelter collection of differentiated
cells or organoid structures (neural tissue,
islands of cartilage, etc) embedded in a myxoid
stroma
D. Collection of normal cells or tissues in
abnormal locations
E. None of the above
381 What is the most common site of metastasis of SCOTT RILEY ONG, BACK-UP
lung cancers? MD (TOP 5 - AUG MIDTERM
A. Bone 2014 MED BOARDS; EXAM - FEB
B. Liver TOPNOTCH MD) 2015
C. Adrenal
D. Brain
E. Kidney

382 The pathognomonic inclusion bodies in nerve SIMILAR TO PREVIOUS BOARD EXAM SCOTT RILEY ONG, BACK-UP
cells infected by the rabies virus are most CONCEPT/PRINCIPLE. The inclusion bodies refer to MD (TOP 5 - AUG MIDTERM
commonly found in which part of the CNS? Negri bodies, which are most commonly seen in the 2014 MED BOARDS; EXAM - FEB
A. Cerebral cortex hippocampus. Cerebellum is the 2nd most common TOPNOTCH MD) 2015
B. Hippocampus site.
C. Basal ganglia
D. Cerebellum
E. Medulla oblongata

383 In which of the following conditions is fibrinous Caseous and adhesive pericarditis are the forms SCOTT RILEY ONG, BACK-UP
pericarditis least likely to occur? associated with TB. MD (TOP 5 - AUG MIDTERM
A. Dressler syndrome 2014 MED BOARDS; EXAM - FEB
B. Tuberculosis TOPNOTCH MD) 2015
C. Systemic lupus erythematosus
D. Rheumatic heart disease
E. None of the above

384 Which of the following laboratory findings will SCOTT RILEY ONG, BACK-UP
you expect in patients with hemophilia A? MD (TOP 5 - AUG MIDTERM
A. Prolonged prothrombin time 2014 MED BOARDS; EXAM - FEB
B. Prolonhged partial thromboplastin time TOPNOTCH MD) 2015
C. Decreased platelet count
D. Prolonged bleeding time
E. Both A and D

385 ABO blood type incompatibility is an example SCOTT RILEY ONG, BACK-UP
of which hypersensitivity reaction? MD (TOP 5 - AUG MIDTERM
A. Type I hypersensitivity reaction 2014 MED BOARDS; EXAM - FEB
B. Type II hypersensitivity reaction TOPNOTCH MD) 2015
C. Type III hypersensitivity reaction
D. Type IV hypersensitivity reaction
E. None of the above

386 The following laboratory findings are SIMILAR TO PREVIOUS BOARD EXAM SCOTT RILEY ONG, BACK-UP
consistent with iron-deficiency anemia except: CONCEPT/PRINCIPLE. MD (TOP 5 - AUG MIDTERM
A. Hypochromic RBCs 2014 MED BOARDS; EXAM - FEB
B. Decreased MCV TOPNOTCH MD) 2015
C. Increased RDW
D. Anisocytosis
E. Decreased MCHC

387 Which of the following is not a small-vessel Polyarteritis nodosa is classically a medium-vessel SCOTT RILEY ONG, BACK-UP
vasculitides? disease. MD (TOP 5 - AUG MIDTERM
A. Polyarteritis nodosa 2014 MED BOARDS; EXAM - FEB
B. Henoch-Schonlein purpura TOPNOTCH MD) 2015
C. Churg-Strauss syndrome
D. Wegener granulomatosis
E. None of the above

388 A renal biopsy that shows hypercellular SCOTT RILEY ONG, BACK-UP
glomeruli on light microscopy, "starry sky" MD (TOP 5 - AUG MIDTERM
pattern of immunofluorescence and 2014 MED BOARDS; EXAM - FEB
subepithelial immune complex humps on TOPNOTCH MD) 2015
electron microscopy is most consistent with
which of the following diagnosis?
A. Rapidly progressive glomerulonephritis
B. Acute poststreptococcal glomerulonephritis
C. IgA nephropathy
D. Membranoproliferative glomerulonephritis
E. Lupus nephritis
389 Which of the following describes the most A: type I. B: type II. C: type III. D: type IV. E: type V SCOTT RILEY ONG, BACK-UP
common type of choledochal cyst? (Caroli disease) MD (TOP 5 - AUG MIDTERM
A. Fusiform dilatation of the common bile duct 2014 MED BOARDS; EXAM - FEB
B. Diverticulum arising from the common bile TOPNOTCH MD) 2015
duct and attached to it by a narrow stalk
C. Focal dilatation of the intraduodenal portion
of the common bile duct
D. Multiple saccular dilatations of the intra- and
TOPNOTCH MEDICAL BOARD PREP PATHOLOGY SUPEREXAM Page 53 of 99
For inquiries visit www.topnotchboardprep.com.ph or email us at topnotchmedicalboardprep@gmail.com
TOPNOTCH MEDICAL BOARD PREP PATHOLOGY SUPEREXAM
For inquiries visit www.topnotchboardprep.com.ph or email us at topnotchmedicalboardprep@gmail.com
Item QUESTION EXPLANATION AUTHOR TOPNOTCH
# EXAM
extra-hepatic bile ducts
E. Saccular dilatations of the intrahepatic bile
ducts without biliary obstruction

390 What is the most common clinical presentation SCOTT RILEY ONG, BACK-UP
of multiple sclerosis? MD (TOP 5 - AUG MIDTERM
A. Nystagmus 2014 MED BOARDS; EXAM - FEB
B. Motor weakness TOPNOTCH MD) 2015
C. Optic neuritis
D. Intention tremor
E. Dementia

391 Which of the following features would make The other choices are more characteristic of Crohn SCOTT RILEY ONG, BACK-UP
you favor a diagnosis of ulcerative colitis over disease. MD (TOP 5 - AUG MIDTERM
Crohn disease? 2014 MED BOARDS; EXAM - FEB
A. Pseudopolyp formation TOPNOTCH MD) 2015
B. Fistula formation
C. Non-caseating granuloma
D. Paneth cell metaplasia
E. Aphthous ulcers

392 Which of the following features would make SIMILAR TO PREVIOUS BOARD EXAM SCOTT RILEY ONG, BACK-UP
you suspect that an ulcer is malignant? CONCEPT/PRINCIPLE. Option B is a classic feature of a MD (TOP 5 - AUG MIDTERM
A. Edematous ulcer collar with overhanging malignant ulcer. In barium studies of the upper GI 2014 MED BOARDS; EXAM - FEB
mucosal edges tract, this will present as the "Carmen meniscus sign". TOPNOTCH MD) 2015
B. Flat-based ulcer with heaped up edges The other options describe a benign ulcer.
C. Radiating folds extending into the crater of
the ulcer
D. Depth of the ulcer is greater than its width
E. Smooth ulcer mound with tapering edges
393 Which of the following viral hepatitis infection Hepatitis C is associated with 50% chronicity but not SCOTT RILEY ONG, BACK-UP
is least associated with fulminant hepatitis? with fulminant hepatitis. Hepatitis D can lead to MD (TOP 5 - AUG MIDTERM
A. Hepatitis B fulminant hepatitis in the setting of co-infection with 2014 MED BOARDS; EXAM - FEB
B. Hepatitis C Hep B. Hepatitis E is associated with fulminant TOPNOTCH MD) 2015
C. Hepatitis D hepatitis among pregnant women.
D. Hepatitis E
E. None of the above

394 Among patients with chronic viral hepatitis SIMILAR TO PREVIOUS BOARD EXAM SCOTT RILEY ONG, BACK-UP
infection who develop liver cirrhosis, what CONCEPT/PRINCIPLE. MD (TOP 5 - AUG MIDTERM
percentage will continue to progress into 2014 MED BOARDS; EXAM - FEB
hepatocellular carcinoma? TOPNOTCH MD) 2015
A. <2%
B. 5%
C. 6-15%
D. 12-20%
E. 50%
395 Which of the following is the most important SIMILAR TO PREVIOUS BOARD EXAM SCOTT RILEY ONG, BACK-UP
prognosticating factor in cases of breast CONCEPT/PRINCIPLE MD (TOP 5 - AUG MIDTERM
carcinoma? 2014 MED BOARDS; EXAM - FEB
A. HER-2/neu status TOPNOTCH MD) 2015
B. Family history
C. Tumor histology
D. Age
E. Axillary node status

396 Which of the following is the most common SCOTT RILEY ONG, BACK-UP
benign neoplasm of the liver? MD (TOP 5 - AUG MIDTERM
A. Hepatic adenoma 2014 MED BOARDS; EXAM - FEB
B. Cavernous hemangioma TOPNOTCH MD) 2015
C. Focal nodular hyperplasia
D. Lipoma
E. Biliary cystadenoma

397 The presence of a hyperpigmented, ulcerating SIMILAR TO PREVIOUS BOARD EXAM SCOTT RILEY ONG, BACK-UP
plaque with irregular borders and asymmetric CONCEPT/PRINCIPLE. Acral lentiginous melanoma, MD (TOP 5 - AUG MIDTERM
shape in which of the following body parts which occurs on the palms and soles and beneath the 2014 MED BOARDS; EXAM - FEB
should make you worry most? nails, has the worst prognosis among the different TOPNOTCH MD) 2015
A. Face types of melanoma.
B. Nape
C. Chest
D. Palms
E. Back

TOPNOTCH MEDICAL BOARD PREP PATHOLOGY SUPEREXAM Page 54 of 99


For inquiries visit www.topnotchboardprep.com.ph or email us at topnotchmedicalboardprep@gmail.com
TOPNOTCH MEDICAL BOARD PREP PATHOLOGY SUPEREXAM
For inquiries visit www.topnotchboardprep.com.ph or email us at topnotchmedicalboardprep@gmail.com
Item QUESTION EXPLANATION AUTHOR TOPNOTCH
# EXAM
398 A 32-year old female presented with symptoms SIMILAR TO PREVIOUS BOARD EXAM SCOTT RILEY ONG, BACK-UP
of fatigue, weight gain, cold intolerance, CONCEPT/PRINCIPLE. Lymphocytic thyroiditis also MD (TOP 5 - AUG MIDTERM
constipation and thinning of her hair. On present with painless enlargement of the thyroid 2014 MED BOARDS; EXAM - FEB
physical examination, you noted a firm, mildly gland, but it does not exhibit Hurthle cells on biopsy. TOPNOTCH MD) 2015
enlarged but painless thyroid gland. Biopsy Subacute thyroiditis presents with painful thryoid
revealed diffuse inflammatory infiltrates, enlargement.
atrophy of the thyroid follicles and
characteristic Hurthle cells. What is your most
likely diagnosis?
A. Hashimoto thyroiditis
B. Lymphocytic thyroiditis
C. Subacute thyroiditis
D. Graves disease
E. Reidel thyroiditis
399 A history of bladder exstrophy is a risk factor Schistosoma infection --> Squamous cell CA. Smoking - SCOTT RILEY ONG, BACK-UP
for the development of which type of bladder -> Transitional cell CA MD (TOP 5 - AUG MIDTERM
cancer? 2014 MED BOARDS; EXAM - FEB
A. Transitional cell carcinoma TOPNOTCH MD) 2015
B. Squamous cell carcinoma
C. Adenocarcinoma
D. Clear cell carcinoma
E. Any of the above

400 Which of the following conditions is SCOTT RILEY ONG, BACK-UP


characterized by findings of giant rugal MD (TOP 5 - AUG MIDTERM
hypertrophy, thickened gastric mucosa, 2014 MED BOARDS; EXAM - FEB
excessive mucus production, hypoproteinemia TOPNOTCH MD) 2015
and hypochlorydia?
A. Non-tropical sprue
B. Eosinophilic gastritis
C. Crohn gastritis
D. Menetrier disease
E. Zollinger-Ellison syndrome
401 A patient with chronic back pain came to you Chronic NSAID use presents with Papillary necrosis on JOSE CARLO DIAGNOSTIC
due to oliguria and had been abusing NSAIDs Renal biopsy. SIMILAR TO PREVIOUS BOARD EXAM MASANGKAY III, MD EXAM - AUG
chronically for the past years, if renal biopsy CONCEPT/PRINCIPLE (TOP 8 - FEB 2014 2014
will be made, which of the following would be MED BOARDS;
seen? TOPNOTCH MD)
A. Acute Tubular Necrosis
B. Papillary Necrosis
C. Effaced foot processes
D. Coagulative Necrosis of the nephrons
E. Nodular Sclerosis
402 A 3 year-old patient was brought to you by his This is a case of Wilm's Tumor, it is the most common JOSE CARLO DIAGNOSTIC
mother due to an abdominal mass which she renal tumor in childhood, it does not cross the midline MASANGKAY III, MD EXAM - AUG
palpated while bathing the child with a urine and mostly involves bilateral kidneys, biopsy will (TOP 8 - FEB 2014 2014
described as "iced-tea-like", upon your present with Triphasic combinations of blastema;, MED BOARDS;
thorough PE you noticed that the mass does not stromal and epithelial cell combinations. TOPNOTCH MD)
cross the midline and involves a synchronous
tumor. Your primary consideration would most
likely reveal which of the following biopsy
findings?
A. Varying amounts of immature
neuroepithelium, cartilage, bone, muslce and
others
B. Flexner-Wintersteiner Rosettes
C. Homer-Wright Pseudorosettes
D. Blastemal, stromal and epithelial cell
combinations
E. glomerulus-like structures composed of a
central blood vessel enveloped by germ cells
403 An elderly patient with a pulmonary mass and Squamous Cell CA will present with paraneoplastic JOSE CARLO DIAGNOSTIC
hypercalcemia underwent right pulmonectomy, hypercalcemia and is usually central in location. MASANGKAY III, MD EXAM - AUG
upon biopsy the pathologist noted the mass to SIMILAR TO PREVIOUS BOARD EXAM (TOP 8 - FEB 2014 2014
have grown from the alveolar duct and alveolar CONCEPT/PRINCIPLE MED BOARDS;
sacs, what would be the primary consideration? TOPNOTCH MD)
A. Small Cell Lung CA
B. Large Cell Lung CA
C. Squamous Cell Lung CA
D. Lung Adenocarcinoma
E. Metastatic Lung CA
404 A patient with Small Cell Lung cancer Lung cancers may metastasize using lymphatic or JOSE CARLO DIAGNOSTIC
underwent PET scan, a metastasis was noted, hematogenous routes to the Adrenals (50%), Liver MASANGKAY III, MD EXAM - AUG
which of the following sites would be the most (30-50%), Brain (20%), Bone (20%) (TOP 8 - FEB 2014 2014
common site of metastasis? MED BOARDS;
A. Bone (Ribs) TOPNOTCH MD)
B. Liver
C. Brain
D. Adrenals
E. Heart

TOPNOTCH MEDICAL BOARD PREP PATHOLOGY SUPEREXAM Page 55 of 99


For inquiries visit www.topnotchboardprep.com.ph or email us at topnotchmedicalboardprep@gmail.com
TOPNOTCH MEDICAL BOARD PREP PATHOLOGY SUPEREXAM
For inquiries visit www.topnotchboardprep.com.ph or email us at topnotchmedicalboardprep@gmail.com
Item QUESTION EXPLANATION AUTHOR TOPNOTCH
# EXAM
405 A 45-year old male patient with recurrent SIMILAR TO PREVIOUS BOARD EXAM JOSE CARLO DIAGNOSTIC
respiratory infections came to your office with CONCEPT/PRINCIPLE MASANGKAY III, MD EXAM - AUG
a peripheral smear result with noted Sea-blue (TOP 8 - FEB 2014 2014
histiocytes by the pathologist, what would be MED BOARDS;
your primary consideration? TOPNOTCH MD)
A. AML
B. ALL
C. CLL
D. CML
E. Hairy Cell Leukemia
406 A walk-in patient with multiple purpuric rashes vWD will present with a prolonged bleeding time due JOSE CARLO DIAGNOSTIC
came to your clinic asking for your to inadequate adhesion of platelets to the severed MASANGKAY III, MD EXAM - AUG
interpretation of his self-requested lab results: epithelium and a prolonged PTT due to the lack of (TOP 8 - FEB 2014 2014
Platelet Count 245,000/L and INR of 1.00 Factor 8 from vWF. MED BOARDS;
Bleeding Time was 15 mins, PTT was TOPNOTCH MD)
prolonged. What would be your impression?
A. TTP
B. Bernard-Soulier Syndrome
C. Glanzmann's Thrombasthenia
D. von Willebrand Disease
E. Hemophilia
407 A 78 year old male patient with a parotid mass Papillary Cystadenoma Lymphomatosum or Warthin's JOSE CARLO DIAGNOSTIC
and a chronic smoking history underwent tumor is the second most common tumor of the MASANGKAY III, MD EXAM - AUG
parotidectomy with findings of follicular parotid gland, occurs virtually in the parotid gland (TOP 8 - FEB 2014 2014
germinal centers and cystic spaces on biopsy, only and is classically seen in elderly patients with a MED BOARDS;
but because of your thirst for knowledge you chronic smoking history. TOPNOTCH MD)
read about it and you knew that this tumor is
virtually restricted to the parotid gland only
and is the second most common salivary
neoplasm. What is this tumor?
A. Pleomprphic Adenoma
B. Adenoid Cystic Adenoma
C. Papillary Cystadenoma Lymphomatosum
D. Mucoepidermoid adenoma
E. Parotid Metastasis from a Lung CA
408 Seborrheic Keratoses in a patient with Gastric JOSE CARLO DIAGNOSTIC
Adenocarcinoma is termed as what sign? MASANGKAY III, MD EXAM - AUG
A. Breslow's Sign (TOP 8 - FEB 2014 2014
B. Lesser-Trelat Sign MED BOARDS;
C. Charcot-Marie-Tooth Sign TOPNOTCH MD)
D. Ormond Sign
E. Zellballen Sign

409 A 34 year old man diagnosed as having type II The reason for this is that with decreased levels of JOSE CARLO DIAGNOSTIC
diabetes mellitus. Laboratory evaluation of his insulin with diabetes mellitus there is increased MASANGKAY III, MD EXAM - AUG
serum also finds hypertriglyceridemia, which is mobilization of free fatty acids from adipose tissue (TOP 8 - FEB 2014 2014
due to his diabetes. The most common type of (increased lipolysis). This increases delivery of free MED BOARDS;
secondary hyperlipidemia associated fatty acids to the liver, which increases production and TOPNOTCH MD)
with diabetes mellitus is characterized by secretion of VLDL by the liver.
elevated serum levels of which one of the
following substances?

A. Chylomicrons
B. High-density lipoproteins
C. Intermediate-density lipoproteins
D. Low-density lipoproteins
E. Very-low-density lipoproteins
410 Which of the following cytokines is secreted by IL1 and TNF-alpha are secreted by Macrophages, IL2 JOSE CARLO DIAGNOSTIC
macrophages and functions as a major Tcells, IL3 BM stem cells, IL4 stimulates IgE, IL5 MASANGKAY III, MD EXAM - AUG
mediator of acute inflammation by stimulating stimulates IgA (TOP 8 - FEB 2014 2014
acute phase reactions with increasing vascular MED BOARDS;
permeability and stimulating fibroblasts? TOPNOTCH MD)

A. Interleukin-1
B. Interleukin-2
C. Interleukin-3
D. Interleukin-4
E. Interleukin-5
411 You encountered a 35-year old patient with This is a classic case of Wegener's Granulomatosis, a JOSE CARLO DIAGNOSTIC
necrotizing granulomas along the respiratory small vessel vasculitis, positive for c-ANCA. MASANGKAY III, MD EXAM - AUG
tract and the lungs. He eventually died of (TOP 8 - FEB 2014 2014
crescentic glomerulonephritis. Which of the MED BOARDS;
following markers would turn out positive in TOPNOTCH MD)
this patient?
A. c-ANCA
B. p-ANCA
C. Anti-centromere
D. ANA
E. Anti-Smith

TOPNOTCH MEDICAL BOARD PREP PATHOLOGY SUPEREXAM Page 56 of 99


For inquiries visit www.topnotchboardprep.com.ph or email us at topnotchmedicalboardprep@gmail.com
TOPNOTCH MEDICAL BOARD PREP PATHOLOGY SUPEREXAM
For inquiries visit www.topnotchboardprep.com.ph or email us at topnotchmedicalboardprep@gmail.com
Item QUESTION EXPLANATION AUTHOR TOPNOTCH
# EXAM
412 Trinucleotide repeat Mutations are Fragile X Syndrome is the 2nd most common cause of JOSE CARLO DIAGNOSTIC
amplifications of a sequence of three mental retardation next to Down's Syndrom, classic MASANGKAY III, MD EXAM - AUG
nucleotides, they also are associated with an finding is macroorchidism (TOP 8 - FEB 2014 2014
Anticipation-type of inheritance, An example is MED BOARDS;
the second most common cause of Mental TOPNOTCH MD)
retardation with a prominent PE finding of
Macroorchidism. What Nucleotide repeat will
be seen in this patient?
A. CAG
B. GAA
C. CTG
D. CGG
E. GCA
413 A 1 year old patient was brought to you by his This is a case of Down Syndrome/ Trisomy 21, 95% of JOSE CARLO DIAGNOSTIC
mother due to noticeable developmental delay, cases are due to nondisjunction of chromosomes on MASANGKAY III, MD EXAM - AUG
you noticed that the patient has specific facies cell division causing 3 chromosome 21. (TOP 8 - FEB 2014 2014
like flat facial profile, epicanthal folds, MED BOARDS;
macroglossia and a prominent Simian Crease. TOPNOTCH MD)
What is the most common chromosomal
structural abnormality seen in this type of
congenital disorder.
A. Nondisjunction
B. Mosaicism
C. Deletion
D. Translocation
E. Transversion
414 A patient came to your office with a chief Contact Dermatitis is a Type IV/Delayed JOSE CARLO DIAGNOSTIC
complaint of erythema and intense pruritus on Hypersensitivity Reaction MASANGKAY III, MD EXAM - AUG
both hands after wearing Latex gloves. What (TOP 8 - FEB 2014 2014
type of Hypersensitivity reaction is this patient MED BOARDS;
having? TOPNOTCH MD)
A. Anaphylactic
B. Cytotoxic
C. Immune Complex
D. Delayed
E. None of the Above
415 An AIDS patient was diagnosed to have JOSE CARLO DIAGNOSTIC
Pneumocystis jiroveci Pneumonia, What is MASANGKAY III, MD EXAM - AUG
probably the patient's CD4 count? (TOP 8 - FEB 2014 2014
A. <500 MED BOARDS;
B. <300 TOPNOTCH MD)
C. <200
D. <100
E. <50

416 What is the most common type of congenital Endocardial Cushion defect and ASD primum type are JOSE CARLO DIAGNOSTIC
heart defect seen in a patient with Down the most common CHD in Down Syndrome patients. MASANGKAY III, MD EXAM - AUG
Syndrome? (TOP 8 - FEB 2014 2014
A. Total Anomalous Pulmonary Venous MED BOARDS;
Return TOPNOTCH MD)
B. Endocardial Cushion Defect
C. Transposition of Great Arteries
D. Atrial Septal Defect, Secundum Type
E. None of the above
417 A 50 year old patient died of Myocardial the histologic description is that of a Coagulative type JOSE CARLO DIAGNOSTIC
Infarction, upon Autopsy/Histologic studies of of necrosis, In the cardiac muscles, this is usually MASANGKAY III, MD EXAM - AUG
his cardiac muscles there was noted evident as early as 4 hours (4-12 hours) after the (TOP 8 - FEB 2014 2014
preservation of architecture of dead tissues, onset of MI. MED BOARDS;
noted as Ghost cells. If the MI occurred at 1:00 TOPNOTCH MD)
PM what is the earliest possible time of the
autopsy?
A. 1:30 PM
B. 2:00 PM
C. 3:30 PM
D. 5:00 PM
E. 6:30 PM
418 Dilated Cardiomyopathy maybe due to the Friedrich's Ataxia may cause a restrictive type of JOSE CARLO DIAGNOSTIC
following, EXCEPT: cardiomyopathy MASANGKAY III, MD EXAM - AUG
A. Pregnancy (TOP 8 - FEB 2014 2014
B. Friedrich's Ataxia MED BOARDS;
C. Chaga's Disease TOPNOTCH MD)
D. Doxorubicin
E. Coxsackie B Myocarditis

419 An autopsy of a patient who died of Libman- LSE is seen in SLE. B:NBTE, C:IE, D:RHD JOSE CARLO DIAGNOSTIC
Sacks Endocarditis will have lesions on the MASANGKAY III, MD EXAM - AUG
heart grossly described as which of the (TOP 8 - FEB 2014 2014
following vegetation descriptions: MED BOARDS;
A. Small or medium sized vegetations on both TOPNOTCH MD)
sides of the valve leaflets
B. Small bland vegetations, usually attached at
the line of closure
C. Large irregular masses on the valve cusps
that extend into the chordae
D. small warty vegetations along the lines of
closure of the valve leaflets

TOPNOTCH MEDICAL BOARD PREP PATHOLOGY SUPEREXAM Page 57 of 99


For inquiries visit www.topnotchboardprep.com.ph or email us at topnotchmedicalboardprep@gmail.com
TOPNOTCH MEDICAL BOARD PREP PATHOLOGY SUPEREXAM
For inquiries visit www.topnotchboardprep.com.ph or email us at topnotchmedicalboardprep@gmail.com
Item QUESTION EXPLANATION AUTHOR TOPNOTCH
# EXAM
E. Large, multiple, regular masses on the valve
cusps that extend into the chordae

420 A patient working for an Aircraft Industry was SIMILAR TO PREVIOUS BOARD EXAM JOSE CARLO DIAGNOSTIC
diagnosed to have Lung Carcinoma, what CONCEPT/PRINCIPLE MASANGKAY III, MD EXAM - AUG
substance may be responsible for this patient's (TOP 8 - FEB 2014 2014
morbidity? MED BOARDS;
A. Silicon TOPNOTCH MD)
B. Cadmium
C. Arsenic
D. Vinyl Chloride
E. Beryllium
421 A 52-year old female came for consult due to on Here we have the typical findings in leukemia - WEBSTER ALINDOG, MIDTERM 1
and off fever, easy bruising and easy recurrent infection/fever (defective immune cells), MD (TOP 3 - FEB EXAM - AUG
fatigability. Significant in P.E. were pallor, pallor and other signs of anemia, and 2014 MED BOARDS; 2014
wasting and presence of hepatosplenomegaly. bruising/bleeding (thrombocytopenia). Sea-blue TOPNOTCH MD)
CBC showed WBC count of >100,000/mm3 and histiocytes (clincher) are found in CML and in other
thrombocytopenia; sea-blue histiocytes were myelodysplastic syndromes (MDS). These cells have
also noted in her bone marrow. The most likely wrinkled, green blue cytoplasm hence the name.
diagnosis for this patient is:
A. Acute promyelocytic leukemia
B. Chronic myelogenous leukemia
C. Histiocytosis X
D. Non-Hodgkin lymphoma
422 Which segment of the respiratory tree is In centriacinar/centrilobular emphysema, the central WEBSTER ALINDOG, MIDTERM 1
primarily affected in centriacinar emphysema or prximal parts of the acini are affected, whereas the MD (TOP 3 - FEB EXAM - AUG
(smoking-related)? distal parts are spared. These are more common and 2014 MED BOARDS; 2014
A. Terminal bronchioles usually more severe in the upper lobes, particularly in TOPNOTCH MD)
B. Respiratory bronchioles the apical segments. In contrast, panacinar
C. Alveolar ducts emphysema involves segments from the respiratory
D. Alveolar sacs bronchiole to the terminal blind alveoli. It tends to
occur more commonly in the lower zones and most
severe at the bases.
423 Achalasia is characterized by: WEBSTER ALINDOG, MIDTERM 1
A. Increased resting tone of LES MD (TOP 3 - FEB EXAM - AUG
B. Lack of esophageal peristalsis 2014 MED BOARDS; 2014
C. Incomplete LES relaxation during TOPNOTCH MD)
deglutition
D. All of the above

424 What is the most frequent type of WEBSTER ALINDOG, MIDTERM 1


hyperfunctioning pituitary adenoma? MD (TOP 3 - FEB EXAM - AUG
A. Gonadotroph adenoma 2014 MED BOARDS; 2014
B. Somatotroph adenoma TOPNOTCH MD)
C. Lactotroph adenoma
D. Corticotroph adenoma

425 A 49-year old hypertensive male is noted to be This is a case of renal artery stenosis and occlusion by WEBSTER ALINDOG, MIDTERM 1
refractory to most oral medications save for an atheromatous plaque is the most common cause MD (TOP 3 - FEB EXAM - AUG
ACE inhibitors. Upon examination, a bruit is (70%, Robbins). This pathology is most common in 2014 MED BOARDS; 2014
heard on auscultation of his kidneys. Elevated elderly males. Although fibromuscular dysplasia can TOPNOTCH MD)
plasma renin was also determined. If we are to also cause renal artery stenosis, we find it less
biopsy the kidneys of this patient we are most appropriate in this case because it is frequent in
likely to identify: younger females. Again, the rule is to choose the best
A. Occlusion of renal artery by an answer. =)
atheromatous plaque
B. Fibromuscular dysplasia of renal artery
C. Adventitial hyperplasia of renal artery
D. Endotheliolysis and vasculogenesis
426 Nodular hyperplasia of the prostate, a common And hence the obstructive symptoms of BPH. Prostatic WEBSTER ALINDOG, MIDTERM 1
disorder of men above 50, arises most tumors, on the other hand, tend to originate from the MD (TOP 3 - FEB EXAM - AUG
frequently and almost exclusively in which part periphery. 2014 MED BOARDS; 2014
of the gland? TOPNOTCH MD)
A. Peripheral zone
B. Posterior aspect
C. Transition zone
D. Any portion of the prostate

427 This is a rare clinical syndrome with major WEBSTER ALINDOG, MIDTERM 1
features of adenomatous colonic polyposis and MD (TOP 3 - FEB EXAM - AUG
CNS tumors, including medulloblastomas and 2014 MED BOARDS; 2014
gliobastomas: TOPNOTCH MD)
A. Gardner syndrome
B. HNPCC syndrome
C. Crohn syndrome
D. Turcot syndrome

TOPNOTCH MEDICAL BOARD PREP PATHOLOGY SUPEREXAM Page 58 of 99


For inquiries visit www.topnotchboardprep.com.ph or email us at topnotchmedicalboardprep@gmail.com
TOPNOTCH MEDICAL BOARD PREP PATHOLOGY SUPEREXAM
For inquiries visit www.topnotchboardprep.com.ph or email us at topnotchmedicalboardprep@gmail.com
Item QUESTION EXPLANATION AUTHOR TOPNOTCH
# EXAM
428 A 55-year old chronic alcoholic male exhibiting This is a case of liver cirrhosis. It has 3 defining WEBSTER ALINDOG, MIDTERM 1
ascites, splenomegaly, periumbilical caput characteristics: 1. bridging fibrous septae; 2. MD (TOP 3 - FEB EXAM - AUG
medusae and skin spider angiomata was parenchymal nodules; and 3. disruption of the 2014 MED BOARDS; 2014
admitted due to anorexia and severe architecture of the entire liver. The vascular TOPNOTCH MD)
malnutrition. Hepatic pathology was reorganization and disruption of the normal structure
immediately suspected and a liver biopsy was leads to portal hypertension (intrahepatic). Finally, it
eventually done over the course. Which of the is said that once cirrhosis has developed, reversal is
following may not be true about the patient's already rare; correction of malnutrition may improve
condition? the patient's clinical presentation but will not cure
A. The biopsy will most likely show bridging definitively the liver problem.
fibrous septae and parenchymal nodules in the
liver.
B. There is reversibility upon correction of the
malnutritive state.
C. It is the predominant cause of intrahepatic
portal hypertension.
D. None of these.
E. All of these.
429 What is the most common clinically significant WEBSTER ALINDOG, MIDTERM 1
congenital anomaly of the pancreas? MD (TOP 3 - FEB EXAM - AUG
A. Annular pancreas 2014 MED BOARDS; 2014
B. Pancreatic agenesis TOPNOTCH MD)
C. Pancreas divisum
D. Ectopic pancreas

430 What consitute the membrane attack complex WEBSTER ALINDOG, MIDTERM 1
of the complement system? MD (TOP 3 - FEB EXAM - AUG
A. C1q,r,s 2014 MED BOARDS; 2014
B. C1q, C3b TOPNOTCH MD)
C. C3b, C4a, C5b
D. C5b, C6-9

431 All of the following are immune-mediated Master the 4 hypersensitivity class and their WEBSTER ALINDOG, MIDTERM 1
hypersensitivity (type III) except for: respective examples. (high yield) MD (TOP 3 - FEB EXAM - AUG
A. Type I DM 2014 MED BOARDS; 2014
B. SLE TOPNOTCH MD)
C. PSGN
D. Serum sickness

432 A 42-year old female came in for consult for This is a case of follicular thyroid carcinoma. It has WEBSTER ALINDOG, MIDTERM 1
presence of a slowly enlarging non-tender little tendency to invade the lymphatics, but vascular MD (TOP 3 - FEB EXAM - AUG
nodule in her anterior neck. Surgical removal invasion is common, with spread to bone, lungs, liver 2014 MED BOARDS; 2014
was done and studies revealed a thyroid tumor and elsewhere. Buzz words: papillary thyroid CA - TOPNOTCH MD)
with light-tan appearance containing small foci orphan Annie eye, psammoma; follicular thyroid CA -
of hemorrhage; (+) fairly uniform cells forming vascular invasion, Hurthle cell; medullary thyroid CA -
small follicles containing colloid with some amyloid deposits, C-cell hyperplasia.
Hurthle cells but no psammoma bodies. Should
it progresses, one can assume that this would
most likely metastasize thru:
A. Direct seeding
B. Lymphatic spread
C. Hematogenous dissemination
D. Metastasis not possible
433 What is the most accepted theory about the WEBSTER ALINDOG, MIDTERM 1
pathogenesis of vitiligo? MD (TOP 3 - FEB EXAM - AUG
A. Post-infectious 2014 MED BOARDS; 2014
B. Poor nutrition TOPNOTCH MD)
C. Autoimmunity
D. Chemical toxicity

434 Which of the following statements is most WEBSTER ALINDOG, MIDTERM 1


consistent with brain abscess? MD (TOP 3 - FEB EXAM - AUG
A. It is a discrete, non-encapsulated lesion 2014 MED BOARDS; 2014
with central liquefactive necrosis. TOPNOTCH MD)
B. It usually originates from adjacent
structures to the brain via the lymphatics.
C. CSF is under increased pressure, with
elevated WBC and protein levels but normal
sugar.
D. The most favored site is the cerebellum.
E. All of the above.
435 What clotting factor is lacking in Hemophilia A? Hemophilia A - factor VIII; Hemophilia B - factor WEBSTER ALINDOG, MIDTERM 1
A. Factor VII IX/Christmas factor; Hemophilia C - factor XI. All will MD (TOP 3 - FEB EXAM - AUG
B. Factor VIII have increased/prolonged PTT. 2014 MED BOARDS; 2014
C. Factor IX TOPNOTCH MD)
D. Factor XIII
E

TOPNOTCH MEDICAL BOARD PREP PATHOLOGY SUPEREXAM Page 59 of 99


For inquiries visit www.topnotchboardprep.com.ph or email us at topnotchmedicalboardprep@gmail.com
TOPNOTCH MEDICAL BOARD PREP PATHOLOGY SUPEREXAM
For inquiries visit www.topnotchboardprep.com.ph or email us at topnotchmedicalboardprep@gmail.com
Item QUESTION EXPLANATION AUTHOR TOPNOTCH
# EXAM
436 A potent vasodilator released during NO = nitric oxide/EDRF; N2O = nitrous oxide/laughing WEBSTER ALINDOG, MIDTERM 1
inflammation which reduces platelet gas, your inhalational anesthetic. MD (TOP 3 - FEB EXAM - AUG
aggregation and adhesion, inhibits several 2014 MED BOARDS; 2014
features of mast cell-induced inflammation and TOPNOTCH MD)
serves as an endogenous regulator of leukocyte
recruitment. It is released from endothelial
cells, hence also known as endothelial-derived
relaxing factor:
A. Leukotrienes
B. Bradykinin
C. Nitric oxide
D. Nitrous oxide
E. Prostaglandin
437 Tay-Sachs disease is a lysosomal storage Disease: enzyme deficient: major accumulating WEBSTER ALINDOG, MIDTERM 1
disease predominated with affectation of metabolite --- Tay-Sachs: hexosaminidase alpha: GM2 MD (TOP 3 - FEB EXAM - AUG
neurons in the brain and the retina. The ganglioside --- Gaucher: glucocerebrosidase: 2014 MED BOARDS; 2014
enzyme deficient is hexosaminidase alpha glucocerebroside --- Neimann-Pick: TOPNOTCH MD)
subunit and the major accumulating metabolite sphingomyelinase: sphingomyelin --- Hurler: alpha-L-
is: iduronidase: dermatan sulfate, heparan sulfate ---
A. Glycogen Hunter: L-iduronosulfate sulfatase: dermatan sulfate,
B. Glucocerebroside heparan sulfate
C. Dermatan sulfate
D. GM2 ganglioside
438 The carcinogenic agent that is mainly found in WEBSTER ALINDOG, MIDTERM 1
missile fuel and space vehicles: MD (TOP 3 - FEB EXAM - AUG
A. Beryllium 2014 MED BOARDS; 2014
B. Radon TOPNOTCH MD)
C. Chromium
D. Benzene

439 This virus causes a benign, self-limited WEBSTER ALINDOG, MIDTERM 1


lymphoproliferative disorder with absolute MD (TOP 3 - FEB EXAM - AUG
lymphocytosis (atypical lymphocytes); 2014 MED BOARDS; 2014
associated with hairy leukoplakia and a number TOPNOTCH MD)
of neoplasms including lymphomas and
nasopharyngeal carcinoma:
A. CMV
B. EBV
C. Adenovirus
D. HTLV-1
440 This refers to the ratio of thickness of the WEBSTER ALINDOG, MIDTERM 1
mucous gland layer in the airways to that of the MD (TOP 3 - FEB EXAM - AUG
wall between the epithelial lining and the 2014 MED BOARDS; 2014
cartilage. It is utilized to assess chronic TOPNOTCH MD)
bronchitis:
A. Reid index
B. De Ritis ratio
C. Gleason score
D. Air passage ratio

441 Hemolysis and vasoocclusive crisis are common Sickle-cell anaemia is caused by a point mutation in JULIET KRISTINE MIDTERM 2
in this form of anemia caused by mutation of the β-globin chain of haemoglobin, causing the EVANGELISTA, MD EXAM - AUG
glutamic acid to valine at amino acid 6 of beta hydrophilic amino acid glutamic acid to be replaced (TOP 9 - FEB 2014 2014
chain with the hydrophobic amino acid valine at the sixth MED BOARDS;
A. Hemoglobin C position. TOPNOTCH MD)
B. Thalassemia
C. Hereditary spherocytosis
D. Sickle cell anemia
E. G6PD Deficiency
442 A 58 year-old male, hypertensive was rushed to A subarachnoid hemorrhage is a bleeding into the JULIET KRISTINE MIDTERM 2
the Emergency Room due to loss of subarachnoid space, the area between the arachnoid EVANGELISTA, MD EXAM - AUG
consciousness. His son recalled that his father membrane and the pia mater surrounding the brain. (TOP 9 - FEB 2014 2014
suddenly complained of very severe headache This may occur spontaneously, usually from a MED BOARDS;
after their breakfast followed by loss of ruptured cerebral aneurysm, or may result from head TOPNOTCH MD)
consciousness. What is the most probable injury. Symtpoms of SAH include a severe headache
diagnosis? with a rapid onset (thunderclap headache), vomiting,
A. Intraparenchymal Hemorrhage confusion or a lowered level of consciousness, and
B. Epidural Hemorrhage sometimes seizures.
C. Subarachnoid Hemorrhage
D. Subdural Hemorrhage
E. Interventricular Hemorrhage
443 Which of the following malignant neoplasms is Paraneoplastic syndromes are common in lung cancer, JULIET KRISTINE MIDTERM 2
correctly paired with the appropriate and may be the first manifestation of the disease or its EVANGELISTA, MD EXAM - AUG
paraneoplastic manifestation? recurrence. Hypercalcemia is frequently found in (TOP 9 - FEB 2014 2014
A. Squamous cell carcinoma - hypercalcemia patients with MED BOARDS;
B. Glioblastoma multiforme - hyperglycemia squamous cell carcinoma. It may arise from bone TOPNOTCH MD)
C. Adrenal cortical carcinoma - hyperuricemia metastasis but can also be induced in a paraneoplastic
D. Hepatocellular carcinoma - polycythemia manner by secretion of parathyroid hormone-related
E. Gastric carcinoma - achlorydia protein (PTHrP).

TOPNOTCH MEDICAL BOARD PREP PATHOLOGY SUPEREXAM Page 60 of 99


For inquiries visit www.topnotchboardprep.com.ph or email us at topnotchmedicalboardprep@gmail.com
TOPNOTCH MEDICAL BOARD PREP PATHOLOGY SUPEREXAM
For inquiries visit www.topnotchboardprep.com.ph or email us at topnotchmedicalboardprep@gmail.com
Item QUESTION EXPLANATION AUTHOR TOPNOTCH
# EXAM
444 The sequence of cellular events in inflammation The sequence of cellular events in inflammation is: JULIET KRISTINE MIDTERM 2
is: Margination-diapedesis-chemotaxis-phagocytosis EVANGELISTA, MD EXAM - AUG
A. Diapedesis-margination-chemotaxis- (TOP 9 - FEB 2014 2014
phagocytosis MED BOARDS;
B. Margination-diapedesis-chemotaxis- TOPNOTCH MD)
phagocytosis
C. Diapedesis-chemotaxis-margination-
phagocytosis
D. Margination-chemotaxis-diapedesis-
phagocytosis
E. Chemotaxis-margination-diapedesis-
phagocytosis
445 A male infant was born at term. No congenital X-linked hypogammaglobulinemia or Bruton's JULIET KRISTINE MIDTERM 2
anomalies were noted at birth. About 6 months agammaglobulinemia occurs due to low levels of all EVANGELISTA, MD EXAM - AUG
later, he was noted of failure to thrive and has immunoglobulins, a virtual absence of B cells due to (TOP 9 - FEB 2014 2014
been getting bacterial pneumonia with tyrosine kinase mutation. Male infants at about 6 MED BOARDS;
Hemophilus influenzae and Streptococcus months of age present with recurrent pyogenic TOPNOTCH MD)
pneumoniae cultured from his sputum. You bacterial infections caused by Strep pneumoniae and
strongly suspect that he has: Haemophilus influenzae.
A. Selective IgA deficiency
B. Severe Combined Immunodeficiency
C. EBV Infection
D. X-linked Hypommaglobulinemia
E. Di-George Syndrome
446 A pigeon raiser suddenly developed stiff neck Cryptococcus neoformans is a type of fungus that is JULIET KRISTINE MIDTERM 2
and deteriorating sensorium was rushed to the found in the soil worldwide, usually in association EVANGELISTA, MD EXAM - AUG
ER and was admitted. Work-up was done for with bird droppings. Cryptococcal meningitis is (TOP 9 - FEB 2014 2014
probable diagnosis. Lumbar puncture was done believed to result from dissemination of the fungus MED BOARDS;
and CSF was obtained and should be stained from either an observed or unappreciated pulmonary TOPNOTCH MD)
with: infection. India ink of the CSF is a traditional
A. Acid fast stain microscopic method of diagnosis.
B. Methylene blue
C. India ink stain
D. Gram Stain
E. H&E Stain
447 A 5-year old was brought in at the out-patient Kwashiorkor is caused by protein deprivation greater JULIET KRISTINE MIDTERM 2
department because of generalized edema than reduction in caloric intake. It is the most common EVANGELISTA, MD EXAM - AUG
accompanied by easily pluckable hair, form of protein-energy malnutrition which is (TOP 9 - FEB 2014 2014
listlessness and loss of appetite. The child is associated with severe loss of the visceral protein MED BOARDS;
most likely suffering from: compartment. Clinical findings include generalized or TOPNOTCH MD)
A. Anorexia dependent edema, skin lesions, hair changes and
B. Marasmus enlarged, fatty liver.
C. Kwashiorkor
D. Bulimia
E. Vitamin Deficiency
448 A 22 year-old man has lifelong hemorrhagic Hemophilia is a group of hereditary genetic disorders JULIET KRISTINE MIDTERM 2
diathesis. The PT and bleeding time are normal, that impair the body's ability to control blood clotting EVANGELISTA, MD EXAM - AUG
but the aPTT is prolonged. The most likely or coagulation. In Hemophilia, platelet count, bleeding (TOP 9 - FEB 2014 2014
cause of the bleeding disorder is: time and prothrombin time are all normal. Only the MED BOARDS;
A. Von Willebrand's Disease aPTT is prolonged such as in Hemophilia A and B. TOPNOTCH MD)
B. Hemophilia
C. Vitamin K Deficiency
D. Factor VII Deficiency
E. Bernard-Soulier Syndrome
449 The histologic hallmark of chronic bronchitis: Chronic bronchitis is defined clinically as persistent JULIET KRISTINE MIDTERM 2
A. Inflammation and fibrosis cough and sputum production for at least 3 months in EVANGELISTA, MD EXAM - AUG
B. Hypertophy of goblet cells at least 2 consecutive years. Although the number of (TOP 9 - FEB 2014 2014
C. Hyperplasia of mucosal glands goblet cells increase slightly, the major histologic MED BOARDS;
D. Marked narrowing of bronchial lumen change is in the size of the mucous gland TOPNOTCH MD)
E. Reversible bronchoconstriction (hyperplasia). This increase is assessed by the ratio
of the thickness of mucous gland layer to the thickness
of the wall between the epithelium and the cartilage,
the Reid Index.
450 A 55 year-old man was suffering from recurrent Charcot-Leyden crystals are collections of crystalloid JULIET KRISTINE MIDTERM 2
attacks of persistent cough. He was rushed to made up of eosinophil lysophospholipase binding EVANGELISTA, MD EXAM - AUG
the Emergency Room due to severe defficulty of protein called galectin-10. Curshmann spirals are (TOP 9 - FEB 2014 2014
breathing. PE revealed wheezes over lung mucus plugs containing whorls of shed epithelium. MED BOARDS;
fields. Mucous plugs were collected and These result from mucus plugging in subepithelial TOPNOTCH MD)
histologically examined revealing collections of mucus gland ducts or bronchioles which later become
cystalloid made up of eosinophil membrane extruded.
protein. These are:
A. Charcot-Leyden Crystals
B. Reinke Crystals
C. Curschmann spirals
D. Councilman Bodies
E. Psammoma Bodies
451 The most important prognostic feature in Prognostic information is important in counseling JULIET KRISTINE MIDTERM 2
invasive breast cancer is: patients about the likely outcome of their disease and EVANGELISTA, MD EXAM - AUG
A. Grade of tumor choosing appropriate treatment. Axillary lymph (TOP 9 - FEB 2014 2014
B. Histologic type of tumor node status is the most important prognostic MED BOARDS;
C. Size of tumor factor for invasive carcinoma in the absence of TOPNOTCH MD)
D. Status of sentinel lymph nodes distant metastasis. The size of an invasive
E. Metastasis carcinoma is the second most important
prognostic factor.

TOPNOTCH MEDICAL BOARD PREP PATHOLOGY SUPEREXAM Page 61 of 99


For inquiries visit www.topnotchboardprep.com.ph or email us at topnotchmedicalboardprep@gmail.com
TOPNOTCH MEDICAL BOARD PREP PATHOLOGY SUPEREXAM
For inquiries visit www.topnotchboardprep.com.ph or email us at topnotchmedicalboardprep@gmail.com
Item QUESTION EXPLANATION AUTHOR TOPNOTCH
# EXAM
452 An 8 year-old female can bend her thumb back Inherited defects in the synthesis or structure of JULIET KRISTINE MIDTERM 2
to touch her forearm. She can pull her skin out fibrillar collagen results to Ehler's Danlos syndrome. EVANGELISTA, MD EXAM - AUG
from her abdomen about 10cm and a cut to her Skin of patients with this defect is extraordinarily (TOP 9 - FEB 2014 2014
skin gapes open with difficulty in repairing. stretchable, extremely fragile and vulnerable to MED BOARDS;
Inherited defect is suspected causing the above trauma. Affected individual also has joint TOPNOTCH MD)
findings which is defect in: hypermobility, joint laxity, dislocations or easy
A. LDL receptor bruising.
B. factor VIII
C. dystrophin
D. alpha-1 antitrypsin
E. collagen
453 Honesto is a 6 year-old boy who had fever Reye's syndrome is a rare but serious condition that JULIET KRISTINE MIDTERM 2
associated with vesicular lesions in different causes swelling in the liver and brain. Reye's EVANGELISTA, MD EXAM - AUG
ages all over his body. He was given aspirin and syndrome most often affects children and teenagers (TOP 9 - FEB 2014 2014
subsequently developed fatty change of the recovering from a viral infection (Varicella and Flu) MED BOARDS;
liver. The most likely diagnosis is: who has been given Aspirin. TOPNOTCH MD)
A. Viral Hepatitis
B. Varicella-Zoster Infection
C. Reye's Syndrome
D. Subactue sclerosing panecephalitis
E. Impetigo
454 A 45 year-old male office worker has Sjogren's syndrome is is a chronic autoimmune JULIET KRISTINE MIDTERM 2
keratoconjunctivitis. He has oral mucosal disease which destroys the exocrine glands, EVANGELISTA, MD EXAM - AUG
atrophy with buccal mucosal ulceration. A specifically the salivary and lacrimal glands which (TOP 9 - FEB 2014 2014
biopsy of minor salivary glands revealed leads to the development of xerostomia and MED BOARDS;
plasma cell infiltrates and biopsy of his lips keratoconjunctivitis sicca, which takes place in TOPNOTCH MD)
revealed marked lymphocytic infiltrates. The association with lymphocytic infiltration of the glands.
antibody that is most likely found to this The antibody that is most likely found in this patient is
patient is: anti-SS-A.
A. anti-dsDNA
B. anti-centromere
C. anti-SS-A
D. anti-Scl-70
E. anti-RNP
455 Compromise of heart function because the Constrictive pericarditis is diagnosed when heart is JULIET KRISTINE MIDTERM 2
pericardium is stiff is: completely encased by a dense fibrosis that it cannot EVANGELISTA, MD EXAM - AUG
A. Hypertrophic cardiomyopathy expand normally during diastole. Fibrinous (TOP 9 - FEB 2014 2014
B. Restrictive cardiomyopathy pericarditis is seen in patients with uremia and or MED BOARDS;
C. Cardiac tamponade viral infection. The exudate imparts an irregular TOPNOTCH MD)
D. Constrictive pericarditis appearance to the pericardial surface (bread and
E. Fibrinous pericarditis butter pericarditis).

456 It refers to a special form of necrosis usually Fibrinous pericarditis is a special form of necrosis JULIET KRISTINE MIDTERM 2
seen in immune reactions involving blood usually seen in immune reactions involving blood EVANGELISTA, MD EXAM - AUG
vessels due to deposits of fibrin complexes, vessels due to deposits of fibrin complexes, together (TOP 9 - FEB 2014 2014
together with fibrin that has leaked out of with fibrin that has leaked out of vessels resulting in a MED BOARDS;
vessels resulting in a bright pink amorphous bright pink amorphous appearance in H&E stains, TOPNOTCH MD)
appearance in H&E stains: called "fibrinoid" (fibrin-like) lesions.
A. Fat necrosis
B. Fibrinous necrosis
C. Coagulative necrosis
D. Liquefactive necrosis
E. Gangrenous necrosis
457 A 42 year-old female was diagnosed with Typical thyroid adenoma is a solitary, spherical, well- JULIET KRISTINE MIDTERM 2
follicular adenoma. The characteristics of the encapsulated lesion that is well-demarcated from the EVANGELISTA, MD EXAM - AUG
diagnosis are enumerated below, except: surrounding thyroid parenchyma. Follicular (TOP 9 - FEB 2014 2014
A. Usually solitary and spherical adenomas contain multiple nodules on their cut MED BOARDS;
B. Papillary structures within the lesion surface. Papillary formation is not a typical feature of TOPNOTCH MD)
C. Fibrous encapsulation follicular adenoma.
D. Contain multiple nodules on cut surface
E. Morphology within and outside the capsule
are different
458 A 2-month old infant, first born male was Congenital hypertrophic pyloric stenosis is a condition JULIET KRISTINE MIDTERM 2
brought to the Emergency Room for persistent which causes projectile non-bilious vomiting. It most EVANGELISTA, MD EXAM - AUG
nonbilious projectile vomiting. Physical often occurs in the first 2 to 6 weeks of life. The (TOP 9 - FEB 2014 2014
examination revealed a palpable olive-shaped pyloric hypertrophy is felt classically as an olive- MED BOARDS;
mass on the abdomen. The most probable shaped mass in the middle upper part or right upper TOPNOTCH MD)
diagnosis is: quadrant of the infant's abdomen.
A. Duodenal atresia
B. Intussusception
C. Volvulus
D. Hypertrophic pyloric stenosis
E. Hirschprung's disease
459 In viral hepatitis infection, the folllowing is Bridging fibrosis is the deposition of fibrous tissue JULIET KRISTINE MIDTERM 2
considered the histologic marker of irreversible with linking of fibrous septa which is the hallmark of EVANGELISTA, MD EXAM - AUG
liver injury: chronic hepatitis. (TOP 9 - FEB 2014 2014
A. Piecemeal necrosis MED BOARDS;
B. Bridging necrosis TOPNOTCH MD)
C. Bridging fibrosis
D. Interface hepatitis
E. Ductular reaction

TOPNOTCH MEDICAL BOARD PREP PATHOLOGY SUPEREXAM Page 62 of 99


For inquiries visit www.topnotchboardprep.com.ph or email us at topnotchmedicalboardprep@gmail.com
TOPNOTCH MEDICAL BOARD PREP PATHOLOGY SUPEREXAM
For inquiries visit www.topnotchboardprep.com.ph or email us at topnotchmedicalboardprep@gmail.com
Item QUESTION EXPLANATION AUTHOR TOPNOTCH
# EXAM
460 A bone marrow aspirate was obtained in a 48 CML is a disease primary of adults between ages 25 JULIET KRISTINE MIDTERM 2
year-old male, nosmoker who complained of and 60 years. Patient presents with mild to moderate EVANGELISTA, MD EXAM - AUG
easy fatigability, weakness, weight loss and anemia, easy fatigability, weakness, weight loss, (TOP 9 - FEB 2014 2014
anorexia associated with LUQ abdominal pain. anorexia, abdominal pain due to splenomegaly which MED BOARDS;
Laboratory revealed anemia. The bone marrow may lead to splenic infarction. The bone marrow TOPNOTCH MD)
aspirate revealed numerous scattered aspirate of CMP shows numerous scattered
macrophage with abundant wrinkled green- macrophage with abundant wrinkled green-blue
blue cytoplasm called "sea-blue histiocytes". cytoplasm called "sea-blue histiocytes".
The most probable diagnosis is:
A. Chronic myelogenous leukemia
B. Multiple myeloma
C. Hodgkin's lymphoma
D. Burkitt's lymphoma
E. Adult T-cell lymphoma
461 A 7 year old boy presents with pallor and this is a case of acute lymphoblastic leukemia, the LUISA SARANILLO, BACK-UP
recurrent fever. CBC showed increased WBC most common cancer of children. It has hypercellular MD (TOP 6 - FEB MIDTERM
and decreased RBC. Bone marrow analysis marrow packed with lymphoblasts - immature T or B 2014 MED BOARDS; EXAM AUG
showed hypercellularity with predominance of cells. It is MPO-negative, PAS positive, TDT and CALLA TOPNOTCH MD) 2014 - FOR
lymphoblasts, MPO negative, PAS positive, TDT positive. It may spread to the CNS and testes. It is the INCLUSION IN
and CALLA positive. What is not true of this most responsive to chemotherapy, and the primary THE SAMPLEX
disease? drug used is asparaginase..
A. it may spread to CNS and testes
B. least responsive to chemotherapy
C. the primary drud used for this is
asparaginase
D. it is composed of immature B or T cells
E. all of the choices are correct
462 A 35 year old male complained of bone pain The primary consideration is multiple myeloma LUISA SARANILLO, BACK-UP
and recurrent fever. His Hemoglobin was low presenting with bone pain, recurrent infection, MD (TOP 6 - FEB MIDTERM
and his creatinine is moderately high. On anemia, and renal insufficiency. Lytic bone lesions and 2014 MED BOARDS; EXAM AUG
imaging, you see lytic bone lesions with punch punched-out defects are characteristics of multiple TOPNOTCH MD) 2014 - FOR
out defects. What is your primary myeloma, which are not found in waldenstroms INCLUSION IN
consideration? macroglobulinemia. MGUS is asymptomatic. CKD will THE SAMPLEX
A. chronic kidney disease not present with bone pain and lytic bone lesions.
B. multiple myeloma
C. waldenstrom's macroglobulinemia
D. monoclonal gammopathy of undetermined
significance
E. none of the choices
463 In chronic myeloproliferative disorders, one of CML has no JAK2 mutations. LUISA SARANILLO, BACK-UP
the following has no JAK2 mutation. MD (TOP 6 - FEB MIDTERM
A. Polycythemia vera 2014 MED BOARDS; EXAM AUG
B. Essential thrombocytosis TOPNOTCH MD) 2014 - FOR
C. Primary myelofibrosis INCLUSION IN
D. Chronic myelogenous leukemia THE SAMPLEX
E. No exception

464 What is the major red cell hemoglobin present In Cooley's anemia or Beta-thalassemia major, the LUISA SARANILLO, BACK-UP
in Cooley's anemia? major red cell hemoglobin is Hemoglobin F. MD (TOP 6 - FEB MIDTERM
A. Hemoglobin F Hemoglobin A is absent. Hemoglobin H is present in 2014 MED BOARDS; EXAM AUG
B. Hemoglobin A hemoglobin H disease, while hemoglobin Barts is TOPNOTCH MD) 2014 - FOR
C. Hemoglobin H present in hydrops fetalis. INCLUSION IN
D. Hemoglobin Barts THE SAMPLEX
E. A and B

465 What is the most common inherited bleeding LUISA SARANILLO, BACK-UP
disorder? MD (TOP 6 - FEB MIDTERM
A. Hemophilia A 2014 MED BOARDS; EXAM AUG
B. Hemophilia B TOPNOTCH MD) 2014 - FOR
C. Von Willebrand disease INCLUSION IN
D. Bernard-Soulier syndrome THE SAMPLEX
E. Glanzmann's thrombasthenia

466 A 65 year old male smoker had a mass in the Warthin tumor or papillary cystadenoma LUISA SARANILLO, BACK-UP
mandibular area. Biopsy of the mass revealed lymphomatosum arises almost exclusively in the MD (TOP 6 - FEB MIDTERM
an epithelial and lymphoid elements, with parotid gland, and is associated with smoking. It is 2014 MED BOARDS; EXAM AUG
cystic spaces and follicular germinal centers. composed of epithelial and lymphoid elements with TOPNOTCH MD) 2014 - FOR
What is the diagnosis? cystic spaces and follicular germinal centers. INCLUSION IN
A. Pleomorphic adenoma THE SAMPLEX
B. paraganglioma
C. mucocoele
D. warthin tumor
E. mucoepidermoid carcinoma
467 A 5 year old girl with atopic dermatitis lichenification - thick rough skin with prominent LUISA SARANILLO, BACK-UP
constantly scratching her elbow and arm markings; excoriation- linear, trauma, epidermal MD (TOP 6 - FEB MIDTERM
producing a thickened rough skin with breakage; plaque - elevated, flat lesion, larger than 2014 MED BOARDS; EXAM AUG
prominent markings. What is the skin lesion? 5mm; scale- dry, platelike, from conification; wheal - TOPNOTCH MD) 2014 - FOR
A. plaque pruritic, erythematous with dermal edema. INCLUSION IN
B. lichenification THE SAMPLEX
C. excoriation
D. scale
E. Wheal

TOPNOTCH MEDICAL BOARD PREP PATHOLOGY SUPEREXAM Page 63 of 99


For inquiries visit www.topnotchboardprep.com.ph or email us at topnotchmedicalboardprep@gmail.com
TOPNOTCH MEDICAL BOARD PREP PATHOLOGY SUPEREXAM
For inquiries visit www.topnotchboardprep.com.ph or email us at topnotchmedicalboardprep@gmail.com
Item QUESTION EXPLANATION AUTHOR TOPNOTCH
# EXAM
468 What is the most common form of cancer in prostate adenocarcinoma is the most common form of LUISA SARANILLO, BACK-UP
men? cancer in men. Lung cancer is the leading cause of MD (TOP 6 - FEB MIDTERM
A. Testicular cancer cancer mortality overall. 2014 MED BOARDS; EXAM AUG
B. Lung cancer TOPNOTCH MD) 2014 - FOR
C. Colon cancer INCLUSION IN
D. Prostate adenocarcinoma THE SAMPLEX
E. Gastric cancer

469 A 50 year old female complained of vaginal this is a case of lichen sclerosus or chronic atrophic LUISA SARANILLO, BACK-UP
dryness, stiffening, and constriction. On vulvitis. All choices describe lichen sclerosus MD (TOP 6 - FEB MIDTERM
examination, there is a white parchment like 2014 MED BOARDS; EXAM AUG
patches of vulvar skin and atrophy of labia. TOPNOTCH MD) 2014 - FOR
Which one of the following is/are true? INCLUSION IN
A. It can lead to cancer THE SAMPLEX
B. there is thinning of the epidermis
C. there is hydropic degeneration of basal cells
D. there is lymphocytic infiltrate
E. all of the choices are correct
470 Osteopetrosis is the first genetic disease treated osteopetrosis aka marble bone disease; osteogenesis LUISA SARANILLO, BACK-UP
with bone marrow transplantation. It is also imperfecta aka brittle bone disease; paget disease aka MD (TOP 6 - FEB MIDTERM
known as: osteitis deformans; osteitis fibrosa cystica is seen in 2014 MED BOARDS; EXAM AUG
A. Marble bone disease hyperparathyroidism. TOPNOTCH MD) 2014 - FOR
B. Brittle bone disease INCLUSION IN
C. Osteitis deformans THE SAMPLEX
D. Osteitis fibrosa cystica
E. None of the choices

471 A 30 year old male was riding on his this is epidural hematoma with lucid interval and LUISA SARANILLO, BACK-UP
motorcycle, suddenly he had a head on collision lenticular shaped lesion on CT scan. The source of MD (TOP 6 - FEB MIDTERM
with another vehicle. He was thrown 1meter bleed is middle meningeal artery; Subdural hematoma 2014 MED BOARDS; EXAM AUG
away from his motorcycle hitting his head on has delayed presentation with fluctuating levels of TOPNOTCH MD) 2014 - FOR
the ground. He was brought to the ER consciousness, and on CT scan it has crescent shaped INCLUSION IN
unconscious, but later regain his consciousness. lesions. The source of bleed is the tearing of bridging THE SAMPLEX
On CT scan, there was a lenticular shaped lesion veins.
most probably a bleed. What is the source of
bleed?
A. middle cerebral artery
B. brain parenchyma
C. middle meningeal artery
D. bridging veins
E. ventricles
472 A 21 year old male noticed a painless testicular The route of lymphatic spread of testicular tumor is as LUISA SARANILLO, BACK-UP
mass. If it is a testicular tumor, what lymph follows: para-aortic nodes > mediastinal nodes > MD (TOP 6 - FEB MIDTERM
node is directly involved in its lymphatic supraclavicular nodes 2014 MED BOARDS; EXAM AUG
spread? TOPNOTCH MD) 2014 - FOR
A. Para-aortic nodes INCLUSION IN
B. Inguinal nodes THE SAMPLEX
C. Obturator nodes
D. Iliac nodes
E. All of the choices
473 What is the most common malignant primary LUISA SARANILLO, BACK-UP
brain tumor in adults? MD (TOP 6 - FEB MIDTERM
A. meningioma 2014 MED BOARDS; EXAM AUG
B. Cystic cerebellar astrocytoma TOPNOTCH MD) 2014 - FOR
C. ependymoma INCLUSION IN
D. Glioblastoma multiforme THE SAMPLEX
E. Medulloblastoma

474 A 34 year old female complained of polyuria diabetes insipidus will present with polyuria and LUISA SARANILLO, BACK-UP
and thirst. You are considering a posterior thirst. There are 2 types: nephrogenic and central DI. MD (TOP 6 - FEB MIDTERM
pituitary syndrome. If this is cause by a In central DI, there is lack of ADH due to transection of 2014 MED BOARDS; EXAM AUG
transection of the pituitary stalk. What is the pituitary stalk, hypothalamic disease or posterior TOPNOTCH MD) 2014 - FOR
treatment? pituitary disease. The treatment for central DI is INCLUSION IN
A. thiazides desmopressin. Indomethacin and thiazides are used THE SAMPLEX
B. indomethacin for nephrogenic DI. Water restriction is indicated in
C. desmopressin SIADH. demeclocycline can cause nephrogenic DI.
D. water restriction
E. demeclocycline
475 A 45 year old female sought consultation to a The following are the morphologic feature of each LUISA SARANILLO, BACK-UP
surgeon due to a mass on the upper outer cancer type: invasive lobular CA - signet ring cells MD (TOP 6 - FEB MIDTERM
quadrant of her Right breast. The surgeon arranged in Indian file pattern, or dyscohesive 2014 MED BOARDS; EXAM AUG
decided to have biopsy of the mass which infiltrating tumor cells arranged in single file or in TOPNOTCH MD) 2014 - FOR
revealed a dyscohesive infiltrating tumor cells loose clusters or sheets; invasive ductal CA- has INCLUSION IN
arranged in single file or in sheets. characteristic grating sound due to small, central THE SAMPLEX
A. invasive lobular carcinoma pinpoint foci or streaks of chalky-white elastotic
B. invasive ductal carcinoma stroma; phyllodes tumor - is a lobulated tumor with
C. inflammatory breast CA cystic spaces; inflammatory breast CA - tumor emboli
D. phyllodes tumor in dermal lymphatics.
E. fibroadenoma

TOPNOTCH MEDICAL BOARD PREP PATHOLOGY SUPEREXAM Page 64 of 99


For inquiries visit www.topnotchboardprep.com.ph or email us at topnotchmedicalboardprep@gmail.com
TOPNOTCH MEDICAL BOARD PREP PATHOLOGY SUPEREXAM
For inquiries visit www.topnotchboardprep.com.ph or email us at topnotchmedicalboardprep@gmail.com
Item QUESTION EXPLANATION AUTHOR TOPNOTCH
# EXAM
476 One of the following embryologic remnant is Right umbilical vein degenerates and has no remnant. LUISA SARANILLO, BACK-UP
not correctly paired. Left umbilical vein becomes the ligamentum teres MD (TOP 6 - FEB MIDTERM
A. Umbilical arteries : medial umbilical hepatis 2014 MED BOARDS; EXAM AUG
ligaments TOPNOTCH MD) 2014 - FOR
B. Foramen ovale : fossa ovalis INCLUSION IN
C. Ductus venosus : ligamentum venosum THE SAMPLEX
D. Right umbilical vein : ligamentum teres
hepatis
E. ductus arteriosus : ligamentum arteriosum
477 A 40 year old female with anterior neck mass diffuse nontoxic goiter is characterized by diffusely LUISA SARANILLO, BACK-UP
came in for consultation. You palpated a enlarged thyroid gland without nodularity. MD (TOP 6 - FEB MIDTERM
diffusely enlarged thyroid gland without Multinodular goiter is characterized by 2014 MED BOARDS; EXAM AUG
nodularity. The rest of the physical examination multilobulated, assymetrically enlarged thyroid gland. TOPNOTCH MD) 2014 - FOR
is unremarkable. What is your primary In graves disease, there is also diffuse enlargement of INCLUSION IN
consideration? thyroid gland but ophthalmopathy is prominent. in THE SAMPLEX
A. Multinodular goiter thyroid adenoma, there is a discrete solitary mass
B. Diffuse nontoxic goiter
C. thyroid adenoma
D. Graves disease
E. none of the choices
478 Hemorrhagic or red infarct occurs in tissues LUISA SARANILLO, BACK-UP
with dual circulation such as: MD (TOP 6 - FEB MIDTERM
A. liver 2014 MED BOARDS; EXAM AUG
B. lungs TOPNOTCH MD) 2014 - FOR
C. intestines INCLUSION IN
D. All of the above THE SAMPLEX
E. A and B only

479 A 50 year old male came in for an eye LUISA SARANILLO, BACK-UP
examination due to blurring of vision. After a MD (TOP 6 - FEB MIDTERM
thorough eye examination, he was told by the 2014 MED BOARDS; EXAM AUG
ophthalmologist that he has cataract. What are TOPNOTCH MD) 2014 - FOR
the possible systemic diseases causing cataract? INCLUSION IN
A. diabetes mellitus THE SAMPLEX
B. wilson disease
C. atopic dermatitis
D. all of the above
E. A and C only
480 A 37 year old female has a salmon-colored The lesion contains suprapapillary plates with dilated, LUISA SARANILLO, BACK-UP
plaques on her elbows, knees, and scalp with tortuous blood vessels within papillae which leads to MD (TOP 6 - FEB MIDTERM
adherent silvery-white scales. Upon scratching pinpoint bleeding when scrathed called the auspitz 2014 MED BOARDS; EXAM AUG
the lesion, pinpoint bleeding was noted. Which sign. When formation of the lesion is induced by local TOPNOTCH MD) 2014 - FOR
of the following is true: trauma, it is called koebner phenomenon. INCLUSION IN
A. there is dilated, tortuous blood vessels THE SAMPLEX
within the papillae
B. this is called koebner phenomenon
C. this is called spongiform pustules
D. this is called munro microabscesses
E. bleeding is caused by acanthosis
481 A 50 year-old man presents with recurrent SIMILAR TO PREVIOUS BOARD EXAM ANGELIS ANDREA FINAL EXAM -
fever, infections, bleeding tendencies and CONCEPT/PRINCIPLE. Sea-blue histiocytes and COCOS, MD (TOP 1 - AUG 2014
pallor. On bone marrow biopsy, sea-blue massive splenomegaly are the buzzwords for CML. FEB 2014 MED
histiocytes were seen. The most likely diagnosis BOARDS; TOPNOTCH
is: MD)
A. AML
B. CML
C. CLL
D. ALL
482 Miliary tuberculosis spreads via which route? SIMILAR TO PREVIOUS BOARD EXAM ANGELIS ANDREA FINAL EXAM -
A. hematogenous CONCEPT/PRINCIPLE. COCOS, MD (TOP 1 - AUG 2014
B. lymphogenous FEB 2014 MED
C. lymphohematogenous BOARDS; TOPNOTCH
D. Contiguous MD)

483 A patient who suffered a massive stroke years Coagulative necrosis happens in the heart, spleen, ANGELIS ANDREA FINAL EXAM -
ago would be expected to have: kidney, caseous necrosis for TB in the lungs, and COCOS, MD (TOP 1 - AUG 2014
A. Liquefactive necrosis fibrinoid necrosis for blood vessels. FEB 2014 MED
B. Coagulative necrosis BOARDS; TOPNOTCH
C. Fibrinoid necrosis MD)
D. Caseous necrosis
484 The movement of a leukocyte towards the site SIMILAR TO PREVIOUS BOARD EXAM ANGELIS ANDREA FINAL EXAM -
of injury is called: CONCEPT/PRINCIPLE. There was a big confusion here COCOS, MD (TOP 1 - AUG 2014
A. chemotaxis but to be strict on the definition: Diapedesis is the FEB 2014 MED
B. diapedesis travel of the leukocyte between endothelial cells as it BOARDS; TOPNOTCH
C. migration exits the blood vessel, migration is the condition MD)
D. rolling where in the leukocyte travels through the
interstitium to the site of injury or infection while
chemotaxis is the orientation of a cell towards a
chemical stimulus.
485 The diagnosis of endometriosis can be SIMILAR TO PREVIOUS BOARD EXAM ANGELIS ANDREA FINAL EXAM -
confirmed by the presence of: CONCEPT/PRINCIPLE. COCOS, MD (TOP 1 - AUG 2014
A. Endometrial stroma FEB 2014 MED
B. Endometrial glands BOARDS; TOPNOTCH
C. Both A and B MD)
D. Either A or B

TOPNOTCH MEDICAL BOARD PREP PATHOLOGY SUPEREXAM Page 65 of 99


For inquiries visit www.topnotchboardprep.com.ph or email us at topnotchmedicalboardprep@gmail.com
TOPNOTCH MEDICAL BOARD PREP PATHOLOGY SUPEREXAM
For inquiries visit www.topnotchboardprep.com.ph or email us at topnotchmedicalboardprep@gmail.com
Item QUESTION EXPLANATION AUTHOR TOPNOTCH
# EXAM
486 A patient palpated a mass on her left breast. Malignant breast masses are usually hard, fixed and ANGELIS ANDREA FINAL EXAM -
Which characteristic would point to a possible nontender on palpation. COCOS, MD (TOP 1 - AUG 2014
malignancy? FEB 2014 MED
A. The mass is firm and doughy. BOARDS; TOPNOTCH
B. It is movable. MD)
C. It is tender on palpation.
D. There is a palpable node on the axilla.

487 A 40 pack-year smoker presents with dyspnea Centroacinar emphysema (mostly affects respiratory ANGELIS ANDREA FINAL EXAM -
on exertion. On PE he has a barrel chest and bronchioles) is found in smokers, while panacinar COCOS, MD (TOP 1 - AUG 2014
hyperresonant lungs. Based on spirometry and emphysema (respiratory bronchioes, alveolar ducts, FEB 2014 MED
chest xray, you diagnosed him to have alveoli) is more common for those with anti- BOARDS; TOPNOTCH
emphysema. Which part of the respiratory trypsin/anti-elastase deficiency. SIMILAR TO MD)
system is damaged the most? PREVIOUS BOARD EXAM CONCEPT/PRINCIPLE.
A. respiratory bronchiole
B. alveoli
C. alveolar duct
D. major bronchi
488 A term neonate is delivered via cesarean This is a classic case describing TTN. The condition is ANGELIS ANDREA FINAL EXAM -
section because of cephalopelvic disproportion. due to retained lung fluid, commonly in term infants COCOS, MD (TOP 1 - AUG 2014
The amniotic fluid is clear and the infant cried delivered by cesarean section. FEB 2014 MED
almost immediately after birth. Minutes after BOARDS; TOPNOTCH
birth however, there was noted increase in MD)
respiratory rate and grunting. A chest xray
showed fluid in the fissures and prominent
pulmonary vascular markings. Which is the
most likely diagnosis?
A. meconium aspiration
B. idiopathic hyaline membrane disease
C. pneumonia
D. transient tachypnea of the newborn
489 A 6-day-old premature infant born at 29 weeks Necrotizing enterocolitis is a life-threatening ANGELIS ANDREA FINAL EXAM -
AOG presents with gross bloody stools, condition seen mostly in premature infants. The COCOS, MD (TOP 1 - AUG 2014
abdominal distention and autonomic instability. characteristic finding on plain radiograph is FEB 2014 MED
What is the initial diagnostic step? pneumatosis intestinalis. BOARDS; TOPNOTCH
A. Stool culture MD)
B. Plain abdominal xray
C. Meckel scan
D. Barium enema
490 A patient presents with bilateral acoustic NF1 is the same with Von Recklinghausen which ANGELIS ANDREA FINAL EXAM -
schwannoma. Which is the most likely presents with cutaneous neurofibromas among COCOS, MD (TOP 1 - AUG 2014
diagnosis? others. Von Hippel Lindau also predisposes FEB 2014 MED
A. Neurofibromatosis 1 individuals to tumors and cysts (eg. BOARDS; TOPNOTCH
B. Neurofibromatosis 2 Pheochromocytoma, and other CNS tumors). SIMILAR MD)
C. Von Hippel Lindau TO PREVIOUS BOARD EXAM CONCEPT/PRINCIPLE.
D. Von Recklinghausen
491 This type of cancer predisposes to a The syndrome described is known as Lambert-Eaton ANGELIS ANDREA FINAL EXAM -
paraneoplastic syndrome wherein antibodies syndrome which presents with muscle weakness. COCOS, MD (TOP 1 - AUG 2014
against presynaptic calcium channels are Small cell lung carcinoma is notorious for causing this FEB 2014 MED
produced: syndrome. BOARDS; TOPNOTCH
A. Squamous cell lung carcinoma MD)
B. Small cell lung carcinoma
C. Lung Adenocarcinoma
D. Lymphoma of the lung
492 In aortic dissection, the blood accumulates SIMILAR TO PREVIOUS BOARD EXAM ANGELIS ANDREA FINAL EXAM -
between: CONCEPT/PRINCIPLE. COCOS, MD (TOP 1 - AUG 2014
A. Tunica intima and tunica media FEB 2014 MED
B. The layers of tunica media BOARDS; TOPNOTCH
C. Tunica media and tunica adventitia MD)
D. Any of the above
493 The basic pathology of atherosclerosis is: Endothelial dysfunction or injury is the key event in ANGELIS ANDREA FINAL EXAM -
A. Fat deposition on the tunica intima the development of atherosclerosis. SIMILAR TO COCOS, MD (TOP 1 - AUG 2014
B. Thickening and loss of elasticity of arterial PREVIOUS BOARD EXAM CONCEPT/PRINCIPLE. FEB 2014 MED
walls BOARDS; TOPNOTCH
C. Endothelial injury MD)
D. Deposition of atheromatous plaque causing
clogged arteries
494 A patient complains of chest heaviness Myoglobin rises for the first 2-3 hours (has high ANGELIS ANDREA FINAL EXAM -
unrelieved by nitrates and rest. Which among sensitivity but poor specificity), CKMB rises 4-6 hours COCOS, MD (TOP 1 - AUG 2014
the following enzymes is expected to rise first? after, Troponins rise 6-12 hours after an MI. FEB 2014 MED
A. troponins BOARDS; TOPNOTCH
B. CKMB MD)
C. myoglobin
D. LDH

495 A patient presents with compressive symptoms The keyphrase fibrous tissue pertains to Reidel's ANGELIS ANDREA FINAL EXAM -
due to a fixed, hard and painless goiter. thyroiditis. A differential would be thyroid carcinoma, COCOS, MD (TOP 1 - AUG 2014
Thyroidectomy showed massive amounts of probably anaplastic type. FEB 2014 MED
fibrous tissue. Which is the likely diagnosis? BOARDS; TOPNOTCH
A. Papillary carcinoma MD)
B. Follicular carcinoma
C. Riedel's thyroiditis
D. de Quervain's thyroiditis

TOPNOTCH MEDICAL BOARD PREP PATHOLOGY SUPEREXAM Page 66 of 99


For inquiries visit www.topnotchboardprep.com.ph or email us at topnotchmedicalboardprep@gmail.com
TOPNOTCH MEDICAL BOARD PREP PATHOLOGY SUPEREXAM
For inquiries visit www.topnotchboardprep.com.ph or email us at topnotchmedicalboardprep@gmail.com
Item QUESTION EXPLANATION AUTHOR TOPNOTCH
# EXAM
496 A very active 17-year-old boy is noted by the A classic description for osteosarcoma is the key ANGELIS ANDREA FINAL EXAM -
family to have recurrent deep pains in his thigh phrase "sunburst appearance" (in contrast to Ewing COCOS, MD (TOP 1 - AUG 2014
that awaken him from sleep. The family brings sarcoma which is "onion skin appearance"). Periosteal FEB 2014 MED
him to your office with a complaint of swelling lifting is also a clue to the diagnosis. Osteosarcoma BOARDS; TOPNOTCH
over his distal thigh which he said is probably usually occurs at the metaphysis of the distal femur or MD)
due to playing basketball a week ago. A proximal tibia.
radiograph of the leg showed sunburst
appearance. Which would likely explain the
condition of the patient?
A. osteosarcoma
B. ewing's sarcoma
C. osteomyelitis
D. bone fracture
497 A young boy presents with gait instability and SIMILAR TO PREVIOUS BOARD EXAM ANGELIS ANDREA FINAL EXAM -
diplopia. On CT scan, there was a cerebellar CONCEPT/PRINCIPLE Craniopharyngoma is a tumor COCOS, MD (TOP 1 - AUG 2014
tumor which is hyperdense, and noncalcified. from the Rathke's pouch and ependymoma arises FEB 2014 MED
Which of the following is the most likely from the lining of the ventricles. BOARDS; TOPNOTCH
diagnosis? MD)
A. cerebellar glioma
B. craniopharyngoma
C. ependymoma
D. medulloblastoma
498 An ICU patient has "muddy brown" casts on AGN presents with RBC casts, nephrotic syndrome ANGELIS ANDREA FINAL EXAM -
urinalysis. This finding is pathognomonic for: with Fatty casts and Acute pyelonephritis with WBC COCOS, MD (TOP 1 - AUG 2014
A. Acute tubular necrosis casts. FEB 2014 MED
B. Nephrotic syndrome BOARDS; TOPNOTCH
C. Acute pyelonephritis MD)
D. Acute glomerulonephritis
499 Which is FALSE regarding autosomal dominant Congenital hepatic fibrosis is associated with ANGELIS ANDREA FINAL EXAM -
polycystic kidney disease (ADPKD)? autosomal recessive polycystic kidney disease, COCOS, MD (TOP 1 - AUG 2014
A. It is due to mutations in PKD1 or PKD2. previously infantile polycystic kidney disease. FEB 2014 MED
B. It is associated with congenital hepatic BOARDS; TOPNOTCH
fibrosis. MD)
C. It has been noted in patients with mitral
valve prolapse and benign hepatic cysts.
D. It presents with multiple, large bilateral
cysts.
500 A patient presents with ptosis and weakness Myasthenia gravis is under type 2 ANGELIS ANDREA FINAL EXAM -
more notable during the end of the day. (cytotoxic/antibody-mediated) hypersensitivity. The COCOS, MD (TOP 1 - AUG 2014
Tensilon test is positive. Which type of body produces antibodies against acetylcholine FEB 2014 MED
hypersensitivity does this patient exhibit? receptors on NMJ. BOARDS; TOPNOTCH
A. Type 1 MD)
B. Type 2
C. Type 3
D. Type 4
501 Which of the following is made up of Type 2 JAN CHARMAINE BACK-UP
collagen?? PALOMAR, MD (TOP MIDTERM
A. Cornea and lens 9 - FEB 2014 MED EXAM AUG
B. Vitreous humor and nucleus pulposus BOARDS; TOPNOTCH 2014
C. Basal lamina and bone MD)
D. Late wound repair
E. None of the above

502 Burton's lines is found in which poisoning? Burton's lines are lines on the gingiva and on the JAN CHARMAINE BACK-UP
A. Mercury epiphysis of long bones in lead poisoning. PALOMAR, MD (TOP MIDTERM
B. Copper 9 - FEB 2014 MED EXAM AUG
C. Lead BOARDS; TOPNOTCH 2014
D. Arsenic MD)
E. None of the above

503 The following are part of the Duke's Major Robbins 7th ed., 598 JAN CHARMAINE BACK-UP
Criteria for Infective endocarditis, except: PALOMAR, MD (TOP MIDTERM
A. Positive blood culture indicating 9 - FEB 2014 MED EXAM AUG
charcteristic organism BOARDS; TOPNOTCH 2014
B. New valvular regurgitation MD)
C. Echocardiographic finding of valve-related
mass or abscess
D. All of the above
E. None of the above
504 These are small erythematous or hemorrhagic, SIMILAR TO PREVIOUS BOARD EXAM JAN CHARMAINE BACK-UP
macular, nontender lesions on the palms and CONCEPT/PRINCIPLE:Osler nodes- are small, tender PALOMAR, MD (TOP MIDTERM
soles and are a consequence of septic embolic subcutaneous nodules that develop in the pulp of the 9 - FEB 2014 MED EXAM AUG
events: digitd or occassionally more proximally in the fingers BOARDS; TOPNOTCH 2014
A. Janeway lesions and persist for hours to several days; Roth spots-are MD)
B. Osler nodes oval retinal hemorrhages with pale centers Robbins
C. Roth spots 7th ed., 598
D. All of the above
E. None of the above

TOPNOTCH MEDICAL BOARD PREP PATHOLOGY SUPEREXAM Page 67 of 99


For inquiries visit www.topnotchboardprep.com.ph or email us at topnotchmedicalboardprep@gmail.com
TOPNOTCH MEDICAL BOARD PREP PATHOLOGY SUPEREXAM
For inquiries visit www.topnotchboardprep.com.ph or email us at topnotchmedicalboardprep@gmail.com
Item QUESTION EXPLANATION AUTHOR TOPNOTCH
# EXAM
505 The following features can be seen in rice bodies are aggregation of organizing fibrin JAN CHARMAINE BACK-UP
rheumatoid arthritis except: covering portions of the synovium and floating in the PALOMAR, MD (TOP MIDTERM
A. Rice bodies joint space; pannus is a mass of synovium and 9 - FEB 2014 MED EXAM AUG
B. Pannus formation synovial stromaconsisting of inflammatory cells, BOARDS; TOPNOTCH 2014
C. Heberden nodes granulation tissue, and fibroblasts, which grows over MD)
D. All of the above the articular cartilage and causes its erosion. Both rice
E. None of the above bodies and pannus is found in RA. Heberden nodes in
the fingers of patients with osteoarthritis represent
prominent osteophytes at the distal interphalangeal
joints. 1305
506 Which of the following is true regarding Sickle SIMILAR TO PREVIOUS BOARD EXAM JAN CHARMAINE BACK-UP
cell disease? CONCEPT/PRINCIPLE:Robbins 7th ed., 628 PALOMAR, MD (TOP MIDTERM
A. It is caused by a point mutation at the 6th 9 - FEB 2014 MED EXAM AUG
position of the Beta-globin chain leading to the BOARDS; TOPNOTCH 2014
substitution of a valine residue for a glutamic MD)
acid residue.
B. It is caused by a point mutation at the 9th
position of the Beta-globin chain leading to the
substitution of a valine residue for a glutamic
acid residue.
C. It is caused by a point mutation at the 6th
position of the Beta-globin chain leading to the
substitution of a glutamic acid residue for a
valine residue.
D. It is caused by a point mutation at the 9th
position of the Beta-globin chain leading to the
substitution of a glutamic acid residue for a
valine residue.
E. None of the above
507 This is the most common leukemia of adults SIMILAR TO PREVIOUS BOARD EXAM JAN CHARMAINE BACK-UP
and the elderly. Peripheral blood smear shows CONCEPT/PRINCIPLE PALOMAR, MD (TOP MIDTERM
smudge cells and nucleated red blood cells. 9 - FEB 2014 MED EXAM AUG
What is the condition? BOARDS; TOPNOTCH 2014
A. ALL MD)
B. CLL
C. AML
D. CML
E. None of the above
508 Which type of Hodkin's lymphoma has the best JAN CHARMAINE BACK-UP
prognosis? PALOMAR, MD (TOP MIDTERM
A. Nodular sclerosis 9 - FEB 2014 MED EXAM AUG
B. Mixed cellularity BOARDS; TOPNOTCH 2014
C. Lyphocyte-predominant MD)
D. Lymphocyte-rich
E. Lymphocyte-depleted

509 The following characteristics refer to Crohn Noncaseating granulomas-Crohn; No granuloma in JAN CHARMAINE BACK-UP
Disease but not to Ulcerative colitis: Ulcerative colitis PALOMAR, MD (TOP MIDTERM
A. Skip lesions or sharp demarcation of 9 - FEB 2014 MED EXAM AUG
diseased bowel segment from adjacent BOARDS; TOPNOTCH 2014
uninvolved bowel MD)
B. Fistula or sinus tract formation
C. Absence of granulomas
D. Transmural inflammation
E. All of the above statements refer to Crohn
Disease.
510 The following statements are true regarding Robbins 7th ed., 865-66 JAN CHARMAINE BACK-UP
colorectal cancer, except: PALOMAR, MD (TOP MIDTERM
A. The most common site of colorectal cancer 9 - FEB 2014 MED EXAM AUG
is in the rectosigmoid area. BOARDS; TOPNOTCH 2014
B. Carcinomas in the distal colon tend to be MD)
annular, encircling lesions that produce so-
called napkin-ring constrictions of the bowel.
C. Left-sided colonic cancers usually presnt
with occult bleeding, changes in bowel habit, or
crampy left lower quadrant discomfort.
D. Cecal or right-sided colonic cancers usually
present with fatigue, weakness and iron-
deficiency from bleeding.
E. None of the above
511 Which liver disease is characterized JAN CHARMAINE BACK-UP
histologically by hepatocyte swelling and PALOMAR, MD (TOP MIDTERM
necrosis, mallory bodies, neutrophillic reaction 9 - FEB 2014 MED EXAM AUG
and fibrosis? BOARDS; TOPNOTCH 2014
A. Fatty liver MD)
B. Alcoholic hepatitis
C. Alcoholic cirrhosis
D. All of the above
E. None of the above

TOPNOTCH MEDICAL BOARD PREP PATHOLOGY SUPEREXAM Page 68 of 99


For inquiries visit www.topnotchboardprep.com.ph or email us at topnotchmedicalboardprep@gmail.com
TOPNOTCH MEDICAL BOARD PREP PATHOLOGY SUPEREXAM
For inquiries visit www.topnotchboardprep.com.ph or email us at topnotchmedicalboardprep@gmail.com
Item QUESTION EXPLANATION AUTHOR TOPNOTCH
# EXAM
512 Hemosiderin would most likely deposit in in decreasing order of severity:liver, pancreas, JAN CHARMAINE BACK-UP
which organ in patients with hereditary myocardium, pituitary gland, adrenal gland, thyroid PALOMAR, MD (TOP MIDTERM
hemochromatosis? and parathyroid glands, joints and skin (detected by 9 - FEB 2014 MED EXAM AUG
A. Pituitary gland Prussian blue histologic reaction) p909 BOARDS; TOPNOTCH 2014
B. Myocardium MD)
C. Liver
D. Myocardium
E. Thyroid gland

513 Kayser-Fleischer ring of Wilson's disease is the JAN CHARMAINE BACK-UP


green to brown deposits of copper in which PALOMAR, MD (TOP MIDTERM
layer of the corneal limbus? 9 - FEB 2014 MED EXAM AUG
A. Corneal epithelium BOARDS; TOPNOTCH 2014
B. Bowman's layer MD)
C. Corneal stroma
D. Descemet's membrane
E. Corneal endothelium

514 Which of the following conditions refer to A. Obstruction of a single main hepatic vein by JAN CHARMAINE BACK-UP
Budd-chiari syndrome? thrombosis is clinically silent. C. Obliteration of PALOMAR, MD (TOP MIDTERM
A. Obstruction of a single main hepatic vein by hepatic vein radicles by varying amounts of 9 - FEB 2014 MED EXAM AUG
thrombosis. subendothelial swelling and fine reticulated collagen - BOARDS; TOPNOTCH 2014
B. Obstruction of 2 or more major hepatic veno-occlusive disease.D. Sinusoidal dilation with MD)
veins produces liver enargement, pain and impediment of hepatic blood efflux - Peliosis hepatica.
ascites.
C. Obliteration of hepatic vein radicles by
varying amounts of subendothelial swelling and
fine reticulated collagen.
D. Sinusoidal dilation with impediment of
hepatic blood efflux.
E. None of the above
515 These are membrane proteins that recognize a SIMILAR TO PREVIOUS BOARD EXAM JAN CHARMAINE BACK-UP
variety of microbe-derived molecules and CONCEPT/PRINCIPLE:Robbins 7th ed., 195 PALOMAR, MD (TOP MIDTERM
stimulate innate immune responses againsts 9 - FEB 2014 MED EXAM AUG
the microbes: BOARDS; TOPNOTCH 2014
A. Toll-like receptors MD)
B. Cancer antigen
C. Hemagglutinin
D. Lipopolysaccharide
E. Endotoxin
516 These mediators effect increase in vascular SIMILAR TO PREVIOUS BOARD EXAM JAN CHARMAINE BACK-UP
permeability and neutrophil recruitment to site CONCEPT/PRINCIPLE: p.208 PALOMAR, MD (TOP MIDTERM
of injury: 9 - FEB 2014 MED EXAM AUG
A. Opsonins BOARDS; TOPNOTCH 2014
B. Coagulation factors MD)
C. Complement molecules
D. Leukotrienes
E. None of the above

517 Which of the following karyotypes is associated SIMILAR TO PREVIOUS BOARD EXAM JAN CHARMAINE BACK-UP
with the classic pattern of Klinefelter CONCEPT/PRINCIPLE: Klinefelter syndrome is best PALOMAR, MD (TOP MIDTERM
syndrome? defined as male hypogonadism that occurs when there 9 - FEB 2014 MED EXAM AUG
A. 45, X are 2 or more X chromosomes and 1 or more Y BOARDS; TOPNOTCH 2014
B. 46, XY chromosomes. P.179 MD)
C. 47, XXY
D. 47, XYY
E. None of the above

518 This disease is characterized by a distinctive SIMILAR TO PREVIOUS BOARD EXAM JAN CHARMAINE BACK-UP
heliotrope discoloration of the upper eyelids CONCEPT/PRINCIPLE PALOMAR, MD (TOP MIDTERM
with periorbital edema that may accompany or 9 - FEB 2014 MED EXAM AUG
precede the onset of muscle disease? BOARDS; TOPNOTCH 2014
A. Inclusion body myositis MD)
B. Dermatomyositis
C. Myotonic Dystrophy
D. Myasthenia gravis
E. None of the above
519 These are benign tumors of adults, usually SIMILAR TO PREVIOUS BOARD EXAM JAN CHARMAINE BACK-UP
attached to the dura, that arise from the CONCEPT/PRINCIPLE PALOMAR, MD (TOP MIDTERM
meningothelial cell of the arachnoid. 9 - FEB 2014 MED EXAM AUG
A. Primary CNS lymphoma BOARDS; TOPNOTCH 2014
B. Meningioma MD)
C. Medulloblastoma
D. Ganglion cell tumor
E. None of the above

520 In Tetralogy of Fallot, the heart is often SIMILAR TO PREVIOUS BOARD EXAM JAN CHARMAINE BACK-UP
enlarged and may be boot-shaped owing to CONCEPT/PRINCIPLE: p. 569 PALOMAR, MD (TOP MIDTERM
what? 9 - FEB 2014 MED EXAM AUG
A. Marked right ventricular hypertrophy, BOARDS; TOPNOTCH 2014
particularly of the apical region MD)
B. Marked right ventricular hypertrophy,
particularly of the base of the heart
C. Marked left ventricular hypertrophy,
particularly of the apical region
D. Marked left ventricular hypertrophy,

TOPNOTCH MEDICAL BOARD PREP PATHOLOGY SUPEREXAM Page 69 of 99


For inquiries visit www.topnotchboardprep.com.ph or email us at topnotchmedicalboardprep@gmail.com
TOPNOTCH MEDICAL BOARD PREP PATHOLOGY SUPEREXAM
For inquiries visit www.topnotchboardprep.com.ph or email us at topnotchmedicalboardprep@gmail.com
Item QUESTION EXPLANATION AUTHOR TOPNOTCH
# EXAM
particularly of the base of the heart
E. None of the above

521 An otherwise healthy 16 year old girl comes to MIGUEL RAFAEL MIDTERM 1
the physician because of a 4 year history of RAMOS, MD (TOP 3 - EXAM - FEB
heavy bleeding with menses. She has a history FEB 2012 MED 2013
of excessive bleeding after a dental extraction BOARDS; TOPNOTCH
but has never had spontaneous bleeding. Her MD)
father has a history of frequent nosebleeds and
post-operative bleeding. Her mother, sisters,
and brother have no history of bleeding
disorders. Examination of the patient shows no
abnormalities except for pallor. Laboratory
studies show Hb 8, Hct 25%, Reticulocyte count
2%, platelet count 200000, bleeding time 12
min, INR 1, APTT 60 seconds. Pelvic
ultrasonography shows no abnormalities.
Which of the following is the most likely
mechanism of this patient’s excessive bleeding?
A) Abnormal structure of von Willebrand factor
B) Capillary fragility
C) Inadequate production of factor VIII
D) Autoimmune platelet destruction
522 A 67 year old man comes to the physician for a MIGUEL RAFAEL MIDTERM 1
follow-up examination. Three years ago, he RAMOS, MD (TOP 3 - EXAM - FEB
underwent radical dissection of a T3 N0 M0 FEB 2012 MED 2013
epidermoid carcinoma of the floow of his BOARDS; TOPNOTCH
mouth and supraomohyoid dissection of his MD)
neck. He currently takes no medications. He is a
120 pack year smoker but stopped 3 years ago.
Vital signs are within normal limits.
Examination shows well-healed surgical scars.
There are no signs of local recurrence. An X-ray
of the chest shows a 3 cm mass in the medial
upper lobe of the right lung. Which of the
following is the most likely cause of these
findings?
A) Bronchioalveolar carcinoma
B) Metastatic carcinoma
C) Primary squamos cell carcinoma
D) Mesothelioma
523 A previously healthy 57 year old man comes to MIGUEL RAFAEL MIDTERM 1
the physician because of impotence for 1 year. RAMOS, MD (TOP 3 - EXAM - FEB
Examination shows bronze-colored skin. His FEB 2012 MED 2013
serum ferritin concentration is 4050 ng/mL. BOARDS; TOPNOTCH
This patient is at increased risk for which of the MD)
following complications?
A) Hepatocellular carcinoma
B) Interstitial lung disease
C) Progressive pancytopenia
D) Renal failure
524 A 42-year-old computer science professor is MIGUEL RAFAEL MIDTERM 1
brought to the physician by her husband, who RAMOS, MD (TOP 3 - EXAM - FEB
reports insidious changes in his wife's FEB 2012 MED 2013
personality and behavior. He reports that she BOARDS; TOPNOTCH
believes that aliens have been speaking to her MD)
and tampering with their heating and air-
conditioning systems. He says that she was
upset when she turned 40 years old, and her
symptoms have developed since that time. She
was adopted, and her family history is
unknown. Physical examination shows
vermicular movements of the tongue and
bilateral writhing motions of the upper
extremities. Mental status examination shows
indifference to her condition and mild to
moderate difficulty with memory and
calculations. What is the mechanism of the
most likely diagnosis?
A) Substantia nigra degeneration
B) Build up of tau proteins in neurons
C) CAG repeats
D) Copper accumulation in tissue

TOPNOTCH MEDICAL BOARD PREP PATHOLOGY SUPEREXAM Page 70 of 99


For inquiries visit www.topnotchboardprep.com.ph or email us at topnotchmedicalboardprep@gmail.com
TOPNOTCH MEDICAL BOARD PREP PATHOLOGY SUPEREXAM
For inquiries visit www.topnotchboardprep.com.ph or email us at topnotchmedicalboardprep@gmail.com
Item QUESTION EXPLANATION AUTHOR TOPNOTCH
# EXAM
525 A 14-year-old boy is brought to the physician MIGUEL RAFAEL MIDTERM 1
by his parents because of a 2-year history of RAMOS, MD (TOP 3 - EXAM - FEB
increasing academic problems. His parents say FEB 2012 MED 2013
that he has always been hyperactive and BOARDS; TOPNOTCH
distractible, but now his academic performance MD)
has deteriorated to the point that he is failing
ninth grade. His teachers say that his
hyperactivity is disrupting the classroom. He
weighs 54 kg (120 lb) and is 152 cm (60 in) tall.
Sexual development is Tanner stage 5;
examination shows macro-orchidism, which
was not shown on previous examinations. He
has a high forehead and long, protruding ears.
He exhibits poor eye contact during the
examination. Psychoeducational testing shows
an IQ of 70. Which of the following is the most
likely diagnosis?
A) Lesch-Nyhan syndromE
B) Prader-Willi syndrome
C) Fragile X syndrome
D) Klinefelter's syndrome
526 A 19-year-old man comes to the physician MIGUEL RAFAEL MIDTERM 1
because of frequent nosebleeds over the past 3 RAMOS, MD (TOP 3 - EXAM - FEB
weeks. He has bipolar disorder currently well FEB 2012 MED 2013
controlled with lithium carbonate, bupropion, BOARDS; TOPNOTCH
and valproic acid. Physical examination shows MD)
no abnormalities except for dried blood in the
nares. Mental status examination shows an
anxious mood and slight motor restlessness.
Serum studies show a lithium carbonate level of
1.3 mEq/L (therapeutic range=0.6–1.2), and
valproic acid level of 77 μg/mL (therapeutic
range=40–100). Which of the following is the
most appropriate next step in management?
A) Measurement of serum aspartate
aminotransferase (AST, GOT) activity
B) Measurement of serum bupropion level
C) Platelet count
D) Discontinuation of lithium carbonate
therapy
527 An asymptomatic 32-year-old man comes for a buzz phrase >> anaphylactic reaction to blood MIGUEL RAFAEL MIDTERM 1
routine health maintenance examination. He transfusion RAMOS, MD (TOP 3 - EXAM - FEB
has a 10-year history of frequent sinus and FEB 2012 MED 2013
pulmonary infections. He had an anaphylactic BOARDS; TOPNOTCH
reaction to a blood transfusion following a MD)
motor vehicle collision 3 years ago. His
temperature is 37 C (98.6 F). Examination
shows mild erythema in the posterior pharynx.
The lungs are clear to auscultation. A complete
blood count and serum protein electrophoresis
are within normal limits. Which of the following
is the most likely cause of the frequent
infections?
A) Colonization with Streptococcus
pneumoniae
B) Common variable immunodeficiency
C) HIV infection
D) Selective IgA deficiency
528 A 67-year-old man with long-standing signs MIGUEL RAFAEL MIDTERM 1
and symptoms of congestive heart failure is RAMOS, MD (TOP 3 - EXAM - FEB
admitted to the hospital because of progressive FEB 2012 MED 2013
shortness of breath. Examination shows no BOARDS; TOPNOTCH
other abnormalities. An x-ray film of the chest MD)
shows cardiomegaly, cephalization of blood
vessels, and a right-sided pleural effusion.
Which of the following sets of pleural fluid
findings is most likely in this patient?
Leukocyte Segmented
Protein Glucose count neutrophils
Monocytes
(g/dL) (mg/dL) (/mm3) (%) (%)

A) 2.5 10 10,000 50 50
B) 2.5 90 2000 60 40
C) 3.8 40 30,000 80 20
D) 4.5 60 10,000 20 80

TOPNOTCH MEDICAL BOARD PREP PATHOLOGY SUPEREXAM Page 71 of 99


For inquiries visit www.topnotchboardprep.com.ph or email us at topnotchmedicalboardprep@gmail.com
TOPNOTCH MEDICAL BOARD PREP PATHOLOGY SUPEREXAM
For inquiries visit www.topnotchboardprep.com.ph or email us at topnotchmedicalboardprep@gmail.com
Item QUESTION EXPLANATION AUTHOR TOPNOTCH
# EXAM
529 A 6-year-old boy is brought to the physician by adrenoleukodystrophy MIGUEL RAFAEL MIDTERM 1
his mother because of progressive visual loss RAMOS, MD (TOP 3 - EXAM - FEB
over the past year. Over the past 2 years, he has FEB 2012 MED 2013
had deterioration of his hearing, speech, BOARDS; TOPNOTCH
writing, and intellectual performance. His MD)
maternal uncle had similar symptoms. Visual
acuity is 20/200 bilaterally. Funduscopic
examination shows optic atrophy. His hearing
is markedly impaired. There is weakness and
spasticity of all extremities. Deep tendon
reflexes are extremely hyperactive. Babinski's
sign is present bilaterally. On mental status
examination, he is not oriented to place, year,
month, or the names of his siblings. An MRI of
the brain shows marked symmetric white
matter disease involving all lobes. Diagnostic
studies are most likely to show which of the
following?
A) Abnormally decreased serum cholesterol
level
B) Acanthocytes on blood smear
C) An excess of very long chain fatty acids
D) Normal nerve conduction studies
530 A 5-year-old boy is brought to the emergency hyaline casts can be seen in setting of dehydration MIGUEL RAFAEL MIDTERM 1
department 30 minutes after he fainted at RAMOS, MD (TOP 3 - EXAM - FEB
home after standing up from a sitting position. FEB 2012 MED 2013
His symptoms began 3 days ago with diarrhea BOARDS; TOPNOTCH
and vomiting. He has had no urine output for 18 MD)
hours. He is alert but quiet. His temperature is
37.5 C (99.5 F), blood pressure is 75/45 mm Hg,
pulse is 120/min, and respirations are 28/min.
Examination shows dry lips and tenting of the
skin. There is no abdominal tenderness. Bowel
sounds are hyperactive. The remainder of the
examination shows no abnormalities. His
capillary refill time is 5 seconds. Intravenous
bolus doses of 0.9% saline are administered.
Bladder catheterization yields 5 mL of urine.
Urinalysis is most likely to show which of the
following?
A) Oxalate crystals
B) Erythrocyte casts
C) Hyaline casts
D) Leukocyte casts
531 A county health officer investigates an outbreak MIGUEL RAFAEL MIDTERM 1
of illness among persons attending a church RAMOS, MD (TOP 3 - EXAM - FEB
picnic. The illness is characterized by the onset FEB 2012 MED 2013
of nausea and vomiting 3 to 4 hours after BOARDS; TOPNOTCH
attending the picnic. All affected persons MD)
recover without specific therapy. The
investigation implicates egg salad as the vehicle
of transmission. This episode is consistent with
a foodborne outbreak caused by which of the
following?

A) Clostridium perfringens
B) Giardia lamblia
C) Salmonella species
D) Staphylococcus aureus
532 A 4-year-old girl is brought to the physician hereditary spherocytosis MIGUEL RAFAEL MIDTERM 1
because of pallor and jaundice for 2 days. She RAMOS, MD (TOP 3 - EXAM - FEB
had previously been well, although she was FEB 2012 MED 2013
treated for jaundice with phototherapy for 2 BOARDS; TOPNOTCH
weeks while a newborn. Her mother and two MD)
additional maternal relatives underwent
splenectomy during childhood for unknown
reasons. Examination of the patient shows
jaundice. The spleen tip is palpated 4 cm below
the left costal margin. Which of the following
blood smear findings is most likely to explain
this family's condition ?
A) Elliptocytes
B) Howell-Jolly bodies
C) Schistocytes
D) Spherocytes

TOPNOTCH MEDICAL BOARD PREP PATHOLOGY SUPEREXAM Page 72 of 99


For inquiries visit www.topnotchboardprep.com.ph or email us at topnotchmedicalboardprep@gmail.com
TOPNOTCH MEDICAL BOARD PREP PATHOLOGY SUPEREXAM
For inquiries visit www.topnotchboardprep.com.ph or email us at topnotchmedicalboardprep@gmail.com
Item QUESTION EXPLANATION AUTHOR TOPNOTCH
# EXAM
533 A 57-year-old man comes for a routine follow- MIGUEL RAFAEL MIDTERM 1
up examination. He has a 10-year history of an RAMOS, MD (TOP 3 - EXAM - FEB
intermittent facial rash. He has been taking FEB 2012 MED 2013
propranolol for 2 months for hypertension. BOARDS; TOPNOTCH
Examination shows several erythematous MD)

You might also like